Вы находитесь на странице: 1из 1011

Practice Exercise 1

Directions (Q. 1-14): The following line chart shows the ratio of export to import of five compa-
nies A, B, C, D and E in years 2000 to 2004.
2.75
2.5
2.25
2 A
1.75 B
Ratio

1.5 C
1.25
D
1
E
0.75
0.5
0.25
0
2000 2001 2002 2003 2004
Year

The following Radar graph shows the projected % increase in export in year 2005 with respect to
2004. (It is assumed that the import in year 2005 is equal to the import in year 2004.)
A

K KUNDAN
80

60
40
E B
20
0

D C

1. In which year was export of company E the maximum?


1) 2000 2) 2002 3) 2003 4) Can’t say
2. The difference between export and import of company D is the maximum in the year
1) 2000 2) 2001 3) 2002 4) Can’t say
3. In year 2003 the difference between export and import is minimum of company
1) A 2) D 3) C 4) Can’t say
4. The export of company A in year 2001 is what percentage more/less than that in year 2000?
1) 24% less 2) 17% more 3) 11% less 4) Can’t be determined
5. The export of company C is twice that of company D in year 2001. The import of company D in year
2001 is 70 million more than the export. The import of company C in year 2001 is
1) 280 million 2) 220 million 3) 240 million 4) 180 million
6. The trade deficit of company B in year 2003 is 75% more than the trade deficit of company A. The
ratio of import of company B to that of company A in year 2003 is
1) 13 : 5 2) 4 : 9 3) 6 : 3 4) 7 : 2
7. If the ratio of export of company E in 2003 to that in 2004 is 4 : 5, the combined ratio of export to
import of company E in year 2003 and 2004 together is
1) 30 : 19 2) 17 : 9 3) 34 : 13 4) 29 : 16
8. The total transactions (export + import) of companies A, B and C in year 2004 are in the ratio 3 : 4
: 2. The export and import of companies A, B and C in year 2004 together are in the ratio of
1) 334 : 213 2) 226 : 179 3) 174 : 97 4) None of these
9. The ratio of export to import of company C in year 2005 as per the projection is
1) 6 : 7 2) 6 : 5 3) 4 : 3 4) 4 : 5

10. Total transaction (ie export + import) of company E in year 2003 is 33 1 % lower than the total
3
transaction of company E in year 2004. Then the ratio of export to import of company E in the year
2003 and 2004 together is
1) 28 : 17 2) 17 : 28 3) 27 : 17 4) 3 : 2
11. If the projected growth in export of company C and company D together in year 2005 is 40% with
respect to the previous year, the ratio of export to import of company C and D together in year 2004
is
1) 9 : 15 2) 7 : 10 3) 9 : 11 4) 5 : 6
12. As per the projection, how many companies have the import more than the export in 2005?
1) 2 2) 1 3) 3 4) None
13. If the exports of companies C, D and E in year 2004 are in the ratio 1 : 3 : 2, the overall % increase
in the export of company C, D and E as per the projected percentage increase in 2005 is
1) 48.3% 2) 54.6% 3) 57.8% 4) 64.8%
14. If the difference between export and import of company A is 60% more than that between export
and import of company B in year 2004, the difference between export and import of company B is

K KUNDAN
what % more/less than that of company A in year 2005?
1) 7.5% less 2) 11.6% more 3) 15% less 4) 17.4% more
Directions (Q. 15-18): The following pie-charts show the revenue (income) and profit of MG Fi-
nance for the financial year 2004-05.
Share of Revenue (incom e) Share of Profit

Others
12% Fertilisers Others
Tube 22.5%
22.0%
Investments
25%
Carborun-
dum
Universal
EID Parry Tube
Fertilisers 9.5%
12% Investments
45% 23.0%
Carborun-
EID Parry
dum
23.0%
Universal
6%
15. The profit from Tube Investments is what % more than the profit from Fertilisers?
1) 4.5% 2) 5.5% 3) 6.6% 4) Can’t be determined
16. The expenditure of Fertilisers is what % more than the expenditure of Tube Investments?
1) 24% 2) 36% 3) 48% 4) Can’t be determined
17. The minimum expenditure of MG Finance is in
1) Fertilizers 2) Others
3) Carborundum Universal 4) Can’t be determined
18. If the total revenue of the company in the financial year 2004-05 is Rs 6250 crores and the total
profit of the company is Rs 600 crores, the profit of which type of investment has the maximum for
per rupee revenue?
1) Fertilizers 2) Carborundum Universal
3) EID Parry 4) Tube Investments
Directions (Q. 19-22): The following graphs show the result of a survey. Refer to the graphs to
answer the questions that follow.

Consum er's Requirem ent from liquid soaps (in %)


60

50

40

30

20

10

0
Skin care Fragrance Freshness Cleansing Action
Reasons For Trial Reasons for Use

K KUNDAN Recomme-
ndation

Price
20%
Size
12%

Freebies
10%

Advertising
18%
Low price
22%

Adverti-
sing
Word of
Mouth
18%
Used it
abroad
10%

25% 15% New to


Sachets Market
15% 35%

19. If 5000 users were questioned for the survey, in which 8% required fragrance and freshness, 3%
required skin care and fragrance, 7% required skin care and cleansing action, 3% required cleans-
ing action and freshness, and 5% required all the qualities from the liquid soap, how many people
required only skin care from liquid soaps? (There are no consumers who required exactly three
qualities from liquid soaps.)
1) 1800 2) 1750 3) 1600 4) 1900
20. If 4200 people were covered under the survey, what is the ratio of the people who tried the soap
because of recommendation and those who did so because of size?
1) 5 : 3 2) 3 : 5 3) 8 : 5 4) 5 : 8
21. What per cent of the people who tried the soap because of recommendation continued using it for
a similar-mentioned purpose?
1) 80% 2) 95% 3) 85% 4) 90%
22. If 7800 people were covered under the survey, what is the difference between the number of
people who use the liquid soaps because it is new to market and the number of people who use it
because of its advertisement?
1) 1750 2) 1560 3) 1800 4) 1500
Answers and explanations
1. 4; Only the ratio of export to import of each of the companies is given. Therefore it can’t be deter-
mined.
2. 4; Only the ratio of export to import of each of the company is given. Therefore we can’t find in
which year the difference between export and import of company D is maximum. It depends
upon the constant involved in each of the ratios.
3. 2; This question is similar to the above problem but we have to find the minimum difference. The
minimum difference between export and import will be when Export = Import, ie ratio = 1.
Hence company D.
4. 4; Different constants are involved in the ratios of export and import of company A in year 2001
and 2000. Therefore we can’t determine.
5. 3; Let K 1 and K 2 be present in the ratio of export to import of company C and company D in year
2001.
Export of company 7
 1.75 
Import of company 4

K KUNDAN
 Export of company C = 7K 1
Import of company C = 4K1
Similarly, export of company D = 3K 2
Import of company D = 4K 2
According to the question,
7K1
2
3K 2

K1 6
 7K1  6K 2   .... (1)
K2 7
Also, 4K 2  3K 2  70  K 2  70
6 6
As per (1)  K1  K 2   70  60
7 7
 Import of company C = 4K1  4  60  240 million.
6. 4; Let K 1 and K 2 be present in the ratio of export to import of company A and company B respec-
tively in year 2003.
Export of company A = K 1
Import of company A = 2K1
Export of company B = 3K 2
Import of company B = 4K 2
Trade deficit (Import – Export) of company A = K 1
Trade deficit of company B = K 2
According to the question,
 75 
K 2  K 1 1  
 100 

7
 K 2  K1 
4
7K1
4
Import of company B 4K 2 4 7
 
Import of company A 2K1 2K1 2
 Required ratio = 7 : 2
7. 1; Let K 1 and K 2 be present in the ratio of export to import of company E in 2003 and 2004 respec-
tively.
Export of company E in 2003 = 5K1
Import of company E in 2003 = 4K1
Export of company E in 2004 = 2K 2
Import of company E in 2004 = K 2
According to the question,
5K1 4
  25K  8K

K KUNDAN
2K 2 5 1 2

K1 8 K .8
   K1  2
K 2 25 25

 8K 2 
5   2K 2
 25  90 30
 Required ratio    = 30 : 19
 8K 2  57 19
4   K2
 25 

8. 2; Let K 1 , K 2 and K 3 be present in the ratio of export to import of companies A, B and C respec-
tively in year 2004.
Total transaction (export + import) of company A = 3K1  2K1  5K1
Total transaction (export + import) of company B = 5K 2  4K 2  9K 2
Total transaction (export + import) of company C = K 3  K 3  2K 3
According to the question,
5K1 3 9K 2 4
 and 
9K 2 4 2K 3 2

 20K1  27K 2  18K 2  8K 3


 K1 : K 2  27 : 20  K 2 : K 3  8 : 18  4 : 9
K1 : K 2 : K 3
 27 : 20 :
4 : 9
 K1 : K 2 : K 3  108 : 80 : 180  54 : 40 : 90

Total export of companies A, B and C 3K1  5K 2  K 3



Total import of companies A, B and C 2K1  4K 2  K 3

3  54  5  40  90 452 226
  
2  54  4  40  90 358 179
 Required ratio = 226 : 179
9. 2; Let K 1 be present in the ratio of export to import of company C in year 2004.
Export of company C in year 2004 = K1
Import of company C in year 2004 = K1
From the radar graph,
% increase in export of company C = 20%
 20  6K 1
 Export of company C in 2005 = K 1 1  100   5 .
 
Import of company C in 2005 = K 1 (same as that of 2004)

6K1
6
 Required ratio = 5   6 : 5
K1 5

10. 1; Let K 1 and K 2 be present in the ratio of export to import of company E in years 2003 and 2004
respectively.

K KUNDAN
Total transaction of company E in year 2003 = 5K1  4K1  9K1
Total transaction of company E in year 2004 = 2K 2  K 2  3K 2
According to the question,

K1 2
 1
9K 1  3K 2 1    2K 2
 3

 
K2 9

2
5   2
Total export of E 5K1  2K 2 9 28
  = 
Total import of E 4K1  K 2 2 17
4   1
9
 Required ratio = 373 : 212
11. 3; % growth in export of company C in year 2005 w.r.t. 2004 = 20%
and, % growth in export of company D = 50%
But companies C and D together have increased by 40% . Obviously, 40% is the weighted mean
of 20% and 50% .
C D
20% 50%
40%
10 20
1 : 2
 Ratio of export of company C to company D in year 2004 = 1 : 2
Let K 1 and K 2 be present in the ratio of export to import of companies C and D in year 2004.
Export of company C = K 1 ; and import of company C = K 1
Export of company D = 3K 2 ; and import of company D = 4K 2

K1 1
and,   2K  3K
3K 2 2 1 2

K1 3
 
K2 2

Export of company C and D K1  3K 2


Required ratio = 
Import of company C and D K1  4K 2

3
K 2  3K 2
9
 2   9 : 11
3 11
K 2  4K 2
2
12. 4; In year 2004, only company D has import less than export. From year 2004 to 2005 export of the
company D increase by 50% .
Therefore, no company has import more than export as per the projection.
13. 1; As per the projected percentage increase, the overall % increase
1 3 2 290
  20   50   60   48 .33 %
6 6 6 6
14. 2; Let K 1 and K 2 be present in the ratio of export to import of companies A and B respectively in

K KUNDAN
year 2004.
Difference between export and import of company A in year 2004 = 3K1  2K1  K1
Difference between export and import of company B in year 2004 = 5 : 4 = 5K1  4K 2  K 2
According to the question,

K 1  K 2 1 

60  8
  K1  K 2  
100  5
 160  24K 1
Export of company A in 2005 = 3K 1  
 100  5

 180 
Export of company B in 2005 = 5K 2    9K 2
 100 
Difference between export and import of company B = 9K 2  4K 2  5K 2
24
Difference between export and import of company A = K1  2K1
5
14K 1 14  8K 2  112K 2
   
5 5  5  25

112K 2
5K 2 
25  100 1300
Required % = 112K 2 =  11 .6%
112
25
11.6% more than that of A.
Practice Exercise 2
Directions (Q. 1-5): Refer to the bar graph below and answer the questions that follow.
The following bar graph gives age-wise as well as offence-wise distribution of male offenders in
England and Wales in the year 2003. Value written on the top of each bar gives the total number of
offences in thousands occurring in that particular category of offence.

100
Percentage of male offenders

80
Ov er 16

60 14-16

12-14
40
10-12

20

0
Burglary Theft Robbery Violence Others

K KUNDAN
Category of offence

1. The second highest number of crimes is committed by the boys in the age group
1) 10-12 2) 12-14
3) 14-16 4) Over 16
2. Which of the following conclusions is true?
1) Boys in age group 14-16 commit more burglaries than theft.
2) Boys in age group 12-14 are the most violent.
3) In the boys’ over-16 age group the number of burglaries committed is more than that of violent
acts.
4) Burglary is the second most common offence amongst the juvenile delinquent below the age of
12.
3. If the total offences were shown by a pie-chart what sector angle would approximately represent
robberies?
1) 10 2) 3
3) 24 4) 65
4. The total number of offences committed by those over 16 is greater than the total number of
offences committed by those in the age group of 12 to 16 by
1) 10% 2) 5%
3) 26% 4) 1%
5. If no child committed more than one offence, given that the total number of boys in the age group
10-12 was 1457682, what was the approximate percentage of children who were away from crime
in that age group?
1) 5 2) 82
3) 87 4) 96
Directions (Q. 6-11): Refer to the table below and answer the questions that follow.
Statewise area and production of tobacco
(A = Area in ’000 hectares; P = Production in ’000 tonnes)
Note: Figures in brackets show the percentage share of Virginia Tobacco.

Year

State 1997-1998 1998-1999 1999-2000 2000-2001 2001-2002

A P A P A P A P A P

Andhra Pradesh 209.7 262.8 198.7 190.6 171.6 154.9 169.4 164 182.8 192.5
(66) (60) (78) (71) (74) (61) (7.5) (67) (75) (67)
Gujarat 81.5 125.3 74.6 153.6 114.7 174.9 121.5 185.2 113 209.5
Karnataka 42.1 30.5 43.5 29.6 44.5 26.6 51.6 33.9 50.8 30.5
(24) (24) (23) (23) (29) (25) (30) (21) (31) (30)
Tamil Nadu (TN) 11.3 16.7 12.7 19 12.9 19.1 14.5 22.4 17.2 26.1
West Bengal 14.9 14.5 13.2 12.7 12.7 11.5 18.9 17.4 14.1 13.3
(3) (2) (4) (2) (2) (2) (1) (1) (1) (1)
Others 63.9 43.8 66.6 48.3 69 51.5 75.6 57.9 70.4 53.4

All India Total 504.4 493.6 409.3 453.8 425.4 438.5 451.5 480.8 448.3 525.3

6. In which of the following years was the productivity (tonnes per hectare) of tobacco in Andhra
Pradesh greater than one?

K KUNDAN
1) 1998-99 2) 1999-2000
3) 2000-2001 4) 2001-2002
7. The all-India percentage share of Virginia Tobacco in the total tobacco production in 1997-1998
was approximately
1) 50 2) 86 3) 33 4) 27
8. If in 2002-2003 the area under cultivation of tobacco was increased to 18,000 hectares in Tamil
Nadu, the tobacco production would then approximately be (in ’000 tonnes, assuming productivity
in 2002-2003 remains the same as in 2001-2002):
1) 32.6 2) 16.3
3) 25 4) 27.3
9. Tobacco production in which of the following States increased consistently over the five-year
period?
1) Karnataka & TN
2) Gujarat & TN
3) Karnataka & Gujarat
4) W Bengal & TN
10. Total production of Virginia Tobacco was higher in which year?
1) 1997-1998
2) 1998-1999
3) 2000-2001
4) 2001-2002
11. Which of the following is true?
1) Production of Virginia Tobacco is increasing every year in Andhra Pradesh.
2) Production of Virginia Tobacco is second highest in year 1999-2000.
3) Production of Virginia Tobacco in West Bengal is decreasing.
4) All-India productivity of Tobacco is minimum in year 2000-01.
Directins (Q. 12-16): Refer to the line graph below and answer the questions that follow.
DIRECT SELLING BUSINESS
14 2100
No. of distributors Turnover 1950

No. of distributors (in lakhs)


12 1800
11.2
Turnover (Rs crores)

1400
10 1500

8 1000 1200
8.2
5.2
6 900
6.4
4 600
300 600
2 300
1.2
0 0
1998 1999 2000 2001 2002

Turnover in 2002 (in Rs No. of distributors in 2002


Rank Company
Crores) (in lakhs)
1. Wamay 472 3.65
2. Codimare 165 4.95
3. Nova 110 0.68
4. Balife 78 0.2

K KUNDAN
12. What is the average simple annual growth rate of turnover of direct selling business in India
during the given period?
1) 42%
3) 78%
2) 60%
4) 137.5%
13. If Wamay and Nova were the only companies in the direct selling business in India till 1998 with
the Wamay market share three times that of Nova, then what is the percentage growth in Nova’s
turnover during the given period?
1) 10% 2) 15.6%
3) 35% 4) 46.7%
14. Balife was launched in 2000. Since then, the number of its distributors is increasing by 25%
every year and its turnover by 20% . Then what was the turnover-to-number of distributors ratio of
Balife during its launching year?
1) 36120 2) 39820
3) 42320 4) 45720
15. Which of the following is definitely false?
1) The ratio of turnover to number of distributors is maximum for Balife during 2002.
2) Top four companies together have more than 80% of total number of distributors in the direct
selling business in India during 2002.
3) There are not more than 18 companies in direct selling business in India during 2002.
4) None of these
16. During which year, has the ratio of turnover-to-number of distributors shown maximum percent-
age increase over the previous year?
1) 1999 2) 2000
3) 2001 4) 2002
Directions (Q. 17-20): Refer to the diagrams below and answer the questions that follow.
Given below is the data of a consumer confidence survey.

28 32
II I Jaipur II I Jaipur
(35, 24) (17, 23)
21 24
Mumbai
14 (16, 11) 16 Kolkata
Bangalore (-8, 8)
Kolkata (29, 5)
7 8
(0, 1)
Bhubaneshwar
0 0
III Delhi IV III Delhi(-5, -6) Mumbai IV
Bhubaneshwar (-9, -10) (0, -2)
–7 (-7, -1) (2, -3) –8
Bangalore
Coimbatore Ahmedabad Coim- batore (11, –16)
–14 (-18, -17) (-5, -16) –16 (–8, –11)
Ahmedabad
(-16, -13)
–24 –24
–40 –30 –20 –10 0 10 20 30 40 –20 –15 –10 –5 0 10 15 20
PSI (Gain/Loss) over January 2004 PSI (Gain/Loss) over September 2004

FEI  Future Expenditure Index


PSI  Present Situation Index
FEI
CCI  Consumer Confidence Index 
PSI
17. For how many cities has PSI changed from a positive value to negative value?
1) 0 2) 1 3) 2 4) 3
18. How many cities have shown greater than or equal to 100% change in FEI?

K KUNDAN
1) 2

1) Ahmedabad
2) 3

2) Kolkata
3) 4

3) Delhi
20. For how many cities, FEI as well as PSI decreased over the period?
1) 3

Answers and explanations


1-5:
2) 4 3) 5
4) 5
19. Which of the following cities have CCI more than 1 during September 2004?
4) Bangalore

4) 6

Please note that the bar chart gives the % distribution of each age group in various kinds of
offences.
1. 3; For all kinds of offences, the percentage of crimes committed is the highest in the age group
over 16 and second highest in age group 14-16. Thus, number of crimes is second highest in
the age group 14-16. Hence, (3).
2. 4; Statement 1:
Not true, as the percentage of boys committing burglaries and thefts is almost the same, but
the number of thefts is much more than burglaries. Thus, thefts would be more than burglaries
in the age group 14-16.
Statement 2:
Note true, as the boys in the age group over 16 are the most violent.
Statement 3:
For age group over 16:
Number of burglaries = 35% × 82  28.7 thousand
Number of violent acts = 55% of 53  29.2 thousand
Hence, this is not true.
Statement 4:
Below the age of 12, the most common offence is theft, followed by burglary. Hence, this state-
ment is true.
3. 2; Total offences = 82 + 238 + 4 + 53 + 74 = 451 thousand.
4  360
 In a pie-chart the angle of the sector representing robberies = 451
 3º.
4. 4; Total number of offences in ’000 in the age group over 16
35 40 50 55 60
=  82   238  4  53   74
100 100 100 100 100
 29  95  2  29  44  199
50 45 40 40 35
In age group 12 to 16 =  82   238  4  53   74
100 100 100 100 100
 41  107  2  21  26  197
5. 4; Number of crimes in age group 10 to 12
 15 15 10 5 5 
=  82   238  4  53   74 
100 100 100 100 100 
= (12 + 36 + 0.4 + 3 + 4) × 1000  55 × 1000  55000
55000
 Percentage of children committing crimes =  100  4 %
1457682
 Percentage of children away from crimes = 100 - 4 = 96% .
6. 4; In the row of Andhra Pradesh choose the year in which P > A.
262.8  0.6  30.5  0.24  14.5  0.02
7. 3;  100  33%
493.6
18

K KUNDAN
8. 4;  261
.  27.3 (Assuming same productivity).
17.2
9. 2; Gujarat and Tamil Nadu both show increasing trends of production.
10. 1; As Andhra Pradesh is the sole biggest contributor, it is sufficient to note that in 1997-1998 the
production of Virginia Tobacco in AP was greater than in any other year by a significant margin.
11. 3; The production of Virginia Tobacco is decreasing every year in West Bengal.
1950  300 1650
12. 4; Average annual growth rate =  100  = 137.5% .
300  4 12
1
13. 4; Nova’s turnover in 1998 =  300 = Rs 75 crores
4
Nova’s turnover in 2002 = Rs 110 crores
110  75 35  100
 Percentage growth =  100   46 .7%
75 75
14. 3 At 25% per annum, the number of distributors increases by 56.25% in two years. At 20% per
annum, the turnover increases by 44% in two years.
 Turnover-to-number of distributors for Balife in 2000
78  100 lakh
1.44 7800 1.5625
=    42318
0.2 lakh 1.44 0.2
1.5625
15. 3; Nothing can be inferred about statement 1 as data is given for only top 4 companies.
Statement 2 is definitely true as top 4 companies have approximately 85% of total number of
distributors.
Total turnover of top 3 companies = 472 + 165 + 110 = Rs 747 crores
Turnover of all other companies = 1950 - 747 = Rs 1203 crores
Number of companies in the direct selling business will be minimum if all companies other
than top 3 companies have turnover almost equal to the turnover of fourth company, ie Balife
1203
 Minimum number of companies (excluding top 3) =  15 .4  16
78
 Minimum number of companies = 16 + 3 = 19
 Statement 3 is definitely false.
16. 2;

Year Ratio (approx) Percentage increase


1998 25000 -
1999 11500 -ve
2000 15600 4100/115 > 30%
2001 17100 1500/156 < 10%
2002 17400 300/17100 < 10%
17. 2; Only for Delhi, PSI has changed from 2 to –9, ie positive value to negative.
18. 4; City % Change in FEI
1
Jaipur  100  100 %
24

K KUNDAN
5   16  2100
Bangalore  100   100%
5 5
11   2 1300
Mumbai  100   100 %
11 11
1 8
Kolkata  100  700 %  100 %
1
1 6
Bhubaneshwar  100  500 %  100 %
1
 17   11 600
Coimbatore  100   100 %
 17 17

 16   13  300
Ahmedabad  100   100 %
 16 16

 3   6  300
Delhi  100   100 %
3 3
Only Jaipur, Ahmedabad and Coimbatore have less than 100% change in FEI.
 Required number of cities = 8 – 3 = 5
19. 3; To have CCI more than 1, |FEI| must be greater than |PSI| during the given period. Only Delhi
has [FEI] more than |PSI|.
20. 2; FEI and PSI of Jaipur have decreased from (35, 24) to (17, 23) over the period.
Similarly, FEI and PSI of Mumbai, Bangalore and Delhi have decreased over the period.
 Total no. of cities = 4.
Practice Exercise 3
Directions (Q. 1-5): Refer to the pie-charts below and answer the questions that follow.
Monthly income and expenditure for a semi-government institute

INCOM E EXPENDITURE

Facilities
31%

Go vernment
Students 35%
40%

Wages Awards
55% 5%

Reno vation
9%
Ins titute
25%

There are some constraints:


i) Income from students can be used for awards and facilities; otherwise it has to be kept aside.

K KUNDAN
ii) Renovation can be done only with donation from institute.
iii) Government and institute’s contributions do not increase even if any new course is intro-
duced for/till first 2 years.
1. If government’s contribution was Rs 10 lakh and the management could save Rs 60000, then
what would be the expenditure on wages? (approximately)
1) Rs 1504000 2) Rs 1538000 3) Rs 1602000 4) Rs 1571000
2. The total income is Rs 35 × 106. The percentage utilization of the income from the students is
100% . If expenditure on awards and facilities is met only with income from students, then what is
the total expenditure? (approximately)
1) Rs 31.5 × 106 2) Rs 35 × 106
6
3) Rs 39 × 10 4) Rs 40 × 106
3. Approximately what is the ratio of the expenditure on facilities to that on wages?
1) 5 : 11 2) 11 : 6
3) 6 : 11 4) Cannot be determined
4. If no money is spent on renovation then the income and the expenditure are the same, otherwise
there is a shortage of Rs 650000. Then what is the total income?
1) Rs 72 × 105 2) Rs 65.7 × 105 3) Rs 59.5 × 105 4) Rs 54.5 × 105
5. Students’ contribution is totally used in awards and facilities but Rs 2  10 6 is still left. Find the
total expenditure (in Rs).
1) 5  106 2) 7.5  10 6 3) 4×106 4) Data inadequate
Directions (Q. 6-10): Refer to the bar graph below and answer the questions that follow.
In the game of basketball, points for the correct throws are 1, 2 or 3. In a match the number of
attempts to basket the ball and accuracy are given for all players of the team below. Indian Railways’
players are A, B, C, D and E.
100 25

80 20

60 15

40 10

20 5

A B C D E
1-pointer attempts (%) 2-pointer attempts (%)

3-pointer attempts (%) Total number of attempts

Accuracy
Player
1-pointer 2-pointer 3-pointer
A 66.66 33.33 20
B 100 66.66 100

K KUNDAN
C 75 75 100
D 88.88 100 50
E 100 71.42 83.33

6. How many points were scored by player A?


1) 13 2) 19 3) 21 4) 39
7. What was the accuracy of the most accurate player? (Accuracy means no. of baskets per attempt.)
1) 78% 2) 80%
3) 83% 4) 87%
8. What percentage of total points was scored by player D?
1) 13% 2) 21% 3) 30% 4) 37%
9. What percentage of total points was scored through 2-pointers?
1) 22% 2) 32% 3) 42% 4) 52%
10. Point scored by all players from 3-pointers is what percentage (approx.) more/less than those from
2-pointers?
1) 15% more 2) 10% less
3) 15% less 4) 10% more
Directions (Q. 11-15): Refer to the graphs below and answer the questions that follow.
Graph (i) shows volumewise share of various companies in car market in India, for March 2004.
Graph (ii) shows total number of cars sold since October 2003 to March 2004.
Graph (iii) shows ratios of market prices of cars of the companies which are written next to each
 price of Bajaj car 
other, ie   = 0.66
 price of Toyota car 
Graph (i)
Bajaj

Honda
M&M
Toyota

Maruti
Graph (ii) Graph (iii)
Num ber of cars sold (in thousands)
80 2.5
70 2
2
60
50 1.5
40 1.1
30 1
0.66 0.625
20
0.5
10
0
0 0
Oct Nov Dec Jan Feb Mar Toyota Bajaj Maruti Honda M&M
2003 2003 2003 2004 2004 2004

K KUNDAN
11. What is the share of M&M in total sale (in Rs) of cars in March 2004?
1) 32% 2) 48% 3) 60% 4) Cannot be determined
12. If Toyota had 20% share in volume in November 2003, then what is the ratio of its sales (units) in
November 2003 to that in March 2004?
1) 11 : 10 2) 8 : 5 3) 44 : 25 4) None of these
13. If a Honda car costs Rs 2.5 lakhs, then what were the sales of Bajaj cars in March 2004?
1) Rs 125 crores 2) Rs 160 crores 3) Rs 250 crores 4) Rs 300 crores
14. How many cars cost more than the average price of cars in March 2004 among the given group of
cars?
1) 1 2) 2 3) 3 4) Cannot be determined
15. If the cost of Bajaj car is Rs 108000 less than that of M&M, then the income from the selling of a
car by Maruti is what % more/less than that by Honda in March 2004?
1) 60% less 2) 50% less 3) 40% more 4) 25% less
Directions (Q. 16-20): The following table shows the percentage population above poverty line
and ratio of male to female above and below poverty line for states A, B, C, D and E in year 2003.

% population above Ratio of male to female


Name of State
poverty line Below poverty line Above poverty line
A 72 4:3 4:5
B 64 8 : 10 9:7
C 56 6:5 13 : 15
D 84 7:9 11 : 10
E 76 5:7 9 : 10
16. In which state is the maximum male population below poverty line?
1) C 2) B
3) A 4) Can’t be determined
17. If the female population below poverty line in state D is 6.3 million, find the female population
above poverty line.
1) 38 million
2) 28 million
3) 40 million
4) 42 million
18. If the female population above poverty line in state A is 3.6 million more than the female popula-
tion above poverty line in state D, find the difference between total population of State A and state
D.
1) 9 million
2) 8 million
3) 10 million
4) Can’t say
19. If the percentage of male population below poverty line in state C and state E together is 15% , find
the % male population above poverty line in states C and E together.
1) 45.2% 2) 32.4% 3) 37.8% 4) Can’t say
20. If the female population above poverty line in state C is greater than the female population above
poverty line in state B, which of the following is definitely true?
1) Population of state C < population of state B
2) Population of state C > population at state B

K KUNDAN
3) Population of state C = population of state B
4) Can’t say

Answers and explanations


1. 2; Government’s contribution = 35% of the total income
10 6  100
 Total income =  2857143
35
Savings = Rs 60000  Total expenditure = Total income - savings = 2857143 - 60000 = 2797143.
Expenditure on wages = 55% of total expenditure  1538429.
2. 3; From condition (i), 100% income from students is utilized.
 income from students = expenditure on awards and facilities.
 40% of income = 36% of expenditure.
100 40 6 6
 Expenditure = 36  100  35  10  388889  3900000  39  10
31 6
3. 3; Facilities-to-wages ratio = 
55 11
4. 2; From the given information, total income = total expenditure - 9% of total expenditure
= total expenditure - 650000
100
=  650000  650000  6572222  6570000
9
5. 4; Let the total income be Rs x and total expenditure be Rs y.
According to the question,
40% of x – 36% of y = 2  105
But we can’t find the value of y.
Hence, data inadequate.
6-9:

No. of attempts for Number of baskets


Player Player
1-pointer 2-pointer 3-pointer 1-pointer 2-pointer 3-pointer

A 6 9 5 A 4 3 1

B 3 6 1 B 3 4 1

C 8 4 3 C 6 3 3

D 9 3 6 D 8 3 3

E 4 14 6 E 4 10 5

Total 30 36 21 Total 25 23 13
6. 1; Points scored by player A = 4 × 1 + 3 × 2 + 1 × 3 = 4 + 6 + 3 = 13
Alternatively:
Player A has overall accuracy of around 40% with quite a low accuracy of 3 pointers.
With total 20 attempts, assuming average score of 2 per attempt, approximate score would be
0.4 × 2 × 20 = 16.
7. 2;

P layer A B C D E

K KUNDAN
Accuracy 8/20 = 0.4 8/10 = 0.8 12/15 = 0.8 14/18 < 0.8 19/24 < 0.8
Players B and C have the maximum accuracy of 80% .
8. 2; Total points = 25 × 1 + 23 × 2 + 13 × 3 = 25 + 46 + 39 = 110
Points scored by player D = 8 × 1 + 3 × 2 + 3 × 3 = 23
23
 Percentage of points scored by player D = 110  100  21 %
Alternatively:
Approximately 20% of total attempts were made by player D with the same accuracy (approxi-
mately) as the other players except A, whose accuracy is quite low. Hence, player D must have
scored approximately 20% of total points.
9. 3; Number of points scored through 2-pointers = 2 × 23 = 46
46
 Percentage of points through 2-pointers = 110  100  42 % .
10. 3; Total points scored from 3-pointers = 13 × 3 = 39
Total points scored from 2-pointers = 23 × 2 = 46
46  39
Required percentage =  100  15 .2%
46
11. 1; From graph (iii)
Price of Bajaj 2 Price of Maruti
5 Price of Honda Price of M & M
 ; ;   2;  1.1
Price of Toyota 3 Price of Bajaj 8 Price of Maruti Price of Honda
 If price of Bajaj car is 8x, then prices of Toyota, Maruti, Honda and M&M cars are 12x, 5x, 10x
and 11x respectively.
From graph (i),
Percentage shares of Honda and Toyota are 12.5% each while those of Bajaj, M&M and Maruti
are 25% each.
25
Sales of M&M cars (in value) in 2004 = 11x   50000
100
25 12.5
Total sales (in value) = (8x + 11x + 5x) × × 50000 + (12x + 10x) ×  50000
100 100
25 25
= (24x + 11x) × × 50000 = 35x × × 50000
100 100
11
Percentage share of sales of M&M cars =  100  32 %
35

20
 55
Sales of Toyota in November 2003 100 220 44
  
12. 3; Sales of Toyota in March 2004 12.5 125 25
 50
100

2.5
13. 3; If a Honda car costs Rs 2.5 lakhs, then a Bajaj car will cost  0 .625  Rs 2 lakhs.
2
25
Sales of Bajaj cars in March 2004 =  50000  2 = 25000 lakhs.
100
14. 3; Ratio of price of all the given companies is

K KUNDAN
Toyoto : Bajaj : Maruti : Honda : M & M
3 : 2
8 : 5
1 : 2
10 : 11
 3  8  1  10 : 2  8  1  10  : 2  5  1  10  : 2  5  2  10  : 2  5  2  11
 240 : 160 : 100 : 200 : 220  12 : 8 : 5 : 10 : 11
Volumes of production of Toyota, Bajaj, Maruti, Honda and M&M in March 2004 are in the ratio
1:2:2:1:2
Let x be present in the ratio of the price of each of the cars.
12x  1  8 x  2  5 x  2  10 x  1  11x  2 70 x
Average price in March 2004 =   8.75 x
1 2  2 1 2 8
Therefore the prices of Toyota, Honda and M&M are greater than the average price of all cars in
March 2004.
15. 2; From solution (11) : Ratio of market prices is Bajaj : M&M = 8x : 11x
 11x – 8x = 3x = 1,08,000
 x = 36,000
Now market price of Maruti = 5x = 180,000
Market price of Honda = 10x = 3,60,000
 required answer = 50% .
Quicker Approach:
Ratio of price of Honda to Maruti = 2 : 1
2 1
 Required ratio =  100  50 %
2
Note: There is no need of the cost of Bajaj car in March 2004.
16. 4; Total population of each of the states is not given. Hence data inadequate.
9
17. 2; part of 16%  6.3 million
16
 9%  6.3 milion
10 6.3
 part of 84% ie 40%   40  28 million.
21 9
18. 1; At first glance, data seems inadequate for the question.
Let total population of state A and state D be x and y respectively.
5
Female population above poverty line in state A = part of 72% of x = 40% of x.
9
10
Female population above poverty line in state D = part of 84% of y = 40% of y.
21
According to the question,
40% of x – 40% of y = 3.6
 40% of x  y   3.6
 x  y = 9 million
Note: If percentages are different then we can’t solve the problem.
6
19. 2; Percentage male population below poverty line in state C = part of 44% = 24%
11
5
and that in state E = part of 24% = 10%
12

K KUNDAN
Combining both states, we have 15% male population below poverty line.
It means that 15% is the weighted mean of 10% and 24% .
C
24%

5
15%
E
10%

9
Therefore, total populations of C and E are in the ratio 5 : 9.
13
Now, % male population above poverty line in state C = part of 56% = 26%
28
9
and that in state E = part of 76% = 36% .
19
26  5  36  9 130  324 454
Required % =    32 .4%
59 14 14
20. 4; Let the total population of state C and state B be x and y respectively.
According to the question,
15 7
part of 56% of x > part of 64% of y
28 16
 30% of x > 28% of y
 x > y or x < y or x = y.
Practice Exercise 4
Directions (Q. 1-5): Read the information to solve the questions.

Num ber of bikes produced by Hero Honda Market share of bikes for year
2001 (Market Size - 1,50,000 bikes)
30000
25000
20000 Others CD100
CD100
30% 35%
15000 Splendor
10000 CBZ

5000 CBZ
Splendor
0 20%
15%

K
2000 2001

1. If Hero Honda company wants all its CBZ which are produced in 2000 and 2001 to be sold then
what should have been the number of CBZ produced in 2001?
1) 15,000 2) 18,000
3) 17,000 4) Indeterminable
2. If in 2001 Yamaha accounts for 8% of market share, what per cent of Others category does it
account for?
1) 24% 2) 25%

KUNDAN
2
3) 26 % 4) 28%
3
3. If Hero Honda plans to launch a new model Crazy in 2002 and if the market increases by 20% and
the market share of Hero Honda increases by 10% , then what is the minimum number of Splen-
dor that needs to be produced to meet the demand?
1) 27,000 2) 25,000
3) 25,500 4) Indeterminable
4. If in 2002 LML enters the market and is able to capure 30% of the 50,000 market then what is the
percentage difference between market share of LML and Splendor? (assuming percentage mar-
ket share of Hero Honda models and Others is the same as in the previous year)
1) 24.5%
2) 19.5%
3) 12%
4) None of these
5. If the market share of Others remains the same, what is the total number of Bajaj bikes sold in
2001, if Bajaj enters as a new player in market in 2001 and has the same market share as Hero
Honda?
1) 52,500 2) 53,500
3) 52,000 4) 54,000
Directions (Q. 6-11): Following graph gives details about the fortunes of the company Mul-
tiple Investment. Productivity is given here as the produced value per employee in ’00000 $.
Profitability is defined here as the profit as a percentage of produced value. The number of em-
ployees is given in ’00s.

19
17
15
13
11
9
7
5
91 92 93 94 95

Profitability Productivity No. of employees

6. What was the percentage change in the production in the given period for Multiple Investment?
1) 78% 2) 86% 3) 68% 4) 92%
7. Which year showed the sharpest change in production?

K
1) 92 2) 93 3) 94 4) 95
8. Which year showed the sharpest increase in profits?
1) 92 2) 93 3) 94 4) 95
9. In which year were the profits per employee the highest?
1) 91 2) 93 3) 94 4) 95
10. In how many years has the profitability shown an inverse trend to that of the number of employ-
ees?
1) 0 2) 1 3) 2 4) 3
11. Which of the following years has the lowest profits per employee?
1) 92 2) 93 3) 94 4) 95

KUNDAN
Directions (Q. 12-16): The tabular data given below gives the information about the adult
population (in lacs) in the four different regions of a city in 1996 along with the death and birth
rates in the respective regions.

Male
Female
Birth Rate
Region I
12
10
5%
Region II
8
11
8%
Region III
14
9
12%
Region IV
22
12
3%
Death Rate 0.5% 2% 1% 3%

12. What was the total adult population of all the four regions combined?
1) 81 2) 83 3) 85 4) 98
13. What was the number of deaths in the given year for regions I and II? (in thousands)
1) 35 2) 43 3) 49 4) 54
14. Which region had the highest population growth rate in the given year?
1) Region I 2) Region II
3) Region III 4) Region IV
15. Which region had the highest number of births?
1) Region I 2) Region II
3) Region III 4) Region IV
16. Which region had the lowest number of deaths?
1) Region I 2) Region II 3) Region III 4) Region IV
Directions (Q. 17-20): Refer to table below and answer the questions that follow.
BSNL announced a cut in STD rates on 27.12.2001. The new rates and slabs are given in the
table below and are to be implemented from 14.01.2002. Assume you are given this question
on 01.01.2002.

Rates (Rs/min)
Distance
Peak Rates Off Peak
(in km)
Old New Old New
50-200 4.8 2.4 1.2 1.2
200-500 11.6 4.8 3.00 2.4
500-1000 17.56 9.00 4.5 4.5
>1000 17.56 9.00 6.00 4.5
17. The maximum percentage reduction in costs will be experienced for calls over which of the follow-
ing distance (in km)?

K
1) 50-200 2) 200-500 3) 500-1000 4) >1000
18. The percentage difference in the cost of a set of telephone calls made on the 13th and 14th
January having durations of 4 minutes over a distance of 350 km, 3 minutes for a distance of 700
km, and 3 minutes for a distance of 1050 km is (if all the three calls are made in peak times)
1) 51.2% 2) 51.75% 3) 59.8% 4) Can’t be determined
19. If one of the three calls in Q. 18 were made in an off peak time on both days, then the percentage
reduction in the total cost of the calls between 13th and 14th January will
1) Definitely reduce
2) Definitely increase
3) Depend on which particular call was made in off peak time

KUNDAN
4) Can’t be determined
20. A person talks for a certain no. of minutes at peak rates and off peak rates on 13.01.2002 for 50-
200 km distance such that his averge cost of talking per minute is Rs 3.7/minute. Find the least
time that he talked (assuming he talks only in multiples of a minute).
1) 27 minutes 2) 43 minutes 3) 36 minutes 4) Can’t say
Answers and explanations
1. 4; We don’t know the market share of 2000. We can’t find the number of CBZ sold in year 2000.
2. 3; x% of 30% of total = 8% of total
8  100 2
x   26 %
30 3
3. 4; We don’t know the percentage distribution of Hero Honda models in 2002.
15
4. 2; % market share of LML = 30% and percentage share of Splendor =  70  10 .5%
100
Required percentage difference = 30 – 10.5 = 19.5%
5. 1; Others have 30% share.
Bajaj has 35% share.
Hero Honda has 35% share (as percentage shares of Bajaj and Hero Honda are the same).
and 35% of 150000 = 52500.
6. 1; Production = Productivity × No. of employee
70
The increase was from 9 × 10 = 90 to 16 × 10 = 160. So 100  78% .
90
7. 4; In 1995 both the productivity and the number of employees showed the sharpest increases.
Since the production is given as the product of these two, ’95 had the sharpest incease in
production.
 
8. 2; Profit in 1991 = 12% of 9  105  10  10 2  1080  105

Profit in 1992 = 11% of 8  10  12  10   1056  10


5 2 5

Profit in 1993 = 12% of 8.5  10  13  10   1326  10


5 2 5

Profit in 1994 = 10% of 8.5  10  15  10   1275  10


5 2 5

Profit in 1995 = 10% of 10  10  16  10   1600  10


5 2 5

1326  1056
% change in profit for 1992-93 =  100  25 .56 % .
1056
1600  1275
% change in profit for 1994-95 =  100  25 .50 % .
1275
9. 1; See the previous soln.

K
Profit per employee in year 1991 is maximum.
10. 3; In ’92 and ’94 the trends shown by productivity and number of employees were inverse.
11. 3; In ’92 the profit per employee was 1056/12, and in ’94 the value was 1275/15. The value of ’94
is the smaller one.
12. 4; Simply add up all the figures of the male and female populations of the four regions and the
answer comes to 98 lacs. Otherwise the numbers are all odd except 98, and from the figures,
there are two odd numbers; therefore the sum has to be even.
13. 3; To find the number of deaths of region I and II, just find 0.5% of 22 and 2% of 19. The sum comes
to 49 thousand.
14. 3; The highest growth rate is of region III, which is 12 - 1 = 11% .
15. 3; Region III, which has a population of 23 lacs and a birth rate of 12% , will have the highest

KUNDAN
number of births.
16. 1; The lowest number of deaths is of region I, which is 0.5% of 22 lacs.
17. 2; Clearly, for 200-500 km distance there is maximum percentage reduction.
4  11.6  3  17.56  3  17.56  4  4.8  3  9  3  9
18. 2; Required % = × 100
4  11.6  3  17.56  3  17.56
151 .76  73 .2 78 .56
  100   100  51 .75 %
151 .76 151 .76
19. 1; Definitely reduce.
20. 3; On 13.01.2002 old rate was applicable.
Rs 3.7/minute is the weighted mean of Rs 4.8/min and Rs 1.2/minute.
Peak rate Off peak
4.8 1.2
3.7
2.5 1.1
 25 : 11
Since 25:11 can’t be simplified further,
Therefore the least time that he talked = 25 + 11 = 36 minutes.
Practice Exercise 5
Directions (Q. 1-5): Refer to the charts below and answer the questions that follow.
Figure (i) Figure (ii)
Total custom ers = 1350000

AT&T
BPL
Idea 8%
20%
9%

Dolphin
15% Orange
13%

K
Airtel Reliance
17% 18%

Figure (iii)
5

KUNDAN 2

0
0

Orange BPL
4

Dolphin
6

Airtel
Figure (i) shows share of various telecom companies in Indian market.
8

Reliance
10

Figure (ii) shows % of pre-paid and post-paid customers of various plans of various companies.
Figure (iii) shows average number of SMS sent in a day (x-axis) by customers and its cost in Rs (y-
axis).
1. What is the ratio of the number of BPL pre-paid plan 2 customers to the number of Idea post-paid
plan C customers?
1) 9 : 10 2) 10 : 9 3) 40 : 9 4) 9 : 40
2. Which of the following is true?
I. The total income of Airtel through SMS in a day is Rs 33 lakhs.
II. The number of Dolphin pre-paid plan 1 customers is equal to that of AT&T post-paid plan C
customers.
III. The amount collected by Reliance through SMS is greater than that by Orange, in a week.
IV. The number of Orange post-paid customers is equal to the number of Idea pre-paid customers.
1) Only I 2) Only I and III 3) Only II and IV 4) Only I, III and IV
3. What is the difference between the number of BPL post-paid plan A customers and that of Idea pre-
paid plan 3 customers?
1) 0.79 lakh 2) 0.87 lakh 3) 0.95 lakh 4) 1.13 lakhs
4. Orange and BPL pre-paid customers send 4 SMS in a day. Then what is the approximate ratio of
the incomes of Orange and BPL from SMS service given to post-paid customers?
1) 1 : 2 2) 2 : 3 3) 1 : 3 4) 2 : 5
5. Reliance announced an attractive offer for pre-paid customers and 20% of BPL post-paid custom-
ers accepted this new offer. Then what would be the ratio of the numbers of pre-paid and post-paid
customers for Reliance?
1) 5 : 13 2) 5 : 7 3) 7 : 9 4) 7 : 13
Directions (Q. 6-10): Refer to the bar-graphs below and answer the questins that follow.
Movement of share prices of four companies in five trading sessions and the volumes on these
sessions.
Share Price
70
65
Goetze Raasi Cement Kesoram Cement Tata Telecom

K
60
55
(Rs.)

50
45
40
35
31.7.97 1.8.97 4.8.97 5.8.97 6.8.97

Volumes
20000

N KUNDAN
18000
16000
(Units)

14000
12000
10000
8000
31.7.97 1.8.97 4.8.97 5.8.97 6-8.97

Goetze Raasi Cement Kesoram Cement T ata T elecom

6. Which of the following scrips shows the highest increase in the share price between 31/7 and 6/
8 and by how much?
1) Goetze  9.5 2) Raasi  11
3) Tata Telecom  7 4) Kesoram  10
7. Which of the following scrips clocks the highest turnover on 31/7?
(Turnover = Volume × Share price)
1) Goetze 2) Raasi 3) Kesoram 4) Tata Telecom
8. Which of the following sets of scrips has shown an increase in volumes from 31/7 to 6/8?
1) Goetze, Raasi 2) Raasi, Kesoram
3) Kesoram, Goetze 4) Raasi, Tata Telecom
9. What is the percentage change in the turnover of Tata Telecom from 31/7 to 6/8?
1) -5% 2) -9% 3) 5% 4) 9%
10. The price of which of the following scrips shows the highest percentage increase from 31/7 to 6/
8?
1) Goetze 2) Raasi 3) Kesoram 4) Tata Telecom
Directions (Q. 11-15): Refer to the pie-charts below and answer the questions that follow.
YEAR 1999-2000
BREAK-UP OF NATIONAL INCOME AND NATIONAL EXPENDITURE

RUPEE EARNED RUPEE SPENT


Indus trial
develo pment Adminis tr-
Agriculture External
14% atio n
23% bo rro wings 23%
9% Debt
s ervicing
8%

Internal
borro wings Direct taxes Defence
17% 18% 11%
Others

K
12%

Cus to ms
Excis e Agriculture
12%
21% 32%

debt servicing cost


11. If debt service ratio = , by what percentage should agricultural income rise
national income
(keeping all other incomes constant) so as to bring the debt-service ratio to 7.5% ?
1) 7% 2) 29% 3) 40% 4) 125%

KUNDAN
12. If it is known that out of total external borrowings, 65% is spent on industrial development, 28%
on defence, and the rest on debt servicing, then what is the component of foreign finance in
industrial development? (Assume rupees earned = rupees spent)
1) 74% 2) 53% 3) 42% 4) 27%
13. If the total collection by way of direct taxes in 1999-2000 was estimated at Rs 123,000 million, how
much extra money was pumped into the agricultural sector during 1999-2000? (Assume rupees
earned = rupees spent)
1) Rs 20,000 million 2) Rs 47,000 million 3) Rs 61,500 million 4) Rs 72,000 million
14. If the internal borrowings are to be decreased by 50% , by what per cent should the rupee earned
from other resources be increased if external borrowings cannot be increased?
1) 14.5% 2) 20% 3) 30% 4) 17%
15. If the sectoral allocation is to change so that our country spends as much on industrial develop-
ment as on agriculture (with total spending on industrial development and agriculture remaining
the same), then the allocation of agricultural sector should reduce by approximately
1) 28.13% 2) 17% 3) 36% 4) 28%

Answers and explanations


Number of BPL prepaid plan 2 customers 0.2  0.1 10
1. 2; = 
Number of Idea postpaid plan C customers 0.09  0.2 9
2. 2; Number of Airtel customers = 0.17 × 1850000
Total income of Airtel through SMS = 7 × 1.5 × 0.17 × 1850000  33,00,000
 Statement I is true.
Percentage of Dolphin pre-paid plan 1 customers = 15 × 0.15 = 2.25%
Percentage of AT&T post-paid plan C customers = 8 × 0.15 = 1.2%
 Statement II is false.
Amount collected by Reliance through SMS in a week (A) = 0.18 × 1850000 × 1 × 8 × 7
Amount collected by Orange through SMS in a week (B) = 0.13 × 1850000 × 1.5 × 3 × 7
A > B  Statement III is true.
Percentage of Orange post-paid customers = 0.13 × 40 = 5.2% .
Percentage of Idea pre-paid customers = 0.09 × 40 = 3.6%
 Statement IV is false.
3. 4; Number of BPL post-paid A customers = 0.2 × 0.35 × 1850000
Number of IDEA pre-paid plan 3 customers = 0.09 × 0.1 × 1850000
 Difference = (0.07 - 0.009) × 1850000 = 0.061 × 1850000  1.13 lakhs.
4. 1; Orange and BPL pre-paid customers send 4 SMS in a day.
3  100  4  60
 Average number of SMS sent in a day by Orange post-paid customers =  1.5
40
6  100  4  30 48
Averate number of SMS sent in a day by BPL post-paid customers = 

K
70 7
Income from Orange post - paid customers 15
.  15
.  13  40 13  90 39 1
 = 48   
Income from BPL post - paid customers 2400 80 2
 2.5  20  70
7
5. 3; In overall customer base, percentage of Reliance pre-paid customers = 18 × 0.35 = 6.3%
Percentage of Reliance post-paid customers = 18 × 0.65 = 11.7%
Increase in Reliance pre-paid customers in terms of percentage of overall customer base
70 20
= 20    2.8%
100 100

N KUNDAN
 New percentage of Reliance prepaid customers = 6.3 + 2.8 = 9.1%
91
. 7
 Ratio =  .
11.7 9
6. 2; By observation, Raasi Cement shows the highest increase in the share price.
7. 2; By observation, it is either Tata or Raasi.
Tata’s turnover = 20,000 × 42 = Rs 840,000
Raasi’s turnover = 15,000 × 58 = Rs 870,000
Hence, Raasi’s turnover is highest on 31/7.
8. 3; By observation, it is Goetze and Kesoram.
9. 3; Tata’s turnover on 31/7 was Rs 840,000. On 6/8, it was 49 × 18,000 = Rs 882,000.
882000  840000
Percentage change =  5%
840000
10. 3; By observation, it is either Raasi or Kesoram.
69  58
For Raasi, % increase =  19%
58
50  415.
For Kesoram, % increase =  20%
415
.
Debt servicing cost
11. 2; Debt service ratio =
National Income
8
To bring down the ratio from 8% to 7.5% , national income must rise by ,
7.5
16 1
ie  100  6.67%
times, ie by
15 15
All other income except Agriculture remains constant.
6.67
Agricultural income should rise by  100  29%
23
12. 3; External borrowing = 9% of resources available.
65% of external borrowings spent on industrial development
65
=  9%  5.85% of total resources.
100
Assuming the budget to be a zero-deficit one, ie rupee earned - rupee spent = 0, we find the
proportion of foreign finance (external borrowings) in industrial development
5.85
=  100  42% .
14
13. 3; Again, assuming a zero-deficit budget, 18% of rupee earned = 123,000 million.
9

K
Extra money pumped = 32% - 23% = 9% , which is  123,000 = Rs 61,500 million..
18
14. 1; Internal borrowings are decreased by 50% .
 Internal borrowings = 0.5 × 17 = 8.5
External borrowings cannot be increased.
8.5
 Rupee earned by other resources should increase by  100 = 14.5%
74
15. 1; Total spend on agriculture and industry = 32 + 14 = 46%
 If total spendings on industry and agriculture are the same then 23% will be spent on each.
9

KUNDAN
 Required reduction =  100  2813%
.
32
Practice Exercise 6
Directions (Q. 1-4): These questions are based on the graphs given below.
Percentagewise break-up of the shareholding pattern of United Lever
(for the period 20th Nov to 20th Dec 2003)

FII Others
4% 14%

GDR
General Motors
2%
(GM)
42%

Mutual

K
Funds/Banks
38%

GDR = Global Depository Receipts


FII = Foreign Institutional Investors
Market value of a United Level share from Market value of a General Motors share from
20th Nov to 20th Dec of 2003 (in $) 20th Nov to 20th Dec of 2003 (in $)
370 450

KUNDAN
350
400
330
350
310
290 300
270
250
250
230 200
20th Nov 27th Nov 4th Dec 12th Dec 20th Dec 20th Nov 27th Nov 4th Dec 12th Dec 20th Dec

1. What is the percentage increase in the value of GM’s holding in United Lever from 10th Dec 2002
to 10th Dec 2003, given that the value of GM’s holding in United Lever as on 10th Dec 2002 was
124.88 million dollars and the difference in the value of the shares of United Lever held in the
form of GDRs and that of those held by FIIs as on 15th Dec 2003 was 17.84 million dollars?
1) 100% 2) 200% 3) 300% 4) Cannot be determined
2. What is the number of shares of United Lever held by ‘Others’ as on 20th Dec 2003, given that the
difference in the value of the shares of United Lever held by GM and Mutual Funds/Banks as on
20th Dec 2003 was 250 million dollars?
1) 2.75 million 2) 3 million 3) 3.5 million 4) Cannot be determined
3. Which of the following statements are true as per the graphs given?
1) The market value of a United Lever share is directly proportional to the market value of a
General Motors share.
2) The peak value attained by the United Lever share is exactly 80% of the peak value of the
General Motors share during the period 20th Nov to 20th Dec 2003.
3) The percentage decrease in the value of the United Lever scrip from 12th Dec to 20th Dec 2003
is less than the percentage decrease in the value of the GM scrip from 27th Nov to 20th Dec
2003.
4) None of these
4. The number of shares held by Mutual Funds/Banks on 29th Dec is what % more/less than the
number of shares held by Others on 12th Dec in United Lever?
1) 39% more 2) 17% less 3) 24% more 4) Can’t be determined
Directions (Q. 5-9): These questions are based on the table given below.

Kerb
Fuel Price on Fuel
No. of Engine Max. Front Rear Boot Tyre Wt
tank Road (in consu-
Model cylin- displace- Power Suspe- Suspe- space Size (in (with
cap. Rs lakh mption
ders ment (cc) (bhp) nsion nsion (litres) mm) Fuel in
(litres) in Delhi) (km/L)
kg)
M800 8 796 39 60 H LS 120 90 1.8 20 740
Zen
Alto

K
Santro
Palio
Matiz
Indica
16
8
8
8
8
8
997
997
1016
1457
796
980
55
45
60
60
58
62
50
50
30
80
100
120
LS
LS
LS
LS
LS
LS
MS
MS
H
ITA
LS
MS
150
90
140
160
100
140
90
80
100
100
80
80
3.4
3
3.3
3.7
3.4
3
15
10
25
10
10
35
900
850
950
1200
1000
1400

KUNDAN
Siena 16 1600 100 140 MS ITA 250 90 4.5 10 1500
Corsa 16 1392 70 200 MS ITA 230 110 4.8 10 1400
Astra 16 1810 80 300 H MS 300 110 5.2 15 1500
Mondeo 16 2600 120 100 H MS 600 120 9.6 6 1750
Sonata 16 2600 120 250 MS ITA 750 120 13.5 8 2200
Ikon 16 2400 90 170 H MS 750 100 6.2 12 1300
Baleno 8 1900 65 290 MS MS 800 90 7.5 10 1750
Mercedes 16 3800 220 420 MS LS 1100 140 35 5 3000
Bolero 16 3500 140 300 LS LS 1600 150 5.3 8 3500
Armada 16 2500 90 250 ITA ITA 1800 160 7 14 2200
Corolla 16 2200 130 200 LS MS 800 120 18.6 15 1800
Octavia 8 2300 170 175 MS ITA 900 110 12.1 12 2000
H = Hydraulic; LS = Leaf Spring; MS = Mcpherson Struts; ITA = Independent Trailing Arm
Engine displacement (in cc)
5. If Cylinder size (in cc) = , what is the maximum cylinder size among
Number of Cylinders
the vehicles having a maximum power of less than 100 BHP?
1) 124.6 2) 237.5 3) 182.1 4) 287.5
6. What percentage of the cars given in the table cannot finish a rally race which involves 2 legs
each of length 900 km, given that each car is allowed to refuel only once during the rally after the
start (all cars start the rally with a full tank of fuel)?
1) 25% 2) 26% 3) 27% 4) 28%
7. How many cars here have a boot space-to-fuel tank capacity ratio of more than 2.9 and whose
price is less than Rs 8 lacs?
1) 9 2) 8 3) 7 4) 5
8. If all the cars are ranked on the basis of the number of revolutions of the tyre required to cover a
distance of 1 km (top ranker makes the least number of revolutions), then find the average fuel
consumption for the top six ranked cars?
1) 11.66 km/litre 2) 9.33 km/litre 3) 10.20 km/litre 4) 6.54 km/litre
9. What is the ratio of the number of vehicles which have a kerb weight of less than 1200 kg and also
use at least one leaf spring in their suspension to the number of vehicles whose engine displace-
ment (in cc) is more than their kerb weight (in kg)?
1) 5 : 14 2) 1 : 3 3) 2 : 5 4) 6 : 13
Directions (Q. 10-13): These questions are based on the table and the graph given below.

K
Statistics of Credit Card Expenditure (CCE) and Personal Consumption Expenditure (PCE) for
various countries
Country P CE (billion $) CCE as % of PCE*
India 320 0.5
South Korea 214 36.0
Taiwan 195 12.0
Indonesia 100 1.8

KUNDAN
Hong Kong 96 19.0
Thailand 64 4.0
Philippines 60 2.7
Malaysia 38 11.5
Singapore 12 40.0
Statistics regarding Credit Card usage in various countries
3000
Credit card spending per card

2500 2500

2000 2000
(in $)

1500 1500
1250
1000 750 1000
500 750
500 500

0
India Taiw an Indonesia Hong Thailand Philippines Malaysia Singapore South
Kong Korea
Nam e of country
* All cards are used only for PCE.
Note: PCE is part of Gross Domestic Expenditure.
10. Assuming that every card holder has only one card, what is the total number of credit card holders
in India?
1) 3.2 lacs 2) 4 million 3) 7.5 lacs 4) None of these
11. If the Gross Domestic Expenditure (GDE) of the country whose CCE as a percentage of PCE is the
highest, is Rs 2,400 billion dollars, then what percentage of the GDE of that country is spent
through credit cards?
1) 40% 2) 20% 3) 2% 4) 0.2%
12. What is the female population of Thailand given that the number of males and the number of
females in Thailand are in the ratio of 3 : 1 and the number of credit cards in Thailand is 40% of
the total population?
1) 2.56 million 2) 3.2 million 3) 0.8 million 4) Cannot be determined
13. What is the value of the global gold market provided that India accounts for a sixth of it and 20% of
the card spend in India is on purchase of gold, which in turn accounts for 5% of the total gold
purchased in India?

K
1) $ 22.6 billion 2) $ 60.5 billion 3) $ 38.4 billion 4) Cannot be determined
Directions (Q. 14-18): Refer to the chart below and answer the questions that follow.
The given diagram shows the number of hours taken to complete the various activities involved
in constructing a house. Also, the sequence of the letters shows which activity is preceded strictly
by which activity. For example, flooring takes 4 hours and can be done only after joisting.

KUNDAN RA
FT
E RS

14. If a certain sequence of operations must take exactly one day, what operation must follow ‘FOUND’?
1) Walls 2) Joists 3) Pour 4) Scape
15. In the sequence which takes one day, the process of roofing is replaced with the process of floor-
ing. What relation does the time taken now have with the time taken in the minimum time-
taking sequence?
1) Greater than the minimum time-taking sequence
2) Less than the minimum time-taking sequence
3) Both are equal
4) No relation
16. If A and I are always to be included, which of these are essential operations?
1) Dig 2) Found
3) Scape 4) There is more than one essential operation.
17. If all the operations are to be covered such that one always moves from node to node, and no
operation is repeated, then which node will one end at, if the conditions given in the question are
followed?
1) H 2) F 3) I 4) Such a sequence is not possible.
18. What is the time taken by the sequence requiring the maximum time as a percentage of the time
taken by all the operations together?
1) 48% 2) 50% 3) 46% 4) 44%
Directions (Q. 19-23): Refer to the charts below and answer the questions that follow.
Transport of goods in India in 2001 Commodities Transported
(in Rs crores)

Railw ays Minerals


23475 Others 45°
Water 85°
7328
Metal
Others Products
3577 80°

K Roads
28493
Air
3079
Food
Products
90°

Zonal Distribution of Railways


Northern Railway
White
Goods
60°

17%

KUNDAN
Western Railway 33%
Southern Railway 26%
Eastern Railway 15%
North-Eastern Railway 9%
19. What is the central angle corresponding to water in the pie-chart showing various modes of
transport?
1) 36° 2) 40° 3) 45° 4) 60°
20. What per cent of the total transport of goods in India is contributed by Northern Railway?
1) 6% 2) 5% 3) 7.5% 4) Cannot be determined
21. If 40% of white goods are transported by roads, what per cent of goods transported by roads consist
of white goods?
1) 26% 2) 55.5% 3) 38.5% 4) 15.5%
22. If half of the minerals are transported by Eastern Railway, what per cent of goods carried by East-
ern Railway consists of food products, white goods and metal products?
1) 26% 2) 74% 3) 48% 4) Cannot be determined
23. In 2002, due to an increase in economic activity, total transporation of goods increases by 7% but
the distribution of goods transported remains the same. Transport by Railways increases by 15%
and the share of Western Railway increases to 38% . If 60% of goods transported by Western Rail-
way consists of Food Products, what per cent of Food Products are transported by Western Railway?
1) 50% 2) 22% 3) 35% 4) 38%
Answers and explanations
1. 2; Difference in GDR and FII in 2003 is 17.84 mn dollars, which represents 2% of United Lever’s
total share value.
 1% = $ 8.92 mn
 100% of United Lever in 2003 = $ 892 mn
892  42
 GM’s holding in 2003 =  $ 374 .64 mn
100
374.64  124.88
Increase in value of GM’s holding from 2002 to 2003 =  100  200%
124.88
2. 3; Difference in the holding of GM and MF/Banks = 4%
4% of holding = $ 250 mn
250  100
 100% of holding =  $6250 mn
4
Value of 1 United Lever share on 20th Dec 2003 = $ 250
6250 mn $

K
 Total no. of shares of United Lever on 20th Dec 2003 =  25 mn
250 $
25  14
Number of shares held by others =  3.5 million.
100
3. 3; Clearly, the market value of United Lever is not in tandem with GM market value. Hence
choice (1) is false.
Peak value of United Lever is 330 as against peak value of GM, which is 400. The percentage is
more than 80% .
Hence choice (2) is also false.
The percentage decrease in United Lever from 12th Dec to 20th Dec

KUNDAN
80 8
   100  25 %
330 33
100
The percentage decrease of GM from 27th Nov to 20th Dec is  100 = 25%
400
United Lever scrip’s decrease is less than GM’s decrease.
4. 4; Since the total value of shares of United Lever on 20th Dec and 12th Dec is not given, hence
data inadequate.
5. 2; For Torque to be maximum the value of Engine Capacity should be as high as possible and the
number of cylinders as low as possible.
This is true in the case of Baleno.
1900
Torque =  237.5
8
6. 2; The total distance of 1800 km has to be covered using only one refuelling. For example for M800
Total distance travelled on Full Tank = 60 × 20 = 1200
 M800 can cover a distance of 2400 km on one additional fuelling.
The number of cars that can complete the rally is 14.
The number of cars that cannot finish the rally is 5.
5
 % of cars that do not finish the rally = 19  100  26 .31 %
7. 4; By inspection, the number of cars satisfying the condition is 5.
8. 2; The car that requires the least number of revolutions has the largest tyre size. Hence cars with
the maximum tyre size are ranked from 1-6. The cars that get this ranking are Armada (1),
Bolero (2), Mercedes E-320 (3). Corolla, Sonata and Mondeo have equal tyre size and hence
complete the top six.
14  8  5  15  6  8
Average fuel consumption = = 9.33 km/litre
6
9. 2; The number of vehicles where weight is less than 1200 kg and which use a Leaf Spring in their
suspension is 5. The number of vehicles whose engine displacement is more than their kerb
weight is 15.
 The ratio is 5 : 15 = 1 : 3
320  0.5
10. 4; Total amount spent through cards = = $ 1.6 bn
100
$1.6 bn
Average amount spent per card in India =
$500
16000  100000
Total number of cards =
500
Total number of cards = 3200000 = 3.2 mn

K
(1 bn = 1000 mn; 1 mn = 10 lacs)
11. 4; Country with highest card spend percentage is Singapore.
12  40
Total amount spent by cards =  $4.8 bn
100
4.8
Amount spent by cards as a percentage of GDE =  100  0.2%
2400
64  4
12. 3; For Thailand, amount spent by credit cards =  $2.56 bn
100

KUNDAN
$2.56 bn
 Total number of credit cards =
$2000
25600  100000
 Total number of credit cards = = 1280000 = 12.8 lacs
2000
Credit cards are 40% of total population
12.8  100
 Total population =  32 lacs
40
32  1
 Number of females =  8 lacs = 0.8 mn
4
320  0.5
13. 3; Total amount spent by card in India =  $1.6 bn
100
$1600 mn  20
Gold purchased through credit cards =  $320 mn
100
$ 320 mn is 5% of the Indian gold market.
320  100
 Total value of gold purchased in India =  $6400 mn
5
India is 1/6 of the Global Market.
 Global mrket = 6400 × 6 = $ 38400 mn = $ 38.4 bn
14. 3; The sequence which takes a total time of 24 hours is A – B – C – E – G – H – I.
In this path, the operation POUR follows FOUND.
15. 3; The given sequence takes 24 hours. If the process of roofing is replaced with that of flooring, it
now takes 21 hours. The sequence which takes minimum time takes 21 hours (A - B - C - E - I).
Thus, both are equal.
16. 4; If A and I are to be included, any sequence taken must always include ‘DIG’ and ‘FOUND’. Thus
there is more than one essential operation.
17. 4; A sequence which covers all operations in such a way that no operation is repeated is a se-
quence like A – B – C – E – C – D – F – H – G – E – I – H. However, the question specifies that E
cannot precede C.
18. 2; Time taken by sequence requiring maximum time = 26 hours. (A-B-C-E-G-H-I)
Total time for all operations = 52 hours.
26
Required percentage = × 100 = 50%
52
19. 2; Total goods transported by all modes = Rs 65952 crores
Central angle corresponding to ‘Water’ in the pie chart showing various modes of transport
7328

K
= × 360° = 40°.
65952
20. 1; Northern Railway forms 17% of total goods transported by railways in India.
17 23475  100
 Required percentage = 100  65952
= 6%

60 40
21. 4; White goods transported by roads    65952 = Rs 4396.8 crores
360 100

4396 .8
White goods as percentage of goods transported by road =  100 15 .5% .

KUNDAN
28493
22. 4; The composition of various goods transported by Eastern Railway is not given. Although the
question includes Food Products, White Goods and Metal Products, the share of ‘other’ goods is
not known.
23. 3; Total transport of goods = 65952 × 1.07 = 70568.64 crores
90
Transport of food products = 70568.6 × = 17642.16 crores
360
Transport of goods by Railways = 23475 × 1.15 = Rs 26996.25 crores
38
Transport of goods by Western Railway = 26996.25 ×  10258 .58 crores.
100
Food Products transported by Western Railway = 60% of 10258.58 = 6155.15 crores
6155 .15
 Percentage of Food Products transported by Western Railway =  100  35 % .
17642 .16
Practice Exercise 7
Direction (1-5): The bar graphs give the percentage increase in income and expenditure of
various types of banks during a period of 2 years. Refer to the graphs to answer the questions
that follow.

% increase in incom e over the last year % increase in expenditure over the last year
50
44
50 44 45 42
45 1998-99 1999-00
40 1998-99 1999-00
40 37
35
35
30
30
25
25 20
17 20 17
20 15 15
14 13 12
15 12 15 11
9
10 10
4
5 5

K
0 0
PSU Banks Old Pvt New Pvt Foreign PSU Banks Old Pvt Banks New Pvt Foreign
Banks Banks Banks Banks Banks

1. Let the income of the PSU Banks in 1997-98 be equal to Rs 500000 crores. If the expenditure of
PSU Banks in 1999-00 is equal to the income of PSU Banks in 1997-98, then the income of PSU
Banks in 1999-00 will be what per cent more than the expenditure of these Banks in 1998-99?
1) 32% 2) 44%
3) 56% 4) 64%
2. If the expenditure of Foreign Banks in 1997-98 is equal to their income in that year and is equal

KUNDAN
to Rs 30000 crores then, in 1999-00, what is the difference in income and expenditure for the
foreign Banks?
1) Rs 1500 crores 2) Rs 2000 crores
3) Rs 15000 crores 4) Rs 20000 crores
3. Let the income of New Pvt. Banks in 1998-99 be Rs 4000 crores. If the expenditure of New Pvt.
Banks in 1998-99 is the same as their income then the difference in incomes of New Pvt. Banks
in 1998-99 and 1999-00 will be what per cent of the difference of expenditures of New Pvt. Banks
in 1998-99 and 1999-00?
1) 52% 2) 76%
3) 84% 4) 118%
4. In 1998-99, if the income of PSU Banks is twice the expenditure of Foreign Banks then what will
be the ratio of the income of PSU Banks to the expenditure of Foreign Banks in 1999-00?
1) 1 : 2 2) 21 : 10
3) 5 : 1 4) 1 : 5
5. In 1998-99, if the income of Foreign Banks is four times their expenditure, then what will be the
ratio of the income to the expenditure of the Foreign Banks in 1999-00?
1) 1 : 4 2) 4 : 1 3) 5 : 1 4) 1 : 5
Directions (Q. 6-9): The bar graph shows the oil reserves and the line graph shows the esti-
mated number of years for which the reserves will last at the present oil extraction rates for
various countries. The pie-chart shows the % distirbution of the oil reserves regionwise. Refer to
the following graphs to answer the questions that follow.
K Latin America
9%

Europe
8%
7%
Reserves by region %

Africa
Asia-Pacific
4%

KUNDAN
1) 0.4
North America
6%
Middle East
66%

6. What is the ratio of oil extracted by Saudi Arabia to that by Mexico per year (in billion barrels)?
2) 2.67
3) 3.16 4) 15.23
7. If the oil reserves held by Saudi Arabia form 32% of the share of Middle East countries then what
percentage share of global oil reserves is in Iraq?
1) 20.92% 2) 16.82%
3) 12.08% 4) 9.6%
8. What will be the difference between countries extracting maximum amount of oil to those ex-
tracting minimum amount of oil? (in billion barrels)
1) 2.57 2) 2.85
3) 2.14 4) 1.98
9. The ratio of the number of years for which the reserves will last to the total available reserves in
billion barrels is maximum for
1) USA 2) Saudi Arabia
3) Azerbaizan 4) Iraq
Directions (Q. 10-15): These questions are based on the following table.
Expected market of four brands of automobiles in the four metros
(In terms of percentage of number of units sold)

M umbai Delhi Calcutta M adras


1996 1997 1996 1997 1996 1997 1996 1997
PAL-Peugeot 32 27 38 35 40 42 26 35
DCM-Daewoo 42 35 33 26 31 28 41 33
Opel Astra 19 27 24 32 23 23 29 26
Mercedes-Benz 7 11 5 7 6 7 4 6

10. What will be the average percentage increase in the market share of Opel Astra in the metros?
1) 25% 2) 9.5%
3) 16% 4) 5.5%
11. PAL-Peugeot’s sales in the four metros have been
I. always more than those of Opel Astra.

K
II. always maximum in Calcutta.
III. always increasing.
1) I only
2) I and II only
3) I, II and III
4) II only
12. If the combined sales of all four is expected to go from 11000 to 12500 in Mumbai between 1996
and 1997 the increase in Mercedes-Benz’s revenues will show a _____ per cent growth, if the price
were to go up by 12% in 1997.
1) 10% 2) 50%

KUNDAN
3) 75% 4) 100%
13. Given that the cost of a PAL-Peugeot and a DCM-Daewoo is same, an Opel Astra costs twice as
much as a PAL-Peugeot, and a Mercedes-Benz is twice as costly as a PAL-Peugeot, the net rev-
enue from the four metros was maximum for
1) PAL-Peugeot and DCM-Daewoo
2) Opel Astra
3) Mercedes-Benz
4) Cannot be answered
14. If in 1996 sales were 11000, 10000, 9000 and 8000 respectively in the four metros, then the
following are almost equal:
1) PAL-Peugeot sales in Mumbai and Calcutta
2) DCM-Daewoo sales in Delhi and Madras
3) Opel-Astra sales in Mumbai and Madras
4) Mercedes-Benz sales in Delhi and Calcutta
15. The combined share of Opel Astra in Mumbai and Madras is 22% . The sales volume of Opel Astra
in Mumbai is how many times that in Madras?
7 5
1) 2)
3 4
8
3) 4) None of these
5
Directions (Q. 16-20): These are based on the following pie-chart given below:
Contributions of different companies to the turnover of Prudential Group of companies

Prudential Prudential
Services Prudential Services Prudential
16% Bank 14% Bank
24% 23%

Prudential
Finance Prudential
19% Finance
26%
Prudential Prudential Prudential
Consultants Insurance Prudential Insurance
8% 33% Consultants 31%
6%

1990 1995
1990: Prudential Insurance turnover was Rs 27 bn.

K
16. If the turnover of Prudential Insurance remained almost unchanged, the group turnover in 1995
was almost Rs
1) 87 bn 2) 93 bn 3) 90 bn 4) 89 bn
Answer the questions 17 to 20 with reference to the data in question 16.
17. The turnover of Prudential Finance increased by
1) 20% 2) 30% 3) 45% 4) 60%
18. If 40% of Prudential Consultants’ revenue came from project exports, its income from domestic
activity was (in 1990)
1) 1 bn 2) 2 bn 3) 3 bn 4) 4 bn
19. The number of companies whose turnover decreased in 1995 as compared with 1990 was

KUNDAN
1) 1 2) 2 3) 3 4) 4
20. The company which showed the biggest percentage drop in turnover is
1) Prudential Consultants 2) Prudential Insurance
3) Prudential Services 4) Prudential Bank

Answers and explanations


1. 4; Income of PSU Banks in 1997-98 = Rs 500000 crores
 Income of PSU Banks in 1998-99 = 500000 × 1.15 = Rs 575000 crores.
And income of PSU Banks in 1999-00 = 575000 × 1.17 = Rs 672750 crores.
Now, expenditure of PSU Banks in 1999-00 = Rs 500000 crores.
500000
Expenditure of PSU Banks in 1998-99 = = Rs 409836 crores.
1.22
Income is more than expenditure by 672750 - 409836 = Rs 262914 crores.
(672750  409836) 262914
Required % =  100  100  64.15%
409836 409836
2. 2; Here, expenditure of Foreign Banks in 1997-98 = Rs 30000 crores.
Expenditure of Foreign Banks in 1998-99 = 30000 × 1.04 = Rs 31200 crores.
Expenditure of Foreign Banks in 1999-2000 = 31200 × 1.11 = Rs 34632 crores.
And income of Foreign Banks in 1997-98 = Rs 30000 crores.
Income of Foreign Banks in 1998-99 = 30000 × 1.09 = Rs 32700 crores.
Incomes of Foreign Banks in 1999-2000 = 32700 × 1.12 = Rs 36624 crores.
 required difference = 36624 - 34632 = Rs 1992 crores
3. 3;  Income of New Pvt. Banks in 1998-99 = Rs 4000 crores
 Income of New Pvt. Banks in 1999-00 = 4000 × 1.37 = Rs 5480 crores
Difference = 5480 - 4000 = Rs 1480 crores
Again, expenditure of New Pvt. Banks in 1998-99 = Rs 4000 crores
 Expenditure of New Pvt. Banks in 1999-00 = 4000 × 1.44 = Rs 5760 crores
Difference = 5760 - 4000 = Rs 1760 crores
1480
 Required percentage =  100 = 84
1760
4. 2; Let the expenditure of Foreign Banks in 1998-99 = Rs x crores
Then income of PSU Banks in 1998-99 = Rs 2x crores
Then income of PSU Banks in 1999-00 = 2x × 1.17 = Rs 2.34x crores
And expenditure of Foreign Banks in 1999-00 = x × 1.11 = Rs 1.11x crores
2.34x 2.1
 Required ratio = 1.11x  1 = 21 : 10.
5. 2; Let expenditure of Foreign Banks in 1998-99 = P
Then income of Foreign Banks in 1998-99 = 4 P

K
And income of Foreign Banks in 1999-00 = 4P × 1.12 = 4.48 P
Expenditure of Foreign Banks in 1999-00 = P × 1.11 = 1.11 P
4.48P 4
 required ratio = 1.11P  1
6. 2; Reserves of oil held by Saudi Arabia is for 88 years.
264
It means Saudi Arabia extracts  3 billion barrels of oil per year.
88
28 3
Similarly, for Mexico = 1.12 billion barrels of oil per year; thus required ratio is  2.67 .
25 112
.

KUNDAN
7. 4; For Saudi Arabia percentage of global oil reserves is

For Iraq it would be


2112
264
.
 120 = 9.6.
32
100
 66 = 21.12

8. 2; The oil extraction of Saudi Arabia is 3 billion barrels per year while that of Azerbaijan is 0.15
billion barrels per year. Hence the difference will be 2.85 billion barrels.
9. 3; The ratio of the number of years for which the reserves will last to the total available reserves
170
in billion barrels is maximum for Azerbaijan, ie  34 .
5
10. 3; Average increase in market share of Opel Astra:
27  19
Increase in Bombay =  100 = 42%
19
Similarly, Increase in Delhi = 33%
Increase in Calcutta = 0%
Increase in Madras = = -11%
Thus average increase = (42 + 33 + 0 – 11)/4 = 16% .
11. 4; Only statement II holds good as of all the metro sales of PAL, it is maximum in Calcutta.
12. 4; Sales of Mercedes-Benz in 1996 = 7% of 11000 = 770 and in 1997 = 11% of 12500 = 1375. Since
the prices increase by 10% , revenue in 1997 will be 1.12 × 1375 = 1540; a growth of 100% from
770.
13. 4; As the number of units sold is not known, the question can not be answered.
14. 2;

PAL Mumbai: 11000 × 32% = 3520 Calcutta: 9000 × 40% = 3600


DCM Delhi: 10000 × 33% = 3300 Madras: 8000 × 41% = 3280
Opel Mumbai: 11000 × 19% = 2090 Madras: 8000 × 29% = 2320
Mere Delhi: 10000 × 5% = 500 Calcutta: 9000 × 6% = 540

The closest is the sales of DCM.


15. 1; 22% is the weighted mean of percentage sale of Opel Astra in Mumbai and Madras respectively.
Mumbai Madras
19% 29%
22%
7 3
Required ratio of sales volume = 7 : 3.
7

K
 Sales volume in Mumbai is 3 times that in Madras.
16. 1; Prudential Insurance turnover = Rs 27 billion, which forms 31% in 1995.
100
Thus total turnover in 1995 =  27 = 87.09 = 87 bn.
31
17. 3; In 1990, turnover of Prudential Insurance = 27 bn, which forms 33% of total turnover.
100
Total turnover =  27 = 81.8 bn. = 82 bn.
33
Prudential Finance turnover increased by (26% of 87 - 19% of 82)/19% of 82 = 45% .

KUNDAN
18. 4; In 1990, income of Prudential Consultants =

Thus domestic income = 60% of 6.56 = 3.93 = 4 bn.

19-20: Decrease of Prudential Consultants =

Decrease of Prudential Services =


6.56
8
100

6.56  5.22
 82 = 6.56 bn

 100  20%

16% of 82  14% of 87
16% of 82
 7% drop.
Thus there is decrease in turnover of Prudential Consultants and Prudential Services only.
19. 2 20. 1
Practice Exercise 8
Directions (Q. 1-2): Refer to the pie-charts below and answer the questions that follow.

Revenue of Jet Airways 2000-2002

Year Total cost as a percentage of revenue Total revenue (Rs crore)

2000-01 85 12562

2001-02 94 15872

A break-up of airline costs


Aircraft spaces
Engine Insurance 4% Salaries & w ages Passenger
maintenance 1% 5% service fee
5% 8%

K
Landing, parking Financial charges
and other f ees 3%
4%
Of fice &
Catering administration
8% 9%
Selling distribution
4%
Depreciation
1%
Miscellaneous

KUNDAN
expenses
Aircraf t f uel Lease rentals 1%
36% 11%

Note: The break-up of the airline costs is the same in both the years.
1. What is the percentage change in the Landing, Parking and Other fees in 2001-02 over 2000-01?
1) 20% 2) 30% 3) 40% 4) 45%
2. What is the average cost of the Passenger Service Fee and Aircraft Spaces for both the years (in
Rs crores)?
1) 1385 2) 1536 3) 1825 4) 1575

Directions (Q. 3-6): Refer to the table below and answer the questions that follow.
Given below is the production cost and the price per unit of a product and also the number of units
produced at that cost and price.

Units 50 100 150 200 250 300 350 400

Price per unit (Rs) 22 19 17 16 14 12 11 10

Cost per unit (Rs) 21 18 16.5 15 13.25 11 9 8.5

Value = Quantity × (Price - cost)


3. For the quantity with how many units is the value highest and how much is it?
1) 200, 200 2) 400, 600 3) 350, 700 4) Can’t be determined
4. If the factory operated in such a manner that it produced 50 units on first day and then 100, 150,
....... 400 till the 8th day, what was the value (average) generated per unit of the product in the
period of 8 days?
1) Rs 1.23 2) Rs 1.46 3) Rs 1.09 4) Rs 1.14
5. What has been the average daily growth rate in the value for the given period (Use the data from
the previous question)?
1) 74% 2) 29% 3) 136% 4) 157%
6. Which of the following is not true?
1) The highest cost-to-price ratio is at a production of 150 units.
2) The average cost-to-price ratio for the given 8 levels is approximately 0.927 : 1.
3) Average value for the given 8 levels is approximately Rs 276.5.
4) All the above are false.
Directions (Q. 7-9): Refer to the charts below and answer the questions that follow.
Given below are two pie charts and a bar graph. The first pie chart gives the usage (in percentage)
of leading Indian toothpaste brands, and the second pie chart gives the usage of leading Indian mouth-
washes as a percentage of the total usage of all mouthwashes. The bar graph gives the sales as well
as usage of mouthwashes, toothpastes and dental powder as percentages of population in Kolkata.

K
Assume that the individual distribution of mouthwashes and toothpastes in the Others category is
very small. Do not assume that all graphs represent the same total number of people.

Usage o f To o thpaste s
Usage of M outhwashes
C OL VIC
3% 8%
B AL
TGV
3%
7%
CLO GN
3% 5%
OTHER S

KUNDAN
P EP GBH
82% 2% 4%
P RO OTHERS HG
2% P S U 67% J HL3%
1% 2%
NEE
2% CF
DW 2%
M IN 2%
2%

Total population = 50 million

Total population = 55 million

80 68
% of population

70
60 53.5
46
50
40 31.2
30 21.4
14.8
20
10
0
Mouthw ashes Toothpastes Dental Pow ders

Sales Usage
7. What is the ratio of the total usage of the top 4 brands of mouth washes to the total usage of
toothpastes in Kolkata?
1) 0.4 2) 0.5 3) 0.6 4) 0.7
8. If in Kolkata the sales of mouthwashes goes up by 20% , the sales of toothpastes goes down by 19%
and the sales of dental powder goes up by 24% , then by what per cent will the total sales (of
mouthwashes, toothpastes and dental powers) increase or decrease?
1) 16.4% decrease 2) 15.3% increase 3) 14.4% decrease 4) 14.8% increase
9. A number Ni is defined such that Ni is equal to the difference between the number of people using
the ith ranked mouthwash and the ith ranked toothpaste. The 1st ranked toothpaste/mouthwash
is the one that has the highest sales. Consider the ‘Others’ category as one category.
9

If  N i = number of people in Kolkata who are prospective dental powder users, then what is the
i 1
ratio of the number of prospective dental powder users to the current users of dental powder in
Kolkata? (If two ranks are same, consider the first one in the clockwise direction to be the higher
ranked one).
1) 0.39 2) 0.57 3) 0.23 4) 0.46

K
Directions (Q. 10-11): Refer to the charts below and answer the questions that follow.

Mid-level
Office Office staff
Helpers



Junior
Executives
Senior

KUNDAN
Executives

Distribution of workforce

Number of People (in hundreds)

50 50
44 45 45
40 40
35

2000 2001 2002 2003

Men Women

The above graphs are related to a company’s statistics of its workforce. The workforce and men-
women census for a given year is taken on 31st December in that year.
10. What was the net per cent increase in the number of women from the beginning of 2000 to that of
2003, and its ratio to the net per cent decrease in the number of men, for the same period?
1) 28.5% , 2.85 2) 28.5% , 2.14 3) 42.8% , 2.14 4) 42.8% , 2.85
11. While making the workforce distribution chart for 2002, John, a junior executive, noticed some-
thing interesting. He noticed that the percentages of senior executives, office helpers and mid-
level staff were in arithmetic progression. John also knew that people of his designation were
always one-fourth of the workforce. Find the value of , ,  ,  for the year 2002. John also knows
that there are 1125 senior executives in 2002.
1) (30, 90, 150, 90) 2) (45, 90, 135, 90) 3) (60, 90, 120, 90) 4) (15, 90, 165, 90)
Directions (Q. 12-13): The following table gives the number of households in a country during
the period 1970-90 and the pie charts below show the distribution of households based on the
number of children for the years 1970 and 1990. Refer to the table and pie charts to answer the
questions that follow.

No. of households No. of households No. of households


Year Year Year
(M illion) (M illion) (M illion)

1970 120 1978 132 1986 150

1972 123 1980 135 1988 155

1974 126 1982 140 1990 160

K
1976 129 1984 145

1970 1990

3
3 Children Children
No Child 25%
20%
35% No Child
40%

KUNDAN
2 Children
18%
2 Children
20%
1 Child
1 Child
20%
22%

12. Which of the following statements about the households is true?


1) There were more households with children in 1970 than in 1990.
2) There were more households with 3 or more children in 1970 than in 1990.
3) The number of households with two children decreased slightly from 1970 to 1990.
4) None of the above
13. Assuming that the average number of children per household in 1970 was 3, how many house-
holds were without any children in 1970?
1) 24 million
2) 42 million
3) 14 million
4) Cannot be determined

Directions (Q. 14-16): Refer to the following bar graph and pie chart to answer the questions
that follow.
The following bar graph shows the percentage of banks of different sectors providing Internet
Banking facilities (five types of services)
120
Rural Development Banks P rivate Sector Banks
100 Regional Banks Co-operative Banks

80

60

40

20

0
Online Loan Online Credit Card Online Deposit Online Corporate Online Bill
Application Application Application Cash Management P ayment

Number of banks offering Internet Banking service in India (Total Banks = 600)

K
Regional
Banks
Private 22%
Sector
Banks
40%
Rural
Develop-
ment Banks
Co- 20%
operative

KUNDAN
Banks
18%

14. What is the ratio of the number of Regional Banks providing Online Deposit Application to total
number of Regional Banks?
1) 9 : 7 2) 7 : 9 3) 3 : 4 4) 4 : 3
15. The difference between the number of Private Sector Banks providing Online Bill Payment and
the number of Regional Banks providing Online Corporate Cash Management is
1) 171 2) 152 3) 165 4) 185
16. If the names of Rural Development Banks and Private Sector Banks are interchanged, what will
be the difference in Rural Development Banks providing Online Credit Card Application after
interchanging with original value?
1) 30 2) 10 3) 12 4) 6
Directions (Q. 17-21): These questions are based on the following table which provides the
profile of the batch of students of 2001 at IIM Bangalore.
Sex
Male Female Total
147 33 180
Age
19-24 24-26 >26 Total
115 60 5 180
Qualification (Graduate in)
Engg/Tech Engg/Tech
Arts Commece Science Total
(Non IIT) (IIT)
27 43 27 52 31 180
Work Experience (in months)
0 < 12 12-24 24-36 36-48 > 48 Total
88 23 21 33 13 2 180
17. If all but 9 girls have no work experience, then the boys with no work experience at all form what
percentage of the total students?
1) 35.5% 2) 36.3% 3) 12.2% 4) 43.3%
18. If all the commerce and arts graduates in the batch take finance as their specialization, what
percentage of the batch comprises finance specialists?
1) 38.88% 1) 38.5% 3) 33.33% 4) Cannot be determined
19. If all the students with 3 years and more experience are engineering graduates and are more
than 24 years in age and all of them except 4 are from the IITs then what is the ratio of IITians to

K
non-IITians among the <3-year work experience category?
1) 11: 4 2) 5 : 12 3) 11 : 24 4) Indeterminate
20. If all those with a non-Engg background are less than 24 years in age, then what percentage of
those less than 24 years are engineers?
1) 21.6% 2) 15.6% 3) 18.4% 4) 10%
21. If the batch of 2002 has 21.2% more girls than the previous batch, 10% of the girls are from an
Engg (IIT) qualification and 20% of the girls are from Engg (non-IIT) background, then what is the
ratio of the non-Engg girls to the total strength of the batch?
1) 15.5% 2) 13.3% 3) 17.2% 4) Cannot be determined

Answers and explanations

KUNDAN
1. 3; Total cost for 2000-01 = 0.85 × 12562  Rs 10680 crore
Total cost for 2001-02 = 0.94 × 15872  Rs 14920 crore
 Landing, parking and other fees for 2000-01 = 0.04 × 10680  Rs 430
 Landing, parking and other fees for 2001-02 = 0.04 × 14920  Rs 600

 Percentage change =
170
430
 100  40%
2. 2; Total cost of aircraft spaces 0.04 × (10680 + 14920)  Rs 1024
Total cost of passenger fees 0.08 × (10680 + 14920)  Rs 2048
 Average = Rs 1536 crore.
3. 3; The values generated in all the quantities are
(22 - 21) × 50 = 50
(19 - 18 ) × 100 = 100
(17 - 16.5) × 150 = 75
(16 - 15) × 200 = 200
(14 - 13.25) × 250 = 187.5
(12 - 11) × 300 = 300
(11-9) × 350 = 700
(10 - 8.5) × 400 = 600
So, 350 and 700 is the answer.
4. 1; The total quantity produced in 8 days: 50 + 100 + ..... + 400 = 1800 units.
And the value generated = 50 + 100 + 75 + 200 + 187.5 + 300 + 700 + 600
2212.5
Average value generated per unit = Rs 2212.5 = = Rs 1.23.
1800
5. 4; From Rs 50 it went up to Rs 600 in given 7 days.
So, 1100% is the growth in 7 days.
1100
Average daily growth rate =  157%
7
16.5
6. 4; (1) is true. is the highest.
17
112.5 121
(2) is also true as the average cost is and average price is .
8 8
2212
(3) is also true as it is = 276
8
(4) is not correct.
7. 4; The top 4 brands of mouthwashes are VIC, TGV, GN and GHB, which account for a total of
24
8 + 7 + 5 + 4= 24% =  50 = 12 million.
100

K
31.2
Total usage of toothpastes =  55 = 17.16 million
100
12
 Required ratio =  0.699  0.7 .
1716
.
8. 2; Sales of mouthwashes goes up to 68 × 1.2 = 81.6%
Sales of toothpastes falls to 14.8 × 0.81 = 11.988%
Sales of dental powder goes up to 21.4 × 1.24 = 26.536%
Initial sales = 104.2%
New sales = 120.124%

KUNDAN
120124
.  104.2
 Percentage change =  100 = 15.28% .
104.2
9. 1; N1 (Others category) = 36 - 33.5 = 2.5 million
 67 
  50  33.5
 100 
N 2 = 4 - 1.6 = 2.4 million (COL and VIC)
N 3 = 3.5 - 1.2 = 2.3 million (BAL and TGV)
N 4 = 2.5 - 1.1 = 1.4 million (CLO and GN)
N 5 = 2 - 1.8 = 1.2 million (PEP and GHB)
N 6 = 1.5 - 0.8 = 0.7 million (PRO and HG)
N 7 = 1 - 0.7 = 0.3 million (NEE and JHL)
N 8 = 1 - 0.7 = 0.3 million (MIN and CF)
N 9 = 1 - 0.5 = 0.5 million (PSU and DW)
9

  Ni = 2.5 + 2.4 + 2.3 + 1.4 + 1.2 + 0.7 + 0.3 + 0.3 + 0.5 = 11.6 million
i 1
53.5
Current users of dental powder =  55 = 29.43 million.
100
11.6
The required ratio = = 0.39.
29.43
45  35
10. 1; Percentage increase for women =  100  28.5%
35
50  45 1
Percentage decrease for men =  100   100  10%
50 10
 Ratio = 2.85.
11. 2; Since the number of Senior Executives (S), the number of office helpers (H) and the number of
mid-level staff (M) are in AP, we can write:
S = a - d, H = a, M = a + d
As junior executives are one-fourth of the workforce,
 1 
  90 0    360 0 
 4 
 S + H + A = 360 - 90  3a = 270  a = 90   = 90°
1125 1125
Percentage of senior executives =  100   100  12.5%
5000  4000 9000
12.5

K
0
   360  45
100
 d = 45°  M = 135°. Hence, (2).
12. 4; Let us examine the statements
Statement 1: Number of households with children :
120  100  35
in 1970 : million  78 million
100
160  100  40
in 1990 : million = 96 million
100

KUNDAN
This statement is not true.
Statement 2: Number of households with 3 or more children:
120  25
in 1970 : million = 30 million
100
160  20
in 1990 : million = 32 million
100
Statement 3: Number of households with 2 children:
120  20
in 1970: million = 24 million
100
160  18
in 1990: million = 28.8 million
100
This statement is also not true. Hence statement 4 is true.
120  35
13. 2; Number of households without children in 1970 = = 42 million.
100
(The information that the average number of children per household in 1970 was 3 is not
required for answering the question).
75
14. 3; Number of Regional Banks providing Online Deposit Application =  132  99
100
99 9 3
Required ratio =   .
132 12 4
75 3
Quicker Approach: Required ratio = 
100 4
85
15. 1; Number of Private Sector Banks providing Online Bill Payment =  240  204
100
25
Number of Regional Banks providing Online Corporate Cash Management =  132  33
100
Required difference = 204 - 33 = 171.
16. 1; Number of Rural Development Banks providing Online Credit Card Application originally
5
=  120  6
100
Number of Rural Development Banks providing the same function after interchanging
15
=  240  36
100
 the required difference = 36 - 6 = 30.

K
17. 1; Total number of girls = 33
with work experience = 9
 No work experience = 24 girls
Boys with no work experience = 88 - 24 = 64
 percentage of boys with no work experience = 64/180 × 100 = 35.5%
18. 4; All commerce and arts graduates take finance as specialization but nothing is mentioned about
others. Apart from arts and commerce graduates, others may opt finance.
Hence, can’t be determined.
19. 2; Students with >3 years experience = 13 + 2 = 15
IITians with >3 years experience = 15 - 4 = 11
IITians with <3 years experience = 31 - 11 = 20

KUNDAN
Non-IITians with >3 years experience = 4
 Non-IITians with <3 years experience = 52 - 4 = 48
Answer = 20 : 48 = 5 : 12
20. 2; Non-Engg background = 180 – 83 = 97
Number of students who are <24 years old = 115
115  97
 Engineers =  100 = 15.6%
115
Choices are far apart, so you can rule out options (1), (3) and (4)
21. 4; Data regarding strength of boys in the batch of 2002 is missing. As a result, we cannot estimate
the total strength of the batch of 2002. Therefore, the answer cannot be determined.
Practice Exercise 9
Directions (Q. 1-4): The following bar graph shows the total Indian Export (in $ billion) during
a period of 8 months. The pie chart shows the breakup of this Export during this period. Refer to
the graphs to answer the questions that follow.

INDIA'S EXPORT IN $ BILLION


45
40
35
$ BILLION

30
25
20
15
10
5
0

K
APR MAY JUN JUL AUG SEP OCT N OV
MONTHS

COSM-
ETICS 11%

GEMS &
JEWELLERY OTHERS
30% 16%
7

KUNDAN
GARMENTS
TEXTILES
19%
24%

1. The exports of Textiles and Others in the month of July is approximately equal to the exports of
Gems & Jewellery and Others in the month of
1) April 2) August 3) October 4) November
2. What is the ratio of the exports of first four months to those of the last four months?
1) 1.12 2) 0.89 3) 1.5 4) 0.75
3. If the target for the export of Gems and Jewellery for the complete year is set at $120 billion, then
what should be the total value of exports of the same for the remaining four months (assume
April-March as the Financial Year)?
1) $ 46 billion 2) $ 38 billion 3) $ 32 billion 4) $ 44 billion
4. If the government charges 12% tax on all exports of Textiles and 15% on Gems and Jewellery,
what is the revenue earned from these sectors during the given eight-month period?
1) $ 15 billion 2) $ 18 billion 3) $ 22 billion 4) $ 25 billion
Directions (Q. 5-10): The following tables show the percentage distribution of India’s popula-
tion by age group and sex in the given years. Refer to the tables to answer the questions that
follow.
DISTRIBUTION OF INDIA’S POPULATION BY AGE AND SEX (1901-1971)
(All figures are percentages of Males/Females to respective total)
1901 1911 1921 1931
AGE
M ale Female M ale Female M ale Female M ale Female
GROUPS
0-4 12.5 13.3 13.3 14.3 12.1 13.2 14.7 16.0
5-9 14.0 13.8 13.8 13.8 14.8 15.0 13.3 12.8
10-14 12.7 10.9 11.7 10.0 12.5 10.8 12.0 11.2
0-14 39.2 38.0 38.8 38.1 39.4 39.0 40.0 40.0
15-24 16.5 17.2 16.7 17.6 16.0 16.8 17.9 19.2
25-34 17.2 17.5 17.2 17.5 16.9 17.3 16.4 16.2
15-34 33.7 34.7 33.9 35.1 32.9 34.1 34.3 35.4
35-44 12.6 12.2 12.6 11.9 12.6 11.9 11.9 11.0
45-59 9.9 9.6 9.9 9.4 10.1 9.5 9.9 9.4
35-59 22.5 21.8 22.5 21.3 22.7 21.4 21.8 20.4
60 and above 4.6 5.5 4.8 5.5 5.0 5.5 3.9 4.2
Total 100.0 100.0 100.0 100.0 100.0 100.0 100.0 100.0

K
1941 1951 1961 1971
AGE
M ale Female M ale Female M ale Female M ale Female
GROUPS
0-4 13.2 14.0 13.1 13.7 14.7 15.5 14.3 15.1
5-9 13.6 13.6 12.6 12.9 14.6 14.9 15.0 15.1
10-14 11.3 10.8 11.4 11.3 11.6 10.8 12.7 12.1
0-14 38.1 38.4 37.1 37.9 40.9 41.2 42.0 42.3
15-24 18.1 18.3 18.9 19.1 16.3 17.1 16.5 16.7
25-34 15.9 16.3 15.4 15.3 15.2 15.5 13.5 14.6
15-34 34.0 34.6 34.3 34.4 31.5 32.6 30.0 31.3

KUNDAN
35-44 12.1 11.6 12.0 11.3 11.4 10.6 11.4 11.0
45-59 10.9 10.5 11.1 10.6 10.7 9.8 10.7 9.4
35-59 23.0 21.1 23.1 21.9 22.1 20.4 22.1 20.4
60 and above 4.9 4.9 5.5 5.8 5.5 5.8 5.9 6.0
Total 100.0 100.0 100.0 100.0 100.0 100.0 100.0 100.0

5. For both the sexes in the given years, the most thickly populated age group is
1) 0-14 2) 35-59 3) 15-34 4) 15-24
6. The total number of times a 1: 1 ratio of male to female percentage is displayed in any age group
(all the mentioned groups) is
1) Four 2) Three 3) Five 4) One
7. If the population in 1961 was 400 crores, the difference between the number of males and females
is highest in which of the given age groups?
1) 0-4 2) 10-14 3) 35-44 4) 45-59
8. In the given tables, how many times is the percentage value for any group (in any year) above
20%?
1) 36 2) 44 3) 48 4) 50
9. Over the given period (1901 - 1971), the least number of males and females belong to which of the
following age groups?
1) 0-4 2) 45-49 3) 10-14 4) 60 and above
10. If the total population in the age group 0-4 years in year 1971 is 14.68% , find the sex ratio (ie ratio
of males to females) in year 1971.
1) 21 : 19 2) 37 : 29 3) 17 : 13 4) 29 : 27

Directions (Q. 11-13): The following bar graphs show the data regarding Export, Production
and Per Capita Availability of Coffee. Refer to the graphs to answer the questions that follow.

215 800

Production (million kg)


210 700
Export (million kg)

600
205
500
200
400
195
300
190
200
185 100
180 0
1980-81 1981-82 1982-83 1983-84 1984-85 1980-81 1981-82 1982-83 1983-84 1984-85

K
Years Years

700
Per Capita Availability

600

500
(in grams)

400

300

KUNDAN
200

100

0
1980-81 1981-82 1982-83 1983-84 1984-85

Years

11. If the area under coffee production was less by 10% in 1984-85 than that in 1983-84, then the rate
of increase in productivity of coffee in 1984-85 was
1) 4% 2) 27% 3) 2.3% 4) 13.6%
12. Which year, except 1980-81, represents the highest proportion of coffee exported by India out of its
production?
1) 1983-84 2) 1982-83 3) 1984-85 4) 1981-82
13. The population of India in 1983-84 was
1) 395 million 2) 790 million 3) 1,185 million 4) 670 million
Directions (Q. 14-17): Answer the questions on the basis of the information given below.
The table below provides certain results of a survey taken among 46 people. The parameters are:
gender, number of servants owned, and age of the participants. The first number in each cell is the
number of people in the that group. The minimum and maximum age of people in each group is given
in brackets. For example, there are 10 female participants with 0 servant and among these 10 the
youngest is 34 years old, while the oldest is 46.

No. of Servants M ale Female Total


0 2(38, 38) 10(34, 49) 12
1 2(32, 32) 16(35, 63) 18
2 4(32, 33) 4(27, 40) 8
3 4(32, 33) 4(27, 40) 8
Total 12 34 46
14. The percentage of participants aged less than 40 years is at least
1) 35% 2) 16.67% 3) 43% 4) 27%
15. Given the information above, the percentage of people older than 35 can be at most
1) 69.6% 2) 73.33% 3) 30% 4) 90%
16. The percentage of people that fall into the 35-to-40-years age group (both inclusive) is at least
1) 10.86% 2) 26.67% 3) 8.33% 4) 6.67%
17. The maximum no. of persons having at least two servants and age at least 35 years is
1) 2 2) 4 3) 6 4) 0

K
Directions (Q. 18-20): Answer the questions on the basis of the information given below.
The correspondence coming to a certain office can be classified under various headings. The
following table shows the percentage distribution of such correspondence over time. The total num-
ber of correspondence received during December 1998 was larger than the number received in June
1999. The total number of correspondence received during September 1998 was larger than the num-
ber received in March 1999.

Category Sep - 98 Dec - 98 M ar - 99 Jun - 99


Business 40 33 19 17
Feedback 25 30 37 44

KUNDAN
Government 11 19 5 17
Products 3 3 10 6
Advertisement 4 7 10 12
Stocks 5 6 11 2
Inventory 12 2 8 2
18. In which category was the percentage of correspondence increasing but at a decreasing rate?
1) Feedback 2) Stocks 3) Advertisement 4) Cannot be determined
19. In the Government category, the number of the correspondence received in December 1998 as
compared to June 1999
2) was larger 2) was smaller 3) was equal 4) Cannot be determined
20. In the Feedback category, the number of corespondence received in September 1998 as compared
to March 1999
1) was larger 2) was smaller 3) was equal 4) Cannot be determined

Answers and explanations


40
1. 3; Export of Textiles and Others in July =  32 = $12.8 billion
100
46
Export of Gems and Jewellery and Others in October = × 28 = $12.8 billion.
100
Quicker Approach: Now here the trick is to realize that since for Textiles and Others, we are
calculating 40% of $32 billion for July, the only month where we would get a near equal for
exports of Gems and Jewellery and Others, accounting to 46% of the export’s basket, would be
for a month in which the total exports is just near but less than that of July. Hence directly from
observation, we can mark on the month of October.
2. 1; Export for the first 4 months (Apr-July) = 35 + 22 + 42 + 32 = $ 131 billion
Export for the last 4 months (Aug-Nov) = (38 + 18 + 28 + 33) = $ 117 billion
 Ratio = 131/117 = 1.119  1.12.
3. 1; Gems and Jewellery export for the first 8 months (Apr-Nov) of the year = 0.3 × 248 = $74.4 billion.
Total value of the exports for the remaining 4 months (Dec-Mar) of the year
= Target - $74.4 billion = $120 - $74.4 = $45.6 billion  $46 billion.
4. 2; Textile exports over the period = 0.24 × 248 = $59.52 billion.
Gems and Jewellery exports over the period = $74.4 billion
Revenue from Textile exports = 0.12 × 59.52 = $7.14 billion
Revenue from Gems and Jewellery exports = 0.15 × 74.4 = $11.16 billion.
Total revenue from these sectors = 7.14 + 11.16 = 18.3  $18 billion.
5. 1; 0-14 years is made up of three groups, viz 0-4, 5-9, 10-14. It gives the maximum percentages.
6. 1; If we refer to the table, we get the ratio 1 : 1 between male and female 4 times.
In 1911, age group 5-9.

K
In 1931, age group 0-14.
In 1941, age groups 5-9 and 60 & above.
7. 4; In 1961, for 0-4 group, difference = 15.5 - 14.7 = 0.8
For 10-14 group, difference = 11.6 - 10.8 = 0.8
For 35-44 group, difference = 11.4 - 10.6 = 0.8
But for the age-group 45-59, it is more than this value.
8. 3; It is more than 20% for age groups 0-14, 15-34 and 35-39, for males and females in all the years,
i.e. 8 × 3 × 2 or 48.
9. 4; Clearly, the least number of males and females belong to age group 60 and above.
10. 1; Let the total population of males and females in 1971 be X and Y respectively.

KUNDAN
According to the question,
14.3% of x  15.1% of y
 100  14.68
x y
 14.3x  15.1y  14.68x  14.68y
 0.42y  0.38x
x 0.42 21
  
y 0.38 19
Quicker Method: By the method of alligation,
14.68% is the weighted mean of 14.3% and 15.1% .
M F
14.3 15.1
14.68
0.42 0.38
 21 : 19
Total coffee production
11. 4; Productivity = Area under production . Let area under production in 1983-84 be x hectares.

645
 Productivity in 1983-84 = .
x
660 733
Productivity in 1984-85 = [Area is less by 10% ] = .
0.9x x

733 645

x x  88  100  13.6%
Rate of increase in productivity in1984-85 = 645 645 .
x

Exports in that year


12. 1; Proportion of coffee exported in any year = Production in that year .

Calculate and check with the value to get the answer as 1983-84.
13. 2; Per capita availability in 1983-84 = 545 gram = 0.545 kg.
Total production of coffee - export of coffee
Per capita availability = total population

645 - 210 millionkg


Total population = = 798 million (approx).

K
0.545kg
Option (2) is closest to this value.
14. 1; Minimum number of participants aged less than 40 = 2 + 1 + 2 + 1 + 4 + 1+ 4 + 1 = 16
16
 Percentage =  100  35% .
46
15. 1; Maximum number of participants older than 35 = 2 + 9 + 15 + 3 + 3 = 32
32
 Percentage = 46 × 100 < 70% .

KUNDAN
16. 1; Minimum number of participants that fall into 35 to 40 years age group = 2 + 1 + 1 + 1 = 5
5
 Percentage =  100 = 10.86% .
46
17. 3; Maximum no. of participants having age at least 35 years and at least 2 servants = 3 + 3 = 6.
18. 3; By observation we can say that the percentage of correspondence is increasing for both Feed-
back and Advertisement, but it is increasing at a decreasing rate for Advertisement.
19. 1; Correspondence in Government category for December 1998 = 0.19 × [December 1998 total].
Correspondence in Government category in June 1999 = 0.17 × [June 1999 total].
19% of December 1998 total > 17% of June 1999 total.
 Correspondence in Government category was greater in December 1998.
20. 4; Number of correspondence in September 1998 > March 1999.
For feedback in September 1998 = 0.25 × [September 1998 total]
For feedback in March 1999 = 0.37 × [March 1999 total]
Since we de not know the exact amount of correspondence in both the categories, we cannot
compare the values.
Practice Exercise-10
Directions (Q. 1-3): Answer the questions on the basis of the information given below.
The data below shows the rankings of twenty-five companies on the basis of various parameters as
shown below.

Total Income Net Profit Net Worth NPM RONW ROCE


Rank Companies
2004 2004 2004 2004 2004 2004

2004 Rs Crore % chg Rs Crore % chg Rs crore % chg % % %

1 Tata Sons 4410.86 35.39 863.29 20.97 3396.38 25.91 19.57 25.42 14.55

2 Tata International 1970.32 -9.28 -830 — 108.15 -7.13 -0.42 -7.67 -2.92

3 Teech Pacific (India) 1772.39 40.95 26.9 -9.1 87.79 43.76 1.52 30.64 12.11

TV Sundram
4 1754.07 4.79 34.28 101.7 130.95 7.76 1.95 36.18 11.96
Iyengar & Sons

5 Nirma Consumer Care 1604.4 -12.76 0.39 77.27 3.13 13.82 0.02 12.46 0.99

6 Bennett, Coleman & Co. 1457.18 19.95 49.73 -75.85 910.14 5.52 3.41 5.46 5.46

7 Haldia Petrochemicals 1434.91 15.686 -501.55 — 614.09 -44.96 -35 -81.67 -9.24

8 Toyota Kirloskar Motor 1695.26 22.25 -102.01 — 452.38 -18.41 -6.02 -22.55 -12.25

9 Adani Wilmar 1166.78 179.11 11.63 5.73 58.42 58.84 1.00 19.91 11.67

10 Ford India 1064.79 — -60.52 — 321.87 -15.83 -5.68 -18.8 -3.87

11 Gitanjali Gems 1032.47 -74.87 21.9 -51.72 207.94 11.77 2.12 10.53 4.71

12 Hero Cycles 985.04 5.38 68.56 83.46 264.36 24.06 6.96 25.93 16.4

13 Riddisiddhi Bullions 952.67 130.67 0.21 505.5 1.85 190.41 0.02 11.55 3.12

14 Samsung Electronics 941.81 44.36 4.91 -4.11 34.94 16.36 0.52 14.06 6.43

15 Godrej & Boyce Mfg Co. 889.54 5.07 11.45 -5.99 280.65 3.75 1.29 4.08 1.57

16 Allanasons 562.87 -0.4 4.78 33.87 64.51 1.82 0.55 7.42 5.75

17 Honda Siel Cars India 855.45 18.89 26.48 — 235.3 12.68 3.1 11.25 5.22

18 Bharat Aluminium Co. 826.96 -17.88 18.76 — 588.06 -2.98 2.27 3.19 2.42

19 Lafarge India 816.46 69.31 -14.39 — 649.73 137.02 -1.76 -2.21 -1.06

Reliance Ports &


20 806.67 49.23 -20.97 — 491.38 -4.09 -2.6 -4.27 -0.69
Terminals

Tractors & Farm


21 805.42 -6.13 23.83 -0.96 416.18 5.16 2.96 5.73 4.51
Equipment

22 Bhushan 805.12 34.38 26.18 13.28 240.84 12.16 3.25 10.87 4.06

23 Jaypee Cement 1200.44 117.88 63.39 — 245.05 26.05 5.28 25.87 6.88

24 Essar Power 1140.51 -15.21 62.39 -34.35 781.71 21.29 5.47 7.98 2.77

25 Bharti Cellular 725.73 38.96 98.63 16.05 287.89 42.15 13.59 34.26 4.64

1. In how many companies was the percentage change for net worth greater than the percentage
change for total income?
1) 13 2) 20
3) 12 4) 16
2. Which of the following is true?
1) The percentage ROCE for the top five companies is always less than the percentage RONW.
2) The percentage ROCE for the top five companies is always greater than the percentage RONW.
3) The percentage ROCE for the top five companies is greater than the percentage RONW for only
one company.
4) None of the above is true.
3. Which of the following statements is not true?
1) As many companies are ranked above RiddhiSiddhi Bullions as below it.
2) The net worth as a percentage of total income for Hero cycles is 26.8%
3) If the top ten companies were ranked on the basis of net profit, from highest to lowest, the fifth-
ranked company would be Adani Wilmar.
4) All are true
Directions (Q. 4-6): Answer the questions on the basis of the information given beow.
Each point in the graph below shows the profit and turnover for a company. Each company belongs
to one of the three industries: Service, Shipping and Banking.
Profit

1000

4. For how many companies does the profit exceed 10% of the turnover?
1) 8 2) 7
3) 6 4) 5
5. For how many banking companies with a turnover of more than 2000 is the profit less than 300?
1) 0 2) 1
3) 6 4) 7
6. An investor wants to buy stocks of only banking or shipping companies with a turnover of more
than 1000 and profit exceeding 10% of turnover. How many choices are availiable to the investor?
1) 6 2) 7
3) 4 4) 5
Directions (Q. 7-9): Answer the questions on the basis of the information given below.
The following table shows the data about ages, height and weight of randomly selected 100 chil-
dren of CATMOS Montessori. Table 1 provides data about ages of the children. For the age given in the
first column, the second column gives the number of children not exceeding that age.
For example, the first entry indicates that there are 9 children aged 4 years or less. Tables 2 and
3 provide data on the heights and weights respectively on the same group of 100 children in a similar
format. Assuming that an older child is always taller and weighs more than a younger child, answer
the following questions.
TABLE 1 TABLE 2 TABLE 3
Age (years) Number Height (cm) Number Weight (kg) Number
4 9 115 6 30 8
5 12 120 11 32 13
6 22 125 24 34 17
7 35 130 36 36 28
8 42 135 45 38 33
9 48 140 53 40 46
10 60 145 62 42 54
11 69 150 75 44 67
12 77 155 81 46 79
13 86 160 93 48 91
14 100 165 100 50 100

7. What is the number of children of age 9 years or less whose height does not exceed 135 cm?
1) 48 2) 45 3) 3 4) Cannot be determined
8. How many children of age more than 10 years are taller than 150 cm and do not weigh more than
48 kg?

K
1) 16 2) 40 3) 9 4) Cannot be determined
9. Among the children older than 6 years but not exceeding 12 years, how many weigh more than 38
kg?
1) 34 2) 52 3) 44 4) Cannot be determined

Directions (Q. 10-12): Answer the questions on the basis of the information given below.
Nature’s Best Juices (NBJ) is in the business of manufacturing fruit juices. NBJ buys APPLE,
MANGO, GRAPE, ORANGE and LITCHEE juice. ORANGE juice can be made by adding artificial flavour
to APPLE and MANGO juice in equal proportions. Similarly, LITCHEE juice can also be made by APPLE
and GRAPE juice. Among other juices, NBJ sells MIXED JUICE (formed by mixing GRAPE and MANGO

KUNDAN
juice in the ratio 70 : 30), TROPICAL DELIGHT JUICE (formed by mixing equal amounts of ORANGE
and LITCHEE juice) and SUMMER SONG juice (formed by mixing equal amounts of ORANGE and GRAPE
juice). The following table provides the price at which NBJ buys the juices.
FLAVOUR Rs/Litre
APPLE 20.00
MANGO 25.00
GRAPE 15.00
ORANGE 22.00
LITCHEE 18.00
10. The cheapest way to manufacture TROPICAL DELIGHT juice would cost
1) Rs 19.50 per litre 2) Rs 19.75 per litre
3) Rs 20.00 per litre 4) Rs 20.25 per litre
11. SUMMER SONG can be manufactured by mixing
1) MIXED JUICE and APPLE in the ratio 14 : 10
2) MIXED JUICE and APPLE in the ratio 3 : 1
3) MANGO and LITCHEE in the ratio 1 : 1
4) APPLE, MANGO and GRAPE in the ratio 1 : 1 : 2
12. Assume that TROPICAL DELIGHT, MIXED JUICE and SUMMER SONG each sell for the same price.
Which of the three is the most profitable to the manufacturer?
1) TROPICAL DELIGHT 2) MIXED JUICE
3) SUMMER SONG 4) Data is insufficient
Directions (Q. 13-17): Answer the questions on the basis of information given below.
The following radar graph shows the percentage increase in the sale of companies A, B, C, D and E in
year 2004 with respect to previous year.
A
80

60

40
E B
20

D C

K
13. Which company has the maximum sale in year 2004?
1) A 2) C 3) E 4) Can’t be determined
14. The ratio of sale of companies A, B, C, D and E in year 2003 is 5 : 4 : 3 : 2 : 6. Find the overall %
increase in the sale of all the five companies together in year 2004.
1) 46% 2) 54% 3) 59% 4) 64%
15. The overall % increase in the sale of company C and company D together is 55% in year 2004.
Find the ratio of sale of company C and D in year 2004?
1) 80 : 13 2) 192 : 17
3) 84 : 31 4) 192 : 53
16. If the ratio of sale of company C, D and E in year 2004 is 5 : 3 : 4 then find the overall % increase

KUNDAN
in the sale of company C, D and E together from 2003 to 2004 (approx.).
1) 41% 2) 39% 3) 54% 4) 46%
17. The overall % increase in the sale of companies B & C together is 57% and that of companies C &
D together is 47% in year 2004. Find the ratio of sale of companies C, D and E in year 2003.
1) 3 : 7 : 23 3) 5 : 9 : 13
3) 6 : 7 : 11 4) None of these

Directions (Q. 18-20): Read the following information given below and answer the questions
that follow.
The Shyam Dairy company is setting up a plant for manufacture and sale of flavoured milk. The
investment of the plant is Rs 10 crores (to be invested in plant, machinery, advertising, infra-
structure etc).
The following table shows the cost of different bottle sizes.

Sale Dealer
Bottle size Bottling cost Cost of liquid Transportation cost
Price Margin
300 ml Rs 2 Rs 8 10 paise per bottle Rs 14 Rs 1.5
500 ml Rs 5 Rs 10 15 paise per bottle Rs 21 Rs 2
1.5 lit Rs 10 Rs 25 20 paise per bottle Rs 52 Rs 5
18. For which bottle should Shyam Dairy try to maximise sale to maximise its profit (assume that the
total number of litres of flavoured milk sold is constant irrespective of break-up of the sales in
terms of the bottle size).
1) 1500 ml bottle
2) 500 ml bottle
3) 300 ml bottle
4) Can’t say
19. If the company sells only 300ml bottles in the first year, how many bottles shoud it sell to recover
the investment made in the first year only?
1) 41.66  106 2) 35.84  10 6
3) 44.56  106 4) Can’t say
20. If the ratio of sales of 300ml bottles to that of 500ml bottles is 4 : 1, and there is no sale of 1500ml
bottles, how many 500ml bottles will be required to recover the investment?
1) 17.96  106 2) 24.8  10 6
3) 7.43  10 6 4) Can’t say

Answers and explanations

K
1. 1; The percentage change to net worth is greater than the percentage change for total income in
13 companies.
2. 3; The percentage ROCE for the top five companies is greater than the percentage RONW for only
the company Tata International.
3. 3; 1st to 5th are: Tata Sons, Bharti, Hero, Jaypee and Essar respectively.
4. 2; Draw a line from the bottom left corner to top right corner. The symbols lying above the line are
ones in which profit exceeds 10% of turnover. There are 6 such companies.
5. 3; Six companies lie below 300 profit and more than 2000 turnover.
6. 4; Only five companies fulfil the requirement.

KUNDAN
7. 2; Number of children aged 9 years or less = 48
Number of children having height 135 cm or less = 45
Thus 45 children satisfy both conditions.
8. 1;

Number of children
age > 10 years 40
height > 150 cm 25
weight > 48 kg 9

Thus required number of children = 25 – 9 = 16.


9. 3;

Number of children
12 years > age > 6 years 55
weight > 38 kg 67

Required number of children = 67 – 23 = 44.


10. 2; TROPICAL DELIGHT juice would cost minimum when its constituents have the minimum pos-
sible price. TROPICAL DELIGHT is made by mixing equal amounts of ‘ORANGE’ and ‘LITCHEE’.
We have the following possibilities:
From the table we have the minimum cost Rs 1975.
11. 4; The possible combinations for SUMMER SONG are given below.

Combination Ratio

K
ORANGE + GRAPE 1:1
(APPLE AND MANGO) + GRAPE 1:1:2
12. 2; From Q. 10 we have: the least possible price for TROPICAL DELIGHT is Rs 19.75 per litre.
Similarly, least possible price for MIXED JUICE is Rs 18 per litre (when GRAPE + MANGO are
mixed in the ratio 70 : 30). And least possible price for SUMMER SONG is Rs. 18.5 pre litre (when
ORANGE + GRAPE is mixed in the ratio 1 : 1).
Hence profitability is maximum for MIXED JUICE .
13. 4; Only the percentage increase in the sales of each of the company is given, not the previous
year’s sale. Hence data inadequate.

KUNDAN
14. 2; The overall % increase in the sales of all the five companies together
5 4 3 2 6
  80   50   60   30   40
20 20 20 20 20
 20  10  9  3  12  54 %
15. 1; 55% increase in the sales of company C and D together is the weighted mean of 60% and 30% .
C D
60% 30%
55%
25 5
5 : 1 (Ratio of sale in year 2003)
Sale of company C in year 2004
Now,
Sale of company D in year 2004

160
5
100  80
=
130 13
1
100
 Required ratio = 176 : 13
16. 4; Let K be present in each of the ratios.
2004 2003
100
C 5K 5K ×  3.12
160
100
D 3K 3K ×  2.3
130
100
E 4K 4K × = 2.8
140
Total 12K 8.2K
12K  8.2K 3.8K
The overall % increase =  100 =  100  46%
8.2K 8.2K
17. 1; Similar to solution of Q. 15.
B C C D
50% 60% 60% 30%
57% 37%
3 : 7 7 : 23
 B : C : D  3 : 7 : 23
18. 3; Profit from one 300ml bottle = 14 – (2 + 8 + 1.5 + 0.1) = Rs 2.4
Profit from one 500ml bottle = 21 – (5 + 10 + 0.15 + 2) = Rs 3.85

K
Profit from one 1500ml bottle = 52 – (10 + 25 + 0.20 + 5) = Rs 11.80
Selling 1500ml from 300ml bottles we get 2.4 × 5 = Rs 12 as profit.
Similarly, selling 1500 ml from 500ml bottles we get = 3.85 × 3 = Rs 11.55 as profit
Selling 1500ml from 1500ml bottle we get Rs 11.80 as profit.
Therefore, Shyam Dairy should maximise the production of 300ml bottles. (As the number of
litres is constant)
19. 1; Let no. of bottles it should sell be x.
Cost of production of x bottles = x (2 + 8 + 0.1 + 1.5) = 11.6x
Fixed cost = 108

KUNDAN
Now, to recover the cost
10 8  11.6x 10 8
 14  x   41.66  10 6 bottle
x 2.4
Quicker Approach: From solution of Q. 18.
Profit per bottle = Rs 2.4
108
 Total no. of bottles to sell =
2.4
20. 3; Let K be present in the ratio.
The overall selling cost of 300 ml and 500 ml (where they are sold in the ratio 4 : 1)
4  14  1  21 77
=   15 .4 .
4 1 5
Cost of production of 300ml bottle = Rs 11.6
Cost of production of 500ml bottle = Rs 17.15
4K  11.6   K  17.15   10 8
Now, 15.4 
4K  K
 77K  46.4K  17.15K  108  13.45K  10 8

108
K  7.43  10 6 bottles
13.45
Practice Exercise 11
Directions (Q. 1-6): These questions are based on the following information regarding the
price changes that a certain pharmaceutical company is considering for its products.

P roduct Existing Price (Rs.) Revised P rice (Rs.)


Antacid 1.50 2.50
Anti-Hypertensive 10.00 12.50
Expectorant 18.00/bottle 24.00/bottle
Anti-Asthmatic 20.00 26.00
Anti-Pyretic 5.00 8.00
Anti-Flatulent 7.50 9.00
The prices for all the products except Expectorant are the prices of 10 tablets.

K
1. A man is prescribed a combination of Antacid and Anti-Hypertensive in the ratio 2 : 3 for the first
week and of Anti-Hypertensive and Anti-Flatulent in the ratio 3 : 4 for the second week. The
purchased all the medicines under the existing price. His expenditure in the second week is
what % more than in the first week?
1) 24% more 2) 18% less 3) 26% more 4) Data Inadequate
2. If a family has a hypertensive and an asthmatic patient, where the person with hypertension has
to consume three tablets of Anti-Hypertensive per day and the asthmatic patient has to take two
tablets of Anti-Asthmatic every alternate day, what will be the increase in expenditure on the two
patients for 30 days?
1) Rs 40.50 2) Rs 42.75 3) Rs 46.50 4) Rs 38.50

KUNDAN
3. What is the percentage increase in the expenditure of a person for one year if he consumes 32
tablets of Antacid in one week?
1 1 2
1) 7 % 2) 6 % 3) 6 % 4) None of these
2 2 3
4. A person is prescribed to take two spoonfuls of Expectorant thrice everyday for a period of 20
weeks. Assuming that each bottle of Expectorant contains 90 spoonfuls, find the expenditure ac-
cording to the existing prices.
1) Rs 200 2) Rs 180 3) Rs 168 4) Rs 240
5. A person is prescribed a combination of Anti-Pyretic and Anti-Asthmatic such that he has to take
one of these before breakfast, the other after lunch and the one he had at breakfast after dinner
also; if he consumed an Anti-pyretic at the end of the dinner on the 7 th day of the course, he
started the course with
1) Anti-Asthmatic 2) Anti-Pyretic
3) Not possible to determine 4) None of the above
6. In the question no. (1), average cost per tablet for the first week is what % less than the average
cost per tablet for the second week?
1) 17.9% 2) 22.35% 3) 24.5% 4) Can’t say

Directions (Q. 7-11): These questions are based on the pie diagrams given below.
Shefali, a first-year student of management from a well-known institute of management in
western India, was doing her internship with a leading public sector bank in India. Her project
involved analyzing the market shares of various Indian companies that manufacture and sell
fuels and lubes. Halfway through her project, she managed to collect the following information
from the sales figures of various companies:

Percentage shares of various companies Percentage shares of various compani es


in total sales of l ubes (by val ue) i n total sal es of fuel s (by value)
P RL P RL
OT HERS
OT HERS 10% 10%
15%
25%

P HCL
20% P HCL
25%

OICL
OICL
30%
15%

P BCL

K
P BCL
30%
20%

Total sales of lubes for the year Total sales of fuels for the year
2000-2001 = Rs 22,400 crores 2000-2001 = Rs 11,200 crores
Shefali’s project guide, after reviewing the above information, pointed out the fact that the above
figures were inclusive of the considerable volumes of inter-company sales that occur every year.
Therefore the correct market shares of the companies should be arrived at after deducting the
inter-company sales figures from the present figures. Shefali then further collected the follow-
ing information regarding the inter-company sales.

KUNDAN
SELLER
PRL

OICL
Sale value as a percentage of the total sales of the selling company

Fuels
Lubes
Fuels
Lubes
PRL




OICL
50
30



PHCL
10

15
20
PBCL

10
20

OTHERS
40
60
10
40
PHCL Fuels — 20 — 20 20
Lubes — 10 — 25 40
PBCL Fuels — 30 10 — 35
Lubes — 15 5 — 25
OTHERS Fuels — 10 5 10 —
Lubes — 15 5 15 —
7. By approximately what percentage did Shefali overestimate the correct value of the total sales of
fuels?
1) 135% 2) 200% 3) 110% 4) 180%
8. If the correct sales figures are considered, then which of the following has the largest percentage
share by value of the sales of fuels and lubes put together?
1) PRL 2) OTHERS 3) OICL 4) PBCL
9. If for any company, Sales - Purchases = Profit, then neither fuels nor lubes were profitable for
1) PRL 2) PHCL 3) OTHERS 4) OICL
10. Assuming the information given in the above question to be true, which of the following had the
maximum profitability for fuels?
1) OICL 2) PHCL 3) PBCL 4) OTHERS
11. Which of the following had the second largest percentage share by value when the correct sales
figures of fuels and lubes put together are considered?
1) OTHERS 2) PHCL 3) OICL 4) PBCL
Directions (Q. 12-15): Refer to the pie-charts below and answer the questions that follow.
The following pie-charts represent the shareholding pattern of various investor groups in the
company XYZ Ltd as on 31.03.1999 and 31.03.2000 respectively.

SHAREHOLDING PATTERN

31-3-1999 FIIs 31-3-2000


15% FIIs
20%

K
P ublic
Public 30%
35%

Govt.
Institutions
25% Govt.
MF Ins titutio ns
10% 25%
MF Promoters P ro mo ters
15% 10% 15%

KUNDAN
Market Price = Rs 138 per share
Market Capitalisation = Rs 559 cr

31.03.2000?
1) 10% 2) 15%
Market Price = Rs 167 per share
Market Capitalisation = Rs 846 cr
Market capitalisation (market value) = Number of outstanding shares × Market price of share.

12. The number of outstanding shares have increased by what percentage from 31.03.1999 to

3) 25% 4) 30%
13. If you have more than 50% shareholding in a company, then you can control the management of
that company. Then, which of the following statements is are true?
In 1999 (i.e. the year ending on 31.03.99)
I. Management control can be with a coalition of two investor groups.
II. Management control can be with FIIs.
III. Management control can be with promoters.
1) I only 2) II only
3) III only 4) All three
14. Market value of shares held by FIIs has gone up by what percentage from 31.03.1999 to 31.03.2000?
1) 200% 2) 100%
3) 50% 4) 5%
15. If the FIIs together cannot hold more than 24% of outstanding shares, then what is the maximum
value of shares that the FIIs can purchase as on 31.03.2000?
1) 2 lakhs 2) 0.2 lakhs
3) 2 crore 4) 20 lakhs
Directions (Q. 16-19): Refer to the graph below and answer the question that follows.
XYZ Ltd manufactures locks.

97 160
Sales Expenses
96 140
95

Expenses (in Rs lakh)


120
Sales (in per cent)

94
100
93
80
92
60
91
90 40

89 20

88 0
1996 1997 1998 1999 2000

K
The above graph shows the sales revenues of XYZ Ltd. in terms of percentage of target achieved
and the expenses in Rs. lakhs for the years 1996 to 2000. The target sales were constant at Rs.
180 lakhs over the period.
16. In which year did the company earn the most profit?
1) 1996 2) 1997 3) 1998 4) 1999
17. What is the maximum drop in sales between any two consecutive years?
1) Rs 1.2 lakhs 2) Rs 4 lakhs 3) Rs 5.4 lakhs 4) Rs 6.3 lakhs
18. During the 5-year period, what is the highest ratio of sales to expenses?
1) 1.2 2) 1.9 3) 1.4 4) 1.64
19. What is the increase in sales (in Rs lakhs) from 1999 to 2000?

KUNDAN
1) 6.20 2) 10.80 3) 8.00 4) 8.64

Directions (Q. 20-24): Following graph shows the production at different plants (A, B, C, D) of
Torrent Ltd for four years. Ordinate is production figures in ’000 MTs.
180
160
140
120
100
80
60
40
20 A B C D
0
1993 1994 1995 1996

(i) Capacity utilization = Production × 100 / Capacity


(ii) Production is undertaken uniformly during the year.
(iii) Capacity of four plants (in ’000 MTs) in 1996 is as follows: A = 222, B = 160, C = 180, D = 190
(iv) Raw material availability at the plants in 1996 is to produce following quantities (’000 MTs):
A = 123, B = 148, C = 185, D = 198
20. Which plant has the highest capacity utilization in 1996?
1) B 2) A 3) D 4) C
21. Plant D operated only for 8 months in 1993. What would have been the capacity utilization in 1993
if the plant had run for the entire year? [Total capacity of D in 1993 = 190000 MTs]
1) 60% 2) 50.67% 3) 40% 4) 70%
22. If plant C operated at 60% capacity utilization in 1993 and 80% in 1994, what was the capacity
addition during that period?
1) 20 2) 15 3) 25 4) Nil
23. What is the maximum possible production, taking into account raw material availability con-
straint in 1996?
1) A-222, B-160, C-180, D-190 2) A-123, B-148, C-185, D-198
3) A-123, B-148, C-180, D-190 4) A-123, B-148, C-180, D-198
24. In 1996 bonus is to be given to employees of the plant which produces at least 25% of total com-
pany production and which achieves capacity utilization of more than that achieved by the whole
company. Which plants received the bonus in 1996?
1) C & D 2) B, C & D 3) D 4) None of these

Answers and explanations


1. 4; The constant present in the first week combination is either similar or different from the

K
constant present in the second week combination. Hence we can’t find the expenditures of first
and second weeks.
Hence data inadequate.
2. 1; Three tablets/day would mean the hypertensive has to be taken 90 times in 30 days.
The increase is of Rs 2.50 for 10 tablets
Hence increase will be of 2.50 × 9 = 22.50 for 90 tablets
Asthmatic has to consume 30 tablets.
 increase = 6.00 × 3 = 18.00
 total increase = 22.50 + 18 = 40.50.

KUNDAN
1.00 2 2
3. 4;  100   100  66 % .
1.50 3 3
4. 2; He has to consume = 2 × 3 × 7 × 20 = 840 spoonfuls
90 spoonfuls --- 1 bottle
840 spoonfuls ---- 10 bottles
1
(since he cannot buy 9 bottles)
3
Hence expenditure according to the existing prices = 10 × 18.00 = Rs 180.
5. 3; Since he may change the order in which he takes the tablets, it is not possible to determine.
2  1.5  3  10 1 33
6. 2; Average cost per tablet for the first week =  =  0 .66
23 10 5  10

3  10  4  7 .5 1
Average cost per tablet for the second week    0.85
34 10

0.85  0.66 1900


Required % =  100   22 .35 %
0.85 85
7. 1; To arrive at the correct value of the total sales of fuels the inter-company sales figures should
be subtracted from the present total sales.
To be subtracted from 100% :
PRL  50 + 10 +40 = 100% , i.e. has sold all its sales to other companies only  correct sales = 0%
OICL  15 + 20 + 10 = 45  (100 – 45)% of 30% = 16.5%
PHCL  20 + 20 + 20 = 60
 (100 - 60)% of 25% = 10%
PBCL  30 + 10 + 35 = 75
 (100 - 75)% of 20% = 5%
OTHERS  10 + 5 + 10 = 25
 (100 - 25)% of 15% = 11.25%

 Actual sales = (0 + 16.5 + 10 + 5 + 11.25) = 42.75% of given sales

100  42.75   100  135 %


 % by which total sales of fuels were overestimated =
42 .5
8. 3; The correct sales figures = (100 - % sales to other companies) × % share of total sales given
= (100 – M) × p (say)
 M should be minimum and p maximum. By observation, this is true for OICL in case of fuels
and for PBCL for lubes.
(Note that OTHERS and PRL are eliminated.)

K
Calculations between PBCL and OICL:
OICL = (100 – 45) × 30% × 25 + (100 – 60) × 15% × S
(where S = sales of lubes and since sales of fuels = 25)  45
For PBCL = (100 – 75) × 20% × 25 + (100 – 45)% × 30% × S  0.275S
Clearly, it is maximum for OICL.
9. 3; We need to find that company for which total sales are less than total purchases = S – P is
minimum (and –ve)
i.e. S is minimum and P is maximum.
S is from the pie chart and P is the sum of purchases obtained from the columns in the table.

KUNDAN
By mere observatio,
OTHERS have purchased far more than any of the rest.
And its sales are also minimal for both lubes and fuels.
10. 2; For maximum profitability, Sales - Purchases should be maximum.
 S – P must be maximum.
For ICL, the total purchases are 17.5% [i.e. 50% of PRL + 20% of PHCL + 30% of PBCL + 10% of
OTHERS]
 Profitability = 30 – 17.5 = 12.5%
Similarly, for PHCL, profitability = 25 – 8.25 = 16.75%
For PBCL, profitability = 20 – 12.5 = 7.5%
And for Others, there is negative profitability.
11. 1; The correct sales figures for fuels have already been calculated in solution (7) and in similar
manner we calculate those of lubes.
OTHERS  [100 -(15+5+15)] × 25% = 16.25%
PHCL  [100 - (10+25+40)] × 20% = 5%
OICL  [100 - (20+40)] × 15% = 6%
PBCL  [100 -(15+5+25)] × 30% = 16.5%
1
Now total sales of lubes = × that of fuels
2
 values for above are
OTHERS  8.125%
PHCL  2.5% (of total sales of fuels)
OICL  3%
PBCL  8.25%
The total correct sales value of fuels and lubes put together:
OTHERS  11.25 + 8.125 = 19.375%
PBCL  5 + 8.25 = 13.25%
OICL  16.5 + 3 = 19.5%
PHCL  10 + 2.5 = 12.5%
 OTHERS is second in terms of (correct) total value of sales of fuels and lubes put together.

559 560
12. 3; Number of outstanding shares as on 31.03.1999   = 4 cr.
138 140
846 850
Number of outstanding shares as on 31.03.2000 =  = 5 cr.
167 170
54 1
Therefore, percentage increase in outstanding shares   = 25% .

K
4 4
13. 1; It is clear that Public (35% ) and Govt Institutions (25% ) can form a coalition and control man-
agement.
14. 2; Market value of shares held by FIIs as on 31.03.1999 = 15% × 559  84
Market value of shares held by FIIs as on 31.03.2000  20 %  846  169
169  84 85
 Percentage increase =  100 =  100  100 % .
84 84
15. 4; Total outstanding shares as on 31.03.2000 = 5 cr. FIIs can buy 4% more as they already hold
20% of the shares, i.e.

KUNDAN
4
 5 cr = 0.2 cr = 20 lakhs.
100
16. 4; Calculating the sales revenue and the profit, year 1999 has the highest profits.
Year Revenue (Sales) Cost P rofit
1996 (93.2% of 180) 168 110 58
1997 (95.3% of 180) 170 150 20
1998 167 135 32
1999 164 100 64
2000 173 125 48
17. 2;

Sales Percentage of target Difference

1996 93.2
1997 95.3 2.10%
1998 93.1 -2.2%
1999 91.2 -1.9%
2000 96 4.80%

Since the year 1997-1998 has the largest fall of 2.2% of Rs. 180 lakhs, the value is Rs. 3.96
lakhs.
18. 4; Calculate from table in solution 16. Highest ratio is 1.64 for year 1999.
96  91 .2
19. 4; Increase in sales =  180 = Rs 8.64 lakh.
100
20. 3; Capacity utilization for the plants is
A = 75 × 100/222  33.8%
B = 135 × 100/160  85%
C = 145 × 100/180  80%
D = 170 × 100/190  90%
21. 1; Plant D produced 75 (’000 MTs) in 1993 in 8 months. If the plant had been operative throughout
the year it would have produced 12 × 75/8 = 112.5 (’000 MTs)
100
The capacity utilization = 112.5 ×  60%
190
22. 2;

Year % Capacity Utlz. Total Capacity

K
1993 60 85 × 100/60 140
1994 80 124 × 100/80 = 156.25
Increase in capacity  15
23. 3; Using both the constraints: Capacity of four plants: A = 222, B = 160, C = 180, D = 190; and Raw
material availability at the plants in 1996 is to produce following quantities (’000MTs): A = 123,
B = 148, C = 185, D = 198. Hence minimum value between Capacity and Raw material availabil-
ity is the maximum production possible. Hence answer option is (3).
24. 4;

KUNDAN
Plant Capacity considering constraints Actual Production % Utilization
A 123 75 61
B 148 135 91
C 180 145 81
D 190 170 90
Total 641 525 82
25% of Total 131.25
Only B & D satisfy the given condition.
Practice Exercise 12

Directions (Q. 1-6): These questions are based on the following information.
The bar graphs show the coal reserves of various nations in metric tonnes where as the line
graph shows the number of years up to which the reserves would last. Data for the bar graphs have to
be read from the left side of the Y-axis and those for the line graph have to be read from right side of

K KUNDAN
the Y-axis.

Africa Asia
8% Africa 16%
22%
Australia
America 8%
25%

Asia
53%
Europe
19%
Europe
8%
America
Australia 35%
6%

Share of the continents in the world Breakup of the share of various continents
population of 6 billion in the global coal resource
1. What is the expected average consumption of coal per annum per thousand people in India if
India’s population is 50% of Asia’s population? (Assuming that the world population remains con-
stant)
1) 3 gm 2) 3 kg 3) 0.003 gm 4) 30 kg
2. By how much (in metric tonnes) is the coal reserve of America more than that of Asia, if Kenya
and South Africa contribute to 37.5% of Africa’s coal reserves?
1) 524 2) 434 3) 334 4) 234
3. By how many kg is the annual average consumption of coal of Korea more than that of Libya?
1) 1450 2) 1740 3) 1850 4) 2050
4. If after 50 years, the world population increases by 12.5% (with each continent having the same
share of the world population), by how much per cent will the average per annum per capita
consumption of coal in Germany change, if it is known that out of every 10 persons in Europe,
approx 3.33 live in Germany? (Use data from previous question if required.)
1) 6.83% 2) 7.7% 3) 9.41% 4) 12.32%
5. If the European nations plan to sell off 41% of their coal reserves equally to the nations of other
continents (except to Africa), then the total coal reserves of India and New Zealand will increase
by what percentage? (Given that India has 13.8% of Asia’s coal reserves and New Zealand has
6.4% of Australia’s coal reserves. Use data from previous question if required.)
1) 20% 2) 25% 3) 27.23% 4) 31.62%
6. The difference between the ratio of coal reserves of Africa to those of Asia and the ratio of coal
reserves of Europe to those of Australia is
1) 1.21 2) 1.35 3) 1.0 4) None of these
Directions (Q. 7-12): These questions are based on the following information.
The table shows the numbers of persons commuting between different cities of India in five con-
secutive years (in thousands).
1991 1992 1993 1994 1995
Mumbai to Pune 9372 11252 6127 12345 9877
Delhi to Bangalore 10765 8328 7056 9362 13125
Kolkata to Raipur 12823 11675 13157 14106 16132
Pune to Delhi 7352 9137 11346 13451 15769
Chennai to Mumbai 8767 10789 12523 14323 16239

% share of various m odes of transport M odes of road transport

Rail
26.2% Taxi
Deluxe
22.7%
27.8%

Road Private
55.0% 16.9%
Govt Bus
Air 32.6%
18.8%

K KUNDAN
Modes of rail transport Percent share of various airline services

Special ANZ
Deluxe
7% Airw ay
17%
Superf ast 18%
Indian
33%
Airlines
Sahara
46%
Airlines
20%
P assenger
43% Jet Airw ays
16%
7. In 1995, what is the difference (in thousands) between the number of people commuting by De-
luxe buses and that of those by Jet Airways?
1) 8522 2) 10145 3) 9132 4) 8738
8. What is the percentage change from percentage increase in the number of persons going by
Deluxe train from Mumbai to Pune from 1991 to 1992 to the percentage increase in those going by
Indian Airlines from Pune to Delhi?
1) 10.21% 2) 20.21% 3) 15.42% 4) 30.42%
9. If a Sahara Airlines ticket from Chennai to Mumbai costs Rs 3800 and a deluxe train A/C ticket

K KUNDAN
from Pune to Delhi costs Rs 2375, what is the difference in revenues generated (in crore rupees)
by Sahara Airlines and Indian Railways on these routes over the period? (Assume that all trains
are run by the Indian Railways.)
1) 320 2) 291.5 3) 250 4) 190.9
10. What is the percentage increase in the number of people travelling by ANZ Airways over the
period?
1) 45% 2) 34% 3) 58% 4) 62%
11. What is the ratio of the number of people travelling from Mumbai to Pune (from 1991 to 1993) by
Deluxe Trains to that of those travelling from Chennai to Mumbai (from 1993 to 1995) by Indian
Airlines?
1) 0.41 2) 0.53 3) 0.32 4) 0.25
12. What is the highest percentage growth in the number of persons travelling by passenger train
between two consecutive years?
1) 20.89% 2) 26.64% 3) 32.12% 4) 30.21%
Directions (Q. 13-18): These questions are based on the following information.
The following graphs and pie chart indicate the economic condition of chatonline.com, a dotcom
company, over five years (1996-2000).
GROSS FIXED ASSETS TURNOVER
100
2000
80
1999
Rs in crores

60
Years

1998
40
1997
20
1996
0
1996 1997 1998 1999 2000
0 50 100 150
Years Rs crore

Gross profit for all five years (Rs 50 crore), before depreciation and tax

1996
2000 18%
28%
The net profit is calculated as (G – D – T)
where G = Gross profit
1997 D = Depreciation
15%
T = Tax
Depreciation and Tax are calculated on the
1999 Gross profit.
1998
22%
17%
13. If in 1996, there was 6.25% depreciation and 13.75% tax deduction, then the net profit in 1996 (Rs
crore) was
1) 5.21 2) 6.82 3) 7.29 4) 6.0
14. For which year was the ratio of turnover to gross fixed assets is the maximum?
1) 1997 2) 1998 3) 1996 4) 2000
15. What is the percentage increase in the ratio of gross fixed assets to gross profit from 1996 to 1999?
1) 96% 2) 135% 3) 236% 4) 310%
16. If in 1998, there was 9.65% depreciation and 11.63% tax deduction, then the ratio of net profit in
1998 to that in 1996 works out to
(Use data from previous questions if required.)
1) 0.92 2) 0.68 3) 1.82 4) 2.97
17. For which year was the ratio of turnover to gross profit the maximum?
1) 1996 2) 1998 3) 1999 4) 2000
18. For how many years is the ratio of (gross fixed assets + turnover) to gross profits lesser than the
succeeding year?
1) 1 2) 2 3) 3 4) 4
Directions (Q. 19-22): Refer to the tables below:
The following tables give information related to fertilisers (nitrogen and phosphate).

Fertilizer output in Nineties


Nitrogen Phosphates
Year Cap. util. (%) Cap. util. (%)
(lac tonnes) (lac tonnes)
1990-91 69.93 85.80 20.51 74.50
1991-92 73.01 88.50 25.62 93.10
1992-93 74.30 88.00 23.06 82.10
1993-94 72.31 83.90 18.16 64.30
1994-95 79.45 91.20 24.93 88.30
1995-96 87.77 96.90 25.58 90.60
1996-97 (Est.) 90.23 97.00 26.80 91.00

Year Fertilizer consumption (lac tonnes)


1990-91 125.46
1991-92 127.28

K KUNDAN 1992-93
1993-94
1994-95
1995-96
121.55
123.66
135.64
144.30
19. By what percentage was consumption of fertilizers greater than the production of fertilizers in the
year 1992-93?
1) 22 2) 23 3) 25 4) 26
20. What was the increase in total production capacity of phosphate fertilizers between 1991-97?
1) 1.5 lac tonnes 2) 2 lac tonnes 3) 2.5 lac tonnes 4) 1 lac tonnes
21. Total fertilizer production in 1995-96 was what percentage of total fertilizer consumption (approxi-
mately)?
1) 78 2) 79 3) 77.5 4) 78.5
22. Between 1990-96, production of nitrogen fertilizer was what percentage of total consumption of
fertilizers?
1) 50 2) 45 3) 60 4) 65

K KUNDAN
Answers and explanations
290
1. 1; Expected per annum consumption of coal in ndia = = 4.8 metric tonnes per year
60
Population of India = 50% of (53% of 6 billion)  1.6 billion
 expected average consumption of coal per annum per thousand people

 

 4.8 metric tonnes   4.8  10 6 kg 
 0.003 kg = 3 gm.
 1.6 billion   
1.6  10 9
2. 3; Total coal reserves of S Africa and Kenya = 90 + 60 = 150 metric tonnes
But 145 = 37.5% of Africa’s coal reserves

So, Africa’s coal reserves


150  8   400
metric tonnes
3
Again, 400 = 22% of global coal reserves
So, the global coal reserves = 1820 metric tonnes
 required difference = 19% of 1820 metric tonnes  346 metric tonnes
320
3. 2; Annual average consumption of coal in Korea = = 3.55 metric tonnes
90
190
Annual average consumption of coal in Libya = = 1.81 metric tonnes
105
 required difference = 1.74 metric tonnes = 1740 kg.
4. 2; Present population of Germany = 33% of (8% of 6 bn)  0.16 bn
Present average consumption of coal per annum per capita in Germany
145  10 6
=  0.013 gm.
70  0.16  10 9
145
Amount of coal consumed in Germany in 50 years =  50  103 .6 metric tonnes
70
So, amount of coal left = 41.4 metric tonnes.
After 50 years, population of Germany = (1.125) × (0.16)  0.18 bn
 41.4 
After 50 years, average per annum per capita consumption =  20  0.18  10 6  kg  0.012 gm.
 
 

 13  12  
 % change =  13  × 100% = 7.7%
 
5. 1; Total coal reserves of European nations = 19% of 1820  346 metric tonnes
Amount of coal sold off by European nations = 41% of 346  142 metric tonnes
So, increase in the coal reserves of the three continents (except Africa)
142
= = 47.33 metric tonnes.
3
Originally, the coal reserves of Asia = 291 metric tonnes
and the coal reserves of Australia = 146 metric tonnes
So, the original coal reserves of India and NZ = 13.8% of 291  6.4% of 146
 49.5 metric tonnes.
Now, the increased coal reserves of Asia = 291 + 47.33 = 338.33 metric tonnes
and the increased coal reserves of Australia = 146 + 47.33 = 193.33 metric tonnes
Now, the increased total coal reserves of India and N.Z = (13.8% of 338.33) + (6.4% of 193.33)
 59 metric tonnes.
 59 .43  49 .5  
So, the % increase in the total coal reserves of India and NZ =    20%
 49 .5 

19 22
6. 3; The required difference =  = 2.375 – 1.375 = 1.0.
8 16
7. 4; Total number of people commuting in 1995 = 71142.
Number of people commuting by road = 55% of 71142 = 39128.
So, the number of people commuting by deluxe buses = 27.8% of 39128 = 10878.
Number of people commuting by air route = 18.8% of 71142 = 13375.
So, the number of people commuting by Jet Airlines = 16% of 13375 = 2140.
Hence, the required difference = 10878 - 2140 = 8738.
8. 2; Number of persons going by deluxe train from Mumbai to Pune in 1991
= 17% of (26.2% of 9372) = 417.
Number of persons going by deluxe train from Mumbai to Pune in 1992
= 17% of (26.2% of 11252) = 501.
 % increase in the number of persons going by deluxe train from Mumbai to Pune from 1991
 501  417  
to 1992 = 
417   100% = 20.14% .
 
Number of persons going by Indian Airlines from Pune to Delhi in 1991
= 46% of (18.8% of 7352) = 636.
Number of persons going by Indian Airlines from Pune to Delhi in 1992
= 46% of (18.8% of 9137) = 790.
 % increase in the number of persons going by deluxe train from Pune to Delhi from1991 to
 790  636  
1992 = 
636   100% = 24.21% .
 
 24 .21  20 .14  
 % change in % increase =  
20.14   100% = 20.21% .

K KUNDAN

9. 2; Number of people travelling from Chennai to Mumbai by Sahara Airlines over the period
= 20% of (18.8% of 62641)= 2355 thousands.
Total revenue generated by Sahara Airlines over the period on the Chennai to Mumbai route
= Rs (3800 × 2355) thousand = Rs 8949000 thousand  Rs 895 crores.
Number of people travelling from Pune to Delhi by deluxe train over the period
= 17% of (26.2% of 57055)= 2541 thousand.
Total revenue generated by Indian Railways over the period on the Pune to Delhi route
= Rs (2375 × 2541) thousand = Rs 6034875 thousand  Rs 603.5 crores.
 the difference in the revenues generated = Rs 291.5 crores.
71142  49079
10. 1; Just simply calculate  100 = 45% (approx.)
49079
11. 3; Number of people travelling from Mumbai to Pune from 91 to 93 by deluxe trains
= 17% of (26.2% of 26751) = 1191 thousand (approx.)
Number of people travelling from Chennai to Mumbai from 93 to 95 by Indian Airlines = 46% of
(18.8% of 43085) = 3726 thousand (approx.)
1191
So, the required ratio = = 0.32
3726
12. 2; The highest % growth in the total number of people is from 93 to 94.

K KUNDAN
Number of people travelling by passenger trains in 1993 = 43% of (26.2% of 50209) = 5657
thousand
Number of people travelling by passenger trains in 1994
= 43% of (26.2% of 63587) = 7164 thousand
 7164  5657  
So, the required % growth = 
5657   100% = 26.64% .
 

 63857  50209  
OR just calculate 
50209   100 = 26.69%
 
13. 2; Gross Profit of the company in 1996 = 18% of 50 = Rs 9 crores
Net Profit of the company in 1996 = 9 – (6.25 + 13.75)% of 9 = Rs 7.29 cr.
14. 3
15. 4; Gross Fixed Assets of the company in 1996 = Rs 10 crores.
Gross Profit of the company in 1996 = 18% of 50 = Rs 9 crores.
10
So, the ratio of Gross Fixed Assets to the Gross Profit of the company in 96 = = 1.11
9
Gross fixed assets of the company in 1999 = Rs 50 crores.
Gross Profit of the company in 1999 = 22% of 50 = Rs 11 crores.
50
So, the ratio of Gross Fixed Assets to the Gross Profit of the company in 1999 =  4.5
11

 4.5  1 .11 
So, the required % increase =    100  310%
 1.11 
16. 1; Gross Profit of the company in 1998 = 17% of 50 = Rs 8.50 crore.
Net Profit of the company in 1998 = 8.5 (100 – 9.65 – 11.63)% = 6.7 cr
From soln (13), Net profit of the co. in 1996 = Rs 7.29 cr
 required ratio = 0.92
17. 2; We have to compare ratio of Turnover (T) to Gross Profit (GP) for the years mentioned in the
options.
The ratios for different years are as follows:
50 50
For 1996, required ratio = 
18 % of 50 18 x
65
Similarly, for 1997 
15 x
115
1998 
17 x
80
1999 
22x
110
2000 
28 x
It’s obvious that the ratio is maximum in 1998 because only it is more than 6.
18. 3; For the years 96, 97 and 99, the required ratio is less than the succeeding years.
19. 3; For 1992-93:
Production = 74.30 + 23.06 = 97.36
Consumption = 121.55
121 .55  97 .36
 Percentage lead of consumption over production =  100  25 %
97 .36
Production 20.51
20. 2; Production capacity in 1991 =   28
Capacity utilisation 0.7450

26 .8
Production capacity in 1997 =  30 .
0.91
Hence increase in capacity = 30 – 28 = 2.
21. 4; Total fertilizers production in 1995-96 = 87.77 + 25.58 = 113.35
Total consumption in 1995-96 = 144.36
113 .35
 required percentage = 144 .3  100  78 .5%
22. 3; Production of nitrogen fertilizers from 1990-96
= 69.93 + 73.01 + 74.3 + 72.31 + 79.45 + 87.77 = 456.77
Consumption of fertilizers from 1990-96
= 125.46 + 127.28 + 121.55 + 123.66 + 135.64 + 144.3 = 777.89
Production 456.77
 Required percentage = Consumption  100  777.84  100  60%

K KUNDAN
Practice Exercise 13
Directions (Q. 1-6): The following charts give data about the “total” and “segment-wise” mar-
ket shares of all the rubber companies in India, for the year 1995-1996.

All Segments 1995-96 Truck & Bus Segment


Ceat Others Ceat
16.5% 39.8% Others
13%
36.6%

MRF
17%

MRF
23.3%

K
Modi Apollo Modi
Apollo Rubber
Rubber 16%
8.3% Dunlop 8.3% 14%
Dunlop
3.8%
3.4%

Car Segment
Two- and Three-Wheelers
Ceat Others
21.3% 28.9%

KUNDAN
MRF Ceat
Apollo 22% 17%
2%

Dunlop
Modi 5%
Rubber
Modi Others
7.4%
Rubber 49%
MRF 5%
Dunlop
34.6%
Apollo 2.6%
5.2%

The total sales of Apollo in 1995-96 were Rs. 1245 crore; this was because of a 25% growth over the
previous year’s sales. Apollo’s sales in the car segment was Rs 208 crore. The truck and bus
segment conssitituted 40% of the total sales in 1995-96.
1. Apollo’s sales in the “two-and three-wheelers” segments was Rs
1) 82 crore 2) 75 crore 3) 70 crore 4) Can’t be determined
2. The sales in the truck and bus segment exceeded those in the car segment by
1) 50% 2) 75% 3) 100% 4) 150%
3. Which of the following had the maximum value of sales?
1) Apollo in truck & bus segment 2) Ceat in two- and three-wheelers
3) Ceat in car segment 4) Modi Rubber in all segments
4. The sales by MRF in the truck and bus segment was
1) Rs 840 crore 2) Rs 1384 crore 3) Rs 1020 crore 4) Rs 1395 crore
5. Apollo’s sales in 1994-95 was
1) Rs 996 crore 2) Rs 1156 crore 3) Rs 750 crore 4) None of these
6. If the sales of Car segment and Two- and Three-Wheeler segments are mixed, the overall share of
Ceat is 18.7% in year 1995-96. The total share of Two- and Three-Wheeler segment is what %
more/less than that of the Car segment?
1) 13.7% less 2) 44.4% more 3) 37.7% more 4) Can’t be determined

Directions (Q. 7-12): Refer to the charts below and answer the questions that follow.
Household Consumptions (%) during 2001-02
India Thailand Singapore
Food and beverages 48 37 14
Clothing 4 13 4
Rent and utilities 12 10 16
Health expenditure 5 7 6

K
Transport and communication 13 13 22
Education 4 9 17
Household equipment 3 8 7
All except above mentioned expenses are savings
21
No. of households (in crores)
18
Monthly income per household (in rupees '000)
15

KUNDAN
12

0
India Thailand Singapore

7. What is the difference between the average annual spendings per household on clothing in India
and in Singapore during 2001-02?
1) Rs 1200 2) Rs 9120 3) Rs 7920 4) Rs 28440
8. What percentage of average household savings in Singapore during 2001-2002 is the average
household savings in Thailand?
1) 10.7% 2) 22.1% 3) 32.1% 4) 50%
9. By what percentage is the average monthly spendings per household on education in Thailand
more than that on health expnditure in India during 2001-2002?
1) 80% 2) 140% 3) 280% 4) 584%
10. The number of households in Thailand is expected to grow by 15% next year while the average
monthly salary per household is expected to fall by 13% . If the percentage distribution of house-
hold expenditures remains the same, what will be the total monthly expenditure on rent and
utilities next year?
1) Rs 6920 crores 2) Rs 6540 crores 3) 7600 crores 4) Rs 8740 crores
11. Which of the following is true for the given three countries during 2001-2002?
1) Average monthly expenditure per household on food and beverages is maximum for India.
2) Total annual household expenditure on health is maximum for Thailand.
3) Total annual household expenditure on household equipment is maximum for India.
4) None of these
12. Find the percentage household consumption on education of India and Thailand together.
1) 7.1% 2) 6.8% 3) 8.2% 4) Can’t say

Directions (13-17): Refer to the bar graph below and answer the questions that follow.

248.23

K 12.47

KUNDAN
India’s domestic passenger car sales in January-December 2003; total = 4.84 lakh units
1. Maruti Udyog Ltd 2. Hyundai Motors India Ltd
3. Tata Motors Ltd 4. Ford India Ltd
5. General Motors Ltd 6. Honda Seie Cars India Ltd
7. Hindustan Motors Ltd 8. Fiat India Pvt Ltd
9. Toyota Kirloskar Motor Pvt Ltd 10. Daimler-Chrysler India Pvt Ltd

13. In 2003, how many units are sold by companies other than those given in the chart?
1) 2320 2) 1810 3) 3805 4) 2830
14. How many units of cars were sold by the given companies in the year 2002?
1) 480000 2) 420340
3) 382120 4) 342140
15. If annual percentage change remains the same for Ford India Ltd and Hindustan Motors Ltd for
the year 2004, then what will be the difference between the numbers of cars sold by these two
companies in year 2004?
1) 9740 2) 11400 3) 10739 4) 12742
16. In year 2002, how many companies sold more than 10000 cars?
1) 6 2) 5 3) 4 4) 2
17. What percentage of combined sales of General Motors Ltd, Fiat India Pvt Ltd and Tata Motors Ltd in
year 2003 is the combined sales of Hindustan Motors Ltd, Ford India Ltd, Hyundai Motors Ltd?
1) 100% 2) 80% 3) 120% 4) 95%
Directions (Q. 18-20): The following line chart shows the percentage increase in the sale of
companies A, B, C, D and E in year 2002 with respect to year 2001.
70
60 60
50
40 40
Value %

30 30
25
20
10
0
-10 A B C D E
-15
-20
Nam e of Com pany

18. The ratio of sales of company A, B, C, D and E in year 2001 is 5 : 4 : 3 : 2 : 6. Find the overall %
increase in the sale of all the five companies together.

K
1) 33% 2) 37% 3) 39% 4) 42%
19. If the overall percentage increase in the sale of companies B and C together is 19% in year 2002,
find the ratio of sale of companies B and C in year 2001.
1) 11 : 7 2) 34 : 21 3) 17 : 14 4) Can’t say
20. Which company has the maximum sale in year 2002?
1) E 2) B 3) D 4) Can’t say

Answers and explanations


1-6: In the beginning, please note that all the pie charts add up to hundred, so the shares given are
the per cent shares. Apollo’s share in all segments is 8.3% in 1995-96 = Rs 1245 crore.

KUNDAN
 Total sales in 1995-96 = (1245) (100)/8.3 = Rs 15000 crore;
Apollo’s share in car segment is 5.2% = Rs 208 crore.
 Total sales in car segment = (208) (100)/5.2 = Rs 4000 crore;
Total sales in trucks & buses segment = 40% of total sales = (40) (15000)/100 = Rs 6000 crore;
Total sales in two- & three-wheeler segment = (15000 - 4000 - 6000) = Rs 5000 crore.
1. 2; Apollo’s sales in two- & three-wheeler segment = (1.5) (5000) / 100 = Rs 75 crore.
2. 1; Sales in the truck & bus segment exceeded that in the car segment by = (6000 - 4000) (100)/
(4000) = 50% .
3. 4; Apollo’s sales in truck & bus segment = 16% of 6000 = Rs 960 crore.
Ceat’s sales in two- and three-wheeler segment = 16.9% of 5000 = Rs. 845 crore.
Ceat’s sales in car segment = 21.3% of 4000 = Rs. 852 crore.
Modi Rubber sales in all segments = 8.3% of 15000 = Rs 1245 crore
Among the above, Modi Rubber sales in all segments is the maximum.
4. 3; MRF sales in truck & bus segment = (6000) (17)/(100) = Rs 1020 crore.
5. 1; Apollo’s sales in 1994-95 = (1245)/(1.25) = Rs 996 crore.
6. 2; 18.7% is the overall share of Car segment and Two- and Three-Wheeler segment. Therefore as
per the method of alligation discussed in theory part:
18.7% is the weighted mean of 21.3% and 16.9% .
Car Segment Two- and Three-Wheelers Segment
21.3% 16.9%
18.7%
1.8 2.6
 9 : 13
Now, total sales of the Two- and Three-Wheeler Segment is greater than that of the Car seg-
ment.
13K  9K 400
Required % =  100   44.4%
9K 9
7. 3; Average annual spending per household on clothing in India during 2001-2002
4
=× 2500 × 12 = Rs. 1200.
100
Average annual spending per household on clothing in Singapore during 2001-2002
4
= × 19000 × 12 = Rs. 9120.
100

K
 Difference = 9120 - 1200 = Rs. 7920.
Alternative Method:
Since percentage shares of spending on clothes are the same,
4 4
difference = × (19000 - 2500) × 12 = × 16500 × 12 = Rs 7920.
100 100
8. 1; Household savings in Singapore and Thailand are 14% and 3% of household income respec-
tively.
 Ratio of total household savings in Thailand and Singapore during 2001 -2002
= Ratio of % of household savings × Ratio of average household income
3 9500 3

KUNDAN
=   = 0.107, i.e. 10.7% .
14 19000 28

9
9. 4; Average monthly spending per household on education in Thailand =  9500 = Rs 855.
100
5
Average monthly spending per household on health in India =  2500 = Rs 125.
100
855  125
 Required % =  100  584 %
125
 The former is 584% = (6.84 – 1) × 100) more than the latter.

10. 3; Number of households in Thailand next year = 8 × 1.15 = 9.20 crores.


Average monthly salary per household next year = 9500 × 0.87 = Rs 8265.
Total monthly expenditure on rent and utilities next year
10
= × 8265 × 9.20  Rs 7600 crores.
100
11. 4; Statement (1) is false as for Thailand, average monthly expenditure per household on food and
beverages is maximum.
Statement (2) is false as total annual household expenditure on health is maximum for Singapore.
Statement (3) is definitely false for India.
12. 1; Nos. of households in India and Thailand are in the ratio 17 : 8.
Monthly incomes per household in India and Thailand are in the ratio 2.5 : 9.5 = 5 : 19
17  5  0.04  8  19  0.09  100  3.4  13.68  100  7.2%
Required % =
17  5  8  19 237
13. 2; Total number of units sold by the given companies
= 248.23 + 91.63 + 76.22 + 14.04 + 12.47 + 12.38 + 10.07 + 8.34 + 7.69 + 1.12
= 482.19 = 482190
Total number of units sold = 484000.
Cars sold by companies other than those given in the chart = 484000 - 482190 = 1810.
14. 3;
Sales in 2002 (in '000)
Maruti Udyog Ltd 192.42
Hyundai Motors India Ltd 75.1
Tata Motors Ltd 51.85
Ford India Ltd 10.71
General Motors Ltd 5.74

K
Honda Seil Cars India Ltd 9.52
Hindustan Motors Ltd 13.25
Fiat India Pvt Ltd 22.54
Toyota Kirloskar Motors Pvt Ltd 0.26
Daimer-Chrysler India Pvt Ltd 0.73
Total 382.12
15. 3; Sales of Ford India Ltd in 2004 = 14.04 × 1.31 = 18.3924

KUNDAN
Sale of Hindustan Motors Ltd in 2004 = 10.07 × 0.76 = 7.6532
Difference = 18392 - 7653 = 10739.
16. 1; Refering to the table from soln of Q 14 we can find that only 6 companies managed to sell more
than 10000 cars in year 2002.
17. 3; In 2003, combined sales of Hindustan Motors Ltd, Ford India Ltd and Hyundai Motors Ltd
= 10.07 + 14.04 + 91.63 = 115.74
Combined sale of General Motors Ltd, Fiat India Pvt Ltd and Tata Motors Ltd
= 12.47 + 8.34 + 76.22 = 97.03
115 .74
Required % =  100 = 119.28  120%
97 .03
5 4 3 2 6 660
18. 1; Required percentage increase =  25   40   15   30   60   33 %
20 20 20 20 20 20
19. 2; 19% increase is the weighted mean of 40% and –15% . The base year is 2001.
B C
40% –15%
19
34 21
Required ratio of sales of companies B and C in year 2001 = 34 : 21.
20. 4; Since, sales of company in year 2001 is not given. Hence, data inadequate.
Practice Exercise 14
Directions (Q. 1-4): Refer to the charts below and answer the questions that follow.
Out of 100 people who took CAT last year, 3 got a call from IIM - A, 7 from IIM - B, 13 from IIM - C
and 17 from one or more of the other IIMs (L, K and I). Every person who had a call from IIMs A, B & C
also had a call from one of the other IIMs (L, K & I). Only one person was lucky enough to get a call from
A, B and C. 5 people had calls from both B and C, and nobody had a call from only A and C. 2 people had
calls from both A & B.
1. How many people had a call from IIM - C, but not from A or B?
1) 10 2) 9 3) 8 4) 7
2. How many people received a call from any one or more of the IIMs L, K & I but not from A, B and C?
1) None 2) 1 3) 3 4) Data insufficient
3. In all, how many people received calls from the IIMs?
1) 17 2) 18 3) 20 4) 33
4. What is the minimum number of people who have received calls from at least 3 IIMs?
1) 1 2) 2 3) 5 4) 6

K
Directions (Q. 5-8): Refer to the charts below and answer the questions that follow.
The following pie charts give the data regarding the world-wide tea and coffee production for the
year 1999-2000.

Tea in m n tonnes Coffee in m n tonnes


(Total 685 m n tonnes) (Total 408 m n tonnes)
Indo-
Indonesia
nesia
10%
4%
Brazil

KUNDAN
Sri Brazil 19%
Lanka 24%
Japan
22% Sri Lanka
3%
23%
Japan
India
3%
China India 21%
25% 22% China
24%

Price movement of tea and coffee (US $ per tonne)

2400
2200
2000
1800
1600
1400
1200
1000
May
Feb

Mar

Sep
Aug

Oct
Jun
Jan

Jul

Dec
Apr

Nov

Coffee T ea

5. The tea production of India exceeded its coffee production by ...... % .


1) 60 2) 54 3) 75 4) 82
6. The average value of Japan’s tea production during 1999-2000 was ....... (in billion US $).
1) 35.5 2) 26.25 3) 40.25 4) Can’t be determined
7. If 30% of the world’s tea and 20% of the world’s coffee was produced in the month of May then the
value of production of these two commodities for the month of May would be ....... (in billions of US
$).
1) 396 2) 356 3) 508 4) 459
8. Brazil’s tea production exceeded that of Indonesia and Japan put together by ........ % .
1) 200 2) 156 3) 240 4) 298
Directions (Q. 9-11) Refer to the charts below and answer the questions that follow.
Volum e grow th (%)
60
50
50 B - Segment C - Segment

40 42

30
25
20 15

K
10
10 6
0 0
2000 2001 2002 2003
-10
-15
-20

All the models under a particular segment follow the same trend as that of the entire segment.
C-Segment
M odel price (Rs lakh) 2001 2003

KUNDAN
Maruti Esteem
Tata Indigo
Ford Ikon
Hyundai Accent
Honda City
Mitsubishi Lancer
4.9
-
5.35
6.00
6.86
7.1
4.66
4.63
4.95
5.69
6.67
6.5
9. The volume of Ford Ikon cars is 670000 in 1999. Then, what is the collective cost of all Ford Ikons
in year 2001?
1) Rs 8044625 lakh 2) Rs 4480625 lakh 3) Rs 44506150 lakh 4) Rs 804462 lakh
10. If for Maruti 800, which is a segment B car, the volume is 530000 in 1999, then what is the
difference between the volumes of Maruti 800s and Ford Ikons in year 2002? (Refer data from
previous question.)
1) 186000 2) 219800 3) 161000 4) 147000
11. If the volume of B segment cars is the same as that of C segment cars in 1999 as well as in 2004
and there is no increase in the volume of B segment cars in 2004, then what must be the in-
crease in the volume of C segment cars in 2004?
1) 22.22% 2) 36.66% 3) 43.33% 4) 30.73%
Directions (Q. 12-15): Refer to the table below and answer the questions that follow.
The number of students in five Engineering Colleges P, Q, R, S and T is 20 boys and 20 girls each.
The table gives the average marks obtained by each boy and girl in five subjects from these colleges.
Subject M ax. Colleges
M arks P Q R S T
B G B G B G B G B G
Physics 200 145 170 160 150 120 130 165 170 155 160
Applied Mechanics 200 100 110 90 100 100 110 100 90 130 120
Mathematics 200 120 110 95 85 130 130 75 80 130 135
Computer 200 105 125 110 120 115 115 85 90 140 135
Electronics 200 100 100 100 70 110 100 100 110 120 130
12. In which of the following subjects did girls have the highest average marks?
1) Computer 2) Electronics 3) Mathematics 4) Physics
13. Which of the following Engineering colleges has the least pooled average (of boys and girls) in all
subjects?
1) S 2) P 3) Q 4) R
14. Which of the following Enginnering colleges has the highest difference between the marks scored

K
by the girls in Mathematics and that by the boys in Applied Mechanics?
1) Q 2) P 3) R 4) S
15. What was the difference between the Mathematics marks of boys of college P and girls of
college R?
1) 120 2) 130 3) 100 4) None of these
Directions (Q. 16-20): Refer to the table below and answer the questions that follow.
Salary range for an organisation for all male and female employees

Range (in Rs) M ales Females


Salary < 3000 70 45

KUNDAN
3000 < Salary < 5000 140 90
5000 < Salary < 8000 48 28
8000 < Salary < 12000 16 8
12000 < Salary < 20000 9 3
20000 < Salary 3 1
16. What is the approximate average monthly income of males in the organisation?
1) Rs 6248 2) Rs 5840 3) Rs 7102 4) Data Insufficient
17. What is the minimum average monthly income (approximately) of the female employees who do
not earn less than Rs 3000?
1) Rs 4077 2) Rs 3946 3) Rs 4388 4) Data Insufficient
18. What is the ratio of the maximum average monthly salaries of all the male employees to all the
female employees who earn equal?
1) 19 : 10 2) 5 : 4 3) 4.5 : 3.0 4) Data Insufficient
19. What per cent of the total employees earn less than Rs 12000 but more than or equal to Rs 5000 as
their monthly salary?
1) 17.4% 2) 21.7% 3) 19.8% 4) Data Insufficient
20. Which of the following cannot be deduced from the given data?
1) On an average, out of 66 employees in the organisation approximately 25 are females.
2) Less than 3% employees of the organisation earn more than Rs 20000 per month.
3) Majority of the employees earn more than or equal to Rs. 3000 but less than or equal to Rs 5000
as per month’s salary.
4) Both (1) and (2)
Answers and explanations
1-4:
IIM A IIM B

1 1 1

1
0 4

IIM C
1. 3; 8 people had a call from IIM-C, but not from A or B.
2. 2; From the figure nA  B  C  = 16
All these people got a call from one of L, K, I
Total who received call from L, K, I = 17  17 – 16 = 1 person received a call from one of L, K and
I and not from A, B or C. Hence (2).

K
3. 1; Since every person who received calls from A, B, or C also received calls from L, K and I and the
total from L, K, I is 17, it follows that in all only 17 people received calls.
4. 4; All those who received a call from more than one of IIMs A, B and C, also received a call from at
least one among L, K, and I. This is the minimum number of people receiving calls from at least
three IIMs.
1 (A, B, and C) + 1(A and B) + 4(B and C) = 6 people minimum.
5. 3; Tea production in India = 22% of 685 = 151
Coffee production in India = 21% of 408  86.
151  86
 Required percentage =  100  75 %
86

KUNDAN
6. 4; Since we do not know which month’s tea was produced and sold in Japan, average value cannot
be determined. Hence, (4)
7. 4; (30% of 685 × 1400) + (20% of 408 × 2100)
 (30 × 700 × 14) + (20 × 400 × 21) = (294000 + 168000) million $
= 462000 × 106 $ = $462 billion. The nearest option is 459.
8. 3; Brazil’s tea production = 24% of 685
Indonesia and Japan’s tea production = 7% of 685
24  7%
of 685
 243% .
 % excess = 7% of 685
9. 2; Volume of Ford Ikon in 1999 is 670000
Volume in 2000 = 670000 × 1.25 = 837500
Volume in 2001 = 837500 × 1.00 = 837500
So, total cost of all Ford Ikons = 837500 × 5.35 = Rs 4480625 lakhs.
10. 2; From the previous question,
volume of Ford Ikon in 2001 = 837500
and volume of Ford Ikon in 2002 = 837500 × 1.15 = 963125
Volume of Maruti 800 in 2002 = 530000 × 1.5 × 0.85 × 1.1 = 743325
So, required difference = 963125 - 743325 = 219800
11. 4; Let the volume in 1999 be 100 for both the segments.
Volume of B segment cars in 2004 = 100 × 1.5 × 0.85 × 1.1 × 1.42 × 1.00 = 199.155 = 199
Let percentage increase in the volume of segment C cars be x in 2004.
 x  1  x 
Volume of C segment cars in 2004 = 100 × 1.25 × 1.00 × 1.15 × 1.06 × 1  100  = 152  
   100 

x 
Equating the volumes, 1521    199  x  30.92% .
 100 
Note: Use the multiplying factor where necessary.
12. 4; Average marks obtained by girls from all Engineering colleges is as follows:
1 1
Average marks in Physics = (170  150  130  170  160 ) = (780 )  156
5 5
Similarly, average marks in Applied Mechanics = 106, Mathematics = 108, Computer = 117 and
Electronics = 102.
Hence, the average is highest for Physics.
13. 1;
Enginnering Average M arks of Average M arks of P ooled Average
Colleges Boys Girls M arks
P 114 123 118.5

K Q
R
S

T
111
115
105

135
The minimum pooled average marks is from college S.
105
117
108

136

14. 3; 130 – 100 = 30 is the highest difference in college R.


108
116
106.5
(Minimum)
135.5

15. 4; Required difference = (130 – 120) × 20 = 200.

KUNDAN
16. 4; Unless we know the exact salary of the employees, we can’t calculate the average salary.
17. 1; As we are talking about the minimum average salary, we will assume the minimum value of
the range for each group. For example, 90 females will have Rs 3000 as the minimum salary
and 28 female employees will have Rs 5000 as the minimum salary.
90  3000  28  5000  8  8000  3  12000  1  20000
 Rs 4077
90  28  8  3  1
18. 4; We do not know exact salaries of those 3 male employees and 1 female employee who earn more
than Rs 20000. So we can’t say anything about this.
19. 4; Total no. of employees = 461
No. of employees who earn more than or equal to 5000 but less than 12000
= 48 + 28 + 16 + 8 = 100
100
 required % =  100  21 .7%
461
20. 3; There are 286 male employees and 175 female employees in the company. (1) and (2) are easily
inferred from the data but (3) is not, as we cannot find the number of employees earning more
than or equal to 3000 but less than or equal to 5000.
Practice Exercise 15
Directions (Q. 1-6): These questions are based on the following information.
Use data from previous questions if required.
The bar-graph gives the amount spent by India on defence equipment purchase over different
years. The values mentioned above are in US dollars. For fighter planes and Others, the values are in
crores while, the values for AK-47 guns and tankers are in lakhs.
4500
4000 Fighter Planes Tankers AK-47 guns Others
3500
3000
2500
2000
1500
1000
500

K
0
1985 1990 1995 2000

The following table gives the sources of India’s purchase of defence equipment for the year 1995.

Country Value of Purchase (in billion US $)


Russia 12
Sweden 3
North Korea 2

KUNDAN
Britain 2
USA 2
Others 1.34
The following pie-chart gives the breakup of expenditure (in billion US $) involved in purchase of
different fighter planes in the year 2000.
3

Sukhoi
8
3
F-20

Jaguar

Mirage

3 Mig-29
4
1. The average price of all the purchased fighter planes combined together is $20 million in 1985.
The average price of a plane increases by $1 million every year. What is the increase in the no. of
fighter planes bought in 2000 as compared to 1985?
1) 300 2) 350 3) 400 4) Indeterminable
2. Sukhoi is a fighter plane bought only from Russia. In 2000, price of a Sukhoi becomes $50 million
after witnessing an increase of 25% over its 1995 value. The expenditure on Sukhoi in 2000 also
increased by 33.3% over the expenditure in 1995. The no. of Sukhoi aircraft purchased in year
1995 is what percentage of the total number of Sukhoi aircraft purchased in year 2000?
1) 79.5% 2) 84.25% 3) 93.75% 4) Indeterminable
3. What is the average price of all other aircraft excluding Sukhoi in 1995?
1) $ 20 m 2) $ 24 m 3) $ 30 m 4) Indeterminable
4. In the year 2000, 60% of the amount spent under ‘Others’ was utilized for importing electric
fencing equipment for the Indian border with its neighbouring countries. India has a land border
of 10000 km with its neighbouring countries, of which 30% is land bordering with Pakistan. Fur-
ther, installation, maintenance, labour and other charges of Rs 5 million per km were required.
What was the total amount spent on the entire process of fencing the Indo-Pak border? (Given 1
US $ = Rs 45)
1) Rs 6.9 billion 2) $ 6.9 billion 3) $ 6.06 billion 4) $ 5.73 billion
5. In 2000, purchases from Britain increased by 50% . So, what per cent of total purchases is done
from Britain in the year 2000?
1) 5.5% 2) 6.5% 3) 8% 4) 10%

K
6. The AK-47 is a fast and light machine gun. For every purchase of an AK-47, 1000 rounds of free
bullets are distributed as a complimentary gift. The price of an AK-47 is $ 0.016 million in 1990.
How many free bullets were obtained for free in the year 1990?
1) 5 million 2) 5 lakhs 3) 4 million 4) 4 lakhs

Directions (Q. 7-11): These questions are based on the following pie-charts.

Noodles: Market Size Cheese: M arket Size Ketchup: Market Size


Rs 350 crores Rs 115 crores Rs 61 crores
Vadilal Verka Others
11% 9% 15%
Top

KUNDAN
Ramen Vijaya Others
20% 14% 5% Maggi
40%
Kissan
Maggi 45%
80%

7. In the chart of cheese, what is the difference Amul


between the central angles formed by Amul and
Vadilal?
1) 180° 2) 186° 3) 189° 4) 178°
8. A town named Khanapur is exactly representative of the total market for these products, except
that it is one-thousandth the size of the total market. If it has a population of 52000 people, what
is the average total per capita expenditure (in Rupees) on these three items?
1) 97 2) 110 3) 105 4) 101
9. The elders in Khanapur recalled that a couple of decades ago, when the population of the town was
just 30000 (present population = 52000), they used to spend Rs 125 per capita for these items as
well as bread. At that time, the total market share of bread must have been ______ lakh rupees.
Assume per capita expenses growth at 1% p.a.
1) 7.15 2) 7.51 3) 8.15 4) Indeterminable
10. If Kissan diversified into the Noodles market, capturing 15% of the market held by Maggi, what
would be the share of Kissan in the entire food market given (Rs crores)?
1) 75 2) 65 3) 70 4) 72
11. If Maggi produces Superior and Ordinary types of its products - noodles and ketchup - in the ratio
of 3 : 2 and 7 : 3 respectively, find the total market share (in Rupees crore) of the Ordinary type of
products of Maggi.
1) 119.3 2) 124.3 3) 109.3 4) 115.3

Directions (Q. 12-14): These questions are based on the following bar graph. Data from ques-
tions may be used in subsequent questions.
The FMCG com pany incentive pay
25

Incentives to Sales Force Incentives to w orkers


20
% of net profit

15

10

K
0
1991 1992 1993 1994 1995 1996

12. If the total incentive given to workers and the sales force in 1991 was Rs 14 lakhs what was the
net profit of the company in that year?
1) Rs 50 lakh 2) Rs 1 crore 3) Rs 90 lakh 4) Rs 1.5 crore
13. In 1992 the net profit increased by 25% over the previous year. What was the amount (in Rs lakhs)
disbursed by way of incentives to the workers?
1) 13.5 2) 14.5 3) 15.625 4) 16.5
14. In 1993 the net profit increases by 20% over the previous year. If each member of the sales force

KUNDAN
received Rs 10000 by way of incentive, how many people constituted the sales force?
1) 42 2) 52 3) 62 4) 72
Directions (Q. 15-17): The following pie-charts show the Exports and Imports of India. Refer to
the graphs to answer the questions that follow.

Others
EC 41%
Others 25% Japan
33% 8%
Russia
6%

USA USA
Japan 16% 12%
10% EC
Russia
33%
16%

Exports from India Imports to India


15. Out of the total exports to Russia, the export earnings from watches amounted to US $ 1.5 million.
So India’s total imports are more than its exports. This statement
1) is true 2) is false
3) requires data on total exports to Russia 4) is uncertain
16. If 50% of the exports to Japan now get diverted to the EC, then the exports to EC would increase by
20% . This statement
1) is true 2) is false
3) requires data on exports to Japan and EC 4) is uncertain
17. If total exports are one-tenth the total imports, what is the deficit in India’s trade with Japan as a
percentage of trade deficit with Russia?
1) 59% 2) 63% 3) 159% 4) uncertain
Directions (Q. 18-20): The following table shows the marks obtained by 100 students in Maths,
Science and the average of the two subjects.

M arks obtained out of 100


Subjects
0-20 20-40 40-60 60-80 80-100
Maths 9 13 17 38 23
Science 12 16 18 32 22
Average of the two subjects 11 17 19 34 19

K
18. Find the number of students who got 60% or more marks in both subjects.
1) 15 2) 61 2) 53 3) Can’t be determined
19. If to qualify in the examination one has to secure a minimum of 60% marks in either of the
subjects, find the least number of students who have qualified the examination.
1) 53 2) 54 3) 61 4) Can’t be determined
20. If to qualify in the examination one has to secure a minimum of 60% marks in both subjects, find
the maximum number of students who have qualified the examination.
1) 15 2) 54 3) 32 4) 53

Answers and explanations

KUNDAN
1. 3; In 1985, amount spent on fighter planes is $ 400 crore = $ 4000 million. So, no. of planes bought
4000
=  200 . In 2000, average price becomes $ 35 m (increase of $ 1 m per year from 1985). In
20
2000, amount spent on fighter planes = 2100 crores = 21000 million. So, no. of planes bought =
21000
= 600. So, increase in the no. of planes bought = 600 – 200 = 400.
35

 100 
2. 3; In 1995, the average price of Sukhoi aircraft = 50   40 million
 100  25 
In 1995, total amount spent on Sukhoi aircraft
3
= 8 billion   = 6 billion US $ = 6000 million US $
4

6000
 Total number of Sukhoi aircraft purchased =  150
40
8000
And, the total number of Sukhoi aircraft purchased in year 2000 =  160 million
50
150
Required % =  100  93 .75 % .
160
3. 2; In 1995, spending on other planes = $ 12b – $ 6b = $ 6b. Other aircraft bought = 400 – 150 = 250
(from previous solution).
6000
Hence, average price of other planes = = $ 24 m.
250
4. 4; In 2000, total amount spent on Others is $ 30b. 60% of 30 = $ 18 b is spent on purchasing
electric fencing equipment. Equipment required for fencing Pakisan border with India = 30% of
$ 18 b = $ 5.4 billion.
Land border with pakistan = 30% of 10000 = 3000 km
So, other costs = Rs 3000 × 5 million = Rs 15000 million = Rs 15 billion
15
=$ = US $ 0.33 billion.
45
So, total costs = 5.4 + 0.33 = $ 5.73 b.
5. 1; In 1995, purchase from Britain = $ 2b.
It increased by 50% , so purchase from Britain becomes $ 3b for 2000.
In 2000, total purchases = 21b + 0.4b + 0.12b + 30b = $ 51.52b.
 3 
So, required % =    100 approx. < 6% = 5.5% .
 51.52 

K
6. 1; In 1990, no. of AK-47 bought =
80
0.016
m = 5000.

So, no. of free bullets = 5000  10 3 = 5 million bullets.


7. 1; Amul and Vadilal hold 61% and 11% of the cheese market respectively. This is a difference of
50% , ie a difference of 180° between the respective central angles.
8. 4; The total market for cheese, ketchup and noodles is 526 crores. Khanapur has a market which

is
1
the total market, ie 52.6 lakhs. As the population of Khanapur is 52000, the average
1000

KUNDAN
total per capita expenditure is 101.15.
9. 4; The rate of expansion is not given. Therefore we can’t calculate the total market share.
10. 3; If Kissan diversified into the noodles market and captured 15% of Maggie’s share, Kissan’s total
share would be (45% × 61) + (15% of 80% × 350) = 27.45 + 42 = 69.45.
11. 1; Maggie’s market share for ketchup and noodles is 40% × 61 = 24.4 and 80% × 350 = 280 respec-
3
tively. The share of ordinary ketchup is  24 .4  7.32 and the share of ordinary noodles is
10
2
 280  112 . So the total market share of ordinary products is 119.3.
5
12-14: From the data given we can deduce the following information:

Year 1991 1992 1993 1994 1995 1996


% P to sales force 2% 2.5% 2.75% 4% 6% 8%
% P to workers 12% 12.5% 12.75% 13% 13.5% 13.5%
Total % P 14% 15% 15.5% 17% 19.5% 21.5%

12. 2; In 1991, 14% of profit amounted to Rs 14 lakhs. So total profit was Rs 1 crore.
13. 3; In 1992 net profit increased by 25% and amounted to Rs 1.25 crores. The amount dispersed to
workers is 12.5% , ie Rs 15.625 lakhs.
14. 1; In 1993 net profit increased by 20% to become Rs 1.25 × 1.2 = 1.5 crores and the number of
2.75 % of 1.5 crore
people in the sales force =  42
10000
15. 4; It is not possible to calculate either India’s total imports or its total exports. So, the validity of
the given statement cannot be determined. Option (3) also can’t be the answer as not only total
exports to Russia, but also total imports from it should be known. So, it is uncertain.
16. 1; If 50% of exports to Japan, ie 5% of the total exports, are diverted to the EC, the exports to the EC
become 30% of total exports. This is an increase of 20% . So, the given statement is true.
17. 3; Let imports be = 1000y; then exports = 100y. Trade deficit with Japan = 80y – 10y. Trade deficit
 70y 
with Russia = 60y – 16y. So, trade deficit as % =  44y   100  160% .
 
18. 4; The particular student who got 60% or more marks in Maths may or may not get 60% or more
marks in Science. Hence we can’t find the number of students who got 60% or more marks in
both subjects.
Therefore, data inadequate.
19. 3; The number of students who got 60% or more marks in any one subject qualifies the examina-
tion.

K
Therefore, the least number of students who have qualified the examination = 38 + 23 = 61.
20. 2; For maximum number of students to get 60% or more marks in both subjects, 32 + 22 = 54
students who got 60% or more marks in Science must get 60% or more marks in Maths.

KUNDAN
Practice Exercise 16
Directions (Q. 1-5): These questions are based on the following information.
The following table gives information about the total mineral extraction in India (in thousand
tonnes) for different years.

Year Total Minerals extracted Iron and Coal extracted


1989 220000 22000
1990 250000 25000
1991 270000 32400
1992 230000 32200
1993 240000 24000
1994 240000 21600

K
1995 220000 33000
1996 240000 38400
1997 240000 43200
1998 220000 35200
1999 250000 45000
1. In which year was the percentage of iron and coal extracted with respect to the total minerals
extracted exactly twice that in 1994?
1) 1997

KUNDAN
2) 1999
3) Both (1) and (2)
4) Neither (1) nor (2)
2. In which year was the total minerals extracted 900% more than iron and coal extracted in the
previous year?
1) 1993 2) 1994
3) 1995 4) 1996
3. How many times was the percentage share of iron and coal extraction with respect to the total
mineral extraction a multiple of three?
1) 6 2) 5
3) 4 4) 3
4. What is the average percentage share of iron and coal extracted with respect to the total minerals
extracted, for the given period?
1) 13.4% 2) 14.4%
3) 15.4% 4) 16.4%
5. How many times has the iron and coal extraction, as a percentage of the total mineral extraction,
shown a decline over the previous year?
1) Once 2) Twice
3) Thrice 4) Four times
Directions (Q. 6-9): These questions are based on the following bar graphs. These show the
passenger fares for daily services from Mumbai (in Rs).
Tata Airlines 1948
280

240
200
160
120

80
40

Colombo

Delhi
Calcutta

Trivandrum
Ahmedabad

Cochin

Porbandar
Hyderabad

Madras
Karachi
Bhavanagar

Indian Airlines 1998


5000
4500

K
4000
3500
3000
2500
2000
1500
1000
Colombo

Delhi
Calcutta

Trivandrum
Ahmedabad

Cochin

Porbandar
Madras
Hyderabad

Karachi
Bhavanagar

KUNDAN
6. The total fares from Mumbai to those places to which Tata Airlines did have a flight but Indian
Airlines does not, is what % of the total of fares of Tata Airlines from Mumbai to all places?
1) 20%
3) 32%
2) 10%
4) 42%
7. The total fares of flights of Tata Airlines from Mumbai to other cities in India in 1948 form what
per cent of the total fares of Indian Airlines from Mumbai to other cities in India?
1) 6%
3) 9%
2) 8%
4) 10%
8. If the annual simple rate of inflation is 7.5% , how would the fare from Mumbai to Calcutta by Tata
Airlines compare with its counterpart Indian Airlines? (Assume the rate of inflation to be con-
stant for all years)
1) Indian Airlines fare would be 4.5 times the Tata Airlines fare.
2) Indian Airlines fare would be 3.5 times the Tata Airlines fare.
3) Tata Airlines fare would be 5.5 times the Indian Airlines fare.
4) Tata Airlines fare would be 3.5 times the Indian Airlines fare.
9. Indian Airlines declared a discount of 37.5% on the fares for any person who travelled from Mumbai
to all other places in India where they have flights, within a period of one year. What will such a
person effectively have to pay (in Rs)?
1) 15000 2) 13000
3) 17000 4) 19000
Directions (Q. 10-13): These are based on the following bar graph.
(Number of Industrial Townships in 1978 is 1,60,000)

Work Force in China


80
70
% of total employment

60
50
40 1978 1991
30
20
10
0
State Enterprises Collective Urban Collectives Industrial Tow nship Private
Agriculture

10. Which was the first year when people were employed in private enterprises?
1) 1978 2) 1991 3) 1989 4) Indeterminable
11. If it is known that the total work force in China increases at a simple rate of 10% per annum,

K
then by what per cent does the number of people employed in the Urban Collectives change?
1) 150 2) 190 3) 250 4) Indeterminable
12. It can be inferred that, in absolute terms,
1) Collective agriculture has lost its charm for the work force.
2) Collective agriculture remains at the 1978 level, in terms of attracting the work force.
3) Collective agriculture has succeeded in attracting a larger work force in 1991 than in 1978.
4) None of the above can be inferred
13. How many new industrial townships have been set up in China from 1978 to 1991?
1) 150 2) 1500 3) 15000 4) Indeterminable

KUNDAN
Directions (Q. 14-18): The following radar graphs show the Trade Growth (in $ billion) of World
and of China from the previous year for the years 1977 to 1985. Refer to the graphs to answer the
questions that follow.
1977 1977
300 600
1985 250 1978 1985 500 1978
200 400
150 300
100 200
1984 50 1979 1984 100 1979
0 0

1983 1980 1983 1980

1982 1981 1982 1981


World China

14. If the total trade of the World in the year 1976 is $ 5267 billion, what will it be in the year 1985?
1) $ 6176 billion 2) $ 6967 billion 3) $ 6965 billion 4) $ 6987 billion
15. If the total trade of China in the year 1979 is $ 1200 billion, what will it be in the year 1985?
1) $ 3456 billion 2) $ 3786 billion 3) $ 3954 billion 4) $ 3450 billion
16. What is the ratio of the total World trade to the total trade of China in the year 1985, if the total
trade of the World in 1976 is $ 5267 billion and the total trade of China in 1979 is $ 1200 billion?
1) 2 : 1 2) 3 : 1
3) 2 : 3 4) 3 : 2
17. Average world trade growth is what per cent more or less than the average trade growth of China
during the entire shown period?
1) more, 25% 2) less, 30%
3) less, 39% 4) more, 35%
18. What is the per cent increase in trade growth of China in the year 1980 over that of the same in
1979?
1) 70% 2) 10%
3) 17% 4) 80%
Directions (Q. 19-22): The following graphs show the cost of different weapons (in Rs crores) in
the year 1999. Refer to the graphs to answer the questions that follow.

Cost in year 1999 (in Rs crores)


2500

K
2200
2000
Cost per unit

1500 1200
1350
1000
340
500
600 320
0
T-72 Smersh multi- Medium tow ed Unmanned aerial Weapon- Medium self-
barrel rocket guns vehicle mark-2 locating radars propelled guns
launchers

KUNDAN 900
800
700
Weapons
Cost in year 1999 (in Rs crores)
800
Cost per unit

600
500
400
300
160
200 80 200
100 10 10
0
Kamal anti-tank Air target Thermal imaging Assault rifles Anti-mine Global
guided missiles trainers system boosts positioning
systems

19. India purchased 22 ‘T-72’ tanks from Russia in 1991. 10 ‘T-72’ tanks were destroyed in Kargil war
and India sold 8 ‘T-72’ tanks to Sri Lanka at Rs 5500 crore per tank. India has returned the
remaining tanks to Russia and got Rs. 300 crore per tank. On maintenance of each tank, India
spent an amount similar to their cost in 1999. The total profit/loss % for India, if India purchased
these tanks from Russia at Rs, 1000 crore per tank, is
1) 25.4% loss 2) 31.0% loss
3) 35.8% loss 4) 39.2% loss
20. India made 48 Weapon-locating radars in the year 1999 and sold 12 radars to Nepal on 20% profit,
6 radars to Australia on 16% profit, and 13 radars to Zimbabwe on 17% loss, and kept the remain-
ing radars for self-use. What was the profit/loss of India?
1) 3.2% profit
2) 3.3% loss
3) 4.0% loss
4) 4.2% profit
21. Which of the following statements is/are true for the graph?
I. The cost of the Air target trainers is approximately 1/5 of the cost of Assault rifles.
II. The cost of Anti-mine boost is double the cost of Global positioning system.
III. The cost of Smersh multi-barrel rocket launcher is thrice the cost of Thermal imaging sys-
tem.
1) Statement II only
2) Statements I and II
3) Statements II and III
4) Statements I and III
22. In the condition of sudden attack on India, Indian military is in need of 27 ‘T-72’ tanks, 6 Smersh
multi-barrel rocket launchers, 12000 Medium towed guns, 2 Thermal imaging systems and 100

K
Anti-mine boosts. What is the approximate cost the Indian military will have to pay for the above
arms?
1) Rs 14484800 crores 2) Rs 14464480 crores
3) Rs 14644800 crores 4) Rs 14464400 crores

Answers and explanations


21600
1. 3; In 1994, the % of iron and coal extraction w.r.t. total mineral extraction =  100 = 9% .
240000

KUNDAN
43200
In 1997, the % of iron and coal extraction w.r.t. total mineral extraction =  100  18 % .
240000
45000
In 1999, the % of iron and coal extraction w.r.t. total mineral extraction =  100  18 % .
250000
In both cases, it’s twice the required % in 1994.
2. 2; Look for the values of total mineral extraction in a year, when it is ten times the value of iron
and coal extraction in the previous year.
In 1994, total mineral extraction = 240000.
In 1993, iron and coal extraction = 24000. So, total mineral extraction is more than iron and
 240000  24000  
coal extraction by    100  900 % .
 24000 
3. 2; The % share of iron and coal extraction in total mineral extraction is a multiple of 3 in five
cases. In 1991 (12% ), 1994 (9% ), 1995 (15% ), 1997 (18% ), 1999 (18% ).
4. 1; The total iron and coal extraction after adding all the values = 352000.
The total mineral extraction after adding all the values = 2620000.
352000
So, required % =  100 = 13.4 approx.
2620000
5. 3; After calculating the percentage share for every year, its seen that the decline occurs thrice: in
1993 (14% to 10% ), 1994 (10% to 9% ), 1998 (18% to 16% ).
6-9:
Tata Indian
Ahmedabad 70 1700
Bhavnagar 60 1400
Calcutta 230 4800
Cochin 210 —
Colombo 250 —
Delhi 150 3600
Hyderabad 90 2400
Karachi 160 2600
Madras 160 3400
Porbandar 70 —
Trivandrum 240 4000

K
Total 1690 23900
6. 3; Tata Airlines’ total fare was 1690.
The fare for Colombo, Porbandar and Cochin was (250 + 70 + 210) = 530. This formed around
31% of the total fare.
7. 1; Indian Airlines’ total fare to cities in India was 21300. Tata Airlines’ total fare to cities in India
was 1280. This amount was around 6% of the fare of Indian Airlines.
8. 1; If the simple rate of inflation is 7.5% , it means that we have to pay 7.5% more over the previous
year’s fare. Since it is simple rate of inflation, the increase in fare every year would remain the
same. So, we can use the Simple Interest formula to calculate the total increase in price in 50
years for Tata Airlines.

KUNDAN
In 1948, for Tata Airlines, the Mumbai-Calcutta fare was 230
7.5
Total increase = 230 × 50 × = 862.5.
100
So, the fare now becomes 230 + 862.5 = Rs 1092.50.
In 1998, Indian Airlines’ Mumbai-Calcutta fare is 4800, which is around 4.5 times the fare
charged by Tata Airlines.
9. 1; A person travelling to all other places in the year has to pay a total of Rs 23900. A person
5
 23900 = 15000.
availing of the 37.5% discount will have to pay just 62.5% of the total fare, ie
8
10. 4; Although we can definitely say that the people were employed in Private Enterprises after 1978
and before or in 1991, we do not have data prior to 1978 and intermediate years’ data. So, cannot
be determined.

  10  
11. 3; Let total work force in 1978 be ‘n’. Then in 1991, it will be = n   100   13 years  n = 2.3n. So,
   
people employed in urban collectives in 1978 = 0.04n; in 1991 = 0.06 × 2.3n = 0.138n.

 0.138n  0.04n    9.8 


So change =    100 %   %  245 %
 0 .04n    0.04 
12. 4; Option (1) does not define ‘charm’ which may have different interpretations. If it means the
percentage of populace, collective agriculture is still the largest employer. Options (2) and (3)
require the actual total work force figures, which are not aailable. So, none of the given state-
ments can be inferred in absolute terms.
13. 4; The number of Industrial Townships in 1978 is known. But that for 1991 is not known (this
could’ve been determined if we had data for total work force in 1991). Hence, cannot be deter-
mined.
14. 2; Total trade of World in year 1976 = $ 5267 billion.
So it will be in year 1985 = 5267 + 100 + 150 + 175 + 175 + 150 + 200 + 225 + 250 + 275
= $ 6967 billion
15. 4; Total trade of China in year 1979 = $ 1200 billion.
Trade of China in year 1985 = 1200 + 275 + 275 + 300 + 350 + 500 + 550 = $ 3450 billion
16. 1; From questions 14 and 15, we know that the total trade of World in year 1985 = $ 6967 billion
and the total trade of China in year 1985 = $ 3450 billion. Hence, the ratio = 1 : 1/2 (approxi-
mately)
or 2 : 1.
100  150  175  175  150  200  225  250  275
17. 3; Average world trade growth =  188.89  190
9

K
Average trade growth of China
100  200  250  275  275  300  350  500  550
=  31111
.  310
9
310  190
Required per cent =  100  39%
310
(275  250)  100
18. 2; Percentage growth in trade of China in 1980 =  10%
250
19. 3; The total cost (purchase + maintenance) of T-72 Tanks = (22 × 1000) +(22 × 2200)

KUNDAN
= 22000 + 48400 = Rs 70400 crores.
The total earning = 8 × 5500 + 4 × 300 = 44000 + 1200 = Rs 45200 crores.
70400  45200   100 
25200
 100  35 .8% .
Hence the loss % =
70400 70400
20. 2; Total cost = 48 × 320 = Rs 15360 crores.
12  320  120 6  320  116 13  320  83 320
Total earning =  +  1440  696  1079 
100 100 100 100
= Rs 10288 crores.
15360  10288
Loss percentage =  100  3.3% .
15360
21. 4; I: The cost of Air target trainers = Rs 160 approximately.
1
The cost of Assault rifles = Rs 800 approximately. Hence, the cost of the Air target trainers is
5
th of the cost of Assault rifles. Statement I is true.
II: By visualising the graph,
the cost of Anti-mine boost is equal to that of the Global positioning system. Statement II is
false.
III: The cost of Smersh multi-barrel rocket launcher = Rs 600 crores approximately.
The cost of Thermal imaging system = Rs 200 crores (approximately).
Hence, the cost of Smersh multi-barrel rocket launcher is thrice that of the Thermal imaging
system.
Statement III is true.
Quicker Approach: Check the 1st condition and get the answer.
22. 4; The cost that Indian military have to pay
= (27 × 2200) + (6 × 600) + (12000 × 1200) + (2 × 200) + (100 × 10)
= 59400 + 3600 + 14400000 + 400 + 1000 = Rs 14464400 crores.

K
KUNDAN
Practice Exercise 17

K
Directions (Q. 1-5): The following bar graphs shows the number of employment provided by
both public and private sectors in different industries in the year 1985, 1990 and 1995. While
the pie charts show the percentage of employment provided by both private and public sectors in
different industries in the year 2000. Refer to the graphs to answer the questions that follow.
Figure within brackets indicate the total for the corresponding year.
Public Sector Public Sector (20000)
Agricultural
Agricultural Mining Iron
4% Mining
Health Trading Transport
6%

7511
Services

KUNDAN
8000
6756
5781

7000
Iron
Health
6000 12%
8%
5000
Services
2754

4000
2449
2247

52%
3000
1590
1212

1101
1002

2000
899

815
752

Transport
450

Trading
353

361
295
280
250

246
174

1000 15%
3%
0
1985 (10611) 1990 (12706) 1995 (14467)

Private Sector Private Sector (10000)

Agricultural Iron Trading


Transport Agricultural
Services Mining Health
2% Services 12% Mining
Transport
4678

18% 4%
5000
4110
3968

Trading
4000
6%
3000
1461
1269

2000
1036

Health
862
804

821

Iron
361

1000 4%
298

277

281

54%
161

163

145
98

83

75

71

61

0
1985 (6726) 1990 (6799) 1995 (7559)

1. The increase in employment opportunity in the Iron industry within the public sector from 1985-
95 formed what per cent of the private sector growth over the same period for the same industry?
1) 91% 2) 560% 3) 1785% 4) 112%
2. The number of employed persons in the industry with the largest employment share in the public
sector formed what per cent of the number of employed persons in the industry with the largest
employment share in private sector in 2000?
1) 146.25% 2) 54.5% 3) 183% 4) 192%
3. If 2 out of every 5 of those who were recruited in the public sector in year 2000 were graduates,
how many non-graduates did the public sector’s transport industry recruit in that year?
1) 1912 2) 1664 3) 1800 4) 1572
Q. 4-5: Refer to the data given below to answer the questions that follow.
If the values in the year 2000 are taken as base = 100, then the values of Public Sector in the year
2005 are given below. The values of Private Sector are given within the brackets.
Agriculture = 115 (110) Mining = 100 (110)
Iron = 120 (110) Health = 120 (90)
Trading = 80 (105) Transport = 105 (80)
Services = 110 (120)
4. The Health industry in the Private sector has witnessed what per cent growth in employment
from 1990 to 2005?
1) 203% 2) 247% 3) 279% 4) 333%
5. The growth in employment within the services industry in the Private sector in 2005 has wit-
nessed what per cent deviation from the growth within the Iron industry in the Public sector in
2005?
1) -25% 2) -16.66% 3) +16.66% 4) +25%
Directions (Q. 6-11): The following line and bar graphs show the number of banking centres in
urban, semi-urban and rural areas of different states in the year 1969. Refer to the graphs to
answer the questions that folow.
240
Semi-urban Urban
220
Number of banking centres

200
Rural 1750
180
160
140
120 1020
100 650
620
80
60 460
40 10 60
20

K
0
Jammu &
Himachal

Punjab

Rajasthan

Bihar

Kerala
Haryana

Pradesh

Kashmir
Jammu &
Himachal

Punjab

Rajasthan

Bihar

Kerala
Haryana

Pradesh

Kashmir

6. If the banking centres have increased at a steady rate of 35% in all states of India, then what will
be the number of banking centres in Haryana in the year 1975?
1) 4840 2) 4600 3) 5630 4) 3990
7. If Kerala Government decided to increase banking centres in rural areas at 46% per year and in
semi-urban areas at 53% per year, then what will be the total number of banking centres in
Kerala in 1973?

KUNDAN
1) 180 2) 150 3) 328 4) 282
8. If in 1969 the average number of banking centres in rural areas of Haryana, J&K, Punjab, Rajasthan,
Bihar, Kerala, Himachal Pradesh and Madhya Pradesh is 670, then the number of banking cen-
tres in rural areas of Madhya Pradesh will be:
1) 790 2) 400 3) 820 4) 500
9. In the year 1967, the Government decided to increase the banking centres according to the popu-
lation increase in that particular state. If according to 1971 census, the semi-urban population of
Punjab has increased by 25% from 1969, then what will be the number of banking centres that
the Government has to increase in the year 1971 in semi-urban parts of Punjab?
1) 22 2) 28 3) 50 4) 38
10. Which of the following statements is not true?
1) The number of banking centres in semi-urban areas of Jammu & Kashmir is equal to the
number of banking centres in urban areas of Rajasthan in 1969.
2) The number of banking centres in rural areas of Bihar is equal to the number of banking

K
centres in urban areas of Haryana in the year 1969.
3) The number of banking centres in rural areas of Rajasthan is almost 2.8 times the number of
banking centres in rural areas of Himachal Pradesh in the year 1969.
4) The number of banking centres in semi-urban areas of Himachal Pradesh is 4 times the num-
ber of banking centres in rural areas of Kerala in 1969.
11. In which of the following states the ratio of rural banking centres to total banking centres the
second lowest?
1) Bihar 2) Punjab 3) Haryana 4) Himachal Pradesh
Directions (Q. 12-15): Refer to the charts below and answer the questions that follow.

KUNDAN
The first bar graph gives the Gross Domestic Product (GDP), the per capita Gross Domestic Product
(PCGDP) and the Gross Domestic Product in constant dollar value in the year 1990 (GDPC) for the US
in various years. Here, we define PCGDP for any year as follows:
GDP
PCGDP =
Population
To get the value of the GDP and GDPC in dollars multiply the corresponding value in the bar graph
by 10 8 .
The second bar graph gives the number of people below the poverty level in the US for various
years. The data is given under three categories (i) All, (ii) Whites and (iii) All people of Hispanic origin.
Note that people of Hispanic origin may be whites or blacks.
To get the actual numbers multiply the values in the bar graph by 1000.
80000 45000
40000 All White Hispanic
70000
PCGDP GDP GDPC
35000
60000
30000
50000
25000
40000 20000
30000 15000

20000 10000
5000
10000
0
0 1975 1980 1985 1990 1995 2000
1975 1980 1985 1990 1995 2000

12. A different calculation for the GDPC takes the base year as 1975; i.e. the value of the dollar in
1975 is taken as 1. Which of the following values will be closest to the GDPC for 2000 based on this
calculation?
8 8 8 8
1) 10,000 × 10 dollars 2) 15,000 × 10 dollars 3) 20,000 × 10 dollars 4) 25,000 × 10 dollars
13. In the year 2000, what percentage of the US population was below the poverty level in the US?
1) 10% 2) 16% 3) 20% 4) 25%
Additional information for questions 14 to 15:
Given below are two statements.
S1: The number of people of Hispanic origin below the poverty level increased from 1975 to 2000.
S2: The ratio of the number of people of Hispanic origin below the poverty level to the total number
of people below the poverty level increased from 1975 to 2000.
14. Which of the following is true?
1) Only S1 is true. 2) Only S2 is true.
3) Both S1 and S2 are true. 4) Neither S1 nor S2 is true.
15. Assume that 50% of the people of Hispanic origin below the poverty line were whites in 1995. Let
x be defined as the percentage of whites not of Hispanic origin who are below the poverty line with
respect to the total number of people below the poverty line. For the year 1995, which of the
following values best approximates x?
1) 65% 2) 56% 3) 47% 4) 38%
Directions (Q. 16-19): Refer to the table below and answer the questions that follow.
In the table below, data for five countries is given. The data given is about the population, the
gross domestic product (GDP) and the relative purchasing power (RPP). The RPP for a country is de-
fined as the average ratio of the amount in dollars required to buy a certain item in USA to the
amount in dollars required to buy the same item in that country. Also given are the annual percent-
age growth rates for the population, the GDP and the RPP. The growth rates may be positive or nega-
tive. Assume the following:
1) A positive annual growth rate of 7% of a particular quantity means that the quantity will double
itself in 10 years. A negative annual growth rate of 7% means that the quantity will get halved
in 10 years.
2) The doubling time for positive growth rates is inversely proportional to the growth rate. A simi-
lar result holds for the halving time.
3) For all calculations involving quantities at a later time, assume that the growth rates remain
constant.

Population Growth GDP Growth Growth


Country RPP
in million (Popln)% (billion dollars) (GDP)% (RPP )

USA 264 0 6950 2.33 1 0

West Germany 65 0 1476 2.33 1 0

East Germany 15 0 153 7 1.5 0

K India

Indonesia
956

204
1.75

1.75
1550

750
7

7
2.5

2.5
-1.75

-1.75
16. The per capita income (PCI) in dollars of a country is the value of the GDP of the country in dollars
divided by its population. If we arrange the countries in increasing order of their per capita in-
comes, which of the following is correct?
1) Indonesia, West Germany, East Germany
3) Indonesia, India, USA
2) India, USA, West Germany
4) East Germany, West Germany, USA
17. Based on the figures in the table, after how many years will the per capita income of India become

KUNDAN
equal to the per capita income of the USA? (Use data from previous question, if required.)
1) between 20 and 40 years 2) between 40 and 60 years
3) between 60 and 80 years 4) more than 80 years
18. Define a quantity called the modified per capita income (MPCI) in dollars as the product of the per
capita income in dollars and the RPP. In how many years will the modified per capita income of
India double itself?
1) 10 years 2) 15 years 3) 20 years 4) 25 years
19. Due to the reunion of West Germany and East Germany per capita GDP of East Germany in-
creased by
1) 87% 2) 91% 3) 95% 4) 100%
Answers and explanations
1. 4; In the iron industry (public sector), increase in employment opportunities from 1985 to 1995
= 1590 – 795 = 795.

K
In the Iron industry (private sector), a similar increase = 4678 - 3968 = 710.
100
Thus the percentage that public sector forms of private sector  795   112 % .
710
20000
2. 4; Services has the public sector’s largest share with 52% of 20000, i.e. 52   10400
100
10000
Iron industry has the private sector’s largest share with 54% of 10000, i.e. 54  = 5400
100
100

KUNDAN
So, the required percentage = 10400   192 % (approx)
5400
3. 3; If 2 out of every 5 are graduates then 3 out of 5 are non-graduates.
Employment in transport industry of public sector = 15% of 20000 = 3000.
So, that non-graduates recruited in transport industry of public sector = 3000 × (3/5) = 1800
4. 4; Private sector’s health industry figures in the year 2000 = 10000 × (4/100) = 400

In the year 2005  400  90  360 and in the year 1990 = 83


100
100 100
 The percentage increase = (360 - 83) ×  277   333 % (approx).
83 83
5. 1; Since the 2005 values of the index for the private sector’s service industry and the public
sector’s iron industry are the same (120), the 2005 percentage deviation will be the same as
the deviation for the 2000 figures.
Number of employment provided in private sector = 10000 × 0.18 = 1800
and number of employment provided in public Iron sector = 20000 × 0.12 = 2400.
Since 1800 and 2400 are in the ratio 3 : 4,
100
 Required percentage deviation = 3  4    25 % .
4
6. 2; Total banking centres in Haryana in year 1969 = 100 + 10 + 650 = 760
6
 35 
Hence, the total number of banking centres in Haryana in year 1975 = 7601    4600.
 100 
7. 3; The number of banking centres in rural areas of Kerala in year 1969 = 60
 The number of banking centres in rural areas of Kerala in year 1973
4
 46 
= 601    273 (approx.)
 100 
The number of banking centres in semi-urban areas of Kerala in year 1969 = 10
 The number of banking centres in semi-urban areas of Kerala in year 1973

4
 53 
= 101    55 (approx.)
 100 
Hence, the total number of banking centres in Kerala in year 1973 = 273 + 55 = 328.
8. 1; The number of banking centres in rural areas of Madhya Pradesh
= 670 × 8 - (650 + 620 + 460 + 1020 + 1750 + 10 + 60) = 5360 - 4570 = 790.
9. 2; Because the population has increased by 25% from 1969 to 1971,
the banking centres have also increased by 25% .
 The number of banking centres in semi-urban parts of Pubjab in the year 1971
25
 110   137 .5  138 .
100
 increase in the number of banking centres = 28.
10. 4; By visualising the graph, option (4) is not true for the graph.
1020 1020 102
11. 3; In Punjab ratio of rural banking centres is  
110  10 120 12
650 65
Similarly, for Haryana = 
110 11
460 46
For Jammu and Kashmir = 
20 2
620 62
For Himachal Pradesh = 
20 2
1750 175
For Rajasthan = 
210  20 23
60
For Kerala = 6
10
10 1
For Bihar = 
90 9
 Haryana has the second lowest ratio of the rural banking centres with respect to the total
banking centres.
GDPC
12. 3; Dollar value =

K
GDP

D1 D2
X(GDP) Y(GDPC) (Dollar Value) (Dollar Value)
(Base-1990) (Base-1975)
1975 11000 28000 2.545 1
1990 46000 46000 1 0.393
2000 68000 54000 0.794 0.312

KUNDAN
When base is changed to 1975
1 0.794
D2 (1990 )   0.393 D2 (2000) 
 0.312
2.545 2.545
 For GDPC in 2000, GDPC = 68000 × 0.312  21000.
13. 2; The population of the US can be obtained from the first graph as GDP divided by PCGDP. This
8
68000  10 8
gives a value of  2.6  10 = 260 million to the year 2000. The number of people
26000
below the poverty level for 2000 (from second graph) is 41 million.
41
 Required percentage = 260  100  16 %

K
14. 3; By visual inspection of the graph, S1 is true.
Ratio of the number of people of Hispanic origin below the poverty level to the total number of
people below the poverty level in
2500  1000 1 8000  1000 1
1975   2000  
23000  1000 9 41000  1000 5
 The ratio is increased. This statement is also true.
15. 2; For the year 1995, we get the values 34 million, 22 million and 6 million for the total, the
number of whites and the number of people of Hispanic origin below the poverty level respec-
tively. 3 million of the people of Hispanic origin are whites. The number of whites not of His-
panic origin are, therefore, 19 million.

KUNDAN
19
 Required percentage = x =  100  56%
34
6950 7000 1476 1400
16. 4; USA    27 ; WG    22 ;
264 260 65 65
153 150 1550 155
EG    10 India    1.6
15 15 956 95
750
Indonesia   3.6 .
204
17. 4; The present per capita income (based on the table) is $27,000 for the USA and $1600 for India -
the ratio is approximately 16. All we need for the problem is the relative rate of growth of PCI for
India with respect to USA.
Growth rate for PCI (India) = 7 - 1.75 and growth rate for PCI (USA) = 2.33 - 0 = 2.33
 The relative percentage growth rate is [(7 – 1.75) – (2.33 – 0)] = 2.92
Now, initially the PCI of USA is 16 times that of India. We have to make it equal to 1.
7
The ratio is halved (PCI of USA = 8 × PCI India) in × 10 years = 24 years.
2.92
The ratio is further halved (PCI of USA = 4 × PCI of India) in 24 × 2 = 48 years
Similarly, the ratio is further halved (ie PCI of USA = 2 × PCI of India) in 24 × 3 = 72 years
 PCI of USA will be euqal to PCI of India in 24 × 4 = 96 years.
 GDP 
18. 3; MPCI =  Population   RPP .
 
The annual growth rate of this quantity can be very well approximated by the growth rate of GDP
– growth rate of population + growth rate of RPP
= 7 – 1.75 - 1.75 = 3.5% . Hence, the doubling time will be 20 years.
1476  153 1629
19. 4; Per capita GDP of West Germany and East Germany together =   20 .36
65  15 80
153
Per capita GDP of East Germany =  10 .2
15
20 .36  10 .2
Required percentage increase =  100  100 %
10 .2
Practice Exercise 18
Directions (Q. 1-5): Answer the questions on the basis of the information given below.
In the final-year exam six students S1 , S 2 , S 3 , S 4 , S 5 and S 6 obtained the percentage marks in
five theoretical papers P1 , P2 , P3 , P4 and P5 and two lab papers L1 and L 2 as shown below:

P1 P2 P3 P4 P5 L1 L2 Total
S1 90 83 87 89 91 78 82
S2 98 92 59 61 50 82
S3 82 88 81 79 70 98 97 82.5
S4 835 875 78 82 90 94
S5 65 55 83 87 70 68 70.6
S6 71 69 73 67 90 80 73.25

K
The weightages associated with each of the theoretical papers P1 , P2 , P3 , P4 and P5 are the
same. Also, the weightages given to lab papers are the same, but different from that for theoretical
papers. The maximum score for all the papers P1 , P2 , P3 , P4 , P5 , L1 , L 2 together is 600.
1. What is the maximum score for any of the theoretical papers and that for either of the lab papers?
1) 50 and 25 2) 100 and 50
2. What is S 6 ’s percentage score in paper L1 ?
1) 59 2) 72
3) 80 and 90 4) 50 and 90

3) 80 4) 62
3. If S1 has got the highest marks among all the students, what can be a possible score that S 4 can

KUNDAN
get in paper P1 ?
1) 97 2) 98 3) 99 4) 96
4. S 5 ’s percentage score in lab paper L 2 is
1) 29.6 2) 59.2 3) 88.8 4) 44.4
5. If S 2 and S 4 finally get the same score, what is the difference between S 4 ’s score in paper P1 and
S 2 ’s score in paper P2 ?
1) 20 2) 21 3) 22 4) 23
Directions (Q 6-10): Answer the questions on the basis of the information given below.
The following diagram and table give the information regarding students writing the civil service
exam over the last few years.
1800
Male Female
(Figures in thousand)

1600
400
1400
1200 250 200
200 350
1000
800
600 1300
1100 1150
1000 950
400
200
0
2000 2001 2002 2003 2004
Details of Students’ Background (Figures in percentage)

Year Arts Pure science Engineering Others


2000 25.00 10.00 58.33 6.67
2001 22.22 8.80 59.25 9.73
2002 15.00 8.33 70.00 6.67
2003 18.50 5.92 71.14 4.44
2004 16.50 11.10 61.80 10.60
6. The number of students who wrote the exam saw the greatest percentage increase over the
previous year in
1) 2001 2) 2002 3) 2004 4) 2003
7. The approximate percentage change in the number of students with pure science background
who wrote the exam from 2000 to 2004 is
1) 57 2) 47 3) 51 4) 43
8. The number of female engineering graduates who wrote the exam in 2001 was approximately.

K
1) 1,70,000 2) 75,000 3) 1,50,000 4) Cannot be determined
9. The number of engineering students writing the exam saw the greatest increase over the previ-
ous year in
1) 2000 2) 2001 3) 2002 4) 2003
10. Which of the following statements is correct?
I. The number of students who wrote the exam from engineering background has been increas-
ing every year.
II. The percentage of female students who wrote the exam was the highest in the year 2004.
III. The number of students who wrote the exam from the Others category was the same in 2000
and 2002.
1) Both II and III 2) Only II 3) Only I 4) Both I and II

KUNDAN
Directions (Q. 11-15): Study the following table and answer the questions that follow.
The table shows the number of articles produced (P) and sold (S) by five units of a company over
the years.

Unit
Year I II III IV V
P S P S P S P S P S
1998 29 22 39 34 56 50 42 40 53 46
1999 52 45 49 43 69 64 45 37 58 51
2000 46 42 32 31 43 32 53 50 49 42
2001 56 52 54 45 52 47 62 58 55 48
2002 49 47 28 23 77 65 64 53 61 58
2003 60 55 32 27 68 61 69 62 66 62
11. What is the approximate overall percentage of rejection for unit IV for all the given years?
1) 13% 2) 21% 3) 10% 4) 17%
12. What is the average number of articles selected in the year 2002?
1) 37 2) 51.2 3) 49.2 4) 41
13. A continuous increase in production and a continuous decrease in the number of articles sold is
shown respectively by which of the following units?
1) II, IV 2) None, III 3) II, III 4) IV, None
14. What is the ratio of the total number of articles sold in 1999 to that by unit IV over the years?
1) 2 : 3 2) 4 : 5 3) 6 : 7 4) 2 : 7
15. In which year was the largest percentage of articles sold out of the articles produced by unit II?
1) 2000 2) 2001 3) 1999 4) 2003
Directions (Q. 16-20): Study the following chart to answer the questions given below:
Proportion of population of five villages in 2004
Rampur
12%
Village % population below poverty line
Maheshpur
Rampur 35 24%
Narayanpur
Maheshpur 40 35%

Narayanpur 38 Dinapur
Dinapur 45 9%
Laxmipur
Laxmipur 32 20%

K
16. In 2005, the population of Rampur as well as Dinapur is increased by 12% from the year 2004. If
the population of Rampur in 2004 was 4000 and the percentage of population below poverty line in
2005 remains the same as in 2004, find approximately the population of Dinapur below poverty
line in 2005.
1) 1510 2) 1520 3) 1535 4) 1500
17. If in 2006 the population of Maheshpur is increased by 8% , the population of Narayanpur is
reduced by 10% from 2004, and the population of Narayanpur in 2004 was 7000, what will be the
total population of Maheshpur and Narayanpur in 2006?
1) 11,550 2) 11,484 3) 11,200 4) 12,285

KUNDAN
18. If in 2004 the total population of five villages together was 35,000 approximately, what will be
population of Narayanpur in that year below poverty line?
1) 4600 2) 4650 3) 4665 4) 4655
19. If the population of Laxmipur below poverty line in 2004 was 1240, what was the population of
Maheshpur in 2004?
1) 4600 2) 4650 3) 4750 4) 4660
20. If the population of Narayanpur is 6300 in 2004, what will be the ratio of the population of Narayanpur
below poverty line to that of Maheshpur below poverty line in that year?
1) 133 : 96 2) 122 : 95 3) 96 : 133 4) 233 : 96

Answers and explanations


1-5: Let the maximum score of each of theoretical papers be ‘a’ and the maximum score of either of
lab papers be ‘b’.
For the student S 3
a b 82 .5
(82 + 88 + 81 + 79 + 70) + (93 + 97) = (5a + 2b) ×
100 100 100
 400a + 190 b = 412.5a + 165 b
 25b = 12.5 a  2b = a  a = 2b
The maximum marks for all the papers together is 600
 5a + 2b = 600  10b + 2b = 600  b = 50
So a = 100
1. 2
2. 1; S 6 ’s score in the paper

73.25  600 (71  69  73  67  90) 80  50 


L1    100 
100  100 100  = 439.5 – 370 – 40 = 29.5

100
 S 6 ' s percentage score = 29 .5   59 %
50
(90  83  87  89  91)  100 78  82
3. 4; Total score of S1    50 = 440 + 80 = 520
100 100
S 4 ’s score without considering the score in paper P1 is given by
83 .5  87.5  78  82  100  90  94  50
= 331 + 92 = 423
100 100
Hence S 4 scores less than 520 - 423 = 97 marks in the papers. So S 4 can score, say, 96. Hence
choice ‘4’.
70.6 (65  55  83  87  70) 68 

K
4. 2; S 5 ’s score in lab paper L 2 is 100  600   100
 100 
100
 50
 
= 423.6 - (360 + 34) = 29.6
29 .6
 S5 ’s percentage score in paper L 2 is  100 = 59.2
50
5. 3; S 4 ’s score without the paper P1 is 423.
(98  92  59  61  50 ) 82
S 2 ’s score without the paper L 2 =  100   50 = 360 + 41 = 401
100 100
Hence if S 2 and S 4 finally get the same score, then the difference in S 4 ’s score in paper P1 and

KUNDAN
S 2 ’s score in paper L 2 is 423 – 401 = 22 marks.
6-10: Total number of students in 2000 is 12,00,000
2001 is 13,50,000
2002 is 13,00,000
2003 is 13,50,000
2004 is 17,00,000
6. 3; Clearly the greatest percentage increase in the number of students writing the exam (increase
over the previous year) is in 2004.
17,00,000  13,50,000
% increase =  100 = 35  100  25 .92
13,50,000 135
7. 1; In 2000, the number (in thousand) is 1200 × 0.1 = 120
In 2004, the number (in thousand) is 1700 × 0.111  190
70
% change =  100  57 %
120
8. 4; The number of females writing the exam in 2001 is given and the number of engineering
students writing the exam can be calculated. But, the number of female engineering graduates
writing the exam can’t be calculated.
9. 2; Number of engineering students writing the exam in
2000  0.5833 × 1200 = 700
2001  0.5925 × 1350 = 800
2002  0.7 × 1300 = 910
2003  0.711 × 1350 = 960
2004  0.618 × 1700 = 1050
100
Greatest percentage increase =  100 = 14.28% in 2001
700
10. 3; Statement I is obviously true from solution to Q. 34.
400
Statement II: In 2004, percentage of female students writing the exam =  100  23 .52
1700
350
In 2002, the percentage was  100  26 .92
1300
 Statement II is false.
6.67
Statement III: Students from ‘Others’ category writing the exam in 2000 was  1200 .
100
6.67
In 2002 the number became  1300 .
100
So statement III is false.
11. 3; The total number of articles produced by unit IV for the given years = 335
The total number of articles rejected by unit IV for all the given years = 35

K
35
 Required percentage rejection = 335  100  10 .44 %  10 %
12. 3; Total number of articles selected in 2002 = 246
246
 Required average =  49 .2
5
13. 4; The continuous increase in production of articles is seen in the unit IV and none of the units
show continuous decrease in the number of articles sold.
14. 2; The total number of articles sold in 1999 = 240
The total number of articles sold by unit IV over the years = 300

KUNDAN
 the required ratio = 240 : 300 = 4 : 5
15. 1; It is obvious from the table.
9
16. 1; Population of Dinapur in 2004 = 4000   3000
12
112
Population of Dinapur in 2005 = 3000   3360
100
3360
 Population below poverty line = 45% of 3360 = 45  100  1512  1510

24
17. 2; Population of Maheshpur in 2004 = 7000   4800
35
108
Population of Maheshpur in 2006 = 4800   5184
100
90
Population of Narayanpur in 2006 = 7000   6300
100
 Total population = 5184 + 6300 = 11484
35 38
18. 4; Population of Narayanpur below poverty line = 35000   = 133 × 35 = 4655
100 100
100 24
19. 2; Population of Maheshpur in 2004 = 1240    4650
32 20
38
20. 1; Population of Narayanpur below poverty line = 6300   2394
100
24 40
Population of Maheshpur below poverty line = 6300    1728
35 100
2394
 Ratio =  133 : 96
1728

K
KUNDAN
Practice Exercise 19
Answer Questions 1 to 5 on the basis of the information given below:
The Venn-diagram shown below gives the number of students who study Mathematics, Physics,
Chemistry and Biology.

The number of students studying the given subjects in the above given order is 64, 76, 63 and 72
respectively.
1. The number of students studying only Physics is

K
1) 11 2) 20 3) 12 4) 14
2. What is the number of students who study Mathematics only?
1) 1 2) 2 3) 3 4) Data insufficient
3. How many students study Mathematics, Biology, Physics and Chemistry?
1) 7 2) 6 3) 12 4) 22
4. What is the total number of students who study either Physics and Mathematics or Chemistry
and Biology?
1) 67 2) 60 3) 53 4) 45
5. The ratio of the number of students who study Physics, Chemistry and Biology to that of those who
study Mathematics, Chemistry and Biology is

KUNDAN
1) 1 : 1 2) 11 : 13 3) 15 : 17 4) 9 : 10
Answer Questions 6 to 10: Read the data carefully and answer the questions that follow:
Details of the Indian Widget Industry

Year
Ratio
2000 2001 2002 2003 2004 2005

Profit Margin 0.27 0.30 0.24 0.30 0.33 0.36

DS Ratio 0.70 0.75 0.90 1.00 1.10 1.20


EXIM Ratio 0.60 0.64 0.72 0.50 0.60 0.68

Average selling price per widget


Profit margin = 1
Average cost price per widget
Industry demand (by volume) for widget
DS Ratio = Industry supply (by volume) for widget

Volume of exports of widget


EXIM Ratio = Volume of imports of widget
Note: 1: (i) Industry demand = Domestic demand + Export demand
(ii) Industry supply = Domestic supply + imported supply
(iii) The average export price per widget = The average selling price per widget
(iv) The average import price per widget = The average cost price per widget
6. Find the volume of widgets exported in the year 2004 as a percentage of the industry demand for
widgets in that year.
1) 24% 2) 65% 3) 37.5% 4) Can’t be determined
7. If the total values of widgets imported in the year 2003 was Rs 300 crore, then what was the total
value of widgets exported in that year?
1) 195 crore 2) 200 crore 3) 325 crore 4) Can’t be determined
8. Find the ratio of domestic demand and domestic supply in the year 2001?
1) 0.75 2) 0.30 3) 0.80 4) Can’t be determined
9. If the volume of widgets imported increased by a steady 20% every year then during which of the
following periods did the volume of widgets exported increase by the maximum percentage?
1) 2000-2001 2) 2001-2002 3) 2004-2005 4) 2003-2004
10. Find the volume of the widgets imported in the year 2002 as a percentage of the industry supply
for widgets in that year.
2
1) 83.5% 2) 66 % 3) 37½% 4) Can’t be determined
3
Note: Questions 11 to 30 carry two marks each.
Given below is a survey conducted in four cities about people liking different film stars. The total

K
number of people surveyed was 2000 and they were distributed as given in the chart. The table shows
the percentage of people liking the film stars in that particular city. An individual can have liking for
more than one star.

Film Star Mumbai Delhi Kolkata Indore


Amitabh
62 52 44 48
Bachchan
Shahrukh Khan 29 32 22 27
Salman Khan 21 18 13 23

KUNDAN
Aamir Khan 36 34 26 33
Hritik Roshan 19 26 11 29
Fardeen Khan 12 14 6 19
Percentage in 4 cities out of 2000 people surveyed.

Indore Delhi
22% 25%

Kolkata
23%
Mumbai
30%

11. Which city has maximum people liking Fardeen Khan in the given survey?
1) Mumbai 2) Delhi 3) Kolkata 4) Indore
12. If all the fans of Aamir Khan also like Amitabh Bachchan in Mumbai, then how many people in
Mumbai like Amitabh but not Aamir Khan?
1) 172 2) 156 3) 216 4) 148
13. If Salman and Shahrukh don’t have common fans then how many of the total people surveyed in
all the four cities together like neither of them?
1) 947 2) 1023 3) 1117 4) 1069
14. What can be the maximum number of people surveyed in Delhi who don’t like any of the above
film stars?
1) 240 2) 250 3) 269 4) 131
15. What can be the maximum number of people surveyed in Kolkata who like all the stars given in
the table.
1) 218 2) 21 3) 27 4) 276
Directions (Q. 16-20): The following table shows the specialisation of the students in percent-
age at the four premier B-schools in Mumbai in the year 2004.
Code B-School
A MNIMS
B BJIMS
C JP Jain
D KITIE
100%

K
25%
75%
Systems
50% Finance
50% C
D
A 75%
25%
B
100%

100% 75% 50% 25%


Marketing

The number of students in a B-school (in 2004) is given by the following graph.

KUNDAN
300
240
250

200
200 160
150

100
80
50

0
A B C D

16. In which of the following B-schools have the maximum number of students opted for finance?
1) MNIMS 2) KITIE 3) JP Jain 4) BJIMS
17. The difference between the number of sudents who opted for finance in MNIMS and JP Jain is
what per cent of the number of students who opted for systems in BJIMS?
1) 150% 2) 200% 3) 300% 4) 350%
18. If in 2005, at MNIMS the number of Marketing students increases by 10% and if the number of
students in Systems remains the same, then find the percentage increase in Finance students if
the total intake in 2005 increases by 10% at MNIMS.
1) 30% 2) 20% 3) 15% 4) 12.5%
19. The total number of students in Systems at the four B-schools forms what percentage of the total
number of Finance students at these four B-schools?
1) 78.4% 2) 73.6% 3) 70.4% 4) 58.9%
20. There exists a student exchange programme between the two institutes MNIMS and JP Jain.
During this programme 50% of Systems students from JP Jain go to MNIMS and continue their
Systems specialisation. Then find the percentage change in the Systems students at MNIMS on
account of the students’ exchange programme, if no student of Systems in MNIMS goes to any
other college.
1) 60% 2) 80% 3) 70% 4) 40%
Directions (Q. 21-25): The following pie-chart and the line graph represents the total number
of companies visiting the campuses of B-schools in different specialisation fields from 2000 to
2004. The different fields are as following:
Code Field
M Marketing
F Finance
H HR

250

K 200

150

100 H
20%
M

H
14%
F
44% 42%
M
43%
H
F
34%

23%
M
40%

H
25%
F
35%
M
52% H
F
26%

22%

KUNDAN
M
F 45%
50 35% F = 182
F = 210 F = 187 H = 160

H = 72
0
2000 2001 2002 2003 2004
Years
21. On an average what percentage of the companies that visited the campuses of B-School between
2000 to 2004 were HR companies?
1) 11.4% 2) 25.04% 3) 21.2% 4) Can’t be determined
22. It can be said that for every three finance companies, ‘x’ number of marketing companies visited
the campuses of B-schools in the given five years. Then the value of x is
1) 2 2) 3 3) 4 4) Can’t be determined
23. By what per cent on an average (compounded) is the total no. of companies visiting the campus
increasing?
1) 18 2) 34 3) 29 4) 40
24. What is the difference between the number of marketing companies and that of finance compa-
nies visiting the campuses for the above five years?
1) 239 2) 309 3) 283 4) 301
25. The no. of HR companies visiting the campuses in 2002 is what per cent more as compared to that
of HR companies visiting the campuses in 2001?
1) 34% 2) 95% 3) 67% 4) 80%
Directions (Q. 26-30): Study the table given below and answer the questions.
‘SA RE GA MA PA’ is an entertaining TV programme in which some shortlisted singers perform
once every week. After listening to the singers Indian viewers cast their votes for their favourites.
The singer getting lowest no. of votes for that particular week goes out of the contest. So, next week
the no. of contestants performing reduces by one. This way the contest goes on till a single winner is
decided. The Indian viewers are divided into four regions called North India, West India, South India
and East India. In a particular week of the contest, five singers were performing named: Hemu,
Himani, Vineet, Debu and Nihira. One performance means singing one song, ie every week the
remaining contestants sing one song.
The candidatewise break-up of votes from different regions is given in the following table for that
week in which the above five singers performed.

Region
North India West India South India East India
Candidate

Hemu 06 00 51 11

Himani 19 100 10 17

Debu 09 00 18 46

Nihira 06 00 07 11

K Vineet 60

100% 100%
00

tition, in total how many songs were sung by the singers in the contest?
1) 78 2) 66 3) 77
14

100%

4) 65
27. In the week when the above five contestants performed, who got the highest votes from Indian
viewers?
1) Himani 2) Hemu 3) Vineet
15

100%
26. If the single winner was decided after the performance of the 11th week from start of the compe-

4) Can’t be determined

KUNDAN
28. The contestant who will not perform next week is
1) Debu 2) Nihira 3) Hemu 4) Can’t be determined
29. Among the five contestants, the one who got highest votes from the viewers of ‘Mumbai’ is (Mumbai
is a city falling in the region of West India)
1) Debu 2) Hemu 3) Himani 4) Can’t be determined
30. If the nos. of viewers voting from North India, West India, South India and East India are in the
ratio of 2 : 1 : 3 : 4, then by what percentage Vinit’s vote is more than Hemu’s?
1) 6.2% 2) 5.8% 3) 7.1% 4) None of these

Answers and explanations


1-5: The complete Venn-Diagram is as given below:
Mathematics Physics

2 20
5
8 6 6 10 Chemistry
13 7 11 2
Biology
14 9 12 4

1. 2; No. of students studying Physics only = 20.


2. 2; No. of students studying Mathematics only = 02.
3. 1; By observation only 7 students study all the 4 subjects.
4. 3; No. of students studying Physics & Maths = 5 + 6 + 7 + 9 = 27
No. of students studying Chemistry & Biology = 13 + 7 + 11 + 2 = 33.
But 7 students are common in both above.
required no. = 27 + 33 – 7 = 53.
5. 4; Students studying Physics and Chemistry and Biology = 7 + 11 = 18
Students studying Mathematics, Chemistry & Biology = 13 + 7 = 20
18
required ratio = 9 : 10
20
Exports Imports
6. 4; The ratio cannot be determined independent of the ratio Supply .
Demand
Supply Exports 0.60
Therefore in 2004, we can find Demand Imports as 1 .10 .
Exports
But we cannot find only .
Demand
Hence the question can’t be answered.
7. 1; The value of widgets imported in 2003 = Rs 300 crores.
= Average import price per widget × Volume of Imports.
= Average cost price per widget × Volume of import
Now, value of widgets exported = Average selling price per widget × Volume of exports.

K
Average cost price Volume of imports 1 1
Hence, Average selling price Volume of exports 1 0.3 0.5
Value of exports = 300 × 0.65 = Rs 195 crores
8. 4; Clearly, we can’t determine the ratio of domestic demand and domestic supply.
9. 4; Since the imports increased by a steady percentage every year, to find the year in which the
exports increased by the highest percentage we need to consider only EXIM ratios. By the obser-
vation of data, it is the highest from 2003 to 2004.
10. 4; Data inadequate.
11. 4; Let x be the percentage of total population surveyed in any city and y be the percentage of people
surveyed in the city who like Fardeen Khan.

KUNDAN
Then Fardeen has maximum liking in that city where ‘x × y’ is maximum.
For Bombay = 30 × 12 = 360
Delhi = 14 × 25 = 350
Kolkata = 6 × 23 = 138
Indore = 19 × 22 = 418
12. 2; Amitabh’s fans who don’t like Aamir = (62 – 36)% of (30% of 2000) = 26% of 600 = 156
13. 4; In Mumbai, 100 29 21 % = 50% don’t like either Salman or Shahrukh.
50% of (30% of 2000) = 300
Same way: in Kolkata = 299
Delhi = 250
Indore = 220
total = 1069
14. 1; In Delhi, for calculating maximum no. of people surveyed who don’t like any of the given stars,
let us assume that 52% fans who like Amitabh also like the other five stars.
So those who don’t like any of the above = 100 52 48% of (25% of 2000) 240
15. 3; At maximum the required no. can be 6% of 23% of 2000 = 27.6 27
16. 1; Code No. of students who opted for Finance
A 0.5 × 200 = 100
B 0.375 × 240 = 90
C 0.25 × 160 = 40
D 0.5 × 80 = 40
Hence, MNIMS
17. 2; From the previous question,
difference = 100 – 40 = 60
No. of students in BJIMS studying Finance = 12.5% of 240 = 30
Ans = 200%
18. 3; Specialisation 2004 2005
110
Marketing (MNIMS) 200 × 25% = 50 50 55
100
Systems (MNIMS) 200 × 25% = 50 50
Marketing (MNIMS) 200 × 50% = 100 —
110
Total (MNIMS) 200 220 200
100
Marketing (MNIMS) in 2005 = 220 – 50 – 55 = 115
% increase in Marketing (MNIMS) = 15%
19. 3; Total students opting for Systems
25% × 200 + 240 × 12.5% + 160 × 50% + 37.5 × 80 = 190
Total Finance students = 100 + 90 + 40 + 40 = 270
190
100 70 .4%
270

K
20. 2; % change =
80 50 %
200 25 %
100
40
50
100 80 %
21. 3; The answer can be determined and will be between 14% and 25% .
(Average of samples always lies between the lowest and the greatest sample.)
Alternative Method:
HR companies = 72 + 70 + 138 + 160 + 154 = 594
Total companies = 360 + 500 + 600 + 640 + 700 = 2800
594
2800
100 21 .2%

KUNDAN
22. 3; Total no. of Marketing companies = 162 + 220 + 258 + 256 + 364 = 1260
Total finance companies = 126 + 210 + 204 + 224 + 182 = 946
946 3
1260 x
4
x
23. 1; 360 1 700 x 18 %
100
Alternative Method: Compound rate of interest is always lower than simple rate of interest.
700 360 1
100
360 4 is less than 25% .
Answer can be only (1) since other options are more than 25% .
24. 2; From Q. No. 82, in this set it is = 1238 – 929 = 309.
210
25. 4; HR camp. in 2001 = 14 70
42
204
HR camp. in 2002 = 23 138
34
138 70
95 %
70

10th week
12 13
26. 3; The total songs = 2
 + 3 + 4
 + ... + 12
 = 1 77
11th week 9th week first week 2
27. 4; We can’t determine the answer because we don’t know the weightage of different regions in
total votes and the data is too messed up to point out a single singer with highest no. of votes.
28. 2; Whatever be the weightage of different regions, Nihira has got lowest % of votes from all the four
regions.
So she must be getting lowest votes overall from the Indian viewers and will be out of the
context next week.
29. 3; Himani has got 100% votes of West India
She has got 100% votes from Mumbai viewers also.
60 2 0 1 14 3 15 4 6 2 0 1 3 51 11 4
30. 1; Required % =
6 2 0 1 3 51 11 4
222 209 1300
= 100 6.2%
209 209

K
KUNDAN
Practice Exercise 20
Direction (Q. 1-6): Following table shows the assets and liabilities of the Reserve Bank of India
for the given years. Make use of data from previous questions if required.
ASSETS AND LIABILITIES OF RESERVE BANK OF INDIA
(All figures in Rs crore)
2000 1999 1998 1991 1981
Liabilities
Notes in circulation 192483 172541 148520 53784 13733
Govt deposits 541 68 453 94 789
Deposits of banks 80460 65359 59427 33829 4162
Other deposits 5551 6330 5239 4619 1570
Assets

K
Gold coin and bullion 12973 12624 13412 6654 226
Foreign assets 25294 12116 10038 14208 4320
Rupee securities 140967 145583 125956 46924 1110
Investments 3916 2916 2767 40286 1593
Loan and advances 37890 19876 13963 18516 3504
Other assets 5467 4398 6133 4103 4630
1. In the year 1999, 20% of investments of the government was in ‘Golden forest scheme’ and the

KUNDAN
same amount was expected to be invested in the year 2000. What was the share of investment for
‘Golden forest scheme’ in 2000?
1) 13% 2) 20% 3) 14.90% 4) 12.8%
2. If the percentage increase in 2001 over the previous decade is the same as that of 1991 over the
year 1981, the percentage increase in the foreign assets in 2001 over the previous year is
1) 22% 2) 15% 3) 9% 4) None of these
3. From year 1981 to year 2000, in how many years was there an increase in the number of notes in
circulation with respect to previous year?
1) 3 2) 2 3) 4 4) Indeterminable
4. Net liability (percentage) is defined as the percentage of net liabilities over total assets, and net
liabilities is the difference between total liabilities and total assets. In which year, out of those
mentioned above, is Net liability (% ) the maximum?
1) 1999 2) 1998 3) 2000 4) None of these
5. What is the rate of average annual compound growth rate of total assets for the decade 1981-91?
1) 20% 2) 24% 3) 18% 4) 16%
6. Textile units, steel-producing units and many manufacturing units are under Government’s Other
assets. Some of these public sectors are declared as sick units of NPA (Non-performing assets).
NPA are the government undertaking units which are running at a loss. 20% of other assets in
1999 are NPA, and in 2000 one more steel-producing unit of assets equal to 120 crores is declared
as NPA. What is the percentage of NPA in 2000 with respect to the Other assets?
1) 20% 2) 15% 3) 18% 4) Indeterminable
CAT’S EYE - CC - 129
Directions (Q. 7-11): Following table shows the indices (index numbers) of major industrial
production in a country. All the information is tabulated taking production in 1993-94 = 100.

Index numbers of industrial production (1993-94 = 100)


W eight 1998-99 1997-98 1996-97 1995-96 1994-95

General index 100.0 143.1 137.6 129.1 122.3 108.4


Mining & quarrying 10.5 120.3 122.4 115.6 117.9 107.6
Electricity 10.2 138.4 130.0 122.0 117.3 108.5
Manufacturing 79.4 146.7 140.6 131.8 123.5 108.5
Food 9.1 134.7 133.8 134.3 129.8 121.6
Beverages etc 2.4 178.5 158.1 132.4 116.7 103.0
Cotton textiles etc 5.5 115.9 125.6 122.7 109.5 99.1
Jute textile etc 0.6 106.0 114.3 97.8 102.4 95.1

K
Textile products including
2.5 153.8 158.7 146.3 133.7 98.5
garments
Wood and products 2.7 121.0 128.5 131.9 123.2 99.3
Paper and products 2.7 169.8 146.4 136.9 125.5 108.6
Leather and products 1.1 119.9 110.8 108.4 99.1 86.8
Rubber, plastic & petroleum
5.7 138.7 124.6 118.4 116.1 107.7
products

KUNDAN
Chemicals and products 14.0 149.8 140.5 122.7 117.2 105.3
Non-metallic mineral products 4.4 174.6 161.4 141.8 131.7 108.0
Basic metal and alloys 7.5 139.9 143.5 139.8 131.0 113.1
Metal products 2.8 141.6 120.2 110.9 100.6 104.7
Electrical machinery etc 9.6 152.1 149.6 141.7 134.7 112.8
Transport equipment 4.0 177.9 153.8 149.9 132.8 113.2
Other products 2.6 128.4 120.4 123.8 117.7 104.4
7. Of the following data, in how many categories in 1994-95 is the industrial production more than
the production in 1993-94?
1) 15 2) 13 3) 12 4) 14
8. How many categories have shown a consistent increase in production every year in the given
period of time?
1) 10 2) 11 3) 12 4) 13
9. How many categories have shown an increase of more than 50% in production in the period of
1994-99?
1) 8 2) 7 3) 5 4) None of these
10. The ‘Average index number’ of any category is defined as the summation of index number divided
by number of years. Which category has minimum average index number?
1) Cotton textiles 2) Paper and products 3) Mining and quarrying 4) Jute textile etc
11. In the given categories, which categories have an average index number of more than 120 points?
1) Mining & quarrying and Electricity 2) Transport equipment and Electrical machinery
3) Manufacturing and Other products 4) All of these
Directions (Q. 12-17): Study the graphs given below carefully and answer the questions that
follow. Use information given in previous question if required.
Production of selected Flow of institutional credit to
commodities (million tonnes) agriculture (Rs crores × 1000)
350 Co-operatives Commercial & RRB's
Foodgrains Pulses

300 Oilseeds Sugarcane


25
250
20
200
15
150
10
100

50 5

K
0 0
1990-91 1998-99 1980-81 1990-91 1998-99 1999-00

Import as percentage of gross fertiliser availability (%)

Potassic
1998-99
Phosphatic

Nitrogenous

KUNDAN
1990-91

0 20 40 60 80 100 120

12. Per capita net availability of foodgrain ( ) is defined as: Total foodgrain production (kg)/Popula-
tion. If in 1998-99 value of was registered as 360 and in 1999-2000 it became 400, then the
percentage increase in the population of India in the year 1999-2000 over the previous year was
1) 16.66% 2) 20% 3) 25% 4) Indeterminable
13. In the year 2000-01 value of was 420 and the foodgrain production was 250 m tonnes. What was
the increase in the population in year 2000-01 over 1990-91 (assuming that in 1998-99 was the
same as it was in 1990-91)?
1) 11 billion 2) 11 million 3) 110 million 4) Indeterminable
14. If in 1990-91, 20% of co-operative credit was sanctioned for sugarcane production then the yield of
sugarcane per 100 rupees of co-operative credit for the year 1990-91 was
1) 35 kg 2) 350 kg 3) 3500 kg 4) Indeterminable
15. Gross fertiliser consumption was registered as 150 tonnes, 200 tonnes and 250 tonnes of Nitrog-
enous, Phosphatic and Potassic respectively for the year 1998-99. Assuming that the import of
fertiliser was taken into consideration in order to fulfil the total consumption, then what was the
total fertiliser imported in 1998-99?
1) 200 tonnes 2) 250 tonnes 3) 215 tonnes 4) 260 tonnes
CAT’S EYE - CC - 131
16. If in 1990-91 the total production of agricultural products accounts for 52% of Gross Domestic
Product, then the Gross Domestic Product in 1990-91 was equivalent to the agricultural produc-
tion of ______. (In million tonnes)
1) 800 2) 850 3) 900 4) indeterminable
17. 30% of sugarcane was exported in 1990-91 and 33.33% yearly increase in the export of sugarcane
was reported next year. (Export rate was $ 89 per tonne). What was the change in the percentage
of the revenue generated from the export of sugarcane in 1991-92?
1) 30% 2) 33.33% 3) 25% 4) indeterminable
Directions (Q. 18-22): Figures given below show the percentage change in the key economic
indicators and major industrial growth for the period of 1996-97 to 2000-01, over the previous
year. Use data from previous questions if required.
KEY ECONOMIC INDICATORS MAJOR INDUSTRIAL GROWTH
10 30
Sof tw are
8 Consultancy
25

Percentage variation
6 Pharmaceutical
20
4

K
2

-2

-4

-6
1996-97 1997-98
GDP
Agricultural Production
Industrial Production

1998-99 1999-00

18. Which of the following is/are true?


2000-01
15

10

0
1996-97 1997-98 1998-99 1999-00 2000-01

KUNDAN
I) Growth of Software industries is the key contributor to the growth of Industrial production.
II) In 1998-99 Agricultural production has shown maximum change in percentage over the pre-
vious year.
III) GDP has shown consistent increase every year over the period 1996-01.
1) Only I 2) I and III only 3) I, II and III 4) II and III only
19. If index of GDP is considered as 100 in 1995-96, then how many years have GDP more than the
average GDP in the given period?
1) 2 2) 3 3) 1 4) None of these
20. If in 2000-01 Software contributed 15% to the total Industrial production and 10% growth was
registered in total Industrial growth next year, what should be the index for total Industrial pro-
duction in 2001-02 (considering index for Software in 1995-96 as 100)?
1) 1080 2) 1520 3) 1160 4) indeterminable
21. 80% of the Software production in 1996-97 was for international projects and this share reduced
to 70% in the year 1999-00. Percentage change in Software production for the international projects
in the given period
1) Decreased by 22% 2) Increased by 24% 3) Decreased by 30% 4) Increased by 30%
22. Which of the following statements are false?
I) Software production is the only sector which has shown continuous growth.
II) In all Industrial production over the mentioned period, the percentage change in growth is
maximum for Consultancy industry.
III) Software, Consultancy and Pharmaceutical industries contribute more than half of the total
Industrial production.
IV) Software, Consultancy and Pharmaceutical industries have shown consistent increase in
their production.
1) I & II only 2) II & III only 3) I, II and III only 4) I, II, III and IV
Directions (Q. 23-27): Study the following graph carefully to answer the questions given below
it.
Production of paper (in lakh tonnes) by three different companies A, B & C over the years
70 A B C

65
60

60
60
50

50

50

50
50
45

45

45

40
40

35
30

30

30

30
30

20

20
20

10

0
2001 2002 2003 2004 2005 2006

K
23. What is the difference (in tonnes) between the production of company C in 2001 and that of com-
pany A in 2004?
1) 500000 2) 5000000 3) 50000 4) 50000000
24. What is the percentage increase in production of company A from 2004 to 2005?
1) 85.71 2) 46.15 3) 84.23 4) 45.25
25. For which of the following years the percentage of rise/fall in production from the previous year is
the maximum for company B?
1) 2002 2) 2003 3) 2004 & 2005 4) 2005
26. The total production of company C in 2002 and 2006 is what percentage of the total production of
company C in 2004 and 2005?

KUNDAN
1) 170 2) 175 3) 178 4) 180
27. What is the difference (in lakh tonnes) between the average production per year of the company
with highest average production and that of the company with the lowest average production?
1) 20.83 2) 28.3 3) 21.38 4) 22.32
Answers and explanations
1. 3; In 1999 total investment was 2916 crores. Investment in “Golden forest” scheme = 0.2 ×2916
= 583.2 crores. Next year same amount is (583.2/3916)×100 = 14.89% .
2. 4; % increase in the foreign assets (1981 to 1991) = [(14208 - 4320) / 4320] × 100 230% approx.
Expected foreign assets in 2001 = 3.3 × 14208 = 46886.4 crores.
Percentage increase in 2001 over previous year = 46886.4 - 25294/25294) × 100 = 85.3% .
3. 4; In the given information, increase in circulation of notes over previous year cannot be calcu-
lated as the number of notes in circulation in 1982 is not known.
4. 4; From the given table we can calculate the net liabilities(% ):
1981 1991 1998 1999 2000

Total liabilities 20254 92326 213639 237968 279037

Total assets 15383 130691 172269 197513 226507

Net liabilities 4871 --- 41370 40455 5528

Net liability (%) 31.66% --- 24% 20.5% 23.29%


CAT’S EYE - CC - 133
5. 2; From the above calculation, if R is the rate of average annual growth, then
130691 = 15383 (1 + R/100)n, where n = 10. R = 24% approx.
6. 3; NPA in 1999 = 20% of 4398 = 880 crores.
NPA in 2000 = 880 + 120 = 1000 crores.
1000
% of NPA with respect to Other assets = × 100 = 18% approx.
5467
7. 4; Number of categories can be calculated directly from the table itself.
8. 1; 10 categories of the given have shown to have consistent increase in production.
9. 3; Beverages, Textile products including garments, Paper and products, Non-metallic mineral prod-
ucts and Transport equipment are the categories out of the given, which have shown increase
of more than 50% in the period 1994-99.
10. 4; In these types of problem we can directly add the index and check for the minimum value.
No need to actually calculate the value of ‘Average index number’ as the base is 5 for all the
cases.
(1) Cotton textile = 115.9 + 125.6 + 122.7 + 109. 5 + 99.1 = 572.8
(2) Paper and products = 169.8 + 146.4 + 136.9 + 125.5 + 108.6 = 687.2
(3) Mining and quarrying = 120.3 + 122.4 + 115.6 + 117.9 + 107.6 = 583.8
(4) Jute textile etc = 106.0 + 114.3 + 97.8 + 102.4 + 95.1 = 515.6

K
11. 2; In all categories given in the answer options, average index number is more than 120 points for
(2), that is Transport equipment = 145 and electrical machinery = 138.2.
12. 4; As the foodgrain production in 1999-2000 is not known, we cannot calculate population in 1999-
2000.
13. 3;
Year Foodgrain production in million tonnes Population in million
1990-91 360 175 486.1
2000-01 420 250 595.2
Hence increase in the population was 110 million (approx).

KUNDAN
14. 3 Sugarcane production in 1990-91 was 240 m tonnes.
Loan = 20% of 3500 crores = 700 crores
240 m tonnes
Production of sugarcane per 100 Rs of loan = 100 3428 kg
700 crore
15. 4;

Year Nitrogenous Phosphatic Potassic


1998-99 (Consumption) 150 tonnes 200 tonnes 250 tonnes
Import (% of gross fertilisers available) 8 65 102
Import (tonnes) 11.11 78.8 126.24

I = import, C = consumption, A = Availability


I
For Nitrogenous, 8% .08 and Availability = Consumption – Import
A
I 0.08
Thus, 0.08 I 150 11.11 tonnes.
C I 1.08
Similarly, we can calculate import for other fertilizers.
Total import = 11.11 + 78.8 + 126.26 = 216.15 tonnes.
16. 4; We do not know the total of agricultural production in 1990-91.
17. 4; As the export rate for the year 1991-92 is not given, so it cannot be calculated.
18. 4; Statement I is not true as the percentage contribution by Software to the total industrial pro-
duction is not given.
Statement II is true. As per the given information, agricultural production has shown maxi-
mum percentage increase over previous year in 1998-99 = {(100.1-93.1)/93.1} × 100 = 8.6% .
Statement III is also correct as GDP has shown consistent increase in the given period of time.
19. 1; Considering GDP for the year 1995-96 = 100 points,

1995-96 1996-97 1997-98 1998-99 1999-00 2000-01

100 101 105.5 111.5 118.25 124.25

560 .5
And average GDP = 112 .1
5
20. 3; In 2000-01 index for software was 100 (1.10) (1.14) (1.08) (1.15) (1.02) = 159.
15% of (Total industrial growth) = 162 points.
Thus total industrial growth = (162/15) × 100 = 1060 points
In the year 2001-02, 10% growth was registered.
Total industrial growth in 2001-02 = (1080 × 110) / 100 = 1166 approx.
21. 2; In 1996-97, 80% of 110 = 88 points.
In 1999-00, 70% of 156 = 109 points approx.

K
% change = [(109 – 88)/88] × 100 = 24% increase approx.
22. 3; Only statement IV is a true statement.
23. 1
65 35
24. 1; Percentage increase of A from 2004 to 2005 100 85 .71
35
25. 3; Percentage rise/fall in production for B

2002 2003 2004 2005 2006


11.11 20 50 50 33.33

KUNDAN
70
26. 2; Percentage production = 100 175
40
27. 1; Average production of A = 51.66
Average production of B = 43.33
Average production of C = 30.83
Difference of production = 51.66 - 30.83 = 20.83

CAT’S EYE - CC - 135


264 DATA INTERPRETATION

Chapter-9

Venn Diagram
Introduction
Pictorial representation of sets gives most of the ideas about sets and their properties in a much easier way
than the representation of sets given in language form. This pictorial representation is done by means of dia-
grams, known as Venn Diagram.
The objects in a set are called the members or elements of the set.
If A = {1, 2, 3, 4, 5, 6}, then 1, 2, 3, 4, 5 and 6 are the members or elements of the set A.
If B = {x : x is a positive integer divisible by 5 and x < 25} or, B = {5, 10, 15, 20}, then 5, 10, 15 and 20 are the
elements of the set B.

K
A  B (read as set A intersection set B) is the set having the common elements of both the sets A and B.
A  B (read as set A union set B) is the set having all the elements of the sets A and B. A - B (read as set A minus
set B) is the set having those elements of set A which are not in set B.
In other words, A - B represents the set A exclusively, ie A – B have the elements which are only in A.
Similarly, B - A represents the set B exclusively. We keep it in mind that n(A  B) = n(B  A) and n(A  B)
= n(B  A).
The number of elements of a set A is represented by n(A), but n(A - B)  n(B - A)

KUNDAN
Now, by the above Venn diagram it is obvious that
n(A) = n(A - B) + n(A  B) ..... (1)
n(B) = n(B - A) + n(A  B) ..... (2)
n(A  B) = n(A - B) + n(A  B) + n(B - A) .... (i)
Adding (1) and (2) we get,
n(A) + n(B) = n(A - B) + n(B - A) + n(A  B) + n(A  B)
or, n(A) + n(B) - n(A  B) = n(A - B) + n(B - A) + n(A  B) ... (ii)
From (i) and (ii), we have
n(A  B) = n(A) + n(B) – n(A  B) .... (3)
Let us see some worked out examples given below:

Solved Examples
Ex. 1: In a class of 70 students, 40 like a certain magazine and 37 like another certain magazine. Find
the number of students who like both the magazines simultaneously.
Soln: We have, n(A  B) = 70, n(A) = 40, n(B) = 37
Now, 70 = 40 + 37 – n(A  B)  n(A  B) = 77 – 70 = 7.
Ex. 2: In a group of 64 persons, 26 drink tea but not coffee and 34 drink tea. Find how many drink (i) tea
and coffee both, (ii) coffee but not tea.
Soln: (i) n(T  C) = 64, n(T - C) = 26, n(T) = 34
We have, n(T) = n(T - C) + n(T  C)
or, 34 = 26 + n(T  C)  n(T  C) = 34 – 26 = 8
(ii) Again, we have
n(T  C) = n(T) + n(C) – n(T  C)
or, 64 = 34 + n(C) – 8
 n(C) = 38
Now, n(C) = n(C - T) + n(T  C)
or, 38 = n(C - T) + 8
 n(C – T) = 38 - 8 = 30
Venn Diagram 265
Ex. 3: In a class of 30 students, 16 have opted Mathematics and 12 have opted Mathematics but not
Biology. Find the number of students who have opted Biology but not Mathematics.
Soln: n(M  B) = 30, n(M) = 16, n(M - B) = 12, n(B - M) = ?
We have, n(M) = n(M - B) + n(M  B)
or, 16 = 12 + n(M  B)
 n(M  B) = 16 - 12 = 4
Again, we have, n(M  B) = n(M) + n(B) - n(M  B)
or, 30 = 16 + n(B) – 4
or, n(B) = 30 - 12 = 18
Now, n(B) = n(B – M) + n(M  B)
or, 18 = n(B - M) + 4
 n(B – M) = 18 – 4 = 14
Ex. 4: In a class of 70 students, 40 like a certain magazine and 37 like another while 7 like neither.
(i) Find the number of students who like at least one of the two magazines.
(ii) Find the number of students who like both the magazines simultaneously.
Soln: We have, total number of students = 70 in which 7 do not like any of the magazines.
For our consideration regarding liking of magazines, we are left with (70 – 7 =) 63 students.
Thus, n(A  B) = 63, n(A) = 40, n(B) = 37

K
(i) The number of students who like at least one of the two magazines = n(A  B) = 63.
(ii) The number of students who like both the magazines simultaneously = n(A  B) = ?
We have, n(A  B) = n(A) + n(B) – n(A  B)
or, 63 = 40 + 37 – n(A  B)  n(A  B) = 77 – 63 = 14
Ex. 5: In a school, 45% of the students play cricket, 30% play hockey and 15% play both. What per cent
of the students play neither cricket nor hockey?
Soln: n(C) = 45, n(H) = 30, n(C  H) = 15
 n(C  H) = 45 + 30 - 15 = 60
ie, 60% of the students play either cricket or hockey or both.
So, the remaining (100 - 60 =) 40% students play neither cricket nor hockey.

KUNDAN
Ex. 6: Out of a total of 360 musicians in a club 15% can play all the three instruments — guitar, violin
and flute. The number of musicians who can play two and only two of the above instruments is 75.
The number of musicians who can play the guitar alone is 73.
(i) Find the total number of musicians who can play violin alone and flute alone.
(ii) If the number of musicians who can play violin alone be the same as the number of musicians
who can play guitar alone, then find the number of musicians who can play flute.
Soln: (i) Total number of musicians = 360

15% of 360 = 54 musicians can play all the three instruments.


Given that x + y + z = 75
Now, 73 + f + v + (x + y + z =) 75 + 54 = 360
 v + f = 360 – (73 + 75 + 54) = 158
(ii) Now we have v = 73
The number of musicians who can play flute alone,
f = (v + f) – v = 158 – 73 = 85
and the number of musicians who can play flute = f + x + y + 54 = 85 + 54 + (x + y)
We have x + y + z = 75, x + y = 75 - z.
As either x + y or z is unknown, we cannot find out the number of musicians who can play flute.
Hence, data is inadequate.
266 DATA INTERPRETATION
Ex. 7: Out of a total 85 children playing badminton or table tennis or both, total number of girls in the
group is 70% of the total number of boys in the group. The number of boys playing only badminton
is 50% of the number of boys and the total number of boys playing badminton is 60% of the total
number of boys. The number of children playing only table tennis is 40% of the total number of
children and a total of 12 children play badminton and table tennis both. What is the number of
girls playing only badminton?
7x
Soln: Let the number of boys be x, then x + = 85  x = 50
10
Number of girls = 85 - 50 = 35

Exercise
Directions (Q. 1-2): Study the following information carefully and answer accordingly:
Out of a total of 240 musicians in a club, 7.5% can play all the three instruments — guitar, violin and flute.
The number of musicians who can play two and only two of the above instruments is 45. The number of musi-
cians who can play the guitar alone is 60.
1. Find the total number of musicians who can play flute alone and violin alone.
1) 115 2) 117 3) 118 4) 121 5) None of these

K
2. If the number of musicians who can play violin only be the same as the number of musicians who can play
only guitar, then find the number of musicians who can play flute.
1) 56 2) 57 3) 162 4) Cannot say 5) None of these
Directions (Q. 3-8): Study the following information carefully and answer accordingly:
There are five high schools A, B, C, D and E in a certain town. Total number of high school students of the
town is 1800. The strength of school A is 20% and B is 37.5% of the total number of students of the town. D and
E have equal strengths. 40% students of A know only one language - Hindi. 60% students of D know only one
language - English. There are 111 more students in B who know Hindi exclusively than the number of students
of D who know English only. 55 students of C know Hindi but not English. 15 students of D know both the
languages. The strength of C is 37.5% of the strength of A. Two-fifths of students of B know both the languages.
The number of students of C who know English but not Hindi is 40 less than the number of the same category
of B. 97 students of E know only English and 20% students of A know both the languages. 28 students of E know

KUNDAN
both the languages.
3. What is the percentage of the number of students who know both the languages?
1) 22.33 2) 22.66 3) 22.22 4) 22.5 5) None of these
4. What is the difference between the number of students who know English and those who know Hindi exclu-
sively?
1) 250 2) 200 3) 400 4) 360 5) None of these
5. The number of students who know only Hindi of C is how many times those who know both the languages of
the same school?
2 1 1 2
1) 4 2) 3 3) 4 4) 3 5) None of these
3 3 3 3
6. Find the percentage of number of students who know Hindi exclusively.
1) 44.44 2) 55.55 3) 33.33 4) 66.33 5) None of these
7. What is the number of schools in which the number of students who know English only is more than the
average number of students who know English only?
1) 1 2) 2 3) 3 4) 4 5) None of these
8. What is the maximum difference between the number of students of a certain school who know only Hindi
and only English?
1) 195 2) 93 3) 165 4) 97 5) None of these
Directions (Q. 9-13): Study the following information carefully and answer accordingly:
1
In a group of 1440 persons, like Coca-Cola only, 37.5% like Pepsi only and 510 like Mirinda. 6.25% of
6
them like all the three drinks while 6 do not like even one of the drinks. The number of persons who like both
1
Mirinda and Pepsi only is half the number of persons who like both Coca-Cola and Pepsi only. like both Coca-
8
Cola and Mirinda only.
9. How many persons like Mirinda only?
1) 174 2) 160 3) 168 4) Data inadequate 5) None of these
Venn Diagram 267
10. What is the difference between the number of persons who like Coca-Cola and those who like Pepsi only?
1) 300 2) 118 3) 192 4) Data inadequate 5) None of these
11. Find the percentage of number of persons who like more than one drink.
1) 27.5 2) 33.9 3) 33.75 4) Data inadequate 5) None of these
12. In a class of 55 students 35 take tea, 27 take coffee and 12 take both. Find the number of students who take
neither tea nor coffee.
1) 10 2) 5 3) 15 4) 8 5) None of these
13. There are 1000 students, out of which 650 drink tea and 390 drink coffee and 30 students do not drink either
tea or coffee. How many students drink both tea and coffee?
1) 80 2) 90 3) 70 4) Data inadequate 5) None of these
Directions (Q. 14-18): Following Venn diagram shows the specialisation in different fields of some
players out of 120 players.

K
14. What is the percentage of those players who have specialised in bowling?
1) 12.50% 2) 30%

2) 12.50%
3) 37.50%

3) 5.83%
4) Can’t be determined

4) 23.33%
5) None of these
15. What is the percentage of those players who have specialised in any of the two departments?
1) 7.50% 5) None of these

KUNDAN
16. What is the percentage of those players who have specialised in only one department?
1) 32.43% 2) 45.83% 3) 54.39% 4) 60% 5) None of these
17. In a class of 150 students, 65 play football, 50 play hockey, 75 play cricket, 35 play hockey and cricket, 20
play football and cricket, 42 play football and hockey and 8 play all the three games. Find the number of
students who do not play any of these three games.
1) 101 2) 49 3) 51 4) Can’t say 5) None of these
18. In a class there are 200 students. 70% of them like Hindi, 30% like English and 20% like Sanskrit. Find the
maximum possible percentage of students who like all the three languages.
1) 20 2) 10 3) 5 4) Can’t say 5) None of these
Directions (Q. 19-23): Study the following information carefully and answer accordingly:
In the figure shown below circle I represents readers of BSC magazine, Circle II represents the students who
have joined Correspondence Course of BSC (Banking Services Chronicle), and circle III represents the students
who have joined Classroom Coaching of BSC Academy.

19. The students who have joined the Classroom Coaching but are neither readers of BSC nor associated with
BSC through Correspondence Course, are represented by the portion
1) G + D 2) G + F 3) C 4) C - (D + G + F)
268 DATA INTERPRETATION
20. The portion which represents the students who are readers of BSC as well as are pursuing Correspondence
Course is
1) G 2) E + G 3) A + B 4) None of these
21. Ranjan Mukherjee is a regular reader of BSC Magazine and is pursuing its Correspondence Course too but has
not joined its Classroom Coaching. Then which of the following groups does he belong to?
1) A 2) G 3) E + G 4) E
22. Priya is a regular reader of BSC Magazine, is pursuing its Correspondence Course too and is determined to
leave behind Ranjan Mukherjee after joining BSC Classroom Coaching. Then which of the following groups
does she belong to?
1) A 2) G 3) E + G 4) E
23. The readers of BSC Magazine have been represented by the portion
1) A + E + D + G 2) A + E + G 3) A 4) None of these
Directions (Q. 24-27): Study the following information carefully and answer accordingly:
Note: Use additional information given in any question for answering subsequent questions.

K
24. How many students study Geography or English?
1) 108 2) 91

2) 67
3) 62

3) 52
4) 130

4) 59
5) 115
25. If 32 students study only Geography, how many students study English?
1) 63 5) Can’t say

KUNDAN
26. If there are 123 students in the class, how many students study Economics?
1) 67 2) 62 3) 63 4) 52 5) None of these
27. How many students study Economics or Geography or both but not all three?
1) 28 2) 60 3) 68 4) 54 5) None of these
Directions (Q. 28-29): Study the following information carefully and answer accordingly:
There are 120 students in a class, who read Maths or History or English. It is known that no student can read
all three subjects. 24 read only Maths and History, 8 read only History and English and 21 read only Maths and
English. 32 read only Maths and 13 only History.
28. How many students read English?
1) 22 2) 30 3) 51 4) 54 5) None of these
29. If 9 of the students who read only Maths start to read all three subjects, find the percentage of students who
read History.
1) 50% 2) 53.33% 3) 60% 4) 40% 5) None of these
Directions (Q. 30-34): Study the following information carefully and answer accordingly:
A survey was conducted among 770 people who speak one or more languages from among Hindi, English and
Urdu. It was also found that 500 speak Hindi, 400 English and 300 Urdu.
(i) 30% of the Urdu-speaking people speak all three languages, which is 10% less than those who speak
Hindi and English both but not Urdu.
1
(ii) Number of people who speak Hindi and Urdu both but not English is 33 % less than the number of
3
people who speak only English.
(iii)Number of people who speak English and Urdu both but not Hindi is 30.
30. How many people speak only Hindi?
1) 190 2) 170 3) 120
4) Can’t be determined 5) None of these
31. How many people speak only English?
1) 190 2) 100 3) 90
4) Can’t be determined 5) None of these
Venn Diagram 269
32. How many people speak Hindi and Urdu both but not English?
1) 180 2) 120 3) 90
4) 150 5) None of these
33. By what per cent the number of people who speak only Urdu is less than those who speak Hindi and English
both but not Urdu?
2 1
1) 66 % 2) 33 % 3) 40%
3 3
4) Can’t be determined 5) None of these
34. By what per cent the number of people who speak only English is more than those who speak Hindi and Urdu
both but not English?
2
1) 40% 2) 66 % 3) 50%
3
4) Can’t be determined 5) None of these
Directions (Q. 35-36): Study the following information carefully and answer accordingly:
There are 200 students in graduation. Out of these 165 are supposed to study at least one of the subjects
from among Physics, Chemistry and Mathematics. 110 students study Physics, 80 students study Chemistry and
90 students Mathematics. 40 students study Physics and Chemistry but not Mathematics, 35 students study
Physics and Mathematics but not Chemistry and 20 students study Chemistry and Mathematics but not Physics.

K
35. How many students study all three subjects?
1) 10 2) 12 3) 15 4) Can’t say 5) None of these
36. What is the percentage of those students who study all the three subjects with respect to those admitted in
graduation?
1) 5.40% 2) 6.06% 3) 4% 4) Can’t say 5) None of these
Directions (Q. 37-42): Study the following information carefully and answer accordingly:
There are three companies A, B and C. The employees of the company speak at least one of the three languages,
viz English, Hindi and French, in following manner:
(i) In company A, 700 employees speak Hindi, 600 speak English and 555 French. In company B, 650
speak Hindi, 580 speak English and 700 speak French. And in company C, 500 speak Hindi, 600
English and 700 French.

KUNDAN
(ii) The number of employees of company A who speak only Hindi is equal to that of company C who speak
English and French but not Hindi. It is also equal to that of company B who speak all the three lan-
guages.
(iii) The number of employees of company C who speak only French is equal to 180, which is 20% more than
the number of employees of company B who speak only Hindi.
(iv) The ratio of the number of employees of company C who speak only English to the number of employees
of company A who speak only French to the number of employees of company B who speak only
Hindi is 2 : 4 : 5.
(v) The number of employees of company A who speak only English is equal to the number of employees of
company B who speak only French, which is equal to 180, which is also 25% less than those who speak
English and French but not Hindi in company C.
(vi) The number of employees of company C who speak Hindi and French but not English is equal to the
number of employees of company A who speak Hindi and English but not French, which is equal to the
number of employees of company B who speak English and French but not Hindi.
(vii) The number of employees of company A who speak French and Hindi but not English is 165, which is
10% more than those who speak Hindi and French but not English in company C.
37. How many employees speak Hindi and English but not French in company C?
1) 130 2) 80 3) 150 4) 170 5) None of these
38. How many employees speak all the three languages in company A?
1) 145 2) 125 3) 130 4) 150 5) None of these
39. How many employees speak any two of the three languages in company B?
1) 540 2) 410 3) 670 4) Can’t say 5) None of these
40. The number of employees of company A who speak English and French but not Hindi is what per cent more
than the number of those who speak only Hindi in company C?
1) 125% 2) 60% 3) 150% 4) 100% 5) None of these
41. What is the difference between the number of employees of company C who speak all the three languages and
the number of employees of company B who speak only English?
1) 10 2) 20 3) 50 4) 110 5) None of these
270 DATA INTERPRETATION
42. By what approximate per cent the number of employees of company B is more than that of C?
1) 4% 2) 6% 3) 8% 4) 10% 5) 12%
Directions (Q. 43-47): The following questions are based on the diagram given below:
P = Physics C = Chemistry M = Mathematics Class strength = 260.

Number of students passed in a subject


43. What is the percentage of students who have failed in all three subjects?
1) 5.8 2) 17.5 3) 35 4) 22.5 5) None of these
44. What is the percentage of students who have passed in two or more subjects?
1) 33 2) 29 3) 36 4) 25 5) 20

K
45. What is the percentage of students who have failed in at least one subject?
1) 96.5 2) 5.8 3) 65.0 4) 75.5 5) None of these
46. Taking any two subjects, which pair of subjects has the maximum number of students passed in at least one
of them?
1) Physics, Chemistry 2) Physics, Mathematics 3) Chemistry, Mathematics
4) Cannot be determined 5) None of these
47. To be promoted to the next class it is essential to pass in Mathematics and at least in one of Physics and
Chemistry. How many students are likely to be promoted to the next class?
1) 245 2) 160 3) 97 4) 48 5) Can’t be determined
Directions (Q. 48-52): Answer these questions on the basis of the information given below:
(i) In a class of 80 students the girls and the boys are in the ratio of 3 : 5. The students can speak only Hindi
or only English or both Hindi and English.

KUNDAN
(ii) The number of boys and the number of girls who can speak only Hindi is equal and each of them is 40%
of the total number of girls.
(iii)10% of the girls can speak both the languages and 58% of the boys can speak only English.
48. How many girls can speak only English?
1) 12 2) 29 3) 18 4) 15 5) None of these
49. In all how many boys can speak Hindi?
1) 12 2) 9 3) 24 4) Data inadequate 5) None of these
50. What percentage of all the students (boys and girls together) can speak only Hindi?
1) 24 2) 40 3) 50 4) 30 5) None of these
51. In all how many students (boys and girls together) can speak both the languages?
1) 15 2) 12 3) 9 4) 29 5) None of these
52. How many boys can speak either only Hindi or only English?
1) 25 2) 38 3) 41 4) 29 5) None of these
Directions (Q. 53-55): Study the following information carefully and answer accordingly:
i) In a school, a total of 220 students are studying together in two sections A and B in the ratio of 5 : 6. The
students are studying only English or only Sanskrit or both English and Sanskrit.
ii) The numbers of students studying only English from section A and of those studying both Sanskrit and
English from Section B are equal and each of them is 40% of the students who are studying only English
from section B.
iii) The number of students studying only Sanskrit from section A is 30% of the number of students studying
in section B and 60% of the students studying only English from section B.
53. How many students are studying both English and Sanskrit from section A?
1) 48 2) 16 3) 40 4) 36 5) None of these
54. How many students are studying only Sanskrit from section B?
1) 36 2) 10 3) 12 4) 24 5) None of these
55. Number of students studying only English from section B is what per cent more than that of the students
studying only English from section A?
1) 150% 2) 100% 3) 75% 4) 20% 5) None of these
Venn Diagram 271
Directions (Q. 56-57): Study the following informations carefully and answer accordingly:
A survey was conducted by an agency in 25000 houses. It was found that 48% used Head & Shoulders, 48%
used Clinic Plus and 53% used Pentene Shampoo. 12% used both Head & Shoulders and Clinic Plus only and
10% used both Clinic Plus and Pentene only.
56. How many people used both Head & Shoulders and Pentene only if 8% used all the three?
1) 2750 2) 2500 3) 3000 4) 2000 5) Data inadequate
57. How many people used only Pentene if 8% used all the three shampoos?
1) 5000 2) 6000 3) 8750 4) 8000 5) None of these
Directions (Q. 58-62): Read the following data to answer the questions that follow:
In a class of 106 students, each student studies at least one of the three subjects Maths, Physics and
Chemistry. 48 of them study Maths, 51 Physics and 53 Chemistry. 16 study Maths and Physics, 17 study Maths
and Chemistry and 18 study Physics and Chemistry.
58. The number of students who study exactly two subjects is
1) 31 2) 32 3) 33 4) 36
59. The number of students who study more than one subject is
1) 39 2) 41 3) 40 4) 42
60. The number of students who study all the three subjects is
1) 5 2) 6 3) 7 4) 4

K
61. The number of students who study exactly one subject is
1) 45 2) 55 3) 65 4) 70
62. The number of students who study Physics and Maths but not Chemistry is
1) 9 2) 11 3) 10 4) 12
Directions (Q. 63-67): Study the following Venn diagram and answer accordingly:
The following Venn diagram represents the results of a survey conducted by a market research firm NSD Ltd
to ascertain the profiles of a sample group. The diagram below shows the number of people who are Poets,
Sportsmen, Graduates or Orators. Refer to the diagram to answer the questions that follow:

KUNDAN
Note:
(1) P = Poets, S = Sportsmen, G = Graduates, O = Orators
(2) The figures in any region of the above diagram pertain to the “only” value for that region. For example, 3
persons are only (Orators + Sportsmen + Graduates) etc.
63. Number of Sportsmen who have at least three specialities is
1) 12 2) 21 3) 9 4) 30
64. Total number of people having at least one speciality is
1) 403 2) 321 3) 343 4) 340
65. Number of people having only one speciality exceeded the number of people having exactly two specialities by
1) 113 2) 111 3) 112 4) 110
66. The number of people having at least one of the described specialities for what percentage of the total sample?
1) 38% 2) 62% 3) 44% 4) Cannot be determined
67. Orators who were neither Sportsmen nor Graduates exceeded Poets who were neither Orators nor Graduates
by a margin of
1) 32 2) 61 3) 43 4) 27
Directions (Q. 68-72): Refer to the following data to answer the questions that follow:
The result of an exam is given below:
Out of 1000 students who appeared
(i) 658 failed in Physics
(ii) 166 failed in Physics and Chemistry
(iii) 372 failed in Chemistry, 434 failed in Physics and Maths
(iv) 590 failed in Maths, 126 failed in Maths and Chemistry
272 DATA INTERPRETATION
68. The number of students who failed in all the three subjects is
1) 178 2) 73 3) 106 4) 126
69. The number of students who failed in Maths but not in Chemistry is
1) 464 2) 392 3) 387 4) 472
70. The number of students who failed in Physics but not in Maths is
1) 318 2) 224 3) 378 4) 232
71. The number of students who failed in Chemistry but not in Physics is
1) 318 2) 198 3) 213 4) 206
72. The number of students who failed in Physics or Maths but not in Chemistry is
1) 558 2) 718 3) 628 4) 692
Directions (Q. 73-75): These questions are based on the following information:
A sports club has 80 members, out of which male and female members are in the ratio of 9 : 7 respectively.
All the members play either badminton or table tennis (TT) or both. 40% of the male members play only badminton.
20% of the female members play both the games, which is equal to the number of female members playing only
TT. Number of male members playing only TT is more than that of male members playing both the games by 3.
73. Number of female members playing badminton is what per cent of the total number of female members in the

K
club?
1) 80 2) 60 3) 75 4) 40 5) None of these
74. In all how many members play TT?
1) 39 2) 15 3) 22 4) 19 5) None of these
75. How many male members play both the games?
1) 17 2) 12 3) 19 4) 16 5) None of these
Directions (Q. 76-80): These questions are based on the following information:
In a class of 84 students boys and girls are in the ratio 5 : 7. Among the girls 7 can speak Hindi and English.
50 per cent of the total students can speak only Hindi. The ratio of the number of students speaking only Hindi
to that speaking only English is 21 : 16. The ratio of the number of boys speaking English only to that of girls
speaking English only is 3 : 5.

KUNDAN
76. What is the number of boys who speak both the languages ?
1) 4 2) 5 3) 3 4) 2 5) None of these
77. What is the number of girls who speak English only ?
1) 12 2) 20 3) 22
4) Cannot be determined 5) None of these
78. What is the ratio of the number of boys who speak Hindi only to that of girls who speak Hindi only?
1) 10 : 11 2) 11 : 10 3) 2 : 5
4) Cannot be determined 5) None of these
79. How many girls can speak Hindi ?
1) 29 2) 22 3) 27 4) 23 5) None of these
80. What is the ratio of the number of boys who speak English to that of girls who do so?
1) 3 : 5 2) 3 3) 5 : 8 4) 5 5) None of these
Directions (Q. 81-83): Study the following information to answer the given questions:
In a school, three languages are taught. Out of the total 600 students each one is required to study at least
one of the three, viz Gujarati, Tamil, Hindi. 20 students study all the three languages. 202 study only Hindi and
111 study only Gujarati. In all, 250 study Tamil. 57 study Hindi and Gujarati. 194 study only Tamil.
81. How many students, along with Tamil, study either Gujarati or Hindi (but not both)?
1) 36 2) 56 3) 16 4) Cannot be determined 5) None of these
82. In all, how many students study Gujarati?
1) 199 2) 181 3) 163 4) Cannot be determined 5) None of these
83. Which of the following statements is definitely true?
1) The total number of students studying Hindi cannot be less than 290.
2) The total number of students studying Hindi cannot be less than 260.
3) The total number of students studying Gujarati cannot be more than 199.
4) Not more than 93 students study more than one language.
5) None of these
Venn Diagram 273
Directions (Q. 84-88): Study the following information carefully to answer the questions:
The teachers’ colony has 2800 members, out of which 650 members read only English newspaper. 550 members
read only Hindi newspaper and 450 members read only Marathi newspaper. The number of members reading all the
three newspapers is 100. Members reading Hindi as well as English newspaper are 200. 400 members read Hindi as
well as Marathi newspaper and 300 members read English as well as Marathi newspaper.
84. Find the difference between the number of members reading English as well as Marathi newspaper and the
number of members reading English as well as Hindi newspaper.
1) 300 2) 200 3) 100
4) 50 5) None of these
85. How many members read at least two newspapers?
1) 600 2) 800 3) 500
4) 1000 5) None of these
86. Find the number of members reading Hindi newspaper.
1) 750 2) 980 3) 1000
4) 1020 5) None of these

K
87. How many members read only one newspaper?
1) 1560 2) 1650 3) 1640
4) 1540 5) None of these
88. Find the number of members reading no newspaper.
1) 150 2) 460 3) 550
4) 750 5) None of these
Directions (Q. 89-93): Study the following information carefully and answer the questions given
below it:
There are 2500 residents in a village. 1,375 residents from this village speak only their local language. 200
residents of the village speak the local language as well as English. The number of residents in the village who speak

KUNDAN
the local language as well as Hindi is 625. 300 residents of the village speak all the three languages ie, English, Hindi
and the local language.
89. The number of residents who speak English as one of the languages forms what per cent of the total residents
in the village?
1) 12 2) 8 3) 20
4) 18 5) None of these
90. The number of residents who speak only the local language forms what per cent of the total number of
residents in the village?
1) 45 2) 55 3) 58
4) 40 5) None of these
91. The number of residents who speak Hindi as one of the languages is approximately what per cent of the
number of residents who speak only the local language?
1) 67 2) 70 3) 61
4) 59 5) 63
92. What is the ratio of the number of residents who speak all the three languages to the number of residents who
speak the local language as well as Hindi?
1) 12 : 55 2) 10 : 25 3)14 : 55
4) 12 : 25 5) None of these
93. If 25 more people who can speak all the three languages come to reside in the village and 45 more people who
can speak the local language and Hindi come to reside in the village, what would be the difference between
the number of residents who can speak all the three languages and the number of residents who can speak
the local language and Hindi?
1) 325 2) 330 3) 340
4) 355 5) None of these
274 DATA INTERPRETATION

Answers and explanations


(1-2): 2. 4; Now, given that v = 60
1. 2; 7.5% of 240 = 18 f = 117 - 60 = 57
But, the number of musicians who can play flute
= f + (x + y) + 18 = 57 + 18 + (x + y). Since x + y is
not known so, the number of musicians who can
play flute cannot be determined.
(3-8): We symbolize the number of students who know
only Hindi, ie Hindi but not English by H - E, the
number of students who know only English by E
- H, the number of students who know both the
Given that x + y + z = 45 languages by H  E and the total strength of
Now, 60 + 18 + (x + y + z =) 45 + (f + v) = 240 schools by T.
or, 123 + (f + v) = 240 We have T = (H - E) + (E - H) + (H  E)
Now collecting the given pieces of information and
 f + v = 240 - 123 = 117 using the above formula, we get

K
KUNDAN
3. 3; Required percentage

=
400
1800
 100 
200
9
 22.22
(9-13):

4. 2; Required difference = (600 + 400) - 800 = 200


11 2
5. 4; 55 = x × 15  x = 3  33

 800  400 1200  100


6. 5; Required percentage =
1800 We have, 240 + x + 540 + 510 = 1440 - 6

200 2 x
=  66 or, x = 1434 – 1290 = 144 and = 72
3 3 2
9. 3; The number of persons who like Mirinda only
7. 2; Average number of students who know English
= 510 – (180 + 90 + 72) = 168
600 10. 5;Required difference
only = = 120.
5 = 240 + 144 + 180 + 90 – 540 = 654 – 540 = 114
So, A and D are the two desired schools. 11. 3;Total number of persons who like more than one
drink = 180 + 144 + 72 + 90 = 486
8. 1; Clearly for B, the difference is maximum and it
is (300 – 105 =) 195 486
 Required percentage = 1440  100 = 33.75%
Venn Diagram 275
12. 2;Required number of students = 55 - (23 + 12 + (19-23):
15) = 55 - 50 = 5 19. 3 20. 2 21. 4 22. 2 23. 1
(24-27):
24. 1;Students studying Geography or English
= (c + 13 + a + 14) + 16 + 25 = 67 + 16 + 25 = 108
25. 1; According to the question, c = 32
 a = 67 - (13 + 14 + 32) = 8
 Students studying English
= 14 + 8 + 16 + 25 = 63
13. 3;Let x be the number of students who drink both. 26. 4;67 + b + 16 + 25 = 123 or, b = 123 - 108 = 15
Students studying Economics
= 13 + 15 + 8 + 16 = 52
(with the help of Q.No. 25)
27. 5;Students studying Economics or Geography or
both but not all three = (67 - 8) + 15 + 16 = 90
(28-29):

K
650 – x + x + 390 – x + 30 = 1000
or, –x = 1000 – 1070 or, x = 70
(14-18):
14. 3;Total number of players who have specialised in
bowling = 15 + 11 + 9 + 10 = 45
45
 Required percentage = 120 × 100 = 37.50% 28. 3;32 + 24 + 13 + 8 + 0 + 21 + E = 120
E = Number of students who read only English
E = 120 - 98 = 22
 7  10  11
15. 4;Required percentage =    100  total number of students who read English
120 = 22 + 8 + 21 = 51
28 29. 5;Total number of students who read History

KUNDAN
=  100 = 23.33% = 24 + 9 + 8 + 13 = 54
120 54
Required % =  100 = 45%
 22  18  15  120
16. 2;Required percentage =    100 (30-34):
120

55
=  100 = 45.83%
120
17. 2; Number of students who play at least one game
= n(F H C) = 65 + 50 + 75 – 35 – 20 – 42 + 8
= 101
 Number of students who don’t play any of the
three games = 150 – 101 = 49.
18. 2;

(i) 30% of Urdu = 30% of 300 = 90


Number of people who speak Hindi and
English both but not Urdu = 100
(ii) Number of people who speak English and
Urdu both but not Hindi = 30
Therefore, Number of people who speak only
English = 400 - (100 + 90 + 3) = 180 ... (A)
S = 40 (iii) Now, with the help of (A),
For x to be maximum the othe r common Number of people who speak Hindi and Urdu
sections should be zero. Now, both but not English = 120 ... (B)
(140 – x) + (60 – x) + (40 – x) + x = 200 Therefore, number of people who speak only
 x = 20 Urdu = 300 – (120 + 90 + 30) = 60 ... (C)
 Required % = 10 Similarly, number of people who speak only
Hindi 500 – (100 + 90 + 120) = 190 ... (D)
276 DATA INTERPRETATION
30. 1;From (D). From (iii),
31. 5;From (A). Number of employees of company C who speak
32. 2;From (B). only French = 180
33. 3;Number of people who speak only Urdu  Number of employees of company B who speak
= 300 – (120 + 90 + 30) = 60 180
only Hindi =  100  150
100  60 120
Required less % =  100  40%
100 Combining (iii) and (iv), we have
:5 = 150  :4 = 120 and :2 = 60
180  120
34. 3;Required more % =  100  50% . From (v),
120 Number of employees of company A who speak
(35-36): only English = Number of employees of company
B who speak only French = 180
 Number of employees of company C who speak
English and French but not Hindi
180
=  100  240
75
Now, combining this with (ii), we have

K
Number of employees of company A who speak
only Hindi = Number of employees of company B
who speak all the three languages = Number of
employees of C who speak English and French
but not Hindi = 240
Let x be the number of students who study all
From (vii),
the three subjects. Then the number of students
Number of employees of company A who speak
who study only Physics = (35 – x)
French and Hindi but not English = 165
Number of students who study only Chemistry
= (20 – x)
 Number of employees of company C who speak
Hindi and French but not English
Number of students who study only Mathematics
= (35 – x) 165
 100  150

KUNDAN
=
Now, 110 + (20 – x) + 20 + (35 – x) = 165 110
or, x = 10 Now, when we combine this with (vi), the rest of
35. 1 our Venn-diagram will be filled.
10  100 37. 4;Number of employees of company C who speak
36. 5;Required % =  5% all the three languages = 700 - (180 + 240 + 150)
200
= 130
(37-42): Now, the number of employees of company C who
Try to depict all the given informations in Venn- speak Hindi and English but not French
diagram. = 600 - (240 + 130 + 60) = 170
A B 38. 1;Number of employees of company A who speak
all the three languages
= 700 - (240 + 150 + 165) = 145
39. 2;Number of employees of company B who speak
Hindi and English but not French
= 580 - (60 + 150 + 240) = 130
Number of employees of company B who speak
Hindi and French but not English
= 700 - (180 + 150 + 240) = 130
Total number of employees of company B who
C speak any two of the three languages
= 130 + 130 + 150 = 410
40. 3;Number of employees of company A who speak
English and French but not Hindi = 125
Number of employees of company C who speak
only Hindi = 50
125  50
 Required % =  100  150%
50
41. 5;Required difference = 130 - 60 = 70
Venn Diagram 277
42. 2;Number of employees in company B
= 700 + 60 + 130 + 150 = 1040
Number of employees in company C
= 700 + 60 + 170 + 50 = 980
1040  980
 Required % =  100  6%
980
(43-47):
Class strength = 260
Students passing in P + C + M = 9
Students pasing in P + C = 28 – 9 = 19
Students passing in P + M = 42 – 9 = 33
Students passing in M + C = 15 – 9 = 6
Students passing only in C = 63 – 19 – 6 – 9 = 29
Students passing only in M = 97 – 6 – 33 – 9 = 49
Students passing only in P = 85 – 9 – 19 – 33
= 24
Total students passing in at least one subject

K
= 63 + 97 + 85 – 28 – 42 – 15 + 9 = 169
53. 3 54. 1
43. 3;Students who have failed in all subjects
= 260 – 169 = 91 60  24
55. 1;Required % =  100 = 150%
44. 4;Students who have passed in two or more 24
subjects = 9 + 19 + 33 + 6 = 67 (56-57):
67
 Required %   100  25%
260
45. 1;Total number of students who have failed in at
least one subject = 260 – 9 = 251
251
 % value = 260  100  96.5%

KUNDAN
46. 3;P, C = 19 + 9 + 24 + 29 + 33 + 6 = 120
P, M = 33 + 9 + 24 + 49 + 6 + 19 = 140
M, C = 6 + 9 + 49 + 29 + 33 + 19 = 145
56. 1;Let x% people use both Head & Shoulders and
47. 4;9 + 33 + 6 = 48
Pentene only.
5  Percentage of people who used only Head &
(48-52): Number of boys in the class =  80  50
8 Shoulders = (28 - x)
Percentage of people who used only Pentene
 Number of girls in the class = 80 – 50 = 30
= (35 - x)
 28 – x + 12 + 18 + 8 + 10 + x + 35 – x = 100
or, 111 – x = 100  x = 11%
 Number of people who used both Head Shoul-
ders and Pentene only = 11% of 25000
= 2750
57. 2; Number of people who used only Pentene
= 24% of 25000 = 6000
(58-62): We have
48. 4 49. 5 50. 4
51. 2 52. 3
(53-55):
Number of students in section A
5
=  220 = 100
11
Number of students in section B
= 220 - 100 = 120
278 DATA INTERPRETATION
a + b + c + d + e + f + g = 106 72. 3;Number of students who failed in Physics or
a + e + d + b = 48 Maths but not in Chemistry
c + b + d + f = 51  n  P  M  C  n  C  1000  372  628
g + e + d + f = 53 (73-75):
b + d = 16; d + e = 17; d + f = 18 The whole information is as follows:
and from the standard formula, Total members : 80
n  A  B  C  n  A   n  B  n C   n  A  B
 n  B  C  n  C  A   n  A  B  C 
We get, 106 = 48 + 51 + 53 – 17 – 18 – 16 + d
 d = 5. Now, all the values can be obtained as
shown in the figure and all the questions can be
answered.
58. 4;b + e + f = 36
59. 2;b + d + e + f = 41
60. 1;d = 5
61. 3;a + c + g = 65 21  7  100  80%
62. 2;b = 11 73. 1: Required per cent =
35

K
Note: This question, and its solution, is so mechanical 74. 5; 19 + 22 = 41
and direct that with proper practice, you should 75. 2;It is obvious from the above figure.
be able to solve it very quickly. (76-80):
(63-67):
63. 2; 2 + 7 + 9 + 3 = 21
64. 4; Adding up all the values, we get required answer
= 340.
65. 3; Only one speciality = 19 + 63 + 101 + 28 = 211
Exactly two specialities = 53 + 11 + 23 + 12 = 99
 Required answer = 211 – 99 = 112
66. 4; The number of people having at least one speci-
ality is 340. But the total number of people sur-

KUNDAN
veyed is not known. Hence, percentage cannot 76. 3 77. 2 78. 1 79. 1
be determined. 80. 5; 12 + 3 : 20 +7 = 15 : 27 = 5 : 9
67. 1; (53 + 19) – (28 + 12) = 32 (81-83):
(68-72):
Let P be the set of the students who failed in
Physics, C be the set of the students who failed
in Chemistry, and M be the set of the students
who failed in Maths. Then
n(P) = 658, n(P  C) = 166,
n(C) = 372, n(P  M) = 434
n(M) = 590, n(M  C) = 126 and
n(P  M  C) = 1000
68. 3;The number of students who failed in all the three
subjects = n(P  M  C)
 n  P  M  C   n  P   n  M  n  C We have been given
 n  P  M  n  P  C  n  M  C A = 20, E = 111, F = 194, G = 202,
= 100 – 658 – 590 – 372 + 434 + 166 + 126 A + D = 57 and A + B + C + F = 250
= 106
69. 1;Number of students who failed in Maths but not
in Chemistry
 n  M  C  n  M  n  M  C  590  126  464
70. 2;Number of students who failed in Physics but
not in Maths
 n  P  M  n  P   n  P  M  658  434  224.
71. 4;Number of students who failed in Chemistry but
not in Physics
 n  C  P   n  C  n  C  P   372  166  206
Venn Diagram 279
Here, we have 86. 5; Number of members reading Hindi newspaper
A + B + C + D + E + F + G = 600 = 550 + 400 + 200 + 100 = 1250
 B + C = 600 - (111 + 37 + 194 + 20 + 202) 87. 2; Number of members who read only one news-
= 600 - 564 = 36 paper = 550 + 650 + 450 = 1650.
We can get B + C through other ways also. 88. 1; Number of members reading no newspaper
Note that A + B + C + F = 250 = 2800 - (650 + 550 + 450 + 400 + 300 +
or 20 + B + C + 194 = 250 200 + 100)
 B + C = 250 - (194 + 20) = 36. = 150.
81. 1; Here we need to find out the values of B and C (89-93)
together ie, 36. 89. 3; L  local language, E  English, H  Hindi
82. 4; Here we need to find out the sum of the values of
A, B, D and E. Since value of B is not known,
hence sum of the values of A, B, D and E can’t
be determined.
83. 4; Total number of students who study more than
one language = A + B + C + D = 20 + 36 + 37 = 93
(84-88):

K
84. 3;  Difference = (E + M) - (H + E) = 300 - 200
90. 2
Required percentage 

91. 1;Required percentage 


500
2500

925
1375
 100  20%

 100  67

92. 4;Required ratio = 300 : 625 = 12 : 25


93. 5;After addition people who speak all the three
languages = 300 + 25 = 325
After addition people who speak local language
= 100

KUNDAN
85. 4; Number of members who read at least 2 news- as well as Hindi = 625 + 45 = 670
papers = 400 + 300 + 200 + 100 = 1000.  Required difference = 670 - 325 = 345.
402 Concept of Arithmetic

Chapter-23

Time and Work


Introduction on Time and Work, there is a proportional
relation, hence we solve them by Unitary
In most of the problems based on Time and Work, Method and Ratio and Proportion (Also see
either the amount of time taken to finish a given the Chapter 11 and 12).
job or the amount of work done is to be calculated. (v) If to finish a certain piece of work, some
Unless otherwise specified, the amount of work men are employed and they finish the work
done is generally taken as unity (1). Also if it is in a certain time, then the relation between
given that a person (P) can finish a job in D days, work, man and time will be as follows:
then it implies that P alone can do the job in D (a) Work and Man: Number of persons
days. employed to do the work is directly
proportional to the amount of work
Important Points to Remember
done. (More t he number of per sons
( i ) Capacity of persons to do a piece of work is employed, more the work done).
always constant. That is, the person does (b) Time and Work: The number of days
the equal work every day. is directly proportional to work done.
( i i) In the question based on ‘Time and Work’, (More the number of days for which a
generally we find amount of work done in work was done, more shall be the total
unit time (1 day, 1 hour etc). Thus, if a

K KUNDAN
amount of work done).
man can do a piece of work in x days (or (c) Time and Man: The number of persons
hours or any other unit of time), then the employed is inversely proportional to
the number of days required to finish a
1
work done by him in one day will be   work. (More the number of persons
x  employed, less will be the time required
of the total work. to finish the work).
( i ii ) If A is twice as good as a workman as B, W e hav e alr eady discussed some pr oblems
then A will take half the time B takes to related to Time and Wor k in Chapter 11 and
finish a piece of work. Chapter 12. Her e, we will discuss some more
( iv) Application of the Concept of Unitary questions related to Time and Work in greater
Method and Variation: Since in problems details to understand deeply.

Solved Examples
Ex. 1: Amit can do a piece of work in 4 days So, work done by Amit and Sumit in 1 day
and Sumit can do it in 6 days. How
1 1 32 5
long will they take, if both Amit and =    
Sumit work together?  4 6   12  12
Soln: We have, time taken by Amit to do the Hence, Amit and Sumit can do the piece
work = 4 days
12 2
Time taken by Sumit to do the work of work in days ie 2 days.
= 6 days. 5 5
Ex. 2: Chhotu and Nitu together plough a field
1
 Work done by Amit in 1 day = in 4 days. Nitu alone takes 6 days to
4 plough the same field. In how many
days can Chhotu alone plough the field?
1
Work done by Sumit in 1 day = Soln: We have, time taken by Chhotu and Nitu
6 to plough the field together
= 4 days
Time and Work 403

Time taken by Nitu to plough the field 1


= 6 days. B and C’s 1 day’s work =
15
1 1
 Chhotu and Nitu’s 1 day’s work = C and A’s 1 day’s work =
4 20
1 Adding, we get 2 (A + B + C)’s 1 day’s
Nitu’s 1 day’s work = wor k
6
1 1 1 5  4  3 12 1
Now, =     
Chhotu’s 1 day’s work 12 15 20 60 60 5
= (Chhotu’s and Nitu’s 1 days’ work) 1 1
– (Nitu’s 1 day’s work)  (A + B + C)’s 1 day’s work =  .
2  5 10
1 1 32 1 Thus, A, B and C together can finish the
=  = 
4 6 12 12 work in 10 days.
Hence, Chhotu can plough the field in 12 Now,
days. A’s 1 day’s work = (A, B and C’s 1 day’s
Ex. 3: Kami, Kavya and Kirti can together work) - (B and C’s 1 day’s work)
weave a car pet in 4 days. Kami by  1 1  32 1
her self can weav e t he sam e si zed =    
 10 15   30  30
carpet in 12 days and Kirti can do it in
10 days. How long will Kavya take to So, A alone can finish the work in 30
do the work by herself? days.
Soln: We have, Now, B’s 1 day’s work = (A, B and C’s 1
time taken by Kami, Kavya and Kirti to day’s work) – (A and C’s 1 day’s work)
weave the carpet = 4 days.  1 1   2 1 1
Time taken by Kami to weave the carpet = =    
 10 20   20  20
12 days
and, time taken by Kirti to weave the carpet So, B alone can finish the work in 20

K KUNDAN
= 10 days. days.
 Kami, Kavya and Kirti’s 1 day’s work C’s 1 day’s work = (A, B and C’s 1 day’s
work) - (A and B’s 1 day’s work)
1
=  1 1  65 1
4 =    
 10 12   60  60
1 So, C alone can finish the work in 60
Kami’s 1 day’s work = and, Kirti’s 1
12 days.
Alternative Method:
1 Let A, B and C alone can finish the work
day’s work =
10 in x, y and z days respectively.
Now,  Work done by them for 1 day will be
Kavya’s 1 day’s work 1
1 1
= (Kami, Kavya and Kirti’s 1 day’s , y and respectively.
wor k) - (Kami’s 1 day’s work) - x z
(Kirti’s 1 day’s work) Now, according to the question
1 1 1 15  5  6 4 1 1 1 1
      ..... (1)
= = x y 12
4 12 10 60 60 15
Hence Kavya can do the whole of the work 1 1 1
  ..... (2)
in 15 days. y z 15
Ex. 4: A and B can do a piece of work in 12
1 1 1
days, B and C in 15 days, C and A in   ..... (3)
20 days. In how many days will they z x 20
finish it together and separately? On adding (1), (2) and (3),
Soln: We have, We have
A and B can finish the work in 12 days. 1 1 1 1 1 1
B and C can finish the work in 15 days. 2     =  
C and A can finish the work in 20 days. x y z  12 15 20
543 1
1 = 
 A and B’s 1 day’s work = 20 5
12
404 Concept of Arithmetic

the work in 10 days, find for how many


1 1 1 1
 x  y  z  10 .... (4) days B worked?
1
Now, on subtracting equations (1), (2) and Soln: A’s 1 day’s work =
(3) from the equation (4), we have 20

1 1 1 65 1 1
    B’s 1 day’s work =
z 10 12  60  60 30
 z = 60 (A + B)’s 1 day’s work

1 1 1 32 1 1 1 32 5 1
    =    
x 10 15  30  30 20 30 60 60 12
 x = 30 1 1
A’s 10 days’ work =  10 
1 1 1 1 20 2
  
y 10 20 20 Remaining work done by (A + B)
 y = 20 1 1
 A, B and C can finish the work in 30 = 1 
2 2
days, 20 days and 60 days respectively.
And from equation (4), A, B and C together 1
Now of the work is done by (A + B) in
can finish the work in 10 days. 12
Ex. 5: A can do a piece of work in 25 days 1 day.
and B can finish it in 20 days. They
work together for 5 days and then A 1
 of the work is done by (A + B) in 1 ×
goes away. In how many days will B 2
finish the remaining work?
Soln: We have, 1
12 × = 6 days.
A can finish the work in 25 days. 2

K KUNDAN
B can finish the work in 20 days. Hence, B worked for 6 days.
1 Ex. 7: A can do a piece of work in 36 days, B
 A’s 1 day’s work = in 54 days and C in 72 days. All the
25 three began the work together but A
1 left 8 days and B 12 days before the
B’s 1 day’s work = com pl et ion of t he wor k. How m any
20
days in all did C put in till the entire
So, (A + B)’s 1 day’s work work was finished?
 1 1  45 9 1
=    
 25 20   100  100 Soln: A’s one day’s work = ,
36
 9  9 1
 (A + B)’s 5 day’s work =  5   B’s one day’s work = and
 100  20 54
 9  11 1
Remaining work = 1   C’s one day’s work =
 20  20 72
The remaining work is done by B. Now suppose the work lasts for x days.
Now,
x 8
the whole work is done by B in 20 days. A’s (x – 8) days’ work = ,
36
11
 of t he wor k is done by B in x  12
20
B’s (x – 12) days’ work = and
54
 11 
 20   days = 11 days. x
 20 
C’s x days’ work =
Hence, the remaining work is done by B 72
in 11 days. x  8 x  12 x
Ex. 6: A and B can separately do a piece of Hence,   1
work in 20 and 30 days respectively. 36 54 72
They work together for some days and Where 1 denotes total work
then B stops. If A completes the rest of Multiplying both sides by 216, we have
Time and Work 405

6x – 48 + 4x – 48 + 3x = 216 Soln: We have, length of the platform = 600 m


or, 13x = 216 + 96
2  2
or, 13x = 312  rd of the platform =  600   m
or, x = 24 3  3
Hence, C worked for 24 days. = 400 m
Ex. 8: A and B can do a piece of work in 12
1
days. B and C together do it in 15 days. Now, in 2 hours, Somari sweeps 600
If A is twice as good a workman as C, 2
find in what time B will alone do it? m.
Soln: According to the question, A and B can  in 1 hour, Somari sweeps
do a piece of work in 12 days.
   
1  600   600  2
 (A + B)’s 1 day’s work =  m    m   600   = 240 m
12  21   5   5
   
 2   2 
1
or, A + B = .... (i)
12 1
Again B and C can complete the work in In 1 hours, Imarati sweeps 400 m.
2
15 days.
 In 1 hour, Imarati sweeps
1
 B+ C= ..... (ii)    
15  400   400  2
Subtracting (ii) from (i), we have,  m  m   400   m
 11   3   3
1 1 54 1    
    2   2 
A – C =
12 15 60 60
2
1 = 266 m
or, A – C = .... (iii) 3
60
Clearly, Imarati sweeps more length in 1

K KUNDAN
Also according to the question, A is twice hour than Somari sweeps in the same
as good a workman as C. time.
 A = 2C Hence, Imarati sweeps more speedily than
Substituting the value of A in (iii), Somari.
1 Ex. 10: A and B together can do a piece of work
2C – C = in 12 days which B and C together can
60
do in 16 days. After A has been working
1 at it for 5 days and B for 7 days, C
or, C = finishes it in 13 days. In how many days
60
could each do the work by himself?
Substituting the value of C in (ii), Soln: Let the whole work be 1.
1 1 1
B+ = A and B in 1 day do of the work.
60 15 12
1 1 4 1 3 1
or, B =     1
15 60 60 60 20 B and C in 1 day do of the work.
16
1 A’s 5 days’ work + B’s 7 days’ work + C’s
or, B’s 1 day’s work = . 13 days’ work = 1.
20
or, A’s 5 days’ work + B’s 5 days’ work +
Hence, B can complete the work in 20
B’s 2 day’s work + C’s 2 days’ work + C’s
days.
11 days’ work = 1
Ex. 9: Somar i sweeps 600 m long railway
1 5 2
plat for m in 2 hour s. Hi s wi fe   + C’s 11 days’ work = 1.
2 12 16
2  5 2  11
I mar ati sweeps
3
r d of the sam e  C’s 11 days’ work = 1     .
 12 16  24
1
platform in 1 hours. Who sweeps 1
2  C’s 1 day’s work =
more speedily? 24
406 Concept of Arithmetic

Now, on subtracting equation (ii) from


1 1 1
 B’s 1 day’s work =   equation (i),
16 24 48
4 1
1 1 1 
  y 10
 A’s 1 day’s work = .
12 48 16
 y = 4 × 10 = 40 days
 A, B and C can do the work in 16, 48 On putting the value of y in equation (ii)
and 24 days respectively. and (iii), we have,
Ex. 11: To do a certain piece of work B would
take three times as long as A and C 1 1 3
  0
together and C twice as long as A and x z 40
B together. The three m en by t heir
united exertions can complete the work 1 1 3
or,   .... (iv)
in 10 days. How long would each take x z 40
by himself?
Soln: According to the question, 3 times B’s daily 1 1 2
and   0
work = (A + C)’s daily work. x 40 z
Add B’s daily work to both sides.
2 1 1
 4 times B’s daily work or,   .... (v)
z x 40
1
= (A + B + C)’s daily work = Now, on adding equations (iv) and (v), we
10 have
1 1 2 3 1
 B’s daily work = .... (1)   
40 z z 40 40
Also, 2 times C’s daily work = (A + B)’s
3 1
daily work. or, 
Add C’s daily work to both sides. z 10
 3 times C’s daily work = (A + B + C)’s  z = 30 days

K KUNDAN
Now, on putting the value of z in equation
1
daily work = . (v), we have
10
2 1 1
 
1 30 x 40
 C’s daily work = .... (2)
30
1  1 1  83 1
Now A’s daily work  x   15  40    120   24
1  1 1  1
=    .... (3)  x = 24 days
10  40 30  24 Hence A, B and C can do the work in 24,
 A, B and C can do the work in 24, 40 40 and 30 days respectively.
and 30 days respectively. Ex. 12: A and B can finish a piece of work in
Alternative Method: 15 days. A and C can take 2 days more
Let A, B and C separately do the work in than B and C take to finish the same
x, y and z days respectively. piece of work. The three together can
finish the work in 8 days. In how many
1 1
Hence their 1 day’s work will be , y days coul d each do t he wor k by
x himself?
Soln: Let A, B and C separately can finish the
1
and respectively. work in x, y and z days respectively.
z Again, suppose that B and C together can
Now, according to the question, finish that work in a days.
Hence, A and C together will finish the
1 1 1 1
   ..... (i) work in (a + 2) days.
x y z 10 Now, according to the question,
1 1 3 1 1 3 1 1 1
  or, x  z  y  0 .... (ii)   .... (i)
x z y x y 15

1 1 2 1 1 2 1 1 1
  or, x  y  z  0 .... (iii)   .... (ii)
x y z y z a
Time and Work 407

Ex. 13: Mohan and Sohan working separately


1 1 1
  .... (iii) and can dig a trench in 10 days and 12 days
x z a 2 respectively. If they are working for 1
1 1 1 1 day alternately, Mohan beginning, in
   how many days will the trench be dug?
x y z 8 .... (iv)
1
On subtracting (i) from (iv), we have Soln: Mohan’s 1 day’s work =
10
1 1 1 7
   1
z 8 15 120 Sohan’s 1 day’s work =
12
120 (Mohan and Sohan)’s 10 days’ working
 z = days
7
1 1 11
Now, on putting the value of z in equations alternately = 5  5 
(ii) and (iii), we have, 10 12 12

1 7 1 11 1
  Remaining work = 1  
y 120 a 12 12

1 1 7 1
or, y  a  120 .... (v) Now of work done by Mohan in 1 day.
10

1 7 1
1
   of the work done by Mohan in 1 ×
x 120 a  2 12

1 1 7 1 5
or,   .... (vi) 10 × days = days
x a  2 120 12 6
Hence, trench will be dug in
1 1
5

K KUNDAN
On putting the value of and y in  5
x = 10   days = 10 days.
 6 6
equation (i), we have
Ex. 14: Two women, Gangi and Jamni, working
1 7 1 7 1 separately can mow a field in 8 and 12
   
a  2 120 a 120 15 hours respectively. If they work for an
hour alternately, Gangi beginning at 9
1 1 1 7 7 am, when will the mowing be finished?
or,    
a  2 a 15 120 120
1
Soln: In the first hour Gangi mows of the
2a  2 11 8
or, a (a  2)  60 field.
or, 120a + 120 = 11a2 + 22a 1
or, 11a2 – 98a – 120 = 0 In the second hour Jamni mows of
12
or, 11a2 – 110a + 12a – 120 = 0
the field.
or, 11a (a – 10) + 12 (a – 10) = 0
or, (a – 10) (11a + 12) = 0 1 1  5
 a = 10  In the first 2 hours    or of
 8 12  24
Now, on putting the value of a in equation
the field is mown.
(v) and (vi), we have,
1 1 7 5 1 5 5
     In 8 hours  4 or of the field is
y 10 120 120 24 24 6
 y = 24 mown.
1 1 7 3 1  5 1
    Now 1    of the field remains to be
x 12 120 120 40  6 6
or, x = 40 mown.
Hence, A, B and C separately can finish
1
120 In the 9th hour Gangi mows of the
t he wor k in 40, 24 and days 8
7
respectively. field.
408 Concept of Arithmetic

1 1 or, Spinning 1 hour daily, Kanta can spin


Jamni will finish the mowing of    1 kg cotton-balls in 6 × 3 = 18 days
6 8
or, Spinning 1 hour daily, Kanta can spin
1  1 1  1 10 kg cotton-balls in (18 × 10) = 180 days
or, of the field in    or of
24  24 12  2 or, Spinning 4 hours daily, Kanta can spin
an hour. 180
1 10 kg cotton-balls in days = 45 days
 4
 The total time required is  8  1   or
 2 Alternative Method:
1 Here there are three quantities: Hours, Kg
9 hours. and Days. We have to find the number of
2 days, hence days should be in the last
Since, the work was started at 9 am, it column as given below:
1
would be finished at 6 pm.
2
Ex. 15: A is thrice as good a workman as B,
and is therefore able to finish a piece Following relationship exists:
of work in 60 days less than B. Find (i) More hours , l es s number of days
the time in which they can do it working (Inverse)
together. (ii) More kgs, more number of days (Direct)
Soln: ‘A is thrice as good a workman as B’ means Hence,
that if A does a piece of work in 1 day 4 : 3
:: 12 : x
then B does it in 3 days. Hence if the 2:10 
difference be 2 days, then A does the work
in 1 day and B in 3 days. But here the or, 4 × 2 : 3 × 10 :: 12 : x
difference is 60 days. Therefore A does (Compounding the ratios)
the work in 30 days and B in 90 days. or, 4 × 2 × x = 3 × 10 × 12
Now (A + B)’s 1 day’s work (  Product of extreme terms = Product of

K KUNDAN
mean terms)
1 1 2
=   . 3  10  12
30 90 45 x= = 45 days
4 2
45 1 [Note: To understand the above process,
 the required time is or 22 days.
2 2 please refer to the Chapter 12 (Ratio and
Ex. 16: A certain number of men can do a work Proportion)].
in 60 days. If there were 8 men more it Ex. 18: If 3 men or 4 women can reap a field in
could be finished in 10 days less. How 43 days, how long will 7 men and 5
many men are there? women take to reap it?
Soln: The original number of men together with Soln: 3 men or 4 women means 3 men’s work =
8 men more can finish the work in (60 – 4 women’s work.
10 =) 50 days. 1
Now 8 men can do in 50 days what the 3 men reap of the field in 1 day.
original number of men can do in 10 days. 43
 In 50 days, a work is done by 8 men. 1
 In 1 day, a work is done by (8 × 50)  1 man reaps of the field in 1 day.
129
men.
.... (1)
 8  50 
 In 10 days, a work is done by  
 10  1
4 women reap of the field in 1 day.
40 men. 43
Ex. 17: Spinning 3 hours daily, Kanta can spin
2 kg cotton-balls in 12 days. Spinning 1
 1 women reaps of the field in 1
4 hours daily, how many days will she 172
take to spin 10 kg cotton balls? day. ..... (2)
Soln: Spinning 3 hours daily, Kanta can spin 2  7 men and 5 women reap
kg cotton-balls in 12 days.
 Spinning 3 hours daily, Kanta can spin  7 5  1
   of the field in 1 day.
 129 172  12
 12 
1 kg cotton-balls in   = 6 days.  7 men and 5 women will reap the whole
 2  field in 12 days.
Time and Work 409

Alaternative Method: or, x2 - 12x - 2x + 24 = 0


3 men = 4 women or, (x - 12) (x - 2) = 0
or, x = 12 or x = 2
4
 1 man = women But x cannot be less t han 6. So,
3 x = 12.
4  28 Hence, B alone can finish the work in 12
 7 men =   7   women. days.
3  3
Ex. 20: If 12 men and 16 boys can do a piece of
 28  43
 7 men + 5 women =   5  work in 5 days and 13 men and 24 boys
 3  3 can do it in 4 days, compare the daily
women. work done by a man with that done by
The question now becomes a boy.
‘If 4 women can reap a field in 43 days, Soln: 12 men and 16 boys can do the work in 5
43 days .... (1)
how long will women take to reap it?’ 13 men and 24 boys can do the work in 4
3
days .... (2)
Here, there are two quantities: Women and Now it is easy to see that if the number of
Days. workers be multiplied by any number, the
Inver se r elat ionship exist s bet ween time must be divided by the same number.
women and days ie more women, less days. Hence multiplying the number of workers
in (1) and (2) by 5 and 4 respectively, we
get
60 men and 80 boys can do the work in 1
day.
52 men and 96 boys can do the work in 1
43
Hence, : 4 :: 43 : x day.
3  60 men + 80 boys = 52 men + 96 boys
43  60 men - 52 men = 96 boys - 80 boys
or, × x = 43 × 4  8 men = 16 boys

K KUNDAN
3
 1 man = 2 boys.
(  Product of extreme terms = Product of
mean terms) a man' s work 2
 a boy' s work  1
4  43  3
 x = = 12 days. Ex. 21: If 12 men and 16 boys can do a piece of
43
work in 5 days and 13 men and 24 boys
Ex. 19: A takes 6 days less than the time taken
can do it in 4 days, how long will 7
by B to finish a piece of work. If both A
men and 10 boys take to do it?
and B together can finish it in 4 days,
Soln: We have, as in the above example,
find the time taken by B to finish the
5 × (12 men + 16 boys)
work.
= 4 × (13 men + 24 boys)
Soln: Suppose B alone takes x days to finish
 60 men + 80 boys = 52 men + 96 boys.
the work. Then, A alone can finish it in
 8 men = 16 boys
(x - 6) days.
 1 man = 2 boys.
Now, (A’s one day’s work) + (B’s one day’s
Now,
1 1  12 men + 16 boys = 24 boys + 16 boys
work) =    = 40 boys.
x x  6
7 men + 10 boys = 14 boys + 10 boys
1 = 24 boys.
and, (A + B)’s one day’s work = . The question now becomes
4
“If 40 boys can do a piece of work in 5
1 1 1 days how long will 24 boys take to do it?”
  
x x 6 4 40 boys do a piece of work in 5 days.
 1 boy does a piece of work in 5 × 40
x 6x 1
or,  days.
x ( x  6) 4
5  40
2x  6 1  24 boys do the piece of work in
or,  24
x 2  6x 4
or, 8x - 24 = x2 - 6x 1
days ie 8 days.
or, x2 - 14x + 24 = 0 3
410 Concept of Arithmetic

Ex. 22: If 5 men and 3 boys can reap 23 hectares


 3 1
in 4 days and if 3 men and 2 boys can Hence all men must work till 1   
reap 7 hectares in 2 days, how many  4  4
boys must assist 7 men in order that of the field is reaped.
they may reap 45 hectares in 6 days.
1  20 
Soln: 5 men + 3 boys can reap 23 hectares in 4 Now 25 men reap of the field in  
days. ..... (1) 4  4 
3 men + 2 boys can reap 7 hectares in 2 5 days.
days. ..... (2) Hence, 15 men should leave the work after
 from (1), 5 days.
14 (5 men + 3 boys) can reap 23 × 14 Ex. 24: 8 men and 12 boys can finish a piece of
hectares in 4 days .... (3) work in 10 days while 6 men and 8
From (2), boys can finish it in 14 days. Find the
23 (3 men + 2 boys) can reap 7 × 2 × 23 time taken by one man alone and that
hectares in 4 days ..... (4) by one boy alone to finish the work.
 14 (5 men + 3 boys) Soln: Suppose that one man alone can finish
= 23 (3 men + 2 boys), the work in x days and one boy alone can
 70 men + 42 boys = 69 men + 46 boys. finish it in y days. Then,
 1 man = 4 boys.
1
Now 5 men + 3 boys = 23 boys. One man’s one day’s work =
 23 boys can reap 23 hectares in 4 days x
 1 boy can reap 1 hectare in 4 days 1
 4 boys can reap 1 hectare in 1 day and, one boy’s one day’s work = y
 4 × 45 boys can reap 45 hectares in 1
day
8
4  45  8 men’s one day’s work =
x
 boys can reap 45 hectares in 6
6
12
days.

K KUNDAN
and, 12 boys’ one day’s work = y
 30 boys can reap 45 hectares in 6 days
But 30 boys = 28 boys + 2 boys Since 8 men and 12 boys can finish
= 7 men + 2 boys the work in 10 days.
Hence 2 boys must assist 7 men.
Ex. 23: 25 men can reap a field in 20 days.  8 12 
 10   1
When should 15 men leave the work, if x y 
1 80 120
the whole field is to be reaped in 37  1
2 or, x y .... (i)
days after they leave the work?
Soln: 25 men can reap the field in 20 days. Again, 6 men and 8 boys can finish
the work in 14 days.
 20  25 
 10 men can reap the field in   6 8
 10   14    1
50 days. x y
When 15 men leave the work, 10 men 84 112
remain. or,  1 .... (ii)
x y
 In 50 days, 10 men can reap the whole
of the field. 1
1
1 Putting = u and y = v in (i) and (ii),
x
 In 1 day, 10 men can reap of the
50 we get
whole of the field. 80u + 120v – 1 = 0
1 84u + 112v – 1 = 0
 In 37 days, 10 men can reap By cross-multiplication, we have
2
u v
 1  
 37   120  112  80  84
 2   3 of the field.
 50  4 1
  
  80  112  120  84
Time and Work 411

Height and man: Less height, less men


u v 1
or,   (Direct)
 8  4  1120 Days and man: Less days , more men
(Inverse)
8 1
or, u =  and Hence,
 1120 140
4 1
v= 
 1120 280

1 1 1
Now, = u    x = 140
x x 140 6 : 9
1 1 1 18 : 32

and, y = v  y  280  y = 280. 2 : 3 :: 8 : x
12 : 9
Thus, one man alone can finish the work 
8 :10 
in 140 days and one boy alone can finish
the work in 280 days. or, 6 × 18 × 2 × 12 × 8 :
Note: Let a 1x + b 1 y + c 1 = 0 and 9 × 32 × 3 × 9 × 10 :: 8 : x
a 2 x + b 2y + c 2 = 0 be a system of (Compounding the ratios)
simultaneous linear equations in two or, 6 × 18 × 2 × 12 × 8 × x
= 9 × 32 × 3 × 9 × 10 × 8
a1 b1
variables x and y such that a  b ie (  Product of extreme terms = Product of
2 2 mean terms)
a1b2  a2b1  0 . Then the system has a 9  32  3  9  10  8
 x= = 30
unique solution given by 6  18  2  12  8
 required number of men = 30 men
 b1c 2  b2c1   c1a 2  c 2a1 
x =  a b  a b  and y =  a b  a b  Ex. 26: A garrison of 3300 men had provisions

K KUNDAN
 1 2 2 1  1 2 2 1 for 32 days, when given at the rate of
The above solution is generally written 850 grams per head. At the end of 7
as days a reinforcement arrives and it
x y z was found that now the provisions will
  last for 17 days more, when given at
b1c 2  b2c1 c1a 2  c 2a1 a1b2  a 2b1 the rate of 825 gm per head. What is
x y z the strength of the reinforcement?
or, b c  b c  a c  a c  a b  a b Soln: Let the strength of reinforcement be x.
1 2 2 1 1 2 2 1 1 2 2 1
According to the given condition,
For mor e det ails see the chapter 3300 × 32 × 850 = 3300 × 850 × 7 + (3300
“Elementary Algebra”. + x) × 825 × 17
Ex. 25: If 8 men, working 9 hours per day can or, (3300 × 32 × 850) - (3300 × 850 × 7) =
build a wall 18 metres long, 2 metres (3300 + x) × 825 × 17
wide and 12 metres high in 10 days, or, 3300 × 25×850 = (3300+x) × 825×17
how many men will be required to build or, 3300 × 50 = (3300 + x) × 33
a wall 32 metres long, 3 metres wide or, 5000 = 3300 + x
and 9 metres high by working 6 hours or, x = 1700
a day in 8 days? So, the strength of new reinforcement
Soln: Here following are the quantities: Hours, = 1700
length, width, height, days and men. We Ex. 27: A and B together can do a job in 12
have to find the number of men. Hence, hours. Had A done half the job and
men should be in the last column as given then B, the remaining half, the whole
below. Relationships of different quantities job would have been carried out in 25
given in the question with man are given hours. How much time would it take
below. for A and B separately to do the job, if
Hours and man: Less hours, more men A is a better worker?
(Inverse) Soln: Suppose A takes x hours and B takes y
Length and man: More length, more men hours to do the job.
(Direct)
Width and man: More width, more men 1 1 1
Then, x  y  12
(Direct)
412 Concept of Arithmetic

x y 1 Ther efore, if addit ional men were not


or,  ........ (1) employed, the work would have lasted (45
xy 12
– 40 =) 5 days behind the schedule time.
1 1 Ex. 29: If 9 men or 12 women or 18 boys can
And x  y = 25
2 2 do a piece of work in 20 days working
or, x + y = 50 ....... (2) 9 hours a day, then working 10 hours
Solve for x and y a day in how many days will 3 men, 2
12 (x + y) = xy women and 3 boys together do twice
or, 500 = xy the work?
Soln:  9 men = 18 boys
600
or, x +  50 18
x  1 man = = 2 boys
9
or x2 - 50x + 600 = 0
or (x - 30) (x - 20) = 0 Again, 12 women = 18 boys
 x = 30 or 20  18 
As A is a better worker, hence A and B  2 women =   2  = 3 boys
 12 
take 20 hours and 30 hours respectively.
Ex. 28: A builder decided to build a farmhouse  3 men + 2 women + 3 boys
in 40 days. He employed 100 men in = 6 + 3 + 3 = 12 boys
the beginning and 100 more after 35 Her e, t here are f our quant ities: Boys,
days and completed the construction Work, Hours and Days. And we have to
i n st ipul at ed t i m e. I f he had not find number of days, hence it should be
empl oyed the addi ti onal m en, how last in the column.
many days behind the schedule would
it have been finished?
Soln: Let only 100 men complete the work in x
days. The above arrangement is based on the
Work done by 100 men in 35 days + Work following relationships:

K KUNDAN
done by 200 men in (40 - 35 =) 5 days = 1 Less boys, more days (Inverse)
Case I: Work done by 100 men in 35 days. More work, more days (Direct)
 100 men, in x days, do the whole of More hours, less days (Inverse)
the work.
 12 :18 
 35  
 100 men, in 35 days, do the   of 1: 2 :: 20 : x
 x 
10 : 9
the work.
Case II: Work done by 200 men in 5 days. or, 12 × 1 × 10 : 18 × 2 × 9 :: 20 : x
 100 men, in x days, do the whole of (Compounding the ratios)
the work. or, 12 × 1 × 10 × x = 18 × 2 × 9 × 20
1 (  Product of extreme terms = Product of
 100 men, in 1 day, do the of the mean terms)
x
work. 18  2  9  20
 x = = 54 days.
1 12  1  10
 1 man, in 1 day, do the of the
100x Ex. 30: 9 men and 3 boys together can do a
work. piece of work in 7 days and 12 men
and 5 boys together can do the same
 200 
 200 men, in 1 day, do the   of piece of work in 5 days. How many boys
 100 x  together with 6 men will finish twice
the work. the work in 21 days?
 200  5  Soln:  In 7 days a piece of work is done by (9
 200 men, in 5 days, do the   of
 100 x  men + 3 boys)
the work.  In 1 day a piece of work is done by 7(9
men + 3 boys)
35 200  5
or,  1  In 5 days the same piece of work is
x 100 x
done by (12 men + 5 boys)
45  In 1 day the same piece of work is done
or, 1 by 5(12 men + 5 boys)
x
 7(9 men + 3 boys) = 5(12 men + 5 boys)
 x = 45 days.
Time and Work 413

or, 63 men + 21 boys = 60 men + 25 boys or, 60 + 15x – 60 + 10x – 20 = 180


or, 63 men – 60 men = 25 boys – 21 boys or, 25x = 200
or, 3 men = 4 boys
200
 9 men + 3 boys = (12 + 3 =) 15 boys  x = = 8 days
Here, there are three quantities: Work, 25
Days and Boys. W e hav e t o find t he Ex. 32: A, B and C can do a pi ece of work
number of boys, hence it should be in the together in 10 days. They worked at it
last column in the following arrangement: together for three days, and then A left.
B and C worked together for next two
days and then B left. C completed the
3
Abov e ar r angement is based on t he remaining of the work in 36 days.
5
following relationships: How m any days would each take
More work, more boys (Direct) separately to finish the work?
More days, less boys (Inverse)
3
 1: 2
:: 15 : x
Soln:  C completes 5 of the work in 36 days.
21: 7 
 C completes the whole of the work in
or, 1 × 21 : 2 × 7 :: 15 : x
(Compounding the ratios)  36  5 
   60 days
or, 1 × 21 × x = 2 × 7 × 15  3 
(  Product of extreme terms = Product of  (A + B + C) working for 3 days.
mean terms)
2  7  15 3
 x = = 10 boys  (A + B + C) complete of the work in
1  21 10
Since, 6 men are already employed. 3 days.
Now, from the question, B and C work for
4 
Hence, 6 men =   6   8 boys.

K KUNDAN
3    3 3  1
2 days and complete 1      of
 required number of boys = (10 – 8)   5 10   10
= 2 boys the work.
Ex. 31: A can do a piece of work in 15 days, B  (B + C) can do the whole of the work in
in 12 days and C in 18 days. A began 20 days.
the work. After 2 days, B and C joined
the work. But due to illness, 3 days 1
 (B + C)’s 1 day’s work = and
after the joining of B, A left the job. 20
Again due to injury to B, he left the job
2 days before the completion of the 1
(A + B + C)’s 1 day’s work =
work. C completed the remaining work. 10
In how many days the whole work is
completed? 1 1 1
 A’s 1 day’s work =  
Soln: Let the work be completed in x days. 10 20 20
 A worked for (2 + 3 =) 5 days  A can complete the whole of the work
B worked for (x – 2 – 2 =) x – 4 days in 20 days.
C worked for (x – 2) days 1
and 1 day’s work of A, B and C  (B + C)’s 1 day’s work = 20 and
1 1 1
=
, and 1
15 12 18 C’s 1 day’s work =
 Work done by A, B and C 60
5 x 4 x 2  B’s 1 day’s work
=  
15 12 18 1 1 3 1 2 1
Now, according to the question, =    
20 60 60 60 30
5 x 4 x 2  B can do the same work in 30 days.
  1 Hence, A, B and C can take 20 days, 30
15 12 18
days and 60 days respectively to finish
(5  12)  15 (x  4)  10 (x  2) the work.
or, 1
180
414 Concept of Arithmetic

Ex. 33: A started a work and left after working Now,


for 2 days. Then B was called and he [(A + B) + (B + C) + (A + C)]’s 1 day’s work
finished the work in 9 days. Had A left
1 1 1 456
the work after working for 3 days, B =   
would have finished the remaining work 30 24 20 120
in 6 days. In how many days can each
15 1
of t hem , worki ng alone, fi ni sh the = 
whole work? 120 8
Soln: Suppose A and B do the work in x and y 1
days respectively. Now, work done by A or, 2(A + B + C)’s 1 day’s work =
8
in 2 days + work done by B in 9 days = 1
2 9 or, (A + B + C)’s 1 day’s work
or, x  y  1 1 1
= 
8  2 16
3 6
Similarly, x  y  1  (A + B + C) can do the piece of work in
16 days.
To solve t he abov e equation put They all work together for 10 days.
1 1 10
= u and y = v. Hence (A + B + C)’s 10 days’ work =
x 16
Thus 2u + 9v = 1 ...... (1)
 10  6
and 3u + 6v = 1 ...... (2)  Remaining work = 1  
Performing (2) × 3 - (1) × 2 we have  16  16
5u = 1
 6 
1 Remaining work   is done by A as B
u =  16 
5
and C leave the work.
1 1

K KUNDAN
or, x = = 5 days.  (A + B + C)’s 1 day’s work = 16
u
1 1
Now putting the value of u = in equation (B + C)’s 1 day’s work =
5 24
(i), we have
 A’s 1 day’s work
1
2 × + 9v = 1  1 1  32 1
5 =    
 16 24   48  48
 2 3
or, 9v = 1     A can do the whole of the work in 48
 5 5 days.
3 1 1 6  48  6 
v=   
5 9 15  A can do of the work in 
16  16 
1 18 days.
and y = = 15 days. Ex. 35: A can do a piece work in 120 days and
v
B can do it in 150 days. They work
Ex. 34: A and B can do a piece of work in 30
together for 20 days. Then B leaves and
days while B and C can do the same
A continues the work alone, 12 days
work in 24 days and C and A in 20
after that C joins A and the work is
days. They all work for 10 days when
com plet ed in 48 days more. In how
B and C leave. How many days more
many days can C do i t if he wor ks
will A take to finish the work?
alone?
Soln:  (A + B) can do a piece of work in 30 Soln: Let C take x days to complete the job.
days. A takes 120 days and B takes 150 days to
1 complete the same job.
 (A + B)’s 1 day’s work = Now, A works for (20 + 12 + 48 =) 80
30
days, B works for 20 days and C works
Similarly, (B + C)’s and (A + C)’s 1 day’s
for 48 days.
1 1 Again,
work = and respectively. A’s 80 days’ work + B’s 20 days’ work +
24 20
C’s 48 days’ work = 1
Time and Work 415

 A, B and C together complete the entire


80 20 48
or,   1 80
120 150 x work in days.
3
400 x  80 x  28800 Now,
or, 1
(600 ) x 1
(A + B)’s 1 day’s work =
or, 480x + 28800 = 600x 40
or, 120x = 28800 3
(A + B + C)’s 1 day’s work =
 28800  80
 x =   = 240  C’s 1 day’s work = (A + B + C)’s 1 day’s
 120 
work – (A + B)’s 1 day’s work
Hence C takes 240 days to complete the
work. 3 1 1
=  
Ex. 36: A and B could do a piece of work in 40 80 40 80
days. After working for 10 days they  C completes the job in 80 days.
are assi sted by C and t he wor k i s Again, according to the question,
finished in 20 days more. If C does as
1 1
much work in 2 days as B does in 3 C’s 2 days’ work = 2 
days, in how many days could each of 80 40
them do the same work alone? 1
Soln: A and B can do a piece of work in 40  B does 40 of the work in 3 days.
days.
They work together for 10 days.  B does whole of the work in
(40 × 3 =) 120 days.
10 1 A’s 1 day’s work
 (A + B)’s 10 days’ work = 
40 4 = (A + B + C)’s 1 day’s work – B’s 1
day’s work – C’s 1 day’s work
  1 3
 Remaining work = 1  4   4 of the 3

1

1
=

K KUNDAN
80 80 120
work.
Now A, B and C together complete the 9  (3  2) 4 1
=  
3 240 240 60
th of the work in 20 days.  A completes the job in 60 days.
4

Practice Exercise

1. A and B can polish the floors of a building in 5. A and B can do a piece of work in 12 days
and B and C can do it in 15 days. If all the
1
25 days. A alone can do of this job in 15 three work together, it can be finished in 10
3 days. How long will it take for A and C to do
days. In how many days can B alone polish it together?
the floors of the building? 6. A can do a piece of work in 10 days and B in
2. P can complete a peice of work in 15 days, Q is 20 days. They work together but 2 days before
50% more efficient than P. How long will P the completion of the work, A leaves. In how
and Q together take to complete the work? many days was the work completed?
3. Suneeta can embroider a saree in 15 days. 7. A can do a piece of work in 24 days, B in 36
Her sister-in-law Abha can do the job in 10 days and C in 48 days. All the three began
days. They star t embr oidering t he saree the work together but after 4 days C left the
together, but two days later Abha gives up the wor k. B lef t t he wor k 3 days bef or e the
work and goes to her parents. In how many completion of the work. In how many days
days will Suneeta finish the remaining work the whole work will be finished.
of embroidering the saree? 8. A can complete a job in 9 days, B can do the
4. A and B can do a work in 45 and 40 days same job in 10 days and C in 15 days. B and
respectively. They began the work together, C together started the work but had to leave
but A left after some time and B finished the the work after 2 days. Find the number of
remaining work in 23 days. After how many days in which A alone will be able to complete
days did A leave? the remaining job.
416 Concept of Arithmetic

9. A can complete a work in 10 days, B in 12 letters. In how many days of 8 hours each
days and C in 15 days. All of them began the will 10 compositor s set 1000 pages of a
work together, but A had to leave the work manuscript, each page of 45 lines with 50
after 2 days of the start and B 3 days before letters in each line?
the completion of the work. How long did the 2 0 . 75 boys do a piece of work in 24 days. How
work last? many men will finish the double work in 20
10. A and B can complete a piece of work in 8 days when one day work of 2 men is equal to
days, B and C in 12 days and C and A in 15 one day’s work of 3 boys?
days. In how many days can C alone complete 21. If 38 men, working 6 hours a day, can do a
the work? piece of work in 12 days find the number of
11. A certain number of persons can complete a days in which 57 men, working 8 hours a
work in 100 days. If there were 10 persons day, can do a piece of work twice as great,
less, it would have taken 10 days more for supposing that 2 men of the first set do as
the work to be completed. How many persons much work in one hour as 3 men of t he
were in the beginning?
1
12. Hariram is thrice as good a workman as Kamal second set do in 1 hours.
and takes 10 days less than Kamal to complete 2
a piece of work. In how much time can Kamal 2 2 . A can do a piece of work in 30 days. B and C
complete that work? together can do the same work in 20 days. A
13. A is twice as good a workman as B and together
2
they can complete a piece of work in 14 days. and B worked together and completed of
In how many days can it be done by A alone? 3
14. 10 persons begin to work together on a job the work in 12 days. How many days will be
but after some days 4 persons leave. As a required by C to complete the remaining work?
r esult, t he job which could hav e been 3
completed in 40 days is completed in 50 days. 23. A does of a work in 12 days and finishes
10
How many days after the commencement of
the work did the 4 persons leave? the remaining work with the assistance of B
15. A fort is provisioned for 32 days for some in 7 days. In how many days can B alone do

K KUNDAN
soldiers. After 4 days, a reinforcement of 150 the whole work?
soldiers arrives and the food will last for 21 24. 15 persons working 8 hours a day can complete
days only. How many soldiers were there in a work in 21 days. How many days will 14
the fort in the beginning? 1
16. A, B and C completes a piece of work in 25, persons take to complete a work 1 times as
2
20 and 24 days respectively. All work together
for 2 days and then A and B leave the work. great, if they work 6 hours a day?
25. If 4 men or 6 boys complete a work in 20
3 days, then in how many days will 6 men and
C wor ks for next 8 days and t hen A
5 11 boys working together complete the same
alongwith D joins C and all finish the work wor k?
in next three days. In how many days D alone 26. If 4 men or 8 boys complete a work in 15
can complete the whole work? days, then in how many days will 3 men and
17. 64 persons can dig a trench 50 m long, 2 m 4 boys working together complete thrice of
wide and 2 m deep in 5 days working 12 the work?
hours daily. In how many days, working 8 27. 1 man or 2 women or 3 boys complete a certain
hours daily, will 80 persons dig anot her work in 44 days. In how many days will 1
trench 75 m long, 4 m wide and 3 m deep? man, 1 woman and 1 boy working together
18. A contractor undertakes to build a house in 8 complete it?
months. He plans to complete the work in 28. If 8 men and 12 boys can finish a piece of
time with 210 men working 20 days in a work in 12 days, in what time will 40 men
month at 6 hours a day. However, he manages and 45 boys finish another piece of work 3
to get only 140 men and is allowed 12 months times as great supposing 16 men can do as
to complete the work. If his men worked for much work in 8 hours as 12 boys do in 24
24 days a month, how many hours a day hours?
should they work to complete the work? 29. A man, a woman and a boy can do a piece of
19. 5 press compositors can set 600 pages of a work in 3, 4 and 12 days respectively. How
many boys must assist one man and one woman
1 to complete the work in one day?
manuscript in 16 days of 10 hours each.
2 30. A can do a piece of work in 10 days, while B
Each page has 60 lines and each line has 40 can do the same in 30 days and C in 60 days.
Time and Work 417

If A is assisted by B and C respectively on wall the other would take 10 hours. However,
alternate days, find the number of days in he knew from experience that when t hey
which the work will be finished. worked together, 10 fewer bricks got laid per
31. If 3 men and 5 women do a piece of work in hour. He put both the men on the job and
8 days and 2 men and 7 children do the same found that it takes exactly 5 hours to build
piece of work in 12 days, show that 10 women the wall. How much bricks did it contain?
can do as much work in a day as 21 children. 38. A contractor undertook to finish a piece of
32. 2 men and 7 children together complete a job work in 150 days. He employed 20 men, 30
in 16 days while 3 men and 8 childr en women and 75 children for it, but at the end
complete it in 12 days. In how many days of 60 days, finding that one fourth of the work
will 8 men and 8 children working together was done, he dismissed all the women and
complete twice the job? 50 of the children and employed more men.
33. A and B could do a piece of work in 30 days. The work was then finished 5 days before
After working for 10 days, they are assisted the stipulated time. Assuming that 3 men
by C and the work is finished in 10 days. If C could do as much work as 5 women and 2
does as much work in 2 days as B does in 3 women as much as 3 children, find the number
days, in how many days could A do the same of additional men employed by the contractor.
work alone? 39. A and B, working separately, can mow a field
34. It takes 8, 12 and 16 days for X, Y and Z in 12 and 15 hours respectively. They work
respectively to complete a work. How many alternately, each for 1 hour, with A beginning
days will it take to complete the work if X at 5 am. At what time will the mowing be
works on it for 2 days, then Y works on it for completed?
until 25% of the work is left for Z to do, and 40. A and B together can do a piece of work in 12
then Z completes the work? days and B and C together in 15 days. If A is
35. A and B can do a work in 12 days while B twice as good a workman as C, in what time
will B alone do it?
2
and C can do it in 6 days. After A had 41. A contractor undertakes to do a certain job in
3 45 days and employs 15 persons on it. At the
worked on it for 3 days and B for 4 days, C end of 36 days the work is only half-done.

K KUNDAN
finished the work in 7 days. In how many How many extr a persons should he now
days could each do the work separately? employ to finish the work in time?
36. A task can be completed by 3 men working 42. To do a certain piece of work, B takes four
times as long as A and C together and C takes
3
for a day, or by 8 women working for th of three times as long as A and B together. If all
4 of them working together complete the work
a day, or by 18 children working for half a in 12 days, how long would A alone take to
day. If 3 women and 3 children have already do the work?
been hired for a day, how many men should 43. A and B can complete a piece of work in 8
be hir ed and for how many days f or t he and 10 days respectively. However, they work
completion of the task? alternately, one day each, with A beginning
37. A contractor estimated that one of his two the work. In how much time will the work be
masons would take 9 hours to build a certain completed?

Answers and explanations


1. It is given that A and B can polish the floors Now, B’s 1 day’s work = (A + B)’s 1 day’s work
of the building in 25 days.
1 1 95 4
– A’s 1 day’s work =   
1 25 45 225 225
Therefore, (A + B)’s 1 day’s work =
25
225
1 Hence, B alone can polish the floor in
Now, A alone can do of the work in 15 4
3
days. 1
 A alone can do t he complet e wor k in days = 56 days.
4
(3 × 15) = 45 days.
1  100  50  3
2. Efficiency of Q =  P  P
or, A’s 1 day’s work =  100  2
45
418 Concept of Arithmetic

 Time taken by Q to complete the work


23
2  Work done by B in 23 days =
 15 = 10 days 40
=
3 Now, according to the question,
1 85x 23
P’s 1 day’s work =  1
15 45  40 40
1 85x  23  45
Q’s 1 day’s work = or, 1
10 45  40
or, 85x + 23 × 45 = 45 × 40
1 1 23 1
 (P + Q)’s 1 day’s work =    or, 85x = 45 (40 – 23) = 45 × 17
15 10 30 6
45  17
 P and Q together will complete the work in x = = 9
6 days. 85
3. Suneeta can embroider a saree in 15 days.  required answer = 9 days.
Abha can embroider a saree in 10 days
1
1 5. Work done by A and B in 1 day =
12
 Suneeta’s 1 day’s work =
15
1
1 Work done by B and C in 1 day =
15
Abha’s 1 day’s work =
10
1
 (Suneeta + Abha)’s 1 day’s work Work done by A, B and C in 1 day =
10
 1 1  23 5 1
=      1
 15 10   30  30 6  Work done by 2A, 2B and 2C in 1 day =
or, (Suneeta + Abha)’s 2 days’ work 5
Work done by A + 2B + C in 1 day
 1 1

K KUNDAN
= 2    1 1 3
 6 3 =  
12 15 20
1 2  Work done by A and C in 1 day
 Remaining work = 1  
3 3
1 3 1
Now, = 
5 20 20
Suneeta completes the work in 15 days
 A and C together can do it in 20 days.
2 6. Suppose the work is finished in x days.
 Suneet a complet es of t he wor k in
3
x 2 x
Then,  1
 2 10 20
15   days = 10 days.
 3 [  A works for (x – 2) days.]
Hence, Suneet a will finish t he r emaining 2x  4  x
work of embroidering in 10 days. or, 1
20
4. A alone can do the work in 45 days.
B alone can do the work in 40 days. or, 3x – 4 = 20
or, 3x = 24
1 or, x = 8 days
 A’s 1 day’s work = and
45 Hence, the work was completed in 8 days.
7.  A, B and C do the piece of work in 24
1 days, 36 days and 48 days respectively.
B’s 1 day’s work =
40 1
 A’s, B’s and C’s 1 day’s work will be ,
1 1  45  40  24
 (A + B) ’s 1 day’s work =  = 
40 45  45  40  1 1
and respectively.
Let after x days A left the work. Therefore, A 36 48
and B together worked for x days.  Work done by all the three together in 4
 85x  days
 (A + B) ’s x days’ work =  
 45  40   1 1 1  13 13
Again, B works alone for 23 days. = 4     4 
 24 36 48  144 36
Time and Work 419

Again, in last 3 days, work done by B 9. Let the work is completed in x days.
1 1 According to the question,
= 3  A worked for 2 days.
36 12
B worked together for (x – 3) days.
13 1 20 5
 remaining work = 1     C worked for all x days.
36 12 36 9  A completes the work in 10 days.
Now, according t o t he quest ion, t his
remaining work is done by A and B. 1
 The work done by A in 1 day =
 1 1  5 10
(A + B)’s 1 day’s work =   
 24 36  72
2 1
5  The work done by A in 2 days = 
 of the work is done by (A + B) in 1 day. 10 5
72
5 1
 of t he work is done by (A + B) in Similarly, the work done by B in 1 day =
9 12
 The work done by B in (x – 3) days =
 72 5 
    8 days.
 5 9   x 3 1
  and the work done by C in 1 day =
Hence, total time taken to finish the work  12  15
= (4 + 3 + 8) = 15 days.
Alternative Method: x
 The work done by C in x days =
Let the work be finish in x days. 15
 Working time for A = (x – 3) days But the total work done is 1
Working time for B = x days
Working time for C = 4 days 1 x 3 x
   1
 Work done by them 5 12 15
(x  3) x x 12  5 (x  3)  4x
  1 1
24 36 48 or,
60
or, 6(x – 3) + 4x + 12 = 144

K KUNDAN
or, 10x = 150 or, 12 + 5x – 15 + 4x = 60
 x = 15 or, 9x = 63
Hence, total time taken to finish the work is 63
15 days.  x = = 7 days.
9
8.  A can complete the work in 9 days.
1 1
 A’s 1 day’s work = 10. (A + B)’s 1 day’s work =
9 8
 B can complete the work in 10 days.
1
1 (B + C)’s 1 day’s work = and
 B’s 1 day’s work = 12
10
 C can complete the work in 15 days. 1
(C + A)’s 1 day’s work =
1 15
 C’s 1 day’s work =
15 On adding all the three, we have
B and C worked for 2 days 1 1 1
 (B + C)’s 1 day’s work 2 (A + B + C)’s 1 day’s work =  
8 12 15
 1 1   3 2 1
=     15  10  8 33 11
 10 15   30  6 =  
120 120 40
1 1
 (B + C)’s 2 day’s work = 2   11 11
6 3  (A + B + C)’s 1 day’s work = 
40  2 80
 1 2  C’s 1 day’s work
Remaining work = 1    = (A + B + C)’s 1 day’s work –
 3 3
(A+B)’s 1 day’s work
1
Now, A does of the work in 1 day. 11 1 11  10 1
9 =   
80 8 80 80
2 2  C alone can finish the work in 80 days.
 A does of the work in  9 = 6 days.
3 3
420 Concept of Arithmetic

11. Let there be x persons in the beginning. 15. There is provision for food for 32 days in the
Now, x persons complete the work in 100 fort for some soldiers.
days and (x – 10) persons complete the work Let the number of soldiers be x initially.
in 110 days. After 4 days 150 more soldiers arrived.
Here there are two quantities person and day.  Now food is available for x soldiers for
Both are inversely proportional to each other. (32 – 4 =) 28 days.
ie less person, more days. But , due to ar rival of 150 soldier s, f ood
 x : (x – 10) :: 110 : 100 finished in 21 days only.
or, (x × 100) = (x – 10) × 110 Now the above question can be summarised
as “A fort is provisioned for 32 days for x
x 110
or,  soldiers and for 21 days for (x + 150) soldiers.
x  10 100 Find the value of x.”
or, 110x – 1100 = 100x Here there are two quantities: Soldiers and
or, 10x = 1100 Days.
Here inverse relation exists between number
1100
 x = = 110 of soldier s and number of days as when
10 number of soldiers increases, number of days
Hence, there were 110 persons originally. decreases.
12. Harir am is thrice as good a workman as
Kamal.
 Ratio of time taken by Hariram and Kamal
= 1 : 3
If difference of time is 2 days, Kamal takes 3 Hence,
days. 21 : 28 :: x : x + 150
If difference of time is 10 days, Kamal takes or, 21(x + 150) = 28x

3  x 21
 10  = 15 days. or, 
2  x  150 28
13. Let the time taken by B alone to complete the or, 28x = 21(x + 150)

K KUNDAN
work be 2x days. Since A is twice as good a or, 7x = 3150
workman as B, then time taken by A alone to 3150
complete the work = x days. or, x = = 450
7
Now,
Thus, t here were 450 soldiers in t he fort
1 initially.
Work done by A in 1 day =
x 16. Time taken by A to complete the work = 25 days.
Time taken by B to complete the work = 20 days.
1 Time taken by C to complete the work = 24 days.
Work done by B in 1 day =
2x A works for (3 + 2 =) 5 days.
Work done by (A + B) in 1 day 1 1
1 1   2 1  3  A’s 5 days’ work = 5 
=     25 5
 x 2x   2x  2x B works for 2 days.
3 1 2 1
Clearly,  
2x 14  B’s 2 days’ work =
20 10
or, 2x = 42
 3  68
42 C works for  2  8  3  days = days
 x = = 21  5  5
2
Hence, A alone can complete the work in 21 68 68 17
 C’s days’ work = 
days. 5 5  24 30
14. Suppose 4 persons leave after x days. It means Let D complete the work in x days.
that 10 persons work for x days and (10 – 4 =)
6 persons work for (50 – x) days. Then 1
 D’s 1 day’s work =
40 days’ work done by 10 persons = x days’ x
work done by 10 persons + (50 – x) days’ work D works for 3 days.
done by 6 persons
or, 10 × 40 = 10x + 6(50 – x) 3
 D’s 3 days’ work =
or, 4x = 100 x
or, x = 25 days.
Time and Work 421

Now, according to the question, Hence,


1 1 17 3 12 : 8
   1 
5 10 30 x 140 : 210  :: 6 : x
6  3  17 3 24 : 20 
or,  1
30 x or, 12 × 140 × 24 : 8 × 210 × 20 :: 6 : x
26 3 (Compounding the ratios)
or,  1 or, 12 × 140 × 24 × x = 8 × 210 × 20 × 6
30 x
(  Product of extreme terms = Product of mean
3 26 4 2 terms)
or,  1  
x 30 30 15 8  210  20  6
or, x = = 5
15  3 45 1 12  140  24
or, x =   22
2 2 2  the required answer = 5 hours.
19. Here are t he f ollowing quantit ies :
1
Hence D alone can complete the work in 22 Compositors, Pages, Hours, Lines, Letters and
2 Days. We have to calculate number of days.
days. Following relationships exists:
17. Here there are following quantities : Persons, More compositors, less days (Inverse)
Length, Width, Depth, Days and Hours and More pages, more days (Direct)
we have to find the number of days. Less hours, more days (Inverse)
Let the number of days be x. Less lines, less days (Direct)
Relat ionships bet ween days and ot her More letters, more days (Direct)
quantities exist as given below: Clearly,
More persons, less days (Inverse)
More length, more days (Direct)
More width, more days (Direct)
More depth, more days (Direct)
Hence,

K KUNDAN
Less hours, more days (Inverse)

10 : 5
600 : 1000 
1 
8 : 10  :: 16 : x
Hence, 2
80 : 64 60 : 45 
8 : 12 40 : 50 

50 : 75  :: 5 : x
or, 10 × 600 × 8 × 60 × 40 :
2 : 4
 21
2 : 3
5 × 1000 × × 45 × 50 :: 16 : x
or, 80 × 8 × 50 × 2 × 2 : 64 × 12 × 75 × 4 × 3 :: 5 : x 2
(Compounding the ratios) (Compounding the ratios)
or, 80 × 8 × 50 × 2 × 2 × x = 64 × 12 × 75 × 4 × 3 × 5
(  Product of extreme terms = Product of mean or, 10 × 600 × 8 × 60 × 40 × x
terms)
21
64  12  75  4  3  5 = 5 × 1000 × × 45 × 50 × 16
2
or, x = = 27 days
80  8  50  2  2 (  Product of extreme terms = Product of mean
18. Here there are four quantities : Men, Days, terms)
Months and Hours. We have to calculate 5  1000  21  45  50  16
number of hours. x=
10  600  8  60  40  2
Following relationships exist:
Less men, more hours (Inverse) 37800 37800
More days, less hours (Inverse) = = = 16.40
48  48 2304
More months, less hours (Inverse)  required number of days = 16.40 days.
Clearly, 2 0 . 3 boys = 2 men
2
 75 boys =  75 = 50 men
3
 1 work is finished in 24 days by 50 men
422 Concept of Arithmetic

 1 work is finished in 1 day by 50 × 24 men


1 : 2
50  24 
8 : 6 :: 12 : x
 1 work is finished in 20 days by
20 57 : 85 .5 
men
or, 1 × 8 × 57 : 2 × 6 × 85.5 :: 12 : x
 2 wor ks are f inished in 20 days by
(Compounding the ratios)
50  24  2  1 × 8 × 57 × x = 2 × 6 × 85.5 × 12
= 120 men.
20 (  Product of extreme terms = Product of mean
Alternative Method: terms)
From the above we know that 75 boys = 50 2  6  85 .5  12
men or, x = = 27 days.
1  8  57
Here, there are three quantities : Days, Work
and Men. We have to calculate the number of 2 2 . A can do a piece of work in 30 days.
men. 1
Here following relationships exist:  A’s 1 day’s work =
30
Less days, more men (Inverse)
More work, more men (Direct) (B + C) can do the same work in 20 days.
Clearly, 1
 (B + C)’s 1 day’s work =
20

2
(A + B) can do of the work in 12 days.
Hence, 3
 (A + B) can do the whole of the work in
20 : 24
 :: 50 : x 12  3
1 : 2 = 18 days.
2
or, 20 × 1 : 24 × 2 :: 50 : x
(Compounding the ratios) 1

K KUNDAN
or, 20 × 1 × x = 24 × 2 × 50  (A + B)’s 1 day’s work =
18
(  Product of extreme terms = Product of mean B’s 1 day’s work
terms) = (A + B)’s 1 day’s work – A’s 1 day’s work
24  2  50  1 1  2 1
x= = 120 =    
20  18 30  90 45
 required answer = 120 men.  B can complete the work in 45 days.
1 C’s 1 day’s work
21. 1 hour work of 2 men of first group = 1 hours = (B + C)’s 1 day’s work – B’s 1 day’s work
2
work of 3 men of second group 1 1 1
=  
1 hour work of 2 men of first group = 1 hour 20 45 36
work of 4.5 men of second group  C can complete the work in 36 days.
( if 3 men do a work in 1.5 hours, it will be
completed in 1 hour by 3 × 1.5 = 4.5 men)  2 1
 2 men of first group = 4.5 men of second Remaining work = 1   
 3 3
group
 C can do the whole of the work in 36 days.
or, 4 men of first group = 9 men of the second
group 1  1 
 C can do of the work in  36    12
9 3  3 
or, 38 men of the first group = 38  = 85.5
4 days.
men of the second group 23. Let B alone can do the work in x days.
 the problem has changed as follows
1
 1 day’s work of B =
x

3
Here, following relationships exist: A finishes of the work in 12 days.
10
(i) more work, more days (Direct)
(ii) more hours per day, less days (Inverse) 1 3 1
 1 day’s work of A =  
(iii) less men, more days (Inverse) 12 10 40
Time and Work 423

3 7 6 6
Remaining work = 1   or, 6 men = = 9 boys
10 10 4
 6 men + 11 boys = 9 boys + 11 boys = 20 boys
1 1
 1 day’s work of A and B =   6 boys complete a work in 20 days.
x 40  1 boy complete a work in (20 × 6) days.
As A and B finished the remaining work in 7  20 boys complet e the same wor k in
days, the work done by them in 7 days
 20  6 
   6 days.
1 1   20 
= 7  
 x 40  26.  4 men = 8 boys
1 1  7 8 
or, 7    1 man =    2 boys
 x 40  10 4 
1 1 1 3 men + 4 boys = (3 × 2) boys + 4 boys = 10 boys
or,   Here, we have three quantities: Boys, Work
x 40 10
and Days. We have to calculate number of
1 1 1 4 1 3 days. Here following relationships exist:
or,     More boys, less days (Inverse)
x 10 40 40 40
More work, more days (Direct)
1 Clearly,
or, x = 13
3
1
 B alone can do the work in 13 days.
3 Hence,
24. Here t here ar e the following quantities :
Persons, Hours, Work and Days. We have to 10 : 8
:: 15 : x
calculat e number of days. Following 1 : 3

K KUNDAN
relationships exist: or, 10 × 1 : 8 × 3 :: 15 : x
Less persons, more days (Inverse) (Compounding the ratios)
Less hours, more days (Inverse) or, 10 × 1 × x : 8 × 3 × 15
More work, more days (Direct) (  Product of extreme terms = Product of mean
Clearly,
terms)
8  3  15
 x = = 36
10
Hence,  required answer = 36 days.
27. 1 man = 3 boys

14 : 15  2 women = 3 boys

6 : 8  :: 21 : x 3
3  1 woman = boys
1 : 2
2 
 1 man + 1 woman + 1 boy
3
or, 14 × 6 × 1 : 15 × 8 × :: 21 : x 3
2 = 3 boys + boys + 1 boy
2
(Compounding the ratios)
11
3 = boys
or, 14 × 6 × 1 × x = 15 × 8 × × 21 2
2
(  Product of extreme terms = Product of mean
 3 boys complete the work in 44 days.
 1 boy complete the work in (44 × 3) days
terms)
11  44  3  2 
15  8  3  21  boys complete the work in  
 x = = 45 2  11 
14  6  2
24 days.
Hence required answer = 45 days.
28.  The work is completed by 16 men in 8
25. 4 men = 6 boys
hours
6 The work is completed in 1 hour by (16 × 8)
or, 1 man = boys men.
4
424 Concept of Arithmetic

In the same manner,


60
 The work is completed by 12 boys in 24 The whole work is done by A and C = days
hours. 7
 The work is completed in 1 hour by
 2 7 1
(12 × 24) boys.  In the first 2 days    of the work
 (16 × 8) men = (12 × 24) boys  15 60  4
is done.
16  8 4
or, 1 boy = man = man
12  24 9  1  3
 In 6 days   6  of the work is done.
 2 4  4
4 40
Now, 8 men + 12 boys = 8  12   men
9 3  3 1
 Remaining work = 1    of the work.
 4 4
4
and 40 men + 45 boys = 40  45   60 men
9 1 2 7
In 1 day A and B do the work =  
4 15 60
40
 men do 1 work in 12 days Now, we have (A + C)’s turn to work.
3
7
 40  A and C together do of the work in 1 day.
 1 man do 1 work in 12   days and 60
 3  Total time = 6 + 1 + 1 = 8 days.
31. 3 men + 5 women can do the work in 8 days
 12  40 
 60 men do 1 work in   days.  24 men + 40 women can do the work in 1
 60  3  day
Again 2 men + 7 children can do the work in
 12  40  3 
 60 men do 3 work in    8 days 12 days
 3  60  24 men + 84 children can do the work in 1
 The required time is 8 days. day
29. Let x boys assist one man and one woman to Hence, 24 men + 40 women = 24 men + 84

K KUNDAN
complete the work in 1 day. children
or, 40 women = 84 children
1
Part of the work done by the man in 1 day = or, 10 women = 21 children.
3 32.  In 16 days a piece of work is done by
Part of the work done by the woman in 1 day (2 men + 7 children)
1  In 1 day the same piece of work is done by
= 16(2 men + 7 children)
4
 In 12 days a piece of work is done by
x (3 men + 8 children)
Part of the work done by x boys in 1 day =  In 1 day the same piece of work is done by
12
12(3 men + 8 children)
1 1 x  16(2 men + 7 children) = 12(3 men + 8
and,   1 children)
3 4 12
or, 32 men + 112 children = 36 men + 96
x 1 1  7  5 childr en
or,  1      1  
12  3 4   12  12 or, 4 men = 16 children
 1 man = 4 children
x= 5
Now,
 number of boys = 5
2 men + 7 children = 8 children + 7 children
30. According to the question, on the first day A
= 15 children
and B together do the work whereas on the
8 men + 8 children = 32 children + 8 children
second day A and C together do the work.
= 40 children
 1 1  4 2  15 children complete a piece of work in
1 day’s work of A and B =    
 10 30  30 15 16 days.
 1 child completes the same piece of work
15 in (16 × 15) days.
The whole work is done by A and B = days  40 children complete the same piece of work
2
 16  15 
 1 1  7 in   days.
1 day’s work of A and C =     40 
 10 60  60
Time and Work 425

 40 children complete the twice of the work  A’s 1 day’s work


= (A + B)’s 1 day’s work – B’s 1 day’s work
 16  15  2 
in   days = 12 days. 1 1 32 1
 40    
=
Alternative Method: 30 45 90 90
Here, we have three quantities : Children,  A alone can finish the work in 90 days.
Work and Days. We have to calculate the
1
number of days. Here, following relationships 34. X’s 1 day’s work =
exist: 8
More children, less days (Inverse) 1
More work, more days (Direct) Y’s 1 day’s work =
12
Clearly,
1
Z’s 1 day’s work =
16
 X works for 2 days.
Hence, 1  1
 X’s 2 days’ work =   2  
40 : 15
:: 16 : x 8  4
1 : 2  According to the question,
or, 40 × 1 : 15 × 2 :: 16 : x 25% of the work will be completed by Z.
(Compounding the ratios)
or, 40 × 1 × x = 15 × 2 × 16 25 1
Therefore, Z does  of the work.
(  Product of extreme terms = Product of mean 100 4
terms) 1
15  2  16  Z does of the work in 1 day.
16
 x = = 12
40
1  1
 required answer = 12 days.  Z does of the work in 16   = 4 days.
4  4

K KUNDAN
1 Let Y work for x days.
33. (A + B)’s 1 day’s work =
30
1
1 1  Y’s 1 day’s work =
(A + B)’s 10 days’ work = 10   12
30 3
1 2 x
Remaining work = 1    Y’s x days’ work =
3 3 12
Now, Again, since the whole work is 1.

2 1 x 1
   1
 (A + B + C)’s 10 days’ work = 4 12 4
3
x 1 1  1 1
2 1 or,  1      1   
 (A + B + C)’s 1 day’s work =  12 4 4  2 2
3  10 15
 C’s 1 day’s work= (A + B + C)’s 1 day’s 12  1
x = = 6
work – (A + B)’s 1 day’s 2
work Hence Y works for 6 days.
1 1 2 1 1  Total number of days in which work is
=    completed 2 + 6 + 4 = 12 days.
15 30 30 30
35. Let A, B and C’s daily work be x, y and z.
 C alone can finish the work in 30 days. Since A and B completes the work in 12 days,
Now, it is given that C does as much work in
2 days as B does in 3 days. 1
A and B’s one day’s work = x + y =
 The work which C does in 30 days, will be 12
 3  1
done by B in  30    45 days. x= y
 2  12
 B alone will finish the work in 45 days. Similarly, both B and C can complete the work
1 2 20
 B’s 1 day’s work = in 6 days = days
45 3 3
426 Concept of Arithmetic

Time taken by 8 women to complete the work


3
 B and C’s one day’s work = y + z = 3
20
= day
4
3
z = y Time taken by 18 children to complete the
20 wor k
Also, 3x + 4y + 7z = 1
Substituting the above value of x and y, we 1
= day
get 2
Now,
 1   3 
3  y   4y  7 y 1 Time taken by 1 man to complete the work
 12   20  = 3 days
The time taken by 1 woman to complete the
3 ( 1  12y) 7 ( 3  20y)
or,  4y  1 wor k
12 20
3
( 1  12y) 21  140y =  8 = 6 days
or,  4y  1 4
4 20
1
5  60y  80y  21  140y Work completed by 1 woman in one day =
or, 1 6
20 Work completed by 3 women in one day
or, 26 – 120y = 20
or, 120y = 6 3 1
= =
6 2
1
y = Time taken by 1 child to complete the work
20
1
Putting this value of y in the equation given =  18 = 9 days
above 2

1 1

K KUNDAN
or, x = y Work completed by 1 child in 1 day =
12 9
Work completed by 3 children in 1 day
1 1
or, x = 
12 20 3 1
= 
9 3
53
or, x = So, work completed by 3 women and 3 children
60
1 1 5
2 1 in one day =  
or, x =  2 3 6
60 30
5 1
Similarly, Work to be completed = 1  
6 6
3
z = y
20 1
 The time taken by 1 men to complete
6
3 1
or, z = 
20 20 1 1
work = 3  day
6 2
2 1
or, z =  So, 1 man should be hired for one day.
20 10 37. Let the number of bricks in the wall be x.
Part of the wall built by both in one hour will
1 1
A, B and C’s one day’s work is , and be
30 20
1 1 19
1 =  
respectively. 9 10 90
10  time taken to complete the wall when both
 A, B and C can complete the work in 30,
20 and 10 days respectively. 90
are working will be = hour
36. From the question, 19
Time taken by 3 men to complete the work
= 1 day
Time and Work 427

According to the question, Hence,


90 1 3
Both will take hours to fix x numbers of :
19 4 4  :: 68 : x
bricks. 

19 85 : 60 
So, number of bricks fixed in one hour = x
90
1 3
But they will fix x number of bricks in 5 hours. or, × 85 : × 60 :: 68 : x
4 4
x (Compounding the ratios)
Bricks fixed in 1 hour will be
5
1 3
Since when both are working together, they or, × 85 × x = × 60 × 68
both fix 10 bricks less 4 4
(  Product of extreme terms = Product of mean
19x x
  10 terms)
90 5
3  60  68  4
19x  18x  x = = 144
 10 85  4
or,
90 Again, from the question,
or, x = 900 Since all the women and 50 children were
The number of bricks in the wall is 900. dismissed, hence the number of woman = 0
38.  5 women = 3 men and the number of children = (75 – 50 =) 25
Now,
3
 1 woman = man  2
5
20 men + 25 children =  20  25   = 30 men
 3 children = 2 women  5
 Number of additional men = (144 – 30)
2
 1 child = woman = 114 men.

K KUNDAN
3 39. According to the question,
2 3 2 1
=   man Work done by A in first hour =
3 5 5 12
 20 men + 30 women + 75 children
1
 3 2 Work done by B in second hour =
15
=  20  30   75   men
 5 5  Work done in two hours
= 20 + 18 + 30 = 68 men 1 1 54 3
According to the question,  =  
12 15 60 20
1 or, Work done by (A + B) in 12 hours
Time required for of the work = 60 days.
4
3 12 9
 = 
 1 3 20 2 10
 Remaining work = 1    of the work. Now, in 13th hour, it is the turn of A
 4 4
Now, number of days required for the rest of or, Work done in 13 hours
the work = (150 – 60) – 5 = 85 days. 9 1 54  5 59
Here we have three quantities: Work, Day,  = 
10 12 60 60
and Men. We have to calculate number of men.
or, Remaining work after 13 hours
Here following relationships exist:
More work, more men (Direct) 59 1
More days, less men (Inverse) = 1 
60 60
Clearly,
1
or, Time taken by B to do work
60

1 1 15 1
=    hour = 15 minutes
1 60 60 4
15
428 Concept of Arithmetic

 Total time taken to complete the whole work 42. Let A and B together complete the work in x
= 13 hours + 15 minutes days.
Hence, mowing will be completed at
1
5 am + 13:15 hours = 18:15 hours  (A + B)’s 1 day’s work =
= 6:15 pm x
40. Let A alone complete the work in x days. According to the question,
 C alone will take 2x days to complete it.
1
Let B alone complete the work in y days. C’s 1 day’s work =
According to the question, 3x
Again, let (A + C) complete the work in y days.
1 1 1
  ......(i) and 1
x y 12
 (A + C)’s 1 day’s work = y
1 1 1
  ......(ii) 1
y 2x 15
B’s 1 day’s work = 4y
Subtracting equation (ii) from (i),
1 1 1 1 1 1
   Now, A + B = and B = 4y
x 2x 12 15 x

2 1 5  4 1 1
or,   A’s 1 day’s work = x  4y ......(i)
2x 60

1 1 1 1
or, 
2x 60 A + C = y and C =
3x
60 1 1
or, x = = 30
2  A’s 1 day’s work = y  3x ......(ii)
From equation (i),

K KUNDAN
From equations (i) and (ii),
1 1 1
  1 1 1 1
30 y 12   
x 4y y 3x
1 1 1 52 3 1
or, y  12  30  60  60  20 1

1

1

1
or, x 3x 4y y
 B alone will complete the work in 20 days.
41. Here, there are three quantities: Days, Work 4 5
and Men. or, 
3x 4y
Following relationship exists:
Less days, more men (Inverse) or, 16y = 15x ......(iii)
Less work, less man (Direct) 1 1 16
Now, (A + C)’s 1 day’s work =   ;
y 15 x 15 x
16

1
(A + B)’s 1 day’s work = and
x
Where x = number of men 1 1
9 : 36  (B + C)’s 1 day’s work = 4y  3x

 :: 15 : x
1 1 1 1 4 1 9
: =    
2 2  15 x 3x 15 x 3x 15 x
4
1 1 16
or, 9 × × x = 36 × × 15 Now, adding to above three,
2 2
2(A + B + C)’s 1 day’s work
36  15
or, x = = 60 16 1 9 40 8
9 =    
15 x x 15 x 15 x 3x
 Number of additional men = (60 – 15 =) 45.
Time and Work 429

 (A + B + C)’s 1 day’s work


1 1 54
43. Work of A and B for first 2 days  
8 1 4 8 10 40
=  
3x 2 3x
9
Again, according to the question, = of the whole of the work.
40
1
(A + B + C)’s 1 day’s work = 8 9 9
12 Work of A and B for 8 days =   of
2 40 10
4 1 the whole of the work.
 
3x 12 9 1
Remaining work after 8 days = 1   of
or, 3x = 4 × 12 10 10
4  12 the whole of the work.
 x = = 16 Since both A and B work alternately one day
3
each, with A beginning the work. It is the
From equation (iii), turn of A for ninth day.
16y = 15x
or, 16y = 15 × 16 1
 of the whole of the work is completed
15  16 8
y = = 15 by A in 1 day.
16
1
1 1  Remaining of t he wor k will be
 A’s 1 day’s work = x  4y 10

 8  4
1 1 1 1 15  4 11 completed by A in   day.
=       10  5
6 4  15 16 60 240 240
Hence the required time to complete the work
240 4  4
 A alone will complete the work in 
= 8   8

K KUNDAN
11 days.
 5  5
days.
Chapter-26

Speed, Time and Distance


Speed The Relative Speed
If the location of an object changes with time then
it is said to be in motion. A bus running on a road, (i) Objects are moving in opposite directions
an ant crawling on a floor, a monkey climbing up The relative speed of one object with respect to
a greased pole etc — all are the examples of objects t he ot her , will have magnit ude gr eater than
in motion because the locations of these objects individual speed of each object. This is why, for
keep on changing wit h respect to t heir example, a train A moving with speed 10 km/hr
surroundings. will cross another t rain B moving in opposite
Let an object move from a point A to the point B directions with speed 25 km/hr, with a relative
through any path, then the actual length of the speed of (10 + 25 =) 35 km/hr which is greater
path followed by the object is called the distance than the individual speed of either train.
travelled by the object. The rate at which any
moving body covers a particular distance is called (ii) Objects are moving in same direction
its speed. The relative speed of one object with respect to the
Distance travelled other, will have magnitude either less than or greater
 Speed = than individual speed of each object. This is why,
Time taken
for example, a train A moving with the speed of 20
1 km/hr will cross the another train B moving in
If the distance is constant, Speed  same direction with the speed of 15 km/hr, with a
Time
relative speed of (20 – 15 =) 5 km/hr which is less
Distance than the individual speed of either train.
 Time = Speed Take another example, a train A moving with
the speed of 60 km/hr will cross the another train
If the time is constant, Distance  Speed B moving in same direction with the speed of
Distance = Time × Speed. 20 km/hr with a relative speed of (60 - 20 =)
If the speed is constant, Distance  Time 40 km/hr which is less than the tr ain A and
We can say that for constant distance travelled, greater than the train B.
speed is inversely proportional to the time taken.
This can be explained by a simple example. To Average Speed
cover a distance of 100 kms, if a person goes at
the speed of 25 kmph, he will require 4 hours to Total Distance Covered
complete the journey and travelling at a speed of Average Speed =
Total Time Taken
50 kmph, 2 hours will be required.
For example, A person divides his total route
(a) Uniform Speed of journey into three equal parts and decides to
travel the three parts with speed of 40, 30 and
If the object covers equal distance in equal time
15 km/hr respectively. Find his average speed
intervals, howsoever small the interval may be then
during the whole journey.
its speed is called the uniform speed.
Let the three equal parts of journey be x km.
(b) Variable Speed Time taken to travel first part of the journey

If the object travels different distances in equal  x 


=   hour
intervals of time, then its speed is called a variable  40 
speed. In this case t he speed changes fr om Time taken to travel second part of the journey
instance to instance.
 x 
=   hour
 30 
450 Concept of Arithmetic

Time taken to travel third part of the journey An Important Result


 x  If two persons (or trains) A and B start at the same
=   hour
 15  time in opposite directions from two points, and
arrive at the point in a and b hours respectively
 x x x  after having met, then
 Total time taken =     hours
 40 30 15  A’s rate : B’s rate = b : a .
Total distance travelled = x + x + x = 3x km
Proof: Suppose A starts from P and B starts from
Total Distance Travelled Q and they meet at R.
Average Speed =
Total Time Taken

x Let A’s rate be x km per hour, and B’s rate


= km/hr be y km per hour.
 x x x 
    QR = ax km, PR = by km.
 40 30 15 
Now time t aken by A in t r av elling the
= 24 km/hr
by
Units of Measurement distance by km is hours.
x
Time is measured in seconds (s), minutes (min) or And t ime t aken by B in tr avelling the

K
hours (hr) ax
Distance is usually measured in metres (m), distance ax km is hours.
y
kilometres (km), miles, yards or feet.
Speed is usually measured in metres per second Since both start at the same time and meet,
(mps), kilomet res per hour (kmph or those two times must be equal. Hence
km/hr) or miles per hour (mph).
ax by x2 b x b
    
Conversion of units y x y2 a y a
1 hour = 60 minutes = (60 × 60) seconds. For example, A man sets out to cycle f rom
1 kilometre = 1000 metres Bhiwani to Rohtak, and at the same time another
8 kilometres = 5 miles man starts from Rohtak to cycle to Bhiwani. After
1 yard = 3 feet passing each other they complete their journeys

KUNDAN
1 kilometre 1 4
1 kilometre per hour = in 3 and 4 hours respectively. At what rate
1 hour 3 5
1000 metres 5 does the second man cycles if the first cycles 8
= (60  60) seconds  18 metre per second km per hour?
We proceed as follows:
 18 kilometres per hour = 5 metres per second
[To convert kilometres per hour into metres 4
4
per second we multiply the given speed in First man' s rate 5  6

Second man' s rate 1 5
5 3
kilometres per hour with . And to convert 3
18
metres per second into kilometres per hour 8 6
we multiply the given speed in metres per or 
Second man' s rate 5
18
second with .] 5 2
5  second man’s rate = 8 = 6 km per hour.
6 3
5
1 kilometre per hour = mile per hour.
8

22
1 mile per hour = feet per second.
15
Speed, Time and Distance 451

Solved Examples
Ex. 1: A man walks 22.5 km in 5 hours. How Now, 1 hour = 60 minutes
much he will walk in 4 hours? = (60 × 60) seconds
Soln: We have, distance covered = 22.5 km, = 3600 seconds
time taken = 5 hours.  Distance covered by the goods train in
one hour = (Speed × Time)
Distance
 Speed = = (4 × 3600) metres
Time
 3600 
 22 .5  = 4   km
 1000 
or, Speed =   km/hr = 4.5 km/hr
 5 
Now, Distance = Speed × Time 72
=km = 14.4 km
or, Distance covered in 4 hours 5
= (4.5 × 4) km = 18 km. Thus, the distance covered by the goods
Hence, the man will walk 18 km in 4 train in 1 hour = 14.4 km.
hours. Hence, the speed of the t rain is 14.4
Ex. 2: A car travels at the speed of 72 km/hr. km/hr.

K
How many metres will it travel in one Ex. 4: A man travels some distance at a speed
second? of 12 km/hr and returns at 9 km/hr. If
Soln: We have, speed of the car = 72 km/hr the total time taken by him is 2 hours
20 minutes, find the distance.
1
Time = 1 sec = minute Soln: Let the distance be x km. Then,
60
x
time taken at 12 km/hr = hour
 60 seconds  1minute  12
 1 
 1 sec  minute   x 
 60  =   60  minutes = 5x minutes.
 12 
 1 
=   hour
x
 60  60 
Time taken at 9 km/hr = hour
9

KUNDAN
 1 
60 minutes  1 hour 1 minute  60 hour  x  20x
  =   60  minutes = minutes.
 1 minute  1
hour  9  3
 60 60  60  Since the total time taken is 2 hours 20
Now, Distance = Speed × Time minutes ie 140 minutes.
 Distance covered in one second Now, according to the question,
= Speed × Time 20
5x  x  140
 1  3
=  72   km
 3600 
15x  20x
or,  140
 1  3
=  72   1000  m
 3600  35 x
[  1 km = 1000 m] or,  140
3
 1  or, 35x = 3 × 140
=  72   10  m = 20 m.
 36  3  410
Thus, the car will travel 20 metres in 1 or, x =
35
sec.  x = 12
Ex. 3: The speed of a goods train is 4 m/sec. Hence, the distance is 12 km.
What is its speed in km/hr? Ex. 5: Walking at 4 km/hr, a person reaches
Soln: In order to find the speed of the goods his office 5 minutes late. If he walks
t r ain in km/hr , we hav e t o find t he at 5 km/hr, he will be 4 minutes too
distance travelled (in km) by it in one hour. early. Find the distance of his office
We have, from his residence.
Speed = 4 m/sec and Time = 1 hour.
452 Concept of Arithmetic

Soln: Let the required distance be x km. When the person walks at 5 km/hr, then
x x 4
Time taken at 4 km/hr = hours t
4 5 60

x  x 1
=   60  minutes = 15x minutes or, t ....(ii)
4  5 15
Now, subt r act ing equat ion (ii) fr om
x equation (i), we have,
Time taken at 5 km/hr = hours
5
x x  1   1 
x 
 = t    t  
4 5  12   15 
=   60  minutes = 12x minutes
5 
1 1 54 9
Since the diff er ence bet ween t he two =  = =
times taken is (5 + 4) = 9 minutes. 12 15 60 60
 15x - 12x = 9
5 x  4x 9
or, 3x = 9 or, 
20 60
9
or, x = = 3. x 9
3 or, 

K
Hence, the required distance is 3 km. 20 60
Alternative Method I: 9  20
Let the distance to the office be 1 km. x = = 3
60
Then, time taken to cover 1 km at the rate
of 4 km/hr Hence, the required distance is 3 km.
Ex. 6: A gun is fired at a distance of 3.32 km
Distance 1 away from Rohit. He hears the sound
= Speed  4 hour
10 seconds later. Find the speed of the
sound.
1  Soln: Since Rohit is at a distance of 3.32 km
=   60  = 15 minutes from the gun and he hears the sound 10
4 
Time taken to cover 1 km at the rate of 5 seconds later. This means that in 10
km/hr seconds sound covers a distance of 3.32

KUNDAN
km.
Distance 1  Distance covered = 3.32 km
= 
Speed 5 hour = (3.32 × 1000) metres
= 3320 metres
1  and, time taken = 10 seconds.
=   60  = 12 minutes
5  Distance
 Difference in time taken Now, Speed =
Time
= (15 – 12) = 3 minutes
But the actual difference in time 3320
= (5 + 4) = 9 minutes  Speed = m/sec = 332 m/sec.
10
Thus, when the difference in time taken Ex. 7: A man travels a distance of 18 km from
is 3 minutes, the distance to office = 1 km his house to an exhibition by tanga at
 If the difference in time is 9 minutes, 15 km/hr and returns back on cycle at
1  10 km/hr. Find his average speed for
the distance to office =   9  = 3 km
3  the whole journey.
Hence the distance to office = 3 km. Soln: Time taken by a man to reach exhibition
Alternative Method II: from his house
Let x km be the distance of office of the
Distance 18 6
person and t hours be the time required = = hours = hours
to reach the office by the person. Speed 15 5
When the person walks at 4 km/hr, then Time taken by a man to reach his house
x 5 from the exhibition
t
4 60 Distance 18 9
= Speed = hours = hours
10 5
x 1
or, t ....(i)
4 12
Speed, Time and Distance 453

 Total time of journey  They are 16 km apart in


6 9 15 16 32 6
=    hours = hours = 3 hours.  2 hours.
5 5 5 1 13 13
6
Total distance covered 2
= (18 + 18) km = 36 km (iv) From (ii), discussed above,
 Average speed for the whole journey  1 1
They are  3  3   km apart in 1 hour.
Total Distance Covered  2 2
=  They are 16 km apart in
Total Time Taken

36 16
= 32 hours.
=km/hr = 12 km/hr 1
3
2
Ex. 8: Two men A and B start from a place P
Ex. 9: A train travelling 25 km an hour leaves
1 Delhi at 9 am and anot her t rai n
walking at 3 km and 3 km an hour
2 travelling 35 km an hour starts at 2
respectively. How many km will they pm in the same direction. How many
km from Delhi will they be together?
1 Soln: Let the required distance be x km.
be apart at the end of 2 hours,

K
2 A train leaves Delhi at 9 am and another
(i) if they walk in opposite train leaves Delhi at 2 pm in the same
directions? direction.
( i i) if they walk in the same direction? ie difference of time = 5 hours.
What time will they take to be 16 Distance travelled by the first train in 5
km apart, hours = (25 × 5) = 125 km
( i ii ) if they walk in the opposite Now, according to the question,
directions? Time taken by the train to cover (x – 125)
( iv) if they walk in the same direction? km is equal to the time taken by the second
Soln: (i) When they walk in opposite directions, train to cover the distance of x km.
Their relative velocity will be
x x  125
 
 1  1 35 25
3  3  6 km

KUNDAN
 2  2 or, 25x = 35x – 35 × 125
35  125 875 1
1 x=   437
ie, They will be 6 km apart in one hour. 10 2 2
2
 The distance from Delhi after which
1 1
 in 2 hour s t hey will be
2 they will be together = 437 km.
2
 1 1  1 Alternative Method :
 6  2   16 km apart. The first train has a start of 25 × 5 km
 2 2  4
and the second train gains (35 – 25) or 10
(ii) When they walk in same direction, km per hour.
Their relative velocity will be  the second train will gain 25 × 5 km in
 1 1 25  5 1
3  3  km or 12 hours.
 2 2 10 2
1  the required distance from Delhi
ie, They will be km apart in one hour.
2 1 1
= 12  35 = 437 km.
2 2
1 1 1  1
 in 2 hours they will be   2   1 3
2 2 2  4 Ex. 10: Walking of his usual speed, a person
4
km apart.
(iii) From (i), discussed above, 1
is 1 hours late to his office. Find his
 1  1 2
They will be  3  3   6 km apart in usual time to cover the distance.
 2  2
Soln: It is easy to see that if the speed of a train
one hour. or man be changed in the ratio of a : b,
454 Concept of Arithmetic

then the time required to travel a certain Soln: Suppose I have to walk 1 km.
distance will be changed in the ratio of
1
b : a. To walk 1 km, I require hour in the
3
3
Since the man walks at of his usual 1
4 first case, and hour in the second case.
4
4
rate, the time that he takes is of his 1 1
3
Therefore, I save    hour in the first
usual time. 3 4

4 1 1
 of usual time = usual time + 1 hours. case, and hour in the second case.
3 2 4

1 1 1 1
 of usual time = 1 hours. Thefore, I save    hour or 5 minutes
3 2 3 4
in the second case. But , by the question,
1 1
 usual time = 1  3 hours = 4 hours. I save (40 + 30) or 70 minutes.
2 2 Hence the required distance
Alternative Method : = (70  5 =) 14 km.

K
Let the usual speed of the person be x km/ Alternative Method :
hr and the distance of his office = D km. Let the required distance be D km.
His usual time to cover the distance Time taken to cover D km at 3 km/hr
D
=   hours D
x  =   hour
Now, according to the question, 3
Time taken to cover D km at 4 km/hr
3
Speed = of his usual speed D
4
=   hour
4
3  Total difference in time
=  x  km/hr
4  = 40 minutes late + 30 minutes early
Time taken to cover the distance D km

KUNDAN
 70  7
= 70 minutes =   hours
   60  6
 D  4D
=   hours Now, according to the question,
 3x  3x
  D D 7
 4   
3 4 6
Again,
4D D 1 3 1 1 7
 1 hours = hours or, D    
3x x 2 2 3 4 6

D4  3 D 7
  1  or, 
or, 12 6
x 3  2

D 1 3 7  12
or,    D = = 14 km
x 3 2 6
Hence the required distance = 14 km.
D 9 Ex. 12: Two men A and B walk from P to Q a
or, 
x 2 distance of 21 km, at 3 and 4 km an
hour respectively. B reaches Q, returns
9 1 immediately and meets A at R. Find the
 Usual time = = 4 hours.
2 2 distance from P to R.
Ex. 11: I have to be at a certain place at a Soln: When B meets A at R, B has walked the
certain time and find that I shall be distance PQ + QR and A the distance PR.
40 minutes too late, if I walk at 3 km That is both of them have together walked
an hour and 30 minutes too soon, if I twice the distance from P to Q, ie 42 km.
walk at 4 km an hour. How far have I
to walk?
Speed, Time and Distance 455

Now the rates of A and B are 3 : 4 and distance PQ (= 55 km) and when they meet
they have walked 42 km. at S for the second time, they have together
Hence the distance PR travelled by A covered three times the distance PQ or
165 km.
3
= of 42 km. = 18 km.
7
Alternative Method I: 3 32
Now PR =  55 km
1 of PQ = 11
32
2
= 30 km.
Let the required distance be x km. QP + PS is the distance covered by B when
Now, according to the question, he meets A for the second time.
A and B both walk for the same distance
1
 Distance travelled by B 2
= (21 + 21 – x) = (42 – x) km 2
 QP + PS = of 165 km = 75 km.
1
32
 42  x  2
Time taken by B =   hours
 4   PS = 75 - QP = (75 - 55) km = 20 km.
Distance travelled by A = x km  SR = PR - PS = (30 - 20) km = 10 km.

K
Ex. 14: Points A and B are 90 km apart from
x 
Time taken by A =   hours each other on a highway. A car starts
3 from A and another from B at the same
time. If they go in the same direction
x 42  x
  they meet in 9 hours and if they go in
3 4
or, 4x = 126 – 3x 9
opposite directions they meet in
or, 7x = 126 7
hours. Find their speeds.
126
or, x = = 18 Soln: Let X and Y be two cars starting from
7 points A and B respectively. Let the speed
 required distance = 18 km of car X be x km/hr and that of car Y be y
Alternative Method II: km/hr.

KUNDAN
A’s speed = 3 km Case I: When two cars move in the same
B’s speed = 4 km direction:
Let us consider that A and B meets after t
hours.
Distance covered by A in t hours = 3t km
Distance covered by B in t hours = 4t km
Total distance covered by A and B
= 3t + 4t = 7t km Suppose two cars meet at point Q. Then,
But the total distance covered by A and B is distance travelled by car X = AQ,
twice the distance between P and Q. distance travelled by car Y = BQ.
So, 7t = 21 × 2 It is given that two cars meet in 9 hours.
 Distance travelled by car X in 9 hours
21 = 9x km or AQ = 9x km
t = 2
7 Distance travelled by car y in 9 hours
t = 6 hours = 9y km
So, the distance between P and R = Distance  BQ = 9y km
travelled by A = 3 × 6 = 18 km. Clearly, AQ - BQ = AB
Ex. 13: A and B start at the same time from P  9x - 9y = 90
and Q (55 km apart) to Q and P at 3 [  AB = 90 km]
or, x - y = 10 .... (i)
1
and 2 km an hour respectively, meet Case II: When two cars move in opposite
2 directions:
at R, reach Q and P, return immediately Suppose two cars meet at point P. Then,
and meet again at S. Find the distance distance travelled by car X = AP
from R to S. distance travelled by car Y = BP
Soln: When A and B meet at R for the first time,
9
they have together cov er ed the whole In this case, two cars meet in hours.
7
456 Concept of Arithmetic

 Dist ance tr av elled by car X in Case II: When he travels 200 km by train
and the rest by car:
9 9 9
hours = x km or AP = x km If Ved travels 200 km by train, then
7 7 7 distance travelled by car is (600 - 200) km
= 400 km.
9
Distance travelled by car Y in hours Now, time taken to cover 200 km by train
7
200
9 9 = hours
= y km or BP = y km x
7 7 Time taken to cover 400 km by train
Clearly, AP + BP = AB
400
9 9 = y hours
 x  y  90
7 7
In this case the total time of journey is 8
9 hours 20 minutes
or, (x  y )  90
7 200 400
  = 8 hours 20 minutes
or, x + y = 70 ... (ii) x y
Solving (i) and (ii), we get
x = 40 and y = 30. 200 400 1
8

K
or, 
Hence, speed of car X is 40 km/hr and x y 3
speed of car Y is 30 km/hr.
Ex. 15: Ved travels 600 km to his home partly [  8 hours 20 minutes
by train and partly by car. He takes 8 20 1
hours if he travels 120 km by train and = 8 hours = 8 hours]
60 3
the rest by car. He takes 20 minutes
longer if he travels 200 km by train 200 400 25
and the rest by car. Find the speed of or,  
x y 3
the train and the car.
Soln: Let the speed of the train be x km/hr and  8 16  25
the speed of the car be y km/hr. or, 25  
Case I: When he travels 120 km by train and x y  3
the rest by car:
8 16 1

KUNDAN
If Ved travels 120 km by train, then or, x  y  3
distance covered by car is (600 - 120) km
= 480 km.
Now, time taken to cover 120 km by train 24 48
or,  1
x y
120
= hours
x 24 48
or,  1  0 .... (ii)
x y
 Distance 
 Time  
 speed  1 1
Putting  u and  v in (i) and (ii),
Time taken to cover 480 km by car x y
480 we get
= y hours 15u + 60v - 1 = 0 .... (iii)
It is given that the total time of the journey 24u + 48v - 1 = 0 .... (iv)
is 8 hours. On solving equations (iii) and (iv), we have

120 480 1 1
  8 u = and v =
x y 60 80

 15 60  1 1 1
or, 8  8 Now, u =    x = 60,
 x y  x 60 x

15 60 1 1 1
or, x

y
1 and v = y  80  y  y = 80.

Hence, speed of train = 60 km/hr and


15 60
or,   1  0 .... (i) speed of car = 80 km/hr.
x y
Speed, Time and Distance 457

Ex. 16: X takes 3 hours more than Y to walk


3
30 km. But, if X doubles his pace, he is 10u – 3 = 0  u =
10
1
ahead of Y by 1 hours. Find their 3 1 3
2 Now, u =  
speed of walking. 10 x 10
Soln: Let the speeds of X and Y be x km/hr and 10 1
y km/hr respectively. x = and v =
Then, time taken by X to cover 30 km 3 5

30 1 1
= hours  y  5  y = 5.
x
And, time taken by Y to cover 30 km
10
30 Hence, X’s speed = km/hr and
3
= y hours
Y’s speed = 5 km/hr.
By the given conditions, we have Ex. 17: After covering a distance of 30 km with
a uniform speed there is some defect
30 30 in a trai n engine and therefore, its
 3
x y

K
4
speed is reduced to of its original
10 10 5
or, x  y  1 .... (i) speed. Consequently, the train reaches
it s destinat ion late by 45 m inutes.
If X doubles his pace, then speed of X
Had it happened aft er cov er ing 18
= 2x km/hr
kilometres more, the train would have
 Time taken by X to cover 30 km
reached 9 minutes earlier. Find the
30 speed of the train and the distance of
= hours journey.
2x
Time taken by Y to cover 30 km Soln: Let the original speed of the train be x
km/hr and the distance of the journey be
30
= y
y hours y km. Then, time taken = hours.

KUNDAN
x
According to the giv en conditions, we
Case I: When defect in the engine occurs
have
after covering 30 km.
30 30 1 Speed for first 30 km = x km/hr
 1 and, speed for the remaining (y - 30) km
y 2x 2
4
30 30 3 = x km/hr
or,   5
y 2x 2
30
10 5 1  Time taken to cover 30 km = hours
  x
or, y x 2 Time taken to cover (y - 30) km
10 20 y  30 5
or,  x  y  1 .... (ii) =
4x
hours = (y - 30) hours
4x
5
1 1
Putting  u and  v , equations (i) According to the given condition, we have
x y
30 5 y 45
and (ii) become:  (y  30 )  
10u – 10v = 1 .... (iii) x 4x x 60
–10u + 20v = 1 ... (iv) 30 5y  150 y 3
or,   
Adding (iii) and (iv), we get: x 4x x 4
1
10v – 2 = 0  v  . 120  5y  150 4y  3x
5 or, 
4x 4x
1 or, 5y – 30 = 4y + 3x
Putting v  in (iii), we get:
5 or, 3x – y + 30 = 0
458 Concept of Arithmetic

Case II: When defect in the engine occurs Total time taken by the train if no accident
after covering 48 km.
d 
Speed for first 48 km = x km/hr happens =   hours
Speed for the remaining (y – 48) km x 
Case I:
4x
= km/hr Time taken by the train to cover the whole
5 length of the trip
48
 Time taken to cover 48 km = hours  
x  (d  3x ) 
=  3  1  hours
Time taken to cover (y - 48) km  75 
x
 100  
 
 y  48   5(y  48 ) 
=   hour =   hour  4(d  3x ) 
 4x   4x  = 4   hours
   3x 
 5 
Now, according to the question,
According to the given condition, the train
now reaches 9 minutes earlier ie it is 36 4(d  3x ) d
4  4
minutes late. 3x x

K
48 5(y  48) y 36 4d  12x d
   or, 
x 4x x 60 3x x
or, 4d – 12x = 3d
48 5y  240 y 3
or,    or, d = 12x .... (i)
x 4x x 5 Case II:
192  5y  240 5y  3x If the train had covered 150 km more
or,  before the accident then the distance of
4x 5x
the accident = (3x + 150) km
5y  48 5y  3x Remaining distance
or,  = (d – (3x + 15)) km
4 5
Time taken to cover the whole length of
or, 25y - 240 = 20y + 12x the trip
or, 12x - 5y + 240 = 0

KUNDAN
Thus, we have the following system of 3x  150 d  (3x  150 )
1
simultaneous equations: x 75
x
3x - y + 30 = 0 ... (i) 100
12x - 5y + 240 = 0 ... (ii)
Now, according to the question,
On solving equations (i) and (ii), we have,
x = 30 and y = 120 3x  150 d  (3x  150 ) d 7
1  
Hence, the original speed of the train is x 3x x 2
30 km/hr and the length of the journey 4
is 120 km.
Ex. 18: A train met with an accident 3 hours 3x  150 4d  12x  600 d 7
after starting, which detains it for one or,    1
x 3x x 2
hour, after which it proceed at 75% of
its original speed. It arriv es at the 3x  150 4  12x  12x  600
or, 
dest i nat i on 4 hours l at e. Had t he x 3x
accident taken place 150 km farther 12x 5
along the railway line, the train would  
x 2
1
have arrived only 3
2
hours late. Find  d  12x 
the length of the trip and the original 9x  450  36x  600 29
or, 
speed of the train. 3x 2
Soln: Let the length of the trip be d km and the
original speed of the train be x km/hr. 3x  150  12x  200 29
or, 
As the accident takes place after 3 hours. x 2
 distance covered in 3 hours by the train
= (3 × x) = 3x km 3x  150  12x  200 29
or, 
Remaining distance = (d - 3x) km x 2
Speed, Time and Distance 459

15 x  50 29 6D
or,  or, V(V  6)  4
x 2
or, 30x - 100 = 29x
or, x = 100 4V(V  6)
 D= ....(i)
Hence, speed = 100 km/hr and the length 6
of the trip (d) = 12x = 12 × 100 = 1200 km When the person moves 6 km/hr slower,
Ex. 19: A train covered a certain distance at a then
uniform speed. If the train would have
been 6 km/hr fast er, it would have D D
 6
taken 4 hours less than the scheduled V6 V
time. And, if the train were slower by 6
D D
km/hr, it would have taken 6 hours or,  6
more than the scheduled time. Find the V6 V
length of the journey. D(V  V  6)
Soln: Let t he act ual speed of t he t rain be or, 6
x km/hr and the actual time taken be y V(V  6)
hours. Then, or, D = V(V – 6) ....(ii)
Distance = (xy) km ... (i) Combining equations (i) and (ii), we get
[  Distance = speed × time]

K
4V(V  6)
If the speed is increased by 6 km/hr, then  V(V  6)
time of journey is reduced by 4 hours ie 6
when speed is (x + 6) km/hr, t ime of or, 4V + 24 = 6V – 36
journey is (y - 4) hours. or, 2V = 60
 Distance = (x + 6) (y - 4) km
60
or, xy = (x + 6) (y – 4) [Using (i)]  V = = 30
or, –4x + 6y – 24 = 0 2
or, –2x + 3y – 12 = 0 ... (ii) Hence original speed = 30 km/hr
When the speed is reduced by 6 km/hr, Putting the value of V in equation (ii), we
then the time of journey is increased by 6 have
hours ie when speed is (x - 6) km/hr, D = 30 (30 – 6) = (30 × 24) = 720 km
time of journey is (y + 6) hours Ex. 20: A hare sees a dog 100 metres away
 Distance = (x – 6) (y + 6) from her and scuds off in the opposite

KUNDAN
or, xy = (x – 6) (y + 6) [Using (i)] direction at a speed of 12 km an hour.
or, 6x – 6y – 36 = 0 A minute later the dog perceives her
or, x – y – 6 = 0 .... (iii) and gives chase at a speed of 16 km
On solving equations (ii) and (iii), we get per hour . How soon wi l l t he dog
x = 30 and y = 24. ov er t ake t he har e, and at what
Putting the values of x and y in (i), we distance from the spot whence the hare
obtain took flight?
Distance = (30 × 24) km = 720 km. Soln: Suppose the hare at H sees the dog at D.
Hence, the length of the journey is 720
km.
Alternative Method:  DH = 100 metres.
Let the original speed and distance be V Let K be the position of the hare where
km/hr and D km respectively. the dog sees her.
Time taken to complete the whole journey  HK = the distance gone by the hare in 1
D minute.
= hours 12  1000
V =  1 m = 200 m.
When the train moves 6 km/hr faster, 60
 DK = 100 m + 200 m = 300 m
then
The hare thus has a start of 300 m.
D D Now the dog gains (16 - 12) or 4 km in an
 4 hour.
V6 V
D D 60  300
or,   4  the dog will gain 300 m in or
V 6 V 4  1000
D(V  V  6) 1
or,  4 4 minutes.
V(V  6) 2
460 Concept of Arithmetic

Again, the distance gone by the hare in DH = 100 metres.


Let K be the position when the dog sees
1 hare.
4 minutes.
2
1
12  1000 1 HK = distance gone by hare in min
2
= 4 m = 900 m.
60 2
12  1000 1
 distance of the place where the hare is =  = 100 metres
caught from the spot H whence the hare 60 2
took flight DK = 100 metres + 100 metres = 200 metres
= (200 + 900) m = 1100 m. The hare thus has a strat of 200 metres.
Ex. 21: A hare, pursued by a grey-hound, is 50 Now the dog gains (16 – 12) or 4 km in an
of her own leaps ahead of him. While hour
the hare takes 4 leaps the grey-hound  Distance gained by dog in 1 min
takes 3 leaps. In one leap the hare goes 4  1000 200
=  metres
3 3 60 3
1 metres and the grey-hound 2
4 4
200
metres. In how many leaps will the grey- Now metres is covered in time
3
hound overtake the hare?

K
= 1 min
3 175 200 metres is covered in time
Soln: 50 leaps of the hare = 50  1 m= m
4 2
1 3
=  200 = 3 min.
175 200
 the grey-hound should gain m over
2 Hence, dog will catch hare in 3 minutes.
the hare. Ex. 23: Two guns were fir ed from the same
Now the grey-hound takes 3 leaps whilst place at an interval of 13 minutes but
the hare takes 4 leaps. a person in a train approaching the
 the grey-hound takes 1 leap whilst the place hears t he second r epor t 12
minutes 30 seconds after the first. Find
4 the speed of the train, supposing that
hare takes leaps.
3 sound travels 330 metres per second.

KUNDAN
Soln: It is easy to see that the distance travelled
3 by the train in 12 min 30 seconds could
 the grey-hound goes 2 m whilst the
4 be travelled by sound in (13 min - 12 min
30 seconds) or 30 seconds.
4 3
hare goes 1 m  the train travels 330 × 30 metres in
3 4
1
12 min.
 3 4 3 2
 the grey-hound gains  2   1  or
 4 3 4  the speed of the train per hour
5 330  30  2  60
m in one leap. =
12 25  1000

175 1188 13
 t he gr ey-hound gains m in = or 47 km per hour
2 25 25
Ex. 24: A carriage driving in a fog passed a
 175 12  man who was walking at the rate of 3
    210 leaps.
 2 5  km an hour in the same direction. He
Ex. 22: A hare starts to run at 12 km per hour could see the carriage for 4 minutes
when a dog is 100 metres off. After and i t was v i si bl e t o hi m upt o a
half a minute the dog sees hare and distance of 100 m. What was the speed
pursues at 16 km per hour. How soon of the carriage?
will he catch hare? Soln: The dist ance t rav elled by man in 4
Soln: Suppose the hare H sees the dog at D. minutes
3  1000
=  4 metres = 200 metres.
60
Speed, Time and Distance 461

 distance travelled by carr iage in 4  Distance covered in 1 hour


minutes x
= (200 + 100) or 300 metres. = km
50  15
 speed of carriage In the second case:
300 60 Distance to be covered = 2x km
=  km per hour
4 1000 x x
Speed = 2   km/hr
1 50  15 25  15
= 4 km per hour. Number of hours he walks per day
2
= (24 – 2 × 9 =) 6 hours
Ex. 25: Two trains start at the same time, one  Distance covered in 1 day
from A to B and the other from B to A.
x
If they arrive at B and A respectively 5 = 6 km
hours and 20 hours after they passed 25  15
each other. Show that one travels twice 6x
as fast as the other. Now km is covered in 1 day
25  15
Soln: Let the two trains be P and Q.
 2x km shall be covered in
2x  25  15
= = 125 days.
6x

K
Let the train P starts from A and Q starts
from B and they meet at C. Ex. 27: A person walks from A to B at the rate
Let P’s speed be x km per hour and Q’s of 3 kmph and after transacting some
speed be y km per hour. business which occupies him an hour,
 BC = 5x km; AC = 20y km returns to A by tramway at the rate of
Now time taken by P to travel a distance 5 kmph. He then finds that he has been
absent for 2 hours 20 minutes. Find
20y
20y = hours the distnace from A to B.
x Soln: We have average speed
And time taken by Q to travel a distance
 2 3 5 
=    3.75 kmph
5x  35 
5x = y hours
The time for which he travels

KUNDAN
Since both start at the same time and meet,
 1  1
those two times must be equal. = 2 1  1 hours
 3  3
20y 5 x
 
x y 1  1 
Distance =   3.75  1   = 2.5 km
2
or, 5x = 20y 2 2
  3 
Note: In calculating distance we have
x2
or, 2 = 4  1 1
y multiplied  3.75  1  by . Because we
 3 2
x 2 will get twice the distance otherwise.
or, 
y 1 Ex. 28: A person has to reach a place 40 km
away. He walks at the rate of 4 km/hr
Ex. 26: A can walk a certain distance in 50
for the first 16 km and then travels in a
days when he rests 9 hours each day,
scoot er for t he r est of t he j our ney.
how long it will take for him to walk
However, if he had travelled by scooter
twice as far if he walks twice as fast
for the 16 km and covered the remaining
and rests twice as long each day?
distance on foot at 4 km/hr, he would
Soln: Suppose initially he covers x km in 50
have taken an hour longer to complete
days
t he j our ney. Find the speed of t he
x scooter.
 Distance covered in 1 day = km Soln: Total distance covered = 40 km
50
Since he rests for 9 hours, Distance covered on foot = 16 km
 Distance covered in (24 – 9 =) 15 hours  Distance covered in a scooter
= 40 km – 16 km = 24 km
x Suppose speed of the scooter
= km
50 = x km per hour
462 Concept of Arithmetic

Now, according to the question,


x
speed of the man = 4 km per hour km/hr and km at a speed of 24 km/hr.
Then time taken in the first case 2
Then, time t aken to tr av el t he whole
16 24  24 
=   4   hours
4 x  x   x x 
journey =    hours
Also time taken in the second case  2  21 2  24 
Now, according to the question,
16 24  16 
=    6  hours x x
x 4  x   = 10
Since the time taken in the second case 2  21 2  24
is one hour longer 24x  21x
or, = 10
 16   24  21  24  2
   6   4   1
 x   x 
2  10  21  24
 x =  224 km.
16 24 21  24
or, 64 = 1
x x Ex. 31: A monkey tries to ascend a greased pole
14 metres high. He ascends 2 metres
16 24 in first minute and slips down 1 metre
or,  = 1 – 6 + 4

K
x x in the alternate minute. If he continues
to ascend in this fashion, how long
8 does he take to reach the top?
or,  1
x Soln: In every 2 minutes he is able to ascend
(2 – 1 =) 1 metre. This way he ascends
8
or, =1 upto 12 metres because when he reaches
x at the top, he does not slip down. Thus,
or, x = 8 upt o 12 met res he takes 12 × 2 = 24
Hence, speed of the scooter minutes and for the last 2 metres he takes
= 8 km per hour. 1 minute. Therefore, he takes (24 + 1 =)
Ex. 29: A boy goes to school at a speed of 3 25 minutes to reach the top. That is, in
km/hr and returns to the village at a 26th minute he reaches the top.
speed of 2 km/hr. If he takes 5 hours Ex. 32: Two runners cover the same distance

KUNDAN
in all, what is the distance between the at the rate of 15 km and 16 km per
village and the school? hour respectively. Find the distance
Soln: Let the required distance be x km. travelled when one takes 16 minutes
Then time taken during the first journey longer than the other.
Soln: Let the distance be x km.
x
= hour. x
3
Time taken by the first runner = hours
And time taken during the second journey 15
x Time taken by the second runner
= hour.
2 x
= hours
x x 16
  5
3 2 Now, according to the question,

2x  3x x x 16
5  
or, 15 16 60
6
or, 5x  30 x (16  15 ) 16
or, 
 x = 6 15  16 60
 required distance = 6 km
Ex. 30: A motor car does a journey in 10 hours, 16
x=  15  16 = 64 km.
the fi rst half at 21 km/hr and t he 60
second half at 24 km /hr . Fi nd t he Ex. 33: Without any stoppage a person travels
distance. a certain distance at an average speed
Soln: Let the distance be x km. of 80 km/hr and with stoppages he
x covers the same distance at an average
Then km is travelled at a speed of 21 speed of 60 km/hr. How many minutes
2 per hour does he stop?
Speed, Time and Distance 463

Soln: Let the total distance be x km.


 3  2
Time taken at the speed of 80 km/hr Remaining distance = 1    of the
 5 5
x
= hours. total distance
80
2 5 2
Time taken at the speed of 60 km/hr  of the total distance= 390  
5 3 5
x
= hours. = 260 km.
60  av er age speed f or t he remaining
 x x   260 
 he rested for    hours distance =    65 km/hr.
 60 80   4 
20x x Ex. 36: When a man travels equal distance at
=  hours speeds V 1 and V 2 km/hr, his average
60  80 240 speed is 4 km/hr. But when he travels
 his rest per hour at t hese speeds for equal ti m e hi s
x x x 60 average speed is 4.5 km/hr. Find the
=    difference of the two speeds and also
240 60 240 x
find the values of V1 and V2 .
Soln: Suppose the equal distance = D km

K
1
= hours = 15 minutes. Then time taken with V 1 and V 2 speeds
4
Ex. 34: A man rode out a certain distance by D D
train at the rate of 25 km an hour and are V hours and V hours respectively.
1 2
walked back at the rate of 4 km per
hour. The whole journey took 5 hours Total distance
and 48 minutes. What distance did he  average speed =
Total time
ride?
Soln: Let the distance be x km. 2D 2V1V2
   4 km/hr
Then time spent in journey by train D D V1  V2

x V1 V2
= hours.
25 In second case,

KUNDAN
And time spent in journey by walking V1  V2
average speed =  4.5 km/hr
x 2
= hours.
4 or, V1 + V2 = 9 and V1 V2 = 18

x x Now, V1  V2 2  V1  V2 2  4V1V2


Therefore,   5 hours 48 minutes.
25 4 = 81 – 72 = 9
29x 48 29  V1  V2  3 km/hr .....(i) and
or, 5 
100 60 5 V1 + V2 = 9 km/hr .....(ii)
On solving equations (i) and (ii), we have
100 V1 = 6 km/hr and V2 = 3 km/hr.
 x   20 km
5 Ex. 37: A m an t akes 8 hours t o walk t o a
Ex. 35: A person travels for 3 hours at the certain place and ride back. However,
speed of 40 km/hr and for 4.5 hours he could have gained 2 hours, if he had
at the speed of 60 km/hr. At the end of covered both ways by riding. How long
woul d he hav e t aken t o walk bot h
3 ways?
it, he finds that he has covered of
5 Soln: Walking time + Riding time
the total distance. At what average = 8 hours .... (1)
speed should he travel to cover the 2 Riding time = (8 – 2 =) 6 hours .... (2)
remaining distance in 4 hours? 2 × (1) – (2) gives the result
Soln: Total distance covered in (3 + 4.5) hours 2 × walking time = (2 × 8 – 6 =) 10 hours.
= 3 × 40 + 4.5 × 60 = 390 km.  both ways walking will take 10 hours.
Now, according to the question, Ex. 38: A person travelled 120 km by steamer,
450 km by train and 60 km by horse.
3
of the distance = 390 km It took 13 hours 30 minutes. If the rate
5 of the train is 3 times that of the horse
464 Concept of Arithmetic

and 1.5 times that of the steamer, find Ex. 40: A per son cov ers a di stance i n 40
the rate of horse, train and steamer minutes if he runs at a speed of 45 km
per hour. per hour on an average. Find the speed
Soln: Suppose the speed of horse = x km/hr. at which he must run to reduce the time
Then speed of the train = 3x km/hr of journey to 30 minutes.
and speed of the steamer = 2x km/hr Soln: Let the distance be D km.
Now, according to the question,
D 40 2
120 450 60 = 40 minutes = 
   13.5 hours 45 60 3
2x 3x x
(Since 13 hours 30 minutes = 13.5 hours) 2
 D =  45 = 30 km
360  900  360 3
or,  13.5
6x Let the required speed be x km per hour.
Now, according to the question,
1620
 x   20
1
6  13.5 x = 30
Hence, speed of horse = 20 km/hr 2
 Speed of train = 3x = 3 × 20
= 60 km/hr  1 
Speed of steamer = 2x = 2 × 20  30 minutes  2 hour 
 

K
= 40 km/hr  x = 30 × 2 = 60 km/hr.
Ex. 39: A man covers a certain distance on Ex. 41: The distance between t wo st ati ons,
scooter. Had he moved 3 km/hr faster, Delhi and Amritsar, is 450 km. A train
he would have taken 40 minutes less. starts at 4 pm from Delhi and moves
If he had moved 2 km/hr slower, he towards Amritsar at an average speed
would have taken 40 minutes more. of 60 km/hr. Another train starts from
Find the distance (in km) and original Amr i t sar at 3.20 pm and m ov es
speed. towards Delhi at an average speed of
Soln: Suppose the distance is D km and the 80 km/hr. How far from Delhi will the
initial speed is x km/hr. two trains meet and at what time?
D D 40 Soln: Suppose the trains meet at a distance of x
Then, we have   and km from Delhi. Let the trains from Delhi
x  3 x 60
and Amritsar be A and B respectively.

KUNDAN
D D 40 Th en ,
 
x  2 x 60 [Time taken by B to cover (450 – x) km]
– [Time taken by A to cover x km]
D D 2
or,  
x x 3 3 40
= ..... (see note)
3D 2 60
or, x (x  3)  3 .... (1)
450  x x 40
or,  
D D 2 80 60 60
and  
x 2 x 3  3(450 – x) – 4x = 160
or, 7x = 1190
2D 2 or, x = 170.
or, x (x  2)  3 .... (2) Thus, the trains meet at a distance of 170
km from Delhi.
From (1) and (2), we have
Time taken by A to cover 170 km
3D 2D
  170 
x (x  3) x ( x  2) =   hours = 2 hours 50 min.
 60 
or, 3(x  2)  2(x  3)
So, the trains meet at 6.50 pm
or, 3x  6  2x  6 Note: RHS = 4 : 00 pm – 3.20 pm
 x  12 km/hr 40
Now, if we put this value in (1), we get = 40 minutes = hour
60
2 12  15 LHS comes from the fact that the train
D =  = 40 km.
3 3 from Amritsar took 40 minutes more to
Hence, the distance is 40 km and the travel up to the meeting point because it
original speed is 12 km/hr. had started its journey at 3.20 pm whereas
Speed, Time and Distance 465

the train from Delhi had started its journey


300 x  1500  300 x
at 4 pm and the meeting time is the same or, 2
for both the trains. x 2  5x
Ex. 42: A man leaves a point P at 6 am and or, 2x2 + 10x = 1500
reaches the point Q at 10 am. Another or, x2 + 5x - 750 = 0
man leaves the point Q at 8 am and or, x2 + 30x - 25x - 750 = 0
reaches the point P at 12 noon. At what or, x(x + 30) - 25 (x + 30) = 0
time do they meet? or, (x - 25) (x + 30) = 0
Soln: Let the distance PQ be A km. or, x = 25 or x = -30.
And they meet x hours after the first man Negative value of x should be rejected.
starts. Hence, the original speed of the train is
Average speed of first man 25 km/hr.
Ex. 44: A fast train takes 3 hours less than a
A A
=  km/hr. slow train for a journey of 600 km. If
10  6 4 the speed of the slow train is 10 km/hr
Average speed of second man less than that of the fast train, find the
speeds of the two trains.
A A
=  km/hr. Soln: Let t he speed of t he slow tr ain be
12  8 4 x km/hr. Then, speed of the fast train is
(x + 10) km/hr.

K
Ax
Distance travelled by first man = km Time taken by the slow train to cover 600
4
They meet x hours after the first man 600
km = hours.
starts. The second man, as he starts 2 x
hours late, meets after (x - 2) hours from Time taken by the fast train to cover 600
his start. Therefore, the distance travelled
by the second man 600
km = hours.
x  10
A(x  2)
= km Now, according to the question,
4
600 600
Ax A(x  2)  3
Now,  km = A x x  10
4 4
600( x  10 )  600 x

KUNDAN
or, 2x – 2 = 4 3
or, x (x  10 )
 x = 3 hours.
 They meet at 6 am + 3 hours = 9 am
Ex. 43: A train travels a distance of 300 km 6000
or, 3
at constant speed. If the speed of the x 2  10 x
train is increased by 5 km an hour,
the journey would have taken 2 hours or, 3(x 2  10 x )  6000
less. Find the ori ginal speed of the
train. or, x 2  10x  2000  0
Soln: Let x km/hr be the constant speed of the
or, x 2  50x  40x  2000  0
train.
Then, time taken to cover 300 km or, x x  50   40 x  50   0
300 or, (x + 50) (x - 40) = 0
= hours. or, x = –50 or x = 40
x
or, x = 40
Time taken to cover 300 km when the
[  x cannot be negative]
speed is increased by 5 km/hr
or, Hence, the speeds of two trains are 40
300 km/hr and 50 km/hr.
= hours. Ex. 45: A plane left 30 minutes later than the
x 5
It is given that the time to cover 300 km schedule time and in order to reach its
is reduced by 2 hours. destination 1500 km away in time it
has to increase its speed by 250 km/hr
300 300 from its usual speed. Find its usual
  2
x x 5 speed.
Soln: Let t he usual speed of the plane be
300(x  5)  300 x x km/hr. Then,
or, 2
x (x  5 )
466 Concept of Arithmetic

Time taken to cover 1500 km with the or, x = 600


1500 or, x = -400
usual speed = hours or, x = 600
x
Time taken to cover 1500 km with the [  x cannot be negative]
So, the original speed of the aircraft was
150 600 km/hr.
speed of (x + 250) km/hr =
x  250
 600 
1500 1500 1 Hence, duration of flight =   hour
    x 
x x  250 2
1500 1500 1  600 
or,   =   hour = 1 hour.
x x  250 2  600 
Ex. 47: Two trains leave a railway station at
1500 x  1500  250  1500 x 1 the same time. The first train travels
or, 
x (x  250 ) 2 due west and t he second t r ai n due
north. The first train travels 5 km/hr
1500  250 1 faster than the second train. If after
or, 
x 2  250 x 2 two hours, they are 50 km apart, find
2
or, 750000 = x + 250x the average speed of each train.
Soln: Let t he speed of the second t rain be

K
or, x2 + 250x – 750000 = 0
or, x2 + 1000x – 750x – 750000 = 0 x km/hr. Then, the speed of the first train
or, x(x + 1000) - 750 (x + 1000) = 0 is (x + 5) km/hr.
or, (x + 1000) (x - 750) = 0 Let O be the position of the railway station
or, x = -1000 or x = 750 from which the two trains leave.
or, x = 750 Distance travelled by the first train in 2
hours
[  speed cannot be negative]
= OA = speed × time = 2(x + 5) km
Hence, the usual speed of the plane is
Distance travelled by the second train in
750 km/hr.
2 hours
Ex. 46: In a flight of 600 km, an aircraft was
= OB = speed × time = 2x km
slowed down due to bad weather. Its
By Pythagoras Theorem,
average speed for the trip was reduced
AB2 = OA2 + OB2
by 200 km/hr and the time of flight

KUNDAN
i ncr eased by 30 m inut es. Fi nd t he
duration of flight.
Soln: Let the original speed of the aircraft be x
km/hr.
Then, new speed = (x – 200) km/hr
Duration of flight at original speed
 600 
=   hour
 x 
Duration of flight at reduced speed
 600 
=   hour
 x  200  or,502 = [2(x + 5)]2 + {2x}2
or,2500 = 4(x + 5)2 + 4x2
600 600 1
   or,8x2 + 40x – 2400 = 0
x  200 x 2 or,x2 + 5x – 300 = 0
or,x2 + 20x – 15x – 300 = 0
600 x  600(x  200 ) 1
or,  or,x(x + 20) – 15 (x + 20) = 0
x (x  200 ) 2 or,(x + 20) (x – 15) = 0
or,x = –20 or x = 15
120000 1
or,  [  x cannot be negative]
x 2  200 x 2 Hence, the speed of the second train is
or, x2 – 200x – 240000 = 0 15 km/hr and the speed of the first train
or, x2 - 600x + 400x - 240000 = 0 is 20 km/hr.
or, x(x - 600) + 400 (x - 600) = 0 Ex. 48: A walks half a kmph faster than B and
or, (x - 600) (x + 400) = 0 three quarters of a kmph faster than
or, x - 600 = 0 or x + 400 = 0 C. To walk a certain distance C takes
Speed, Time and Distance 467

three quarters of an hour more than B


or, 4y 2  17y  4  0
and two hours more than A. Find the
distance covered and the time taken 1
by B. Also find his speed. or, y  4,
Soln: Let the speed of B be y kmph 4
Now f r om (5), d = 45 and f r om (3),
 1
 Speed of A =  y   kmph and 1
 2 T = 10 hours. (y = is not possible).
4
 1 Ex. 49: A man started from his house to his
speed of B =  y   kmph and
 2 workplace 8 km away at the rate of 4
kmph so as to reach just in time. After
1 3 1 5 minutes he realised that he had left
speed of C = y   y kmph
2 4 4 some important documents at home,
Let the distance travelled be d km and so he turned back, and now walking
time for A is T. at an increased speed, still succeeded
in reaching his workplace in time. What
d 2d was his increased speed?
Now, for A, T =  ... (1)
1 2y  1 Soln:
y
2

K For C, T + 2 =

or, T 

and for B,

T
4d
4y  1

5 d

4 y
y

2 
d
1
4

4d  8y  2
4y  1

d 5
or, T  y  4
4d
4y  1

... (2)
Let A be the house and B be the workplace.
The workplace is 8 km away. Had he
walked all the way at 4 kmph he would
have t aken 2 hour s to r each his
workplace, which would be just in time.
Therefore, total time = 2 hours
Referring to the above diagram, the total
4d  54 time is made up

KUNDAN
or, T  4y ... (3) (i) walking from A to C for 5 minutes

From (1) and (2), 5 1


= hours = hours
60 12
2d 4d  8y  2
 now, 4 km in 1 hour
2y  1 4y  1
or, d (4y - 1) = (2y + 1) (2d - 4y + 1) 1 1
 km in hours
or, 4dy - d = 4dy - 8y2 + 2y + 2d - 4y + 1 3 12
or, 8y2 + 2y - 3d - 1 = 0 1
8y 2  2y  1 Distance AC = km
or, d = .... (4) 3
3 (ii) If x is the increased speed, time for
and from (2) and (3) walking from C to A
4d  8y  2 4d  5y AC 1
 x hours
4y  1 4y = hours =
x 3
or, 16dy  32y 2  8y  16dy  20y 2  4d  5y (iii) Walking from A to B time taken
AB 8
or, 12y 2  3y  4d  0 =  hours
x x
12y 2  3y Now, according to the question,
or, d  .... (5)
4 1 1 8
From (4) and (5), 2  
12 3x x
8y 2  2y  1 12y 2  3y

3 4 8
 x = 4 kmph
or, 32y 2  8y  4  36y 2  9y 23
468 Concept of Arithmetic

Ex. 50: Dinky is picked up by his father by car by travelling partly by foot and partly by
from school everyday and they reach car, he takes 45 minutes longer than if
home at 5.00 pm. One day, since school he would hav e t r avelled t he whole
got over an hour earlier than usual, he distance by car.
start ed walki ng towar ds home at 3 Therefore time taken to walk the distance
kmph. He met his father on the way BC
and t hey r eached home 15 mi nut es = tw = (7.5 + 45 =) 52.5 minutes.
earlier than their usual time. What is  tw = 52.5 minutes
the speed of the car? But for a constant distance BC,
Soln: sd t
 w
sw td
sd 52.5
 
3 7.5
 s d = 21 kmph
Ex. 51: Two places P and Q are 162 km apart.
Let the speed of car be sd and the speed A train leaves P for Q and at the same
of Dinky be s w time another train leaves Q for P. Both
 s w = 3 kmph the trains meet 6 hours after they start

K
Ref er ring t o the abov e diagr am, if A moving. If the train travelling from P
represents the home and B represents the to Q travels 8 km/hr faster than the
school, the father starts at his usual time other train, find the speed of the two
but meets his son on the way at C. So, in trains.
going from A to C and back to A he saves Soln: Suppose the speeds of the two trains are
the time he would have used commuting x km/hr and y km/hr respectively.
from C to B and back to C. Since they Now,
reach back 15 minutes earlier than usual, Total distance travelled by both the trains
ie the time saved = 15 minutes. Therefore in 6 hours = (6x + 6y) km
time taken to drive from C to B and back Now, according to the question,
is 15 minutes. Therefore time taken to (6x + 6y) = 162
15 162
drive the distance BC = td = or, x  y   27 ....(i)
2 2

KUNDAN
 td = 7.5 minutes and x – y = 8 ....(ii)
Dinky starts 1 hour earlier than usual. Solving equations (i) and (ii), we have
Had he moved at driving speed, ie speed x = 17.5 km/hr and
of the car, he would have reached 1 hour y = 9.5 km/hr.
earlier. But he reaches only 15 minutes Hence, speed of the two trains are 17.5
earlier. Therefore he loses 45 minutes, ie km/hr and 9.5 km/hr.

Practice Exercise

the scheduled time. What is the distance of


6
1. For a journey, walking of his usual speed, the school from her house?
7 4. I will reach my destination 40 minutes late if
a man becomes late by 25 minutes. What is I walk at the rate of 3 km/hr. However, I will
his usual time taken for the journey? reach 30 minutes before time if I walk at the
2. A motorist covers a distance from A to B at a rate of 4 km/hr. Find the distance of my
speed of 20 km/hr and return journey from B destination from the starting point.
to A at a speed of 30 km/hr. If he takes 5 5. A student travels to his school at a speed of 4
hours for the whole journey, find the distance km/hr and reaches the school 15 minutes
from A to B. late. On travelling at a speed of 6 km/hr, he
3. Shivangi starts from her house for her school reaches the school 5 minutes early. At what
at a certain fixed time. If she walks at the speed must he travel to reach the school just
rate of 5 km/hr, she is late by 7 minutes. in time?
However, if she walks at the rate of 6 km/hr, 6. A person travels a certain distance on a bicycle
she reaches the school 5 minutes earlier than at a certain speed. Had he moved 3 km/hr
Speed, Time and Distance 469

faster, he would have taken 40 minutes less. station B meets with an accident one hour
Had he moved 2 km/hr slower, he would have after starting. After stopping there for 30
taken 40 minutes more. Find the distance 4
and original speed of the person. minutes, it proceeds at of its usual speed
5
7. Two places A and B are 80 km apart from and arrives at B 2 hours late. Had the train
each other on a highway. A car starts from A covered 80 km more before the accident, it
and another from B at the same time. If they would have been just one hour late. Determine
move in the same direction, they meet each the original speed of the train and the distance
other in 8 hours. If they move in opposite between A and B.
directions towards each other, they meet in 17. A train after travelling 50 km meets with an
1 hour 20 minutes. Determine the speeds of
3
the cars. accident and then proceeds at of its former
8. A train covers a distance between stations A 4
speed and ar r ives at it s dest inat ion 25
and B in 45 minutes. If the speed is reduced
minutes late. Had the accident occurred 24
by 5 km/hr, it will cover the same distance
km behind, it would hav e reached the
in 48 minutes. What is the distance between
destination only 35 minutes late. Find the
the two stations A and B (in km)? Also, find
speed of the train and the distance travelled
the speed of the train.
by the train.
9. A car covering half of a distance of 100 km
18. Ravi can walk a certain distance in 40 days,
develops some engine trouble and later travels

K
when he rests 9 hours a day. How long will
at half of its original speed. As a result, it
he take to walk twice the distance, twice as
arrives 2 hours later than its normal time.
fast and rest twice as long each day?
What was the original speed of the car?
19. Two men set out at the same time to walk
1 1 towards each other from two points A and B,
10. A train covers a distance of 193 km in 4 72 km apart. The first man walks at the rate
3 4
hours with one stoppage of 10 minutes, two of 4 km/hr. The second man walks 2 km in
of 5 minutes and one of 3 minutes on the 1
way. Find the average speed of the train. the first hour, 2 km in the second hour, 3
2
11. Distance between two places X and Y is 60
km in the third hour and so on. Find the
km. Two persons A and B start from X towards
time after which the two men will meet.
Y at the same time. Speed of B is 4 km/hr
20. Two trains start out towards each other from
less than the speed of A. A reaches Y, returns

KUNDAN
points 650 km apart. If they start out at the
at once and meets B at a distance of 12 km
same time, they will meet in 10 hours, but if
from Y. What is the speed of B?
one of them starts out 4 hours and 20 minutes
12. An army bomb squad man set a fuse f or
after the other, they will pass each other 8
blasting a rock to take place after one minute.
hours following the departure of the latter.
He ran away from the site at the speed of 13
Determine the average speed of each other.
m/s. Sound travels at the speed of 325 m/s.
21. Distance between two stations X and Y is
Upto what distance could the army man run,
220 km. Trains P and Q leave station X at 8
before he heard the sound of blast?
am and 9.51 am respectively at the speed of
13. On a particular day a person starts walking
25 km/hr and 20 km/hr respectively f or
from a place X at 2 am and reaches place Y at
journey towards Y. A train R leaves station Y
5 am. A second person starts walking from a
at 11.30 am at a speed of 30 km/hr for journey
place Y at 4 am and reaches place X at 9 am
towards X. When and where will P be at equal
on the same day. At what time do they cross
distance from Q and R.
each other?
22. Two places P and Q are 336 km apart. A train
14. A covers some distance in 50 days when he
leaves P for Q and at the same time another
rests 9 hours a day. In how many days will
train leaves Q for P. Both trains meet at the
he cover the double distance by resting twice
end of the 6 hours. If one train travels 8 km/hr
as before?
faster than the other, find the speeds of the
15. A man travelled a total distance of 3990 km,
other trains.
part of it by air, part by water and the rest by
23. On a 2-km road, a total number of 201 trees
land. The time he spent in travelling by air,
are planted on each side of the road at equal
water and land was in rat io 1 : 16 : 2
distances. How many such trees in all will
respectively and the average speed of each
be planted on both sides of a 50-km road such
mode of travel was in the ratio 20 : 1 : 3
that the distance between two consecutive
respectively. If his overall average speed was
trees is the same as that of the consecutive
42 km/hr, find the distance covered by water.
trees on the 2-km road?
16. A goods train travelling from station A to
470 Concept of Arithmetic

Answers and explanations


1. Let the usual speed of the person be x km/hr Since the difference between the two times
and the distance of his journey be D km. taken is (7 + 5 =) 12 minutes
D  12x – 10x = 12
His usual time to cover the distance =   hour or, 2x = 12
x 
Now, according to the question, 12
x = = 6
2
6 6  Hence, the required distance is 6 km.
Speed = of his usual speed =  x  km/hr
7 7  Alternative Method:
Time taken to cover the distance D km Let x km be the distance between her house
and school and t hours be the time required
  to reach the school from her house.
 D  7D
=   hour When Shivangi walks at 5 km/hr, then
 6x  6x x 7
  t
 7  ....(i)
5 60
Again, When Shivangi walks at 6 km/hr, then
7D D  25  x 5

K
   t
6x x  60  6 60
D7  5 x 1
or,   1  or t ....(ii)
x 6  12 6 12
Subtracting equation (ii) from equation (i), we
D  5  5
or,   6  hours get
x  12  2
x x  7   1 
  t    t  
1 5 6  60   12 
 Usual time = 2 hours.
2
x 1 7 5  7 12 1
2. Let the distance from A and B is x km. or,     
30 12 60 60 60 5
 Time taken to cover the distance from A to
30
x x = = 6

KUNDAN
B at 20 km/hr = hours. 5
20 Hence the required distance = 6 km
And time taken to cover the distance from B 4. Let the required distance be x km.
x Difference of time taken at different speeds
to A at 30 km/hr = hours. = (40 + 30) minutes = 70 minutes
30
 Total time taken = 5 hours. (Given) 70 7
= hours = hours
60 6
x x
  5
20 30 x
Time taken at 3 km/hr = hours
3x  2x 3
5
60 x
or, 5x = 60 × 5 = 300 Time taken at 4 km/hr = hours
4
300 According to the question,
or, x = = 60 km
5
x x 7
3. Let the required distance be x km.  
3 4 6
x
Time taken to walk at 5 km/hr = hours 4x  3x 7
5 or, 
12 6
x 
=   60  minutes = 12x minutes x 7
5  
or,
12 6
x
Time taken to walk at 6 km/hr = hours
6 7
or, x =  12 = 14
x  6
=   60  minutes = 10x minutes  Distance of the destination = 14 km
6 
Speed, Time and Distance 471

5. Let the distance of the school be x km.


2V V  3
x or, D  ....(i)
Time taken in first case = hours 9
4 When the person moves 2km/hr slower, then
15 1 D D 40
But this time is 15 minutes late or   
60 4 V  2 V 60
hours late
D D 2
 Actual time for reaching the school in time or,  
V2 V 3
x 1
should be    hours D D 2
4 4 or,  
V2 V 3
x
Time taken in second case = hours DV  DV  2D 2
6 
or, V(V  2) 3
 5  1
But this time is 5 minutes early or  
 60  12 2D 2
or, V(V  2)  3
hours early.
 Actual time for reaching the school in time

K
V(V  2)
x 1  or, D  ....(ii)
should be    hours 3
 6 12  Combining equations (i) and (ii), we get
From the above, we have
2V(V  3) V(V  2)

x 1 x 1  9 3
    
 4 4   6 12  or, 2(V + 3) = 3(V – 2)
or, 2V + 6 = 3V – 6
x  1 2x  1
or,  or, 3V – 2V = 6 + 6
4 12 or, V = 12 km/hr
or, 12x  12  8x  8 Putting the value of V in equation (ii),
or, 4x  20 12  10
we get D = = 40 km.

KUNDAN
or, x = 5 km 3
 The distance of the school be 5 km and 7. Case I: When the cars are moving in the same
(x  1) direction.
actual time to reach school in time =
4
= 1 hour
 The required speed is 5 km/hr.
6. Let t he or iginal speed and distance be Let A and B be two places and C be the place
V km/hr and D km respectively. of meeting.
Time taken to complete the whole journey Let the speed of car starting from A be x km/hr
D and the car starting form B be y km/hr.
= hours. Relative speed = (x – y) km/hr
V
When the person moves 3 km/hr faster, then According to the question,
(x – y) × 8 = 80
D D 40 or, x – y = 10 ...(i)
 
V  3 V 60 Case II: When the cars are moving in the
opposite directions and they meet at point C.
D D 2
or,  
V3 V 3
Relative speed = (x + y) km/hr
D D 2 Time taken = 1 hour 20 minutes
or,  
V V3 3
 1  4
DV  3D  DV 2 = 1    hours
  3  3
or, V V  3 3
Again, according to the question,
3D 2 4
or, V V  3  3 (x  y )   80
3
472 Concept of Arithmetic

or, x + y = 60 ...(ii)
Solving equations (i) and (ii), we have 580
x = 35 and y = 25  Average speed of the train = 3 km/hr
 Speeds of the cars 58
= 35 km/hr and 25 km/hr. 15
8. Suppose the distance is x km and the speed 580  15
of the train is y km/hr. = = 50 km/hr
Thus we have two relationships: 58  3
11.
x 45 3 3
(1)   x  y
y 60 4 4

x 48 4 4
(2)    x  (y  5)
y  5 60 5 5
From (1) and (2), we have
Let A and B meet after t hours.
3 4
y  (y  5 ) Let the speed of B be x km/hr.
4 5  Speed of A = (x + 4) km/hr
Distance covered by A in t hours = 60 + 12
4 3
or, y     4

K
= 72 km
5 4 Distance covered by B in t hours = 60 – 12
= 48 km
4  20
or, y = = 80 km/hr Now, according to the question,
16  15 xt = 48 ....(i)
Therefore speed = 80 km/hr and distance (x + 4)t = 72 ....(ii)
3 On dividing equation (ii) by equation (i), we
x =  80 = 60 km have
4
9. Half of the original speed means double the x  4 72 3
 
normal time. It means that the car should x 48 2
have covered half of the distance of 100 km, or, 2x + 8 = 3x
ie 50 km, in 2 hours. or, x = 8
Hence, the original speed of the car  Speed of A = 8 km/hr

KUNDAN
 50  12. Time after which the bomb is set to explode
=   = 25 km/hr = 1 minute = 60 seconds
 2  Speed of the man = 13 m/sec
10. Distance covered by train Distance covered by man in 60 sec
1 580 = 13 × 60 = 780 metres
= 193 km = km So, distance to be travelled by sound before it
3 3
catches up with army man = 780 metres
Time taken by the train to cover this distance Speed of the sound = 325 m/sec (given)
1 17 Since the man and sound are travelling in
= 4 hours = hours the same direction, the relative speed of sound
4 4
= (325 – 13 =) 312 m/sec
Total stoppage during the journey Time taken by sound to travel 780 metres
= 10 × 1 + 5 × 2 + 3 × 1
780
23 = = 2.5 sec
= 23 minutes = hours 312
60
Now, dur ing t his time man would hav e
Actual time taken by the train to cover the travelled further. So, distance covered by man
above distance in 2.5 seconds = 2.5 × 13 = 32.5 m
17 23 The total distance travelled by man
=  = 780 + 32.5 = 812.5 metres.
4 60
13. X P Y
17  15  23 l l l
=
60 Let the speed of the person who starts from X
be x km/hr and speed of the person who starts
255  23 232 58 from Y be y km/hr.
=  = hours
60 60 15 Time taken by the person who starts from X
= 5 am – 2 am = 3 hours
Speed, Time and Distance 473

Time taken by the person who starts from Y 14. Let the distance for A be x km
= 9 am – 4 am = 5 hours Number of hours A walks daily = (24 – 9 =) 15
Again, let the distance between X and Y be D hours
km. Number of days = 50 days
Now, according to the question,
x
D D  Speed (in km/hr) = ..... (1)
50  15
x km/hr = and y km/hr =
3 5 In second situation
If the person starting from X reaches the Let the number of days be Y
meeting point after t hours, person starting Distance = 2x
from Y will reach the meeting point after Number of hours for which A walks daily = 6
(t – 2) hours. Since the person starting from hours
X starts moving at 2 am while the person  Speed in second case (in km/hr)
starting from Y starts moving at 4 am. And
Distance 2x
the difference of time = (4 am – 2 am)= 2 hours =  ..... (2)
 Dist ance (XP) t r avelled by t he person Time Y6
In both the cases, the speed remains the same
D 
starting from X =   t  km 2x 2x
3   
and the distance (YP) travelled by the person Y  6 50  15

K
or, Y × 6 = 50 × 15
D 
starting from Y =  (t  2) km 50  15
5  or, Y = = 125 days
6
Total distance travelled by both before meeting
= Distance travelled by person from X + 15. Total distance travelled = 3990 km
Distance travelled by person from Y Ratio of time spent in travelling by air, water
and land = 1 : 16 : 2
D  D  Ratio of respective speeds = 20 : 1 : 3
=   t     (t  2)  D
3  5  From the given fact, the ratio of respective
distances will be 20 : 16 : 6 = 10 : 8 : 3
 t t 2 Sum of the ratios = 10 + 8 + 3 = 21
or, D   D
3 5  Distance travelled by steamer will be
8

KUNDAN
t t 2
or,  1 = 3990  = 1520 km
3 5 21
16. Let the distance between station A and station
5t  3t  6 B be d km.
or, 1
15 Again, let the initial speed of the goods train
or, 8t – 6 = 15 be x km/hr.
or, 8t = 15 + 6 = 21 As the accident takes place after 1 hour
 distance covered in 1 hour by the goods
21 5 train = x km
or, t = 2 hours
8 8 Remaining distance = (d – x) km
Conv ert ing t his in hour s, minut es and Total time taken, if no accident happened
seconds, we get 2 hours 37 minutes and 30
d 
seconds. =   hours
x 
5 5  Case I:
[2 hours = 2 hours +   60  minutes
8 8  Time taken by the goods train to cover the
distance
 75  1
= 2 hours +    37 minutes
 2  2 30 d  x
= 1 
60 4x
1 5
= 2 hours + 37 minutes + minutes
2
 1 5(d  x ) 
= 1   hours
1   2 4x 
= 2 hours + 37 minutes +   60   Now, according to the question,
2 
30 seconds 1 5(d  x ) d
1   2
= 2 hours 37 minutes and 30 seconds] 2 4x x
474 Concept of Arithmetic

17. Let the distance be D km and speed be the x


(d  x ) 5 d 1
or,   km/hr
4x x 2 From the question, we have
(d  x ) 5 d 1 50 (D  50) 4 D 25 D 5
      
or, x 3x x 60 x 12
4x x 2
150  4D  200 12D  5x
5d  5x  4d 1 or, 
or,  3x 12x
4x 2
4D  50 12D  5 x
d  5x 1 or, 
or,  3x 12x
4x 2 or, 16D  200  12D  5x
or, 2d – 10x = 4x  4D – 5x = 200 ... (i) and
or, 2d = 14x
or, d = 7x ....(i) 50  24 (D  26) 4 D 35
  
Case II: x 3x x 60
If the goods train had covered 80 km more D 7 12D  7x
before the accident, then the distance of site   
x 12 12x
of the accident = (x + 80) km
Remaining distance = [d – (x + 80)] km 26 4D  104 12D  7 x

K
or,  
Time taken to cover the whole of the distance x 3x 12x

  78  4D  104 12D  7x
 (x  80 )  or, 
 30 d  (x  80 )  hours 3x 12x
=    
 x  60 4x  4D  26 12D  7x
  or, 
 5  3x 12x
 According to the question, or, 4D  7x  104 .... (ii)
x  80 30 d  (x  80 ) d Now, subtracting equation (ii) from equation
   1 (i), we have
x 60 4x x
5 2x = 96
 x = 48 km/hr
80 1 5[d  (x  80] d Put the value of x in equation (i) and find the

KUNDAN
or, 1     1 distance (D)
x 2 4x x
or, 4D – 5 × 48 = 200
80 1 5[d  (x  80 )] d or, 4D = 200 + 240 = 440
or,   
x 2 4x x 440
 D = = 110 km.
320  5d  5x  400 1 d 4
or,   18. Time for work per day in first condition
4x 2 x
= (24 – 9 =) 15 hours
5d  5x  80 1 d Time for work per day in second condition
or,  
4x 2 x = (24 – 9 × 2 =) 6 hours
Her e we hav e four quant it ies : Speed,
1 4d  5d  5x  80 Dist ance, Wor k and Days. W e hav e t o
or, 
2 4x calculate number of days. Hence, Days will
or, 2x = 5x – d + 80 be in t he last column. Here f ollowing
Putting the value of d from equation (i), we relationships exist:
have More speed, less days (Inverse)
2x = 5x – 7x + 80 More distance, more days (Direct)
or, 4x = 80 Less hours of work, more days (Inverse)

80
 x = = 20
4
Hence original speed of the train Hence,
= 20 km/hr. 2 : 1 
Distance between the stations A and B 1 : 2  :: 40 : x
= d = 7x (From i) 6 : 15
= (7 × 20) = 140 km. or, 2 × 1 × 6 : 1 × 2 × 15 :: 40 : x
(Compounding the ratios)
Speed, Time and Distance 475

or, 2 × 1 × 6 × x = 1 × 2 × 15 × 40
13
(Product of extreme terms = Distance covered by train A in hours
Product of mean terms) 3

1  2  15  40 13 13x
 x = = 100 = AP = x × =
2 1 6 3 3
Hence the required time = 100 days. Both the trains meet 8 hours after train A
19. Let A starts from point X, B starts from point leaves P. Now if they meet at C1 then
Y and they meet after t hours. PC1 = 8 × x = 8x km
A B BC1 = 8 × y = 8y km
X Y According to the question,
P
13x
 XP = 4 × t = 4t km 8x + 8y = 650 
YP = 2 + 2.5 + 3 + .... t terms 3
This is an AP. 13x
or, 8x  y   650 
n 3
Sum of an AP = 2a  n  1d 
2
13x
where n = number of terms, a = first term or, 8  65  650 
3
and d = common difference

K
t  1 t  t 1 13x
2  2  (t  1)    4    or,  650  520  130
 YP =
2  2 2  2 2 3

130  3
t2 t 7t t 2 7t  t 2 or, x =
13
= 30 km/hr
= 2t     
4 4 4 4 4
Speed of train A = 30 km/hr
But it is given that XY = 72 or XP + PY = 72 Speed of train B = (65 – 30) km/hr
= 35 km/hr
7t  t 2
 + 4t = 72 21. As given, speed of the train P = 25 km/hr
4 Speed of the train Q = 20 km/hr
2
or, 7t + t + 16t = 288 Speed of the train R = 30 km/hr
or, t2 + 23 t - 288 = 0

KUNDAN
or, t2 + 32 t – 9t - 288 = 0 Q 20 t
or, t (t + 32) – 9 (t + 32) = 0 P1
1
P B A Q1 R R
or, (t + 32) )(t – 9) = 0
 t + 32 = 0 X 33 km 25 t 30 t Y
or, t = –32 (Not possible) 87.5 km
 t – 9 = 0
or, t = 9 Distance travelled by train P between 8:00 to
They meet after 9 hours. 11: 30
2 0 . Let the trains A and B travel at speed of x and 1 7 175
y km/hr respectively and meets 10 hours after ie in 3 hours =  25   87.5 km
2 2 2
departure.
Distance travelled by train Q between 9 : 51
to 11 : 30 ie. in
From the figure it can be seen that 39 33
AC = (x × 10) km 1 hour 39 minutes = 1  20   20
60 20
BC = (y × 10) km = 33 km
 AC + BC = x × 10 + y × 10 Assume that trains P and Q are at A and B
or, 650 = 10(x + y) respectively at 11 : 30 am. Also assume that t
or, x + y = 65 minut es aft er 11 : 30 am, t r ain P was
In the second situation when the other train equidistant from train Q and train R. At the
starts after 4 hours and 20 minutes equidistant position train P, Q and R were at
P1, Q1 and R1.
 XP1 = XA + AP1 = (87.5 + 25 t) km
4 hours and 20 minutes XQ1 = XB + BQ1 = (33 + 20 t) km
20 1 13 P1 Q1 = XP1 – XQ1 = (87.5 + 25 t) - (33 + 20 t)
= 4 4  hours = (54.5 + 5 t) km
60 3 3
Distance RR1 = 30 t km
476 Concept of Arithmetic

P1R1 = Total distance - XP1 - RR1 Let the speed of train from P = x km/hr and
= 220 - (87.5 + 25 t) - 30 t that from Q = (x + 8) km/hr
= (132.5 - 55 t) km Both trains meet after 6 hours
 P1Q1 = P1R 1  (x × 6) + (x + 8) × 6 = 336
 5t + 54.5 = 132.5 - 55 t or, 6x + 6x + 48 = 336
78 or, 12x = 336 – 48 = 288
or, 60 t = 78 or t =  60 minutes
60 288
or, t = 78 minutes or, x = = 44
12
So 78 minutes after 11 : 30 am ie at 12 : 48
pm train P will be equidistant from train Q  Speed of one train = 24 km/hr
and R. Speed of the other train = (24 + 8 =)32 km/hr
23. Distance between 2 trees on a 2-km road
78
XP1 = 87.5 + 25 t = 87.5 + 25 ×  2  1000 
60
= 87.5 + 32.5 =   = 10 m
 201  1 
XP1 = 120 km
Number of trees planted on both sides of a
 At 120 km away from station X, trains would
50-km road
be at equal distances.
 50  1000  
22. = 2  10
  1 = 10002

K
  
Let R be the meeting point.

KUNDAN
274 Concept of Arithmetic

Chapter-15

Profit and Loss


Cost Price the cost price, the difference between the cost price
(CP) and the selling price (SP) is called loss.
The amount paid to purchase an article or the price Thus, if SP < CP, then
at which an article is made is known as its cost Loss = CP – SP
price. The cost price is abbreviated as CP. For or, CP = SP + Loss
example, if a watch is purchased in Rs 1250, then and SP = CP – Loss
its cost price (CP) will be Rs 1250. For example, if the above cycle is sold for Rs
1450, then the loss will be Rs (1500 – 1450 =)
Effective Cost Price
50.
Usually, a shopkeeper has to bear some additional
expenses such as taxes, labour charges, freight Profit and Loss Percentage
charges and maintenance charges for the goods In order to compare the profit or loss in two or
before they are sold. Such charges are called more sales, we usually express profit and loss as a
overhead charges. The overhead charges become per cent of the cost price.
a part of the cost price. Thus, the effective cost For example, a shopkeeper buys an article for
price of the goods is equal to the sum of the actual

K KUNDAN
Rs 500 and sells it for Rs 550 and purchases
payment made while purchasing the goods and another for Rs 100 and sells it for Rs 150, the
overhead charges. profits on each of them is Rs 50 but in the first
Effective Cost Price = Payment made while case Rs 50 is profit on Rs 500 and in the second
pur chasing t he goods case Rs 50 is profit on Rs 100. To compare we
+ overhead charges. calculate profit per cent. Now, in the first case we
For example, if a person purchases a car for Rs
400000, he has to pay 4% tax on it and the cost of  50 
have profit per cent =  100   10% and in
transportation is Rs 1000, then the effective cost  500 
price of the car
= 400000 + (4% of 400000 + 1000) = Rs 417000  50 
the second case profit per cent  100   50%.
 100 
Selling Price
Thus profit per cent in the second case is more.
The price at which an article is sold is known as The profit per cent is the profit that would be
its selling price. The selling price is abbreviated obtained for a CP of Rs 100. Similarly, the loss per
as SP. cent is the loss that would be made for a CP of Rs
For example, if a cow is sold for Rs 1200, then 100. Thus, we have
its selling price (SP) will be Rs 1200.
Profit
Profit per cent = Cost Price (CP)  100 and
Profit
If the Selling Price (SP) of an article is greater Loss
than Cost Price (CP), the difference between the Loss per cent = Cost Price (CP)  100
selling price and the cost price is called profit.
Thus, if SP > CP, then For example, if cost price of an article is Rs
Profit = SP – CP 1000 and the selling price of that article is Rs
or, SP = CP + Profit 1100, then the profit per cent will be
and CP = SP – Profit
100
For example, if a cycle is purchased for Rs 1500  100 = 10%
and sold for Rs 1600, then the profit will be Rs 1000
(1600 – 1500 =) 100. Similarly, if the article is sold for Rs 950, then
loss per cent will be
Loss
50
If the selling price (SP) of an article is less than  100 = 5%
1000
Profit and Loss 275

Important Point (100 +100) = Rs 200


 loss on both the horses
Profit or loss of an article is always calculated on
cost price of that article. Hence, to find out profit  20000  200
or loss of any article, selling price and cost price =   200  = Rs
 99  99
of that article must be known.
For example, if two horses are sold for Rs 100 and the loss per cent
each and if the first is sold at 10% gain and the Loss
second at 10% loss, then it never means that there =  100
is neither profit nor loss on both t he horses. Cost Price (CP)
Because, without knowing the cost price of both
the horses we can not come to any conclusion. We 200
can find the cost price of both the horses: 99  100
Gain on first horse = 10% = = 1%
20000
ie if the cost price of the first horse is Rs 100
99
then selling price of that horse will be Rs (100
+ 10) = Rs 110. Hence, there is a loss of 1% on selling both
 When selling price is Rs 110 then cost the horses.
price is Rs 100.
 When selling price is Re 1 then cost price
Rule of Fraction
In this chapter, the use of “Rule of Fraction” is
100
is Re . dominant. We should understand this rule very
110 well because it is going to be used in almost all the
 When selling price is Rs 100 then cost price questions.
100  100 1000 If our required value is greater than the supplied
is Rs = Rs . value we should multiply the supplied value with a
110 11
fraction which is more-than-one. And if our required
Loss on second horse = 10% value is less than the supplied value we should multiply

K KUNDAN
ie if the cost price of the second horse is Rs the supplied value with a fraction which is less-than-
100, then selling price of the horse is Rs 100 one.
– Rs 10 = Rs 90. (a) If there is a gain of x% , the calculating
 When selling price is Rs 90 then cost price figures would be 100 and (100 + x).
is Rs 100. (b) If there is a loss of y% , the calculating
 When selling price is Re 1 then cost price figures would be 100 and (100 - y).
100 (c) If t he r equired v alue is mor e than t he
is Rs . supplied value, our multiplying fraction
90
 When selling price is Rs 100 then cost price 100  x 100
should be or
100 100  y (both are
100  100 1000
is Rs = Rs . greater than 1).
90 9
(d) If t he r equir ed v laue is less than t he
 Cost price of both the horses supplied value, our multiplying fractions
 1000 1000  20000 100 100  y
= Rs    = Rs should be or (both are
 11 9  99 100  x 100
and the selling price of both the horses = Rs less than 1).

Solved Examples

Ex. 1: A box of Al phanso m angoes was Since CP > SP. Therefore, there is loss
purchased by a fruit-seller for Rs 300. given by
However, he had to sell them for Rs Loss = CP – SP
255 because they began to get over ripe. or, Loss = Rs (300 – 255) = Rs 45
What was the loss percentage?
Soln: We have,  Loss 
Now, loss per cent =   100 %
CP = Rs 300 and SP = Rs 255.  CP 
276 Concept of Arithmetic

Soln: We have the LCM of 4 and 5 = 20


 45 
or, Loss per cent =   100 % =15% Hence let the number of balls purchased
 300  be 20.
Hence, there was 15% loss.
3
Ex. 2: Subramaniam bought 100 eggs for Rs  Cost price of balls =  20 = Rs 15
50. Out of these, 4 eggs were found to 4
be broken and he sold the remaining
eggs at the rate of Rs 8.50 per dozen. 4
and Selling price of balls =  20
Find his gain or loss per cent. 5
Soln: We have, CP of 100 eggs = Rs 50. = Rs 16
It is given that 4 eggs were found to be  Gain = Rs 16 – Rs 15 = Re 1
broken. So, the number of remaining eggs  Gain per cent
which were sold in the market
= 100 – 4 = 96. 1 20 2
=  100 = = 6 %
It is given that the remaining eggs were 15 3 3
sold at the rate of Rs 8.50 per dozen. Again,
Therefore, SP of 12 eggs = Rs 8.50  If gain is Re 1, then the number of
8.50 balls = 20
 SP of 1 egg = Re  If gain is Rs 16, then the number of
12
balls = 16 × 20 = 320
 8.50  Ex. 5: A shopkeeper buys some pens at 3 for
Hence, SP of 96 eggs = Rs   96 
 12  Rs 10 and twice the quantity at 4 for
Rs 13. If he sells all the pens at the
= Rs 68.
rate of a dozen for Rs 59, then find his
Clearly, SP > CP
gain or loss per cent.
So, there is gain given by
Soln: Let the shopkeeper buy 1 dozen (12) pens
Gain = SP – CP = Rs 68 - Rs 50 = Rs 18.

K KUNDAN
at the first rate.
Hence, gain per cent
 Cost price of 3 pens is Rs 10
 Gain   18 
=   100 % =   100 % = 36% 10  12
 CP   50   Cost price of 12 pens is = Rs 40
3
Ex. 3: A grocer buys eggs at 10 for Rs 8 and
At the second rate shopkeeper buys 2
sells at 8 for Rs 10. Find his gain or
dozen of pens.
loss per cent.
Soln: We have, LCM of 10 and 8 = 40. So, let  Cost price of 4 pens is Rs 13.
 Cost price of 24 pens is
the number of eggs bought be 40.
Now, CP of 10 eggs = Rs 8.  13  24 
   Rs 78
 8   4 
 CP of 40 eggs = Rs   40  = Rs 32.
 10   Cost price of 3 dozen (36) of pens
= Rs (40 + 78) = Rs 118
Again, SP of 8 eggs = Rs 10
Selling price of 3 dozen (36) of pens
 10  = 3 × 59 = Rs 177
 SP of 40 eggs = Rs   40  = Rs 50.
 8   profit = Rs (177 – 118) = Rs 59
Now, CP = Rs 32 and SP = Rs 50. Profit
Clearly, SP > CP  per cent profit =  100
Cost Price
So, there is gain given by
Gain = SP – CP = Rs 50 - Rs 32 = Rs 18. 59
=  100 = 50%
Hence, Gain per cent 118
 Gain   18  Ex. 6: The selling price of 10 articles is the
=   100 % =   100 % same as the cost price of 11 articles,
 CP   32 
find the gain per cent.
 225  1 Soln: Let t he cost pr ice of each ar ticle be
=  % = 56 % Rs x.
 4  4
Then, CP of 10 articles = Rs 10x
Ex. 4: A shopkeeper buys some balls at 4 for SP of 10 articles = CP of 11 articles
Rs 3 and sell at 5 for Rs 4. As such he = Rs 11x
gains Rs 16. Find the number of balls  Gain on the purchase of 11 articles
purchased by shopkeeper and also find = Rs 11x – 10x = Rs x
the gain percentage.
Profit and Loss 277

Hence, Gain per cent  Selling price of goods of Rs 120 is Rs


 Gain  120  120
=   100 % = Rs 144
 CP  100
 Profit = Rs 144 – Rs 100 = Rs 44
 x 
=   100 % = 10% 44
 10 x   Profit per cent =  100 = 44%
Ex. 7: By selling 200 oranges, there is a profit 100
equal to the selling price of 40 oranges. Ex. 10: A tradesman by means of false balance
Find gain per cent. defrauds to the extent of 20% in buying
Soln: Let the selling price of one orange be Re goods and also defrauds to the extent
1. of 20% in selling. Find the gain per cent.
 Selling price of 200 oranges = Rs 200 Soln: In normal case, suppose the tradesman
Profit = Selling price of 40 oranges = Rs 40 buy x kg of goods in Rs x.
 Cost price of 200 oranges By using false balance tradesman buys,
= Rs (200 – 40) = Rs 160 according to the question,
Gain (x + 20% of x) kg of goods in Rs x.
 Gain per cent =  100  Cost price per kg
CP
x 1
40 = Rs x  20 % of x = Re
=  100 = 25% 1 .2
160
Ex. 8: A dishonest dealer professes to sell his Again, from the question, by using false
goods at cost pr i ce, but he uses a balance tradesman sells (x – 20% of x) kg
weight of 960 gm for the kg weight. of goods in Rs x.
Find his gain per cent.  Selling price per kg
Soln: Suppose cost price of 1 kg (1000 gm) of x 1
goods is Rs 100. = Rs x  20% of x = Re
0.8
 Cost price of 960 gm of goods

K KUNDAN =
100
1000
 960 = Rs 96
Selling price of 960 gm of goods = Cost
price of 1000 gm of goods = Rs 100
[Since dishonest dealer tells customer that
he uses weight of 1 kg and takes Rs 100
which is equal to the cost price of 1 kg of
goods. But, in reality, he gave only 960
gm of goods to the customer of which cost
 1
Profit per kg = Re  
1 

 30  20 
= Re 
 24 

10
24  100

 0.8 1.2 

 = Re
10
24

Profit per cent = 1 = 50%


price was Rs 96. Thus, he sells goods for 1.2
Rs 100 instead of Rs 96.]
 profit = Rs (100 – 96) = Rs 4 Note: You must under st and t he diff er ence
between Ex. 9 and Ex. 10.
4 1 In Ex . 9 t radesman def rauds in t he
 profit per cent =  100 = 4 %
96 6 purchasing amount ie price while in Ex.
Ex. 9: A tr adesm an defr auds 20% whi l e 10 tradesman defrauds in quantit y of
purchasing and selling goods. Find the goods purchased or sold.
gain per cent. Ex. 11: A bookseller sold 300 copies of a book
Soln: Suppose that the tradesman purchases at a profit of 15%. If a book costs him
goods for Rs 100. Rs 12, find the sell ing price of the
Since he def r auds while purchasing books.
goods, hence he purchases goods of Rs Soln: We have, cost of one copy of the book
120 instead of Rs 100. Again, he defrauds = Rs 12
20% while selling the goods. Threfore,  Cost of 300 copies of the book
he takes Rs 120 from the customer instead = Rs (300 × 12) = Rs 3600
of Rs 100. It is given that the bookseller sold 300
 Cost price of goods = Rs 100 and the copies of the book at a profit of 15%
cost pr ice of pur chased goods Therefore, Profit = 15% of CP
= Rs 120. = 15% of Rs 3600
 Selling price of goods of Rs 100 is Rs  15 
120. = Rs   3600  = Rs 540
 100 
278 Concept of Arithmetic

Now, SP = CP + Profit Soln: For the first article, we have


or, SP = Rs 3600 + Rs 540 SP = Rs 375 and Gain = 25%
or, SP = Rs 4140. 100
Ex. 12: Krishnamurthi bought oranges at Rs  CP =  SP
100  Gain %
5 a dozen. He had to sell them at a (By Rule of Fraction)
loss of 4%. Find the selling price of
one orange.  100 
or, CP = Rs   375  = Rs 300
Soln: We have, CP of one dozen oranges = Rs 5.  100  25 
Loss per cent = 4% For the second article, we have,
 4  1 SP = Rs 375 and Loss = 25%
 Loss = 4% of Rs 5 = Re   5  = Re  
 100  5 100
 CP =  SP
Now, SP = CP – Loss 100  Loss %
 1 24  100 
or, SP = Rs  5   = Rs or, CP = Rs   375  = Rs 500
 5 5  100  25 
24  Total cost paid in buying the two articles
Thus, SP of one dozen oranges = Rs = Rs (300 + 500) = Rs 800.
5
Total SP = Rs 375 + Rs 375 = Rs 750.
 24 1  2  CP > SP. So, there is a loss
 SP of one orange = Re    = Re  Total loss = CP - SP
 5 12  5
= Rs 800 – Rs 750 = Rs 50
2
=  100 paise = 40 paise  Loss 
5 Hence, loss per cent =   100 %
 CP 
Ex. 13: A man sells his scooter for Rs 18000
making a profit of 20%. How much did  50  1
 100 % = 6 %

K KUNDAN
the scooter cost him? = 
 800  4
Soln: Let the cost price of the scooter be Rs 100.
Ex. 16: Shridhar bought two buffaloes for Rs
Th en ,
30000. By selling one at a loss of 15%
Profit = Rs 20 [  Profit = 20% (given)]
and other at a gain of 19%, he found
 SP = CP + Profit = Rs 100 + Rs 20 that selling price of both buffaloes is
= Rs 120 the same. Find the cost price of each.
Now, if the SP is Rs 120, then CP Soln: Let t he cost pr ice of one buf falo be
= Rs 100 Rs x. Then, cost pr ice of ot her is
If the SP is Rs 18000, then Rs (30000 – x)
 100  100  loss%
CP = Rs  18000  = Rs 15000  CP
 120  SP of first buffalo =
100
Hence, the cost of the scooter = Rs 15000.
Ex. 14: By selling an A.C. for Rs 18000, I lose 100  15 85x
= x =
20%. What did I pay for it? 100 100
Soln: We have,
SP = Rs 18000, and Loss = 20%. 100  gain%
SP of second buffalo =  CP
Let the CP of A.C. be Rs 100. 100
Then, Loss = CP – SP  100  19 
or, SP = CP – Loss =   (30000  x )
 100 
= Rs 100 – Rs 20 = Rs 80.
Now, if the SP is Rs 80, then CP = Rs 100 119
=  (30000  x )
If the SP is 18000, then CP 100
100 Now, it is given that the selling price of
= Rs  18000 = Rs 22500. both buffaloes is the same.
80
Ex. 15: A man sold two articles at Rs 375 each. 85x 119
  (30000  x )
On one he gains 25% and on the other 100 100
he loses 25%. How much does he gain
or, 5x = 7(30000 – x)
or lose in the whole transaction? Also
or, 12x = 210000
find his gain or loss per cent in the
whole transaction. 210000
 x = = 17500
12
Profit and Loss 279

Hence, the cost price of two buffaloes are  Cost price of the third motor-cycle
Rs 17500 and Rs (30000 - 17500 =) Rs = Rs (20000 – 5000 – 6000)
12500. = Rs 9000
Ex. 17: Raman bought two old scooters for Rs Ex. 19: Three items are purchased at Rs 450
18000. By selling one at a profit of 25% each. One of them is sold at a loss of
and the other at a loss of 20%, he 10%. At what price should the other
neither gains nor loses. Find the cost two be sold so as to gain 20% on the
price of each scooter. whole transaction? What is the gain per
Soln: Let the cost price of one scooter be Rs x. cent on these two items?
Then, the cost price of the second scooter Soln: We have, CP of one item = Rs 450.
= Rs (18000 – x)  CP of three items = Rs (3 × 450)
25x = Rs 1350.
Profit on the sale of first scooter = Gain on the whole transaction = 20%
100
Loss on the sale of second scooter
= 20% of Rs 1350
20
= (18000  x )  20 
100 = Rs  1350  = Rs 270.
 100 
It is given that there is no gain or loss to
Raman.  SP of three items = CP + Gain
 Profit on first scooter = Loss in second = Rs 1350 + Rs 270 = Rs 1620
scooter. It is given that first item is sold at a loss
of 10% . Therefore,
25x 20 Loss on selling first item
  (18000  x )
100 100 = 10% of Rs 450
or, 5x = 4(18000 – x)
 10 
or, 9x = 72000 = Rs   450  = Rs 45
 100 
 x = Rs 8000
 SP of first item = CP – Loss

K KUNDAN
Hence, the cost price of two scooters was
Rs 8000 and Rs 10000 respectively. = Rs 450 – Rs 45 = Rs 405
Ex. 18: A person sold three motor-cycles for Rs SP of three items is Rs 1620 and the SP
6000, Rs 7500 and Rs 5500 of first item is Rs 405.
respectively. By selling first he gains  SP of the remaining two items
20% and by selling second he gains = Rs 1620 – Rs 405 = Rs 1215.
25%. But in selling all the three he loses Thus, the other two items should be sold
5%. Find the cost price of third motor- at Rs 1215.
cycle. CP of the remaining two items
Soln: Cost price of the first motor-cycle = Rs (2 × 450) = Rs 900
 Gain on the remaining two items
100 = SP – CP
=  SP
100  Gain% = Rs 1215 – Rs 900 = Rs 315.
 Gain per cent on the remaining two
100 items
=  6000 = Rs 5000
120
 315 
Cost price of the second motor-cycle =   100 % = 35%
 900 
100
=  SP Ex. 20: If a man were to sell his hand-cart for
100  Gain% Rs 720, he would lose 25%. What must
he sell it for to gain 25%?
100
=  7500 = Rs 6000 Soln: We have, SP of the cart = Rs 720
125 Loss = 25%
Cost price of all the three motor-cycles
 100 
 CP =   SP 
100  100  Loss% 
=  SP
100  Loss%
 100 
100 or, CP = Rs   720 
=  (6000  7500  5500)  100  25 
95
4 
100 = Rs   720  = Rs 960
=  19000 = Rs 20000 3 
95
Thus, CP of the cart = Rs 960.
280 Concept of Arithmetic

Desired gain = 25% And selling price at 15% profit= Rs115


Difference in the selling prices
100  Gain%
 SP =  CP = Rs (115 – 110) = Rs 5
100
 When difference in selling price is Rs
 100  25  5, cost price is Rs 100.
or, SP = Rs   960  When difference in selling price is Rs 40,
 100 
100
 125  cost price is  40 be Rs 800.
= Rs   960  5
 100 
Ex. 24: A person sells an article at a profit of
5  10%. If he had bought it at 10% less
= Rs   960  = Rs 1200.
4  and sold it for Rs 3 more, he would
Ex. 21: If a person sells an article for Rs 360, have gained 25%. Find the cost price.
Soln: Let the actual cost price be Rs 100.
1 Actual seling price at 10% profit = Rs 110
gaining th of its CP. Find gain per
5 Supposed cost price at 10% less = Rs 90
cent. Supposed selling price at 25% gain
Soln: Let the CP of the article be Rs x. 125
= Rs 90  = Rs 112.5
x 100
Then, Gain =
5  the differene in the selling prices
= Rs 112.5 – Rs 110 = Rs 2.5
1 If the difference is Rs 2.5, the CP = Rs 100
[  Gain = th of CP (given)]
5 If the difference is Rs 3, the CP

 Gain  100
Now, Gain% =  100  =  3 = Rs 120
 CP  2. 5

K KUNDAN
Ex. 25: A shopkeeper sells a watch at a loss
x  of 5%. If he had bought it at 10% less
  and sold it for Rs 140 more, he would
 5 100 
or, Gain% = x %
  1
  have gained th of the cost price.
4
1  What is the cost price of the watch?
=  100  % = 20% Soln: Suppose cost price of the watch = Rs 100
5 
Selling price of the watch at 5% loss
Ex. 22: If a person sells an article for Rs 600, = (100 – 5) = Rs 95
1 Cost price at 10% less = Rs 90
gaining th of its selling price. What
5 1
 profit = th of the cost price
4
1
must he sell it for to lose th of its
4 90
= Rs = Rs 22.50
cost price? 4
1 Now, selling price = Rs (90 + 22.50)
Soln: Profit = th of 600 = Rs 120 = Rs 112.50
5
Difference in selling prices
 Cost price = Selling price – Profit = 112.50 – 95
= Rs (600 – 120)= Rs 480 = Rs 17.50
1  If second selling price is Rs 17.50 more,
Loss = th of 480 = Rs 120 then cost price = Rs 100
4
 If second selling price is Rs 140 more,
 Selling price = Cost price – Loss
= Rs (480 – 120)= Rs 360 100  140
Ex. 23: A cow is sold at a profit of 10%. Had then cost price = = Rs 800
17.50
it been sold for Rs 40 more, then profit Ex. 26: A man sold his house for Rs 400 at
would have been 15%. Find the cost some loss. If he had sold it for Rs 500,
price of cow.
Soln: Let the cost price of cow be Rs 100. 2
 Selling price of the cow at 10% profit then he would have gained rd of the
3
= Rs 110
Profit and Loss 281

first loss. Find the cost price of the  Cost price of 150 quintals of rice
house. = 150 × 300 = Rs 45000
Soln: Suppose cost price of the house is  Selling price of it at 8% profit
Rs x.
By selling it for Rs 400, loss 108
=  45000 = Rs 48600
= Rs (x – 400) and 100
by selling it for Rs 500 gain = Rs (500 – x)  Selling price of the remaining 30
Now, according to the question, quintals of rice
= Rs (48600 – 29700 – 8550)
2
500 – x = (x – 400) = Rs 10350
3  Selling price of the remaining 1 quintal
or, 3(500 – x) = 2(x – 400) of rice
or, 1500 – 3x = 2x – 800
or, 5x = 2300 10350
= = Rs 345
30
2300
 x = = Rs 460 Ex. 29: A man bought 300 quintals of sugar.
5 He sold 200 quintals of sugar at the
Ex. 27: A shopkeeper sold his TV set for Rs rate of Rs 126 per quintal and he gained
4000 at some profit. If he had sold it 5% on it. At what rate should he sell
for Rs 3200, then he would have lost the remaining to gain 10% of the whole?
3 Soln:  Selling price of 200 quintals of rice
th of the first gain. Find the cost = 200 × 126 = Rs 25200
5  Cost price at 5% profit
price of TV set.
Soln: Suppose cost price of t he TV set is 100
=  25200 = Rs 24000
Rs x. 105
 By selling it for Rs 4000, profit  Cost price of 300 quintals of rice
= Rs (4000 – x) and

K KUNDAN
24000  3
By selling it for Rs 3200, loss = = Rs 36000
= Rs (x – 3200) 2
Now, according to the question,  Selling price at 10% profit
3 110
(x – 3200) = (4000 – x) =  36000 = Rs 396000
5 100
or, 5(x – 3200) = 3(4000 – x)  Selling price of t he r emaining
or, 5x – 16000 = 12000 – 3x (300 – 200 =) 100 quintals of sugar
or, 8x = 12000 + 16000 = 28000 = Rs (396000 – 25200) = Rs 14400
 Selling price of the remaining sugar per
28000
 x = = Rs 3500 14400
8
quintal = Rs = Rs 144.
 Cost price of the TV set = Rs 3500 100
Ex. 28: A merchant bought 150 quintals of rice
1
at a rate of Rs 300 per quintal. 90 Ex. 30: Ramu bought 150 quintals of rice.
quintals of which he sold at a gain of 4
10% and 30 quintals at a loss of 5%. of which he sold at a loss of 10%. At
At what price per kg should he sell the what per cent gain should he sell the
remaining rice to gain 8% of his total remaining to gain 10% of the whole?
cost price? Soln: Suppose selling price of 1 quintal of rice
Soln:  Cost price of 90 quintals of rice is Rs 100.
= 90 × 300 = Rs 27000 1 75
 Selling price of it at 10% profit of 150 quintals of rice = quintals
4 2
 100  10  of rice
= 27000   = Rs 29700
 100  75
 Cost price of 30 quintals of rice  Cost price of quintals of rice
2
= 30 × 300 = Rs 9000
 Selling price of it at 5% loss 75
=  100 = Rs 3750
95 2
=  9000 = Rs 8550
100
282 Concept of Arithmetic

 Selling price of it at 10% loss Now, selling price of 1 kg of mixture


= Rs 12
90
=  3750 = Rs 3375  Cost price of it at 20% profit
100
100
 Cost price of 150 quintals of rice =  12 = Rs 10
= 150 × 100 = Rs 15000 120
 Selling price of it at 10% gain Hence,
14 – 6x = 10
110 or, 6x = 4
=  15000 = Rs 16500
100
4 2
 Selling price of t he r emaining x= 
6 3
 75  225  Ratio of both kind of tea
150   quintals of rice
 2  2
2  2
= Rs (16500 – 3375) = Rs 13125 = x : (1 – x) = : 1  
3  3
225
and cost price of the r emaining 2 1
2 = : = 2 : 1
quintals of rice 3 3
= Rs (15000 – 3750) = Rs 11250 Ex. 33: A man buys 5 horses and 13 cows for
 Profit = Rs (13125 – 11250) = Rs 1875 Rs 10000. He sell s the hor ses at a
profit of 15% and cows at a loss of
1875 2 10%. If his total gain is Rs 375 then
 Profit per cent =  100 = 16 %
11250 3 find what price does he pay for a horse
Ex. 31: A vendor purchased 40 dozen bananas and a cow?
for Rs 250. Out of these, 30 bananas Soln: Let the price of a horse be Rs x and the
were rotten and could not be sold. At price of a cow be Rs y.

K KUNDAN
what rate per dozen should he sell the  5x + 3y = 10000 .... (i)
remaining bananas to make a profit Now, profit on 5 horses
of 20%?
3x
Soln: We have, CP of bananas = Rs 250 = 15% of Rs 5x = Rs and
Gain required = 20% 4
 Gain = 20% of CP = 20% of Rs 250 13y
loss on 13 cows = 10% of 13y = Rs
 20  10
= Rs   250  = Rs 50
 100 
3x 13y
Now, SP = CP + Gain  Total profit =  = 375
4 10
or, SP = Rs 250 + Rs 50
or, SP = Rs 300 or, 15x – 26y = 7500 .... (ii)
Number of good bananas Now, equation (i) × 2 + equation (ii), we
= (400 × 12 - 30) = 450. have,
Now, SP of 450 bananas = Rs 300 25x = 27500
 x = 1100
 300  Now, putting the value of x in equation
 SP of 12 bananas = Rs  12  = Rs 8
 450  (i), we have
Hence, t he v endor should sell t he 5500 + 13y = 10000
remaining bananas at Rs 8 per dozen to  13y = 10000 – 5500 = 4500
make a profit of 20%. 4500 2
Ex. 32: A tradesman buys a kind of tea at Rs y = = Rs 346
13 13
4 per kg and he mixes it with another
kind of tea at Rs 14 per kg. In what Hence, cost of one horse and one cow are
proportion must he mix the two kinds 2
of teas so that the mixture is sold at Rs 1100 and Rs 346 respectively.
13
Rs 12 per kg to gain 20%?
Alternative Method:
Soln: Suppose mixture is 1 kg in which quantity
Let the cost of 1 horse be Rs x, then total
of first kind of tea is x kg and the second
selling price
kind is (1 – x) kg.
Cost price of this mixture  115  90
= 8x + 14 (1 – x) = (14 – 6x) = 5x    (10000  5 x )
 100  100
Profit and Loss 283

Now, according to the question, Retailer’s gain = 25%


 Retailer’s SP
 115  90
5x    (10000  5 x ) = 10375
 100  100  100  25 
= Rs   126 .5 
or, 575x + 90 × 10000 – 450x  100 
= 10375 ×100
5 
or, 125x = 137500 = Rs   126 .5  = Rs 158.125
4 
137500
 x = = Rs 1100 Now,
125 If the retailer’s SP is Rs 158.125, the cost
Cost of 13 cows = 10000 – 5 × 1100 of production = Rs 100
If the retailer’s SP is Re 1, the cost of
10000  5500
 Cost of 1 cow = 100
13
production = Re
158.125
4500 2
= = Rs 346 If the retailer’s SP is Rs 1265, the cost of
13 13 production
Hence, cost of 1 horse and 1 cow are Rs
100
2 = Rs  1265 = Rs 800.
1100 and Rs 346 respectively. 158.125
13 Hence, the cost of production = Rs 800.
Ex. 34: Ram purchased a radio set for Rs 300 Ex. 36: Mohan bought 20 dining tables for Rs
and sold it to Shyam at a profit of 25%. 12000 and sold them at a profit equal
Shyam sold it to Mohan at a loss of to the SP of 4 dining tables. Find the
10%. For how much did Mohan buy it? SP of 1 table.
Soln: CP of the radio set for Ram = Rs 300 Soln: Let the SP of 1 table be Re 1.
Gain = 25% of Rs 300 Then, SP of 20 tables = Rs 20.
Profit = SP of 4 tables = Rs 4.
 25

K KUNDAN

= Rs   300  = Rs 75  CP = SP – Profit
 100  = Rs 20 – Rs 4 = Rs 16.
 SP of the radio set for Ram = CP + Gain
= Rs 300 + Rs 75 = Rs 375  Gain 
 Gain% =   100 %
 CP of the radio set for Shyam = SP of  CP 
the radio set for Ram = Rs 375
Since Shyam sold the radio set to Mohan  4 
=  100  = 25%
at a loss of 10% , therefore  16 
Loss = 10% of Rs 375 Now,
CP of 20 dining tables = Rs 12000
 10 
= Rs   375  = Rs 37.50  CP of 1 table = Rs 600
 100  Gain% = 25%
SP of the radio set for Shyam = CP – Loss
= Rs 375 - Rs 37.50 = Rs 337.50  100  Gain% 
 SP =    CP
 CP of the radio set for Mohan = Rs 337.50 100 
Hence, Mohan bought the radio set for Rs or, SP of one table
337.50
Ex. 35: If the manufacturer gains 10%, the  100  25 
= Rs   600  = Rs 750
wholesale dealer 15% and the retail  100 
dealer 25%, then what is the cost of Ex. 37: By selling 144 hens, Kamal lost the SP
product of a water heater, the retail of 6 hens. Find his loss per cent. Had
price of which is Rs 1265? he purchased them for Rs 7200, what
Soln: Let the cost of production be Rs 100. would have been the SP of one hen?
We have, manufacturer’s gain = 10% Soln: Let the SP of one hen be Re 1. Then,
 Manufacturer’s SP SP of 6 hens = Rs 6.
 100  10   Loss = SP of 6 hens = Rs 6
= Rs   100  = Rs 110.
 100  SP of 144 hens = Rs 144
Wholesaler’s gain = 15%  CP of 144 hens = SP + Loss
 Wholesaler’s SP = Rs 144 + Rs 6
= Rs 150
 100  15 
= Rs   110  = Rs 126.5
 100 
284 Concept of Arithmetic

Loss 20x
Hence, Loss% =  100 or, CP = ..... (i)
CP 21
When SP is reduced by Rs 50, there is
6
=  100 = 4% loss of 5% . Therefore,
150
If CP = Rs 7200, then 100
CP =  (x  50)
Loss = 4% of Rs 7200 100  5

 4   100 
= Rs   7200  = Rs 288. Using : CP  100 - Loss%  SP 
 100   
 SP = CP – Loss = Rs (7200 - 288)
20
= Rs 6912. or, CP = (x  50 ) .... (ii)
19
 6912  From (i) and (ii), we get
So, SP of one hen = Rs   = Rs 48.
 144 
20x 20
Ex. 38: Some lollipops are bought at 11 for a  (x  50)
rupee and the same number at 9 a rupee. 21 19
If the whole lot is sold at 10 a rupee, x 1
find the gain or loss per cent. or,  (x  50)
21 19
Soln: We have,
LCM of 9, 10 and 11 = 9 × 10 × 11 = 990. [Dividing both sides by 20]
So, let us assume that 990 lollipops of or, 19x = 21(x – 50)
each kind are bought. [Using cross multiplication]
Now, CP of 990 lollipops of first kind or, 19x = 21x – 1050
or, 21x – 19x = 1050
1  or, 2x = 1050
= Rs   990  = Rs 90.

K KUNDAN
 11 
1050
CP of 990 lollipops of second kind or, x = = 525.
2
1  Hence, the original SP of the article is Rs
= Rs   990  = Rs 110.
9  525.
Total CP of 1980 lollipops of both kinds Ex. 40: By selling 90 ball-pens for Rs 160 a
= Rs 90 + Rs 110 = Rs 200. person loses 20%. How many ball-pens
SP of 10 lollipops = Re 1. should be sold for Rs 96 so as to have
 SP of these 1980 lollipops a profit of 20%?
Soln: We have,
 1  SP of 90 ball-pens = Rs 160
= Rs  1980  = Rs 198.
 10  Loss per cent = 20
Clearly, SP < CP. So, there is a loss.  100 
 CP = Rs    SP
Loss = CP - SP = Rs 200 - Rs 198 = Rs 2.  100  Loss% 
Loss 2  100 
 Loss% =  100 =  100 = 1% = Rs    160 = Rs 200
CP 200  100  20 
Hence, ther e is 1% loss in the whole Thus, CP of 90 ball-pens = Rs 200
transaction.  CP of a ball-pen
Ex. 39: By reducing the selli ng pri ce of an
article by Rs 50, a gain of 5% turns  200  20
= Rs   = Rs
into a loss of 5%. Find the original  90  9
selling price of the article. Suppose x ball-pens are sold to earn a
Soln: Let the original selling price of the article profit of 20%
be Rs x. It is given that on this SP there
is a gain of 5%. Therefore, 20x
CP of x ball-pens = Rs
9
 100 
CP =  x SP of x ball-pens = Rs 96
 100  5 
 20 x 
 100   Profit = Rs  96  
 9 
Using : CP  100  Gain%  SP 
  It is given that on selling x ball-pens for
Rs 96, profit earned is 20% .
Profit and Loss 285

Ex. 43: A seller uses 840 gm in place of one kg


20 x to sell his goods. Find his actual per
96 
 9  100 = 20 cent profit or loss
20 x (a) when he sells his article on 4% loss
9 on cost price.
(b) when he sells his article on 4% gain
 20 x  20 x
or,  96   5  on cost price.
 9  9 Soln: Suppose the cost price of 1000 gm is Rs
20 x 4x 100.
or, 96  
9 9 100
 Cost price of 840 gm = (840)
24x 1000
or,  96
9 = Rs 84
For (a), selling price of 840 gm
96  9
 x = = 36 = Rs (100 - 4) = Rs 96
24  Profit = SP - CP = 96 - 84 = Rs 12
Hence, 36 ball-pens should be sold for Rs
96 to earn a profit of 20%. 12  100 100 2
 % profit =   14 %
Ex. 41: Rajeshwar bought 16 dozen ball-point 84 7 7
pens and sold them by and by. Due to a For (b), selling price of 840 gm
calculation mistake in fixing selling = Rs (100 + 4) = Rs 104
price, he lost an amount equal to SP of  Profit = SP - CP = 104 - 84 = Rs 20
4 dozen pens. Find the loss per cent.
20  100 17
Find the SP of one dozen pens, if he  % profit =  23 %
purchased these 16 dozen pens for Rs 84 21
240. Ex. 44: A dealer sold a radio at a loss of 2.5%.
Soln: Let the SP of one dozen pens be Rs x. Had he sol d it for Rs 100 more, he
Then SP of 16 dozen pens = Rs 16x woul d hav e gained 7.5%. For what

K KUNDAN
Loss = SP of 4 dozen pens = Rs 4x value should he sell it in order to gain
 CP = SP + Loss = Rs 16x + Rs 4x
1
= Rs 20x 12 %?
2
 4x 
 Loss per cent =  100  = 20% Soln: Suppose he bought t he r adio f or
 20 x 
Rs x.
Now, CP of 16 dozen pens = Rs 240
Then selling price at 2.5% loss
 CP of one dozen pens = Rs 15
Loss = 20%  100  2.5  97.5x
 Loss on one dozen pens = Rs x   = Rs
 100  100
 20  and selling price at 7.5% gain
= Rs  15  = Rs 3
 100   100  7.5  107.5x
Hence, SP of one dozen pens = CP – Loss = Rs x   = Rs
 100  100
= Rs 15 – Rs 3 = Rs 12.
Now, according to the question,
Ex. 42: A grocer sells rice at a profit of 10%
and uses a weight which is 20% less. 107 .5x 97 .5x
 = Rs 100
Find his total percentage gain. 100 100
Soln: Suppose the grocer bought at Rs x per kg. or, 10x = 100 × 100
 110x  80  x = Rs 1000
Then he sells at Rs   per kg Therefore to gain 12.5% , he should sell
 100  100
for
110x 100
or, at Rs  per kg  100  12 .5 
100 80 Rs 1000   = Rs 1125
 100 
11x
or, at Rs per kg Ex. 45: An article is sold at a profit of 20%. If
8 both the cost price and selling price are
11x Rs 100 less, the profit would be 4%
x more. Find the cost price.
8 300
Now, % profit =  100 = Soln: Suppose the cost price of that article is
x 8 Rs x.
= 37.5%
286 Concept of Arithmetic

 120   225  100 


The selling price = Rs x   or, T   = 1250
 100   36 
 T = 360
New cost price and selling pr ice is  Price of a table = Rs 360
Rs (x - 100) Ex. 47: An article is sold at 20% profit. If its
  120   cost price is increased by Rs 50 and at
and Rs  x    100 respectively. the same time if its selling price is also
  100  
increased by Rs 30, the percentage of
New profit
1
  120   profit decreases by 3 % . Find the cost
3
= Rs  x   100   (x  100)
  100   price.
Soln: Suppose the cost price = Rs x
 120   20 
= Rs  x  1 = Rs x   120
x = Rs
6
x
 100   100  Then SP = Rs
100 5
 New percentage profit Now, new CP = Rs (x + 50)
 20  120 
x  new SP = Rs  x  30 
 100   100  20 x % 100 
=
x  100 x  100 6 
We are also given that the new percentage = Rs  x  30 
5 
of profit = (20 + 4 =) 24%
Now, the new per cent profit
20x
or,  24  1  2 50
x  100 =  20  3   16 %  %
 3  3 3
or, 4x = 2400

K KUNDAN
 x = 600 Thus,
Thus cost of the article = Rs 600  50  6
Ex. 46: A person sells his table at a profit of 100  % of (x + 50) = (x + 30)
 3  5
1 1
12 % and the chair at a loss of 8 %  350  6
2 3 or,  ( x  50 )  x  30
 300  5
but on the whole he gains Rs 25. On
the other hand if he sells the table at 7 175 6
or, x  x  30
1 6 3 5
a loss of 8 % and the chair at a profit
3 6 7 175
or,    x   30
1 5 6 3
of 12 % then he neither gains nor 1 85
2
or, x 
loses. Find the cost price of the table. 30 3
Soln: Suppose the cost price of a table = Rs T  x = Rs 850.
and cost price of a chair = Rs C. Then,
1
1  1  Ex. 48: rd of a commodity is sold at 15%
12 % of T +   8 %  of C = 25 and 3
2  3 
1
1  1 profit, th is sold at 20% profit and
 4
  8 %  of T + 12 % of C = 0
 3  2 the rest at 24% profit. If a total profit
of Rs 62 is earned, then find the value
25 25 of the commodity.
or, T – C = 2500 ...(1)
2 3 Soln: Suppose the value of the commodity was
25 25 x x
T + C = 0 ...(2) Rs x. Then was sold at 15% profit,
3 2 3 4
(1)  2 + (2)  3 gives was sold at 20% pr of it and

25 25   x x   5x
T - T = 1250  x   3  4   12 was sold at 25% profit.
4 9    
Profit and Loss 287

Now, profit B gains Rs 810 (the same as A loses) and


his investment in this transaction is Rs
x  15  x  20  5x  24 
      = Rs 62 8100.
3  100  4  100  12  100 
810
 B’s per cent gain =  100  10%
x x x 8100
or,   = 62
20 20 10 Ex. 51: A fruit merchant makes a profit of 25%
by selling mangoes at a certain price. If
4x
or, = 62 he charges Re 1 more on each mango,
20 he would gain 50%. Find what price
62  20 per mango did he sell at first. Also find
 x = = Rs 310. the cost price per mango.
4 Soln: Suppose the cost price of a mango be Rs
Ex. 49: A person bought some oranges at the x.
rate of 5 per rupee. He bought the same Then, first selling price
number of oranges at the rate of 4 per
rupee. He mixes both the types and sells  100  25  5x
= Rs x   = Rs
at 9 for rupees 2. In this business he  100  4
bears a loss of Rs 3. Find out how many If he charges Re 1 more and gets 50%
oranges he bought in all? profit then there exists a relationship:
Soln: Suppose he bought x oranges of each
quality. 5x  100  50 
1  x 
Then his total investment 4  100 
x x 9x 5x 3x
= = Rs or, 1 
5 4 20 4 2
Total selling price
3x 5 x
2x  2 4x or,  = 1

K KUNDAN
= Rs = Rs 2 4
9 9  x = Rs 4
 total loss  Cost price per mango = Rs 4 and
9x 4x 81x  80 x x  125 
=    first selling price = 4   = Rs 5
20 9 180 180  100 
x Ex. 52: A man sells two horses for Rs 1710.
then Rs = Rs 3 The cost price of the first is equal to
180
the selling price of the second. If the
 x  180  3  540 first is sold at 10% loss and the second
Ther ef or e he bought 2 × 540 = 1080 at 25% gain, what is his total gain or
oranges in total. loss (in rupees)?
Ex. 50: A horse worth Rs 9000 is sold by A to Soln: Let the cost price of the first horse be Rs
B at 10% loss. B sells the horse back x.
to A at 10% gain. Who gains and who  Cost price of the second horse
loses? Find also the values. = Rs (1710 – x)
 First horse is sold at 10% loss.
 90 
Soln: A sells to B for Rs 9000   = Rs 8100  Selling price of the first horse
 100 
 100  10   9 
 110  = x  = Rs  x
 100   10 
Again, B sells to A for Rs 8100  
 100  Again, second horse is sold at 25% gain.
= Rs 8910  Selling price of the second horse
Thus, A loses Rs (8910 - 8100) = Rs 810.
In this whole transaction, A’s investment  100  25 
= (1710  x )  
is only Rs 9000 (the cost of the horse)  100 
because the horse returned to his hand.
 A’s per cent loss  (1710  x ) 5 
= Rs  
 4 
810
=  100  9% Now, according to the question,
9000 Cost price of the first horse is equal to the
selling price of the second horse.
288 Concept of Arithmetic

(1710  x ) 5 1
x= There is a loss of 12 % when selling
4 2
or, 4x = 1710 × 5 – 5x
x
or, 9x = 1710 × 5 price =
2
1710  5
 x = = 950 x 100 
9  
 cost price =
 Cost price of the first horse and the 2  100  12.5 
selling price of the second horse = Rs 950
100 x 4x
Now, selling price of the first horse =
175 7
9 9
= x =  950 = Rs 855 Now, when selling price is Rs x, % profit
10 10
 Total selling price = Rs (950 + 855) 4x
x
= Rs 1805  7  100  7x  4x  100
 Profit = Selling Price – Cost Price 4x 4x
= Rs 1805 – Rs 1710 = Rs 95 7
Ex. 53: Rakesh calculates his profit percentage
on the selling price whereas Ramesh 3
  100  75%
calculates his profit on the cost price. 4
They find that the difference of their Ex. 55: Ramesh sold his goat for Rs 31.25, and
profits is Rs 100. If the selling price of got a percentage of profit equal to the
both of them are the same and both of cost price. Find the cost of the goat.
them get 25% profit, find their selling Soln: Let the cost price of the goat be Rs x.
price.  percentage profit = x%
Soln: Suppose the selling price for both of them  profit = x% of Rs x

K KUNDAN
is Rs x.
Now, cost price of Rakesh x x2
= x
= Rs
 100  25  3 100 100
= x  x
But, Profit = Selling price – Cost price
 100  4
and cost price of Ramesh = (31.25 – x)
Now, according to the question,
 100  4
= x  x
x2
 100  25  5 = 31.25 – x
100
3 x
Rakesh’s profit = x  x  or, x2 = 3125 – 100x
4 4 or, x2 + 100x – 3125 = 0 ....(*)
or, x2 + 125x – 25x – 3125 = 0
4 x
Ramesh’s profit = x  x  or, x(x + 125) – 25(x + 125) = 0
5 5 or, (x – 25) (x + 125) = 0
Now, difference of their profits is given.  x = 25 or –125
x x Now, value of x ie cost price cannot be
  = Rs 100 negative, threfore, value of x cannot be
4 5
–125.
x  x = 25 ie cost price of goat = Rs 25.
or, = 100 Note: (*) Quadratic equation can be solved
20
with the help of formula given below. If a
 x = Rs 2000 quadratic equation is in the form of ax2 +
Thus selling price = Rs 2000. bx + c = 0, then
Ex. 54: What will be the percentage profit after
selling an article at a certain price if  b  b 2  4ac
1 x =
ther e is a loss of 12 % when the 2a
2 In the above equation (*),
article is sold at half of the previous a = 1, b = 100 and c = –3125
selling price?
Soln: Suppose the previous selling price = Rs x
 100  (100 )2  4  1  ( 3125 )
 x =
x 2 1
Now, the later selling price = Rs
2
Profit and Loss 289

or, (x – 40) (x + 240) = 0


=
1
2
 100  10000  12500   x = 40 or –240
 Cost price of watch = Rs 40.
Ex. 58: A man sold a chair and a table together
=
1
2

 100  22500  for Rs 1520 thereby making a profit of
25% on chair and 10% on table. By
1 selling them together for Rs 1535 he
=  100  150 would have made a profit of 10% on
2
the chair and 25% on the table. Find
1 the cost price of each.
=  100  150 or 1  100  150 Soln: Let the cost price of one chair be Rs x and
2 2
that of one table be Rs y.
= 25 or –125 Profit on a chair = 25%
For detail see the chapter “Elementary  selling price of one chair
Algebra”.
Ex. 56: A person sold an article for Rs 9 and 25 125
= x x  x
got a percentage of loss equal to the 100 100
cost price. Find the cost price of the Profit on a table = 10%
article.  selling price of one table
Soln: Let the cost price of the article be Rs x.
 percentage loss = x% 10y 110
= y  y
 loss = x% of Rs x 100 100
According to the given condition, we have
x x2
=x = Rs 125 110
100 100 x y = 1520
But, Loss = Cost price – Selling price 100 100
= (x – 9) or, 125x + 110y = 152000
Now, according to the question, or, 25x + 22y = 30400 ....(i)

K KUNDAN
If profit on a chair is 10% and on a table
x2 is 25%, then total selling price is Rs 1535.
= x – 9
100
 10   25 
or, x2 = 100x – 900  x  x   y  y  = 1535
 100   100 
or, x2 – 100x + 900 = 0
or, x2 – 90x – 10x + 900 = 0 110 125
or, x(x – 90) – 10(x – 90) = 0 or, x y  1535
or, (x – 10) (x – 90) = 0 100 100
 x = 90 or 10 or, 110 x  125 y  153500
 Cost price of the article = Rs 90 or Rs 10. or, 22x + 25y = 30700 ....(ii)
Ex. 57: A person sold his watch for Rs 48, and Subtracting (ii) from (i), we get
got a percentage of profit equal to the 3x - 3y = –300
half of the cost price. Find the cost or, x – y = –100 ....(iii)
price of watch. Adding (ii) to (i), we get
Soln: Suppose cost price of the watch = Rs x. 47x + 47y = 61100
x = x + y = 1300 ....(iv)
 percentage profit = % Solving (iii) and (iv), we get
2
x = 600 and y = 700.
x Hence, the cost price of a chair is Rs 600
x x 2 and that of a table is Rs 700.
 profit = % of Rs x  2  x
2 100 200 Ex. 59: On selling a tea-set at 5% loss and a
lemon-set at 15% gain, a crockery seller
But, Profit = Selling price – Cost price gains Rs 7. If he sells the tea-set at 5%
= (48 – x) gain and the lemon-set at 10% gain, he
Now, according to the question, gains Rs 13. Find the actual price of
x2 the tea-set and the lemon-set.
= 48 – x Soln: Let the cost prices of the tea-set and the
200
lemon-set be Rs x and Rs y respectively
or, x2 = 9600 – 200x Case-I: When tea-set is sold at 5% loss
or, x2 + 200x – 9600 = 0 and lemon-set at 15% gain.
or, x2 + 240x – 40x – 9600 = 0
or, x(x + 240) – 40(x + 240) = 0
290 Concept of Arithmetic

Now, if the difference is Rs 15, CP of the


 5x   x 
Loss on tea-set = Rs   = Rs   horse = Rs 100
 100   20  If the difference is Rs 600, CP of the horse
 15y   3y   100 
Gain on lemon-set = Rs   = Rs   =   600  = Rs 4000
 100   20   15 
 CP of the tonga = Rs 7500 – Rs 4000
 3y x 
 Net gain = Rs    = Rs 3500
 20 20   CP of the horse = Rs 4000 and the CP
of the tonga = Rs 3500
3y x
or,  = 7 Alternative Method:
20 20 CP of tonga + CP of horse = Rs 7500
or, 3y – x = 140 Total selling price at a profit of 19%
or, x – 3y + 140 = 0 .... (i)
119
Case II: When tea-set is sold at 5% gain = Rs 7500 
and the lemon-set at 10% gain. 100
Let the CP of tonga be Rs x.
 5x   x 
Gain on tea-set = Rs   = Rs   Then the CP of horse = Rs (7500 – x)
 100   20   SP of tonga at 11% profit
 10y   y   100  11  111x
Gain on lemon-set = Rs   = Rs   = x  = Rs
 100   10   100  100
SP of horse at 26% profit
 x y 
 Total gain = Rs   
 20 10   100  26 
= (7500  x )   
 100 
x y
or,   13 Now according to the question,

K KUNDAN
20 10
or, x + 2y = 260 111x 126 119
 (7500  x ) = 7500 
or, x + 2y – 260 = 0 ....(ii) 100 100 100
Subtracting (ii) from (i), we get or, 111x + (7500 – x)126 = 7500 × 119
–5y + 400 = 0 or, 111x –126x
or, y = 80 = –7500 × 126 + 7500 × 119
Putting y = 80 in (i), we get or, –15x = 7500 (–7)
x – 240 + 140 = 0
7500  7
or, x = 100  x = = Rs 3500
Hence, cost prices of tea-set and lemon- 15
set are Rs 100 and Rs 80 respectively.  CP of tonga = Rs 3500 and the
Ex. 60: A man bought a tonga and a horse for CP of horse = Rs (7500 – 3500)
Rs 7500. He sold the t onga at 11% = Rs 4000
profit and the horse at 26% profit, Ex. 61: A scooter dealer bought two scooters
making 19% profit on the whole. Find for Rs 13000. By selling one at a profit
the cost price of the tonga and the of 20% and the other at a loss of 25%,
horse separately. he found that selling prices of the two
Soln: CP of tonga + CP of horse = Rs 7500 are same. Find the cost price of each
Gain on the whole transaction = 19% scooter.
Soln: Let the cost price of the first scooter be Rs
7500  119
 total SP = Rs = Rs 8925 x.
100 Then the cost price of the second scooter
If he had sold both at a profit of 11% , = Rs (13000 – x)
then SP of both would have been Profit of first scooter = 20%
Selling price of the first scooter
7500  111
Rs = Rs 8325 x  120
100
= Rs
Difference in two SP’s 100
= Rs 8925 – Rs 8325 = Rs 600
6x
This difference is due to the SP of the = Rs ....(i)
horse at 26% prof it inst ead of 11% 5
(26 – 11)% or 15% higher profit. Loss on second scooter = 25%
Profit and Loss 291

 SP of the second scooter or, 24x = 195000 – 15x


or, 24x + 15x = 195000
 (13000  x )75 
= Rs   or, 39x = 195000
 100 
195000
 39000  3x   x = = 5000
= Rs   ...(ii) 39
 4 
 CP of the first scooter = Rs 5000 and
As the two SP’s are equal, we have from the CP of the second scooter
equations (i) and (ii) = Rs 13000 – Rs 5000
6x 39000  3x = Rs 8000.

5 4

Practice Exercise
1. A shopkeeper buys a cycle for Rs 450 and 16. Rama sells an article at a loss of 10%. If she
sells it for Rs 495. Find his profit per cent. had sold it for Rs 30 more, she would have
2. A man bought a scooter for Rs 8500 and sold gained 5%. Find the cost price of the article.
it for Rs 8720. Find his loss per cent. 17. A machine was sold at a profit of 10%. Had it
3. Sohan bought a house for Rs 65000. He spent been sold for Rs 400 less, there would have
Rs 10000 on its repairs. Then he sold it for been a loss of 10%. Find the cost price of the
Rs 90000. Find his gain per cent. machine.
4. A man buys a colour TV for Rs 6800. For how 18. The expenditure on transportation of goods
much should he sell it so as t o is 5% of the cost price. Find the cost price of
(i) gain 10% (ii) lose 5% on it. the goods sold for Rs 9450 at a gain of 20%.
5. A shopkeeper sold a fridge for Rs 4850 at a 19. A manufacturer sells a battery to a wholesaler

K KUNDAN
loss of 3%. Find his cost price. at a profit of 18%, the wholesaler sells it to a
6. By selling a radio for Rs 528, Ramesh gains retailer at a profit of 20%. The retailer sells it
10% . Find his cost price. to a customer at a profit of 25%. If the customer
7. A shopkeeper buys 144 eggs at Rs 3 each. On pays Rs 30.09 for it, find the cost of the
the way 20 eggs are spoiled. He sells the manufacturer.
remainder at Rs 4 each. Find his gain or loss 2 0 . A dishonest dealer professes to sell his goods
per cent. at cost price but uses a false weight of 800
8. Subramaniam bought 100 eggs for Rs 50. Out gm instead of 1 kg. Find his profit per cent.
of these, 4 eggs were found to be broken and 21. Kundan sold a watch at a profit of 15%. Had
he sold the remaining eggs at the rate of Rs he bought it at 10% less and sold it for Rs 28
8.50 per dozen. Find his gain or loss per cent. less, he would have gained 20% . Find the
9. A grocer buys eggs at 10 for Rs 8 and sells at cost price of the watch.
8 for Rs 10. Find his gain or loss per cent. 2 2 . An article is sold at a profit of 20% . If both
10. Toffees are bought at the rate of 6 for Re 1 the cost price and the selling price were to
and sold at the rate of 5 for Re 1. What is the be Rs 20 less, the profit would be 10% more.
gain per cent? Find the cost price of the article.
11. Selling price of 9 articles is equal to the cost 23. On selling a watch for Rs 280, a man earns
price of 15 articles. Find the gain or loss per 1
cent in the transaction. of the cost price. Determine the cost price
3
12. Shashi sold a radio to Kanta at gain of 10% of the watch and his gain per cent.
and Kanta sold it to Shama at a gain of 15%. 24. A man sold two articles at Rs 375 each. On
Shashi had bought it for Rs 500, what did it one he gains 25% and on the other he loses
cost to Shama? 25% . How much does he gain or lose in the
13. A sells a goods to B at a profit of 20% and B whole transaction? Also find his gain or loss%
sells it to C at a profit of 25%. If C pays Rs in the whole transaction.
225 for it, what was the cost price of A? 25. A t r adesman by means of f alse balance
14. A shopkeeper loses 20% by selling tea at Rs defrauds to the extent of 8% in buying goods
32 per kg. How should he sell it to gain 10%? and also defrauds to the extent of 8% in
15. A television manufacturer earns 20% profit selling. Find his gain per cent.
by selling each TV set for Rs 14400. If the 26. A person sells a table at a profit of 25% and a
production cost is increased by 15% , what
2
should be the new selling price of a set so as chair at a loss of 16 % and on the whole he
to gain 15% . 3
292 Concept of Arithmetic

gains Rs 50. On the other hand, if he sells gain% must he sell the remainder so as to
gain 10% of the whole?
2
the table at a loss of 16 % and the chair at 36. A man purchased a table and a chair for Rs
3 2000. He sold the table at a profit of 20% and
a profit of 25% , he neither gains nor loses. the chair at a profit of 30%. His total profit
Find the cost price of the table and that of was 23% . Find the cost price of the table.
the chair.
1
27. A man purchases 5 horses and 10 cows for 37. A person purchased some goods. He sold rd
Rs 10000. He sells the horses at 15% profit 3
and the cows at 10% loss. Thus he gets Rs
3
375 as profit. Find the cost of 1 horse and 1 goods at 14% profit, th goods at 17.5%
cow separately. 5
28. Two articles were purchased for Rs 1000. One profit and rest at 20% profit. Find his gain in
was sold at a loss of 12% and the other at a this transaction.
gain of 8% . If there was no loss or gain in 38. Three tables are purchasd for Rs 2500 each.
the transaction, find the selling price of the First is sold at a profit of 8%, the second is
two articles separately. sold at a loss of 3%. If their average selling
29. A person bought a horse and a carriage for Rs price is Rs 2575, find the profit per cent on
10000. He sold the horse at a gain of 20% the third.
and the carriage at a loss of 10%, thus gaining 39. A sells his goods 20% cheaper than B and
2% on the whole transaction. Find the cost 20% dearer than C. A man buys goods worth
price of the horse. Rs 9600 from A. Would it have been more
30. A shopkeeper sells 100 kg of sugar partly at profitable or less profitable and by how much
10% profit and the remaining at 20% profit. to have bought half the goods from B and half
If he gains 12% on the whole transaction, from C?
how much sugar does he sell at 20% profit? 40. A man sells a TV at a profit of 25% of the
31. A owned an article worth Rs 10000. He sold cost. Had he sold it at a profit of 25% of the

K KUNDAN
it to B at a profit of 10% based on the worth selling price, his profit would have increased
of the article. B sold the article back to A at a by 5% of the cost price plus Rs 100. Find the
loss of 10% . How much did A make in these cost of TV.
transactions? 41. A bought an article X for Rs 8000 and an
32. By selling an article for Rs 144, a merchant article Y for Rs 2000. He sold them to B and
gains such that gain per cent equals the cost B sold them to C, each making a profit of
price. Find the cost price of the article. 25% on X and each incurring a loss of 10%
33. A shopkeeper has two varieties of tea. One on Y. If A were to sell them to C directly at
quality costs him Rs 24 per kg and the other the same selling price as that at which B had
Rs 28 per kg. He mixes them in the ratio of 3 sold to C, what per cent would A have gained
: 2 and sells the blended tea at Rs 26.88 per on the whole?
kg. Find his gain per cent. 42. A man sold an article at a loss of 5%. Had he
34. 450 pens each of the same cost were classified sold it at a gain of 10%, he would have fetched
into three categories for the purpose of sale. Rs 525 more. Find the cost of the article.
They were sold at 9% , 10% and 12% profit 43. A watch is sold at a profit of 20%. If both the
respectively. The first two categories gave an cost price and the selling price of the watch
are decreased by Rs 100, the profit would be
3 5% more. Find the original cost price of the
overall profit of 9 % . All the three categories
7 wat ch.
gave an overall profit of 10%. Determine the 44. A sells his goods 10% cheaper than B and
number of pens put in the first category. 10% dear er t han C. How much would a
35. A dealer bought 80 cricket bats for Rs 50 each. customer of B save by purchasing an article
He sells 20 of them at a gain of 5%. At what of worth Rs 100 from C?
Profit and Loss 293

Answers and explanations


1. Cost price of the cycle = Rs 450  If selling pirce is Rs 97, then cost price is
Selling price of the cycle = Rs 495 Rs 100.
Profit = Selling price – Cost price  If selling price is Rs 4850, then cost price
= Rs 495 – Rs 450 = Rs 45
 100 
45 = Rs   4850  = Rs 5000
Profit per cent =  100 = 10%  97 
450
6. Let the cost price of radio be Rs 100
2. Cost price of the scooter = Rs 8500 Gain = 10%
Selling price of the scooter = Rs 7820 Selling price = Rs 100 + Rs 10 = Rs 110
Loss = Cost price – Selling price
 When selling price is Rs 110, cost price is
= Rs 8500 – Rs 7820 = Rs 680
Rs 100.
680  When selling price is Rs 528, cost price is
Loss per cent =  100 = 8%
8500 100
3. Cost price of the house Rs  528 = Rs 480
110
= Rs 65000 + Rs 10000 = Rs 75000
7. Cost price of 144 eggs = Rs 144 × 3 = Rs 432
Selling price of the house = Rs 90000
Selling price of (144 – 20 =) 124 eggs
 Gain = Rs 90000 – Rs 75000 = Rs 15000
= Rs 124 × 4 = Rs 496
15000 Gain = Rs 496 – Rs 432 = Rs 64
 Gain per cent =  100 = 20%
75000 64
4. ( i ) Cost price of TV = Rs 6800 Gain per cent =  100 = 14.8%
432
Gain = 10% of Rs 6800
8. We have, CP of 100 eggs = Rs 50.
 10  It is given that 4 eggs were found to be broken.
= Rs   6800  = Rs 680

K KUNDAN
So, the number of remaining eggs which were
 100 
sold in the market = 100 – 4 = 96.
 Selling price = Rs 6800 + Rs 680 It is given that the remaining eggs were sold
= Rs 7480 at the rate of Rs 8.50 per dozen.
( i i) Loss = 5% of Rs 6800 Therefore, SP of 12 eggs = Rs 8.50
6800  5 8.50
= Rs = Rs 340  SP of 1 egg = Re
100 12
 Selling price in this case
= Rs 6800 – Rs 340 = Rs 6460  8.50 
Hence, SP of 96 eggs = Rs   96 
Alternative Method:  12 
( i ) Let the cost price be Rs 100
= Rs 68.
Gain = Rs 10
Clearly, SP > CP. So, there is gain given by
Selling price = Cost price + Gain (profit)
Gain = SP - CP = Rs 68 - Rs 50 = Rs 18.
= Rs 100 + Rs 10 = Rs 110
If the cost price is Rs 100, then the selling  Gain 
price of TV is Rs 110. Hence, gain per cent =   100 %
 CP 
If the cost pr ice is Rs 6800, then the
110  18 
selling price of TV =  6800 = Rs 7480 =   100 % = 36%
100  50 
( i i) Let the cost price be Rs 100. 9. We have, LCM of 10 and 8 = 40. So, let the
 Loss is Rs 5. number of eggs bought be 40.
Selling price = Rs (100 – 5) = Rs 95 Now, CP of 10 eggs = Rs 8.
 If cost price is Rs 100, then the selling
price is Rs 95.  8 
 CP of 40 eggs = Rs   40  = Rs 32.
 If cost price is Rs 6800, then the selling  10 
95 Again, SP of 8 eggs = Rs 10
price is Rs  6800 = Rs 6460
100  10 
5. Suppose cost price of the TV = Rs 100  SP of 40 eggs = Rs   40  = Rs 50.
 8 
Loss = 3%
 Selling price = Rs 100 – Rs 3 = Rs 97 Now, CP = Rs 32 and SP = Rs 50.
294 Concept of Arithmetic

Clearly, SP > CP. So, there is gain given by 13. CP of the goods for C = 225
Gain = SP - CP = Rs 50 - Rs 32 = Rs 18. SP of the goods for B = CP of the goods for C
= Rs 225
 Gain 
Hence, Gain per cent =   100 %  100 
 CP  CP of the goods for B = Rs    225
 100  25 
 18  225 1
=   100 % = % = 56 %  100 
 32  4 4 = Rs   225 
 125 
10. See the solution of Q.No. 9. This question
= Rs 180
can also be solved by the above mentioned
CP of the goods for B = SP of the goods for A
method. Let us see how?
= Rs 180
100 50
Cost price of 1 toffee = = paise  100 
6 3  CP of the goods for A = Rs    180
 100  20 
100
Selling price of the 1 toffee = = 20 paise  100 
5 = Rs  180 
 120 
 50   60  50  10 = Rs 150
Profit =  20   =   = paise 14. Selling Price = Rs 32 per kg
 3   3  3
Loss = 20%
10  100 
10 3  Cost price = Rs   32  per kg
3  100  20 
 Profit per cent = 50  100    100
3 50
3 100
= Rs  32 = Rs 40 per kg
= 20% 80

K KUNDAN
11. Let the cost price of each article be Rs x. Again,
Then, cost price of 9 articles = Rs 9x. Gain = 10%
Selling price of 9 articles = Cost price of 15
 100  10 
articles = Rs 15x Selling price = Rs   40  per kg
 Gain on the purchase of 15 articles  100 
= Rs 15x – Rs 9x = Rs 6x
110
= Rs  40 = Rs 44 per kg
 Gain  100
Hence, Gain per cent =   100 %
 CP   He should sell tea at Rs 44 per kg to gain
10% .
 6x  200 2 15. SP of TV set = Rs 14400
=   100 %  %  66 % Profit = 20%
 9x  3 3
 CP of the TV set
12. CP of the radio set for Shashi = Rs 500
Gain = 10% of Rs 500  100 
=   14400  = Rs 12000
 100  20 
 10 
= Rs   500  = Rs 50 Now, accor ding t o t he quest ion, t he
 100  production cost is increased by 15%
SP of the radio set for Shashi  New CP = 115% of Rs 12000
= CP + Gain = Rs 500 + Rs 50 = Rs 550
 115 
 CP of the radio set for Kanta = Rs  12000  = Rs 13800
= SP of the radio set for Shashi  100 
= Rs 550  Selling price of the TV set to gain 15%
Since Kanta sold the radio to Shama at a gain
 100  15 
of 15% =   13800 
 Gain = 15% of Rs 550  100 

 15   115 
= Rs   550  = Rs 82.5 =  13800 
 100   100 
SP of radio set for Kanta = Rs 15870
= CP + Gain = Rs 550 + Rs 82.5 16. Let the cost price of the article be Rs 100.
= Rs 632.50 Loss = 10% = Rs 10
 CP of the radio set for Shama = Rs 632.50
Profit and Loss 295

 Selling price of the article  If the customer pays Rs 177, the cost of
= Rs 100 – Rs 10 = Rs 90 the manufacturer is Rs 100.
Selling price of the article to gain 5%  If the customer pays Rs 30.09, the cost of
= Rs 100 + Rs 5 = Rs 105
100
Difference in two selling prices the manufacturer =  30.09 = Rs 17
= Rs 105 – Rs 90 = Rs 15 177
 When difference in SP is Rs 15, CP of the 2 0 . Suppose goods costs the dealer Re 1 per kg.
article is Rs 100. He sells for Re 1 what costs him Re 0.80.
When difference in SP is Rs 30, CP of the  Gain on Re 0.80 = Re 1 – Re 0.80 = Re 0.2
article 0.2
 Gain on Rs 100 =  100 = Rs 25
100 0.80
=  30 = Rs 200
15  Gain per cent = 25%
 Cost price of the article = Rs 200 21. Let the actual cost price of watch be Rs 100.
17. Let the cost price of the machine be Rs 100. Actual selling price of watch at the profit of
Profit = 10% = Rs 10 15%
 Selling price of the machine = Rs 100 + Rs 15 = Rs 115
= Rs 100 + Rs 10 = Rs 110 Supposed cost price of the watch at 10% loss
Selling price of the machine at a loss of 10% = Rs 100 – Rs 10 = Rs 90
= Rs 100 – Rs 10 = Rs 90 Supposed selling price of the watch at 20%
Difference in two selling prices gain
= Rs 110 – Rs 90 = Rs 20
 120 
 When difference in SP is Rs 20, CP of the = Rs  90   = Rs 108
machine is Rs 100.  100 
 When difference in SP is Rs 400, CP of the  the difference in selling prices
100 = Rs 115 – Rs 108 = Rs 7
machine =  400 = Rs 2000  If the difference is Rs 7, the CP is Rs 100.
25
 If the difference is Rs 28 the CP is Rs

K KUNDAN
 Cost price of the machine = Rs 2000.
18. Let the cost price of goods be Rs x.  100 
  28  = Rs 400
 effective cost price = x + 5% of x  7 
5x x 21x  Cost price of the watch = Rs 400
= x x = Rs 2 2 . Suppose the cost price (CP) of the article is
100 20 20
Rs x.
Selling price = Rs 9450 To get a profit of 20%, selling price (SP) must
Now, according to the question,
 100  20 
21x  100  20  be Rs   x = 1.2x.
  = 9450  100 
20  100 
New cost price = Rs (x – 20)
21x 120 Also, the new selling price = Rs (1.2x – 20)
or,  = 9450 Now, according to the question
20 100
(x – 20) × 130% = (1.2x – 20)
9450  20  100 130
or, x = = Rs 7500 or, (x – 20) × = (1.2x – 20)
21  120 100
19. Let the cost of the manufacturer be Rs 100.
or, (x – 20)1.3 = 1.2x – 20
Gain of the manufacturer = 18%
or, 1.3x – 1.2x = –20 + 26
Then CP of the wholesaler
or, x = 60
= Rs 100 + Rs 18 = Rs 118
Thus, the required cost price of the article is
Gain of the wholesaler = 20%
Rs 60.
New, SP of the wholesaler = CP of the retailer
23. Let the cost price of the watch be Rs 100.
 120 
= Rs 118   = Rs 141.60 1 100
 100  Profit = rd of the cost price = Rs
3 3
Gain of the retailer = 25%
 CP of the customer 100 400
Selling price = Rs 100 + Rs = Rs
3 3
 125 
= Rs 141 .60   = Rs 177 400
 100 
 If selling price is Rs , cost price is Rs
3
100.
296 Concept of Arithmetic

 If selling price is Rs 280, cost price is 26. Let the cost price of table and chair be Rs T
and C respectively.
 100  3 
Rs   280  = Rs 210 Total CP of a table and a chair = Rs (T + C)
 400  In First Case:
Proft = Selling Price – Cost Price
= Rs (280 – 210) = Rs 70  25  5T
SP of a table = T 1   = Rs
 100  4
70 1
Profit per cent =  100 = 33 %
210 3  50  5C
24. For the first article, we have SP of a chair = C 1   = Rs
 100  6
SP = Rs 375 and Gain = 25%
100 5 T 5C
 SP Total SP of a table and chair = 
 CP = 4 6
100  Gain%
On the whole, there is a gain of Rs 50,
 100 
or, CP = Rs   375  = Rs 300  5 T 5C 
 100  25  ie     (T  C)  50
For the second article, we have,  4 6 
SP = Rs 375 and Loss = 25%
T C
or,   50
100 4 6
 CP =  SP
100  Loss% or, 3T – 2C = 600 ....(i)
In Second Case:
 100 
or, CP = Rs   375  = Rs 500
 100  25   50  5T
SP of a table = T 1   = Rs
 Total cost paid in buying the two articles  300  6
= Rs (300 + 500) = Rs 800.
 25  5C

K KUNDAN
Total SP = Rs 375 + Rs 375 = Rs 750. SP of a chair = C 1   = Rs
 100  4
 CP > SP. So, there is a loss
 Total loss = CP – SP 5 T 5C
= Rs 800 – Rs 750 = Rs 50 Total SP of a table and chair = 
Hence, loss per cent 6 4

 Loss   50  1 5 T 5C
=   100 % =   100   6 % Also,   TC
 CP   800  4 6 4
25. In normal case, suppose the tradesman buys 5C 5T
x kg of the goods in Rs x. or, C  T
4 6
By using f alse balance t r adesman buys,
according to the question, (x + 8% of x) kg of C T
goods in Rs x. or, 
4 6
1
 Cost price per kg = Rs 3C
1.08 or, T 
2
Again, from the question
Selling price per kg of goods Now, putting the value of ‘T’ in equation (i),
we get,
x 1
= Rs x  8% of x = Rs 3  3C
0.92  2C  600
2

 1 1  9C  4C
   or,  600
 0.92 1.08   100 2
Profit % = 1
 
  5C
 1.08   600
or,
2
 1.08  0.92  16
   100   100 600  2
 0.92  92 or, C = = Rs 140 and
5
400 9 3C 3  240
  17 % T =  = Rs 360
23 23 2 2
Profit and Loss 297

27. Let the cost of 1 horse be Rs x, then total


120x  900000  90x  1000000
selling price =
100
 115   90 
= 5x    ( 10000  5 x)   30 x  100000
 100   100   ..... (ii)
100
Now, according to the question,
From equations (i) and (ii), we have,
 115   90  30x  100000
5x    (10000  5x )   = 10375  200
 100   100  100
or, 575x + 90 × 10000 – 450x = 10375 × 100 or, 30x  100000  20000
or, 125x
or, 30x  120000
137500
 x = = Rs 1100 120000
125
or, x = = Rs 4000
Therefore, the cost of one horse = Rs 1100 30
 The CP of the horse = Rs 4000 and CP of
10000  5  1100
and the cost of one cow = the carriage = Rs (10000 – 4000) = Rs 6000.
10 30. Let the shopkeeper sell x kg of sugar at 20%
= Rs 450 profit.
28. Let the CP of first article be Rs x. Then, the shopkeeper sold (100 – x) kg of
 the CP of second article be Rs (1000 – x). sugar at 10% profit.
Loss on first article = 12% of x SP of x kg of sugar at 20% profit
12 3x  20  6x
=  x = Rs
100 25 = x 1  
 100  5
Profit on second article = 8% of (1000 – x)
SP of (100 – x) kg of sugar at 10% profit
8 2

K KUNDAN
=  (1000  x )  (1000  x ) 10 
100 25
= 100  x  1  
As there is no loss or gain in the transaction,  100 
 Loss on first article = Profit on second
article 10
= 100  x  10  x 
3x 2 100
 (1000  x )
25 25 11x
or, 3x = 2(1000 – x) = 110 
10
or, 3x = 2000 - 2x SP on the whole transaction
or, 5x = 2000
or, x = Rs 400  12 
 CP of first article = Rs 400 = 1001    112
 100 
CP of second article = Rs (1000 – 400)
= Rs 600 6x 11x
SP of first article = 88% of 400 = Rs 352 Then,  110   112
5 10
SP of second article = 108% of 600 = Rs 648.
29. Let the CP of the horse be Rs x 6x 11x
or,  = 112 - 110 = 2
 The CP of the carriage be Rs (10000 – x) 5 10
120x 12x  11x
SP of the horse = or, 2
100 10
9010000  x  x
SP of the carriage = or, 2
100 10
Total gain in the transaction or, x = 20 kg
= 2% of Rs 10000 = Rs 200 ... (i) 31. Worth of article = Rs 10000
But total gain CP for B = 110% of Rs 10000
= SP of both things – CP of both things
 110  10000 
120 x 9010000  x  = Rs   = Rs 11000
=   10000  100 
100 100
B sells the article back to A at a loss of 10%.
298 Concept of Arithmetic

 CP for A = 90% of Rs 11000 From equation (i),


 90  3x
= Rs  11000  = Rs 9900 3x   900
 100  2
 Gain for A = Rs 11000 – Rs 9900 = Rs 1100
9x
32. Let the cost price of the the article be Rs x. or,  900
 gain per cent = x% 2
 x = 200
 x2  Hence, t he number of pens in t he f ir st
 
 gain = x% of x = Rs  100  category = 200
 
35. We have CP of one bat = Rs 50
Gain = Selling Price – Cost Price  CP of 20 bats = Rs (50 × 20) = Rs 1000
= Rs (144 – x) Gain on these bats = 5%
Now, according to the question,
 100  Gain% 
x2  SP of 20 bats = Rs    CP
= 144 – x  100 
100
2
or, x = 14400 – 100x  100  5 
= Rs   1000 
or, x2 + 100x – 14400 = 0  100 
or, x(x + 180) – 80(x + 180) = 0 = Rs 1050
or, x2 + 180x – 80x – 14400 = 0 Now, CP of 80 bats = Rs (80 × 50) = Rs 4000.
or, (x – 80) (x + 180) = 0 Desired gain = 10%
or, x = –180 or 80
Value of x cannot be negative, hence x = 80.  100  Gain% 
 Required SP = Rs    CP
Hence, cost price of the article is Rs 80.  100 
33. Suppose he buys 3x kg of tea of first quality
and 2x kg of tea of second quality.  100  10 
= Rs   4000 

K KUNDAN
Then, CP of 5 kg of tea  100 
= Rs 24 × 3x + Rs 28 × 2x = Rs 4400
= Rs 72x + Rs 56x = Rs 128x Thus, desired SP of 60 bats
SP of 5x kg of tea = Rs 26.88 × 5x = Rs 134.40x = Rs 4400 – Rs 1050 = Rs 3350.
 Profit = Rs 134.40x – Rs 128x = Rs 6.40x CP of 60 bats = Rs (60 × 50) = Rs 3000.
6.40x Desired gain on 60 bats
 Profit per cent =  100% = 5% = Rs 3350 – Rs 3000 = Rs 350.
128x
Hence, desired gain%
34. Let the number of pens in the first and the
second categories be x and y respectively.  350  2
=   100 % = 11 %
 Number of pens in third category  3000  3
= (450 – x – y) 36. Let the cost price of the table be Rs x
Let the cost of each pen be Re 1. Cost price of the chair will be Rs (2000 – x)
According to the question, Selling price of the table at 20% profit
x  109 y  110  66  x (100  20) 6x
  x  y  1  % 
100 100  7  =
100 5
109 x 110y  66  Selling price of the chair at 30% profit
or,   x  y  1  
100 100  700  100  30
= 2000  x 
109 x 110 y  766  100
or,   x  y   
100 100  700 
130
or, 7(109x + 110y) = 766x + 766y = 2000  x  
100
or, 763x + 770y = 766x + 766y
or, 3x = 4y ...(i) 13
Again, = 2600  x
10
109x 110y 112 450  110 His total SP at the total profit of 23% is
  450  x  y   
100 100 100 100 100  23
or, 109x + 110y + 450 × 112 – 112x – 112y = 2000 
100
= 450 × 110
or, 3x + 2y = 450(112 – 110) 123
or, 3x + 2y = 900 = 2000  = Rs 2460
100
Profit and Loss 299

Now, accroding to the question, Second Case:


He purchased goods worth Rs 4800 from B.
6  13x 
x   2600    2460 As A sells his goods 20% cheaper than B.
5  10  If the cost price of A is Rs 80, CP of B is Rs
100.
6x 13x
  2460  2600 Now if the cost of A is Re 1, the CP of B is
5 10
100
12x  13x Rs
or,  140 80
10 If the cost price of A is 4800, the CP of B is
or, –x = –140 × 10
or, x = 1400 100  4800
= = Rs 6000
The cost price of the table is Rs 1400. 80
37. Suppose the value of the commodity was Rs He purchases goods worth Rs 4800 from C
x. As A sells his goods 20% dearer than C.
x 3x
Then, was sold at 14% profit, was If the CP of A is Rs 120, the CP of C is 100
3 5
100
 x 3x  x If the CP of A is Re 1, the CP of C is
120
sold at 17.5% profit and x    
3 5  15 If the CP of A is Rs 4800, the CP of C is
was sold at 20% profit.
 100  4800 
Now, profit =    Rs 4000
 120 
x  14  3x  17 .5  x  20 
=       Total cost price of man when he purchases
3  100  5  100  15  100  half from B and half from C
= Rs 6000 + Rs 4000 = Rs 10000.
14x 10.5x 4x
=   Thus it can be seen that it is profitable for a

K KUNDAN
300 100 300 person to buy the whole amount from A as it
will cost him. Rs 400 cheaper in comparison
14x  31.5x  4x 49.5 x
= = to second case.
300 300 40. Let the cost price be Rs x
49.5x 25 x
300  100 = 16.5% Profit = of x = Rs
Gain per cent = 100 4
x
x 5x
38. The cost price of each table = Rs 2500 Selling price = x  = Rs
4 4
 100  8 
 Selling price of first table = 2500    5x 1
 100  25% profit on selling price = Rs 
= Rs 2700 4 4
 Selling price of second table 5x
= Rs
 100  3  16
= 2500    = Rs 2425
 100  According to the question,
Selling price of three tables Profit in second case – Profit on first case
= Rs 2575 × 3 = Rs 7725 = 5% of cost price + 100
Selling price of third table 5x x 5
= Rs (7725 – 2700 – 2425) or,  x  100
16 4 100
= Rs 2600
Profit on third table = Rs 2600 – Rs 2500 5 x  4x x
or,   100
= Rs 100 16 20
 % Profit on the third table
x x
or,   100
100 16 20
=  100  4%
2500
x
39. First Case: or,  100
The man purchased goods worth Rs 9600 80
from A. or, x = 8000
 Cost price = Rs 8000.
300 Concept of Arithmetic

41. In case of A Case I:


Cost price of article X = Rs 8000 Cost price = Rs (x – 100)
Cost price of article Y = Rs 2000
 6x 
Now, according to the question in case of B Selling price = Rs  100 
Cost price of article X  5 
8000  125 Profit = Selling Price – Cost Price
= Rs = Rs 10000
100  6x 
Cost price of article Y =   100  x  100 
 5 
2000  90
= Rs = Rs 1800  6x  x
100 = Rs   x  = Rs
 5  5
Also in case of C
Cost price of article X According to the question,

10000  125  x 
= Rs = Rs 12500  
100  5   100
Cost price of article Y  100  = 25
 
 
1800  90
= Rs = Rs 1620
100 20x
Total cost price of X and Y for A or, = 25
x  100
= Rs 8000 + Rs 2000 = Rs 10000
Total selling price of B to C 4x
= Rs 12500 + Rs 1620 = Rs 14120 or, = 5
x  100
Profit for A in case he sells to C at the same or, 5x – 500 = 4x
price as B = Rs 14120 – Rs 10000 = Rs 4120 or, x = 500

K KUNDAN
4120  CP of the watch = Rs 500
Profit % =  100  41.2% 44. Let B sell an article for Rs 100.
10000
 SP of A = Rs (100 – 10) = Rs 90 and
Hence, the required answer is 41.2% .
42. Let the cost price of the article be Rs x then,  100   100 
SP of C =   90  =   90 
according to the question,  100  10   110 
 100  10   100  5  900
 x   x = Rs 525
 100   100  = Rs
11
110 95
or, x x = Rs 525 900
100 100  When C sells an article for Rs , B
11
22 19
or, x x = Rs 525 sells it for Rs 100.
20 20  If C sells an article for Rs 100, B will sell it
22x  19 x 100  11 1100
or, = Rs 525 for Rs  100 = Rs
20 900 9

 525  20  1100
 x = Rs   = Rs 175 × 20 = Rs 3500  Required savings =  100
 3  9
43. See the solution of Q.No. 22.
1100  900
Let the CP of the watch be Rs x. =
Case I: 9

 100  20  120 x 6x 200


Selling price =  x  = Rs = Rs
 100  100 5 9
Chapter-16

Discount
Marked Price First discount = 20%
Amount of first discount
You might have seen while buying goods that on
every article there is a price marked. This price is 100  20
= Rs = Rs 20
known as the marked price. Marked price is also 100
called list price or printed price. Marked price is  Net amount = Rs 100 – Rs 20 = Rs 80
abbreviated as MP. Second discount = 5%
Amount of second discount
List Price
80  5
Elect rical goods, electr onics and ot her t hings = Rs = Rs 4
which are manufactured in a factory are marked 100
accroding to the price list supplied by the factory,  Price to be paid by the customer
at which the retailer is supposed to sell them. = Rs 80 – Rs 4 = Rs 76
This price is known as the list price. For books ( i ) Maruti Suzuki
etc, the printed price is marked price. Discount series = 5% , 20%
First discount = 5%
Discount Amount of first discount
In order to clear the stocks or to increase sales, 100  5
sometimes shopkeepers offer a certain per cent of = Rs = Rs 5
100
rebate on the marked price for cash payments. This
 Net amount = Rs 100 – Rs 5 = Rs 95
rebate is known as discount.
Second discount = 20%
The customer or buyer pays t he diff er ence
Amount of second discount
between the marked price and discount. Thus,
Selling Price = Marked Price – Discount 95  20
It should be noted that discount is given on = Rs = Rs 19
100
the marked price only.
 Price to be paid by the customer
Successive Discounts = Rs 95 – Rs 19 = Rs 76
Thus, selling prices of both the companies are
When two or more discounts are allowed one after equal. There is no difference in selling price and
the ot her , then such discounts are known as the customer may buy goods from any one.
successive discounts. Note: We have seen in the above example that it
In successiv e discount s, f ir st discount is makes no difference if we change the order
subtracted from the marked price to get net price of the discount series. By cummulative law
after the first discount. Taking this price as the of multiplication discount series 30% and
new mar ked pr ice, t he second discount is 10% is equal to the discount series 10%
calculated and it is subtracted from it to get net and 30% . Similarly series 25% , 15% and
price after the second discount. Continuing in this 15%, 25% are also equal.
manner, we finally obtain the net selling price. In the abov e example, selling pr ices of
The following examples will illustrate the above  80 95 
procedure. t wo f ir ms are Rs 100    and
 100 100 
“Hyundai Motors offers a series of discounts
20% and 5% and Maruti Suzuki offers a  95 80 
Rs 100    are equal.
discount series 5% and 20% . Which of the  100 100 
two offers is beneficial to the customer?”
Let us see how we can calculate the single Cash Discount
discount equivalent to successive discounts. When a retailer purchases some goods from a
( i ) Hyundai Motors manufacturer or wholesaler, he gets an invoice.
Discount series = 20% , 5% The meaning of invoice is ‘a list of goods sold or
302 Concept of Arithmetic

services provided together with the prices charged’. included the tax known as Value Added Tax (VAT).
The terms of the payment of the bills are written For example, Samir bought the following articles
on it. If the retailer makes the payment according from a departmental store:
to them, then he gets some discount for cash
payment. This discount is called cash discount. Rate per Rate of
Item Quantity
item (Rs) sales tax
2 1 n
The terms are like this: , , , ..... ie a Shirts 4 200.00 8%
10 20 30
discount of 2% if the payment is made within 10 Pair of shoes 2 350.00 10%
days, a discount of 1% if the payment is made Television 1 10900.00 10%
between 10th and 20th days and the payment must Tea Set 1 750.00 8%
be made within 30 days. The buyer thinks whether Calculate the total bill paid, including sales tax,
he should make the payment early or not—as on by Samir to the departmental store.
one side he gets 2% discount for cash payment— Here, We have,
on the other he can have interest on the money CP of 4 shirts = Rs 200 × 4 = Rs 800
for 1 month. Rate of sales tax = 8%
For example, an invoice is given below, what  Sales tax = 8% of Rs 800
will be payment if the payment is made in 2 days?
Quantity Article Rate  8 
= Rs   800  = Rs 64
4 dozen Pencils Rs 6 per dozen  100 
3 dozen Pens Rs 36 per dozen So, amount paid for 4 shirts
4 dozen Erasers Rs 12 per dozen = Rs (800 + 64) = Rs 864
CP of 2 pairs of shoes = Rs 350 × 2 = Rs 700
3 n Rate of sales tax = 10%
Terms : ,
10 30  Sales tax = 10% of Rs 700
First of all, we will find the amount of the bill  10 
for the goods mentioned in the invoice. = Rs   700  = Rs 70
 100 
Now, cost of 4 dozen pencils So, amount paid for 2 pairs of shoes
= Rs 6 × 4 = Rs 24 = Rs (700 + 70) = Rs 770

K KUNDAN
cost of 3 dozen pens CP of television set = Rs 10900
= Rs 36 × 3 = Rs 108 Rate of Sales tax = 10%
cost of 4 dozen erasers Sales tax = 10% of Rs 10900
= Rs 12 × 4 = Rs 48
 10 
Total amount = Rs 24 + Rs 108 + Rs 48 = Rs 180 = Rs  10900  = Rs 1090
Accor ding to t he t er ms ment ioned in t he  100 
invoice, 3% discount will be given if the payment So, amount paid for television set
is made withing 10 days. = Rs (10900 + 1090) = Rs 11990
 Discount for payment in 2 days CP of tea set = Rs 750
Rate of sales tax = 8%
 3   Sales tax = 8% of Rs 750
= Rs 180   = Rs 5.40
 100 
 8 
 Amount of payment = Rs 180 – Rs 5.40 = Rs   750  = Rs 60
 100 
= Rs 174.60
So, amount paid for tea set
Sales Tax and Value Added Tax (VAT) = Rs (750 + 60) = Rs 810
Hence, total amount of the bill
We know that the government imposes different = Rs (864 + 770 + 1190 + 810) = Rs 14434
types of taxes. Sales tax is one of these taxes. It is See another example, Waheeda bought an air
levied at the specified rate on the sale price of the cooler for Rs 3300 including a tax (VAT) of 10% .
items and it differs from item to item and state to Find the price of the air cooler before VAT was
state. Sales tax is calculated on selling price added.
(SP). Thus, if discount is given,first discount is The price includes the VAT, ie, the Value Added
calculated and then sales tax is calculated on the Tax. Thus, a 10% VAT means if the price without
selling price of the article. If there is no discount, VAT is Rs 100 then price including VAT is Rs
then sales tax is calculated on the marked (list) 110.
price of the article. Thus, the sales tax is charged Now, when pr ice including VAT is Rs 110,
by the government on the sale of an item. It is original price is Rs 100.
collected by the shopkeeper from the customer and Hence, when price including tax is Rs 3300,
given to the government. This is, therefore, always
on the selling price of an item and is added to the  100 
the original price = Rs   330  = Rs 3000.
value of the bill. These days, however, the prices  110 
Discount 303

Solved Examples
Ex. 1: At a clearance sale, all goods are on Ex. 5: A trader marks his goods at 20% above
sale at 45% discount. If I buy a skirt the cost price. If he allows a discount
marked Rs 600, how much would I need of 5% for cash payment, what profit
to pay? per cent does he make?
Soln: We have, Soln: Let the CP of the article be Rs 100.
MP = Rs 600, Discount = 45%  Marked price of the article = Rs 120.
 Discount = 45% of Rs 600 The trader allows a discount of 5% .
 SP = (100 – 5)% of Rs 120
 45 
= Rs   600  = Rs 270
 100   120  95 
= Rs   = Rs 114
 SP = MP – Discount  100 
= Rs 600 – Rs 270 = Rs 330 Since CP of the goods is Rs 100, hence
Thus, t he amount I need t o pay is the profit per cent = (114 – 100 =) 14%.
Rs 330. Ex. 6: A shopkeeper offers his customers 10%
Ex. 2: A fan marked at Rs 540 is offered at discount and still makes a profit of
Rs 496.80 due to off season. Find the 26%. What is the actual cost to him of
rate of discount offered. an article marked Rs 280?
Soln: Marked price of the fan = Rs 540 Soln: We have, marked price = Rs 280.
Off season price = Rs 496.80 Discount = 10% on marked price
Amount of off season discount
 10 
= Rs 540 – Rs 496.80 = Rs 43.20 = Rs   280  = Rs 28
 100 
43.20
 Rate of discount =  100 = 8%  SP = MP – Discount = Rs (280 – 28 )
540 = Rs 252
Ex. 3: List price of a Video cassette is Rs 100. Now, SP = Rs 252 and Gain = 26%
A dealer sells three Video cassettes for

K KUNDAN
100
Rs 274.50 after allowing discount at  CP =  SP
certain rate. Find the rate of discount 100  Gain%
allowed.
 100 
Soln: We have, = Rs   252 
List price of one Video cassette = Rs 100  100  26 
 List price of three Video cassettes
 100 
= Rs 300 = Rs   252  = Rs 200
SP of three Video cassettes = Rs 274.50  126 
 Discount = Rs (300 – 274.50) = Rs 25.50 Hence, the actual cost of the article is Rs
200.
 25.50 
 Rate of discount =  100  = 8.5% Ex. 7: The marked price of a watch is Rs 400.
 300  After allowing a discount of 25% on
Ex. 4: After allowing a discount of 12% on the the marked price, there was a loss of
marked price of an article, it is sold Rs 20. Determine the loss per cent.
for Rs 880. Find its marked price. Soln: Marked price of watch = Rs 400
Soln: Let the marked price be Rs 100. Discount of 25% of the marked price
Discount = 12% on Marked Price
 25 
= 12% of Rs 100 = Rs 12 = Rs  400   = Rs 100
 SP = MP – Discount = Rs (100 – 12 )  100 
= Rs 88 Selling price of the watch
Now, = Rs (400 – 100 =) 300
When SP is Rs 88, MP = Rs 100 Cost price of the watch = Selling price of
the watch + Loss
100
When SP is Re 1 MP = Rs = Rs 300 + Rs 20 = Rs 320
88
When SP is Rs 880, MP  20 
 required loss per cent =  100 
 320 
 100 
= Rs   880  = Rs 1000
 88   25  1
=  6 %
Hence, the marked price of the article is  4  4
Rs 1000.
304 Concept of Arithmetic

Ex. 8: A shopkeeper marks his goods at such SP of the article = MP – Discount


a price that after allowing a discount
 x 7x
of 12.5% for cash payment, he still = Rs  x   = Rs
m akes a pr ofi t of 10%. Fi nd t he  8 8
marked price of an article which costs
 7x 
him Rs 245. Profit = SP – CP = Rs   245 
Soln: We have,  8 
CP of the article = Rs 245
Gain = 10%   7x  
  8  245  
 
100  Gain %
 CP Profit % =  245
 100 
 SP =  
100  
 100  10  Now, according to the question,
= Rs   245 
 100 
  7x  
 110    8  245  
 245  = Rs 269.50  
= Rs    100  = 10
 100   245 
Now, let the marked price be Rs 100.  
Then, discount allowed
= 12.5% of MP = Rs 12.5 70x
or,  2450 = 245
 SP of the article = MP – Discount 8
= Rs 100 – 12.5 = Rs 87.5
Thus,  (2450  245 )  8 
When SP is Rs 87.5, MP = Rs 100 or, x = Rs  
 70 
100
When SP is Re 1, MP = Rs  2695  8   21560 
87.5 = Rs   = Rs  
 70   70 
When SP is Rs 269.50, MP
= Rs 308

K KUNDAN
 100  Hence, the marked price of the article is
= Rs   269 .50  = Rs 308
 87 .5  Rs 308.
Ex. 9: A cycl e m erchant al lows 25%
Hence, marked price of the article is Rs
commission on his advertised price and
308.
still makes a profit of 20%. If he gains
Alternative Method I:
Rs 60 over the sale of one cycle, find
Let the Marked Price (MP) be Rs 100.
his advertised price.
Then, Discount = 12.5
Soln: Let the advertised price be Rs 100.
 SP = Rs (100 – 12.5) = Rs 87.5
Commission on advertised price = 25%
 SP  100   8.75  100  = Rs 25
 CP =  100  Profit %  = Rs  100  10   SP = Advertised price – Commission
   
= Rs 100 – Rs 25 = Rs 75
 875  We have, profit = 20%
= Rs  
 11  100
 CP =  SP
875 100  Gain%
 If CP is , then marked price is Rs
11
 100 
100. = Rs   75 
 100  20 
 If CP is Rs 245, then marked price is
 100  11  245   100 
=   75  = Rs 62.5
Rs   = Rs 308
 120 
 875 
Hence, the marked price of the article is  Gain = SP – CP = Rs 75 – 62.5
Rs 308. = Rs 12.5
Alternative Method II: Now,
Let the Marked Price of the article be Rs If the gain is Rs 12.5, advertised price
x, we have, = Rs 100
CP of the article = Rs 245 If the gain is Re 1, advertised price

 12 .5  x 100
Discount = Rs   x  = Rs = Rs
 100  8 12 .5
Discount 305

If the gain is Rs 60, advertised price


 100  10 
100 Soln: Selling price = 480   = Rs 432
 60 = Rs 480  100 
= Rs
12.5
Hence, advertised price of the cycle is Rs  100 
Cost price = 432   = Rs 400
480.  100  8 
Ex. 10: A cycl e m erchant al lows 25% If there is no discount, SP = Rs 480
commission on his advertised price and
480  400
still makes a profit of 20%. If he gains  % profit =  100 = 20%
Rs 60 over the sale of the one cycle, 400
find his cost price. Ex. 13: A deal er bought a hor se at 20%
Soln: Let the advertised price be Rs 100. discount on its original price. He sold
Commission on advertised price = 25% it at a 40% increase on the original
= Rs 25 price. What percentage of profit did he
 SP = Advertised price – Commission get?
= Rs 100 – Rs 25 = Rs 75 Soln: Let the original CP = Rs 100
We have, profit = 20% Dealer’s CP = 100 – 20% of 100 = Rs 80
Dealer’s SP = 100 + 40% of 100 = Rs 140
100
 CP =  SP
100  Gain% 140  80
Dealer’s profit % = = 75%
80
 100 
= Rs   75  Ex. 14: If a discount of 10% is given on the
 100  20  m arked pr i ce of an ar t i cl e, t he
shopkeeper gets a profit of 20%. Find
 100 
= Rs   75  = Rs 62.5 hi s per cent profi t i f he offers a
 120  discount of 20% on the same article.
 Profit = SP – CP Soln: Suppose the marked price = Rs 100
= Rs 75 – Rs 62.5 = Rs 12.5 Then selling price at 10% discount
Now, = Rs (100 – 10) = Rs 90

K KUNDAN
 If the gain is Rs 12.5, then CP is Rs Since he gets 20% profit, his cost price
62. 5.
 100 
 If the gain is Rs 60, then CP is Rs = 90   = Rs 75
 120 
 62.5  Now, at 20% discount, the selling price
  60  = Rs 300.
 12.5  = Rs (100 – 20) = Rs 80
Ex. 11: Sat ish marks his goods 25% abov e Thus his % profit
cost price but allows 12.5% discount 80  75 500 20 2
for cash payment. If he sells the article =  100   6 %
for Rs 875, find his cost price. 75 75 3 3
Soln: Let the cost price of goods be Rs 100. Ex. 15: An article is marked at a price which
 Marked price = Rs (100 + 25) = Rs 125 gives a profit of 25%. After allowing a
 Selling price = Rs (125 –12.5% of 125) certain discount, the profit reduces to

 125  1
= Rs 125   125  12 %. Find the discount per cent.
 1000  2
Soln: Let the CP of an article be Rs 100. Then,
 125  875 to attain a profit of 25% , marked price
= Rs 125   = Rs
 8  8 must be 25% more than the cost price.
 marked price = Rs 100 + Rs 25
875 = Rs 125
 If the selling price is Rs 8
, then
After allowing a certain discount the profit
cost price = Rs 100 1
 If the selling price is Rs 875, then cost reduces to 12 %.
2
 8  100  875  1
price = Rs   = Rs 800
Therefore, Profit = 12 % of CP
 875  2
Ex. 12: The marked price of a radio is Rs 480.
1 1
The shopkeeper allows a discount of = 12 % of Rs 100 = Rs 12
10% and gains 8%. If no discount is 2 2
allowed, find his gain per cent.
306 Concept of Arithmetic

1  200 
 SP = CP + Profit = Rs 100 + 12  
2 7 100  = 40%
 increase per cent = 
 500 
1  
= Rs 112  7 
2
Ex. 17: If a shopkeeper marks the price of goods
Now, MP = SP + Discount
50% more than their cost price and
 Discount = MP – SP
allows a discount of 40%, what is his
1 1 gain or loss per cent?
= Rs 125 – Rs 112 = Rs 12 Soln: Let the cost price be Rs x.
2 2
 Marked price = Rs (x + 50% of x)
 Discount 
 Discount % =  100   50x  3x
 MP  = Rs  x   = Rs
 100  2
 1  Discount = 40% of the marked price
 12 
=  2 100  = 10 3x
 125  = 40% of Rs
  2
 
 3x 40  3x
Hence, discount = 10% =    = Rs
Ex. 16: A tradesman allows a discount of 15%  2 100  5
on the written price. How much above Selling Price = Marked Price – Discount
the cost price must he mark his goods
 3x 3x  9x
to make a profit of 19%? =    = Rs
Soln: Let the CP be Rs 100  2 5  10
We have, Gain = 19% of CP = Rs 19 Since Selling Price < Cost Price, there is
 SP = CP + Gain = Rs 100 + Rs 19 a loss and it is given by
= Rs 119 9x 
 x
The trader allows a discount of 15%. This Loss = CP – SP = Rs  x   = Rs

K KUNDAN
 10  10
means that when marked price is Rs 100,
then SP is Rs 85.
Now,  x 
 
If Rs 85 is the SP, then marked price  Loss % =  10 100  = 10%
= Rs 100  x 
 
If Re 1 is the SP, then marked price  
100 Hence, there is a loss of 10% .
= Rs
85 Ex. 18: A shopkeeper offers 5% discount on all
If Rs 119 is the SP, then his goods t o al l hi s cust om er s. He
offers a further discount of 2% on the
 100 
Marked Price = Rs  119  = Rs 140 reduced price to those customers who
 85  pay cash. What will you actually have
Hence, the trader must mark his goods to pay for an article in cash if its MP
40% above the cost price. (Marked Price) is Rs 4800?
Alternative Method: Soln: We have, MP of the article = Rs 4800
Let the marked price be Rs 100. First discount = 5% of MP
Discount = 15% = 5% of Rs 4800
 Selling Price = Rs 100 – Rs 15 = Rs 85
Profit = 19%  5 
= Rs   4800 
 100 
 100 
 Cost Price =   SP  = Rs 240
 100  Profit %  Net price after discount
= Rs 4800 – Rs 240 = Rs 4560
 100  500 Second discount = 2% of Rs 4560
=   85  = Rs
 119  7
 2 
Increase in marked price with respect to = Rs   4560 
cost price  100 
= Rs 91.20
500 200 Net price after discount
= Rs 100 – Rs = Rs
7 7 = Rs 4560 – Rs 91.20 = Rs 4468.80
Discount 307

Alternative Method: ( By Rule of Soln: Marked price of the scooter = Rs 18000


Fraction) First discount = 10% of Rs 18000
Marked Price = Rs 4800
 10 
First discount = 5% = Rs  18000 
Second discount = 2%  100 
Net selling price of the goods is the actual = Rs 1800
price you have to pay Net price after first discount
 Selling Price = Rs 18000 – Rs 1800 = Rs 16200
Second discount = 5% of Rs 16200
 100  First discount 
= Marked Price  
 5 
 100  16200 
= Rs 
 100 
 100  Second discount 
  = Rs 810
 100  Net price after second discount
= Rs 16200 – Rs 810 = Rs 15390
  100  5   100  2   Third discount = 2% of Rs 15390
= Rs 4800    
  100   100  
 2 
= Rs  15390 
 4800  95  98   100 
= Rs  
 100  100  = Rs 307.80
= Rs 4468.80 Net price after third discount
Ex. 19: Find the single discount equivalent to = Rs (15390 – 307.80) = Rs 15082.20
successive discounts of 15% and 20%. Hence, net selling price is Rs 15082.20
Soln: Let the marked price be Rs 100. Alternative Method: (By Rule of Fraction)
Then, first discount = 15% of Rs 100 Net selling price
= Rs 15  100  10   100  5 
Net price after first discount = Rs 18000   
= Rs 100 – Rs 15 = Rs 85  100   100 
Second discount = 20% of Rs 85

K KUNDAN
 100  2 
 20   
= Rs   85   100 
 100 
= Rs 17 90 95 98
= Rs 18000   
Net price after second discount 100 100 100
= Rs 85 – Rs 17 = Rs 68
Total discount = Rs (15 + 17) = Rs 32 18  9  95  98
= Rs
Hence, the equivalent single discount 100
= Rs 32 on MP of Rs 100 = 32% = Rs 15082.20
Alternative Method: ( By Rule of Ex. 21: Two successive discounts of 20% and
Fraction) 5% are allowed on an article whose
Let the marked price be 100 net selling price is Rs 380. Find its
First discount = 15% and marked price.
Second discount = 20% Soln: Let the marked price be Rs 100.
Net selling price of the goods is the actual Then, first discount = 20% of Rs 100
price you have to pay = Rs 20
 Selling Price Net price after first discount
= Rs (100 – 20) = Rs 80
  100  15   100  20   Second discount = 5% of Rs 80
= Rs 100    
  100   100  
 5 
= Rs   80 
100  85  80   100 
= Rs   = Rs 68
 100  100  = Rs 4
 Equivalent discount Net price after second discount
= Marked Price – Selling Price = Rs (80 – 4) = Rs 76
= 100 – 68 = 32% Thus, net selling price after two discounts
Ex. 20: An ol d scoot er i s sol d at t hr ee = Rs 76
successive discounts of 10%, 5% and Now,
2%. If the marked price of the scooter If SP of Rs 76, MP = Rs 100
is Rs 18000, find its net selling price.
308 Concept of Arithmetic

100 Ex. 23: A dealer buys an article listed at Rs


If SP is Re 1, MP = Rs 100 and gets successive discounts of
76
10% and 20%. He spends 10% of the
 100  cost price on transportation etc. At
If SP is Rs 380, MP = Rs   380 
 76  what price should he sell the article to
= Rs 500 earn a profit of 15%?
Hence, marked price = Rs 500. Soln: List price of the article = Rs 100
Alternative Method: (By Rule of Fraction) First discount = 10% of Rs 100 = Rs 10
Selling Price = Rs 380 Net price after first discount
Let the marked price be Rs x. = Rs (100 – 10) = Rs 90
Now, according to the question, Second discount = 20% of Rs 90

 100  20   100  5   20 
= Rs   90  = Rs 18
380 = x     100 
 100   100 
Net price after second discount
80 95 = Rs (90 – 18) = Rs 72
or, 380 = x  
100 100 Transportation cost = 10% of Rs 72

380  100  100  10 


 x = = Rs 500 = Rs   72 
80  95  100 
Ex. 22: The list price of a watch is Rs 160. = Rs 7.20
After two successive discounts, it is  CP of the article = Rs (72 + 7.20)
sold for Rs 122.40. If the first discount = Rs 79.20
i s 10%, what i s the rat e of second Profit = 15%
discount? 100  Profit %
Soln: List price of watch = Rs 160.  SP =  CP
First discount = 10% of Rs 160 100

 10   100  15 
160  = Rs   79 .20  = Rs 91.08
= Rs 

K KUNDAN
 100   100 
= Rs 16 Ex. 24: An invoice for a machinery plant is
 Net price after first discount stated as follows: Cost of machinery
= Rs (160 – 16) = Rs 144 plant = Rs 18575.
SP of watch = Rs 122.40 2 1 n
Second discount = Rs (144 – 122.40) Terms: Cash 3, , , . Find
10 20 30
= Rs 21.60
Since second discount is on Rs 144 ie the discount if,
price after first discount. (i) the bill is paid immediately.
 Rate of second discount (ii) the bill is paid on the 9th day.
(iii) the bill is paid on the 17th day.
 21.60  (iv) the bill is paid on the 28th day.
=   100 % = 15%
 144  Soln: Cost of the plant = Rs 18575
Alternative Method: (By Rule of Fraction) (i) As the bill is paid immediately, cash
List price = Rs 160, discount of 3% is allowed.
The first discount = 10% and  18575  3 
the selling price = Rs 122.40  discount = Rs  
 100 
Let the rate of second discount be x% .
Now, according to the question, = Rs 557.25
(ii) As the payment is made of 9th day,
 100  10   100  x  2% discount is allowed.
122.40 = 160     
 100   100 
 18575  2 
 discount = Rs  
90 (100  x )  100 
or, 122.40 = 160  
100 100 = Rs 371.50
(iii) When the payment is made on 17th
12240  100 day, 1% discount is allowed.
or, 100 – x = = 85
160  90
 18575  1 
 x = 100 – 85 = 15%  discount = Rs  
Hence, the second discount is 15% .  100 
= Rs 185.75
Discount 309

(iv) In case the payment is made after 20 Hence, the basic price of the TV set is Rs
days, no discount is given. 12300.
 When the payment is made on the Ex. 27: Sam i r bought a shir t for Rs 336,
28th day, there is no discount. including 12% sales tax and a necktie
Ex. 25: George bought a VCR at the list price for Rs 110 including 10% sales tax.
of Rs 18500. If the rate of sales tax Find the printed price (without sales
was 8%, find the amount he had to tax) of shirt and necktie together.
pay for purchasing the VCR. Soln: Let the printed price of the shirt be Rs x
Soln: List price of VCR = Rs 18500 and that of necktie be Rs y. Then,
Rate of sales tax = 8% Sales tax on shirt = 12% of Rs x
 Sales tax = 8% of Rs 18500
12x 3x
8 = Rs = Rs
 18500 = Rs 1480 100 25
=
100 Sales tax on necktie = 10% of Rs y
So, total amount which George had to pay
10y y
for purchasing the VCR = Rs = Rs
= Rs 18500 + Rs 1480 = Rs 19980 100 10
Ex. 26: The price of a TV set inclusive of sales  Selling price of shirt
tax is Rs 13530. If the rate of sales 3x 
 28x
tax is 10%, find its basic price. = Rs  x   = Rs and,
Soln: Let the basic price of TV set be Rs x.  25  25
Th en , Selling price of necktie
Sales tax at the rate of 10% on Rs x y 
 11y
= Rs  y   = Rs
 10   x   10  10
= Rs   x  = Rs  
 100   10  But selling prices of shirt and necktie are
Thus, the sale price of the TV set Rs 336 and Rs 110 respectively.

 x  11x 28x 11y


= Rs  x   = Rs  = 336 and = 110

K KUNDAN
 10  10 25 10
It is given that the sale price of the TV set 336  25 110  10
is Rs 13530. or, x = and y =
28 11
11x
 = 13530 or, x = 300 and y = 100
10 Hence, the total printed price of the shirt
13530  10 and necktie
or, x = = 12300 = Rs (300 + 100) = Rs 400
11

Practice Exercise

1. List price of a Video cassette is Rs 100. A Find the marked price of the article which
dealer sells three Video cassettes for Rs 274.50 costs him Rs 1400.
after allowing discount at certain rate. Find 5. A trader marks his goods at 25% above the
the rate of discount allowed. cost price. If he allows a discount of 8% for
2. A shopkeeper marks his goods 20% above his cash payment, what profit per cent does he
cost price. He gives 15% discount on the make?
marked price. Find his gain per cent. 6. A dealer marks his goods 20% above the cost
3. A shopkeeper allows a discount of 10% on price. He then allows some discount on it
the marked price of an item but charges a and earns a profit of 14%. What is the rate of
sales tax of 8% on the discounted price. If a discount offered by the dealer?
customer pays Rs 680.40 as the price of the 7. On selling an article at a discount of 20% ,
item including sales tax, find the marked the profit is 20% . Find the profit per cent if
price of the item. the article is sold at a discount of 10%.
4. A shopkeeper marks his goods at such a price 8. By selling an umbrella f or Rs 300, a
shopkeeper gains 20% . During a clearance
1
that after allowing a discount of 12 % on sale, the shopkeeper allows a discount of 10%
2 on the marked price. Find his gain per cent
the marked price, he earns a profit of 20% . during the sale season.
310 Concept of Arithmetic

9. How much per cent above the cost price must profit when all the goods are sold and the
a person mark his goods so that even after amount realised?
giving a discount of 10% , a profit of 10% is 19. Reena goes to a shop to buy a radio, costing
made? Rs 2568. The rate of sales tax is 7% . She
10. A trader bought some goods at a discount of tells the shopkeeper to reduce the price of
20% of the list price. He wants to mark them the radio to such an extent that she has to
at such a price that he can give a discount of pay Rs 2568, inclusive of sales tax. Find the
20% on the marked price and still make a reduction needed in the price of the radio.
profit of 25% . Find the per cent of the list 20. David purchased a pair of shoes for Rs 441
price at which he should mark the goods. including sales tax. If the sales price of the
11. In a shop, the prices of all goods are marked shoes is Rs 420, find the rate of sales tax.
15% above the cost price and 5% discount is 21. Amit purchases a motorcycle, having marked
allowed on all sales. In a certain month, the price Rs 46000 at a discount of 5%. If sales
business expenses of the shop amounted to tax is charged at the rate of 10% , find the
Rs 6,500 and the owner realised a profit of amount Amit has paid to pur chase the
6%. Find the cost prices of goods sold by the motorcycle.
shop during the month. 22. The list price of an air-conditioner is Rs
12. A company gives discount to its customers at 25630. The rate of sales tax is 10% . The
15% on the list price and thus makes a profit cust omer r equest s the dealer t o allow a
of 19% . If the cost of production goes up by discount to such an extent that the cost of
12% , company issues a new price list in the air-conditioner amount s to Rs 25630
which cost of all goods have been increased inclusiv e of sales t ax. Find the rat e of
by 10%. If company continue to give discount discount.
of 15% on t he list pr ice, f ind the pr ofit 23. Shilpa buys a washing machine quoted at Rs
percentage.
4 2 n
13. Which is the profitable bargain for a consumer 16800 on the terms of cash 5, , , .
(i) The successive discount s of 20% and 10 20 30
15% . Find the amount she will have to pay if the
(ii) The successive discount s of 10% and payment is made on the 7th day.

K KUNDAN
25% . 24. A shopkeeper fixed selling price of his goods
14. Two dealers offer an article at the same list at 20% above cost price. He sells half the
price. The first allows discount 20% , 10% stock at this price, one quarter of the stock at
and 5% , the other of 15% , 12% and 8% . the discount of 15% and the remaining at
Which is a better offer for the customer? the discount of 50% on the marked selling
15. The marked price of an article is Rs 800. A price. What is his gain or loss per cent?
retailer purchases it after two successive 25. A sells an item at Rs 100 less than the list
discounts for Rs 540. The first discount of it price and receives 10% of his selling price
was 25%. Find the rate of second discount. as commission. B sells the same item at Rs
16. By how much above the cost should the goods 200 less than the list price and receives 20%
be marked for sale, so that after allowing a of his selling price as his commission. If they
trade discount of 20% and a cash discount of both get the same commission. What is the
6.25%, a net gain of 20% on the cost is made? list price of the item?
17. A dealer buys a table listed at Rs 1500 and 26. Two retailers A and B purchase one computer
gets successive discounts of 20% and 10% . each from a wholesaler at the r ate of Rs
He spends Rs 20 on transportation and sells 40000. First retailer A sells the computer at a
it at a profit of 10%. Find the selling price of profit of 20% while t he second ret ailer B
the table. enhances t he rat e by 30% and sells the
18. A manufacturer marks his goods at 40% above computer at 8% rebate. Find the profit of each
the cost price. He allows a discount of 10% A and B.
for the cash customers and 5% to the credit 27. A shopkeeper marks his goods at 20% above
his cost price. He sells three-fourth of his
3
customers. th of the goods are sold for cash goods at t he mar ked pr ice. He sells t he
5 remaining goods at 50% of the marked price.
and rest on credit. What is the percentage of Determine his profit per cent on the whole
transaction.
Discount 311

Answers and explanations


1. We have, 5. Let the cost price be Rs x.
List price of one Video cassette = Rs 100 Marked price = Rs (x + 25% of x)
 List price of three Video cassettes = Rs 300
 25x  5x
SP of three Video cassettes = Rs 274.50 = Rs  x   = Rs
 Discount = Rs (300 – 274.50) = Rs 25.50  100  4
Discount = 8% of the marked price
 25.50 
 Rate of discount =  100  = 8.5%
 300   5x 8  x
= Rs    = Rs
2. Let the cost price be Rs 100.  4 100  10
100  120  Selling Price = Marked Price – Discount
 Marked Price = Rs = Rs 120
100  5x x   25 x  2x 
Rate of discount = 15% = Rs    = Rs  
 4 10   20 
120  85
 Selling Price = Rs = Rs 102 23x
100 = Rs
20
 Gain % = Rs 102 – Rs 100 = 2%
3. Let the marked price of the item be Rs 100.  23 x  3x
Discount = 10%  Gain = SP – CP = Rs   x  = Rs
 20  20
 Discounted price for the shopkeeper
= Rs (100 – 10) = Rs 90
Sales tax = 8% of the discounted price  3x 
 
= 8% of Rs 90 Gain % =  20 100  = 15%
 x 
 8   
 90  = Rs 7.20  
= Rs 
 100 
Alternative Method:
 Selling price for the shopkeeper

K KUNDAN
Let the CP of the article be Rs 100.
= Rs 90 + Rs 7.20 = Rs Rs 97.20  Marked price of the article = Rs 125
Now, The trader allows a discount of 8% .
 If selling price is Rs 97.20, then marked  SP = (100 – 8)% of Rs 125
price is Rs 100.
 If selling price is Rs 680.40, then marked 125  92
= Rs = Rs 115
100
 680 .40  100 
price is Rs   = Rs 700. Since the CP of the article is Rs 100, hence
 97 .20  the profit per cent = (115 – 100 =) 15%
 Marked Price = Rs 700. 6. Let the cost price be Rs x.
4. We have, CP of the article = Rs 1400  Marked price = Rs (x + 20% of x)
Gain = 20%
 20x  6x
 100  Gain%  = Rs  x   = Rs
  CP  100  5
 SP = 
 100  Gain % = 14%
 100  20 
= Rs   1400   100  Gain% 
 100  Selling Price =   CP 
 100 
 120  1400 
= Rs   = Rs 1680  100  14  57x
 100  = Rs   x  = Rs
 100  50
Now, let the marked price be Rs 100.
Then, discount allowed = 12.5% of MP Discount = Marked Price – Selling Price
= Rs 12.5  6x 57 x  3x
 SP of the article = MP – Discount = Rs    = Rs
 5 50  50
= Rs (100 – 12.5) = Rs 87.5
Thus, 3x
 When SP is Rs 87.5, MP = Rs 100
50  100  3x  5  100
Rate of discount =
 100  6x 50 6x
 When SP is Rs 1680, MP = Rs  1680 
 87.5  5
= Rs 1920 = 5%
312 Concept of Arithmetic

7. Suppose the marked price = Rs 100 He wants to make a profit of 25% on this cost
Then selling price at 20% discount price.
= Rs (100 – 20) = Rs 80
 25  80 
Since he gets 20% profit, his cost price  Selling price = Rs  80  
 100 
 100   80  5  200
= Rs 80   = Rs   = Rs = Rs (80 + 20) = Rs 100
 100  20   6  3 This becomes final selling price of the goods
Now, at 10% discount, the selling price after discount of 20% .
= Rs (100 – 10) = Rs 90 Now, let the marked price of goods be Rs x.
Profit = Selling Price – Cost Price Now, according to the question,
 200  70 20  x
= Rs  90   = Rs x  100
 3  3 100
Thus, his per cent profit
100 x  20 x
or,  100
70 100
3  100  70  3  100 or, 80x  10000
= = 35%
200 3 200
3 10000
or, x =  125
8. Selling price of the umbrella = Rs 300 80
Gain% = 20%  The marked price of the goods should be
Rs 125.
 100  Hence the trader should mark the price of
Cost price = Rs 300    = Rs 250
 100  20  the goods (125 – 100 =) 25% above the list
Here, marked price is the selling price price.
= Rs 300. 11. Let the cost price of the goods sold during
 New selling price (during the sale season) the month be Rs 100x.
Marked price, 15% above the cost price
 100  10  = Rs 115x
= Rs 300    = Rs 270

K KUNDAN
 100  Sales price after 5% discount on marked price
 Gain = Selling Price – Cost Price
95  115 x
= Rs (270 – 250) = Rs 20 
100
20 Expenses during the month = Rs 6500
 Gain % =  100 = 8%
250 Owners’ profit = 6%
9. Let the CP be Rs 100. So, according to the question,
We have, Gain = 10% of CP = Rs 10 95  115x
SP = CP + Gain = Rs 100 + Rs 10 = Rs 110  6500  106x
The person allows a discount of 10% . This 100
means that when marked price is Rs 100, or, 95 × 115x – 106x × 100 = 650000
then SP is Rs 90. or, 10925x – 10600x = 650000
Now, or, 325x = 650000
If Rs 90 is the SP, then marked price is Rs 650000
100. or, x = = Rs 2000
325
If Rs 110 is the SP, then marked price is
But CP is 100x = 100 × 2000 = Rs 200000.
 100  1100 12. Let the marked price be Rs 100
Rs  110  = Rs
 90  9  Selling price = Rs (100 – 15) = Rs 85
Hence, the person must mark his goods 100
Cost price at 19% profit = 85  (100  19)
 1100  200 2
  100    22 % above the cost
 9  9 9
85  100 500
price. = = Rs
119 7
10. Let the list price be Rs 100.
Cost price after 20% discount New cost of production after increase of 12%

 20  500 112
=  = Rs 80
= Rs 100   100 
7 100
 100 
= Rs (100 – 20) = Rs 80 New list price is Rs (100 + 10) = Rs 110
Discount 313

New selling price after discount of 15% Cost price after 8% discount
 100  15  110  85 8  74.80
= 110    = = Rs 93.50 = 74.80  = Rs 68.816
 100  100 100
Profit = Selling Price – Cost of Production It can be seen from the above that the first
= Rs (93.50 – 80) = Rs 13.50 discount series is more favourable.
15. Marked price of an article = Rs 800
13.50
Profit % =  100 = 16.875% First discount = 25% of Rs 800
80
 25 
13. ( i ) Let the list price be Rs 100 = Rs  800   = Rs 200
In first case,  100 
Net price after first discount
20
First discount of 20% = Rs 100  = Rs (800 – 200) = Rs 600
100 SP of article = Rs 540
= Rs 20 Second discount = Rs (600 – 540) = Rs 60
Price after first discount = Rs (100 – 20) Since second discount is on Rs 600 ie price
= Rs 80 after first discount.
Now, second discount of 15%  60 
 Rate of second discount =   100 %
 15  80   600 
= Rs   = Rs 12 = 10%
 100 
16. Let the marked price be Rs 100
Net price = Rs (80 – 12) = Rs 68 Price after discount of 20%
( i i) First discount 10%
 20  100 
10 = Rs 100   = Rs 80
= Rs 100  = Rs 10  100 
100
Price after discount of 6.25%
Price after first discount = Rs (100 – 10)
= Rs 90  6.25  80 
Now, = Rs  80   = Rs 75
 100 

K KUNDAN
Second discount of 25% =
25  90
100
= Rs 22.50
Net price = Rs 90 – 22.50 = Rs 67.50
This way it can be seen that the second
offer is more beneficial to the consumer.
14. Let the marked price be Rs 100
First discount series 20%, 10% and 5%
 Single equivalent discount

 SP = Rs 
= (100 – 75) = 25%
Again, let the CP be Rs 100.
We have gain % = 20%
 100  20 
 100 
  100 = Rs 120

Now a discount of 25% is allowed. This means


that when marked price is Rs 100, then SP
Cost price after 20% discount
is Rs (100 – 25) = Rs 75
20  100 Now,
= 100  = Rs 80
100  If Rs 75 is the SP, then maked price is
Cost price after 10% discount Rs 100.
 If Rs 120 is the SP, then maked price is
10  80
= 80  = Rs 72  100 
100 Rs  120  = Rs 160
 75 
Cost price after 5% discount
Hence marked price will be (160 – 100) = 60%
5  72 more than the cost price.
= 72  = Rs 68.40
100 17. Marked Price = Rs 1500
Second discount series 15%, 12% and 8% Successive discounts = 20% and 10%
Cost price after 15% discount After 20% discount,
Price = 80% of Rs 1500
15  100
= 100  = Rs 85  80 
100 = Rs  1500  = Rs 1200
Cost price after 12% discount  100 
After 10% discount,
12  85 Price = 90% of Rs 1200
= 85  = Rs 74.80
100
 90 
= Rs  1200  = Rs 1080
 100 
314 Concept of Arithmetic

Transportation cost = Rs 20  Selling price of the radio


 Effective cost price = Rs 1080 + Rs 20
 7x  107x
= Rs 1100 = Rs  x   = Rs
Profit = 10%  100  100
But the selling price of the set is Rs 2568.
 100  Profit% 
 Selling Price =    Cost Price 107x
 100   = 2568
100
 100  10 
= Rs   1100  2568  100
 100  or, x = = Rs 2400.
107
 110  Hence, the reduction needed in the price of
= Rs  1100  = Rs 1210
 100  the radio = Rs (2568 – 2400) = Rs 168
18. Let the cost price for manufacturer be Rs 100 2 0 . Let the rate of sales tax be x%. Then
Marked price of the goods Sales tax = x% of Rs 420

40  100  x  21x
= Rs   420  = Rs
= 100  = Rs (100 + 40) = Rs 140  100  5
100
According to the question,  21x 
 Selling price of shoes = Rs  420  
3  5 
He sells th of his goods on cash at 10% But selling price of shoes is Rs 441.
5
discount 21x
 420  = 441
5
3
 Marked price of th goods
5 21x
or, = 21
5
3
=  140 = Rs 84  x = 5
5
Hence, the rate of sales tax is 5%.

K KUNDAN
10% discount = Rs 8.40 21. Marked price of motorcycle = Rs 46000
3 Discount = 5% of Rs 46000
Selling price of th goods  5 
5  46000  = Rs 2300
= Rs 
= Rs 84 – Rs 8.40 = Rs 75.60  100 
 Net price of motorcycle
2 = Rs 46000 – Rs 2300 = Rs 43700
He sells th of the goods on credit at 5%
5 Sales tax = 10% of Rs 43700
discount.  10 
= Rs   43700  = Rs 4370
2  100 
So, marked price of th goods  SP of motorcycle = Rs (43700 + 4370)
5
= Rs 48070
2 Hence, Amit has paid Rs 48070 to purchase
=  140 = Rs 56 the motorcycle.
5
2 2 . The list price of air-conditioner = Rs 25630
5 Rate of sales tax = 10%
5% discount = 56  = Rs 2.80 Selling price of air-conditioner with sales tax
100
25630  110
2 = Rs = Rs 28193
Selling price of th goods 100
5
Let the rate of discount given = x%
= Rs (56 – 2.80) = Rs 53.20
According to the question,
Selling price of goods under both category
= Rs (75.60 + 53.20) = Rs 128.80 28193  x
28193 – = 25630
Profit = Selling price – Cost price 100
= Rs (128.80 – 100) = Rs 28.80 or, 2819300 – 28193x = 100 × 25630
Profit = 28.80%
19. Let the reduced price, excluding the sales 2819300  2563000 100 1
or, x =  9
tax, of the radio be Rs x. Then, 28193 11 11
7x 1
Sales tax = 7% of Rs x = Rs Hence, the rate of discount = 9 %
100 11
Discount 315

23. Cost price of washing machine = Rs 16800.


 20 
As the payment is made on the 7th day, cash B’s commission = Rs   ( x  200 )
discount of 4% is allowed. 100 

 16800  4   x  200 
 Discount = Rs   = Rs 672 = Rs  
 100   5 
 Required amount paid by her Now, according to the question,
= Rs (16800 – 6782) = Rs 16128
x  100 x  200
24. Let the shopkeeper sell 100 articles of cost 
price Rs 100 each. 10 5
The shopkeeper fixes the marked price at 12% or, 10x – 2000 = 5x – 500
above the CP. or, 10x – 5x = 2000 – 500
 Marked price of each article or, 5x = 1500
= Rs (100 + 20) = Rs 120
1500
He sells half the stock at marked price.  x = = 300
 SP of 50 articles = Rs (120 × 50) = Rs 6000 5
Total marked price of 25 articles Hence, the marked price of the article
= Rs (25 × 120) = Rs 3000 = Rs 300
26. Cost price of first retailer A = Rs 40000
 15  He sells it at a profit of 20%.
Discount (at 15%) = Rs  3000   = Rs 450
 100 
20
 Selling price = Rs (3000 – 450) = Rs 2550  His profit = 40000  = Rs 8000
100
Remaining 25 ar t icles ar e sold at 30%
discount. CP of second retailer ie B = Rs 40000
 Marked price (total) = Rs 3000 He marks it at 30% profit.

 30  100  30
 Discount = Rs  3000   = Rs 900  His marked price = 40000 
 100  100
SP = Rs (3000 – 900) = Rs 2100 = 400 × 130 = Rs 52000

K KUNDAN
Hence, total SP = Rs (6000 + 2550 + 2100) B’s SP after giving 8% rebate becomes
= Rs 10650 (100  8)
Total CP = Rs (100 × 100) = Rs 10000 = Rs 52000 
100
Profit = Rs (10650 – 10000) = Rs 650
= Rs (520 × 92) = Rs 47840
650  100  B’s profit = Rs (47840 – 40000) = Rs 7840
 Gain % = = 6.5% 27. Let the shopkeeper have 100 articles and the
10000
25. Let the marked price of article be Rs x. cost of each article be Rs 100.
For salesman A,  Total CP = Rs (100 × 100) = Rs 10000
SP of article = Rs (x – 100) Now, CP of each article be Rs 120.
According to the question,
 10 
A’s commission = Rs   (x  100 ) 3 1 
100  Total SP = Rs   100  120   100  60 
4 4 
 x  100  = Rs (9000 + 1500) = Rs 10500
= Rs  
 Gain = Rs (10500 – 10000) = Rs 500
 10 
For salesman B, 500
SP of article = Rs (x – 200)  Gain % =  100 = 5%
10000
Ratio & Proportion and k-Method 359

Chapter-17

Ratio & Proportion and k-Method

Important Definitions and Related Concepts


1. Some Useful Results on Proportion Subtracting one from each side, we have,

If four quantities a, b, c and d are in proportion, then a c a b c d


1  1  
we can see that the following results are true: b d b d
( i ) Invertendo  a – b : b :: c – d : d.
If four quantities be in proportion they keep in This operation is called Dividendo.
proportion even when they are taken inversely. (v) Componendo and Dividendo
If a : b :: c : d, then b : a :: d : c
When four quantities are in proportion, the sum
a c of the first and second is to their difference as
Since  dividing unity by each of these equal
b d the sum of third and fourth is to their difference.
sides, we have, If a : b :: c : d, then a + b : a – b :: c + d : c – d

a c a c a b c d
Since,  ,   ....(i)

K
1: = 1 : b d b d
b d
[By Componendo]
b d
   b : a :: d : c a b c d
a c And  ....(ii)
b d
This result is called Invertendo.
[By Dividendo]
( i i) Alternendo
If four quantities be proportionals, they remain a b c d
Dividing (i) by (ii), we get 
proportionals when they are taken alternately. a b c d
If a : b :: c : d, then a : c :: b : d This operation is known as Componendo and
Dividendo.
a c b
Since  , multiplying both sides by , we
b d c a c a b c d
Also, if  , then 
get, b d a b c d

KUNDAN
This oper at ion is known as Dividendo and
a b c b a b
     Componendo.
b c d c c d
 a : c :: b : d. This result is called Alternendo. 2. The k-Method
( i ii ) Componendo This method requires that each of the given ratios be
When four quantities are in proportion, the first put equal to constant k, the values of the numerators
together with the second is to second as the third (antecedents) found in terms of k and denominators
together with the fourth is to the fourth. (consequent s) and such v alues as ar e obt ained
If a : b :: c : d, then a + b : b :: c + d : d substituted in the two sides of the equality to be proved.
As the method becomes almost mechanical after a
a c little practice, the students will find it very convenient
Given, 
b d and helpful, though not much instructive in solving
Adding one to each side, we have, problems concerning equal ratios.
Note:
a c a b c d ( i ) Ther e is simple method popular ly known as
1  1  
b d b d ‘k-method’ to solve problems on equal ratios.
 a + b : b :: c + d : d. ( i i) In this method, we assume each of the given ratios
This operation is said to be Componendo. equal to k.
( iv) Dividendo 3. Theorem On Equal Ratios
When four quantities are in proportion, the excess
of the first over the second is to the second as a c a a b c
(i) If  , then   .
the excess of the third over the fourth is to the b d b b d d
fourth.
a b c d a c e
 ( i i) If  = , then each of these ratios is equal
If a : b :: c : d, then b d f
b d
a c
Given,  a c e
b d to b  d  f .
360 SSC Advanced Maths
( vi) Three or more quantit ies are said to be in
a c e
( i ii ) If  = = ......, then each ratio is equal to continued proportion when the ratio of the first
b d f and the second is equal to the ratio of second
and t hir d and so on. Thus, a, b, c are in
a  c  e  ...... Sum of antecedent s
= . a b
b  d  f  ...... Sum of consequent s  .
continued proportion if
b c
4. Important Points a b
(vii) When  , we get b2 = a × c. If three quantities
(i) Four quantities a, b, c, d are said to be in b c
proportion (or proportional) if a : b :: c : d. In a, b, c are in continued proportion, then b is
short, proportion means equality of two ratios. said to be the mean proportional between a and
( i i) Proportion is often expressed as a : b :: c : d c. c is called the third proportional to a and b.
and is read as “a is to b as c is to d”. ( vi ii) The concept of proportion need not be restricted
( i ii ) The terms ‘a’ and ‘b’ are called extremes (end to only two equal ratios. It may be extended
terms) and ‘b’ and ‘c’ are called means (middle
terms). a c e
thus. If  = = ........, then a, b, c, d, e, f
( iv) The f our t h ter ms ‘d’ is called t he f ourth b d f
proportional to a, b and c. ...... are said to be in proportion.
(v) The f our quantit ies, a, b, c and d ar e in ( ix ) The concept of continued proportion may be
proportion if and only if the product of the extended as given below:
extremes is equal to the product of the means.
Thus, if a : b :: c : d, then ad = bc and conversely, a b c
If    ......, then a, b, c, d, .... are said to

K
b c d
a c
let ad = bc, then  . be in continued proportion.
b d

Solved Examples

a c 4a  9b 4c  9d (a  2b  3c  4d )  (a  2b  3c  4d )
Ex. 1: (a) If = , show that  . =
b d 4a  9b 4c  9d (a  2b  3c  4d )  (a  2b  3c  4d )

a c 2(a  2b ) 2(a  2b )
( b) If = , pr ov e t hat ( 2a + 3b) ( 2c – 3d)  
b d 2(3c  4d ) 2(3c  4d )
= (2a – 3b)(2c + 3d).
a  2b a  2b
 

KUNDAN
a c 3c  4d 3c  4d
Soln. (a) We have, 
b d By alternendo, we get,
4a 4c 4
  [Multiplying both sides by ] a  2b 3c  4d
9b 9d 9 
a  2b 3c  4d
4a  9b 4c  9d By componendo and dividendo, we get,
 
4a  9b 4c  9d
a  2b  a  2b 3c  4d  3c  4d
[By Componendo and Dividendo] 
a  2b  a  2b 3c  4d  3c  4d
a c
(b) We have,  2a 6c a 3c
b d    
4b 8d 2b 4d
2a 2c 2  4ad = 6bc  2ad = 3bc
  [Multiplying both sides by ]
3b 3d 3
x a x b 2ab
2a  3b 2c  3d Ex. 3: Find the value of  , when x = .
  x a x b a b
2a  3b 2c  3d
[By Componendo and Dividendo] 2ab
Soln. When x =
 (2a + 3b)(2c – 3d) = (2a – 3b)(2c + 3d) a b
[By cross-multiplication] Dividing both sides by a, we have,
Ex. 2: If (a – 2b – 3c + 4d)(a + 2b + 3c + 4d) = (a + 2b – x 2b
3c – 4d) (a – 2b + 3c – d), prove that 2ad = 3bc. 
a a b
a  2b  3c  4d a  2b  3c  4d
Soln.  x  a 3b  a
a  2b  3c  4d a  2b  3c  4d  
x a b a
By componendo and dividendo, we get,
[By componendo and dividendo]
(a  2b  3c  4d )  (a  2b  3c  4d )
x a 3b  a
(a  2b  3c  4d )  (a  2b  3c  4d )   .....(i)
x a a b
Ratio & Proportion and k-Method 361

2ab 1  px 1  qx
Again, x = (e) 1  px 1
a b 1  qx
Dividing both sides by b, we get,
x 2b x 3  3x
341
 Soln. (a) We have, 
b a b 3x 2  1 91
By componendo and dividendo, we get,
x  b 3a  b
  .....(ii)
x b a b x 3  3x  3x 2  1 341  91

[By componendo and dividendo] x 3  3x  3x 2  1 341  91
Adding (i) and (ii), we get,
x a x b 3b  a 3a  b (x  1)3 432 216
 =    
3 250 125
x a x b a b a b (x  1)

3b  a  3a  b 2a  2b 2(a  b ) 3 3
= = = = 2.  x 1 6 x 1 6
a b (a  b ) (a  b )      
 x 1  5 x 1 5

 p  2x p  2y  By componendo and dividendo, we get,


4 xy
Ex. 4: If p = x  y , find the value of  p  2x  p  2y  . x  1  x 1 6  5

x 1 x 1 6  5
4xy p 2y
Soln: p = x  y  2x  x  y 2x 11

K
   x = 11
2 1
By componnendo and dividendo, we get,
p  2x 2y  (x  y ) x  4  x  10 5
 (b) We have, 
p  2x 2y  ( x  y ) x  4  x  10 2
By componendo and dividendo, we get,
p  2x 3y  x
  ....(i) x  4  x  10  x  4  x  10 52
p  2x yx 
x  4  x  10  x  4  x  10 52
4xy p 2x
Again, p = x  y  
2y x  y x 4 7
 
By componendo and dividendo, we get, x  10 3
p  2y 2x  (x  y ) On squaring both sides, we get,

KUNDAN

p  2y 2x  (x  y ) x 4 49

x  10 9
p  2y 3x  y
  ....(ii)  9x + 36 = 49x – 490
p  2y x y
 –40x = –526
Adding (i) and (ii), we get,
526 263
p  2x p  2y 3y  x 3x  y  x 
+ = + 40 20
p  2x p  2y yx x y
x  1  x  1 4x  1
3y  x 3x  y 3y  x  3x  y (c) We have, 
= – = x  1  x 1 2
yx yx yx
By componendo and dividendo, we get,
2y  2x 2(y  x ) x  1  x  1  x  1  x  1 4x  1  2
= = = 2. 
yx yx x  1  x  1  x  1  x  1 4x  1  2
Ex. 5: Find the value of x in the following equations: 2 x  1 4x  1
 
3 2 x  1 4x  3
x  3x 341
(a) 
3x 2  1 91 x  1 4x  1


x  1 4x  3
x  4  x  10 5 On squaring both sides, we get,
(b) 
x  4  x  10 2
x  1 16x 2  8 x  1

x  1  x 1 4 x 1 x  1 16x 2  24x  9
(c) 
x 1  x 1 2 By componendo and dividendo, we get,

a x  a x x 1 x 1 (16x 2  8x  1)  (16x 2  24x  9)


(d) k =
a x  a x x 1 x 1 (16x 2  8 x  1)  (16x 2  24x  9)
362 SSC Advanced Maths
Note: x = 0, also satisfies the given equation.
2x 32x 2  16 x  10
  Ex. 6: Show that 3nx2 – 2mx + 3n = 0, if
2 32x  8
32x – 8x = 32x2 – 16x + 10
2 m  3n  m  3n
x
 –8x + 16x = 10 m  3n  m  3n
5
8x = 10  x m  3n  m  3n
4 Soln. x 
m  3n  m  3n
a x  a x By componendo and dividendo, we get,
(d) We have, k
ax  ax
x 1 ( m  3n  m  3n )  ( m  3n  m  3n )
By componendo and dividendo, we get, =
x 1 ( m  3n  m  3n )  ( m  3n  m  3n )
a  x  a  x  a  x  a  x k 1

a  x  a  x  a  x  a  x k 1 x 1 m  3n
 =
x 1 m  3n
a  x k 1 On squaring both sides, we get,
 
a  x k 1
On squaring both sides, we get, x 2  2x  1 m  3n

x 2  2x  1 m  3n
a  x k 2  2k  1
 By componendo and dividendo, we get,
a  x k 2  2k  1
(x 2  2x  1)  (x 2  2x  1)

K
By componendo and dividendo, we get, m  3n  m  3n

(x 2  2x  1)  (x 2  2x  1) m  3n  m  3n
a  x a  x k 2  2k  1  k 2  2k  1
= 2
a  x a  x k  2k  1  k 2  2k  1 x2 1 m
 
2x 3n
2a 2k 2  2
  3n(x2 + 1) = 2mx  3nx2 + 3n = 2mx
2x 4k
3nx2 – 2mx + 3n = 0.

a k2 1 2ak
   x 15 a 2  4 b 2 47
x 2k k2 1 Ex. 7: If  , find the values of
15a 2  4 b 2 7
1  px 1  qx
(e) 1  px 1  qx  1 a a 3  3b 3

KUNDAN
(i) (ii)
b 3b 3
1  px 1  qx
 1  px  1  qx 15a 2  4b2 47
Soln. (i) 
15a 2  4b 2 7
On squaring both sides, we get,
By componendo and dividendo, we get,
1  p2x 2  2 px 1  qx
 (15a 2  4b 2 )  (15a 2  4b 2 ) 47  7
1  p 2x 2  2 px 1  qx
2 2 2 2

(15a  4b )  (15a  4b ) 47  7
By componendo and dividendo, we get,
30a 2 54 54 8 9 a2
(1  p 2x 2  2 px )  (1  p 2x 2  2 px )      
2
2 2 2 2 8b 40 b 2 40 30 25
(1  p x  2 px )  (1  p x  2 px )
Taking the positive square roots of both the
1  qx  1  qx sides, we get,
=
1  qx  1  qx a 9 3
  
b 25 5
1  p 2x 2 1 2p
   1 + p2x2 =
2 px qx q a 3 a3 27
(ii)   3 
b 5 b 125
2p 2p  q
 p2x2 = – 1 =
q q
a327 9
  
2 2p  q 3b 3 375 125
 x 
p2q Using componendo, we get,

a 3  3b 3 9  125 a 3  3b 3 134
1 2p  q 3 =  =
 x p q 3b 125 3b 3 125
Ratio & Proportion and k-Method 363

x2  y2 xy x y p q x2  y2  z2  px  qy  rz 
2
Ex. 8: If  , prove that x  y  p  q . (ii)  
p2  q2 pq a 2 2
b c 2
 pa  qb  rc 

x 2  y2 xy x 2  y2 p2  q 2 x y z
Soln.    Soln. (i) Since   = k (say)
2 2 pq xy pq a b c
p q
 x = ak, y = bk, z = ck
x 2  y2 p2  q2 3
 
2xy 2 pq  a 2x 2  b 2y 2  c 2z 2  2
LHS =  
By componendo and dividendo, we get,  a 3 x  b 3y  c 3z 
x 2  y 2  2xy p 2  q 2  2 pq
 3
x 2  y 2  2xy p 2  q 2  2 pq  a 2 . a 2k 2  b 2 . b 2k 2  c 2 . c 2k 2  2
=  
 a 3 . ak  b 3 . bk  c 3 . ck 
(x  y )2 ( p  q )2
 
2
(x  y ) ( p  q )2 3
3
 k 2 (a 4  b 4  c 4 )  2
x y pq =    k2
Hence,   k (a 4  b 4  c 4 ) 
x y p q
xyz ak . bk . ck
a c e a 3  c3  e3 ace RHS = =

K
Ex. 9: If b  d  f , prove that 3  . abc abc
b  d 3  f 3 bdf
k 3 . abc 3
a c e = =
Soln. Let    k (say) abc k3  k 2
b d f
Hence, LHS = RHS
Then, a = bk, c = dk, e = fk
x y z
( i i) Since   = k (say)
a3  c 3  e3 b3k 3  d 3k 3  f 3k 3 3 a b c
LHS = = k ..(i)
b3  d 3  f 3 b3  d 3  f 3  x = ak, y = bk, z = ck

ace bk . dk . fk x 2  y2  z 2 a 2k 2  b 2k 2  c 2k 2
RHS = =  k3 ..(ii) LHS = =
bdf bdf a 2  b2  c 2 a 2  b2  c 2

KUNDAN
From (i) and (ii), we get,
LHS = RHS k 2(a 2  b 2  c 2 )
=  k2
3 3 3 a2  b2  c 2
a c e ace
 3 3 = 2 2
b d f3 bdf  px  qy  rz   p . ak  q . bk  r . ck 
RHS =   = 
 pa  qb  rc  pa  qb  rc 
x y z x3 y3 z3 xyz
Ex. 10: If   , show that    . 2
a b c a 3 b 3 c 3 abc  k ( pa  qb  rc ) 2
=   k
 pa  qb  rc 
x y z
Soln. Let   = k Hence, LHS = RHS
a b c
 x = ak, y = bk, z = ck x y z
Substituting these values of x, y and z in LHS. Ex. 12: If   , then show that
b c c a a b
We have,
(i) x + y + z = 0 and (ii) ax + by + cz = 0
a 3k 3 b 3k 3 c 3k 3
LHS =    k3  k3  k3  k3 x

y

z
3 3
a b c3 Soln. Let
b c c a a b
= k (say)

ak . bk . ck  x = k(b – c), y = k(c – a), z = k(a – b)


RHS =  k3 (i) LHS = x + y + z
abc
LHS = RHS [ Each side = k3] = k(b – c) + k(c – a) + k(a – b)
= k(b – c + c – a + a – b)
x y z = k(0) = 0.
Ex. 11: If   , show that
a b c (ii) LHS = ax + by + cz
3 = a{k(b – c)} + b{k(c – a)} + c{k(a – b)}
 a 2 x 2  b2y 2  c2z 2  2 xyz = k(ab – ac + bc – ab + ac – bc)
(i)  3 3 3
 
 a x  b y  c z  abc = k(0) = 0.
364 SSC Advanced Maths

a b c a b c
Ex. 13: If   , prove that each is equal Ex. 15: If  
b c c a a b x  2 y  3z y  2 z  3 x z  2 x  3 y , show

1 a b c
to or –1. that each ratio is equal to 2 ( x  y  z ) .
2

a b c a b c
Soln. Let   = k ....(i)  
b c c a a b Soln.
x  2y  3z y  2z  3x z  2x  3y
Then, a = k(b + c), b = k(c + a), c = k(a + b) Then by the theorem on equal ratios, we get,
 a + b + c = k(b + c + c + a + a + b) Sum of antecedents
= 2k(a + b + c) Each given ratio =
Sum of consequents
 a + b + c – 2k(a + b + c) = 0
a b c
(a + b + c) (1 – 2k) = 0 =
( x  2y  3z )  (y  2z  3x )  (z  2x  3y )
1 a b c a b c
 Either a + b + c = 0 or 1 – 2k = 0  k = = =
2 2x  2y  2z 2(x  y  z )
If a + b + c = 0, then b + c = –a
a a Hence,
so that   1
b  c a a b c b c
= =
b c x  2y  3z y  2z  3x z  2x  3y

K
Similarly,  1 ;  1
c a a b a b c
= .
2(x  y  z )
1 1
If k = , then from (i), each fraction = Ex. 16: Find the third proportional to (x – y), (x2 – y2).
2 2
Soln. Let the third proportional be p, then
1
Hence, each fraction is equal to or –1. x – y : x2 – y2 : : x2 – y2 : p
2
 p (x – y) : (x2 – y2)(x2 – y2)
ay  bx cx  az bz  cy
Ex. 14: If   , then prove that  p (x – y) = (x2 – y2)(x + y)(x – y)
c b a
( x 2  y 2 )(x  y )( x  y )
x y z p =
  . (x  y )

KUNDAN
a b c
 p = (x + y)(x2 – y2)
ay  bx cx  az bz  cy
Soln. We have,  
c b a Ex. 17: Find the fourth proportional to 2xy, x2, y2 .
Sum of the antecedents Soln. Let the fourth proportional be p, then
Each ratio =
Sum of the consequents 2xy : x2 : : y2 : p

ay  bx  cx  az  bz  cy Since, product of extremes = product of means


=
a b c  2xy × p = x2 × y2

x (c  b )  y(a  c )  z (b  a ) x 2y 2 xy
= p = p=
a b c 2xy 2
x y z
Let   = k  x = ak, y = bk, z = ck Ex. 18: Find the mean proportional between (x – y),
a b c
(x3 – x2y2).
ak (c  b )  bk (a  c )  ck (b  a )
Each ratio = Soln. Let the mean proportional be p, then
a b c

0
p = ( x  y )(x 3  x 2y ) = ( x  y ) x 2 (x  y )
=
a b c
= (x  y )2 x 2 = x (x – y) = x2 – xy
x y z
   is true.
a b c
Ratio & Proportion and k-Method 365

Exercise
12. Solve:
6 pq x  3 p x  3q
1. If x  p  q , find the value of x  3 p  x  3q .
4x  1  2x 1

a) 0 b) –1 c) 2 d) 1 4x  1  2x 5
a) –1 b) 5 c) 0 d) 2
4 6 x 2 2 x 2 2
2. If x  , find the value of  .
2 3 x 2 2 x 2 2 7 x  4x  3
13. If  6 , then find the value of x.
a) 2 b) 0 c) 1 d) None of these 7x  4x  3
3. Solve the following equation for x: a) 6 b) 5 c) 7 d) None of these

x  7  x 1 2 (2x  1)2  (2x  1)2 17


 
x  7  x 1 1 14. If 18 , then find the value of x.
(2x  1)2  (2x  1)2
a) 1 b) 0 c) –2 d) 2 Where x > 0.
x 2 x3 1
4. If  5 , then find the value of x. a) 2 b)
x 2 x 3 8
a) 2 b) 7 c) 8 d) 6 c) Both (a) and (b) d) None of these

a  a 2  2ax x 3  y3 91

K
 b , then find the value of x.  2x 2  3y 2
5. If 15. If 37 , determine the value of .
2
a  a  2ax x 3  y3 3y 2

2ab 2ab 5 1 27
2a 2 2b 2 a) b) c) d) 3
a) b) c) d) 27 5 5
(b  1)2 b 1 a b (a  b )2

x2  x 1 x 2  x 1
p2  q 2  p 2  q 2 16. If  and x  y, x  0, y  0, then
2
6. If x  , then find the value of y  y 1 y2  y 1
p 2  q 2  p2  q 2
find the value of xy.
q2x2 – 2p2x + q2. a) 0 b) 2 c) –1 d) 1
a) 0 b) 1 c) –1 d) 2
1 1

KUNDAN
a 9 5a 2  6b 2 ( p  1) 3  ( p  1) 3
7. If  , find the values of . 17. If y = 1 1 , then f ind t he value of
b 5 5a 2  6b 2
( p  1) 3  ( p  1) 3
17 37 13 37
a) b) c) d) y3 – 3py2 + 3y – p.
37 17 37 13 a) 1 b) 0 c) –1 d) None of these

d
8. If a + b = 1; c + d = 1 and a – b = , then find the y 2  xy  y 2  xy
c 18. If p = , then find the value of
value of c – d. y 2  xy  y 2  xy

b a xp2 – 2yp + x.
a) b) c) a + b d) ab a) 0 b) –1 c) 2 d) 1
a b

8ab x  4a x  4b x y z
9. If x = , find the value of  . 19. If   , then find the value
a b x  4a x  4b b c a c a b a b c
a) 4 b) 3 c) 2 d) 1 of (b – c)x + (c – a)y + (a – b)z.
a) 1 b) –1 c) 0 d) None of these
12 pq x  6 p x  6q 2 0 . If a + b : b + c = c + d : d + a, then find the value of
10. If x  p  1 , find the value of x  6 p  x  6q .
a + b + c + d.
a) 1 b) 0 c) –2 d) 2 b
a) 0 b) –1 c) d) None of these
a
a  b 1 a 2  ab  b 2
11. If  , find the value of .
a  b 2 a 2  ab  b 2 x y z
21. If   , then find the value of
r 2  pq p 2  qr q 2  pr
91 73 71 93
a) b) c) d) px + qy + rz.
73 91 93 71
a) –1 b) 1 c) 0 d) 2
366 SSC Advanced Maths

x y z a b c a b c
2 2 . If bc (b  c )  ca (c  a )  ab (a  b ) , then find the value 32. If   , then
c
is equal to
3 4 7
of a(b + c)x + b(c + a)y + c(a + b)z. a) 0 b) 1 c) 2 d) 3
a) 0 b) 1 c) 2 d) –2
xy  y 2
33. If x : y = 7 : 3, then the value of is
2x  y 3x  z z y x2  y2
23. If  
3y  z 2y  x x  z , then f ind t he value of
3 4 3 7
x + y + z. a) b) c) d)
4 3 7 3
2
a) 1 b) 3a  5b
3 34. If  5 , then a : b is equal to
3a  5b
c) 0 d) Cann’t be determined a) 2 : 1 b) 5 : 3 c) 3 : 2 d) 5 : 2

x y z ax  by 35. If p : q = r : s = t : u = 2 : 3, then (mp + nr + ot) : (mq +


24. If   , then find the value of
a b c (a  b )(x  y ) . ns + ou) equals
a) 3 : 2 b) 2 : 3 c) 1 : 3 d) 1 : 2
a) 1 b) 0 c) –1 d) 2
36. If x : y = 3 : 4, then (7x + 3y) : (7x – 3y) is equal to
a) 5 : 2 b) 4 : 3 c) 11 : 3 d) 37 : 19
a b c a 3  b3  c 3
25. If   , then find the value of 3 .
b c d b  c 3  d3 x 3 6 yx
37. If y  4 , then the value of 7  y  x is

K
b c a b
a) b) c) d)
d d d c 2 3 3
a) 1 b) c) d) 1
7 7 7
x y z
26. If   , then find the value of 38. If x : y = 2 : 1, then (5x2 – 13xy + 6y2) is equal to
a b c
3 4 55
ax  by by  cz cz  ax a) b) c) 0 d)
  . 4 3 4
(a  b )(x  y ) (b  c )(y  z ) (c  a )(z  x )
a) 0 b) 2 c) 1 d) 3 a c e 2a 2  3c 2  4e 2
39. If    3 , then ?.
x y z b d f 2b 2  3d 2  4 f 2
27. If   ,
(b  c )(b  c  2a ) (c  a )(c  a  2b ) (a  b )(a  b  2c ) a) 2 b) 3 c) 4 d) 9

KUNDAN
then find the value of x + y + z.
a) 0 b) –1 c) –2 d) 1 y2  x2
40. If 1.5x = 0.04y, then the value of is
y 2  2xy  x 2
ax  by bx  az ay  bz
28. If   and x + y + z  0, then
xy xz yz 730 73 73 74
find the value of each of the ratios. a) b) c) d)
77 77 770 77
a b a b a b a b
a) b) c) d) a b c a b c
2 3 2 3 41. If   , then is equal to
2 3 5 c
a) 2 b) 4 c) 5 d) 6
x 2  y2  p2 z 2  2xy 2z 2  p 2
29. If 2
 2
 , then f ind t he
x yz y zx z 2xy 5 x  2y
42. If x : y = 3 : 4, then the value of is
value of each of the ratios. 7 x  2y

1 1 1 1 1 1 7 7 7 7
a) xy  yz  zx b) x 2  y 2  z 2 a) b) c) d)
25 23 29 17

1 1 1 a b 2a  3b
43. If  , then the value of is
c) x  y  y  z  z  x d) None of these 3 2 3a  2b
12 5 12
30. If x : y = 3 : 2, then the ratio 2x2 + 3y2 : 3x2 – 2y2 is a) b) c) 1 d)
equal to 5 12 7

a) 12 : 5 b) 6 : 5 c) 30 : 19 d) 5 : 3 4ab x  2a x  2b
44. If x  (a  b ) , then the value of  is
a b x  2a x  2b
2a  5b 4
31. If  , then a : b is equal to a) a b) b c) 2ab d) 2
3a  6b 7
a) 21 : 36 b) 2 : 59 c) 59 : 2 d) 36 : 21
Ratio & Proportion and k-Method 367
45. If x varies inversely as (y 2 – 1) and is equal to 24
when y = 10, then the value of x when y = 5 is 3 x  3 x
51. If = 2, then x is equal to
a) 99 b) 12 c) 24 d) 100 3 x  3x

2x  y 1 3x  y 5 12 5 7
46. If  , then the value of is a) b) c) d)
x  2y 2 3x  y 12 5 7 5

1 3 4
a) b) c) d) 1  1 x  1 x 
5 5 5 3  
52. If x  , then the value of   is
2  1 x  1 x 
x y xy
47. If x – y = = , then the numerical value of xy a)  3 b) –1 c) 1 d) 3
7 4
is 53. If a : b : c = (y – z) : (z – x) : (x – y), then the value of ax
+ by + cz is
4 3 1 1
a) b) c) d) a) 1 b) 3 c) 0 d) –1
3 4 4 3
x 4  x 4
 x  y 54. If = 2, then x is equal to
x 4 x 4
48. If y : x = 4 : 15, then the value of  x  y  is
a) 2.4 b) 3.2 c) 4 d) 5
11 19 4 15 55. Find the fourth proportional to x3 – y3, x4 + x2y2 + y4,
a) b) c) d) x – y.
19 11 11 19
a) x2 – xy b) x3 – y2

K
49. If ( 3x – y) : (x + 5y) = 5 : 7, then the value of (x + y) : c) x2 + y2 – xy d) x2 – y2 + xy
(x – y) is
a) 2 : 3 b) 3 : 2 c) 3 : 1 d) 1 : 3 a b
56. Find the third proportional to  , a2  b 2 .
b a
x
50. If (5x2 – 3y2) : xy = 11 : 2, then the positive value of a
y
a) ab b) a + b c) a2 + b2 d)
is b

22 7 3 7 57. Find the mean proportional between (a + b) (a – b)3,


a) b) c) d) (a + b)3 (a – b).
7 2 2 2
a) (a2 + b2)2 b) (a + b)2 c) (a2 – b2)2 d)(a – b)2

Answers and explanations

1. c;

KUNDAN
x 

x  3p
6 pq x
p  q  3p

2q  p  q

Again, x 
6 pq
pq
2q
p q
By componendo and dividendo, we get,
x  3p
=
2q  p  q
=


3q  p
qp

x

2q
3q p  q
By componendo and dividendo, we get,
....(i)
By componendo and dividendo, we get,
x 2 2 2 3 2 3 3 3 2

x2 2 2 3 2 3

Again, x =

2 3
x

4 6
2 3

3 2

2 3

.
4 6

2 3
1
...(i)

x  3q 2p  p  q 3p  q x 2 2
 
= = ...(ii) 2 3 2 3
x  3q 2p  p  q p q
Adding (i) and (ii), we get, By componendo and dividendo, we get,

x  3p x  3q 3q  p 3p  q x 2 3 2 2 2 3 3 2 3
+ = –   ...(ii)
x  3p x  3q qp p q x2 3 2 2 2 3 2 3
From (i) and (ii), we get,
3q  p  3 p  q 2q  2 p q  p 
= = = 2  q  p  = 2. x 2 2 x 2 3 3 3 2 3 2 3
qp qp   + = –
x2 2 x2 3 3 2 3 2

4 6 3 3  2 3 2 3 2 3 2 2
2. a; x = =
2 3 3 2 3 2
 3 2
x 4 6 1 x 2 3  
 = .  = = 2  = 2.
2 2 2 2 2 3 2 2 2 3  3 2
368 SSC Advanced Maths

x  7  x 1 2 2ab – 2bx = b2x + x


3. d; We have, 
x  7  x 1 1 2ab = b2x + 2bx + x
By componendo and dividendo, we get, 2ab = x(b2 + 2b + 1)

( x  7  x  1)  ( x  7  x  1) 2  1 2ab
 2ab = x(b + 1)2  x 
( x  7  x  1)  ( x  7  x  1) 2  1 (b  1)2

2 x7 3 x7 3
    x p2  q 2  p2  q 2
2 x 1 1 x 1 1
6. a; We have, 1 
Squaring both sides, we get, p  q  p2  q2
2 2

x 7 9

x 1 1 x  1 2 p2  q 2 p2  q 2
 x  1  
 1(x + 7) = 9(x – 1) [By cross-multiplication] 2 p2  q 2 p2  q 2
 x + 7 = 9x – 9
 9x – x = 7 + 9 Squaring both sides, we get,
 8x = 16  x = 16 ÷ 8 = 2
(x  1)2 p2  q 2
Hence, x = 2.  
(x  1)2 p2  q2
4. b; Applying componendo and dividendo, we get,
Applying componendo and dividendo, we get,
x 2  x 3  x 2  x 3 5 1

K
x 2  x 3  x 2  x 3 5 1 (x  1)2  (x  1)2 2p2
 
2 2
(x  1)  (x  1) 2q 2
2 x2 6 x 2 3
 

2 x 3 4
or
x 3 2 2(x 2  1) p 2
 
4x q2
x 2 9
Squaring both sides, we get, 
x 3 4 x 2  1 p2
4(x + 2) = 9(x – 3) (By cross-multiplication)   q2x2 + q2 = 2p2x
2x q2
4x + 8 = 9x – 27
4x – 9x = –27 – 8  –5x = –35 q2x2 – 2p2x + q2 = 0
 5x = 35  x = 35 ÷ 5 = 7.
a 9 a2 81

KUNDAN
7. b;    
a  a  2ax 2 b 5 b2 25
5. a; Since, b
a  a 2  2ax
5a 2 5 81 5
Applying componendo and dividendo, we get,    [Multiplying both sides by ]
6b 2 6 25 6

a  a 2  2ax  a  a 2  2ax b 1
 5a 2
27
b 1  
a  a 2  2ax  a  a 2  2ax 6b 2 10
2a b 1 a b 1  By componendo and dividendo, we get,
   
2 a  2ax2 b 1 a  2ax b  1
2 5a 2  6b 2 27  10 37
= =
Squaring both sides, we get, 5a 2  6b 2 27  10 17

a2 b 2  2b  1 d
 8. a; a + b = 1 and a – b =
2
a  2ax b 2  2b  1 c

a b 1 c
a 2  2ax b 2  2b  1   
  [By invertendo] a b d d
a2 b 2  2b  1 c
Applying componendo and dividendo, we get, By componendo and dividendo, we get,
a 2  2ax  a 2 b 2  2b  1  b 2  2b  1 (a  b )  (a  b ) c d

2
a  2ax  a 2 2 2
b  2b  1  b  2b  1 (a  b )  (a  b ) = c  d

2a 2  2ax 2b2  2 2a (a  x ) 2(b 2  1) 2a



1
     (  c + d = 1)
2ax 4b 2ax 4b 2b c  d

b
(a  x ) b 2  1 a  x b2  1   c d (By invertendo)
    a
x 2b x 2b
Ratio & Proportion and k-Method 369

8ab x 2b a  b 1
9. c; x=   
a b 4a a b 11. a; We have,
a  b 2
By componendo and dividendo, we get,
By componendo and dividendo, we get,
x  4a x  4b 2b  (a  b )
 = ( a  b ) ( a  b ) 1 2
x  4a x  4b 2b  (a  b ) 
( a  b)( a  b ) 1 2
x  4a 3b  a
  ...(i)
x  4a b a 2 a 3 a 3
   =
2 a 1 b 1
8ab x 2b
Again, x =  
a b 4b a  b a
By componendo and dividendo, we get, Squaring both sides, we get, = 9
b
x  4b 2a  (a  b )
 2
x  4b 2a  (a  b ) a  a 
2 2      1
a  ab  b b  b 
x  4b 3a  b Now, = 2
  ..(ii) a 2  ab  b 2 a  a 
x  4b a b      1
b  b 
Adding (i) and (ii), we get,
[Dividing numerator and denominator by b2]
x  4a x  4b 3b  a 3a  b
+ =  +
x  4a x  4b b a a b (9)2  9  1 81  9  1 91
= = = .

K
(9)2  9  1 81  9  1 73
3b  a 3a  b 3b  a  3a  b
= – =
b a b  a b a
4x  1  2x 1
2b  2a 2(b  a ) 12. d; We have, 
=  2 4x  1  2x 5
b a b a
By componendo and dividendo, we get,
12pq x 2q
10. d; Since, x    ( 4x  1  2x )  ( 4x  1  2x ) 1  5
pq 6p p  q 
( 4x  1  2x )  ( 4x  1  2x ) 1  5
By componendo and dividendo, we get,
x  6 p 2q  ( p  q ) 2 4x  1 6 4x  1 3
    
x  6 p 2q  ( p  q ) 2 2x 4 2x 2

KUNDAN
x  6 p 3q  p 4x  1 9
 Squaring both sides, we get, 
 ....(i) 2x 4
x  6p qp
4(4x + 1) = 9 × 2x [By cross-multiplication]
12pq x 2p  16x + 4 = 18x
Again, x   
pq 6p p  q 2x = 4  x = 4 + 2 = 2.
By componendo and dividendo, we get,
7 x  4x  3
x  6q 2 p  ( p  q ) 13. c; 6
 7x  4x  3
x  6q 2 p  ( p  q )
Applying componendo and dividendo, we get,
x  6q 3 p  q
  ....(ii) ( 7 x  4x  3)  ( 7x  4x  3) 6  1
x  6q pq 
( 7 x  4x  3)  ( 7x  4x  3) 6  1
Adding (i) and (ii), we get,
x  6p x  6q 3q  p 3p  q 2 7x 7 7x 7
   
+ = + 2 4x  3 5 4x  3 5
x  6p x  6q qp p q
Squaring both sides, we get,
x  6p x  6q 3q  p 3p  q
 + = + 7x 49
x  6p x  6q qp qp  
4x  3 25
3q  p  3 p  q 2q  2 p 2(q  p )  25(7x) = 49(4x – 3) [By cross-multiplication]
= qp = qp = qp 2  175x = 196x – 147
 196x – 175x = 147
x  6p x  6q 21x = 147  x = 147 ÷ 21 = 7
Hence, + = 2. Hence, x = 7.
x  6p x  6q
370 SSC Advanced Maths
x2y + y – xy2 – x = 0
(2x  1)2  (2x  1)2
17 x2y – xy2 – x + y = 0
14. c; We have, 
(2x  1)2  (2x  1)2 18 xy(x – y) – 1(x – y) = 0
(x – y)(xy – 1) = 0
By componendo and dividendo, we get,
x – y = 0 or xy – 1 = 0
[(2x  1)2  (2x  1)2 ]  [(2x  1)2  (2x  1)2 ] 17  8 x = y or xy = 1
 But, x  y (given),  xy = 1.
2 2 2 2 17  8
[(2x  1)  (2x  1) ]  [(2x  1)  (2x  1) ]
1 1
2(2x  1)2
25 (2x  1)2 25 ( p  1) 3  ( p  1) 3
    17. b; We have, y = 1 1
2 9 2 9
2(2x  1) (2x  1)
( p  1) 3  ( p  1) 3
2x  1 5 2x  1 5
    y 1
2x  1 3 2x  1 3 Applying componendo and dividendo, we get,
y 1
6x + 3 = 10x – 5  6x + 3 = – 10x + 5
–4x = –8  16x = 2
 1 1  1 1
1 ( p  1)3  ( p  1)3   ( p  1)3  ( p  1)3 
x = 2 or x =    
8
   
=
 1 1  1 1
x 3  y3 91 ( p  1)3  ( p  1)3   ( p  1)3  ( p  1)3 
15. a; We have, 
x 3  y3 37 ,    

K
   
Using componendo dividendo, we get,
1
(x 3  y 3 )  (x 3  y 3 ) 91  37 3
 y  1 ( p  1)3  y  1 p 1
3 3 3 3 91  37     
(x  y )  (x  y ) y 1 1  y  1 p 1
( p  1)3
2x 3 128 x3 64
   
3 3 27
2y 54 y y 3  3y 2  3y  1 p 1
 
y 3  3y 2  3y  1 p 1
x 4 x2 16
    Using componendo and dividendo, we get,
y 3 y2 9
y 3  3y 2  3y  1  (y 3  3y 2  3y  1)

KUNDAN
2 

2x

32 y 3  3y 2  3y  1  (y 3  3y 2  3y  1)
3y 2 27
( p  1)  ( p  1)

2
2x  3y 2
32  27 ( p  1)  ( p  1)
  [By dividendo]
3y 2 27
2y 3  6y 2p y 3  3y
    p
6y 2  2 2 3y 2  1
2x 2  3y 2 5
 
3y 2 27 y3 + 3y = p(3y2 + 1)
y3 – 3py2 + 3y – p = 0
18. a; We have,
x2  x 1 x 2  x 1
16. d; We have, 2
 2
y  y 1 y  y 1 p y 2  xy  y 2  xy

By alternendo 1 y 2  xy  y 2  xy
x2  x 1 y2  y  1 By componendo and dividendo, we get,

2
x  x 1 y2  y  1
p 1 ( y 2  xy  y 2  xy )  ( y 2  xy  y 2  xy )
By componendo and dividendo, we get, =
p 1 ( y 2  xy  y 2  xy )  ( y 2  xy  y 2  xy )
(x 2  x  1)  (x 2  x  1) (y 2  y  1)  (y 2  y  1)

(x 2  x  1)  (x 2  x  1) (y 2  y  1)  (y 2  y  1) 2 p 1 y 2  xy
p  1 2 y  xy 
  
p  1 2 y 2  xy p 1 y 2  xy
2x 2  2 2y 2  2
 
2x 2y Squaring both sides, we get,

p 2  2p  1 y 2  xy
x 2  1 y2  1 
   y = (x2 + 1) = x(y2 + 1) 2
p  2p  1 y 2  xy
x y
Ratio & Proportion and k-Method 371
By componendo and dividendo, we get, yz + y2 – x2 – xz = 0
2 2 2 2 z(y – x) + (y2 – x2) = 0
( p  2 p  1)  ( p  2 p  1) (y  xy )  (y  xy )
 z(y – x) + (y – x)(y + x) = 0
( p2  2 p  1)  ( p 2  2 p  1) (y 2  xy )  (y 2  xy ) (y – x) (x + y + z) = 0
Either y – x = 0 or x + y + z = 0
2( p 2  1) 2y 2 p2  1 y x = y or x + y + z = 0.
   
4p 2xy 2p x
x y z
x(p2 + 1) = 2yp xp2 – 2yp + x = 0. 24. a; Let   k
a b c
x y z x = ak, y = bk, z = ck
19. c; Let   k
b c a c a b a b c The given expression
 x = (b + c – a)k
 y = (c + a – b)k ax  by a(ak )  b(bk )
 z = (a + b – a)k = (a  b )(x  y ) =
(a  b )(ak  bk )
Now, the given expression
= (b – c)x + (c – a)y + (a – b)z
k (a 2  b 2 ) k (a 2  b2 )
= (b – c)(b + c – a)k + (c – a)(c + a – b)k = = = 1.
+ (a – b)(a + b – c)k k (a  b )(a  b ) k (a 2  b2 )
= (b2 + bc – ab – bc – c2 + ca)k + (c2 + ac – bc – ac – a2
+ ab)k + (a2 + ab – ac – ab – b2 + bc)k a b c
25. c; Let   = k
= (b – ab – c + ca)k + (c2 – bc – a2 + ab)k
2 2
b c d
+ (a2 – ac – b2 + bc)k Then c = dk, b = dk2 and a = dk3
= (a2 – ab – c2 + ca + c2 – bc – a2 + ab The given expression
+ a2 – ac – b2 + bc)k

K
=0×k=0 a 3  b3  c 3 (dk 3 )3  (dk 2 )3  (dk )3
= =
a b c d
3
b c d3 3
(dk 2 )3  (dk )3  d 3
20. a; Let  k
b c d a
a + b = (b + c)k ....(i) d 3k 9  d 3k 6  d 3k 3
=
c + d = (d + a)k ....(ii) d 3k 6  d 3k 3  d 3
Adding (i) and (ii), we get,
a + b + c + d = (b + c + d + a)k d 3k 3(k 6  k 3  1) a
=  k3 
(a + b + c + d) – (a + b + c + d)k = 0 d 3(k 6  k 3  1) d
(a + b + c + d) (1 – k) = 0
Now either a + b + c + d = 0 or 1 – k = 0
x y z
x y z 26. d; Let   k
a b c

KUNDAN
21. c; Let  2  2 = k
r 2  pq p  qr q  pr Then, x = ak, y = bk, z = ck, we have
x = k(r – pq); y = k(p – qr); z = k(q2 – pr)
2 2

px + qy + rz = pk(r2 – pq) + qk(p2 – qr) + rk(q2 – pr) ax  by a(ak )  b(bk )


= k{pr2 – p2q + p2q – q2r + q2r – r2p} (a  b )(x  y ) = (a  b )(ak  bk )
=k× 0 = 0
x y z k (a 2  b 2 ) k (a 2  b2 )
22. a; Let bc (b  c )  ca (c  a )  ab (a  b ) = k =
(a  b )k (a  b )
=
k (a 2  b2 )
= 1

x = bc(b – c)k, y = ca(c – a)k, z = ab(a – b)k


by  cz cz  ax
 a(b + c)x = abc(b + c)(b – c)k = abc(b2 – c2)k Similarly, = 1 and = 1
Similarly, b(c + a)y = abc(c2 – a2)k; c(a + b)z (b  c )(y  z ) (c  a )(z  x )
= abc(a2 – b2)k Hence,
Adding up, we get,
ax  by by  cz cz  ax
a(b + c)x + b(c + a)y + c(a + b)z + + = 3.
= abc(b2 – c2)k + abc(c2 – a2)k + abc(a2 – b2)k (a  b )(x  y ) (b  c )(y  z ) (c  a )(z  x )
= abc . k(b2 – c2 + c2 – a2 + a2 – b2)
= abc × k(0) = 0. x y
27. a; Let =
(b  c )(b  c  2a ) (c  a )(c  a  2b )
2x  y 3x  z z y z
23. c; 3y  z  2y  x  x  z = = k
(a  b )(a  b  2c )
By the theorem on equal ratios, we get, Then, x = k(b – c)(b + c – 2a) ....(i)
2x  y  3x  z  z  y 5x x y = k(c – a)(c + a – 2b) ....(ii)
Each given ratio =   z = k(a – b)(a + b – 2c) ....(iii)
3y  z  2y  x  x  z 5y y
Adding (i), (ii) and (iii), we get,
z y x x + y + z = k{(b – c)(b + c – 2a) + (c – a)(c + a – 2b) +
 x  z  y (a – b)(a + b – 2c)}
= k[(b – c)(b + c) + (c – a)(c + a) + (a – b)(a + b)]
y(z + y) = x(x + z) [By cross-multiplication] – 2[a(b – c) + b(c – a) + c(a – b)] = 0.
yz + y2 = x2 + xz
372 SSC Advanced Maths

ax  by bx  az ay  bz a b c
28. c; We have,   32. c; Let   = k
x y x z yz 3 4 7
By the theorem on equal ratios, a = 3k, b = 4k, c = 7k

(ax  by )  (bx  az )  (ay  bz ) a b c 3k  4k  7k 14k


Each ratio =  = = = 2
( x  y )  ( x  z )  (y  z ) c 7k 7k

ax  bx  ay  by  az  bz x 7
= 33. a; We have, y  3
x y x z y z
Now,
x  (a  b )  y (a  b )  z (a  b )
= 2x  2y  2z xy  y 2 y(x  y ) y 1
 = 
x2  y2 ( x  y )( x  y ) x y x
1
(a  b )( x  y  z ) a b y
= =
2(x  y  z ) 2
1 1 3
[  x + y + z  0] =  
7 73 4
1
a b 3 3
 Each ratio =
2
3a  5b 5
34. d; We have, 
x 2  y 2  p2 z 2  2xy 2z 3  p 2 3a  5b 1
29. a; We have, = = By componendo and dividendo, we get,

K
x 2yz y 2zx z 2xy
By the theorem on equal ratios, we have, 3a  5b  3a  5b 5  1

3a  5b  3a  5b 5  1
(x 2  y 2  p 2 )  (z 2  2xy )  (2z 2  p 2 )
Each ratio = 6a 6 a 6 10 5
x 2yz  y 2zx  z 2 xy      
10b 4 b 4 6 2
x 2  y 2  2xy  z 2 ( x  y )2  z 2 5 : 2
= 
xyz ( x  y  z ) xyz (x  y  z ) p r t 2
35. b; We have,   
(x  y  z )(x  y  z ) x y z q s u 3
= = (If x  y  z) p q
xyz (x  y  z ) xyz    k p = 2k and q = 3k
x y z 1 1 1 2 3
= xyz  xyz  xyz  yz  zx  xy Similarly, r = 2k, s = 3k, t = 2k, u = 3k

KUNDAN
mp  nr  ot
Now, mq  ns  ou
x 3
30. c; Here y  2
m.2k  n .2k  o.2k 2k (m  n  o ) 2
= = =
2 2 m.3k  n .3k  o.3k 3k (m  n  o ) 3
x 3 9
 2
   2 : 3
y 2
  4
x 3 7x 7 3 7
 x2  36. c; Since,      
2 2 2 2   3 y 4 3y 3 4 4
2x  3y y 
  By componendo and dividendo, we get,
Now, =
3x 2  2y 2  x2 
3 2   2 7x  3y 7  4 11
y  =  11 : 3
  7x  3y 74 3
[On dividing numerator and denominator by y2]
x 3
37. a; We have, 
 9 9 96 y 4
2   3 3
 4 2 2 15 4 x
= = 27 = 27  8 =  1
 9 2 2 19 6 yx 6 y
3    2 Now,  = 
 4 4 4 7 yx 7 1 x
y
30
= = 30 : 19 [Dividing numerator and denominator by y]
19

2a  5b 4 3
31. c; We have,  6 1 4 6 43
3a  6b 7 =  = 
7 1 3 7 43
 14a – 35b = 12a + 24b  2a = 59ab 4
a 59 6 1
 = = 59 : 2 =  = 1
b 2 7 7
Ratio & Proportion and k-Method 373

x x  2a x  2b 3b  a 3a  b
38. c; We have, y = 2 x = 2y x – 2y = 0   =  +
x  2a x  2b b a a b
5x2 – 13xy + 6y2 = 5x2 – 10xy – 3xy + 6y2
= 5x (x – 2y) – 3y (x – 2y) 3b  a  3a  b 2b  2a 2(b  a )
=  = = 2
= (x – 2y)(5x – 3y) b a b a b a
= 0 × (5x – 2y) = 0
1
a c e 45. a; We have, x 
39. d; We have, b  d  f  3 y2 1
a = 3b; c = 3d; e = 3f k
2  9b 2  3  9d 2  4  9 f 2  x  ; where k is a constant.
2a 2  3c 2  4e 2 y2 1
 
2
2b  3d  4 f2 2 2b 2  3d 2  4 f 2
When y = 10, x = 24, then
9(2b 2  3d 2  4 f 2 ) k k
= 9
2b 2  3d 2  4 f 2 24 =  24 =
102  1 9
40. b; We have, 1.5x = 0.04y k = 24 × 99
x 0.04 4 2 When y = 5, then
   
y 1.5 150 75 k 24  99 24  99
y 75 x =    99
  y2 1 52  1 24
x 2
y2  x2 (y  x )(y  x ) 2x  y 1
46. b; We have, x  2y  2

K
Now, 2 2
= 2
y  2xy  x (y  x )
4x – 2y = x + 2y
y 75 3x = 4y
1 1
yx 2 73
=  x = 75  x 4
yx y 77  
1  1 y 3
x 2

 x   4 
a b c y  3  1 3   1
41. a; Let   k 3x  y  y   3  4 1 3
2 3 5   = = 
3x  y  x   4  4 1 5
a = 2k; b = 3k; c = 5k y  3  1  3   1
 y  3
a b c 2k  3k  5k 10k
 = = 2 x y xy
c 5k 5k 47. a; x – y = = k

KUNDAN
7 4
x 3
42. c; We have, y  4 x – y = k; x + y = 7k
 (x + y)2 – (x – y)2 = 49k2 – k2
4xy = 48k2
x 3 15  8
5 2 5 2  16k = 48k 2
5x  2y y 4 4 7
 = x = 3 = 21  8  1
7x  2y 7 2 29 k=
7 2 3
y 4 4
1 4
 xy = 4k = 4 × =
a b a 3 3 3
43. a; We have,   
3 2 b 2 48. a; Since, y : x = 4 : 15  x : y = 15 : 4
x 15
a 3  
2a  3b 3 2 23 y 4
b 2 6 12
 = = =  By componendo and dividendo, we get,
3a  2b a 3 94 5
3 2 3 2 x y 15  4 11
b 2 2 = =
x y 15  4 19
4ab x 2b
44. d; Since, x     3x  y 5
a b 2a a b 49. c; We have, 
x  5y 7
By componendo and dividendo, we get, 21x – 7y = 5x + 25y
x  2a 2b  a  b 3b  a 21x – 5x = 25y + 7y
   16x = 32y
x  2a 2b  a  b b a
x 32 2
Again,   
y 16 1
x 2a
 x  y 2 1
2b a  b  x  y  2  1 = 3 : 1
x  2b 2a  a  b 3a  b
   [By componendo and dividendo]
x  2b 2a  a  b a b
374 SSC Advanced Maths

5x 2  3y 2 11 a b c
50. b; Since,
xy

2 53. c; Let y  z  z  x  x  y  k

11 a = k(y – z); b = k(z – x); c = k(x – y)


 5x2 – 3y2 = xy ax + by + cz = k(xy – xz + yz – xy + xz – yz) = 0
2
2 2
 10x – 6y – 11xy = 0
x 4  x 4 2
 10x2 – 15xy + 4xy – 6y2 = 0 54. d; We have, =
5x(2x – 3y) + 2y(2x – 3y) = 0 x 4 x 4 1
(2x – 3y) (5x + 2y) = 0 By componendo and dividendo, we get,
 2x = 3y
2 x4 3
x 3 
 2 x 4 1

y 2 On squaring both sides, we get,
x 4 9
3 x  3x 2 
51. b; We have,  x 4 1
3 x  3x 1
9x – 36 = x + 4
By componendo and dividendo, we get, 9x – x = 36 + 4
2 3  x 2 1 8x = 40
  3  x = 5
2 3  x 2 1
Squaring on both sides, we get, 55. c; Let the fourth proportional be p, then
x3 – y3 : x4 + x2y2 + y4 : : x – y : p

K
3x p(x3 – y3) = (x4 + x2y2 + y4) (x – y)
9
3x
3 + x = 27 – 9x (x 4  x 2y 2  y 4 )(x  y )
 p
9x + x = 27 – 3 = 24 x 3  y3
24 12
x =  [( x 2  y 2 )  ( xy )2 ](x  y )
10 5 =
(x  y )(x 2  y 2  xy )

3 1 2
52. d; Since, x    (x 2  y 2  xy )(x 2  y 2  xy )(x  y )
2 x 3 =
( x  y )(x 2  y 2  xy )
By componendo and dividendo, we get,
= x2 + y2 – xy

KUNDAN
1 x 2  3 56. a; Let the third proportional be x, then

1 x 2  3
a b
   : a2  b 2 : : a2  b 2 : x
2 2 b a

1 x
=
 
2 3
=
2  3 
a b 2
1 x 2  3 2  3 43
 x    =
b a a 2  b 2 
1 x 2
 = 2  3
1 x  a 2  b2 
 x  ab  = a2 + b2
1 x 2  3  
 
1 x 1  x = ab
By componendo and dividendo, we get, 57. c; Let the mean proportional be x, then
1 x  1 x 2  3 1 x = (a  b )(a  b )3 (a  b )3 (a  b )

1  x  1  x 2  3 1
= (a  b )4 (a  b )4 = (a + b)2 (a – b)2
=
3 3

3  3 1  3 = [(a + b)(a – b)]2 = (a2 – b2)2
3 1 3 1
316 Concept of Arithmetic

Chapter-17

Simple Interest
Lender and Borrower uniformly on the original principal throughout the

K KUNDAN
loan period, it is called simple interest.
Generally in transactions involving large sums of To explain the meaning of simple interest let
money such as buying a house or a car etc we us suppose that you have borrowed a sum of Rs
borrow money either from a bank or an individual 100 at 10% per annum for 2 years from Mr X. This
or some other agency. The bank or an individual means that at the end of 2 years you would pay Rs
or some other agency from which we borrow money 20 as interest in addition to the principal. Now
is called the lender and the person or a company suppose you had borrowed Rs 100 for 1 year only
who borrows money is called the borrower. The from Mr X. Then you would have to return Rs 100
money borrowed is known as the loan. The amount + Rs 10 = Rs 110 at the end of the year. Mr X has
of loan may be big or small depending upon the Rs 110 now. If Mr X lends this sum again to you
requirement of the borrower. for 1 more year at the same rate, then at the end of
When a borrower borrows some money from a one year he would get his Rs 110 back together
lender, he or she also makes a promise to return it with the interest on this sum. The interest on Rs
after a specified period of time. At the end of the 110 at the rate of 10% per annum would be Rs
specified period, the borrower has not only to pay
the money which he had borrowed but also to pay  10 
 110  = Rs 11. Hence, Mr X would be having
some additional money for using lender’s money.  100 
Rs 110 + Rs 11 = Rs 121 at the end of 2 years.
Some Important Terms
Clearly, this amount differs from the amount in
In borrowing and lending money the following the first case. This happens because in the second
terms are commonly used: case the interest is charged on the interest which
(i) Principal: The money borrowed by a borrower he got from you at the end of first year. When the
from the money-lender is known as the principal. interest is charged in this manner, it is known as
(ii) Interest: The additional money paid by the compound interest (We will discuss ‘Compound
borrower to the lender for having used his (her) Interest’ in detail in the next chapter). If t he
money is called interest. interest is calculated on principal only, it is known
(iii) Amount: The total money paid by t he as simple interest.
borrower to the lender at the end of the specified In this chapter the term interest will be used
period is called the amount. in the sense of simple interest only.
Thus, Amount = Principal + Interest

K KUNDAN
or, A = P + I; Methods for Finding Simple Interest
where, A stands for the Amount, P for Principal
and I for Interest. (i) By Unitary Method:
(iv) Rate: The interest paid for keeping Rs 100
In this method, by using rate per cent per annum
for one year is known as the rate per cent per
(interest on Rs 100 for 1 year) we calculate simple
annum.
interest on the given sum for given period of time.
If the money is borrowed at the rate of 12% per
For example, suppose a man lends Rs 500 at
annum, then the interest paid for keeping Rs 100
10% per annum for 2 years. If we want to find the
for one year is Rs 12.
interest he earns we shall proceed as follows:
Clearly, the interest we pay is proportionate to
Principal = Rs 500, Rate = 10%, Time = 2 years.
the money that we borrow and also to the period of
time for which we keep the money ie the more the
 Interest on Rs 100 for one year = Rs 10
money and the time, the more the interest. Interest 10
is also proportionate to the rate of interest agreed  Interest on Re 1 for one year = Rs
100
upon by the lending and the borrowing parties.
 Interest on Rs 500 for one year
Thus, interest varies directly as principal, time
and rate. 10
= Rs  500
(v) Simple Interest: If interest is calculated 100
Simple Interest 317

 Interest on Rs 500 for two years we proceed as follows:


We have, Principal (P) = Rs 7300, Rate
500  10  2
= Rs = Rs 100 (R) = 10% per annum and Time (T) = 15th
100 May 2000 to 8th October 2000.
Thus, we say that Rs 100 is the interest on Rs Number of days
500 for 2 years at 10% per annum. This interest = May + June + July + August +
is called Simple Interest. It is abbreviated as SI. Septermber + October
= 16 + 30 + 31 + 31 + 30 + 8
(ii) By Formula: = 146 days
We shall find the simple interest on the sum of Rs
 146  2
P invested at R% per annum for T years, from the  Time (T) =   year = year
above method as given below:  365  5

K KUNDAN
 Interest on Rs 100 for 1 year = Rs R
PRT  7300  10 2 
Now, SI = = Rs   
R 100  100 5
 Interest on Re 1 for 1 year = Rs
100 = Rs 292
In this context, you should remember the
RT following:
 Interest on Re 1 for T years = Rs  
 100  73, 146, 219 and 292 days are respectively
 Interest on Rs P for T years
1 2 3 4
R , , and of a year.
   P  R T  5 5 5 5
= Rs  P  T  = Rs  
 100   100  ( i i) From the above example, it is clear that one
Thus, the simple interest (SI) on Rs P at the should know the number of days in each
rate of R% per annum for T years is given by month.
Mo n t h Number of Days
PRT January 31
SI = ie
100 February 28 or 29*
March 31
Principal  Rate  Time
Simple Interest = Apr il 30
100 May 31
For example, in the above example, we have June 30
P = Rs 500, R = 10% and T = 2 years July 31
August 31
PRT 500  10  2
 SI = = = Rs 100 September 30
100 100 October 31
From this formula, we have November 30
100  SI December 31
(i) P = * February is always of 28 days except the
RT Leap Year. In the Leap Year the month
100  SI February contains 29 days.
(ii) R = Leap Year: Every year which is exactly

K KUNDAN
PT
divisible by 4 such as 1988, 1992, 1996
100  SI etc. is called a leap year.
(iii) T = Also every 4th century is a leap year. The
PR
other centuries, although divisible by 4, are
Important Points not leap years. Thus, for a century to be a
leap year, it should be exactly divisible by
( i ) In the above formulae, time T is in years. If 400. For example,
the time is given in months or days, then (1) 400, 800, 1200, etc are leap years since
we convert it into years by dividing by 12 they are exactly divisible by 400.
or 365 according as the time is given in (2) 700, 600, 500 etc are not leap years
months or days. since they are not exactly divisible by
Note that for counting the number of 400.
days we do not count the day on which ( i ii ) To find Amount, we can also use formula
the money is deposited but we count the as given below:
day of withdrawal.  Amount = Principal + Interest
For example, if we have to find the simple
interest on Rs 7300 from 15th may 2000 to PRT  RT
 A= P = P 1  
8th October 2000 at 10% per annum, then 100  100 
318 Concept of Arithmetic

per rupee per annum and time = 219 days.


 Rate  Time 
or, Amount = Principal 1   we have,
 100  P = Principal = Rs 1000
For example, suppose we hav e to f ind R = Rate = 10 paise per rupee per
Amount if the Principal is Rs 800, rate is annum
6% per annum and the time is 4 years. = 100 × 10 paise per cent per
 64 annum
Amount = Rs 800 1   = Rs 10 per cent per annum
 100 
= 10 per cent per annum
 6  and T = Time = 219 days
= Rs 800 1  
 25   219  3

K KUNDAN
=   year = year
 31   365  5
= Rs  800   
 25 
 3 
= Rs (32 × 31) = Rs 992 PRT  1000  10  
 SI = = Rs  5 
( iv) If the interest is paise per rupee not rupees 100  100 
per cent, then we convert it into per cent  
 
by multiplying it by 100.
The following example will illustrate the = Rs 60
computation of simple interest. Now, Amount = Principal + SI
Find the simple int er est and amount = Rs 1000 + Rs 60
when, principal = Rs 1000, rate = 10 paise = Rs 1060

Solved Examples

Ex. 1(i): Find the simple interest on Rs 2400 at Ex.1(iii): A man borrowed Rs 8500 at 12% per
the rate of 10% per annum for 3 years. annum for 3 years. Find the amount he
had to pay after 3 years.
PRT
Soln: Simple Interest = Soln: Principal = Rs 8500
100 Rate = 12% per annum
Time = 3 years
2400  10  3
= = Rs 720
100 PRT  8500  12  3 
 SI = = Rs  
Ex.1(ii): Ajay deposited Rs 6000 in post office 100  100 
for 6 months. If the post office pays = Rs 3060
1  Amount to be paid after 3 years
interest at 5 % per annum, find the = Rs 8500 + Rs 3060 = Rs 11560
2
Alternative Method:
interest Ajay got after 6 months.

K KUNDAN
Soln: Here, Principal = Rs 6000  RT
Amount = A = P 1  
1  100 
Time = 6 months = year
2 12  3 

1 = Rs 8500 1  
 100 
Rate = 5 % per annum
2
 8500  34 
11 = Rs   = Rs 11560
= % per annum  25 
2
Ex.2(i): Find the simple interest on Rs 800 at
PRT  6000  11  1  t he r ate of 5% per annum fr om 2
 SI = = Rs  
100  100  2  2  January 1980 to 26 May 1980.
= Rs 165 Soln: Here Principal (P) = Rs 800;
 Ajay got Rs 165 as int er est on his Rate (R) = 5% per annum
deposit. Time (T) = From 2 January 1980 to 26
Note: The rate of interest is per annum. May 1980.
Therefore, the time must be expressed in = January + February + March
years. + April + May
Simple Interest 319

= 29 + 29 + 31 + 30 + 26  When amount is Re 1, principal


 146  100
= 146 days =   years = Rs
 365  136
 When amount is Rs 9520, principal
2
= years  100 
5  9520  = Rs 7000
= Rs 
Since, the year 1980 is exactly divisible  136 
by 4, it is a leap year. Therefore, the month Note: From the above example, we see that
of February will be of 29 days.
A  100
PRT 800  5  2 Principal =
100  R  T
 SI = = Rs

K KUNDAN
100 100  5 Ex. 4(i):At what rate per cent per annum will
= Rs 16 Rs 300 produce Rs 18 as interest in 2
Ex.2(ii): Find the simple interest on Rs 500 at years?
t he r ate of 4% per annum fr om 21
100  SI 100  18
January 1900 to 4 April 1900. Soln: Rate = = = 3%
Soln: P = Rs 500, R = 5% per annum PT 300  2
T = From 21 January 1900 to 4 April Ex.4(ii): At what rate per cent per annum will
1900 Rs 800 amounts to Rs 1000 in 2 years?
= January + February + March + April Soln: Let the required rate per cent be R% per
= 10 + 28 + 31 + 4 annum.
We have,
 73  1 Principal (P) = Rs 800,
= 73 days =   year = year
 365  5 Amount = Rs 1000
Since 1900 is div isible by 4 but not  SI = Amount – Principal
divisible by 400, therefore, it is not a leap = Rs 1000 – Rs 800 = Rs 200.
year . Hence, t he mont h of Febr uar y Time (T) = 2 years.
consists of 28 days.
SI  100
Now, R 
PRT  500  4  1  PT
 SI = = Rs   = Rs 4
100  100  5 
200  100 25
Ex. 3(i):What sum lent out at 10% per annum  R = % %  12.5%
800  2 2
simple interest would produce Rs 150
as interest in 5 years? Hence, required rate per cent = 12.5% per
Soln: We have, SI = Rs 150, R = Rate per cent = annum.
10% per annum and T = Time = 5 years. Ex.5(i): In what time will Rs 250 produce Rs
We have to find the principal. 125 as interest at the rate of 10% per
annum?
SI  100
Now, Principal = 100  SI 125  100
RT Soln: Time = = = 5 years
PR 250  10
 150  100 

K KUNDAN
Ex.5(ii): In what time will Rs 8500 amount to
 Principal = Rs  
 10  5  Rs 15767.50 at 9% per annum simple
= Rs 300. interest?
Hence, required sum = Rs 300. Soln: Amount = Rs 15767.50
Ex. 3(ii): What sum will amount to Rs 9520 at Principal = Rs 8500
9% per annum in 4 years at simple SI = Rs 15767.50 – Rs 8500 = Rs 7267.50
interest? Rate = 9% per annum
Soln: Rate = 9% , Time = 4 years and Amount 100  SI 7267.50  100
= Rs 9520. Time = =
PR 8500  9
Let principal be Rs 100.
Then simple interest 1
= 9 years
 100  9  4  2
= Rs   = Rs 36
Ex. 6: A person deposits Rs 4200 in a bank
 100 
at the rate of 8% per annum interest
 Amount = Rs 100 + Rs 36 = Rs 136
and Rs 1400 in a post office at the rate
Now,
of 6% per annum interest. At what rate
 When amount is Rs 136, principal per cent will he get interest on his total
= Rs 100
deposits?
320 Concept of Arithmetic

Soln: Let the time be equal and he deposits his  Amount in 5 years = Rs (100 × 2)
money with the bank and the post office = Rs 200
for 1 year.  Interest = Rs (200 – 100) = Rs 100
PRT SI  100 100  100
 Interest of bank = Rate = = = 20%
100 PT 100  5
Now in T years Amount = Rs (3 × 100)
 4200  8  1 
= Rs   = Rs 336 = Rs 300
 100   Interest in T years = Rs 300 – Rs 100
= Rs 200
PRT
and interest of post office = SI  100 200  100
100
 Time = T = =

K KUNDAN
PR 100  20
 1400  6  1 
= Rs   = Rs 84 = 10 years
 100  Ex. 8: What sum lent out at 6.25% per annum
 Total interest = Rs 336 + Rs 84 produces the same simple interest in 2
= Rs 420 and years as Rs 1500 lent out at 5% produce
Total deposits = Rs 4200 + Rs 1400 in 1 year and 4 months?
= Rs 5600 Soln: Let t he r equired sum of money be
Rs P.
100  SI  420  100  1
 Rate = =   = 7 % When this sum is lent out at 6.25% per
PT  5600  1  2 annum for 2 years, we have,
Ex.7(i): In how many years will a sum of money
doubl e i tself at 18.75% per annum  25 
P 2
simple interest?  4 
SI = Rs
Soln: Let the principal be Rs P. Then, Amount  100 
 
= Rs 2P.  
We have, Principal = Rs P,
Amount = Rs 2P P
 SI = Rs .....(i)
 SI = Amount – Principal 8
= Rs 2P - Rs P = Rs P
R = rate per cent  PRT 
= 18.75% per annum. Using : SI  100 , where 
 
Let the required time be T years. Then, R  6.25%  25 
% and T  2 years
SI  100  4 
T =
PR For the computation of SI on Rs 1500 at
5% per annum lent out for 1 year 4
P  100
 T = years months, we have,
P  18.75 P = Rs 1500, R = 5% per annum and
16 16 4
= years = 5 years 4 months Time (T) = years = years
3 12 3

K KUNDAN
Ex.7(ii): At what rate per cent per annum will a

Soln:
sum treble itself in 16 years?
Let the sum be Rs P. Then,
amount = Rs 3P.
 SI = Rs (3P – P =) Rs 2P.
We have, Time (T) = 16 years.
We have to find the value of R.

Now, R =
100  SI

 1500  5 
 SI = Rs 



= Rs 100
100
4
3






.....(ii)
As given in the question that SI is the
same in both the cases.
PT P
 = 100
 100  2P  8
 R =   % = 12.5%
or, P = 800.
 P  16 
Ex.7(iii): A sum of money doubles itself in 5 Hence, the required sum = Rs 800.
years. In how many years will it treble Ex. 9: The simple interest on a sum of money
itself? 1
Soln: Suppose the sum of money is Rs 100. is of the principal and the number
4
Simple Interest 321

of years is equal to the rate per cent  When the amount is Rs 150, then the
per annum. Find the rate per cent. sum = Rs 100
Soln: Let the Principal be Rs 100.  When the amount is Re 1, the sum
1 100
 Interest = of Rs 100 = Rs 25 = Re
4 150
 When the amount is Rs 7500, the sum
Suppose rate per cent = x%
 time = x years  100  7500 
= Rs   = Rs 5000
PRT  150 
Now, SI = Alternative Method:
100
Let the sum be Rs x.
100  x  x

K KUNDAN
or, 25 = x
100  Interest = Rs
2
or, x2 = 25
 Amount = Principal + Interest
 x = 5%
Ex. 10: A sum of money lent out at si mple  x 3x
interest amounts to Rs 2200 in one year = Rs  x   = Rs
 2 2
and Rs 2800 in 4 years. Find the sum
Now, according to the question,
of money and the rate of interest.
Soln: We have, Amount in 4 years 3x
= 7500
= (Principal + SI for 4 years) = Rs 2800 2
Amount in 1 year 7500  2
= (Principal + SI for 1 year)  x = = Rs 5000
3
= Rs 2200
On subtracting, we get  Principal = Rs 5000 and Interest
SI for 3 years = Rs (2800 - 2200) = Rs 600 5000
= Rs = Rs 2500
600 2
 SI for 1 year = Rs = Rs 200.
3 SI  100 2500  100 1
But, Principal  Rate = = = 12 %
PT 5000  4 2
= Amount in 1 year - SI for 1 year
 Principal = Rs 2200 – Rs 200 Ex. 12: A sum of money invested at 20% per
= Rs 2000. annum simple interest amounts to Rs
Now, Principal (P) = Rs 2000, 1
650 in 1 years. What will it amount
SI = Rs 200 and Time (T) = 1 year. 2
to in 2 years at 12% per annum simple
SI  100 interest?
R =
PT Soln: Let the given sum of money be Rs 100.
When it is invested at 20% per annum
200  100
 R = % = 10%
2000  1 1
for 1 years, we have
2

K KUNDAN
Hence, Principal = Rs 2000 and rate
= 10% per annum.
Ex. 11: A sum of money amount to Rs 7500 in  3 
100  20 
4 years. If the simple interest is half PRT  2

SI =    = Rs 30
of the sum, then find the sum of money 100  100 
 
and the rate of interest.  
Soln: Suppose the sum of money is Rs 100.
 Amount = Principal + SI
 Simple interest = Half of the sum
= Rs 100 + Rs 30 = Rs 130.
= Rs 50
Now,
 Interest on Rs 100 for 4 years = Rs 50 If amount is Rs 130, then principal
 Interest on Rs 100 for 1 year
= Rs 100
50 1
= Rs = Rs 12 100
4 2 If amount is Re 1, then principal = Re
130
1 If amount is Rs 650, then principal
 Rate = 12 %
2
Now, Rs 100 in 4 years amounts to 100
= Rs  650 = Rs 500.
Rs (100 + 50) = Rs 150 130
322 Concept of Arithmetic

Thus, the giv en sum of money is  Other amount = Rs (3100 - x)


Rs 500. = Rs (3100 - 1600)
When this sum of money is invested at = Rs 1500
12% per annum for 2 years, we have Hence, ratio of money lent
= 1600 : 1500 = 16:15
PRT
SI = Ex. 15: Divide Rs 1550 into two parts such that
100 if one part be lent out at 15% per annum
and the other at 24% per annum, the
 500  12  2 
 SI = Rs   = Rs 120. total yearly income is Rs 300.
 100  Soln: Let the first part be Rs x.
 Amount = Principal + SI Then the second part = Rs (1550 – x).
= Rs 500 + Rs 120 = Rs 620. Now, SI on Rs x at 15% per annum for 1

K KUNDAN
Hence, the given sum of money amounts year
to Rs 620.
Ex. 13: A money-lender lends Rs 3600 to Ram  x  15  1  3x
= Rs   = Rs
for 2 years and Rs 2400 to Shyam for  100  20
3 years. If he gets Rs 864 as interest SI on Rs (1550 - x) at 24% per annum for
find the rate of interest per annum. 1 year
Soln: Let the rate of interst be x% per annum
 Interest on Rs 3600 for 2 years  1550  x   24  1 
= Rs  
 100 
 3600  x  2 
= Rs   = Rs 72x
1550  x   6
 100 
= Rs
and the interest on Rs 2400 for 3 years 25

 2400  x  3   9300  6x 
= Rs   = Rs 72x = Rs  
 100   25 
 Total interest = Rs (72x + 72x)
 3x 9300  6x 
= Rs 144x  Total SI = Rs   
Now, according to the question,  20 25 
144x = 864 But, total SI = Rs 300. (Giv en in t he
question)
864
 x = = 6% 3x 9300 – 6x
144   300

Ex. 14: A sum of Rs 3100 was lent partly at 20 25
5% and partly at 8% interest. Total or, 15x + 4 (9300 – 6x) = 300 × 100
interest received after 3 years was Rs [Multiplying both sides by the
600. Find the ratio of the money lent LCM of 20 and 25 ie 100]
at 5% and 8%. or, 15x + 37200 – 24x = 30000
Soln: Suppose money lent at 5% = Rs x or, –9x = 30000 –37200
 Money lent at 8% = Rs (3100 - x) or, –9x = –7200
Time (T) = 3 years
7200

K KUNDAN
Interest in the first case or, x = = 800
9
x 53 15x
= = Rs So the first part = Rs 800 and the second
100 100 part = Rs (1550 – 800) = Rs 750.
Interest in the second case Ex. 16: Divide Rs 4350 in two parts so that
3100  x   8  3 the simple interest on the first when
= Rs deposited for one year at 9% per annum
100
and that on the second when deposited
24 (3100  x ) for two years at 10% per annum in a
= Rs
100 bank are the same.
Now according to the question, Soln: Let the first part be Rs x.
Then the second part = Rs (4350 - x).
5x 243100  x  For the f irst part, we hav e Principal
  600
100 100 = Rs x, rate of interest = 9% per annum
or, 15x + 74400 – 24x = 60000 and Time = 1 year
or, 9x = 14400
 x  9 1 9x
14400  SI = Rs   = Rs
or, x = = Rs 1600  100  100
9
Simple Interest 323

For the second part, we have Am an, find the rat e of interest per
Principal = Rs (4350 - x), rate of interest annum.
= 10% per annum and Time = 2 years. Soln: We have, sum borrowed by Amit = Rs 3000
Sum borrowed by Aman = Rs 2500
(4350  x )  10  2
 SI = Rs  Difference of sum
100 = Rs 3000 - Rs 2500 = Rs 500
It is given that Amit paid Rs 175 more
204350  x 
= Rs interest than Aman. Therefore,
100
It is given that the interest earned on two 1
SI on Rs 500 for 2 years is Rs 175
parts are same. 2
9x 204350  x  Now, Principal = Rs 500,

K KUNDAN
 
100 100 5
Time = years and SI = Rs 175
or, 9x = 20 (4350 - x) 2
[Multiplying both sides by 100]
or, 9x = 87000 – 20x 100  SI 100  175
 Rate =  % = 14%
or, 20x + 9x = 87000 PT 5
500 
[Transposing 20x on LHS] 2
or, 29x = 87000 Hence, rate of interest is 14% per annum.
87000 Ex. 18: If Rs 640 amounts to Rs 784 in 2 years
or, x = = 3000. 6 months, what will Rs 860 amount to
29
in 4 years at the same rate per cent
 First part = Rs 30000 per annum?
Second part = Rs (4350 - 3000) Soln: Let the rate per cent be R% per annum.
= Rs 1350. We have, Principal P = Rs 640
Hence, the two parts are of Rs 3000 and Amount = Rs 784 and
Rs 1350. Time = T = 2 years 6 months
Alternative Method:
Let SI in each case be Re 1. Then, 1 5
= 2 years = years.
P1 = Principal in first case 2 2
 100  1  100  SI = Amount - Principal
= Rs   = Rs = Rs 784 – Rs 640 = Rs 144
 9 1  9
SI  100
 100  SI  Now, R =
PT
Using : P  R  T 
 
P2 = Principal in second case 144  100
 R = % = 9%
5
 100  1  640 
= Rs   = Rs 5 2
 10  2 
Now, if Rs 860 are invested at the same
100 rate for 4 years. Then,

K KUNDAN
 P1 : P2 = : 5 = 20 : 9
9  860  9  4 
SI = Rs   = Rs 309.60
 100 
 20 
So, first part = Rs  4350  
 29   PRT 
= Rs 3000 and Using : SI  100 , P  Rs 860, 
 
 20  R  9% and T  4 years 
second part = Rs  4350  
 29   Amount = Principal + SI
= Rs 1350. = Rs 860 + Rs 309.60
Hence, the two parts are of Rs 3000 and = Rs 1169.60.
Rs 1350. Ex. 19: The simple interest on a certain sum
Ex. 17: Aman and Amit borrowed Rs 2500 and for 2.5 years at 12% per annum is Rs
Rs 3000 respectively at the same rate 40 less than the simple interest on the
same sum for 3.5 year s at 10% per
1 annum. Find the sum.
of simple interest for 2 years. If
2 Soln: Let the required sum be Rs P.
Amit paid Rs 175 more interest than Case: I. W hen, Pr incipal = Rs P,
324 Concept of Arithmetic

Rat e = 12% per annum and Time Soln:  Yearly interest on Rs 1500 at the rate
= 2.5 years. of 10% per annum
In this case, we have SI
PRT  1500  10  1 
 P  12  2.5  3P = = Rs  
100  100 
= Rs   = Rs
 100  10 = Rs 150
Case: II. W hen, Pr incipal = Rs P, and the yearly interest on Rs 1000 at the
Rat e = 10% per annum and rate of 8% per annum
Time = 3.5 years.
In this case, we have  1000  8  1 
= Rs   = Rs 80
 100 
 P  10  3.5  7P
SI = Rs   = Rs  Total yearly interest received by both

K KUNDAN
 100  20 the sums = Rs 150 + Rs 80 = Rs 230
It is given that the SI in the first case is Both sums = Rs 1500 + Rs 1000 = Rs 2500
Rs 40 less than the SI in the second case.  Remaining sum = Rs 4000 – Rs 2500
7P 3P = Rs 1500
   40 Now,
20 10 Yearly interest on Rs 4000 at the rate of
7P  6P 9.125%
or,  40
20  4000  9.125  1 
= Rs   = Rs 365
 100 
P
or, = 40  Remaining interest
20
= Rs 365 – Rs 230 = Rs 135
or, P = 40 × 20 = 800  He wants Rs 135 as a interest on the
Hence, the required sum = Rs 800. remaining sum Rs 1500.
Alternative Method:
Let the principal be Rs 100. SI  100 135  100
 Rate = = = 9%
Case I: When, Rate = 12% per annum and PT 1500  1
Time = 2.5 years. In this case, we have
2
P Ex. 21: A man lends of his capital at the
SI = Rs = Rs 30 5
20
Case II: When, Rate 10% per annum and 3
rate of 8% per annum, of his capital
Time = 3.5 years. In this case we have 8
 100  12  3.5  at the rate of 10% per annum and the
SI = Rs   = Rs 35 r em ai ni ng at the rat e of 12% per
 100 
annum. If his annual income is Rs 965,
Difference in the simple interests in the then find his capital.
two cases = Rs (35 - 30) = Rs 5 Soln: Suppose the capital = Rs 100
Now,
If the difference is Rs 5, the principal 2
of Rs 100 = Rs 40

K KUNDAN
= Rs 100 5
 If the difference is Re 1, the principal
3 75
100 of Rs 100 = Rs = Rs 37.50
= Rs 8 2
5  Remaining capital
If the difference is Rs 40, the principal = Rs (100 – 40 – 37.50) = Rs 22.50
Annual interest on Rs 40 at the rate of
 100 
= Rs   40  = Rs 800
 5   40  8  1 
8% = Rs   = Rs 3.20
Hence, the required sum = Rs 800  100 
Ex. 20: A person has Rs 4000. He lends Rs Annual interest on Rs 37.50 at the rate of
1500 at the rate of 10% per annum and
Rs 1000 at the rate of 8% per annum  37.50  10  1 
10% = Rs   = Rs 3.75
interest. At what rate of interest would  100 
he lend the remaining amount so that Annual interest on Rs 22.50 at the rate of
his t ot al yearl y income shoul d be
9.125% of the total sum.  22.50  12  1 
12% = Rs   = Rs 2.70
 100 
Simple Interest 325

 Total annual income on Rs 100


y  t  r 100  y
= Rs (3.20 + 3.75 + 2.70) = Rs 9.65 or,   z x
100 r t
 When annual income is Rs 9.65, then
capital = Rs 100 [Putting the value of x
 W hen annual income is Re 1, then from equation (i)]
00  y2 = zx (proved)
capital = Rs Ex. 24: Divide Rs 2379 into three parts so that
9.65
their amount after 2, 3 and 4 years
 When annual income is Rs 965, then
may be equal, the rate of interest being
 00  5% per annum.
capital = Rs   965  = Rs 10000
 9.65  Soln: Let t he sum in f irst , second and
t hir d case be Rs x, Rs y and Rs z
Ex. 22: Divide Rs 2000 into two sums such that,

K KUNDAN
respectively.
if the first be put out at simple interest
Now, according to the question,
1 The amount in first case of Rs x.
for 6 years at 3 per cent, and the
2
 x  25  11x
1 = Rs  x   = Rs
second for 3 years at 4 per cent, the  100  10
2
i nter est of t he fi rst sum woul d be The amount in second case of Rs y
double that of the second.
 y  35  23y
Soln: Let the first part be Rs x, then = Rs  y   = Rs
second part be Rs (2000 – x).  100  20
The amount in third case of Rs z
x 67 21x
Interest on first part = = Rs  z 45  6y
100  2 100
= Rs  z   = Rs
Interest on second part  100  5
Again, according to the question,
2000 – x   3  9
2000  x   27
= = 11x 23y 6y
100  2 200   k
According to the question, 10 20 5
21x 2  2000  x   27 10k 20k 5k
 x = ; y = ; and z =
100 200 11 23 6
or, 21x + 27x = 2000 × 27
10k 20k 5k
x: y : z= : :
2000  27 11 23 6
or, x 
48
20k 20k 20k 1 1 1
 x = 1125 = : : = : :
Hence, first part = Rs 1125 and 22 23 24 22 23 24
the second part = Rs (2000 – 1125) 22  23  24 22  23  24 22  23  24
= Rs 875. = : :
Ex. 23: x, y, z are three sums of money such 22 23 24

K KUNDAN
that y is the simple interest on x, and = 276 : 264 : 253
z is the simple interest on y for the  Dividing Rs 2379 into three parts in
same time and rate. Show that y2 = zx. the ratio 276 : 264 : 253, we get
Soln: Let the rate be r per cent per annum and 276
the time be t years. x=  2379
276  264  253
Now, according to the question,
x t r 276
y =  2379 = Rs 828
100 793
100y 264
or, x  ....(i) y =  2379 = Rs 792
t r 793
Again,
y t r 253
z z =  2379 = Rs 759
100 793
Hence, t he t hr ee par t s ar e Rs 828,
y t r
or, x  zx Rs 792 and Rs 759.
100
[Multiplying both sides by x]
326 Concept of Arithmetic

Ex. 25: What annual payment will discharge a = Rs 480x + Rs 600(x + 1)


debt of Rs 770 due in 5 years, the rate = Rs 480x + Rs 600x + Rs 600
of interest being 5 per cent per annum? = Rs 1080x + Rs 600
Soln: Let Rs P be the annual payment. Total interest paid = Rs 9240 (Given in
The amount of Rs P in 4 years at 5% the question)
 1080x + 600 = 9240
 4  5  120P
= P 1   or, 1080x = 9240 – 600 = 8640
 100  100
8640
The amount of Rs P in 3 years at 5% or, x = = 8
1080
 5  3  115P
= P 1    First rate is 8% and second is 9%.
 100  100 Ex. 27: A sum of money was lent at simple

K KUNDAN
The amount of Rs P in 2 years at 5%
1
5  2  110P int erest at 11% per annum for 3
 2
= P 1  
 100  100
1
The amount of Rs P in 1 year at 5% years and 4 years respectively. If the
2
 5  1  105P difference in interest for two periods
= P 1  
 100  100 was Rs 412.50, then find the sum?
These four amounts together with the last Soln: Suppose the Principal (P) = Rs 100;
annual payment of Rs P will discharge Rate = 11%
the debt of Rs 770. 1
If the Time (T) = 3 years
120P 115P 110P 105P 2
     P  770
100 100 100 100
100  11  7 77
Then SI = Rs = Rs
550P 100  2 2
or,  770
100
1
If the time (T) = 4 years
770  100 2
or, P = = 140.
550
100  11  9 99
Hence annual payment = Rs 140. Then, SI = Rs = Rs
100  2 2
Note: W e hav e a separ ate chapter on
‘Instalment’ in which various types of  99 77 
questions have been discussed in detail.  Difference in SI = Rs   
 2 2 
Ex. 26: A man borrowed Rs 16000 at certain
rate of interest and Rs 20000 at 1% 22
= Rs = Rs 11
higher than the first. He paid Rs 9240 2
as interest in 3 years. Find the two If the difference is Rs 11, Principal
rates at which he borrowed the money. = Rs 100
Soln: Interest on Rs 20000 at 1% for 3 years If the difference is Rs 412.50, Principal

K KUNDAN
 20000  1  3  100
= Rs   = Rs 600 = Rs  412.50 = Rs 3750.
 100  11
 Interest on Rs 16000 + Rs 20000 at Ex. 28: A sum of money amounts to Rs 944 in
equal rate for 3 years 3 years at a simple interest. If the rate
= Rs 9240 – Rs 600 = Rs 8640 of interest be raised by 25 per cent, the
sum am ount s t o Rs 980 duri ng t he
8640  100 same period. Find the sum and the rate
 Rate = = 8%
36000  3 of interest.
 First rate = 8% and Soln: Let the rate be R
second rate = (8 + 1) = 9% The amount in 3 years = Rs 944
Alternative Method: Let the Principal be P
Let the first rate be x% and the second be Then interest = Rs (944 - P)
(x + 1)%. PRT
Then total interest on Rs 16000 and Rs But SI =
100
20000 in 3 years
PR3
 16000  x  3   20000  (x  1)  3  or, 944 – P = .... (1)
= Rs   + Rs   100
 100   100 
Simple Interest 327

In the second case or, (5450 + 3000 – 7000)


The rate per cent is increased by its 25%
7000  3  r 4000  2  r
Then the rate per cent = 
100 100
25 5R
=R R  or, 1450 = 210r + 80r
100 4
1450
Amount after three years = Rs 980  r = = 5%
 Interest after three years 290
= Rs (980 - P) Ex. 31: A person invested some amount at the
rate of 12% simple interest and some
R other amount at the rate of 10% simple
P 5 3
980 – P = 4 ..... (2) interest. He received yearly interest of

K KUNDAN
100 Rs 130. But if he had interchanged the
Dividing equation (1) by equation (2), we am ount s i nvest ed, he woul d hav e
get received Rs 4 more as interest. How
much amount did he invest at different
4 944 – P rates?

5 980 – P Soln: Suppose the person invested Rs x at the
or, 3920 – 4P = 4720 – 5P rate of 12% simple interest and Rs y at
or, 5P – 4P = 4720 – 3920 the rate of 10% simple interest.
or, P = 800
12x 10y
Putting the value of P in equation (1) Then, yearly interest =  .
We get, 100 100

800  R  3 12x 10y


944 – 800 =   = 130
100 100 100
or, 14400 = 2400R or, 12x + 10y = 13000
or, 6x + 5y = 6500 .... (i)
14400 If the invested amounts are interchanged,
or, R = = 6%
2400 then yearly interest increases by Rs 4.
 Rate = 6% 10 x 12y
Ex. 29: The simple interest on a sum of money   = 134
100 100
will be Rs 300 after 5 years. In the next
5 years principal is trebled, what will or, 10x + 12y = 13400
be the total interest at the end of the or, 5x + 6y = 6700 .... (ii)
10th year? Subtracting (ii) from (i), we get
Soln: Simple interest for 5 years = Rs 300 x – y = –200 .... (iii)
Now, when pr incipal is tr ebled, t he Adding (ii) to (i), we get
simple interest for 5 years will also be 11x + 11y = 13200
t reble t he simple int er est on original or, x + y = 1200 .... (iv)
principal for the same period. Thus SI for Adding (iii) and (iv), we get
last 5 years when principal is trebled 2x = 1000

K KUNDAN
= 3 × 300 = Rs 900  x = 500.
 Total SI for 10 years = Rs (300 + 900) Putting x = 500 in (iii), we get y = 700.
= Rs 1200 Thus, the person invested Rs 500 at the
Ex. 30: Ramesh borrows Rs 7000 from a bank rate of 12% per year and Rs 700 at the
at SI. After 3 years he paid Rs 3000 to rate of 10% per year.
the bank and at the end of 5 years from Ex. 32: The rate of interest for the first 2 years
the date of borrowing he paid Rs 5450 is 3% per annum, for the next 3 years
to the bank to settle the account. Find is 8% per annum and for the period
the rate of interest. beyond 5 years 10% per annum. If a
Soln: Any sum that is paid back to the bank man gets Rs 1520 as a simple interest
before the last instalment is deducted from for 6 years, how much money did he
the principal and not from the interest. deposit?
Thus, Soln: Let the deposit be Rs 100.
Total interest = Interest on Rs 7000 for 3 Interest for first 2 years at 3% simple
years + Interest on (Rs 7000 - Rs 3000 =)  100  3  2 
Rs 4000 for 2 years. interest = Rs   = Rs 6
 100 
328 Concept of Arithmetic

Interest for next 3 years at 8% simple  Total interest = Rs 6 + Rs 24 + Rs 10


= Rs 40
 100  8  2 
interest = Rs   = Rs 24  When inter est is Rs 40, deposited
 100  amount is Rs 100.
Interest for the last year at 10% simple  When interest is Rs 1520, deposited
 100  10  1   100 
interest = Rs   = Rs 10 amount = Rs  1520  = Rs 3800
 100   40 

Practice Exercise

K KUNDAN
1.

2.
Rishi deposit ed Rs 20000 in a f inancial
institution on 28th March, 2000 and closes
his account on 21st August, 2000. If the
institution pays an interest of 9% per annum,
what amount does Rishi get?
One can borrow money from a bank for setting
up a dairy or a poultry farm. Vijay set up a
poultry farm and had to invest Rs 8888 for
9. The simple interest on Rs 1650 will be less
than the interest on Rs 1800 at 4% simple
interest by Rs 30. Find the time.
10. If simple interest on Rs 1800 exceeds the
interest on Rs 1650 in 3 years by Rs 45. Find
the rate per cent per annum.
11. A farmer borrowed Rs 2400 at 12% interest
1
this purpose. Under the rules for loan, the per annum. At the end of 2 year s, he
bank loans at the most 75% of the tot al 2
investment and the person concerned has to cleared his account by paying Rs 1200 and a
inv est t he r emaining 25% of t he t ot al cow. Find the cost of the cow.
investment. The rate of simple interest is 12. Two equal amounts of money are deposited
in two banks each at 15% per annum for 3.5
1 year s and 5 year s respect ively. If t he
12 % per annum. Vijay borrowed as much
2 difference between their interests is Rs 144,
money from the bank as he could under the find each sum.
13. The difference between the interest received
1
rules. After 1 years, he cleared the loan. from two different banks on Rs 500 for 2 years
2 is Rs 2.5. Find the difference between their
How much money did he return to the bank? rates.
3. In how much time will the simple interest 14. A sum was put at SI at a certain rate for 2
on a certain sum be 0.125 times the principal years. Had it been put at 3% higher rate, it
at 10% per annum? would have fetched Rs 300 more. Find the
sum.
1
4. The simple interest on a sum of money is 15. A person lent a certain sum of money at 4%
9 simple interest and in 8 years the interest
of the principal, and the number of years is amounted to Rs 340 less than the sum lent.
equal to the rate per cent per annum. Find Find the sum lent.

K KUNDAN
the rate per cent. 16. Arun and Ramu are friends. Arun borrowed a
5. A sum of money doubles itself in 4 years at a sum of Rs 400 at 5% per annum simple interest
simple interest. In how many years will it fr om Ramu. He ret urns the amount with
amount to 8 times itself? interest after 2 years. Ramu returns to Arun
1 2% of the total amount returned. How much
6. A money lender wants of t he amount did Arun receive?
5
17. A person wants to divide a sum of Rs 375000
loaned every year as interest. What is the rate between his son and daughter aged 12 years
of interest? If a farmer borrows Rs 5000 for 1 and 14 years respectively so that when they
year f rom the money lender, what is t he attain the age of 18 years, the amounts to be
amount that he has to pay back altogether? received by each at 5% simple interest per
7. If Rs 450 amounts to Rs 540 at 10% simple annum will be the same. Find the share of
interest, find the principal that amounts to each.
Rs 708 at 9% simple interest in the same 18. Divide Rs 7053 into three parts so that the
period. amount after 2, 3 and 4 years respectively
8. A sum of money lent out at simple interest may be equal. The rates of interest being 4%
amounts to Rs 1440 in 2 years and Rs 2040 in per annum.
7 years. Find the rate of interest per annum. 19. A sum of money at simple interest amounts to
Simple Interest 329

Rs 9440 in 3 years. If the rate of interest is was the original sum borrowed?
increased by 25% , the same sum of money 24. A boy aged 10 years is left with Rs 50000
amounts to Rs 9800 in the same time. Find which is under trust. The trustees invest the
the sum and the rate of interest. money at 4% per annum and pay the minor
20. A person deposited Rs 8000 in a bank. After boy a sum of Rs 1200 for his pocket money at
2 years he withdrew Rs 3000 and at the end the end of each year. The expenses of trust
of 4 years he received an amount of Rs 6800. come out to be Rs 300 per annum. Find the
Find the rate of simple interest. amount that will be handed over to the minor
21. When the rate of interest in a bank is increased boy after he attains the age of 18 years.
from 5% to 6% per annum, a person deposits 25. A man deposits some money in a savings bank
Rs 1000 more into his account. If the annual and at the end of one year earns Rs 15 as
interest now received by him is Rs 110 more interest. He put in another Rs 85 and deposits

K KUNDAN
than that before, find his original deposit. the money for another one year. After the
22. A man derives his income from the investment expiry of this period he gets Rs 420 as the
of Rs 4150 at a certain rate of interest and Rs sum total of the principal and the interest.
3500 at 1 per cent higher than the first. His What amount was originally deposited and
whole income for 4 years is Rs 1211. Find what rate of interest the bank paid?
the rates of interest. 26. Naresh borrowed Rs 28000 from a bank at
23. A sum of money was borrowed at 6% per simple interest. After three years he paid back
annum simple interest. At the end of first Rs 12000 to bank and after two more years
year Rs 6800 was paid off and the rate of he paid back 21800 and settled the account.
interest on the balance was reduced to 5% Find the rate of interest charged by the bank.
per annum. If the interest for the second year 27. At what rate per cent per annum of simple
11 interest, will a sum of money double itself in
was of the interest for the first year, what 12 years?
20

Answers and explanations


1. Here Principal P = Rs 20000 and 2. Total investment = Rs 8888
Rate R = 9% per annum. Total loan received by Vijay = 75% of Rs 8888
In calculating the time, we count the number
 75 
of days, excluding t he date on which the = Rs   8888  = Rs 6666.
money was deposited and including the date  100 
on which the money was withdrawn.
1
Thus, we have Now, Principal (P) = Rs 6666, Rate (R) = 12 %
Number of days = 3 days (March) + 30 days 2
(April) + 31 days (May) + 30 days (June) + 31
1
days (July) + 21 days (Aug) = 146 per annum and Time (T) = 1 years
2
146 2
 Time = 146 days = year = year.

K KUNDAN
365 5 25 3
PRT 6666  
Now, interest on Rs 100 for one year = Rs 9  SI = = Rs 2 2
100 100
2  2
Interest on Rs 100 for year = Rs  9   6666 3
5  5
= Rs = Rs 1249.875
16
2  Amount = Principal + SI
Interest on Rs 20000 for year
5 = Rs 6666 + Rs 1249.875
= Rs 7915.875.
 2 1 
= Rs  9    20000  = Rs 720. Hence, Vijay returned Rs 7915.875 to the
 5 100  bank.
 Interest = Rs 720. 3. Let the principal be Rs P.
Now, Amount = Principal + Interest We have, Principal = Rs P, SI = 0.125 P and
= Rs 20000 + Rs 720 R = 10% per annum.
= Rs 20720. We have to find the time T.
Thus, the man receives Rs 20720 on 21st
SI  100
August 2000. Now, T =
PR
330 Concept of Arithmetic

7. Amount in the first case = Rs 540


0.125P  100
 T = years Principal = Rs 450
P  10 SI = Rs 540 – Rs 450 = Rs 90
Rate = 10%
12.5P 2.5
= years = years = 1.25 years. SI  100 90  100
10P 2
 Time = = = 2 years
4. Let Principal = P, time = R years, rate = T PR 540  10
PTT P Now, in second case, amount = Rs 708
Then  Rate = 9% and Time = 2 years
100 9 Let the principal be Rs 100
2 100 PRT 100  9  2
 T  Then, SI = = = Rs 18

K KUNDAN
9 100 100
10 1  Amount = Principal + Simple Interest
 T = 3 = Rs 100 + Rs 18 = Rs 118
9 3
 When amount is Rs 118, principal
1 = Rs 100
 rate = 3 %
3 100
 When amount is Re 1, principal = Rs
5. Let the sum of money be Rs 100. 118
 Amount in 4 years = 100 × 2 = Rs 200  When amount is Rs 708, principal
 Simple interest = Rs 200 – Rs 100
100  708
= Rs 100 = Rs = Rs 600
118
SI  100 100  100 Note: When, amount, rate per cent per annum
 Rate = = = 25% and time is given, to find sum we can apply
PT 100  4
Now, amount in T years = Rs 100 × 8 the formula as given below:
= Rs 800 100  Amount
 Interest in T years Sum = 100  R  T
= Rs (800 – 100) = Rs 700
In the above case,
SI  100 700  100
 Time = T = = = 28 years  100  708  100  708 
PR 100  25
Sum = Rs   = Rs  
Alternative Method: 100  (2  9)   100  18 
A sum of money doubles in 4 years.
3 times in 4 × 2 = 8 years 100  708 
= Rs   = Rs 600
4 times in 4 × 3 = 12 years  118 
... ... ... ... ... ... 8. According to the question,
... ... ... ... ... ... Principal + SI for 2 years = 1440 ....... (i)
... ... ... ... ... ... Principal + SI for 7 years = 2040 ....... (ii)
8 times in 4 × 7 = 28 years Subtracting (i) from (ii), we have
1 SI for 5 years = Rs 600
 SI for 1 year = Rs 120

K KUNDAN
6. Since t he money lender wants of the
5
From (i), we have
amount loaned every year as interest. So, if Principal + 120 × 2 = 1440
money lender gives a loan of Rs 100, then or, Principal = 1200
1  100  SI 100  120
interest after one year = Rs  100  = Rs 20.  Rate of interest =   10%
5  PT 1200  1
Thus, the rate of interest is 20% per annum. 9. We may consider that Rs (1800 - 1650) gives
Now, interest on Rs 100 for one year = Rs 20. interest of Rs 30 at 4% per annum.
20 30  100
Interest on Re 1 for one year = Re  Time = = 5 years.
100 150  4
Hence, interest on Rs 5000 for one year 10. First principal = Rs 1800
Second principal = Rs 1650
 20 
= Rs   5000  = Rs 1000 Difference = Rs 1800 – Rs 1650 = Rs 150
 100   Rs 45 in SI on Rs 150 in 3 years.
Amount to be paid = Principal + Interest
= Rs 5000 + Rs 1000 = Rs 6000 SI  100 45  100
 Rate % = = = 10%
PT 150  3
Simple Interest 331

11. We have, Principal P = Rs 2400, or, xy + 3x – xy = 15000


Rate of Interest R = 12% per annum and or, x = 5000
Thus, the sum = Rs 5000
1 5
Time T = 2 years = years 15. Let the sum be Rs x.
2 2 Now, according to the question,
ie Interest on Rs 100 for one year = Rs 12 x 84
 x  340
5 100
 Interest on Rs 100 for years
2 8x
or,  x  340
5 25
= Rs 12 × = Rs 30. or, 8x = 25x – 25 × 340
2
or, 17x = 25 × 340

K KUNDAN
5  30  340  25
Interest on Re 1 for years = Re  
 x = = Rs 500
2  100  17
5  Sum lent = Rs 500
Hence, interest on Rs 2400 for years 16. Here, borrowed sum = Rs 400
2
Rate of interest = 5% per annum
30 Time = 2 years
= Rs  2400 = Rs 720
100  400  5  2 
 Amount = Principal + Interest SI = Rs   = Rs 40
 100 
= Rs 2400 + Rs 720 = Rs 3120.
After 2 years amount returned to Ramu
It is given that the farmer cleared his account
= Principal + SI
by paying Rs 1200 and a cow.
= Rs (400 + 40) = Rs 440
Therefore, Amount = Rs 1200 + Cost of the
Amount returned to Arun = 2% of Rs 440
cow
or, Rs 3120 = Rs 1200 + Cost of the cow 2
or, Cost of the cow = Rs 3120 – Rs 1200 =  440 = Rs 8.80
100
= Rs 1920.
17. The son aged 12 years will attain the age of
12. Let the sum be Rs x, then
18 years after (18 – 12 =) 6 years.
x  15  5 x  15  7 The daughter aged 14 years will attain the
  144
100 200 age of 18 years after (18 – 14 =) 4 years.
Let the sum allotted to the son be Rs x
or, 150x – 105x = 144 × 200
 Sum allotted to the daughter
144  200 = Rs (375000 – x)
 x = = Rs 640 Then, according to the question,
45
Amount of Rs x for 6 years at 5% rate of
500  2  r1 interest
13. I1   10 r1
100 = Amount of Rs (375000 – x) for 4 years at 5%
rate of interest.
500  2  r2

K KUNDAN
I2   10 r2 x 65 (375000  x )  4  5
100  x  (375000  x ) 
100 100
I1  I 2  10 r1  10 r2 = 2.5 3x 1

or, x   (375000  x ) 1  
2.5 10  5
or, r1  r2 = 0.25%
10 13x 6
or,  (375000  x )
14. Let the sum be Rs x and the original rate be 10 5
y% per annum. Then, new rate = (y + 3)%
per annum. 13x
or,  6 (375000  x )
Now, according to the question, 2
x (y  3)  2 x (y )  2 or, 13x = 4500000 – 12x
  300 or, 13x + 12x = 4500000
100 100
2xy  6x  2xy 4500000
or,  300 or, x = = 180000
100 25
 Sum allotted to the son = Rs 180000 and
xy  3x  xy the sum allotted to the daughter
or,  300
50 = Rs (375000 – 180000) = Rs 195000
332 Concept of Arithmetic

18. Let the sum in first, second and third case be Dividing equation (1) by equation (2), we get
Rs x, y and z respectively.
4 9440  P
Now, according to the question, 
The amount in first case of Rs x 5 9800  P
 P = 8000
 x  2 4   27x 
= Rs  x   = Rs   Putting the value of P in equation (1), we get
 100   25  R = 6%.
The amount in second case of Rs y Alternative Method:
Let the principal be Rs P and the rate of
 y  3 4   28y 
= Rs  y   = Rs   interest be R%
 100   25  In first case, applying the SI formulae
The amount in third case of Rs z
RT

K KUNDAN

 z 44  29z  A = P 1  
 100 
= Rs  z   = Rs  
 100   25 
 3R 
Again, according to the question, 9440 = P 1  
 100 
27x 28y 29z
  k 9440  100
25 25 25 or, P
100  3R
25k 25k 25k In second case
x= , y = and z =
27 28 29
 5R  3 
25k 25k 25k 1 1 1 9800 = P 1  
: : : :  4  100 
x: y : z = =
27 28 29 27 28 29
or, x : y : z  400  15R 
or, 9800  P  
 400 
27  28  29 27  28  29 27  28  29
= : :
27 28 29 9800  400
or, P
or, 812 : 783 : 756 400  15R
But the principal in both the cases is same.
812
Now x =  7053 = Rs 2436 So computing the principal in both the cases,
812  783  756 we get
783 9440  100 9800  400
y =  7053 = Rs 2349 
812  783  756 100  3R 400  15R
756 944 980  4
z =  7053 = Rs 2268. or, 
812  783  756 100  3R 400  15R
19. Let the rate be R.
or, 944 400  15R   980  4100  3R 
The amount in 3 years = Rs 9440.
Let the principal be P. or 944  400  944  15R  980  400  980  12R
Then, interest = Rs (9440 – P)

K KUNDAN
or, 377600  14160R  392000  11760R
PRT or, 14160R  11760R  392000  377600
But SI =
100
or, 2400R  14400
PR3
or, (9440 – P) = ....(1) 14400
100 or, R =  6%
2400
In the second case,
 The required rate percentage is 6% .
The rate per cent is increased by its 25%.
2 0 . Let the rate of interest be R% per annum.
25 5R Since Rs 8000 were with the bank for 2 years
Then, the rate per cent = R  R 
and the balance ie Rs (8000 - 3000) = Rs 5000
100 4
Amount after three years = Rs 9800 were with the bank for 4 years. Therefore,
 Interest after three years = Rs (9800 – P) Total SI earned = SI on Rs 8000 for 2 years +
SI on Rs 5000 for 4 years
5R
P 3 8000  R  2 5000  R  4
or, (9800 – P) = 4 ....(2) = Rs + Rs
100 100
100
= Rs 160R + Rs 200R = 360 R
Simple Interest 333

Also, SI earned = Rs (3000 + 6800 - 8000) 23. Let the sum be Rs x.


= Rs 1800 Simple Interest at 6% for one year
 360 R = 1800 6 1  x 6x
 
1800 100 100
 R = = 5
360
6x 106x
Hence, required rate of interest = 5% per Amount = x  
annum. 100 100
Alternative Method: Principal for second year after return of Rs
We have, interest received 6800
= Rs (3000 + 6800 - 8000) = Rs 1800 106x
(SI on Rs 8000 for 2 years) + (SI on Rs 5000   6800
for 4 years) = Rs 1800 100

K KUNDAN
or, (SI on Rs (8000 × 2) for 1 year) + (SI on Rs Simple interest at 5% on this amount
(5000 × 4) for 1 year) = Rs 1800  106 x  5
or, (SI on Rs 16000 for 1 year) + (SI on Rs   6800  
20000 for 1 year) = Rs 1800  100  100
or, SI on Rs 36000 for 1 year = Rs 1800 Now according to the question,
or SI on Rs 100 for 1 year 6x 11  106 x  5
   6800  
 1800  100 20  100  100
= Rs  100  = Rs 5
 36000 
66x 106x 6800  5
Hence, required rate of interest or,  5
2000 10000 100
= 5% per annum.
21. Let the original deposit be Rs P. 6800  5 106x  5 66x
Then, simple interest on Rs P for one year at or,  
100 10000 2000
(6 – 5)% = 1% per annum + SI on Rs 1000 for
one year at 6% per annum = Rs 110. 530x 66x 530x  330x
or, 340   =
P  1  1 1000  6  1 10000 2000 10000
or,   110
100 100 200x
P or, 340 
 60  110 10000
or,
100
340  10000
P or, x   17000
or,  110  60 200
100
 the required amount is Rs 17000.
P 24. Principal in this case = Rs 50000
or, = 50 Rate of interest per annum = 4%
100
So, interest for one year
or, P = 50 × 100 = 5000
Hence, original deposit = Rs 5000. 4
=  50000 = Rs 2000
2 2 . Let the first rate be x% and the second be 100
(x + 1)%.

K KUNDAN
Expenditure per annum = Pocket money of
Then total interest on Rs 4150 and Rs 3500 the boy + expenses of trust
in 4 years = Rs (1200 + 300) = Rs 1500
 4150  x  4   3500  (x  1)  4   Net earnings per annum
= Rs   + Rs   = Rs 2000 – Rs 1500 = Rs 500
 100   100 
Time after which boy will attain the age of 18
= Rs 166x + Rs 140 (x + 1) years = 8 years
= Rs (306x + 140) Total income at the end of 8 years
Total income = Rs 1211 = 8 × 500 = Rs 4000
 306x + 140 = 1211 Total amount given to the boy at the end of 18
or, 306x = 1211 – 140 = 1071 years = Rs 50000 + Rs 4000 = Rs 54000.
1071 7 1 25. Let the Principal be P and the Rate R%
or, x =  3 %
306 2 2 P  R 1
 15 
1 100
 First rate is 3 % and the second rate is
2 PR
or, = 15
 1  1 100
 3 %  1  4 % .  PR = 1500
 2  2
334 Concept of Arithmetic

Changed principal = P + 15 + 85 26. Let the rate of simple interest be x% per


= Rs (P + 100) annum.
After 3 years
 R 
 Amount = (P + 100) 1   Principal = Rs (28000 – 12000) = Rs 16000
 100  Interest charged by the bank
= Rs (12000 + 21800 – 28000)
PR
or, 420 = P + + 100 + R = Rs 5800
100 According to the question,
1500 28000  3  x 16000  2  x
or, 420 = P + + 100 + R  = 5800
100 100 100
= P + 15 + 100 + R [  PR = 1500] or, 840x + 320x = 5800

K KUNDAN
or, P + R = 305 or, 1160x = 5800
or, (P  R)2  (P  R)2  4PR  (305) 2  4  1500 5800
or, x = = 5
2
or, (P  R)  93025  6000  87025 1160
 Rate of interest = 5% per annum.
or, (P  R)2  87025  295 27. Let the principal be Rs P
Now, P + R = 305 .....(i)  Amount = Rs 2P
P – R = 295 .....(ii)  SI = Rs (2P – P) = Rs P
Now, on adding equations (i) and (ii), we have, SI  100 P  100
2P = 600  Rate = Principal  Time  P  12
600
or, P = = 300 25 1
2 = 8
On putting the value of P in either equation 3 3
(i) or in equation (ii), we have, 1
R = 5 Hence the required rate per cent = = 8 %.
3
 Principal = Rs 300 and Rate = 5%.

K KUNDAN
Compound Interest 335

Chapter-18

Compound Interest

K KUNDAN
Introduction
You know that, if Principal = Rs P, Rate = R% per
annum and Time = T (in years), then the Simple
Interest (SI) in Rs is given by

SI =
PRT
100
Clearly, compound interest at the end of certain
specified period is equal to the difference between
the amount at the end of the period and the original
principal ie CI = Amount – Principal.

Conversion Period
The fixed interval of time at the end of which the
interest is calculated and added to the principal at
For example, if Principal = Rs 5000 and
rate of interest = 10% per annum, then t he beginning of t he inter v al is called t he
conversion period.
 5000  10  1  In other words, the period at the end of which
SI for 1 year = Rs   = Rs 500
 100  the interest is compounded is called the conversion
period.
 5000  10  2  When the interest is calculated and added to
SI for 2 years = Rs   = Rs 1000
 100  the principal every six months, the conversion
period is six months. Similarly, the conversion
 5000  10  3  period is thr ee mont hs when t he int er est is
SI for 3 years = Rs  
 100  calculated and added quarterly.
= Rs 1500 and so on. Note: If no conversion period is specified, the
Clearly, in computing SI the principal remains conversion period is taken to be one year.
constant throughout. But the above method of
computing interest is generally not used in banks,
Computation of Compound Interest
insurance corporations, post offices and other
money lending and deposit taking companies. They (i) By the method when the interest is
use a different method for computing interest. In calculated and added to the principal
this method, the borrower and the lender agree to every interval
fix up a certain time interval, say a year or a half-
For example, the compound interest on Rs 1000
year or a quarter of a year for the computation of
for 2 years at 4% per annum is Rs 81.60. Let us
interest and amount. At the end of first interval
see. How?
the interest is computed and is added to the original

K KUNDAN
Principal for the first year = Rs 1000
principal. The amount so obtained is taken as the
Interest for the first year
pr incipal for the second inter val of time. The
amount of this principal at the end of the second  1000  4  1 
interval of time is taken as the principal for the = Rs   = Rs 40
 100 
third interval of time and so on. At the end of
certain specified period, the difference between  P  R  T
the amount and the money borrowed ie the original Using : Interest  100 

principal is computed and it is called the compound
Amount at the end of first year
interest (abbreviated as CI) for that period. Thus,
= Rs 1000 + Rs 40 = Rs 1040
we may define the compound interest as follows:
Principal for the second year = Rs 1040
If the borrower and the lender agree to fix up a
Interest for the second year
certain interval of time (say, a year or a half-year
or a quarter of a year etc) so that the amount ( =  1040  4  1 
Principal + Interest) at the end of an interval = Rs   = Rs 41.60
 100 
becomes the principal for the next interval, then
Amount at the end of second year
the total interest over all the intervals calculated
= Rs 1040 + Rs 41.60
in this way is called the compound interest and
= Rs 1081.60
is abbreviated as CI.
336 Concept of Arithmetic

 Compound interest = Rs (1081.60 – Rs 1000) 2


= Rs 81.60  R   R 
= Rs P 1   1  
 100   100 
(ii) By the Formula
3
We have seen in the above example that it takes a  R 
= Rs P 1  
lot of time to find compound interest. Hence, we  100 
explore below a formula for finding compound
interest.  2 
 R 
Let principal be Rs P.  P 1   becomes a common factor 
Rate = R%   100  

K KUNDAN
Time = T years Proceeding in the same manner, amount at the
 Interest for first year end of T years
 P  R 1  PR  T
= Rs   = Rs    R 
 100   100  = Rs P 1  
 100 
 Principal for second year
CI (Compound Interest) for T years
 PR   R 
= Rs P + Rs   = Rs P 1   T
 100   100   R 
= Rs P 1   – Rs P
 Interest for second year  100 

 R    T 
 R 
 P  1  100   R  1  = Rs P 1  100   1
= Rs       
 100 
  Thus, we get the formula for finding the amount
in case of compound interest as
PR  R  Time
= Rs 1    Rate 
100  100  A = P 1  
 100 
 Amount at the end of second year or principal
Now, we solve the above example, by using the
for third year
above formula
 R  PR  R  2
= Rs P 1   + Rs 1    4 
 100  100  100  A = Rs 1000 1  
 100 
 R  R   1000  104  104 
= Rs P 1   1   = Rs   = Rs 1081.60
 100   100   100  100 
2  CI = Rs 1081.60 – Rs 1000 = Rs 81.60
 R 
= Rs P 1  
 100  Computation of Compound Interest when
Interest is Compounded Half-Yearly or
  R  
 P 1  100  becomes a common factor Quarterly

K KUNDAN
   
In compound interest, the time from one specified
Interest for the third year period to the next is known as the conversion
2
period as stated earlier. If time is one year, there
  R   is one conversion period a year. If the time is six
 P  1    R  1
  100   months, there are two conversion periods a year.
= Rs   If t he t ime is t hr ee mont hs, t her e ar e four
100
  conversion periods a year.
 
As stated earlier the rate of interest is usually
quoted as per cent per annum. Thus when the
2
PR  R  interest is calculated :
= Rs 1   ( i ) quarterly, the rate of interest per conversion
100  100 
 Amount at the end of third year 1
period is of the rate stated yearly.
4
2 2
 R  PR  R  ( i i) half-yearly, the interest rate per conversion
= Rs P 1   + Rs 1  
 100  100  100  1
period is of the rate stated yearly.
2
Compound Interest 337

For example, we have to find the compound Computation of Compound Interest, when
1 Time is not an Exact Number of Years
interest on Rs 12000 for 1 years at 16% per
2
annum, interest being compounded (a) quarterly Suppose we have to find Compound Interest (CI)
and (b) half-yearly. We proceed as follows: 1
(a) Principal = Rs 12000 for 2 years on a certain sum at a certain rate per
2
Rate of Interst = 16% per annum
1
16 cent. The Compound Interest for 2 years will be
= = 4% per quarter 2
4

K KUNDAN
equal to the Compound Interest for 2 years at the
1 3  given rate together with Compound Interest for 1
Time = 1 years =   4  = 6 quarters
2 2  1
year at of the given rate. Thus interest for any
T 2
 R  fraction of a year is the same as the interest for
 A = P 1  100 
  full year at the rate equal to the same fraction of
6 the rate.
 4  For example, if we have to find the compound
= Rs 12000 1  
 100 
1
interest on Rs 25000 at 13% per annum for 2
6 2
 104 
= Rs 12000   = Rs 15183.83 years, we proceed as follows:
 100 
Principal = Rs 25000
 CI = Rs 15183.83 – Rs 12000 Rate = 12%
= Rs 3183.83
(b) Principal = Rs 12000 1
Time = 2 years
Rate of Interest = 16% per annum 2
T
 16   R
=   8% per half-year
 2   A = P 1  100 
 
1 3   12 
Time = 1 years =   2  = 3 half-years 12  
2 
2 2  25000

1  1  2 
= Rs  
 100   100 
T  
 R   
 A = P 1  100  1
 
[  Interest for year at 12%
3 2
 8 
= Rs 12000 1    12 
 100    6%]
= Interest for 1 year at 
 2 
12000  108  108  108
= Rs
100  100  100  12 
2
6 

K KUNDAN
 
= Rs 15116.54 = Rs 25000 1  100  1  
100 
   
 CI = Rs 15116.54 – Rs 12000
= Rs 3116.54
 112 112 106 
Thus, if Principal = P, Time = T years and Rate = Rs  25000    
= R% per annum  100 100 100 
(i) Amount (when int erest compounded = Rs 33241.60
quarterly)  CI = Rs 33241.60 – Rs 25000 = Rs 8241.60
4T Thus, let P be the principal and the rate of
 R 
  R 
4T int er est be R% per annum. If t he interest is

 P 1  4   P 1   compounded annualy but time is the fraction of a
 100   400 
  1
(ii) Amount (when interest compounded half- year say 2 years, then amount A is given by
2
yearly)
2T  R 
 R 
R   
2T 2
   R   2  and CI = A – P
 P 1  2   P 1  A = P 1   1 
 100   200   100  

100 

   
338 Concept of Arithmetic

Alternative Method Here, P = Rs 4000,


R1 = 5% per annum and
In this method, we calculate Compound Interest
R2 = 15% per annum.
for the exact number of years by the formula and
 Amount after 2 years
Simple Interest for the remaining time. And this
Simple Interest should be added to the Compound  R1   R 
= P 1   1  2 
Interest.  100   100 
For example, solve the above example.
Principal = Rs 25000  5  15 
= Rs 4000  1   1  
Rate = 12%  100   100 

K KUNDAN
1
Time = 2 years  1  3 
2 = Rs 4000  1   1  
 20   20 
Compound Interest for the first 2 years = A – P
T
21 23
 R  = Rs 4000  
 A = P 1  100  20 20
  = Rs 4830.
2
Thus, the refrigerator will cost Rs 4830 to Ram
  12  Singh.
= Rs 25000 1  100  
   
General Formula for Computing Amount
 25000  112  112  Let P be the principal, the rate of interest be R%
= Rs   = Rs 31360 per annum and time be T years and the interest is
 100  100 
 Compound Interest for the first 2 years compounded after each month.
= Rs 31360 – Rs 25000 = Rs 6360  Interest on Rs 100 for 1 year = Rs R
For the next year, Rs 31360 will be Principal  Interest on Rs 100 for x months

1 R 
= Rs  x
 Interest for next year  12 
2
 Interest on Rs P for x months
 1 
PRT  31360  12   P R  x 
= = Rs  2  = Rs  
100  100   100  12 
  Amount at the end of x months
 
= Rs 1881.60  Rx 
 Total Compound Interst PRx  
= Rs 6360 + Rs 1881.60 = Rs 8241.60 = P = P 1  12 
100  12  100 
 
Computation of Compound Interest, when
Interest is Compounded Annually but This amount is considered as principal for the
next x months.
Being Different for Different Years

K KUNDAN
 According to the above,
Let P be the principal and the rate of interest be Amount at the end of next x months
R1% for the first year, R2% for second year, R3% = Principal for the first x months +
for third year and so on and in the last Rn% for the Interest of the next x months
nth year. Then the amount A and the compound
interest (CI) at the end of the n years is given by  Rx   Rx 
    Rx

= P 1 12   P 1  12  

 R1   R   R 
A = P 1   1  2  .......1  n  and  100   100  100  12
 100   100   100     
   
CI = A – P
For example, Ram Singh bought a refrigerator 2
for Rs 4000 on credit. The rate of interest for the  Rx   Rx   Rx 
    
first year is 5% and of the second year is 15% . P 1  12  1  12   P 1  12 
=
How much will it cost him if he pays the amount  100   100   100 
    
after two years?     
To solve this, we proceed as follows:
Compound Interest 339

Similarly, amount for the next x months (e) If the rate of interest is calculated annually,
then x = 12
3
 Rx 
  12T
= P 1  12   R  12  12
T
 100     R 



  Amount = P 1  12  = P 1  
 100   100 
 
Now,  
12 Thus, we can conclude that the above formula
 Number of x months in 1 year = x

K KUNDAN
is the General Formula and it is very useful in
computing Amount (Principal + Compound Interest)
12T and Compound Interest, whether rate is calculated
 Number of x months in T years =
x quarterly, four-monthly, half-yearly, nine-monthly
etc.
12T For example, to find the Compound Interest on
 Rx  x
  1
 Amount (A) for T years = P 1  12 
 Rs 12000 for 1 years at 12% per annum, interest
 100  4
 
  being compounded five-monthly, we proceed as
follows:
(a) If the rate of interest is calculated quarterly, Here, P = Rs 8000,
then x = 3 R = 12%,
12T 1 5
4T
 R3  3  R  T = 1 = and
    4 4
 Amount = P 1  12
  = P 1 4 

x = 5
 100   100 
   
    12T
 Rx  x
(b) If the rate of interest is calculated half-  
12
yearly, then x = 6  Amount (A) = P 1  100 
2T
 
12T
 R   
 R6  6  
  P 1  2 
 Amount = P 1  12
  =  100   125 
 100      12  5  45 
     
   12 
8000 1  
= Rs   100 

(c) If the rate of interest is calculated nine-    
monthly, then x = 9    
 
12T 4T
 R9  9  3R  3  3
     5  
= Rs 8000 1   

K KUNDAN
P 1  12  P 1  4 
 Amount = =   100  
 100   100 
   
   
 21 21 21 
= Rs 8000   
(d) If the rate of interest is calculated four-  20 20 20 
monthly, then x = 4
= Rs 9261
12T  Compound Interest = Rs 9261– Rs 8000
3T
 R4  4  R  = Rs 1261
   
 Amount = P 1  12  = P 1  3 
 100   100 
   
   
340 Concept of Arithmetic

Solved Examples
Ex. 1: Find the com pound i nt er est on Rs 3
12000 for 3 years at 10% per annum  25 
= Rs 64000 × 1  
compounded annually.  1000 
Soln: We know that the amount A at the end of
T years at the rate of R% per annum when 3
 1 
the interest is compounded annually is = Rs 64000 × 1  
 40 
given by

K KUNDAN T 3
 R   41 
A = P 1   = Rs 64000 ×  
 100   40 
Here, P = Rs 12000, R = 10% per annum 41 41 41
and T = 3 years = Rs 64000 ×  
40 40 40
3 = Rs 68921
 R 
 Amount A after 3 years = P 1   Hence, compound interest payable after 3
 100 
years = Rs 68921 - Rs 64000 = Rs 4921.
3 Ex. 3: Find the compound interest at the rate
 10 
= Rs 12000 × 1   of 10% per annum for four years on
 100  the principal which in four years at the
rate of 4% per annum gives Rs 1600
3
 1  as simple interest.
= Rs 12000 × 1  
 10  Soln: Let Rs P be the principal.
This principal gives Rs 1600 as SI in four
3 years at the rate of 4% per annum.
 11 
= Rs 12000 ×  
 10  SI  100
 P =
RT
11 11 11
= Rs 12000 ×  
10 10 10 1600  100
or, P = Rs = Rs 10000
= Rs (12 × 11 × 11 × 11) 4 4
= Rs 15972 Now, we have P = Rs 100000
Now, R = 10% and T = 4.
Compound Interest = A – P
T
 Compound Interest  R 
 Amount after 4 years = P 1  
= Rs 15972 – Rs 12000  100 
= Rs 3972.
Ex. 2: Vij ay obt ai ns a l oan of Rs 64000 4
 10 
against his fixed deposits. If the rate = Rs 10000 × 1  
 100 
of interest be 2.5 paise per rupee per
annum, calculate the compound interest

K KUNDAN
4
payable after 3 years.  1 
= Rs 10000 × 1  
Soln: Here, P = Rs 64000, T = 3 years, and  10 
R = 2.5 paise per rupee per annum
4
= (2.5 × 100) paise per hundred rupees  11 
per annum = Rs 10000 ×  
 10 
 2.5  100 
= Rs   per hundred rupees 11 11 11 11
  
 100  = Rs 10000 ×
10 10 10 10
per annum
= Rs 14641.
= 2.5% per annum.
 Compound interest
 R 
3 = Rs 14641 – Rs 10000
 Amount A after 3 years = P 1   = Rs 4641.
 100  Ex. 4: Compute the compound interest on Rs
3 12000 for 2 years at 20% per annum
 2.5  when compounded half-yearly.
= Rs 64000 × 1  
 100  Soln: Here, Principal P = Rs 12000,
Compound Interest 341

Rate = R = 20% per annum Soln: Here, Principal = P = Rs 320000


Rate = R = 20% per annum
20
= = 10% per half-year 20
2
= = 5% per quarters
T = 2 years = 2 × 2 = 4 half-years 4
T
T = 1 year = 1 × 4 = 4 quarters
 R 
 A = P 1  100   R 
T
  Amount (A) after 1 year = P 1  
 100 
 4
10  

K KUNDAN

= Rs 12000 1  100     5  
4
     320000 1   
= Rs 100  
 
 4
 1  
= Rs 12000 1  10     1  
4
     320000 1   
= Rs 20  
 
 4
 11  
= Rs 12000   10    4
 21  
     320000   
= Rs
  20  
 11 11 11 11 
= Rs 12000    
 10 10 10 10   320000  21  21  21  21 
= Rs  
 20  20  20  20 
14641
= Rs 12000  = Rs 388962
10000  Compound interest
= Rs 17569.20 = Rs 388962 – Rs 320000 = Rs 68962.
 Compound interest Alternative Method:
= Rs 17569.20 – Rs 12000
= Rs 5569.20 12T
 Rx  x
Alternative Method:  
We can solve this question by General Amount (A) for T years = P 1  12 
Formula also.  100 
 
 
12T
 Rx  x Here,
  P = Rs 320000
Amount (A) for T years = P 1  12 
 100  R = 20% per annum
  T = 1 year and
 
x = 3
Here,
P = Rs 12000  121 
R = 20% per annum   20  3  3 

K KUNDAN
   
T = 2 years and = Rs 320000 1  12
  
x = 6   100  
 
   
 122   
  20  6  6 
    4
= Rs 12000 1  12  
  5  
 100  = Rs 320000 1   
     100  
    
  Now, do as the above.
Ex. 6: What sum of m oney at com pound
 4
 1   interest will amount to Rs 2249.52 in 3
= Rs 12000 1    years, if the rate of interest is 3% for
  10  
the first year, 4% for the second year
Now, do as the above. and 5% for the third year?
Ex. 5: Find the com pound i nt er est on Rs Soln: The general formula for such question is:
320000 for one year at the rate of 20%
 R1   R  R   R 
per annum . I f t he i nt er est i s A= P 1   1  2  1  3  ... 1  n 
compounded quarterly.  100   100   100   100 
342 Concept of Arithmetic

Where A = Amount, P = Principal and R1,  Compound interest


R2, R3, ...., Rn are the rates of interest for = Rs (133.10 - 100) = Rs 33.10
different years. Now,
In the above case, If compound interest is Rs 33.10, principal
= Rs 100
 3  4  5 
2249.52 = P 1   1   1   If compound interest is Re 1, principal =
 100   100   100 
100
2249.52 = P (1.03) (1.04) (1.05) Rs
33.10
2249 .52
If compound interest is Rs 331, principal

K KUNDAN
 P = (1.03)  (1.04 )  (1.05 ) = Rs 2000
 100 
Ex. 7: Find the com pound i nt er est on Rs = Rs   331  = Rs 1000
 33.10 
1 Hence, principal = Rs 1000.
24000 at 15% per annum for 2
3 Alternative Method:
years. Rate = 10% per annum
Soln: Here, P = Rs 24000 Time = 3 years
R = 15% per annum and Amount = Rs 331
1 T
 R 
Time = 2 years.  A = P 1  
3  100 
1
 Amount after 2 3 years A 331
 P = T
 3
 R   10 
1   1  
 1   100   100 
R 
R  
2
 3
= P 1    1  
 331  10  10  10 
 100   100  =   = Rs 1000
   11  11  11 
 
Ex. 9: What sum will become Rs 9826 in 18
 1  months if the rate of interest is 5%
 15 
15  
2 per annum and t he i nt er est i s

= Rs 24000 1    1  3  compounded half-yearly?
 100   100 
  Soln: Let the required sum ie the principal, be
 
Rs P.
We have,
 2
 3   1  Principal = Rs P,
= Rs 24000 1    1  
  20   20   Amount = Rs 9826,
R = 5% per annum
 2
 23   21   5
% per half-year and
= Rs 24000      =
  20   20   2

K KUNDAN
= Rs 33327 18 3
T = 18 months = years = years.
 Compound interest = Rs (33327 – 24000) 12 2
= Rs 9327.
Ex. 8: Find the principal, if the compound 3 
=   2   3 half-years
interest compounded annually at the 2 
rate of 10% per annum for three years T
is Rs 331.  R 
 A = P 1  
Soln: Let the principal be Rs 100. Then,  100 
Amount after three years
3
 5 
 3  
 10  
= Rs 100  1  100   or, 9826 = P 1  2 
     200 
 
 
 3
 11  
= Rs 100   100    1 
3
    or, 9826 = P 1  
 80 
= Rs 133.10.
Compound Interest 343

3 T
 81  882  21 
or, 9826 = P   or,  
 80  800  20 

3 2
 80  441  21 
or, P = 9826 ×   = 9466.54 or,  
 81  400  20 
Hence, required sum = Rs 9466.54. T 2
Alternative Method:  21   21 
or,    
Let the required sum be Rs 100. Then,  20   20 

K KUNDAN 3  T = 2.
the amount after 18 months ie years at Hence, required time is 2 years.
2
Ex. 11: In what time will Rs 64000 amount to
the rate of 5% compounded half-yearly, is
Rs 68921 at 5% per annum, interest
given by
being compounded half-yearly?
3 Soln: Here, Principal (P) = Rs 64000
 2 
  5  2 Amount (A) = Rs 68921
    Rate (R) = 5% per annum.
Amount = Rs 100  1  2  
  200   5
  = % per half-year
    2
 
Let the time be T years = 2T half-years
 3 Therefore,
 81  
= Rs 100     T
  80    R 
A = P 1  
 100 
531441
= Rs 2T
5120  5 
Now,  
68921  64000 1  2 
531441  100 
 
If amount is Rs ,  
5120
then principal = Rs 100 2T
If the amount is Re 1, then principal 68921  1 
or,  1  
64000  40 
 100  5120 
= Rs  
 531441   41 
3
 41 
2T

If the amount is Rs 9826, then principal or,    


 40   40 
 100  5120  or, 2T = 3
= Rs   9826  = Rs 9466.54
 531441  3 1
years = 1

K KUNDAN
Hence, required sum = Rs 9466.54.  T = years.
2 2
Ex. 10: In what time Rs 800 amount to Rs 882
Ex. 12: At what r ate per cent per annum
at 5% per annum compounded
compound i nt er est wil l Rs 10000
annually?
amount to Rs 13310 in three years?
Soln: Here, Amount (A) = Rs 882,
Soln: Let the rate be R% per annum.
Principal (P) = Rs 800 and
We have, P = Principal = Rs 10000,
Rate (R) = 5% per annum.
A = Amount = Rs 13310 and
T T = 3 years.
 R 
 A = P 1  
 100   R 
T
 A = P 1  100 
T  
 5 
or, 882 = 800 1  
 100   R 
3
or, 13310  10000 1  
T  100 
882  1 
or,  1  
800  20  13310  R 
3
or,  1  
10000  100 
344 Concept of Arithmetic

1331  R 
3 11 R
 1   or,  1
or, 10 200
1000  100 

3
R 11 1
113  R  or,  1 
or,  1   200 10 10
10 3  100 
200
R 11  R = = 20%
or, 1   10
100 10
Hence, the rate of interest

K KUNDAN
R 11 = 20% per annum.
or,  1 Ex. 14: Determine the rate of interest for a sum
100 10
216
R 1 that becomes times of itself in 3
or,  125
100 10
years, compounded annually.
100 Soln: Let the principal be Rs P and the rate of
or, R = = 10 interest be R% per annum compounded
10
annually.
Hence, rate = 10% per annum.
It is given that the amount at the end of 3
Ex. 13: Nikhil invested Rs 6000 in a company
at com pound i nter est compounded 216
semi-annual ly. He recei ves Rs 7986 years must become P. Therefore,
125
after 18 months from the company.
Find the rate of interest per annum. 3
216  R 
Soln: We have, P = Principal = Rs 6000 P  P 1  
125  100 
A = Amount = Rs 7986 and
 T
3  R  
T = 18 months = years. Using A  P 1   
2   100  

3 
=   2   3 half-years 216  R 
3
2  or,  1  
Let the rate of interest be R% per annum 125  100 

R 3 3
= % per half-year. 6  R 
2 or,    1  
5  100 
(Since the interest is compounded semi-
annually) 6 R
or,  1
T 5 100
  R
 A = P 1  100  R 6
  or,  1
100 5

K KUNDAN
3
 R  R 1
  
or, 7986  6000 1  2  or,
100 5
 100 
  or, R = 20
 
Hence, the rate of interest is 20% per
3 annum.
 R 
or, 7986  6000 1   Ex. 15: The difference between the compound
 200  interest and simple interest on a certain
3 sum of money at 10% per annum for 2
7986  R 
or,  1   years is Rs 500. Find the sum when
6000  200  the interest is compounded annually.
3 Soln: Let the sum be Rs 100.
1331  R 
or,  1   Computation of compound interest: We
1000  200  have, Principal = Rs 100,
3 3 R = 10% per annum, and
 11   R  T = 2.
or,    1  
 10   200 
Compound Interest 345

 2 15
 10    R 
Amount = Rs 100  1  100   or, 2  1   ....(i)
     100 

 2 Putting A  2P and T  15 in 
 11   
= Rs 100   10   = Rs 121. T
     R  
  A  P 1  
  100  
 CI = Rs 121 – Rs 100 = Rs 21.
Computation of simple interest : We have, Suppose the money becomes 8 times ie
Principal = Rs 100, R = 10% and 8P in T years. Then,

K KUNDAN
Time = 2 years. T
 R 
 100  10  2  8P  P 1  
 SI = Rs   = Rs 20.  100 
 100 
T
Thus, Difference in CI and SI  R 
or, 8  1  
= Rs (21 – 20) = Re 1  100 
Now, T
 If difference between CI and SI is Re 1, 3  R 
or, 2  1  
Sum = Rs 100  100 
 If difference between CI and SI is Rs 3
 15 T
500, Sum = Rs (100 × 500) = Rs 50000. R    R 
Alternative Method: or, 1     1  
 100    100 
Let the sum be Rs P.
[Using (i)]
PRT 45 T
Simple Interest =  R   R 
100 or, 1    1  
 100   100 
 P  10  2  P  or, T = 45
=  = Rs   Hence, the money will become 8 times in
 100  5
45 years.
Compound Interest = Amount – Principal Ex. 17: A farmer wants to divide Rs 390300
 2  between his two daughters who are 16
  10   years and 18 years old respectively in
= Rs  P  1  100   P 
    such a way that the sum invested at
the rate of 4% per annum, compounded
 2 
  11   annually will give the same amount to
= Rs  P   10   P  each, when they attain the age of 21
   
years. How should he divide the sum?
 P  11  11  Soln: Suppose the farmer gives Rs P to 16 years
= Rs   P
 10  10  old daughter and the remaining Rs
(390300 – P) to 18 years old daughter.
 21P  At the age of 21 years, each daughter gets
= Rs  

K KUNDAN
 100  the same amount. This means that the
amount of Rs P invested for 5 years is
Now, according to the question,
same as the amount of Rs (390300 – P)
21P P invested for 3 years ie
 = Rs 500
100 5 5 3
 4   4 
P 1    390300  P  1  
21P - 20P  100   100 
or, = Rs 500 2
100  4 
or, P 1    390300  P 
 P = Rs (500 × 100) = Rs 50000  100 
Ex. 16: A sum of money doubl es i t sel f at
2
compound interest in 15 years. In how  1 
or, P 1    390300  P 
many years will it become eight times?  25 
Soln: Let the sum of money be Rs P invested at
2
the rate of R% per annum. It is given that  26 
or, P    390300  P 
the money doubles itself in 15 year s.  25 
Therefore,
 26  2 
15
 R  or, P  25   1  390300
2P  P 1     
 100 
346 Concept of Arithmetic

 676  625  2
 R 
or, P    390300 9680  P 1   ...(i) and
 625   100 
1301  3
or, P    390300  R 
 625  10648  P 1   ...(ii)
 100 
390300  625
or, P = = 187500 Now, dividing equation (ii) by equation (i),
1301 we have
Therefore, the daughter aged 16 years gets 3
Rs 187500 and the daughter aged 18 years  R 
P 1  

K KUNDAN
gets Rs (390300 – 187500) = Rs 202800. 10648  100 
 2
Alternative Method: 9680  R 
Let the equal amount in each case be Rs P 1  
 100 
100 and P1, P2 be the principals for the
two daughters. 10648 R
or,  1
In case of the first daughter, 9680 100
A = Rs 100, T = 5 years, R = 4%
5 5 R 10648
 4   26  or,  1
 100  P1 1    P1   100 9680
 100   25 
R 968
100 100  (25 )5 or, 
or, P1   100 9680
5
 26  (26)5
  968
 25  or, R =  100 = 10
9680
In case of the second daughter,
A = Rs 100, T = 3 years, R = 5% R 
2

3
Putting R = 10 in 9680  P 1   , we
 5   100 
 100  P2 1  
 100  get
2
 10 
100
3 9680  P 1  
 25   100 
or, P2  3
 100  
 26   26 
  2
 1 
 25  or, 9680  P 1  
 10 
 Ratio between their parts is
2
3  11 
100  (25)5  25  or, 9680  P  
P1 : P2 = : 100 
(26)5  26   10 
2
 10 
 25 
2 or, P = 9680   
=   :1  11 
 26 
100
or, P = 9680 

K KUNDAN
= 252 : 262 = 625 : 676 = 8000
121
 We shall divide Rs 390300 in the ratio
of 625 : 676 Hence, principal = Rs 8000 and rate of
 Daughter aged 16 years old gets interest = 10% per annum.
Ex. 19: A sum of money is put at compound
 390300 
Rs   625  interest for 2 years at 20% per annum.
 625  676  I t would fet ch Rs 482 m or e, if the
390300  625 interest were payable half-yearly than
= Rs = Rs 187500
1301 if it were payable yearly. Find the sum.
and the daughter aged 18 years old gets Soln: Let t he r equired sum of money be
= Rs (390300 – 187500) = Rs 202800. Rs P.
Ex. 18: A sum amounts to Rs 9680 in 2 years Case I: When interest is payable yearly.
and to Rs 10648 in 3 years compounded In this case, let the amount be A1. Then,
annually. Find the sum (principal) and
2 2
the rate of interest per annum.  20  6 36P
A1  P 1    P  
Soln: Let the sum (principal) be Rs P and the  100  5 25P
rate of interest be R% per annum. Then,
Case II: When interest is payable half-
yearly.
Compound Interest 347

In this case, Principal = P,


 2 
R = 20% per annum R 
or, 110  P 1  100   1 and PR = 5000
20   
= = 10% per half-year
2
 2R R2 
T = 2 years = 2 × 2 = 4 half-years or, 110  P 1  100  10000  1 and
Let A2 be the amount at the end of 2 years.  
Th en , PR = 5000
T
 R  2PR PR 2

K KUNDAN
A 2  P 1   or, 110   and PR = 5000
 100  100 10000

 10 
4 PR PR
or, A 2  P 1   or, 110    R and PR = 5000
 100  50 10000

4 5000 5000
 11  or, 110   R
or, A 2  P   50 10000
 10  [Putting PR = 5000]
It is given that A2 – A1= 482 R
or, 110 = 100 +
 11 
4
6
2 2
or, P    P    482
 10  5 R
or, 10 =
2
 11  4 2
6 or, R = 20.
or, P  10    5    482
Putting R = 20 in PR = 5000, we get 20P
    
= 5000
4 2  P = 250.
 11   12  Hence, principal = Rs 250 and rate
or, P  10    10    482
     = 20% per annum.
Alternative Method:
  11  2 12    11 2 12   Difference between Compound Interest
      482 and Simple Interest
or, P   10   10   
  10  10  
    = Rs 110 – Rs 100 = Rs 10
 Simple Interest for 2 years = Rs 100
 241   1 
or, P     482 100
 100   100   Simple Interest for 1 year = Rs
2
482  100  100 = Rs 50
 P = = 20000 Because inter est is r eckoned year ly
241
Compound Int erest and t he Simple
Hence, the sum of money was Rs 20000.
Interest for the first year will be the same.
Ex. 20: Reena borrowed from Kamal certain
 Rs 10 is the interest on Rs 50 for 1

K KUNDAN
sum for two years at simple interest.
year.
Reena lent this sum to Hamid at the
 Principal = Rs 50, T = 1 year and
same rate for two years at compound
SI = Rs 10
interest. At the end of two years she
received Rs 110 as compound interest SI  100 10  100
but paid Rs 100 as simple interest. Find  Rate = PT =
50  1
the sum and rate of interest.
Soln: Let the principal be Rs P and the rate of = 20% per annum
interest be R% per annum. Now, SI = Rs 100, R = 20% and
We have, CI = Rs 110, SI = Rs 100 and T = 2 years
Time = 2 years. SI  100  100  100 
2  P = RT =   = Rs 250
 R   20  2 
 110  P 1    P and
 100  Hence, principal = Rs 250 and
PR2 rate = 20% per annum.
100  Ex. 21: The simple interest on a sum at 4% per
100
annum for 2 years is Rs 80. Find the
 2  compound interest on the same sum for
R  PR
or, 110  P 1    1 and 100  the same period.
 100   50
348 Concept of Arithmetic

Soln: Let the sum be Rs P. 43 3


 R  5 125
PRT Now, 1     
SI =  100  4 64
100
12
SI  100 80  100  R  125  75 9375
or, 1    
 P = RT = = Rs 1000  100  64  75 4800
2 4
T 12
 R   R 
CI = P 1   P or, 4800 1    9375
 100   100 

K KUNDAN
 2  The above equation shows that Rs 4800
 4 
= Rs 1000 1  100   1000  becomes Rs 9375 after 12 years.
    Ex. 24: Find the present value of Rs 40960 due
2
 1000  26  26  3 year s hence at 6 % per annum
= Rs   1000  3
 25  25 
compound interest.
  676  3
= Rs 1000  1  20 
  625  Soln: Present value 1   = Rs 40960
 3  100 
1000  51
= Rs = Rs 81.6 40960
625  Present value = Rs 3
Ex. 22: The compound interest on a certain sum  20 
1  
of money for 2 years at 10% per annum  3  100 
is Rs 420. Find the simple interest at
the same rate and for the same time.  15 15 15 
= Rs  40960    
Soln: Let the sum be Rs P.  16 16 16 
T = Rs 33750
  R
 CI = P 1  100   P Ex. 25: What annual payment will discharge a
  debt of Rs 8116 due in 3 years at 8%
per annum compound interest?
2
 10  Soln: Let Rs x be the amount of each instalment.
or, 420  P 1   P
 100  Then the instalments of Rs x are paid at
the end of 1 year, 2 years and 3 years
 11  11  respectively.
or, 420  P  P
Present values of these instalments are
 10  10 

21    





 
or, 420  P
 x   x   x 
100  
2 and  3 
 8 , 8   8 
 1   1   
420  100  100     
100    1  100  
 P = = Rs 2000   
21  Total present value of these instalments

K KUNDAN
PRT  25 625 15625   50725x 
 Simple Interest = = Rs x     = Rs  19683 
100  27 729 19683   
Also, present value of Rs 8116 due 3 years
 2000  10  2 
= Rs   hence
 100 
= Rs 400  
 
Ex. 23: Rs 4800 becomes Rs 6000 in 4 years  8116 
at a certain rate of compound interest.  3 
= Rs   8  
What will be the sum after 12 years?
 1  100  
Soln: Let the rate of interest be R%.   
Now, we have
 8116  15625 
R 
4 = Rs  
  19683 
4800 1    6000
 100  50725x 8116  15625
 =
4 19683 19683
 R  6000 5
or, 1      x = Rs 2500
 100  4800 4
Compound Interest 349

Practice Exercise
1. Abhay lent Rs 8000 to his friend for 3 years at 15. Two partners A and B together lend Rs 2523
the rate of 5% per annum compound interest. at 5% compound int er est compounded
What amount does Abhay get after 3 years? annually. The amount A gets at the end of 3
2. Find the compound interest on Rs 1000 at years is the same as B gets at the end of 5
the rate of 10% per annum for 18 months years. Determine the share of each.
when interest is compounded half-yearly. 16. Two partners A and B together lends Rs 84100

K KUNDAN
3. What will be compound interest of Rs 24000 at 5% compound int er est compounded
1 annually. The amount which A gets at the
for 2 years at the rate of 15% per annum? end of 3 years is the same as what B gets at
3
4. Ramesh deposited Rs 7500 in a bank which the end of 5 years. Determine the ratio of the
pays him 12% int er est per annum shares of A and B.
compounded quarterly. What is the amount 17. A sum of money put at compound interest
which he receives after 9 months? amounts to Rs 8820 in two years and to Rs
5. At what rate per cent per annum of compound 9261 in three years. Find the sum and the
interest will Rs 1600 amount to Rs 1852.20 annual rate of interest.
in 3 years? 18. Find the present value of Rs 4913 due 3 years
6. Find the sum of money which will amount to 1
hence at 6 % per annum compound interest.
Rs 26010 in 6 months at the rate of 8% per 4
annum when the interest is compounded 19. A person borrowed Rs 4000 at 5% per annum
quarterly. compound interest compounded annually.
7. Govardhan deposited Rs 7500 in a bank for 6 After 2 years, he repaid Rs 2210 and then
months at the rate of 8% per annum, interest after 2 more years, he repaid the balance with
compounded quarterly. Find the amount he interest. Find the total interest paid by him.
received after 6 months. 2 0 . A sum of money lent out at compound interest
8. A certain sum invested at 4% per annum increases in value by 50% in 5 years. A person
compounded semi-annually amounts to Rs wants to lend three different sums of money
7803 at the end of one-year. Find the sum. X, Y and Z for 10, 15 and 20 years respectively
9. Rs 16000 inv ested at 10% per annum at the above rate in such a way that he gets
compounded semi-annually amounts to Rs back equal sums of money at the end of their
18522. Find the time period of investment. respective periods. Find the value of X : Y : Z.
10. The dif f erence in simple and compound 21. A person closes his account in a bank by
interest on a certain sum for 2 years at 5% withdrawing Rs 110000. One year earlier,
per annum compounded annually is Rs 75. he had withdr awn Rs 65000. Two year s
Find the sum. earlier, he had withdrawn Rs 125000. How
11. The simple interest on a sum of money at much money had he deposited in the bank at
some rat e for 3 years is Rs 225 and t he the time of opening the account three years
compound interest on the same sum of money ago if t he annual interest r at e was 10%
and at the same rate for 2 years is Rs 153. compounded annually?

K KUNDAN
Find the sum and the rate per cent per annum. 2 2 . Compound interest and simple interest on a
12. A principal sum of money is lent out at certain sum for 2 years are Rs 104 and Rs
compound interest compounded annually at 100 respectively. Find the rate per cent and
the rate of 20% per annum for 2 years. It the principal.
would give Rs 2410 more if the interest is 23. The difference between compound and simple
compounded half-yearly. Find the principal interests on a certain sum of money at the
sum. 1
13. A money-lender borrows a certain sum of interest rate of 10% per annum for 1 years
2
money at 3% per annum simple interest and is Rs 183, when the interest is compounded
lends it at 6% per annum compound interest semi-annually. Find the sum of money.
compounded half-yearly. If he gains Rs 618 24. In how many years will a sum of Rs 800 at
in a year, find the sum of money borrowed by 10% per annum compound inter est ,
him. compounded semi-annually, becomes Rs
14. The compound interest on a certain sum of 926.10.
money for 2 years at 5% per annum is Rs 25. If the difference between compound interest,
102.50. What will be the compound interest compounded half-yearly and simple interest
on the same sum of money for the same period on a sum of money at 8% per annum for 1
at 4% per annum. year is Rs 30. Find the sum.
350 Concept of Arithmetic

Answers and explanations


1. Here, P = Rs 8000, R = 5% per annum and R 3. Here, Principal (P) = Rs 24000
= 3 years. Rate (R) = 15%

 R 
3 1
 Amount after 3 years = P 1   Time (T) = 2 years
 100  3

3  R 

K KUNDAN
 5 
R   
2
= Rs 8000  1    3 
 100  A = P 1   1 
 100   100 
 
3  
1 
= Rs 8000  1  
 20 
 15 
15   
2

 21 
3
= 24000 1   1 3 
= Rs 8000     100   100 
 20   
 
21 21 21
= Rs 8000    = Rs 9261. 2
 23   21 
20 20 20 = 24000     
Thus, Abhay gets Rs 9261 at the end of 3  20   20 
years.
23 23 21
2. Here, P = Rs 1000, = 24000    = Rs 33327
R = 10% per annum 20 20 20
 Compound Interest = Rs 33327 – Rs 24000
10
= % = 5% per half-year = Rs 9327
2 Alternative Method:
Principal (P) = Rs 24000
 18  3
T = 18 months =   years Rate (R) = 15%
 12  2
1
Time (T) = 2 years
3  3
=   2   3 half-years Compound Interest for the first 2 years
 2 
= Amount – Principal
Now, from the formula
T
T  R 
 R   A = P 1  
A = P 1    100 
 100 
 2
 3
5    15  
 = Rs 24000 1   
= Rs 1000 1      100  

K KUNDAN
  100   

 3  24000  23  23 
 1   = Rs   = Rs 31740
= Rs 1000 1     20  20 
  20  
 Compound Interest for the first 2 years
3 = Rs 31740 – Rs 24000 = Rs 7740
  21  
= Rs 1000    For the next year Rs 31740 will be principal

  20   1
 Interest for next years
3
 1000  21  21  21 
= Rs  
 20  20  20  PRT  31740  15  1 
= = Rs   = Rs 1587
= Rs 1157.625 100  3  100 
Hence,  Total Compound Interest
Compound Interest = Amount – Principal = Rs 7740 + Rs 1587
= Rs 1157.625 – Rs 1000 = Rs 9327
= Rs 157.625
Compound Interest 351

4. Here, Principal = Rs 7500


Rate = 12% per annum  
 
 12  A  26010 
=    3% per quarter and  P = T
= Rs  2
 4   R   1  2  
1  
 9  3  100    100  
Time = 9 months =   year
 12  4
3   26010  50  50 
=   4   3 quarters = Rs   = Rs 25000
 51  51 
4 

K KUNDAN T 7. Principal (P) = Rs 7500


 R 
A = P 1  
 100   6 1
Time (T) = 6 months =   year
Amount after 9 months  12  2

3
  3   =  1  4   2 quarters
 
= Rs 7500 1  100   2 
   
Rate (R) = 8% per annum
7500  103  103  103
= Rs 8 
100  100  100 =    2% per quarter
4 
= Rs 8195.45
5. Let the rate be R% per annum. T
We have  R 
 Amount = P 1  
Principal (P) = Rs 1600  100 
Amount (A) = Rs 1852.20 and
Time (T) = 3 years  2
 2  
 
T = Rs 7500 1  100  
 R     
 A = P 1  
 100 
 2
3  51  
R   

or, 1852.20 = 1600 1   = Rs 7500   50  
100     

3
 R  1852.20  7500  51  51 
or, 1   = = 1.157625 = (1.05)3 = Rs   = Rs 7803
 100  1600  50  50 
R 8. A = Rs 7803
or, 1  = 1.05 R = 4% per annum
100
R 4 
or, = (1.05 – 1) = 0.05 =    2% per half-year
2 

K KUNDAN
100
T = 1 year = (1 × 2 =) 2 half-years
 R = 0.05 × 100 = 5%
(Since t he interest is compounded semi-
6. Here, Amount (A) = Rs 26010
annually)
Rate (R) = 8% per annum
T
8   R 
=    2% per quarter  A = P 1  
4   100 
 6 1  
Time (T) = 6 months =   year  
 12  2 A  7803 
 P = T
= Rs  2
 R   1  2  
=  1  4   2 quarters 1  
 
 100  100  
2 
Now,
 7803  50  50 
T = Rs   = Rs 7500
 R   51  51 
 A = P 1   ;
 100  Thus, the sum invested is Rs 7500.
where P = Principal or required sum
352 Concept of Arithmetic

9. We have, Principal (P) = Rs 16000


41x x
Amount (A) = Rs 18522 Difference in CI and SI = 
Rate (R) = 10% per annum 400 10

41x  40 x x
=  10   5% per half-year = 
 2  400 400
But it is given that the difference of CI and SI
Let the time be T years = (2 × T) half-years
is Rs 75.
Now,
T
x
 R    75

K KUNDAN
 A = P 1   400
 100  or, x = 75 × 400 = Rs 30000.
 The sum is Rs 30000.
2T
 5  11.  Simple interest for 3 years is Rs 225
or, 18522  16000 1  
 100  225
 Simple interest for 1 year is = Rs 75
2T
3
18522  21 
or,   225
16000  20   Simple interest for 2 years is 2
3
2T = Rs 150
9261  21 
or,   and, compound interest for 2 years is Rs 153.
8000  20 
Difference between compound interest and
3 2T simple interest for 2 years = 153 – 150 = Rs 3.
 21   21  As we know t hat, diff er ence bet ween
or,    
 20   20  compound interest and simple interest for 2
or, 2T = 3 years = Interest on simple interest for 1 year.
Let the rate of interest be R%.
3 1
 T = = 1 years 75  R
2 2 Then,  3 or R = 4%
100
Therefore, time period of investment is one-
and-a-half year. PR P4
10. Let the sum be Rs x and  75 or  75
100 100
Simple interest on Rs x for 2 years at 5%
75  100
x  2 5 x or, P = = Rs 1875
= = Rs 4
100 10
Sum = Rs 1875 and rate per cent = 4% per
 Time  annum.
Rate 
 Compound Interest = P 1    1 12. Let the sum be Rs P.
 100    Compound interest when it is compounded
annually,
 Compound interest on Rs x for 2 years at

K KUNDAN
5% 20 
2

= P 1   P
 5 
2   100 
 
= x 1  100   1 Compound interest when it is compounded
   half-yearly,
22
 21 2   20 
   
= x  20   1 2 
   = P 1  P
 100 
 
 
 441  4
= x 1  10 
 400  = P 1   P
 100 
x  441  400  Now, according to the question, we have
=
400
  4    2 
10  20 
41x P 1    P   P 1    P  = 2410
=   100     100  
400    
Compound Interest 353

4 2
  10609  
or, P 1  10   P 1  20  = 2410 = Rs P    1
 100   100    10000  

 4 2 609P
10   20   = Rs
or, P 1    1    = 2410 10000
 100   100  
 Now, according to the question,

 11  4  12 2 
or, P      = 2410  609P 3P 

K KUNDAN
 10   10      = Rs 618
  10000 100 

 2
2 2  609P - 300P 
  11   12   or,   = Rs 618
or, P         = 2410  10000 
10   10  
    or, 309P = Rs (618 × 10000)
 P = Rs 20000
  11 2 12   11 2 12  14. Let the principal be Rs P. Then,
     
or, P          = 2410 Compound Interest
  10  10   10  10 
   2
 5 
= Rs P 1   P
   100 
or, P  121  12   121  12   = 2410
  100 10   100 10   2
 21 
= Rs P   P
 241 1   20 
or, P    = 2410
 100 100  2
 441 
= Rs P   P
2410  100  100  400 
or, P = = 100000
241
Hence, the required sum = Rs 100000 41P
= Rs
13. Let the required sum of money be Rs P. 400
Now, according to the question,
PRT
Case I: Simple Interest = 41P
100
= Rs 102.50
400
 P  3  1  3P 
= Rs   = Rs  
 102.50  400 
 100   100  or, P = Rs   = Rs 1000
Case II: Compound Interest  41 
= Amount – Principal Now,

K KUNDAN
T 2
 R   4  
= P 1   P Compound Interest = Rs 1000 1    1
 100   100  
 
Here, R = 6% per annum

6   26  2 
=    3% per half-year = Rs 1000    1
2   25 



T = 1 year = 2 half-years
(Since the interest is compounded half-yearly)  676 
= Rs 1000   1
 Compound Interest  625 
  2 
3   676 - 625 
= Rs P 1    P = Rs 1000  
  100    625 
 

  103 2  51
= Rs 1000 
= Rs P    P 625
  100  
  = Rs 81.60
354 Concept of Arithmetic

15. Let the share of A be Rs x and share of B be


5
Rs (2523 – x).  21 
According to the question,  
x 20 
Amount received by A after 3 years at 5% or,  
y  21 3
interest  
3 3
 20 
 5   21 
 x 1    x 
100  2
  20  x  21  441
or,   
Similarly, amount received by B after 5 years y  20  400

K KUNDAN  5 
5  The required ratio = 441 : 400.
= 2523  x   1   17. Let the Principal be Rs P and the rate be R%
 100 
per annum.
5 2
 21   R 
= 2523  x     8820  P1   ......... (i)
 20   100 
Again, according to the question, 3
 R 
 21 
3
 21 
5 and 9261  P 1   ......... (ii)
x   2523  x     100 
 20   20  Now, equation (ii)  equation (i) gives
5
9261 R
 21   1
  8820 100
x 20 
or,   R 441
2523  x   21 3 or, 
  100 8820
 20 
441 100
2 or, R = = 5
x  21  441 8820
or,     R = 5%
2523  x   20  400
Using equation (i)
or, 400x  2523  441  441x
2 2
 5   21 
or, 400x  441x  2523  441 8820  P 1     P
 100   20 
or, 841x  2523  441
441
2523  441 or, 8820  P
or, x =  1323 400
841
or, P = Rs 8000
 The share of A = Rs 1323
 Principal = Rs 8000 and Rate = 5%
The share of B = Rs 2523 – 1323 = Rs 1200
18. We know that
16. Solve as the Q.No. 15. Try yourself.
T
Alternative Method:  R 
A  P 1  

K KUNDAN
Let the amount lent by A be Rs x and amount  100 
lent by B be Rs y.
Now, according to the question, 3
 25 
Amount received by A after 3 years at 5%  
interest or, 4913  P 1  4  ;
 100 
3 3
 
 5   21   
= x 1   = x  
 100   20  [where P is the present worth]
Similarly, amount received by B after 5 years 3
 25 
at 5% interest or, 4913  P 1  
 400 
5 5
 5   21 
= y  1   = y  
   
 100   20 
   
Again, according to the question,  4913   4913 
or, P =  3
 3
 21 
3
 21 
5  1  1     17  
x   y    16     16  
 20   20 
Compound Interest 355

4913  16  16  16 3Y 9Z
or, P = = 4096 Suppose X   k
17  17  17 2 4
Hence the present worth is Rs 4096.
2 4
19. After 2 years amount at CI  X = k, Y = k and Z = k
3 9
2 2
 5   1 
= 4000 1    4000 1   2 4
 100   20   X : Y : Z = k: k: k
3 9
21 21

K KUNDAN
= 4000   = Rs 4410 2 4
20 20 or, X : Y : Z = 1 : :
3 9
After 2 years the person repaid Rs 2210,
 X : Y : Z = 9 : 6 : 4
hence the amount borrowed
(multiplying each term by 9)
= Rs (4410 – 2210) = Rs 2200
21. The person withdraws Rs 110000.
Now, compound interest on Rs 2200 for 2
One year earlier the person had
years at 5% per annum is
 10 
 5 
2
Rs 110000    100000
Rs 2200 1    2200  11 
 100 
Two years earlier the person had
 21 21  10
= Rs  2200    2200  (100000  65000)  = Rs 150000
 20 20  11
= Rs (2425.50 – 2200) At the time of opening the account, the person
= Rs 225.50 had
Total interest paid by him
10
= Rs (225.50 + 410) = Rs 635.50 (150000  125000)  = Rs 250000
2 0 . Rate of interest = 50% in 5 years 11
Now, according to the question, 2 2 . Difference between CI and SI
X is lent for 10 years, Y for 15 years and Z for = Rs 104 – Rs 100 = Rs 4
20 years.  SI for 2 years = Rs 100
X 3X 100
After 5 years X becomes = X  SI for 1 year = Rs = Rs 50
2 2 2
This becomes the principal for next years and Because interest is reckoned yearly, CI and
after 10 years the X will be SI for the first year will be the same.
2
 Rs 4 is the interest on Rs 50 for 1 year.
3  3X   3   Principal = Rs 50, T = 1 year and
    X
2  2  2 SI = Rs 4
Similarly, after 15 years Y will be 4  100
3  Rate = % = 8%
3  3  3   3  50  1

K KUNDAN
       Y Now, SI = Rs 100
2  2  2   2 
 R = 8% and
and after 20 years Z will be T = 2 years
4
3  3  3  3  3  100  100 
    Z    Z  P = Rs   = Rs 625
2  2  2  2  2  82 
Now, according to the question, 23. Let the principal be Rs P.
2 3 4 Rate = R = 10% per annum
3 3 3
  X  Y  Z
2  2  2 10
= = 5% per half-year
9X 27Y 81Z 2
or,  
4 8 16 1 3
Time = T = 1 years = years
9 2 2
(dividing each term by )
4 3
=  2 = 3 half-years
3Y 9Z 2
or, X  
2 4
356 Concept of Arithmetic

T 2T
 R   5 
Compound Interest (CI) = A = P 1   P or, 926.10 = 800 1  
 100   100 
  T 
R  2T
or, CI = P 1    1 926.10  1 
  100   or, = 1  
  800  20 

  3  2T
5  9261  21 
= P 1    1 or, =  
  100   8000  10 

K KUNDAN
 
3 2T
  9261    21   21 
= P  - 1  or,   =  
 10   10 
  8000  
or, 2T = 3
 1261 
= Rs P   and 3 1
 8000   T = 1
2 2
3 1
P  10  Hence the required time = 1 years.
PRT 2 3P 2
SI = = = Rs
100 100 20 25. Let the sum of money be Rs P and rate of
Now, according to the question, interest = 8% per annum.
1261P 3P Rate of interest compounded half-yearly = 4%
 = Rs 183  Difference between CI and SI
8000 20
  2  PR2
1261P  1200P R 
 
or, = Rs 183 = P 1  100   1  100
8000    
 183  8000   2 
or, P = Rs   = Rs 24000
1 
2R  R  2R
 61  = P      1
 100  100  100 
 required sum = Rs 24000  
24. Here, principal (P) = Rs 800
2
Rate (R) = 10% per annum  R 
= P 
10  100 
= = 5% per half-year
2 2
 4 
Amount (A) = Rs 926.10 or, Rs 30 = P   
Let the time be T years.  100 
 Time = 2T half-years
P 1
Time or, Rs 30 =
 Rate  625

K KUNDAN
 Amount = Principal 1  
 100   P = Rs (30 × 625) = Rs 18750
Quadratic Equations

1. Quadratic Equations
Let p(x) be a quadratic polynomial. Then, the equation p(x) = 0 is
called quadratic equation. The values of x satisfying p(x) = 0 are
called its roots or zeros. For example, 25x2 – 30x + 9 = 0 is a
3
quadratic equation. And the value of x  is the solution of
5
3
the given equation. Since, if we put x  in 25x2 – 30x + 9 = 0,
5
2
 3 3
we have LHS = 25     30   9 = 9 – 18 + 9 = 0 = RHS.
 5 5
The general form of a quardratic equation is ax2 + bx + c = 0;
where a, b and c are real numbers and a  0 .

2. Roots of a Quadratic Equation


(i) If  and  are the two roots of ax2 + bx + c = 0, then
b  b 2  4ac b  b 2  4ac
 and  
2a 2a
b
(ii) Sum of the roots (  )  
a
c
(iii) Product of the roots () 
a
(iv) A quardratic equation whose roots are  and  is given by
x 2  (  )x    0 , ie x 2 –(sum of roots)x + pr oduct
of roots = 0
(v) In ax2 + bx + c = 0 the expression D = b2 – 4ac is called its
discriminant.
3. Nature of Roots of ax2 + bx + c = 0
Let D = b2 – 4ac be the discriminant of the given equation. Then
roots of the equation ax2 + bx + c = 0 are
(i) real and equal, if D = 0.
(ii) real, unequal and rational, when D > 0 and D is a perfect
square.
(iii) real, unequal and irrational, when D > 0 and D is not a
perfect square.
(iv) imaginary, if D < 0.
70 Magical Book on Arithmetical Formulae
(v) integers, when a = 1, b and c are integers and the roots are
rational.
4. Methods of Solving Quadratic Equations
(i) By Factorization
This can be understood by the examples given below:
Ex. 1: Fi nd t he s olut ions of the quad rati c eq uati on
x 2 + 6x + 5 = 0 and check the solutions.
Soln: The quadratic polynomial x 2  6x  5 can be factorized as
follows:
= x2 + 6x + 5 = x2 + 5x + x + 5
= x (x  5)  1(x  5)
= (x  5)(x  1)
Therefore the given quadratic equation becomes
(x  5)(x  1)  0
This gives x = – 5 or, x = – 1
Therefore, x = – 1, – 5 are the required solutions of the
given equation.
Check: We substitute x = – 1 and x = – 5 in the given
equation and get

K KUNDAN
(a) (–1)2 + 6(–1) + 5
=1–6+5=0
(b) (–5)2 + 6(–5) + 5
= 25 – 30 + 5 = 0
Therefore, the solutions are correct.
(ii) By Using Method of Completing Square
It is not always easy to factorise polynomials and solve qua-
dratic equations as discussed above. For example, consider
the quadratic equation x2 + 5x + 5 = 0. If we want to factorise
the left-hand side of the equation using the method of split-
ting the middle term, we must determine two integral factors
of 5 whose sum is 5. But, the only factors of 5 are 1 and 5 or
–1 and –5. In both the cases, the sum is not 5. Therefore,
using factorisation, we are unable to solve the quadratic equa-
tion x2 + 5x + 5 = 0. Here, we shall discuss a method to solve
such quadratic equations. Let us consider the following ex-
ample:
Ex. 2: Solve: x2 + 3x + 1 = 0.
Soln: We have
x2 + 3x + 1 = 0
1
We add and subtract ( coefficient of x)2 in LHS and get
2
Elementary Algebra 71
2 2
 3  3
x 2  3x  1        0
 2  2
2 2
2  3  3  3
 x  2  x       1  0
 2  2  2
2
 3 5
 x     0
 2 4
2 2
 3  5 3 5
  x      or x  
2  2  2 2

This gives x 

 3 5
or x 

3  5
2 2
3  5 3  5
Therefore, x   , are the solutions of the
2 2
given equation.
(iii) By Using Quadratic Formula
If the equation is ax2 + bx + c = 0, then

b  b 2  4ac
x 
2a

K KUNDAN
or, x 
b  b 2  4ac
2a
and
to as quadratic formula.

2
b

2
 b
2a
2
 4ac
is often referred

(a) When b – 4ac = 0, ie b = 4ac, then   


b
and
2a
b
 where  and  are the two roots of the above
2a
equation ie both the roots are equal.
(b) When b2 – 4ac > 0 ie b2 > 4ac, then the equation has
two distinct real roots ,  given by
b  b 2  4ac b  b 2  4ac
 and  
2a 2a
Ex. 3: Solve the following equation:
2x2 + 5x – 6 = 0.
Soln: Here, the given equation :
2x2 + 5x – 6 = 0
ie a = 2, b = 5, c = –6
b  b 2  4ac
x =
2a
72 Magical Book on Arithmetical Formulae

5  (5)2  4  2  (6) 5  25  48
= =
22 4
5  73 5  73 5  73
= = ,
4 4 4

5. Finding Roots of a Quadratic Equation Having Rational


Roots
Suppose we have to find the roots of 10 x 2  x  21  0 .
Note the following steps:
10 x 2  x  21

Step I: –21 × 10 = –210 (Multiply the coefficient of x2


and the constant term)
Step II: 14 –15 (Find the factors of –210 which give
coefficient of x ie (–1) in any
possible way: 14 × (–15) = –210 and
14 + (–15) = –1))
 
14 7 15 3
Step III:   (Divide the factors obtained in
10 5 10 2

K KUNDAN
Step IV:

7
5

3
2
step II by coefficient of x2)

(Change the sign of values


obtained in step III)
7 3
Hence the roots are and
5 5
Now see the examples given below:
Ex. 4: Find he roots of 12x2 + 25 x – 117 = 0
Soln:

52 13 27 9
 
12 3 12 4
 
13 9
3 4
Elementary Algebra 73
13 9
Therefore, the roots are and .
3 4
Ex. 5: Find the roots of 35x2 + x – 12 = 0
Soln: 35x2 + x – 12 = 0

35   12

21 –20
 
21 3 20 4
 
35 5 35 7
 
3 4
5 7
3 4
Therefore, the roots are and
5 7
Ex. 6: Find the roots of 91x2 + 20x + 1 = 0.
Soln: 91x2 + 20x + 1 = 0

K KUNDAN
91
7 13
 
7 1 13 1
 
91 13 91 7
 
1 1
13 7
1 1
Therefore, the roots are and .
13 7
Ex. 7: I. 6x2 – x – 35 = 0
II. 6y2 + 41y + 63 = 0
Compare the roots of the quadratic equations in I and
II. Which of the following is true?
1) x > y
2) x < y
3) x > y
4) x < y
5) x = y
Soln: 3;
I. 6x 2  x  35  0 II. 6y 2  41y  63  0
74 Magical Book on Arithmetical Formulae
–35 × 6 6 × 63

–15 14 27 14
   
15 14 27 9 14 7
 
6 6 6 2 6 3
   
5 7 9 7
2 3 2 3
Comparing the pair of values obtained from quadratic
equations (I) and (II), we get x > y.
6. Condition for Common Roots
Let a1x 2  b1x  c1  0 and a 2x 2  b2x  c 2  0 be two quadratic equa-
tions such that a1, a1  0 and a1b2  a2b1.
Let  be the common root of these two equations.
Then, a1 2  b1  c1  0

and a 2 2  b2  c 2  0
Solving these two equations by cross-multiplication, we get

K KUNDAN
2



1
b1c 2  b2c1 c1a 2  c 2a1 a1b2  a 2b1

 2 
b1c 2  b2c1
a1b2  a 2b1 and
c a  c 2a1
 1 2
a1b2  a 2b1
Eliminating , we get
2
 b1c 2  b2c1   c1a 2  c 2a1 
 a b  a b    a b  a b 
1 2 2 1 1 2 2 1

2
 b1c 2  b2c1 a1b2  a2b1   c1a 2  c 2a1 
The above is the required condition for the two quadratic equa-
tions to have a common root.
The common root is given by
c1a2  c 2a1 b c  b2c1
  1 2
a1b2  a 2b1 or c1a 2  c 2a1
Note: (i) To find the common root of two equations, make the
coefficient of second degree terms in two equations
equal and subtract. The value of x so obtained is the
required common root.
(ii) If the two equations have both roots common, then
Elementary Algebra 75

a1 b1 c1
 
a2 b2 c 2

Ex. 8: Fin d th e v a lue of K , so tha t th e eq uati ons


x 2  x  12  0 and Kx 2  10 x  3  0 may have one root
common. Also find the common root.
Soln: Let  be the common root of the two equations.
Hence, 2 – – 12 = 0 and
K2 + 10+ 3 = 0
Solving the two equations,
2  1
 
117 12K  3 10  K
2
  12K  3  117 10  K 
2
 9  4K  1  117 10  K 
 16K 2  8K  1  130  13K
 16K 2  5K  129  0
 16K2 – 48K + 43K – 129 = 0

K KUNDAN
 16K (K – 3) + 43(K – 3) = 0
 (16K + 43) (K – 3) = 0
43
K =  or 3
16
12K  3
 = = –3 or 4
10  K
7. Maximum or Minimum Value of a Quadratic Expression
As we have already seen, equation of the type ax2 + bx + c = 0
(where, a  0) is called a quadratic equation. An expression of
the type ax2 + bx + c is called a “quadratic expression”.
The quadratic expression ax2 + bx + c takes different values as x
takes different values.
As x varies from – to +, the quadratic expression ax2 + bx + c
(i) has a minimum value whenever a > 0. The minimum value of
 4ac  b 2 
the quadratic expression is  and it occurs at
 4a 
b
x  .
2a
(ii) has a miximum value whenever a < 0. The miximum value of
 4ac  b 2 
the quadratic expression is  and it occurs at
 4a 
76 Magical Book on Arithmetical Formulae

b
x  .
2a
Ex. 9: Fi nd t he m axim um o r mi nimu m v a lue of
–5x2 + 20x + 40.
Soln. A quadratic expression of the form ax2 + bx + c, will have
a minimum value when a > 0 and maximum value when
a < 0. Its maximum or minimum value is given by
4ac  b 2 b
and it occurs at x   .
4a 2a
Given, a = –5, b = 20 and c = 40
Since, a < 0, the expression has a maximum value.
4(5)(40)  202
 the maximum value = = 60
4(5)
Ex. 10: In the prev ious exa mple , fi nd t he v alue of
x for which the maximum value occurs.
Soln. The maximum vlaue of the expression occurs at
b 20
x   2
2a 2(5)

K KUNDAN
Chapter-41

Inequality

1. Quadratic Expression
An expression of the form ax2 + bx + c (a  o); where a, b, c are
real numbe rs is called a quadratic expre ssion in x. The
corresponding equation of the expression ax 2 + bx + c is
ax2 + bx + c = 0
2. Real Number Line

On the real number line, as we move right the value becomes


greater.
Therefore, 2 < 3, –3 < –2. [Since on the real number line as
–2 is in the right side of –3, therefore, –2 is greater than –3]
Also, –2 < 0, –2 < 1, –1.5 < –0.5, –1.999 > –2 and so on.
3. Sign Scheme For the Quadratic Expression ax2 + bx + c
The sign scheme for the quadratic expression is always meant
for the real values of x. We cannot compare any two imaginary
numbers. Therefore, to say that ri > 2i or 4i < wi is absolutely
incorrect.
A given quadratic expression in one variable, say x, could be
either positive or negative depending on the values of x. For a
quadratic expression ax2 + bx + c, all such real values of x can
be found for which the given expression in x would be positive
or negative. To find the sign scheme of the quadratic expression
ax2 + bx + c, find the roots of the corresponding equation
ax2 + bx + c = 0 and do the following:
(a) If roots are real and equal or imaginary, then ax2 + bx + c
will have same sign as that of the coefficient of x2 for all
real values of x. Conversely, if a quadratic expression has
same sign for all re al value s of x, the n roots of its
corresponding equation must be imaginary or real and
unequal, ie the discriminant of the corresponding equation
must be < 0.
(b) If roots of the equation ax2 + bx + c are  and , such that ,
 are real and unequal, ie real and distinct, then sign
scheme for ax2 + bx + c is obtained using the number line
as given below:
324 Magical Book on Arithmetical Formulae

It is to be noted that a is the coefficient of x2 and  is the


smaller root and  is the greater root.
For example, find for what real values of x, (a) the expression
x2 + 2x – 3 > 0 and (b) the expression x2 + 2x – 3 < 0.
To solve the above example let us see the following:
Given quadratic expression is x2 + 2x – 3.

K
Corresponding equation is x2 + 2x – 3 = 0.
Now, we find the roots of the above corresponding equation,
x2 + 2x – 3 = 0
x2 + 2x – 3 = 0
x2 + 3x – x – 3 = 0
x(x + 3) – 1(x + 3) = 0

KUNDAN
(x – 1) (x + 3) = 0
x = 1 or – 3
Therefore, = –3 and = 1
Since the roots are real and distinct and coefficient of x2 in
x2 + 2x – 3 is positive, the sign scheme for the expression
x2 + 2x – 3 is as given below:

(a) x2 + 2x – 3 > 0, ie expression x2 + 2x – 3 is either zero or


positive when, x < –3 and x > 1.
(b) x2 + 2x – 3 < 0, ie expression x2 + 2x – 3 is either zero or
negative when, –3 < x < 1.
Note: To answer such questions quickly, always remember
the following:
When the inequality is less than zero (ie < 0),
the value of x is in the smaller range (ie  < x < )
and when the inequality is more than zero (ie > 0)
the value of x is in the larger range (ie – < x <  or
 < x < + ). Try to solve the above example by applying
this quicker method. (Always Remember)
4. Wavy Curve Method to Solve Inequation
Let f (x) = (x + 2) (2x – 5)2 (x – 6)3 be any function. We have to find
the set of values of x for which
(i) f (x) > 0
(ii) f (x) < 0
(iii) f (x) > 0
(iv) f (x) < 0
Inequality 325
Procedure to Solve Such Type of Problems
Step I: Factorize the given algebraic function and make the
coefficient of x positive in each of the factors.
Step II: Find the roots of the given function.
Step III: Arrange the roots in ascending order.
Step IV: Put the roots on Real Number Line.
Step V: Notice whether powers of the factors are even or
odd.
Step VI: Draw the diagram as given below.

K
For drawing a diagram, we begin from the greatest root. The
greatest root will lie in the extreme right on the Real Number
Line. Hence, we start drawing the line from the upper part of
the right-hand side (see the diagram given below) and come to
the point where the greatest root of the given function lies on
the Real Number Line. Check the power of the factor containing
the greatest root, whether it is even or odd. If the power is

KUNDAN
odd, cross the number line and come to the next root (just
next to the greatest root) lying on the Real Number Line, other-
wise don’t cross the number line but remain on the same side
of the number line and come to the next root. Following fig-
ures will illustrate our points.
Case I: If the power of the factor containing the greatest root
is odd:

Case II: If the power of the factor containing the greatest root
is even:
326 Magical Book on Arithmetical Formulae
At point B on the number line, again we have to check the
power of the factor containing root B whether it is odd or even
and follow the same above-discussed procedure.
Convention: We take the upper part of the Real Number Line
as positive (+ve) and the lower part of it as negative (–ve). (See
the above diagram.)
Note: +(ve) and (–ve) are the signs of f(x).
Now, let us see the solution of the sample question given
above.
Roots of f(x) = –2, 5/2, 6

K
Putting the roots on Real Number Line, we get the following
diagram:

KUNDAN
(i) f  x   0 , when x   , 2   6, 
In another way
f  x   0 , when

   x  2  6  x  
(ii) f  x   0 , when

 2  x  5 2   5 2  x  6
(iii) f  x   0 , when

   x  2   6  x  
(iv) f  x   0 , when

 2  x  5 2  5 2  x  6 .
For illustration see the examples given below:
Ex. 1: Solve (3x – 1) (x – 2) < 0.
Soln: Dividing by 3 on both sides (because the term 3x is
there, so to get x we have to divide it by 3),
Inequality 327

 1
then  x    x  2  0
3

1
  x  2
3
1
 3x 2

K Alternative Method: (Wavy Curve Method)


1
Roots of the equation  3x  1 x  2  0 are
3
and 2.

Now, put these roots on the Real Number Line and draw
a curve as given below.

KUNDAN
1
We have crossed the number line at 2 and , because
3
powers of  3x  1 and  x  2 are 1, ie odd.
We have to find the set of solutions of the inequation
 3x  1 x  2  0 . From the above diagram we can eas-
1
ily find the required set of solutions =  x  2.
3
Ex. 2: Solve (2 – x) (x – 5) < 0.
Soln: Multiply by (–1) on both sides and that is why ‘<’ sign
will change to ‘>’
ie (–1) (2 – x) (x – 5) > 0(–1)
  x  2 x  5  0
 

2 5
 x  2   x  5 ie x  R   2, 5 
Alternative Method: (Wavy Curve Method)
Making the coefficient of x + ve,
 2  x  x  5  0   x  2 x  5  0
328 Magical Book on Arithmetical Formulae

(Both the powers are odd, hence we cross the Real Num-
ber Line at 5 and 2).
 2  x  x  5  0 if  x  2 x  5  0

K
Required set of solutions is
 ,2   5,   or,  x  2   x  5
Ex. 3: Solve 3x2 – 7x – 6 > 0.
Soln: 3x 2  9 x  2x  6  0
 3x  x  3  2  x  3  0   x  3 3x  2  0

KUNDAN


2
  x  3  x    0 (dividing both sides by 3)
3

 x  3 x   
2 
  0
3 

 2
 x      x  3
 3
Alternative Method: (Wavy Curve Method)
Roots of the equation 3x 2  7 x  6  0
2
 x = 3 and 
3
Putting the roots on the Real Number Line and drawing
a curve, we get the following:

We have to find the set of solutions of the inequation


3x 2  7 x  6  0 . From the above diagrams required set

 2
of solutions = x      x  3
 3
Inequality 329
Ex. 4: Solve 5x2 + 6x + 1 < 0
Soln: 5x 2  6x  1  0  5x 2  5x  x  1  0
 5x(x + 1) + x + 1 < 0
 (x + 1) (5x + 1) < 0
1 
  x  1  x    0  1  x  
5 5
Alternative Method: (Wavy Curve Method)

K
1
Roots of the equation 5 x 2  6x  1  0 are –1 and  .
5
Putting these roots on the Real Number Line and draw-
ing the diagram, we have the following:

KUNDAN
We have to solve 5 x 2  6x  1  0
From the above diagram it is clear that the required set
1
of solutions = 1  x  
5
Ex. 5: Solve x3 – 6x2 + 11x – 6 < 0.
Soln: Putting x = 1, we get x 3  6 x 2  11x  6  0
Hence (x – 1) is a factor of given equation.

x 3  6 x 2  11x  6 =  x  1 x  5x  6
2

  x  1 x  2 x  3
We have to solve,  x  1 x  2 x  3  0

Required set of solutions is


 x  1   2  x  3 or,  , 1   2, 3
330 Magical Book on Arithmetical Formulae
5. Maximum or Minimum Value of a Quadratic Expression
We know that the equation of the type ax2 + bx + c = 0 (where
a  o) is called a quadratic equation. An expression of the type
ax2 + bx + c is called a “quadratic expression”. The quadratic
expression ax2 + bx + c takes different values as x takes different
values.
As x varie s from –  to +, the quadratic e xpre ssion
ax2 + bx + c
(a) has a minimum value whenever a > 0. The minimum value

K
 4ac  b 2 
of the quadratic expression is  and it occurs at x
 4a 
b
=  .
2a
(b) has a maximum value whenever a < 0. The maximum value
 4ac  b 2 

= KUNDAN
of the quadratic expression is 
b
2a
.
 4a 

Ex. 6: Find the maximum or minimum value of the


and it occurs at x

expression –5x2 + 20x + 40. Also find the value of x


for which maximum or minimum value occurs.
Soln: From the above discussion we know that the quadratic
equation of the form ax2 + bx + c will have a minimum
value when a > 0 and a maximum value when a < 0. Its
 4ac  b 2 
maximum or minimum value is given by 
 4a 
b
and it occurs at x =  .
2a
Here, a = –5, b = 20 and c = 40.
Since, a < 0, the expression –5x2 + 20x + 40 has a
maximum value.

4(5)(40)  202
 the maximum value =  60.
4(5 )
The maximum value of the given expression occurs at
b 20 20
x= = = = 2.
2a 2(5) 10
Simplification
We must learn to do the basic calculations faster. It saves few 11. VBODMAS Rule:
seconds from each question. And at the end, we find that at V stands for Vinculum or bar
least 5-10 minutes have been saved. B Stands for brackets and operation of brackets in
the order (), {}, []
What to do for faster basic calculation: O stands for “of”
1. Remember the tables upto30-40. D stands for division (÷)
2. Learn the method of multiplication. M stands for multiplication (×)
3. Learn the method of addition - substraction in a single A stands for addition (+)
line or column. S stands for substraction (-)
4. Learn the method of approximation. 12. Some algebraic formulae:
5. Learn to compare two fractions.
1. (a + b)2 = a2 + b2 + 2ab
6. Remember square roots, squares, cubes and cube
roots. 2. (a – b)2 = a2 + b2 – 2ab
7. Remember fractional value of percentage. 3. (a + b)2 + (a – b)2 = 2(a2 + b2)

K
8. Remember decimal values of reciprocals. 4. (a + b)2 – (a – b)2=4ab
9. Do calculations mentally. Don’t write unnecessary 5. (a + b) (a – b) = a2 – b2
steps. 6. (a + b)3 = a3 + 3a2b + 3ab2 + b3
10. Learn the laws of surds and indices. 7. (a – b)3 = a3 – 3a2b + 3ab2 – b3
8. a3 + b3 = (a + b) (a2 – ab + b2 )
9. a3 – b3 = (a – b) (a2 + ab + b2)

Exercise

KUNDAN
Directions (Q. 1-51): What will come in place of the
6. 17 7.5  17 3.5  17 4.5  17 ?
question mark(?) in the following equations?
1) 6 2) 8.5 3) 7.5
5 4 3 4) 8.5 5) None of these
1. of of of 222  ?
8 9 5 7. 125% of 320 + ?% of 125 = 440
1) 42 2) 43 3) 39 1) 46 2) 42 3) 50
4) 37 5) None of these 4) 32 5) None of these
2. 56% of 450 + ? = 300 3 4 5
1) 52 2) 48 3) 42 8. of of of ?  36
5 9 8
4) 56 5) None of these
1) 200 2) 328 3) 216
3. 271.5  27 3.5  27 ? 4) 260 5) None of these
1) 5 2) 7 3) 3
4) 2 5) None of these
1
4. 27.06 × 25 – ? = 600 9. 84  3
 85  8 2  8 ?
8
1) 76.3 2) 76.7 3) 76.5
4) 76.2 5) None of these 1) 7 2) 2 3) 3
4) 4 5) None of these
7 4 10. –(a – b) × ? = b – a
5. 4 2  ?
8 13 1) –1 2) 1 3) -a
4) a 5) None of these
1 1 4
1) 11 2) 11 3) 11 11. (a + b) = ? × (– a – b)
3 13 13 1)1 2) –a 3) –1
3 4) –b 5) None of these
4) 11 5) None of these 12. |? + 14| = 11
8
1) –3 2) –25 3) 25
4) 3 5) Either –3 or –25
13. 16 + 26 × 2 = ? 25. 14 × 18.6 ÷ 12 + 19.3 = ?
1) 84 2) 44 3) 40 1) 41 2) 33.5 3) 291.9
4) 832 5) None of these 4) 8.32 5) None of these
26. 84.2  64 2.1  7 8.4  563.5  56?
14. 2567 ÷ 17 × 3 = ? + 180
1) 18.2 2) 9.8 3) 11.9
1) 51 2) 271 3) 273
4) 12.6 5) None of these
4) 73 5) None of these
15. 7.5% of 140 + 2.5% of 80 = ? 27. 4 2  32  ?
1) 125 2) 18.5 3) 145 1) 25 2) 5 3) 125
4) 14.5 5) None of these 4) 425 5) None of these
3 4 7 28. 53% of 120 + 25% of 862 = ?% of 500
16. of of of 1375  ? 1) 42.50 2) 55.82 3) 63.68
5 7 12
1) 185 2) 175 3) 285 4) 38.89 5) None of these
4) 275 5) None of these
17. 32.05 × 15 + ? = 500 29. 872.905 ÷ 9.013 ÷ 1.898 = ?
1) 19.75 2) 19.25 3) 20.75 1) 194 2) 50 3) 102

K
4) 20.25 5) None of these 4) 55 5) 72

18. 10 7.5  52.5  2 2.5  10? 30. 3


86300  ?
1) 10 2) 12.5 3) 9.5 1) 51 2) 35 3) 53
4) 11.5 5) None of these 4) 38 5) 44
31. 0.493 × 1.864 × 3.554 = ?
1) 3.5 2) 6.5 3) 4.8
8 2 1 1 4) 5.2 5) 6.9
19. 1  3  2  11  ?
9 7 7 6
? 55.985
32.   2.167
12 25 25 2.951 13.010
1) 7 2) 14 3) 14
223 223 126

KUNDAN
1) 35 2) 40 3) 16
11 4) 26 5) 18
4) 7 5) None of these 33. 948.991 - 621.052 + ? = 723.486
126
20. 56% of 958 + 67% of 1008 = ?% of 2000 1) 305 2) 416 3) 396
1) 60.592 2) 47.622 3) 42.86 4) 348 5) 443
4) 91.455 5) None of these
34. (3158 + 4602 + ?) ÷ 39 = 347
21. 7 2.3  49 4.7  633.4  815.85  63? 1) 5483 2) 5883 3) 5783
1) 16.25 2) 15.1 3) 13.4 4) 5913 5) None of these
4) 18.9 5) None of these 35. (0.08% of 363 + 0.6% of 241) × 500 = ?
22. ?2  1642  3072  272 1) 846.2 2) 868.2 3) 84.62
4) 86.82 5) None of these
1) 151 2) 189 3) 211
18696 11916
4) 259 5) None of these 36.  ?
20853 28728
915849  795664  ?
2
23.
181 164 155
1) 1849 2) 79 3) 33 1) 2) 3)
331 441 246
4) 37 5) None of these
161
4) 5) None of these
3 5 4 1 241
24. 1  1  2   ?
4 8 5 2 37. 3
328509  ?
7 7 3 1) 63 2) 59 3) 73
1) 2 2) 1 3) 2
40 40 40 4) 69 5) None of these
3 38. (47045 ÷ 9.7) + (2035 ÷ 3.7) = ?
4)1 5) None of these 1) 5400 2) 5445 3) 54
40
4) 54.45 5) None of these
50. 1984 + 523 - ? = 1899
39. 1965 ÷ 17161 ÷ 3 = ?
1) 718 2) 608 3) 708
1) 15 2) 9 3) 5 4) 12 4) 618 5) None of these
5) None of these 51. 3 + 33 + 333 + 3.33 = ?
40. 4225  1225  6  ? 1) 362.3 2) 372.33 3) 702.33
4) 702 5) None of these
1) 1156 2) 1600 3) 1444
4) 1296 5) None of these
52. What should come in place of the question mark in the
41. (31% of 260) × ? = 12896
following question ?
1) 150 2) 140 3) 160
4) 180 5) None of these ? 72

 2 1 4 24 ?
42. 16  12   3  ? 1) 12 2) 16 3) 114
 5 15  81
4) 144 5) None of these
9 2 8 [SBI-PO Exams-1999]
1) 1 2) 3 3) 2 53. What should come in place of the question mark (?) in
19 19 13
the following equation ?

K
9
4) 1 5) None of these 28 ?
13 
? 112
43. 183  43.2 = ? 1) 70 2) 56 3) 48
1) 135 2) 136 3) 137 4) 64 5) None of these
4) 138 5) None of these
54. Which of the following are in descending order of their
value?
3
44. 19683  ?  3
5 7 8 11 5 8 11 7
1) 90 2) 27 3) 3 1) 9 , 11 , 15 , 17 2) 9 , 15 , 17 , 11
4) 18 5) None of these

KUNDAN
11 7 8 5 11 7 5 8
45. 15152  ?  1515 3) 17 , 11 , 15 , 9 4) 17 , 11 , 9 , 15
1) 3030 2) 2295225 3) 4485 5) None of these
4) 5115 5) None of these
46. 60 = ?% of 400 55. Which of the following has fractions in ascending order?
1) 6 2) 12 3) 20 2 3 7 9 8 3 2 7 9 8
4) 15 5) None of these 1) , , , , 2) , , , ,
3 5 9 11 9 5 3 9 11 9
47. 1400 × ? = 1050
1) 1 4 2) 3 4 3) 3 5 8 9 7 3 2 3 2 9 7 8
3) , , , , 4) , , , ,
9 11 9 5 3 5 3 11 9 9
4) 2 3 5) None of these
48. 40% of ? = 240 8 9 7 2 3
5) , , , ,
1) 60 2) 6000 3) 960 9 11 9 3 5
4) 600 5) None of these
49. 35 + 15 × 1.5 = ?
1) 75 2) 25.25 3) 57.5
4) 51.5 5) None of these
Answers and explanations
or, ? + 14 = 11
5 4 3
1. 4; ? =    222  37 or, -11
8 9 5
 ? = -25
2. 2; Let the number be x. or, -3
56 13. 5; 16 + 26 × 2 = 16 + 52 = 68
  450  x  300 14. 3; 2567 ÷ 17 × 3 = ? + 180
100
or, 151 × 3 = ? + 180
or, x = 300 – 252 = 48
or, 453 = ? + 180
3. 1; 271.5  213.5  27 ?  ? = 453 - 180 = 273
15. 5; 7.5% of 140 + 2.5% of 80
or, 27?  27 1.53.5
75  140 25  80

 x
 ? = 5 a  a  a
y x  y   =
10  100 10  100
= 10.5 + 2 = 12.5

4. 3; Let the number be x. 16. 4

K
 27.06 × 25 – x = 600 17. 2
or, x = 676.5 – 600 = 76.5 18. 1; 10 7.5  5 2.5  2 2.5 = 10 7.5  10 2.5  1010
39 30 45 1 Hence, the question mark (?) should be replaced by
5. 5; ?     11 10.
8 13 4 4
6. 2; 17 7.5  17 3.5 17 4.5  17 ? 8 2 1 1
19. 3; 1  3  2  11
9 7 7 6
or, 17 7.5 17 3.5  17 4.5  178.5
or, ? = 8.5 8 2 1 1
 1  3  11  2       
7. 4; 125% of 320 + ?% of 125 = 440 9 7 6 7

KUNDAN
?  125 125
or,  440   320 8 1 1 25
100 100  13      = 14
 9 7 6  126
100 20. 1; 56% 958 + 67% of 1008
 ? = 40   32
125 = 536.48 + 675.36 = 1211.84
Now, 1211.84 = ?% of 2000
3 4 5
8. 3; of of of ? = 36  ? = 60.592
5 9 8
21. 2; 7 2.3  49 4.7  633.4  815.85
5 8 9
or, ?  36    or ? = 216 = 7 2.3  7 4.7  7 4.7  633.4  95.85  95.85
3 5 4
= 7 2.3 4.7  4.7  9 (5.855.85)  633.4
4 1 1
9. 4; 8   85  2 = 8 4  3 5 2  8 4
83 8 = 6311.7  633.4 = 6315.1
 ?=4 Hence, ? = 15.1
10. 2; -(a - b) . x = b - a 22. 4; ? 2  164 2  307 2  272
Put x replacing ‘?’ (question mark)
or -[-(a - b)x] = -[b - a] or (a - b)x = a - b  ?  2  3072  272  1642
ab = 94249  272  26896 = 67081
or x  1
ab
11. 3; a + b = ? × (-a - b) Hence, ? = 67081  259
or a + b = x . (-a - b)
23. 5; 915849  795664
[Put x replacing ‘?’ (question mark)]
or a + b = -x (a + b) = 957 + 892 = 1849
or x = -1 Now, ? 2  1849
12. 5; |? + 14| = 11
 ? = 43
3 5 4 1 37. 4; 3
328509  69
24. 4; 1  1  2 
4 8 5 2 Go through the given options. First of all look for the
digit at the unit's place. Reject 1) because 3 × 3 × 3 =
3 5 1 4
= 1  1  2       ....7. Reject 3) on the same ground. Reject 2) because
4 8 2 5
603  216000 . Now, check 4).
30  25  20  32 43 3 38. 1 39. 3 40. 4 41. 3
=  1
40 40 40 42. 5; 1 8 19
25. 1 43. 1
26. 3; 84.2  64 2.1  78.4  563.5 44. 5; 3
19683  ?  3
= 84.2  8 2 2.1
 78.4  563.5 3
19683 27
 ?=  9
= 84.2  8 4.2  78.4  563.5 3 3

 
= 88.4  78.4  563.5 45. 5; 15152  ?  1515

K
= 568.4  563.5  56(8.43.5)  5611.9 ? = 15152  1515  15153 = 3477265875

27. 5; ? = 625 46. 4; 60 = ?% of 400
28. 2; 53% of 120 + 25% of 862
60  100
= 63.6 + 215.5 = 279.1  ?  15
Now, x% of 500 = 279.1 400
47. 2; 1400 × ? = 1050
279.1  100
x = 55.82% 1050 3
500  ? 
29. 2; 872.905 ÷ 9.013 ÷ 1.898 1400 4
 872 ÷ 9 ÷ 2  49 48. 4; 40% of ? = 240

KUNDAN
30. 5; Note that 30 3  27,000 240  100
 ?  600
40
40 3  64,000 49. 3; ? = 35 + 15 × 1.5 = 35 + 22.5 = 57.5
50. 2; 1984 + 523 - ? = 1899
50 3  1,25,000
 ? = 1984 + 523 - 1899 = 608
Hence, the value of 3
86300 will be lie between 40 51. 2;
and 50. Hence, we have only one such option. 3
Therefore, opt 5) as answer. 33
31. 1; 0.493 × 1.864 × 3.554 333
 0.5 × 1.9 × 3.5  3.325 + 3.33
372.33
? 55.985 ? 13
32. 4;   2.167    2. 2
2.951 13.010 3 56
52. 4; ? ?  24  72
3  56  2.2 Squaring both the sides,
 ?   28.43
13
(?2  ? ) ?3  (8  3)  (8  9)  (8  9)
33. 3; ? = 723.486 + 621.052 - 948.991
 723 + 621 - 948  396 = (8 )23839 3
34. 5; (3158 + 4602 + ?) ÷ 39 = 347
 ?  2  8  9  144
or, 3158 + 4602 + ? = 347 × 39
or, ? = 13533 - 3158 - 4602 = 5773 28 ?
53. 2; 
35. 2; (0.08% of 363 + 0.6% of 241) × 500 ? 112
= (0.2904 + 1.446) × 500 = 868.2
 ?  28 112  56
18696  11916 164  1358424 164
36. 2; = = 54. 4
20853  28728 441 1358424 441 55. 2
Approximation
In this type of question you have to find the approximate Ex. 2: 5003 × 550 = ?
value not the exact value. Some times the choices have very 1) 27,51,000 2)27,59,000
small difference and we are confused with our result. Usually 3) 27,50,000 4) 28,00,000
five types of calculations are asked under this section. These Soln.: 5000 × 550 = 2750000
are Sum, Multiplication, Division, Root and Percentage. In
each case we will apply different type of calculations. +3 0 = 1650
27,51,650
Sum (Addition and Substraction)
Req. Ans. = 703100 + 1480 = 704580  704500
1) Replace the large number by the numbers which have If the options differ by a large number like 50,000 we
maximum possible no. of zeros in the end. For example need not do this Calculation.
230818 use 231000 (If the choices are in thousand) Division: In this case the dividend and the divisor should
Use 230800 (If choices are in hundreds) either be incresed or decresed simultaneously. It should
Use 230820 (If choices are in tens) be shifted to their nearest multiple of ten, hundred or
2) If there are two large numbers to be added then try to thousand depending upon the options.

K
increase one and decrease the other. Ex. 2: 810 ÷ 3.9 = ?
3) If a large number is to be substracted from another 1)205 2)207
large number then both the numbers should be either 3)209 4)211
increased or decresed simultaneously. Be carefull about such type of questions. The options
Multiplication: At the time of multiplication finding are very close. If we slightly increase the divisor by 0.1 and
approximate value is very sensitive. A little change in keep the dividend constant then we will get the answer 202.5.
the number leads to a very much deviated result. So we But the actual result is 207.692  207. So a slight increase or
can not adopt the formulae used in addition and dcrease in one side may cause a great deviation. So both the
substraction. For example dividend and divisor should be increased or deeresed. But if
789 × 893 = 704577 the divisor is increased by 0.1, then Dividend will be increased

KUNDAN
But if we use 790 for 789 and 890 for 893 then the result by how much?
will be 703100. And if the options are 703000, 703500, See the formula
704000 and 704500 then our answer will be 703,000 but The value by which the dividend should be increased
the correct result is 704500. = increase in divisor × approximate value of the quotient
To overcome this problem let us see the formula given Using this formula we well get the the increase in dividend
below. will be (. 1) × 200 = 20.
Ex. 1: Find the aproximate value “789 × 893”. So the dividend will be 830.
Soln.: 790 × 890 = 703100 Thus 830 ÷ 4 = 207.5
Apart from these you have to remember the “BODMAS”
-1 +3 = (-1) 890 + 3(790) = 1480 Rule mentioned in the previous chapter.
Req. Ans. = 703100 + 1480 = 704580  704500

Exercise
Directions (Q. 1-49): What approximate value will come 3. (21 + 99) × (30 – 19.02) = ?
in place of the question-mark (?) in the following questions? 1) 3581 2) 131 3) 1290
(You are not expected to calculate the exact value). 4) 1600 5) 1320
1. 103 × 1003 + 999999999 = 10? + 10?
2 6 2 3
1) 6, 9 2) 9, 9 3) 6, 12 4.    ?
4) 16, 9 5) 6, 18 3 8 3 5
2. 134% of 3894 + 38.94% of 134 = ? 1) 0.45 2) 0.5 3) 1.45
1) 5000 2) 5300 3) 5500 4) 0.2 5) 0.55
4) 5270 5) 4900
5. 1000000.0000001  ?
1) 1000 2) 100 3) 10000 20. 63.9872 × 9449.8780 ÷ 243.0034 = ?2
4) 999 5) 99 1) 2489 2) 2500 3) 50
6. 22.9782 + 0.02 + ? = 23 4) 45 5) 150
1) 0.08 2) 20.08 3) 0.02 21. 5237.897 - 6629.010 + 7153.999 - 2205.102 = ?
4) 0.007 5) 0.80 1) 6340 2) 4688 3) 5240
7. 21 + 3.7 × 2.9 = ? 4) 3558 5) 6290
1) 74 2) 70 3) 27 22. 4985.0346 ÷ 215.987 - 3768.112 ÷ 206.868 = ?
4) 32 5) 44 1) 8 2) 5 3) 18
4) 11 5) 15
8. 447. 75 ÷ 28 × 4.99 = ?
23. 956240  ?
1) 60 2) 70 3) 72
4) 80 5) 75 1) 979 2) 864 3) 1009
4) 647 5) 783
9. 3.52 19.25  ?  275 24. 459% of 849.947 + 266% of 6284.012 - 1486.002 = ?
1) 15 2) 20 3) 30 1) 20330 2) 12640 3) 15000

K
4) 28 5) 40 4) 22160 5) 19130
10. 85% of 225 + 32.91 × 5.01 = ?
1) 340 2) 355 3) 375 25. (9615.36 + 1247.18) ÷ (2435.72 + 1937.92) = ?
4) 345 5) 370 1) 4 2) 9 3) 2
4) 7 5) 8
11. 15.962  75% of 285  ?
1) 435 2) 485 3) 440 26. 5646  3982  39  ?
4) 420 5) 470 1) 77 2) 109 3) 66
12. 1679 ÷ 14.95 × 5.02 = ? 4) 99 5) 119
1) 540 2) 525 3) 545 27. (48 × 296) ÷ (19 × 173) = ?
4) 565 5) 520 1) 2 2) 4 3) 8

KUNDAN
4) 9 5) 3
13. 425 ÷ 16.95 × ? = 225 28. 0.5% of 449 × 8.2% of 674 = ?
1) 11 2) 0.8 3) 9 1) 124 2) 139 3) 146
4) 19 5) 0.9 4) 115 5) 100
14. 198.995 × 12.005 + 16.25 × 6.95 = ? 29. 3784 ÷ 28 + 538 = ?
1) 2580 2) 2550 3) 2400 1) 600 2) 623 3) 651
4) 1450 5) 2500 4) 636 5)673

1 3 98 30. 6,23,898 × 99 = ? × 60,000


15. 12 5  ?  99 1) 1000 2) 1030 3) 1050
99 205 99
1) 40 2) 30 3) 33 4) 1065 5) 1010
4) 45 5) 50 4 3 6 5
16. 145% of 700.05 + 22.99 × 15.05 = ? 31.    ?
5 7 7 9
1) 1300 2) 1425 3) 1395
4) 1280 5) 1360 9 20 18
1) 2) 3)
2
17. 1428.025 ÷ 12.005 + (?) = 240 17 49 25
1) 121 2) 13 4) 123 1 4
4) 15 5) 11 4) 5)
2 7
18. 179.99 ÷ 3.001 × 2.005 = ?
1) 45 2) 120 3) 30 32. 399.982  ?
4) 150 5) 90 1) 160000 2) 15999 3) 1600
4) 1599 5) 16000
625.25  4.01  ?
2
19.
1) 240 2) 480 3) 100
4) 400 5) 320
47. 399.9 + 206 × 11.009 = ?
1
33. 624.9995  4.9989   ? 
2
1) 2800 2) 6666 3) 4666
4.9900865 4) 2400 5) 2670
1) 6 2) 50 3) 10 2 7 17 6
4) 125 5) 15 48.    ?
5 8 19 5
34. 989.001 + 1.00982 × 76.792 = ? 1 1
1) 1000 2) 1100 3) 1065 1) 1 2) 3) 2
2 2
4) 110 5) 100
3 9
4) 4 5) 11
35. 59.99% of 255.012 + 22.98% of 182.005 = ?
1) 162 2) 146 3) 195 49. (299.99999)³ = ?
4) 225 5) 178 1) 27000000 2) 9000000000 3) 180000
4) 2.7 × 109 5) 2700000
36. 1000  ?
1) 10 2) 24 3) 45 50. What approximate value should come in place of the
4) 18 5) 32 question mark (?) in the following equation?
37. 15.002 × ? × 25.0210 = 7113.918 1

K
1) 19 2) 26 3) 11 33 % of 768.9 + 25% of 161.2 - 68.12 = ?
3
4) 31 5) 35
1) 230 2) 225 3) 235
38. 81.38  81.63  ? 4) 220 5) 240
1) 680 2) 218 3) 726
4) 512 5) 134 51. What approximate value should come in place of the
39. 12 × 958 ÷ 17 = ? question mark (?) in the following equation?
1) 532 2) 676 3) 765 39.05 × 14.95 - 27.99 × 10.12 = (36 + ?) ×5
4) 483 5) 806 1) 22 2) 29 3) 34
4) 32 5) 25
40. (8423 + 3120 + 6543) ÷ (1536 + 377 + 189) = ?

KUNDAN
1) 5 2) 14 3) 9 52. What approximate value should come in the place of
4) 18 5) 3 question mark (?) in the following equation?
41. (13% of 7439) × (3.23% of 537) = ?
3
1) 16243 2) 16135 3) 16674 1325 17  508.24 of 20% – 85.39 of 4 = ?
4) 16824 5) 16774
1) 5500 2) 5200 3) 5800
7 3 4 4) 4900 5) 5900
42. 121 ÷  5  8  5   ?
1) 168 2) 288 3) 208 53. What approximate value will come in place of the question
4) 298 5) 198 mark (?) in the following equation?
43. (96) 2  3 78961  ? 625.04  16.96  136.001  17  ?
1) 215 2) 210 3) 220 1) 418 2) 441 3) 425
4) 224 5) None of these 4) 433 5) 449
44. (15.28 × 3.56) ÷ 3.15 = ?
1) 12 2) 9 3) 21 54. If 3167 is added to 4093 and the sum is divided by 145,
4) 17 5) 24 approximately what will be the outcome?
1) 50 2) 75 3) 60
45. 4) 90 5) 80
45689  ?
1) 180 2) 415 3) 150
55. What approximate value will come in place of the
4) 210 5) 300
questions mark (?) in the following equation?
10008.992  3589  0.4987  ?
2070.50 ÷ 15.004 + 39.001 × (4.999)2 = ?
46. 1) 1005 2) 997 3) 1049
10009.001
4) 1213 5) 1113
1) 3000 2) 300000 3) 3000000
56. What approximate value should come in place of the
4) 5000 5) 9000000
question mark (?)? 4) 3635 5) 3824
36.0001 ÷ 5.9998 × ? = 108.0005
59. What approximate value should come in place of the
1) 18 2) 16 3) 256 question mark (?) in the following equation?
4) 316 5) 325
2
6.39 × 15.266 + 115.8 of =?
57. What approximate value should come in the place of 5
question mark (?) in the following equation? 1) 145 2) 165 3) 180
98.98 ÷ 11.03 + 7.014 × 15.99 = (?)2 4) 130 5) 135
1) 131 2) 144 3) 12
4) 121 5) 11 60. What approximate value should come in place of the
question mark (?) in the following equation?
58. What approximate value should come in place of the 857 of 14% - 5.6 × 12.128 = ?
question mark (?) in the following equation? 1) 48 2) 36 3) 60
31% of 3581 + 27% of 9319 = ? 4) 52 5) 46
1) 2630 2) 3625 3) 2625

K
1. 2; Here, 10 3  100 3  999999999
3
 
= 10  10 2 3

= 103  106  10 9
= 103 6  109
 10 9
Answers and explanations
8. 4; 447.75 ÷ 28 × 4.99 = ?
or, ?  448  28  5  80
9. 5; 3.52 19.25  ?  275
1 1
or, 12  19  ?  275
4 4
= 109  109

KUNDAN
1 1 1
Therefore, question mark will be replaced by 9. or, 12  19  12   19    ?  275
2. 4; 134% of 3894 + 38.94% of 134 4 4 4
= 134% of 3894 + 38945 of 1.34 or, ?  275  235  40
= 134% of 3894 + 1.34% of 3894 10. 2; ? = 85% of 225 + 32.91 × 5.01
= 135.34% of 3894 85
= 5270.1396   225  33  5
100
= 5270
3. 5; (21 + 99) × (30 – 19.02)  191.25 + 165  356.25
= 120 × 10.98 11. 5; ? = 15.962  75% of 285
= 120 × 11 = 1320
2 75
4. 4;
2 6 2 3
    16   100  285
3 8 3 5
 256 + 213.75  469.75
72 1 12. 4; 1679 ÷ 14.95 × 5.02 = ?
= = = 0.2
72  5 5  ? = 1680 ÷ 15 × 5  112 × 5  560
13. 3; 425 ÷ 16.95 × ? = 225
5. 1; 1000000.0000001
425  ?
or,  225
= 1000000 = 1000 17
6. 3; 22.9782 + 0.002 + ? = 23
225  17
? = 23 – 22.9802 or, ?  9
= 0.0198 = 0.02 425
7. 4; 21 + 3.7 × 2.9 14. 5; 198.995 × 12.005 + 16.25 × 6.95 = ?
= 21 + 3.7 × 3 ?  199 × 12 + 16 × 7  2388 + 112
= 21 + 11.1 = 32. 1 or, ?  2500
15. 1 16. 5 17. 5 36. 5; 1000  10 10 10  10 10
18. 2 19. 4  10 × 3.16
20. 3; 63.9872 × 9449.8780 ÷ 243.0034  31.6
 64 × 9450 ÷ 243 37. 1; 15.002 × ? × 25.0210 = 7113.918
 64 × 39  2496
7113.918
Now, ? 2  2496  ? =
15.002  25.0210
?  50
21. 4; 5237.897 - 6629.010 + 7153.999 - 2205.102 7100
  18.93
= 3557.784  3558 15  25
22. 2; 4985.0346 ÷ 215.987 - 3768.112 ÷ 206.868 38. 4; 81.38  81.63  81.38 1.63
 4985 ÷ 216 - 3768 ÷ 207
 23 - 18  5 = 83.01  83  512
23. 1; 39. 2; 12 × 958 ÷ 17  676
956240  978
40. 3 41. 5 42. 2
24. 5; 459% of 849.947 + 266% of 6284.012 - 1486.002 43. 1 44. 4
 460% of 850 + 265% of 6285 - 1486

K
 3910 + 16655 - 1486 45. 4; ?  45689  213.75  210
 19079
25. 3 10008.992 
46. 2; ?  3589  0.4987
26. 4 10009.001
27. 2
28. 1  100092  3600  0.50
29. 5  10009  60  0.50  300000
30. 2; We have 47. 5; ? = 399.9 + 206 × 11.009
6,23,898 × 99 = ? × 60,000  400 + (200 + 6) × 11
= 400 + 2200 + 66  2670
6,23,898  99

KUNDAN
 ?
60,000 2 7 17 6
48. 1; ?    
5 8 19 5
623898 100  623898
=  1030 2 7 17 5
60,000    
31. 3; We have 5 8 19 6

4 3 6 5 4 3 7 9 18 2 595
   =    =   0.40  0.60  1.0
5 7 7 9 5 7 6 5 25 5 912
49. 1; ? = (299.99999)³
32. 1; 399.982  400 2  160000  (300)³ = 27000000
33. 3; We have 1
1
50. 1; ? = 33 % of 768.9 + 25% of 161.2 - 68.12
3
624.9995  4.9989 2 ?
4.9900865
1 1
1
= of 768.9 + of 161.2 - 68.12
3 4
 625  52 = ?
5 = 256.3 + 40.3 - 68.12  230
51. 5
1
 ?25  25  10 3
5 52. 1; ? = 1325 17  508.24 of 20% – 85.39 of
34. 3; 989.001 + 1.00982 × 76.792 4
 990 + 1 × 76.8  1066.8  1325 17  500 of 20% – 85 × 0.75
35. 3; 59.99% of 255.012 + 22.98% of 182.005
= 5460 + 100 – 60 = 5500
 60% of 255 + 23% of 182
 153 + 41.86 53. 4; ?  625.04  16.96  136.001  17  25 × 17 + 8
 194.86 = 425 + 8 = 433
57. 5; 98.98 ÷ 11.03 + 7.014 × 15.99 = (?)2
3167  4093 7260
54. 1; Reqd no. =   50 Suppose ? = x
145 145 Then 100 ÷ 11 + 7 × 16  121 (taking approximate
55. 5; 2070.50 ÷ 15.004 + 39.001 × (4.999)2= ? value)
or, ?  2070 ÷ 15 + 39 × 5 × 5  x = 11
= 138 + 975 = 1113 58. 2; ? = 31% of 3581 + 27% of 9319
36 = 1110.11 + 2516.13  3625
56. 5;  ?  108
6 2
59. 1; ? = 6.39 × 15.266 + 115.8 of
3
108
or, ? 60. 4; ? = 857 of 14% - 5.6 × 12.128
6  857 of 14% - 5.6 × 12  120 - 67  52
or, ?  18
or, ? = 324  325.

K
KUNDAN
654 Arithmetic

Chapter-34

Permutation and Combination


Introduction 1 1 x
or, 9!  109!  11  10  9!
In this section, we shall introduce the term and
notation of factorial which will be often used in 1 1   x 1
this chapter. or, 1   
9!  10   11  10  9!
Factorial  1  x
or, 1  
The continued product of first n natural numbers  10  11  10
is called the “n factorial” and is denoted by n! or 11 x
or, 
n 10 11  10
ie n! = 1 × 2 × 3 × 4 × 5 × ... × (n – 1) × n.  x = 11 × 11 = 121
Thus, 3! = 1 × 2 × 3 = 6 Ex. 3: Find the value of n, if
4! = 1 × 2 × 3 × 4 = 24 (i) (n + 2)! = 2550 × n!
5! = 1 × 2 × 3 × 4 × 5 = 120 (ii) (n + 1)! = 12 × (n – 1)!
6! = 1 × 2 × 3 × 4 × 5 × 6 = 720 etc. Soln: (i) (n + 2)! = 2550 × n!
Clearly, n! is defined for positive integers only. or, (n + 2) (n + 1) × n! = 2550 × n!
or, (n + 2) (n + 1) = 2550

K KUNDAN
Zero Factorial or, n2 + 3n – 2548 = 0
As we will require zero factorial in the later sections or, (n + 52) (n – 49) = 0
of this chapter and it does not make any sense to or, n = 49 [ n  0]
define it as the product of the integers from 1 to (ii) (n + 1)! = 12 × (n – 1)!
zero. So, we define 0! = 1. or, (n + 1) × n × (n – 1)! = 12 × (n – 1)!
or, n(n + 1) = 12
 Factorials of proper fractions or negative or, n2 + n – 12 = 0
integers are not defined. or, (n + 4) (n – 3) = 0
 Factor ial n is defined only for whole or, n = 3 [ n  0]
numbers. n! n!
Ex. 4: If 2! (n  2)! and 4 ! (n  4 )! are i n t he
Property of Factorial n!
ratio of 2 : 1, find the value of n.
We know that n! = 1 × 2 × 3 × 4 × ... × (n – 1) × n
= [1 × 2 × 3 × 4 × ... × (n – 1) × n n! n!
= n[(n – 1)!] Soln: We have 2! (n  2)! : 4! (n  4)! = 2 : 1
Thus n! = n[(n – 1)!]
n! 4! (n  4)! 2
For example, 8! = 8(7!), 6! = 6(5!), 4! = 4(3!) etc. or, 2! (n  2)!  
The following examples will illustrate the use n! 1
of this property of factorial n. 4! (n  4)! 2
Ex. 1: Find the LCM of 4!, 5! and 6!. or, 2! (n  2)  (n  3)  (n  4)!  1
Soln: We have 5! = 5 × 4!
6! = 6 × 5 × 4!
4  3  2! 2
 LCM of 4!, 5!, 6! or, 2!  (n  2)  (n  3)  1
= LCM of [4!, 5 × 4!, 5 × 6 × 4!]
= (4!) × 5 × 6 = 6! = 720 or, (n – 2) (n – 3) = 6
1 1 x or, n2 – 5n = 0
Ex. 2: If   , find the value of x. or, n(n – 5) = 0
9 ! 10 ! 11!
or, n = 0, 5
1 1 x But for n = 0, (n – 2)! and (n – 4)! are not
Soln:  
9! 10! 11! meaningful. So n = 5
Permutation and Combination 655

Ex. 5: Prove that (n! + 1) is not divisible by any  25  33 < 26


natural number between 2 and n.  s = 5
Soln: Let m be divisible by k and r be any natural So, E2 (33!)
number between 1 and k. If (m + r ) is
divided by k, t hen we obtain r as the  33   33   33   33   33 
=      
remainder.  2   22   23   24   25 
Since n! = 1 × 2 × 3 × 4 × ... × (n – 1) × n, = 16 + 8 + 4 + 2 + 1 = 31
it follows that n! is divisible by ev ery Hence, the exponent of 2 in 33! is 31 ie
natural number bet ween 2 and n. So 33! is divisible by 231. But 231 is divisible
(n! + 1), when divided by any nat ural by 215 also. Hence, 33! is divisible by 215
number between 2 and n, leaves 1 as the and the largest integer n such that 33! is
remainder. Hence (n! + 1) is not divisible divisible by 2n is 31.
by any natural number between 2 and n. Ex. 8: Find the exponent of 15 in 100!
Soln: We have 15 = 3 × 5
Exponent of Prime p in n!
[On prime factorizing 15, we get 3 × 5]
Let p be a prime number and n be a positive integer, Now, E3(100!) = 48 [See Ex. 6]
n  100  100 
then   denotes the greatest integer less than E5(100!) =   
p  5   52 
[ 52 < 100 < 53  s = 2]
n = 20 + 4 = 24
or equal to p
 Exponent of 15 in 100!
= min(24, 48) = 24
10   12  15  Ex. 9: Find the exponent of 6 in 33!
For example,   = 3,   = 2,   = 5 etc.
3 5  3 Soln: We have 6 = 2 × 3
Let Ep(n) denote the exponent of the prime p in E2(33!) = 31 [See Ex. 7]
the positive integer n. Then,  33   33   33 
  

K KUNDAN
E3(33!) = 
n   n   n   3   32   33 
Ep(n!) =  p    2   ...   s  ; [ 33 < 33 < 34  s = 3]
    p 
  p 
= 11 + 3 + 1 = 15
Where s is the largest positive integer such that Hence, exponent of 6 in 33!
ps  n  ps + 1. = min(15, 31) = 15
The following examples will illustrate the above. Ex. 10: Find the number of zeros at the end
Ex. 6: Find the exponent of 3 in 100!. 100!?
Soln: Let Ep(n) denote the exponent of p in n. Soln: In t erms of pr ime f act or s 100 can be
written as 2a 3b 5c 7d ...
n   n   n 
Then, Ep(n!) =  p    2   ...   s  ; Now, E2(100!)
   p   p 
100  100  100  100 
Where s is the largest positive integer =     
such that ps  n  ps + 1  2   22   23   24 
Here, n = 100, p = 3 and
 34 < 100 < 35  s = 4 100  100 
 5  6 
2   2 
100  100  100  100 
So, E3(100!) =      [ 26 < 100 < 27]
 3   32   33   3 4 
= 50 + 25 + 12 + 6 + 3 + 1 = 97
= 33 + 11 + 3 + 1 = 48
Ex. 7: Prove that 33! is divisible by 215. What 100  100 
  [ 52 < 100 < 53]
 5   52  
is the largest integer n such that 33! is E5(100!) = 
divisible by 2n? = 20 + 4 = 24
Soln: Let Ep(n) denote the exponent of prime p Therefore,
in n. 100! = 297 × 3b × 524 × 7d × ...
Then, we know that = 273 × (2 × 5)24 × 3b × 7d × ...
n   n   n   n  = 1024 × 273 × 3b × 7d × ...
Ep(n!) =  p    2    3   ...   s  ; Thus, the number of zeros at the end of
   p   p   p  100! is 24.
Where s is the largest integer such that
ps  n  ps + 1
Here, n = 33, p = 2
656 Arithmetic

Fundamental Principles of Counting The total number of signals when r


flags are used at a time from 5 flags is
In this section we shall discuss two fundamental equal to the number of arrangements of 5,
principles viz. principle of addition and principle taking r at a time ie 5Pr. Since r can take
of multiplication. These t wo pr inciples will v alues 1, 2, 3, 4, 5. Hence by t he
enable us to under st and per mut ation and fundamental principle of addition, the total
combination. In fact these two principles form the number of signals
basis of permutation and combination. = 5P1 + 5P2 + 5P3 + 5P4 + 5P5
Fundamental Principle of Multiplication: If
there are two jobs such that one of them can be 5! 5! 5! 5! 5!
completed in m ways, and when it has been = (5  1)!  (5  2)!  (5  3)!  (5  4)!  (5  5 )!
completed in any one of these m ways, second job
can be completed in n ways; then the two jobs in 5! 5! 5! 5! 5!
=    
succession can be completed in m × n ways. 4! 3! 2! 1! 0!
Ex. 11: In a class there are 10 boys and 8 girls. = 5 + (5 × 4) + (5 × 4 × 3) + (5 × 4 × 3 × 2)
The teacher wants to select a boy and + (5 × 4 × 3 × 2 × 1)
a gi r l t o repr esent t he cl ass in a = 5 + 20 + 60 + 120 + 120 = 325
function. In how many ways can the Difference Between the Two Principles: As
teacher make this selection? we have discussed in the pr inciple of
Soln: Here the teacher is to perform two jobs: multiplication a job is divided or decomposed into
(i) selecting a boy among 10 boys, and a number of sub-jobs which are connected to each
(ii) selecting a girl among 8 girls. other and the job is said to be performed if each
The first of these can be performed in sub-job is performed.
10 ways and t he second in 8 ways. While in the principle of addition, there are a
Therefore by the fundamental principle number of independent jobs and we have to perform
of multiplication, the required number of one of them. So, the t otal number of ways of
ways is 10 × 8 = 80 completing any one of the sub-jobs is the sum of
In general, the above principle can be the number of ways of completing each sub-job.
extended for any finite number of jobs as

K KUNDAN
Ex. 14: There are 3 candidates for a classical,
stated below: 5 for a Mat hemat i cal , and 4 for a
If there are n jobs J1, J2, ..., Jn such that Natural science scholarship.
job Ji can be performed independently in mi (i) I n how m any ways can t hese
ways; where i = 1, 2, 3, ..., n. Then the total scholarships be awarded?
number of ways in which all the jobs can be (ii) I n how m any ways one of t hese
performed is m1 × m2 × m3 × ... × mn. scholarships be awarded?
Fundamental Principle of Addition: If there Soln: Clear ly, classical scholarship can be
are two jobs such that they can be performed awar ded t o, any one of t he t hree
independently in m and n ways, then either of the candidat es. So, t here are 3 ways of
two jobs can be performed in (m + n) ways. awarding the classical scholarship.
Ex. 12: In a class there are 10 boys and 8 girls. Similarly, Mathematical and Natural
The teacher wants to select either a Science scholarships can be awarded in
boy or a girl to represent the class in a 5 and 4 ways respectively.
function. In how many ways the teacher So,
can make this selection? (i) Number of ways of awar ding three
Soln: Here the teacher is to perform either of scholarships = 3 × 5 × 4 = 60
the following two jobs: [By Fundamental Principle
(i) selecting a boy among 10 boys, or of Multiplication]
(ii) selecting a girl among 8 girls And
The first of these can be performed in (ii) Number of ways of awarding one of the
10 ways and t he second in 8 ways. three scholarships = 3 + 5 + 4 = 12
Therefore, by fundamental principle of [By Fundamental Principle
addition, either of the two jobs can be of Addition]
performed in (10 + 8 =) 18 ways. Hence,
the teacher can make the select ion of Permutations
either a boy or a girl in 18 ways.
Ex. 13: How many different signals can be given A per mut ation is an arr angement in a
using any number of flags from 5 flags definite order of a number of objects taken some or
of different colours? all at a time. Consider the following examples.
Soln: The signals can be made by using at a time
one or two or three or four or five flags.
Permutation and Combination 657

Ex. 15: Write down all the permutations of the taken 3 at a time.
set of three letters A, B, C. So, required number of ways
Soln: The permutations of three letters A, B, C
7! 7! 7  6  5  4!
taking all at a time are: = 7P3 = (7  3)!  4!  = 210
ABC, ACB, BCA, BAC, CBA, CAB 4!
Clearly, there are 6 permutations. Alternative Method: First prize can be
Ex. 16: Write down all the permutations of the won in seven ways. Second prize can be
v owels A, E, I , O and U i n Engl ish won by any one of t he r emaining six
alphabet taking three at a time and athletes in 6 ways. Now, five athletes are
starting with A. left. So, third prize can be won by any
Soln: The permutations of vowels A, E, I, O, U one of the remaining 5 athletes in 5 ways.
taking three at a time and starting with A Hence by the fundamental principle of
are: counting, the required number of ways
AEI, AIE, AEO, AOE, AEU, AUE, AIO, AIU, = 7 × 6 × 5 = 210
AUI, AOU, AUO Ex. 19: The flag of a newly formed forum is in
Clearly, there are 12 permutations. the form    of three blocks, each
Number of permutations of n distinct things to be coloured differently. If there are
taken r at a time: If n and r are positive integers 6 different col ours on t he whole to
such t hat 1  r  n, t hen t he number of all choose from, how many such designs
permutations of n distinct things, taken r at a time are possible?
is denoted by P(n, r) or nPr and and it is equal to Soln: The total number of possible designs is
n! the number of arrangements of 6 different
colours taking 3 colours at a time.
(n  r )!
Hence required number of designs
n! 6! 6! 6  5  4  3!
Thus, nPr or P(n, r) = (n  r )! = Total number of = 6P3 = (6  3)!  3!  3!
= 6 × 5 × 4 = 120
permutations of n distinct things taken r at a time.

K KUNDAN
Alternative Method:
From the above we can have, the number of Since there are 6 colours to choose from,
all permutations n of distinct things, taken all therefore, first block can be coloured in 6
at a time is n!. ways.
Now, consider the following examples. Now, the second block can be coloured
Ex. 17: In how many ways three different rings by any one of the remaining colours in
can be worn in four fingers with at five ways. So, there are five ways to colour
most one in each finger? the second block.
Soln: The total number of ways is same as the After colouring first two blocks only
number of arrangements of 4 finger s, four colours are left. The third block can
taken 3 at a time. So, required number of now be colour ed by any one of t he
ways remaining four colours. So, there are four
4! 4! ways to colour the third block. Hence by
= 4P3 = (4  3)!  1! = 4! fundamental principle of multiplication,
the number of flag-designs = 6 × 5 × 4 =
= 4 × 3 × 2 × 1 = 24 120
Alternative Method: Ex. 20: How many different signals can be made
Let R1, R2, R3 be three rings. Since R1 can by 5 flags from 8 flags of different
be put in any one of the four fingers. So, colours?
there are four ways in which R 1 can be Soln: The total number of signals is the number
worn. Now, R2 can be worn in any one of of arrangements of 8 flags by taking 5 flags
the remaining three fingers in 3 ways. In at a time.
the remaining 2 fingers ring R 3 can be Hence the required number of signals
worn in 2 ways. So, by the fundamental 8! 8!
principle of counting the total number of = 8P5 = (8  5)!  3!
ways in which three different rings can
be worn in four fingers = 4 × 3 × 2 = 24. 8  7  6  5  4  3!
Ex. 18: Seven athletes are participating in a = = 6720
3!
race. In how many ways can the first
three prizes be won? Ex. 21: In how many ways can 6 persons stand
Soln: The total no. of ways in which first three in a queue?
pr izes can be won is t he number of Soln: The number of ways in which 6 persons
arrangements of seven different things can stand in a queue is same as the number
658 Arithmetic

of arrangements of 6 different things taken Ex. 25: How many four-digit numbers are there
all at a time. with distinct digits?
Hence, the required number of ways = 6P6 Soln: The total number of arrangements of 10
digits 0, 1, 2, 3, 4, 5, 6, 7, 8 and 9 taking
6! 6!
= (6  6)!  0! = 6! 4 at a time is 10P4. But these arrangements
also include those numbers which have
= 6 × 5 × 4 × 3 × 2 × 1 = 720 0 at thousand’s place. Such numbers are
Ex. 22: I t is r equir ed to seat 5 m en and 4 not four-digit numbers.
women in a r ow so that the women When 0 is fixed at thousand’s place,
occupy the even places. How many such we have to arrange remaining 9 digits by
arrangements are possible? taking 3 at a time. The number of such
Soln: In all 9 persons are to be seated in a row arrangements is 9P3.
and in the row of 9 positions there are So, the total number of numbers having 0
exact ly f our ev en places viz second, at thousand’s place = 9P3.
fourth, sixth and eighth. It is given that Hence, t he t ot al number of four -digit
these four even places are to be occupied numbers
by 4 women. This can be done in 4P4 ways = 10P4 – 9P3
(ways of arr anging 4 women in 4 10! 9! 10! 9!
positions). The remaining 5 positions can = (10  4)!  (9  3)!  6!  6!
be filled by the 5 men in 5P5 ways. So, by
the fundamental principle of counting the = (10 × 9 × 8 × 7 – 9 × 8 × 7)
number of required sitting arrangements = 5040 – 504 = 4536
Ex. 26: How many four-letter words with or
4! 5!
= 4P4 × 5P5 = (4  4)!  (5  5)! without meaning, can be formed out of
the letters of the word ‘LOGARITHMS’,
= 4! × 5! = 24 × 120 = 2880 if repetition of letters is not allowed?
Ex. 23: Three men have 4 coats, 5 waist coats Soln: Ther e ar e 10 let t er s in t he wor d
and 6 caps. In how many ways can they ‘LOGARITHMS’. So, the number of four-

K KUNDAN
wear them? letter words = number of arrangements of
Soln: The total number of ways in which three 10 letters, taken 4 at a time = 10P4
men can wear 4 coats is the number of 10! 10!
arrangements of 4 different coats taken 3 = (10  4)!  6!
at a time. So, 3 men can wear 4 coats in
4
P3 ways. Similarly, 5 waist coats and 6 = 10 × 9 × 8 × 7 = 5040
caps can be worn by three men in 5P 3 Ex. 27: A room has 6 doors. In how many ways
and 6P3 ways respectively. Hence, by the can a man enter the room through one
fundamental principle of counting, the door come out through a different door?
required number of ways Soln: Here a man has to do two jobs:
= 4P3 × 5P3 × 6P3 (i) entering the room and
(ii) coming out through the door.
4! 5! 6! 4! 5! 6!
= (4  3)!  (5  3)!  (6  3)! =   Clearly, a person can enter the room
1! 2! 3! through any one of the 6 doors. So there
= (4 × 3 × 2 × 1) × (5 × 4 × 3) × (6 × 5 × 4) are 6P1 ways of entering into the room.
= 172800 After entering into the room, the man
Ex. 24: How many numbers lying between 100 can come out through any one of the
and 1000 can be formed with the digits remaining 5 doors. So, he can come out
1, 2, 3, 4, 5 if the repetition of digits is through a different door in 5P1 ways.
not allowed? Hence the number of ways in which a
Soln: Every number lying between 100 and 1000 man can enter a room through one door
is a three-digit number. Therefore, we and come out through a different door
have to find the number of permutations = 6P1 × 5P1
of five digits 1, 2, 3, 4 and 5 taken 3 at a 6! 5! 6! 5!
time. = (6  1)!  (5  1)!  5!  4!
Hence, the required number of numbers
= 6 × 5 = 30
5! 5! 5  4  3  2!
= P3 = (5  3)!  2! 
5 Ex. 28: How m any wor ds ( wi t h or wi t hout
2! meaning) of three distinct letters of the
= 5 × 4 × 3 = 60 English alphabets are there?
Soln: Ther e ar e 26 let t er s in t he English
alphabet. The total number of words (with
Permutation and Combination 659

or without meaning) of 3 distinct letters Ex. 30: How m any di ffer ent words can
is the number of arrangements of 26 letters be form ed wi t h t he l ett er s of t he
taking 3 letters at a time. ‘MISSISSIPPI’?
Hence the required number of words Soln: Here, there are 11 objects (letters) of which
= 26P3 there are 4 S’s, 4 I’s, 2 P’s and the rest M
is different.
26! 26!
= (26  3)!  23! So, total number of words is the number
of arrangements of 11 things, of which 4
= 26 × 25 × 24 = 15600. are similar of one kind, 4 are similar of
Alternative Method: second kind and 2 are similar of third
Here we have to fill up three places by 11!
distinct letters of the English alphabets. kind ie .
Since there are 26 letters of the English 4!4!2!
alphabet, the first place can be filled by Hence, the total number of words
any of these letters. So, there are 26 ways 11!
of f illing up the fir st place. Now, t he = = 34650
4!4!2!
second place can be filled up by any of
the remaining 25 letters. So, there are 25 (ii) Permutations When Objects Can Repeat:
ways of filling up the second place. After The number of per mutat ions of n dif ferent
filling up t he first two places only 24 things, taken r at a time, when each may be repeated
letters are left to fill up the third place. any number of times in each arrangement, is nr .
So the third place can be filled in 24 ways. Ex. 31: Find the number of four-letter words,
Hence, the r equired number of wor ds with or without meaning, which can be
= 26 × 25 × 24 = 15600 formed out of the letters of the word
ROSE, where the repetition of the letters
Permutations Under Certain Conditions is allowed.
In this section we shall discuss permutations Soln: Accor ding to t he abov e r ule, since
where either distinction between some of the items repetition is allowed, the required number

K KUNDAN
are ignored or repetitions of items are allowed or a of words = 44 = 256
particular item occurs in every arrangement etc. Ex. 32: How many numbers of three digits can
Such t ype of per mutat ions ar e known as be formed with the digits 1, 2, 3, 4, 5
per mutat ions under cert ain condit ions as when digits may be repeated?
discussed below. Soln: The unit’s place can be filled in 5 ways.
Since, the repetition of digits is allowed,
(i) Permutations when all the objects are therefore, ten’s place can be filled in 5
not distinct objects. ways and hundred’s place can also be filled
(ii) Permutations when object can repeat ie in 5 ways. Therefore, by fundament al
when repetition is allowed. pr inciple of count ing, the requir ed
(iii) Permutations when particular object(s) number of three-digit numbers
occur(s) together or do(es) not occur = 5 × 5 × 5 = 53 = 125
together. Ex. 33: Find the number of numbers of five
di gi ts that can be form ed wi th t he
(i) Permutations When all the Objects are not digits 0, 1, 2, 3, 4 if the digits can be
Distinct Objects: repeated in the same number.
Soln: In a five-digit number 0 cannot be put in
The number of permutations of n objects, where
ten thousand’s place. So, the number of
p1 objects are of one kind, p2 are of second kind
ways of filling up the 10 thousand’s place
..., pk are of kth kind and the rest, if any, are of
n! 4! 4!
different kind is p ! p !...p ! . = 4P1 = (4  1)!  3! = 4
1 2 k
Ex. 29: Find the number of permutations of the Since the repetition of digits is allowed,
letters of the word ALLAHABAD. therefore, each of the other places can be
Soln: Here, there are 9 objects (letters) of which filled in 5 ways.
there are 4 A’s, 2 L’s and rest are all Hence, the required number of numbers
different. = 4 × 5 × 5 × 5 × 5 = 2500
Ther efor e t he r equir ed number of Ex. 34: How many four-digit numbers are there,
arrangements when a di gi t may be r epeated any
9! 98765 number of times?
=  = 7560 Soln: In a four-digit number 0 cannot be placed
4!2! 2
660 Arithmetic

at thousand’s place. So, thousand’s place (iii) Since any one of the 4 boys may get all
can be filled with any digit from 1 to 9. the prizes. So, the number of ways in
Thus, thousand’s place can be filled in which a boy gets all the three prizes = 4
9
P1 = 9 ways. So, t he number of ways in which
Since repetition is allowed, therefore, each a boy does not get all the prizes = 64 – 4
of the remaining 3 places can be filled in = 60.
10 ways by using the digits from 0 to 9.
Hence, the required number of numbers ( i ii ) Permutation when particular object( s)
= 9 × 10 × 10 × 10 occur(s) together or do( es) not occur
= 9 × 103 = 9000. together:
Ex. 35: In how many ways 5 rings of different Ex. 38: In how many ways can the letters of
types can be worn in 4 fingers? the word ‘PENCIL’ be arranged so that
Soln: The first ring can be worn in any of the 4 (i) N is always next to E?
fingers. So, there are 4 ways of wearing (ii) N and E are always together?
it. Similarly, each one of the other rings Soln: (i) Let us keep EN together and consider it
can be worn in 4 ways. as one letter. Now we have 5 letters which
Hence the requisite number of ways can be arranged in a row in 5P5 = 5! = 120
= 4 × 4 × 4 × 4 × 4 = 45 ways.
Ex. 36: In how many ways can 5 let ters be Hence t he t otal number of ways in
posted in 4 letter boxes? which N is always next to E is 120.
Soln: Since each letter can be posted in any (ii) Keeping E and N together and considering
one of the four letter boxes. So, a letter it as one letter, we have 5 letters which
can be posted in 4 ways. Since there are can be arranged in 5P5 = 5! = 120 ways.
5 letters and each letter can be posted in But E and N can put together in 2P2
4 ways. So, total number of ways in which = 2! = 2 ways.
all the five letters can be posted Hence total number of ways
= 4 × 4 × 4 × 4 × 4 = 45 = 5! × 2! = 240
Ex. 37: I n how many ways can 3 pr izes be Ex. 39: How m any different wor ds can be

K KUNDAN
distributed among 4 boys, when formed with the letters of the word
(i) no boy gets more than one prize? EQUATION so that
(ii) a boy m ust get any number of (i) the words begin with E?
prizes? (ii) the words begin with E and end
(iii) no boy gets all the prizes? with N?
Soln: (i) The total number of ways is the number (iii) the words begin and end with a
of arrangements of 4 taken 3 at a time. consonant?
So, the requisite number of ways Soln: Clear ly, the giv en word EQUATION
= 4P3 = 4! = 24 contains 8 letter out of which 5 are vowels
Alternative Method: and 3 are consonants.
The first prize can be given away in 4 (i) Since all words must begin with E. So,
ways as it may be given to any one of the we fix E at the first place. Now, remaining
4 boys. The second prize can be given 7 letters can be arranged in 7P7 = 7! ways
away in 3 ways, because the boy who got So, total number of words = 7!
the first prize cannot receive the second (ii) Since all words must begin with E and
prize. The third prize can be given away end with N. So, we fix E at the first place
to anyone of the remaining 2 boys in 2 and N at the last place.
ways. Now, r emaining 6 lett er s can be
So, the number of ways in which all arranged in 6P6 = 6! ways.
the prizes can be given away = 4 × 3 × 2 = Hence the required number of words
24 = 6P6 = 6!
(ii) The first prize can be given away in 4 (iii) There are 3 consonants and all words
ways as it may be given to anyone of the 4 should begin and end with a consonant.
boys. The second prize can also be given So, first and last places can be filled with
away in 4 ways, since it may be obtained 3 consonants in 3P2 = 3! ways.
by the boy who has already received a Now, the remaining 6 places are to be
prize. Similarly, third price can be given filled up with the remaining 6 letters in
away in 4 ways. 6
P6 = 6! ways
Hence, the number of ways in which Hence the required number of words
all the prizes can be given away = 4 × 4 × = 3! × 6! = 6 × 720 = 4320
4 = 43 = 64
Permutation and Combination 661

Ex. 40: How many words can be formed with be in the second row. If the two corners
the letters of the word ‘ORDINATE’ so of the second row are reserved for the
that vowels occupy odd places? two tallest students, interchangeable
Soln: There are 4 vowels and 4 consonants in only between them, and if the middle
the word ORDINATE. seat of the front row is reserved for
We have to arrange 8 letters in a row such the principal how many arrangements
that vowels occupy odd places. are possible?
There are 4 odd places 1, 3, 5 and 7. [MAT–2006]
Four vowels can be arranged in these 4 Soln: Since the middle seat of the front row is
odd places in 4P4 = 4! ways. reserved for the principal, the remaining
Remaining 4 even places viz 2, 4, 6, 8 are 6 teachers can be arranged in the front
to be occupied by the 4 consonants. row in 6P6 = 6! ways
This can be done in 4P4 = 4! ways. The two corners of the second row are
Hence the total number of words in which reserved for the two tallest students. They
vowels occupy odd places = 4! × 4! = 576 can occupy these two places in 2P2 = 2!
Ex. 41. In how many ways 5 boys and 3 girls ways.
can be seated in a row so that no two The remaining 18 seats may be occupied
girls are together? by the remaining 18 student s in 18 P 18
Soln: × B × B × B × B × B × = 18! ways.
The 5 boys can be seated in a row in Hence, by the fundamental principle of
(5P5 = )5! ways. counting the total number of arrangements
In each of these arrangements 6 places = 6! × (18! × 2!) = 18! × 1440
are created, shown by the cross-mark, as Ex. 44: How many even numbers are there with
given above. three digits such that if 5 is one of the
Since no two girls are to sit together, so digits, then 7 is the next digit?
we may arrange 3 girls in 6 places. Soln: We have to determine the total number of
This can be done in 6P 3 ways ie 3 girls even numbers formed by using the given
can be seated in 6P3 ways. conditions.

K KUNDAN
Hence, t he t ot al number of seating So, at unit’s place we can use one of the
arrangements digits 0, 2, 4, 6, 8. If 5 is at ten’s place
6! then, as per the given condition, 7 should
= 5P5 × 6P3 = 5! × (6  3)! be at unit’s place. In such a case the
number will not be an even number. So, 5
= 5! × 6 × 5 × 4 = 14400
cannot be at t en’s and unit’s places.
Ex. 42: How many words can be formed from
Hence, 5 can be only at hundred’s place.
the letters of the word DAUGHTER so
Now, two cases arise.
that
Case-I: When 5 is at hundred’s place.
(i) the vowels always come together?
If 5 is at hundred’s place, then 7 will be
(ii) the vowels never come together?
at ten’s place.
Soln: Ther e ar e 8 lett er s in the wor d
Hence unit ’s place can be f illed in
‘DAUGHTER’, including 3 vowels (A, U, 5
P1 = 5 ways by using the five digits 0, 2,
E) and 5 consonants (D, G, H, T, R).
4, 6 and 8.
(i) Considering three vowels as one letter,
So, total number of even numbers
we have 6 letters which can be arranged
= 1 × 1 × 5 = 5
in 6P6 = 6! ways.
Case II: When 5 is not at hundred’s
But corresponding each way of these
place.
arrangements, the vowels A, U, E can be
Now, hundred’s place can be filled in 8P1
put together in 3P3 = 3! ways.
= 8 ways
Hence, the required number of words
(  0 and 5 cannot be used at hundred’s
= 6! × 3! = 720 × 6 = 4320
(ii) The total number of words formed by using place)
all the eight letters of the word DAUGHTER In ten’s place we can use any one of the
is 8P8 = 8! = 4032 ten digits except 5. Hence ten’s place can
So, the total number of words in which be filled in 9P1 = 9 ways.
vowels are never together = Total numbers At unit’s place we have to use one of the
of words – Number of words in which even digits 0, 2, 4, 6, 8. So, unit’s place
vowels are always together can be filled in 5P1 = 5 ways
= 40320 – 4320 = 36000 So, total number of even numbers
Ex. 43: When a group photograph is taken, all = 8 × 9 × 5 = 360
the seven teachers should be in the first Hence, the total number of required even
row and all the twenty students should numbers = 360 + 5 = 365.
662 Arithmetic

Ex. 45: A code word i s t o consi st of two is divisible by 4.


distinct English alphabets followed by The digits at unit’s and ten’s places can
two distinct numbers from 1 to 9. For be arranged as follows:
example, DE45 is a code word. Thousand’s Hundred’s Ten’s Unit’s
(i) How m any such code words are × × 1 2
there? × × 2 4
(ii) How many of them end with an even × × 3 2
integer? × × 5 2
Soln: (i) There 26 English alphabets. Now, corresponding to each such way the
So, first two places in the code word remaining three digits at thousand’s and
can be filled in 26P2 ways. hundred’s places can be arranged in 3P2
In last two places we have to use two ways.
distinct numbers from 1 to 9. Hence, last Hence, the required number of numbers
two places can be filled in 9P2 ways. = 3P2 × 4 = 3! × 4 = 24
Hence, by the fundamental principle Ex. 48: Find the number of ways in which 5
of count ing, the total number of code boys and 5 girls be seated in a row so
words that
= 26P2 × 9P2 = 650 × 72 = 46800 (i) No two girls may sit together.
(ii) Number of code words ending with an (ii) All the girls sit together and all
even integer. the boys sit together
In this case, the code word can have (iii) All the girls are never together.
any of the numbers 2, 4, 6, 8 at t he Soln: (i) Sitting arrangement of the 5 boys and 5
extreme right position. girls, as per the given condition, will be
Hence the extreme right position can as given below.
be filled in 4P1 = 4 ways. × B × B × B × B × B ×
Now, next left position can be filled 5 boys can be seated in a row in 5P5
with any one of the remaining 8 digits in = 5! ways
8
P1 = 8 ways and the two extreme left Now, in the 6 gaps as shown above, 5 girls

K KUNDAN
positions can be filled by two English
6!
alphabets in 26P2 ways. can be arranged in 6P5 = (6  5 )! = 6! ways.
Hence, the total number of code words
which end with an even integer Hence the number of ways in which no
= 4 × 8 × 26P2 = 4 × 8 × 650 two girls sit together = 5! × 6P5 = 5! × 6!
= 20800 (ii) The two groups of girls and boys can be
Ex. 46: How many numbers between 400 and arranged in 2P2 = 2! ways.
1000 can be formed with the digits 0, 5 girls can be arranged among themselves
2, 3, 4, 5, 6 if no digits is repeated in in 5P5 = 5! ways.
the same number? Similarly, 5 boys can be arranged among
Soln: Numbers between 400 and 1000 consist themselves in 5P5 = 5! ways
of three digits with digit at hundred’s place Hence, by the fundamental principle of
greater than or equal to 4. counting, the total number of requisite
Hundred’s place can be filled, by using seating arrangements = 2!(5! × 5!) = 2(5!)2
the digits 4, 5, 6 in 3P1 = 3 ways. (iii) The total number of ways in which all
Now, ten’s and unit’s places can be filled the girls are never together
by the remaining 5 digits in 5P2 ways. = Total number of arrangements – Total
Hence the required number of numbers number of arrangements in which all the
= 3 × 5P2 girls are always together
5! Hence, total number of arrangements
= 3  (5  2)! = 10P10 = 10!
Total number of arrangements in which
all the girls are always together
5!
= 3 = 3 × 5 × 4 = 5P5 × 6P6 = 5! × 6!
3!  required number of arrangements
= 60 = 10! – 5! × 6!
Ex. 47: How many four-digit numbers divisible Ex. 49: Five boys and five girls form a line with
by 4 can be made with the digits 1, 2, the boys and girls alternating. Find the
3, 4 and 5 if the repetition of digits is number of ways of making the line.
not allowed? Soln: 5 boys can be ar r anged in a line in
Soln: Recall that a number is divisible by 4 if 5
P5 = 5! ways.
the number formed by the last two digits Since the boys and girls are alternating,
Permutation and Combination 663

so, the corresponding each of the 5! ways A’s. Also there are 4 even places viz 2nd,
of arrangements of 5 boys we obtain 5 4th, 6th and 8th.
places marked by cross as shown below: So, these 4 even places can be occupied
(i) B1 × B2 × B3 × B4 × B5 ×
4!
(ii) × B1 × B2 × B3 × B4 × B5 by 4 vowels in = =1 way
Clearly, 5 girls can be arranged in 5 places 4!
marked by cross in (5! + 5!) ways. Now, we are left with 5 places in which
Hence, the total number of ways of making 5 letters, of which 2 are alike (2 L’s) and
the line = 5! × (5! + 5!) = 2(5!)2 others are distinct, can be arranged in
Ex. 50: (i) How many different words can be
5!
formed with the letters of the word ways.
HARYANA? 2!
(ii) How many of these begin with H Hence the total number of words in which
and end with N? vowels occupy the even places
(iii) In how many of these H and N are 5! 4! 5!
together? =   = 5 × 4 × 3 = 60
2! 4! 2!
Soln: (i) There are 7 letters in the word HARYANA
of which 3 are A’s and remaining all are (ii) Consider both L together and treat them
each of its own kind. as one letter we have 8 letters out of which
Hence, the total number of words A repeats 4 times and others are distinct.

7! 7  6  5  4  3! 8!
=  These 8 letters can be arranged in
3! 3! 4!
= 7 × 6 × 5 × 4 = 840 ways.
(ii) After fixing H in first place and N in last So, the number of words in which both L
place, we have 5 letters out of which three come together
are alike ie A’s and remaining all are each 8!
of its own kind. = = 1680
4!

K KUNDAN
So, total number of words
Hence the number of words in which both
5! 5  4  3! L do not come together
=  = 20
3! 3! = Total number of words – Number of
(iii) Considering H and N together we have words in which both L come together
(7 – 2 + 1 =) 6 letters out of which three = 7560 – 1680 = 5880
are alike ie A’s and others are each of its Ex. 52: (i) How many arrangements can be
own kind. These 6 letters can be arranged m ade wi t h l et ter s of the wor d
MATHEMATICS?
6! (ii) In how many of them vowels are
in ways.
3! together?
But H and N can be arranged amongst [MAT–2005]
themselves in 2P2 = 2! ways. Soln: (i) Ther e ar e 11 let t er s in t he wor d
Hence, the requisite number of words MATHEMATICS of which t wo are M’s,
two are A’s, two are T’s and all others
6! are dist inct . So, r equir ed number of
=  2! = 120 × 2 = 240
3! arrangements
Ex. 51: How m any different wor ds can be 11!
formed by using all the letters of the = = 4989600
2!2!2!
word ALLAHABAD? (ii) Ther e ar e 4 v owels v iz A, E, A, I.
(i) In how many of them vowels occupy Consider ing t hese f our vowels as one
the even positions? letter we have 8 letters (M, T, H, M, T, C,
(ii) In how many of them both L do not S and one letter obtained by combining
come together? all vowels), out of which M occurs twice,
Soln: Ther e ar e 9 lett er s in the wor d T occurs twice and the rest all are distinct.
ALLAHABAD out of which 4 are A’s, 2 are These 8 letters can be arranged in
L’s and the rest are all distinct. 8!
So, the requisite number of words ways.
2!2!
9! But the four vowels (A, E, A, I) can be put
= = 7560
4!2! 4!
(i) There are 4 vowels and all are alike ie 4 together in ways.
2!
664 Arithmetic

Hence, the total number of arrangements Ex. 55: How many num ber s gr eat er than a
in which vowels are always together million can be formed with the digits
2, 3, 0, 3, 4, 2, 3?
8! 4!
=  Soln: Any number greater than a million will
2!2! 2! contain all the seven digits.
= 10080 × 12 = 120960 Now, we hav e to ar range these sev en
Ex. 53: If all the letters of the word AGAIN be digits, out of which 2 occurs twice, 3
arranged as in a dictionary, what is occurs thrice and the rest are distinct.
the fiftieth word? The number of such arrangements
Soln: In dictionary the words at each stage are
7!
arranged in alphabetical order. Starting = = 420
with the letter A, and arranging the other 2!3!
four letters G, A, I, N we obtain 4P4 = 4! These arrangements also include those
= 24 words numbers which contain 0 at the million’s
Thus, there are 24 words which start with place.
A. These are the first 24 words. Keeping 0 fixed at the millionth place,
Then, starting with G, and arranging the we have 6 digits out of which 2 occurs
other four letters A, A, I, N in different twice and 3 occurs thrice and the rest are
distinct.
4! 22
ways, we obtain  = 12 words
2! 2 6!
These 6 digits can be arranged in
Thus, there are 12 words which start with 2!3!
G. = 60 ways
Now, we start with I. The remaining 4 Hence the number of required numbers
= 420 – 60 = 360.
4!
letters A, G, A, N can be arranged in
2! Circular Permutations
= 12 ways If we arrange the objects along a closed curve viz a
Hence, there are 12 words, which start

K KUNDAN
circle etc, the permutations are known as circular
with I. permutations. So f ar , we have discussed
Thus, we have so f ar constr uct ed 48 permutations of objects (or things) in a row. This
words. type of permutations are generally known as linear
The 49th word should be started with the permutations.
letter N. As we have seen in the earlier sections of this
So, the 49th word is NAAGI and hence chapter t hat ev er y linear ar r angement has a
the 50th word is NAAIG. beginning and an end, but there is nothing like
Ex. 54: If the different permutations of the beginning or end in a circular permutation. Thus
word EXAMINATION are listed as in a in a circular permutation, we consider one object
dictionary, how many items are there as fixed and the remaining objects are arranged as
in the list before the first word starting in case of linear arrangements.
with E? The number of circular arrangements (permutations)
Soln: In a dictionary the words at each stage of n different things taken all at a time is (n – 1)! if
are arranged in alphabetical order. In the clockwise and anticlockwise orders are taken as
given problem we have to find the total different arrangements.
number of words starting with A, because Consider the following circular permutations.
the very next word will start with E. We observe that in both, the order of the circular
For finding the number of words starting arrangement is A1, A2, A3, A4.
with A, we have to find the number of
arrangements of the remaining 10 letters
E, X, M, I, N, A, T, I, O, N, of which there
are 2 I’s, 2 N’s and the others each of its
own kind.
The number of such arrangements
10!
= = 907200
2!2!
Hence, t he required number of it ems Figure-1 Figure-2
= 907200. In Figure 1, the order is anticlockwise whereas
in Figure 2, the order is clockwise. Thus t he
number of circular permutations of n things in
Permutation and Combination 665

which clockwise and anti-clockwise arrangements Soln: Let B1 and B2 be two brothers among 20
give rise to different permutations is (n – 1)!. persons and let M be a person.
For example, the number of permutations of 6 Clearly, person M can be chosen from 18
persons seated around a table is (6 – 1)! = 5!. persons (excluding B1 and B2) in
Because with respect to table, the clockwise and
18!
anti-clockwise arrangements are distinct. 18
P1 = (18  1)! = 18 ways
If anti-clockwise and clockwise order of
arrangements are not distinct e.g. arrangements Considering the two brothers B 1 and B2
of beads in a necklace, arrangement of flowers in and person M as one person, we have
gar land etc. , t hen t he number of cir cular (20 – 3 + 1 = ) 18 persons in all.
1 These 18 persons can be arranged around
permutations of n distinct items is (n  1)! . a circle in (18 – 1)! = 17! ways.
2 But B 1 and B 2 can be arranged among
Ex. 56: (i) In how many ways can 5 persons themselves in 2P2 = 2! = 2 ways ie (i) B1MB2
be seated around a circular table? and (ii) B2MB1
(ii) I n how m any of t hese ar r ange- Hence, by the fundamental principle of
ments will two particular persons counting, the total number of ways
be next to each other? = 18 × 17! × 2! = 2 × 18!
Soln: (i) 5 persons can be seated around a circular Ex. 59: In how many ways can a party of 4
table in (5 – 1)! = 4! = 24 ways m en and 4 wom en be seated at a
(ii) Considering two particular persons as one circular table so that no two women
person, we have 4 persons in all. These are adjacent?
4 persons can be seated around a circular Soln: The 4 men can be seated at the circular
t able in (4 – 1)! = 3! ways. But two table such that there is a vacant seat
particular persons can be arranged among between every pair of men in (4 – 1)! = 3!
themselves in 2! ways. ways.
Hence, the total number of arrangements Now, 4 vacant seats can be occupied by 4
= 3! × 2! = 12 women in 4P4 = 4! ways

K KUNDAN
Ex. 57: (i) If 20 persons were invited for a Hence, the required number of sitting
party, in how many ways can they arrangements
and t he host be seated at a = 3! × 4! = 144
circular table? Ex. 60: A round table conference is to be held
(ii) I n how m any ways wi l l t wo between 20 delegates of 2 countries. In
particular persons be seat ed on how many ways can they be seated if
either side of the host? two particular delegates are (i) always
Soln: (i) Clearly, there are 21 persons, including together? (ii) never together?
the host to be seated around a circular Soln: (i) Let D1 and D2 be two particular delegates.
table. These 21 persons can be seated Considering D1 and D2 as one delegate,
round a circular table in (21 – 1)! = 20! we have (20 – 1 =) 19 delegates in all.
ways. These 19 delegates can be seated round a
(ii) Let P1 and P2 be the two particular persons circular table in (19 – 1)! = 18! ways.
and H be the host. Number of arrangements But t wo part icular delegat es can be
of these two particular persons can be arranged among themselves in
seated on either side of the host = 2P2 = 2! 2
P2 = 2! = 2 ways ie D1D2 and D2D1.
= 2 ways ie (i) P1HP2 and (ii) P2HP1 Hence, the total number of ways
Consider the two particular persons and = 2 × 18! = 2(18!)
t he host as one person, we hav e 19 (ii) To find the number of ways in which two
persons in all. These 19 persons can be particular delegates never sit together, we
seated round a circular table in (19 – 1)! = subtract the number of ways in which
18! ways. they sit together from the total number of
But two particular persons can be seated sitting arrangements of 20 persons.
on either side of the host in 2 ways. So, Clearly, 20 persons can be seated round a
the number of ways sitting 21 persons at circular table in (20 – 1 =) 19! ways.
a circular table with two particular persons Hence, the required number of sitting
on either side of the host = 18! × 2 arrangements
Ex. 58: There are 20 persons among whom are = 19! – 2 × 18!
two brothers. Find the number of ways = 19 × 18! – 2 × 18!
in which we can arrange them around = 18! (19 – 2) = 17(18!)
a circle so that there is exactly one Ex. 61: There are 5 gentlemen and 4 ladies to
person between the two brothers. dine at a round table. In how many
666 Arithmetic

ways can they seat themselves so that Ex. 64: Find the number of ways in which 10
no two ladies are together? di ffer ent beads can be arr anged to
Soln: Five gentlemen can be seated at a round form a necklace.
table in (5 – 1)! = 4! ways. Soln: 10 different beads can be arranged in
circular form in (10 – 1)! = 9! ways. Since
t her e is no dist inction bet ween t he
clockwise and ant iclockwise arr ange-
ment s. So, t he r equir ed number of
1
arrangements = (9! )
2
Ex. 65: Find the number of ways in which 8
different flowers can be strung to form
Now, 5 places are created (as shown in a garland so that 4 particular flowers
figure marked by cross-lines) in which 4 are never separated.
ladies can be arranged in Soln: Considering 4 particular flowers as one
5! flower, we have (8 – 4 + 1 =) 5 flowers
5
P4 = (5  4)! = 5! ways which can be strung to form a garland
Hence, the total number of ways in which = (5 – 1)! = (4!) ways
no two ladies sit together (Since the moment we fix 4 particular
= 4! × 5! = 2880 flowers, the clockwise and anticlockwise
Ex. 62: In how many ways can seven persons arrangements in the garland are distinct.)
sit around a table so that all shall not But 4 particular flowers can be arranged
have the same neighbours in any two in 4P 4 = 4! ways. Thus t he r equir ed
arrangements? number of ways = (4!) × 4! = 576
Soln: Clearly, 7 persons can sit at a round table
in (7 – 1)! = 6! ways Combination
But, in clockwise and ant iclockwise A combination is a grouping of selection of all or
arrangements, each person will have the part of a number of things without reference to the

K KUNDAN
same neighbours. ar rangement of t he things selected. Thus t he
So, the required number of ways number of combinations of the three letters A, B
1 and C taken 2 at a time are AB, AC and BC. Note
= (6! ) = 360
2 t hat AB and BA ar e 1 combinat ion but 2
Ex. 63: Three boys and three girls are to be permutations of the letters A and B.
seated around a table in a circle. Among n 
them, the boy X does not want any girl The symbol nCr or C(n, r) or  r  represents the
 
neighbour and the girl Y does not want number of combinations (selections, groups) of n
any boy neighbour. How many such different things taken r at a time. Thus 9C4 denotes
arrangements are possible? the number of combinations of 9 different things
[MAT– 2002] taken 4 at a time.
Soln: Let B1, B2 and X be three boys and G1, G2 Generally we use the word ‘arrangement’ for
and Y be three girls. Since the boy X does per mutat ion and the wor d ‘ selection’ f or
not want any girl neighbour. Therefore combinations.
boy X will have his neighbours as boys B1
and B2 as shown in the figure. Similarly, Notation and Important Properties
girl Y has her neighbours as girls G1 and
G2 as shown in fig. ( i ) The number of all combinations of n distinct
objects, taken r at a time is given by
n
n! Pr
n
Cr = 
(n  r )! r ! r!
n (n  1)((n  2)...(n  r  1)
=
1.2.3...r
n
( i i) Cn = nC0 = 1
n
But the boys B1 and B2 can be arranged ( i ii ) Cn – r = nCr
n
among themselves in 2! ways and the ( iv) Cr + nCr-1 = n+1Cr
girls G1 and G2 can be arranged among (v) If n and r are non-negative integers such
themselves in 2! ways. n n 1
that 1  r  n, then nCr = . C r -1
Hence, t he r equir ed number of r
n n
arrangements = 2! × 2! = 4 ( vi) If Cx = Cy, then either x = y or x + y = n
Permutation and Combination 667

( vii) If n is even, then the greatest value of nCr (0 Ex. 72: If there are 12 persons in a party, and
r  n) is nCn/2. if each two of them shake hands with
(viii) If n is odd, then the greatest value of nCr (0  each other , how m any handshakes
n n
happen in the party?
C n1 C n1 Soln: It is to note here that, when two persons
r  n) is or .
2 2 shake hands, it is count ed as one
Ex. 66: If nC7 = nC4, find the value of n. handshake, not two. Because if X shaking
Soln: We have, nCx = nCy  x + y = n or x = y hands with Y and Y with X will not be
So, nC7 = nC4  n = 7 + 4 = 11 two different handshakes. Here order is
Ex. 67: Find the value of 10C3. not important. So, this is a question of
n n 1 ‘combination’.
Soln: We have nCr = . C r-1 The total number of handshakes is
r
same as the number of ways of selecting
10 9
 10C3 =  C2 2 persons among 12 persons.
3 12! 12!
10 9 8 = 12C2 = (12  2)!2!  10!2!
=   C1
3 2
12  11  10!
10 9 8 7 = 2  10! = 66
=    C0
3 2 1 Ex. 73: A question paper has two parts. Part
10 9 A and Par t B, each cont ai ni ng 10
=  8  1 [  7C = 1] questions. If the student has to choose
3 2 0

= 120 8 from Part A and 5 from Part B, in


Ex. 68: If nC15 = nC8, find the value of nC21. how m any ways can he choose t he
Soln: n
C15 = nC8  n = 15 + 8 = 23 questions?
[  nCx = nCy  x + y = n] Soln: There are 10 questions in Part A out of
 nC21 = 23C21 = 23C23-21 [  nCr = nCn-r] which 8 questions can be chosen in 10C8
ways.

K KUNDAN
23 22 Similarly, 5 questions can be chosen from
= 23
C2 =  C1
2 Part B containing 10 questions in 10C 5
ways.
23 22 21
=   C0 Hence, by the fundamental principle of
2 1 counting, the total number of ways of
23 22 selecting 8 questions from Part A and 5
=   1 [ nC = 1] from Part B
0
2 1
= 23 × 11 = 253 10! 10!
= 10C8 × 10C5 = (10  8)!8!  (10  5)!5!
Ex. 69: If 10Cx = 10Cx + 4, find the value of x.
10
Soln: Cx = 10Cx + 4  x + x + 4 = 10 10! 10!
 2x = 6  x = 3 =  = 11340
2!8! 5!5!
Ex. 70: From a class of 32 students, 4 are to Ex. 74: In how many ways a committee of 5
be chosen for a competition. In how members can be selected from 6 men
many ways can this be done? and 5 women, consisting of 3 men and
Soln: The required number of ways 2 women?
32 32! Soln: 3 men out of 6 men can be selected in 6C3
C4 
= 28!4! ways. 2 women out of 5 women can be
Ex. 71: Three gentlemen and three ladies are selected in 5C 2 ways. Therefore, by the
candidates for two vacancies. A voter fundamental principle counting, 3 men
has to vote for two candidates. In how out of 6 men and 2 women out of 5 women
many ways can one cast his vote? can be selected in 6C3 × 5C2
Soln: Clearly, there are 6 candidates and a voter  6 5 4 5 4 
has to vote for any two of them. So, the =    = 200 ways
 3  2 1 2 1 
required number of ways is the number of Ex. 75: In how many ways can a cricket eleven
ways of selecting 2 out of 6 ie 6C2. be chosen out of a batch of 15 players
Hence, the required number of ways if
6! 6! (i) t her e is no r est r i ct i on on t he
= 6C2 = (6  2)!2!  4!2! selection?
(ii) a par ti cular pl ayer i s al ways
6  5  4! chosen?
= 4!  2 = 15
(iii) a particular player is never chosen?
668 Arithmetic

Soln: (i) The total number of ways of selecting 11 So, total number of committees in which
players out of 15 is men are in majority
15
C11 = 15C15-11 = 15C4 = (9C5 × 8C7) = (126 × 8) = 1008
Ex. 77: A committee of 5 is to be formed out of
15  14  13  12
= = 1365 6 gents and 4 ladies. In how many ways
4  3  2 1 this can be done, when
(ii) If a particular player is a ways chosen. (i) at least two ladies are included?
This means that 10 players are selected (ii) at most two ladies are included?
out of the remaining 14 players. Soln: (i) A committee of 5 persons, consisting of at
 required number of ways least two ladies, can be formed in the
= 14C10 = 14C14-10 = 14C4 = 1001 following ways
(iii) If a particular player is never chosen. This (a) Selecting 2 ladies out of 4 and 3 gents
means that 11 players are selected out of out of 6. This can be done in 4C2 × 6C3
the remaining 14 players. ways.
 required number of ways (b) Selecting 3 ladies out of 4 and 2 gents
= 14C11 = 14C14-11 = 14C3 = 364 out of 6. This can be done in 4C3 × 6C2
Ex. 76: A committee of 12 is to be formed from ways.
9 women and 8 men. In how many ways (c) Selecting 4 ladies out of 4 and 1 get
this can be done if at least five women out of 6. This can be done in 4C4 × 6C1
have to be included in a committee? In ways.
how many of these committees (i) the Since the committee is formed in any of
women are in majority (ii) and the men t he abov e ways, t her efor e, by t he
are in majority? fundamental principle of addition, the total
Soln: Ther e ar e 9 women and 8 men. A number of forming the committee
committee of 12, consisting of at least 5 = (4C2 × 6C3) + (4C3 × 6C2) + (4C4 × 6C1)
women, can be formed by choosing: = 120 + 60 + 6 = 186
(I) 5 women and 7 men (ii) A committee of 5 persons, consisting of at
(II) 6 women and 6 men most two ladies can be constituted in the
(III) 7 women and 5 men

K KUNDAN
following ways:
(IV) 8 women and 4 men (a) Selecting 5 gents only out of 6.
(V) 9 women and 3 men This can be done in 6C5 ways.
In (I), (b) Selecting 4 gents only out of 6 and one
Number of ways of choosing 5 women out lady out of 4.
of 9 women = 9C5 This can be done in (6C4 × 4C1) ways.
Number of ways of choosing 7 men out of (c) Selecting 3 gents only out of 6 and two
8 men = 8C7 ladies out of 4.
 Number of ways of f or ming t he This can be done in (6C3 × 4C2) ways.
committee in this case = 9C5 × 8C7 Since the committee is formed in any of
Similarly, in (II), (III), (IV) and (V), we have, the above ways, so, the total number of
the number of ways of forming committee ways of forming the committee
are (9C6 × 8C6), (9C7 × 8C5), (9C8 × 8C4) and = 6C5 + (6C4 × 4C1) + (6C3 × 4C2)
(9C9 × 8C3) respectively. = 6 + 60 + 120 = 186
Since committee is formed in any of the Ex. 78: A box contains 5 different red and 6
above ways. di ffer ent whi te bal ls. I n how many
Hence, total number of ways of forming ways can 6 balls be selected so that
t he committ ee, by t he f undament al there are at least two balls of each
principle of addition colour?
= ( 9C 5 × 8C 7) + ( 9C 6 × 8C 6) + ( 9C 7 × 8C 5 ) Soln: The selection of 6 balls, consisting of at
+ (9C8 × 8C4) + (9C9 × 8C3) least two balls of each colour from 5 red
= (126 × 8) + (84 × 28) + (36 × 56) and 6 white balls, can be made in the
+ (9 × 70) + (1 × 56) = 6062 following ways:
(i) Clearly, women are in majority in (III), (IV) Red Ball White Ball
and (V) cases as discussed above. (i) 2 4
So, total number of committees in which (ii) 3 3
women are in majority (iii) 4 2
= (9C7 × 8C5) + (9C8 × 8C4) + (9C9 × 8C3) (i) By selecting 2 red balls, out of 5 and 4
= (36 × 56) + (9 × 70) + (1 × 56) white balls out of 6, this can be done
= 2702 in (5C2 × 6C4) ways.
(ii) Clearly, men are in majority in only (I) (ii) By selecting 3 red balls out of 5 and 3
case as discussed above.
Permutation and Combination 669

white balls out of 6, this can be done (i) Three par t icular student s join t he
in (5C3 × 6C3) ways. excursion party
(iii) By selecting 4 red balls out of 5 and 2 In t his case, we hav e t o choose 7
white balls out of 6, this can be done st udents f rom the remaining 22
in (5C4 × 6C2) ways. students. This can be done in 22C7 ways.
Since t he select ion of 6 balls can be (ii) Three particular students do not join
completed in any one of the above ways. the excursion party.
Hence, by the fundamental principle of In this case, we hav e to choose 10
addition, the total number of ways to select st udents f rom the remaining 22
the balls students. This can be done in 22C 10
= (5C2 × 6C4) + (5C3 × 6C3) + (5C4 × 6C2) ways.
= (10 × 15) + (10 × 20) + (5 × 15) Hence, by the fundamental principle of
= 425 addition, the required number of ways
Ex. 79: In how many ways can a cricket team = 22C7 + 22C10 = 817190
be selected from a group of 25 players Ex. 82: A boy has 3 library tickets and 8 books
containing 10 batsmen, 8 bowlers, 5 of his interest in the library. Of these
all rounders and 2 wicket keepers? 8, he does not want t o borr ow
Assume that the team of 11 players Chemistry Part II, unless Chemistry
requires 5 batsmen, 3 all-rounders 2 Part I is also borrowed. In how many
bowlers and 1 wicket keeper. ways can he choose the three books to
Soln: The selection of team is divided into four be borrowed?
phases: [MAT–1999]
(i) Selection of 5 batsmen out of 10. This Soln: We have the following two possibilities:
can be done in 10C5 ways. (i) When Chemistry Part I is borrowed.
(ii) Selection of 3 all-rounders out of 5. In t his case, t he boy may bor row
This can be done in 5C3 ways. Chemistry Part II. So, he has to select
(iii) Selection of 2 bowlers out of 8. This now two books out of the remaining 7
can be done in 8C2 ways. books of his interest. This can be done

K KUNDAN
(iv) Selection of one wicket keeper out of in 7C2 ways.
2. This can be done in 2C1 ways. (ii) When Chemistry Part I is not borrowed.
The selection of team is completed by In this case, the boy does not want to
completing all the four phases. borrow Chemistry Part II. So, he has to
 By t he f undament al pr inciple of select three books from the remaining
multiplication, the team can be selected 6 books. This can be done in 6C3 ways.
in 10C5 × 5C3 × 8C2 × 2C1 = 141120 ways. Hence, by the fundamental principle of
Ex. 80: A person wishes to make up as many addition, the required number of ways
different parties as he can out of his = 7C2 + 6C3 = 21 + 20 = 41
20 fr i ends such t hat each par t y Ex. 83: In how many ways can 7 plus (+) signs
consi sts of t he same num ber of and 5 minus (–) signs be arranged in a
persons. How many friends should he row so that no two minus (–) signs are
invite? together?
Soln: Suppose he invites r friends at a time. Soln: The plus signs (+) can be arranged in only
Then the total number of parties is 20Cr . one way, because all are identical, as
We have to find the maximum value of shown below:
20
Cr, which is for r = 10.
We know [See Property (vii) in the section + + + + + + +
Notation and Important Properties] that, A blank box in the above arrangement
n shows available space for the minus (–)
if n is even, then nCr is maximum for r = . signs. Since there are 7 plus signs (+),
2
Hence, he should invite 10 friends at a the number of blank boxes is therefore 8.
time in order to form the maximum number The f ive minus signs are now t o be
of parties. arranged in the 8 boxes so that no two of
Ex. 81: From a class of 25 students, 10 are to them are together.
be chosen for an excursion party. There Now, 5 boxes out of 8 can be chosen in
8
are 3 students who decide that either C 5 ways. Since all minus signs ar e
all of them will join or none of them ident ical, so 5 minus signs can be
will join. In how many ways can they arranged in 5 chosen boxes in only one
be chosen? way.
Soln: We have the following possibilities: Hence, t he number of possible
arrangements = 1 × 8C5 × 1 = 56
670 Arithmetic

Ex. 84: In how many ways can 21 identical = Number of ways of selecting 2 out of n
books one English and 19 identical n (n  1)
books on Hindi be placed in a row on a = nC2 =
2
shelf so that two books on Hindi may Out of these lines, n lines are the sides of
not be together? the polygon.
Soln: In order that no two books on Hindi are  Number of diagonals of the polygon
together, we must first arrange all books
on English in a row. Since all English n (n  1) n (n  3)
= n 
books ar e identical, so t hey can be 2 2
arranged in a row in only one way as Ex. 88: A polygon has 44 diagonals. Find the
shown below: number of its sides.
× E × E × E × E × E ..... × E × E × [FMS-DU–2002]
Here, E denotes the position of an English Soln: Let there be n sides of the polygon.
book and (×) that of a Hindi book. We know that the number of diagonals of
Since, there are 21 books on English, the
number places mark (×) are therefore 22. n (n  3)
n-sided polygon is
Now, 19 books on Hindi are to be arranged 2
in these 22 places so that no two of them
are together. n (n  3)
Therefore, = 44
Out of 22 places 19 places for Hindi books 2
can be chosen in 22C19 ways. or, n2 – 3n – 88 = 0
Since, all books on Hindi are identical, or, (n – 11) (n + 8) = 0
so, 19 books on Hindi can be arranged in or, n = 11 and -8 [ n > 0]
19 chosen places in only one way.  n = 11
Hence the required number of ways Hence, there are 11 sides of the polygon.
= 1 × 22C19 × 1 = 1540 Ex. 89: I f m par all el l i nes i n pl ane ar e
Ex. 85: For the post of 5 teachers, there are intersected by a family of n parallel lines.
23 applicants, 2 posts are reserved for Find the num ber of paral l el ogram s

K KUNDAN
SC candi dat es and t here are 7 SC form ed.
candidates among the applicants. In Soln: A parallelogram is formed by choosing two
how many ways can the selection be straight lines from the set of m parallel
m ade? lines and two straight lines from the set
Soln: Clearly, there are 7 SC candidates and 16 of n parallel lines.
other candidates. Two straight lines from the set of m
We have to select 2 out of 7 SC candidates parallel lines can be chosen in mC2 ways
and 3 out of 16 other candidates. and two straight lines from the set of n
This can be done in 7C2 × 16C3 ways parallel lines can be chosen in nC2 ways.
 The number of ways of making t he Hence the number of parallelograms
selection = 7C2 × 16C3 = 11760 formed
m (m  1) n (n  1)
Application of Combinations in = mC2 × nC2 = 
2 2
Geometrical Problems
mn (m  1)(n  1)
Ex. 86: How many triangles can be formed by =
4
joining the vertices of a hexagon?
Soln: There are 6 vertices of a hexagon.
One triangle is formed by selecting a group
Mixed Problems on Permutations and
of 3 vertices from the given 6 vertices. Combinations
This can be done in 6C3 ways Ex. 90: Out of 7 consonants and 4 vowels, how
6! 6! many wor ds of 3 consonants and 2
 Number of triangles = (6  3)!3!  3!3! = 20 vowels can be formed?
Ex. 87: How many diagonal s are ther e in a Soln: Three consonants out of 7 and 2 vowels
polygon with n sides? out of 4 can be chosen in 7C3 × 4C2 ways.
Soln: A polygon of n sides has n vertices. Thus there are ( 7C3 × 4C 2) groups each
By joining any two vertices of a polygon, containing 3 consonants and 2 vowels.
we obtain either a side or a diagonal of Since each gr oup cont ains 5 let t ers,
the polygon. which can be arranged among themselves
Number of line segments obt ained by in 5P5 = 5! ways.
joining the vertices of a n-sided polygon Hence the required number of words
taken 2 at a time = (7C3 × 4C2) × 5! = 25200
Permutation and Combination 671

Ex. 91: How many four-let ter wor ds can be selected objects is immaterial whereas in
formed using the letters of the word permutation, the ordering is essential. For
FAILURE, so that example, (A), (B) and (B), (A) are same as
(i) F is included in each word? combinations but different as permutations.
(ii) F is not included in any word? ( i ii ) Practically to find the permutations of n
Soln: There are 7 letters in the word FAILURE. different things taken r at a time, we first
( i ) To include F in every four-letter word, select r items from n items and then arrange
we first select four letters from the 7 them. So, usually the number of permutations
letters of the word FAILURE such that exceeds the number of combinations.
F is included in every selection. This ( iv) Each combination corresponds to many
can be done by selecting three letters permut at ions. For example, t he six
from the remaining 6 letters, A, I, L, permutations ABC, ACB, BCA, BAC, CBA
U, R and E in 6C3 ways. Now, there are and CAB corr espond t o the same
4 letters in each of 6C3 selections. combination ABC.
Consider one of these 6C 3 selections. Note: Generally, we use the word “arrangements”
This selection contains 4 letters which for permutations and the word “selections”
can be arranged in 4P4 = 4! ways. Thus, for combinations.
each of 6C3 selections provide 4! words. Ex. 93: How m any words can be form ed by
Hence the total number of words taking 4 letters at a time out of the
= 6C3 × 4! = 480 letters of the word MATHEMATICS?
(ii) If F is not to be included in any word, [MAT–2005]
then we first select 4 letters from the Soln: There are 11 letters viz MM, AA, TT, H,
remaining 6 letters. This can be done E, I, C, S. All these letters are not distinct,
in 6C4 ways. so we cannot use nPr . We can choose 4
Now, ever y select ion has 4 let ter s letters from the following ways:
which can be arranged in a row in 4P4 (i) All the four distinct letters.
= 4! ways. (ii) Two distinct and two alike letters.
Hence, the total number of words (iii) Two alike of one kind and two alike

K KUNDAN
= 6C4 × 4! = 360. of another kind.
Ex. 92: How many five-letter words containing (i) All the four distinct letters: There
3 v owel s and 2 consonant s can be are 8 distinct letters viz M, A, T, H, E,
formed using the letters of the word I, C, S out of which 4 can be chosen in
8
‘EQUATION’ so that the two consonants C4 ways. So, the total number of groups
occur together? of 4 letters = 8C 4. Each such groups
Soln: There are 5 vowels E, U, A, I, O and 3 has 4 letters which can be arranged in
consonant s Q, T, N in the given word (4P4 =)4! ways. Hence the total number
EQUATION. of words = 8C4 × 4! = 8P4 = 1680
3 vowels out of 5 and 2 consonants out of (ii) Two distinct and two alike letters:
3 can be chosen in (5C3 × 3C2) ways There are 3 pairs of alike letters viz.
Hence, there are ( 5C3 × 3C2) groups each MM, AA, TT, out of which one pair can
containing 3 vowels and 2 consonants. be chosen in 3C1 ways. Now we have
Now, each group contains 5 letters which t o choose t wo let ter s out of the
are to be arranged in such a way that 2 remaining 7 different types of letters
consonants occur together. which can be done in 7C 2 ways. So,
Considering 2 consonants as one letter, the total number of groups of 4 letters
we have 4 letters which can be arranged in which two are different and two are
in 4P4 = 4! ways. But 2 consonants can be alike in 3C1 × 7C2. Each such group has
put together in 2! ways. Therefore 5 letters 4 let t er s of which 2 are alike and
in each group can be arranged in 4! × 2! remaining two distinct and they can
ways. 4!
Hence, the required number of words be arranged in ways. Hence the
2!
= (5C3 × 3C2) × (4! × 2!) = 1440 total number of words in which two
Difference between a Permutation and a letters are alike
Combination 4!
= 3C1 × 7C2 × = 756
2!
( i ) In combinat ion only selection is made
(iii) Two alike of one kind and two alike
whereas in permutation not only a selection
of other kind: There are 3 pairs of 2
is made but also an ar rangement in a
alike letters out of which 2 pairs can
definite order is considered.
be chosen in 3C 2 ways. So, there are
( i i) In a combinat ion, t he or der ing of t he
672 Arithmetic

3
C2 groups of 4 letters each. In each Hence, t he t otal number of words
group there are 4 letters of which 2 consisting of two alike letters of one
are alike of one kind and 2 are alike of type and 2 alike letters of second type
other kind. These 4 let t er s can be
4!
4! = 3C2 × 2! 2! = 18
arranged in ways. Hence, the total
2!2!
number of words in which two letters (iii) 2 alike and 2 different letters: Out
are alike of one kind and two alike of of 3 sets of two alike letters one set
other kind can be chosen in 3C1 ways. Now, from
4! the remaining 6 distinct letters, 2 letters
= 3C2 × 2! 2! = 18 can be chosen in 6C 2 ways. Thus, 2
alike letters and 2 distinct letters can
From (i), (ii) and (iii), the total number of be selected in (3C1 × 6C2) ways. So, there
four-letter words are (3C1 × 6C2) groups of 4 letters each.
= 1680 + 758 + 18 = 2454 Now, let t er s of each gr oup can be
Ex. 94: How many four-let ter wor ds can be
formed using the letter of the word 4!
ar ranged among t hemselves in
INEFFECTIVE? 2!
Soln: Ther e ar e 11 let t er s in t he wor d ways.
INEFFECTIVE viz. EEE, FF, II, C, T, N, V. Hence, t he t otal number of words
The four-letter words may consist of: consisting of two alike letters and 2
(i) 3 alike letters and 1 distinct letter. distinct letters
(ii) 2 alike letters of one kind and 2 alike
letters of the second kind. 4!
= 6C1 × 6C2 × = 540
(iii) 2 alike letters and 2 distinct letters. 2!
(iv) all different letters. (iv ) All different letters: There are 7
Now we will discuss these four cases distinct letters E, F, I, T, N, V, C out of
one by one: which 4 can be selected in 7C4 ways.
(i) 3 alike letters and 1 distinct letter: So, there are 7C 4 groups of 4 letters

K KUNDAN
There is one set of three alike letters each. The letters in each of 7C4 groups
viz. EEE. So, three alike letters can be can be arranged in 4! ways. So, the
selected in one way. Out of t he 6 total number of four-letter words in
different letters F, I, T, N, V, C one which all letters are distinct = 7C4 × 4!
let ter can be select ed in 6C 1 ways. = 840
Thus, thr ee alike and one differ ent Hence, the total number of four-letter
letter can be selected in 1 × 6C1 = 6C1 words
ways. So, there are 6C1 groups each of = 24 + 18 + 540 + 840 = 1422
which contains 3 alike letters and one
different letter. These 4 letters can be Derangement
4! If there are n different things and n assigned
arranged in 3!1! ways.
places then total number of arrangements of n
Hence, t he t otal number of words different things such that none of the things goes
consisting of three alike and distinct to assigned places. This is called derangement.
letters Number of derangements
4! 1 1 1 1

= 6C1 × 3!1! = 6C1 × 4 = 24 = n !1    ... ( 1)n 
 1! 2! 3! n! 
(ii) 2 alike letters of one kind and 2 alike
letters of second kind: There are three Ex. 95: There are 4 letters and 4 addressed
sets of two alike letters viz. EE, FF, II. envelopes corresponding to each of the
Out of these three sets 2 can be selected letters. Find the number of ways in
in 3C2 ways. So, there are 3C2 groups which all letters can be put in wrong
each of which contains 4 letters out of envelopes.
which 2 are alike of one type and two Soln: Required number of ways
are alike of second type. Now, 4 letters  1 1 1 1
= 4! 1    
in each group can be ar r anged in  1! 2! 3! 4! 
 4!  = 22 – 24 + 12 – 4 + 1 = 9
  ways.
 2! 2! 
Chapter-35

Probability
Introduction each suit, namely Spades, Clubs, Hearts and
Diamonds. Cards of spades and clubs are black
Historically, probability theory began with the study cards. Cards of hearts and diamonds are red cards.
of games of chance, such as roulette and cards. There are 4 honours of each suit. These are Aces,
Apart from games, uncertainty prevails in business Kings, Queens and Jacks. These are called face
and other aspects of life. cards.
“Probabilit y is a concept which numerically
measures the degree of uncertainty and therefore Sample Space
of certainty of the occurrence of events.”
Before defining probability, we shall give certain The set of all possible outcomes in a r andom
concepts used therein. experiment is called a sample space and it is
generally denoted by S.
Experiment Each element of a sample space is called a
sample point.
An operation which can produce some well-defined Ex. 1: In t ossing a fair coin, t her e ar e t wo
outcomes, is known as an experiment. possible outcomes, namely head (H) and
When we perform an experiment in science or t ail (T). So, t he sample space in t his
engineering and repeat the same under identical experiment is given by S = {H, T}
conditions, we get almost the same result every Ex. 2: When we throw a die it can result in any
time. of the six numbers 1, 2, 3, 4, 5, 6. So the
But, t her e ar e exper iment s, which when sample space is given by
repeated under identical conditions, do not produce S = {1, 2, 3, 4, 5, 6}
the same outcome every time. For example, if we Ex. 3: When two coins are tossed together, the
toss a fair coin, we may get a head or a tail. Now, sample space is
if we make further trials, ie, toss the coin again S = {HT, TH, HH, TT}
and again the outcome of each trial depends on Here HT shows the head on the first coin
chance and it is not the same each time. Sometimes and t ail on t he second. Similarly, TH
the head appear and sometimes the tail. stands for tail on the first one and head
on the second; HH means head on each
Random Experiment
and TT means tail on each.
If in each trial of an experiment conducted under Ex. 4: From a group of 3 boys and 2 girls we
identical conditions, the outcome is not unique, select two children. What would be the
but may be any of the possible outcomes then such sample space of this experiment? If we
an experiment is known as a random experiment. denote the boys as B1, B2, B3 and the girls
In a random experiment, the outcome of each as G 1, G 2 ; then the sample space is
tr ial depends on chance which is beyond our described as
control and as such it cannot be predicted with S = { (B1, B2), (B1, B3), (B1, G1), (B1, G2),
certainty. (B2, B 3), (B 2, G 1), (B2, G2), (B3, G1),
For example, tossing a fair coin; rolling an (B3, G2), (G1, G2)}
unbiased die; drawing card from a well-shuffled Ex. 5: A coin is tossed twice, if the second throw
pack of cards; picking up a ball of certain colour results in a tail, then a die is thrown.
from a bag containing balls different colours are Describe the sample space.
all examples of random experiments. Clearly, the sample space is given by
When we throw a coin, then either a Head (H) S = {HH, TH, HT1, HT2, HT3, HT4, HT5,
or a Tall (T) appears. HT6, TT1, TT2, TT3, TT4, TT5, TT6}
A die is a solid cube, having 6 faces, marked 1, Ex. 6: In a simultaneous toss of a die and a coin,
2, 3, 4, 5 and 6 respectively. In throwing a die, the sample space is
the outcome is the number that appears on the S = {(1, H), (2, H), (3, H), (4, H), (5, H), (6, H),
uppermost face. The plural of die is dice. (1, T), (2, T), (3, T), (4, T), (5, T), (6, T)}
A pack of cards has 52 cards. It has 13 cards of
674 Concept of Arithmetic

Important Point If   E, we say that the event E has occurred.


If we have a random experiment with m outcomes If   E, we say that the event E has not occurred.
x1, x2, x3, ...., xm and another with n outcomes y1, For example, if we throw a die, then
y 2, y 3 , . . . ., y n ; then t he sample space of t he S = {1, 2, 3, 4, 5, 6}
experiment which consists of carrying out the two Let E be the event of getting an even number,
experiments together has (mn) outcomes. The then
sample space is {(xi, yi); 1  i  m and 1  i  n}.
E = {2, 4, 6}.
Ex. 7: In a simult aneous throw of two dice,
following are the possible (6 × 6 =) 36 Now, in a trial, let the outcome be 2. Since
outcomes. 2  E, so in this trial, the event E has occurred.
{ (1, 1), (2, 1), (3, 1), (4, 1), (5, 1), (6, 1) In another trial, let the outcome be 5. Since
(1, 2), (2, 2), (3, 2), (4, 2), (5, 2), (6, 2) 5 E, so in this trial, the event E has not occurred.
(1, 3), (2, 3), (3, 3), (4, 3), (5, 3), (6, 3)
(1, 4), (2, 4), (3, 4), (4, 4), (5, 4), (6, 4) Equally Likely Events
(1, 5), (2, 5), (3, 5), (4, 5), (5, 5), (6, 5)
The given events are said to be equally likely, if
(1, 6), (2, 6), (3, 6), (4, 6), (5, 6), (6, 6)}
none of them is expected to occur in preference to
Events the other.
For example, if we roll an unbiased die, each
Any subset of a sample space is called an event. outcome is equally likely to happen.
For example, in a single throw of a die, the However, the die is so formed that a particular
event of getting a prime number is given by, face occurs most often, then the die is biased. So,
E = {2, 3, 5} in this case, the outcomes are not equally likely to
Clearly, the sample space, S = {1, 2, 3, 4, 5, 6} happen.
Evidently, E  S
Complementary Event
Impossible Event and Sure Event
In a random experiment, let S be the sample space
Let S be a sample space, since   S, so  is an and let E be an event. Then E  S. Clearly, Ec  S.

K KUNDAN
event, called an impossible event. Since S  S, So, Ec is also an event, called the complement of
so S is an event, called a sure event.
For example, in the throw of a die, sample space E. Sometimes we denote Ec by E or E. We call the
S = {1, 2, 3, 4, 5, 6}
event E as not-E. Clearly, E occurs when E does
Let E1 = event of getting a number less than 1.
And E2 = event of getting a number less than 7. not occur. Also, E occurs when E does not occur.
Clearly, the events E and ‘not-E’ are such that
Clearly, no outcome can be a number less than 1.
only one of them can occur in a trial and at least
So, E 1 is an impossible event. Clear ly, each one of them must occur.
outcome is a number less than 7. So, E2 is a sure
event. Mutually Exclusive Events
Two events E 1 and E 2 are said to be mutually
Elementary and Composite Events exclusive, if E 1  E 2 = . In other words, such
An event containing only a single sample point is events where the occurrence of one precludes the
called an elementary or simple event. occur rence of t he ot her , ar e called mut ually
Ev ent s ot her t han element ar y ar e called exclusive events.
composite or compound or mixed events. The events which are not mutually exclusive
are known as compatible events.
For example, in a simultanoues toss of two
Ex. 8: In throwing a die, we have S = {1, 2, 3, 4,
coins, the sample space is, 5, 6}
S = {HT, TH, HH, TT} Let E1 = event of getting a number less
Then, the event E1 = {TT} of getting both the than 3.
t ails is a simple ev ent . Also, t he ev ent And E2 = event of getting a number less
E2 = {HT, TH, TT} of getting at least one tail is a than 4.
compound event. Then, E1 = {1, 2} and E2 = {5, 6}
Clearly, E1  E2 = 
Occurrence of an Event So, E1 and E2 are mutually exclusive.
In a random experiment, let S be the sample space Ex. 9: Let us consider a simultaneous toss of two
and E  S, so that E is an event. coins.
Let  be an outcome of a trial. Then S = {HT, TH, HH, TT}
Probability 675

Let E1 = event of getting heads on first Probability of an Event


coin = {HH, HT}
Suppose a bag contains 90 red balls and 10 white
And E2 = event of getting tails on second
balls, which are similar in shape and size. If the
coin = {HT, TT}
balls are thoroughly mixed and then one ball is
Clearly, E1  E2   drawn at random, it will be either red or white
So, E1 and E2 are compatible events. because number of red balls is more than the white
balls in the bag. Clearly, the ball drawn is more
Mutually Exclusive and Exhaustive likely to be red than white. We express it by saying
System of Events that the event of drawing a red ball has more
probability than the event of drawing a white ball.
We say that the subsets E1, E2, E3, ... Ek of a sample To ev ery event associat ed wit h a r andom
space S form a mutually exclusive and exhaustive experiment we try to attach a numerical value
system of events, if called its probability in such a manner that for
(i) Ei  Ej = ; for i  j and any two events, the event which is more likely to
(ii) E1  E2  ...  Ek = S happen has a higher value for the probability.
For example, consider t he exper iment of In a random experiment, let s be a sample space
drawing a card from a well-shuffled pack of 52 and let E  S. Then, E is an event.
cards. Let E1, E2, E3 and E4 be the events, ‘card The probability of occurrence of the event E is
drawn is spades’, ‘card drawn is clubs’, ‘card drawn defined as
is heart s’ and ‘card dr awn is diamonds’
respectively. Number of outcomes favourable
As the card drawn is necessarily one of the to occurence of E
P(E) =
four types, so one of these events surely occurs. Number of all possible outcomes
Clearly, if one of these events occurs, the others
cannot occur. So, E1, E2, E3 and E4 form a mutually Number of distinct elements in E
exclusive and exhaustive system of events. = Number of distinct elements in S

Independent Events n (E)

K KUNDAN
= n (S)
Two events are said to be independent, if the
occurr ence of one does not depend upon the
occurrence of the other. Odds in Favour of an Event and Odds
The event s which are not independent ar e Against an Event
known as dependent events.
Suppose we toss to unbiased coins. If the number of ways in which an event can occur
Let E1 = event of occurrence of head on first be m and the number of ways in which it does not
coin. occur be n, then
And E2 = event of occurrence of head on second m
coin. (i) odds in favour of the event = and
Clearly, the occurrence of head on second coin n
does not depend upon the occurrence of head on n
first coin. So E1 and E2 are independent events. (ii) odds against the event = .
m
Algebra of Events
Important Results on Probability
In a random experiment, let the sample space be
S. Let E  S and F S. Then, E as well as F is an I. In a random experiment, if S is the sample
event. space, and E is an event, then
We say that (i) P(E)  0; ie probability of occurrence of
(i) (E  F) is an event that occurs only when an event is always non-negative.
each one of E and F occurs ie it denotes (ii) P() = 0; ie probability of occurrence of an
the event of simultaneous occurrence of both impossible event is 0.
the events E and F. (iii) P(S) = 1; ie probability of occurrence of a
(ii) (E  F) is an event that occurs only when E sure event is 1.
occurs or F occurs or both occur. II. If E and F are mutually exclusive events, then
ie it denotes the events of occurrence of at (i) P(E  F) = 0 and
least one of the events E and F. (ii) P(E  F) = P(E) + P(F)
Note: (a) For mutually exclusive events E
(iii) (E – F) denotes the occurrence of event E
and F, we have
but not F.
P(E or F) = P(E  F) = P(E) + P(F)
676 Concept of Arithmetic

(b) If E 1, E 2, . . . , E k are mut ually Soln: Clearly, the sample space is


exclusive events, then S = {HHH, HHT, HTH, THH, TTH, THT,
P(E1  E2 E3  ...  Ek) HTT, TTT}
Let E1, E2, E3, E4 and E5 be the events of
= P(E1) + P(E2) + P(E3) + ... + P(Ek)
getting all heads; two heads; one head; at
III. If E and F are two mutually exclusive and least one head and at least two heads
exhaustive events, then P(E) + P(F) = 1. respectively.
IV. Let E be any event and E be its complementary Th en , E 1 = {HHH}
E 2 = {HHT, HTH, THH}
event, then P( E ) = 1 – P(E) E 3 = {HTT, THT, TTH}
V. If E is an event associated with a random E 4 = {HTT, THT, TTH, HHT,
experiment then 0  P(E)  1. HTH, THH, HHH} and
E 5 = {HHT, HTH, HHH}
Ex. 10: Find the probability of getting a head
in a throw of a coin. n (E1 ) 1 n (E 2 ) 3
 P(E1) =  ; P(E ) =  ;
Soln: Here, sample space, S = {H, T} n (S) 8 2 n (S) 8
Event of getting heads, E = {H}
Clearly, n(S) = 2 and n(E) = 1 n (E 3 ) 3 n (E 4 ) 7
 Probability of getting a head is given by P(E3) =  ; P(E ) =  ;
n (S) 8 4 n (S) 8
n (E) 1
P(E) = n (S)  2 n (E5 ) 4 1
and P(E5) =  
n (S) 8 2
Ex. 11: In a simultaneous throw of two dice,
find the probability of getting a total Ex. 14: What is the probability that a number
of 7. selected from the numbers 1, 2, 3, ...,
Soln: We know that in a throw of two dice the 24, 25, is a prime number, when each
total number of possible outcomes is (6 × of the given numbers is equally likely
6 =) 36. to be selected?
Thus, if S is the sample space, then n(s) Soln: Sample space, S = {1, 2, 3, 4, ... 24, 25}

K KUNDAN
= 36 Let E be the event of selecting a prime
Let E be the event of getting a total of 7. number
Then, E = {(1, 6), (2, 5), (3, 4), (4, 3), (5, 2), Then, E = {2, 3, 5, 7, 11, 13, 17, 19, 23}
(6, 1)} n (E) 9
Thus, n(E) = 6  P(E) = n (S)  25
n (E) 6 1 Ex. 15: Ti ckets numbered from 1 t o 20 are
 P(a total of 7) = n (S)  36  6
mixed up together and then a ticket is
Ex. 12: A coi n is tossed successi v el y three drawn at random . What i s the
times. Find the probability of getting probabi l i ty t hat the ti cket has a
exactly one head or two heads. number which is a multiple of 3 or 7?
Soln: Let S be the sample space. Then, Soln: Clearly, the sample space S has 20 points.
S = {HHH, HHT, HTH, THH, TTH, THT, Let E be the event of getting a multiple of
HTT, TTT} 3 or 7.
Let E be the event of getting exactly one Then E = {3, 6, 9, 12, 15, 18, 7, 14}
head or exactly two heads.  P(a multiple of 3 or 7)
Then, E = {HHT, HTH, THH, HTT, THT, n (E) 8 2
TTH} = P(E) = n (S)  20  5
Clearly, n(E) = 6 and n(S) = 8
Ex. 16: One card is drawn from a pack of 52
n (E) 6 3 car ds, each of t he 52 car ds bei ng
 P(E) = n (S)  8  4
equally likely to be drawn. Find the
Ex. 13: Three unbiased coins are tossed. What probability that
is the probability of getting (i) the card drawn is red;
(i) all heads (ii) the card drawn is king;
(ii) two heads (iii) the card drawn is red and a king;
(iii) one head (iv) the card drawn is either red or a
(iv) at least one head king.
(v) at least two heads? Soln: Let S denote the sample space. Then n(S)
= 52
(i) Let E1 be the event of drawing a red card.
Probability 677

Since the number of red cards is 26, we Ex. 19: A bag contains 8 red and 5 white balls.
have n(E1) = 26 Three balls are drawn at random. Find
the probability that
n (E1 ) 26 1
 P(a red card) = P(E1) =   (i) all the three balls are white;
n (S) 52 2 (ii) all the three balls are red;
(ii) Let E2 be the event of drawing a king. (iii) one ball is red and two balls are
Since the number of kings is 4, we have white.
n(E2) = 4 Soln: Total number of balls = (8 + 5) = 13
Let S be the sample space. Then
n (E 2 ) 4 1
 P(a king) = P(E2) =   n(S) = number of ways of select ing 3
n (S) 52 13 balls out of 13
(iii) Let E3 be the event of drawing a red card 13! 13  12  11
which is a king. Since the number of red = 13
C3 = (13  3)!3!  3  2  1 = 286
kings is 2, we have n(E3) = 2
(i) Let E1 be the event of getting 3 white balls.
n (E3 ) 2 1 Th en ,
 P(a red king) = P(E3) =  
n (S) 52 26 n(E1) = number of ways of selecting 3 balls
(iv) Let E4 be the event of drawing a red card out of 5
or a king. 5! 54
Clearly, there are 26 red cards (including = C3 = (5  3)!3!  2
5
= 10
2 red kings) and there are 2 more kings.
Thus, n(E4) = (26 + 2 =) 28  P(getting 3 white balls)
 P(a red card or a king) n (E1 ) 10 5
n (E 4 ) 28 7 = n (S)  286  143
= P(E4) = n (S)  52  13
(ii) Let E2 be the event of getting 3 red balls.
Ex. 17: What is the probability that a leap year Th en ,
n(E2) = Number of ways of selecting

K KUNDAN
select ed at r andom wi ll cont ain 53
Sundays? 3 balls out of 8
Soln: A leap year cont ains 366 days and 8! 876
therefore 52 weeks and 2 days. Clearly, = 8C3 = (8 - 3)!3!  3  2 = 56
there are 52 Sundays in 52 weeks.
For the remaining 2 days, they may be:  P (getting 3 red balls)
(i) Sunday and Monday
n (E2 ) 56 28
(ii) Monday and Tuesday =  
(iii) Tuesday and Wednesday n (S) 286 143
(iv) Wednesday and Thursday (iii) Let E3 be the event of getting 1 red and 2
(v) Thursday and Friday white balls.
(vi) Friday and Saturday Th en ,
(vii) Saturday and Sunday n(E3) = (number of ways of selecting 1 ball
Now, for having 53 Sundays in the year, out of 8) × (number of ways of
one of the above 2 days must be Sunday. selecting 2 balls out of 5)
Thus, out of the above 7 possibilities, 2 = 8C1 × 5C2 = 8 × 10 = 80
favour the event that one of the two days  P(getting 1 red and 2 white balls)
is a Sunday.
n (E3 ) 80 40
2 =  
 required probability = n (S) 286 143
7
Ex. 18: A bag contains 9 black and 12 white Ex. 20: Two cards are drawn at random from
balls. One ball is drawn at random. a pack of 52 car ds. What i s t he
What is the probability that the ball probability that the drawn cards are
drawn is black? both aces?
Soln: Total number of balls = (9 + 12 =) 21 Soln: Let S be the sample space. Then,
Thus, if S is the sample space, then n(S) n(S) = number of ways of select ing 2
= 21C1 = 21 cards out of 52
And, if E is the event of getting a black
52! 52  51
ball, then n(E) = 9C1 = 9 = 52
C2 = (52  2)!2!  = 1326
2
n (E) 9 3
 P(getting a black ball) = n (S)  21  7 Let E be the event of getting both the aces.
Th en ,
678 Concept of Arithmetic

n(E) = number of ways of selecting aces Hence,


out of 4
Number of cases favourable to E
4! 43 P(E) =
Total number of outcomes
= 4
C2 = (4  2)!2!  2 = 6
3
 P(both aces) =
8
n (E ) 6 1 Ex. 23: Three dice are thrown together. Find
= P(E) = n (S)  1326  221
the probability of getting a total of at
Ex. 21: In a lottery of 50 tickets numbered 1 to least 6.
50, t wo t ickets are drawn Soln: Since the three dice are thrown, the total
simultaneously. Find the probability number of points in the sample space (S)
that is (6 × 6 × 6 =) 216.
(i) both the tickets drawn have prime Let E = event of getting a total of at least
numbers; 6.
(ii) none of the tickets drawn has prime Then, E = event of getting a total of less
numbers. than 6 ie 3 or 4 or 5
Soln: Let S be the sample space. Then,
n(S) = number of ways of select ing 2  E = {(1, 1, 1), (1, 1, 2), (1, 2, 1), (2, 1, 1),
numbers out of 50 (1, 1, 3), (1, 3, 1), (3, 1, 1), (1, 2, 2),
(2, 1, 2), (2, 2, 1)}
50! 50  49
= C2 = (50  2)!2! 
50
2 = 1225  n( E ) = 10

(i) Prime numbers between 1 to 50 are: n (E ) 10


So, P(not E) = P( E ) = 
2, 3, 5, 7, 11, 13, 17, 19, 23, 29, 31, 37, n (S) 216
41, 43, 47  P(E) = 1 – P(not E)
Their number is 15.
 20  206 103
Let E be the event of getting both prime = 1   

K KUNDAN
numbers  216  216 108
Then, n(E) = number of ways of selecting Ex. 24: Six dice are thrown simultaneously.
2 numbers out of 15. Find the probability that
(i) all of them show the same face.
15! 15  14
= 15
C2 = (15  2)!2!  = 105 (ii) all of them show different faces.
2 (iii) Exactly thr ee of them show the
 P(getting both prime) sam e face and r em aining t hree
show different faces.
n (E ) 105 21
= P(E) = n (S)  1225  245 (iv) at least four of them show the
same face.
(ii) Number of non-primes from 1 to 50 Soln: Since the 6 dice are thrown, the total
= (50 – 15) = 35 number of points in the sample space (S)
Let F be the event of getting both non- is 6 × 6 × 6 × 6 × 6 × 6 = 66
prime numbers. (i) Let E1 = event of getting all dice having
Then, n(F) = number of ways of selecting the same face.
2 numbers out of 35. Here, all dice show the same face means
we are getting same number on all six
35! 35  34 dice.
= 35
C2 = (35  2)!2!  2 = 595
Then, n(E1) = number of ways of selecting
 P(getting both non-prime) 1 number out of 6 = 6C1
 P(getting all the dice having the same
n (F ) 595 11
= P(F) = n (S)  1225  35 n (E1 ) 6 C1 6 1
face) =  6  6  5
n (S) 6 6 6
Ex. 22: The odds in favour of an event are 3 : (ii) Let E2 = event of getting all dice showing
5. Find the probability of occurrence of different faces
this event. Then, n(E 2) = t he tot al number of
Soln: Let E be the event. arrangements of 6 numbers 1, 2, 3, 4, 5,
Then, the number of cases: 6 by taking all at a time
(i) favourable to occurrence of E = 3
(ii) against the occurrence of E = 5 6!
 total number of outcomes = 8 = 6P6 = (6  6)! = 6!
Probability 679

 P(getting all the dice having different


n (E 4 )
n (E 2 ) 6!  required probability = n (S )
faces) = n (S)  6
6
(iii) Let E3 = event of getting three of the dice 6 5 6!   6 5 6!  6
 C1  C2     C1  C1    C1
showing same face and remaining three =  4!   5!
showing different faces 66
Then n(E 3) = The number of ways in
Ex. 25: The digits 1, 2, 3, 4, 5, 6, 7, 8 and 9
which three dice show the same face and
are written in random order to form a
the remaining three show distinct faces
nine-digit number. Find the probability
6! that this number is divisible by 4.
= 6C1 × 5C3 × Soln: Let S be the sample space. Then
3!
n(S) = number of arrangements of the given
[Select a number which occurs on three
numbers taken all at a time
dice out of 6 numbers 1, 2, 3, 4, 5, 6.
= 9P9 = 9!
This can be done in 6C1 ways. Now select
Let E = event of getting nine-digit numbers
three numbers out of the remaining 5
divisible by 4.
numbers. This can be done in 5C3 ways.
Out of the these 9! numbers only those
Now we have 6 numbers like 1, 2, 3, 4, 4,
numbers are divisible by 4 which have
4; 2, 3, 6, 1, 1, 1 et c. These can be
their last digits as even natural number
6! and the numbers formed by their last two
arranged in ways. Hence the number digits are divisible by 4.
3!
of ways in which three dice show the The various possibilities of last two digits
same face and the remaining three show are 12, 32, 52, 72, 92, 24, 64, 84, 16, 36,
distinct faces, by fundamental principle 56, 76, 96, 28, 48, 68.
This means t hat t here ar e 16 ways of
6 5 6!  choosing the last two digits. Corresponding
of multiplication, is  C1  C3  

K KUNDAN
 3!  to each of these ways the remaining 7 digits
 required probability can be arranged in 7P7 = 7! ways.
Therefore, the total number of nine-digit
6 6! numbers divisible by 4 is 16 × 7!
C1  5C3 
n (E 3 ) 3!  n(E) = 16 × 7!
= n (S) 
66 16  7! 2
 required probability = 
(iv) Let E4 = event of getting at least 4 of the 9! 9
dice having the same face. Ex. 26: A four-digit number is formed with the
Now, at least four of them show the same digits 1, 3, 4, 5 with no repetition. Find
face means t he chance t hat ( i) t he num ber i s
(a) Four dice show the same f ace and divisible by 5 and (ii) the number is odd.
remaining 2 show distinct faces. This Soln: Let S be the sample space. Then
n(S) = the total number of ways in which
can be done in 6 C1  5C2  64!! ways. four-digit number can be formed is 4P4
= 4! = 24
(b) Fiv e dice show t he same f ace and (i) Let E1 = event of getting numbers divisible
remaining one shows a different face. by 5.
The number is divisible by 5 if its unit’s
This can be done in 6 C1  5C1  65!! place is 5. Therefore, unit’s place can be
filled in 1 way, the remaining 3 places
ways.
can be filled with other three digits in
(c) All the six dice show same face. This
(3P3 =)3! ways. Hence the number of ways
can be done in 6C1.
of forming the number divisible by 5 is
By the fundamental principle of addition,
= 3! = 6
required number of arrangements
 n(E1) = 6
6 5 6!   6 5 6!  n (E1 ) 6 1
=  C1  C2     C1  C1    required probability = n (S)  24  4
 4!   5!
(ii) Let E2 = event of getting odd numbers
6 5 6!  
6 5 6! 
6
 n(E4) =  C1  C 2  4!    C1  C1  5!   C1 To form odd numbers, the unit’s place can
   
be filled with 1, 3, or 5 ie in ( 3P 1 =) 3
680 Concept of Arithmetic

ways. The remaining three places can 2 boxes and this can be done in 29 ways.
then be filled with other digits in (3P3 =)3! Hence the total number of ways
ways. = (12C3 × 29)
Hence the number of ways in which the  n(E) = 12C3 × 29
odd numbers can be formed is 3 × 3! = 18 12
 n(E2) = 18 n (E) C3  29
 required probability = 
n (S) 312
n (E 2 ) 18 3
 required probability = n (S)  24  4 Ex. 29: In shuffling a pack of 52 playing cards,
four are accidently dropped; find the
Ex. 27: What is the probability that four S’s chance that the missing cards should
come consecut i vel y i n the wor d be one from each suit.
‘MISSISSIPPI’? Soln: Let S be the sample space. Then
Soln: Let S be the sample space. Then n(S) = The total number of ways in which
n(S) = The total number of words that can 4 cards can be selected out of 52
be f or med by per muting t he 11 cards = 52C4
letters of the word ‘MISSISSIPPI’. Let E = Event of getting the missing cards
11! that should be one from each suit.
= 4! 4! 2!1! There are 4 suits and each suit contains
13 cards.
Let E = event of getting such words in Hence the number of ways of selecting 4
which four S’s come consecutively. cards, 1 from each suit
Since the sequence of 4 consecutive S’s = (13C1 × 13C1 × 13C1 × 13C1)
may start either from the first place or  n(E) = (13C1 × 13C1 × 13C1 × 13C1)
second place, . . . ., or eight h place.
Therefore, there are 8 possible ways in
 required probability =
13 C1 4  2197
which 4 S’s can come consecutively and 52 20825
C4
in each case the remaining 7 letters viz
 7! 

K KUNDAN
MIIIPPI can be arranged in   ways. Addition Theorems on Probability
 4! 2!1! 
Thus the total number of ways in which So f ar we hav e calculat ed t he pr obabilit y of
occurrence of an event by using the definition only.
 7!  But sometimes it is not convenient to find the
4 S’s can come consecutively =  8  
 4! 2!1!  number of cases favourable to the occurrence of an
event due to which the computation of probability
 7!  from the definition only is not possible. In such
 n(E) =  8  4! 2!1!  cases we calculate the probability of the event from
 
known pr obabilit ies of ot her ev ents. This is
n (E ) possible only when the given event is expressible
 required probability = n (S)
as the union of two or more events.
For example, in the random experiment of
7! 11! 4 drawing 2 cards from a well shuffled pack of 52
= 8  4! 2!1!  4! 4! 2!1!  165 cards the event “getting both red cards or both
kings”, can be expressed as the union of two events
Ex. 28: Twelve ball s ar e di stri buted am ong
viz.
three boxes. What is the probability
E : getting two red cards.
that the first box will contain 3 balls?
F : getting two kings.
Soln: Let S be the sample space. Then
In t his art icle we intend t o discuss some
n(S) = The number of ways in which 12
theorems to find the probability of the union of
balls can be placed in 3 boxes
two or more events.
= Since each ball can go to any one
(i) Theorem-I: (Addition theorem for two events)
of the three boxes there are 3 ways
if E and F are two events associated with a random
in which a ball can go to any one
experiment, then
of the three boxes. Thus there are
312 ways in which 12 balls can be
P(E  F) = P(E) + P(F) – P(E  F)
Corollary: If E and F are mutually exclusive
placed in 3 boxes.
events, then
Let E = event getting first box that will
P(E F) = 0
contain 3 balls.
P(E  F) = P(E) + P(F)
Number of ways in which 3 balls out of
This is t he addit ion theorem for mut ually
12 can be put in the first box is 12C3. Now
exclusive events.
the remaining 9 balls one to be placed in
Probability 681

(ii) Theorem-II: (Addition theorem for three n(F) = number of ways of select ing 2
events) if E, F, G are three events associated with kings out of 4 kings
a random experiment, then = 4C2 = 6
P(E F G) = P(E) + P(F) + P(G) - P(E F) – n(E  F) = number of ways of selecting 2
P(F G) - P(E G) + P(E F G) red kings out of 2 red kings
Corollary: If E, F, G are mutually exclusive = 2C2 = 1
events, then
n (E ) 325
P(E F) = P(F G) = P(E G) = P(E F G) = 0 Hence, P(E) = n (S)  1326
 P(E F G) = P(E) + P(F) + P(G)
This is the addition theorem for three mutually
exclusive events. n (F ) 6 1
P(F) = n (S)  1326  221 and
Ex. 30: A card is drawn at random from a well-
shuffled deck of 52 cards. Find the
probability of its being a spade or a n (E  F ) 1
P(E  F) = 
king. n (S ) 1326
Soln: Let S be the sample space. Then  P(both red or both kings)
n(S) = number of ways of selecting one = P(E or F) = P(E  F)
card out of 52 cards = P(E) + P(F) - P(E  F)
52! (By addition theorem)
= 52C1 = (52  1)!1! = 52
 325 1 1  330 55
Let E = event of getting a spade =     
And F = event of getting a king  1326 221 1326  1326 221
Then, E  F = events of getting a king of Ex. 32: A basket contains 20 apples and 10
spade oranges out of which 5 apples and 3
Clearly, n(E) = 13, n(F) = 4 and n(E F) = 1 oranges are defective. If a person takes
out 2 at random what is the probability
n (E ) 13 1 that either both are apples or both are
 P(F) = n (S)  52  4
good?

K KUNDAN
Soln: Let S be the sample space. Then,
n (F ) 4 1 n(S) = number of ways of selecting 2 out
P(F) = n (S)  52  13 and
of 30

n (E  F ) 1 30! 30  29
P(E F) =  = C2 = (30  2)! 2! 
30
2
n (S ) 52
 P(a spade or a king) = 15 × 29 = 435
= P(E or F) = P(E  F) Let E = event of getting both apples.
= P(E) + P(F) - P(E  F) And F = event of getting both good items.
(By addition theorem) Then, E  F = event of getting 2 good
apples.
1 1 1  16 4 n(E) = number of ways of select ing 2
=     
 4 13 52  52 13 apples out of 20

4 20! 20  19
 P (a spade or a king) = = 20
C2 = (20  2)! 2!  2 = 190
13
Ex. 31: Two cards are drawn at random from Ther e ar e 8 def ectiv e pieces and the
a well-shuffled pack of 52 cards. What remaining 22 are good.
is the probability that either both are n(F) = number of ways of selecting 2 good
red or both are kings? items out of 22
Soln: Let S be the sample space. Then, 22! 22  21
n(S) = number of ways of selecting 2 cards = 22
C2 = (23  2)! 2!  2 = 231
out of 52 cards
= 52C2 = 1326 There are only (20 - 5 =) 15 good apples.
Let E = event of getting both red cards n(E  F) = number of ways of selecting 2
And, F = event of getting both kings good apples out of 15
Then, (E  F) = event of getting two red
15! 15  14
kings = 15
C2 = (15  2)!2!  2 = 105
 n(E) = number of ways of selecting 2
cards out of 26 red cards
= 26C2 = 325
682 Concept of Arithmetic

n (E ) 190 And F = {(3, 6), (6, 3), (4, 5), (5, 4)}
Hence, P(E) = n (S)  435 ; Clearly E  F = . So, E and F are mutually
exclusive.
n (F ) 231
P(F) = n (S)  435 and n (E ) 6 1
 P(E) = n (S)  36  6 and

n (E  F ) 105
P(E  F) =  n (F ) 4 1
n (S ) 435 P(F) = n (S)  36  9
 P(both apple or both good)
So, P(a doublet or a total of 9)
= P(E or F) = P(E  F)
= P(E or F) = P(E  F)
= P(E) + P(F) - P(E  F)
= P(E) + P(F)
[By addition theorem]
[ E and F are
 190 231 105  mutually exclusive]
=    
 435 435 435  1 1 5
=   
 6 9  18
 190  231  105  316  P(neither a doublet nor a total of 9)
=  
 435  435 = 1 - P(a doublet or a total of 9)
Ex. 33: Two di ce ar e tossed once. Fi nd t he
 5  13
probability of getting an even number = 1  
on first die or a total of 8.  18  18
Soln: Let S be the sample space. Then, Ex. 35: A di e is t hrown t wi ce. What is t he
n(S) = 6 × 6 = 36 probability that at least one of the two
Let E = event of getting an even number throws comes up with the number 4?
on first die Soln: Let S be the sample space. Then,
And, F = event of getting a total of 8 n(S) = 6 × 6 = 36
Then, E  F = event of getting an even Let E1 = the event that the first throw
number on first die and a total of 8 shows 4.

K KUNDAN
Now, E = {(2, 1), (2, 2), (2, 3), (2, 4), And, E2 = the event that the second throw
(2, 5), (2, 6), (4, 1), (4, 2), (4, 3), (4, 4), (4, shows 4.
5), (4, 6), (6, 1), (6, 2), (6, 3), (6, 4), (6, 5),  E1  E2 = the event showing 4 in each
(6, 6)} thr ow.
F = {(2, 6), (6, 2), (3, 5), (5, 3), (4, 4)} Then, E1 = {(4, 1), (4, 2), (4, 3), (4, 4),
and E  F = {(2, 6), (6, 2), (4, 4)} (4, 5), (4, 6)};
 n(E) = 18, n(F) = 5 and n(E  F) = 3 E 2 = {(1, 4), (2, 4), (3, 4), (4, 4),
(5, 4), (6, 4)}
n (E ) 18 1
Hence, P(E) = n (S)  36  2 ; and E1  E2 = {(4, 4)}
 n(E1) = 6, n(E2) = 6 and n(E1  E2) = 1

n (F ) 5 6 1
P(F) = n (S)  36 and Hence, P(E1) = 
36 6
6 1
n (E  F ) 3 1 P(E2) =  and
P(E  F) =   36 6
n (S) 36 12
1
 P(even number on first die or a total 8) P(E1  E2) =
36
= P(E or F) = P(E  F)
= P(E) + P(F) – P(E  F)  P(at least one 4) = P(E1 or E2)
= P(E1  E2)
1 5 1  20 5 = P(E1) + P(E2) - P(E1  E2)
=     
 2 36 12  36 9
1 1 1  11
Ex. 34: In a single throw of two dice, find the =    
 6 6 36  36
probability that neither a doublet nor
a total of 9 will appear. Ex. 36: A card is drawn from a deck of 52
Soln: Let S be the sample space. Then, cards. Find the probability of getting
n(S) = 6 × 6 = 36 a king or a heart or a red card.
Let E = the event of getting a doublet. Soln: Let S be the sample space. Then,
And, F = the event of getting a total of 9. n(S) = number of selecting 1 card out of
Then E = {(1, 1), (2, 2), (3, 3), (4, 4), 52.
(5, 5), (6, 6)} = 52C1 = 52
Probability 683

Let E, F, G be the events of getting a king, 13 13


a heart and a red card respectively. C4 C4
P(E3) = 52 and P(E4) = 52
Th en , C4 C4
n(E) = number of ways of selecting 1 king
It is clear t hat E 1 , E 2 , E 3 and E 4 ar e
out of 4
mutually exclusive events
= 4C1 = 4
 P (getting all the 4 cards of the same
n(F) = number of ways of selecting 1 heart
suit)
out of 13
= P(E1 or E2 or E3 or E4) = P(E1) + P(E2)
= 13C1 = 13
+ P(E3) + P(E4)
n(G) = number of ways of selecting a red
card out of 26  13
C 4 
= 26C1 = 26 =  4 

52
C 4 
Clearly, (E  F); (E  G); (F  G) and
(E  F  G) are the events of getting ‘a
 13  12  11  10 4  3  2 1 
king of heart’, ‘a king of red card’, ‘a heart = 4   
and a red card’ and ‘a king of heart and  4  3  2 1 52  51  50  49 
red card’ respectively.
44
 n(E F) = 1; n(E G) = 2; n(F G) = 13 =
and n(E F G) = 1 4165

4

1 13 1
 ; Conditional Probability
Hence, P(E) = ; P(F) =
52 13 52 4
Let E and F be two events associated with a random
26 1 experiment. Then, the probability of occurrence of
P(G) =  ; E under the condition that F has already occured
52 2
and P(F)  0, is called the conditional probability
1 2 1 and it is denoted by P(E/F).
P(E  F) = ; P(E G) =  ; Thus, P(E/F) = Probability of occurrence of E given
52 52 26
that F has already occurred.

K KUNDAN
13 1 Similarly, P(F/E) = Probability of occurrence of F
P(F G) =  given that E has already occured.
52 4
Sometimes, P(E/F) is also used to denote the
1 probability of occurrence of E when F occurs.
and P(E F G) = Similarly, P(F/E) is used to denote the probability
52
 P(a king or a heart or a red card) of F when E occurs.
= P(E or F or G) = P(E  F G) Following examples will illust rat e v ar ious
= P(E) + P(F) + P(G) – P(E F) – P(F G) meanings of these notations.
– P(E G) + P(E F  G) Ex. 38: Suppose a bag contains 5 white and 4
red balls. Two balls are drawn from
 1 1 1 1 1 1 1  the bag one after the other without
=        
 13 4 2 52 4 26 52  replacement. Consider the following
events.
28 7 E = event of drawing a white ball in the
= 
52 13 first draw
Ex. 37: Four cards are drawn at a time from F = event of drawing a red ball in second
a pack of 52 playing cards. Find the draw
probability of getting all the four cards Now, P(F/E) = Probability of drawing a
of the same suit. red ball in second draw given that a white
Soln: Let S be the sample space. ball has already been drawn in the first
Then, n(S) = number of ways of selecting draw.
4 out of 52 Since 8 balls are left after drawing a white
= 52C4 ball in first draw and out of these 8 balls,
Let E1, E2, E3, E4 be the events of getting 4 balls are red, therefore,
‘all spades’, ‘all clubs’, ‘all hearts’ and ‘all 4
C1 4 1
diamonds’ respectively. P(F/E) = 8
 
Then, n(E 1) = 13 C 4 ; n(E 2 ) = 13C 4 ; n(E 3 ) C1 8 2
= 13C4 and n(E4) = 13C4 Note that P(E/F) is not meaningful in this
13 13 experiment because E cannot occur after
C4 C4 the occurrence of F.
Hence, P(E1) = 52 ; P(E2) = 52
C4 C4
684 Concept of Arithmetic

Ex. 39: Consider an experiment of throwing a Multiplication Theorems on Probability


pair of dice.
Let E and F be the events given by Let E and F be two events associated with the
E = the sum of points is 8 same random experiment. Then,
F = there is an even number on first die. P(E  F) = P(E) × P(F/E); where P(E)  0
Th en , or
E = {(2, 6), (6, 2), (3, 5), (5, 3), (4, 4)} and P(E  F) = P(F) × P(E/F); where P(F)  0
F = {(2, 1), (2, 2), (2, 3), (2, 4), (2, 5), Note: (i) By definition of conditional probability,
(2, 6), (4, 1), (4, 2), (4, 3), (4, 4), we have
(4, 5), (4, 6), (6, 1), (6, 2), (6, 3), n (E  F ) n (E  F )
(6, 4), (6, 5), (6, 6)} P(F/E) = n (E ) and P(E/F) = n (F )
E  F = {(2, 6), (6, 2), (4, 4)}
 n(E  F) = 3 (ii) Extension of Multiplication Theorem: If
A1, A2, ..., A n are n events related to a random
n (E ) 5 experiment, then P(A1 A2 A3 ... An) = P(A1) ×
 P(E) = n (S)  36 and
P(A2/A1) × P(A3/A1 A2) ... P(An/A1 A2 ... An-1);
where P(A i /A 1  A 2  .. .  A i-1 ) r epr esent s the
n (F ) 18 conditional probability of the event A i , given that
P(F) = n (S)  36 the events A1, A2, ..., Ai-1 have already happened.
Ex. 41: A bag contains 10 white and 15 black
Now, P(E/F) = Probability of occurrence
balls. Two balls are drawn in succession
of E when F occurs.
wit hout r epl acem ent . What i s the
= Probability of get ting 8 as the sum
pr obabil i t y t hat fi rst is whi t e and
when there is an even number on first
second is black?
die.
Soln: Consider the following events:
n (E  F ) 3 A = event of getting a white ball in first
=  [ n(E  F) =3]
n (E ) 5 draw
B = event of getting a black ball in second
Ex. 40: A die is thrown twice and the sum of

K KUNDAN
draw
t he num bers appear ing is obser ved
Required probability = Probability of getting
t o be 6. What i s the condit i onal
a white ball in first draw and black ball
probabi l i ty t hat the num ber 4 has
in second draw
appeared at least once?
= P(A and B) = P(A  B)
Soln: Consider the events:
= P(A) × P(B/A) ... (i)
E = number 4 appears at least once.
F = the sum of the numbers appearing is 6. n ( A ) 10 C1 10 2
Now, P(A) = n (S)  25  
Th en , C1 25 5
E = {(4, 1), (4, 2), (4, 3), (4, 4), (4, 5), (4, 6), And P(B/A) = Probability of getting a black
(6, 4), (5, 4), (3, 4), (2, 4), (1, 4)} ball in second draw when a white ball
 n(E) = 11 has already been drawn in first drawn.
and, 15
C1
15 5
F = {(1, 5), (2, 4), (3, 3), (4, 2), (5, 1)} =  
24
 n(F) = 5 C1 24 8
(E  F) = {(2, 4), (4, 2)} From (i), required probability
 n(E  F) = 2
2 5 1
The required probability = P(A B) = P(A) × P(B/A) =  
5 8 4
n (E  F ) 2 Ex. 42: Find the pr obabi l it y of drawi ng a
= P(E/F) = 
n (F ) 5 di am ond card i n each of t he t wo
Note: It is to note here that in Ex. 38 consecutive draws from a well-shuffled
only P(F/E) is meaningful whereas in Ex. pack of cards, if the card drawn is
39 P(E/F) and P(F/E) both are meaningful. not replaced after the first draw.
This is due to the reason that in Ex. 38 Soln: Let E = event of drawing a diamond card
events E and F are subsets of two different in the first draw
sample spaces and they occur one after and F = event of dr awing a diamond
the other whereas in Ex. 39, events E card in the second draw
and F are subsets of the same sample space Then, required probability
and they can occur together also. = P(E F) = P(E) × P(F/E) ... (i)
13
C1 13 1
 
 P(E) = 52
C1 52 4
Probability 685

After drawing a diamond card in first draw After drawing 3 white balls in first draw
51 cards are left out of which 12 cards 5 + 8 - 3 =) 10 balls are left in the bag, out
are diamond cards. of which 8 are black balls
 P(F/E) = Pr obabilit y of drawing a 8
diamond car d in second draw when a C3 56 7
 P(F/E) = 10
 
diamond card has already been drawn in C3 120 15
first draw
Hence, required probability
12
C112 4 = P(E  F) = P(E) × P(F/E)
= 51 

C1 51 17 5 7 7
Now, from (i), =  
143 15 429
1 4 1 Ex. 45: Two balls drawn from an urn containing
P(E  F) = P(E) × P(F/E) =  
4 17 17 2 white, 3 red and 4 black balls one by
Ex. 43: A bag contains 19 tickets, numbered one without replacement. What is the
from 1 to 19. A ticket is drawn and probability that at least one ball is
then another ticket is drawn without red?
replacement. Find the probability that Soln: Let E = event of not getting a red ball
both tickets will show even numbers. in first draw
Soln: Let E = event of dr awing an even And F = event of not getting a red ball
numbered ticket in first draw in second draw
And, Then, required probability
F = event of drawing an even numbered = Probability that at least one ball is red
ticket in the second draw. = 1 – Probability that none is red
Th en , = 1 - P(E and F)
Required probability = 1 - P(E  F)
= P(E  F) = P(E) × P(F/E) ... (i) = 1 - P(E) × P(F/E) ... (i)
Since there are 19 tickets, numbered 1 to Now, P(E) = Probability of not getting a
red ball in first draw

K KUNDAN
19, in the bag but of which 9 are even
numbers viz 2, 4, 6, 8, 10, 12, 14, 16, 18. = Probability of getting an other colour
(white or black) ball in first draw
9
Therefore, P(E) = 6 2
19 
Since the ticket drawn in the first draw =
9 3
is not replaced, therefore, second ticket When another colour ball is drawn in first
drawn is from the remaining 18 tickets, draw there are 5 other colour (white or
out of which 8 are even numbered. black) balls and 3 red balls, out of which
8 4 one other colour ball can be drawn in
 P(F/E) =  (5C1 =) 5 ways.
18 9
Hence, required probability 5
= P(E  F) = P(E) × P(F/E)  P(F/E) =
8
9 4 4 From (i), required probability
=   = 1 - P(E) × P(F/E)
19 9 19
Ex. 44: An urn contains 5 white and 8 black 2 5 7
balls. Two successive drawings of three = 1  
3 8 12
balls at a time are made such that the Ex. 46: If A and B are two events such that
balls are not replaced before the second P(A) = 0.4, P(B) = 0.8 and P(B/A) = 0.6,
draw. Find the probability that the find P(A/B) and P(A  B).
fir st dr aw gi v es 3 whit e bal ls and
second draw gives 3 black balls. P(A  B)
Soln: Let E = event of drawing 3 white balls Soln: We know that P(B/A) = P(A)
in the first draw
 P(A  B) = P(A) × P(B/A)
And F = event of drawing 3 black balls
= (0.4 × 0.6) = 0.24
in the second draw
Required probability P(A  B) 0.24
= P(E  F) = P(E) × P(F/E) ... (i) So, P(A/B) = P(B) = 0.8 = 0.3
5
C3 10 5 And, P(A  B) = P(A) + P(B) - P(A  B)
Now, P(E) = 13
 
C3 286 143 = (0.4 + 0.8 - 0.24) = 0.96
686 Concept of Arithmetic

Ex. 47: A die is rolled. If the outcome is an odd Ex. 50: A die is thrown twice and the sum of the
number, what is the probability that it numbers appearing is observed to be 7.
is prime? What is the conditional probability that
Soln: When a die is rolled the sample space is the number 2 has appeared at least
S = {1, 2, 3, 4, 5, 6} once?
Let E = event of getting an odd number Soln: Let E = the event of getting the sum 7.
And, F = event of getting a prime number and, F = the event of getting at least one 2.
Then, E = {1, 3, 5}; F = {2, 3, 5} and Then, E = {(1, 6), (2, 5), (3, 4), (4, 3),
E  F = {3, 5} (5, 2), (6, 1)}
and, F = {(1, 2), (2, 2), (3, 2), (4, 2),
3 1 3 1
 P(E) =  , P(F) =  and (5, 2), (6, 2), (2, 1), (2, 3),
6 2 6 2 (2, 4), (2, 5), (2, 6)}
Then, E  F = {(2, 5), (5, 2)}
2 1
P(E  F) =  Now, we have to find P(F/E)
6 3
P(E  F) 2 1
Suppose E has already occurred and then  P(F/E) =  
P(E) 6 3
F occurs.
Then, we have to find P(F/E) Ex. 51: Two unbiased dice are thrown. Find the
probabi l i ty t hat the sum is 8 or
P(E  F)  1 2  2 greater, if 4 appears on the first die.
Now, P(F/E) =   
P(E) 3 1 3 Soln: Let E = the event of getting 4 on the first
die
Ex. 48: Three fair coins are tossed. Find the
and, F = the event of getting the sum 8
probability that they are all tails, if
or greater
one of the coins shows a tail.
 E = {(4, 1), (4, 2), (4, 3), (4, 4), (4, 5),
Soln: Here S = {HHT, HTH, THH, HTT, THT,
(4, 6)
TTH, HHH, TTT}
and, F = {(4, 4), (4, 5), (4, 6), (5, 4), (5, 5),
Let E = event that one of the coins shows
(5, 6), (6, 2), (2, 6), (3, 5), (3, 6), (5, 3), (6,
a tail
3), (6, 4), (6, 5), (6, 6)}

K KUNDAN
and, F = event that they are all tails
 E  F = {(4, 4), (4, 5), (4, 6)}
Then, E = {HHT, HTH, THH, HTT, THT,
 required probability
TTH, TTT}
and F = {TTT} P(E  F) 3 1
= P(F/E) =  
Clearly, E  F = {TTT} P(E) 6 2
7 1 1 Ex. 52: I n a cl ass 40% st udents r ead
 P(E) = , P(F) = and P(E  F) = mathemat ics, 25% biol ogy and 15%
8 8 8
bot h mathem atics and bi ology. O ne
P(E  F)  1 8  1 student is selected at random. Find the
So, P(F/E) =   
P(E) 8 7 7 probability that
Ex. 49: A coin is tossed t wice and the four (i) he reads mathematics, if it is known
possible outcomes are assumed to be that he reads biology;
equally likely. If E is the event ‘both (ii) he reads biology, if it is known that
head and tail have appeared’ and F be he reads mathematics.
the event, ‘at most one tail is observed’, Soln: Let E = the event of reading mathematics
find P(E), P(F), P(E/F) and P(F/E). and F = the event of reading biology
Soln: Here S = {HH, HT, TH, TT} 40 2 25 1
E = {HT, TH} and F = {HH, HT, TH} Then P(E) =  ; P(F) = 
100 5 100 4
 E  F = {HT, TH}
15 3
2 1 3 and P(E  F) = 
So, P(E) =  ; P(F) = and 100 20
4 2 4
P(E  F)  3 4  3
 (i) P(E/F) =   
2 1 P(F)  20 1  5
P(E  F) = 
4 2
P(E  F)  3 5  3
P(E  F)  1 4  2 (ii) P(F/E) =   
   P(E)  20 2  8
 P(E/F) = P(F) 2 3 3
Ex. 53: A bag contains 5 white, 7 red and 8
black balls. If four balls are drawn one
P(E  F)  1 2 
And, P(F/E) =     1 by one without replacement, what is
P(E) 2 1 the probability that all are white?
Probability 687

Soln: Let A, B, C, D denote events of getting a remaining cards is 49 and there remains
white ball in first, second, third and fourth only one ace.
draw respectively.
1
Then, required probability  on the fourth draw, P(an ace) =
= P(A  B C D) 49
= P(A) × P(B/A) × P(C/A  B) Hence,
× P(D/A B C) ... (i)
 1 1 1 1  1
Now, P(all aces) =     
P(A) = Probabilit y of dr awing a whit e  13 17 25 49  270725
Ex. 55: A box contains 3 red and 5 blue balls.
5 1 Two balls are drawn one at a time at
ball in first draw = 
20 4 random without replacement. Find the
When a white ball is drawn in the first probability of getting 1 red and 1 blue
draw there are 19 balls left in the bag, out ball.
of which 4 are white Soln: 1 red and 1 blue ball can be obtained in 2
ways; either (1st red ball and 2nd blue
4
 P(B/A) = ball) or (1st blue ball and 2nd red ball).
19 Case I: Initially, there are 8 balls in all,
Since t he ball drawn is not replaced, out of which 3 are red.
therefore, after drawing a white ball in
3
second draw there are 18 balls left in the C1 3
 P(1st ball red) = 8

bag, out of which 3 are white. C1 8
3 1 Since replacement is not being made, after
 P(C/A  B) = 
18 6 the first draw, we have 7 balls in all, out
After drawing a white ball in third draw of which 5 are blue.
there are 17 balls left in the bag, out of 5
C1 5
which 2 are white.  P(2nd ball blue) = 7

7

K KUNDAN
C1
2
 P(D/A B C) = Hence, P(1st ball red and 2nd ball blue)
17
Hence, required probability  3 5  15
=   
= P(A  B C D)  8 7  56
= P(A) × P(B/A) × P(C/A  B) Case II: Initially there are 8 balls in all
× P(D/A B C) out of which 5 are blue.
1 4 1 2  1 5
=       P(1st ball blue) =
 4 19 6 17  969 8
Ex. 54: Four cards are drawn successively one Since replacement is not being made, after
after the other from a well-shuffled the first draw, we have 7 balls in all, out
pack of 52 cards. If the cards are not of which 3 are red
replaced, find the probability that all
of them are aces. 3
 P(2nd ball red) =
7
4 1
Soln: On first draw, P(an ace) =  Hence, P(1st ball blue and 2nd ball red)
52 13
Since the card drawn is not replaced, the 5 3 15
=  
second card is drawn from the remaining 8 7 56
51 cards. Also, one ace has already been Now, the above two cases are mutually
drawn, there are now three remaining exclusive
aces.  P(1 red ball and 1 blue ball)
3 1  15 15  15
 on second draw, P(an ace) =  =   
51 17  56 56  28
Now, there remain 50 cards containing 2
aces. Independent Events
2 1 Events are said to be independent, if the occurrence
 on third draw, P(an ace) =  or non-occur r ence of one does not af f ect the
50 25
probability of the occurrence or non-occurrence of
Af ter the thir d dr aw, t he number of
the other.
688 Concept of Arithmetic

Suppose a bag contains 6 white and 3 red balls. Ex. 57: A can solve 90% of the problems given
Two balls are drawn from the bag one after the in a book and B can solve 70%. What
other. Consider the events: is the probability that at least one of
E = drawing a white ball in first draw them will solve a problem, selected at
F = drawing a red ball in second draw random from the book?
If the ball drawn in the first draw is not replaced Soln: Let E = t he ev ent t hat A solv es t he
back in the bag, then events E and F are dependent pr oblem
events because P(F) is incr eased or decreased And, let F = the ev ent t hat B solves
according as the first draw results as a white or the problem
red ball. If the ball drawn in first draw is replaced Clearly, E and F are independent events
back in the bag, then E and F are independent 90 9
events because P(F) remains same whether we get Now, P(E) =  and
100 10
a white ball or a red ball in first draw ie
70 7
P(F) = P(F/E) and P(F/ E ) P(F) = 
100 10
It is evident from the above discussion that if E
and F are two independent events associated with By addit ion t heor em f or independent
a random experiment, then events, we have
P(E/F) = P(E) and P(F/E) = P(F) and vice-versa. P(E  F) = 1  P ( E )  P ( F ) ... (i)

Multiplication Theorems for Independent  9  1


 P(E )  1   and
Events  10  10
 7  3
Theorem-I: If E and F are independent events P(F )  1  
associated with a random experiment, then  10  10
P(E  F) = P(E) × P(F) or, P(E or F) = P(E  F)
Theorem-II: If E1, E2, E3, ... En are independent  1 3  97
events associated with a random experiment, then = 1    = 0.97
 10 10  100
P(E1  E2  E3  ...  En) = P(E1) × P(E2) × ... ×
Alternative Method:

K KUNDAN
P(En)
From the above, we have
Addition Theorem For Independent 9 7
Events P(E) = and P(F) =
10 10
Theorem: If E1, E2, ..., En are n independent events  P(E  F) = P(E) × P(F)
associated with a random experiment, then  9 7  63
P(E1  E2 E3 ... En) =   
 10 100  100
= 1  P(E1 )  P(E2 )  ...  P(En ) [ E and F are independent events]
Ex. 56: An unbiased die is tossed twice. Find Hence, P(E or F) = P(E  F)
the probability of getting a 4, 5 or 6 on = P(E) + P(F) – P(E  F)
the first toss and a 1, 2, 3 or 4 on the
 9 7 63  97
second toss. =     = 0.97
Soln: Let E = event of getting a 4, 5 or 6 on  10 10 100  100
the first toss Ex. 58: The probability that A hits a target is
and, F = event of getting a 1, 2, 3 or 4 1
on the second toss.   and the probability that B hits it
3
Then, clearly, E and F are independent
2 
events. is   . What is the probability that
Sample space in each case is 5 
S = {1, 2, 3, 4, 5, 6} the target will be hit, if each one of A
and B shoots at the target?
3 1 4 2 Soln: Let E = the event that A hits the target
P(E) =  and P(F) = 
6 2 6 3 and F = the event that B hits the target
Hence, P(getting a 4, 5 or 6 on first toss 1 2
and a 1, 2, 3 or 4 on 2nd toss) As given, we have P(E) = and P(F) =
3 5
= P(E  F)
Clearly, E and F are independent events.
1 2 1
= P(E) × F(F) =      P(target is hit) = P(A hits or B hits)
2 3 3 = P(E  F)
(By mult iplication t heor em f or
independent events) = 1  P(E)  P(F)
Probability 689

(By the addition theorem for independent


= P(E) × P( F ) + P(F) × P( E )
events)
 1 2  1 4 1 6  10 2
or, P( E ) = 1 – P(E) = 1    and =      
 3 3  7 5 5 7  35 7
 2 3 (iii) P(none of them will be selected)
P( F ) = 1 – P(F) = 1    = P[(not-E) and (not-F)]
 5 5
= P[(E  F )  P(E)  P(F )
2 3 2 3
 P(target is hit) = 1      1   [ E and F being independent,
3 5 5 5
Alternative Method: E and F are independent]
1 2 2  6 4  24
P(E  F) = P(E) × P(F) =  
3 5 15 =   
 7 5  35
(By multiplication theorem for independent (iv) P(at least one of them will be selected)
events) = 1 – P(none will be selected)
 P(target is hit) = P(A hits or B hits)
= P(E  F) = P(E) + P(F) - P(E  F)  24  11
= 1  
1 2 2  9 3  35  35
=     
 3 3 15  15 5 Ex. 60: A speaks truth in 75% and B in 80% of
Ex. 59: A husband and a wife appear in an the cases. In what percentage of cases
interview for two vacancies in the same are they likely to contradict each other
post . The pr obabil i t y of husband’ s narrating the same incident?
1 [BSRB Chennai PO–2005; ATMA–2006]
sel ect i on   and t hat of wife’ s Soln: Let E = the event that A speaks the truth.
7 
And let F = t he ev ent t hat B speaks
1  the truth.
select ion is   . What i s t he
5 

K KUNDAN
probability that Then, E = the event that A tells a lie,
(i) both of them will be selected? and F = the event that B tells a lie
(ii) only one of them will be selected? Clearly, E and F are independent events.
(iii) none of them will be selected?
(iv) at l east one of t hem wi l l be Hence, E and F as well as E and F are
selected? independent
[MAT–2005]
75 3
Soln: Let E = the event that the husband is Now, P(E) =  and
selected 100 4
and, F = t he event that t he wif e is
80 4
select ed P(F) = 
Clearly, E and F are independent events. 100 5
 3 1
1 1  P( E ) = 1    and
Now, P(E) = and P(F) =  4 4
7 5
1 6  4 1
 P( F ) = 1   
 P( E ) = 1    and  5 5
 7 7
 P(A and B contradict each other)
 1 4
P( F ) = 1    = P[(A speaks the truth and B tells a
 5 5 lie) or (A tells a lie and B speaks the
(i) P(both of them will be selected) truth)]
= P(E and F) = P(E  F) = P(E) × P(F)
= P[(E  F )  (E  F )]
1 1 1
=  
7 5 35 = P[(E  F )  (E  F )]
(ii) P(only one of them will be selected)
= P(E and not-F) or P(F and not-E) [ P[(E  F )  (E  F )  ]
= P[(E  F )  (F  E)] = P(E) × P( F ) + P( E ) × P(F)
= P[(E  F )  (F  E)] 3 1 1 4 7
=     
 4 5 4 5  20
[ P[(E  F )  (F  E  )]
690 Concept of Arithmetic

Hence, the percentage of t he cases in


 1 2
which A and B contradict each other  P( A ) = 1   
 3 3
 7 
=   100 % = 35%
 20   2 5
Ex. 61: A problem is given to three students P( B ) = 1    and
 7 7
1 1
whose chances of solving it are ,  3 5
2 3
P( C ) = 1   
1  8 8
and r especti v el y. What i s t he
4  The probability that exactly one of them
probability that the problem will be will solve it
solved? = P{[A  (not-B)  (not-C)] or [(not-A) 
Soln: Let E1, E2, E3 be the respective events of  B (not-C) or [not-A) (not-B) C]}
solving the problem and, E1 , E2 , E3 be = P{(A B  C )  ( A B C )  ( A  B C)}
the respective events of not solving the
problem. = P{(A B  C ) + ( A B C ) + ( A  B C)}
Then, E 1 , E 2 and E 3 are independent
[  (A B  C )  ( A B C ) 
events.
( A  B C) = ]
 E1 , E2 and E3 are independent events.

1 1 1 = P(A) × P( B ) × P( C ) + P( A ) × P(B) × P( C )
Now, P(E1) = ; P(E2) = and P(E3) =
2 3 4 + P( A ) × P( B ) × P(C)

 1 1 1 5 5 2 2 5 2 5 3
 P( E1 ) = 1    =          
 2 2 3 7 8 3 7 8 3 7 8

 1 2  25 5 5  75 25
P( E2 ) = 1    =     

K KUNDAN
and
 3 3  168 42 28  168 56
Ex. 63: Three groups of children contain 3 girls
 1 3 and 1 boy; 2 girls and 2 boys; 1 girl
P( E3 ) = 1   
 4 4 and 3 boys respectively. One child is
 P(none solves the problem) selected at random from each group.
= P[(not E1) and (not E2) and (not E3)] Find the chance that the three selected
comprise 1 girl and 2 boys.
= P( E1  E2  E3 ) = P( E1 ) × ( E2 ) × ( E3 )
Soln: Let B1, B2, B3 be the events of selecting a
[ E1 , E2 and E3 are independent] boy from first, second and third group
respectively. And let G1, G2 and G3 be the
1 2 3 1 events of selecting a girl from first, second
=     and third group respectively.
2 3 4 4
1
Hence, P(the problem will be solved) C1 1
Then, P(B1) = 
= 1 – P(none solves the problem) 4
C1 4
 1 3
= 1    2
C1 2 1
 4 4  
P(B2) = 4 4 2
Ex. 62: The probabilities of A, B, C solving a C1
1 2 3
problem are , and respectively. 3
3 7 8 C1 3
P(B3) = 4

If all the three try to solve the problem C1 4
simultaneously, find the probability
3
that exactly one of them will solve it. C1 3
P(G1) = 4

Soln: Let A, B and C be the events of solving C1 4
the problem A, B and C respectively.
2
C1 2 1
Then, A , B , C are the respective events P(G2) = 4
 
C1 4 2
of not solving the problem by them
1
1 2 3 C1 1
Now, P(A) = ; P(B) = and P(C) = P(G3) = 4

3 7 8 C1 4
Probability 691

 P(1 girl and 2 boys) 3


C1 3
= P[(G1  B2  B3) or (B1 G2  B3) P(red from second bag) = 10

10 and
or, (B1  B2  G3)] C1
= P(G1  B2 B3) + P(B1  G2  B3) 7
C1 7
+ P(B1  B2  G3) P(black from second bag) = 10

= P(G1) × P(B2) × P(B3) + P(B1) × P(G2) C1 10
× P(B3) + P(B1) × P(B2) × P(G3)  P(two black balls and one red ball)
3 1 3 1 1 3 1 1 1 = P[(1 red from first and 2 black from
=           second) or (1 black from first and 1 red
4 3 4 4 2 4 4 2 4
and 1 black from second)]
 9 3 1  13 = P(1 red from first and 2 black from
=    
 32 32 32  32 second) + P(1 black from first and 1
Ex. 64: A bag contains 4 white and 2 black red and 1 black from second)
balls. Another contains 3 white and 5 = P[(1 red from first) × P(2 black from
black balls. If one ball is drawn from second)] + [P(1 black from first) × P(1
each bag, find the probability that red and 1 black from second)]
(i) both are white; = P[(1 red from first) × P(2 black from
(ii) both are black; second)] + [P(1 black from first) × P(1
(iii) one is white and one is black. red and 1 black from second)]
Soln: (i) P(both balls are white) 7
= P[(white ball from first bag) and (white 4 C2 5 3 C1  7 C1
=  10
  10
ball from second bag)] 9 C2 9 C2
= P(White ball from first bag) × P (White
 4 7 6   5 3 7 
ball from second bag) =      2
 9 10  9   9 10  9 
4 3 1
=     28 7  63 7
6 8 4    
=
(ii) P(both balls are black)  135 27  135 15

K KUNDAN
= P[(black ball from first bag) and (black Ex. 66: A bag contains 4 red and 3 black balls.
ball from second bag)] A second bag contains 2 red and 4 black
= P(black ball from first bag) × P(black ball balls. One bag is selected at random.
from second bag) Fr om t he sel ected bag, one bal l i s
2 5 5 drawn. Find the probability that the
=    ball drawn is red.
 6 8  24
1
(iii) P(one white ball and one black ball) Soln: Probability of choosing first bag =
= P[(black ball from first bag and white 2
from second) (Choosing 1 out of 2).
or, (whit e f r om f ir st and black f rom Probability of drawing a red ball from this
second)] 4
C1 4
= P(black f rom fir st and whit e fr om bag = 7

C1 7
second) + P(white from first and black
from second) P(Choosing first bag and drawing a red
 2 3   4 5   1 5  13 1 4 2
=          ball from it) =    
 6 8   6 8   8 12  24 2 7 7
Ex. 65: A bag contains 4 red and 5 black balls
1
and bag B contains 3 red and 7 black Probability of choosing second bag =
balls. One ball is drawn from bag A 2
and t wo from bag B. Find the Probability of drawing a red ball from this
probability that out of 3 balls, two are 2
black and one is red. C1 2 1
bag = 6
 
[MAT–1998] C1 6 3
Soln: Clearly, we have:
P(Choosing second bag and drawing a red
4
C1 4
P(red from first bag) = 9
 1 1 1
C1 9 ball from it) =    
2 3 6
5
C1 5
P(black from first bag) =  2 1 19
9
C1 9  P(a red ball) =  
7 6 42
692 Concept of Arithmetic

Ex. 67: A, B and C shoot to hit a target. If A And, let F = the event of drawing 3 black
hits the target 4 times in 5 trials; B balls in second draw.
hits it 3 times in 4 trials and C hits it When the balls are replaced before the
2 t i m es i n 3 tr i als; what i s the second trial, E and F are independent
probability that the target is hit by at events.
least 2 persons?  Required probability
[MAT–2009] = P(E  F) = P(E) × P(F)
Soln: Let A, B and C be the events that A hits 5 8
the target, B hits the target and C hits the C3 C3 10 56
= 13
 13
 
target respectively. C3 C3 286 286
Th en ,
140
4 3 2 =
P(A) = , P(B) = , P(C) = 20449
5 4 3
Note: Also see the Ex. 44. Ex. 44 is an
 4 1  3 1 example of condit ional pr obabilit y,
P( A ) = 1    , P( B ) = 1    and whereas the above example is the case of
 5 5  4 4
independent events.
 2 1
P( C ) = 1    The Law of Total Probability
 3 3
Case I: P(A, B and C all hit the target) Let S be the sample space and E1, E2, ..., En be n
= P(A  B  C) mut ually exclusiv e and exhaustiv e ev ent s
= P(A) × P(B) × P(C) associated with a random experiment. If E is any
4 3 2 2 event which occurs with E1 or E2 or E3 or .... or En,
=     then
5 4 3 5
P(E) = P(E1) P(E/E1) + P(E2) P(E/E2) + ... +
Case II: P(A and B hit but not-C) P(En) P(E/En)
= P(A B  C ) The law of total probability as stated above say
that if an event E can occur in n mutually exclusive

K KUNDAN
= P(A) × P(B) × P ( C ) ways, then the probability of occurrence of E is
the sum of the probabilities of various ways.
4 3 1 1
=     Ex. 69: A bag contains 4 red and 3 black balls.
5 4 3 5 A second bag contains 2 red and 4 black
Case III: P(A and C hit but not-B) balls. One bag is selected at random.
Fr om t he sel ected bag, one bal l i s
= P(A  C  B )
drawn. Find the probability that the
= P(A) × P(C) × P( B ) ball drawn is red.
Soln: A red ball can be drawn in two mutually
4 2 1 2
exclusive ways:
=    
 5 3 4  15 (I) Selecting bag I and then drawing a
Case IV: P(B and C hit but not-A) red ball from it.
(II) Selecting bag II and then drawing a
= P( B  C  A )
red ball from it.
= P(B) × P(C) × P( A ) Let E1, E2 and E denote the events defined
as follows:
3 2 1 1
=     E1 = Selecting bag I
 4 3 5  10 E2 = Selecting bag II
All t he abov e cases being mut ually E = Drawing a red ball
exclusive, we have the required probability Since one of the two bags is select ed
2 1 2 1  5 randomly,
=     
 5 5 15 10  6 1 1
Ex. 68: An urn contains 5 white and 8 black therefore P(E1) = and P(E2) =
balls. Two successive draws of three 2 2
balls at a time are made such that the Now, P(E/E1) = Probability of drawing a
balls are replaced before the second r ed ball when t he f ir st bag has been
trial. Find the probability that the first chosen.
drawi ng wil l gi v e 3 whi t e and t he 4
second 3 black balls. = (Since first bag contains 4 red and
7
Soln: Let E = the ev ent of drawing 3 white
balls in first draw 3 black balls)
Probability 693

and P(E/E 2) = Probability of drawing a Ex. 71: In a bolt factory, machines A, B and C
red ball when t he second manufacture respectively 25%, 35% and
bag has been selected. 40% of the total bolts. Of their output
2 5%, 4% and 2% ar e respect i vel y
= (Since the second bag contains 2 defect i ve bol ts. A bol t i s dr awn at
6
red and 4 black balls) random from the product. If the bolt
Using the law of total probability, we have drawn is found to be defective, what is
P(red ball) = P(E) the probability that it is manufactured
= P(E1) P(E/E1) + P(E2) P(E/E2) by the machine B?
Soln: Let E1, E2, E3 and E be the events defined
 1 4 1 2  19 as follows:
=     
 2 7 2 6  42 E1 = the bolt is manufactured by machine A
Ex. 70: In a bolt factory, machines A, B and C E2 = the bolt is manufactured by machine B
manufacture respectively 25%, 35% and E3 = the bolt is manufactured by machine C
40% of the total bolts. Of their output E = the bolt is defective.
5%, 4% and 2% ar e respect i vel y Th en ,
defect i ve bol ts. A bol t i s dr awn at P(E 1) = Probability that the bolt drawn is
random from the product. What is the manufactured by machine A
pr obabil i t y t hat t he bol t dr awn i s 25
defective? =
100
Soln: Let E1, E2, E3 and E be the events defined
as follows: P(E 2) = Probability that the bolt drawn is
E1 = the bolt is manufactured by machine A manufactured by machine B
E2 = the bolt is manufactured by machine B 35
E3 = the bolt is manufactured by machine C =
100
E = the bolt is defective. Then,
P(E 3) = Probability that the bolt drawn is
25 1 35 40 manufactured by machine C
 ; P(E ) =

K KUNDAN
P(E1) = 2
; P(E3) =
100 4 100 100 40
P(E/E1) = Probability that the bolt drawn =
100
is defective given the condition P(E/E1) = Probability that the bolt drawn
that it is manufactured by is def ect ive giv en t hat is
machine A manufactured by machine A
5 5
= =
100 100
4 4
Similarly, P(E/E2) = and Similarly, P(E/E2) = and
100 100
2
2 P(E/E3) =
P(E/E3) = 100
100
Using the law of total probability, we have Now, required probability = Probability that
P(E) = P(E 1 ) P(E/E 1) + P(E 2) P(E/E 2 ) + the bolt is manufactured by machine B
P(E3) P(E/E3) given that the bolt drawn is defective

 25 5   35 4   40 2  P(E 2 ) P(E/E 2 )
=        or, P(E2/E) =
 100 100   100 100   100 100  P(E1 ) P(E/E1 )  P(E 2 ) P(E/E 2 )
 P(E 3 ) P(E/E 3 )
= 0.0345

Baye’s Rule 35

4
Let S be the sample space and E1, E2, ..., En be n 100 100
= 25 5 35 4 40 2
mut ually exclusiv e and exhaustiv e ev ent s     
associated with a random experiment. If E is any 100 100 100 100 100 100
event which occurs with E1 or E2 or, ... or En, then
140 140 28
=  
P(Ei ) P(E/E i ) 125  140  80 345 69
P(Ei/E) = n ; where i = 1, 2, ..., n


i n
P(Ei ) P(E/E i )
694 Concept of Arithmetic

Ex. 72: Three boxes contain 6 red, 4 black; 4 Ex. 73: A dice is thrown 6 times. If “getting an
red, 6 black and 5 red, 5 black balls odd number” is a “success”, then find
r espect iv ely. O ne of t hese boxes i s the probability of 5 “successes”.
selected at random and a ball is drawn [MAT–1998]
from it. If the ball drawn is red, then Soln: In this question, a random experiment is
find the probability that it is drawn performed 6 times.
from the first box.  n = 6
[MAT–1998] p = probability of getting an odd number
Soln: Let E1, E2, E3 and E be the events defined
3 1
as follows: in one throw = 
E1 = box first is chosen 6 2
E2 = box second is chosen [Since there are 3 odd numbers (1, 3, 5)
E3 = box third is chosen and out of 6 numbers (1, 2, 3, 4, 5, 6) on the
E = ball drawn is red dice.]
Since there are three boxes and one of q = probability of non-occurrence of an
the three boxes is chosen at r andom,  1 1
therefore odd number in one throw = 1   
 2 2
1 Since, here, “getting an odd number” is
P(E1) = P(E2) = P(E3) = called a “success”
3
Hence r = 5 (required number of getting
If E1 has already occurred, then box first successes)
has been chosen which contains 6 red  Required probability = nCr pr qn-r
and 4 black balls. The pr obabilit y of
5 6 5 5
6 6 1 1 1 1 3
= C5      6    
drawing a red ball from it is .  2  2 2 2 32
10
Ex. 74: Suppose 6 coi ns ar e fl i pped
6 sim ul taneousl y. Then fi nd t he
Hence, P(E/E1) =
10 probability of getting at least one tail.

K KUNDANSimilarly, P(E/E2) =

P(E/E3) =
5
10
4
10
and

We are required to find P(E1/E) ie given


that the ball drawn is red, what is the
probability that it is drawn from the first
Soln: Let X denote the number of tails in 6
throws of a coin. Then the probability of
getting r tails is given by

P(X = r) =

=
6

6
r
1 1
Cr    
 2  2

1
Cr  
6
6 r
; r = 0, 1, 2, ..., 6

box. 2
By Bay’s rule, Hence, required probability
= P(X  1) = 1 - P(X = 0)
P(E1 ) P(E/E1 )
P(E1/E) = 1 1
6
63
P(E1 ) P(E/E1 )  P(E 2 ) P(E/E 2 ) 6
= 1  C0    1  
 P(E 3 ) P(E/E 3 ) 2 64 64
Ex. 75: A coin is tossed 5 times. What is the
1 6 probability that head appears an odd

3 10 6 2 number of times?
=  
1 6 1 4 1 5 15 5 Soln: We know that if a coin is tossed n times,
    
3 10 3 10 3 10 then the probability that head will turn
1
Binomial Probability up an odd number of times is
2
. Let us
If a random experiment is performed ‘n’ times under see how?
similar condit ions, then the pr obabilit y of t he Let X denote the number of heads in n
occurrence of the event E exactly ‘r’ times in ‘n’ trials trials.
is P(X = r) and P(X = r) = nCr pr qn-r; where r = 0, 1, 2, Th en ,
3, ... n; p = the probability of occurrence of event E
r n r n
in one trial and q = (1 - p) = probability of non- n 1 1 1
P(X = r) = Cr      n Cr  
occurrence of event E in one trial (ie p + q = 1). 2 2 2
Probability 695

Thus, the required probability q = probability that a student is not a


= P(X = 1) + P(X = 3) + P(X = 5) + ... 1
swimmer =
n n n 5
n 1 1 1
= C1   n C3    n C 5    ... r = 4
2 2 2  required probability = nCr pr qn-r
n 4 5 4 4
1 n 5 4 1 4 1
n n
=   ( C1  C3  C5  ...) = C4      5 C4    
 2 5 5 5 5
Ex. 78: 100 i dent i cal coi ns, each wi t h
1 1
= n
(2n 1 )  21  probability, p, of showing up heads are
2 2 t ossed once. I f 0 < p < 1, and t he
[  nC1 + nC3 + nC5 + ... = 2n-1] probabili ty of heads showing on 50
coins is equal to that of heads showing
1 on 51 coins, then find the value of p.
 required probability =
2 [MAT–1999]
Ex. 76: What is the probability of getting at Soln: Let X denote the number of coins showing
least 6 heads if eight coins are tossed heads up.
simultaneously? Then, the number of random experiments
[GGSIU–2002] (n) = 100 and the probability of occurrence
Soln: Let X denote the number of heads in 8 of event is p.
throws of a coin. Then the probability of Now, according to the question,
getting r heads is given by P(X = 51) = P(X = 50)
r 8 r
or, 100C51 p51 q100 – 51 = 100C50 p50 q100 – 50;
8 1 1 (where q = 1 – p)
P(X = r) = Cr     ; r = 0, 1, 2, 3, ..., 8
 2  2 or, 100C51 p51 q49 = 100C50 p50 q50
100
8 p C50 51
8 1  100

q 50

K KUNDAN
= Cr   C51
2
Required probability p 51
= P(X  6) or, 1  p  50
= P(X = 6) + P(X = 7) + P(X = 8)
1  p 50
8 or, 
1 8 8 8 p 51
=   ( C6  C 7  C8 )
 2 1 50 101
or, p  51  1  51
1  8! 8! 8! 
=     51
256  (8  6)! 6! (8  7)! 7! (8  8 )! 8!   p =
101
1 8 7  37 Ex. 79: The probability that a marksman will
=   8  1 
256  2  256 1
hit a target is given as . Then find
Ex. 77: The probability that a student is not a 5
the probability that at least one hit in
1 10 shots.
swimmer is . Then find the probability Soln: Let X denote the number of shots in which
5
t hat out of t he 5 st udent s 4 ar e a marksman hit a target in 10 shots. Then
swimm er s. the probability of r hits is given by P(X = r)
[MAT– 2002] r 10 r
Soln: There are 5 students. Hence the random 10 1 4
= Cr    
experiment is performed 5 times. 5 5
 n = 5 Thus, the required probability
p = probability that a student is a swimmer = P(X  1) = 1 - P(X = 0)
0 10 10
 1 4 10 1 4  4
= 1    = 1  C0      1  
 5 5 5 5 5
Ratio & Proportion and k-Method 359

Chapter-17

Ratio & Proportion and k-Method

Important Definitions and Related Concepts


1. Some Useful Results on Proportion Subtracting one from each side, we have,

If four quantities a, b, c and d are in proportion, then a c a b c d


1  1  
we can see that the following results are true: b d b d
( i ) Invertendo  a – b : b :: c – d : d.
If four quantities be in proportion they keep in This operation is called Dividendo.
proportion even when they are taken inversely. (v) Componendo and Dividendo
If a : b :: c : d, then b : a :: d : c
When four quantities are in proportion, the sum
a c of the first and second is to their difference as
Since  dividing unity by each of these equal
b d the sum of third and fourth is to their difference.
sides, we have, If a : b :: c : d, then a + b : a – b :: c + d : c – d

a c a c a b c d
Since,  ,   ....(i)

K
1: = 1 : b d b d
b d
[By Componendo]
b d
   b : a :: d : c a b c d
a c And  ....(ii)
b d
This result is called Invertendo.
[By Dividendo]
( i i) Alternendo
If four quantities be proportionals, they remain a b c d
Dividing (i) by (ii), we get 
proportionals when they are taken alternately. a b c d
If a : b :: c : d, then a : c :: b : d This operation is known as Componendo and
Dividendo.
a c b
Since  , multiplying both sides by , we
b d c a c a b c d
Also, if  , then 
get, b d a b c d

KUNDAN
This oper at ion is known as Dividendo and
a b c b a b
     Componendo.
b c d c c d
 a : c :: b : d. This result is called Alternendo. 2. The k-Method
( i ii ) Componendo This method requires that each of the given ratios be
When four quantities are in proportion, the first put equal to constant k, the values of the numerators
together with the second is to second as the third (antecedents) found in terms of k and denominators
together with the fourth is to the fourth. (consequent s) and such v alues as ar e obt ained
If a : b :: c : d, then a + b : b :: c + d : d substituted in the two sides of the equality to be proved.
As the method becomes almost mechanical after a
a c little practice, the students will find it very convenient
Given, 
b d and helpful, though not much instructive in solving
Adding one to each side, we have, problems concerning equal ratios.
Note:
a c a b c d ( i ) Ther e is simple method popular ly known as
1  1  
b d b d ‘k-method’ to solve problems on equal ratios.
 a + b : b :: c + d : d. ( i i) In this method, we assume each of the given ratios
This operation is said to be Componendo. equal to k.
( iv) Dividendo 3. Theorem On Equal Ratios
When four quantities are in proportion, the excess
of the first over the second is to the second as a c a a b c
(i) If  , then   .
the excess of the third over the fourth is to the b d b b d d
fourth.
a b c d a c e
 ( i i) If  = , then each of these ratios is equal
If a : b :: c : d, then b d f
b d
a c
Given,  a c e
b d to b  d  f .
360 SSC Advanced Maths
( vi) Three or more quantit ies are said to be in
a c e
( i ii ) If  = = ......, then each ratio is equal to continued proportion when the ratio of the first
b d f and the second is equal to the ratio of second
and t hir d and so on. Thus, a, b, c are in
a  c  e  ...... Sum of antecedent s
= . a b
b  d  f  ...... Sum of consequent s  .
continued proportion if
b c
4. Important Points a b
(vii) When  , we get b2 = a × c. If three quantities
(i) Four quantities a, b, c, d are said to be in b c
proportion (or proportional) if a : b :: c : d. In a, b, c are in continued proportion, then b is
short, proportion means equality of two ratios. said to be the mean proportional between a and
( i i) Proportion is often expressed as a : b :: c : d c. c is called the third proportional to a and b.
and is read as “a is to b as c is to d”. ( vi ii) The concept of proportion need not be restricted
( i ii ) The terms ‘a’ and ‘b’ are called extremes (end to only two equal ratios. It may be extended
terms) and ‘b’ and ‘c’ are called means (middle
terms). a c e
thus. If  = = ........, then a, b, c, d, e, f
( iv) The f our t h ter ms ‘d’ is called t he f ourth b d f
proportional to a, b and c. ...... are said to be in proportion.
(v) The f our quantit ies, a, b, c and d ar e in ( ix ) The concept of continued proportion may be
proportion if and only if the product of the extended as given below:
extremes is equal to the product of the means.
Thus, if a : b :: c : d, then ad = bc and conversely, a b c
If    ......, then a, b, c, d, .... are said to

K
b c d
a c
let ad = bc, then  . be in continued proportion.
b d

Solved Examples

a c 4a  9b 4c  9d (a  2b  3c  4d )  (a  2b  3c  4d )
Ex. 1: (a) If = , show that  . =
b d 4a  9b 4c  9d (a  2b  3c  4d )  (a  2b  3c  4d )

a c 2(a  2b ) 2(a  2b )
( b) If = , pr ov e t hat ( 2a + 3b) ( 2c – 3d)  
b d 2(3c  4d ) 2(3c  4d )
= (2a – 3b)(2c + 3d).
a  2b a  2b
 

KUNDAN
a c 3c  4d 3c  4d
Soln. (a) We have, 
b d By alternendo, we get,
4a 4c 4
  [Multiplying both sides by ] a  2b 3c  4d
9b 9d 9 
a  2b 3c  4d
4a  9b 4c  9d By componendo and dividendo, we get,
 
4a  9b 4c  9d
a  2b  a  2b 3c  4d  3c  4d
[By Componendo and Dividendo] 
a  2b  a  2b 3c  4d  3c  4d
a c
(b) We have,  2a 6c a 3c
b d    
4b 8d 2b 4d
2a 2c 2  4ad = 6bc  2ad = 3bc
  [Multiplying both sides by ]
3b 3d 3
x a x b 2ab
2a  3b 2c  3d Ex. 3: Find the value of  , when x = .
  x a x b a b
2a  3b 2c  3d
[By Componendo and Dividendo] 2ab
Soln. When x =
 (2a + 3b)(2c – 3d) = (2a – 3b)(2c + 3d) a b
[By cross-multiplication] Dividing both sides by a, we have,
Ex. 2: If (a – 2b – 3c + 4d)(a + 2b + 3c + 4d) = (a + 2b – x 2b
3c – 4d) (a – 2b + 3c – d), prove that 2ad = 3bc. 
a a b
a  2b  3c  4d a  2b  3c  4d
Soln.  x  a 3b  a
a  2b  3c  4d a  2b  3c  4d  
x a b a
By componendo and dividendo, we get,
[By componendo and dividendo]
(a  2b  3c  4d )  (a  2b  3c  4d )
x a 3b  a
(a  2b  3c  4d )  (a  2b  3c  4d )   .....(i)
x a a b
Ratio & Proportion and k-Method 361

2ab 1  px 1  qx
Again, x = (e) 1  px 1
a b 1  qx
Dividing both sides by b, we get,
x 2b x 3  3x
341
 Soln. (a) We have, 
b a b 3x 2  1 91
By componendo and dividendo, we get,
x  b 3a  b
  .....(ii)
x b a b x 3  3x  3x 2  1 341  91

[By componendo and dividendo] x 3  3x  3x 2  1 341  91
Adding (i) and (ii), we get,
x a x b 3b  a 3a  b (x  1)3 432 216
 =    
3 250 125
x a x b a b a b (x  1)

3b  a  3a  b 2a  2b 2(a  b ) 3 3
= = = = 2.  x 1 6 x 1 6
a b (a  b ) (a  b )      
 x 1  5 x 1 5

 p  2x p  2y  By componendo and dividendo, we get,


4 xy
Ex. 4: If p = x  y , find the value of  p  2x  p  2y  . x  1  x 1 6  5

x 1 x 1 6  5
4xy p 2y
Soln: p = x  y  2x  x  y 2x 11

K
   x = 11
2 1
By componnendo and dividendo, we get,
p  2x 2y  (x  y ) x  4  x  10 5
 (b) We have, 
p  2x 2y  ( x  y ) x  4  x  10 2
By componendo and dividendo, we get,
p  2x 3y  x
  ....(i) x  4  x  10  x  4  x  10 52
p  2x yx 
x  4  x  10  x  4  x  10 52
4xy p 2x
Again, p = x  y  
2y x  y x 4 7
 
By componendo and dividendo, we get, x  10 3
p  2y 2x  (x  y ) On squaring both sides, we get,

KUNDAN

p  2y 2x  (x  y ) x 4 49

x  10 9
p  2y 3x  y
  ....(ii)  9x + 36 = 49x – 490
p  2y x y
 –40x = –526
Adding (i) and (ii), we get,
526 263
p  2x p  2y 3y  x 3x  y  x 
+ = + 40 20
p  2x p  2y yx x y
x  1  x  1 4x  1
3y  x 3x  y 3y  x  3x  y (c) We have, 
= – = x  1  x 1 2
yx yx yx
By componendo and dividendo, we get,
2y  2x 2(y  x ) x  1  x  1  x  1  x  1 4x  1  2
= = = 2. 
yx yx x  1  x  1  x  1  x  1 4x  1  2
Ex. 5: Find the value of x in the following equations: 2 x  1 4x  1
 
3 2 x  1 4x  3
x  3x 341
(a) 
3x 2  1 91 x  1 4x  1


x  1 4x  3
x  4  x  10 5 On squaring both sides, we get,
(b) 
x  4  x  10 2
x  1 16x 2  8 x  1

x  1  x 1 4 x 1 x  1 16x 2  24x  9
(c) 
x 1  x 1 2 By componendo and dividendo, we get,

a x  a x x 1 x 1 (16x 2  8x  1)  (16x 2  24x  9)


(d) k =
a x  a x x 1 x 1 (16x 2  8 x  1)  (16x 2  24x  9)
362 SSC Advanced Maths
Note: x = 0, also satisfies the given equation.
2x 32x 2  16 x  10
  Ex. 6: Show that 3nx2 – 2mx + 3n = 0, if
2 32x  8
32x – 8x = 32x2 – 16x + 10
2 m  3n  m  3n
x
 –8x + 16x = 10 m  3n  m  3n
5
8x = 10  x m  3n  m  3n
4 Soln. x 
m  3n  m  3n
a x  a x By componendo and dividendo, we get,
(d) We have, k
ax  ax
x 1 ( m  3n  m  3n )  ( m  3n  m  3n )
By componendo and dividendo, we get, =
x 1 ( m  3n  m  3n )  ( m  3n  m  3n )
a  x  a  x  a  x  a  x k 1

a  x  a  x  a  x  a  x k 1 x 1 m  3n
 =
x 1 m  3n
a  x k 1 On squaring both sides, we get,
 
a  x k 1
On squaring both sides, we get, x 2  2x  1 m  3n

x 2  2x  1 m  3n
a  x k 2  2k  1
 By componendo and dividendo, we get,
a  x k 2  2k  1
(x 2  2x  1)  (x 2  2x  1)

K
By componendo and dividendo, we get, m  3n  m  3n

(x 2  2x  1)  (x 2  2x  1) m  3n  m  3n
a  x a  x k 2  2k  1  k 2  2k  1
= 2
a  x a  x k  2k  1  k 2  2k  1 x2 1 m
 
2x 3n
2a 2k 2  2
  3n(x2 + 1) = 2mx  3nx2 + 3n = 2mx
2x 4k
3nx2 – 2mx + 3n = 0.

a k2 1 2ak
   x 15 a 2  4 b 2 47
x 2k k2 1 Ex. 7: If  , find the values of
15a 2  4 b 2 7
1  px 1  qx
(e) 1  px 1  qx  1 a a 3  3b 3

KUNDAN
(i) (ii)
b 3b 3
1  px 1  qx
 1  px  1  qx 15a 2  4b2 47
Soln. (i) 
15a 2  4b 2 7
On squaring both sides, we get,
By componendo and dividendo, we get,
1  p2x 2  2 px 1  qx
 (15a 2  4b 2 )  (15a 2  4b 2 ) 47  7
1  p 2x 2  2 px 1  qx
2 2 2 2

(15a  4b )  (15a  4b ) 47  7
By componendo and dividendo, we get,
30a 2 54 54 8 9 a2
(1  p 2x 2  2 px )  (1  p 2x 2  2 px )      
2
2 2 2 2 8b 40 b 2 40 30 25
(1  p x  2 px )  (1  p x  2 px )
Taking the positive square roots of both the
1  qx  1  qx sides, we get,
=
1  qx  1  qx a 9 3
  
b 25 5
1  p 2x 2 1 2p
   1 + p2x2 =
2 px qx q a 3 a3 27
(ii)   3 
b 5 b 125
2p 2p  q
 p2x2 = – 1 =
q q
a327 9
  
2 2p  q 3b 3 375 125
 x 
p2q Using componendo, we get,

a 3  3b 3 9  125 a 3  3b 3 134
1 2p  q 3 =  =
 x p q 3b 125 3b 3 125
Ratio & Proportion and k-Method 363

x2  y2 xy x y p q x2  y2  z2  px  qy  rz 
2
Ex. 8: If  , prove that x  y  p  q . (ii)  
p2  q2 pq a 2 2
b c 2
 pa  qb  rc 

x 2  y2 xy x 2  y2 p2  q 2 x y z
Soln.    Soln. (i) Since   = k (say)
2 2 pq xy pq a b c
p q
 x = ak, y = bk, z = ck
x 2  y2 p2  q2 3
 
2xy 2 pq  a 2x 2  b 2y 2  c 2z 2  2
LHS =  
By componendo and dividendo, we get,  a 3 x  b 3y  c 3z 
x 2  y 2  2xy p 2  q 2  2 pq
 3
x 2  y 2  2xy p 2  q 2  2 pq  a 2 . a 2k 2  b 2 . b 2k 2  c 2 . c 2k 2  2
=  
 a 3 . ak  b 3 . bk  c 3 . ck 
(x  y )2 ( p  q )2
 
2
(x  y ) ( p  q )2 3
3
 k 2 (a 4  b 4  c 4 )  2
x y pq =    k2
Hence,   k (a 4  b 4  c 4 ) 
x y p q
xyz ak . bk . ck
a c e a 3  c3  e3 ace RHS = =

K
Ex. 9: If b  d  f , prove that 3  . abc abc
b  d 3  f 3 bdf
k 3 . abc 3
a c e = =
Soln. Let    k (say) abc k3  k 2
b d f
Hence, LHS = RHS
Then, a = bk, c = dk, e = fk
x y z
( i i) Since   = k (say)
a3  c 3  e3 b3k 3  d 3k 3  f 3k 3 3 a b c
LHS = = k ..(i)
b3  d 3  f 3 b3  d 3  f 3  x = ak, y = bk, z = ck

ace bk . dk . fk x 2  y2  z 2 a 2k 2  b 2k 2  c 2k 2
RHS = =  k3 ..(ii) LHS = =
bdf bdf a 2  b2  c 2 a 2  b2  c 2

KUNDAN
From (i) and (ii), we get,
LHS = RHS k 2(a 2  b 2  c 2 )
=  k2
3 3 3 a2  b2  c 2
a c e ace
 3 3 = 2 2
b d f3 bdf  px  qy  rz   p . ak  q . bk  r . ck 
RHS =   = 
 pa  qb  rc  pa  qb  rc 
x y z x3 y3 z3 xyz
Ex. 10: If   , show that    . 2
a b c a 3 b 3 c 3 abc  k ( pa  qb  rc ) 2
=   k
 pa  qb  rc 
x y z
Soln. Let   = k Hence, LHS = RHS
a b c
 x = ak, y = bk, z = ck x y z
Substituting these values of x, y and z in LHS. Ex. 12: If   , then show that
b c c a a b
We have,
(i) x + y + z = 0 and (ii) ax + by + cz = 0
a 3k 3 b 3k 3 c 3k 3
LHS =    k3  k3  k3  k3 x

y

z
3 3
a b c3 Soln. Let
b c c a a b
= k (say)

ak . bk . ck  x = k(b – c), y = k(c – a), z = k(a – b)


RHS =  k3 (i) LHS = x + y + z
abc
LHS = RHS [ Each side = k3] = k(b – c) + k(c – a) + k(a – b)
= k(b – c + c – a + a – b)
x y z = k(0) = 0.
Ex. 11: If   , show that
a b c (ii) LHS = ax + by + cz
3 = a{k(b – c)} + b{k(c – a)} + c{k(a – b)}
 a 2 x 2  b2y 2  c2z 2  2 xyz = k(ab – ac + bc – ab + ac – bc)
(i)  3 3 3
 
 a x  b y  c z  abc = k(0) = 0.
364 SSC Advanced Maths

a b c a b c
Ex. 13: If   , prove that each is equal Ex. 15: If  
b c c a a b x  2 y  3z y  2 z  3 x z  2 x  3 y , show

1 a b c
to or –1. that each ratio is equal to 2 ( x  y  z ) .
2

a b c a b c
Soln. Let   = k ....(i)  
b c c a a b Soln.
x  2y  3z y  2z  3x z  2x  3y
Then, a = k(b + c), b = k(c + a), c = k(a + b) Then by the theorem on equal ratios, we get,
 a + b + c = k(b + c + c + a + a + b) Sum of antecedents
= 2k(a + b + c) Each given ratio =
Sum of consequents
 a + b + c – 2k(a + b + c) = 0
a b c
(a + b + c) (1 – 2k) = 0 =
( x  2y  3z )  (y  2z  3x )  (z  2x  3y )
1 a b c a b c
 Either a + b + c = 0 or 1 – 2k = 0  k = = =
2 2x  2y  2z 2(x  y  z )
If a + b + c = 0, then b + c = –a
a a Hence,
so that   1
b  c a a b c b c
= =
b c x  2y  3z y  2z  3x z  2x  3y

K
Similarly,  1 ;  1
c a a b a b c
= .
2(x  y  z )
1 1
If k = , then from (i), each fraction = Ex. 16: Find the third proportional to (x – y), (x2 – y2).
2 2
Soln. Let the third proportional be p, then
1
Hence, each fraction is equal to or –1. x – y : x2 – y2 : : x2 – y2 : p
2
 p (x – y) : (x2 – y2)(x2 – y2)
ay  bx cx  az bz  cy
Ex. 14: If   , then prove that  p (x – y) = (x2 – y2)(x + y)(x – y)
c b a
( x 2  y 2 )(x  y )( x  y )
x y z p =
  . (x  y )

KUNDAN
a b c
 p = (x + y)(x2 – y2)
ay  bx cx  az bz  cy
Soln. We have,  
c b a Ex. 17: Find the fourth proportional to 2xy, x2, y2 .
Sum of the antecedents Soln. Let the fourth proportional be p, then
Each ratio =
Sum of the consequents 2xy : x2 : : y2 : p

ay  bx  cx  az  bz  cy Since, product of extremes = product of means


=
a b c  2xy × p = x2 × y2

x (c  b )  y(a  c )  z (b  a ) x 2y 2 xy
= p = p=
a b c 2xy 2
x y z
Let   = k  x = ak, y = bk, z = ck Ex. 18: Find the mean proportional between (x – y),
a b c
(x3 – x2y2).
ak (c  b )  bk (a  c )  ck (b  a )
Each ratio = Soln. Let the mean proportional be p, then
a b c

0
p = ( x  y )(x 3  x 2y ) = ( x  y ) x 2 (x  y )
=
a b c
= (x  y )2 x 2 = x (x – y) = x2 – xy
x y z
   is true.
a b c
Ratio & Proportion and k-Method 365

Exercise
12. Solve:
6 pq x  3 p x  3q
1. If x  p  q , find the value of x  3 p  x  3q .
4x  1  2x 1

a) 0 b) –1 c) 2 d) 1 4x  1  2x 5
a) –1 b) 5 c) 0 d) 2
4 6 x 2 2 x 2 2
2. If x  , find the value of  .
2 3 x 2 2 x 2 2 7 x  4x  3
13. If  6 , then find the value of x.
a) 2 b) 0 c) 1 d) None of these 7x  4x  3
3. Solve the following equation for x: a) 6 b) 5 c) 7 d) None of these

x  7  x 1 2 (2x  1)2  (2x  1)2 17


 
x  7  x 1 1 14. If 18 , then find the value of x.
(2x  1)2  (2x  1)2
a) 1 b) 0 c) –2 d) 2 Where x > 0.
x 2 x3 1
4. If  5 , then find the value of x. a) 2 b)
x 2 x 3 8
a) 2 b) 7 c) 8 d) 6 c) Both (a) and (b) d) None of these

a  a 2  2ax x 3  y3 91

K
 b , then find the value of x.  2x 2  3y 2
5. If 15. If 37 , determine the value of .
2
a  a  2ax x 3  y3 3y 2

2ab 2ab 5 1 27
2a 2 2b 2 a) b) c) d) 3
a) b) c) d) 27 5 5
(b  1)2 b 1 a b (a  b )2

x2  x 1 x 2  x 1
p2  q 2  p 2  q 2 16. If  and x  y, x  0, y  0, then
2
6. If x  , then find the value of y  y 1 y2  y 1
p 2  q 2  p2  q 2
find the value of xy.
q2x2 – 2p2x + q2. a) 0 b) 2 c) –1 d) 1
a) 0 b) 1 c) –1 d) 2
1 1

KUNDAN
a 9 5a 2  6b 2 ( p  1) 3  ( p  1) 3
7. If  , find the values of . 17. If y = 1 1 , then f ind t he value of
b 5 5a 2  6b 2
( p  1) 3  ( p  1) 3
17 37 13 37
a) b) c) d) y3 – 3py2 + 3y – p.
37 17 37 13 a) 1 b) 0 c) –1 d) None of these

d
8. If a + b = 1; c + d = 1 and a – b = , then find the y 2  xy  y 2  xy
c 18. If p = , then find the value of
value of c – d. y 2  xy  y 2  xy

b a xp2 – 2yp + x.
a) b) c) a + b d) ab a) 0 b) –1 c) 2 d) 1
a b

8ab x  4a x  4b x y z
9. If x = , find the value of  . 19. If   , then find the value
a b x  4a x  4b b c a c a b a b c
a) 4 b) 3 c) 2 d) 1 of (b – c)x + (c – a)y + (a – b)z.
a) 1 b) –1 c) 0 d) None of these
12 pq x  6 p x  6q 2 0 . If a + b : b + c = c + d : d + a, then find the value of
10. If x  p  1 , find the value of x  6 p  x  6q .
a + b + c + d.
a) 1 b) 0 c) –2 d) 2 b
a) 0 b) –1 c) d) None of these
a
a  b 1 a 2  ab  b 2
11. If  , find the value of .
a  b 2 a 2  ab  b 2 x y z
21. If   , then find the value of
r 2  pq p 2  qr q 2  pr
91 73 71 93
a) b) c) d) px + qy + rz.
73 91 93 71
a) –1 b) 1 c) 0 d) 2
366 SSC Advanced Maths

x y z a b c a b c
2 2 . If bc (b  c )  ca (c  a )  ab (a  b ) , then find the value 32. If   , then
c
is equal to
3 4 7
of a(b + c)x + b(c + a)y + c(a + b)z. a) 0 b) 1 c) 2 d) 3
a) 0 b) 1 c) 2 d) –2
xy  y 2
33. If x : y = 7 : 3, then the value of is
2x  y 3x  z z y x2  y2
23. If  
3y  z 2y  x x  z , then f ind t he value of
3 4 3 7
x + y + z. a) b) c) d)
4 3 7 3
2
a) 1 b) 3a  5b
3 34. If  5 , then a : b is equal to
3a  5b
c) 0 d) Cann’t be determined a) 2 : 1 b) 5 : 3 c) 3 : 2 d) 5 : 2

x y z ax  by 35. If p : q = r : s = t : u = 2 : 3, then (mp + nr + ot) : (mq +


24. If   , then find the value of
a b c (a  b )(x  y ) . ns + ou) equals
a) 3 : 2 b) 2 : 3 c) 1 : 3 d) 1 : 2
a) 1 b) 0 c) –1 d) 2
36. If x : y = 3 : 4, then (7x + 3y) : (7x – 3y) is equal to
a) 5 : 2 b) 4 : 3 c) 11 : 3 d) 37 : 19
a b c a 3  b3  c 3
25. If   , then find the value of 3 .
b c d b  c 3  d3 x 3 6 yx
37. If y  4 , then the value of 7  y  x is

K
b c a b
a) b) c) d)
d d d c 2 3 3
a) 1 b) c) d) 1
7 7 7
x y z
26. If   , then find the value of 38. If x : y = 2 : 1, then (5x2 – 13xy + 6y2) is equal to
a b c
3 4 55
ax  by by  cz cz  ax a) b) c) 0 d)
  . 4 3 4
(a  b )(x  y ) (b  c )(y  z ) (c  a )(z  x )
a) 0 b) 2 c) 1 d) 3 a c e 2a 2  3c 2  4e 2
39. If    3 , then ?.
x y z b d f 2b 2  3d 2  4 f 2
27. If   ,
(b  c )(b  c  2a ) (c  a )(c  a  2b ) (a  b )(a  b  2c ) a) 2 b) 3 c) 4 d) 9

KUNDAN
then find the value of x + y + z.
a) 0 b) –1 c) –2 d) 1 y2  x2
40. If 1.5x = 0.04y, then the value of is
y 2  2xy  x 2
ax  by bx  az ay  bz
28. If   and x + y + z  0, then
xy xz yz 730 73 73 74
find the value of each of the ratios. a) b) c) d)
77 77 770 77
a b a b a b a b
a) b) c) d) a b c a b c
2 3 2 3 41. If   , then is equal to
2 3 5 c
a) 2 b) 4 c) 5 d) 6
x 2  y2  p2 z 2  2xy 2z 2  p 2
29. If 2
 2
 , then f ind t he
x yz y zx z 2xy 5 x  2y
42. If x : y = 3 : 4, then the value of is
value of each of the ratios. 7 x  2y

1 1 1 1 1 1 7 7 7 7
a) xy  yz  zx b) x 2  y 2  z 2 a) b) c) d)
25 23 29 17

1 1 1 a b 2a  3b
43. If  , then the value of is
c) x  y  y  z  z  x d) None of these 3 2 3a  2b
12 5 12
30. If x : y = 3 : 2, then the ratio 2x2 + 3y2 : 3x2 – 2y2 is a) b) c) 1 d)
equal to 5 12 7

a) 12 : 5 b) 6 : 5 c) 30 : 19 d) 5 : 3 4ab x  2a x  2b
44. If x  (a  b ) , then the value of  is
a b x  2a x  2b
2a  5b 4
31. If  , then a : b is equal to a) a b) b c) 2ab d) 2
3a  6b 7
a) 21 : 36 b) 2 : 59 c) 59 : 2 d) 36 : 21
Ratio & Proportion and k-Method 367
45. If x varies inversely as (y 2 – 1) and is equal to 24
when y = 10, then the value of x when y = 5 is 3 x  3 x
51. If = 2, then x is equal to
a) 99 b) 12 c) 24 d) 100 3 x  3x

2x  y 1 3x  y 5 12 5 7
46. If  , then the value of is a) b) c) d)
x  2y 2 3x  y 12 5 7 5

1 3 4
a) b) c) d) 1  1 x  1 x 
5 5 5 3  
52. If x  , then the value of   is
2  1 x  1 x 
x y xy
47. If x – y = = , then the numerical value of xy a)  3 b) –1 c) 1 d) 3
7 4
is 53. If a : b : c = (y – z) : (z – x) : (x – y), then the value of ax
+ by + cz is
4 3 1 1
a) b) c) d) a) 1 b) 3 c) 0 d) –1
3 4 4 3
x 4  x 4
 x  y 54. If = 2, then x is equal to
x 4 x 4
48. If y : x = 4 : 15, then the value of  x  y  is
a) 2.4 b) 3.2 c) 4 d) 5
11 19 4 15 55. Find the fourth proportional to x3 – y3, x4 + x2y2 + y4,
a) b) c) d) x – y.
19 11 11 19
a) x2 – xy b) x3 – y2

K
49. If ( 3x – y) : (x + 5y) = 5 : 7, then the value of (x + y) : c) x2 + y2 – xy d) x2 – y2 + xy
(x – y) is
a) 2 : 3 b) 3 : 2 c) 3 : 1 d) 1 : 3 a b
56. Find the third proportional to  , a2  b 2 .
b a
x
50. If (5x2 – 3y2) : xy = 11 : 2, then the positive value of a
y
a) ab b) a + b c) a2 + b2 d)
is b

22 7 3 7 57. Find the mean proportional between (a + b) (a – b)3,


a) b) c) d) (a + b)3 (a – b).
7 2 2 2
a) (a2 + b2)2 b) (a + b)2 c) (a2 – b2)2 d)(a – b)2

Answers and explanations

1. c;

KUNDAN
x 

x  3p
6 pq x
p  q  3p

2q  p  q

Again, x 
6 pq
pq
2q
p q
By componendo and dividendo, we get,
x  3p
=
2q  p  q
=


3q  p
qp

x

2q
3q p  q
By componendo and dividendo, we get,
....(i)
By componendo and dividendo, we get,
x 2 2 2 3 2 3 3 3 2

x2 2 2 3 2 3

Again, x =

2 3
x

4 6
2 3

3 2

2 3

.
4 6

2 3
1
...(i)

x  3q 2p  p  q 3p  q x 2 2
 
= = ...(ii) 2 3 2 3
x  3q 2p  p  q p q
Adding (i) and (ii), we get, By componendo and dividendo, we get,

x  3p x  3q 3q  p 3p  q x 2 3 2 2 2 3 3 2 3
+ = –   ...(ii)
x  3p x  3q qp p q x2 3 2 2 2 3 2 3
From (i) and (ii), we get,
3q  p  3 p  q 2q  2 p q  p 
= = = 2  q  p  = 2. x 2 2 x 2 3 3 3 2 3 2 3
qp qp   + = –
x2 2 x2 3 3 2 3 2

4 6 3 3  2 3 2 3 2 3 2 2
2. a; x = =
2 3 3 2 3 2
 3 2
x 4 6 1 x 2 3  
 = .  = = 2  = 2.
2 2 2 2 2 3 2 2 2 3  3 2
368 SSC Advanced Maths

x  7  x 1 2 2ab – 2bx = b2x + x


3. d; We have, 
x  7  x 1 1 2ab = b2x + 2bx + x
By componendo and dividendo, we get, 2ab = x(b2 + 2b + 1)

( x  7  x  1)  ( x  7  x  1) 2  1 2ab
 2ab = x(b + 1)2  x 
( x  7  x  1)  ( x  7  x  1) 2  1 (b  1)2

2 x7 3 x7 3
    x p2  q 2  p2  q 2
2 x 1 1 x 1 1
6. a; We have, 1 
Squaring both sides, we get, p  q  p2  q2
2 2

x 7 9

x 1 1 x  1 2 p2  q 2 p2  q 2
 x  1  
 1(x + 7) = 9(x – 1) [By cross-multiplication] 2 p2  q 2 p2  q 2
 x + 7 = 9x – 9
 9x – x = 7 + 9 Squaring both sides, we get,
 8x = 16  x = 16 ÷ 8 = 2
(x  1)2 p2  q 2
Hence, x = 2.  
(x  1)2 p2  q2
4. b; Applying componendo and dividendo, we get,
Applying componendo and dividendo, we get,
x 2  x 3  x 2  x 3 5 1

K
x 2  x 3  x 2  x 3 5 1 (x  1)2  (x  1)2 2p2
 
2 2
(x  1)  (x  1) 2q 2
2 x2 6 x 2 3
 

2 x 3 4
or
x 3 2 2(x 2  1) p 2
 
4x q2
x 2 9
Squaring both sides, we get, 
x 3 4 x 2  1 p2
4(x + 2) = 9(x – 3) (By cross-multiplication)   q2x2 + q2 = 2p2x
2x q2
4x + 8 = 9x – 27
4x – 9x = –27 – 8  –5x = –35 q2x2 – 2p2x + q2 = 0
 5x = 35  x = 35 ÷ 5 = 7.
a 9 a2 81

KUNDAN
7. b;    
a  a  2ax 2 b 5 b2 25
5. a; Since, b
a  a 2  2ax
5a 2 5 81 5
Applying componendo and dividendo, we get,    [Multiplying both sides by ]
6b 2 6 25 6

a  a 2  2ax  a  a 2  2ax b 1
 5a 2
27
b 1  
a  a 2  2ax  a  a 2  2ax 6b 2 10
2a b 1 a b 1  By componendo and dividendo, we get,
   
2 a  2ax2 b 1 a  2ax b  1
2 5a 2  6b 2 27  10 37
= =
Squaring both sides, we get, 5a 2  6b 2 27  10 17

a2 b 2  2b  1 d
 8. a; a + b = 1 and a – b =
2
a  2ax b 2  2b  1 c

a b 1 c
a 2  2ax b 2  2b  1   
  [By invertendo] a b d d
a2 b 2  2b  1 c
Applying componendo and dividendo, we get, By componendo and dividendo, we get,
a 2  2ax  a 2 b 2  2b  1  b 2  2b  1 (a  b )  (a  b ) c d

2
a  2ax  a 2 2 2
b  2b  1  b  2b  1 (a  b )  (a  b ) = c  d

2a 2  2ax 2b2  2 2a (a  x ) 2(b 2  1) 2a



1
     (  c + d = 1)
2ax 4b 2ax 4b 2b c  d

b
(a  x ) b 2  1 a  x b2  1   c d (By invertendo)
    a
x 2b x 2b
Ratio & Proportion and k-Method 369

8ab x 2b a  b 1
9. c; x=   
a b 4a a b 11. a; We have,
a  b 2
By componendo and dividendo, we get,
By componendo and dividendo, we get,
x  4a x  4b 2b  (a  b )
 = ( a  b ) ( a  b ) 1 2
x  4a x  4b 2b  (a  b ) 
( a  b)( a  b ) 1 2
x  4a 3b  a
  ...(i)
x  4a b a 2 a 3 a 3
   =
2 a 1 b 1
8ab x 2b
Again, x =  
a b 4b a  b a
By componendo and dividendo, we get, Squaring both sides, we get, = 9
b
x  4b 2a  (a  b )
 2
x  4b 2a  (a  b ) a  a 
2 2      1
a  ab  b b  b 
x  4b 3a  b Now, = 2
  ..(ii) a 2  ab  b 2 a  a 
x  4b a b      1
b  b 
Adding (i) and (ii), we get,
[Dividing numerator and denominator by b2]
x  4a x  4b 3b  a 3a  b
+ =  +
x  4a x  4b b a a b (9)2  9  1 81  9  1 91
= = = .

K
(9)2  9  1 81  9  1 73
3b  a 3a  b 3b  a  3a  b
= – =
b a b  a b a
4x  1  2x 1
2b  2a 2(b  a ) 12. d; We have, 
=  2 4x  1  2x 5
b a b a
By componendo and dividendo, we get,
12pq x 2q
10. d; Since, x    ( 4x  1  2x )  ( 4x  1  2x ) 1  5
pq 6p p  q 
( 4x  1  2x )  ( 4x  1  2x ) 1  5
By componendo and dividendo, we get,
x  6 p 2q  ( p  q ) 2 4x  1 6 4x  1 3
    
x  6 p 2q  ( p  q ) 2 2x 4 2x 2

KUNDAN
x  6 p 3q  p 4x  1 9
 Squaring both sides, we get, 
 ....(i) 2x 4
x  6p qp
4(4x + 1) = 9 × 2x [By cross-multiplication]
12pq x 2p  16x + 4 = 18x
Again, x   
pq 6p p  q 2x = 4  x = 4 + 2 = 2.
By componendo and dividendo, we get,
7 x  4x  3
x  6q 2 p  ( p  q ) 13. c; 6
 7x  4x  3
x  6q 2 p  ( p  q )
Applying componendo and dividendo, we get,
x  6q 3 p  q
  ....(ii) ( 7 x  4x  3)  ( 7x  4x  3) 6  1
x  6q pq 
( 7 x  4x  3)  ( 7x  4x  3) 6  1
Adding (i) and (ii), we get,
x  6p x  6q 3q  p 3p  q 2 7x 7 7x 7
   
+ = + 2 4x  3 5 4x  3 5
x  6p x  6q qp p q
Squaring both sides, we get,
x  6p x  6q 3q  p 3p  q
 + = + 7x 49
x  6p x  6q qp qp  
4x  3 25
3q  p  3 p  q 2q  2 p 2(q  p )  25(7x) = 49(4x – 3) [By cross-multiplication]
= qp = qp = qp 2  175x = 196x – 147
 196x – 175x = 147
x  6p x  6q 21x = 147  x = 147 ÷ 21 = 7
Hence, + = 2. Hence, x = 7.
x  6p x  6q
370 SSC Advanced Maths
x2y + y – xy2 – x = 0
(2x  1)2  (2x  1)2
17 x2y – xy2 – x + y = 0
14. c; We have, 
(2x  1)2  (2x  1)2 18 xy(x – y) – 1(x – y) = 0
(x – y)(xy – 1) = 0
By componendo and dividendo, we get,
x – y = 0 or xy – 1 = 0
[(2x  1)2  (2x  1)2 ]  [(2x  1)2  (2x  1)2 ] 17  8 x = y or xy = 1
 But, x  y (given),  xy = 1.
2 2 2 2 17  8
[(2x  1)  (2x  1) ]  [(2x  1)  (2x  1) ]
1 1
2(2x  1)2
25 (2x  1)2 25 ( p  1) 3  ( p  1) 3
    17. b; We have, y = 1 1
2 9 2 9
2(2x  1) (2x  1)
( p  1) 3  ( p  1) 3
2x  1 5 2x  1 5
    y 1
2x  1 3 2x  1 3 Applying componendo and dividendo, we get,
y 1
6x + 3 = 10x – 5  6x + 3 = – 10x + 5
–4x = –8  16x = 2
 1 1  1 1
1 ( p  1)3  ( p  1)3   ( p  1)3  ( p  1)3 
x = 2 or x =    
8
   
=
 1 1  1 1
x 3  y3 91 ( p  1)3  ( p  1)3   ( p  1)3  ( p  1)3 
15. a; We have, 
x 3  y3 37 ,    

K
   
Using componendo dividendo, we get,
1
(x 3  y 3 )  (x 3  y 3 ) 91  37 3
 y  1 ( p  1)3  y  1 p 1
3 3 3 3 91  37     
(x  y )  (x  y ) y 1 1  y  1 p 1
( p  1)3
2x 3 128 x3 64
   
3 3 27
2y 54 y y 3  3y 2  3y  1 p 1
 
y 3  3y 2  3y  1 p 1
x 4 x2 16
    Using componendo and dividendo, we get,
y 3 y2 9
y 3  3y 2  3y  1  (y 3  3y 2  3y  1)

KUNDAN
2 

2x

32 y 3  3y 2  3y  1  (y 3  3y 2  3y  1)
3y 2 27
( p  1)  ( p  1)

2
2x  3y 2
32  27 ( p  1)  ( p  1)
  [By dividendo]
3y 2 27
2y 3  6y 2p y 3  3y
    p
6y 2  2 2 3y 2  1
2x 2  3y 2 5
 
3y 2 27 y3 + 3y = p(3y2 + 1)
y3 – 3py2 + 3y – p = 0
18. a; We have,
x2  x 1 x 2  x 1
16. d; We have, 2
 2
y  y 1 y  y 1 p y 2  xy  y 2  xy

By alternendo 1 y 2  xy  y 2  xy
x2  x 1 y2  y  1 By componendo and dividendo, we get,

2
x  x 1 y2  y  1
p 1 ( y 2  xy  y 2  xy )  ( y 2  xy  y 2  xy )
By componendo and dividendo, we get, =
p 1 ( y 2  xy  y 2  xy )  ( y 2  xy  y 2  xy )
(x 2  x  1)  (x 2  x  1) (y 2  y  1)  (y 2  y  1)

(x 2  x  1)  (x 2  x  1) (y 2  y  1)  (y 2  y  1) 2 p 1 y 2  xy
p  1 2 y  xy 
  
p  1 2 y 2  xy p 1 y 2  xy
2x 2  2 2y 2  2
 
2x 2y Squaring both sides, we get,

p 2  2p  1 y 2  xy
x 2  1 y2  1 
   y = (x2 + 1) = x(y2 + 1) 2
p  2p  1 y 2  xy
x y
Ratio & Proportion and k-Method 371
By componendo and dividendo, we get, yz + y2 – x2 – xz = 0
2 2 2 2 z(y – x) + (y2 – x2) = 0
( p  2 p  1)  ( p  2 p  1) (y  xy )  (y  xy )
 z(y – x) + (y – x)(y + x) = 0
( p2  2 p  1)  ( p 2  2 p  1) (y 2  xy )  (y 2  xy ) (y – x) (x + y + z) = 0
Either y – x = 0 or x + y + z = 0
2( p 2  1) 2y 2 p2  1 y x = y or x + y + z = 0.
   
4p 2xy 2p x
x y z
x(p2 + 1) = 2yp xp2 – 2yp + x = 0. 24. a; Let   k
a b c
x y z x = ak, y = bk, z = ck
19. c; Let   k
b c a c a b a b c The given expression
 x = (b + c – a)k
 y = (c + a – b)k ax  by a(ak )  b(bk )
 z = (a + b – a)k = (a  b )(x  y ) =
(a  b )(ak  bk )
Now, the given expression
= (b – c)x + (c – a)y + (a – b)z
k (a 2  b 2 ) k (a 2  b2 )
= (b – c)(b + c – a)k + (c – a)(c + a – b)k = = = 1.
+ (a – b)(a + b – c)k k (a  b )(a  b ) k (a 2  b2 )
= (b2 + bc – ab – bc – c2 + ca)k + (c2 + ac – bc – ac – a2
+ ab)k + (a2 + ab – ac – ab – b2 + bc)k a b c
25. c; Let   = k
= (b – ab – c + ca)k + (c2 – bc – a2 + ab)k
2 2
b c d
+ (a2 – ac – b2 + bc)k Then c = dk, b = dk2 and a = dk3
= (a2 – ab – c2 + ca + c2 – bc – a2 + ab The given expression
+ a2 – ac – b2 + bc)k

K
=0×k=0 a 3  b3  c 3 (dk 3 )3  (dk 2 )3  (dk )3
= =
a b c d
3
b c d3 3
(dk 2 )3  (dk )3  d 3
20. a; Let  k
b c d a
a + b = (b + c)k ....(i) d 3k 9  d 3k 6  d 3k 3
=
c + d = (d + a)k ....(ii) d 3k 6  d 3k 3  d 3
Adding (i) and (ii), we get,
a + b + c + d = (b + c + d + a)k d 3k 3(k 6  k 3  1) a
=  k3 
(a + b + c + d) – (a + b + c + d)k = 0 d 3(k 6  k 3  1) d
(a + b + c + d) (1 – k) = 0
Now either a + b + c + d = 0 or 1 – k = 0
x y z
x y z 26. d; Let   k
a b c

KUNDAN
21. c; Let  2  2 = k
r 2  pq p  qr q  pr Then, x = ak, y = bk, z = ck, we have
x = k(r – pq); y = k(p – qr); z = k(q2 – pr)
2 2

px + qy + rz = pk(r2 – pq) + qk(p2 – qr) + rk(q2 – pr) ax  by a(ak )  b(bk )


= k{pr2 – p2q + p2q – q2r + q2r – r2p} (a  b )(x  y ) = (a  b )(ak  bk )
=k× 0 = 0
x y z k (a 2  b 2 ) k (a 2  b2 )
22. a; Let bc (b  c )  ca (c  a )  ab (a  b ) = k =
(a  b )k (a  b )
=
k (a 2  b2 )
= 1

x = bc(b – c)k, y = ca(c – a)k, z = ab(a – b)k


by  cz cz  ax
 a(b + c)x = abc(b + c)(b – c)k = abc(b2 – c2)k Similarly, = 1 and = 1
Similarly, b(c + a)y = abc(c2 – a2)k; c(a + b)z (b  c )(y  z ) (c  a )(z  x )
= abc(a2 – b2)k Hence,
Adding up, we get,
ax  by by  cz cz  ax
a(b + c)x + b(c + a)y + c(a + b)z + + = 3.
= abc(b2 – c2)k + abc(c2 – a2)k + abc(a2 – b2)k (a  b )(x  y ) (b  c )(y  z ) (c  a )(z  x )
= abc . k(b2 – c2 + c2 – a2 + a2 – b2)
= abc × k(0) = 0. x y
27. a; Let =
(b  c )(b  c  2a ) (c  a )(c  a  2b )
2x  y 3x  z z y z
23. c; 3y  z  2y  x  x  z = = k
(a  b )(a  b  2c )
By the theorem on equal ratios, we get, Then, x = k(b – c)(b + c – 2a) ....(i)
2x  y  3x  z  z  y 5x x y = k(c – a)(c + a – 2b) ....(ii)
Each given ratio =   z = k(a – b)(a + b – 2c) ....(iii)
3y  z  2y  x  x  z 5y y
Adding (i), (ii) and (iii), we get,
z y x x + y + z = k{(b – c)(b + c – 2a) + (c – a)(c + a – 2b) +
 x  z  y (a – b)(a + b – 2c)}
= k[(b – c)(b + c) + (c – a)(c + a) + (a – b)(a + b)]
y(z + y) = x(x + z) [By cross-multiplication] – 2[a(b – c) + b(c – a) + c(a – b)] = 0.
yz + y2 = x2 + xz
372 SSC Advanced Maths

ax  by bx  az ay  bz a b c
28. c; We have,   32. c; Let   = k
x y x z yz 3 4 7
By the theorem on equal ratios, a = 3k, b = 4k, c = 7k

(ax  by )  (bx  az )  (ay  bz ) a b c 3k  4k  7k 14k


Each ratio =  = = = 2
( x  y )  ( x  z )  (y  z ) c 7k 7k

ax  bx  ay  by  az  bz x 7
= 33. a; We have, y  3
x y x z y z
Now,
x  (a  b )  y (a  b )  z (a  b )
= 2x  2y  2z xy  y 2 y(x  y ) y 1
 = 
x2  y2 ( x  y )( x  y ) x y x
1
(a  b )( x  y  z ) a b y
= =
2(x  y  z ) 2
1 1 3
[  x + y + z  0] =  
7 73 4
1
a b 3 3
 Each ratio =
2
3a  5b 5
34. d; We have, 
x 2  y 2  p2 z 2  2xy 2z 3  p 2 3a  5b 1
29. a; We have, = = By componendo and dividendo, we get,

K
x 2yz y 2zx z 2xy
By the theorem on equal ratios, we have, 3a  5b  3a  5b 5  1

3a  5b  3a  5b 5  1
(x 2  y 2  p 2 )  (z 2  2xy )  (2z 2  p 2 )
Each ratio = 6a 6 a 6 10 5
x 2yz  y 2zx  z 2 xy      
10b 4 b 4 6 2
x 2  y 2  2xy  z 2 ( x  y )2  z 2 5 : 2
= 
xyz ( x  y  z ) xyz (x  y  z ) p r t 2
35. b; We have,   
(x  y  z )(x  y  z ) x y z q s u 3
= = (If x  y  z) p q
xyz (x  y  z ) xyz    k p = 2k and q = 3k
x y z 1 1 1 2 3
= xyz  xyz  xyz  yz  zx  xy Similarly, r = 2k, s = 3k, t = 2k, u = 3k

KUNDAN
mp  nr  ot
Now, mq  ns  ou
x 3
30. c; Here y  2
m.2k  n .2k  o.2k 2k (m  n  o ) 2
= = =
2 2 m.3k  n .3k  o.3k 3k (m  n  o ) 3
x 3 9
 2
   2 : 3
y 2
  4
x 3 7x 7 3 7
 x2  36. c; Since,      
2 2 2 2   3 y 4 3y 3 4 4
2x  3y y 
  By componendo and dividendo, we get,
Now, =
3x 2  2y 2  x2 
3 2   2 7x  3y 7  4 11
y  =  11 : 3
  7x  3y 74 3
[On dividing numerator and denominator by y2]
x 3
37. a; We have, 
 9 9 96 y 4
2   3 3
 4 2 2 15 4 x
= = 27 = 27  8 =  1
 9 2 2 19 6 yx 6 y
3    2 Now,  = 
 4 4 4 7 yx 7 1 x
y
30
= = 30 : 19 [Dividing numerator and denominator by y]
19

2a  5b 4 3
31. c; We have,  6 1 4 6 43
3a  6b 7 =  = 
7 1 3 7 43
 14a – 35b = 12a + 24b  2a = 59ab 4
a 59 6 1
 = = 59 : 2 =  = 1
b 2 7 7
Ratio & Proportion and k-Method 373

x x  2a x  2b 3b  a 3a  b
38. c; We have, y = 2 x = 2y x – 2y = 0   =  +
x  2a x  2b b a a b
5x2 – 13xy + 6y2 = 5x2 – 10xy – 3xy + 6y2
= 5x (x – 2y) – 3y (x – 2y) 3b  a  3a  b 2b  2a 2(b  a )
=  = = 2
= (x – 2y)(5x – 3y) b a b a b a
= 0 × (5x – 2y) = 0
1
a c e 45. a; We have, x 
39. d; We have, b  d  f  3 y2 1
a = 3b; c = 3d; e = 3f k
2  9b 2  3  9d 2  4  9 f 2  x  ; where k is a constant.
2a 2  3c 2  4e 2 y2 1
 
2
2b  3d  4 f2 2 2b 2  3d 2  4 f 2
When y = 10, x = 24, then
9(2b 2  3d 2  4 f 2 ) k k
= 9
2b 2  3d 2  4 f 2 24 =  24 =
102  1 9
40. b; We have, 1.5x = 0.04y k = 24 × 99
x 0.04 4 2 When y = 5, then
   
y 1.5 150 75 k 24  99 24  99
y 75 x =    99
  y2 1 52  1 24
x 2
y2  x2 (y  x )(y  x ) 2x  y 1
46. b; We have, x  2y  2

K
Now, 2 2
= 2
y  2xy  x (y  x )
4x – 2y = x + 2y
y 75 3x = 4y
1 1
yx 2 73
=  x = 75  x 4
yx y 77  
1  1 y 3
x 2

 x   4 
a b c y  3  1 3   1
41. a; Let   k 3x  y  y   3  4 1 3
2 3 5   = = 
3x  y  x   4  4 1 5
a = 2k; b = 3k; c = 5k y  3  1  3   1
 y  3
a b c 2k  3k  5k 10k
 = = 2 x y xy
c 5k 5k 47. a; x – y = = k

KUNDAN
7 4
x 3
42. c; We have, y  4 x – y = k; x + y = 7k
 (x + y)2 – (x – y)2 = 49k2 – k2
4xy = 48k2
x 3 15  8
5 2 5 2  16k = 48k 2
5x  2y y 4 4 7
 = x = 3 = 21  8  1
7x  2y 7 2 29 k=
7 2 3
y 4 4
1 4
 xy = 4k = 4 × =
a b a 3 3 3
43. a; We have,   
3 2 b 2 48. a; Since, y : x = 4 : 15  x : y = 15 : 4
x 15
a 3  
2a  3b 3 2 23 y 4
b 2 6 12
 = = =  By componendo and dividendo, we get,
3a  2b a 3 94 5
3 2 3 2 x y 15  4 11
b 2 2 = =
x y 15  4 19
4ab x 2b
44. d; Since, x     3x  y 5
a b 2a a b 49. c; We have, 
x  5y 7
By componendo and dividendo, we get, 21x – 7y = 5x + 25y
x  2a 2b  a  b 3b  a 21x – 5x = 25y + 7y
   16x = 32y
x  2a 2b  a  b b a
x 32 2
Again,   
y 16 1
x 2a
 x  y 2 1
2b a  b  x  y  2  1 = 3 : 1
x  2b 2a  a  b 3a  b
   [By componendo and dividendo]
x  2b 2a  a  b a b
374 SSC Advanced Maths

5x 2  3y 2 11 a b c
50. b; Since,
xy

2 53. c; Let y  z  z  x  x  y  k

11 a = k(y – z); b = k(z – x); c = k(x – y)


 5x2 – 3y2 = xy ax + by + cz = k(xy – xz + yz – xy + xz – yz) = 0
2
2 2
 10x – 6y – 11xy = 0
x 4  x 4 2
 10x2 – 15xy + 4xy – 6y2 = 0 54. d; We have, =
5x(2x – 3y) + 2y(2x – 3y) = 0 x 4 x 4 1
(2x – 3y) (5x + 2y) = 0 By componendo and dividendo, we get,
 2x = 3y
2 x4 3
x 3 
 2 x 4 1

y 2 On squaring both sides, we get,
x 4 9
3 x  3x 2 
51. b; We have,  x 4 1
3 x  3x 1
9x – 36 = x + 4
By componendo and dividendo, we get, 9x – x = 36 + 4
2 3  x 2 1 8x = 40
  3  x = 5
2 3  x 2 1
Squaring on both sides, we get, 55. c; Let the fourth proportional be p, then
x3 – y3 : x4 + x2y2 + y4 : : x – y : p

K
3x p(x3 – y3) = (x4 + x2y2 + y4) (x – y)
9
3x
3 + x = 27 – 9x (x 4  x 2y 2  y 4 )(x  y )
 p
9x + x = 27 – 3 = 24 x 3  y3
24 12
x =  [( x 2  y 2 )  ( xy )2 ](x  y )
10 5 =
(x  y )(x 2  y 2  xy )

3 1 2
52. d; Since, x    (x 2  y 2  xy )(x 2  y 2  xy )(x  y )
2 x 3 =
( x  y )(x 2  y 2  xy )
By componendo and dividendo, we get,
= x2 + y2 – xy

KUNDAN
1 x 2  3 56. a; Let the third proportional be x, then

1 x 2  3
a b
   : a2  b 2 : : a2  b 2 : x
2 2 b a

1 x
=
 
2 3
=
2  3 
a b 2
1 x 2  3 2  3 43
 x    =
b a a 2  b 2 
1 x 2
 = 2  3
1 x  a 2  b2 
 x  ab  = a2 + b2
1 x 2  3  
 
1 x 1  x = ab
By componendo and dividendo, we get, 57. c; Let the mean proportional be x, then
1 x  1 x 2  3 1 x = (a  b )(a  b )3 (a  b )3 (a  b )

1  x  1  x 2  3 1
= (a  b )4 (a  b )4 = (a + b)2 (a – b)2
=
3 3

3  3 1  3 = [(a + b)(a – b)]2 = (a2 – b2)2
3 1 3 1
Series
A Series is a sequence of numbers obtained by (iii) Series of squares, cubes etc
some particular predefined rule and applying that pre- These series can be formed by squaring or cub-
defined rule it is possible to find out the next term of ing every successive number.
the series. For example,
A series can be created in many ways. Some of (i) 2, 4, 16, 256, ...
these are discussed below : (ii) 3, 9, 81, 6561, ....
(i) Arithmetic Series (iii) 2, 8, 512, ..... etc.
An arithmetic series is one in which successive are such series. (In the first and second, every
numbers are obtained by adding (or subtracting) number is squared to get the next number while
a fixed number to the previous number. For ex- in the third it is cubed).
ample, (iv) Mixed Series
(i) 3,5,7,9,11,...... A mixed series is basically the one we need to
(ii) 10,8,6,4,2,...... have a sound practice. Because it is generally
(iii) 13,22,31,40,49,..... the mixed series which is asked in the examina-
(iv) 31,27,23,19,15,.....etc. tion. By a mixed series, we mean a series which is
are arithmetic series because in each of them the created according to any non-conventional (but
next number can be obtained by adding or sub- logical) rule. Because there is no limitation to
tracting a fixed number. (For example, in 3,5,7,9,11, people’s imagination, there are infinite ways in
..... every successive number is obtained by add- which a series can be created and naturally it is
ing 2 to the previous number). not possible to club together all of them. Still we
(ii) Geometric Series are giving examples of some more popular ways
A geometric series is one in which each succes- of creating these mixed series. (We shall be giv-
sive number is obtained by multiplying (or divid- ing them names, which are not generalised and
ing) a fixed number by the previous number. probably not found in any other book, but which
For example, are given with the purpose of clarifying their logic
(i) 4,8,16,32,64,.... without difficulty).
(ii) 15,-30,60,-120,240,..... I) Two-tier Arithmetic Series. We have seen
(iii) 1024,512,256,128,64,..... that in an arithmetic series the difference of any two
(iv) 3125,-625,125,-25,5,...... successive numbers is fixed. A Two-tier Arithmetic
are geometric series because, in each of them, Series shall be the one in which the differences of
the next number can be obtained by multiplying successive numbers themselves form an arithmetic
(or dividing) the previous number by a fixed num- series.
ber. (For example, in : 3125,-625, 125,-25,5,... ev- Examples
ery successive number is obtained by dividing (a) 1, 2, 5, 10, 17, 26, 37, .....
the previous number by -5.) (b) 3, 5, 9, 15, 23, 33, ..... etc.
2 K KUNDAN

are examples of such series. [In 1, 2, 5, 10, 17, 26, is an arithmetico-geometric series. (Each succes-
37, .....; for example, the differences of succes- sive term is obtained by first adding 1 to the
sive numbers are 1, 3, 5, 7, 9, 11,... which is an previous term and then multiplying it by 3).
arithmetic series. Note : The differences of successive numbers should
Note: Two-tier arithmetic series can be denoted as a be in Geometric Progression.
quadratic function. In this case, the successive differences are 5, 15,
For example, the above series 45, 135, ..... which are in GP.
b) Geometrico-Arithmetic Series. As the name
(a) is 0 2  1, 12  1, 2 2  1, 32  1, .... which can be
suggests, a geometrico-arithmetic series should be
denoted as the one in which each successive term is found by
f x   x 2  1 , where x = 0, 1, 2, .... first multiplying (or dividing) the previous term by a
fixed number and then adding (or deducting ) another
Similarly example (b) can be denoted as
fixed number.
f(x) = x 2  x  3 , x = 0, 1, 2, 3, .... For example
II) Three-tier Arithmetic Series. This, as the 3, 4, 7, 16, 43, 124, ....
name suggests, is a series in which the differences of is a geometrico-arithmetic series. (Each succes-
successive numbers form a two-tier arithmetic series; sive term is obtained by first multiplying the pre-
whose successive term’s differences, in turn, form an vious number by 3 and then subtracting 5 from
arithmetic series. it.)
For example Note: The differences of successive numbers should
a) 336, 210, 120, 60, 24, 6, 0, .... be in geometric progression. In this case, the
is an example of three-tier arithmetic series. successive differences are 1, 3, 9, 27, 81, .....
[The differences of successive terms are which are in GP.
126, 90, 60, 36, 18, 6, ...... IV) Twin Series. We shall call these twin series,
The differences of successive terms of this new because they are two series packed in one .
series are 1, 3, 5, 1, 9, -3, 13, -11, 17, ........
36, 30, 24, 18, 12, ....... is an example of twin series. (The first, third, fifth
which is an arithmetic series.] etc. terms are 1, 5, 9, 13, 17 which is an arithmetic
Note: Three-tier arithmetic series can be denoted as a series. The second, fourth, sixth etc. are 3, 1, -3, -
cubic function. 11 which is a geometrico-arithmetic series in
For example, the above series is (from right end) which successive terms are obtained by multi-
plying the previous term by 2 and then subtract-
13  1, 23  2, 33  3, 43  4, ... which can also be
ing 5.)
denoted as V) Other Series. Besides, numerous other se-
f(x) = x 3 – x; x = 1, 2, ... ries are possible and it is impossible to even think of
III) We know that, (let alone write them down ) all of them. It is only
i) In an arithmetic series we add ( or deduct ) a through a lot of practice and by keeping abreast with
fixed number to find the next number, and the latest trends that one can expect to master the
ii) In a geometric series we multiply (or divide ) a series.
fixed number to find the next number.
We can combine these two ideas into one to form Suggested steps for solving series ques-
a) Arithmetico-Geometric Series. As the name tions
suggests, in this series each successive term should Despite the fact that it is extremely difficult to lay
be found by first adding a fixed number to the previ- down all possible combinations of series, still, if you
ous term and then multiplying it by another fixed num- follow the following step-by-step approach, you may
ber. solve a series question easily and quickly:
For example Step I: Preliminary Screening
1,6, 21, 66, 201...... First check the series by having a look at it. It
K KUNDAN 3

may be that the series is very simple and just a first the series may be irregularly increasing or decreas-
look may be enough and you may know the next term. ing. In such cases, the rise (or fall ) may be sharp then
Some examples are given below, where preliminary slow and then again sharp and so on .
screening is sufficient to tell you the next term. For example, consider the series: 4, 5, 7, 10, 14,
Ex. i) 4, -8, 16, -32, 64, ? 19, 25. Here the series increases and the increase is
ii) 1, 4, 9, 16, 25, 36, 49, ? slow. A gradual, slow increase. So you should try to
iii) 1, 3, 6, 10, 15, 21, ? test for an arithmetic type of increase. Indeed, it turns
iv) 2,6, 18, 54, 162, ? out to be a two-tier arithmetic series, the differences
Answer i) Each term is multiplied by -2. 1, 2, 3, 4, 5, forming a simple series.
Next term : -132. Again, consider the series: 1, 2, 6, 15, 31, 56. Here
ii) The series is +3, +5, +7, +9,+11, +13, +15. you may immediately ‘feel’ that the series rises very
Next term: 49 + 15 = 64. sharply. So, you should try to test for a geometric
Another approach: The series is,12,22, 32 etc. type of increase. On trial you may see that the series
Next term: 82 = 64. is not formed by successive multiplications. So, you
iii) The series is +2, +3, +4, +5, +6, +7. should check for addition of squared numbers, cubed
Next term: 21 + 7 =28. numbers etc. Indeed the series turns out to be 1, 1+12,
iv) Each term is multiplied by 3. 1+ 12+ 22, 1+ 12+ 22+ 32 etc. Another similar example
Next term: 162 x 3 = 486. could be of the series 1, 5, 14, 30, 55, 91. This is 12 ,
Step II: Check Trend: Increasing / Decreasing / Al- 12+ 22, 12+ 22 + 32, 12 + 22 + 32 + 42 etc. Another example
ternating could be : 2, 9, 28, 65, 126, 217. This is : 13+1, 23+1, 33+1
If you fail to see the rule of the series by just etc.
preliminary screening you should see the trend of the [Note: We have seen that there may be two ways in
series. By this we mean that you should check whether which a geometric increase (or decrease) may take
the series increases continuously or decreases con- place. In one case it is because of multiplications (or
tinuously or whether it alternates, ie, increases and divisions) by terms and in other case it is because of
decreases alternately. For example, the series i) and ii) addition (or substraction) of squared or cubed terms.
in the following examples are increasing, the series iii) How do we differentiate between the two? We can
is decreasing and the series iv) is alternating. differentiate between the two by looking at the trend
Ex. i) 3, 10, 21, 36, 55, 78. of the increase. If the increase is because of addition
ii) 5,10,13,26,29, 58. of squared or cubed terms, the increase will be not
iii) 125, 123, 120, 115, 108, 97. very sharp in the later terms (fourth, fifth, sixth terms
iv) 253, 136, 352, 460, 324, 631, 244. etc.) For example, watch the series: 1, 2, 6, 15, 31, 56.
Step III (A) ( to be employed if the series is increas- Here the series appears to rise very steeply: 1, 1 × 2 =
ing or decreasing) Feel the rate of increase or de- 2, 2 × 3 = 6, 6 × 2.5 = 15, 15 × 2  31, 31 × (1. something)
crease = 56. Thus we see multiplications are by 2, 3, 2.5, 2,
For an increasing (or decreasing) series, start with respectively. That is, the rise is very sharp initially
the first term and move onwards. You will notice that but later it slows down. The same can be said to be
the series proceeds either arithmetically or geometri- true of the series: 1, 5, 14, 30, 55, 91. Here, 1 × 5=5, 5 ×
cally or alternately . By an arithmetic increase, we 3  14, 14 × 2  30, 30 × 1.8  55, 55 × 1.6  91. Here
mean that there is an increase (or decrease) of terms too, the rise is very sharp initially, but later it slows
by virtue of addition (or subtraction). In such cases down. In such cases, therefore, where the rise is
you will ‘feel’ that the series rises (or falls) rather very sharp initially but slows down later on, you should
slowly. By a geometric increase (or decrease) we mean check for addition of squared or cubed numbers.]
that there is an increase (or decrease) of terms by As our next example, consider the series : 3, 5, 11,
virtue of multiplication (or division) or if there is addi- 25, 55, 117. We see that this series, too, rises very
tion it is of squares or of cubes. In such cases, you sharply. Hence, there must be a geometric type of
will ‘feel’ that the series rises (or falls ) very sharply. increase. Further, the rate of increase does not die
By an alternative increase (or decrease ) we mean that down in later terms. In fact, it picks up as the series
4 K KUNDAN

progresses. Hence, this time the geometric increase On the other hand, 1, 3, 5, 10, 14, 29, 30 is an
should be of the first kind, i.e., through multiplica- increasing series having alternating increase.)
tion. The series must be formed by multiplications by For an alternating series you should check for
2 and some further operation. Now it is easy. A little two possibilities: One, that the series may be a mix of
more exercise will tell us that the series is : ×2–1, ×2+1, two series (twin series) and two, that two different
×2+3, ×2+5, ×2+7 etc. Another and similar example kinds of operations may be going on. For example,
could be : 7, 8, 18, 57, 232, 1165. Here the series is: × consider the series: 4, 8, 6, 12, 9, 16, 13. This is an
1+1, ×2+2, ×3+ 3, ×4+ 4, ×5+ 5. alternating series. It is a mix of two simple series: 4, 6,
As our last example we will take up a series which 9, 13 and 8, 12, 16 etc. Again, consider the series: 800,
shows an alternating increase. In such cases there 1200, 600, 1000, 500, 900. Here, two different kinds of
are two possibilities: one, that two different series operations are going on. One, addition of 400 and
may be intermixed or the other, that two different kinds two, division by 2.
of operations may be being performed on successive
terms. To understand this, let us see the following A summaryof the three steps
examples. Consider the series: 1, 3, 5, 10, 14, 29, 30, 84. [Very Important]
You can see that this series increases gradually and Step I: Do a preliminary screening of the series. If it is
hence it is an increasing series but the increase, in a simple series you will be able to solve it easily.
itself, is irregular, haphazard. In fact, it is a mix of two Step II: If you fail in preliminary screening then deter-
series: 1, 5, 14, 30 which is a series: 1, 1+22,1+22+32, 1+ mine the trend of the series. Determine whether it
22+ 32+ 42; and the other series: 3,10, 29, 84 which is is increasing, decreasing or alternating.
another series: × 3+ 1, × 3-1, ×3-3 etc. Again, consider Step III (A): Perform this step only if a series is in-
the series: 3, 13, 20, 84, 91, 459. This is also an in- creasing or decreasing. Use the following rules:
creasing series with a haphazard increase ( alternat- i) If the rise of a series is slow or gradual, the
ing increase) with sharp and then slow rises coming series is likely to have an addition-based in-
alternately. Here, two different kinds of operations crease; successive numbers are obtained by
are being performed alternately: the first operation is adding some numbers.
that of multiplication by 3, 4, 5 successively and add- ii) if the rise of a series is very sharp initially but
ing a constant number 4 and the second operation is slows down later on, the series is likely to be
that of adding 7. Hence the series is: ×3+4, + 7, ×4+4, formed by adding squared or cubed num-
+7, ×5+4. bers.
Step III (B) ( to be employed if the series is neither iii) if the rise of a series is throughout equally
increasing nor decreasing but alternating ) Check sharp, the series is likely to be multiplication-
two possibilities based; successive terms are obtained by
For an alternating series, where the terms in- multiplying by some terms (and, maybe, some
crease and decrease alternately, the rules remain, more addition or subtraction could be there, too.)
or less, the same as those for a series showing alter- iv) if the rise of a series is irregular and haphaz-
nating increase. ard there may be two possibilities. Either
(Note: Please note the difference between an alter- there may be a mix of two series or two differ-
nating increase and a series having alternat- ent kinds of operations may be going on al-
ing increase carefully. In an alternating increase ternately. (The first is more likely when the
terms increase, decrease alternately. But a series increase is very irregular: the second is more
having alternating increase increases continu- likely when there is a pattern, even in the
ously [and on having alternating decrease de- irregularity of the series).
creases continuously]. The increase may be hap- Step III (B): (to be performed when the series is alter-
hazard and irregular - alternately, sharp and slow nating)
- but the increase is continuous. For example, 15, [Same as (iv) of step (iii). Check two possibili-
22, 20, 27, 25 is an alternating series because ties]
there is increase and decrease in terms, alternately.
K KUNDAN 5

Some solved examples multiple. On checking the series is: × 2 + 4. Next


Ex. Find the next number of the series term = 316 × 2 + 4 = 636
i) 8, 14, 26, 50, 98, 194 x) Series increases sharply but then its speed of
ii) 8, 8, 9, 9, 11, 10, 14, 11 rise slows down. Likely to be addition of squared
iii) 325, 259, 204, 160, 127, 105 or cubed numbers. On checking, the series is:13–
iv) 54, 43, 34, 27, 22, 19 3, 23– 4, 33 – 5, 43 – 6 ..... Next term =73 – 9 = 334
v) 824, 408, 200, 96, 44, 18
Finding wrong numbers in a series
vi) 16, 17, 21, 30, 46, 71
In today’s examinations, a series is more likely to
vii) 3, 3, 6, 18, 72, 360
be given in the format of a complete series in which
viii) 3, 4, 8, 17, 33, 58
an incorrect number is included. The candidate is re-
ix) 6, 16, 36, 76, 156, 316
quired to find out the wrong number.
x) –2, 4, 22, 58, 118, 208
Obviously, finding the wrong number in a series
Solutions
is very easy once you have mastered the art of under-
i) Sharp increase and terms roughly doubling ev-
standing how the series is likely to be formed. On
ery time. On checking with 2 as multiple the se-
studying a given series and applying the concepts
ries is:
employed so far you should be able to understand
next term = previous term ×2 -2. Next term = 382.
and thus “decode” the formation of the series. This
ii) Irregular. Very irregular. Likely to be, therefore,
should not prove very difficult because usually six
mixed. On checking it is a mix of two series:
terms are given and it means that at least five correct
8, 9, 11, 14, (+1, +2, +3 etc.) and 8, 9, 10, 11.
terms are given. This should be sufficient to follow
Next term =14+4=18.
the series.
iii) Gradual slow decrease. Likely to be arithmetical
We are giving below some solved examples on
decrease. Check the differences of successive
this particular type where you are required to find out
terms. They are: 66, 55, 44, 33, 22. Hence, next
the wrong numbers in a series:
decrease will be : 11.
Next term= 105–11= 94. Selected number series
iv) Gradual slow decrease. Likely to be arithmetical Which Of The Following Does Not Fit In The Se-
decrease. Check differences. They are 11, 9, 7, 5, ries?
3. Hence, next decrease will be 1. 1) 2, 6, 12, 27, 58, 121, 248
Next term =19 – 1 = 18. 2) 3, 9, 18, 54, 110, 324, 648
v) Sharp decrease and terms roughly being halved 3) 1, 1.5, 3, 6, 22.5, 78.75, 315
everytime. Checking with 2 as divisor the series 4) 190, 166, 145, 128, 112, 100, 91
is: 5) 895, 870, 821, 740, 619, 445, 225
Next term = (previous term - 8) ÷ 2. Next term = 5. 6) 1, 2, 6, 21, 86, 445, 2676
vi) Preliminary screening tells us that each term is 7) 864, 420, 200, 96, 40, 16, 6
obtained by adding 12,22,32,42,52 ...., respectively. 8) 4, 12, 30, 68, 146, 302, 622
Next term = 71 + 62= 107 9) 7, 10, 12, 14, 17, 19, 22, 22
vii) Sharp increase. The series is: × 1, × 2, × 3, × 4, × 5, 10) 196, 168, 143, 120, 99, 80, 63
.... Next term =360 × 6 = 2160 11) 258, 130, 66, 34, 18, 8, 6
viii) Sharp increase that slows down later on. (Ratios 12) 2, 6, 24, 96, 285, 568, 567
of successive terms rise sharply from 4 ÷ 3=1.3 to 13) 6072, 1008, 200, 48, 14, 5, 3
8 ÷ 4 = 2 to 17 ÷ 8 =2.125 and then start falling to 14) 2, 1, 10, 19, 14, 7, 16
33 ÷ 17  1.9 and then to 58 ÷ 33  1.8). Hence 15) 318, 368, 345, 395, 372, 422, 400, 449
likely to be addition of squared or cubed num- 16) 2807, 1400, 697, 347, 171, 84, 41, 20
bers. On checking the series is : +12,+22, +32, +42, 17) 824, 408, 396, 96, 44, 18, 5
+52, ..... Next term = 58 + 62 = 94. 18) 5, 7, 13, 25, 45, 87, 117
ix) Sharp increase with terms roughly doubling each 19) 2185, 727, 241, 79, 30, 7, 1
time. Likely to have geometrical nature with 2 as
6 K KUNDAN

20) 2, 3, 10, 15, 25, 35, 50, 63 48×4+8=200; 200 × 5+10=1010)


21) 2, 7, 28, 60, 126, 215, 344 14) 19;(2÷2=1; 1+9=10; 10÷2=5; 5+9=14; 14÷2 =7;
22) 0, 4, 19, 48, 100, 180, 294 7+9=16)
23) 1, 2, 7, 34, 202, 1420 15) 400;(There are two series; S1 =318+27=345;
24) 823, 734, 645, 556, 476, 378, 289 345+27 =372; 372+27=399; S2=368+27=395;
25) 1, 4, 11, 34, 102, 304, 911 395+27=422;——)
26) 5, 8, 20, 42, 124, 246, 736 16) 347;(20×2+1=41; 41×2+2 =84; 84×2+3=171;——
27) 13700, 1957, 326, 65, 16, 6, 2 —)
28) 1, 1.5, 3, 20.25, 121.5, 911.25, 8201.25 17) 396;[(824–8) ÷2 =408; (408-8) ÷2 =200; (200-8)
29) 3, 6, 10, 20, 33, 62, 94 ÷2=96;————]
30) 0, 6, 23, 56, 108, 184, 279 18) 87;(Add 2,6,12,20,30 and 42 to the successive
31) 1, 2, 6, 12, 66, 197, 786 numbers)
32) 1, 2, 6, 144, 2880, 86400, 3628800 19) 30;[(2185–4)÷3=727; (727–4)÷3=241; (241-
33) –1, 5, 20, 59, 119, 209, 335 4)÷3=79;—————]
34) 1, 2, 4, 8, 15, 60, 64 20) 25;[12+1=2; 22-1=3; 32+1=10; 42-1=15; 52+1=26;—
35) 49, 56, 64, 71, 81, 90, 100, 110 —————]
36) 1, 3, 10, 29, 74, 172, 382 21) 60;[13+1=2; 23-1=7; 33+1=28; 43-1=63;————
37) 25, 26, 24, 29, 27, 36, 33 ———]
38) 36, 54, 18, 27, 9, 18.5, 4.5 22) 19;[13–12=0; 23-22=4; 33-32=18; 43-42=48 ;53-
39) 144, 132, 125, 113, 105, 93, 84, 72, 61, 50 52=100;—————]
40) 3, 9, 36, 72, 216, 864, 1728, 3468 23) 202;[1×2-1=1; 1×3-1=2; 2×4-1=7; 7×5-1=34; 34×6-
41) 1, 1, 1, 4, 2, 1, 9, 5, 1, 16 1=203;——]
24) 476;[Hundred-digit of each number is decreas-
Answers ing by one and unit-and ten-digits are increasing
1) 6;(2×2+1=5; 5×2+2=12; 12×2+3=27; 27×2+4=58; by one.]
and so on) 25) 102;[1×3+1=4; 4×3-1=11; 11×3+1=34————]
2) 110;(Multiply by 3 and 2 alternately) 26) 20; [Series is ×2 - 2, × 3 - 2, × 2 - 2, ×3 -2, ......]
3) 6;(1×1.5=1.5; 1.5×2=3; 3×2.5=7.5; 7.5×3=22.5;— 27) 6; [Series is -1 ÷ 7, –1 ÷6, –1 ÷ 5, –1 ÷4, –1 ÷3, .....]
—————) 28) 3; [Series is ×1.5, ×3, × 4.5, × 6, × 7.5, × 9]
4) 128;(190-24=166; 166-21=145; 145-18=127; 127- 29) 33; [Series is ×2, × 1.5 + 1, × 2, × 1.5 + 1, × 2, × 1.5
15=112;——) + 1]
5) 445;(reduce the successive numbers by 5 2,
30) 108; [Series is 13  2 0 , 2 3  21 , 33  2 2 ,
72,92,112,———)
6) 86;( 1×1+1=2; 2×2+2=6; 6×3+3=21; 21×4+4=88;— 4 3  2 3 , 53  2 4..... ]
———) 31) 12; [Series is × 3- 1, × 4 - 2, × 3 - 1, × 4 - 2, .....]
7) 96;(Start from right end; 2(6+2)=16; 2(16+4)=40; 32) 6; [Series is × 1 × 2, ×2 × 3, ×3 × 4, × 4 × 5, × 5 × 6,
2(40+6)=92; 2(92+8)=200 ————) .....]
8) 302;(Add 8,18, 38, 78,158 and 318 to the succes-
sive numbers) 33) 20; [Series is 13  2, 2 3  3, 3 3  4, 4 3  5,
9) 19;(There are two series; S 1 =7,12,17,22;
53  6, ..... ]
S2=10,14,18,22)
10) 196;(Add 17,19,21,23,————to the successive 34) 8; [Series is × 2, +2, ×3, +3, ×4, +4, ........]
numbers from RE) 35) 71; [Series is
11) 8;(Add 4,8,16,32,64,128 to the successive num- 2 2 2 2 2 2
7 , 7  7, 8 , 8  8, 9 , 9  9, .... ]
bers from RE) 36) 172; [Series is ×2 + 1, ×2 + 4, ×2 + 9, ×2 + 16, ×2 +
12) 24;(2×6-6=6; 6×5-5=25; 25×4-4=96; 96×3-3=285;— 25, .......]
———)
13) 1008;(From RHS; 3×1+2=5; 5×2+4=14; 14×3+6=48; 37) 24; [ Series is  12  1,  2 2  2,32 ,3,..... ]
K KUNDAN 7

38) 18.5; [Series is ×1.5, ÷3, ×1.5, ÷3, ×1.5, ÷3] stant fraction). In this case the values are de-
39) 61; [Series is –12, –7, –12, –8, –12, –9, –12, –10, –
5 1
12,....] creased by or .
40) 3468; [Series is ×3, ×4, ×2, ×3, ×4, ×2, ×3,.....] 15 3
41) 5; [Series is 4 23 18 12 8 3
2 1 0 2 1 0 2 1 0 2 2) , , , , ,
1 , 1 , 1 , 2 , 2 , 2 , 3 , 3 , 3 , 4 ,.... ] 5 35 35 35 35 35
Soln: By the above rule if we change all the fractions
Some Unique Series with the same denominators, the series is
These series may be asked in examinations, so
28 23 18 12 8 3
you must be aware of them. , , , , , .
35 35 35 35 35 35
I. Series of Date or Time
1) Which of the following doesn’t fit into the se- 12
ries? We see that numerators decrease by 5, thus
35
5-1-96, 27-1-96, 18-2-96, 12-3-96, 2-4-96
Soln: Each successive date differs by 22 days. If you 13
should be replaced by .
recall that 96 is a leap year, you will find that 12- 35
3-96 should be replaced by 11-3-96. Now, we conclude that the above fractions de-
2) Which of the following doesn’t fit into the se-
ries? 5 1
crease successively by or .
5.40, 8.00, 10.20, 12.30, 3.00, 5.20 35 7
Soln: Each successive time differs by 2 hrs 20 min- 118 100 82 66 46 28
utes. So 12.30 should be replaced by 12.40. 3) , , , , ,
225 199 173 147 121 95
Note: Keep in mind that the problem of series may be
based on dates or times. Sometimes it doesn’t Soln: We see that all the denominators differ, so we
strike our mind and the question is solved can’t use the above rule. In this case usually, the
wrongly. numerators and denominators change in a defi-
nite pattern. Here, numerators decrease succes-
II. Fractional series sively by 18 whereas denominators decrease
Which of the following doesn’t fit into the se-
ries? 66
successively by 26. Thus should be re-
147
4 7 1 1 8
1) , , , ,
5 15 15 5 15 64
placed by .
Soln: Whenever you find that most of the fractions 147
have the same denominators, change all the de-
nominators to the same value. For example, in 12 15 18 21 24 27
4) , , , , ,
this question, the series becomes: 89 86 82 80 77 74
12 7 1 3 8 Soln: Numerators increase successively by 3 whereas
, , , , denominators decrease successively by 3. Thus
15 15 15 15 15
Now, it is clear that numerators must decrease 18 18
should be replaced by .
1 82 83
successively by 5. Therefore, should be re-
15 Note: More complicated questions based on fractions
are not expected in the exams because it is not
2 easy to find the solution in complicated cases.
placed by .
15
III. Some numbers followed by their LCM or HCF
Note: The above method is useful when the fractional 1) 1, 2, 3, 6, 4, 5, 6, 60, 5, 6, 7, ..... (Fill up the blank)
values are decreased by a constant value (a con- Soln : The series can be separated in three parts. 1, 2,
8 K KUNDAN

3, 6/ 4, 5, 6, 60/5, 6, 7 .... In each part fourth num- which one is different from others.
ber is LCM of first three numbers. Thus the an- 1) 22, 44, 88, 132, 165, 191, 242. Find the number
swer should be 210. which doesn’t fit in the above series (or group).
2) 8, 6, 24, 7, 3, 21, 5, 4, 20, ...., 9, 18 Soln: 191; Others are divisible by 11 or 191 is the
1) 1 2) 3 3) 4 4) 5 5) 6 single prime number.
Soln : 8, 6, 24/ 7, 3, 21/ 5, 4, 20/_ , 9, 18 2) Which one of the following series doesn’t fit
Third number in each part is LCM of first two into the series?
numbers. Thus, the answer should be 6. 29, 31, 37, 43, 47, 51, 53
3) 8, 4, 4, 7, 8, 1, 3, 9, 3, 2, 1, .... Soln: 51; All other are prime numbers.
1) 1 2) 2 3) 3 A note on Arithmetic Progressions. Arithmetic
4) 5 5) None of these progression is basically the arithmetic series.
Soln : 8, 4, 4/ 7, 8, 1/ 3, 9, 3/ 2, 1 ... A succession of numbers is said to be in Arith-
In each part, third number is HCF of first to num- metic Progression (A.P.) if the difference between any
bers. Thus our answer should be 1. term and the term preceding it is constant through-
out. This constant is called the common difference
IV. Some numbers followed by their product
(c.d.) of the A.P.
1) 2, 3, 6, 18, 108, 1844
To find the nth term of an A.P. Let the first term of
Which of the above numbers does not fit into
an A.P. be a and the common difference be d.
the series?
Then the A.P. will be a, a+d, a+2d, a+3d, ........
Soln: 2 × 3 = 6
Now first term t1 = a = a + (1-1)d
3 × 6 = 18
second term t2 = a + d = a + (2-1)d
6 × 18 = 108
third term t3 = a + 2d = a + (3-1)d
18 × 108 = 1944
fourth term t4 = a + 3d= a + (4-1)d
Thus, 1844 is wrong.
fifth term t5 = a + 4d= a + (5-1)d
2) 5, 7, 35, 8, 9, 72 , 11, 12, 132, _, 3, 6. Fill up the
Proceeding in this way, we get nth term tn= a+
blank.
(n-1)d
Soln: 5, 7, 35/ 8, 9, 72/ 11, 12, 132/ 2, 3, 6
Thus nth term of an A.P. whose first term is a and
In each group third number is the multiplication
common difference is d is given by tn = a + (n-1)d
of first and second. Thus our answer is 2.
V. By use of digit-sum Some Solved Examples
1) 14, 19, 29, 40, 44, 51, 59, 73 Ex. 1: Find the first five terms of the sequence for
Which of the above numbers doesn’t fit into the which t1=1,t2=2 and tn+2=tn+tn+1.
series? Soln: Given, t1 = 1, t2 = 2, tn+2 = tn+tn+1
Soln: Next number = Previous number + Digit-sum of Putting n = 1, we get t3 = t1+t2 = 1+2 = 3
previous number n = 2, we get t4 = t2+t3 = 2+3 = 5
Like, 19 = 14 + (4 + 1) n = 3, we get t5 = t3+t4 = 3+5 = 8
29 = 19 + (1 + 9) Thus the first five terms of the given sequence
40 = 29 + (2 + 9) are 1,2,3,5 and 8.
Thus, we see that 51 should be replaced by 52. Ex. 2: How many terms are there in the A.P.
2) 14, 5, 18, 9, 22, 4, 26, 8, 30, 3, __, __. Fill up the 20,25,30,...100?
blanks. Soln: Let the number of terms be n.
Soln: 1st, 3rd, 5th, 7th, .... numbers follow the pattern Given tn = 100, a = 20, d = 5, we have to find n.
of +4 (14 + 4 = 18, 18 + 4 = 22, ....). Whereas 2nd, Now tn = a+(n-1)d  100 = 20+ (n-1)5
4th, 6th are the digit-sums of their respective pre- or 80 = (n-1)5 or, n-1 = 16  n = 17.
vious number (5 = 1 + 4, 9 = 1 + 8), ....) Thus, our Ex. 3: A person was appointed in the pay scale of Rs.
answer is 34 and 7. 700-40-1500. Find in how many years he will
VI. Odd number out reach maximum of the scale.
Sometimes a group of numbers is written out of Soln: Let the required number of years be n.
K KUNDAN 9

Given tn = 1500, a = 700, d = 40, to find n. Soln: The first series is ÷6, ÷5, ÷4, ÷3
 tn = a+(n-1)d  a = 3720 ÷ 6 = 620, b = 620 ÷ 5 = 124,
 1500 = 700+(n-1)40 c = 124 ÷ 4 = 31, and finally d = 31 ÷ 3 = 10.33
or, (n-1)40 = 800 or, n-1 = 20 or, n = 21. Ex. 6: 27 44 71 108 155
Two-line number series 34 a b c d e
Nowadays this type of number series is also be- What value should replace e?
ing asked in examinations. Soln: The differences of two successive terms of
In this type of no. series one complete series is the series are 17, 27, 37, 47.
given while the other is incomplete. Both the series  a = 34 + 17 = 51, b = 51 + 27 = 78,
have the same definite rule. Applying the very defi- c = 78 + 37 = 115,
nite rule of the complete series, you have to deter- d = 115 + 47 = 162, and finally e = 162 + 57 = 219
mine the required no. of the incomplete series. For Ex. 7: 108 52 24 10 3
example: 64 a b c d e
Ex. 1: 4 14 36 114 460 What is the value of c?
2 a b c d e Soln: The series is –4 ÷ 2
Find the value of e.  a = (64 - 4) ÷ 2 = 30, b = (30 – 4) ÷ 2 = 13,
Soln: The first series is ×1 + 10, ×2 + 8, ×3 + 6, ×4 + 4, c = (13 – 4) ÷ 2 = 4.5
.... Ex. 8: –4 –2 1 8 31
–1 a b c d e
 a = 2 × 1 + 10 = 12, b = 12 × 2 + 8 = 32, c = 32
× 3 + 6 = 102, Find the value of b.
d = 102 × 4 + 4 = 412, and Soln: The series is repeated as ×2 + 6 and ×3 + 7
finally e = 412 × 5 + 2 = 2062 alternately.
Ex. 2: 5 6 11 28 71 160  a = –1 × 2 + 6 = 4 and b = 4 × 3 + 7 = 19
2 3 a b c d e Ex. 9: 5 8 41 33 57 42 61
What is the value of e? 3 4 a b c d e
Soln: The differences of two successive terms of Find the value of d.
the first series are 1, 5, 17, 43, 89, the sequence Soln: This is an alternate number series having two
of which is series: S1 = 5 41 57 61. The differences be-
tween two successive terms are 36 (= 62), 16 (=
0   12 , 13  2 2 , 23  32 , 33  4 2 , 43  52 . 42), 4 (= 22); and
 a = 3 + 5 = 8, b = 8 + 17 = 25, c = 25 + 43 = 68, S2 = 8 33 42
d = 68 + 89 = 157, and finally e = 157 + The differences between two successive terms
3 2 are 25 (= 52), 9 (=32)
( 5  6  125  36 =) 161 = 318  b = 4 + 25 = 29 and d = 29 + 9 = 38
Ex. 3: 1296 864 576 384 256 Remember: In such type of series the first and
1080 a b c d e the second term of the two series may and may
What should replace c? not have the similar relationship. As here, for
Soln: The first series is ÷ 3 × 2 the first series 8 - 5 = 3 but for the second
 a = 1080 ÷ 3 × 2 = 720, b = 720 ÷ 3 × 2 = 480, series 4 – 3 = 1  3. However, the series 3 a c
and finally c = 480 ÷ 3 × 2 = 320 e will always follow the same property as that
Ex. 4: 7 13 78 83 415 of the series S1 and the series 4 b d will always
3 a b c d e follow the same property as that of the series
Find the value of b. S2.
Soln: The first series is +6, ×6, +5, ×5 Ex. 10: 1 3 2 10 4 28
 a = 3 + 6 = 9 and b = 9 × 6 = 54 2 a b c d e
Ex. 5: 3240 540 108 27 9 What is the value of e?
3720 a b c d e Soln: This series is of grouping-type. Here we con-
What is the value of d? sider each two terms of the series separately
10 K KUNDAN

and each group separately. That is, for the first


516 = 83  4 , 733 = 9 3  4
series: the first group g1 = 1 and 3; g2 = 2 and
10; g3 = 4 and 28. Here for the two numbers of  
Now, we get 68  4 1 3  4 3
13
4
each group we have to find the relevant prop-
erty. For example g1, holds the property ×3, g2 So, b = (4 + 1 =) 53  4  129 and d = (5 + 1 =)
holds the property ×5 and g3 holds the prop- 63 + 4 = 220
erty ×7. Ex. 12: 2 5 17.5 43.75 153.125
The property of multiplication by 3, 5 and 7 is 1 a b c d e
a relevant property. Find the value of c.
Here, if we consider these groups in the way Soln: The series is ×2.5, ×3.5, ×2.5, ×3.5, ....
that the differences between the two numbers
 a = 1 × 2.5 = 2.5, b = 2.5 × 3.5 = 8.75 and c =
of the groups are 2, 8 and 24. It is not as rel- 8.75 × 2.5 = 21.875
evant as the former property of multiplication Here, after finding out the property of the given
by 3, 5, and 7. series as the direct repeated multiplication by
After determining the property between the
2.5 and 3.5 (the series is not of the type ×m 
two numbers of each group, to determine the
n that is, ×2.5 + 2, ×3.5 – 6, ×3 – 2 etc.), we also
property between the groups we consider the
observe that 1, the first no. of the second se-
first numbers only of each group in the fash-
ries is half of 2, the first no. of the first series.
ion 1, 2 and 4.
So, without finding a and b, we can directly
The property is ×2.
find out c as it is equal to half of the corre-
Now, we directly conclude e = 7 × d
sponding number of the first series. i.e. c =
and b = 2 × 2 = 4 and d = 2 × 4 = 8
Thus, e = 7 × 8 = 56. 43.75
= 21.875
Note: When the alternate no. series fails to deter- 2
mine the property of the given series, then the Ex. 13: 3 6 24 72 144 576
grouping type of series is applied. Here, for a 1 a b c d e
moment, if we consider for alternate no. series, What value should replace e?
we get Soln: The series is ×2, ×4, ×3, ×2, ×4, .....
S1 = 1 2 4. The property is ×2  a = 1 × 2 = 2, b = 2 × 4 = 8, c = 8 × 3 = 24, d =
S2 = 3 10 28. From merely these three numbers 24 × 2 = 48, e = 48 × 4 = 192
it is not proper to say that S2 holds a property The property of the first series is direct re-
of ×3 + 1 and ×3 – 2 (as 3 × 3 + 1 = 10 and 10 × peated multiplication by 2, 4 and 3.
3 – 2 = 28) or it holds the property of 3, 32 + 1 So, we can find out e directly as e = one-third
and 33 + 1 (as in this very case 3 should be of the corresponding number of the first se-
replaced by 31 + 1 i.e. 4). Thus we observe that 576
ries, i.e. = 192
the property of the given series cannot be ob- 3
tained by applying the method of the alternate Ex. 14: 575 552 533 518 507
no. series. So we proceed for the method of 225 a b c d e
the grouping no. series. Find the value of e.
Ex. 11: 220 96 347 77 516 60 733 Soln: The difference of the successive terms of the
68 a b c d e first series are 23, 19, 15, 11.
What is the value of d?  a = 225 – 23 = 202, b = 202 – 19 = 183, c = 183
Soln: Clearly, this no. series is of the type of alter- – 15 = 168,
nate no. series. So, to find out the value of d, d = 168 – 11 = 157, and finally e = 157 – (11 – 4=)
we are only concerned about the series 7 = 150.
S1 = 220 347 516 733 Note: When the series holds the property of the dif-
ference of the successive terms, you can di-
We observe that 220 = 6 3  4 , 347 = 7 3  4 , rectly proceed as follows:
K KUNDAN 11

Difference between the first terms of the two  a = 3 × 2 + 3 = 9, c = 9 × 2 + 3 = 21 and e = 21


series = 575 – 225 = 350 × 2 + 3 = 45.
 d = corresponding number of the first se- Also, in order to determine the value of d, we
ries i.e. 507 – 350 = 157 are only concerned with the series S2 for the
And then we have e = 157 – (11 – 4) = 150.
second given series as 11 b d.
Ex. 15: 15 31 11 23 5 11
 b = 11 × 2 + 7 = 29 and d = 29 × 2 + 7 = 65
21 43 a b c d e
Thus e = 45 and d = 65
What is the value of d?
Note: If we solve this sum by the process of group-
Soln: As the numbers are regularly increasing and
ing no. series:
then decreasing so you can consider for the
alternate no. series in the way: For the first given series: g1 = 5, 17; g 2 13, 41;
S1 = 15 11 5; the difference of the successive g 3 = 29, 89; the property is ×3 + 2.
terms are 4 and 6 and S2 = 31 23 11; the Also for the second given series g1 = 3, 11.
difference of the successive terms are 8 (= 4 × The property is ×3 + 2.
2) and 12 (= 6 × 2) Now the first numbers of the groups are 5, 13,
Now, in order to determine the value of d, we have 29, 61; the property is ×2 + 3.
to consider S2 for the second given series as 43 b  a = 3 × 2 + 3 = 9 and c = 9 × 2 + 3 = 21 and
e = 21 × 2 + 3 = 45.
d.
d = c × 3 + 2, i.e. 21 × 3 + 2 = 65
 b = 43 - 8 = 35 (As the numbers of S1 and Thus, we get the same result. However, the
S2 for the first given series are continuously grouping process fails in the previous solved
questions 9 and 11.
decreasing, we cannot have the difference of
You can check it yourself.
the successive term = 8 as b = 43 + 8 = 51)
We finally suggest you to apply the process
Finally, d = b – 12 = 35 – 12 = 23.
of alternate series first and only if it fails to
Note: Here, if we apply the process of grouping type
serve the purpose, then proceed for grouping-
no. series, for the first given series:
type number series.
g1 = 15, 31, g 2 = 11, 23; g 3 = 5, 11. Ex. 17: 9 19 39 79 159
The property between the numbers of each 7 a b c d e
group is ×2 + 1. What is the value of e?
For the second given series: g1 = 21, 43; the Soln: First method: The series is ×2 + 1, i.e. 9 × 2 + 1
= 19, 19 × 2 + 1 = 39, 39 × 2 + 1 = 79, and 79 × 2
property where is also ×2 + 1.
+ 1 = 159
Now, the first numbers of the groups are 15,
 a = 7 × 2 + 1 = 15, b = 15 × 2 + 1 = 31, c = 31
11, 5; the property is –4, –6, –8, ......
× 2 + 1 = 63, d = 63 × 2 + 1 = 127, and finally e
 a = 21 – 4 = 17
= 127 × 2 + 1 = 255
and c = 17 – 6 = 11 and then d = 11 × 2 + 1 = 23.
Other method: The difference between the
Thus, we get the same result.
successive terms of the first series are (19 – 9
Ex. 16: 5 17 13 41 29 89 61
=)10, (39 – 19 =) 20, (79 – 39 =) 40 and (159 – 79
3 11 a b c d e
=) 80. These numbers are in geometric pro-
What is the value of e and d?
gression having common ratio = 2. It is obvi-
Soln: S1 = 5 13 29 61, the property is ×2 + 3 ously a systematic sequence of numbers. Ap-
S2 = 17 41 89, the property is ×2 + 7 plying this very property for the second se-
ries, we get
In order to determine the value of e, we are
a = 7 + 10 = 17, b = 17 + 20 = 37, c = 37 + 40 =
only concerned with the series S1 for the sec- 77, d = 77 + 80 = 157 and e = 157 + (2 × 80 =) 160
ond given series as 3 a c e. = 317
12 K KUNDAN

Here we see that the values of each of a, b, c, d the series, your answer is ‘1’. If it is not necessary to
and e is entirely different from the values ob- interchange the position of the numbers to establish
tained by the first method. Both the methods the series, give 5 as your answer. Remember that
have their respective systematic properties, but when the series is established, the numbers change
which of the two has to be applied depends on from left to right (i.e. from the unmarked number to
the provided options. the last marked number) in a specific order.
In such a case, in exams, you have to answer Ex. 18: 17 16 15 13 7 –17
according to the suitability of the given op- (1) (2) (3) (4) (5)
tions. Soln: 5; The series is: -0!, –1!, -2!, -3! ....
Note: Whenever the chain rule is single throughout Ex. 19: 2 1 195 9 40 4
the series of the type ×m  n(where m and n (1) (2) (3) (4) (5)
are integers, e.g. ×2 + 1, ×2 - 3, ×4 + 6, ×3 + 7, Soln: 2; The series is: ×1 – 1, ×2 + 2, ×3 – 3, ×4 + 4 ....
etc.) this difference of answers will come; so Replace (2) with (4).
be cautious. In the chain rule when it is not Ex. 20: 16 15 29 343 86 1714
single (e.g. × 2 + 1 and then ×2 - 1 alternately, (1) (2) (3) (4) (5)
×3 + 2 and then ×2.5 alternately etc, or ×2 + 1, Soln: 3; The series is: ×1 – 12 , ×2 – 12 , ×3 – 12 , ×4 –
×2 + 3, ×2 + 5, ....., ×3 -7, ×3 - 14, ×3 - 21, ...., ×3,
×2, ×4 and again ×3, ×2, ×4 etc.) this difference 12 , ....
will not appear. Replace (3) with (4).
Directions (Ex. 18-22): In each of the follow- Ex. 21: 1728 1452 1526 1477 1607 1443
ing questions, a number series is established if the (1) (2) (3) (4) (5)
positions of two out of the five marked numbers are Soln: 1; The series is: -112, -92, -72, -52, ....
interchanged. The position of the first unmarked Replace (1) with (4).
number remains the same and it is the beginning of Ex. 22: 1 1 1 2 8 4
the series. The earlier of the two marked numbers (1) (2) (3) (4) (5)
whose positions are interchanged is the answer. For Soln: 4; The series is: 1, 12, 13, 2, 22, 23, ....
example, if an interchange of number of marked ‘1’ Replace (4) with (5).
and the number marked ‘4’ is required to establish

Exercise-1
Directions (Q. 1-5): In each of the following ques- 1. 120 15 105 21.875 87.5 17.5
tions, a number series is established if the positions 1) 2) 3) 4) 5)
of two out of the five marked numbers are inter- 2. 7 14 17 51 15 90
changed. The position of the first unmarked number 1) 2) 3) 4) 5)
remains the same and it is the beginning of the se- 3. 40 14 60 24 80 19
ries. The earlier of the two marked numbers whose 1) 2) 3) 4) 5)
positions are interchanged is the answer. For ex- 4. 15 240 71 192 111 160
ample, if an interchange of the number marked ‘1’ 1) 2) 3) 4) 5)
and the number marked ‘4’ is required to establish 5. 9 10 24 7 10 55
the series, your answer is ‘1’. If it is not necessary to 1) 2) 3) 4) 5)
interchange the positions of the numbers to estab- [SBI-PO-Exams-2000]
lish the series, give ‘5’ as your answer. Remember Directions (Q. 6-11): In each of the following
that when the series is established, the numbers questions, a number series is given. After the se-
change from left to right (i.e. from the unmarked ries, below it, a number is given followed by (1), (2),
number to the last marked number) in a specific (3), (4) and (5). You have to complete the series start-
order. ing with the given number following the sequence of
K KUNDAN 13

the given series. Then answer the questions given 14. 214 18 162 62 143 90 106
below it. 1) -34 2) 110 3) 10
6. 11 15 38 126 4) 91 5) 38
7 (1) (2) (3) (4) (5) 15. 160 80 120 180 1050 4725 25987.5
Which of the following will come in place of (3)? 1) 60 2) 90 3) 3564
1) 102 2) 30 3) 2140 4) 787.5 5) 135
4) 80 5) 424 16. 2 3 7 13 26 47 78
7. 2 3 8 27 1) 11 2) 13 3) 15
5 (1) (2) (3) (4) (5) 4) 18 5) 20
Which of the following will come in place of (5)? [SBI-PO-Exams-1999]
1) 184 2) 6 3) 925 Directions (Q. 17-21): In each of the questions
4) 45 5) 14 given below there is a mathematical series. After the
8. 2 3 9 40.5 series a number is being given followed by a, b, c, d
4 (1) (2) (3) (4) (5) and e. You have to create another series after under-
Which of the following will come in place of (2)? standing the sequence of the given series which starts
1) 486 2) 81 3) 3645 with the given number. Then answer the questions
4) 18 5) 6 given below.
9. 12 28 64 140 17. 1 9 65 393
37 (1) (2) (3) (4) (5) 2 (1) (2) (3) (4) (5)
Which of the following will come in place of (5)? Out of the following numbers which would come
1) 1412 2) 164 3) 696 in the place of c?
4) 78 5) 340 1) 490 2) 853 3) 731
10. 5 12 60 340 4) 729 5) None of these
7 (1) (2) (3) (4) (5) 18. 8 8 12 24
Which of the following will come in place of (4)? 36 (1) (2) (3) (4) (5)
1) 172 2) 5044 3) 1012 Out of the following numbers which would come
4) 20164 5) 28 in the place of e?
[BSRB-Mumbai-PO-1998] 1) 810 2) 36 3) 54
11. In the following number series, a wrong number 4) 108 5) None of these
is given. Find out that wrong number. 19. 424 208 100 46
11 24 66 164 462 1086 2414 888 (1) (2) (3) (4) (5)
1) 462 2) 164 3) 24 What number would come in the place of b?
4) 1086 5) 66 1) 20 2) 440 3) 216
[BSRB-Mumbai-PO-1998] 4) 56 5) None of these
Directions (Q. 12-16): One number is wrong in 20. 4 5 9.75 23.5
each of the number series given in each of the follow- 7 (1) (2) (3) (4) (5)
ing questions. You have to identify that number and What number would come in the place of d?
assuming that a new series starts with that number 1) 32.5 2) 271.5 3) 8
following the same logic as in the given series, which 4) 14.25 5) None of these
of the numbers given in (1), (2), (3), (4) and (5) given 21. 5 294 69 238
below each series will be the third number in the new 13 (1) (2) (3) (4) (5)
series? Which of the following numbers would come in
12. 3 5 12 38 154 914 4634 the place of e?
1) 1636 2) 1222 3) 1834 1) 246 2) 206 3) 125
4) 3312 5) 1488 4) 302 5) None of these
13. 3 4 10 34 136 685 4116 [Bank of Baroda-PO-1999]
1) 22 2) 276 3) 72 Directions (Q. 22-26): In each of the following
4) 1374 5) 12 questions a number series is given. Only one num-
14 K KUNDAN

ber is wrong in each series. Find out that wrong 1) 118.75 2) 118.25 3) 108.25
number, and taking this wrong number as the first 4) 118.125 5) None of these
term of the second series formed following the same 31. 25 146 65 114
logic, find out the third term of the second series. 39 (1) (2) (3) (4) (5)
22. 1 2 8 21 88 445 Which of the following numbers will come in
1) 24.5 2) 25 3) 25.5 place of (5)?
4) 25 5) None of these 1) 122 2) 119 3) 112
23. 6 7 18 63 265 1365 4) 94 5) None of these
1) 530 2) 534 3) 526 [Guwahati-PO-1999]
4) 562 5) None of these Directions (Q. 32-36): In each of the following
24. 7 23 58 127 269 555 questions a number series is given. A number in the
1) 263 2) 261 3) 299 series is suppressed by letter ‘A’. You have to find
4) 286 5) None of these out the number in the place of ‘A’ and use this num-
25. 5 4 9 18 66 195 ber to find out the value in the place of the question
1) 12 2) 25 3) 20 mark in the equation following the series.
4) 18 5) None of these 32. 36 216 64.8 388.8 A 699.84 209.952
26. 2 7 28 146 877 6140 A ÷ 36 = ?
1) 242 2) 246 3) 252 1) 61.39 2) 0.324 3) 3.24
4) 341 5) None of these 4) 6.139 5) 32.4
[SBI-Associates-PO-1999] 33. 42 62 92 132 A 242 312
Directions (Q. 27-31): In each of the following A + 14 = ? × 14
questions a number series is given. After the series, 6 5
a number is given followed by (1), (2), (3), (4) and (5). 1) 11 2) 14 3) 12
7 7
You have to complete the series starting with the 1
1
number given following the sequence of the given 4) 12 5) 12
2 6
series. Then answer the question given below it.
34. 4 7 12 19 28 A 52
27. 9 19.5 41 84.5
A² - 4 = ?
12 (1) (2) (3) (4) (5)
1) 1365 2) 1353 3) 1505
Which of the following numbers will come in
4) 1435 5) 1517
place of (3)?
35. 18 24 A 51 72 98 129
1) 111.5 2) 118.5 3) 108.25
4) 106.75 5) None of these 3 4
A   ?
28. 4 5 22 201 7 5
7 (1) (2) (3) (4) (5) 23 12
1) 12 2) 11 3) 12
Which of the following numbers will come in 35 35
place of (4)? 2 2
1) 4948 2) 4840 3) 4048 4) 14 5) 10
5 7
4) 4984 5) None of these
29. 5 5.25 11.5 36.75 3 3 9 9 27 27
36. A
3 (1) (2) (3) (4) (5) 8 4 16 8 32 16
Which of the following numbers will come in
A ?
place of (3)?
1) 34.75 2) 24.75 3) 24.5 3 6 6
4) 34.5 5) None of these 1) 2) 3)
2 8 4
30. 38 19 28.5 71.25
18 (1) (2) (3) (4) (5) 3 9
4) 5)
Which of the following numbers will come in 4 8
place of (4)?
K KUNDAN 15

[BSRB-Mumbai-PO-1999] 1) 12 2) 11 3) 75
Directions (Q. 37-41): In each of the following 4) 72 5) None of these
questions, a number series is given. After the se- 44. 3 4 12 45 190 1005 6066
ries, a number is given followed by (1), (2), (3), (4) 1) 98 2) 96 3) 384
and (5). You have to complete the series starting with 4) 386 5) None of these
the number given, following the sequence of the given 45. 6 10.5 23 59.5 183 644 2580
series. 1) 183.5 2) 182.5 3) 183
37. 15 16 25 50 4) 182 5) None of these
189 (1) (2) (3) (4) (5) 46. 2 7 19 43 99 209 431
Which of the following numbers will come in 1) 181 2) 183 3) 87
place of (5)? 4) 85 5) None of these
1) 354 2) 273 3) 394 [BSRB-Hyderabad-PO-1999]
4) 426 5) None of these Directions (Q. 47-51): In each of the following
38. 6 3.5 4.5 8.25 question a number series is given. A number in the
40 (1) (2) (3) (4) (5) series is suppressed by ‘P’ mark. First you have to
Which of the following numbers will come in find out the number in the place of the ‘P’ mark and
place of (3)? use this number to find out the answer of the ques-
1) 20.5 2) 21.5 3) 33.75 tion following the series.
4) 69.5 5) None of these 47. 188 186 P 174 158 126
39. 9 10 22 69
5 (1) (2) (3) (4) (5)
P  13  ?
Which of the following numbers will come in 1) 14.03 2) 14.10 3) 13.00
place of (2)? 4) 13.67 5) None of these
1) 15 2) 28 3) 14 48. 3.2 4.8 2.4 3.6 P 2.7
4) 45 5) None of these 0.06% of 54 ÷ P = ?
40. 2 10 27 60 1) 0.18 2) 1.62 3) 0.62
5 (1) (2) (3) (4) (5) 4) 18.0 5) 0.018
Which of the following numbers will come in 1 2 1 2
place of (2)? 49. 4 6 8 P 13 15
3 3 3 3
1) 39 2) 13 3) 34
30% of (P² + 13²) = ?
4) 38 5) None of these
1) 78.73 2) 87.00 3) 98.83
41. 5 149 49 113
4) 172.80 5) None of these
146 (1) (2) (3) (4) (5)
50. 220 182 146 114 84 58 P
Which of the following numbers will come in
place of (4)? 1
P ?
1) 290 2) 234 3) 254 256
4) 218 5) None of these
[BSRB-Calcutta-PO-1999] 1 1
Directions (Q. 42-46): In each of the following 1) 2 2) 2 3) 2
8 4
questions, a number series is given in which one
number is wrong. You have to find out that number 7
4) 3 5) None of these
and have to follow the new series which will be started 8
by that number. By following this, which will be the 51. 25 37 51 67 85 P 127
third number of the new series?
42. 1 2 6 33 148 765 4626 
20% of P  625  ? 
1) 46 2) 124 3) 18 1) 625 2) 550 3) 450
4) 82 5) None of these 4) 525 5) None of these
43. 2 9 5 36 125 648 3861 [NABARD-1999]
16 K KUNDAN

Directions (Q. 52-56): In each of the following 58. 150 299 601 1197 2401 4793
questions a number series is given. A number is given 60 (1) (2) (3) (4) (5)
after the series and then (1), (2), (3), (4) and (5) are What will come in place of (5)?
given. According to the given series, you have to form 1) 1917.2 2) 1913 3) 1922
a new series which begins with the given number, 4) 861 5) None of these
and then answer the question asked. 59. 3 9 81 6561
52. 6 3.0 4.5 2.25 2 (1) (2) (3) (4) (5)
40 (1) (2) (3) (4) (5) What will come in place of (4)?
Which of the following numbers will come in 1) 656 2) 4374 3) 1024
place of (3)? 4) 65536 5) None of these
1) 20.5 2) 21.5 3) 33.75 60. 3 7 24 101 512
4) 69.5 5) 15 5 (1) (2) (3) (4) (5)
53. 5 9 26 90 What will come in place of (3)?
13 (1) (2) (3) (4) (5) 1) 55 2) 149 3) 168
Which of the following numbers will come in 4) 752 5) None of these
place of (5)? 61. 2 6 40 1606 2579244
1) 2880 2) 2292 3) 1716 1 (1) (2) (3) (4) (5)
4) 3432 5) None of these What will come in place of (4)?
54. 4 9 25 103 1) 45 2) 95 3) 30489
3 (1) (2) (3) (4) (5) 4) 30633 5) None of these
Which of the following numbers will come in [BSRB-Bhopal-PO-2000]
place of (3)? 62. What will come in place of the question mark (?)
1) 391 2) 81 3) 91 in the following number series?
4) 79 5) None of these 2 9 28 65 ?
55. 6 10 32 126 1) 96 2) 106 3) 126
2 (1) (2) (3) (4) (5) 4) 130 5) None of these
Which of the following numbers will come in [BSRB-Bhopal-PO-2000]
place of (1)? Directions (Q. 63-67): In each of the following
1) 4 2) 6 3) 2 questions a number series is given. One term of the
4) 3 5) None of these series is denoted by ‘N’. You have to calculate the
56. 1260 628 312 154 value of ‘N’ and using the value of ‘N’ so obtained you
788 (1) (2) (3) (4) (5) have to replace the question mark (?) with suitable
Which of the following numbers will come in value.
place of (4)? 63. 99 163 N 248 273 289
1) 194 2) 45.5 3) 48
4) 72.5 5) None of these
2 N  17  ?
[BSRB-Chennai-PO-2000] 1) 20.5 2) 20.0 3) 21.5
Directions (Q. 57-61): In each of the following 4) 19.5 5) 21.0
questions a number series is given. After the series, 1
64. 6 2 6 1 6N
a number is given below it followed by (1), (2), (3), (4) 2
and (5). You are required to complete the series start- 150% of N = ?
ing with the number given following the sequence of 1) 9.00 2) 1.25 3) 1.80
the given series. Then answer the given questions. 4) 1.50 5) 1.875
57. 12 24 96 576 4608 1 1
9 (1) (2) (3) (4) (5) 65. N 12 9 7 6 5
5 7
What will come in place of (4)?
1) 1080 2) 676 3) 3608 18% of N + 24% of N = ?
4) 3456 5) None of these 1) 6.72 2) 8.40 3) 15.12
K KUNDAN 17

4) 7.56 5) 2.52 74. 510 254 N 62 30 14 6


66. 125 N 1127 1176 9408 9472 40% N + ? = 9²
N² - 2N = ? 1) 31.4 2) 29.8 3) 50.4
1) 25599 2) 22499 3) 25920 4) 30.6 5) None of these
4) 26243 5) 16899 [BSRB-Delhi-PO-2000]
67. 14.8 17.2 N 22.0 2.8 41.2 Directions (Q. 75-79): A number series is given
25% of 25 N = ? in each of the following questions. A number is given
1) 71.25 2) 77.5 3) 76.25 after the series followed by (1), (2), (3), (4) and (5).
4) 55.00 5) 167.50 First you have to understand how the series is formed
[BSRB-Bangalore-PO-2000] and then starting with that number you have to com-
68. In the following number series, one number is plete the second series. Now answer the given ques-
wrong. Find out the wrong number. tions.
3 10 35 172 885 5346 37471 75. 13 14 5 18 0.5
1) 10 2) 5346 3) 885 19 (1) (2) (3) (4) (5)
4) 35 5) 172 What would come in place of (5)?
[BSRB-Delhi-PO-2000] 1) 13.75 2) 27 3) 18.75
69. In the following number series, one number is 4) 6.75 5) None of these
wrong. Find out the wrong number. 76. 17 21.5 30.5 44 62
318 158 76 38 18 8 3 21 (1) (2) (3) (4) (5)
1) 38 2) 18 3) 158 What would come in place of (5)?
4) 318 5) 76 1) 84.5 2) 88.5 3) 86
[BSRB-Delhi-PO-2000] 4) 88 5) None of these
Directions (Q. 70-74): In each of the following 77. 1 8 10 35 136
questions a number series is given. A number in the 2 (1) (2) (3) (4) (5)
series is suppressed by letter ‘N’. You have to find What would come in place of (3)?
out the number in the place of ‘N’ and use this num- 1) 40 2) 42 3) 51
ber to find out the value in the place of the question 4) 49 5) None of these
mark in the equation following the series. 78. 12 26 11 36 9
70. 68 68.5 69.5 71 N 75.5 78.5 7 (1) (2) (3) (4) (5)
N × 121 + ? = 10000 What would come in place of (3)?
1) 1160 2) 1200 3) 1150 1) 7 2) 21 3) 4
4) 1180 5) None of these 4) 11 5) None of these
71. 19 20 24 33 49 74 N 159 79. 2 3 6 15 45
N² ÷ 10000 = ? 16 (1) (2) (3) (4) (5)
1) 121.0 2) 12.1 3) 1.21 What would come in place of (4)?
4) 0.121 5) None of these 1) 360 2) 120 3) 300
72. 51 43 N 30 25 21 18 4) 240 5) None of these
N² - 2N = ? [BSRB-Patna-PO-2001]
1) 1155 2) 1224 3) 1295 Directions (Q. 80-84): In each of the following
4) 1368 5) None of these number series only one number is wrong. If the
73. 2 5 14 41 122 365 N wrong number is corrected, the series gets estab-
lished following a certain logic. Below the series a
2
N – 16 % of 5670 – (?)² = 10² number is given followed by (1), (2), (3), (4), (5) and
3 (f). You have to complete the series following the same
1) 7 2) 149 3) 49 logic as in the given series after correcting the wrong
number. Now answer the following questions giving
4) 7 5) None of these the correct values for the letter in the questions.
80. 2 3 2 15 76 245 1434
18 K KUNDAN

3 (1) (2) (3) (4) (5) (f) 90. 10 15 24 35 54 75 100


What will come in place of (3)? 1) 35 2) 75 3) 24
1) 16 2) 22 3) 24 4) 15 5) 54
4) 21 5) None of these 91. 1 3 4 7 11 18 27 47
81. 1 2 8 33 148 740 4626 1) 4 2) 11 3) 18
4) 7 5) 27
2 (1) (2) (3) (4) (5) (f)
92. 3 2 3 6 12 37.5 115.5
What will come in place of (4)? 1) 37.5 2) 3 3) 6
1) 156 2) 164 3) 168 4) 2 5) 12
4) 152 5) None of these 93. 2 8 32 148 765 4626 32431
82. 2 4.5 11 30 93 312 1136 1) 765 2) 148 3) 8
1 (1) (2) (3) (4) (5) (f) 4) 32 5) 4626
What will come in place of (2)? 94. 2 3 11 38 102 229 443
1) 6 2) 81 3) 16.75 1) 11 2) 229 3) 120
4) 18.75 5) None of these 4) 38 5) 3
83. 2 14 18 46 82 176 338 [IBPS Bank PO Exam-2002]
4 (1) (2) (3) (4) (5) (f) Directions (Q. 95-99): What will come in place
What will come in place of (5)? of the question mark(?) in the following number se-
1) 238 2) 338 3) 218 ries?
4) 318 5) None of these 95. 2 9 30 105 ? 2195
84. 1 3 7 11 21 43 85 1) 432 2) 426 3) 440
4 (1) (2) (3) (4) (5) (f) 4) 436 5) None of these
What will come in place of (f)? 96. 3 4 12 45 ? 1005
1) 275 2) 279 3) 277 1) 152 2) 198 3) 144
4) 273 5) None of these 4) 192 5) None of these
[SBI-Bank-PO-2001] 97. 1 3 9 31 ? 651
Directions (Q. 85-89): In each of the following 1) 97 2) 127 3) 129
questions a number series is given with one wrong 4) 109 5) None of these
number. Find that wrong number. 98. 5 ? 4 7.5 17 45
85. 2 3 6 15 45 156.5 630 1) 3.5 2) 3 3) 2.5
1) 3 2) 45 3) 15 4) 2 5) None of these
4) 6 5) 156.5 99. 15 30 ? 40 8 48
86. 36 20 12 8 6 5.5 4.5 1) 10 2) 20 3) 18
1) 5.5 2) 6 3) 12 4) 12 5) None of these
4) 20 5) 8 [Andhra Bank SO Exam-2002]
87. 1 3 9 31 128 651 3313 Directions (Q. 100-104): In each of the follow-
1) 651 2) 128 3) 31 ing questions a number series is given. After the
4) 9 5) 3 series a number is given followed by (a), (b), (c), (d)
88. 2 3 10 40 172 855 5346 and (e). You have to complete the series starting with
1) 3 2) 855 3) 40 the number given and following the sequence of the
4) 172 5) 10 original series. Answer the questions that follow the
89. 5 8 16 26 50 98 194 series.
1) 8 2) 26 3) 50 100. 5 6 16 57 244 1245
4) 16 5) 98 2 (a) (b) (c) (d) (e)
[Corporation Bank PO-2002] What will come in place of (d)?
Directions (Q. 90-94): In each of these ques- 1) 46 2) 39 3) 156
tions a number series is given. Only one number is 4) 173 5) None of these
wrong in each series. You have to find out the wrong 101. 3 5 9 17 33 65
number. 7 (a) (b) (c) (d) (e)
K KUNDAN 19

What will come in place of (d)? 112. 7 9 16 25 41 68 107 173


1) 95 2) 51 3) 99 1) 107 2) 16 3) 41
4) 49 5) None of these 4) 68 5) 25
102. 7 4 5 9 20 52.5 113. 4 2 3.5 7.5 26.25 118.125
3 (a) (b) (c) (d) (e) 1) 118.125 2) 26.25 3) 3.5
4) 2 5) 7.5
What will come in place of (c)?
114. 16 4 2 1.5 1.75 1.875
1) 4.5 2) 2 3) 6
1) 1.875 2) 1.75 3) 1.5
4) 7 5) None of these
4) 2 5) 4
103. 3 10 32 111 460 2315
[Canara Bank PO Exam-2003]
2 (a) (b) (c) (d) (e)
Directions (Q. 115-119): In each of the follow-
What will come in place of (b)?
ing number series, a wrong number is given. Find
1) 29 2) 30 3) 26
out that number.
4) 28 5) None of these
115. 5 10 17 27 37 50 65
104. 5 8 6 10 7 12
1) 10 2) 17 3) 27
7 (a) (b) (c) (d) (e)
4) 37 5) 50
What will come in place of (c)?
116. 108 54 36 18 9 6 4
1) 14 2) 16 3) 9
1) 54 2) 36 3) 18
4) 11 5) None of these
4) 9 5) 6
[RBI Grade ‘B’ Officer’s Exam-2002]
117. 2 3 5 8 14 23 41 69
Directions (Q. 105-109): In each of these ques-
1) 5 2) 8 3) 14
tions a number series is given. Only one number is
4) 41 5) 69
wrong in each series. You have to find out the wrong
118. 0 1 9 36 99 225 441
number.
1) 9 2) 36 3) 99
105. 1 2 4.5 11 30 92.5 329
4) 225 5) 441
1) 92.5 2) 4.5 3) 11
119. 3 7.5 15 37.5 75 167.5 375
4) 2 5) 30
1) 167.5 2) 75 3) 37.5
106. 2 5 7 12 19 32 50
4) 15 5) 7.5
1) 7 2) 12 3) 32
[IBPS Jr Executive Exam-2002]
4) 19 5) 5
Directions (Q. 120-124): In each of the follow-
107. 2 13 65 271 817 1639 1645 ing questions a number series is given. After the
1) 13 2) 65 3) 271 series, a number is given followed by (a), (b), (c), (d)
4) 817 5) 1639 and (e). You have to complete the series starting with
108. 3 4 16 75 366 1945 11886 the number given following the sequence of the given
1) 16 2) 75 3) 366 series and answer the question given below the se-
4) 1945 5) 4 ries.
109. 2 14 91 546 3002 15015 120. 3 12 30 66 138 282
1) 15015 2) 91 3) 14 7 (a) (b) (c) (d) (e)
4) 3002 5) 546 What will come in place of (b)?
[NABARD Assistant Manager-2002] 1) 34 2) 70 3) 46
4) 62 5) None of these
Directions (Q. 110-114): In the following num-
ber series, a wrong number is given. Find out that 121. 2 3 10 39 172 885
wrong number. 5 (a) (b) (c) (d) (e)
110. 2 11 38 197 1172 8227 65806 What will come in place of (d)?
1) 11 2) 38 3) 197 1) 244 2) 175 3) 208
4) 1172 5) 8227 4) 196 5) None of these
122. 3 5 22 13.5 35 19
111. 16 19 21 30 46 71 107
1) 19 2) 21 3) 30 1 (a) (b) (c) (d) (e)
4) 46 5) 71 What will come in place of (a)?
1) 3 2) 2 3) 5
20 K KUNDAN

4) 4 5) None of these 4) 938 5) None of these


123. 2 3 7 25 121 721 133. 3 5 22 13.5 35 19
3 (a) (b) (c) (d) (e) 5 (a) (b) (c) (d) (e)
What will come in place of (c)? What will come in place of (c)?
1) 31 2) 49 3) 45 1) 16 2) 15.5 3) 14.5
4) 39 5) None of these 4) 13 5) None of these
124. 4 2 3 7.5 26.25 118.125 134. 9 10 16 57 212 1085
6 (a) (b) (c) (d) (e) 2 (a) (b) (c) (d) (e)
What will come in place of (c)? What will come in place of (c)?
1) 12.25 2) 11.5 3) 12.5 1) 37 2) 15 3) 18
4) 11.125 5) None of these 4) 25 5) None of these
[PNB Management Trainee Exam-2003] 135. 7 20 46 98 202 410
Directions (Q. 125-130): In each of the follow- 4 (a) (b) (c) (d) (e)
ing number series, a wrong number is given. Find What will come in place of (d)?
out the wrong number. 1) 166 2) 184 3) 146
4) 162 5) None of these
125. 2 3 6 18 109 1944 209952
[Bank of Maharashtra PO-2003]
1) 3 2) 6 3) 18 Directions (Q. 136-140): In each of the follow-
4) 109 5) 1944 ing number series only one number is wrong. Find
126. 1 3 6 11 20 39 70 out that wrong number.
1) 3 2) 39 3) 11 136. 24 14 26 17 28 16 30
4) 20 5) 6 1) 14 2) 17 3) 36 4) 16 5) 28
127. 2 13 27 113 561 3369 23581 137. 5 6 16 57 248 1245 7506
1) 13 2) 27 3) 113 1) 6 2) 16 3) 57 4) 248 5) 1245
4) 561 5) 3369 138. 2 9 32 105 436 2195 13182
128. 50 51 47 56 42 65 29 1) 2195 2) 105 3) 436 4) 9 5) 32
1) 51 2) 47 3) 56 139. 11 6 8 12 26 67.5 205.5
1) 8 2) 6 3) 26 4) 67.5 5) 12
4) 42 5) 65
140. 3 16 113 673 3361 13441 40321
129. 3 9 23 99 479 2881 20159 1) 13441 2) 673 3) 16
1) 9 2) 23 3) 99 4) 3361 5) 113
4) 479 5) 2881 [Allahabad Bank PO (Mains)-2004]
130. 2 4 5 8 13 21 34 Directions (Q. 141-145): What will come in place
1) 4 2) 5 3) 8 of the question mark (?) in the following number
4) 13 5) 21 series?
[SBI Bank PO Exam-2003] 141. 3 10 32 100 ?
Directions (Q. 131-135): In each of the follow- 1) 345 2) 460 3) 308
ing questions a number series is given. After the 4) 440 5) None of these
series a number is given followed by (a), (b), (c), (d) 142. 5 3 4 ? 38
and (e). You have to complete the series starting with 1) 8.5 2) 6 3) 7.5
the number given, following the sequence of the given 4) 8 5) None of these
series, and answer the questions given below the se- 143. 5 6 ? 57 244
ries. 1) 21 2) 16 3) 17
131. 4 2.25 3.25 7.125 18.25 51.875 4) 15 5) None of these
7 (a) (b) (c) (d) (e) 144. 3 10 21 ? 51
What will come in place of (b)? 1) 34 2) 32 3) 33
1) 4.125 2) 5.25 3) 6.75 4) 25 5) None of these
4) 4.75 5) None of these 145. 5 11 ? 55 117
132. 3 40 176 537 1078 1079 1) 21 2) 27 3) 23
1 (a) (b) (c) (d) (e) 4) 25 5) None of these
What will come in place of (e)? [Syndicate Bank PO-2004]
1) 839 2) 738 3) 829 Directions (Q. 146-150): In each of the
K KUNDAN 21

following series, only one number is wrong. You have 4) 12 5) 9


to find out that wrong number. 159. 15 28 45 64 85 116 147
146. 27 17 22 42 92 237.5 718.5 1) 85 2) 28 3) 64
1) 17 2) 42 3) 237.5 4)116 5) 45
4) 22 5) 92 160. 1 4 27 258 3125 46656 823543
1) 27 2) 258 3) 3125
147. 7 11 18 35 67 131 259
4) 4 5) 46656
1) 18 2) 35 3) 11 [Andhra Bank PO-2005]
4) 131 5) 67 Directions (Q. 161-165): In each of the following
148. 7 4 5 9 18 52.5 160.5 number series only one number is wrong. Find
1) 4 2) 5 3) 9 out that wrong number.
4) 52.5 5) 18 161. 4 11 25 53 108 221 445
149. 5 6 33 158 374 1230 2561 1) 221 2) 108 3) 53
1) 374 2) 33 3) 6 4) 11 5) 25
4) 1230 5) 158 162. 7 8 16 46 107 232 448
150. 12 11 18 45 196 795 4734 1) 8 2) 16 3) 46
1) 795 2) 196 3) 45 4) 107 5) 232
4) 18 5) 11 163. 1 8 28 99 412 2078 12462
[Oriental Bank of Commerce PO-2005] 1) 99 2) 28 3) 8
Directions (Q. 151-155): In each of these ques- 4) 2078 5) 412
tions a number series is given. Only one number is 164. 2 5 13 55 271 1632 11411
wrong in each series. You have to find out the wrong 1) 271 2) 13 3) 1632
number. 4) 5 5) 55
151. 289 288 279 254 205 128 3 165. 16 8 12.5 30 105 472.5 2598.75
1) 288 2) 254 3) 205 1) 12.5 2) 8 3) 30
4) 128 5) None of these 4) 472.5 5) 105
152. 2 10.5 53 265.5 1327.5 6640.5 [Syndicate Bank (RDO)-2005]
1) 10.5 2) 1327.5 3) 6640.5 Directions (Q. 166-170): In the following num-
4) 265.5 5) None of these ber series only one number is wrong. Find out the
153. 16 18 32 52 86 138 224 wrong number.
1) 52 2) 86 3) 138 166. 14 13 22 55 212 1035
4) 18 5) None of these 1) 55 2) 13 3) 212
154. 6 35 173 689 2063 4125 4115 4) 22 5) None of these
1) 689 2) 35 3) 4125 167. 217 224 213 226 210 228
4) 2063 5) None of these 1) 213 2) 226 3) 210
155. 4 5 18 81 385 2065 4) 228 5) None of these
1) 385 2) 18 3) 2065 168. 153 495 712 837 901 928
4) 81 5) None of these 1) 712 2) 837 3) 901
[Punjab National Bank (MT)-2005] 4) 928 5) None of these
Directions (Q. 156-160): In the following num- 169. 11 42 214 1045 4148 12417
ber series only one number is wrong. You have to
1) 42 2) 214 3) 1045
find out the wrong number.
156. 25 23 27 19 34 3 67 4) 4148 5) None of these
1) 23 2) 3 3) 34 170. 488 245 124 64 35 20.25
4) 19 5) 27 1) 124 2) 64 3) 245
157. 1 2 12 63 316 1704 10446 4) 35 5) None of these
1) 1704 2) 316 3) 63 [IDBI PO-2005]
4) 12 5) 2 Directions (Q. 171-175): In each of the follow-
158. 7 9 12 28.5 73 205 645 ing number series only one number is wrong. Find
1) 205 2) 28.5 3) 73 out that wrong number.
22 K KUNDAN

171. 3 5 11 29 87 314 184. 15 12 17 10 ? 8 25 6


1) 87 2) 11 3) 29 1) 3 2) 7 3) 21
4) 314 5) None of these 4) 19 5) None of these
172. 12 13 20 48 112 237 185. 1 ? 27 64 125
1) 13 2) 237 3) 20 1) 8 2) 4 3) 6
4) 9 5) None of these
4) 48 5) None of these
186. 2 5 7 12 19 31 50 ?
173. 8.1 9.2 17.3 26.5 43.8 71.5 114.1 1) 53 2) 81 3) 69
1) 17.3 2) 26.5 3) 43.8 4) 74 5) None of these
4) 9.2 5) None of these 187. 1 6 36 240 1960 ?
174. 10 16 48 320 4864 154628 1) 19660 2) 3680 3) 36800
1) 154628 2) 4864 3) 320 4) 19600 5) None of these
4) 16 5) None of these [Corporation Bank PO-2006]
175. 48 24 72 34 108 54 Directions (Q. 188-192): What should come in
1) 54 2) 34 3) 72 place of the question mark(?) in the following num-
4) 108 5) None of these ber series?
[Bank of Baroda PO-2005] 188. 24 ? 109 134 150 159
1) 71 2) 65 3) 86
Directions (Q. 176-180): In each of the follow-
4) 53 5) None of these
ing number series one of the given numbers is wrong. 189. 17 9 10 ? 35 90
Find out the wrong number. 1) 21 2) 27.5 3) 19
176. 439 778 1456 2812 5624 10948 4) 16.5 5) None of these
1) 5624 2) 1456 3) 778 190. 3 20 78 332 1680 ?
4) 2812 5) None of these 1) 8410 2) 9836 3) 10098
4) 1150 5) None of these
177. 156 468 780 1094 1404 1716
191. 13 30 66 140 ? 592
1) 468 2) 1094 3) 1716 1) 210 2) 290 3) 428
4) 780 5) None of these 4) 430 5) None of these
178. 5040 3014 1814.4 1088.64 653.184 391.9104 192. 3 5 15 ? 1125 84375
1) 1088.64 2) 391.9104 3) 1814.4 1) 75 2) 20 3) 45
4) 80 5) None of these
4) 653.184 5) None of these [Central Bank of India PO-2006]
179. 113 130 164 215 293 368 Directions (Q. 193-197): What should come in
1) 215 2) 130 3) 164 place of the question mark (?) in the following num-
4) 293 5) None of these ber series?
180. 36 54 135 472.15 2126.25 11694.375 193. 15 18 16 19 17 20 ?
1) 135 2) 54 3) 472.15 1) 23 2) 22 3) 16
4) 11694.375 5) None of these 4) 18 5) None of these
194. 1050 420 168 67.2 26.88 10.752 ?
[Bank of Maharashtra (SO)-2006]
Directions (Q. 181-187): What will come in place 1) 4.3008 2) 6.5038 3) 4.4015
of the question mark (?) in the following number 4) 5.6002 5) None of these
series? 195. 0 6 24 60 120 210 ?
181. 12 14 17 13 8 14 21 13 4 ? 1) 343 2) 280 3) 335
1) 14 2) 13 3) 15 4) 295 5) None of these
4) 2 5) None of these 196. 32 49 83 151 287 559 ?
182. 4 6 12 30 90 315 ? 1) 1118 2) 979 3) 1103
1) 945 2) 1102 3) 1260 4) 1120 5) None of these
4) 1417.5 5) None of these 197. 462 552 650 756 870 992 ?
183. 25 16 ? 4 1 1) 1040 2) 1122 3) 1132
1) 3 2) 6 3) 12 4) 1050 5) None of these
4) 18 5) None of these
[Andhra Bank PO-2006]
K KUNDAN 23

Directions (198-202): What will come in place 1) 19.2466 2) 17.2244 3) 16.8824


of the question mark (?) in the following number 4) 18.6624 5) None of these
series? 211. 9 4.5 4.5 6.75 13.5 33.75 ?
198. 3 20 87 392 ? 1) 101.25 2) 103.75 3) 99.75
1) 1612 2) 1963 3) 847 4) 105.50 5) None of these
212. 705 728 774 843 935 1050 ?
4) 2025 5) None of these 1) 1190 2) 1180 3) 1185
199. 12 28 75 316 1555 ? 4) 1187 5) None of these
1) 9366 2) 9294 3) 7811 [Bank of Baroda PO-2007]
4) 9336 5) None of these Directions (Q. 213-217): What should come in
200. 112 111 119 92 156 31 ? place of question mark (?) in the following number
1) 375 2) 287 3) 387 series?
4) 247 5) None of these 213. 1 4 27 256 3125 46656 ?
201. 1 15 16 31 47 78 125 ? 1) 117649 2) 279936 3) 705894
1) 172 2) 203 3) 139 4) 16807 5) 823543
4) 167 5) None of these 214. 30 46 78 126 190 270 ?
202. 55 60 67 78 91 108 ? 1) 356 2) 366 3) 382
1) 125 2) 121 3) 127 4) 398 5) 414
4) 89 5) None of these 215. 380 465 557 656 762 875 ?
[IOB PO-2006] 1) 955 2) 1015 3) 975
Directions (Q. 203-207): What will come in place 4) 995 5) 1025
of the question mark (?) in the following series? 216. 1250 500 200 80 32 12.8 ?
203. 12 22 69 272 1365 ? 1) 5.12 2) 6.4 3) 4.3
1) 8196 2) 8184 3) 8195 4) 6.02 5) 5.16
4) 6830 5) None of these 217. 23 26 24 27 25 28 ?
204. 1 ? 27 64 125 1) 27 2) 29 3) 26
1) 14 2) 4 3) 9 4) 24 5) 21
4) 8 5) None of these [IDBI Bank Officers-2007]
205. 104 153 189 214 ? Directions (Q. 218-222): In each of the follow-
1) 239 2) 225 3) 264 ing questions a series of numbers is given. Only one
4) 235 5) None of these number in the series is wrong. Find out that wrong
206. 15 17 32 49 81 130 ? number.
1) 179 2) 211 3) 194 218. 15 16 20 28 45 70 106
4) 226 5) None of these 1) 16 2) 20 3) 28
207. 15 17 21 29 45 77 ? 4) 45 5) 70
1) 109 2) 125 3) 141 219. 2 8 26 90 372 1876 11226
4) 173 5) None of these 1) 8 2) 26 3) 90
[Andhra Bank IT Officer-2007] 4) 372 5) 1876
Directions (Q. 208-212): What should come in 220. 5 7 11 19 36 67 131
place of question mark (?) in the following number 1) 7 2) 11 3) 19
series? 4) 36 5) 67
208. 13 14 30 93 376 1885 ? 221. 8 9.5 11.5 14.5 17 20.5 24.5
1) 10818 2) 10316 3) 11316 1) 17 2) 14.5 3) 9.5
4) 11318 5) None of these 4) 11.5 5) 20.5
209. 4 6 9 13.5 20.25 30.375 ? 222. 11 12 22 47 111 236 452
1) 40.25 2) 45.5625 3)42.7525 1) 12 2) 22 3) 47
4) 48.5625 5) None of these 4) 111 5) 236
210. 400 240 144 86.4 51.84 31.104 ? [PNB Management Trainee-2007]
24 K KUNDAN

Exercise-2
Directions (Q. 1-5): In each of the following ques- 8. 15 17 20 29 45
tions, a number series is given based on a certain a b c d e
sequence. Below it another number series is given. What no. should come in place of e?
You have to follow the same sequence in the second 1) 35 2) 39 3) 47
number series and then answer the question below 4) 69 5) None of these
it. 9. 20 41 78 153 304
1. 7 35 40 240 246 a b c d e
8 a b c d e What no. should come in place of d?
Find the value of c. 1) 150 2) 140 3) 135
1) 260 2) 280 3) 290 4) 129 5) None of these
4) 300 5) None of these 10. 136 64 26 10 1
2. 8 5 5 8 23 a b c d e
9 a b c d e What no. should come in place of c?
What is the value of d? 1) 10 2) 0.75 3) 5
1) 49 2) 52 3) 47 4) 0.5 5) None of these
4) 42 5) None of these Directions (Q. 11-15): In each of the following
3. 41 73 95 109 117 questions, a number series is given. After the se-
9 a b c d e ries, below it, a number is given followed by a, b, c, d
What should replace d? and e. You have to complete the series starting with
1) 109 2) 80 3) 79 the number given following the sequence of the given
4) 85 5) None of these series. Then, answer the questions given below it.
4. 6 10 18 34 66 11. 24 12 12 18
7 a b c d e 18 a b c d e
Find the value of e. What value should come in place of d?
1) 131 2) 132 3) 133 1) 13.5 2) 15 3) 21
4) 134 5) None of these 4) 24 5) None of these
5. 5 13 23 67 131 12. 7 7 13 33
4 a b c d e 13 a b c d e
What is the value of c? What should replace e?
1) 47 2) 48 3) 49 1) 135 2) 76 3) 142
4) 50 5) None of these 4) 87 5) None of these
Directions (Q. 6-10): In each of the following 13. 8 29 152 1073
questions a number series is given, in which a wrong 12 a b c d e
no. is given. Find out the wrong no. and start the What value should come in place of b?
second series with that no. Answer the questions 1) 186 2) 198 3) 204
below it. 4) 212 5) None of these
6. 2 5 11 27 58 14. 9 22 50 110 236
a b c d e 5 a b c d e
What no. should come in place of d? What should come in place of d?
1) 29 2) 49 3) 79 1) 172 2) 184 3) 196
4) 99 5) None of these 4) 208 5) None of these
7. 1 2 5 10 677 15. 731 649 714 664
a b c d e 585 a b c d e
What no. should come in place of c? What value would replace e?
1) 51 2) 81 3) 101 1) 429 2) 512 3) 521
4) 621 5) None of these 4) 492 5) None of these
K KUNDAN 25

Directions (Q. 16-20): One number is wrong in What should come in place of  ?
each of the number series given in each of the follow-
ing questions. You have to identify that number and 1) 2 2) 0 3) 4
assuming that a new series starts with that number 4) -2 5) 6
following the same logic as in the given series, which 25. 13 18 28 48
of the numbers given in 1, 2, 3, 4 and 5 given below 9    
each series will be the third number in the new se-
What should come in place of  ?
ries?
16. 2, 3, 6, 15, 46, 157.5, 630 1) 74 2) 84 3) 76
1) 240 2) 238 3) 345 4) 96 5) 116
4) 138 5) 328 Directions (Q. 26-30): In each of the following
17. 2, 3, 8, 28, 112, 565 questions a no. series is given. After the series, a no.
1) 56 2) 60 3) 58 is given below it, followed by a, b, c, d and e. You have
4) 62 5) 70 to complete the series starting with the no. given and
18. 3, 6, 9, 17, 33, 65, 129 follow the same property as in the given no. series.
1) 21 2) 22 3) 23 Then answer the questions given below it.
4) 20 5) 18 26. 2 -3 -33 -178 -723
19. 582, 605, 588, 611, 634, 617, 600 -3 a b c d e
1) 642 2) 632 3) 634 What value should come in place of b?
4) 630 5) 640 1) -178 2) -243 3) 178
20. 60, 121, 131, 260, 284, 572, 601 4) -263 5) None of these
1) 531 2) 542 3) 521 27. 7 8 4 13 -3 22
4) 522 5) 540 13 a b c d e
Directions (Q. 21-25): In each of the following What value should come in place of d?
questions a number series is given based on a cer- 1) 3 2) -6 3) 22
tain sequence. Below it a number is given followed by 4) -42 5) None of these
28. 374 371 362 383 338 407 302
 ,  ,  ,  . You have to follow the same sequence 235 231 a b c d e
and answer the questions. What is the value of b?
21. 5 -30 -210 1680 1) 335 2) 312 3) 286
3     4) 243 5) None of these
What should come in place of  ? 29. 0.75 2.5 7 17 38
0.175 a b c d e
1) -126 2) 1008 3) 1224
What would be value of c?
4) -676 5) 896
1) 10.5 2) 12.4 3) 14.5
22. 1 5 14 30
4) 15.7 5) None of these
3     30. 332 336 171 59 15.75
What should come in place of  ? 226 a b c d e
1) 37 2) 45 3) 57 What value should come in place of c?
4) 63 5) 71 1) 112 2) 68 3) 45
23. 5 12 60 340 4) 42.5 5) None of these
7    Directions (Q. 31-35): In each question a num-

ber series is given which contains a wrong term.
What should come in place of  ? Find the wrong no. B (say) and answer the question
1) 172 2) 5044 3) 3012 given below.
4) 20164 5) 9068 31. 6 7 10 20 46
24. 7 10 24 88 What will be the value of (40 ÷ B - 2)?
3     1) 4 2) 0 3) 7
4) 5 5) None of these
26 K KUNDAN

32. 2.5 8 19 40 85 4) 3800 5) 4000


What will be the value of (B2 - 3B)/37? 42. 12 12.25, 13.25, 15.25 19.50 25.75
1) 25 2) 30 3) 35 What is   0.25 of 180% ?
4) 40 5) None of these
1) 36 2) 27 3) 30
33. 5 11 21 44 85 171
4) 40 5) 45
What will be the value of (3B + 17) ?
43. 356 397 447 503 570
1) 40 2) 70 3) 149
What is 20% of 5  ?
4) 261 5) None of these
1) 444 2) 440 3) 450
34. 18 27 55 135 405 1417.5
4) 480 5) 447
What will be the value of (3.5 B + 10)?
44. 28 327 464 5120 6216
1) 202.5 2) 73 3) 10.45
What is  ÷ 4 ?
4) 482.5 5) None of these
1) 1280 2) 2460 3) 2560
35. 4 3 11 18 39 73
4) 630 5) 780
What will be the value of (B2 - 2B)?
45. 4 5 12 39 205 805
1) 312 2) 288 3) 144
4) 280 5) None of these What is  2  10  25 ?
Directions (Q. 36-40): In each of the following 1) 4000 2) 40,000 3) 48760
series you have to find the wrong term. Now, taking 4) 57680 5) None of these
wrong term as B, answer the following questions. Directions (Q. 46-50): In each of the following
36. 2 5 19 39 122 243 questions, a number series is given. Only one num-
What is the value of 3B ÷ 8? ber is wrong in this series. Find out that wrong num-
1) 42 2) 41.75 3) 144.25 ber, and taking this wrong number as the first term
4) 45.75 5) 127 of the second series formed, following the same logic,
37. -3 1 9 25 56 121 find out the fourth term of the second series.
What is the value of 20% of 5B? 46. 80 20 24 45 180
1) 121 2) 56 3) 25 1) 18 2) 13.5 3) 17.5
4) 9 5) None of these 4) 22 5) 24.50
38. 4 10 22 47 94 190 47. 18 16 36 66 244 1190
Starting from B, write the second term of the se- 1) 202 2) 224 3) 174
ries which is based on the same logic. 4) 176 5) 182
1) 224 2) 94 3) 194 48. 30 10 15 37.50 131.25
4) 296 5) None of these 1) 56.25 2) 58.60 3) 62.50
39. 120 59 27 11.5 3.75 4) 70.625 5) 48.75
What is the value of B ÷ 2 + 21? 49. 583 511 450 413 383 363
1) 50.5 2) 45.5 3) 140.5 1) 290 2) 380 3) 250
4) 124.5 5) None of these 4) 280 5) 230
40. 4 14 38 114 460 50. 4374 729 234 162 216 576
Write the second term of a similar series starting 1) 13 2) 9.66 3) 12.50
with B. 4) 7.86 5) 8.66
1) 114 2) 48 3) 104 Directions (Q. 51-55): In each of the following
4) 96 5) None of these questions, a number series is given. Only one num-
Directions (Q. 41-45): In each of the following ber is wrong in this series. Find out that wrong num-
questions, a number series is given in which one ber, and taking this wrong number as the first term
number is wrong (say  ). You have to identify that of the second series formed, following the same logic,
number and answer the following questions. find out the fourth term of the second series.
41. 3 5.5 16.75 64 293.3125 51. 1 5 17 53 113 229
What is   42 ? 1) 566 2) 589 3) 661
1) 3200 2) 3400 3) 3600 4) 680 5) None of these
K KUNDAN 27

52. 15 16 16 22 19 28 61. 1 1.75 6.5 26.25 117 603.75


1) 34 2) 32 3) 30 1) 2) 3) 4) 5)
4) 28 5) None of these 62. 0 81 98 343 6705 1336
53. 664 617 537 464 418 337 1) 2) 3) 4) 5)
1) 454 2) 490 3) 504 63. 962 121 313 225 754 850
4) 512 5) 520 1) 2) 3) 4) 5)
54. 2 4 3 6 5 9 6.75 64. 60 32 40 160 120 192
1) 12.25 2) 13.50 3) 15.25 1) 2) 3) 4) 5)
4) 15 5) 16.25 65. 100 47 94 55 92 51
55. 16 16 18 20 8 28 -2 1) 2) 3) 4) 5)
1) 16 2) 22 3) 24 Directions (Q. 66-70): In each of the following
4) 25 5) None of these questions a no. series is given. After the series, a no.
Directions (Q. 56-60): In each of the following is given below it, followed by a, b, c, d and e. You have
questions, a number series is given. Only one num- to complete the series starting with the no. given and
ber is wrong in this series. Find out that wrong num- following the same property as in the given no. se-
ries. Then answer the questions below it.
ber, and taking this wrong number as the first term
66. 1 5 14 39 88 209
of the second series formed, following the same logic, -13 (a) (b) (c) (d) (e)
find out the fourth term of the second series. What should come in place of (e)?
56. 256 257 518 1560 6268 1) 195 2) 185 3) 74
1) 3124 2) 9387 3) 5864 4) 101 5) 155
4) 6932 5) None of these 67. 2520 280 2240 320 1920 384
57. 10 1 0.3 1.5 0.105 0.0945 504 (a) (b) (c) (d) (e)
1) 0.225 2) 0.0450 3) 0.0225 What should come in place of (c)?
4) 0.4976 5) 2.250 1) 448 2) 384 3) 74
58. 97 97 91 113 69 4) 120 5) 64
1) 117 2) 123 3) 125 68. 659 130 491 266 387 338
1009 (a) (b) (c) (d) (e)
4) 127 5) 137
What should come in place of (d)?
59. 2 10 72 363 1093 1098 1) 616 2) 737 3) 762
1) 678 2) 768 3) 468 4) 726 5) None of these
4) 568 5) 758 69. 12 13 -1 122 145 1454
60. 112 56 20 6 0 92 (a) (b) (c) (d) (e)
1) -4 2) -5 3) -7 What should come in place of (b)?
4) -8 5) -1 1) 132 2) 149 3) 159
Direction (Q. 61-65): In each of the following 4) 169 5)None of these
questions, a number series is established if the posi- 70. 67 82 69 80 71 78
tions of two out of the five marked numbers are in- 123 (a) (b) (c) (d) (e)
terchanged. The position of the first unmarked num- What should come in plce of (e)?
1) 138 2) 125 3) 136
ber remains the same and it is the beginning of the
4) 127 5) 134
series. The earlier of the two marked numbers whose
Directions (Q. 71-75): In each of the following
positions are interchanged is the answer. For ex-
questions, a number series is established if the posi-
ample, if an interchange of the number marked ‘1’
tions of two out of the five marked numbers are in-
and the number marked ‘4’ is required to establish
terchanged. The position of the first unmarked num-
the series, your answer is ‘1’. If it is not necessary to
ber remains the same and it is the beginning of the
interchange the position of the numbers to establish
series. The earlier of the two marked numbers whose
the series, give ‘5’ as your answer. Remember that
positions are interchanged is the answer. For ex-
when the series is established, the numbers change
ample, if an interchange of the number marked ‘1’
from left to right (i.e. from the unmarked number to and the number marked ‘4’ is required to establish
the last marked number) in a specific order.
28 K KUNDAN

the series, your answer is ‘1’. If it is not necessary to Directions (Q. 81-85): In each of the following
interchange the positions of the numbers to estab- questions a number series is given. A number in the
lish the series, give 5 as your answer. Remember series is suppressed by letter ‘A’. You have to find
that when the series is established, the numbers out the number in the place of ‘A’ and use this num-
change from left to right (i.e. from the unmarked ber to find out the value in the place of the question
mark in the equation following the series.
number to the last marked number) in a specific
81. 300 A 240 16 160 32
order. 12% of A + 0.56 = ?
71. 1142 1144 1132 1148 1140 1164 1) 14.96 2) 15 3) 2
(1) (2) (3) (4) (5) 4) 2.36 5) 1
72. -5 225 10 15 40 0 82. 41 42 23 72 A 115
(1) (2) (3) (4) (5)
73. 389 100 269 188 212 213 4 6
? 3 of 1  A
(1) (2) (3) (4) (5) 5 19
74. -3 -2 0 3 28 705 1) 37 2) 27 3) 22
(1) (2) (3) (4) (5) 4) 17 5) 47
75. 10395 945 8505 2025 6075 1215 83. 2529 3058 3787 4748 5973 A
(1) (2) (3) (4) (5) A+6 =?
1) 7500 2) 7367 3) 7600
Directions (Q. 76-80): In each of the following
4) 7486 5) None of these
questions a no. series is given. After the series, a no. 84. 107 322 1287 A 38615
is given below it, followed by a, b, c, d and e. You have A - 10 = ?
to complete the series starting with the no. given and 1) 6436 2) 6446 3) 6416
following the same property as in the given no. se- 4) 6410 5) None of these
ries. Then answer the questions below it. 1 2 1 2
76. -1 0 -4 -27 -448 85. 4 6 8 A 13 15
3 3 3 3
15 (a) (b) (c) (d) (e) 30% of (A2 + 132) = ?
What should come in place of (c)? 1) 78.73 2) 87 3) 98.83
1) 531 2) 60 3) 549 4) 172.80 5) None of these
4) 8768 5) None of these Directions (Q. 86-90): In each of the following
77. 561 440 271 46 335 696 questions a no. series is given. After the series, a no.
1341 (a) (b) (c) (d) (e) is given below it, followed by a, b, c, d and e. You have
What should come in place of (b)? to complete the series starting with the no. given and
1) 1115 2) 1436 3) 1326 following the same property as in the given no. se-
4) 1176 5) 1051 ries. Then answer the questions below it.
78. 60 10 50 54 51 25.5 86. 60 30 20 15 12 10
90 (a) (b) (c) (d) (e) 90 (a) (b) (c) (d) (e)
What should come in place of (c)? What should come in place of (d)?
1) 79 2) 75 3) 69 1) 18 2) 22.5 3) 20
4) 70 5) None of these 4) 15 5) None of these
79. 5 42 297 1786 8935 35746 87. 25 24 52 465 1876 46895
6 (a) (b) (c) (d) (e) 5 (a) (b) (c) (d) (e)
What will come in place of (d)? What should come in place of (e)?
1) 10615 2) 10610 3) 42460 1) 9379 2) 10895 3) 13375
4) 10722 5) None of these 4) 11795 5) None of these
80. 2 10 27 60 127 260 88. 1567 1688 1607 1656 1631 1640
4 (a) (b) (c) (d) (e) 3687 (a) (b) (c) (d) (e)
What should come in place of (d)? What should come in place of (b)?
1) 254 2) 259 3) 149 1) 3566 2) 3485 3) 3727
4) 159 5) None of these 4) 3737 5) None of these
K KUNDAN 29

89. 100 110 121 133.10 146.41 161.051 4) 686 5) None of these
60 (a) (b) (c) (d) (e) 98. 4.5 9.5 21 50 132 392
What should come in place of (c)? 22 (a) (b) (c) (d) (e)
1) 72.60 2) 87.846 3) 79.86 What should come in place of (c)?
4) 75 5) 65 1) 44.5 2) 412 3) 190
90. 687 869 713 845 735 825 4) 210 5) None of these
535 (a) (b) (c) (d) (e) 99. -1 0 -8 3 -52 -135
What should come in place of (a)?
21 (a) (b) (c) (d) (e)
1) 560 2) 692 3) 716
4) 726 5) None of these What should come in place of (b)?
Directions (Q. 91-95): In each of the following 1) 116 2) 36 3) 25
questions, a number series is established if the posi- 4) 30 5) None of these
tions of two out of the five marked numbers are in- 100. 3000 191 2216 847 1688 959
terchanged. The position of the first unmarked num- 3435 (a) (b) (c) (d) (e)
ber remains the same and it is the beginning of the What should come in place of (a)?
series. The earlier of the two marked numbers whose 1) 626 2) 746 3) 636
positions are interchanged is the answer. For ex- 4) 596 5) None of these
ample, if an interchange of number marked ‘1’ and Directions (Q. 101-105): In each of the follow-
the number marked ‘4’ is required to establish the ing questions, a number series is established if the
series, your answer is ‘1’. If it is not necessary to positions of two out of the five marked numbers are
interchange the position of the numbers to establish interchanged. The position of the first unmarked
the series, give 5 as your answer. Remember that number remains the same and it is the beginning of
when the series is established, the numbers change the series. The earlier of the two marked numbers
from left to right (i.e. from the unmarked number to whose positions are interchanged is the answer. For
the last marked number) in a specific order. example, if an interchange of the number marked ‘1’
91. 82 83 165 9916 1983 496 and the number marked ‘4’ is required to establish
1) 2) 3) 4) 5) the series, your answer is ‘1’. If it is not necessary to
92. 1200 40 1000 50 750 75 interchange the position of the numbers to establish
1) 2) 3) 4) 5) the series, give ‘5’ as your answer. Remember that
93. 4 3 124 15 14 495 when the series is established, the numbers change
1) 2) 3) 4) 5) from left to right (i.e. from the unmarked number to
94. -1 224 0 16 2 7104 the last marked number) in a specific order.
1) 2) 3) 4) 5) 101. 45 47 45 51 54 43
95. 1890 521 1482 641 809 1170 1) 2) 3) 4) 5)
1) 2) 3) 4) 5) 102. 461 340 565 204 733 4
Directions (Q. 96-100): In each of the following 1) 2) 3) 4) 5)
questions a no. series is given. After the series a no. 103. 10395 1485 13365 1053 15795 1215
is given below it, followed by a, b, c, d and e. You have
to complete the series starting with the no. given and 1) 2) 3) 4) 5)
following the same property as in the given no. se- 104. 6 56 1 19 11 529
ries. Then answer the question below it. 1) 2) 3) 4) 5)
96. 22 45 43 87 85 171 105. 48 16 13 12 17 25.25
37 (a) (b) (c) (d) (e)
1) 2) 3) 4) 5)
What should come in place of (e)?
1) 146 2) 293 3) 147 Directions (Q. 106-110): In each of the follow-
4) 291 5) None of these ing questions a no. series is given. After the series a
97. 8 9 22 75 316 1605 no. is given below it, followed by a, b, c, d and e. You
23 (a) (b) (c) (d) (e) have to complete the series starting with the no. given
What should come in place of (d)? and following the same property as in the given no.
1) 3405 2) 576 3) 624 series. Then answer the questions below it.
30 K KUNDAN

106. 480 480 960 320 1280 256 4) 8.80 5) None of these
120 a b c d e 115. 25 45 26 44 A 90 153 267
What should come in place of e ? A + 50 = ?
1) 320 2) 62 3) 54 1) 100 2) 50 3) 94
4) 60 5) None of these 4) 104 5) None of these
107. 1 9 26 86 352 1768 Directions (Q. 116-120): In each of the follow-
-8 a b c d e ing questions a no. series is given. After the series,
What should come in place of c? a no. is given below it, followed by a, b, c, d and e. You
1) 136 2) 8 3) 32 have to complete the series starting with the no. given
4) 24 5) None of these and following the same property as in the given no.
108. 9405 5684 3083 1402 441 0 series. Then answer the questions given below it.
7843 a b c d e 116. -2 0 3 14 63 326
What should come in place of b? 15 a b c d e
1) 2159 2) 1521 3) 0 What should come in place of c?
4) -2123 5) None of these 1) 37 2) 116 3) 92
109. 1 0 4 3 28 115 4) 106 5) None of these
11 a b c d e 117. -0.5 0.5 0 3 44 1105
What should come in place of d? 1 a b c d e
1) 268 2) 63 3) 1315 What should come in place of d?
4) 258 5) None of these 1) 88 2) 864 3) 908
110. 0.25 1.25 -3 0 -64 4) 912 5) None of these
45 a b c d e 118. 374 310 454 198 598 22
What should come in place of b? 234 a b c d e
1) 1611 2) 176 3) 46 What should come in place of e?
4) 56 5) None of these 1) -118 2) 458 3) 108
Directions (Q. 111-115): In each of the follow- 4) 128 5) None of these
ing questions a number series is given. A number in 119. 45045 9009 63063 7007 77077 5929
the series is suppressed by letter ‘A’. You have to 1575 a b c d e
find out the number in the place of ‘A’ and use this What should come in place b?
number to find out the value in the place of the ques- 1) 3205 2) 2405 3) 2205
tion mark in the equation following the series. 4) 1805 5) None of these
111. 2880 A 96 24 8 4 120. 12 14 29 31 63 65
10% of A = ? 942 a b c d e
1) 42 2) 46 3) 38 What should come in place of b?
4) 58 5) None of these 1) 1889 2) 944 3) 1989
112. 15 33 69 A 285 573 4) 954 5) None of these
A+9 =? Direction (Q. 121-125): In each of the following
1) 130 2) 140 3) 160 questions, a number series is established if the posi-
4) 150 5) None of these tions of two out of the five marked numbers are in-
113. 2 A 4 15 56 285 terchanged. The position of the first unmarked num-
ber remains the same and it is the beginning of the
1
A of 1  4 = ? series. The earlier of the two marked numbers whose
3 positions are interchanged is the answer. For ex-
1) 3 2) 4 3) 1 ample, if an interchange of the number marked ‘1’
4) 1/3 5) None of these and the number marked ‘4’ is required to establish
114. 49 56 64 A 81 90 the series, your answer is ‘1’. If it is not necessary to
15% of A + ? = 18.8 interchange the position of the numbers to establish
1) 10 2) 8 3) 10.80 the series, give 5 as your answer. Remember that
K KUNDAN 31

when the series is established, the numbers change and following the same property as in the given no.
from left to right (ie from the unmarked number to series. Then answer the questions below it.
the last marked number) in a specific order. 131. 4 6 15 49 201 1011
121. 4 0 -7 -45 -20 -94 15 a b c d e
1) 2) 3) 4) 5) What should come in place of d?
122. 6 772 28 130 10 5806 1) 115 2) 465 3) 455
1) 2) 3) 4) 5) 4) 475 5) None of these
123. 2 6 25 96 285 568 132. 0 1 0 9 32 825
1) 2) 3) 4) 5) 4 a b c d e
124. 8544 1420 280 66 5 18
What should come in place of c?
1) 2) 3) 4) 5)
1) 279 2) 81 3) 269
125. 1 2 64 9 625 46656
1) 2) 3) 4) 5) 4) 272 5) None of these
Directions (Q. 126-130): In each of the follow- 133. 2 -1 28 -9 86 -25
ing questions a no. series is given. After the series, 122 a b c d e
a no. is given below it, followed by a, b, c, d and e. You What should come in place of b?
have to complete the series starting with the no. given 1) 124 2) 328 3) 9
and following the same property as in the given no. 4) 228 5) None of these
series. Then answer the questions below it. 134. 101 323 545 767 989 111011
126. -4 2 10 96 6150 34 a b c d e
-10 a b c d e What should come in place of a?
What should come in place of d? 1) 56 2) 78 3) 43
1) -198 2) 186 3) 132 4) 100 5) None of these
4) -762 5) None of these 135. -1 0 10 65 345 1750
127. 4721 4611 4429 4219 3913 3571 -2 a b c d e
871 a b c d e What should come in place of e?
What should come in place of c? 1) -1750 2) 1375 3) -1325
1) 63 2) 367 3) 579
4) -1475 5) None of these
4) 469 5) None of these
Direction (Q. 136-140): In each of the following
128. 96 144 288 720 2160 7560 questions, a number series is established if the posi-
18 a b c d e tions of two out of the five marked numbers are in-
What should come in place of e? terchanged. The position of the first unmarked num-
1) 1417.50 2) 945 3) 405 ber remains the same and it is the beginning of the
4) 927.50 5) None of these series. The earlier of the two marked numbers whose
129. 48 60 110 338 1346 6734 positions are interchanged is the answer. For ex-
120 a b c d e ample, if an interchange of the number marked ‘1’
What should come in place of b? and the number marked ‘4’ is required to establish
1) 132 2) 254 3) 274 the series, your answer is ‘1’. If it is not necessary to
4) 248 5) None of these interchange the positions of the numbers to estab-
lish the series, give ‘5’ as your answer. Remember
130. 4 6 15 79 704 8480
that when the series is established, the numbers
12 a b c d e change from left to right (ie from the unmarked num-
What should come in place a? ber to the last marked number) in a specific order.
1) 18 2) 24 3) 14 136. 6 4 16 11 65 143166
4) 26 5) None of these 1) 2) 3) 4) 5)
Directions (Q. 131-135): In each of the follow- 137. 829 436 661 300 557 508
ing questions a no. series is given. After the series, 1) 2) 3) 4) 5)
a no. is given below it, followed by a, b, c, d and e. You 138. 3 27 61 125 295 191
have to complete the series starting with the no. given 1) 2) 3) 4) 5)
32 K KUNDAN

139. 2 5 26545 177 4424 44 149. 300 496 352 452 388 424
1) 2) 3) 4) 5) 104 a b c d e
140. 1680 420 2100 306.25 2450 350 What should come in place of e?
1) 2) 3) 4) 5) 1) 192 2) 256 3) 328
Direction (Q. 141-145): In each of the following 4) 228 5) None of these
questions, a number series is established if the posi- 150. 1.6 40000 8 14000 40 4900 200
tions of two out of the five marked numbers are in- 4.05 200 a b c d e
terchanged. The position of the first unmarked num- What should come in place of b and e respec-
ber remains the same and it is the beginning of the tively?
1) 210, 507.25 2) 310, 506.25 3) 310, 505.25
series. The earlier of the two marked numbers whose
4) 200, 505.25 5) None of these
positions are interchanged is the answer. For ex- Directions (Q. 151-155): In each of the follow-
ample, if an interchange of the number marked ‘1’ ing questions a no. series is given. After the series,
and the number marked ‘4’ is required to establish a no. is given below it, followed by a, b, c, d and e. You
the series, your answer is ‘1’. If it is not necessary to have to complete the series starting with the no. given
interchange the positions of the numbers to estab- and following the same property as in the given no.
lish the series, give 5 as your answer. Remember series. Then answer the questions below it.
that when the series is established, the numbers 151. 45 44 39 32 13 -8
change from left to right (ie from the unmarked num- 100 a b c d e
ber to the last marked number) in a specific order. What should come in place of e?
141. 1 0 2 -6 32 -190 1) 58 2) 38 3) 46
4) 48 5) None of these
1) 2) 3) 4) 5)
152. 840 560 420 336 280 240
142. 4 548 717 65 675 838 180 a b c d e
1) 2) 3) 4) 5) What should come in place of c?
143. 4 127 9 34 7 646 1) 60 2) 84 3) 90
1) 2) 3) 4) 5) 4) 72 5) None of these
144. 10000 6875 6619 6583 6584 6592 153. 1600 231 1320 479 1104 663
1) 2) 3) 4) 5) 1380 a b c d e
145. 12 26 102 60 80 42 What should come in place of d?
1) 2) 3) 4) 5) 1) 443 2) 894 3) 884
Directions (Q. 146-150): In each of the follow- 4) 259 5) None of these
ing questions a no. series is given. After the series, 154. 1 9 41 169 841 3369
a no. is given below it, followed by a, b, c, d and e. You 16 a b c d e
have to complete the series starting with the no. given What should come in place of b?
and following the same property as in the given no. 1) 69 2) 341 3) 1369
series. Then answer the questions below it. 4) 343 5) None of these
146. 3 5 18 69 350 2095 155. 120 60 180 45 225 37.50
14 a b c d e 576 a b c d e
What should come in place of c? What should come in place of c?
1) 333 2) 84 3) 1670 1) 216 2) 220 3) 864
4) 421 5) None of these 4) 1080 5) None of these
147. 1 9 61 369 1841 7369 Directions (Q. 156-160): In each of the follow-
15 a b c d e ing questions, a number series is established if the
What should come place of b? positions of two out of the five marked numbers are
1) 745 2) 845 3) 865 interchanged. The position of the first unmarked
4) 885 5) None of these number remains the same and it is the beginning of
148. 4 2 32 146 592 1774 the series. The earlier of the two marked numbers
8 a b c d e whose positions are interchanged is the answer. For
What should come in place of c? example, if an interchange of the number marked ‘1’
1) 3952 2) 200 3) 986 and the number marked ‘4’ is required to establish
4) 990 5) None of these the series, your answer is ‘1’. If it is not necessary to
K KUNDAN 33

interchange the position of the numbers to establish 167. 2 11 57 225 679 1353
the series, give ‘5’ as your answer. Remember that 7 a b c d e
when the series is established, the numbers change What should come in place of d?
from left to right (ie from the unmarked number to 1) 825 2) 2579 3) 2459
the last marked number) in a specific order. 4) 2479 5) None of these
156. 7 1 14 47 242 1447 168. 1024 240 865 381 742 486
1) 2) 3) 4) 5) 1328 a b c d e
157. 1 2 -95 3 -4 0 What should come in place of c?
1) 2) 3) 4) 5) 1) 685 2) 625 3) 544
158. 410 320 430 248 152 458 4) 705 5) None of these
1) 2) 3) 4) 5) 169. 1.50 2 6 22.5 98 502.50
159. 20 2430 90 405 30 18225 10 a b c d e
1) 2) 3) 4) 5) What should come in place of b?
160. 2000 996 494 54.75 117.50 243 1) 21 2) 23 3) 73.50
1) 2) 3) 4) 5) 4) 20 5) None of these
Directions (Q. 161-165): In each of the follow- 170. 18 12 9 18 6.75
ing questions, a number series is established if the 30 a b c d e
positions of two out of the five marked numbers are What should come in place of d?
interchanged. The position of the first unmarked 1) 30 2) 12.75 3) 11.75
number remains the same and it is the beginning of 4) 12 5) None of these
the series. The earlier of the two marked numbers Directions (Q. 171-175): Find out the wrong
whose positions are interchanged is the answer. For number in the following given sequences.
example, if an interchange of the number marked ‘1’ 171. 13 6 8 9 20 47.5 145.5
and the number marked ‘4’ is required to establish 1) 47.5 2) 20 3) 6
the series, your answer is ‘1’. If it is not necessary to 4) 145.5 5) 8
interchange the positions of the numbers to estab-
172. 256 128 64 192 96 240 80
lish the series, give ‘5’ as your answer. Remember
that when the series is established, the numbers 1) 128 2) 192 3) 560
change from left to right (i.e. from the unmarked 4) 64 5) 480
number to the last marked number) in a specific 173. 9 -1 1 -2 -11 -285 -10255
order. 1) -1 2) -2 3) -10255
161. 5 14 23 44 90 57 4) 1 5) -11
1) 2) 3) 4) 5) 174. 40 39 33 37 16 35 -8
162. 343 12 54 45 279 177 1) 39 2) -5 3) 38
1) 2) 3) 4) 5) 4) 33 5) 17
163. 20160 5040 1440 96 192 480 175. 121 110 113 138 89 170 49
1) 2) 3) 4) 5) 1) 89 2) 49 3) 110
164. 2 3 1484 93 10 37105
4) 138 5) 170
1) 2) 3) 4) 5)
165. 1260 1190 1180 1150 1160 1140 Directions (Q. 176-180): In each of the follow-
1) 2) 3) 4) 5) ing questions a no. series is given. After the series,
Directions (Q. 166-170): In each of the follow- a no. is given below it, followed by a, b, c, d and e. You
have to complete the series starting with the no. given
ing questions a no. series is given. After the series,
a no. is given below it, followed by a, b, c, d and e. You and following the same property as in the given no.
have to complete the series starting with the no. given series. Then answer the questions below it.
and following the same property as in the given no. 176. 0 1 0 9 32 185
series. Then answer the questions below it. 15 a b c d e
166. 4 2 16 146 2334 29175 What should come in place of c?
1) 99 2) 81 3) 34
6 a b c d e
What should come in place of c? 4) 79 5) None of these
1) 228 2) 220 3) 218 177. 2 3 10 39 196 1175
5 a b c d e
4) 216 5) None of these
What should come in place of d?
34 K KUNDAN

1) 550 2) 556 3) 656 4) 192 5) None of these


4) 603 5) None of these 190. 5 ? 14 56 39.20 156.80 109.76
178. 1680 840 560 420 336 280 1) 20 2) 10 3) 15
720 a b c d e 4) 1 5) None of these
What should come in place of e? Directions (Q. 191-195): Find out the wrong
1) 144 2) 140 3) 240 number in the following number series.
4) 120 5) None of these 191. 12 25 52 55 57 115 117
179. 250 106 206 142 178 162 1) 55 2) 117 3) 25
232 a b c d e 4) 52 5) None of these
What should come in place of c? 192. 2478 819 257 84 24 6 1
1) 251 2) 124 3) 135 1) 257 2) 24 3) 6
4) 120 5) None of these 4) 819 5) 1
180. 132 145 117 167 90 200 193. 2 3 6 15 45 160 630
326 a b c d e 1) 45 2) 630 3) 6
What should come in place of b? 4) 3 5) 160
1) 340 2) 311 3) 261 194. 199 176 195 180 190 184 187
4) 305 5) None of these 1) 180 2) 190 3) 184
Directions (Q. 181-185): Find the wrong num- 4) 187 5) 199
ber in the following number series. 195. 1 5 2 30 28 2620
181. 2 5 4 45 168 4216 25272 1) 5 2) 2620 3) 28
1) 4 2) 2 3) 168 4) 30 5) 2
4) 25272 5) 4216 Directions (Q. 196-200): Find the wrong num-
182. 4 5 1 7 -1 19 7 ber in the following number series:
1) -1 2) 7 3) 19 196. 5 25 30 100 95 475
4) 5 5) 4 1) 25 2) 95 3) 30
4) 5 5) 475
183. 50 25 3 25 29 24 4 28 36 197. 36 96 156 126 86
1) 36 2) 96 3) 156
4) 86 5) 126
1) 50 2) 25 3 3) 36
198. 45 270 54 216 72 146
4) 29 5) 4 1) 146 2) 270 3) 216
184. 1521 560 1785 264 2113 96 2505 4) 54 5) 45
1) 560 2) 2505 3) 96 199. 65 52 1040 832 16540 13312
4) 264 5) 2113 1) 13312 2) 16540 3) 52
185. 1 3 9 82 1065 18106 4) 1040 5) 832
200. 42 83 161 315 613 1209
1) 1065 2) 3 3) 82
1) 42 2) 315 3) 83
4) 18106 5) 1 4) 1209 5) 161
Directions (Q. 186-190): What will come in place Directions (Q. 201-205): In each of the follow-
of the question mark (?) in the following number ing questions, a number series is established if the
series? positions of two out of the five marked numbers are
186. 3 ? 2 33 88 505 2940 interchanged. The position of the first unmarked
1) 8 2) 1 3) -2 number remains the same and it is the beginning of
4) 6 5) None of these the series. The earlier of the two marked numbers
187. 5 1 3 5 21 ? 607 whose positions are interchanged is the answer. For
1) 97 2) 109 3) 99 example, if an interchange of the number marked ‘1’
4) 103 5) None of these and the number marked ‘4’ is required to establish
188. 5 12 60 340 1648 ? 20164 the series, your answer is ‘1’. If it is not necessary to
1) 6724 2) 4046 3) 4036 interchange the positions of the numbers to estab-
4) 6512 5) None of these lish the series, give ‘5’ as your answer. Remember
189. 6 10.5 23 60 ? 644 2580 that when the series is established, the numbers
1) 163 2) 183 3) 202 change from left to right (ie from the unmarked num-
K KUNDAN 35

ber to the last marked number) in a specific order. 1) 225 2) 425 3) 525
201. 4 60 12 4 420 3780 4) 575 5) None of these
1) 2) 3) 4) 5) 213. 7 13 78 83 415
202. 1020 420 840 652 724 568 4 (a) (b) (c) (d) (e)
1) 2) 3) 4) 5) Which of the following numbers will come in
203. 1 4 5888 92 653 17 place of (c)?
1) 2) 3) 4) 5) 1) 65 2) 60 3) 53
204. 864 1728 576 2304 256 2048 4) 48 5) None of these
1) 2) 3) 4) 5) 214. 8 4.5 5.5 9.75
205. 7 4 6 21 1200 135 40 (a) (b) (c) (d) (e)
1) 2) 3) 4) 5) Which of the following numbers will come in
Directions (Q. 206-210): In each of the follow- place of (c)?
ing questions, a number series is established if the 1) 20.5 2) 31.5 3) 33.75
positions of two out of the five marked numbers are 4) 53.75 5) None of these
interchanged. The position of the first unmarked 215. 12 76 27 63
number remains the same and it is the beginning of 136 (a) (b) (c) (d) (e)
the series. The earlier of the two marked numbers Which of the following numbers will come in
whose positions are interchanged is the answer. For place of (e)?
example, if an interchange of the number marked ‘1’ 1) 187 2) 162 3) 168
and the number marked ‘4’ is required to establish 4) 178 5) None of these
the series, your answer is ‘1’. If it is not necessary to Directions (Q. 216-220): Find the missing num-
interchange the positions of the numbers to estab- ber (P) in the following series
lish the series, give ‘5’ as your answer. Remember 216. 126 158 174 P 186 188
that when the series is established, the numbers 1) 180 2) 182 3) 184
change from left to right (ie from the unmarked num- 4) 178 5) None of these
ber to the last marked number) in a specific order.
217. 2.7 P 3.6 2.4 4.8 3.2
206. 120 240 160 480 768 192
1) 2) 3) 4) 5) 1) 1.5 2) 1.8 3) 2.1
207. 295 46 343 118 407 174 4) 2.4 5) 2.5
1) 2) 3) 4) 5) 2 1 2 1
218. 15 13 P 8 6 4
208. 4 6 15 139 561 14031 3 3 3 3
1) 2) 3) 4) 5) 1) 8 2) 9 3) 10
209. 2 3 285 15 56 4 4) 11 5) None of these
1) 2) 3) 4) 5)
219. P 58 84 114 146 182 220
210. 12 13 11 14 10 15
1) 2) 3) 4) 5) 1) 28 2) 30 3) 32
Directions (Q. 211-215): In each of the follow- 4) 34 5) 36
ing questions, a number series is given. After the 220. 127 P 85 67 51 37 25
series a number is given followed by (a), (b), (c), (d) 1) 105 2) 100 3) 95
and (e). You have to complete the series starting with 4) 90 5) None of these
the number given following the sequence of the given Directions (Q. 221-225): Find the missing number
series. 221. 1 2 0 9 20 ?
211. 10 10 15 30
32 (a) (b) (c) (d) (e) 1) 80 2) 100 3) 125
Which of the following numbers will come in 4) 180 5) 200
place of (d)? 222. 1 1 2 4 ? 16
1) 96 2) 120 3) 720 1) 6 2) 8 3) 10
4) 240 5) None of these
4) 12 5) None of these
212. 6 12 36 96 216
5 (a) (b) (c) (d) (e) 223. -1 0 7 26 63 ?
Which of the following numbers will come in 1) 101 2) 310 3) 511
place of (e)? 4) 420 5) None of these
36 K KUNDAN

224. 6 5 9 ? 103 514 What will come in place of (d)?


1) 12 2) 13 3) 14 1) 14.25 2) 15 3) 16
4) 16 5) 26 4) 12 5) None of these
225. 20 21 25 52 ? 238. 5 10 23 48 89 150
1) 100 2) 200 3) 308 6 (a) (b) (c) (d) (e)
What will be the value of (b) : (d)?
4) 400 5) None of these
1) 3 : 8 2) 5 : 17 3) 4 : 17
Directions (Q. 226-230): Find the wrong num- 4) 4 : 15 5) None of these
ber in the sequences given below: 239. 4 4.5 11 37.5 158 802.5
226. 12 24 96 476 4608 7 (a) (b) (c) (d) (e)
(1) (2) (3) (4) (5) What will come in place of (b)?
227. 150 299 600 1197 2401 4793
1) 14 2) 19 3) 17
(1) (2) (3) (4) (5) 4) 21 5) None of these
228. 2 6 40 1606 2579242 240. 732 371 660 435 604 483
(1) (2) (3) (4) (5) 821 (a) (b) (c) (d) (e)
229. 512 101 24 7 2 What will come in place of (e)?
(1) (2) (3) (4) (5) 1) 583 2) 572 3) 693
230. 6561 81 9 3 1 4) 683 5) None of these
(1) (2) (3) (4) (5)
Directions (Q. 241-245): In each of the follow-
Directions (Q. 231-235): In each of these questions
ing number series, a wrong number is given. Find
a number series is given. Only one number is
out the wrong number.
wrong in each series. You have to find out the 241. 2 6 13 26 54 100 197
wrong number. 1) 26 2) 54 3) 100
231. 3, 1.75, 2.75, 6.375, 16.65, 48.125 4) 197 5) 13
1) 1.75 2) 48.125 3) 2.75
242. 56 57 48 73 24 105 -10
4) 16.65 5) 6.375
1) 57 2) 105 3) 73
232. 13, 84.5, 507, 2788, 13942.5, 62741.25
4) -10 5) 24
1) 2788 2) 84.5 3) 13942.5 243. 2 2 13 59 363 2519 20161
4) 62741.25 5) 507 1) 13 2) 20161 3) 2519
233. 17, 103, 523, 2119, 6423, 12967 4) 363 5) 59
1) 2119 2) 103 3) 12967
244. 1 8 66 460 2758 13785 55146
4) 523 5) 6423
1) 460 2) 2758 3) 66
234. 3, 6, 10, 18, 30, 53, 89
4) 8 5) 55146
1) 53 2) 89 3) 30 245. 3 1 3 0.7 3 0.6 3 0.5 3
4) 18 5) 10 1) 1 2) 0.7 3) 0.6
235. 5, 12.5, 43.75, 196.875, 1080.8125, 7038.28125 4) 0.5 5) 3
1) 43.75 2) 12.5 3) 1080.8125
Directions (Q. 246-250): In each of the follow-
4) 7038.28125 5) 196.875 ing questions a number series is given. After the
Directions (Q. 236-240): In each of the follow- series, a number is given followed by (a), (b), (c), (d)
ing questions a number series is given. After the and (e). You have to complete the second series and
series a number is given followed by (a), (b), (c), (d) answer the questions given below the series.
and (e). You have to complete the series starting with 246. 54 56 62 74 94 124
the number given and following the sequence of the
175 (a) (b) (c) (d) (e)
original series. Answer the questions that follow the What will come in place of (c)?
series. 1) 196 2) 195 3) 175
236. 7 16 12 30 23 56 4) 174 5) None of these
11 (a) (b) (c) (d) (e) 247. 6 3 3 4.5 9 22.5
What will come in place of (c)? 8 (a) (b) (c) (d) (e)
1) 38 2) 27 3) 16
What will come in place of (e)?
4) 24 5) None of these 1) 24 2) 28 3) 275
237. 6 3.5 4.5 8.25 18.5 48.75 4) 30 5) None of these
3 (a) (b) (c) (d) (e)
248. 11 13 10 40 8 14
K KUNDAN 37

16 (a) (b) (c) (d) (e) 16 (a) (b) (c) (d) (e)
What will come in place of (b)? What will come in place of (d)?
1) 15 2) 20 3) 18 1) 1384 2) 2642 3) 2808
4) 14 5) None of these 4) 1988 5) None of these
249. 0 3 14 77 548 252. 154 462 231 693 346.5 1039.5
-1 (a) (b) (c) (d) (e) 276 (a) (b) (c) (d) (e)
What will come in place of (d)? What will come in place of (e)?
1) 447 2) 443 3) 445 1) 1746 2) 621 3) 1242
4) 438 5) None of these 4) 983 5) None of these
250. 5 6 10 28 124 724 253. 7 91 1001 7007 35035 105105
12 (a) (b) (c) (d) (e) 14.5 (a) (b) (c) (d) (e)
What will come in place of (b)? What will come in place of (c)?
1) 24 2) 26 3) 52 1) 21132.5 2) 14514.5 3) 20020.5
4) 17 5) None of these 4) 13864.5 5) None of these
Directions (Q. 251-256): In each of the follow- 254. 582 574 601 537 662 446
ing questions a number series is given. After the 204 (a) (b) (c) (d) (e)
series a number is given followed by (a), (b), (c), (d) What will come in place of (d)?
and (e). You have to complete the series starting with 1) 284 2) 68 3) 174
the number given, following the sequence of the origi- 4) 331 5) None of these
nal series and answer the questions that follow the 255. 85 43 44 67.5 137 345
125 (a) (b) (c) (d) (e)
series.
What will come in place of (c)?
251. 12 30 120 460 1368 2730 1) 86 2) 107.5 3) 112.5
4) 97.5 5) None of these
[UBI PO-2005]

Answers and explanations


Exercise-1
(1-5): 8. 4; The series is ×1.5, ×3, ×4.5, ....
1. 3; The series is ÷8, ×7, ÷6, ×5 ... 9. 1; The series is ×2 + 4, ×2 + 8, ×2 + 12, ....
 Replace (3) with (5). 10. 2; The series is ×8 - 28, ×7 - 24, ×6 - 20, .....
2. 1; The given series is an alternate series 11. 2; The series is ×2 + (2 × 1²), ×2 + (2 × 3²), ×2 + (2
ie, S1 = 7 11 15; the property is +4 × 5²), ×2 + (2 × 7²), ... So, 164 should be re-
placed by 182.
S2 = 14 51 90; after interchanging 14 and 90,
(12-16):
we get the series following - 39 and -37. 12. 3; The series is ×1 + 2, ×2 + 2, ×3 + 2, ×4 + 2, ×5 +
 Replace (1) with (5). 2, × 6 + 2. 914 is incorrect. It should be 772. The
3. 3; The given series is an alternate series. new series begins with 914.
ie, S1 = 40 60 80; the property is +20 13. 3; The series is ×1 + 1, ×2 + 2, ×3 + 3, ×4 + 4, ×5 +
S 2 = 14 24 19; after interchanging 24 and 19, 5, × 6 + 6. 34 should be 33 and thus the new
we get the property as +5. series starts with 34.
4. 5; The series is +15², -13², +11², -9² .... 14. 4; The series is -(14)², +(12)², -(10)², +(8)², -(6)²
5. 2; The series is ×1 + 1, ÷2 + 2, ×3 + 3 ... and so on.
 Replace (2) with (3) 1 3 5 7 9 11
(6-10): 15. 5; The series is  , , , , , and so
2 2 2 2 2 2
6. 1; The series is ×1 + 4, ×2 + 8, ×3 + 12, .... on.
7. 3; The series is ×1 + 1, ×2 + 2, ×3 + 3, ....
38 K KUNDAN

16. 1; The series is +1² - 0, +2² - 1, +3² - 2, +4² - 3, +5² 33. 2; The series is +20, +30, +40...... So 182 will come
- 4, +6² - 5. in place of A.
Thus, 7 is the wrong number.
182  14
(17-21): ?  14
17. 4; The series is ×8 + 1, ×7 + 2, ×6 + 3. 14
 a = 2 × 8 + 1 = 17, b = 17 × 7 + 2 = 121, c = 121 34. 5; The series is +3, +5, +7, +9 .... So 39 will come in
× 6 + 3 = 729 place of A.
18. 1; The series is ×1, ×1.5, ×2 ? = 39² - 4 = 1517
 a = 36 × 1 = 36, b = 36 × 1.5 = 54, c = 54 × 2 = 35. 1; The series is +6, +11, +16, +21 ... So 35 will come
108, d = 108 × 2.5 = 270 and e = 270 × 3 = 810 in place of A.
19. 3; The series is ÷2 - 4 3 4
 a = 888 ÷2 - 4 = 440 and b = 440 ÷ 2 - 4 = 216 ?  35    12
7 5
20. 5; The series is × 1 + 1, ×1.5 + 2.25, × 2 + 4, (×2.5 +
6.25, ×3 + 9, .....) 3 81
36. 5; The series is ×2 and  alternately. So will
 a = 7 × 1 + 1 = 8, b = 8 × 1.5 + 2.25 = 14.25, c 4 64
= 14.25 × 2 + 4 = 32.5 and d = 32.5 × 2.5 + 6.25 = come in place of A.
81.25 + 6.25 = 87.5
21. 2; The series is + (17)², -(15)², +(13)², -(11)², +(9)², 81 9
? 
...... 64 8
 c = 13 + (238 - 5 =) 233 = 246, d = 246 - (11)² (37-41):
= 37. 1; The series is +1², +3², +5², +7²,......
246 - 121 = 125 and e = 125 +(9)² = 125 + 81 = 206 38. 3; The series is ×0.5 + 0.5, ×1 + 1, ×1.5 + 1.5, ×2 +
(22-26): 2,....
22. 5; The series is ×1 + 1, ×2 + 2, ×3 + 3, ..... So 8 is 39. 3; The series is ×1 + 1, ×2 + 2, ×3 + 3,......
wrong. Beginning with 8 we get 20 as third 40. 1; The series is ×2 + 6, ×2 + 7, ×2 + 6,....
term. 41. 4; The series is +(12)², -(10)², +(8)², -(6)²,....
23. 5; The series is ×1 + 1², ×2 + 2², ×3 + 3², .... (42-46):
24. 2; The series is ×2 + 9, ×2 + 11, ×2 + 13, ..... 42. 3; The series is ×1 + 1², ×2 + 2², ×3 + 3², ×4 + 4²,....
25. 4; The series is ×1 - 1, ×2 + 2, ×2 - 2, ×3 + 3, ..... 43. 5; Ans = 13. The series is ×1 + 7, ×2 - 11, ×3 + 15,....
26. 4; The series is ×3 + 1, ×4 + 1, ×5 + 1, ...... 44. 4; The series is ×1 + 1², ×2 + 2², ×3 + 3², ×4 + 4²,....
(27-31): 45. 1; The series is ×1.5 + 1.5, ×2 + 2, ×2.5 + 2.5, ×3 +
27. 5; The series is ×2 + 1.5, ×2 + 2, ×2 + 2.5 ......... So, 3....
108.5 should come in place of (c). 46. 2; The series is ×2 + 3, ×2 + 5, ×2 + 7, ×2 + 9,.....
28. 1; The series is ×1² + 1, ×1² + 1, ×2² + 2, ×3² + 3, ×4² (47-51):
+ 4, ......... 47. 3; The series is -2, -4, -8, -16,....
29. 2; The series is ×1 + 0.25 × 1, ×2 + 0.25 × 4, ×3 + So, P = 186 - 4 = 182
0.25 × 9 ...... So 24.75 should come in place of
(c).  ?  P  13  182  13  13
30. 4; The series is ×0.5, ×1.5, ×2.5, ..... So 118.125 48. 5; The series is ×1.5, ÷2, ×1.5, ÷2,.....
should come in place of (d).
31. 3; The series is +11², -9², +7², -5²,.... So 112 should 49. 2; The series is  2 1 in each term.
come in place of (e). 3
(32-36): 50. 1; The series is -38, -36, -32, -30, -26, -24
51. 4; The series is +12, +14, +16, +18.....
3 (52-56):
32. 3; The series is  6, alternately. So, 116.64 will
10 52. The series is ÷2, ×1.5....
come in place of A. 53. 5; Ans = 2860. The series is ×1 + 4, ×2 + 8, ×3 +
116.64 ÷ 36 = 3.24 12,.....
K KUNDAN 39

54. 4; The series is ×2 + 1, ×3 - 2, ×4 +3, ×5 - 4,.... 89. 4; The series is ×2 - 2


55. 3; The series is ×2 - 2, ×3 + 2, ×4 - 2, ×6 + 2,..... (90-94):
56. 2; The series is ÷2 - 2 in each steps 90. 1; The series is +5, +9, +13, +17 .... The differ-
(57-61): ence in successive nos. 9 - 5 = 13 - 9 = 17 - 13
57. 4; The series is ×2, ×4, ×6,...... = .... = 4. Hence, 35 is wrong. It should be 37.
58. 2; The series is ×2 - 1, ×2 + 3, ×2 - 5, ..... 91. 5; The sum of the first two nos. is the third no.
59. 4; Each number is the square of its preceding num- Hence, 27 is wrong. It should be 29.
ber. 92. 5; The series is ×0.5 + 0.5, × 1 + 1, × 1.5 + 1.5 ....
60. 2; The series is ×2 + 1, ×3 + 3, ×4 + 5, ..... Hence, 12 is wrong. It should be 14.
61. 4; Each number is square of its preceding number 93. 4; The series is  2  22 ,  3  32 ,  4  42 ,  5  52
plus 2, 4, 6, ....
.... Hence 32 is wrong It should be 33.
viz, 2² + 2 = 6, 6² + 4 = 40 .......
62. 3; The series is 1³ + 1 = 2, 2³ + 1 = 9, 3³ + 1 = 28,.... 94. 2; The series is  13 ,  23 ,  33 ,  43.... Hence, 224 is
(63-67): wrong. It should be 227.
63. 5; The series is +8², +7², +6², +5²,...... (95-99):
95. 4; The series is ×1 + 1 × 7, ×2 + 2 × 6, ×3 + 3 × 5
1 1
64. 3; The series is  ,3, ,4,..... ...
3, 4
96. 5; The series is 1  12 ,  2  22 ,  3  32 , ...
2 3 4 5 97. 3; The series is ×1 + 2, ×2 + 3, ×3 + 4, ...
65. 4; The series is  , , , ,.....
3 4 5 6 98. 2; The series is ×0.5 + 0.5, ×1 + 1, ×1.5 + 1.5, ...
66. 1; The series is +6², ×7, +7², ×8,........ 99. 1; The series is ×2, ÷3, ×4, ÷5, ...
67. 2; The series is +2.4, -4.8, +9.6, -19.2,........ (100-104):
68. 4; Series is ×2 + 2², ×3 + 3², ×4 + 4²,...... 100. 5; Ans = 172. The series is 1  12 ,  2  2 2 ,  3  32 ,
69. 5; The series is ÷2 - 1 in each term. ....
(70-74): 101. 5; Ans = 97. The series is ×2-1 in each term.
70. 5; The series is +0.5, +1, +1.5, +2,.... 102. 3; The series is ×0.5 + 0.5, ×1+1, ×1.5+1.5, ×2+2,
71. 3; The series is +1², +2², +3², +4²,..... ....
72. 2; The series is -8, -7, -6, -5,...... 103. 2; The series is: 3 × 1 + 7 × 1 = 10; 10 × 2 + 6 × 2 =
73. 1; The series is ×3 - 1 in each term. 32; 32 × 3 + 5 × 3 = 111; 111 × 4 + 4 × 4 = 460;
74. 4; The series is ÷2 - 1 in each term. 460 × 5 + 3 × 5 = 2315 ....
(75-79): 104. 1; The series is ×2 - 2, ÷2 + 2, ×2 - 2, ÷2 +2, ....
75. 3; The series is ×1 + 1, ÷2 - 2, ×3 + 3, ...... (105-109):
76. 2; The series is +4.5, +9, +13.5,...... 105. 1; The series is ×1+1, ×1.5+1.5, ×2+2, ×2.5+2.5, ...
77. 5; The series is ×1 + 7, ×2 - 6, ×3 +5, ×4 - 4,..... 106. 3; The series is 2 + 5 = 7; 7 + 5 = 12; 12 + 7 = 19; ...
78. 2; The series is ×2 + 2, ÷2 - 2, ×3 + 3, ÷3 - 3,... 107. 2; The series is ×6 + 1, ×5 + 2, ×4 + 3, ×3 + 4, ...
79. 1; The series is ×1.5, ×2, ×2.5, ×3,.....
108. 3; The series is  1 13 ,  2  23 ,  3  33 ,
(80-84):
80. 4; The series is ×1 + 1², ×2 - 2², ×3 + 3², ×4 - 4².....  4  43 , ...
81. 5; The series is ×1 + 1², ×2 + 2², ×3 + 3², ×4 + 4²..... 109. 4; The series is ×7, ×6.5, ×6, ×5.5, ...
82. 5; The series is ×2 + 0.5, ×2 + 2, ×2 + 8, ×2 + 32.... (110-114):
83. 1; The series is ×2 + 10, ×2 - 10 alternately. 110. 4; The series is ×3 + 5, ×4 - 6, ×5 + 7, ×6 - 8, ....
(84-89):
111. 1; The series is  12 ,  22 ,  32 ,  42 , ....
84. 3; The series is ×2 + 1, ×2 - 1 alternately.
85. 5; The series is × 1.5, × 2, × 2.5, × 3 and so on. 112. 4; The series is 7 + 9 = 16; 16 + 9 = 25; 25 + 16 = 41;
86. 1; The series is -16, -8, -4, -2, -1, -0.5, and so on. 41 + 25 = 66; 66 + 41 = 107 ....
87. 2; The series is × 1 + 2, × 2 + 3, × 3 + 4, and so on. 113. 3; The series is ×0.5, ×1.5, ×2.5, ×3.5, ...
88. 3; The series is × 1 + 12, ×2 + 22, ×3 + 32 and so on. 114. 2; The series is ×0.25, ×0.5, ×0.75, ×1, ...
40 K KUNDAN

(115-119): 2 4 5 8 13 21 34
115. 3; The series is +5, +7, +9, +11, .... 130. 1;
+2 +1 +3 +5 +8 +13
116. 4; The series is ÷2, ÷1.5 alternately. Hence the series should be as follows:
117. 5; The series is an alternate series, having 2 3 5 8 13 21 34
S1 = 2 5 14 41; ×3 – 1 in each term +1 +2 +3 +5 +8 +13
Here 1 + 2 = 3; 2 + 3 = 5; 3 + 5 = 8; 5 + 8 = 13
S2 = 3 8 23 69; ×3 – 1 in each term (131-135):
118. 3; The differences are 1 - 0 = 1 = 13 ; 9 - 1 = 8 131. 4; The series follows the pattern:
4 × 0.5 + 0.25 × 12 = 2.25
= 23 ; 36 - 9 = 27 = 33 ; 99 – 36 = 63  43 , but
100 - 36 = 64 = 43 ; 2.25 × 1 + 0.25 × 22 = 3.25
3.25 × 1.5 + 0.25 × 32 = 7.125
225 – 100 = 125 = 53 ;
7.125 × 2 + 0.25 × 42 = 18.25
441 – 225 = 216 = 63
119. 1; The series is ×2.5, ×2 alternately. 18.25 × 2.5 + 0.25 × 5 2 = 51.875
(120-124): Hence, the required answer
120. 3; The series is ×2 + 6 in each term.
= 7 × 0.5 + 0.25 × 12 = 3.75 = a
2 2
121. 1; The series is  1  1 ,  2  2 ,  3  3 , ....
2
3.75 × 1 + 0.25 × 22 = 4.75 = b
122. 4; The series is ÷2 + 3.5, ×2 + 12, ÷2 + 2.5, ×2 + 8,
132. 1; The series is based on the pattern
...
123. 2; The series is ×2 - 1, ×3 - 2, ×4 - 3, ....  5  52 ,  4  4 2 ,  3  32 ,  2  2 2 ,  1 12 .
124. 5; The series is ×0.5, ×1.5, ×2.5, .... Hence the required answer will be:
(125-130): 1 × 5 + 5 2 = 30 (=a) × 4 + 42 = 136 (=b) × 3 +
125. 4; 2 3 6 18 109 1944 209952
Here 2 × 3 = 6; 3 × 6 = 18; 6 × 18 = 108; 32 = 417 (= c) × 2 + 22 = 838 (= d) 838 × 1 +
18 × 108 = 1944; 108 × 1944 = 209952
12 = 839.
126. 2; 1 3 6 11 20 39 70 [Note: In this type of series, observing from
Here 1 × 2 + 1 = 3; 3 × 2 + 0 = 6; the right end will be beneficial; e.g. we see
6 × 2 – 1 = 11; 11 × 2 – 2 = 20; that the last element is greater by 1 to its pre-
20 × 2 – 3 = 37; 37 × 2 – 4 = 70 ceding element. While the secondlast is
127. 1; 2 13 27 113 561 3369 23581 greater by double +4 to its predecessor. ... In
Here 2 × 2 + 7 = 11; 11 × 3 – 6 = 27; this way we can conclude the definite pat-
27 × 4 + 5 = 113; 113 × 5 – 4 = 561; tern.]
561 × 6 + 3 = 3369; 3369 × 7 – 2 = 23581. 133. 3; The series follows the pattern:
128. 4; 50 51 47 56 42 65 29
1 1
Here 50  12  51 ;  7  , +6×2,  5 , + 4 × 2 ....
2 2
51  2 2  47 ; 47  32  56 ; Now, the required answer
56  4 2  40 ; 40  5 2  65 ; 1
= 5  7   6 a 
2
65  6  29 2
129. 3; 3 9 23 99 479 2881 20159 1
Here 3 × 2 + 3 = 9; 9 × 3 – 4 = 23; (6 + 6) × 2 = 24 (= b), 24  5  = 14.5(=c)
2
23 × 4 + 5 = 97; 97 × 5 – 6 = 479;
479 × 6 + 7 = 2881; 134. 2; The given series is  1 12 ,  2  2 2 ,  3  32 ,
2881 × 7 – 8 = 20159
 4  4 2 ,  5  52
K KUNDAN 41

Hence, the required answer number by 4 and 2 alternately.


ie +7, +11, +13, +17
= 2 1  12  3 a  ;
 ? = 21 + 13 = 34
3  2 – 2 2  2 b  ; 2  3  32  15 c 145. 4; The series is ×2+1, ×2+3, ×2+5, ×2+7, ...
 ? = 11 × 2 + 3 = 25
135. 5; The successive term is 6 more than the double
(146-150):
of its preceding term. Hence required answer
146. 4; Here 27 × 0.5 + 0.5 × 7 = 17; 17 × 1 + 1 × 6 = 23
= 4 × 2 + 6 = 14(=a);
23 × 1.5 + 1.5 × 5 = 42; 42 × 2 + 2 × 4 = 92
14 × 2 + 6 = 34 (= b);
92 × 2.5 + 2.5 × 3
34 × 2 + 6 = 74(=c);
= 237.5; 237.5 × 3 + 3 × 2 = 718.5.
74 × 2 + 6 = 154 (= d)
147. 1; The series is ×2 – 3; ×2 – 3; ... and so on,
4) 33 5) 17
Hence, 18 should be replaced by 19.
(136-140):
148. 5; The series is ×0.5 + 0.5; ×1 + 1; ×1.5 + 1.5; ×2 +
136. 2; Here the series consists of two series S1 and S 2 : 2; ×2.5 + 2.5; ×3 + 3.
S1: 24 26 28 30 Hence, 18 should be replaced by 20.
+2 +2 +2
S2: 14 15 16 149. 1; Here 5  13  6 ; 6  33  33 ;
+1 +1
3 3
Hence, 17 should be replaced by 15. 33  5  158 ; 158  7 = 501
137. 4; Here the series is 501  93  1230; 1230  113  2561.
5 × 1 + 12 = 6; 6 × 2 + 22 = 16; 16 × 3 + 32 = 57; 57
2 2
× 4 + 42 = 244; 244 × 5 + 52 = 1245; 150. 2; The series is  1 12 ;  2  2 ;  3  3 ;
1245 × 6 + 62 = 7506. 2 2 2
Hence, 248 should be replaced by 244. 4  4 ; 55 ; 6  6 .
138. 5; (2 + 7) × 1 = 9; (9 + 6) × 2 = 30; Hence, 196 should be replaced by 164.
(30 + 5) × 3 = 105; (105 + 4) × 4 = 436; (151-155):
(436 + 3) × 5 = 2195; (2195 + 2) × 6 = 13182 151. 4; The series is
32 should be replaced by 30. 2 2 2 2 2 2
 1 ,  3 ,  5 ,  7 ,  9 ,  11 .
139. 1; Here the series is:
11 × 0.5 + 0.5 = 6; 6 × 1 + 1 = 7; 7 × 1.5 + 1.5 = 12; Obviously, 128 should be replaced by 124.
12 × 2 + 2 = 26; 26 × 2.5 + 2.5 = 67.5; 67.5 × 3 + 152. 2; The series is ×5 + 0.5, ×5 + 0.5, ....
3 = 205.5. Obviously, 1327.5 should be replaced by 1328.
Hence, 8 should be replaced by 7. 153. 5; Look at the series from the right end. You get
140. 3; Here the series is: that 224 is the sum of the two preceding terms
3 × 8 – 7 = 17; 17 × 7 – 6 = 113; 113 × 6 – 5 = 673; (86 and 138). The same is true for 138 also [ 
673 × 5 – 4 = 3361; 52 + 86 = 138]. Obviously, the number 32 should
3361 × 4 – 3 = 13441; 13441 × 3 – 2 = 40321 be replaced by 34 [  16 + 18 = 34].
Hence, 16 should be replaced by 17. 154. 3; The series is ×6 – 1, ×5 – 2, ×4 – 3, ....
(141-145): Obviously, 4125 should be replaced by 4121.
141. 3; The series is ×3+1, ×3+2, ×3+4, ×3+8, ... 155. 1; The series is ×1 + 13, ×2 + 23, ×3 + 33, ....
 ? = 100 × 3 + 8 = 308 Obviously, 385 should be replaced by 388.
142. 5; The series is ×1–2, ×2–2, ×3–2, ×4–2, ... (156-160):
 ? = 4 × 3 – 2 = 10 156. 3; The series is -2, +4, -8, +16 ...
143. 2; The series is Hence, the wrong number is 34. It should be
2 2 2 2
replaced by 35.
1  1 ,  2  2 ,  3  3 ,  4  4 , ... 3 3 3
157. 1; The series is  1  1 ,  2  2 ,  3  3 ...
2
 ? = 6  2  2  16 Hence, the wrong number is 1704. It should be
144. 1; The series is based on increasing previous replaced by 1705.
42 K KUNDAN

158. 5; Here, 7 × 0.5 + 2.5 = 6 153 + 73 = 496


6 × 1 + (2.5 + 3.5) = 12
12 × 1.5 + (2.5 + 3.5 + 4.5) = 28.5 496  63  712
28.5 × 2 + (2.5 + 3.5 + 4.5 + 5.5) = 73 712  53  837
73 × 2.5 + (2.5 + 3.5 + 4.5 + 5.5 + 6.5) = 205
205 × 3 + (2.5 + 3.5 + 4.5 + 5.5 + 6.5 + 7.5) = 645 837  43  901
Hence, 9 should be replaced by 6. 901  33  928
159. 1; The series is; +13, +17, +19, +23, +29 ... (Con- Hence, the wrong number is 495.
secutive prime numbers) 169. 2; Here the series is:
Hence, the wrong number is 85. It should be 11 × 7 - 7 × 5 = 42
replaced by 87. 42 × 6 - 6 × 6 = 216
160. 2; Here, series is: 11, 22 , 33 , 44 , 55 ... 216 × 5 - 5 × 7 = 1045
Hence, the wrong number is 258. It should be 1045 × 4 - 4 × 8 = 4148
replaced by 256. 4148 × 3 - 3 × 9 = 12417
(161-165): Hence, the wrong number is 214.
161. 2; The series is ×2 + 3, ×2 + 3, ... 170. 4; Here, the series is
Hence, 108 should be replaced by 109. 488 ÷ 2 + 1.0 = 245
245 ÷ 2 + 1.5 = 124
162. 3; The series is  13 ,  23 ,  33 , ... 124 ÷ 2 + 2.0 = 64
Hence, 46 should be replaced by 43. 64 ÷ 2 + 2.5 = 34.5
163. 4; Here 2078 should be replaced by 2075. The 34.5 ÷ 2 + 3.0 = 20.25
series goes as follows: Hence, the wrong number is 35.
1×1+7×1=8 (171-175):
8 × 2 + 6 × 2 = 28 171. 1; Here, 3 × 1.5 + 0.5 = 5
28 × 3 + 5 × 3 = 99 5 × 2 + 1 = 11
99 × 4 + 4 × 4 = 412 11 × 2.5 + 1.5 = 29
412 × 5 + 3 × 5 = 2075; and 29 × 3 + 2 = 89
2075 × 6 + 2 × 6 = 12462. 89 × 3.5 + 2.5 = 314
164. 3; The series is ×2 + 1, ×3 - 2, ×4 + 3, ... Hence, in the given series 87 is a wrong num-
Hence, 1632 should be replaced by 1631. ber and should be replaced by 89.
165. 1; The series is ×0.5, ×1.5, ×2.5, ... 172. 3; Here,
Hence, 12.5 should be replaced by 12. 3
(166-170):
3
12  1  13 13  2 = 21
166. 1; Here the series is: 3
21  3  48
3
48  4  112
14 × 1 - 1 = 13
3
13 × 2 - 4 = 22 112  5  237
22 × 3 - 9 = 57 Hence, in the given series 20 is a wrong num-
57 × 4 - 16 = 212 ber and should be replaced by 21.
212 × 5 - 25 = 1035 173. 5; Here,
Hence, the wrong number is 55. 8.1 + 9.2 = 17.3
167. 3; Here the series is: 9.2 + 17.3 = 26.5
217 + 7 = 224 17.3 + 26.5 = 43.8
224 - 11 = 213 26.5 + 43.8 = 70.3
213 + 13 = 226 43.8 + 70.3 = 114.1
226 - 17 = 209 Hence, the wrong number is 71.5. It should be
209 + 19 = 228 replaced by 70.3.
Hence, the wrong number is 210. 174. 1; Here,
168. 5; Here the series is: 10 × 2 - 4 = 16 16 × 4 - 16 = 48
K KUNDAN 43

48 × 8 - 64 = 320 320 × 16 - 256 = 4864 4 + 10 = 14


4864 × 32 - 1024 = 154624 182. 3; 4 × 1.5 = 6 6 × 2 = 12
Hence, the wrong number is 154628. It should 12 × 2.5 = 30 30 × 3 = 90
be replaced by 154624. 90 × 3.5 = 315 315 × 4 = 1260
175. 2; Here,
183. 5; 25 16 ? 4 1
48 ÷ 2 = 24 24 × 3 = 72
2 2
72 ÷ 2 = 36 36 × 3 = 108 ie 5 2
4 3 2 1
2 2

108 ÷ 2 = 54 Hence, ? = 9
Hence, the wrong number is 34. It should be 184. 3; 15 - 3 = 12 12 + 5 = 17
replaced by 36. 17 - 7 = 10 10 + 11 = 21
(176-180): 21 - 13 = 8 8 + 17 = 25
176. 1; Here, 25 - 19 = 6
439 × 2 - 100 = 778 Note that 3, 5, 7, 11, 13, 17 and 19 are consecu-
778 × 2 - 100 = 1456 tive prime numbers
1456 × 2 - 100 = 2812 185. 1; 1 ? 27 64 125
2812 × 2 - 100 = 5524
ie 13 2
3
3
3
4
3
4
3
5524 × 2 - 100 = 10948
Hence the wrong number is 5624.  ?  23  8
177. 2; Here, 186. 2; 2 + 5 = 7 5 + 7 = 12
156 + 312 = 468 7 + 12 = 19 12 + 19 = 31
468 + 312 = 780 19 + 31 = 50 31 + 50 = 81
780 + 312 = 1092 187. 1; (1 + 2) × 2 = 6 (6 + 3) × 4 = 36
1092 + 312 = 1404 (36 + 4) × 6 = 240
1404 + 312 = 1716 (240 + 5) × 8 = 1960
Hence the wrong number is 1094. (1960 + 6) × 10 = 19660
178. 5; Here, (188-192):
5040 × 3 ÷ 5 = 3014 188. 5; 73
3014 × 3 ÷ 5 = 1814.4
1814.4 × 3 ÷ 5 = 1088.64 Here, 24  7 2 = 73
1088.64 × 3 ÷ 5 = 653.184 2
73  6  109
653.184 × 3 ÷ 5 = 391.9104
2
Hence there is no wrong number in the series 109  5  134
given. 2
134  4  150
179. 4; Here,
2
113 + 17 × 1 = 130 130 + 17 × 2 = 164 150  3  159
164 + 17 × 3 = 215 215 + 17 × 4 = 283 Hence, the question mark(?) should be replaced
283 + 17 × 5 = 368 by 73.
Hence the wrong number in the series is 293. 189. 4; 17 × 0.5 + 0.5 = 9
180. 3; Here, 9 × 1 + 1 = 10
36 × 1.5 = 54 54 × 2.5 = 135 10 × 1.5 + 1.5 = 16.5
135 × 3.5 = 472.5 472.5 × 4.5 = 2126.25 16.5 × 2 + 2 = 35
2126.25 × 5.5 = 11694.375 35 × 2.5 + 2.5 = 90
Hence the wrong number is 472.15. Hence, the question mark(?) should be replaced
(181-187): by 16.5.
181. 1; 12 + 2 = 14 14 + 3 = 17 190. 4; 3 × 2 + 14 = 20
17 - 4 = 13 13 - 5 = 8 20 × 3 + 18 = 78
8 + 6 = 14 14 + 7 = 21 78 × 4 + 20 = 332
21 - 8 = 13 13 - 9 = 4 332 × 5 + 20 = 1680
44 K KUNDAN

1680 × 6 + 18 = 10098 756 + 114 = 870


Hence, the question mark should be replaced 870 + 122 = 992
by 10098. 992 + 130 = 1122
191. 2; 13 × 2 + 4 = 30 (198-202):
30 × 2 + 6 = 66 198. 4; 15 + 3 = 18
66 × 2 + 8 = 140 18 – 2 = 16
140 × 2 + 10 = 290 16 + 3 = 19
290 × 2 + 12 = 592 19 – 2 = 17
Hence, the question mark should be replaced 17 + 3 = 20
by 290. 20 – 2 = 18
192. 1; 3 × 5 = 15 199. 1; 1050 × 0.4 = 420
5 × 15 = 75 420 × 0.4 = 168
15 × 75 = 1125 168 × 0.4 = 67.2
75 × 1125 = 84375 67.2 × 0.4 = 26.88
(193-197): 26.88 × 0.4 = 10.752
193. 4; 15 + 3 = 18 10.752×0.4 = 4.3008
18 – 2 = 16 200. 5; 0 6 24 60 120 210
16 + 3 = 19 +6 +18 +36 +60 +90
19 – 2 = 17 Look at the differences of two consecutive
17 + 3 = 20 terms of the series. We get the following pat-
20 – 2 = 18 tern:
194. 1; 1050 × 0.4 = 420 0+ 6×1=6
420 × 0.4 = 168 6 + 6 × 2 = 18
168 × 0.4 = 67.2 18 + 6 × 3 = 36
67.2 × 0.4 = 26.88 36 + 6 × 4 = 60
26.88 × 0.4 = 10.752 60 + 6 × 5 = 90
10.752×0.4 = 4.3008 90 + 6 × 6 = 126
195. 5; 0 6 24 60 120 210 Hence, the required term = 210 + 126 = 336
+6 +18 +36 +60 +90 201. 3; 32 + 17 = 49
Look at the differences of two consecutive 49 + 34 = 83
terms of the series. We get the following pat- 83 + 68 = 151
tern: 151 + 136 = 287
0+ 6×1=6 287 + 272 = 559
6 + 6 × 2 = 18 559 + 544 = 1103
18 + 6 × 3 = 36 202. 2; 462 + 90 = 552
36 + 6 × 4 = 60 552 + 98 = 650
60 + 6 × 5 = 90 650 + 106 = 756
90 + 6 × 6 = 126 756 + 114 = 870
Hence, the required term = 210 + 126 = 336 870 + 122 = 992
196. 3; 32 + 17 = 49 992 + 130 = 1122
49 + 34 = 83 (203-207):
83 + 68 = 151 203. 2; 12 × 2 - 2 = 22
151 + 136 = 287 22 × 3 + 3 = 69
287 + 272 = 559 69 × 4 - 4 = 272
559 + 544 = 1103 272 × 5 + 5 = 1365
197. 2; 462 + 90 = 552 1365 × 6 - 6 = 8184
552 + 98 = 650 204. 2; The series is
650 + 106 = 756 11, 22, 33, 44, 55
K KUNDAN 45

Hence, the question mark will be replaced by 4. 843 + 23 × 4 = 935


205. 5; 104  7 2  153 935 + 23 × 5 = 1050
1050 + 23 × 6 = 1188
2
153  6  189 (213-217):
2 1 2 3 4
189  5  214 213. 5; 1  1; 2  4; 3  27; 4  256;
2 5 6 7
214  4 = 230 5  3125; 6  46656; 7  823543
206. 2; 15 + 17 = 32
214. 2; 30 + 16 × 0 = 30;
17 + 32 = 49
32 + 49 = 81 30 + 16 × 1 = 46;
49 + 81 = 130 46 + 16 × 2 = 78;
81 + 130 = 211 78 + 16 × 3 = 126;
207. 3; 15 + 2 = 17 126 + 16 × 4 = 190;
17 + 4 = 21 190 + 16 × 5 = 270;
21 + 8 = 29 270 + 16 × 6 = 366
29 + 16 = 45 215. 4; 380 + 85 = 465;
45 + 32 = 77 465 + (85+7) = 557;
77 + 64 = 141 557 + (85+14) = 656;
(208-212): 656 + (85+21) = 762;
208. 3; 13 × 1 + 1 = 14
762 + (85+28) = 875;
14 × 2 + 2 = 30
875 + (85+35) = 995
30 × 3 + 3 = 93
93 × 4 + 4 = 376 2 2
376 × 5 + 5 = 1885 216. 1; 1250   500; 500   200;
5
  5
1885 × 6 + 6 = 11316
209. 2; 4 × 1.5 = 6 2 2 2
6 × 1.5 = 9 200   80; 80   32; 32   12.8;
9 × 1.5 = 13.5 5 5 5
13.5 × 1.5 = 20.25
2
20.25 × 1.5 = 30.375 12.8   5.12
30.375 × 1.5 = 45.5625 5
210. 4; 400 × 0.6 = 240
240 × 0.6 = 144 217. 3; 23 26 24 27 25 28
144 × 0.6 = 86.4
86.4 × 0.6 = 51.84
51.84 × 0.6 = 31.104 It is an alternate series. Ans: 26
31.104 × 0.6 = 18.6624 (218-222):
211. 1; 9 × 0.5 = 4.5 218. 3; The series is obtained by:
4.5 × 1.0 = 4.5 +12, +22, +32, +42, +52, ......
4.5 × 1.5 = 6.75 So 28 is wrong. It should be 29.
6.75 × 2.0 = 13.5 219. 5;
13.5 × 2.5 = 33.75
+4 +2 +0 +4 -46
33.75 × 3.0 = 101.25
212. 5; 1188 ×1 + 6 ×2+10 ×3+12 ×4+12 ×5+16 ×6-30

705 + 23 × 1 = 728 2 8 26 90 372 1876 11226


728 + 23 × 2 = 774 If we place 1870 in place of 1876 the series
774 + 23 × 3 = 843 progresses as per the following pattern:
46 K KUNDAN

2 8 26 90 372 1870 11226 221. 2; The series is obtained by


+1.5, +2, +2.5, +3, +3.5, +4, +4.5, ....
×1 + 6 ×2+10 ×3+12 ×4+12 ×5+10 ×6+6
So, 14.5 is wrong. It should be 14.
4 2 0 -2 -4
222. 2; The series is obtained by
220. 4; The series is obtained by +13, +23, +33, +43, +53, +63 ....
+2, +4, +8, +16, +32, +64 So, 22 is wrong. It should be 20.
So, 36 is wrong. It should be 35.

Exercise-2
(1-5): So, value of e would be 529.
1. 5; The series is × 5, +5, × 6, + 6. (16-20):
 a = 8 × 5 = 40, b = (40 + 5 =) 45, c = (45 × 6 =) 16. 4; The series is ×1.5, ×2, ×2.5, ×3 and so on. So,
270 46 is incorrect and new series starts from 46.
2. 3; The series is ×1 - 3, × 2 - 5, × 3 - 7, × 4 - 9 17. 2; The series is ×1 + 1, ×2 + 2, ×3 + 3, ......
 a = 9 × 1 - 3 = 6, b = 6 × 2 - 5 = 7, c = 7 × 3 - 18. 1; The series is ×2 - 1, ×2 - 1, ......
7 = 14, d = 14 × 4 - 9 = 47 19. 5; The series is +23, -17, +23, -17, ......
3. 4; The differences of the two successive terms 20. 1; The series is ×2 + 1, +10, ×2 + 2, +20, ×2 + 3,
are 32, 22, 14, 8. The sequence of which is 62 - +30, ...
4, 52 - 3, 42 - 2, 32 - 1 (21-25):
4. 1; The differences are 4, 8, 16, 32. 21. 2; The series is ×(-6), ×7, × (-8) ....
5. 3; The series is × 3 - 2, and × 2 - 3 alternately 22. 3; The series is +22, +32, +42, ...
 a = 4 × 3 - 2 = 10, b = 10 × 2 - 3 = 17, 23. 1; The series is × 8 - 28, ×7 - 24, ×6 - 20,...
c = 17 × 3 - 2 = 49 24. 2; The series is ×2 - 4, ×3 - 6, ×4 - 8.
(6-10): 25. 2; The series is +5, +10, +20
6. 4; The series is ×2 + 1, ×2 + 2, ×2 + 3, ×2 + 4, ..... (26-30):
 Wrong no. is 11. 26. 2; The series is ×7 - 17, ×6 - 15, × 5 - 13, ×4 - 11, ....
7. 5; The series is, 12  1, 22  1, 52  1, .... 27. 1; The series is +12, -22, +32, -42, ......
28. 4; This is an alternate series. First is -12, -24, -36
So, the wrong no. is 10. Answer will be 10202. and , second is +12, +24, +36....
8. 3; The series is  12 ,22 ,32 ,42 ,.... 29. 2; The series is ×2 + 1, ×2 + 2, ×2 + 3, ×2 + 4 ....
30. 5; The series is ÷1 + 4, ÷2 + 3, ÷3 + 2, ÷4 + 1 ....
The wrong no. is 17.
(31-35):
9. 4; The series is ×2 - 5, ×2 - 4, ×2 - 3, ×2 - 2, ....
31. 2; The series is ×3 - 11, ×3 - 11, .....
The wrong no. is 20.
So wrong term is 20
10. 4; The series is ÷2 - 4, ÷2 - 4, ... Wrong no. is 26.
(11-15):  40 ÷ 20 - 2 = 0
32. 4; The series is ×2 + 3, ×2 + 3 .....
11. 5; The series is × 0.5, × 1.0, × 1.5, × 2,. ......
So wrong term is 40.
12. 5; The series is, +
1600  120
12  1,  32  3,  52  5,  7 2  7....  = 40
37
13. 4; The series is × 3 + 5, × 5 + 7, × 7 + 9, ....
14. 1; The series is × 2 + 4, × 2 + 6, × 2 + 10, × 2 + 16, 33. 3; The series is × 2 + 1, ×2 - 1, ×2 + 1, ×2 - 1, .....
× 2 + 24 ..... So wrong term is 44
15. 5; The series is –82, +65, –50, +37, –26  3 × 44 + 17 = 149.
or, The series is - (92 + 1), + (82 + 1), -(72 +1), + 34. 1; The series is × 1.5, × 2, × 2.5, × 3 ......
(62 + 1), -(52 + 1) So wrong term is 55.
 3.5 × 55 + 10 = 202.5
K KUNDAN 47

35. 2; The series is × 2 - 5, × 2 + 5, × 2 - 5, × 2 + 5 ..... 55. 2; The series is +(12 - 1), -(22 - 2), + (32 - 3) ....
So wrong term is 18. (56-60):
 324 - 36 = 288. 56. 2; The series is ×1 + 12, ×2 + 22, ×3 + 32, ×4 + 42 .....
(36-40): 57. 3; The series is ×0.1, ×0.3, ×0.5, ×0.7 .....
36. 4; The series is: 58. 4; The series is +(12 - 1), -(32 - 3), +(52 - 5), -(72 - 7)
....
 2  12 ,  3  2 2 ,  2  12 ,  3  2 2 , ....
59. 1; The series is ×9 + 1, ×7 + 2, ×5 + 3, ×3 + 4....
 Wrong number = 122 = B
1 1 1 1
 3B ÷ 8  3 × 122 ÷ 8 = 45.75 60. 5; The series is × –6, × –5, × –4, × – 3 ...
37. 2; The series is ×2 + 7 2 2 2 2
 wrong number = 56 = B (61-65):
 20% of 5B  20% of 5 × 56 = 56. 61. 5; The series is ×1 + 0.75 × 12, ×2 + 0.75 × (2)2, .....
38. 3; The series is: (x + 1) × 2 62. 4; The series is ×1 + 92, ×2 - 82, ×3 +72, ×4 - 62 ......
 wrong number = 47 = B Replace (4) with (5).
 2nd number = (47 + 1) × 2 = 96 63. 2; The series is -292, +272, -252, +232 ......
39. 1; The series is –4 followed by ÷2 Replace (2) with (5).
Wrong number = 59 64. 1; The series is ×2, ÷3, ×4, ÷5 ....
 B ÷ 2 + 21 = 59 ÷ 2 + 21 = 50.5 Replace (1) with (4).
40. 2; The series is: ×1 + 10, ×2 + 8, ×3 + 6, .... 65. 3; The series is -53, + 47, -43, +41 .....
So, wrong number is 38. (descending prime numbers)
 2nd number = (38 × 1) + 10 = 48 Replace (3) with (5).
(41-45): (66-70):
41. 3; The series is × 1.5 + 1, ×2.5 + 3, ×3.5 + 5 .... 66. 1; The series is +22, +32, +52 ........ (Consecutive
42. 2; The series is +0.52, +12, +1.52, +22, .... prime numbers)
43. 5; The series is +41, +47, +59, +67, .... (Alternate 67. 5; The series is ÷ 9, ×8, ÷7, ×6 ......
prime No.) 68. 2; The series is -232, +192, -152, + 112 ......
44. 1; First digit from the left of the series is 2, 3, 4; 69. 3; The series is ×1 + 13, ×2 - 33, ×3 + 53, ×4 - 73 .....
other digits of the series are 70. 5; The series is +15, -13, +11, -9, +7 ....
23 = 8, 33 = 27, 43 = 64, .... (71-75):
45. 2; The series is ×1 + 1, ×2 + 2, ×3 + 3, ×4 + 4, ... 71. 2; The series is +2,  (2) 2 ,  (2) 3 ,  (2) 4 ,..........
(46-50):
Replace (2) with (4).
1 2 1 2 1 2 72. 1; The series is ×1+5, ×2+10, ×3-15, ×4-20,
46. 2; The series is   1 ,   2 ,   3 ,... ×5+25,.....
4 4 4
47. 3; The series is 18 × 1 – 1 × 2 = 16 × 2 – 2 × 3 Replace (1) with (5).
= 26 × 3 – 3 × 4 = 66 × 4 – 4 × 5 73. 4; The series is  17 2 ,13 2 ,9 2 ,5 2 ,12 ,......... .
= 244 × 5 – 5 × 6, .... Replace (4) with (5).
48. 1; The series is ×0.5, ×1.5, ×2.5, ×3.5, .... 74. 5; The series is:
49. 4; The series is -(92 - 9), -(82 - 8), -(72 - 7), ...
50. 5; The series is ÷6, ÷3, ÷1.5, ....  12  1,  2  2 2 ,  32  3,  4  4 2 , ....
(51-55): 75. 3; The series is 11,  9,  7,  5,  3,..........
51. 3; The series is ×2 + 3, ×3 + 2 alternately.
Replace (3) with (5).
52. 5; Ans = 27. The series is +12, -0, +22 , -1, +32, .....
(76-80):
53. 1; The series is 664 - 46 = 618 - 81 = 537 - 73
76. 3; The series is:
= 464 .....
 12  12 ,  2 2  2 2 ,  32  32 ,  4 2  4 2 ,...
3 3 77. 5; The series is :
54. 4; The series is ×2, × ,  2,  ,....
4 4
 112 ,  132 ,  15 2 ,  17 2 ,  19 2 ,  212 ,...
48 K KUNDAN

78. 1; The series is: 6,  5,  4,  3,  2, 1,....... (96-100):


96. 4; The series is ×2 + 1, ×1 - 2 alternately.
79. 1; Here 5 × 8 + 2 = 42
Similarly, 97. 5; Ans = 676. The series is  1  12 ,  2  2 2 ,
42 × 7 + 3 = 297
6 × 8 + 2 = 50 (a)  3  32 ,  4  4 2 ...
297 × 6 + 4 = 1786 98. 3; The series is ×2 + 0.5, ×2 + 2, × 2 + 8, ×2 + 32 ....
50 × 7 + 3 = 353 (b) 99. 2; The series is  1  13 ,  2  2 3 ,  3  33 ,
1786 × 5 + 5 = 8935
353 × 6 + 4 = 2122 (c)  4  43 ....
8935 × 4 + 6 = 35746
100. 1; The series is  532 ,  45 2 ,  37 2 ,  29 2 .....
2122 × 5 + 5 = 10615 (d)
80. 4; The series is ×2+6, ×2+7, ×2+6, ×2+7, ........ (101-105):
(81-85): 101. 2; The series is +2,  22 ,  23 , + 3,  32 ,  33.....
81. 3; The series is ÷25, ×20, ÷ 15, × 10 ..... Replace (2) with (5).
82. 2; The series is ×1 + 1, ÷2 + 2, ×3 + 3, ÷4 + 4 .... 102. 5; The series is  112 ,  152 ,  19 2 ,  232 ,  27 2.....
83. 1; The series is +232, +272, +312, +352 .....
103. 3; The series is ÷7, ×9, ÷11, ×13, ....
84. 5; The series is ×3 + 1, ×4 - 1, ×5 + 1, ×6 - 1 ....
Replace (3) with (5)
85. 2; The series is  2 1 in each terms. 104. 1; The series is ×12 +5,  12  10,  22  15,  22  20
3
(86-90): ......
Replace (1) with (4).
1 2 3 4 105. 2; The series is ×0.25 + 4, ×0.5 + 4, ×0.75 + 4, ×1 +
86. 1; The series is  ,  ,  ,  .......
2 3 4 5 4 ......
87. 2; The series is ×1 - 1, ×2 + 2 , ×32 - 3, ×4 + 42 ....
2 2
Replace (2) with (3).
88. 3; The series is +112, -92, +72, -52 .... (106-110):
89. 3; The series is each next term is 10% more than 106. 5; Ans = 64. The series is  1, ×2,  3, ×4,  5,
the preceding term. ........
90. 5; Ans = 717. The series is +(132 + 13), -(122 + Note: We find that first series is on pattern of multi-
12), +(112 + 11), - (102 + 10), ..... plication or division only. In such case, each
(91-95): term of second series will be in the same ratio
91. 3; The series is ×1 + 1; ×2 - 1; ×3 + 1; .... of the corresponding term of the first series.
Replace (3) with (5).  480 ÷ 4 = 120
92. 5; The series is ÷30, ×25, ÷20, ×15 ...... So, e = 256 ÷ 4 = 64
93. 2; The series is  1  13 ,  2  23 ,  3  33 , 107. 3; The series is: ×1+8, ×2+8, ×3+8, ×4+8, .......
108. 2; The series is:  612 ,  512 ,  412 ,  312 , ........
 4  43 ....
Replace (2) with (4). 109. 1; The series is:
94. 1; The series is  21  2 2 ,  2 2  2 3 ,  2 3  2 4 , 1  12 ,  2  2 2 ,  3  32 ,  4  4 2 , ......
110. 2; The series is:
 2 4  2 5 ....
Replace (1) with (4). 12  13 ,  2 2  2 3 ,  32  33 ,  4 2  43 , .....
95. 4; The series is  (37) 2 ,  (31) 2 ,  ( 29) 2 , (111-115):
111. 5; Ans = 48.
 (23) 2 .... The series is ÷6, ÷5, ÷ 4, ÷3, ........
(Prime number) 112. 4; The series is ×2 + 3, ×2 + 3, ×2 + 3 ...
Replace (4) with (5). 113. 3; The series is ×1 + 1, ×2 - 2, ×3 + 3,
114. 2; The series is 72, 7 × 8, 82, 8 × 9, 92,
K KUNDAN 49

115. 1; The series is an alternate series having two 134. 1; The series is: every digit of the no. increases
series: by 2 in each step.
S1 = 25 26 A 153; ×1 + 1, ×2 - 2, 135. 5; Ans = -1375. The series is (+1) × 5, (+2) × 5,
S2 = 45 44 90 267; ×1 - 1, ×2 + 2, ×3 - 3 ..... (+3) × 5 .....
 A = 50 (136-140):
(116-120): 136. 5; The series is: ÷2 + 1, ×3 + 4, ÷4 + 7, ×5 + 10 .....
116. 2; The series is: ×1 + 2, ×2 + 3, ×3 + 5 (sum is 137. 1; The series is:  232 ,  192 ,  152 ,  112 , ......
consecutive prime nos.)
Replace (1) with (3).
117. 3; The series is: ×12 + 1, ×22 - 2, ×32 + 3 ....
138. 4; The series is 22  1, 52  2, 8 2  3, 112  4 .....
118. 1; The series is:  8 2 ,  122 ,  16 2 .
Replace (4) with (5).
119. 3; The series is ÷5, × 7, ÷9, × 11, .....
120. 1; The series is ×1 + 2, ×2 + 1, alternately. 139. 2; The series is ×2 + 1,  32  1, ×4 + 1,  52 1 , .....
(121-125): Replace (2) with (5).
121. 3; The series is × 2 - 8, × 2 - 7, × 2 - 6, × 2 - 5 ...... 140. 3; The series is: ÷4, ×5, ÷6, ×7 ....
Replace (3) with (4). Replace (3) with (5).
122. 1; The series is ×1.5 + 1, × 3 - 2,× 4.5 + 4, ×6 - 8 (141-145):
.... 141. 5; The series is × (-2) + 2, ×(-3) + 2, ×(-4) + 2 ....
Replace (1) with (4). 142. 2; The series consists of two series:
123. 5; The series is ×6 - 6, ×5 - 5, ×4 - 4, ×3 - 3, ....... = 4 65 675 ; 10  52
S1
124. 4; The series is ÷6 - 4, ÷5 - 4, ÷4 - 4, ......
Replace (4) with (5). S2= 548 717 838 ;  132 ,  112 ,  92....
125. 2; The series is: 10 , 21 , 32 , 43 , 5 4 ........ Replace (2) with (3).
143. 1; The series is ×1 + 3, ×2 - 5, ×3 + 7, ×4 - 9, ....
Replace (2) with (3).
Replace (1) with (4).
(126-130):
144. 3; The series is:  55 ,  4 4 ,  33 ,  2 2 , ....
126. 4; The series is ×1 0 + 6,  21  6 ,  32  6 ,
Replace (3) with (5).
 43  6 ..... 145. 2; The series is
127. 5; Ans = 369. The series is  112  11 , 12  11, 2 2  22, 32  33, 4 2  44 ....
2 2 2
 13  13 ,  15  15,  17  17 ..... Replace (2) with (5).
128. 1; The series is ×1.5, ×2, ×2.5, ×3 ..... (146-150):
129. 2; The series is ×1 + 12, × 2 - 10, × 3 + 8, × 4 - 6 ... 146. 1; The series is ×2 – 1, ×3 + 3, ×4 – 3, ×5 + 5, ×5 -
5 .......
130. 3; The series is  21 ,  32 ,  43 ,  54 ......
147. 2; The series is ×8 + 1, ×7 – 2, ×6 + 3, ×5 – 4, ......
(131-135):
148. 3; The series is ×7 - 26, ×6 + 20, ×5 - 14, ×4 + 8, ......
131. 2; The series is ×1 + 2, ×2 + 3, ×3 + 4, ×4 + 5 .....
149. 4; The series is
132. 2; The series is  12  12 , ×2 - 2,  32  3 2 , ×4 - 4,
 14 2 ,  12 2 ,  10 2 ,  82 , .............
.....
7
133. 4; The series is 12  1, 3  2 2 , 5 2  3 , 7  4 2 , 150. 5; The series is ×5 and  alternately..
20
9 2  5 , .... (151-155):
2 151. 5; Ans = 47. The series is
Here, 2 = 1  1 . In the same way
122 = 112  1 . Therefore a = 13  2 2  9 ,
      
 0  12 ,  1  2 2 , 2  32 , 3  4 2 , ..... 
Note: Once you find the pattern in which only addi-
b = 15 2  3  228 , c = 17  4 2  17  16  1 , tion and subtraction is used, then you don’t
and so on. need to find each term of the second series.
50 K KUNDAN

Find the difference of first terms of the two


12  2,  2 2  2,  32  2,  4 2  2, ...
series, ie 100 – 45 = 55.
Now, e – (–8) = 55  e = 47 167. 4; The series is ×6 – 1, ×5 + 2, ×4 - 3, ×3 + 4, ....

2 3 4 5 168. 1; The series is  282 ,  25 2 ,  22 2 ,  19 2


152. 4; The series is  ,  ,  ,  ..... 169. 2; The series is
3 4 5 6
153. 3; The series is  37 2 ,  332 ,  29 2 ,252 , ....  1  0.5 12 ,2  0.5  2 2 ,  3  0.5  32 , ...

154. 2; The series is  2 2  5,  5  2 2 alternately.. 3 3 3 3


170. 5; Ans = 11.25. The series is  ,  ,  , 
2 4 6 8
155. 1; The series is ÷2, ×3, ÷4, ....
(156-160): ....
156. 5; The series is × 2 - 13, ×3 + 11, ×4 - 9, ... (171-175):
157. 2; The series is 1 1 1 1
2 2 2 2
171. 5; The series is × – , × 1 + 1, ×1 – 1 , ×2
1  1,  2  2 ,  3  3,  4  4 , ... 2 2 2 2
Replace (2) with (5). + 2, ...
158. 4; The series is 1 3
172. 4; The series is × , ÷1, × , ÷2, ....
 2
  2
 
 9  9 ,  11  11 , 13  13 ,  15  15 , ... 2
  2
 2 2
Replace (4) with (5). 173. 2; The series is
159. 1; The series is ×1.5, ×3, ×4.5, ×6, ... 12  10,  2 2  5,  32  10,  4 2  5 , .....
Replace (1) with (4).
160. 3; The series is ÷2 - 4 in each term.     
174. 4; The series is + 12  2 , – 2 2  3 , + 32  4 , – 
Replace (3) with (5).
(161-165): 4 2

 5 , .....
161. 4; The series is 175. 3; The series is  12 ,  32 ,  5 2 ,  7 2 , ....
2 2 2 2 2 2 2 2
1  2  0, 2  3  1, 3  4  2, 4  5  3... (176-180):
Replace (4) with (5). 176. 1; The series is
162. 5; The series is an alternate series having 1  12 ,  2  2,  3  32 ,  4  4,  5  5 2 , ....
S1 = 343 54 279; property is 177. 2; The series is ×2 – 1, ×3 + 1, ×4 – 1, ×5 + 1, ×6 –
 17 2 ,  152 ,  132... 1, ......
1 2 3 4
S2 = 12 45 177; property is  22  3 . 178. 4; The series is  ,  ,  ,  , .....
2 3 4 5
2 2 2 2 2
163. 3; The series is  ,  ,  ,  ,  ... 179. 2; The series is  12 2 ,  10 2 ,  8 2 ,  6 2 , ....
8 7 6 5 4
Replace (3) with (5).   
180. 2; The series is  12  2 2  32 ,  2 2  32  4 2 , 
164. 2; The series is
   
 32  4 2  5 2 ,  4 2  52  6 2 , .....
 12  1,2 2  2,  32  3,  4 2  4... (181-185):
Replace (2) with (4). 181. 5; The series is
165. 1; The series is
12  3,  2  6,  32  9,  4  12, ...
 2 2

 9  1 ,  8  2,  7  3 ,  6  4, ... 2 2
 182. 5; The series is
Replace (1) with (2).
(166-170):  
2
 2
   
 1  1 ,  2  2 ,  3  3 ,  4  4 , .... 
166. 3; The series is 183. 1; The series is ÷2, ×3, +4, -5, ÷6, ...
K KUNDAN 51

203. 2; The series is ×1 +3, ×3 +5, ×5 +7, ×7 +9, ...


184. 3; The series is  312 ,  352 ,  39 2 ,  432 , ...
Replace (2) with (5).
185. 2; The series is ×1 + 1, ×5 - 1, ×9 + 1, ×13 - 1, ....
(186-190): 204. 5; The series is ×2, ÷3,  (2) 2 ,  (3) 2 , ...
186. 1; The series is ×1 + 1 ×5, ×2 – 2 ×7, ×3 + 3 ×9, ×4 205. 4; The series is ×1 – 3, ×3 – 6, ×5 – 9, ...
– 4 × 11, .... Replace (4) with (5).
187. 5; Ans = 101. The series is 1 2 2 ,  2  12 , (206-210):
206. 4; The series is ×2, ÷1.5, ×3, ÷2.5, ....
 3  2 2 ,  4  12 , ... Replace (4) with (5).
188. 1; The series is ×8 – 28, ×7 – 24, ×6 – 20, ×5 – 16,
207. 1; The series is  112 ,  132 ,  15 2 ,  17 2 , ....
...
189. 2; The series is ×1.5 + 1.5, ×2 + 2, ×2.5 + 2.5, ×3 + Replace (1) with (5).
3, .... 208. 5; The series is  12  2,  2  3,  32  4,  4  5
7 209. 2; The series is ×1 + 1, ×2 - 2, ×3 + 3, ×4 - 4, ...
190. 1; The series is ×4 and  alternately.. Replace (2) with (5).
10
210. 5; The series is +1, -2, +3, -4, ...
(191-195):
(211-215):
191. 4; The series is ×2 + 1, ×1 + 2 alternately.
211. 4; The series is ×1, ×1.5, ×2.0, ×2.5 ....
192. 1; The series is ÷3 - 7, ÷3 - 6, ÷3 - 5, .....
193. 5; The series is ×1.5, ×2, ×2.5, ×3, .... 212. 2; The series is  2 3  2,  33  3,  4 4  4,
194. 2; The series is -23, +19, -15, +11, -7, +3, ....
+ 5 5  5, .....
195. 3; The series is
213. 1; The series is +6, ×6, +5, ×5 ....
12  4,  2  8,  32  12,  4  16, .... 214. 3; The series is ×0.5 + 0.5, ×1 + 1, ×1.5 + 1.5, ×2 +
(196-200): 2 ....
196. 3; The series is ×5 and –5 alternately. 215. 4; The series is  8 2 ,  7 2 ,  6 2 ,  5 2 , .....
197. 4; The ten’s and hundred’s (if present) digits of
(216-220):
successive numbers are sum of digits of pre-
216. 2; The logic of the sequence is:
vious number while unit’s digit is constant, ie
6, in all the numbers in the given series.  25 ,  2 4 ,  2 3 , 2 2 ,  21
For example : 36 = 6 + 3 / 6 = 96; 217. 2; The logic of the sequence is: ÷1.5, × 2 alter-
9 + 6 / 6 = 156; 1 + 5 + 6 / 6 = 126 nately.
198. 1; The series is ×6, ÷5, ×4, ÷3, ...
199. 2; The series is ÷5 × 4 and ×5×4 alternatively. 1
218. 4; The logic of the sequence is:  2
200. 2; The series is ×2–1, ×2–5, ×2–9, ... 3
(201-205): 219. 4; The logic of the sequence is: +24, +26, +30,
201. 1; The series is ×1, ×3, ×5, ×7, ×9, ×11, .... +32, +36, +38
Replace (1) with (2). 220. 1; The logic of the sequence is -22, -20, -18, -16,
202. 3; The series is based on the following pattern: -14, -12
1020 – 600 ( 25 2  25)  420 (221-225):
221. 3; The logic is  1 12 ,  2  2 2 ,  3  32 ,
2
420 + 420 ( 21  21)  840
 4  4 2 ,  5  52
2
840 – 272 ( 17  17)  568 222. 2; Every term is the sum of all previous terms.
223. 5; The terms of the series are
568 + 156 ( 132  13)  724
03  1,13  1, 2 3  1, 33  1, 4 3  1, 53  1.
724 - 72 ( 9 2  9)  652
224. 5; The logic of the series is ×1 - 1, ×2 - 1, ×3 - 1, ×4
Replace (3) with (5) - 1, ×5 - 1
52 K KUNDAN

225. 3; The difference of the terms are 1, 4, 27, 256 or 361 × 7 - 8 = 2519; 2519 × 8 + 9 = 20161
244. 1; 1 8 66 460 2758 13785 55146
11 , 2 2 , 33 , 4 4.
Here 1 × 9 - 1 = 8; 8 × 8 + 2 = 66; 66 × 7 - 3 = 459;
(226-230): 459 × 6 + 4 = 2758; 2758 × 5 - 5 = 13785; 13785
226. 4; The logic is ×2, ×4, ×6, ×8 × 4 + 6 = 55146
227. 2; The logic is ×2 - 1, ×2 + 3, ×2 - 5, ×2 + 7, ×2 - 9. 245. 2; 3 1 3, 0.7 3 0.6 3
228. 5; 6  2 2  2, 40  6 2  4  1606  40 2  6 1 1
Here 3   1; 1 × 3 = 3; 3   0.75;
2579244  1606  8 2 3 4
229. 5; The logic is –7 ÷ 5, –5 ÷ 4, –3 ÷ 3, –1 ÷ 2 1
230. 5; Every term is the square root of its previous 0.75 × 4 = 3; 3   0.6; 0.6 × 5 = 3
5
term.
(231-235): 1
3  0.5; 0.5  6  3
231. 4; The series is +0.5 × 0.5, +1 × 1, + 1.5 × 1.5, +2 × 6
2, .... (246-250):
[Note: First add and then multiply.]
232. 1; The series is: ×6.5, ×6, ×5.5, ×5, ...       
246. 2;  12  1 ,  2 2  2 ,  32  3 ,  4 2  4 ... 
233. 5; The series is:
1 3 5
 6  13 ,  5  2 3 ,  4  33 ,  3  43 , ... 247. 4;  ,  1,  ,  2, 
2 2 2
234. 3; The series is: 3 + 6 + 1 = 10, 6 + 10 + 2 = 18, 10 248. 1; +2, -3, ×4, ÷5, +6 ...
+ 18 + 3 = 31, ... 249. 2; ×1 + 3, ×3 + 5, ×5 + 7, ×7 + 9, ×9 + 11, ...
235. 3; The series is: ×2.5, ×3.5, ×4.5, ×5.5, ... 250. 3; ×2 - 4, ×3 - 8, ×4 - 12, ×5 - 16, ×6 - 20, ...
(236-240): (251-256):
236. 1; The series is ×2 + 2, ÷2 + 4, ×2 + 6, ÷2 + 8, .... 251. 3; Here the series is as follows:
237. 5; Ans = 14. The series is ×0.5 + 0.5, ×1 + 1, ×1.5 12 × 6 - 6 × 7 = 30
+ 1.5, ×2 + 2 ... 30 × 5 - 5 × 6 = 120
238. 4; The series is: 120 × 4 - 4 × 5 = 460
460 × 3 - 3 × 4 = 1368
 12  2 2 ,  2 2  32 , 32  4 2 ,  4 2  5 2 , ...
1368 × 2 - 2 × 3 = 2730
b = 24, d = 90  ratio = 4 : 15 Similarly,
239. 3; The series is: 16 × 6 - 6 × 7 = 54
×1 + 0.5 × 12, ×2 + 0.5 × 22, ×3 + 0.5 × 32, ... 54 × 5 - 5 × 6 = 240
240. 2; The series is:  19 2 ,  17 2 ,  15 2 ,  132 , ... 240 × 4 - 4 × 5 = 940
(241-245): 940 × 3 - 3 × 4 = 2808
241. 2; 2 6 13 26 54 100 197 Hence, 2808 will come in place of (d).
Here 2 × 2 + 2 = 6; 6 × 2 + 1 = 13; 252. 5; Here the series is as follows
13 × 2 + 0 = 26; 26 × 2 - 1 = 51; 154 × 3 = 462
51 × 2 - 2 = 100; 100 × 2 - 3 = 197 462 ÷ 2 = 231
242. 4; 56 57 48 73 24 105 -10 231 × 3 = 693
693 ÷ 2 = 346.5
Here 56  12  57; 346.5 × 3 = 1039.5
Similarly,
57  3 2  48; 48  5 2  73; 73  7 2  24;
276 × 3 = 828
24  9 2  105; 105  112  16 828 ÷ 2 = 414
243. 4; 2 2 13 59 363 2519 20161 414 × 3 = 1242
Here 2 × 3 - 4 = 2; 2 × 4 + 5 = 13; 1242 ÷ 2 = 621
13 × 5 - 6 = 59; 59 × 6 + 7 = 361; 621 × 3 = 1863
K KUNDAN 53

Hence, 1863 will come in place of (e). 3


662  6  446
253. 2; Here, the series is as follows:
7 × 13 = 91 Here, we have to find the number which will
91 × 11 = 1001 come in place of (d).
1001 × 7 = 7007 Hence, the required number
7007 × 5 = 35035 = 204 + (662 - 582) = 284
35035 × 3 = 105105 255. 4; Here, the series is as follows:
Note that 3, 5, 7, 11 and 13 are consecutive (85 + 1) × 0.5 = 43
prime numbers. (43 + 1) × 1.0 = 44
Hence, the required number (44 + 1) × 1.5 = 67.5
= 14.5 × 13 × 11 × 7 = 14514.5 (67.5 + 1) × 2.0 = 137
254. 1; Here, the series is as follows (137 + 1) × 2.5 = 345
Similarly,
3
582  2  574 (125 + 1) × 0.5 = 63
3 (63 + 1) × 1.0 = 64
574  3  601
(64 + 1) × 1.5 = 97.5
3
601  4  537 Hence, 97.5 will come in place of (c).
3
537  5  662
Chapter 13

Cloze Test
Direction: In the following passages there are 9. 1) increasingly 2) always
blanks, each of which has been numbered. These 3) gradually 4) deliberately
numbers are printed below the passage and 5) badly
against each five words are suggested, one of 10. 1) enlighten 2) validate
which fits the blank appropriately. Find out the 3) negate 4) underestimate
appropriate word in each case. 5) belittle
Absorb = to hold somebody’s attention or
Passage 1 interest completely
Ravage = to damage something badly; to
Economic backwardness of a region is (1) by the destroy something
co-existence of unutilized or underutilized (2)on the The ravages = the damaging effect of something;
one hand, and (3)natural resources, on the other. of something the destruction done by something
Economic development essentially means a process Kingpin = a person of thing essential for
of (4) change whereby the real per capita income of an success
economy (5) over a period of time. Then, a simple but Enhance = to increase or improve further the
good quality, value or status of
meaningful question arises: what causes economic
something
development? Or what makes a country developed? Incredible = difficult to believe; extraordinary
This question has absorbed the (6) of scholars of Diminish = to decrease; to become or make
socio-economic change for decades. Going through the something smaller or less
(7) history of developed countries like America, Russia Degenerate = to pass into a worse physical, mental
and Japan, man is essentially found as (8) in the or moral state that one which is
process of economic development. Japan, whose considered normal or desirable.
economy was (9) damaged from the ravages of the Succumb = to fail to resist an illness, an attack
etc
Second World War, is the clearest example of our time
Pivotal = central; of great importance because
to (10) kingpin role in economic development. other things depend on it.
1. 1) developed 2) cured Vicious = acting or done with evil intentions;
3) improved 4) enhanced cruel and violent.
5) characterised Enlighten = to give somebody greater knowledge
2. 1) sources 2) finances or understanding
3) funds 4) manpower Validate = t o show t hat something is
5) industries reasonable or logical; to make
something legally valid
3. 1) exhaustive 2) unexploited
Negate = to cancel the effect of something; to
3) abundant 4) indefinite nullify something
5) unreliable Belittle = to make a person or an action seem
4. 1) upward 2) drastic unimportant or of little value.
3) negligible 4) incredible
5) sudden Passage 2
5. 1) diminishes 2) degenerates
Although John Wisdom’s writings in philosophy
3) increases 4) succumbs
show clearly the influence of Wittgenstein, they
5) stabilizes
nevertheless also display a (1) originality. Despite the
6. 1) plans 2) attempts
(2) and difficulty of his style, a careful reading of
3) attention 4) resources
Wisdom is seldom (3). He is a unique kind of genius
5) strategy
in philosophy.
7. 1) existing 2) glorious
This essay is an excellent example of Wisdom’s
3) ancient 4) economic
repeate d attempts to (4) the ultimate bases of
5) discouraging
philosophical perplexity. A great deal of the time
8. 1) pivotal 2) neutral
Wisdom is ( 5) inte re ste d in finding out why
3) insignificant 4) enchanted
me taphysicians fe e l (6) to utte r such strange
5) vicious
sentences (e.g. “Time is unreal”, There are no material
410 Test of English Language

things”, etc). According to Wisdom, such sentences Augment = to make something larger in number
are both false (and perhaps meaningless) and yet (7). or size; to increase something
Even more than Wittgenstein, Wisdom has stressed Fortify = to make somebody feed stronger,
braver etc.
the “therapeutic” conception of philosophy, a view that
Explore = to examine something thoroughly in
comes out clearly in this essay where he emphasizes order to test it or found out about it.
the analogy be tween philosophical and neurotic Inadvertent = not done deliber ately or
distress (8) them with other kinds of problems. intentionally
The reader who is interested in gaining a fuller (9) Reluctant = unwilling and therefore slow to act,
with Wisdom’s thought is referred to his famous article agree etc.
“Gods in Philosophy and Psycho-analysis”. Other Minds Allude = to mention somebody/something
is Wisdom’s most (10) discussion of a single topic briefly or indirectly.
Ad apt = to make something suitable for a
and in many ways his finest work.
new use situation etc.
1. 1) concise 2) virtual 3) marked Acquaintance = slight knowledge of something
4) limited 5) relative Prolong = to make something last longer; to
2. 1) individuality 2) novelty extend something
3) originality 4) complexity Prolific = producing many works.
5) creativity
3. 1) unprofitable 2) useful Passage 3
3) advantageous 4) unreliable The latest stage of the continuing (1) between India
5) durable

K
and the United States on the nuclear issue is now
4. 1) jettison 2) delimit punctuated with pleasing diplomatic observations. Our
3) augment 4) fortify latest round of talks with the American Deputy
5) explore Secretary of State is “positive and encouraging”. The
5. 1) admirably 2) primarily US Deputy Secretary of State remarked that “none of
3) inadvertently 4) reluctantly us are pleased to have any clouds over the (2)”. We in
5) happily India know that these clouds have (3) towards the
6. 1) depressed 2) confined subcontinent from the West. The US can easily
3) alluded 4) compelled disperse the clouds if it wants. But the economic
5) adapted sanctions are still in place. The US is only (4) trying
7. 1) illuminating 2) damaging to come to terms with the fact that the nuclear
3) confusing 4) critical weapons are not the (5) of the Permanent Members

KUNDAN
5) unreliable of the Security Council. If they do not recognize India
8. 1) compelling 2) associating as a nuclear power, then what is it that they are (6)
3) contrasting 4) describing to? India will not (7) by their de-recognising the nuclear
5) advocating tests. Both sides can happily close (8) eyes and agree
9. 1) comparison 2) analysis to (9) what has happened. The fact that India is a
3) agreement 4) elaboration sovereign nation, entitled to take decision beneficial
5) acquaintance for its own security, has not been altered by the tests.
10. 1) projected 2) sustained The US has come round to (10) that India has some
3) prolonged 4) prolific say in this matter.
5) attributed 1. 1) adversaries 2) negotiations
Perplexity = conf usion; t he st at e of being
3) strifes 4) strategies
confused or worried
Metaphysics = the branch of philosophy dealing 5) disputes
with the nature of existence, truth 2. 1) relationship 2) struggle
and knowledge 3) matter 4) talks
Utter = t o say somet hing; t o expr ess 5) countries
something in speech. 1) formed 2) eclipsed
Therapeutic = of or connected with healing; having 3) reined 4) covered
a good general effect on the body or 5) floated
the mind
4. 1) spontaneously 2) generously
Neurotic = having or showing an abnormal
anxiet y or obsession about 3) grudgingly 4) gracefully
something; caused by or suffering 5) willingly
from a mental illness that causes 5. 1) threats 2) creations
depression or abnormal behaviour 3) properties 4) monopoly
Concise = brief 5) possessions
Virtual = almost or nearly the thing described, 6. 1) prepared 2) objecting
but not completely. 3) pointing 4) clinging
Jettison = to abandon or reject something that
5) planning
is not wanted.
Cloze Test 411

7. 1) gain 2) differ 2. 1) obvious 2) necessary


3) flourish 4) suffer 3) essential 4) recognised
5) develop 5) prominent
8. 1) their 2) our 3. 1) accept 2) participate
3) naked 4) inward 3) pronounce 4) inculcate
5) both 5) relate
9. 1) imitate 2) undo 4. 1) advocates 2) possessed
3) cherish 4) reiterate 3) exponents 4) indifferent
5) ignore 5) themselves
10. 1) expecting 2) suspecting 5. 1) seriousness 2) beliefs
3) accepting 4) advocating 3) barriers 4) masks
5) rejecting 5) chains
Punctuate = to interrupt something at intervals. 6. 1) snobbery 2) egoism
Disperse = to go in different directions or make 3) brashness 4) boasting
somebody/something do this 5) candour
Adversary = an opponent in a cont est, an
7. 1) projective 2) spontaneous
argument or a battle
Strife = angry or violent disagreement;
3) pious 4) cavaliers
conflict 5) callous
Eclipse = to outshine somebody/something; 8. 1) conflict 2) persuasiveness
t o make somebody/something 3) dedication 4) propensity

K
appear dull or unimportant by 5) jealousy
comparison. 9. 1) pervasiveness 2) boundaries
Rein = to restrain or control somebody/ 3) sluggishness 4) blocking
something
5) enthusiasm
Spontaneous =done, happening, said etc because
of a sudden impulse from within,
10. 1) unanimous 2) uncritical
not planned or caused or suggested 3) uninhabited 4) uncanny
by something/somebody outside. 5) unusual
Grudgingly = reluctantly Interpersonal = existing or done between two
Grudge = t o do or giv e somet hing v er y people
unwillingly Impulsive = noted for or involving sudden action
Cling = to become attached to something; wihtout careful thought
to stick to something Prominent = distinguished or important; easily

KUNDAN
Imitate = to copy somebody/something; to seen
take or follow somebody/something Pronounce = to declare or announce something
as an example. especially formally, solemnly or
Cherish = to keep a feeling or an idea in one’s officially
mind or heart and think of it with Inculcate = to fix ideas, principles etc firmly in
pleasure. somebody’s mind especially by
Reiterate = to repeat something that has already often repeating them
been said, especially for emphasis Exponent = a per son who suppor t s and
promotes a theory, belief, cause etc
Passage 4 Snobbery = attitudes and behaviour that are
characteristic of a snob.
Trust is the basis of human relationship. As trust Snob = a person who believes he or she has
between people grows, (1) change and interpersonal superior taste or knowledge
dynamics are transformed. Diverse skills and abilities B rash = conf ident in a rude, noisy or
become (2) and appreciated as strengths. People begin aggressive way.
to (3) one another’s attitudes and feelings. They learn Candour = the quality of being frank and
honest in one’s behav iour or
to be (4) instead of playing roles. As trust grows the
speech.
(5) that prevent (6) and openness lessen. People Pious = having or showing a deep respect
become more expressive, impulsive, frank and (7). for God and religion
Their communication is efficient and clear. They risk Cavalier = showing a lack of proper concern
(8) and confrontation, opening the doors to deeper Callous = having or showing no sympathy for
communication, involve me nt and commitment. other people’s feelings or suffering
Congestion and (9) lessen. The flow of data is open Propensity = a tendency t o do somet hing
and (10). especially something undesirable.
Pervasive = present and seen or felt everywhere
1. 1) motivations 2) behaviours
Sluggish = moving slowly; not alert or lively
3) patterns 4) aspirations Unanimous = agreed with by everybody in a group
5) commitments Uncanny = not natural; mysterious and slightly
frightening.
412 Test of English Language

Passage 5 Scant = hardly enough; not very much


Primitive = of or at an early stage of social
A good percentage of the population of India is development
tribal. The tribals live in the hills and forests of the
country and have been little (1) by the (2) currents of Passage 6
the plains. Practically all the states of India have their India’s ( 1) ove r the past half ce ntury since
tribal population. The tribes are numerous, computed independence has been unique and (2) in many ways.
to be about 200, some living in (3) regions in dense Yet the record is (3) in relation to what the country
forests, and others on the borders of villages. Some set out to achieve and could certainly have been (4).
tribes are (4) to a few souls, while others like the It is (5) to look at both sides; the alternative is to be
Santhals, run into millions and are steadily (5) in (6) down by unrelieved gloom or unwarranted (7). The
numbers. During the British period some of them were fact is that after eight 5-year plans, about 40 per cent
known as ‘criminal tribes’ for they showed (6) respect of population is (8) below the poverty line. The human
for the Indian Penal Code. After independence they development indices are (9) low, placing India at the
have been named Scheduled Tribes. Under modern 126th position in the world table, far below many
conditions isolation, however, has become (7) and the countries that came into (10) much later than it did.
hill tribes are getting (8). The cultural traffic is two- 1. 1) development 2) domination
way. Social reformers are taking civilization to the 3) predicament 4) history
hills, and the tribes, (9) their old occupations of 5) excellence
hunting and (10) farming, are settling in villages, 2. 1) dubious 2) insignificant

K
towns and cities as labourers and industrial workers. 3) desperate 4) special
1. 1) affected 2) domiciled 5) commendable
3) motivated 4) deprived 3. 1) outshining 2) broken
5) favoured 3) disappointing 4) brighter
2. 1) financial 2) proud 5) played
3) cultural 4) unruly 4. 1) underplayed 2) accomplished
5) swift 3) tampered 4) noteworthy
3. 1) comfortable 2) marshy 5) exaggerated
3) wild 4) unpopulated 5. 1) proposed 2) futile
5) inhospitable 3) impracticable 4) necessary
4. 1) devoted 2) confined 5) suggested
3) susceptible 4) related

KUNDAN
6. 1) laid 2) struck
5) attached 3) cooled 4) weighed
5. 1) constant 2) deteriorated 5) brought
3) developing 4) increasing 7. 1) progress 2) debating
5) decreasing 3) meticulousness 4) haste
6. 1) abundant 2) genuine 5) complacency
3) superficial 4) exorbitant 8. 1) much 2) still
5) scant 3) obviously 4) found
7. 1) crucial 2) necessary 5) far
3) difficult 4) convenient 9. 1) deplorably 2) admirably
5) indispensable 3) surprisingly 4) not
8. 1) civilized 2) demoralised 5) amusingly
3) wiped-out 4) entertained 10. 1) world 2) being
5) reduced 3) independence 4) compete
9. 1) escaping 2) with 5) India
3) enhancing 4) leaving Unrelieved = not changing; continuing
5) continuing Gloom = part ial darkness; a f eeling of
10. 1) productive 2) primitive sadness and depression
3) profitable 4) cultivable Un warr ant ed = not justified or necessary
5) scientific Dominate = to have control or power over or very
Unruly = not easy to control or manage strong influence on somebody/
Marshy = wet and muddy something
Inhospitable = not giving a f riendly or polite Predicament = a difficult or unpleasant situation
welcome to guests especially one in which it is difficult
Susceptible = easily influenced or harmed by to know what to do.
something; sensitive Dubious = doubtful
Exorbitant = much t oo high or gr eat ; Commendable = deserving praise
unreasonable Accomplished = skilled
Tamper = to interfere with or alter something
Cloze Test 413

without authority 10. 1) credited 2) implored


Exaggerate = to make something seem larger, 3) admired 4) flattered
better, worse etc than it really is 5) blamed
Futile = producing no result; having no Tarred with the = having or considered to have
purpose same brush the same faults as somebody
Meticulous = giving or showing great care and (as somebody)
attention to detail. Decent = proper, acceptable, satisfactory
Complacency = a calm feeling of satisfaction with Nurture = t o help t he development of
oneself, one’s work something
Deplore = t o be shocked or of f ended by Abolish = to end the existence of a law, a
something; to condemn practice, an institution etc
Amusing = causing laughter or smile; enjoyable Contaminate = to make something/somebody
Weigh down = to make somebody/something bend impure by adding substances that
by being heavy. are dangerous or carry disease
Impeach = to raise doubts about something; to
Passage 7 question something
In the thirties and forties, geography was (1) subject Placate = to make somebody less angry; to
calm or satisfy somebody.
in schools. Children spent hours tracing maps and
Remedy = to correct , change or improve
(2) about strange places, peoples and customs. something undesirable
Harvard University (3) its geography department after Implore = to ask or beg for something in a
World War II. A string of leading universities in the serious way

K
United States (4) suit. Geography has been tarred with Flatter = to praise somebody too much or in
the racist brush, and no one wants to be (5). an insincere way especially in order
David S Landes, professor of history and economics to gain favour for oneself.
at Harvard University, makes a forceful (6) for
geography in his book, The Wealth and Poverty of Nations.
Passage 8
Geography, he says, tells the unpleasant truth that Fourteen centuries ago when the world was much
nature is unfair, unequal in its (7) and that its younger, the ruler of all India, Rajah Balhait, was (1)
unfairnesses are not easily (8). For Landes, there is about his people. A new game of dice, called nard, had
nothing racist in a geography that links (9) and group (2) the imagination of his subjects. Teaching them
behaviour to nature, no one can be praised or (10) for that chance alone - a roll of the dice - guided the (3)
the temperature of the air, the volume or timing of of men. All who played this game of fortune lost their
rainfall, or the topography. (4) in the virtues of courage, prudence, wisdom and

KUNDAN
1. 1) full-fledged 2) resourceful hope. It bred a fatalism that was (5) the spirit of the
3) decent 4) boring kingdom.
5) famous Rajah Balhait commissioned Sissa, an intelligent
2. 1) knowledge 2) drawings courtier, at his court, to find an answer to this (6).
3) ignored 4) learned After much (7) the clever Sissa invented another game,
5) figures chaturanga, the exact (8) of nard, in which the four
3. 1) established 2) nurtured elements of the Indian army were the key pieces. In
3) intensified 4) developed the game these pieces - chariots, horses, elephants
5) abolished and foot soldiers - joined with a royal counsellor to
4. 1) followed 2) cleared defend their king and defeat the enemy. Forceful (9)
3) prepared 4) wore was demanded of the players not luck. Chaturanga soon
5) filed became more popular than nard, and the (10) to the
5. 1) learned 2) contaminated kingdom was over.
3) neglected 4) prepared 1. 1) concerned 2) confident
5) knowledgeable 3) ignorant 4) indifferent
6. 1) decision 2) impeachment 5) partisan
3) lesson 4) plea 2. 1) propelled 2) enshrined
5) plan 3) captured 4) activated
7. 1) behaviour 2) favours 5) enhanced
3) sources 4) deal 3. 1) communities 2) ways
5) functions 3) abnormalities 4) destinies
8. 1) sensed 2) placated 5) groups
3) remedied 4) over-ruled 4. 1) bravado 2) interest
5) understood 3) peace 4) wealth
9. 1) expediency 2) sentiments 5) faith
3) performance 4) acquisition
5) obedience
414 Test of English Language

5. 1) appalling 2) crushing particular, needs to be complimented for this–have


3) moistening 4) promoting le d to substantially le sse r e ne rgy inte nsity of
5) overwhelming economic growth. However, even the tempered demand
6. 1) apprehension 2) risk numbers are (8) to be below 80Gw. As against this
3) problem 4) game need, the coal supply from domestic sources is unlikely
5) destiny to support more than 25 Gw equivalent capacity.
7. 1) deliberation 2) absorption Imported coal can add some more, but at a much (9)
3) insight 4) hesitation cost. Gas-based electricity generation is unlikely to
5) reluctance contribute anything substantial in vie w of the
8. 1) nature 2) equivalent unprecedented gas supply challenges. Nuclear will be
3) picture 4) opposite (10) in the foreseeable future. Among imported coal,
5) replica gas, large hydro and nuclear, no more than 15-20Gw
9. 1) prediction 2) concentration equivalent can be (11) to be added in the five-year
3) manipulation 4) attack time block.
5) fortune (12) (13) this, capacity addition in the renewable
10. 1) devastation 2) anxiety energy based power generation has touched about 3Gw
3) impeachment 4) nuisance a year. In the coming five years, the overall capacity
5) threat addition in the electricity grid (14) renewable energy
Virtue = behaviour that shows high moral is like ly to range be twe e n 20Gw and 25Gw.
standards; goodness Additionally, over and above the grid-based capacity,

K
Prudent = acting with or showing care and off-grid electricity applications are reaching remote
thought for the future; showing
places and (15) lives where grid-based electricity
good judgement
Fat al = causing or ending in death
supply has miserably failed.
Counsellor = an adviser especially one who has 1. 1) against 2) for
professional training 3) onwards 4) at
Part isan = showing too much support for one 5) on
person, group or cause; biased 2. 1) that 2) inside
Propel = to move, drive or push something/ 3) always 4) who
somebody forward 5) where
Enshrine = to preserve something in a place or
3. 1) forward 2) subject
from it will be remembered and
respected.
3) place 4) demand

KUNDAN
Br av ad o = a display of bold talk or behaviour 5) replace
to impress other people. 4. 1) pass 2) publish
Appalling = shocking; extremely bad 3) feature 4) find
Moisten = to become or make something moist 5) light
Overwhelming = very great; very strong 5. 1) likewise 2) publicity
Replica = a close or exact copy of something 3) next 4) after
of a painting; a model of something
5) earlier
made on a smaller scale
Anxiety = a nervous feeling caused by fear that
6. 1) waste 2) require
something bad is going to happen; 3) highlight 4) generate
worry 5) consumed
Nuisance = a thing, a person or behaviour that 7. 1) structures 2) efforts
is annoying or causes trouble 3) projections 4) practices
5) developmental
Passage 9 8. 1) sure 2) unsure
As the country embarks on planning (1) the 12th 3) unexpected 4) unlikely
Plan (2012-17) period, a key question mark (2) hangs 5) likely
over the process is on the energy requirements. 9. 1) nominal 2) excelled
Growth is energy-hungry and the aspirations of 3) higher 4) lower
growing at 9-10% will (3) huge demands on the energy 5) expected
resource s of the country. In this energy jigsaw, 10. 1) failure 2) success
renewable energy will (4) like never before in the 12th 3) dangerous 4) maximum
Plan and the (5). 5) marginal
By the rule of the thumb, India will (6) about 100 11. 1) certain 2) linked
gigawatts (Gw)-100,000 megawatts of capacity addition 3) remarked 4) expected
in the next five years. Encouraging trends on energy 5) sure
efficiency and sustained (7) by some parts of the 12. 1) When 2) But
government—the Bureau of Energy Efficiency, in 3) However 4) If
5) As
Cloze Test 415

13. 1) for 2) with 6. 1) companion 2) attitude


3) is 4) ever 3) calling 4) friend
5) against 5) abode
14. 1) through 2) project 7. 1) absorbed 2) alarmed
3) versus 4) against 3) attacked 4) attached
5) capacity 5) awed
15. 1) lightening 2) making 8. 1) empowered 2) brute
3) touching 4) saving 3) tall 4) high
5) generating 5) exhibited
Embark on = to start or engage in something 9. 1) domestic 2) durable
new or difficult 3) devastating 4) delicate
Compliment = to express praise or admiration 5) dubious
of somebody
10. 1) hoist 2) puncture
Temper = t o make t he ef fect s of
somet hing less sever e by
3) disturb 4) attack
balancing it with the else 5) deflate
Unprecedented = never having happened, been 11. 1) protect 2) tender
done or been known before 3) abandon 4) pluck
Over and above = besides something; in addition 5) touch
to something Stubborn = determined not to change one’s
Marginal = having little importance; not attitude or position; having a strong
central

K
will
Ji gsaw = a picture printed on cardboard Passion = a strong feeling eg of hate, love or
or wood cut int o var ious anger
different shapes that have to Awe = to fill somebody with a feeling of
be fitted together again; jigsaw respect combined with fear or
puzzle. wonder
Brute = involving physical force only and not
Passage 10 thought or reason
Outrun = t o r un f ast er or f ur ther t han
Bret Bonson loved animals (1) on a family owned somebody/something
Zoo. He had grown up caring for antelope, deer and Instinct = a natural feeling that makes one act
wildcats. He was (2), at times stubbornly, protective. or respond in a particular way.
Once, when a tiger cub was born with a deformed leg, Hoist = to raise something to a higher
position

KUNDAN
the local veterinarian and Bret’s parents (3) the animal
would never live a full life. Even so, the boy bottle-fed Deflate = t o make somebody f eel less
the cub and cared for it. (4) Bret’s mothering, the cub confident than they were or less
important than they thought they
died, but Bret’s mothering (5) lived on.
were.
He worked at a Safari park where, in 1980, he Tender = loving; gentle; easily moved to pity
trained his first African elephant and found his true or sympathy
(6). From the beginning Bonson was (7) by elephants. Pluck = to hold something with the fingers
They have the (8) force to uproot trees and can outrun and pull it.
the fastest human sprinter. But they also have (9)
fine motor skills. The same trunk that could (10) the Passage 11
front end of an automobile or fracture a predator’s Gandhiji once said, “I would say that if the village
skull could gently (11) a peanut from the fingers of a perishes, India will perish too. India will be (1) more
small child. India. Her own mission in the world will get (2). The
1. 1) created 2) constructed (3) of the village is possible only when it is no more
3) built 4) erected (4). Industrialisation on a mass scale will (5) lead to
5) raised passive or active exploitation of the villagers as the
2. 1) methodically 2) carefully problem (6) competition and marketing come in.
3) fiercely 4) suitably Therefore, we have to (7) on the village being self-
5) actually contained, manufacturing mainly for use. Provided this
3. 1) believed 2) valued character of the village industry is (8) there would be
3) expressed 4) imagined no objection to villagers using even the modern
5) exhibited machines and tools that they can make and (9) to
4. 1) Until 2) Unless use. Only, they (10) not be used as a means of
3) Instead 4) Despite exploitation of others.”
5) Although 1. 1) certainly 2) scarcely
5. 1) belief 2) instinct 3) much 4) no
3) love 4) passion 5) any
5) care
416 Test of English Language

2. 1) lost 2) extension new teaching posts in colleges. (10) with this problem,
3) elevated 4) flourished authorities at the university have decided that serving
5) jeopardy teachers belonging to various disciplines will teach
3. 1) rehabilitation 2) pruning the paper.
3) revival 4) devastation 1. 1) Despite 2) Having
5) atonement 3) Enacting 4) Adopting
4. 1) denuded 2) exploited 5) Although
3) contaminated 4) populated 2. 1) contaminate 2) clean
5) ruined 3) filter 4) protect
5. 1) passionately 2) surprisingly 5) pollute
3) scarcely 4) never 3. 1) resulting 2) why
5) necessarily 3) obvious 4) as
6. 1) forming 2) enhancing 5) because
3) between 4) of 4. 1) seldom 2) don’t
5) with 3) hardly 4) perfectly
7. 1) concentrate 2) ponder 5) actually
3) imagine 4) ensure 5. 1) inability 2) deferral
5) decide 3) decision 4) failure
8. 1) regained 2) neglected 5) reluctance
3) maintained 4) thwarted 6. 1) extracts 2) accord

K
5) abolished 3) expects 4) loses
9. 1) prepare 2) afford 5) assumes
3) hesitate 4) propose 7. 1) displeasure 2) antagonism
5) plan 3) hurdles 4) confusion
10. 1) can 2) could 5) priority
3) need 4) would 8. 1) losses 2) constraints
5) should 3) apathy 4) soundness
Perish = to be destroyed; to die 5) independence
Revival = a recovery; the process of bringing 9. 1) receive 2) establish
something back 3) emphasize 4) expect
Jeopardy = at risk
5) sanction
Rehabilitate = to restore somebody/something to

KUNDAN
their/its former higher status or
10. 1) Down 2) Familiarity
position 3) Faced 4) Convinced
Prune = to reduce the extent of something 5) Solution
by cutting unnecessary parts. Stringent = that must be obeyed; strict or severe
Atone = to act in a way that compensates for Assume = to accept something as true before
a previous wrong or error. there is proof
Denude = to make something bare Deferral = delaying something until a later time
Ponder = to think about something carefully Antagonism = a feeling of hostility or opposition
and for a long time especially in Constraint = a thing that limits or restricts
trying to reach a decision; to consider Apathy = a lack of interest, enthusiasm or
Thwart = to prevent somebody doing what concern
they intended to; to oppose a plan
etc successfully. Passage 13
Passionate = caused by or showing st rong
feelings A friend in need is a friend indeed. A man who stands
(1) his friend in (2) is a true friend, Selfless love is
Passage 12 the base of true friendship. True friends share each
other’s joy and sorrow, pain and pleasure. They do
(1) stringent anti-pollution laws, mass awareness not fall (3) in adversity. They have full confidence in
levels in India about the need to (2) the environment each other. They never (4) each other. (5) makes
are low. Which is (3) many people insist that mere friends, adversity tries them. A selfless friend is (6);
laws won’t do; what we (4) need are “environment a selfish friend is a curse. The first is an angel and
conscious” citizens. It is in this context that the the second is a devil. One makes your career while
University’s (5) to introduce environment studies as the other (7) it.
a compulsory paper at the undergraduate level (6) True friendship means great self-sacrifice on the
significance. There was some (7) initially about who part of both. A true frie nd (8) ple asure and
would teach the paper because financial (8) make it convenience . He goes cut of his way and faces
impossible for colleges to (9) approval for new teaching difficulties in his way with joy and even with pride.
posts. In fact, in August 1999, the University Grants Joy and sorrow, success and failure, good fortune and
Commission (UGC) imposed a ban on the creation of
Cloze Test 417

misfortune, are equally (9) by a pair of true friends. Passage 14


They (10) the burden of life equally for they feel that
they sail in the same boat and that they have to sink The Government seems to be in right earnest to
and swim together. ensure more (1) in governance. The Prime Minister’s
1. 1) to 2) with announcement that his Government is (2) drafting
3) for 4) by legilsation to (3) the citizen’s right to information is
5) of indeed welcome. Though the talk on the right to
2. 1) adversity 2) commotion information is not new, we may (4) the bill to be
3) change 4) happiness brought early this time. The previous Government had
5) growth set up a high-level committee to (5) a draft bill. But
3. 1) by 2) to nothing has been heard about the matter since, (6)
3) off 4) with the committee did quite some work. The issue,
5) through however, has come to such a pass that a solution
4. 1) postulate 2) commit cannot be (7) further. Sunlight is the best disinfectant,
3) danger 4) deplete a foreign judge once said, while (8) the unwarranted
5) betray secrecy in an administrative system. When those in
5. 1) Wealth 2) Prosperity authority know that people have the right to ask
3) Man 4) Providence questions and the government is under the (9) to
5) Well-bring provide them with answers, (10) of authority, or of
6. 1) boon 2) force public finances, for personal or party ends is less likely

K
3) blessing 4) calamity to happen.
5) message 1. 1) strictness 2) rudeness
7. 1) throws 2) develops 3) leniency 4) economy
3) constructs 4) mars 5) transparency
5) lacks 2. 1) personally 2) busy
8. 1) foregoes 2) mitigates 3) not 4) reluctantly
3) evolves 4) appraises 5) absolutely
5) prospers 3. 1) presumption 2) absolve
9. 1) built 2) pleased 3) curb 4) question
3) admired 4) advocated 5) establish
5) shared 4. 1) expect 2) wait
10. 1) expect 2) shoulder 3) try 4) frustrate

KUNDAN
3) dislike 4) propose 5) appeal
5) project 5. 1) level 2) regard
Stand by = to support or help somebody 3) prepare 4) enact
Adversity = difficulties; trouble; misfortune 5) unearth
Commotion = noisy confusion or excitement 6. 1) even 2) as
Fall off = to decrease in quantity or quality 3) because 4) until
Postulate = to accept or suggest that something 5) though
is true, especially as a basis for 7. 1) found 2) expected
reasoning or discussion
3) delayed 4) looked
Deplete = to reduce greatly the quantity size,
power or value of something. 5) longed
Betray = t o show a lack of loyalt y t o 8. 1) nurturing 2) criticising
somebody/something 3) demanding 4) appreciating
Providence = the way in which God or nature 5) upholding
cares for and protects all creatures 9. 1) pretentious 2) affect
Blessing = God’s favour and protection 3) substance 4) obligation
Curse = a magical word or phrase spoken 5) property
with the aim of punishing, injuring
10. 1) misuse 2) governance
or destroying somebody/something
Ma r = to damage or spoil something 3) dishonour 4) curbing
Forego = to give up or do without something 5) breach
especially something pleasant In earnest = to a greater extent; with more
Mitigate = to make something less severe, determination and energy
violent or painful Disinfectant = a substance that cleans something
Appraise = to assess the value quality or nature by destroying the bacteria that cause
of somebody/something disease
Lenient = not severe, especially in punishing
people.
Presumption = the action of supposing something
to be true.
418 Test of English Language

Absolve = to declare that somebody is free of 3) maintaining 4) doubling


guilt, blame etc. 5) minimising
Curb = to prevent something from getting 11. 1) damage 2) variable
out of control
3) content 4) yield
Long = to wait something very much; to have
a strong desire for something or to
5) refuge
do something 12. 1) squandered 2) preserved
Uph old = to support or confirm a decision, 3) doubled
belief et c which has been 4) engulfed
questioned 5) coerced
Pretentious = claiming importance, value or style, 13. 1) equilibrium 2) existence
especially without good cause. 3) failure 4) proportion
5) bankruptcy
Passage 15 Conquest = t he act ion or an inst ance of
Man in his (1) of nature and universe has made conquering somebody
the world (2), polluted. The air we breathe is polluted, Foul = very unpleasant; very bad; terrible
Wanton = done deliberately for no good reason
the water we drink is (3). There is (4) felling of trees,
Reclamation = the action of making land fit to
clearing of jungles, (5) natural barriers like the cult ivate, eg by draining it or
mountains and drying up the oceans by way of (6). bringing water to it
This (7) of nature by man is a grave mistake for which Grave = serious and important; giving cause
mankind has to pay the price. Rapid industrialisation for worry

K
means (8) the industrial effluents into the rivers and Vanda lism = behaviour character of a person who
seas. The river water has turned murky. Marine life deliberately destroys or damages
has been (9). The toxic chemicals have made the air works of art, public and private
property, the beauties of nature etc
that we breathe polluted. Pesticides and insecticides
for no good reason.
sprayed on plants and the chemicals and fertilizers Effluent = liquid waste matter, sewage etc that
used for (10) plant yield have poisoned our food. Hence pours out of a factory into a river
what we eat today has high toxic (11). Nature’s Murky = dirty; not clear
plentifulness is a heritage not to be (12) with impunity. Endangered = in danger of becoming extinct
It must be conserved for future generations or its (13) Extinct = no longer in existence
will extinguish all. Squander = to waste something foolishly or
1. 1) pursuit 2) view 3) conquest carelessly
Impunity = freedom from punishment or injury

KUNDAN
4) victim 5) want
Extinguish = to end the existence of a feeling,
2. 1) foul 2) diluted condition etc
3) poor 4) precarious Pursuit = the action of looking for or trying to
5) critical find something
3. 1) disturbed 2) pure Precarious = not safe; dangerous
3) counterproductive 4) suffocated Provocation = the action of making somebody
5) contaminated angr y by deliberat ely doing
4. 1) dubious 2) wanton something annoying or offensive.
Evasion = the act or process of avoiding
3) careful
something that is legally or morally
4) planned required
5) useless Divulge = to make something known especi-
5. 1) attacking 2) projecting ally a secret
3) cutting 4) blasting Menace = a thing or person that threatens to
5) sizing harm somebody/something
6. 1) reclamation 2) inhabitation Culminate = t o reach t he highest point or
3) stabilisation 4) destruction specified conclusion or result.
Forfeit = to giv e up somet hing or have
5) damage
somet hing t aken away as a
7. 1) provocation 2) adventure consequence of or punishment for
3) vandalism 4) abundance having done something wrong.
5) evasion Refuge = shelter or protection from danger,
8. 1) relocating 2) divulging trouble etc
3) menacing 4) culminating Engulf = to surround somebody/something
5) diverting especially so t hat they ar e
9. 1) evaporated 2) endangered completely covered
Coerce = to make somebody do something by
3) devalued 4) eliminated
using force or threats
5) forfeiting Bankruptcy = the state of being unable to pay
10. 1) managing 2) developing one’s debts
Cloze Test 419

Passage 16 13. 1) light 2) day


3) authority 4) person
Once Gurudev Tagore asked Gandhiji: “Gandhiji,are 5) sun
you (1) unromantic? When in the early (2) the morning 14. 1) scorch 2) shine
sun rises does it not (3) your heart with joy to see its 3) bright 4) burn
reddish glow? When the birds (4) does not your heart 5) illuminate
thrill with its (5) music? When the rose opens its 15. 1) brightness 2) shade
petals and blooms in the garden, does its sight not 3) dullness 4) strength
bring (6) to your he art?” The Mahatma re plie d, 5) stairs
“Gurudev, I am not so dumb or (7) as not to be moved Tint = a shade or variety of a colour
by the beauty of the rose or the morning rays of the Sigh = an act or sound of taking long deep
sun or the music of the birds. But what can I do? My breat h t hat can be hear d,
one (8), my one anxiety, my one ambition is: When expr essing sadness, r elief ,
shall I see the red tint of the rose on the cheeks of tiredness etc
Lustre = the soft brightness of a smooth or
(9) (10) millions of my people? When shall I hear the
shining surface; glory; distinction
sweet and melodious song of the birds in place of Da wn = the time of day when light first
their (11) sighs - when will such music (12) out of appears
their soul? And when will that (13) come, when the Flock = a group of sheep, goats or birds of
light of the morning sun will (14) the heart of the the same type either kept together
common man in India? When will I see its lustre and or feeding and travelling together
Divine = wonderful; beautiful

K
(15) on his face?”
1. 1) not 2) genuinely Ar om a = a distinctive usually pleasant smell
Lethargic = lazy
3) seldom 4) so
Scorch = to burn and damage a surface by
5) fairly making it too hot.
2. 1) season 2) dawn Illuminate = to shine light on something
3) monsoon 4) climate
5) days Passage 17
3. 1) involve 2) impeach
Man has always considered himself to be the ruler
3) move 4) fill
of his planet. This (1) and the attendant superiority
5) penetrate
feeling has made him look down (2) other creatures
4. 1) fly 2) nestle
who co-exist with human on this earth. The so-called
3) flock 4) cry

KUNDAN
civilized human race has (3) and ill-treated small and
5) sing
large animal species and birds in an attempt to prove
5. 1) alarming 2) fearful
his (4). It is common knowledge that (5) number of
3) divine 4) irritating
animals have been (6) for centuries under the (7) of
5) loud
conducting scientific experiments or for sports. Till
6. 1) aroma 2) cheer
recently, in the (8) of scientific experiments, monkeys
3) fragrance 4) agony
and frogs have been (9) to dissection and (10) in the
5) fear
laboratory.
7. 1) insensitive 2) lethargic
1. 1) pleasure 2) fact
3) ambitious 4) idle
3) achievement 4) force
5) romantic
5) arrogance
8. 1) slogan 2) request
2. 1) in 2) upon
3) interpretation 4) desire
3) with 4) for
5) demand
5) into
9. 1) old 2) rich
3. 1) criticised 2) devalued
3) happy 4) noble
3) protected 4) abused
5) hungry
5) enlarged
10. 1) naked 2) fashioned
4. 1) supremacy 2) wisdom
3) poor 4) fellow
3) cleverness 4) instinct
5) playful
5) possession
11. 1) encouraging 2) flourishing
5. 1) tall 2) plenty
3) prosperous 4) agonizing
3) countless 4) diverse
5) cheerful
5) numerous
12. 1) play 2) bring
6. 1) tortured 2) exposed
3) come 4) drop
3) treated 4) vanished
5) sing
5) extinct
420 Test of English Language

7. 1) projection 2) criticism 5. 1) maintained 2) illustrated


3) pretext 4) game 3) marginalised 4) bestowed
5) study 5) forsaken
8. 1) matter 2) set 6. 1) vast 2) brief
3) scheme 4) virtue 3) formal 4) clean
5) name 5) distinct
9. 1) confined 2) subjected 7. 1) dormant 2) dedicated
3) condemned 4) allied 3) vital 4) common
5) performed 5) dynamic
10. 1) cruelty 2) deformation 8. 1) strength 2) tips
3) study 4) vivisection 3) clearance 4) sermons
5 ) proliferation 5) ideals
Look down upon = to consider somebody/ 9. 1) informed 2) narrated
something inferior to oneself; to 3) intensified 4) realised
regard somebody/something with 5) invented
contempt
10. 1) encouraging 2) imitating
Under the pretext of = giving the specified reason
as one’s justification.
3) blaming 4) preaching
Dissection = the practice of cutting up dead body, 5) assuming
a plant etc in order to study Deteriorate = to become worse in quality or condition
Vivisection = t he pr act ice of perf or ming Bestow = t o pr esent somet hing as a gif t t o

K
operations etc on live animals for somebody
the purposes of scientific research. Vital = essential to the existence, success or
Vanish = t o disappear completely and operation of something.
suddenly Sermons = a talk on a moral or religious subject
Extinct = no longer in existence usually given by a priest during a religious
Condemn = to say that one disapproves strongly service.
of somebody/something; to criticize
somebody/something Passage 19
Proliferation = a rapid growth or increase in
numbers The social (1) of the Web lifestyle and work style
are enormous. A lot of people (2) that computers and
Passage 18 the Internet will depersonalize experience, creating a
world that is less warm. But these are unfounded as

KUNDAN
In the se days of e conomic libe ralisation, we know that some people were (3) afraid that the
globalisation, etc. materialistic values have assumed telephone would reduce face-to-face contact and will
(1) importance. Money, physical comforts and luxuries (4) society to fall apart. But the (5) actually came true.
are the most sought after aspects. There has been (2) Just as phone and e-mail have increased contact
competition. Such competition (3) undue stress. The between people living in different communities and
stress leads to (4) of health of the people. Indian between people on the go, the PC and the Internet
culture has (5) its striking uniqueness, as against give us (6) way to communicate. They do not take any
the Western culture, in the fact that there is a (6) away. In reality, the ability to use the Internet to
place for spiritualism in the value system in all walks redefine (7) in our communities is strengthening
of life. The spirituality is a very (7) force which helps personal and cultural (8). The Web lifestyle is about
us in maintaining our physical and mental health. It broadening (9), not narrowing them. Community
gives us (8) to cope with the stress. Westerners have building is going to be one of the biggest growth areas
now (9) the importance of spirituality and, therefore, on the Web. It dramatically increases the number of
they have started (10) us in the matter of spirituality. communities you can bond to because of its ability to
1. 1) usual 2) little (10) groups of like-minded people independent of
3) tangible 4) least geography or time zones.
5) greater 1. 1) groups 2) needs
2. 1) critical 2) unhealthy 3) factor 4) teaching
3) unequalled 4) no 5) implications
5) absolute 2. 1) accept 2) dare
3. 1) releases 2) deserves 3) fear 4) propose
3) generates 4) demonstrates 5) reject
5) suppresses 3. 1) strongly 2) initially
4. 1) neglect 2) illness 3) always 4) never
3) generation 4) deterioration 5) possibly
5) encroachment 4. 1) let 2) decay
3) develop 4) cause
5) destroy
Cloze Test 421

5. 1) opposite 2) found 8. 1) distinctive 2) appreciated


3) finding 4) different 3) formative 4) helping
5) negative 5) end
6. 1) cheaper 2) economical 9. 1) evolving 2) spreading
3) another 4) second 3) esteem 4) wisdom
5) many 5) popularity
7. 1) groups 2) ethics 10. 1) desirable 2) manageable
3) culture 4) bonds 3) redundant 4) vulnerable
5) boundaries 5) possible
8. 1) distances 2) connections Tone up = to make one’s body stronger, fitter
3) differences 4) implications etc
5) suggestions Cha os = Complete disorder or confusion
9. 1) horizons 2) values Calamity = an event that causes great harm or
damage; a disaster
3) nations 4) means
Perpetual = without interruption; continuous
5) status Induce = to persuade or influence somebody
10. 1) reduce 2) focus to do something
3) prepare 4) connect Implicate = to show that somebody is involved
5) develop in something, especially in crime.
Enormous = very large; huge; immense Inculcate = to fix ideas, principles etc firmly in
Implication = the conclusion that can be drawn from somebody’s mind especially by

K
something, although it is not explicitly often repeating them
stated. Cherish = to keep a feeling or an idea in one’s
Ethics = moral principles that govern or influence mind or heart and think of it with
a person’s behaviour. pleasure
Vulnerable = that can be hurt, harmed or attacked
Passage 20 easily especially because of being
small or weak.
The urgent need of the hour is to (1) up the moral Redundant = no longer needed; unnecessary
(2) of our society in general and of our student
community in particular, if we want to save ourselves Passage 21
and our socie ty from the pre se nt (3) of mass
Studies (1) the impact of computer models to
indiscipline and (4) of basic human values, which has
support policy-making processes in organisations have
become a (5) phenomenon. We must, therefore, (6)
(2) that client involvement in the model-building

KUNDAN
and practise the most (7) basic human values like co-
process is often a (3) for effective model-building. One
operation, tolerance, patriotism, generosity, truth,
important reason is that the process of model-building
justice and excellence—the ideals which are universal
is frequently more important than the resulting model.
in nature and which are (8) in themselves and which
Model-building itself is largely a (4) process about the
are worthy of (9) for their own sake. These ideals are
problem. Most (5) about the characteristics of an ill-
both personally as well as socially (10).
structured problem are gained during the (6) process
1. 1) give 2) stand
of designing a computer model, rather than after the
3) jack 4) climb
model is finished. Another important reason is that
5) tone
most information in an organisation (7) in the mental
2. 1) fibre 2) enactment
models of organisation members. To support policy-
3) reconstruction 4) situation
making in organisation it is this knowledge which
5) appreciation
needs to be (8) and represented in the model. An
3. 1) polarisation 2) degradation
important topic in client-oriented or (9) model building
3) chaos 4) provocation
thus be come s the (10) of re le vant knowle dge
5) sentiments
contained in the mental models of participants.
4. 1) calamity 2) focus
1. 1) evaluating 2) focussing
3) realisation 4) erosion
3) projecting 4) advocating
5) criticism
5) directing
5. 1) durable 2) universal
2. 1) devised 2) exhibited
3) perpetual 4) segmental
3) convinced 4) attributed
5) prolific
5) indicated
6. 1) incorporate 2) induce
3. 1) support 2) valuation
3) implicate 4) inculcate
3) prerequisite 4) material
5) involve
5) blueprint
7. 1) absorbing 2) cherished
4. 1) valuable 2) durable
3) introspective 4) famous
3) tedious 4) learning
5) productive
5) critical
422 Test of English Language

5. 1) thinking 2) insights when East Asia was experiencing (8) difficulties.


3) planning 4) appreciation However, the one unambiguous Achilles’ heel of
5) opinion the reforms has been the (9) state of government
6. 1) elongated 2) concentrated finances. One of the two crises that India faced in
3) iterative 4) evolving 1990-91 was the unsustainable imbalance between
5) consummate government revenues and (10).
7. 1) resides 2) follows 1. 1) pulsating 2) shocked
3) settles 4) lies 3) commendable 4) promotable
5) committed 5) dipped
8. 1) extended 2) bisected 2. 1) production 2) consumption
3) subjected 4) captured 3) index 4) growth
5) attributed 5) progress
9. 1) revolving 2) interactive 3. 1) moderate 2) lukewarm
3) dogmatic 4) accentuated 3) shaky 4) considerate
5) formative 5) obstinate
10. 1) demarcation 2) formation 4. 1) ledger 2) balance
3) proliferation 4) association 3) equilibrium 4) intention
5) elicitation 5) idea
Attributed = to regard something as belonging 5. 1) demonstrated 2) exercising
t o, caused by or pr oduced by 3) rejecting 4) display

K
somebody/something 5) exhibiting
Prerequisite = a thing required as a condition for
6. 1) substantial 2) exemplary
something to happen or exist
Blueprint = a detailed plan or scheme
3) indicative 4) conservative
Elongate = to make something longer 5) destructive
Iterative = relating to or involving the repetition 7. 1) rationalisation 2) handling
of a process or utterance especially 3) management 4) proportions
of mathematical or computational 5) ration
process. 8. 1) crisis 2) overcoming
Consummate = highly skilled; perfect 3) severe 4) enjoyable
Dogmatic = insisting that one’s beliefs are right
5) wailing
and that others should accept them,
without paying attention to evidence
9. 1) critical 2) vulnerable

KUNDAN
or to other opinions 3) prone 4) attackable
Accentuated = to make something very noticeable 5) easygoing
or pr ominent ; to emphasize 10. 1) surplus 2) measurement
something. 3) thinking 4) incomes
Formative = having an important and lasting 5) expenditure
influence on the development of A far cry from
somebody’s character something = at or to a great distance
Elicit = to draw facts, a response etc from By and large = in general; generally speaking
somebody, somet imes wit h Ample = enough or more than enough
difficulty. Weather = to come safely through a difficult
period etc; to survive something.
Passage 22 Unambiguous = clear in meaning; that cannot be
interpreted in more than one way
In the decade since reforms were introduced, India
Achilles’ heel = a week point or small fault
has achieved substantial success in the sphere of especially in somebody’s
macroeconomics. Overall growth rate has been (1) character, which cannot be used
except for the last couple of years. It bears pointing or attacked by other people to
out that we have now come to view a 6 per cent (2) their advantage
rate as a slowdown! This is a far cry from pre-reforms Pulsate = to expand and contract with
rate of growth of 3 per cent. The price level has by and strong regular movements
large remained (3) both as measured by the WPI and Commendable = deserving praise
Lu kewa rm = only slightly warm
CPI. India’s (4) of payments position has bee n
Considerate = thoughtful; careful not to hurt or
comfortable. Exports, while (5) some sluggishness this trouble others
fiscal, have been growing. Imports, in spite of (6) Obstinate = difficult to overcome, remove etc
liberalisation, have not gone out of hand. This is amply Exemplary = serving as a good example,
reflected in the comfortable current account deficits suitable to be copied.
(CAD); the CAD-to-GDP ratio has remained way below W ai l = t o cr y or complain about
the crisis (7) that it had achieved in 1991. The rupee something in a loud, usually high
has weathered external turbulence rather well even pitched voice.
Cloze Test 423

Prone = likely to suf fer f rom, do or Passage 24


exper ience somet hing
unfortunate Actually everyday we are engaged in this business
of ‘reading’ people. We do it (1). We want to figure
Passage 23 others out. So we (2) make guesses about what others
The weaker sections of the rural population are think, value, want and feel and we do so based on our
mostly from the socially and economically backward (3) beliefs and understandings about human nature.
and (1) sections of the village community. Because of We do so because if we can figure out (4) and
their (2) and financial difficulty, they are not readily intentions of others the possibility of them (5) or
(3) to change their work habits and adopt modern hurting us (6) and this will help us to (7) a lot of
technology. (4) sure about the traditional methods, unnecessary pain and trouble. We also make second-
they are (5) to take to (6) equipment and techniques guesses about what they will do in future, how they
which require some time to get accustomed for (7) will (8) if we make this or that response. We do all
work. this second-guessing based upon our (9) of what we
After holding a number of group meetings with rural believe about the person’s inner nature (10) his or
people (8) to different vocations and spread over the her roles and manners. We mind-read their (11)
entire country, we can safely say that persons in the motives.
villages are not (9) for training to improve upon their Also, everyday we misguess and misread. Why?
traditional and hereditary (10) of working. Because of the complexity, (12), and multidimensional
1. 1) depressed 2) different functioning of people. After all, how well do you ‘read’

K
3) rich 4) privileged your own thoughts, aims, values, motives, beliefs, etc?
5) forward How well do you know your own structuring process
2. 1) ability 2) dependence — your own thinking and (13) styles?
3) illiteracy 4) number 1. 1) vehemently 2) practically
5) majority 3) actually 4) incessantly
3. 1) discarding 2) feeling 5) virtually
3) bending 4) undertaking 2. 1) ably 2) constantly
5) willing 3) partly 4) largely
4. 1) Making 2) Having 5) positively
3) Quite 4) Being 3. 1) futuristic 2) proactive
5) Not 3) reactive 4) decorative
5. 1) forced 2) reluctant 5) assumptive

KUNDAN
3) bound 4) prepared 4. 1) manifestations 2) expressions
5) curious 3) motives 4) hopes
6. 1) farming 2) traditional 5) prospects
3) improved 4) powerful 5. 1) tricking 2) blaming
5) old 3) furthering 4) alarming
7. 1) routine 2) monotonous 5) criticizing
3) excessive 4) wasteful 6. 1) lessens 2) happens
5) effective 3) questions 4) deepens
8. 1) accruing 2) helping 5) laments
3) enabling 4) belonging 7. 1) approach 2) direct
5) referring 3) avoid 4) implement
9. 1) eager 2) capable 5) prepare
3) indifferent 4) antagonistic 8. 1) solve 2) apply
5) unwilling 3) plan 4) approach
10. 1) theories 2) techniques 5) respond
3) desires 4) hours 9. 1) projection 2) exhibition
5) policies 3) situation 4) prediction
Vocation = a person’s job or profession 5) attribution
Privileged = having a special right or advantage 10. 1) organizing 2) underneath
available only to a particular person 3) appreciating 4) proposing
or group of people 5) outside
Reluctant = unwilling and therefore slow to act, 11. 1) cunning 2) visible
agree etc
3) deeper 4) obvious
Accrue = to allow something to collect over a
period of time; to accumulate
5) proposed
Eag er = full of interest or desire; keen 12. 1) abnormality 2) angularity
Antagonistic = showing or feeling opposition; 3) focus 4) layeredness
hostile; aggressive 5) contribution
424 Test of English Language

13. 1) proposing 2) developing 9. 1) ignorant 2) alert


3) upbringing 4) lamenting 3) prepared 4) vigilant
5) emotive 5) aware
Vehement = showing or caused by strong feeling; 10. 1) administrative 2) financial
passionate 3) capacity 4) business
Incessant = not stopping; continual 5) hierarchical
Virtually = almost
Slackness = laziness
Proactive = creating or controlling a situation by
Anticipate = to expect something
causing things to happen rather
Assimilate = to absorb ideas, information etc in
than reacting to events
the mind
Manifestation = an event, an action, an object or a
statement that shows something
clearly, eg illustrating or resulting
Passage 26
from an abstract idea The first proposal I submitted for my dissertation
Lessen = to become or make something less at UCLA was to write a theory of personality. My
Laments = to feel or express great sorrow or
chairman, a kindly man, smiled (1) and told me that
regret for somebody/something
Underneath = beneath somet hing; below perhaps this was a bit ambitious for a young graduate
something student.
Cunning = clever at deceiving people (2), I accepted his verdict and changed my topic,
Angular = thin and having prominent bones; but not my desire. It (3) later, when I had a chance to
stiff and awkward begin to (4) a theory in my research on group dynamics

K
Emotive = arousing or able to arouse intense for the Navy during the Korean War.
f eeling; t ending t o af fect t he As I (5) on the reasons for the persistence of my
emotions
inte rest in the ove rarching theory, I had an (6)
Passage 25 memory. When I was around eight years old, I was a
(7) baseball fan, as was my father. My hero was Lou
The study of accountancy is (1) in demand in the Gohrig. I would approach my father in an attempt to
view of (2) of greater complexity in our business prove to him how good Gohrig really was : “He hit 363,
organisation. Formerly a (3) of day-to-day income and had 49 home runs, batted in 165 runs. He’s terrific!”
e xpe nditure was more than (4) . A busine ss My father’s response caught me off guard : “Yes, but
organisation today has to (5) a clear account of the he can’t field.” I wasn’t prepared for that. From then
(6) it uses, the amounts that are owing to it, the on, my way of (8) with my father’s responses was to
amount that it owes to others, the profit or loss it

KUNDAN
make sure I knew everything about any topic I wanted
has made and the (7) it employs. Without a scientific to talk to him about. Partly as a (9), I became a holist.
(8) of accounting no businessman can be fully (9) of I had to make sure I had (10) for everything.
his real (10) position and run his organisation. 1. 1) usually 2) profusely
1. 1) progressing 2) getting 3) benignly 4) abruptly
3) powering 4) moving 5) decidedly
5) growing 2. 1) Indolently 2) Skilfully
2. 1) demand 2) growth 3) Enchanted 4) Constrained
3) status 4) position 5) Chagrined
5) slackness 3. 1) lamented 2) resurfaced
3. 1) mixture 2) map 3) appreciated 4) provided
3) measure 4) record 5) projected
5) transaction 4. 1) inject 2) involve
4. 1) sufficient 2) anticipated 3) exhibit 4) formulate
3) expected 4) required 5) establish
5) necessary 5. 1) pondered 2) evaluated
5. 1) gather 2) observe 3) developed 4) perfected
3) maintain 4) organise 5) appreciated
5) assimilate 6. 1) interesting 2) obvious
6. 1) manpower 2) infrastructure 3) engulfing 4) esteemed
3) money 4) resources 5) evolving
5) capabilities 7. 1) precarious 2) haunting
7. 1) capital 2) strength 3) deliberate 4) pervasive
3) authority 4) strategies 5) rabid
5) principles 8. 1) patience 2) alliance
8. 1) way 2) plan 3) influence 4) coping
3) system 4) goal 5) questioning
5) purpose
Cloze Test 425

9. 1) custom 2) capacity 2. 1) for 2) was


3) defence 4) preference 3) from 4) with
5) posterity 5) may
10. 1) consideration 2) accounted 3. 1) replacing 2) retailing
3) longing 4) regard 3) rotating 4) re-regulating
5) established 5) reducing
Dissertation = a long essay on a particular subject 4. 1) lie 2) profess
especially one written for a higher 3) exhibit 4) manifest
university degree 5) express
Benignly = kindly; gently; mildly; pleasantly
5. 1) analytical 2) absorbing
Indolent = lazy
Persistence = continuing to do something in spite
3) interesting 4) frightening
of difficulties 5) valuable
Overarching = Covering a wide range of topics, 6. 1) critical 2) obsolete
interests, activities etc 3) modern 4) devastating
Rabid = violent or extreme 5) lamentable
Off guard = not prepared for attack, a surprise 7. 1) durability 2) reactivity
or a mistake 3) activity 4) proactivity
Account for = to give a satisfactory record of
5) capacity
money, etc in one’s care.
Prof use = in large amounts; abundant
8. 1) systems 2) managements
Enchanted = filled with delight 3) processes 4) individuals

K
Chagrined = af fect ed wit h a f eeling of 5) units
disappointed or annoyance at 9. 1) echoed 2) supported
having failed, made a mistake etc. 3) adjusted 4) provided
Ponder = to thing about something carefully 5) developed
and for a long time especially in 10. 1) directing 2) providing
trying to reach a decision; to consider
3) affecting 4) questioning
Engulf = to surround somebody/something
especially so t hat they ar e
5) projecting
completely covered. Ad apt = to make something suitable for a
Evolve = to develop naturally and usually new use, situation etc.
gradually Foster = t o help t he development of
Esteem = t o have a high opinion of something; to encourage or promote
somebody/something; to respect something

KUNDAN
somebody/something greatly Culminate = t o reach t he highest point or
Precarious = not safe; dangerous specified conclusion or result
Haunting = beautiful and sad, making a strong Obsolete = no longer used; out of date
impression and remaining in the Shelf–life = the length of time for which a stored
thoughts. item, especially food, remains in
Pervasive = present and seen or felt everywhere good condition
Deliberate = done intentionally
Posterity = all future generations of people Passage 28
In (1) of constitutional guarantees relating to
Passage 27
equality of opportunity and various other guarantees
In the past, it was thought learning knowledge took of equality before the law, the social and economic (2)
place in school and for some also in further education. of women, especially of poor women in India, is well-
Then it was a matter of (1) practical skills at work at known. We are referring mainly to the poor rural women
the beginning of a career, and with a bit of luck, that who have little or no assets and who (3) the bulk of
(2) it. Now things have changed. Global competition the female population in rural areas. It is not as if
is (3) the shelf-life of products and the knowledge only poor rural women get less wages or suffer from
and skills that (4) behind them. The pace of change social ( 4) be cause the y be long to a particular
can be (5). Knowledge that was leading edge at one community. Even at highe r levels of the socio-
minute can become (6) the next. Therefore, it is the economic hierarchy among the well-to-do groups,
(7) rather than knowledge that is the key. Successful women are not (5) to men. Among the economically
organizations have to learn, adapt and change (6) sections of society, women’s proper place is (7) to
continuously as do the (8) within them. This is (9) in be the home. In rural areas, women of (8) status
the rapid growth of knowledge workers. It is (10) all families, normally do not go out to work. In the (9)
levels of organisations. value system, the re is a gradation of e conomic
1. 1) fostering 2) projecting activities, which is (10) in the socio-economic status
3) acquiring 4) manipulating of the family.
5) culminating Thus, if the women of the family do manual labour
in the fields, it denotes low status. Women earning a
426 Test of English Language

living, or supplementing their family income through long and are unlikely to (3) up together again: a(n) (4)
economic activities like stitching, garment-making, or rise in global oil prices, a monsoon that arrived late
some handicraft work, are also considered low because and a spike in global metal prices. North Sea crude
it clearly shows that their family is poor and they are has crossed $42 per barrel, driven up by low petroleum
forced to make ends meet. It is considered right and (5) and soaring demand in the US as war production
proper for a woman to cook, sew and take up activities heats up. Oil markets are also spooked by the (6) of
like pickle-making for her own family. But, if she were Russian oil supplies falling on the back of the Yukos-
to earn a wage through these same activities, it Sibneft probe. There’s little that the government can
denotes poverty and also, often, low socio-economic do to (7) users from soaring oil prices—indeed, it
status. shouldn’t, if it wants to (8) efficiency. Higher transport
1. 1) support 2) spite costs have pushed up rates of vegetables and fruits.
3) contrast 4) wake Farm produce could also get affected by rains that
5) view arrived too late for kharif sowing. China is (9) up steel
2. 1) condition 2) prosperity and other metals from all over the world to (10) a
3) progress 4) deprivation construction boom ahead of the 2008 Olympics,
5) value making metal prices soar all over the world, and
3. 1) constitute 2) deploy sparking inflation in India.
3) measure 4) define 1. 1) mere 2) moderate
5) exploit 3) retarding 4) vehement
4. 1) status 2) service 5) dull

K
3) indifference 4) ignorance 2. 1) obstinate 2) constitute
5) discrimination 3) persist 4) repeat
5. 1) dedicated 2) accountable 5) normalise
3) equal 4) responsible 3. 1) go 2) scramble
5) antagonistic 3) mount 4) yield
6. 1) marginal 2) significant 5) crop
3) well-off 4) affordable 4. 1) sustained 2) suspicious
5) dependable 3) horrific 4) erratic
7. 1) entitled 2) decided 5) favourable
3) indicated 4) debated 5. 1) lists 2) trades
5) considered 3) services 4) inventories

KUNDAN
8. 1) economic 2) appropriate 5) details
3) ample 4) higher 6. 1) prospect 2) progress
5) social 3) view 4) extent
9. 1) unequal 2) prevailing 5) deposit
3) appropriate 4) commendable 7. 1) support 2) ignore
5) deplorable 3) propel 4) prolong
10. 1) reflected 2) exempted 5) insulate
3) barred 4) considered 8. 1) position 2) promote
5) neglected 3) process 4) pass
Deprivation = the state of not having the benefits 5) form
that most people have, such as a 9. 1) hurrying 2) passing
home and enough food, money etc 3) pairing 4) gobbling
Deploy = to use something effectively
5) throwing
Antagonistic= showing or feeling opposition;
hostile; aggressive
10. 1) keep 2) make
Prevailing = most usual or widespread 3) feed 4) grow
Commendable = deserving praise 5) fight
Deplorable = that is, or should be condemned Scary = causing fear or alarm
Exempt = to make somebody/something free Crop up = t o appear or occur especially
from an obligation, duty or payment unexpectedly
Persist = t o continue t o do somet hing
Passage 29 especially with determination and
inspite of difficulty, opposition,
After ten years of (1) inflation, prices have hiked argument or failure
7.5% in the third week of July. This looks scary—after Soar = to rise quickly to a high level or
all, Indians had got used to prices crawling up by 2% standard
in the last two years, and a 10-year average inflation Spook = to become suddenly frightened by
rate of about 5%—but you shouldn’t worry. This burst something
Probe = a thorough and careful investigation
of inflation is the result of three factors that have
of something
come together unexpectedly, are unlikely to (2) for
Cloze Test 427

Gobble up = to use up all of something very 9. 1) closely 2) previously


quickly. 3) timely 4) hastily
Vehement = showing or caused by strong feeling; 5) questioningly
passionate
10. 1) stately 2) manifold
Obstinate = refusing to change one’s opinion or
decision, despit e at t empt s t o
3) shrinking 4) applicable
persuade one. 5) functioning
Horrific = causing horror Underlying = exiting in relation to a situation but
Inventory = a detailed list eg of goods, furniture not immediately obvious
or jobs to be done. Disseminate = to spread ideas, beliefs etc widely
Propel = to move, drive or push something/ Impetus = a force that encourages a process to
somebody forward develop more quickly
Prolong = to make something last longer Ramification = any of a large number of complex
Insulate = to protect somebody/something or unexpected results that follow an
from the unpleasant effects of action or a decision.
something. Disrupt = to make it difficult for something to
pr oceed, eg by causing noise,
Passage 30 problem, interruptions etc.
Systemic = done or acting according to a system
In recent years, the banking industry has been or plan
undergoing rapid changes, reflecting a number of (1) Relegate = to give somebody/something a
developments. The most significant has been advances lower or less important rank, task
in communication and information technology. Which or state.

K
Hastily = hurriedly
have (2) and broadened the (3) of financial information
Stately = having dignity; impressive; grand
while lowering the costs of many financial activities.
A second key (4) for change has been the increasing Passage 31
competition among a broad (5) of domestic and foreign
institutions in providing banking and (6) financial Tea prices in the domestic (1) continue to rule high
services. Third, financial activity has become larger in the (2) ye ar despite the expectation of a (3)
relative to overall economic activity in most economies. production as compared to the previous year. According
This has meant that any (7) of the financial markets to a preliminary assessment (4) on the weather (5) in
or financial infrastructure has broader economic (8) recent months, tea output in the next year may reach
than might have be e n the case (9) . The se 800 tons as (6) 780 tons last year. During the past
de ve lopme nts have (10) conse que nce s for the three months tea prices have shown an (7). Unlike

KUNDAN
institutional and systemic structure of the financial last year when tea prices were dramatically low, this
sector in general and banking in particular. year prices seem to have (8) at rather high level. In
1. 1) challenging 2) subjective the subsequent four months, the (9) average price
3) situated 4) underlying showed a downtrend, but in September the price has
5) principled (10) hardened to a considerable extent.
2. 1) measured 2) motioned 1. 1) market 2) area
3) habituated 4) processed 3) sector 4) profit
5) accelerated 5) production
3. 1) concealment 2) disagreement 2. 1) last 2) first
3) dissemination 4) sowing 3) current 4) second 4) earlier
5) differentiation 3. 1) lower 2) large
4. 1) force 2) impetus 3) higher 4) maximum
3) pull 4) movement 5) reasonable
5) energy 4. 1) shared 2) based
5. 1) group 2) rank 3) carried 4) strategy
3) place 4) range 5) conducted
5) row 5. 1) pattern 2) forecast
6. 1) personal 2) relegated 3) condition 4) outbreak
3) related 4) noticed 5) out bursts
5) referenced 6. 1) to 2) per
7. 1) disruption 2) dissociation 3) above 4) against
3) shattering 4) split 5) compared
5) dissection 7. 1) upgrade 2) uptrend
8. 1) branches 2) clusters 3) increased 4) increment
3) arrangement 4) ramifications 5) incline
5) subdivisions 8. 1) stabilised 2) surfaced
3) increased 4) moderated
5) synchronised
428 Test of English Language

9. 1) annual 2) weekly making a profit through changes in


3) daily 4) quarterly their value, but with the risk of
5) monthly losing money.
Conform = to follow generally accepted rules,
10. 1) now 2) then
standards etc; to comply
3) since 4) never Legitimate = in accordance with law or rules; legal
5) again Enact = to make or pass a law
Subsequent = later; following
Incline = to lean or slope or cause something Passage 33
to lean or slope, in a certain direction
Synchronise = to operate, move, turn etc at the First aid experts stress that (1) what to do for an
same time, speed etc. (2) victim until a doctor or other trained person gets
to the accident scene can (3) a life, especially in cases
Passage 32 of stoppage of breathing, severe bleeding, and shock.
A National Horticulture Mission is proposed to be People with special (4) problems, such as diabetes,
launched with a goal to (1) horticulture production by cardiovascular disease, epilepsy, or allergy, are (5) to
2011 -12. States have been (2) to join (3) with the wear some sort of emblem identifying the problem, as
Centre in launching this mission and establish a State a safeguard against administration of medication that
Level Cooperative Society for promoting horticulture. might be injurious or even (6). When emergencies do
Farmers will be (4) to (5) into oilseeds through occur, (7) first aid within the first few minutes often
promotion of superior seed technology and through (8) life or death. (9) administering of first aid (10)

K
an (6) policy of price support. medical professionals to provide better care.
A model law on (7) of agricultural produce has been 1. 1) before 2) attempting
circulated and, so far, ten States have (8) legal or (9) 3) regarding 4) knowing
action for ‘direct marketing’ and ‘contract farming’ 5) about
arrangements in line with the model law. The Budget 2. 1) injured 2) inquiring
urged all the States to (10) the model law at an early 3) efficient 4) important
date. 5) accidental
1. 1) channelise 2) market 3. 1) harm 2) comfort
3) mobilise 4) double 3) take 4) soothe
5) sell off 5) save
2. 1) found 2) invited 4. 1) mental 2) ethical
3) reported 4) noticed 3) medical 4) accident

KUNDAN
5) dedicated 5) moral
3. 1) hands 2) themselves 5. 1) prohibited 2) invited
3) them 4) along 3) compelled 4) allowed
5) products 5) urged
4. 1) empowered 2) encouraged 6. 1) appropriate 2) dangerous
3) paid 4) granted 3) beneficial 4) fatal
5) authorised 5) remedial
5. 1) look 2) turn 7. 1) expecting 2) providing
3) diversify 4) involve 3) avoiding 4) ignoring
5) invest 5) neglecting
6. 1) independent 2) encouraging 8. 1) determines 2) offers
3) expensive 4) exact 3) vanishes 4) reflects
5) appropriate 5) begs
7. 1) distribution 2) storage 9. 1) Hasty 2) Careless
3) harvesting 4) marketing 3) Proper 4) Probable
5) investment 5) Reasonably
8. 1) precipitated 2) speculated 10. 1) resists 2) instigates
3) initiated 4) prohibited 3) hinders 4) prevents
5) enforced 5) enables
9. 1) penal 2) conforming Epilepsy = a disease of the nervous system
t hat causes a per son to f all
3) legitimate 4) informal
unconscious, often with violent
5) administrative movements of the body.
10. 1) enact 2) explain Emblem = an object t hat r epr esent s
3) interpret 4) clarify something; a symbol
5) elaborate Fat al = causing or ending in death
Speculate = to guess; to buy and sell goods or Sa f e gu ar d = a thing that serves as a protection
stocks and shares in the hope of from harm, risk, or danger
Cloze Test 429

U rg e = to recommend or advise something discussion and without quarrelling


strongly Meticulous = giving or showing great care and
Remedial = providing or intended to provide a attention to detail.
treatment, medicine etc that cures
of disease or relieves pain. Passage 35
Instigate = to make something begin or happen
Hinder = to prevent or delay the progress of If an (1) is genius, he (2) the penalty of genius. If
somebody/something he has only talent, various cares and worries make
life extremely (3). He takes great pains (4) compose.
Passage 34 He meets with continuous (5) at his inability to reveal
(6). Also he is often (7) with the difficulty of (8) the
New technology has led directly to (1) standards of
public ear. A literary life (9), therefore, mostly an
living, yet science tends to follow market forces as
unhappy (10).
well as to (2) them. It is not surprising that the rich
1. 1) individual 2) ideal
get richer in a continuing cycle of (3) while the poorest
3) invention 4) event
are often left behind. A special (4) should be made by
5) author
the powerhouses of world science to address the
2. 1) tolerates 2) prevents
unmet challenges of the poor. Ending (5) poverty can
3) suffers 4) imposes
relieve many of the pressures on the environment.
5) inflicts
When impoverished households are (6) (7) on their
3. 1) miserable 2) impatient
farms, for example, they face less pressure to cut down
3) comfortable 4) happy
neighbouring forests in (8) of new farmland. Still, even

K
5) bearable
as extreme poverty ends, we must not fuel prosperity
4. 1) about 2) with
with a lack of (9) for industrial pollution and the (10)
3) in 4) to
burning of fossil fuels.
5) and
1. 1) visible 2) declining
5. 1) admiration 2) disappointment
3) improved 4) amicable
3) disapproval 4) criticism
5) rigorous
5) satisfaction
2. 1) fail 2) claim
6. 1) public 2) them
3) market 4) avoid
3) himself 4) literature
5) lead
5) others
3. 1) wealth 2) growth
7. 1) down 2) engaged
3) poverty 4) improvement
3) busy 4) leading

KUNDAN
5) economy
5) faced
4. 1) effort 2) care
8. 1) entering 2) sounding
3) practice 4) occasion
3)awakening 4) gaining
5) sanction
5) listening
5. 1) marginal 2) apparent
9. 1) is 2) governs
3) superficial 4) extreme
3) leads 4) begins
5) dismal
5) wishes
6. 1) abnormally 2) less
10. 1) thing 2) one
3) more 4) excessively
3) ending 4) event
5) unreasonably
5) incidence
7. 1) efficient 2) meticulous
Inflict = t o make somebody/something
3) careful 4) dependent suf f er somet hing; t o make
5) productive somebody expect something that is
8. 1) view 2) search unpleasant or not welcome
3) expectation 4) lust
5) place Passage 36
9. 1) attitude 2) mobility
The Indian Meteorological Department has sought
3) initiative 4) concern
permission to (1) a Doppler weather radar system -
5) ease
use d for long-range we athe r fore casting. The
10. 1) unchecked 2) repeated
Government had (2) the equipment in the wake of
3) periodical 4) occasional
26/7 and (3) to find a suitable location have been on
5) limited
(4) then. The key factor is that the radar’s antenna is
Unmet = not satisfied
to be installed in an (5) area of a few square kilometres
Dismal = less good than expected; very poor;
miserable; gloomy far from highrises, (6) at an altitude. The radar would
Impoverish = to make somebody poor; to make also need to be (7) near the coast as it would be used
somebody poorer or worse in quality to (8) high-intensity storms or cyclones. MHCC has
Amicable = based on or achieved through polite hinted it is willing to (9) clearance for the Colaba site,
430 Test of English Language

but only after (10) the location. It was after (11) around Encumber = to pr ev ent somebody/
for locations across the city that the office proposed something from moving or
to locate the radar near the observatory. But the acting freely and easily
Scout around = to look in various places to find
problem of finding a suitable site within the (12) still
somebody/something
remains. The area is very congested. We will have to Abstain = to keep oneself from doing or
locate a site not only from the heritage (13) of view having something that one
but also the radar needs to be at a height which is likes or enjoys.
higher than all the buildings in the area. The naval Precinct = an area in a town for specific or
residential buildings which are in the area are 13-14 restricted use, especially a
storeys high. If at all it is to be set up at Colaba then shoping area where vehicles
it must be above the (14) structures so that signals may not enter.
reaching the antenna are not (15).
Passage 37
1. 1) detach 2) install
3) launch 4) fix Several studies have (1) that folks who (2) engage
5) attach in mentally challenging activities—like reading, doing
2. 1) granted 2) realised crossword puzzles or playing chess—(3) less likely to
3) abstained 4) seen (4) dementia later in life. The difficulty comes in
5) sanctioned figuring out (5) their good fortune is a direct (6) of
3. 1) try 2) project their leisure activities or whether their continuing
3) commission 4) efforts pursuit of these pleasures merely (7) good genes for

K
5) worked cognitive function.
4. 1) since 2) until A 20-year survey of 469 elderly people living in the
3) already 4) at Bronx, New York, tried to get to the (8) of this chicken-
5) for or-egg question by following subjects who (9) no signs
5. 1) inseparable 2) encumbered of dementia in the first seven years of the study. The
3) unencumbered 4) unpossessed results, which were published in 2003, showed that
5) occupied reading and playing board games or a musical
6. 1) hopelessly 2) hoping instrume nt was (10) with a de cre ase d risk of
3) enacting 4) preferably Alzehimer's disease or other forms of dementia. (11),
5) undesirably those with the strongest habits (12) the greatest
7. 1) erected 2) located benefits. Participants who solved crossword puzzles

KUNDAN
3) stalled 4) tied four days a week, for instance, had a 47% (13) risk of
5) build dementia than those who do the puzzles once a week.
8. 1) deduct 2) examine By the same (14), several studies have suggested
3) feel 4) evaluate that older folks who are socially active — (15), for
5) detect example, do volunteer work or attend religious services
9. 1) advocate 2) launch — have a reduced risk of dementia.
3) pass 4) grant 1. 1) done 2) performed
5) grand 3) found 4) led
10. 1) examining 2) study 5) ensured
3) scanning 4) combing 2. 1) seldom 2) never
5) watching 3) absently 4) reluctantly
11. 1) marching 2) chasing 5) regularly
3) scouting 4) pursuing 3. 1) seem 2) have
5) hunting 3) were 4) refrain
12. 1) campus 2) premises 5) ascertain
3) area 4) perimeter 4. 1) cure 2) engage
5) precinct 3) embarrass 4) develop
13. 1) site 2) point 5) form
3) angle 4) out 5. 1) that 2) low
5) sight 3) when 4) why
14. 1) existing 2) enacted 5) whether
3) demolished 4) planned 6. 1) goal 2) result
5) conceived 3) measure 4) route
15. 1) stopped 2) hurdled 5) offer
3) blocked 4) paused 7. 1) encourages 2) reflects
5) halted 3) enhances 4) engenders
In the wake of = coming af ter or f ollowing 5) threats
something
Cloze Test 431

8. 1) height 2) cause Demonstrate = to show something clearly by


3) bottom 4) dilemma giving proof or evidence
5) anxiety
9. 1) had 2) conceal
Passage 38
3) reserve 4) force The growth story in any developing country cannot
5) accumulate be (1) without (2) its impact on the poverty and
10. 1) bereft 2) together employment situation. The Planning Commission has
3) envisaged 4) associated (3) that India should strive for ‘more inclusive growth’.
5) anticipated The number of people living below the poverty line
11. 1) Luckily 2) Certainly has (4) from 36 per cent in 1993-94 to 22.0 per cent in
3) Intriguingly 4) Unfortunately 2004-05. Again, the issue is to bring more and more
5) Obviously pe ople out of pove rty by (5) the m productive
12. 1) targeted 2) demonstrated employment opportunities. The Approach Paper to 11th
3) deserved 4) demanded Five Year Plan suggests that doubling the growth of
5) expected agricultural GDP to 4 per cent per annum will (6) rural
13. 1) more 2) greatly employment conditions, by raising real wages and
3) sharper 4) steeper reducing underemployment. However, even if this is
5) lower attained, an overall growth of 9 per cent will further
14. 1) token 2) way increase income (7) between agricultural and non-
3) analogy 4) example agricultural households, (8) around 10 million workers

K
5) author currently in agriculture find remunerative non-
15. 1) they 2) always agricultural employment. This (9) a major challenge
3) same 4) who not only in terms of generating non-agricultural
5) many employment but also in (10) its required location and
Dementia = a serious disorder of mind caused type.
by brain disease or injury 1. 1) completed 2) retold
Figure out = t o under st and somebody/
3) achieved 4) constructed
something by thinking about them/
it. 5) narrated
Pursuit = the action of looking for or trying to 2. 1) generating 2) assessing
find something. 3) realising 4) counting
Merely = only; simply 5) finding
Cognitive = of or relating to the action or process 3. 1) desired 2) estimated

KUNDAN
of acquir ing knowledge and 3) focused 4) verified
understanding through thought, 5) stressed
experience or the senses
4. 1) uplifted 2) degraded
By the same = exactly the one or ones referred
token to or mentioned; not different; 3) vanished 4) decreased
identical 5) enhanced
Refrain = to stop oneself doing something, 5. 1) absolving 2) providing
especially something that one would 3) nurturing 4) ignoring
like to do. 5) refusing
Ascertain = to investigate something so that one 6. 1) impact 2) diversify
knows and is certain; to find out 3) lay 4) aggravate
something
5) improve
Engender = to be the cause of a situation or
condition 7. 1) opportunity 2) assessment
Dilemma = a situation in which one has to 3) disparity 4) parity
choose between two undesirable 5) tax
things or courses of action. 8. 1) unless 2) for
Conceal = to hide somebody/something 3) in spite of 4) despite
Bereft = without or having lost a particular 5) by
power or quality; lacking hope, 9. 1) addresses 2) meets
support or ideas
3) poses 4) recognises
Envisage = to imagine something as a future
possibility; to form a mental picture 5) solves
of something 10. 1) exploring 2) acquitting
Anticipate = to expect something; to see what is 3) reciprocating 4) matching
going to happen or what will need 5) solving
to be done and take action to Strive = to try very hard or for a long time to
prepare for it in advance. obtain or achieve something; to fight
Intriguing = interest ing especially because hard against somebody/something
unusual; fascinating or mysterious Remunerative = for which one is well paid
432 Test of English Language

Absolve = to declare that somebody is free of 9. 1) resolve 2) order


guilt blame etc 3) observe 4) diagnose
Parity = the state of being equal especially 5) recommend
as regards status or pay
10. 1) casual 2) good
Disparity = a difference
Explore = to examine something thoroughly in
3) surgeon 4) handsome
order to test it or find out about it 5) insincere
Acquit = to declare somebody to be not guilty 11. 1) with 2) for
of a crime etc; to free or clear 3) at 4) upon
somebody of blame responsibility 5) in
etc 12. 1) dose 2) drug
Reciprocate = to give and receive something in 3) intake 4) nourishment
return; to make a mutual exchange
5) punishment
of something
13. 1) oppose 2) protest
Passage 39 3) subject 4) care
5) object
Though I had hired cabins in Bandra and a house 14. 1) oppose 2) take
in Andheri, divinity would not let me settle down. (1) 3) prescribe 4) describe
had I moved into my new house when my brother 5) propose
Balmukund, who had already been through an (2) 15. 1) hard 2) unkind
attack of jaundice some years back, had a (3) attack 3) easy 4) wise

K
of typhoid, ( 4) with pne umonia and signs of 5) careful
restlessness at night. The doctor was (5) in. He said Divinity = the quality of being God or a god.
medicine would have (6) effect, but eggs and chicken Confer with = to have discussions especially in
both might be given. Balmukund was only five years order to exchange opinions or get
old. To confer with his wishes was out of the question. advice
Being his (7) I had to (8). The doctor was very good. I Nourishment = f ood that keeps somebody/
something alive and well.
told him that we were all vegetarians and that I could
Prescribe = to advise or order the use of a
not possibly give either of the two things to my brother. medicine or medical treatment.
Would he therefore (9) something else? ‘Your brother’s Burly = big and strong; heavily built.
life is in danger,’ said the (10) doctor. ‘We could give
him milk diluted (11) water, but that will not give him Passage 40
enough (12). As you know, I am called in by many

KUNDAN
In our system, a vast gap (1) the life children lead
vegetarian families, and they do not (13) to anything I
at school and what they experience outside. The space
(14). I think you will be well advised not to be so (15)
where they are (2) to learn about life is so far removed
on your brother.’
from (3) that we might as well ask (4) to buy space-
1. 1) Then 2) Hardly
suits for their little ones, instead of school uniforms.
3) Wherever 4) Quicker
(5) they read, listen to and copy from the blackboard
5) Why
is so meticulously deodorised and (6) that it carries
2. 1) heart 2) big
no resonance of experienced reality and (7) in life.
3) acute 4) hard
The school day becomes a (8) of didactic songs and
5) harsh
memorised information. Special (9) are marked by
3. 1) unforgiving 2) hard
elaborate acts of sycophancy and preaching. School
3) burly 4) severe
authorities (10) stop talking about values, but ignore
5) tough
the cynicism felt by the young over the high levels of
4. 1) couple 2) felt
chicanery and verbosity they find in adult talk.
3) combined 4) joint
1. 1) occurs 2) separates
5) adjoining
3) bridges 4) escapes
5. 1) brought 2) called
5) finds
3) invited 4) sent
2. 1) about 2) worried
5) commissioned
3) compelled 4) supposed
6. 1) negligent 2) soothed
5) deprived
3) rough 4) little
3. 1) reality 2) school
5) deep
3) fantasy 4) imagination
7. 1) doctor 2) attendant
5) existence
3) nurse 4) forefather
4. 1) teachers 2) principals
5) guardian
3) schools 4) coaching classes
8. 1) plead 2) hide
5) parents
3) pressurise 4) decide
5) proceed
Cloze Test 433

5. 1) When 2) How and economic settlements. Even though the numbers


3) What 4) Whether involved in violent deaths are (7) by the larger numbers
5) Whenever that (8) from (9) of healthcare, the crude and brutal
6. 1) sanctified 2) written nature of this form of gender inequality makes it a
3) emphasised 4) memorised particularly severe (10) of the deprivation of women.
5) imbibed 1. 1) expectations 2) counting
7. 1) demand 2) culture 3) incidence 4) acceptance
3) miseries 4) joy 5) responses
5) applicability 2. 1) frequency 2) occurrence
8. 1) programme 2) ritual 3) event 4) chance
3) consult 4) tradition 5) blocking
5) store 3. 1) relatively 2) clearly
9. 1) persons 2) leaders 3) surely 4) undoubtedly
3) locations 4) occasions 5) astonishingly
5) indications 4. 1) accomplished 2) acknowledged
10. 1) always 2) deliberately 3) cleared 4) understand
3) seldom 4) relentlessly 5) assured
5) invariably 5. 1) anxiety 2) terrible
Spacesuit = a sealed suit covering the whole 3) surprise 4) power
body and supplied with air, allowing 5) form

K
somebody to survive and move 6. 1) national 2) visible
about in space.
3) social 4) category
Sanctify = the state of being holy or sacred; to
make somebody/something holy; to
5) personal
justify something 7. 1) fewer 2) outshine
Resonance = the power to bring, images, feelings, 3) lean 4) dwarfed
memories etc into the mind of the 5) horrible
reader, listener etc. 8. 1) perish 2) develop
Deodorise = to hide or remove unpleasant smells 3) spoil 4) incline
from something 5) direct
Didactic = designed f or t he purpose of
9. 1) omission 2) attention
teaching something
Sycophancy = the action of gaining people’s favour
3) care 4) effort
by insincere pr aise or always 5) neglect

KUNDAN
agreeing with them 10. 1) remark 2) indication
Cynic = a person who questions whether 3) happening 4) manifestation
something will really happen, 5) rise
whether something is important etc Brutal = cruel; savage; without mercy
Chicanery = the use of clever but misleading talk Incidence = the extent to which something
in order to trick somebody; false happens or has an effect.
argument Remarkable = unusual or exceptional; worth
Verbosity = noun of the word ‘verbose’ noticing
Verbose = using or containing more words than Batter = to hit somebody/something hard
are needed and repeatedly.
Imbibe = to adsorb something Astonishingly = very surprisingly
Misery = great suffering or discomfort of mind Terrible = v er y unpleasant and ser ious;
or body. causing one to feel very unhappy or
Relentless = never ending; constant upset
Invariably = always Dwa rf = to make somebody/something seem
small by contrast or distance.
Passage 41 Perish = to be destroyed; to die
Manifestation = an event, an action, an object or a
One of the most brutal features of gender inequality statement that shows something
takes the form of physical violence against women. clearly eg illustrating and resulting
The (1) of such violence is remarkably high, not only from an abstract idea.
in poorer and less developed economies but also in Accomplish = to succeed in doing something; to
wealthy and modern societies. Indeed the (2) of complete something successfully.
battering wome n e ven in the richest and most Accomplished = skilled; well trained or educated
developed economies is (3) high. Turning to India, it in social skills such as conversation,
art, music etc.
must be (4) first that the frequency of assaults on
women is high in the country. To that (5) general
recognition has to be added the special role of violence
connected with particular (6) features, such as dowry
434 Test of English Language

Passage 42 13. 1) louder 2) fewer


3) magnificent 4) most
With the US military tied down on two fronts and 5) bigger
the rest of the world growing (1) to American power, 14. 1) instability 2) fuel
the challenges for Rice are as (2) as they have been 3) energy 4) peace
for any Secretary of State in the past three decades. 5) atrocity
After six years of tussling with others on Bush’s 15. 1) defusing 2) demolishing
national-security team, Rice has seen off her rivals 3) terminating 4) igniting
and (3) as the principal spokesperson for Bush’s 5) extinguishing
foreign (4). Her reward has been to (5) responsibility Tie down = to restrict somebody/oneself to
for selling a failed policy in Iraq and (6) a legacy for cer t ain conditions or a f ixed
Bush at a time when (7) in the world are in the mood occupation or place.
to help her. “Bush is severely (8) and has very little Tussle = to struggle or fight to get something
(9) or support at home or abroad,” says Leslie Gelb, See off = to force somebody to leave a place
Grim = very bad; of very low quality
former president of the Council on Foreign Relations.
Emanate = to come or flow from something/
“That is (10) true for his Secretary of State. So they somebody or from a place.
are (11) flailing around.” Ignite = to start to burn or make something
That’s a grim assessment, since the (12) to start to burn
international order are (13) today than at any other Subservient = giving too much respect, obedience
time since the end of the cold war. The most immediate etc

K
source of (14) emanates from where the country civil Trivial = of little importance; concerned with
war risks (15) a region-wide conflict. unimportant thing
Daunt = to discourage or frighten somebody
1. 1) resistant 2) subservient
Entrust = to give responsibility for somebody/
3) immune 4) cordial something to somebody
5) indifference Shi rk = to avoid doing work, one’s duty etc
2. 1) obvious 2) trivial because one is lazy, cowardly, not
3) superfluous 4) daunting interested etc.
5) rewarding Inherit = to have features or qualities similar
3. 1) renamed 2) emerged to those of an ancestor.
3) appointed 4) entrusted Visualize = t o f or m a ment al pict ur e of
somebody/something
5) visited
Salvage = to save something from harm,
4. 1) aid 2) recognition

KUNDAN
disaster, difficult circumstances etc.
3) policy 4) acceptability Autocrat = a person who expects to be obeyed
5) minister at all times and pays no attention to
5. 1) shirk 2) avoid the opinions, feelings etc of others
3) transfer 4) visualize Inadvertent = not done deliber ately or
5) inherit intentionally
6. 1) focusing 2) framing Pleasantry = a friendly casual remark usually
made in order to appear polite
3) escaping 4) salvage
Accolade = an award of praise, approval or
5) demolishing honour
7. 1) people 2) few Atrocity = a very wicked or cruel act.
3) diplomats 4) autocrats Extinguish = to cause something to stop burning
5) most Legacy = money or property left to a person
8. 1) intensified 2) master-minded when somebody dies.
3) weakened 4) projected
5) supported Passage 43
9. 1) credibility 2) difficulty Delinking of jobs from degrees is one of the (1)
3) majority 4) power features of our education (2). There has been a (3)
5) enthusiasm fall in (4) in the academic field in recent years. There
10. 1) not 2) uniformly is a (5) of degree holders in the country. As a result,
3) remotely 4) partially university degrees have (6) their value and charm while
5) also the number of students in colleges and universities
11. 1) effectively 2) inadvertently of the country has been (7) rising. Consequently,
3) basically 4) aimlessly thousands of graduates and postgraduates come out
5) not of these institutions and stand in queues waiting to
12. 1) admirations 2) threats get some (8) jobs (9) in the country. Moreover, these
3) pleasantries 4) demands degree holders do not have any technical or vocational
5) accolades knowledge needed for a particular job. As a result,
Cloze Test 435

the number of educated unemployed has been rising find programmes that (9) the benefits of remitted cash
(10). It has created a very serious problem. while (10) some of its downside.
1. 1) minor 2) trivial 1. 1) accelerated 2) grew
3) unachievable 4) irrelevant 3) expand 4) increase
5) salient 5) escalating
2. 1) process 2) policy 2. 1) strike 2) encouraged
3) development 4) guideline 3) astonished 4) convinced
5) procedures 5) disturb
3. 1) expected 2) sheer 3. 1) rise 2) represent
3) rough 4) steep 3) project 4) exceed
5) gentle 5) recover
4. 1) assessment 2) evaluation 4. 1) record 2) tracks
3) competence 4) fees 3) estimate 4) report
5) value 5) surveys
5. 1) flood 2) class 5. 1) Detrimental 2) Minor
3) party 4) mob 3) Profuse 4) Benefited
5) rabble 5) Vital
6. 1) mislaid 2) lost 6. 1) circumstance 2) profit
3) increase 4) found 3) impact 4) status
5) establish 5) quality

K
7. 1) slowly 2) hastily 7. 1) declaring 2) established
3) deeply 4) gradually 3) measuring 4) reforming
5) steadily 5) govern
8. 1) prestigious 2) trivial 8. 1) mask 2) hid
3) menial 4) academic 3) review 4) display
5) managerial 5) supported
9. 1) occurring 2) posted 9. 1) launch 2) predict
3) created 4) available 3) optimum 4) appreciate
5) advertised 5) maximize
10. 1) exponentially 2) awfully 10. 1) augmenting 2) avoiding
3) terribly 4) fast 3) suspend 4) protects
5) incalculably 5) detracting

KUNDAN
Salient = most noticeable or important; main Astonish = to surprise somebody greatly
Consequently = as a result; therefore Remittance = a sum of money sent in payment for
Vocational = of or relating to the qualifications something; the sending of money in
and pr epar at ion needed f or a payment for something
particular job Alleviate = to make something less severe; to
Sheer = complete; nothing more than ease something
Rabble = a large disorderly group of people; a Plight = a serious and difficult situation or
mob. condition
Menial = not requiring much skill and often Put off = put something to a later time or date;
boring to delay something
Awful = ext r emely bad or unpleasant ; Ma s k = to hide or disguise something
terrible Remit = to send money etc to a person or
place especially by post.
Passage 44 Escalate = to increase or develop by successive
stages
Mass migration has produced a huge worldwide Detrimental = harmful
economy of its own which has (1) so fast during the Prof use = in large amounts; abundant
past few years that the figures have (2) experts. Last Optimum = best or most favourable
year remittances sent home by migrants were expected Augment = to make something larger in number
to (3) $232 billion according to the World Bank which or size; to increase something
(4) these figures. (5) though the flow of remittances Detract = to make something seem less good
or of lower value
is to alleviate the plight of the migrant’s family it
cannot on its own lift entire nations out of poverty. Passage 45
Those who study the (6) of remittances argue that
the money allows poor countries to put off basic Some places are so beautiful that they (1) the viewer
decisions of economic management like (7) their tax for all eternity. So it was for Emperor Muhammad
collection systems and building schools. Remittances Zahiruddin Babur, the 16th-century monarch who (2)
to poor countries can also (8) the fact that they do away his time in the pleasure gardens of Kabul before
not produce much at home. The challenge is now to heading south to India in 1525 to (3) the Mughal
436 Test of English Language

Empire. Though Babur built a dynasty that was to last F ra g r an t = having a pleasant or sweet smell
for 300 years, he never (4) his beloved Kabul, and (5) Dwell = to live in or at a place
vast riche s to re cre ate the garde ns (6) the
subcontinent. Those Mughal gardens, as they are now
Passage 46
(7), grace ancient capitals from Delhi to Srinagar with At just (1) midnight on July 1, 1997 in a glittering
their (8) vistas and strict architectural symmetry. But, and poignant ceremony, Hong Kong passed from being
Babur never really (9) at home in India and asked a jewel of the British empire to a (2) of a new global
that (10) his death his body be returned to Kabul and power. Hong Kong people (3) their city’s handover from
laid to rest in his favourite garden. the UK to China with (4) feelings: apprehension over
1. 1) attracted 2) haunt the future, joy at a fresh start, sadness at seeing the
3) fascinated 4) accommodate British go, pride over returning to their motherland.
5) implore On the eve of the handover, the stock market index, a
2. 1) cast 2) fed key barometer of Hong Kong’s wealth, (5) at a record
3) gave 4) whiled 15,200 points and today it (6) near the 21,000 mark.
5) deported Being a part of a booming China almost guarantees
3. 1) establish 2) travelled that Hong Kong will remain (7). But mainland China
3) crown 4) situate is a (8) as well as a partner. China’s new ports, for
5) find example, will siphon trade (9) from Hong Kong and its
4. 1) reached 2) visited lower labour costs will impact the jobs. However, there
3) saw 4) remembered is little doubt that Hong Kong is fortunate to have

K
5) forgot become a part of China at a time when mainland China
5. 1) accumulates 2) confiscated can provide (10) opportunity.
3) exhausted 4) demanded 1. 1) recorded 2) near
5) looted 3) close 4) past
6. 1) into 2) over 5) quite
3) overlooking 4) throughout 2. 1) component 2) premises
5) encroaching 3) captive 4) merger
7. 1) destroyed 2) dilapidated 5) list
3) rebuilt 4) inhabited 3. 1) encounter 2) decided
5) known 3) viewed 4) restrained
8. 1) elegant 2) notorious 5) told

KUNDAN
3) obnoxious 4) fragrant 4. 1) flexible 2) emotional
5) infrequent 3) mixed 4) changed
9. 1) went 2) dwelt 5) negative
3) felt 4) rested 5. 1) plunged 2) rose
5) enjoyed 3) valued 4) climbed
10. 1) before 2) upon 5) stood
3) till 4) in 6. 1) follows 2) pauses
5) at 3) fell 4) hovers
Haunt = to return repeatedly to somebody’s 5) measure
mind; to be impossible for somebody 7. 1) marginalised 2) prosperous
to forget
3) orderly 4) friendly
Eternity = time without end; endless life after
death 5) poor
While away = to pass a period of time in a relaxed 8. 1) competitor 2) representative
way 3) adversary 4) colleague
Grace = a quality of simple elegant beauty 5) member
and smoothly controlled movement. 9. 1) against 2) away
Elegant = gr acef ul and att r act iv e in 3) illegally 4) moving
appearance or manner 5) through
V is ta s = a beautiful view eg of nat ural
10. 1) full 2) risky
scenery, a city etc.
At home = in one’s own country 3) lucky 4) unfair
Implore = to ask or beg for something in a 5) immense
serious way Glittering = magnificent , splendid or
Confiscate = to take somebody’s property away extremely successful
from them by the use of one’s Poignant = affecting one’s feelings deeply;
authority usually as a punishment making one sad or full of pity
Dilapidated = falling to pieces; in a bad state of Apprehension = anxiety about something in the
repair future, fear that something will
Obnoxious = very unpleasant; offensive be unpleasant or that
Cloze Test 437

something unpleasant will 8. 1) paid 2) offered


happen. 3) deserved 4) distracted
On the eve of = the day or evening before an 5) received
event, especially a religious
9. 1) enhancement 2) prevention
festival or holiday.
Hover = to remain near something or in
3) attachment 4) refurbishment
an uncertain state. 5) expedition
Siphon away = to transfer something from one 10. 1) anticipate 2) provoke
place to another often unfairly 3) discourage 4) envisage
or illegally 5) create
Captive = having little or no freedom to Insolvent = unable to pay debt; bankrupt
go elsewhere or t o make Hazard = a thing that can be dangerous or
choices cause damage; a danger or risk
Adversary = an opponent in a contest, an Sa f e gu ar d = a thing that serves as a protection
argument or a battle. from harm, risk or danger
Paramount = more important than anything else;
Passage 47 supreme
Predominantly = mainly; for the most part
India’s approach towards treatment of (1) banks is Eventual = happening at last as a result
yet another interesting issue. Rather than closing Dissolve = t o cause an or ganizat ion or
them down, policymakers in India have shown a arrangement to end officially
preference to (2) such banks with healthy public sector Relegate = to give somebody/something less
important rank, task or state

K
banks. It has been (3) in certain circles that such an
approach may give rise to a moral hazard problem. Anchor = a person or thing that gives security
However, two issues need (4) in this context. First, and confidence
Extradite = to send back somebody accused or
comme rcial banks are the most dominant and
found guilty of a crime to the country
systemically important segment of the financial where the crime was committed.
syste m. Second, ove r 70 pe r ce nt of the bank Imbibe = to absorb something; to drink
depositors in India are small depositors. Therefore, something especially alcohol
systemic concerns coupled with the necessity to (5) Connive = to work together with somebody in
the interest of small depositors have been (6) in the order to do something wrong or
minds of policy makers while (7) with insolvent banks. illegal
This issue had not (8) much attention in the context Cope = to deal successfully with something
difficult; to manage
of a predominantly government-owned banking system.
Converse = to talk to somebody especially in

KUNDAN
As the weight of private banks increases further informal way.
thinking will need to be done on this subject, both in Distracted = unable to concentrate because of
terms of (9) of insolvency through advance regulatory being worried or thinking about
supervision and action, and post-insolvency measures something else
that (10) moral hazard and eventual fiscal cost. Refurbish = to restore and decorate a building
1. 1) insolvent 2) foreign et c; t o develop and improv e
3) cooperative 4) small something
Expedition = an organized journey or voyage with
5) private
a particular aim
2. 1) dissolve 2) relegate Envisage = to imagine something as a future
3) anchor 4) merge possibility; to form a mental picture
5) connect of something
3. 1) resolved 2) felt
3) promised 4) identified Passage 48
5) done
Although he is no longer alive, (1) his influence
4. 1) resolutions 2) decisions
can be felt in the studio (2) he created cartoons and
3) approaches 4) priority
feature films which made him known and (3) around
5) consideration
the world. (4) many people who work to create humour
5. 1) enhance 2) increase
he took it very seriously. He would sit sadly (5) the
3) safeguard 4) rationalize
funniest cartoon concentrating or some way to improve
5) evolve
it. Walt Disney (6) the opinions of those working with
6. 1) paramount 2) superficial
him but the (7) judge me nt was always his. He
3) extradited 4) vested
demanded a lot (8) people but he gave a lot too. When
5) imbibing
the economy was not doing well he gave every one a
7. 1) conniving 2) coping
(9) and though some (10) of this, it gave his employees'
3) absorbing 4) dealing
morale a boost.
5) conversing
438 Test of English Language

1. 1) yet 2) even 1. 1) knowledge 2) security


3) and 4) till 3) presence 4) confidentiality
5) besides 5) guarantee
2. 1) from 2) where 2. 1) negotiate 2) advance
3) which 4) while 3) credit 4) disburse
5) that 5) sanction
3. 1) respect 2) seen 3. 1) pursue 2) interact
3) loved 4) entertained 3) operate 4) enable
5) laughed 5) engage
4. 1) For 2) To 4. 1) drawback 2) hurdle
3) Without 4) Not 3) consequence 4) luxury
5) Like 5) innovation
5. 1) on 2) until 5. 1) Despite 2) Although
3) front 4) through 3) Even 4) Yet
5) in 5) Until
6. 1) saw 2) concluded 6. 1) view 2) realise
3) discussed 4) discouraged 3) display 4) engineer
5) valued 5) assess
7. 1) final 2) ultimately 7. 1) essential 2) obsolete
3) important 4) hasty 3) extant 4) retreat

K
5) lasting 5) expired
8. 1) by 2) from 8. 1) moderately 2) occasionally
3) with 4) to 3) compulsorily 4) indiscriminately
5) many 5) effectively
9. 1) advance 2) share 9. 1) phenomenal 2) gradual
3) fee 4) raise 3) proportionate 4) competitive
5) profit 5) projected
10. 1) credit 2) disapproved 10. 1) discount 2) base
3) criticized 4) offended 3) expansion 4) satisfaction
5) paid 5) relationship
Raise = an increase in amount, number or Bedrock = basic facts or principles
intensity Virtual = almost or nearly the thing described,

KUNDAN
Morale = t he amount of confidence, but not completely
enthusiasm, determination etc that Obsolete = no longer used; out of date
a person or group has at a particular Phenomenal = very remarkable; extraordinary
time. Disburse = to pay out money especially from a
fund collected for a purpose
Passage 49 Extant = still in existence
Retreat = to move back or withdraw when
Traditional bank architecture is based on bank faced with danger or difficulty
branches. These branches ensure the physical (1) of
a customer’s savings. A customer may go there to Passage 50
deposit and withdraw money, (2) loans and (3) in other
Our company has set up a foundation which is (1)
financial transactions. In the past two decades
to spreading literacy. To (2) this cause the foundation
banking architecture has changed– the Automated
has a project called ‘A Library for Every School’ through
Teller Machine (ATM) has been a big (4) and credit
(3) the foundation donates books mainly to government
and debit cards have created new financial spaces.
school libraries so that children have easy (4) to books
(5) the bank branch has remained the bedrock of the
on a variety of subjects. In my (5) as Chairperson of
banking system–after all a person needs a bank
the Foundation I travel (6) in rural areas. All this
account in a branch before he can operate a debit or
travelling has (7) me to understand what children want
ATM card. This may be about to change as technocrats
to read in different parts of the country. (8) my travels
now (6) cell phones as the new architecture of virtual
I frequently stay in the houses of people I meet as (9)
banks. This has the potential to make branches (7).
there are no hotels in small towns and villages that I
Cell phone banking looks especially relevant for India
visit. In India a guest is always treated well; an old
since it can penetrate the countryside cheaply and
Sanskrit saying is Atithi Devo Bhava (10) that God
(8). The world over cell phones are spreading at a (9)
comes in the form of a guest.
rate and in India alone new cell phone connections
1. 1) dedicated 2) responsible
are growing at the rate of six million a month–a rate
3) trying 4) catered
of customer (10) that no bank can dream of.
5) involved
Cloze Test 439

2. 1) awaken 2) further Scientists from India’s space and atomic energy


3) aim 4) contribute departments and in some other places where serious
5) perform science is done can take (a/an) (9) out of the school’s
3. 1) those 2) which book and (10) the way in engaging with school pupils
3) whom 4) where and getting them to do real science.
5) these 1. 1) done 2) unlikely
4. 1) opportunity 2) admission 3) potential 4) promising
3) purchase 4) access 5) possible
5) contact 2. 1) questioned 2) said
5. 1) feeling 2) decision 3) retorted 4) answered
3) role 4) knowledge 5) address
5) order 3. 1) question 2) finding
6. 1) extensively 2) somehow 3) methodology 4) result
3) extremely 4) hastily 5) studies
5) sometime 4. 1) wage 2) create
7. 1) ensured 2) provided 3) execute 4) carry
3) enabled 4) deprived 5) attempt
5) made 5. 1) option 2) lives
8. 1) During 2) Since 3) visual 4) demands
3) From 4) Through 5) perception

K
5) Besides 6. 1) revolutionary 2) radical
9. 1) while 2) usual 3) rote 4) adequate
3) neither 4) often 5) bore
5) either 7. 1) stimulate 2) simulate
10. 1) threatens 2) meaning 3) make 4) peek
3) fearing 4) imply 5) judge
5) naturally 8. 1) cause 2) root
Cater = to provide what is needed or desired 3) reasons 4) issues
by somebody/something 5) sources
9. 1) thread 2) leaf
Passage 51 3) example 4) look
Can an experiment conceived, carried out, and 5) pages

KUNDAN
reported in kids-speak with pencil-coloured figures 10. 1) lead 2) start
and hand-written tables by school children aged 8 to 3) deliver 4) paved
10 years get published in a highly rated international 5) ahead
journal following a peer-reviewing process? Twenty- Peer = to look closely or car ef ully at
seven schoolchildren from a primary school in UK have something especially when unable
to see it well
proved this is (1) — if a simple but novel scientific
Bumble-bee = a large hairy bee that makes a loud
question raised is (2) in a scientific way. Their paper noise as it flies
was published in the Royal Society’s Biology Letters Fo ra ge = to search or hunt for something
journal. Their (3) was that bumble-bees can use a especially food and supplies
“combination of colour and spatial relationships in Spati al = related to space as a physical
deciding which colour of flower to forage from.” dimension
Considering that our understanding of how bees Carry out = to do something as required or
perceive coloured patterns and scenes is inadequate, specified; to fulfil something
Rote learning = learing something in order to be
this inspiring outcome has shown that schoolchildren
able to repeat it from memory, rather
guided by gifted teachers can think and (4) out than in order to understand it.
experiments like any hard-wired scientist. For these Retort = to make a quick, especially angry,
kids, doing science changed their (5) of the subject. reply to an accusation or a challenge
Science also became “cool and fun.” This refreshing Stimulate = to pretend to have or feel an emotion;
approach turns the spotlight on the best methods of to create certain conditions by
teaching science. The (6) learning system adopted means of model etc; to take the
by most schools in India, even classroom study appearance of somet hing/
somebody
combined with some laboratory work with pre-defined
outcomes, does very little to (7) curiosity and interest Passage 52
in science. Is that one of the (8) why out-of-the-box
thinking that produces path-breaking science rarely Does Indian industry need democracy? The Indian
comes out of Indian laboratories? The children at the economy’s sustained growth today is (1) by incomplete
UK school had their gifted teacher to guide them. democracy. While millions of Indians endure poverty,
440 Test of English Language

only a tiny majority (2) prosperity. On the other hand, Egalitarian = showing or holding a belief in equal
many Latin American countries have registered (3) rights, benefits and opportunities
growth rates under military dictators and today one of for everybody
Orthodox = following strictly the older, more
the fastest growing economies in the world — China
traditional practices.
— has an (4) rather than a democratic government. Imply = to suggest something indirectly
So why does India need democracy for sustained rather than stating it directly
growth? To many, democracy (5) slower decision- Gracious = kind, polite and generous especially
making with corrupt politicians and red-tapeism etc. to somebody of a lower social
Industry should therefore be (6) with less, not more, position
democracy. However, while China (7) consumption in Indulge = to become involved in an activity
order to save and invest more than half its output to especially one that is illegal or
disapproved of.
produce 10% growth, India (8) almost two-thirds of
Disperse = to go in different directions or make
its output and manages to achieve 9% growth from somebody/something do this
one-third of its output. (9) India’s democracy is not
inefficient when it comes to making (10) use of Passage 53
resources.
1. 1) deprived 2) hampered A factor that air passengers give little thought to
3) eliminated 4) faced but which is a serious threat to air safety is (1)
5) threaten maintenance. In the current global airline boom
2. 1) pursuit 2) acquisition competition is (2). Which compels airlines to (3) costs

K
3) benefit 4) enjoy and ( 4) output. In India with a se at capacity
5) value considerably (5) of the demand all airlines practise
3. 1) acceptable 2) insignificant severe cost-cutting to (6). Faced with having to cut
3) variable 4) inflated costs to the bone and maximise aircraft utilisation, a
5) affordable surprising number of airlines may cut (7) on aircraft
4. 1) autonomous 2) economical maintenance, even at the (8) of compromising safety.
3) authoritarian 4) egalitarian While commercial aircraft are (9) to take much
5) orthodox punishment, for example, in the event of pilots flying
5. 1) imply 2) mentions into thunderstorms there is a limit to the punishment
3) attracts 4) features that e ve n the toughest aircraft can take whe n
5) means profitability takes (10) over safety.
1. 1) expedient 2) incessant

KUNDAN
6. 1) gracious 2) adapted
3) fascinated 4) pleased 3) routine 4) laborious
5) urged 5) poor
7. 1) bans 2) curtails 2. 1) optimise 2) intense
3) regulate 4) ceases 3) destined 4) guarantee
5) discourage 5) profitable
8. 1) consumes 2) selects 3. 1) falsify 2) lavish
3) indulges 4) disperse 3) minimise 4) incline
5) hoard 5) ration
9. 1) Accordingly 2) Totally 4. 1) depress 2) productive
3) Thus 4) Even 3) curb 4) available
5) Likely 5) maximise
10. 1) ultimately 2) capably 5. 1) ahead 2) less
3) modest 4) secure 3) more 4) saturated
5) effective 5) above
Hamper = t o pr event somebody’s fr ee 6. 1) invest 2) survive
movement or activity; to restrict or 3) live 4) appraise
hinder somebody/something 5) reinforce
Endure = to suffer patiently something that 7. 1) out 2) taxes
is painful or uncomfortable 3) across 4) corners
Authoritarian = favouring complete obedience to 5) short
authority especially that of the state
8. 1) claim 2) formality
at the expense of personal freedom
Acquisition = the action or process of acquiring 3) reminder 4) strain
something 5) risk
Inflate = to make something more important, 9. 1) designed 2) unable
impressive etc than it really is; to 3) rotated 4) originally
exaggerate something 5) standard
Cloze Test 441

10. 1) encouragement 2) influence 2. 1) full 2) voluminous


3) precedence 4) cover 3) substantial 4) limited
5) guard 5) rapid
To cut to the = to reduce something greatly 3. 1) readily 2) tangible
bone 3) routinely 4) securely
To cut corners = to do something in the easiest, 5) unique
quickest or cheapest way, often by
4. 1) process 2) waves
ignoring rules or omitting something
Precedence = the right or requirement to come
3) deliveries 4) connection
before somebody/something else in 5) channels
time, order etc; priority 5. 1) valuable 2) answerable
Expedient = useful or convenient for a particular 3) amenable 4) exposed
purpose, though not necessarily fair 5) responsible
or moral 6. 1) waning 2) stable
Incessant = not stopping; continual 3) proportionate 4) marginal
La vi sh = giv ing or doing somet hing
5) high
generously or excessively; great in
extent, rich in quality and usually
7. 1) archive 2) domain
costing a lot of money 3) purpose 4) component
Ration = to limit the amount of something that 5) aspect
somebody is allowed to have 8. 1) law-abiding 2) tried
Curb = to prevent something from getting 3) reassuring 4) cost-effective

K
out of control; to restrain something 5) stop-gap
Appraise = to assess the value, quality or nature 9. 1) inclined 2) immune
of somebody/something
3) vulnerable 4) surrendered
Reinforce = t o st rengt hen or emphasize a
feeling, an idea, a habit etc
5) pressured
10. 1) person 2) own
Passage 54 3) relatives 4) purses
5) self
Mobile banking (M banking) involves the use of a Readily = without hesitating; willingly
mobile phone or any other mobile device to (1) Amenable = that can be treated in a particular
financial transactions linked to a client’s account. M way
banking is new in most countries and most mobile Domain = a field of knowledge or activity
payment models even in developed countries to date Single out = to choose somebody/something

KUNDAN
operate on a (2) scale. A mobile network offers a (3) from a group for special attention
Vulnerable = that can be hurt, harmed or attacked
available te chnology platform onto which other
easily especially because of being
services can be provided at low cost with effective small or weak.
results. For example, M banking services which use Implication = a thing that is not openly stated; a
(4) such as SMS can be carried at a cost of less than thing that is suggested or implied.
one US cent per message. The low cost of using Disburse = to pay out money especially from a
existing infrastructure makes such services more (5) fund collected for a purpose
to use by customers with lower purchasing power and Lure = a thing that attracts or is used to
opens up access to services which did not reach them attract people or animal
Tangible = clear and definite; real; that can be
earlier due to (6) cost of service delivery. Although M
perceived by touch
banking is one aspect in the wider (7) of e-banking Waning = becoming gradually smaller, weaker,
there are reasons to single it out for focus—especially less powerful or less important
because there are a lot more people with mobile Law-abiding = obeying the law
phones than bank accounts in India. Immune = not aff ected or inf luenced by
M banking could provide a (8) solution to bring more something
“unbanked” people to the financial mainstream.
Without traditional credit, individuals are (9) to Passage 55
exploitation by abusive lenders offering very high In July 2008, one of the most inspiring leaders of
interest rates on short term loans. Also of considerable our times, will (1) his ninetieth birthday. Nelson
importance are public safety implications for the Mandela retired from politics in 1999, but he has
unbanked—they are often victims of crime because remained (2), continuing his work through the Nelson
many operate on a cash-only basis and end up carrying Mandela Foundation. The foundation has launched
significant amounts of cash on their (10) or store cash an Aids awareness campaign, 46664, named (3)
in their homes. Mandela’s prison number. He has also set up a
1. 1) disburse 2) undertake scholarship programme whose (4) was to promote
3) subscribe 4) lure leadership among young Africans.
5) amass
442 Test of English Language

During the 1990s, (5) I worked with Mr Mandela on efficiency by Japan. Mass production and production
his autobiography Long Walk to Freedom, I (6) his for the masses became the bases of new business
leadership firsthand. During his election campaign strategies. Large-scale consumption by all with the
we were on board a plane discussing his book. Twenty social benefit of (3) poverty, became the dominant
minutes (7) to landing the engine failed. Many began economic strategy. The advent of electricity and its
to panic. The only thing that (8) them was looking at large -scale application to lighting, heating and
Mandela, who was reading his paper as if he was a operating machine s added a fresh dimension to
passenger on a morning train to work. The plane manufacturing. By the 1950s came (4) in electronics
landed safely and when we got into the car taking us and transistor devices to be followed by innovations
to the hotel he (9) to me, “I was terrified on the plane in microelectronics, computers and various forms of
!” As a leader he realised he was a model for others sensors all of which (5) altered the manufacturing
and this gave him the strength to (10) over his own sce ne . It is now no longe r ne ce ssary to make
fear. prototypes in a factory or a laboratory to study a new
1. 1) tribute 2) remember product. Many new products can be (6) on computers
3) honour 4) celebrate and their behaviour simulated on them. By choosing
5) rejoice an optimum de sign through such simulations,
2. 1) resigned 2) active compute r programme s can dire ctly (7) the
3) influenced 4) participant manufacturing proce sse s. The se proce sse s are
5) reserved generally called Computer Aided Design (CAD) and
3. 1) by 2) with Computer Assisted Manufacturing (CAM). The se

K
3) after 4) as capabilities are leading to newer forms of (8) by
5) thereafter customers. Each customer can be offered several
4. 1) wish 2) pursuit special options. Customised product design or (9)
3) result 4) plot manufacturing are other popular techniques currently
5) aim in (10) in many developed countries.
5. 1) when 2) that 1. 1) havoc 2) transformation
3) period 4) later 3) destruction 4) violence
5) alongside 5) deforestation
6. 1) felt 2) acquainted 2. 1) discarded 2) resorted
3) experienced 4) underwent 3) indulged 4) perfected
5) learned 5) designated

KUNDAN
7. 1) before 2) sooner 3. 1) removing 2) nurturing
3) close 4) prior 3) appeasing 4) cajoling
5) advance 5) mastering
8. 1) calmed 2) soothing 4. 1) additions 2) gadgets
3) composed 4) restraint 3) modifications 4) variety
5) discipline 5) inventions
9. 1) speaks 2) confided 5. 1) immediately 2) precisely
3) confidentially 4) entrusted 3) irreversibly 4) indefinitely
5) assured 5) measurably
10. 1) success 2) overcame 6. 1) designed 2) produced
3) dominate 4) victory 3) manufactured 4) sold
5) triumph 5) purchased
Name after = to give a name to somebody or 7. 1) inspire 2) cultivate 3) visualise
something 4) drive 5) curtail
On board = on or in a ship or an aircraft 8. 1) uses 2) demands
Confide = to tell a secret to somebody
3) advertisements 4) consumption
which trusting them not to tell
others
5) goods
Terrified = very frightened; filled with 9. 1) visible 2) secure
terror 3) fundamental 4) overt
Triumph over = to be successful; to gain a 5) flexible
victory 10. 1) view 2) wings
Rejoice = to feel or show great joy 3) vogue 4) isolation
Acquainted = familiar with something 5) order
Advent = the approach or arrival of an important
Passage 56 person, event etc
Mankind has seen rapid (1) in the last 150 years Prototype = the first model or design of something
from which other forms are copied or
because of the mass manufacturing techniques (2) in
developed.
western nations and later taken to new levels of
Cloze Test 443

Simulated = artificial, but made to look, feel etc like accommodate all. Only then can we ensure the much-
the real thing needed supply-demand (10) in the education sector.
Vogue = a current fashion 1. 1) with 2) for
Discard = to throw something out or away; to stop
3) on 4) into
using, wearing, etc something that is no
longer useful
5) in
Resort = to make use of something especially bad 2. 1) around 2) near
or unpleasant as a means of achieving 3) into 4) about
something, often because no other course 5) reaching
of action is possible 3. 1) forming 2) translating
Indulge = t o become inv olv ed in an act ivit y 3) having 4) taking
especially one that is illegal or disapproved 5) framing
of
4. 1) affect 2) ideas
Designate = to mark or indicate something clearly; to
specify something
3) practice 4) concept
Nurture = to care for and encourage the growth of 5) procedure
somebody/something 5. 1) benefit 2) merit
Appease = to reduce the intensity of somebody’s 3) chance 4) basis
feelings usually by satisfying their needs 5) method
or demands partly or in full 6. 1) unless 2) until
Cajole = to make somebody do something by 3) executed 4) provided
cleverly persuading, deceiving or flattering
5) exercised
them.

K
Visualise = to form a mental picture of somebody/
7. 1) other 2) any
something 3) two 4) differ
Curtail = to make something shorter or less; to 5) after
reduce something 8. 1) on 2) of
Overt = done or shown openly or publicly; not 3) often 4) taken
secret or hidden 5) off
9. 1) soft 2) more
Passage 57 3) less 4) only
The Right of Children to Free and Compulsory 5) hard
Education (RTE) Act, 2009, which came (1) effect in 10. 1) need 2) equilibrium
April this year, is meant to transform the education 3) expectation 4) attempt
sector and take India closer to the goal of universal 5) aspects

KUNDAN
schooling. But with admissions to the new academic Just around the corner = very near
session just (2) the corner, it is fast becoming clear Translate into = to express something or to be
expr essed in a diff er ent
that (3) well-intentioned ideas into (4) will take some
especially a more practical form
doing. For a start, the guidelines for admissions under Random = done, chosen et c wit hout
the RTE prohibit schools from conducting any sort of method or conscious choice;
stude nt profiling. The stre ss on a random ye t haphazard
justifiable admission process means that schools will Quirk = a strange thing that happens
have to resort to something as quirky as a lottery especially accidently
system. However, leaving admission to a good school The crunch = an impor t ant and oft en
to pure (5) will only incentivise manipulations, unpleasant point, situation or
piece of information
defeating the very essence of RTE.
Mortar = a mixture of lime with cement
The main problem facing the education sector is sand and wat er , used in
that of a resource crunch. The provisions for ensuring building to hold bricks, stones
universal access to education are all very well, (6) we etc together.
have the infrastructure in place first. Brick-and-mortar Precede = to happen before something
schools need to precede open admission and not the The other way = in the opposite position or
(7) way around. In that sense, legislators’ assessment around direction
of ground realities is (8) target when they endorse Endorse = t o give one’s approv al or
support to a claim, statement,
the closure of tens of thousands of low-cost private
course of action etc
schools for not meeting the minimum standards of Specification = a descr iption of what is
land plot, building specifications and playground area required
as laid out in the RTE Act. Instead of bearing down (9) Bear down on = t o mov e quickly t owar ds
on private schools for failing to conform to abstract somebody/something in a
bureaucratic criteria, efforts to bring about universal determined or threatening way
education should focus on upgrading and expanding Conform = t o comply; t o agr ee or be
the existing government school infrastructure to consistent with something
Abstract = general; not based on any
particular person, situation etc.
444 Test of English Language

Passage 58 Passage 59
(1) a country needs money for a development Re ce ntly the World Bank and the Asian
project, what can it do? It can (2) to the World Bank or Development Bank (ADB) (1) separate reports on
Asian Development Bank for aid. A country with a poverty. The World Bank report (2) its benchmark of
foreign currency problem can ask the International extreme poverty by 25 cents from $1 per person per
Monetary Fund for (3). However, (4) there is no way day to $1.25 per person a day. The ADB announced an
out for a country which has shortage of food. The even (3) benchmark of $ 1.35 per person a day. These
country cannot (5) import the food if it is rare like new benchmarks are (4) on surveys in the world’s
pulses which are grown only by a few countries. In poorest countries.
such cases the problem is more (6). Experts often like to (5) that poverty has declined
This situation has led experts to suggest the (7) of because of economic growth in India and China. This
establishing a World Agricultural Bank. The food is wrong and (6). In the past twenty-five years the
situation today is serious since production is not pove rty rate in India has (7) by less than one
keeping (8) with demand. The World Agricultural Bank percentage point a year. (8) we use a poverty line of
can therefore be established by member-countries who $1 per person per day or $1.25 per person per day
have to (9) both capital as well as surplus food to the makes little (9). The number of poor in India is large.
Bank. The stocks would form a corpus which would be The purpose of these statistics is not to dispute them
used to assist members in (10) of distress. but to (10) whether the benefits of economic growth
1. 1) Though 2) Supposed are being shared with the poor.

K
3) Unless 4) That 1. 1) declared 2) released
5) When 3) print 4) issue
2. 1) appeals 2) go 5) publish
3) approach 4) solicit 2. 1) heightened 2) announced
5) requests 3) raised 4) maintained
3. 1) backing 2) helping 5) notified
3) solution 4) assistants 3. 1) better 2) significant
5) relieve 3) plausible 4) higher
4. 1) simply 2) during 5) lower
3) fact 4) presently 4. 1) based 2) collected
5) while 3) inferred 4) derived
5. 1) attempt 2) yet 5) gathered

KUNDAN
3) even 4) try 5. 1) realise 2) claim
5) start 3) discover 4) recommend
6. 1) address 2) acute 5) criticize
3) declined 4) achievable 6. 1) adverse 2) opposing
5) prohibited 3) corrupt 4) rejected
7. 1) object 2) implementation 5) misleading
3) knowledge 4) advice 7. 1) deplete 2) plunge
5) idea 3) declined 4) weaken
8. 1) up 2) ahead 5) fell
3) paced 4) line 8. 1) Unless 2) Despite
5) tune 3) Instead 4) Whether
9. 1) demand 2) share 5) Regardless
3) benefit 4) contribute 9. 1) difference 2) effect
5) fund 3) contrast 4) question
10. 1) controls 2) combats 5) option
3) times 4) needs 10. 1) acknowledge 2) suggest
5) areas 3) care 4) inspire
Backing = help; support 5) study
Acute = very great; severe Plausible = seeming to be right or reasonable
Keep up = to continue without stopping that can be believed
Corpus = a collect ion of wr itt en and/or Deplete = to reduce greatly the quantity, size,
spoken texts power or value of something
Distress = the state of being in danger or Plunge = to jump or fall into something quickly
difficulty and needing help and with force.
Solicit = to ask somebody eagerly or firmly
for something; to try to obtain
something
Combat = t o f ight or str uggle against
somebody/something
Cloze Test 445

Passage 60 Passage 61
Technology (1) lives. But (2) if people want it to. The US is in the (1) of a cleanup of toxic financial
This qualification is important, and (3) to waste that will (2) taxpayers hundreds of billions of
understanding progress. Akio Morita, the founder of dollars, at the very least. The primary manufacturers
Sony, used to make inventions not by writing code of these hazardous products (3) multimillion-dollar
but by making minute, detailed studies of (4) people paychecks for their efforts. So why shouldn’t they (4)
lived the ir live s. It is observable that when he to pay for their mop-up? This is, after all, what the US
re linquishe d dire ct involve me nt in product Congress (5) in 1980 for (6) of actual toxic waste.
development at the company in the 1980s, Sony Under the Superfund law (7) that year, polluters (8)
seemed to lose its (5) of developing a truly radical for the mess they make. Environmental lawyer E
invention like the Walkman that the world takes to Michael Thomas sees no (9) lawmakers couldn’t
en masse. demand the same of financial polluters and (10) them
However much it seems that machines are in (6), to ante up some of the bank bailout money.
they are not. Yet the belief that technology alone holds 1. 1) range 2) depth
the key to (7) the way people work, buy, and do 3) midst 4) essence
business is strong. The rise of dotcoms in the late 5) debate
1990s was (8) by a belief that technology was changing 2. 1) benefit 2) cost
the rules of marketing and employee relationships. 3) earn 4) facilitate
This is not to say there have been no changes in the 5) save

K
new economy; but that they (9) to appear where 3. 1) donated 2) demanded
technology makes it (10) for people to communicate 3) dwindled 4) spent
with each other, or have been unre lated to the 5) pocketed
technology. The dynamic is still a human one. 4. 1) hesitate 2) come
1. 1) ruins 2) changes 3) makes 3) defy 4) have
4) explains 5) shakes 5) admit
2. 1) not 2) occasionally 5. 1) decreed 2) refrained
3) seldom 4) only 3) commented 4) admonished
5) never 5) visualised
3. 1) key 2) primarily 6. 1) consumers 2) advocates
3) encouraging 4) supported 3) exponents 4) producers
5) disastrous 5) users

KUNDAN
4. 1) why 2) where 7. 1) revoked 2) forced
3) when 4) whether 3) squashed 4) abandoned
5) how 5) enacted
5. 1) share 2) profit 8. 1) regain 2) claim
3) knack 4) business 3) pay 4) demand
5) plant 5) consider
6. 1) progress 2) control 9. 1) practice 2) reason
3) action 4) operation 3) compensation 4) issue
5) transition 5) wonder
7. 1) encroaching 2) accomplishing 10. 1) force 2) plead 3) appeal
3) determining 4) highlighting 4) dupe 5) follow
5) informing In the midst of = while something is happening
8. 1) govern 2) successful or being done.
3) underlying 4) disputed Cleanup = t he r emov al of cr iminals,
harmful influences etc; the
5) accompanied
removal of dirt etc from a person
9. 1) tend 2) cease or place
3) fail 4) refuse Toxic = poisonous
5) avoid At the very least = and probably more than that
10. 1) essential 2) laborious H a z ar d o u s = dangerous; risky
3) tough 4) easier Pocket = to keep or take something for
5) awkward oneself especially dishonestly
Relinquish = to stop having, doing or claiming After all = in spite of what has been said,
something; to give something up done or expected.
Knack = a skill at performing some special Decree = t o or der somet hing by a
task; an ability judgement or decision made by
Radical = fundamental; of or from the root or certain lawcourts
base Mess = a dirty or untidy state.
en masse = in a mass or crowd; all together
446 Test of English Language

Ante = money etc risked or gambled 4. 1) prove 2) search


on the unknown result of a 3) application 4) understanding
future event eg a race or a card 5) acknowledge
game
5. 1) law 2) aspects
Facilitate = to make something especially
an action or a process, easy or
3) experts 4) books
easier 5) loop
Dwindle = to become gradually less or 6. 1) equip 2) arm
smaller 3) decorate 4) promote
Refrain = to st op oneself doing 5) load
somet hing especially 7. 1) optional 2) expensive
something that one would like 3) tough 4) deep
to do
5) specialized
Admonish = to give a mild but firm warning
to somebody; to advise urge
8. 1) authentic 2) voluminous
somebody seriously 3) many 4) prompt
Revoke = to withdraw or cancel a law, 5) shining
licence, etc 9. 1) consulting 2) qualified
S q ua s h = to press or crush something so 3) rich 4) merchant
that it changes shape, becomes 5) tired
very soft etc 10. 1) learned 2) powerful
Dupe = to deceive or trick somebody in
3) ready 4) comprehensive

K
doing something
Plead = t o make r epeat ed ur gent
5) prescribed
r equest s to somebody for Pave the way = to create a situation in which
something somebody will be able to do
Defy = to ref use to obey or show something or something can
r espect f or somebody/ happen.
something Lo op = a set of instructions that is
repeated again and again until
Passage 62 a par ticular condit ion is
satisfied
It is a pity that we do not have good books on
insurance written by Indian authors (1) to the steady Passage 63
growth of literature on the subject in other countries, On October 2, 1983 the Grameen Bank Project (1)

KUNDAN
especially the USA, whose insurance laws and (2) are the Grameen Bank. We invited the Finance Minister
very much similar to those of our country. And to be the Chief Guest at our (2) ceremony. But when
students studying in our colleges and the millions of the Ministry came to (3) that the ceremony would take
insurance employees appearing for various insurance place in a remote district, they said it would not be an
examinations have to depend (3) on books written by (4) place to launch a Bank and that the ceremony
foreign authors. As these books mainly deal with the should be (5) in Dhaka so that all the top Government
problems of insurance industry of foreign countries, Officials could (6). We stood firm and (7) to them that
the (4) of the insurance scene in India and the various we did not work in urban areas so it made no (8) to
legal (5) and insurance procedures remains very weak. have the ceremony in a city (9) we had no borrowers.
To (6) the insurance employee s and the college We had the ceremony in a big open field with the
students who have opted for (7) courses in insurance Finance Minister present as Chief Guest. For all of
with different aspects of theory and practice of us who had worked so hard to (10) this it was a dream
insurance, we should have good and (8) textbooks. come true.
The book under review written by VMR Nair himself, 1. 1) reorganised 2) merged
an experienced and (9) expert on insurance law based 3) named 4) converted
on leading Indian cases, will be found very useful by 5) became
students of insurance sector as a (10) guide to the 2. 1) opening 2) closing
principles of insurance. 3) dedicated 4) inaugurate
1. 1) paving 2) corresponding 5) induction
3) following 4) emphasize 3. 1) reveal 2) know
5) correcting 3) aware 4) inform
2. 1) process 2) product 5) acquaint
3) notes 4) currency 4. 1) excellent 2) available
5) procedures 3) inauspicious 4) appropriate
3. 1) slightly 2) upon 5) obvious
3) still 4) at 5. 1) invited 2) assembled
5) until 3) done 4) shifted
5) held
Cloze Test 447

6. 1) present 2) accompany 6. 1) requests 2) bring


3) attend 4) involve 3) emphasises 4) speculates
5) entertain 5) postulates
7. 1) apologised 2) told 7. 1) Sufficient 2) Good
3) explained 4) denied 3) Competent 4) Absence
5) refused 5) Inadequate
8. 1) difference 2) sense 8. 1) grow 2) multiplication
3) difficulty 4) meaning 3) expansion 4) rise
5) point 5) inflation
9. 1) where 2) while 9. 1) reported 2) produced
3) that 4) however 3) develop 4) composed
5) which 5) resulted
10. 1) obey 2) achieve 10. 1) weather 2) if
3) discover 4) built 3) whether 4) unless
5) perform 5) provided
Induction = the action or process of admitting At the expense = with loss or damage to
somebody or of being admitted to of something something
an office or organization Believe = to feel sure of the truth of
Reveal = to make facts etc known something
Acquaint = to make somebody/oneself familiar Consider = to t hink about somebody/
with or aware of something something especially in order

K
to make a decision
Passage 64 Speculate = to guess; to form opinions
without having definite or
Twenty years (1) now, nearly 60% of the world’s complet e knowledge or
population will live in urban areas. The impact of evidence
urbanization might not all be positive on India as Vicinity = the area round a place
urban expansion is happening at a much (2) rate than
infrastructure expansion. Passage 65
Sustainability issue s ne e d to be (3) so that Today, it is (1) recognized that the 21st century
economic development is not at the (4) of public will be driven by knowledge. To (2) the challenges of
health. Some urban services that ought to be in (5) in this century, India needs to usher in a knowledge
a city like water, electricity, transport etc need special revolution that (3) to bring about systemic changes in

KUNDAN
consideration. education.
TERI has put together a detailed report that (6) While our economy has made significant strides,
Sustainability in the provision of basic urban services the education system has not kept (4) with the
in Indian cities. aspirations of the youth. The vast disparity in the
(7) public transport is a major reason for the country today is a result of skewed (5) to knowledge.
proliferation of private vehicles on the road. Respiratory To address this, we need a substantial expansion in
illness in children living in urban areas is on the (8) educational opportunities, with a spe cial (6) on
with more cases of Asthma being (9) because of inclusion of the underprivileged.
pollution. The future of cities of Indian dreams At the bottom of the pyramid, steps must be taken
depends on (10) we can build better cities today. to ( 7) acce ss to quality e ducation. While the
1. 1) on 2) till government has taken steps to ensure education to
3) since 4) from all, where it lacks in its e fforts in the quality
5) after perspective. Being a spirally upward drive, education
2. 1) quick 2) faster can not be (8) to improve at the higher level unless it
3) slower 4) changed improves at the very grassroots level. The top of the
5) speed pyramid, ie higher education, is also uneven. Students
3. 1) understand 2) speculated struggle to compete in the exams which (9) a sound
3) believed 4) imagined knowledge of English. While candidates are expected
5) considered to travel several kilometres to reach school to obtain
4. 1) expense 2) payment any education, the higher education organisations
3) rate 4) costs often (10) candidates from vernacular media through
5) charge State-sponsored exams and proudly affirm them as
5. 1) location 2) abundance ‘unbiased’.
3) large 4) functional 1. 1) thickly 2) widely 3) ample
5) vicinity 4) partly 5) considered
448 Test of English Language

2. 1) adhere to 2) gather better. However, when the economy starts to recover


3) cover 4) contact growth, stocks te nd to recover faster. The re is
5) meet significant disagreement about how health care and
3. 1) sought 2) wanted utilities tend to (6).
3) seeks 4) attempt In 2008, an economic recession was suggested by
5) determined several important indicators of economic downturn.
4. 1) adequate 2) sufficient These (7) high oil prices, which led to (8) high food
3) influence 4) pace prices due to a dependence of food production on
5) ahead petroleum, as well as using food crop products such
5. 1) access 2) approaching as ethanol and biodiesel as an (9) to petroleum; and
3) rights 4) infiltration global inflation; a substantial credit crisis leading to
5) excess the drastic bankruptcy of large and we ll (10)
6. 1) aspiration 2) intensity investment banks as well as commercial banks in
3) important 4) place various, diverse nations around the world; increased
5) emphasis unemployme nt; and signs of conte mporaneous
7. 1) enjoy 2) help economic downturns in major economics of the world,
3) provide 4) diminish a global recession.
5) deepen 1. 1) imagined 2) depict
8. 1) awaited 2) judged 3) shown 4) visualized
3) thought 4) expected 5) characterized

K
5) said 2. 1) increase 2) variance
9. 1) demand 2) has 3) more 4) decrease
3) consume 4) expects 5) abundance
5) wants 3. 1) weakens 2) initiates
10. 1) discourages 2) disobey 3) awakens 4) strengthens
3) contest 4) assume 5) volatile
5) reject 4. 1) maintained 2) yield
To usher in = to mark the start of something; 3) heavy 4) result
to cause something to begin 5) payment
To bring about = to make something happen 5. 1) are 2) want
Stride = one long step; the distance
3) tend 4) yearn
covered by one long step

KUNDAN
Disparity = a difference
5) made
Skewed = not normal or usual; distorted; 6. 1) distribute 2) recover
not straight; crooked 3) wait 4) increased
Spir al = t o increase r apidly and 5) fight
continuously 7. 1) meant 2) show
Vernacular = a language or form of a language 3) numbered 4) included
spoken in particular country or 5) encompass
region or by a particular group
8. 1) fearful 2) dangerous
as compared with a formal or
written language
3) abnormally 4) healthy
Ample = enough or more than enough 5) nutritious
Adhere to = to obey something; to remain 9. 1) alternative 2) variant
attached to something 3) substitute 4) element
Diminish = to become or make something 5) integral
smaller or less; to decrease 10. 1) wealthy 2) costly 3) stand
4) created 5) established
Passage 66 To hold up = to delay or block the movement or
In e conomics, the te rm re ce ssion ge ne rally progress of somebody/something;
t o use or pr esent somebody/
describes the reduction of a country’s Gross Domestic
something as an example
Product (GDP) for at least two quarters. A recession Downturn = a reduction in economic or business
is (1) by rising unemployment, increase in government activity
borrowing, (2) of share and stock prices, and falling Contemporaneous = existing or happening at the
investment. All of these characteristics have effects same time
on people. Some recessions have been anticipated by Variance = the extent to which something
stock market declines. The real-estate market also varies or differs from something else
usually (3) before a recession. However, real-estate Encompass = to include something; to surround
or cover something completely
declines can last much longer than recessions. During
an economic decline, high-(4) stocks such as financial
services, pharmaceuticals and tobacco (5) to hold up
Cloze Test 449

Passage 67 Hail from = to originate from a place


Fetch = to go and find and bring back
Without doubt, there is one thing (1) to all of us. somebody/something
We have played a game at some time in our lives. Alleviate = to make something less severe
Most of us play to relax or have fun, but for many
playing a game or a sport is a way to (2) poverty behind.
Passage 68
In fact, in many African countries, playing a sport The barter system for getting goods and services
professionally can (3) the lives of a person’s entire (1) back many centuries. In most cultures the barter
family. system was used before money was (2). People who
For example, in the small town of Bekoji, in Ethiopia had specific items or services would (3) these with
(4) than a hundred boys and girls can be seen running others for the things they needed. Good negotiation
at dawn everyday. Each of these youth is (5) and was the (4) to making good trades. While the barter
serious and their coach is (6) that one of them will be system (5) based on basic needs, today the barter
a world champion. This seems like an idle (7) but it is syste m continue s to thrive . The barte r syste m
virtually a guarantee in this small community (8) transcends the monetary system. The barter system
mainly farmers. Many of the fastest male and female is making a (6) today. What makes the barter system
distance runners in the world hail from this small even better today than ever before is that it can now
town. A small handpainted sign which greets visitors be done globally. In the past, bartering was simply
outside Bekoji (9) “Welcome to the Village of Athletes”. done with those that were located nearby. Today, the
Children here start running at an early age, (10) great barter system can be used in a much more (7) way

K
distances to fetch water and firewood or to reach than ever before yet it carries with it the same basic
school. At the Olympics, runners from this small town motivation - the need for something that you don’t
are likely to win more medals than those from have and the excess of something that someone else
developed countries. It will give their families a way wants.
out of poverty. The barter system is enjoying (8) interest today.
1. 1) accepted 2) common Bartering allows you to get the things you need without
3) alike 4) similar having to (9) additional money. Instead, you can use
5) popular the things you no longer need or want to get the things
2. 1) alleviate 2) forgot you do need. There are swap markets and online
3) prevent 4) reduce auctions that (10) you to sell or trade your items or to
5) leave purchase items that you want. Negotiation takes place
3. 1) changes 2) arrange just like it did hundreds of years ago.

KUNDAN
3) control 4) transform 1. 1) discovered 2) dates
5) shift 3) began 4) started
4. 1) further 2) more 5) initiated
3) greater 4) over 2. 1) bought 2) imagined
5) larger 3) began 4) emerged
5. 1) concentrated 2) rival 5) invented
3) focused 4) playful 3. 1) buy 2) sell
5) performed 3) exchange 4) give
6. 1) convince 2) optimist 5) return
3) intended 4) privilege 4. 1) important 2) essential
5) confident 3) result 4) key
7. 1) boast 2) suspicion 5) intention
3) risk 4) worship 5. 1) originated 2) stood
5) precaution 3) generated 4) created
8. 1) existing 2) that 5) produced
3) comprising 4) consisting 6. 1) issue 2) comeback
5) for 3) withdrawal 4) recall
9. 1) warn 2) inform 5) fading
3) notices 4) reads 7. 1) primitive 2) appreciated
5) wish 3) promoted 4) sophisticated
10. 1) covering 2) driving 5) better
3) measuring 4) following 8. 1) diminishing 2) revival
5) competing 3) perishing 4) declining
Da wn = the time of day when light first 5) renewed
appears 9. 1) expend 2) exchanging
Bo as t = a statement showing too much pride
3) expand 4) consume
and satisfaction
Virtually = almost
5) cost
450 Test of English Language

10. 1) insist 2) force 6. 1) unless 2) because


3) allow 4) lure 3) against 4) whether
5) constraint 5) that
Barter = to exchange goods, property etc for 7. 1) recommend 2) think
other goods etc without using 3) point 4) refer
money 5) suggest
Thrive = to live, continue, grow or develop
8. 1) respect 2) debt
well and vigorously
Transcend = to be or go beyond the normal limits
3) attention 4) expense
of something 5) compensation
Swap = t o exchange somet hing f or 9. 1) lower 2) attain
something else 3) decline 4) shrunk
Expend = to use or spend resources in doing 5) recover
something 10. 1) difficult 2) interpret
Perish = to be destroyed; to die 3) reveal 4) intended
Cure = to attract or tempt a person or an
5) inferred
animal
To take on = to assume something; to begin to
Constraint = a thing that limits or restricts
something have a particular characteristic,
quality or appearance.
Passage 69 Owing to = because of or on account of
The (1) of India as an economic superpower is not something

K
reflected in the (2) of life enjoyed by its 1.2 billion Enhance = to increase or improve further for
good quality, value or status of
citizens, according to the Human Development Index,
somebody
which (3) India very low among 182 countries. In our Pursuit = the action of looking for or trying to
performance-oriented world, measurement issues find something
have taken on (4) importance as what we measure Compliance = the tendency to agree to do what
affects what we do. In fact, the French President has others want
established an international commission on the Intend = to have a particular purpose or plan
Measurement of Economic Performance and Social in mind
Progress, owing to his (5) and that of others with the
current state of statistical information about the
Passage 70
economy and society. Asteroids are rocks and debris which are the
The big question concerns (6) Gross Domestic leftovers of the construction of our solar system. Most

KUNDAN
Product (GDP) provides a good measure of living are in a belt, which (1) between Mars and Jupiter.
standards. In many cases, GDP statistics seem to (7) However, the gravitational influence of the giant
that the economy is doing far better than most citizens planets, like Jupiter, or an impact by a comet can
fe el it is. More over, the focus on GDP cre ate s knock these large rocks out of their orbit, thus hurling
conflicts—while political leaders are told to maximise them (2) the Earth. Many bodies have struck Earth in
it, citizens also demand that (8), be paid to enhancing the (3), and a widely accepted theory blames the impact
security, reducing air, water and noise pollution all of of an asteroid for the extinction of dinosaurs about
which actually (9) GDP growth. Statistics are (10) to 65 million years ago. The scale of such a disaster can
summarise what is going on in our complex society, it be understood by the example of a relatively small-
is therefore obvious that we can’t reduce everything size asteroid strike in Siberia in early 20th century
to a single number —GDP. which (4) more than half a million acres of forest.
1. 1) pursuit 2) perception However, what relieves the common man of the (5)
3) conversion regarding asteroid impact is the fact that many
4) title scientific groups are dedicated towards tracking the
5) tribute asteroid paths and orbit all around the year. With
2. 1) quality 2) spirit advanced equipment and technology, they can predict
3) span 4) joy any upcoming danger much in (6). According to them
5) loss the chances of finding such an asteroid crossing Earth
3. 1) scored 2) qualified in this or the next five generations’ lifetime is only
3) regard 4) ranked one in thousands. Even if such an asteroid is found
5) counted out, there will be (7) of time to track it, measure its
4. 1) great 2) unduly orbit precisely, and plan a system for (8) it from its
3) trivial 4) considerably orbit away from that of the Earth’s. There will be no
5) negligible great hurry, and no great panic. It would be a project
5. 1) confidence 2) belief for all the world’s nations to take part in. It could be
3) dissatisfaction 4) compliance a globally unifying event. Because it will be (9) long
5) obedience before it actually hits the Earth, it probably would
Cloze Test 451

take only a small measure such as chemical rockets, Passage 71


or perhaps an atomic explosion to divert it from a
threatening path. The World Diabetes Congress has determined that
Thus, in short, it can be said that though the impact India has the largest number of diabetics in the world.
would pose enormous risk to all living forms on Earth, Apart from the loss of productivity, the (1) burden is
the odds of it occurring within our lifetimes is very alarming - $ 2.8 billion annually. Sedentary jobs, (2)
(10) and it is unnecessary to run around believing of electronic entertainment, changing diet patterns
that the sky is falling. and (3) dependence on automobiles have driven the
1. 1) rotates 2) appears activity (4) of Indians’ lives, especially in cities.
3) strikes 4) encircles The (5) is, therefore, to make people physically (6)
5) exists and requires interventions which impact a large (7) of
2. 1) past 2) around the population. Admittedly, physical activity is a (8)
3) towards 4) against of choice and is strongly driven by (9) preferences.
5) inside But policy making needs to shift to (10) moderate
3. 1) future 2) centuries levels of physical activity in the daily lives of people.
3) earliest 4) past One way to accomplish this is to create walkable
5) history communitie s that give re side nts a varie ty of
4. 1) extinct 2) devastated destinations within walking distance.
3) wasted 4) shrivelled 1. 1) health 2) economic
5) fell 3) finance 4) subsidy

K
5. 1) apprehension 2) expectation 5) physical
3) distrust 4) sufferings 2. 1) widespread 2) broadcast
5) hesitation 3) spread 4) prevalent
6. 1) sooner 2) accuracy 5) expand
3) advance 4) time 3. 1) increasing 2) totally
5) distance 3) entirely 4) grown
7. 1) dearth 2) loss 5) mutual
3) most 4) lack 4. 1) outside 2) most
5) plenty 3) out 4) from
8. 1) blocking 2) deflecting 5) through
3) avoiding 4) destroying 5. 1) dispute 2) ultimatum
5) changing 3) hazard 4) sensitivity

KUNDAN
9. 1) experienced 2) harmful 5) challenge
3) perceived 4) noticed 6. 1) qualified 2) equip
5) devastating. 3) built 4) active
10. 1) low 2) large 5) trained
3) narrow 4) high 7. 1) piece 2) section
5) few 3) scale 4) degree
Leftovers = food remaining at the end of a meal; 5) per cent
something that belongs to a past 8. 1) lack 2) want
period and surprisingly still exists 3) matter 4) scarcity
although most other things of that 5) right
period no longer do. 9. 1) individually 2) showing
Hurl = to throw somebody/something
3) given 4) special
violently in a particular direction
Extinct = no longer in existence 5) personal
Apprehension = anxiety about something in the 10. 1) attract 2) pursuit
future; fear that something will be 3) indulge 4) introduce
unpleasant or t hat something 5) insist
unpleasant will happen. Sedentary = done sitting down; spending a
Plenty = a number or an amount that is lot of time sitting down
sufficient for somebody or more than Drive somebody/ = to force somebody/something
they need. something out to leave or disappear
Deflect = to change or make somet hing Intervene = to come or be between
change direction especially after Accomplish = to succeed in doing something;
hitting something to complete somet hing
Devastated = to r uin something; to destr oy successf ully; t o achiev e
something completely. something
Shrivel = to shrink or wrinkle from heat or cold Prevalent = exist ing or happening
or because of being dry generally; widespread
Dearth = a lack or shortage of things or people
452 Test of English Language

Passage 72 Ambiguous = not clearly stated or defined


Revert = to retur n to a former state or
Economic growth figures for the first quarter of this condition
financial year seem to support the claim that the worst Atrocious = very wicked; cruel or shocking
may be over for the Indian economy. The gradual revival Contradict = to say that something a person has
said or written is wrong, and that
is also an indication that the government’s economic
the opposition is true
stimulus package is (1). What could, however, upset Futile = producing no result; having no
the positive outlook is the drought which (2) large purpose
parts of the country and its impact on overall growth.
Even though the monsoon had picked up (3), the rains Passage 73
received were grossly (4). There are clear (5) that farm
There is a considerable amount of research about
output, particularly cereals, will fall drastically.
the factors that make a company innovate. So is it
Insufficient rain is bound to shoot up the (6) of
possible to create an environment (1) to innovation?
agricultural commodities and that would impact the
This is a particularly pertinent (2) for India today.
economy as a whole. The drought would also (7) a
Massive problems in health, education, etc (3) be
drastic re duction in rural e mployme nt and
solved using a conventional approach but (4) creative
consumption besides inflation in the prices of food
and innovative solutions that can ensure radical
articles.
change and (5). There are several factors in India’s
Food prices have been (8) since the past few
(6). Few countries have the rich diversity that India
months, and lower agricultural production is likely to

K
or its large, young population (7). While these (8)
(9) the situation. The government has said that food
innovation policy interventions, certain additional
grain from the buffer stocks will be used to keep prices
steps are also required. These include (9) investment
(10). Subsidised food grain is necessary in these
in research and development by (10) the government
times, but its effectiveness will depend a lot on the
and the private sector, easy transfer of technology
distribution system.
from the academic world etc. To fulfil its promise of
1. 1) impractical 2) ambiguous
being prosperous and to be at the forefront, India must
3) failing 4) working
be innovative.
5) weakening
1. 1) stimuli 2) conducive
2. 1) strike 2) affected
3) incentive 4) facilitated
3) exposed 4) reverted
5) impetus
5) altered
2. 1) objective 2) controversy
3. 1) unseasonably 2) unfavourably

KUNDAN
3) doubt 4) question
3) presently 4) meagrely
5) inference
5) later
3. 1) cannot 2) possibly
4. 1) inadequate 2) enough
3) should 4) never
3) missing 4) ample
5) must
5) atrocious
4. 1) necessary 2) apply
5. 1) contradictions 2) advices
3) need 4) consider
3) reasons 4) results
5) requires
5) indications
5. 1) quantity 2) advantages
6. 1) production 2) requirement
3) increase 4) chaos
3) prices 4) yield
5) growth
5) labour
6. 1) challenges 2) praises
7. 1) trigger 2) lead
3) favour 4) leverage
3) result 4) contribute
5) esteem
5) dampen
7. 1) blessed 2) enjoys
8. 1) improving 2) balanced
3) endows 4) prevails 5) occurs
3) stable 4) increasing
8. 1) aid 2) jeopardise
5) decreasing
3) promotes 4) endure
9. 1) aggravate 2) amend
5) cater
3) smoothen 4) improve
9. 1) acute 2) utilising
5) challenge
3) restricting 4) inspiring
10. 1) unprofitable 2) futile
5) increased
3) maximum 4) growing
10. 1) both 2) besides
5) down
Stimulus = a thing that encourages or excites
3) combining 4) participating
somebody/something to activity, 5) also
greater effort etc Conducive = helping something to happen or
Aggravate = to make a disease, a situation, an making it likely
offence etc worse or more serious Pertinent = relevant to something
Cloze Test 453

Radical = fundamental 4. 1) unexplained 2) doubt


Foref ront = the most forward or important 3) some 4) true
position or place 5) sad
Stimuli = plural of ‘stimulus’.
5. 1) himself 2) sometimes
Facilitate = to make something especially an
action or a process easy or easier
3) proper 4) improve
Impetus = a force that encourages a process to 5) themselves
develop more quickly 6. 1) established 2) created
Leverage = power or influence 3) set 4) wound
Esteem = high regard; a favourable opinion 5) thought
Endow = to provide somebody/something 7. 1) tried 2) mattered
with a good quality, ability, feature 3) meaning 4) supposed
etc.
5) expect
Jeopardise = to cause something to be harmed,
lost or destroyed; to put something
8. 1) renounced 2) showed
in danger of this happening 3) passed 4) negated
Endure = to tolerate a person, an event etc 5) directed
9. 1) shift 2) make
Passage 74 3) turn 4) mull
5) switch
Seed quality is an (1) aspect of crop production.
10. 1) sell 2) equipments
For ages, farmers have traditionally been selecting
3) people 4) techniques
and (2) good quality seed, since it was in their interest

K
5) creatures
to do so. They knew and understood the importance
Advent = the approach or arrival of an
of quality seed in production. important person, event etc
However, with the adve nt of gre en revolution Set something up = to place or build something; to
technology, based (3) on the high-yielding dwarf establish or create something
varieties of wheat and rice, mainstream thinking Staple = main or principal
changed. Agricultural scientists, for reasons that Soar = to rise quickly to a high level or
remain (4), began to doubt the ability of farmers to standard
maintain seed quality (5). Aided by the World Bank, Empathy = the ability to imagine and share
another person’s experience,
the Ministry of Agriculture launched a National Seeds
feelings etc
Project in 1967. Under the project, spread into three Renounce = to give up a habit; abandon
phases, seed processing plants were (6) up in nine something

KUNDAN
states. Six states were covered under phase three. Mull over = to think about or consider
All that the huge processing plants were (7) to do was something long and carefully
to provide ‘certified’ seeds of food crops, mainly self-
pollinating crops, to farmers. In mid-1980s, the Passage 75
International Rice Research Institute (IRRI) in the The world’s climate has always changed and species
Philippines concluded a study which (8) that there have evolved accordingly to survive it. The surprising
was hardly any difference in the crop yields from fact about the (1) between evolution and global
transplante d rice and from the crop sown by warming (2) that it is not linear. (3) temperatures alone
broadcasted seeds. One would wonder why, in the first are not (4) of evolution. Evolution is also the (5) of
instance, were the farmers asked to (9) ove r to seasonal changes. As the environment (6) those
transplanting paddy ? The answe r is simple — species which don’t adapt (7) to exist. But the sheer
probably, to help the mechanical industries grow. (8) of manmade climate change today is (9). ‘Bad things
Since rice is the! staple food in Asia, tractor sales could are happening’ and by one (10) global warming could
only grow if there was a way to move the machine in threaten upto one-third of the world’s species if left
the rice fields. No wonder, the sales of tractors, unchecked. In fact, a lot of the species which will be
puddlers, reapers and other associated (10) soared in able to survive are the ones we consider pests like
rice-growing areas. insects and weeds.
1. 1) irrational 2) main 1. 1) difference 2) similarity
3) brilliant 4) important 3) argument 4) relationship
5) empathetic 5) alliance
2. 1) maintaining 2) trusting 2. 1) being 2) seems
3) selling 4) processing 3) mainly 4) besides
5) creating 5) is
3. 1) necessarily 2) exceptionally 3. 1) However 2) Mounted
3) primarily 4) regularly 3) Rising 4) Elevating
5) truly 5) Inclining
454 Test of English Language

4. 1) means 2) triggers a conscious effort to (10) the right policies in place


3) responses 4) threats soon.
5) stimulus 1. 1) past 2) against
5. 1) result 2) precursor 3) through 4) across
3) resistance 4) cause 5) on
5) provocation 2. 1) earning 2) share
6. 1) conserves 2) stifles 3) venture 4) delivery
3) predicts 4) changes 5) distribution
5) emerges 3. 1) commonly 2) ideally
7. 1) continue 2) halt 3) indefinitely 4) preferably
3) cease 4) terminate 5) invariably
5) discontinue 4. 1) whereas 2) unlike
8. 1) luck 2) value 3) besides 4) although
3) collapse 4) pace 5) despite
5) attention 5. 1) encouraging 2) second
9. 1) threatened 2) pursued 3) lesser 4) beating
3) unprecedented 4) record 5) greater
5) debated 6. 1) affords 2) cures
10. 1) forecast 2) chance 3) visits 4) reaches
3) pattern 4) occasion 5) provides

K
5) imagination 7. 1) look 2) plan
Ad apt = to make something suitable for 3) weigh 4) admire
a new use, situation etc 5) consider
Cease = to come to or bring something 8. 1) persuade 2) ascertain
to an end; to stop
3) influence 4) impede
Sheer = complete; nothing more than
Unprecedented = never having happened; been
5) estimate
done or been known before 9. 1) thought 2) credited
Weed = a wild plant growing where it is 3) identified 4) believed
not wanted especially among 5) supposed
crops or garden plants 10. 1) derive 2) frame
Precursor = a person or thing that comes 3) figure 4) consider
before somebody/something

KUNDAN
5) put
more important, larger or more Poise = to be or keep something balanced
highly developed. or suspended
Stifle = t o suppr ess or contr ol Toil = work that is hard and makes one
something; to feel or make very tired
somebody f eel unable t o Invariably = always
breathe properly because of Impede = to delay or stop the progress or
lack of fresh air. movement of something/somebody
Passage 76 Passage 77
The world is going (1) a deep recession. At such a The large number of natural disasters within a few
time, one thing we need in abundance is jobs for the days in late September has led to two assumptions.
semi-skilled and unskilled. This is the only way in First, we are experiencing more natural calamities
which e qual (2) of we alth can take place . The today (1) ever before, and second, the distribution of
healthcare industry is (3) poised to occupy this disasters (2) unequal. A UN report studied natural
position. The IT industry hires people from the upper- disasters (3) 1975 and 2007 found that not only is the
middle strata and rich families, usually engineers, (4) of catastrophes increasing because of climate
(4) the health care industry hires nurses, to the tune change and environmental (5) but also that the brunt
of eighty per cent of the jobs created, from the lower of tragedies is borne (6) poor countries least equipped
economic strata. to deal with such (7). It is true that some countries
Global health care is a $ 4.5-trillion industry, (5) are disaster-prone but some (8) Japan for example
only to the agro industry. Even then health care (6) have manage d to ove rcome the ir ge ographical
only eight per cent of world’s population. Policymakers disadvantage s. (9) to UN e stimate s, equivalent
should (7) at health care industry as not only an populations in the Philippines and Japan (10) the
industry which addresses pain but also as one which same number of cyclones each year but 17 times more
can (8) the economy. The last century was driven by people perish in the Philippines than in Japan. In
machines that addressed human toil and it is strongly same ways natural disaste rs give de ve lope d
(9) that this century will be driven by health care. economies an excuse for technological improvement
This, however, will only happen if policymakers make
Cloze Test 455

while in poorer ones it feeds a vicious cycle —since illegal fishing and the clearing of land are direct results
they are constantly struggling to recover from natural of urbanization and deforestation. People have (4) and
calamities they cannot afford the disaster prevention damaged almost half of earth’s land, at a very
measures needed. unsustainable rate.
1. 1) as 2) than Global warming is having a serious impact as well.
3) not 4) of A six-degree Celsius increase in global temperature
5) since killed 95% of all species on Earth 251 million years
2. 1) being 2) are ago. An increase of six-degree Celsius is forecast this
3) often 4) is century if a change is not made to (5) the damage
5) seem done to earth. Humans will be one of the 95% of
3. 1) after 2) prior species lost. Noticeable, changes of global warming
3) between 4) separating include migration (6) and the change in se ason
5) affecting timings. Migrating birds are migrating earlier which
4. 1) response 2) dances in turn is causing them to hatch eggs and (7) young
3) occurring 4) damage earlier than they did at the beginning of this century.
5) frequency While this is just the tip of the iceberg many other (8)
5. 1) degradation 2) protection regarding the extinction of plant and animal species
3) detriment 4) audit need addressing. It is more important now than ever
5) summit before to pull our heads out of the sand and make
6. 1) of 2) by changes for the (9) of the earth. Future generations

K
3) with 4) for are (10), as they are a species as well.
5) on 1. 1) killing 2) alive
7. 1) calm 2) misbelieve 3) born 4) left
3) misfortunes 4) faith 5) lost
5) mistake 2. 1) speak 2) told
8. 1) inspite 2) even 3) estimation 4) believe
3) since 4) how 5) consider
5) like 3. 1) shape 2) development
9. 1) Thanks 2) Comparing 3) deterioration 4) warmth
3) Similar 4) According 5) expansion
5) Linked 4. 1) altered 2) created
10. 1) endure 2) incite 3) produced 4) made

KUNDAN
3) enjoys 4) trigger 5) brought
5) encountersor 5. 1) void 2) dissipate
Calamity = an event that causes great harm 3) augment 4) reverse
or damage; a disaster 5) increase
Catastrophe = a sudden great disaster 6. 1) delay 2) birds
Bear the brunt = t o r eceiv e t he mainf orce,
3) slowdown 4) hasten
of something sock or impact of something
Endure = to tolerate an event
5) acceleration
Perish = to be destroyed; to die 7. 1) spare 2) bear
Detriment = causing harm to somebody/ 3) destroy 4) amend
something 5) generation
Incite = to urge or persuade somebody 8. 1) animals 2) difficulty
to do something by making 3) issues 4) humans
them very angry or excited; to 5) problem
create or cause something
9. 1) extinction 2) better
especially conflict or violence
3) wealth 4) stigma
Passage 78 5) demand
10. 1) endangered 2) threaten
Hundreds of plants and animals are (1) every day 3) evaluated 4) living
due to deforestation and urbanization. What might 5) compared
happen if this continues in the future? The last mass Extinct = no longer in existence
extinction of plant and animal species occurred 65 Deterioration = the action of becoming worse
million years ago with the dinosaurs. In all, five mass in quality or condition
extinctions have occurred and scientists (2) earth is Contaminate = to make something/somebody
in the sixth mass extinction. The world as it is now is impure by adding substances
that are dangerous or carry
threatened, including people, who are responsible for
disease
e arth’s ( 3). Pe sticide s contaminating wate r; Hatch = to come out of an egg.
overharvesting of animals and plants; air pollution; B ea r = to give birth to somebody
456 Test of English Language

Endanger = to cause danger to somebody/ Catastrophe = a sudden great disaster


something; to put somebody/ Void = empty; without something;
something in danger. lacking something
Hide one’s head = to pretend that an obvious A figment of some- = a thing that is not real
in the sand problem or danger does not body’s imagination but only imagined
exist
Passage 80
Passage 79
The world is witnessing food price turbulence again.
India has become, in purchasing power parity A bad drought in Russia, rising demand in the US and
terms, the fourth largest economy in the world. India’s developing countries, and Pakistan’s blighted crop
economic (1) since 1980 has been among the (2) rapid. prospe cts after its floods are ke eping price s of
Although India managed its one incipient crisis in commodities such as cereals, sugar, oil and meat high.
the early 1990s, it avoided the catastrophic losses The Food and Agriculture Organization’s monthly food
(3). While many (4) that exposing India’s economy to price (1) is heading north.
global competition would reveal India’s economic (5) India is not (2) from this problem even at the best
it has rather revealed strengths and often unexpected of times. For the week that ended on 11 September,
strengths in new areas no planner would have (6) of. food prices (as (3) by the Wholesale Price Index) rose
India is increasingly taking its (7) on the global stage by 15.86%.
and in inte rnational forums as a 21st-ce ntury Given the robust demand for foodstuffs, a time of
superpower. The generations of politicians and policy price volatility calls for a careful look at the “design”

K
makers who have been (8) of leading India to where it issues surrounding food supply management. At times,
is today can be justifiably (9) of the transformation. even huge food stocks are not able to (4) rising food
But achievements create new (10) , two of which are prices. The fault lies in how food is released to traders
improving service delivery, particularly to the poor, by government agencies such as the Food Corporation
through greater accountability and expanding the of India (FCI). This problem is apart from FCI’s high
benefits of rapid growth—across sectors, regions, and carrying cost of foodgrains. But this is not the problem
people. at (5).
1. 1) decline 2) policy For example, under the open market sales scheme
3) crisis 4) treaty (OMSS) a fixed quantity of grain, usually in multiples
5) growth of 10 metric tonnes, is sold to traders, flour mills
2. 1) several 2) very and other buyers when supplies are (6) or there is
3) most 4) much price volatility. But a combination of price rigidity,

KUNDAN
5) so terms of sale and the quantity sold under OMSS
3. 1) end 2) deterioration defeats its purpose. One reason for this is the large
3) thus 4) together volume in the hands of very few individual buyers.
5) elsewhere This (7) to perverse economic incentives.
4. 1) asked 2) feared Often, the grain sold under this scheme winds up
3) think 4) spoke back with food (8) age ncie s be cause of price
5) believe differentials (the price at which it is sold and the
5. 1) growth 2) space prevailing market price). This has been observed many
3) gain 4) weakness times in states as diverse as Punjab and Uttar
5) void Pradesh. If the number of buyers is (9) and the
6. 1) questioned 2) dreamed quantity sold to each buyer reduced, or the price fixed
3) arranged 4) plan but the amount of grain that can be bought kept
5) wanted flexible, these problems can be (10).
7. 1) matter 2) place This makes for a sensible menu of options. But it
3) life 4) generation needs careful implementation. And if, for some reason,
5) mark changes are required to suit (11) conditions in different
8. 1) part 2) issues states, the economic logic behind these ideas should
3) humans 4) figment not be lost (12) of.
5) thought 1. 1) index 2) state
9. 1) worried 2) angry 3) scheme 4) rate
3) honoured 4) distinguished 5) value
5) proud 2. 1) affected 2) above
10. 1) impossibilities 2) evaluations 3) immune 4) away
3) challenges 4) comparison 5) separate
5) dangers 3. 1) developed 2) increased
Incipient = in its early stages; beginning 3) reported 4) measured
to happen 5) handled
Cloze Test 457

4. 1) arrest 2) identify Turbulence = disturbance, conf usion or


3) find 4) slow conflict
5) stop Blight = to affect the plant with a
disease; to spoil or ruin
5. 1) this 2) juncture
something
3) all 4) best Immune = not affected or influenced by
5) hand something
6. 1) nil 2) short Robust = strong and able to survive
3) plenty 4) enough rough treatment
5) least Call for = to require, demand or need
7. 1) rises 2) leads something
3) gives 4) is Volatile = likely to change suddenly or
sharply; not stable
5) jumps
At hand = near in place or time
8. 1) hoarding 2) storing Perverse = showing a deliber at e and
3) supply 4) producing stubborn desire to behave in a
5) procurement way t hat is wr ong,
9. 1) controlled 2) promoted unreasonable or unacceptable
3) constant 4) increased Wind up = to end up; to arrive finally in a
5) decreased place
10. 1) neglected 2) solve Overcome = to succeed in dealing with or
controlling something
3) overcome 4) indicated
To lose sight = to f ail to consider

K
5) highlighted of somebody/ something; to forget
11. 1) good 2) local something something
3) all 4) similar At this Juncture = at a par t icular especially
5) bad important, stage in a series of
12. 1) weight 2) look events
3) value 4) sight Procurement = t he process of obtaining
5) significant something

Answers

KUNDAN
Passage 1 Passage 8
1. 5 2. 4 3. 2 4. 2 5. 3 1. 5 2. 4 3. 1 4. 3 5. 3
6. 3 7. 4 8. 1 9. 5 10. 2 6. 5 7. 2 8. 1 9. 2 10. 2
Passage 2 Passage 9
1. 3 2. 4 3. 1 4. 5 5. 2 1. 2 2. 1 3. 3 4. 3 5. 3
6. 4 7. 1 8. 2 9. 5 10. 3 6. 2 7. 2 8. 4 9. 3 10. 5
Passage 3 11. 4 12. 5 13. 5 14. 1 15. 3
1. 2 2. 1 3. 5 4. 3 5. 4 Passage 10
6. 3 7. 4 8. 1 9. 5 10. 3 1. 5 2. 5 3. 1 4. 4 5. 4
Passage 4 6. 3 7. 5 8. 2 9. 4 10. 1
11. 4
1. 2 2. 5 3. 1 4. 5 5. 3
6. 5 7. 2 8. 1 9. 4 10. 2 Passage 11
Passage 5 1. 4 2. 1 3. 3 4. 2 5. 5
6. 4 7. 1 8. 3 9. 2 10. 5
1. 1 2. 3 3. 2 4. 2 5. 4
6. 5 7. 3 8. 5 9. 4 10. 2 Passage 12
Passage 6 1. 1 2. 4 3. 2 4. 5 5. 3
6. 5 7. 4 8. 2 9. 1 10. 3
1. 4 2. 5 3. 3 4. 2 5. 4
6. 4 7. 5 8. 2 9. 1 10. 2 Passage 13
Passage 7 1. 4 2. 1 3. 3 4. 5 5. 2
6. 3 7. 4 8. 1 9. 5 10. 2
1. 3 2. 4 3. 5 4. 1 5. 2
6. 4 7. 1 8. 3 9. 2 10. 5 Passage 14
1. 5 2. 2 3. 5 4. 1 5. 3
6. 5 7. 3 8. 2 9. 4 10. 1
458 Test of English Language

Passage 15 Passage 30
1. 3 2. 1 3. 5 4. 2 5. 3 1. 4 2. 5 3. 3 4. 2 5. 4
6. 1 7. 3 8. 5 9. 2 10. 2 6. 3 7. 1 8. 4 9. 2 10. 2
11. 3 12. 1 13. 5 Passage 31
Passage 16 1. 1 2. 3 3. 3 4. 2 5. 2
1. 4 2. 2 3. 3 4. 5 5. 3 6. 4 7. 2 8. 1 9. 5 10. 5
6. 2 7. 1 8. 4 9. 5 10. 1 Passage 32
11. 4 12. 3 13. 2 14. 5 15. 1
1. 4 2. 2 3. 1 4. 2 5. 3
Passage 17 6. 5 7. 4 8. 3 9. 5 10. 1
1. 5 2. 2 3. 4 4. 1 5. 3 Passage 33
6. 1 7. 3 8. 5 9. 2 10. 4
1. 4 2. 1 3. 5 4. 3 5. 5
Passage 18 6. 4 7. 2 8. 1 9. 3 10. 5
1. 5 2. 2 3. 3 4. 4 5. 2 Passage 34
6. 5 7. 3 8. 1 9. 4 10. 2
1. 3 2. 5 3. 2 4. 1 5. 5
Passage 19 6. 3 7. 5 8. 2 9. 4 10. 1
1. 5 2. 3 3. 2 4. 4 5. 1 Passage 35
6. 3 7. 4 8. 2 9. 1 10. 5
1. 5 2. 3 3. 1 4. 4 5. 2

K
Passage 20 6. 3 7. 5 8. 4 9. 1 10. 2
1. 5 2. 1 3. 3 4. 4 5. 2 Passage 36
6. 4 7. 2 8. 1 9. 3 10. 1
1. 2 2. 5 3. 4 4. 1 5. 3
Passage 21 6. 4 7. 2 8. 5 9. 4 10. 1
1. 1 2. 5 3. 3 4. 4 5. 2 11. 3 12. 3 13. 2 14. 1 15. 3
6. 3 7. 4 8. 4 9. 2 10. 5 Passage 37
Passage 22 1. 3 2. 5 3. 1 4. 4 5. 5
1. 3 2. 4 3. 1 4. 2 5. 5 6. 2 7. 1 8. 3 9. 1 10. 4
6. 1 7. 4 8. 3 9. 2 10. 5 11. 3 12. 2 13. 5 14. 1 15. 4
Passage 23 Passage 38

KUNDAN
1. 1 2. 3 3. 5 4. 4 5. 2 1. 1 2. 2 3. 5 4. 4 5. 2
6. 3 7. 5 8. 4 9. 1 10. 2 6. 5 7. 3 8. 1 9. 3 10. 1
Passage 24 Passage 39
1. 4 2. 2 3. 5 4. 3 5. 1 1. 2 2. 3 3. 4 4. 3 5. 2
6. 1 7. 3 8. 5 9. 1 10. 2 6. 4 7. 5 8. 1 9. 5 10. 2
11. 3 12. 4 13. 5 11. 1 12. 4 13. 5 14. 3 15. 1
Passage 25 Passage 40
1. 5 2. 1 3. 4 4. 1 5. 3 1. 2 2. 4 3. 1 4. 5 5. 3
6. 3 7. 1 8. 3 9. 5 10. 2 6. 1 7. 5 8. 1 9. 4 10. 3
Passage 26 Passage 41
1. 3 2. 1 3. 2 4. 4 5. 1 1. 3 2. 1 3. 5 4. 2 5. 2
6. 1 7. 5 8. 4 9. 1 10. 2 6. 5 7. 4 8. 1 9. 5 10. 4
Passage 27 Passage 42
1. 3 2. 2 3. 5 4. 1 5. 4 1. 1 2. 4 3. 2 4. 3 5. 2
6. 2 7. 4 8. 4 9. 1 10. 3 6. 4 7. 2 8. 3 9. 1 10. 5
Passage 28 11. 4 12. 2 13. 5 14. 1 15. 4
1. 2 2. 1 3. 1 4. 5 5. 3 Passage 43
6. 3 7. 5 8. 4 9. 2 10. 1 1. 5 2. 2 3. 4 4. 3 5. 1
Passage 29 6. 2 7. 5 8. 1 9. 4 10. 4
1. 2 2. 3 3. 5 4. 4 5. 2 Passage 44
6. 1 7. 5 8. 2 9. 4 10. 3 1. 1 2. 3 3. 4 4. 2 5. 5
6. 3 7. 4 8. 1 9. 5 10. 2
Cloze Test 459

Passage 45 Passage 61
1. 2 2. 4 3. 1 4. 5 5. 3 1. 3 2. 2 3. 5 4. 4 5. 1
6. 4 7. 5 8. 1 9. 3 10. 2 6. 4 7. 5 8. 3 9. 2 10. 1
Passage 46 Passage 62
1. 4 2. 1 3. 3 4. 3 5. 5 1. 2 2. 5 3. 3 4. 4 5. 2
6. 4 7. 2 8. 1 9. 2 10. 5 6. 1 7. 5 8. 1 9. 2 10. 4
Passage 47 Passage 63
1. 1 2. 4 3. 2 4. 5 5. 3 1. 5 2. 1 3. 2 4. 4 5. 5
6. 1 7. 4 8. 5 9. 2 10. 3 6. 3 7. 3 8. 2 9. 1 10. 2
Passage 48 Passage 64
1. 1 2. 2 3. 3 4. 5 5. 1 1. 4 2. 2 3. 3 4. 1 5. 2
6. 5 7. 1 8. 2 9. 4 10. 2 6. 3 7. 5 8. 4 9. 1 10. 3
Passage 49 Passage 65
1. 3 2. 1 3. 5 4. 5 5. 4 1. 2 2. 5 3. 3 4. 4 5. 1
6. 1 7. 2 8. 5 9. 1 10. 3 6. 5 7. 3 8. 4 9. 1 10. 5
Passage 50 Passage 66
1. 1 2. 2 3. 2 4. 4 5. 3 1. 5 2. 4 3. 1 4. 2 5. 3

K
6. 1 7. 3 8. 1 9. 4 10. 2 6. 2 7. 4 8. 3 9. 1 10. 5
Passage 51 Passage 67
1. 5 2. 4 3. 2 4. 4 5. 5 1. 2 2. 5 3. 4 4. 2 5. 3
6. 3 7. 1 8. 3 9. 5 10. 4 6. 5 7. 1 8. 3 9. 4 10. 1
Passage 52 Passage 68
1. 2 2. 4 3. 1 4. 3 5. 5 1. 2 2. 5 3. 3 4. 4 5. 1
6. 4 7. 2 8. 1 9. 3 10. 5 6. 2 7. 4 8. 5 9. 1 10. 3
Passage 53 Passage 69
1. 5 2. 2 3. 3 4. 5 5. 1 1. 2 2. 1 3. 4 4. 1 5. 3
6. 2 7. 4 8. 5 9. 1 10. 3 6. 4 7. 5 8. 3 9. 1 10. 4

KUNDAN
Passage 54 Passage 70
1. 2 2. 4 3. 1 4. 5 5. 3 1. 5 2. 3 3. 4 4. 2 5. 1
6. 5 7. 2 8. 4 9. 3 10. 1 6. 3 7. 5 8. 2 9. 4 10. 1
Passage 55 Passage 71
1. 4 2. 2 3. 3 4. 5 5. 1 1. 2 2. 3 3. 1 4. 3 5. 5
6. 3 7. 4 8. 1 9. 2 10. 5 6. 4 7. 2 8. 3 9. 5 10. 4
Passage 56 Passage 72
1. 2 2. 4 3. 1 4. 5 5. 3 1. 4 2. 2 3. 5 4. 1 5. 5
6. 1 7. 3 8. 2 9. 5 10. 3 6. 3 7. 1 8. 4 9. 1 10. 5
Passage 57 Passage 73
1. 4 2. 1 3. 2 4. 3 5. 3 1. 2 2. 4 3. 1 4. 3 5. 5
6. 4 7. 1 8. 5 9. 4 10. 2 6. 3 7. 2 8. 1 9. 5 10. 1
Passage 58 Passage 74
1. 5 2. 2 3. 1 4. 4 5. 3 1. 4 2. 1 3. 3 4. 1 5. 5
6. 2 7. 5 8. 1 9. 4 10. 3 6. 3 7. 4 8. 2 9. 5 10. 2
Passage 59 Passage 75
1. 2 2. 3 3. 4 4. 1 5. 2 1. 4 2. 5 3. 3 4. 2 5. 1
6. 5 7. 3 8. 4 9. 1 10. 5 6. 4 7. 3 8. 4 9. 3 10. 1
Passage 60 Passage 76
1. 2 2. 4 3. 1 4. 5 5. 3 1. 3 2. 5 3. 2 4. 1 5. 2
6. 2 7. 3 8. 5 9. 1 51. 4 6. 4 7. 1 8. 3 9. 4 10. 5
460 Test of English Language

Passage 77 Passage 79
1. 2 2. 4 3. 3 4. 5 5. 1 1. 5 2. 3 3. 5 4. 2 5. 4
6. 2 7. 3 8. 5 9. 4 10. 1 6. 2 7. 2 8. 1 9. 5 10. 3
Passage 78 Passage 80
1. 5 2. 4 3. 3 4. 1 5. 4 1. 1 2. 3 3. 4 4. 1 5. 5
6. 5 7. 2 8. 3 9. 2 10. 1 6. 2 7. 2 8. 3 9. 3 10. 3
11. 2 12. 4

K
KUNDAN
Error Detection English Language and Comprehension 257

Chapter 22

Error Detection
Introduction Explanation:
We are prone to commit mistakes. It is because of The subjunctive mood is use d in English
our ignorance of the fundamental rules of grammar specifically in two situations: (i) with the expression
and curre nt usage . On occasions e ve n the of a wish and (ii) to express a condition contrary to
knowledgeable, in their weaker moments. It is in fact actual fact. The present subjunctive is conjugated as
slippery spot which demands of us a cautious approach. follows: I were; We were; You were; He were; They
The following are some of the mistakes commonly were.
made in the use of English language. You will do well 5. (a) He is working hard with a view to win this
to study them together with the clear explanations of match. (Incorrect)
how to correct these errors. He is working hard with a view to winning
1. One of my friends are a doctor. (Incorrect) this match. (Correct)
One of my friends is a doctor. (Correct) (b) I look forward to meet my old friend next
month. (Incorrect)
Explanation I look forward to meeting my old friend next
‘One of’ is followed by a plural noun phrase. It month. (Correct)
means ‘one of them’. It takes a singular verb because Explanation:
the subject is ‘one’. The verb that follows the phrase ‘with a view to’ or
2. (a) I don’t know nothing about him. (Incorrect) ‘look forward to’ is to be always in the ‘-ing’ form.

K KUNDAN
I don’t know anything about him. (Correct) with a view to V1 (ing) + ......
(b) I couldn’t find him nowhere. (Incorrect) look forward to + V1 (ing) + ......
I couldn’t find him anywhere. (Correct) 6. (a) He prides on his wealth. (Incorrect)
(c) He doe s not want no ne of that cake . He prides himself on his wealth. (Correct)
(Incorrect) (b) She absented from her class. (Incorrect)
He does not want any of that cake. (Correct) She ab sent ed her self from he r class.
Explanation: (Correct)
The use of two negatives to express a single (c) I availed of this opportunity. (Incorrect)
negative idea is wrong. Two negatives lead to a positive I availed mysel f of this op portunity.
meaning. One negative word should, therefore, be used (Correct)
for the expression of a negative idea. (d) I enjoyed during the holidays. (Incorrect)
3. (a) I always like to closely ex amine e ve ry I enj oyed mysel f during the holidays.
proposal. (Incorrect) (Correct)
I always like to ex amine closely e ve ry Or, I enjoyed the holidays. (Correct)
proposal. (Correct) (e) He resigned to the will of God. (Incorrect)
(b) He plans to hurriedly complete this work. He resigned himself to the will of God.
(Incorrect) (Correct)
He plans to complete this work hurriedly. Explanation:
(Correct) When verbs like absent, apply, acquit, enjoy, over-
Explanation: reach, resign, and pride are used reflectively (that is,
Both the sentences are examples of split infinitive. when the subject of the verb is also the receiver of
The infinitive is the “to” form of the verb, for example, the action, the action is ‘reflected’) a reflexive pronoun
“to sing”, “to dance”, “to finish”. If a word is placed (I—myself; you—yourself; We—ourselves; The y—
between the two words (eg, “to closely examine”), the themselves; He—himself; She—herself; One—oneself)
infinitive is said to be “split”. Such splittings are to is used after it.
be avoided. 7. (a) I cut me shaving this morning. (Incorrect)
4. (a) I wish I was as tall as my father. (Incorrect) I cut myself shaving this morning. (Correct)
I wish I were as tall as my father. (Correct) (b) We got out of the swimming pool and dried
(b) If he was alive he would help me. (Incorrect) us. (Incorrect)
If he were alive he would help me. (Correct) We got out of the swimming pool and dried
ourselves. (Correct)
258 English Language and Comprehension Error Detection

Explanation: Explanation:
When the same person is the subject and the When a pronoun is the object of a verb or a
object, it is necessary to use the reflexive pronouns: preposition it should be in objective case.
myself, yourself, herself, himself, itself, ourselves, 12. (a) He is taller then me. (Incorrect)
themselves, oneself. He is taller than I (am). (Correct)
8. (a) I, you and he are neighbours. (Incorrect) (b) I love you more than him. (Incorrect)
You, he and I are neighbours. (Correct) I love you more than he (loves you). (Correct)
(b) You, they and we must work togethe r. (c) I love you more than he (Incorrect)
(Incorrect) I love you more than (I love) him. (Correct)
We, you and they must work toge ther. Explanation:
(Correct) The case of the pronoun following ‘than’ and ‘as’ is
Explanation: decided by mentally supplying the verb and completing
When first, second and third person singular the sentence.
pronouns (I, You and He) are used together, they are 13. (a) The Climate of Patna is better than Delhi.
placed in this order: Second person (You), third person (Incorrect)
(he) and then first person (I). In the case of plural The Climate of Patna is better than that of
pronouns ‘we’ comes first, then ‘you’ and then ‘they’. Delhi (Correct)
9. (a) I have read Shakespeare’s works who was a (b) The roads of Delhi are wider than Mumbai.
great dramatist (Incorrect) (Incorrect)
I have read the works of shakespeare who The roads of Delhi are wider than those of
was a great dramatist. (Correct) Mumbai. (Correct)
(b) Ravi’s dog who was my frie nd has died. Explanation:
(Incorrect) The objects of comparison are ‘the climate of Patna’
The dog of Ravi, who is my friend, has died. and ‘the climate of Delhi’; ‘the roads of Delhi’ and ‘the
(Correct) roads of Patna’. To avoid the repetition of a noun in a
Explanation: sentence we use ‘that’ for singular noun and ‘those’
Relative pronoun should be placed as close to its for plural noun.

K KUNDAN
antecedent as possible. 14. (a) One of them has already given up one’s
10. (a) Let he do whatever he likes to do. (Incorrect) studies. (Incorrect)
Let him do whatever he likes to do (Correct) One of them has already given up his studies.
(b) Let you and I solve this riddle. (Incorrect) (Correct)
Let you and me solve this riddle. (Correct) (b) One should not waste his time. (Incorrect)
Explanation: One should not waste one’s time. (Correct)
Pronouns following ‘Let’ must be in the objective Explanation:
case, and not in the nominative case. When ‘one’ means ‘one in number’, the pronoun for
Always keep in mind these forms of Personal it is third person singular pronoun (he, she, it). The
Pronouns: possessive formed from them can be his or her or its.
In the first sentence the meaning is one taken out of
them. Hence the possessive should be ‘his’. In the
second sentence ‘One’ is an indefinite pronoun,
meaning ‘anyone’. The possessive of ‘one’ is ‘one’s’.
Hence the use of ‘one’s’ in place of ‘his’.
15. (a) Eithe r the Chief Minister or his Cabinet
colleagues have submitted his resignation.
(Incorrect)
Eithe r the Chief Ministe r or his cabine t
colleagues have submitted their resignation.
When the pronoun is the subject of a sentence, (Correct)
the nominative case is used. When the pronoun is (b) Neither the officer nor the clerks could get
the object of a sentence, the objective case is used. his salary. (Incorrect)
And whe n the pronoun shows posse ssion, the Neither the officer nor the clerks could get
possessive case is used. their salary. (Correct)
11. (a) These books are for you and I. (Incorrect) Explanation:
These books are for you and me. (Correct) Whe n two nouns joine d by ‘Either....or’ or
(b) Between he and I there is an understanding. ‘Neither....nor’ differ in number, the pronoun must
(Incorrect) agree with the plural noun which comes after ‘or’/
Between him and me there is an understand- ‘nor’.
ing. (Correct)
Error Detection English Language and Comprehension 259

16. (a) The mother and the daughter love one (b) He is the most wisest of all. (Incorrect)
another. (Incorrect) He is the wisest of all. (Correct)
The mother and the daughter love each Explanation:
other. (Correct) Double comparatives and double superlatives must
(b) Those thre e boys love each o ther. not be used.
(Incorrect) 21. (a) He is more wiser than brave. (Incorrect)
Those thre e boys love on e an other. He is more wise than brave. (Correct)
(Correct) (b) He is the more intelligent and wiser than
Explanation: his brother. (Incorrect)
‘Each other’ is used in speaking of two persons or He is wiser and more intelligent than his
things, ‘one another’ in speaking of more than two. brother. (Correct)
17. (a) Neither of the three boys came. (Incorrect) Explanation:
None of the three boys came. (Correct) When two adjectives in the comparative or the
(b) None of the two boys came. (Incorrect) superlative degree are used together, the one formed
Neither of the two boys came. (Correct)
by adding ‘more’ or ‘most’ must follow the other
(c) Either of the four boys has done this work.
adjective.
(Incorrect)
22. (a) You are wiser than old. (Incorrect)
Anyone of the four boys has done this work.
You are more wise than old. (Correct)
(Correct)
(b) He is braver than wise. (Incorrect)
(d) Anyone of the two candidates is fit for this
He is more brave than wise. (Correct)
post. (Incorrect)
Either of the two candidates is fit for this Explanation:
post. (Correct) When we compare two qualities in the same
Explanation: person or thing, the comparative ending ‘er’ is not
‘Either’ or ‘Neither’ is used in reference to two only. used. In all such cases we should use ‘more’ before
‘Anyone’ or ‘None’ is used for more than two. the adjective.
18. (a) Each boy and each girl was in their best 23. (a) He is as wise, if not wiser than his brother.
dress. (Incorrect) (Incorrect)
He is as wise as, if not wiser than his

K KUNDAN
Each boy and each girl was in her best dress.
(Correct) brother. (Correct)
(b) Every soldier and every sailor is in their (b) This book is as good, if not better than that
place. (Incorrect) book. (Incorrect)
Every soldier and every sailor is in his This book is as good as, if not better than
place. (Correct) that book. (Correct)
(c) Every night and every day brings their own Explanation:
responsibility. (Incorrect) When two adjectives with differing degrees of
Every night and every day brings its own comparison are used they should be complete in
responsibility. (Correct) themselves. We should complete the first comparison
Explanation: before taking up the second.
When two singular nouns are joined by ‘and’ and 24. (a) It is the best of the two books. (Incorrect)
preceded by ‘each’ or ‘every’, the pronoun is always It is the better of the two books. (Correct)
singular. (b) He is the bett er of the thre e boys.
19. (a) It is not such a good book which I expected. (Incorrect)
(Incorrect) He is the best of the three boys. (Correct)
It is not such a good book as I expected. (c) Which is the bes t; bre ad or butte r?
(Correct) (Incorrect)
(b) This is the same be ggar who came Which is better; bread or butter? (Correct)
yesterday. (Incorrect) (d) Which is better–bread, butter or fruit?
This the same beggar that came yesterday. (Incorrect)
(Correct) Which is the best–bread, butter or fruit?
(c) My problem is the same which yours. (Correct)
(Incorrect)
(e) Out of these two watches this is the best.
My problem is the same as yours. (Correct)
(Incorrect)
Explanation:
Out of these two watches this is better.
The relative pronoun ‘as’ or ‘that’ should be used
(Correct)
after ‘same’ or ‘such’. Never use ‘who’ or ‘which’ after
Explanation:
‘same’ or ‘such’.
20. (a) He is mor e wis er than his brothe r. We should use comparative degree in comparing
(Incorrect) two things or persons and the superlative degree in
He is wiser than his brother. (Correct) comparing more than two things or persons.
260 English Language and Comprehension Error Detection

25. (a) There are no less than twenty boys in this Explanation:
class. (Incorrect) The comparative adjectives, senior, junior, superior,
There are no fewer than twenty boys in this inferior, posterior, anterior, prior are followed by ‘to’
class. (Correct) instead of ‘than’.
(b) He takes no fewer than one kilo of milk. 30. (a) It is the most unique book. (Incorrect)
(Incorrect) It is a unique book. (Correct)
He takes no less than one kilo of milk. (b) It is the most ideal place. (Incorrect)
(Correct) It is an ideal place. (Correct)
Explanation: (c) It is the most perfect answer. (Incorrect)
‘Less’ refers to quantity only, whereas ‘fewer’ It is a perfect answer. (Correct)
de note s numbe r. One is use d in the case of Explanation:
uncountable things and the other in the case of Some adjectives are not compared because they
countable things; as—fewer people, fewer houses, fewer denote meanings which do not admit of variation of
boxes but less milk, less sunshine, less rice. degree or qualities already possessed by them to the
26. (a) It is a ten-miles walk. (Incorrect) utmost possible extent. Such adjectives are: unique,
It is a ten-mile walk. (Correct) ideal, perfect, extreme, chief, complete, round, square,
(b) It is a four-men committee. (Incorrect) universal, impossible, golden, infinite, perpetual.
It is a four-man committee. (Correct) 31. (a) The higher you go, the cool you feel.
(c) It is a two-hours journey. (Incorrect) (Incorrect)
It is a two-hour journey. (Correct) The higher you go the cooler you feel.
Explanation: (Correct)
When expressions of measurement, amount and The older you get, the wise you grow.
quantity are used as adjectives, they are usually (Incorrect)
singular. The noun occurring after the hyphen is always The older you get, the wiser you grow.
singular notwithstanding the fact that the preceding (Correct)
word indicates plurality. Explanation:
27. (a) It took us one and a half hour. (Incorrect) When two changes happen together, that is, there

K KUNDAN
It took us one and a half hours. (Correct) is parallel increase, it is expressed by: the + comparative
(b) This box weighs one and a half pound. degree + the + comparative degree.
(Incorrect) 32. (a) He r house is be tte r than m y on e.
This box weighs one and a half pounds. (Incorrect)
(Correct) Her house is better than mine. (Correct)
(c) It is 1.5 millimetre in length. (Incorrect) (b) His motor car is more expensive than Ravi’s
It is 1.5 millimetres in length. (Correct) one. (Incorrect)
Explanation: His motor car is more expe nsive than
Plural nouns are used with fraction and decimal Ravi’s. (Correct)
over 1. Explanation:
28. (a) Kapil is better than any bowler. (Incorrect) We cannot use ‘one’ or ‘ones’ immediately after a
Kapil is better than any other bowler. genitive or possessive adjective. If these words are
(Correct) preceded by an adjective, however, they can come after
(b) He is better than any student. (Incorrect) a genitive or a possessive adjective. For example,
He is be tter than any other stude nt. Her new house is better than my old one.
(Correct) My old watch, is in better condition than his
Explanation: new one.
Whe n comparative de gre e is use d in the 33. (a) The Victoria Memorial is a worth seeing
superlative sense it is followed by ‘any other’ and not building. (Incorrect)
by ‘any’. The Victoria Memorial is a building worth
29. (a) He is senior than me. (Incorrect) seeing. (Correct)
He is senior to me. (Correct) (b) This is a worth seeing sight. (Incorrect)
(b) I am junior than him. (Incorrect) That is a sight worth seeing. (Correct)
I am junior to him. (Correct) Explanation:
(c) This book is supe rior than that book. A compound adjective is sometimes formed by the
(Incorrect) combination of ‘worth’ with some participle. It is placed
This book is supe rior to that book. after the noun it qualifies.
(Correct) 34. (a) Have they heard the last news? (Incorrect)
(d) That book is infe rior than this book. Have they heard the latest news? (Correct)
(Incorrect) (b) His last nove l is be ing published next
That book is inferior to this book. (Correct) month. (Incorrect)
Error Detection English Language and Comprehension 261

His latest novel is being published next 39. (a) Few politician can be relied on. (Incorrect)
month. (Correct) A f ew p olit icians can be re lie d on.
(c) ‘Edward II’ was Marlowe’s latest play. (Correct)
(Incorrect) (b) Litt le le arning is a d ange rous thing.
‘Edwad II’ was Marlowe ’s last play. (Incorrect)
(Correct) A little learning is a dangerous thing.
Explanation: (Correct)
We use ‘latest’ for things which are new. But last (c) He has few interest in politics. (Incorrect)
means either ‘before this one’ or ‘at the end of a series’. He has little interest in politics. (Correct)
35. (a) I am looking forward to his nearest visit. Explanation:
(Incorrect) We usually use ‘few’ with plural nouns and ‘little’
I am looking forward to his next visit. with uncountable nouns. ‘Little’ means ‘not much/many’.
(Correct) It is rather negative. ‘A little’ is more positive. It
(b) Excuse me . Where ’s the nex t railway means ‘some’.
station? (Incorrect) 40. (a) He is eno ugh bo ld to take up this
Excuse me. Where’s the nearest railway challenge. (Incorrect)
station? (Correct) He is bol d enough to take up this
Explanation: challenge. (Correct)
We usually use ‘next’ for time. It means ‘nearest in (b) He hasn’t got en ough goo d voice .
the future’. It is generally used when we think of things (Incorrect)
coming one after another in a series. ‘Nearest’ is used He hasn’t got a goo d enough voi ce .
for ‘place’. It means ‘most near’ or ‘closest’. (Correct)
36. (a) There are not some books on the table. (c) He is not driving enough fast. (Incorrect)
(Incorrect) He is not driving fast enough. (Correct)
There are not any books on the table. Explanation:
(Correct) ‘Enough’ can qualify an adjective or adverb. It
(b) Has he brought some books? (Incorrect) usually comes after adjectives and adverbs.
Has he brought any books? (Correct) 41. (a) His all books were burnt. (Incorrect)

K KUNDAN
Explanation: All his books were burnt. (Correct)
‘Some’ is usually used in affirmative clauses (b) His both hands are skinny. (Incorrect)
whereas ‘any’ is used in questions and negative. We Both his hands are skinny. (Correct)
can use some in questions if we expect an affirmative (c) Raju’s all hopes were gone. (Incorrect)
answer, or when we want to encourage people to say All Raju’s hopes were gone. (Correct)
‘yes’. For example, Explanation:
Would you like some more potato chips? The noun of the possessive case (Raju’s Sheela’s)
Could I have some ripe mangoes, please? or the pronoun of the possessive case (mine, ours,
37. (a) He is not as tall as his brother. (Incorrect) theirs, his, her etc.) comes just before that noun for
He is not as tall as his brother is. (Correct) which it is used.
(b) She is richer than you are. (Incorrect) 42. (a) This is a best book. (Incorrect)
She is richer than you. (Correct) This is good book. (Correct)
Explanation: (b) He is a worst scholar. (Incorrect)
When ‘than’ or ‘as’ is followed by third person He is a very bad scholar. (Correct)
pronoun, the verb is repeated. But the verb is omitted Explanation:
if ‘than’ or ‘as’ is followed by first and second person. An adjective of superlative degree is used when
38. (a) He is my older brother. (Incorrect) the noun it qualifies shows the possession of a quality
He is my elder brother (Correct) to a higher degree than any other member of the same
(b) She is my oldest sister. (Incorrect) class.
She is my eldest sister. (Correct) 43. (a) He came latter than you. (Incorrect)
(c) He is the eldest man of this place . He came later than you. (Correct)
(Incorrect) (b) If offered red or white shirt I’d choose the
He is the oldest man of this place . later. (Incorrect)
(Correct) If offered red or white shirt, I would choose
Explanation: the latter. (Correct)
The words ‘elder’ and ‘eldest’ are u se d for (c) I will see you latter. (Incorrect)
comparing the members of the family. They are often I will see you later. (Correct)
used before words brother, sister, son, daughter, Explanation:
grandson, granddaughter. ‘Older’ and ‘oldest’ are used ‘Later’ is the comparative of ‘late’. It means ‘more
with regard to age and in connection with human family late in time’, ‘after wards’. But ‘latter’ is the opposite of
relationship. ‘former’. It means ‘the second of two people or things
262 English Language and Comprehension Error Detection

just mentioned’. ‘Later’ denotes time whereas ‘latter’ (b) I like very much skating. (Incorrect)
denotes position. I like skating very much.
44. (a) Have you any farther questions to ask? Or, I very much like skating. (Correct)
(Incorrect) Explanation:
Have you any further questions to ask? We should not put adverbs between the verb and
(Correct) its object. These are not generally separated.
(b) He made no farther remarks. (Incorrect) 49. (a) I yesterday met him. (Incorrect)
He made no further remarks. (Correct) I met him yesterday. (Correct)
(c) Delhi is further from Gaya than Allahabad. (b) The y’re to mor row le aving for Paris.
(Incorrect) (Incorrect)
Delhi is farther from Gaya than Allahabad. Tom orro w the y’re le aving for Paris.
(Correct) (Correct)
(d) Let’s not walk any further. (Incorrect) Explanation:
Let’s not walk any farther. (Correct) Adverbs of definite time are put at the beginning
Explanation: or end of a clause. They do not go in mid-position.
Farther means ‘at or to a great distance or more 50. (a) You well organise d that function.
distant point’. ‘Further’ means ‘more’, ‘additional’. (Incorrect)
45. (a) She sings beautiful. (Incorrect) You organised that function well. (Correct)
She sings beautifully. (Correct) (b) She badly dances. (Incorrect)
(b) This flower smells sweetly. (Incorrect) She dances badly. (Correct)
This flower smells sweet. (Correct) Explanation:
Explanation: When we use an adverb to evaluate, it generally
To give more information about the action - to say goes in end-position, not in mid-position.
how, where or when it is done - we use adverbs with 51. (a) She is very slower than Reena. (Incorrect)
verbs. When the quality of the subject rather than She is much slower than Reena. (Correct)
the action of the verb is to be expressed, we use (b) You are very older than me. (Incorrect)
adjective with a verb. It is the smell of the flower that You are much older than me. (Correct)

K KUNDAN
has been described in the second sentence. Hence (c) She was walking much slowly. (Incorrect)
the use of the adjective ‘sweet’, not the adverb She was walking very slowly. (Correct)
‘sweetly’. Explanation:
46. (a) He hit the ball hardly. (Incorrect) ‘Very’ is used with adjectives and adverbs in the
He hit the ball hard. (Correct) positive degree and with present participle whereas
(b) You have to work hardly. (Incorrect) ‘much’ is used with adjectives and adverbs in the
You have to work hard. (Correct) comparative degree, and with past participle.
(c) He has got hard any money. (Incorrect) 52. (a) It is nothing else than pride. (Incorrect)
He has got hardly any money. (Correct) It is nothing else but pride. (Correct)
(d) He is hardly pressed for time. (Incorrect) (b) Call me anything els e than a thie f.
He is hard pressed for time. (Correct) (Incorrect)
Explanation: Call me anything else but a thief. (Correct)
‘Hard’ is both an adjective and adverb. Its meaning Explanation:
is quite different from ‘hardly’ which is also an adverb. We usually use the adverb ‘but’ (not, than) after
‘Hardly’ means ‘almost no’ or ‘almost not’. The correct ‘else’.
expression is ‘be hard pressed’ which means ‘be under 53. (a) He seldom or ever goes to his village home.
pressure strained’. (Incorrect)
47. (a) The two first pages of this book are torn. He seldom or never goes to his village
(Incorrect) home. (Correct)
The first two pages of this book are torn. (b) He sel dom o r ev er plays c ricke t.
(Correct) (Incorrect)
(b) The two first chapters of this book are good. He seldom if ever (seldom or never) plays
(Incorrect) cricket. (Correct)
The first two chapters of this book are Explanation:
good. (Correct) ‘Ever’ usually means ‘at any time’. It cannot go with
Explanation: ‘seldom’ which means ‘not often, rarely’. The correct
The ‘two first’ is a meaningless expression. It expression is ‘seldom or never’, or ‘seldom if ever’.
implies that two things may be first. It makes no 54. (a) It is no use to ask he r — she is not
sense. The correct expression is ‘the first two’. interested in it. (Incorrect)
48. (a) He speaks well English. (Incorrect) It is no use asking her — she is not
He speaks English well. (Correct) interested in it. (Correct)
Error Detection English Language and Comprehension 263

(b) Is it any use to try to talk to him? Explanation:


(Incorrect) Some verbs are never used in progressive forms.
Is it any use trying to talk to him? Here is the list of some of the most important ‘non-
(Correct) progressive’ verbs:
Explanation: 1. Relational Verbs : appear, belong to,
In expressions like these, ‘use’ is followed by an consist of, contain,
‘-ing’ form. The correct expression is, it or there is no equal, fit, include, owe,
use + -ing form of verb. require, resemble,
55. (a) His mother kept on to encourage him to seem, suffice
study. (Incorrect) 2. Verbs of Emotion : adore, abhor, care,
His mother kept on encouraging him to detest, dislike, hate,
study. (Correct) like, love, wish
(b) Whate ve r happe ns, keep o n t o tr y. 3. Verbs of Perception : hear, see, smell, taste
(Incorrect) 4. Verbs of Possession : have, own, possess
Whate ve r happe ns, keep on t ryin g. 5. Verbs of Cognition : believe, feel, forget,
(Correct) know, mean, mind,
Explanation: realise, recall, recollect,
remember, suppose,
‘Keep on’ is never followed by an infinitive. It is
think, trust,
always followed by the ‘-ing’ form of verb.
understand
56. (a) She knows to sing and dance. (Incorrect)
Note: Some of the verbs noted above are used in
She kn ows h ow to sing and dance .
progressive form in special cases, as for example,
(Correct)
Appear : (be published) When is your next article on
(b) He kno ws t o pre pare Fre nch toast.
his subject appearing?
(Incorrect)
Hear : (receive information) I have been hearing good
He knows how to prepare French toast.
news about him.
(Correct)
(receive a letter) I have been hearing from him
Explanation:

K KUNDAN
fairly regularly. : (try specially in a legal
‘Know’ is never followed directly by an infinitive. sense) The judge has been hearing this case
We generally use the expression ‘know how to’. for the last five years.
57. (a) He has finished to mend the puncture. See : (imagine, have hallucinations) She is seeing
(Incorrect) things; there is nothing there.
He has finished mending the puncture. (meet) I will be seeing the Managing Director
(Correct) tomorrow.
(b) I enjoy to travel. (Incorrect) Smell : (inhale the odour of) He was smelling the
I enjoy travelling. (Correct) mango to find out whether it was fresh or
Explanation: stale.
After some verbs we use an -ing form, and not an Feel : (go forward carefully) The blind beggar is
infinitive. Here is the list of verbs which are followed feeling his way.
by an -ing form. Think : (reflect upon, recall) he was thinking about
avoid forgive miss days long gone by.
consider give up practise : (examine the possibility of) Now I am thinking
delay go put off of leaving this place.
dislike (can’t) help risk 59. (a) I have not and shall not bear this trouble.
enjoy imagine spend money/time (Incorrect)
excuse keep suggest I have not borne and shall not bear this
feel like mind understand trouble. (Correct)
finish (b) I have never and will never do such a thing.
58. (a) Why is she appearing so sad? (Incorrect) (Incorrect)
Why does she appear so sad? (Correct) I have never done and shall never do such
a thing. (Correct)
(b) I am owing a great deal to my parents.
(Incorrect) Explanation:
I owe a great deal to my parents. (Correct) The present form of a verb cannot be used for
both the present perfect tense and the future indefinite
(c) I am not feeling well today. (Incorrect)
tense. The present perfect tense takes the past
I am not well today. (Correct)
participle form of the verb (bear; bore; borne; do; did;
(d) He is ado ring that political le ade r.
done) and the future indefinite takes the present form
(Incorrect) of the verb.
He adores that political leader. (Correct)
264 English Language and Comprehension Error Detection

60. (a) He hanged the lamp on the wall. (Incorrect) (Correct)


He hung the lamp on the wall. (Correct) (b) I awaited for his arrival (Incorrect)
(b) He was hung for murder. (Incorrect) I awaited (waited for) his arrival. (Correct)
He was hanged for murder. (Correct) (c) He has repaid back his loan. (Incorrect)
Explanation: He has repaid (paid back) his loan.
The word ‘hang’ has two different meanings: (i) to (Correct)
kill a person by hanging; (ii) to suspend from or attach Explanation:
loosely to some other object. The two different forms To avoid such silly mistakes, it is well to remember
of the verb ‘hang’ are: that:
V1 V2 V3 return = come back; await = wait for; repay = pay back;
Hang : Hanged : Hanged (for persons) resume = start again
Hang : hung : hung (for things) 63. (a) The boat was dro wned in the rive r.
61. (a) This book costed me half past twelve (Incorrect)
rupees. (Incorrect) The boat was sunk in the river. (Correct)
This book cost me rupees twelve and fifty (b) A boy has be e n sunk in the rive r.
paise. (Correct) (Incorrect)
(b) The he ns have lain no e ggs today. A boy has been drowned in the rive r.
(Incorrect) (Correct)
The hens have laid no eggs today. (Correct) Explanation:
(c) Let me lay on the bed. (Incorrect) A boat capsizes or sinks; a ship sinks; a person is
Let me lie on the bed. (Correct) drowned. ‘To be drowned’ is used only of living things.
Explanation: 64. (a) Who invented America? (Incorrect)
‘Half past twelve’ is a time expression. We say, ‘It Who discovered America? (Correct)
is half past twelve by my watch’. It is never used for (b) Marconi dis covered the wir e le ss.
denoting the price of something. We usually say, (Incorrect)
‘rupees twelve and fifty paise’. The use of the verbs in Marconi invented the wireless. (Correct)
the sentences above is wrong. Explanation:

K KUNDAN
The correct past and perfect forms of certain verbs ‘To invent’ is to make something that did not exist
are as given below: before. ‘To discover’ is to find something that existed
Present Past Perfect before but was unknown.
lie = rest, be down lay lain 65. (a) If I was you I would not have done so.
lay = place, arrange, deposit, (Incorrect)
put down flat laid laid If I were you I would not have done so.
lie = to tell a lie lied lied (Correct)
leave = go away left left (b) He walks as if he is a king. (Incorrect)
live = be alive, be at home lived lived He walks as if he were a king. (Correct)
hang = to put up hung hung Explanation:
hang = to execute the order When we talk about events which are not certain
of death sentence hanged hanged to happen – which we hope will happen, or imagine
flow (water) flowed flowed might happen or want to happen - we use a special
fly (bird) flew flown group of verb-forms called the subjunctive (e.g., I were,
flee = run away (person) fled fled She be, etc.) The subjunctive form ‘were’ is used
bear = put up with bore borne instead of ‘was’ after ‘if’, ‘as if’, and ‘I wish’.
bore = to make a hole; to 66. (a) Neither his action was just nor unjust.
make tired or (Incorrect)
uninterested bored bored His action was neither just nor unjust.
find = to discover found found (Correct)
found = to establish founded founded (b) I neither saw him nor her. (Incorrect)
fall fell fallen I saw neither him nor her. (Correct)
fell = to cut down (a tree); (c) Neither it is good nor it is bad. (Incorrect)
to knock down It is neither good nor bad. (Correct)
(a person) felled felled Explanation:
feel felt felt Neither....nor is used to join together two negative
fill filled filled ideas. This structure is balanced, so that the same
awake (intransitive) awoke awoke kind of words follow neither and nor.
awake (transitive) awaked awaked 67. (a) Ten students have passed and one failed.
62. (a) He has returned back from De lhi. (Incorrect)
(Incorrect) Ten students have passed and one has
He has returned (come back) from Delhi. failed. (Correct)
Error Detection English Language and Comprehension 265

(b) One of the thieves escaped and two caught. 72. (a) She knew that I am coming. (Incorrect)
(Incorrect) She knew that I was coming. (Correct)
One of the thieves escaped and two were (b) He said that he want s to go home .
caught. (Correct) (Incorrect)
Explanation: He said that he wanted to go home .
The auxiliary verb is usually repeated if the voice (Correct)
or number of one principal verb is not the same as Explanation:
the voice or number of the other. If there is a past tense in the principal clause,
68. (a) Tell me where are you going. (Incorrect) the dependent clause must also be in the past tense.
Tell me where you are going. (Correct) 73. (a) Both Raju as well as his brother were
(b) He aske d me wh at was y our nam e. present. (Incorrect)
(Incorrect) Both Raju and his brother were present.
He asked me what my name was. (Correct) (Correct)
(c) Tell me when are you leaving for New York. (b) Tigers are both found is Asia and in Africa.
(Incorrect) (Incorrect)
Tell me when you are leaving for New York. Tigers are found both in Asia and in Africa.
(Correct) (Correct)
Explanation: Explanation:
Indirect questions normally have the word-order The correlative of ‘Both’ is ‘and’ (Both ...and) and
of affirmative sentences. The auxiliary verb is never not ‘as well as’. The same kind of words generally follow
put before the subject. ‘Both’ and ‘and’.
69. (a) She finished her work when I met her. 74. (a) Will I turn the light on? (Incorrect)
(Incorrect) Shall I turn the light on? (Correct)
She had finished her work when I met her. (b) Shall he come tomorrow? (Incorrect)
(Correct) Will he come tomorrow? (Correct)
(b) The train started before he reached the (c) Shall you do me a favour? (Incorrect)
station. (Incorrect) Will you do me a favour? (Correct)
The train had started before he reached (d) Will we attend the party? (Incorrect)

K KUNDAN
the station. (Correct) Shall we attend the party? (Correct)
Explanation: Explanation:
When two actions take place in the past, the one In interrogative sentences ‘shall’ is used in the
earlier in time is expressed by a verb in the past perfect first person and ‘will’ in the third person. In the second
tense, while the one later in time by that in the past person ‘shall’ and ‘will’ are used in accordance with
simple tense. the answer expected.
70. (a) They will be delighted if you will welcome 75. (a) Please excuse me being late. (Incorrect)
them. (Incorrect) Please excuse my being late. (Correct)
They will be delighted if you welcome (b) She disliked me coming late. (Incorrect)
them. (Correct) She disliked my coming late. (Correct)
(b) I will tell you as soon as I will know about Explanation:
it. (Incorrect) When a noun or pronoun is placed before a gerund,
I will tell you as soon as I know about it. it should be put in the possessive case.
(Correct) 76. (a) Unless you do not work hard, you will cut a
(c) When I shall go to Paris, I shall inform sorry figure. (Incorrect)
you. (Incorrect) Unless you work hard, you will cut a sorry
When I go to Paris, I shall inform you. figure. (Correct)
(Correct) (b) Walk slowly lest you should not fall down.
Explanation: (Incorrect)
We generally use the present instead of the future Walk slowly lest you should fall down.
after if and conjunction of time such as when, as soon (Correct)
as, after, while, until, and before. Explanation:
71. (a) It is time you go to bed. (Incorrect) ‘Unless’ means ‘if not’. It should, therefore, be not
It is time you went to bed. (Correct) used in a sentence or clause which is already negative.
(b) It is time you wash your face. (Incorrect) ‘Lest’ is here a subordinating conjunction expressing
It is time you washed your face. (Correct) a negative purpose. It should not be followed by ‘not’.
Explanation: 77. (a) I thought of attending the party, but could
When we want to say that ‘it’s time’ for somebody not. (Incorrect)
else to do something, we generally use the structure: I had thought of attending the party but
It’s time + subject + past tense verb. could not. (Correct)
266 English Language and Comprehension Error Detection

(b) We hoped that you woul d solve this and is pronounced as ‘yu’, or when a word begins with
problem. (Incorrect) ‘o’ and is pronounced as ‘wa’, it is preceded by the
We had hoped that you would solve this article ‘a’ and not ‘an’. Such words are: unique, united,
problem. (Correct) usual, European, useful, unit, universal, university,
(c) I ex pected not such a turn of events. useless, one-eyed man, one-rupee not, unilateral, etc.
(Incorrect) 82. (a) He plays violin. (Incorrect)
I had not expected such a turn of events. He plays the violin. (Correct)
(Correct) (b) Can you play tabla? (Incorrect)
Explanation: Can you play the tabla? (Correct)
The past perfect tense is used with such verbs as Explanation:
hope, expect, think, intend, mean (=intend), suppose and The definite article ‘the’ is used before musical
want to indicate that a past hope , e xpe ctation, instruments. When ‘play’ means ‘produce music’, its
intention, desire, etc., was not realized. object is always preceded by an article.
78. (a) I had gone to Chandigarh. (Incorrect) 83. (a) He has no knowledge and interest in music.
I went to Chandigarh. (Correct) (Incorrect)
Or, I had been to Chandigarh. (Correct) He has no knowledge of and interest in
(b) I had slept for hours. (Incorrect) music. (Correct)
I slept for hours. (Correct) (b) He did not agree but differed from my
Explanation: opinion. (Incorrect)
We do not usually used past perfect tense singly He did not agree to but differed from my
in a sentence. We can use simple past tense instead. opinion. (Correct)
When the verb ‘go’ is used in the sense of ‘go and Explanation:
come back from’, ‘been’ is used in place of ‘gone’. Sometimes a single preposition can’t be used for
79. (a) He needs not seek my help. (Incorrect) two words that take two different prepositions. In such
He need not seek my help. (Correct) a situation both the prepositions should be used.
(b) She dares not walk in the dark. (Incorrect) 84. (a) There is no end of troubles. (Incorrect)
She dare not walk in the dark. (Correct) There is no end to troubles. (Correct)

K KUNDAN
Explanation: (b) I am busy in my work. (Incorrect)
In affirmative sentence the singular form of dare/ I am busy with my work. (Correct)
need (that is, dares/needs) is used with singular (c) Send this letter on my address. (Incorrect)
subject. But need not and dare not admit of no change Send this letter to my address. (Correct)
even if the subject is third person singular. Explanation:
80. (a) The re is a HE school in my village . We usually say, end to one’s troubles, busy with
(Incorrect) some work, send something to someone’s address.
The re is an HE school in my village . 85. (a) Entering the room, the boys were found
(Correct) quarrelling. (Incorrect)
(b) We have filed a FIR. (Incorrect) Entering the room, he found the boys
We have filed an FIR. (Correct) quarrelling. (Correct)
(c) He is a NCC officer. (Incorrect) (b) Walking in the garden, a snake bit him.
He is an NCC officer. (Correct) (Incorrect)
(d) He has set up a X-ray plant. (Incorrect) While he was walking in the garden, a snake
He has set up an X-ray plant. (Correct) bit him. (Correct)
Explanation: (c) Barking furiously I led the dog out of the
Whether ‘a’ or ‘an’ is used before initials depends compound. (Incorrect)
on how the initial is pronounced. A, E, F, H, I, L, M, I led the dog, barking furiously, out of the
N, O, R, S and X all begin with a vowel sound; hence compound. (Correct)
an LEA School, an MA, an MP but a BBC production, a (d) Going out of the room, the door was left
BA, a PhD, etc. open by her. (Incorrect)
81. (a) He is an university professor. (Incorrect) Going out of the room, she left open the
He is a university professor. (Correct) door. (Correct)
(b) It is an unit of measurement. (Incorrect) (e) On examining the answer books, many silly
It is a unit of measurement. (Correct) mistakes were detected. (Incorrect)
(c) It is an universal truth. (Incorrect) On ex amining the answe r books, we
It is a universal truth. (Correct) detected many silly mistakes. (Correct)
Explanation: Explanation:
We use ‘a’ before a consonant sound, even if it is These sentences are examples of unattached
written with a vowel. When a word begins with ‘U’ and participles. When the first word of a sentence is a
is pronounced as ‘yu’, or when a word begins with ‘EU’ participle, it must refer to some noun or pronoun it
Error Detection English Language and Comprehension 267

qualifies. That noun or pronoun should be the subject Explanation:


of the main clause. Though preferable is not a comparative yet it has
86. (a) I saw her to go. (Incorrect) a comparative force. It is, therefore, wrong to write
I saw her go. (Correct) ‘more preferable’. Prefer takes to, not than.
(b) I observed him to limp. (Incorrect) 91. (a) He regards me his guardian. (Incorrect)
I observed him limp. (Correct) He regards me as his guardian. (Correct)
Explanation: (b) They portrayed Nehru a dictator. (Incorrect)
We usually leave out to, the sign of infinitive, The y portraye d Ne hru as a dict ator.
after sensory verbs like feel, hear, see, observe, perceive, (Correct)
mark, behold, survey, view, watch. Explanation:
87. (a) He insisted to go. (Incorrect) Certain verbs are always followed by ‘as’. Such
He insisted on going. (Correct) verbs are: describe, depict, define, mention, portray,
(b) I am tired to sing and dance. (Incorrect) regard, represent, treat.
I am tired of singing and dancing. (Correct) 92. (a) She called me as a dullard. (Incorrect)
(c) He succeeded to win the match. (Incorrect) She called me a dullard. (Correct)
He succe e de d in winning t he match. (b) He was appointed as principal. (Incorrect)
(Correct) He was appointed principal. (Correct)
Explanation: Explanation:
An infinitive is generally not used after words Certain verbs are not followed by ‘as’ or ‘to be’.
which take a preposition after them. Such words are: Such verbs are: appoint, choose, elect, call, consider,
Verbs: insist, object, prevent, succeed, think make, name, think.
Nouns: insistence, objection, intention, habit, 93. (a) She looks as if she suspects foul play.
resistance, view (Incorrect)
Adjectives: equal, fond, tired, used She looks as if she suspected foul play.
The infinitive s should be change d into the (Correct)
corresponding gerunds. (b) You act as though everything is in your
88. (a) Avoid to go there. (Incorrect) hands. (Incorrect)
Avoid going there. (Correct) You act as though everything was in your

K KUNDAN
(b) Stop to worry. (Incorrect) hands. (Correct)
Stop worrying. (Correct) Explanation:
(c) Keep on to try. (Incorrect) We usually avoid the use of present tense after
Keep on trying. (Correct) ‘as if’ and ‘as though’.
(d) He is busy to write letters. (Incorrect) 94. (a) He did nothing but to play. (Incorrect)
He is busy writing letters. (Correct) He did nothing but play. (Correct)
Explanation: (b) She did nothing but to sing. (Incorrect)
The infinitives are changed into the corresponding She did nothing but sing. (Correct)
gerunds after certain words which do not take (c) She did no more than to cry. (Incorrect)
prepositions after them. Such words are: She did no more than cry. (Correct)
Verbs: avoid, enjoy, finish, go on, keep on, mind, Explanation:
remember, can’t help, stop, give up We usually use the infinitive without ‘to’ after the
Adjectives: busy, worth preposition ‘but’ and ‘than’.
89. (a) It is no good to cry ove r spilt milk. 95. (a) She will wash up before she will go to bed.
(Incorrect) (Incorrect)
It is no good crying ove r spilt milk. She will wash up before she goes to bed.
(Correct) (Correct)
(b) There is no harm to do thi s work. (b) You won’t know how good pudding is till you
(Incorrect) will have tasted it. (Incorrect)
There is no harm in doing this work. You won’t know how good pudding is till you
(Correct) have tasted it. (Correct)
Explanation: Explanation:
The infinitive is changed into the corresponding The future simple is not used in time clauses.
gerund after such phrases as these: It is no use, It is The simple present tense is used instead. Nor is future
no good, Have the pleasure of, There is no harm in. perfect tense used in time clauses. The present perfect
90. (a) Death is more preferable than dishonour. is used instead.
(Incorrect) 96. (a) It is I who is responsible for this mistake.
Death is preferable to dishonour. (Correct) (Incorrect)
(b) I prefer swimming than walking. (Incorrect) It is I who am responsible for this mistake.
I prefer swimming to walking. (Correct) (Correct)
268 English Language and Comprehension Error Detection

(b) It is you who is responsible for this mistake. (c) He did not receive any message up till now.
(Incorrect) (Incorrect)
It is you who are re sponsible for this He has not received any message up till
mistake. (Correct) now. (Correct)
Explanation: (d) So f ar he did not re ac h the station.
The verb governed by ‘who’ should follow its (Incorrect)
antecedent. In the first sentence the antecedent is So far he has not reached the station.
‘I’. Hence the verb ‘am’. In the second sentence the (Correct)
antecedent is ‘you’. Hence the verb ‘are’. Explanation:
97. (a) I want an armchair for an old man with The present perfect tense can alone be used with
sliding back. (Incorrect) adverbials such as these: already, since, yet, so far, up
I want, for an old man, an armchair with till now.
sliding back. (Correct) 101. (a) This tragic incident has taken place last
(b) He shot himself dead after bidding his wife year. (Incorrect)
goodbye with a pistol. (Incorrect) This tragic incident took place last year.
He shot himself dead with a pistol after (Correct)
bidding his wife goodbye. (Correct) (b) She has gone to bed at 9 o’ clock.
Explanation: (Incorrect)
Both the sentences are examples of wrong word- She went to bed at 9 o’ clock. (Correct)
order impeding clarity of expression. It is well to (c) I have met him yesterday. (Incorrect)
remember that an adjective, adjective-substitute or I met him yesterday. (Correct)
adjective phrase should be put as near its antecedent Explanation:
as possible. This applies to relative clauses, too. Only the simple past can be used with adverbials
98. (a) No body has over helped you, has he? mentioning a definite time in the past.
(Incorrect) 102. (a) Reeta was married by Rakesh. (Incorrect)
Nobody has ever helped you, have they? Rakesh married Reeta. (Correct)
(Correct) (b) He married his youngest daughter with an

K KUNDAN
(b) Don’t do that any more, do you? (Incorrect) engineer. (Incorrect)
Don’t do that any more, will you? (Correct) He married his youngest daughter to an
(c) I am taller than him, isn’t I? (Incorrect) engineer. (Correct)
I am taller than him, aren’t I? (Correct) (c) His youngest daughter was married with
Explanation: an engineer. (Incorrect)
If anybody, anyone, everybody, everyone, somebody, His youngest daughter was married to an
someone, nobody or no one is the subject of the main engineer. (Correct)
clause, the subject of the question tag is normally Explanation:
‘they’. If the main clause is in the form of a negative When ‘marry’ is used in the sense of ‘take as
request or command, the question tag normally begins husband or wife’, it can never be used in the passive
with ‘will’. The question tag after ‘I am’ is ‘aren’t I’. form. It can be used both in the active and passive
99. (a) Bring a cold glass of water. (Incorrect) forms in the sense of ‘give in marriage’. It is to be
Bring a glass of cold water. (Correct) followed by ‘to’, not ‘with’.
(b) He has purchased a fresh basket of apples. 103. (a) She come s to colle ge by a bicycl e.
(Incorrect) (Incorrect)
He has purchased a basket of fresh apples. She comes to college by bicycle.
(Correct) Or, She comes to college on a bicycle.
(c) I’d love to have a hot cup of tea. (Incorrect) (Correct)
I’d love to have a cup of hot tea. (Correct) (b) He came back by my car. (Incorrect)
Explanation: He came back in my car. (Correct)
In the present context the expression, ‘cold glass’, Explanation:
‘fresh basket’ and ‘hot cup’ are meaningless. The When the name of a vehicle is used in a general
adjectives cold, fresh and hot qualify the noun ‘water’, sense, the preposition ‘by’ is used. In all such cases
‘apples’ and ‘tea’ respectively. Hence the correct no article is used before the name of the vehicle. If
expressions will be cold water, fresh apples and hot the reference is to a particular vehicle, ‘by’ is not used.
tea. We use ‘in’ or ‘on’ instead.
100. (a) He already left for Delhi. (Incorrect) 104. (a) Supposing if he does not come, what will
He has already left for Delhi. (Correct) you do? (Incorrect)
(b) He did not speak to me since that incident. Supposing he does not come, what will you
(Incorrect) do? OR,
He has not spoken to me since that If be does not come, what will you do?
incident. (Correct) (Correct)
Error Detection English Language and Comprehension 269

Explanation: believe, suppose and imagine, we make the first verb


‘Supposing’ and ‘If’ are not used together. Either (think, believe, etc.) negative, not the second.
of the two should be used. 108. (a) Do sit down, will you? (Incorrect)
105. (a) He informed that everybody had gone Do sit down, won’t you? (Correct)
against him. (Incorrect) (b) Give me sufficie nt time , won’t yo u?
He informed me (us, him, them, etc.) that (Incorrect)
everybody had gone against him. (Correct) Give sufficient time, will you? (Correct)
(b) She told that she was interested in chess. (c) Shut up, can you? (Incorrect)
(Incorrect) Shut up, can’t you? (Correct)
She told me (us, him, them etc.) that she Explanation:
was interested in chess. (Correct) After imperative we use (i) won’t you? — to invite
(c) He assured that every help would be given. people to do things. (ii) will you?/would you?/could
(Incorrect) you?/can’t you? — to tell people to do things.
He assured me (us, him, them, etc.) that 109. (a) He pays more attention to films than books.
every help would be given. (Correct) (Incorrect)
Explanation: He pays more attention to films than to
The verbs assure, inform, remind and tell are not books. (Correct)
immediately followed by that–clause as their object. (b) I can rely more on you than her. (Incorrect)
There has to be an indirect object between the verb I can rely more on you than on he r.
and the clause. (Correct)
106. (a) Flour is made of wheat. (Incorrect) Explanation:
Flour is made from wheat. (Correct) When there is comparison between two objects,
(b) Your chair is not made fr om wood. the preposition is placed before each of the two objects.
(Incorrect) 110. (a) He is declared to pass in the first division.
Your chair is not made of wood. (Correct) (Incorrect)
Explanation: He is declared to have passed in the first
When reference is made to the material of which division. (Correct)
something is made, we use ‘of’. The original material (b) She is supposed to commit this murder.

K KUNDAN
of which something is made can still be recognised. (Incorrect)
But we use ‘from’ when something is changed beyond She is supposed to have committed this
recognition. murder. (Correct)
107. (a) I thin k you hav en’t me t my fathe r. Explanation:
(Incorrect) Perfect infinitive is used if it refers to a time prior
I do not think you have met my father. to that which is expressed by the finite verb.
(Correct) 111. (a) How do you do? Fine, thanks. (Incorrect)
(b) I believe you haven’t seen the Taj Mahal. How do you do? How do you do? (Correct)
(Incorrect) Explanation:
I don’t believe you have seen the Taj Mahal. Don’t confuse How do you do? with How are you? It
(Correct) is a formula used when people are formally introduced.
Explanation: The reply is exactly the same: How do you do?
When negative ideas are introduced with think,

Correct Usage
Here are two lists of such words and expressions earning earnings
as are commonly misused. Their correct forms will equipments equipment
help you avoid mistakes you occasionally commit. furnitures furniture/pieces of furniture
Incorrect Correct gentries gentry
advices advice/pieces of advice lecturership lectureship
arm (Weapon) arms machineries machinery/machines
auspice auspices offsprings offspring
blotting blotting paper outskirt outskirts
boarding boarding house, hostel poetries poems
bowel bowels sceneries scenery/scenes
breads pieces/slices/loaves of bread scissor scissors
cattles cattle stationeries stationery
270 English Language and Comprehension Error Detection

traffics traffic with black and blue black and blue


trouser trousers with heart and soul heart and soul
a coward man a cowardly man/a coward with tooth and nail tooth and nail
a miser person a miserly person/a miser good in studies good at studies
a man of his words a man of his word clever in figure works clever at figure works
a flight of stair a flight of stairs bad in studies bad at studies
a man of letter a man of letters in trice in a trice
Arrear Bill Arrears Bill in hurry in a hurry
a serial of lectures a series of lectures What to speak of Not to speak of
birth date date of birth abstain to speak ill of.. abstain from speaking ill of...
cousin brother/ aim to do good aim at doing good
sister cousin bent to do bent upon doing
custom duty customs duty desirous to go desirous of going
family members members of his family do the needful do whatever is necessary
famous criminal notorious criminal (have) passion to read (have) passion for reading
head pain headache persist to say persist in saying
stomach pain/ refrain to go refrain from going
tooth pain stomachache/toothache repent to do repent of doing
(He has a headache/toothache/stomachache.) succeed to win succeed in winning
in class tenth in class ten or in the tenth class take pride to do take pride in doing
in the campus on the campus think to do think of doing
in the committee on the committee build a home build a house
in leave on leave cut jokes crack jokes
mutual friend common friend cut the pencil sharpen the pencil

K KUNDAN
no place (in a bus, cook bread bake bread
train etc.) no room (in a bus, train etc) describe about describe
passing marks pass marks discuss about discuss
Tennis field Tennis court drink tea take tea
cheque of Rs. 200 cheque for Rs 200 excel to speak excel in speaking
The back side give a speech deliver a speech
of a building The back of a building give goodbye bid goodbye
The front side give the examination take the examination;
of a building The front of a building appear at the examination;
today morning this morning sit for the examination
today afternoon this afternoon eat the poor feed the poor
today evening this evening give order give orders
today night tonight make a lecture deliver a lecture
two dozens pens two dozen pens make a goal score a goal
three thousands people three thousand people make noise make a noise
(But we say, dozens of pens, thousands of people to de- open the knot untie the knot
note unspecified number).
pray God pray to God
Saving Bank Savings Bank
rise the lid raise the lid
sworn enemies avowed enemies
see the pulse feel the pulse
vacant vessels empty vessels
speak a lie tell a lie
white hair grey hair
stick the button sew the button
worth seeing sight a sight worth seeing
take out one’s shoes take off one’s shoes
9.30 o’clock train 9.30 train
to have headache to have a headache
details upon details detail upon detail
to have temperature to have a temperature
miles after miles mile after mile
to steal in the exami- to use unfair means
with bag and baggage bag and baggage
nation in the examination
Chapter 3

Error Detection in Specific Words


Directions: In each sentence below four words that desicion. 4)/ All correct 5)
that the printed in bold have been lettered (1), 16. The importanse 1)/ given to content-oriented
(2), (3), (4) and (5). One of them may be wrongly 2)/ approach has affected the methodology 3)/
spelt or inappropriate in the context of the of this project 4)/. All correct 5)
sentence. Find out the word, which is wrongly 17. Almost all risk-taking 1)/ work involve 2)/
spelt or inappropriate if there is any. The letter decision 3)/ making under uncertainty. 4)/
of that word is the answer. If all the words, which All correct 5)
are printed in bold, are correctly spelt and 18. In developing 1)/ countries there is increesing
appropriate in the context of the sentence, mark 2)/ concern for fostering 3)/ human potential.
(5) as the answer ie All orrect. 4)/ All correct 5)
1. Gandhiji is known 1)/ for his successful 2)/ 19. I want to express 1)/ my appreciation 2)/ of
afforts 3)/ to liberate 4)/ India. All correct 5) the help offered 3)/ by my former colleages.
2. The cruelties 1)/ of history 2)/ are perpetrated 4)/ All correct 5)
3)/ in the name of nobal 4)/ causes. All correct 20. The research 1)/ reported 2)/ in this valume
5) 3)/ assumes importance. 4)/ All correct 5)
3. The fear of universal 1)/ destruction 2)/ hangs 21. Almost 1)/ two hours have elapsed 2)/ since
3)/ over us like a dark cloud 4)/. All correct 5) he fell 3)/ asleep. 4) All correct 5)
4. The e nvironme nt has a pro found e 1) / 22. Have you noticed 1)/ that the country is on
inf luen ce 2)/ on the way a Society 3)/ the brink 2)/ of a serious 3)/ dissaster? 4)/
develops 4)/. All correct 5) All correct 5)
5. The atmosphere 1)/ was fragrent 2)/ with the 23. The messanger’s 1)/ story that appreared 2)/
scent 3)/ of rose flowers 4)/. All correct 5) incredible 3)/ has turned out 4)/ to be grue-
6. Shareholders used to be liabel 1)/ for the debts some. All correct 5).
2)/ of the company in proportion 3)/ to the 24. She shade 1)/ tears as if to display her grief,
size 4)/of their holdings. All correct 5) 2)/ but they were not a genuine 3)/ expression
7. The popular 1)/ understanding of the 2)/ of sorrow. 4) / All correct 5)
incidence 3)/ was that he had resigned 4)/. 25. As a consequence 1)/ of 2)/ that earthquack
All correct 5) 3)/ many families have been, ruined. 4)/ All
8. He was felicitated 1)/ for his roll 2)/ in correct 5)
resolving 3)/ the coniflict 4)/ in the region. 26. Rising 1)/ prices 2)/ of foodgrains will have an
All correct 5) adverse 3)/ impac on developing 4)/countries.
9. He firmly 1)/ denied 2)/ that the document All correct 5)
3)/ existed 4)/. All Correct 5) 27. To deal effectively 1)/ with a crisis 2)/ quick
10. Today banks offer 1)/ the facility 2)/ of instint decisions 3)/ are requited 4)/. All correct 5)
3)/ transfer 4)/ of funds to their customers. 28. The IT Company has succeeded 1)/ in achieving
All correct 5). high growth rate despite 2)/ facing 3)/ several
11. Their sole 1)/ concern 2)/ was how they could 4)/problems. All correct 5) .
assisst 3)/ their colleague in his hour of crisis. 29. On an average 1)/ there are very few persons
4)/ All correct 5) wil lingl y 2)/ to take on 3)/ such
12. We shall be disabled 1)/ to justify 2)/ this responsibility. 4)/ All correct 5)
excess 3)/ expenditure. 4)/ All correct 5) 30. More than halve 1)/ of the budget 2)/ has been
13. The lease on these premises 1)/ has expired spent 3)/ on modernising 4)/ the factory. All
2)/ and we have incured 3)/ significant debt. correct. 5)
4)/ All correct 5) 31. My gole 1)/ is to acquire 2)/ a position 3)/ of
14. Your statement 1)/ that you received 2)/ no aut horit y 4)/ and re spe ct within the
prier 3)/ intimation 4)/ is not plausible. All organization. All correct 5)
correct 5) 32. The refusal 1)/ of the Ministry to clear pending
15. Mr Sharma refused to acknowledge 1)/ that 2)/ dues is a couse 3)/ for concern 4)/. All
he had committed 2)/ an error 3)/ while taking correct 5)
196 Test of English Language

33. He was convinced 1)/ that discipline 2)/ and 53. Our firm 1)/ manages 2)/ the investment of
hard work would result 3)/ in dividents 4)/ in several 3)/ foreigner 4)/ companies in India.
the long run. All correct 5) All correct 5)
34. This is a company which believes 1)/ in 54. The panel 1)/ is suppose 2)/ to meet tomorrow
transparency 2)/ and appointment is done to sanction 3)/ the purchase 4)/ of ne w
strictly 3)/ on merit 4)/. All correct 5) premises. All correct 5)
35. There will be a decline 1)/ in the probability 55. It is our intention 1)/ to keep a check 2)/ on
2)/ of oil companies because of the hike 3)/ in the expences 3)/ ncurred 4)/ by them. All
oil prices 4)/. All correct 5) correct 5)
36. Private 1)/ companies offer more competitive 56. Since petroleum products are taxed 1)/ heavily
2)/ salaries and opportunities 3)/ for carrier they are a majar 2)/ source 3)/ of revenue 4)/
4)/ growth. All correct 5) for the government. All correct 5)
37. The company is trying 1)/ to raise 2)/ its 57. Even though the proposal appearrs 1)/ practical
revenu 3)/ by offering more products and 2)/ the committee should discuss 3)/ it at
services 4)/. All correct 5) length 4)/. All correct 5)
38. He has proved 1)/ his reliability 2)/ and 58. The Indian stock market has been one of the
integrity 3)/ in accomplishing 4)/ a task. All well 1)/ performing 2)/ markets globally 3)/
correct 5) during the current 4)/ year. All correct 5)
39. Many farmers still relie 1)/ on credit 2)/ from 59. Had the scheme been allowed 1)/ to continue
mone yle nde rs, which is risky 3)/ and 2)/ it would have generated 3)/ attracted 4)/
inconvenient 4)/. All correct 5) returns. All correct 5)
40. There is a possibly 1)/ that the merger will have 60. People should constantly 1)/ upgrade their
a negative 2)/ impact 3)/ on consumers 4)/. skills 2)/ in order to be effective 3)/ and
All correct 5) efficient 4)/. All correct 5)
41. China attracts 1)/ plen ty 2)/ of fore ign 61. Every single 1)/ decision will be reveiwed 2)/
investors 3)/ and has huge forex reserves 4)/ at the regular 3)/ monthly 4)/ meeting. All
All correct 5) correct 5)
42. India should adopt 1)/ these strategies 2)/ to 62. We often 1)/ ask our customers to give us their
handle the threat 3)/ of global warming opinion s 2)/ and suggest ions 3)/ for
successively 4)/ All correct 5) improvement 4)/. All correct 5)
43. The shortage 1)/ of rice has led 2)/ to a severe 63. I tried to convince 1)/ him that the situation
food crises 3)/ in the region 4)/ All correct 5) 2)/ was not as worse 3)/ as it appeared 4)/. All
44. You should delay 1)/ your plan to acquier 2)/ correct 5)
that company owing 3)/ to the risk involved. 64. Underneath 1)/ the new law 2)/ the managing
4)/ All correct 5) director will no longer 3)/ be appointed 4)/ by
45. The panal 1)/ is of the view 2)/ that the the government. All correct 5)
restrictions 3)/ should be implemented 4)/ 65. After the meeting I discussed 1)/ the issue
immediately. All correct 5) 2)/ with my colleagues 3)/ who were very
46. The sit e 1)/ of the acciden t 2)/ was helpfull 4)/. All correct 5)
surrounded 3)/ by a lot of spectators 4)/ but 66. Inspite of my attempts 1)/ to encourage 2)/
none helped the victims. All Correct 5) him to continue 3)/ studying 4)/ he decided
47. Forecasts 1)/ of oil consumption 2)/ are to take up a job. All correct 5)
controvercial 3)/ because of excessive 4)/ use 67. This is the first time that he has been selected
of vehicles. All correct 5) 1)/ to give a speech 2)/ at the anual 3)/
48. The flour 1)/ was cleaned using detergents conference 4)/. All correct 5)
2)/and all the stains 3)/ were removed 4)/. All 68. To find a solution 1)/ to the problem, we met
correct 5) 2)/ daily after work and contributed 3)/ our
49. The thief who was in the possession 1)/ of the ideals 4)/. All correct 5)
police escaped 2)/ but the police refrained 3)/ 69. In my opinion it will be difficullt 1)/ to
from chassing 4)/ him. All correct 5) persuade 2)/ the employees to accept 3)/ these
50. We figured 1)/ that no one would dear 2)/ to changes 4)/ in the rules. All correct 5)
come close 3)/ to us because of our weapons 70. After the presentation 1)/ it was clearly 2)/
4)/. All correct 5) that they were not interested 3)/ in financing
51. In the latest 1)/ few months the number of 4)/ the project. All correct 5)
con tract s 2)/ we have secured 3)/ has 71. We are now facing stiff 1)/ competing 2)/ from
declined 4)/ sharply. All correct 5) foreign companies manufacturing 3)/ similar
52. No amount 1)/ of money can compensate 2)/ 4)/ products. All correct 5)
you for the losses 3)/ you have experienced 72. I have mentioned 1)/ this case to indicate
4)/. All correct 5)
Error Detection in Specific Words 197

2)/ how deliberate 3)/ it will be to achieve our 92. Rajan would 1)/ pick 2)/ up his children from
goal 4)/. All correct 5) 3)/ the school and sup erwis e 4)/ t he ir
73. Today, thank 1)/ to our support 2)/she has homework. All correct 5)
managed 3)/ to purchase 4)/ her own house. 93. The milk vendor 1)/ studied hard 2)/ for four
All correct 5) years before topped 3)/the national level 4)/
74. As per your request 1)/ this is a detail 2)/ list exam. All correct 5)
of our basic 3)/ requirements 4)/. All correct 94. The issues 1)/ of hunger and poorty 2)/ are
5) left behind 3)/ as we have progressed 4)/ in
75. The final decision 1)/ of whether 2)/to go ahead technology. All correct 5)
3)/ or not does not rest 4)/ with the manager. 95. Indian peoples 1)/ invest 2)/ as much 3)/ in
All correct 5) gold as in bank savings accounts 4)/. All correct
76. It is true 1)/ that credit card companies cannot 5)
import 2)/ such 3)/ high charges 4)/ on 96. Oportunities 1)/ multiply 2)/ when they are
customers. All correct 5) seized 3)/ and die 4)/ when they are not. All
77. In our opinion the se norm s 1)/ ar e not correct 5)
applicant 2)/ in this kind 3)/ of situation 97. He realized 1)/ he was alone 2)/ in the house
4)/ All correct 5) and rushed 3)/ to bolt 4)/all the doors and
78. The government is trying 1)/ to control 2)/ windows. All correct 5)
the price raise 3)/ but this is not the method it 98. The information 1)/ provided 2)/ to the staff
should adopt 4)/. All correct 5) was not adecuate 3)/ and everyone retaliated.
79. To ensure 1)/ there has been no fraud 2)/ the 4)/ All correct 5)
accounts of the previous 3)/ year need to be 99. Worried 1)/ that he will fail in the exams,
carelessly 4)/ examined. All correct 5) Satish stayed 2)/ up the whole knight 3)/ and
80. In this time of recession you should consider studied. 4)/ All correct 5)
1)/ the possibility 2)/ of reducing 3)/ your 100. Thomas could not cook 1)/very well and thus
expenditure 4)/. All correct 5) had to stay 2)/ food from 3)/ a restaurant. 4)/
81. It is impossible 1)/ to complete 2)/ the entire All correct 5)
3)/ project within the specified 4)/ time frame. 101. As it was the tenth 1)/ day of the festival 2)/
All correct 5) the constables 3)/ on duty were tried 4)/ of
82. The Government is certain 1)/ to amend 2)/ patrolling. All correct 5)
the law to prevail 3)/ the crisis 4)/. All correct 102. Rita was tending 1)/ to her flower 2)/ beds
5) with joy 3)/ and pride. 4)/ All correct 5)
83. Except 1)/ for a few, the majority 2)/ of our 103. The most won derfulles t 1)/ thing about
staff has been recruited 3)/ locally 4)/. All miracles 2)/ is that the y sometimes 3)/
correct 5) happen. 4)/All correct 5)
84. The y pat ientl y 1)/ ex p lained 2)/ the 104. It is better 1)/ to die 2)/ on your feet then 3)/
procedure 3)/ for appling 4)/ for a loan to the live 4)/ on your knees. All correct 5)
villagers. All correct 5) 105. He glanced 1)/ around the room suspisiously,
85. One can succeed 1)/ in business unless 2)/ 2)/ sure they were 3)/ hiding somewhere. 4)/
one is prepared 3)/ to take risks 4)/. All correct All correct 5)
5) 106. They failed to adhear 1)/ to the terms of the
86. Ex pl ain 1)/ why you believe 2)/ the se agreement 2)/ on which they 3)/ had agreed
practices 3)/ should be discontinued. 4)/ All upon 4)/ earlier. All correct 5)
correct 5) 107. The main problem 1)/ of education is that 2)/
87. If you empl oy 1)/ m ore sys tematic 2) / it is not ex cessible 3)/ to all the pe ople
methods 3)/ you can lesser your costs. 4)/ All conveniently. 4)/ All correct 5)
correct 5) 108. Last night the ambassador 1)/ was summoned
88. On the basics 1)/ of this data 2)/ it is 2)/ to the foreign office to discuss 3)/ the cricis.
worthwhile 3)/ to take this risk 4)/ All correct 4)/ All correct 5)
5) 109. He was an amature 1)/ singer till 2)/ the age
89. Our present 1)/ projects are running 2)/ on of forty, when 3)/ he turned a professional.
schedule 3)/ and will be completely on time, 4)/ All correct 5)
4)/ All correct 5) 110. It never ceases 1)/ to amazing 2)/ me how he
90. We have noticed 1)/ you in advance 2)/ can talk for so long without ever 3)/ saying
regarding 3)/ the regional 4)/ manager’s anything interesting. 4)/ All correct 5)
conference. All correct 5) 111. Although 1)/ it was the first time the King had
91. A good 1)/ leader is of prime 2)/ importance lead 2)/ his troupes to the battle, he showed
for develop 3)/ of any organisation 4)/. All 3)/ remarkable 4)/ courage. All correct 5)
correct 5) 112. As the dinne r was read y 1)/ the hosts
198 Test of English Language

requested 2)/ everyone to precede 3)/ to the 4)/. All coorrect 5)


dining 4)/ hall. All correct 5) 132. Salaries 1)/ of bank employees 2)/ are likely
113. The police stopped 1)/ him and asked for the 3)/ to go above 4)/ by ten per cent. All correct
lisence 2)/ which he unfortunately 3)/did not 5)
carry while going 4)/ for the drive. All correct 5) 133. The Chairman has promissed 1)/ to look 2)/
114. His father thought 1)/ that he had more into the matter 3)/ and take necessary 4)/
potential 2)/ to become 3)/ an engineer then action. All correct 5)
4)/ a doctor. All correct 5) 134. Since 1)/ the interest rate on bank deposits is
115. The doctors at the hospital says 1)/ that he low 2)/ at present, people prefer 3)/ to deposit
will be absolutely 2)/ fine within a week’s time, their savings 4)/ in post office schemes. All
given 3)/ that he follows their advice. 4)/ All correct 5)
correct 5) 135. Most 1)/ projects are delayed 2)/ because of
116. He was unable 1)/ to give a satisfactory 2)/ the absents 3)/ of proper planning 4)/. All
e xplanation for his absense 3)/ from the correct 5)
meeting. 4)/ All correct 5) 136. A large number of celebrities 1)/ have joined
117. Much 1)/ countries are starting to turn 2)/ their an NGO involved 2)/ in the movement for
attention 3)/ to new sources 4)/ of energy. All protection 3)/ of animal writes 4)/. All correct
correct 5) 5)
118. As the ship was sinking 1)/ fast, the captain 137. The teacher liked the poem 1)/ so much that
2)/ gave orders to abandon 3)/ it immediately. she requested 2)/ Saba to read it allowed 3)/
4)/ All correct 5) to the whole 4)/ class. All correct 5)
119. The council 1)/ denied having any hand 2)/ in 138. As the wind blue 1)/ harder every minute, 2)/
the recently 3)/ unearthed scandle. 4)/ All people got a fairly 3)/ good idea that a storm
correct 5) was approaching 4)/ the town. All correct 5)
120. Their 1)/ has been a series of abductions 2)/ 139. The family had to bear 1)/ a leaky sealing 2)/
of young children 3)/ of the schools in the area throughout the rainy season 3)/ as they could
4)/ All correct 5) not afford 4)/ to get it repaired. All correct 5)
121. In such cir cums tans es 1)/ do not take 140. Breathe 1)/ deeply and inhale the sents 2)/ of
unnecessary 2)/risks 3)/with your savings 4)/ roses and daisies in the landscaped gardens
All correct 5) 3)/ surrounding 4)/ my house. All correct 5)
122. RBI is unwilling 1)/ to enforce 2)/ the se 141. Drivers 1)/ who exceed 2)/ the speed limit
re gulations as the se will dis courage 3 )/ 3)/ are duly find 4)/ by the traffic police
investment 4)/. All correct 5) personnel. All correct 5)
123. Many Indian firms have ent ry 1)/ into 142. This project 1)/ is in its final faze 2)/ of
partnership 2)/ with foreign 3)/ ones of late completion and is expected 3)/ to be ready 4)/
4)/. All correct 5) by the end of this year. All correct 5)
124. This is a prime 1)/ example 2)/ of what the 143. The bored 1)/ of governors meets 2)/ once every
government can achieve 3)/ if it is determine month to discuss 3)/ the organisation’s policy
4)/. All correct 5) 4)/. All correct 5)
125. They managed 1)/ to accomplice 2)/ this by 144. There was a slight 1)/ reduction 2)/ in his
coming 3)/ up with unique 4)/ schemes. All wait 3)/ after almost a week 4)/ of strict dieting.
correct 5) All correct 5)
126. He was a member 1)/ of the orignal 2)/ 145. He has not been able to attend 1)/ the office
committe e which draf ted 3)/ the se 2)/ for last four 3)/ days be cause he is
recommendations 4)/. All correct 5) suffering 4)/ from fever. All correct 5)
127. Do not precede 1)/ with the transfer 2)/ of 146. Ways and Means Advances is a facility 1)/ under
funds until you re ce ive furt her 3 )/ which the government can borrow 2)/ from RBI
instructions 4)/. All correct 5) to meet its revenue 3)/ requirments 4)/. All
128. Our branch is located 1)/ in a remote 2)/ area correct 5)
where water is scarcity 3)/ available 4)/. All 147. We are finding 1)/ it difficult to meet our
correct 5) targets 2)/ because of short 3)/ of funds 4)/.
129. To gain 1)/ the ir support, 2)/ you must All correct 5)
convenience 3)/ them of the benefits 4)/ of 148. The company has received 1)/ a good responds
the proposal. All correct 5) 2)/ from people who have started 3)/ using its
130. The deal we negotiated 1)/ has not be en new services 4)/ . All correct 5)
approved 2)/ by RBI and needs 3)/ to be 149. The government has ready 1)/ adopted 2)/ many
revized 4)/. All correct 5) recommen datio ns 3)/ of the Narsimhan
131. He found 1)/ the company in 1980 to provide Committee on banking sector reforms 4)/. All
2)/ electricity to people living 3)/ in rural areas correct 5)
Error Detection in Specific Words 199

150. Statistics are crucial 1)/ for the government to 170. The tennis 1)/ and cricket matches 2)/were
frame 2)/ effective 3)/ policies 4)/. All correct interpted 3)/as it began 4)/to rain. All correct5)
5) 171. The car 1)/ was praked 2)/ near 3)/ her building
151. Besides 1)/ the duties of compassion 2)/ 4)/. All correct 5)
harmlessness and 3)/ forgiveness there are 172. A vaccination 1)/drive was held 2)/to prevent
still duties that we owe to others 4)/. All correct the outbreak 3)/of an epidemic 4)/. All correct5)
5) 173. Lina was 1)/the first 2)/one to reech 3)/ the
152. There was once a Brahmana who had made a venue 4)/. All correct 5)
vow 1)/ that he would only eat food that he 174. She could 1)/ not attend 2)/ the function 3)/
could 2)/ gather 3)/ in the feilds 4)/. All correct as her flight got canselled 4)/. All correct 5)
5) 175. Mohan trried 1)/to help 2)/but his friend 3)/
153. By far the most urgent 1)/ need of industry in refused to take his help 4)/. All correct 5)
journal 2)/ and of IT industry in particular 176. The organization preferred 1)/ to hire locale
3)/ is the need for skilled 4) / manpower. All 2)/ population as they understood the language
correct 5) 3)/ and customer preferences. 4)/All correct 5)
154. The King gave away 1)/ all that he had, and he 177. In our opinion the exicting 1)/ assessment
and his famly 2)/ went without food so that 2)/ system required immediate 3)/ revision.
they might 3)/ feed the hungry. 4)/ All correct5) 4)/ All correct 5)
155. We must feal 1)/ love for all, no matter 2)/ 178. In responds 1)/ to the advertisement a sizeable
whether 3)/ they are of our own family or 2)/ number of candidates have submitted 3)/
strangers or whether they are rich or poor 4)/ their applications. 4)/ All correct 5)
All correct 5) 179. There is no guarantee 1)/ that if this model is
156. Quality 1)/ is never an accident 2)/ and is adopted 2)/ the entire 3)/ sector will prosper.
always the result of intelligent 3)/ effort 4)/. 4)/ All correct 5)
All correct 5) 180. With this uniq ue 1)/ in itiative 2)/ the
157. Sharad consoled 1)/ Vijay and asured 2)/ him company hopes to sustain 3)/ its currant 4)/
that his son would return 3)/ home by sunset growth rate. All correct 5)
4)/. All correct 5) 181. The income 1)/ of many people in rural 2)/
158. One of the monkeys was keeping 1)/ a track India is not adequate 3)/ to satisfy 4)/ their
2)/ of the things 3)/ done by the king’s men basic needs. All correct 5)
from a distance 4)/. All correct 5) 182. He is always 1)/ prompt 2)/ in caring 3)/ out
159. The Swan lived in a pawned 1)/and had striking instructions 4)/. All correct 5)
2)/ golden 3)/ feathers 4)/. All correct 5) 183. The revized 1)/ rates 2)/ of interest will be
160. The mother and her daughter 1)/ were happily effective 3)/ immediately 4)/. All correct 5)
selling milk which got them enough 2)/ money 184. Such transactions 1)/ are quiet 2)/ expensive
to leed 3)/ a comfortable 4)/ life. All correct 5) 3)/ and time consuming 4)/ for customers. All
161. Information 1)/ about the exam 2)/ was correct 5)
displaid 3)/ on the notice 4)/ board. All correct 185. The guidelines 1)/ of the new scheme 2)/ are
5) expected 3)/ to be finally 4)/soon. All correct5)
162. Richa promised 1)/ to kleen 2)/ her room 3)/ 186. We have incurred 1)/ an expense 2)/ of over
on Sunday 4)/. All correct 5) fifty thousands 3)/ this year alone 4)/. All
163. The robers 1)/ tried to get 2)/ into the house correct 5)
3)/ through the balcony 4)/. All correct 5) 187. World Earth Day is celeberated 1)/ as a means
164. The strike 1)/ continued for three days, because 2)/ to make pe ople awar e 3)/ about the
2)/of which the company 3)/ underwent a huge environment 4)/. All correct 5)
4)/ loss. All correct 5) 188. The key issue 1)/ discused 2)/ at the meeting
165. There 1)/ were many 2)/ seagulls 3)/ on the was how to resolve 3)/ the food crisis 4)/. All
beach 4)/. All correct 5) correct 5)
166. The completion 1)/ of the tunnel 2)/ has been 189. He deserves 1)/ some recognition 2)/ for
held 3)/ up owning 4)/ to a strike. All correct working diligently 3)/ for the passed 4)/ five
5) years. All correct 5)
167. The Directr 1)/ spoke clearly and distinctly,2)/ 190. The re are many emp loymen t 1) /
therefore the audience 3)/ could understand opportunities 2)/ for fresh graduates 3)/ in
4)/every word he spoke. All correct 5) the market 4)/ today. All correct 5)
168. Shall I write 1)/ a lettr 2)/ to him or would you 191. There were many objectives 1)/ from employees
prefer 3)/ to call 4)/ him? All correct 5). to the pro posal 2)/ to am en d 3)/ the
169. It has always be en the policy 1)/ of the regulations 4)/. All correct 5)
company 2)/ to promote existing 3)/ staff to 192. Since he has provided 1)/ over halve 2)/ the
senior positions 4)/. All correct 5) finance for the infrastructure 3)/ he should
200 Test of English Language

be in charge 4)/. All correct 5) the phone and ultimately 3)/ ended up not
193. The scheme permits 1)/ investors 2)/ to buy finishing 4)/ her work. All correct 5)
the shares from foreign 3)/ companies at a 213. She had not eaten 1)/ anything 2)/ for a very
ficsed 4)/ price. All correct 5) long time now and her stomach 3)/ was groling
194. A leader who relies 1)/ on his team members 4)/. All correct 5)
2)/ for advice 3)/ is respected 4)/. All correct 214. Half of the harm 1)/ that is done in this world
5) 2)/ is due to people 3)/ who want to feel
195. He is in complete 1)/ agreement 2)/ with your important 4)/. All correct 5)
analyze 3)/ of the situation 4)/. All correct 5) 215. Life is like a mirror; 1)/ smile at it, and it’s
196. RBI has attempted 1)/ to spend 2)/ financial charmeng; 2)/ frown 3)/ at it, and it becomes
3)/ awareness 4)/ through this programme. All sinister 4)/. All correct 5)
correct 5) 216. The designer will showcash 1)/ her collection
197. In order to succeed 1)/ it is crucial 2)/ for an 2)/ at an upcoming 3)/ fashion 4)/ event in
organisation to constantly 3)/ improve 4)/. the city. All correct 5)
All correct 5) 217. Our mind is like a garden 1)/ which can either
198. With some assistance 1)/ from her son she be intelligently 2)/ cultivated 3)/ or be
was enable 2)/ to settle 3)/ her debts 4)/ on allowed to run wilde 4)/. All correct 5)
time. All correct 5) 218. Researchers 1)/ have identified 2)/ the early
199. Though the government initiated 1)/ a large master cells make up the human heart could
sum 2)/ of money in the scheme 3)/ it was a be used to make patches 3)/ to fix damaged
failure 4)/. All correct 5) 4)/ hearts. All correct 5)
200. We have prepared a detailed 1)/ report giving 219. The girl spends 1)/ every knight 2)/ studying,
various 2)/ solutions 3)/ to resort 4)/ the as she wants to graduate 3)/ from a well known
problem. All correct 5) college 4)/ with good marks. All correct 5)
201. Under existing 1)/ regulations we are not 220. The next time your gym instructor 1)/ tells
permitted 2)/ to owe 3)/ more than a forty per you to do some stretching 2)/ exersises 3)/
cent share 4)/ of the family business. All correct before starting the workout, 4)/ say no. All
5) correct 5)
202. In case of any land dispute 1)/ panchayat 221. Discussion 1)/ is an exchange of knowledge
officials 2)/ will determine 3)/ how the property 2)/ whereas arguement 3)/ is a depiction 4)/
is to be dividend 4)/. All correct 5) of ignorance. All correct 5)
203. The World Bank has consented 1)/ to sanction 222. He was arrested 1)/ for the crime 2)/ and was
2)/ the necessary 3)/ finance 4)/ for the project. charged 3)/ with attempt 4)/ to murder. All
All correct 5) correct 5)
204. To obtain 1)/ a refund you will have to fill 2)/ a 223. Commit 1)/ yourself to lifelong learning 2)/
claim 3)/ with the appropriate 4)/ authority. as the most valuable 3)/ aset 4)/ you will have
All correct 5) is your mind. All correct 5)
205. Experts predict 1)/ there will be shortaze 2)/ 224. Belive 1)/ that life is worth 2)/ living and your
of investment 3)/ in the infrastructure 4)/ belief will create 3)/ the fact 4)/. All correct 5)
sector. All correct 5) 225. The best educated 1)/ human bing 2)/ is the
206. When the young artist returned 1)/ to his one who understands 3)/ most about the life
village, his family held a festive 2)/ dinner on in which 4)/ he is placed. All correct 5)
it's lawn to ce le brate his tr ium pan t 3)/ 226. The city’s fashion-conscious ladies 1)/ came
homecoming 4)/. All correct 5) together at a city hotel to check out an exibition
207. She trusted Mira with all her heart 1)/ and thus 2)/ by various 3)/ designers 4)/ and labels.
handled 2)/ over her life's 3)/ savings to her All correct 5)
instantly 4)/. All correct 5) 227. The ministry’s proposal 1)/ for an autonomous
208. It is difficult 1)/ to see the picture 2)/ when 2)/ ove rarching aut horit y 3)/ for highe r
you are inside 3)/ the frame 4)/. All correct 5) education and research was finally approval
209. Had she not suppressed 1)/ all the details of 4)/. All correct 5)
her Company's project 2)/ her Company would 228. Silense 1)/ is to retreat 2)/ in wordless prayer,
have bagged 3)/ the contract 4)/. All correct 5) gazing 3)/ out the window of your heart, and
210. The whole 1)/ time she walked with her child going for slow meandering 4)/ walks in a garden.
in her arms the only thing 2)/ that worried 3)/ All correct 5)
her was her son's feature 4)/. All correct 5) 229. A majority of Army tanks continue to grope 1)/
211. It is not unusual 1)/ for guests of the hotel to in the dark, stricken 2)/ as they are with an
carry 2)/ souveniers 3)/ back with them when ecute 3)/ case of night blindness 4)/. All
they return 4)/ to their homes. All correct 5) correct 5)
212. She vested 1)/ her time in chatting 2)/ over 230. Back home, the ever affable 1)/ Bollywood singer
Error Detection in Specific Words 201

shares the ex citement 2)/ of having 3)/ 4)/ All correct 5)


performed 4)/ at the Royal Hall in London. All 248. I appealed 1)/ to her to look after my children
correct 5) 2)/ during my absence 3)/ and she acceded to
231. Even though the state has been witnessing my request. 4)/ All correct 5)
1)/ deaths on a daily basis, 2)/ it has not 249. You have absolutely 1)/ no authority 2)/ to
hindered 3)/ the festivity 4)/ spirit of the interphere 3)/ in my affairs. 4)/ All correct 5)
people. All correct 5) 250. This time I was compelled 1)/ to leave him
232. The player was arrestted 1)/ for kicking 2)/ unpunished 2)/ although he deserved severe
and punching 3)/ a driver outside a fast-food 3)/ punishment for breech of rules. 4)/ All correct
outlet 4)/ in the city. All correct 5) 5)
233. The clever disciple 1)/ had decided to proved 251. The cine ma transfor ms 1)/ abs tract 2)/
2)/ his skills by reciting 3)/ the holy verse 3)/ themes 3)/ into concret stories. 4)/ All correct
from the book. All correct 5) 5)
234. In just one year Beena has gained 1)/ around 252. He adviced 1)/ me not to pick 2)/ up quarrel
eight kilograms 2)/ and doctors fear she might 3)/ with neighbours on trivial matters. 4)/ All
be prune 3)/ to heart-related ailments 4)/. All correct 5)
correct 5) 253. Imported items are costlier 1)/ than there 2)/
235. On being threatened 1)/ by the king’s servants, domestic 3)/ counterparts 4)/. All correct 5)
the poor gardener 2)/ blurted 3)/ out that he 254. Although 1)/ I was paid significantly 2)/ low,
had stolen the jewels 4)/. All correct 5) I found my salary to be insufficient 3)/ or
236. It is indeed recommendable 1)/ that the apex rather adequate. 4)/ All correct 5)
court has deemed 2)/ it necessary to remind 255. An exhorbitantly 1)/ rigid attitude may prove
the government of its duties in promoting 3)/ 2)/ very dangerous, 3)/ even fatal 4)/. All correct
education and investing 4)/ in it. All correct 5) 5)
237. The perception 1)/ of animal life was even more 256. There is hardly any resemblence 1)/ between
ambigous 2)/ because of anthropomorphic 3)/ the faces 2)/ of the so-called identical 3)/
characterisations of animal behaviour 4)/All twins 4)/. All correct 5)
correct 5) 257. India has progressed 1)/ remarkbly 2)/ in
238. Policy of permitting 1)/ legal 2)/ import of gold ex ercisin g 3)/ our com mitme nts in
has stimulated 3)/ its consumation 4)/. All international affair 4)/. All correct 5)
correct 5) 258. They vouched 1)/ to lunch 2)/ the programme
239. His continually 1)/ defending 2)/ his stand 3)/ without much fanfare 4)/. All correct 5)
on the issue has risen 3)/ doubts in the mind 259. The Gove rnme nt officials appe ar to be
of the jury 4)/. All correct 5) committed 1)/ to implement 2)/ the poverty
240. The government’s strategy to encourage 1)/ elevation 3)/ project with due sincerity 4)/.
entreprenurship 2)/ gathers momentum 3)/ All correct 5)
with unenvisaged 4)/ response. All correct 5) 260. Preventive majors 1)/ have controlled 2)/ the
241. The roll 1)/ of the institute is to provide enormous 3)/ growth rate of terrorist 4)/
technical support to other institutions and to activities. All correct 5)
constantly monitor 2)/ their facilities 3)/ and 261. Tackling 1)/ potentially 2)/ violent 3)/
performance. 4)/ All correct 5) creaminals 4)/ is not free from risk. All correct
242. The competitive 1)/ edge for survival 2)/ lays 5)
3)/ in the effective 4)/ use of information 262. With the unusually 1)/ heavy rainfall, the
technology. All correct 5) prediction 2)/ made by meteorological 3)/
243. The most popular 1)/ method adopted 2)/ by department was proved 4)/ wrong. All correct
an organisation to com municate 3)/ job 5)
vacancie s to the public is through 263. The economic imperatives 1)/ for acquiring
advertisement. 4)/ All correct 5) 2)/ technological strengths do not warrant 3)/
244. The act of extending 1)/ preferential 2)/ repeatition 4)/ here. All Correct 5)
treatment to service providers was high 3)/ 264. The combination of a base of imported technology
appreciated. 4)/ All correct 5) and capabilities 1)/ built up indigenous 2)/
245. The significant 1)/ future 2)/ is that none of led in itiall y 3)/ to product and proce ss
the ancient Indian scientists claimed 3)/ involvement 4)/. All correct 5)
originality 4)/ of their theories. All correct 5) 265. If a country does not learn to master 1)/ these
246. Owing 1)/ to scarsit y 2)/ of wate r, the new realities of life, our aspirations 2)/ to
metropolitan 3)/ city of Mumbai is passing ensure the prosperity 3)/ of our people may
through 4)/ a hard time. All correct 5) come to not 4)/. All correct 5)
247. I repriminded 1)/ him for using humiliating 266. Since vegetable and fruit consumtion 1)/ will
2)/ language while addressing 3)/the audience. increase in future, an appropriate 2)/ choice
202 Test of English Language

considering agro-climate 3)/ input needs and 286. The objective 1)/ of the seminar was to raise
economic returns 4)/ should be arrived at for 2)/ awareness 3)/ about the consequenses 4)/
every region. All correct 5) of corruption. All correct 5)
267. An environmental 1)/ concern 2)/ that is likely 287. The government is commited 1)/ to providing
to have implications 3)/ for Indian agriculture world-class infrastructure to sustain 2)/
is the emission 4)/ of gases like methane and extensive 3)/ growth 4)/ of industries. All
carbon dioxide. All correct 5) correct 5)
268. The turgid 1)/ article 2)/ on liberalization is 288. Non-communicable 1)/ diseases 2)/ are a
very complicated 3)/ to following 4)/. All major 3)/ concerned 4)/ for people in this
correct 5) region. All correct 5)
269. The suspicion 1)/ was paralysed 2)/ due to 289. The industry is poised 1)/ for a quantum 2)/
the impact of the tortuous 3)/ interrogation jump as it has ventured into Indian markets
4)/. All correct 5) which have excellent 3)/ growth opportunities
270. While con vincin g 1)/ othe rs about false 4)/. All correct 5)
claims, he maintains 2)/ a strait 3)/ face 4)/ 290. The Government administration is required to
All correct 5) undergo 1)/ a radicle 2)/ transformation for
271. Due to financial 1)/ constrents, 2)/ I can’t better service orientation 4)/. All correct 5)
help you solve 3)/ your problems 4)/. All correct 291. The squwad 1)/ that was entrusted with the
5) task of detection 2)/ of explosives and their
272. Meditation 1)/ enables 2)/ us to acquire piece neutralizatio n 3)/ has com ple te d the
3)/ and tranquility 4)/. All correct 5) assignment 4)/. All correct 5)
273. Sania competed 1)/ with the excellent 2)/ 292. The equipments 1)/ used for surveillance 2)/
players from each state in our country 3)/ and must be reliability 3)/ and of appropriate 4)/
won the first prige 4)/ All correct 5) quality. All correct 5)
274. The approach 1)/ of parents’ participation 2)/ 293. We are aware 1)/ that the funds have been
in e ducation acknowleges 3)/ the social utilise d to me e t working 2)/ capitel 3)/
dimensions 4)/ of this issue. All correct 5) expenditure 4)/. All correct 5)
275. Vinayak is a shrewd 1)/ person, any moment 294. The slowdown of the global economy should be
2)/ he may create 3)/ a truoble 4)/ for you. All considired 1)/ an opportunity 2)/ rather than
correct 5) a challenge 4)/. All correct 5)
276. An exclucive 1)/ proposal will be presented 295. The conversion 1)/ of large investment banks
2)/ for discussion and fav ourabl e 3) / into commercial 2)/ banks meant that there
consideration 4)/ All correct 5) were fewer 3)/ employers in the sector 4)/. All
277. After several 1)/ rounds of negotiations 2)/ correct 5)
the flat owner axceeded 3)/ to our request 4)/ 296. Employees frequently 1)/ avail of loan facilites
All correct 5) 2)/ provided 3)/ by their employers at reduced
278. Mr Banerjee is a popular 1)/ leader since he 4)/ rates of interest. All correct 5)
del igat es 2)/ autho rity 3)/ to his 297. This outlet 1)/ of the multinational company
subordinates 4)/. All correct 5) is ex pected 2)/ to e mploy additional 3)/
279. The manager was accompanied 1)/ by several personal 4)/. All correct 5)
2)/ e xpe rts whe n he s urv eyed 3)/ the 298. We plan to comple te the ex haus tive 1)/
devastation 4)/ caused by the fire. All correct performance 2)/ review 3)/ undertook 4)/ by
5) us by next week. All correct 5)
280. He failed 1)/ to secure 2)/ that crucial 3)/ 299. Home loan borrowers 1)/ will be definitely 2)/
contract so he loss 4)/ his job. All correct 5) affected 3)/ since banks have raised their
281. The presents 1)/ of all parties 2)/ involved lending 4)/ rates of interest. All correct 5)
3)/ in the dispute is essential 4)/. All correct5) 300. The Finance Ministe r has imp ressed 1)/
282. Mahesh has taken a momentous 1)/ decision satisfaction 2)/ over the progress 3)/ made
to sell his very profitible 2)/ business 3)/ to by regional 4)/ rural banks. All correct 5)
his biggest rival 4)/. All correct 5) 301. If they want to maintain 1)/ their current rate
283. The entire 1)/ process 2)/ of assiging 3)/ of expansion 2)/ they have to consider 3)/
projects has undergone 4)/ a change. All correct these parametres 4)/. All correct 5)
5) 302. India’s overall 1)/ trade defecit 2)/ continues
284. In his speech Mr Leeladhar revealed 1)/ that 3)/ to be a major source of concern for analysts
banks have to comply 2)/ with Basel norms 4)/. All correct 5)
3)/ last by March next year. All correct 5) 303. After severel 1)/ rounds of discussions 2)/
285. The scheme 1)/ was launched to allow 3)/ a the Manager assented 3)/ to our proposal 4)/
customer to know the states 4)/ of his loan All correct 5)
application. All correct 5) 304. The indafatigable 1)/ zeal and fearlessness
Error Detection in Specific Words 203

2)/ with which the youth worked rejoiced 3)/ 322. In order to confront 1)/ the threat 2)/ of global
me beyond measure 4)/. All correct 5) warming it is imperative 3)/ that we work
305. He sliped 1)/ away in the night 2)/ and reached altogether 4)/. All correct 5)
3)/ there in the morning 4)/. All correct 323. This firm which is based 1)/ in France is
306. He let the laugher 1)/ wash away 2)/ the prepaired 2)/ to finance 3)/ small Indian
tension 3)/ and then held 4)/ up her hand. All businesses which produce 4)/environment-
correct 5) friendly products. All correct 5)
307. The author also advocated 1)/ fresh vegetable 324. Despite intense 1)/ pressure 2)/ from his
2)/ and fruits 3)/ as naturel 4)/ diet of aged supe riors he refuse d to discl oze 3)/ the
peoples. All correct 5) findings 4)/ of the report. All correct 5)
308. The document 1)/ he gave me was long and 325. Corporates have benefited tremendously 1)/
complicated 2)/ and I st ruggled 3)/ to from the government’s timely 2)/ decision to
understand 4)/ it. All correct 5) waive 3)/ various tax 4)/. All correct 5)
309. We shall have to await 1)/ and see if these 326. Since you are unable 1)/ to repay the loan you
measures 2)/ are sufficient 3)/ to address have no alternate 2)/ but to seek 3)/ an
4)/ the problem. All correct 5) extension 4)/. All correct 5)
310. They are negotiating 1)/ to try and reach 2)/ 327. A major 1)/ disadvantage 2)/ of this deal is
an agreement which will beneficial 3)/ everyone that we shall have to bear 3)/ the cost 4)/ of
concerned 4)/. All correct 5) training. All correct 5)
311. The company has decided 1)/ to allott 2)/ a 328. The bank’s fluctuating 1)/ performance over
substantial 3)/ portion 4)/of its profits to the prior 2)/ year has been a major 3)/ cause
research and development. All correct 5) 4)/ for concern. All correct 5)
312. It remains 1)/to be seen whether 2)/ these 329. An economy relies 1)/ on its access 2)/ to
reforms 2)/ will be acceptable 4)/ by the Board. dependable 3)/ and affordable 4)/ sources of
All correct 5) energy. All correct 5)
313. His main reasoning 1)/ for applying 2)/ for 330. Researches have used data prevalent 1)/ to
this promotion is the possibility 3)/ of an manufacturing companies to illustrate 2)/ the
overseas posting 4)/. All correct 5) harmful 3)/ impacts 4)/ of technology on the
314. Since our financial resources 1)/ are limitless environment. All correct 5)
2)/ we cannot afford 3)/ this alternative. All 331. Such a situation 1)/ is neither feasible 2)/
correct 5) nor desirable 3)/ in a democratic country like
315. Your marketing strategy 1)/ should be devised ours 4)/. All correct 5)
2)/ dependent 3)/ on the kind of business you 332. The gradual 1)/ withdrawal 2)/ of such safety
own. All correct 5) 3)/ mechanisims 4)/ will affect small and
316. To better understand the challenges 1)/ of medium industries the most. All correct 5)
starting a ne w ven ture 2)/ he read u p 333. In order to curtale 1)/ the substantial 2)/
extensively 4)/. All correct 5) export of iron ore recently 3)/ the government
317. The file contents 1)/ a revised 2)/ estimate has imposed 4)/ an export tax All correct 5)
3)/ of the banking industry’s manpowe r 334. Several 1)/ mediam 2)/ and small sized 3)/
requirements 4)/for the next year. All correct5) companies successfully survived 4)/the global
318. Though these programmes have proved 1)/ to financial crisis of 2008. All correct 5)
be extremely 2)/ effective 3)/ they do have 335. One of the tan gibl e 1)/ benefit s 2)/ of
certain drawbacks 4)/. All correct 5) appearing 3)/ for a loan under this scheme is
319. Any failure 1)/ to complicit 2)/ with these the interest rate concessions 4)/ All correct 5)
fundamental 3)/ regulations will result in a 336. With effect 1)/ from April, non-banking finance
fine 4)/. All correct 5) companies with good performances 2)/ may be
320. Every organisation needs to be proactive 1)/ granted 3)/ licences to convert 4)/ into banks.
in devising 2)/ stratergies 3)/ to ensure the All correct 5)
retention 4)/of staff. All correct 5) 337. In 2009, the largest 1)/ remittances 2)/ sent
321. According to these estimates 1)/ our profitable to India were from oversees 3)/ Indians living
2)/ margin 3)/ will be higher if we adopt 4)/ 4)/ in North America. All correct 5)
this approach. All correct 5)
204 Test of English Language

Answers and explanations


1. 3; The correct spelling is ‘efforts’. 55. 3; The correct spelling is ‘expenses’.
2. 4; The correct spelling is ‘noble’. 56. 2; The correct spelling is ‘major’.
3. 5; All correct. 57. 1; The correct spelling is ‘appears’.
4. 1; The correct spelling is ‘profound’. 58. 5; All correct.
5. 2; The correct spelling is ‘fragrant’. 59. 4; Replace ‘attracted’ with ‘attractive’.
6. 1; The correct spelling is ‘liable’. 60. 5; All correct.
7. 2; The word ‘outcome’ should be used in place of 61. 2; The correct spelling is ‘reviewed’.
‘understanding’. 62. 5; All correct.
8. 2; The correct spelling is ‘role’. 63. 3; Change ‘worse’ to ‘bad’.
9. 5; All correct. 64. 1; Change ‘underneath’ to ‘under’ or ‘As per’ or
10. 3; The correct spelling is ‘instant’. ‘According to’.
11. 2; The correct spelling is ‘assist’. 65. 4; Change ‘helpfull’ to ‘helpful’.
12. 1; Replace ‘disabled’ with ‘capable’. 66. 4; Replace ‘studying’ with ‘study’.
13. 3; The correct spelling is ‘incurred’. 67. 3; The correct spelling is ‘annual’.
14. 3; The correct spelling is ‘prior’. 68. 4; Replace ‘ideals’ with ‘ideas’.
15. 4; The correct spelling is ‘decision’. 69. 5; All correct.
16. 1; The correct spelling is ‘importance’. 70. 2; Replace ‘clearly’ with ‘clear’.
17. 1; The appropriate word should be ‘risky’. 71. 2; The word ‘competing’ is inappropriately used.
18. 2; The correct spelling is ‘increasing’. The correct word is ‘competition’.
19. 4; The correct spelling is ‘colleagues’. 72. 3; The correct spelling is ‘deleberate’.
20. 3; The correct spelling is ‘volume’. 73. 5; All correct.
21. 5; All correct. 74. 2; Replace ‘detail’ with ‘detailed’.
22. 4; The correct spelling is ‘disaster’. 75. 5; All correct.
23. 1; The correct spelling is ‘messenger’s’. 76. 2; Replace the word ‘import’ with ‘impose’.
24. 1; The appropriate word is ‘shed’. 77. 2; Replace ‘applicant’ with ‘applicable’.
25. 3; The correct spelling is ‘earthquake’. 78. 3; Replace ‘raise’ with ‘rise’.
26. 3; The correct spelling is ‘impact’. 79. 4; Replace ‘carelessly’ with ‘carefully’.
27. 4; The correct spelling is ‘required’. 80. 5; All correct.
28. 5; All correct. 81. 5; All correct.
29. 2; The appropriate word is ‘willing’. 82. 3; Replace ‘prevail’ with ‘solve’.
30. 1; The appropriate word is ‘half’. 83. 5; All correct.
31. 1; The correct spelling is ‘goal’. 84. 4; The correct spelling is ‘applying’.
32. 3; The correct spelling is ‘cause’. 85. 5; All correct.
33. 4; The correct spelling is ‘dividends’. 86. 4; It is proper to use ‘prohibited’.
34. 5; All correct. 87. 4; It is proper to use ‘lessen’.
35. 2; The appropriate word should be ‘profitability’. 88. 1; It is appropriate to use ‘basis’.
36. 4; The appropriate word is ‘career’. 89. 4; It is proper to use ‘completed’.
37. 3; The correct spelling is ‘revenue’. 90. 3; It is proper to use ‘during’.
38. 5; All correct. 91. 3; Replace the word ‘develop’ with ‘development’.
39. 1; The correct spelling is ‘rely’. 92. 4; The correct spelling is ‘supervise’.
40. 1; The appropriate word is ‘possibility’. 93. 5; All correct.
41. 4; Replace the word ‘reserves’ with ‘reserve’. 94. 2; The appropriate word should be ‘poverty’.
42. 4; Re place the word ‘successively’ with 95. 1; The appropriate word should be ‘people’.
‘successfully’. 96. 1; The correct spelling is ‘opportunities’.
43. 3; The correct spelling is ‘crisis’. 97. 5; All correct.
44. 2; The correct spelling is ‘acquire’. 98. 3; The correct spelling is ‘adequate’.
45. 1; The correct spelling is ‘panel’. 99. 3; The appropriate word should be ‘night’.
46. 5; All correct. 100. 2; The appropriate word should be ‘buy’.
47. 3; The correct spelling is ‘controversial’. 101. 4; Replace ‘tried’ with ‘tired’.
48. 1; The appropriate word should be ‘floor’. 102. 1; Replace ‘tending’ with ‘attending’.
49. 4; The correct spelling is ‘chasing’. 103. 1; Replace ‘wonderfullest’ with ‘wonderful’.
50. 2; The appropriate word should be ‘dare’. 104. 3; Replace ‘than’ with ‘then’.
51. 1; Replace ‘latest’ with ‘last’. 105. 2; The correct spelling is ‘suspiciously’.
52. 5; All correct. 106. 1; The correct spelling is ‘adhere’.
53. 4; Replace ‘foreigner’ with ‘foreign’. 107. 3; The correct spelling is ‘accessible’.
54. 2; Replace ‘suppose’ with ‘supposed’. 108. 4; The correct spelling is ‘crisis’.
Error Detection in Specific Words 205

109. 1; The correct spelling is ‘amateur’. 168. 2; The correct spelling is ‘letter’.
110. 2; Replace ‘amazing’ with ‘amaze’. 169. 5; All correct.
111. 2; The appropriate word is ‘led’. 170. 3; The correct spelling is ‘interrupted’.
112. 3; The appropriate word is ‘proceed’. 171. 2; The correct spelling is ‘parked’.
113. 2; The correct spelling is ‘licence’. 172. 5; All correct
114. 4; The appropriate word is ‘than’. 173. 3; The correct spelling is ‘reach’.
115. 1; Replace ‘says’ with ‘say’. 174. 4; The correct spelling is ‘cancelled’.
116. 3; The correct spelling is ‘absence’. 175. 1; The correct spelling is ‘tried’.
117. 1; Replace ‘Much’ with ‘Many’. 176. 2; The correct spelling is ‘local’.
118. 5; All correct. 177. 1; The correct spelling is ‘existing’.
119. 4; The correct spelling is ‘scandal’. 178. 1; The appropriate word should be ‘response’.
120. 1; Replace ‘Their’ with ‘There’. 179. 5; All correct
121. 1; The correct spelling is ‘circumstances’. 180. 4; The correct spelling is ‘current’.
122. 5; All correct. 181. 5; All correct.
123. 1; Replace ‘entry’ with ‘entered’. 182. 3; Replace ‘caring’ by ‘carring’.
124. 4; Replace ‘determine’ with ‘determined’. 183. 1; The correct spelling is ‘revised’.
125. 2; The appropriate word is ‘accomplish’. 184. 2; ‘quiet’ should be replaced by ‘quite’.
126. 2; The correct spelling is ‘original’. 185. 4; ‘finally’ should be replaced by ‘final’.
127. 1; The appropriate word should be ‘proceed’. 186. 3; The correct use is ‘thousand’.
128. 3; Replace ‘scarcity’ with ‘scarcely’. 187. 1; The correct spelling is ‘celebrate’.
129. 3; Replace ‘convenience’ with ‘convince’. 188. 2; The correct spelling is ‘discussed’.
130. 4; The correct spelling is ‘revised’. 189. 4; Use ‘past’ in place of ‘passed’.
131. 1; The appropriate word is ‘founded’. 190. 5; All correct.
132. 4; The appropriate word is ‘up’. 191. 1; The appropriate word should be ‘objections’.
133. 1; The correct spelling is ‘promised’. 192. 2; Replace ‘halve’ with ‘half’.
134. 5; All correct. 193. 4; Replace ‘ficsed’ with ‘fixed’.
135. 3; The appropriate word is ‘absence’. 194. 5; All correct.
136. 4; The appropriate word is ‘rights’. 195. 3; Replace ‘analyze’ with ‘analysis’.
137. 3; The appropriate word is ‘aloud’. 196. 2; Change ‘spend’ to ‘awaken’.
138. 1; The appropriate word is ‘blew’. 197. 5; All correct.
139. 2; The appropriate word is ‘ceiling’. 198. 2; Change ‘enable’ to ‘able’.
140. 2; The appropriate word is ‘scent’. 199. 1; Change ‘initiated’to’spent’.
141. 4; The appropriate word is ‘fined’. 200. 4; Change ‘resort’ to ‘resolve’.
142. 2; The correct spelling is ‘phase’. 201. 5; All correct.
143. 1; The appropriate word is ‘bord’. 202. 4; The appropriate word should be ‘divided’.
144. 3; The appropriate word is ‘weight’. 203. 5; All correct.
145. 5; All correct. 204. 4; All correct.
146. 4; The correct spelling is ‘requirements’. 205. 2; The correct spelling is ‘shortage’.
147. 3; The appropriate word is ‘shortage’. 206. 2; The correct spelling is ‘triumphant’.
148. 2; The appropriate word is ‘response’. 207. 2; Replace ‘handled’ with ‘handle’.
149. 1; The appropriate word is ‘readily’. 208. 5; All correct.
150. 5; All correct. 209. 5; All correct.
151. 5; All correct. 210. 4; The appropriate word is ‘future’.
152. 4; The correct spelling is ‘field’. 211. 3; The correct spelling is ‘souvenirs’.
153. 2; The appropriate word is ‘general’. 212. 1; The appropriate word should be ‘wasted’.
154. 4; The correct spelling is ‘family’. 213. 4; The correct spelling is ‘growling’.
155. 1; The correct spelling is ‘feel’. 214. 5; All correct.
156. 5; All correct. 215. 2; The correct spelling is ‘charming’.
157. 2; The correct spelling is ‘assured’. 216. 1; The correct spelling is ‘showcase’.
158. 5; All correct. 217. 4; The correct spelling is ‘wild’.
159. 1; The appropriate word is ‘pond’. 218. 5; All correct.
160. 3; The correct spelling is ‘lead’. 219. 2; The correct spelling is ‘night’.
161. 3; The correct spelling is ‘displayed’. 220. 3; The correct spelling is ‘exercises’.
162. 2; The correct spelling is ‘clean’. 221. 3; The correct spelling is ‘argument’.
163. 1; The correct spelling is ‘robbers’. 222. 5; All correct.
164. 5; All correct. 223. 4; The correct spelling is ‘asset’.
165. 5; All correct. 224. 1; The correct spelling is ‘Believe’.
166. 4; The appropriate word is ‘owing’. 225. 2; The correct spelling is ‘being’.
167. 1; The correct spelling is ‘Director’. 226. 2; The correct spelling is ‘exhibition’.
206 Test of English Language

227. 4; Replace ‘approval’ with ‘approved’. 280. 4; The appropriate word is ‘lost’.
228. 1; The correct spelling is ‘silence’. 281. 1; The appropriate word is ‘presence’.
229. 3; The correct spelling is ‘acute’. 282. 2; The correct spelling is ‘profitable’.
230. 5; All correct. 283. 3; The correct spelling is ‘assigning’.
231. 4; Replace ‘festivity’ with ‘festive’. 284. 4; The appropriate word is ‘latest’.
232. 1; The correct spelling is ‘arrested’. 285. 4; The appropriate word is ‘status’.
233. 2; Replace ‘proved’ with ‘prove’. 286. 4; The correct spelling is ‘consequences’.
234. 4; Replace ‘prune’ with ‘prone’. 287. 1; The correct spelling is ‘committed’.
235. 5; All correct. 288. 4; The appropriate word is ‘concern’.
236. 5; All correct 289. 5; All correct.
237. 2; The correct spelling is ‘ambiguous’. 290. 2; The correct spelling is ‘radical’.
238. 4; The correct spelling is ‘consumption’. 291. 1; The correct spelling is ‘squad’.
239. 3; It should be ‘raised’ in place of ‘risen’. 292. 3; The appropriate word is ‘reliable’.
240. 2; The correct spelling is ‘entrepreneurship’. 293. 3; The correct spelling is ‘capital’.
241. 1; The word ‘roll’ makes no sense here. So replace 294. 1; The correct spelling is ‘considered’.
it with ‘role’, which means ‘function’. 295. 5; All correct.
242. 3; Replace ‘lays’ with ‘lies’. 296. 2; The correct spelling is ‘facilities’.
243. 5; All correct. 297. 4; The appropriate word is ‘personnel’.
244. 3; Replace ‘high’ with ‘highly’. 298. 4; The appropriate word is ‘undertaken’ .
245. 2; It should be ‘feature’ instead of ‘future’ to make 299. 5; All correct.
the sentence meaningful. 300. 1; The appropriate word is ‘expressed’.
246. 2; The correct spelling is ‘scarcity’. 301. 4; The correct spelling is ‘parameters’.
247. 1; The correct word will be ‘reprimand’. The 302. 2; The correct spelling is ‘deficit’.
meaning of ‘reprimand’ is ‘to express severe 303. 1; The correct spelling is ‘several’.
disapproval of somebody or their actions 304. 1; The correct spelling is ‘indefatigable’.
especially officially’. 305. 1; The correct spelling is ‘slipped’.
248. 5; All correct 306. 1; The correct spelling is ‘laughter’.
249. 3; The correct spelling is ‘interfere’. 307. 4; The correct spelling is ‘natural’.
250. 4; The correct spelling is ‘breach’. 308. 5; All correct.
251. 4; The correct spelling is ‘concrete’. 309. 1; The appropriate word is ‘wait’.
252. 1; The correct usage will be ‘advised’. 310. 3; The appropriate word is ‘benefit’.
253. 2; The appropriate word is ‘their’. 311. 2; The correct spelling is ‘allot’.
254. 3; The appropriate word is ‘sufficient’. 312. 4; The appropriate word is ‘accepted’.
255. 1; The correct spelling is ‘exorbitantly’. 313. 1; The appropriate word is ‘reason’.
256. 1; The correct spelling is ‘resemblance’. 314. 2; The appropriate word is ‘limited’.
257. 5; All correct. 315. 3; The appropriate word is ‘depending’.
258. 2; The appropriate word is ‘launch’. 316. 5; All correct.
259. 3; The correct spelling is ‘alleviation’. 317. 1; The appropriate word is ‘contains’
260. 1; The appropriate word is ‘measures’. 318. 5; All correct.
261. 4; The correct spelling is ‘criminals’. 319. 2; The appropriate word is ‘comply’.
262. 5; All correct. 320. 3; The correct spelling is ‘strategies’.
263. 4; The correct spelling is ‘repetition’. 321. 2; The appropriate word is ‘profit’.
264. 2; The appropriate word is ‘indigenously’. 322. 4; The appropriate word is ‘together’.
265. 4; The appropriate word is ‘nought’. 323. 2; The correct spelling is ‘prepared’.
266. 1; The correct spelling is ‘consumption’. 324. 3; The correct spelling is ‘disclose’.
267. 5; All correct. 325. 4; The appropriate word is ‘taxes’.
268. 4; The appropriate word is ‘follow’. 326. 2; The appropriate word is ‘alternative’.
269. 1; The appropriate word is ‘suspect’. 327. 5; All correct.
270. 3; The appropriate word is ‘straight’. 328. 2; The appropriate word is ‘previous’.
271. 2; The correct spelling is ‘constraints’. 329. 5; All correct.
272. 3; The appropriate word is ‘peace’. 330. 1; The appropriate word is ‘relevant’.
273. 4; The appropriate word is ‘prize’. 331. 5; All correct.
274. 3; The correct spelling is ‘acknowledges’. 332. 4; The correct spelling is ‘mechanisms’.
275. 4; The correct spelling is ‘trouble’. 333. 1; The correct spelling is ‘curtail’.
276. 1; The correct spelling is ‘exclusive’. 334. 2; The correct spelling is ‘medium’.
277. 3; The correct spelling is ‘acceded’. 335. 3; The correct spelling is ‘applying’.
278. 2; The correct spelling is ‘delegates’. 336. 2; The appropriate word is ‘performance’.
279. 5; All correct. 337. 3; The correct spelling is ‘overseas’.
match battle conspiracy
war blow defence
legislation war bulwark
fight safeguard campaign
Appropriate Prepositions insurance shield

ABOUT AS

(i) Here are some verbs which are followed by about: (i ) Here are some transitive verbs which are followed
agree care forget by as:
moan talk argue acknowledge class designate
chat fret muse name address classify
te ll ask complain diagnose nominate adopt
fuss protest think conceive disguise perceive
boast consult groan brand condemn elect
rave warn brag project cast consider
disagree grumble read employ regard categorise
wonder brood dream construe establish stamp

K
inquire speak worry certify count hail
(ii) Here are some nouns which are followed by about: use characterise denounce
advice decision misgivings interpret choose depict
row agreement fuss label cite describe
news anxiety idea mark
opinion book information (ii) He re are some intransitive verbs which are
phobia chat joke followed by as:
concern judgement prediction
act double pose
consultation lecture quarrel
begin function serve
debate letter question
come pass work
(iii)Here are some adjectives which are followed by

KUNDAN
about: AT
adamant embarrassed optimistic (i) Here are some verbs which are followed by at:
unclear angry enthusiastic
gape glance look
passionate unconcerned annoyed
gaze glare stare
fussy pleased undecided
bark shout swear
anxious guilty positive
scream snap yell
uneasy apprehensive happy
beam grin smile
sceptical unhappy bothered
ignorant sensitive unsure wave frown scowl
certain indignant sentimental sneer wink aim
upset complacent miserable grasp snatch clutch
serious vague concerned guess strike grab
mistaken sorry wary shoot throw dab
crazy nervous uncertain hack pick tear
worried gnaw nibble poke
work exclaim marvel
AGAINST shudder frown rail
smile jeer rave
(i) Here are some verbs which are followed by against: sneer laugh scoff
agitate compete play wander
align conspire plot
ally fight side (ii) Here are some adjectives which are followed by
advise guard insure at:
vaccinate counsel immunize aghast annoyed embarrassed
insulate protect fight pleased alarmed appalled
inoculate militate warn furious surprised amzaed
(ii) Here are some nouns which are followed by astonished impatient unhappy
against: amused bewildered indignant
aggression boycott crime upset angry disappointed
victory ally fight irritated
BETWEEN prevent transform cut
get protect turn
(i ) He re are some nouns which are followe d by deflect glean receive
between: withhold
agreement consultation link IN
alliance contact merger
antagonism co-ordination misunderstanding (i) Here are some verbs which are followed by in:
balance correspondence partnership assist fail intervene
battle encounter rapport mediate believe gain
band feud relationship invest revel collaborate
breach fight split glory involve share
collision friendship truce dabble implicate join
connection interplay understanding specialise deal indulge
contrast disparity gulf luxuriate wallow (to enjoy)
similarity difference distinction engage interfere meddle
inequality discrepancy gap
(ii) Here are some nouns which are followed by in:
parity

K
belief decrease fluctuation
(ii) He re are some ve rbs which are followe d by
skill breach delight
between:
improvement slump catch
advertise bargin grope development increase stake
search aim fight change exercise interest
hunt send apply upsurge confidence experiment
fish look wait part voice cut
appeal call hope faith pride decline
lust press ask fall say difference
clamour hunger pine equality similarity disparity
wish beg hanker inequality variation
long pray yearn

KUNDAN
INTO
BY
Here are some verbs which are followed by into:
Here are some verbs which are followed by by: bully deceive galvanize
abide live stick seduce coax delude
go stand swear pressurize shame coerce
frighten push trick
FOR blossom develop make
Here are some nouns which are followed by for: change divide transform
admiration disdain enthusiasm convert evolve translate
nostalgia affection dislike degenerate grow turn
hatred partiality affinity
OF
disregard love passion
appreciation disrespect lust (i ) Here are some intransitive verbs which are followed
predilection contempt distaste by of:
mania weakness approve conceive dispose
FROM learn smell beware
consist dream partake
Here are some verbs which are followed by from: speak boast despair
ban detach graduate hear savour think
remove beg deter complain disapprove know
guard separate borrow smack weary
disconnect hide shelter (ii) Here are some transitive verbs which are followed
buy discourage inherit by of:
shield change dissuade
absolve avail cure
insulate stop come
make accuse cheat
disqualify keep switch
denude notify acquit
conceal elicit obtain
cleanse deprive purge
translate convert fall
advise convict divest
warn assure convince (vi) Here are some verbs which are followed by on:
inform advise elaborate muse
(iii) Here are some adjectives which are followed by remark agree expand
of: ponder report comment
afraid beloved contemptuous lecture pronounce speculate
expressive appreciative bereft decide meditate reflect
critical fond apprehensive vote dwell
capable devoid forgetful OVER
ashamed certain empty
free aware characteristic (i) Here are some verbs which are followed by over:
enamoured full bare argue fight muse
conscious envious guilty brood fret quarrel
ignorant intolerant scared clash fuss quibble
uncertain impatient jealous disagree gloat row
sceptical unconscious incapable enthuse grieve wrangle
nervous scornful unsure
(ii) Here are some nouns which are followed by over:
independent oblivious short

K
unworthy indicative possessed anxiety debate outcry
unafraid wary insensible battle fuss quarrel
proud unware worthy concern misunderstanding row

ON THROUGH

(i ) Here are some transitive verbs which are followed Here are some verbs which are followd by through:
by on: browse glance look
base confer impose thumb flick go
lavish spend blame run wade flip
force inflict model leaf skim work

KUNDAN
(ii) Here are some intransitive verbs which are followed TO
by on:
bet count gamble (i) Here are some verbs which are followed by to:
intrude build depend beckon give point
impinge lean capitalise signal demostrate offer
embark impose plan show wave announce
check encroach improve explain say apologize
prevail choke focus mention speak complain
infringe rely concetrate mutter talk confide
frown insist settle report write change
verge full return convert
(iii) Here are some nouns which are followed by on: promote turn
assault boycott embargo (ii) Here are some adjectives which are followed by
limitation attack burden to:
emphasis onslaught attempt courteous fair kind
claim focus restriction sympathetic cruel faithful
ban effect limit loyal unfair devoted
(iv) Here are some nouns which are followed by on: friendly nice unfaithful
disloyal indifferent partial
advice debate lecture unkind accustomed identical
remark agreement decision proportional vulnerable adjacent
legislation report book immune sensitive allergic
ideas outlook verdict impervious similar central
commit judgment indebted subordinate comparable
(v) Here are some verbs which are followed by on: inferior superior equal
adamant dependent fair irrelevant susceptible equivalent
hooked intent bent married unaccustomed essential
easy hard incumbent parallelled useful fundamental
keen preferable vital
WITH 3. Call

(i) Here are some verbs that are followed by with: at to visit somebody's place
ally confer merge for to demand
amalgamate conspire negotiate in to send for
associate dance share on to go to somebody's house to
chat debate speak Call meet him
coexist discuss trade off to postpone
collide interact work over to call out out the rolls
argue disagree row up to remember
bargain feud struggle upon to plead
clash fight vie 4. Die
compete haggle wrangle
of disease, illness, hunger, thirst, grief
(ii) Here are some adjectives which are followed by f rom a wound, over-eating
with: f or one's ideals, one's country
alarmed furnished overloaded Die in an accident, battle
suffused cluttered heavy by violence, one's own hands

K
painted swamped crawling (ie, commit suicide)
infested patterned teeming through neglect
crowded inlaid peopled 5. Go
thick draped inaden
piled tinged embellished af ter to chase
littered riddled filled against to oppose
overgrown studded angry Go f or to chase
displeased happy satisfied into to look into
annoyed dissatisfied impressed of f to explode
unhappy bored fascinated
6. Fall
infatuated unimpressed content

KUNDAN
fed up obsessed unsatisfied out to quarrel
disapoointed furious pleased back on use as a last resort if the
acquainted conversant impatient main thing fails
occupied afflicted engaged into to come into close contact
incompatible parallel comparable Fall in with meet by chance
faced inconsistent patient on pounce upon
compatible familiar infected upon come to pass
popular confronted frank through become unsuccessful
intoxicated unacquainted conneted off to be shed
free level unconnected
consistent friendly mixed up 7. Get
unfamiliar at to know
along to enjoy the company
Summary away to escape
Look at the charts given below: Get back to return
1. Break down to leave the train, bus or taxi
on with to maintain good relation
down of taxi, car, bus over to surmount
into a house
Break off relation 8. Look
up a meeting, class, etc after someone
with someone at see intently
2. Bring back upon recall
Look down upon dislike
down cause to fall into examine, investigate
f orth produce on consider
Bring f orward through study
produce, supply
out publish
9. Put 12. Turn
at lodge and entertain
across down reject
convey
aside on switch on
save Turn
forward out come out for special purposes
advance
Put in up arrive
submit
off delay, postpone
on wear Look at these charts, too:
out extinguish
(1)
10. Run
in an armchair, the car
away steal and disappear
She is at a table, a desk
back over reconsider
sitting on the chair, the ground, the floor,
down become weak or exhausted
af ter a horse, that seat
try to catch
Run of f make copies of

K
out come to an end (2)
over knock down and pass over
up against come car
face, encounter
go bus
11. Take travel train
by
boat
back withdraw plane
down write down bicycle
Take off deduct
up absorb, soak

KUNDAN
512 Test of English Language

Chapter 18

Sentence Arrangement

Exercise-1
Directions: In each of the questions below four To start something= to begin someting from the
sentences are given which are denoted by A, B, C from scratch very beginning without advantage
and D By using all the four sentences you have or preparation, especially when
building or devel oping
to frame a meaningful para. The correct order of
something
the sentences is your answer. Choose from the Spectacular = impressive or extraordinary
five alternatives the one having the correct order 4. (A) They think that India will disintegrate like
of the sentences. the Soviet Union or Yugoslavia.
1. (A) It was with this invincible spirit that Netaji (B) What will be the exact shape of India in 2000

K
opposed Wavell’s offer. AD can only be a matter of surmise.
(B) “Japan’s surrender is not India’s surrender,” (C) On the contrary, the blind patriots foresee a
he said. very bright future for India.
(C) The revolutionary spirit of Netaji was never (D) The prophets of doom say that the future of
dampened even after the surrender of Japan. India is doomed.
(D) He knew that a war of liberation demanded 1) BCDA 2) BDAC 3) DABC
great spirit, great sacrifice, courage and 4) DBAC 5) BCAD
patience. Surmise = a guess; guessing
1) BCDA 2) BCAD 3) CBAD Foresee = to predict something
4) DCBA 5) CBDA Doom = to cause somebody/something to
Invincible = too s trong to be overcome or suffer inevitable death destruction,
defeated failure etc

KUNDAN
Dampen = to make something less strong 5. (A) I wish I had more time, so that I could visit
2. (A) But all work is not education. the odd nooks and corners of India.
(B) In India, a majority of our people do hard (B) And yet I have not seen many parts of the
work, strenuous physical work, but all are country we love so much and seek to serve.
not educated. (C) Our own country is a little world by itself with
(C) It aims at concrete and objective realization an infinite variety and places for us to
of the ideas and is of great educative value. discover.
(D) “Work” is that activity of man which has a (D) I have travelled a great deal in this country
definite objective. and I have grown in years.
1) DCAB 2) BCDA 3) BACD 1) DCBA 2) DBCA 3) ADBC
4) DBCA 5) CBAD 4) CDBA 5) CBAD
Strenuous = requiring great effort or energy Every nook and corner = every part of a place;
Concrete = definite; positive everywhere
3. (A) Hari Prasad Nanda is one such person who A great deal = much; a lot
worked his way to the top from the scratch. 6. (A) By incre asing the se conne ctions the
(B) A few of them had a spark of proved adventure intelligence is also increased.
and their initiative, dedication and sincerity (B) Classical music has been proved to be very
brought them spectacular success. helpful in child development.
(C) The partition of India into India and Pakistan (C) Many doctors now a days use it for
made a numbe r of migrants to India therapeutic purposes which are based upon
penniless. the findings of this research.
(D) He rose to be come a first-ge ne ration (D) The research has demonstrated that listening
entrepreneur with the second largest complex to classical music increases the rate of
to his credit. nervous connections in brain.
1) ADBC 2) ADCB 3) CBAD 1) BDCA 2) DBAC 3) ACBD
4) CBDA 5) BCDA 4) BDAC 5) BCAD
Sentence Arrangement 513

Nowadays = at the present time, in contrast with 10. (A) Their invention has been proved to be a boon
the past to the society since many diseases caused
Therapeutic = of or connected with healing; having by the microbes could be cured by these
a good general effect on the body or
antibiotics.
the mind
Demonstrate =to show something clearly by giving
(B) The scientists, therefore, face a continuous
proof or evidence challenge to keep inventing newer drugs to
7. (A) This right, however, comes along with the counter this problem.
responsibility which the press is forced to (C) A major hurdle in the use of antibiotics,
work with. howe ve r, is that the microbe s de ve lop
(B) It means that the pressmen have every right resistance to the antibiotics, thus rendering
to expose matters re lated to public and these ineffective.
national interest. (D) Antibiotics are chemical substances produced
(C) Such re sponsibilitie s include not by microbes which are capable of inhibiting
broadcasting matters related to national the growth of other microbes.
security and other sensitive issues. 1) BACD 2) CBAD 3) DABC
(D) Freedom of press is symbolic of democracy. 4) DACB 5) ABDC
1) DBCA 2s) DBAC 3) ABCD Boon = a thing that is good or helpful for
somebody; a benefit; an advantage
4) BDAC 5) BCDA
Microbe = a tiny living thing that can only be
8. (A) A popular example of this damage is the Taj seen under a microscope
Mahal, whose marble has been corroded due Render = to cause somebody/something to be
to acid rain. in a specific condition

K
(B) It changes the acidic content of water bodies, Inhibit = to restrict or prevent a process or an
thus affecting the lives of the living organisms action
of this habitat. 11. (A) Goa is no exception.
(C) Acid rain has been a major factor responsible (B) Nevertheless, what has just transpired in the
for the degradation of the environment. state, le ading to the dissolution of the
(D) Along with affecting the living beings, it is asse mbly whe n its te rm was le ss than
also responsible for the corrosion of several halfway done, borders on the bizarre.
heritage buildings, thus causing irreparable (C) The country is used to receiving unusual
damages to them. political reports from states with small
1) CBAD 2) CDBA 3) BADC legislatures, mainly concerning defections
4) BDAC 5) CBDA that make it easy to change the colour of
Corrode = to destroy something sl owly, governments.

KUNDAN
especially by chemical action (D) Sensing that his game might be up, the chief
Habitat = the natural environment of an ministe r hurrie d to the gove rnor and
animal or a plant recommended the dissolution of the House.
Corrosion = the action of destroying something
1) ABCD 2) DCBA 3) CABD
slowly, especially by chemical action
Heritage = things such as works of art, cultural
4) BDCA 5) DABC
achievements and customs that Transpire = to become known; to prove to be so;
have been passed on from earlier to happen
generations Nevertheless = In spite of this; however; still
Bizarre = very strange; not at all logical
9. (A) Along with missing their education, these
12. (A) It has intensified patrolling in key areas and
children also face life-threatening dangers by
is keeping a close watch on rowdy elements.
working in hazardous chemical factories.
(B) The communally sensitive city has been
(B) Child labour has been interfering with the
largely peaceful, barring stray incidents.
education of millions of children across India.
(C) Amid the frenzy that has gripped Gujarat,
(C) However, there is a dire need to reinforce
Surat is an island of sanity.
these laws strictly throughout the country.
(D) However, the police does not want to take
(D) Many laws have been framed in order to curb
any chances.
this evil.
1) ADBC 2) DACB 3) BCDA
1) BADC 2) CBAD 3) DBAC
4) CBAD 5) CBDA
4) BDAC 5) BACD
Patrol = to go round an area to check that it
Vitally = extremely
is secure or safe and that there is
Hazar dous = dangerous; risky
no trouble
In dire need = extreme
Rowdy = creating a lot of noise and disorder
Reinforce = to cause a process to continue or
Stray = separated from a group; not in the
increase in intensity
right place
Curb = to restrain something; to prevent
Amid = in the middle of something; during
something from getting out of control
something, especially something
Dire = terrible; very serious or urgent
that causes excitement, alarm etc
514 Test of English Language

Frenzy = a s tate of extreme excitement; (D) For the first time in so many years a broad
extreme and wil d activity or consensus has been achieved, across the
behaviour political spectrum, on the direction and pace
Sanity = the state of having a normal healthy
of economic reforms.
mind; the state of being sensible or
reasonable; good sound judgement
1) CBAD 2) ABCD 3) BDAC
13. (A) Those who conduct the poll surveys defend 4) ADBC 5) CBDA
Albeit = although
their predictions saying there is a ‘slight
17. (A) A person living a totally worldly life is like a
permissible’ error up to two per cent.
line which is not a tangent.
(B) They only mislead the already confused
(B) The world we live in is like a limited size curve.
voters.
(C) He has no rules of living and follows a path
(C) But in the present era of close results, this
which suits him at a particular point of time.
is a big deviation which can tilt the whole
(D) In other words, he lives a directionless life.
predictions for some party.
1) ABCD 2) BACD 3) DCBA
(D) The results of the recent assembly elections
4) CDAB 5) DABC
have once again proved that poll surveys and
18. (A) Be it floods, cyclones, drought or deaths due
exit polls are highly unreliable.
to starvation, this state has witnessed them
1) DBAC 2) ACDB 3) ADBC
all.
4) DCBA 5) BCAD
Tilt = to move or make something move
(B) Ironically, whenever a calamity befalls the
into a sloping position state, things are given a political colour.

K
14. (A) There is a need for reformation in Muslim (C) People of Orissa have a bizarre fate.
education and in the thought process of the (D) The state is prone to all kinds of natural
community. disasters.
(B) Prophet Mohammed said that if in acquiring 1) CDAB 2) DCBA 3) ABCD
knowledge one had to travel to the distant 4) CADB 5) BADC
land of ‘Chin’, one should do so. Starvation = suffering or death caused by lack of
food.
(C) The basic teachings of Islam, as emphasised
Calamity = an event that causes great harm or
in the Quran, state that knowledge wherever damage; a disaster
found should be acquired by the Muslims. Befall = to happen
(D) What he intended was that Muslims should Bizarre = very strange; not at all logical
acquire modern knowledge. Prone = likely to suffer from, do or get
1) ABCD 2) DCBA 3) ACDB something

KUNDAN
4) CDAB 5) ACBD 19. (A) But for a poor country like India, corruption
15. (A) They epitomise a people’s civilisation that is a matte r which vitally touche s the
goes back to the dawn of history. developmental process.
(B) The Himalayas have nurtured this land with (B) Existence of rampant corruption in carrying
life -sustaining wate r and e nable d a out the works of deve lopment is wide ly
civilisation to blossom. acknowledged.
(C) If these majestic mountains were not there, (C) In a wealthy country, corruption may be only
the rain clouds sweeping up from the Indian a moral issue.
Ocean would have passed over our sub- (D) So, there is an urgent need to root out this
contine nt into Central Asia, leaving it a evil from the system if we need to see a
burning desert. developed India.
(D) The Himalayas are not merely a geographical 1) ABCD 2) DCBA 3) BCAD
feature, a mere chain of mountain ranges. 4) BADC 5) CABD
1) ABCD 2) DCBA 3) CDBA Rampant = existing or spreading everywhere in
4) ACBD 5) DACB a way that cannot be controlled
Epitomise = to be a perfect example of Carry out = to do something as required or
something specified; to fulfil something; to
Dawn of something = the beginning or first sign of perform or conduct an experiment
something Root out = to f ind and destroy or remove
Nurture = to help the development of something completely
something 20. (A) If some step is not taken to check this trend,
16. (A) Albeit slowly, but things have started to move. only God knows what will be the fate of this
(B) The last decade has been a landmark of our country in coming years.
economy in more ways than one. (B) Every political party is responsible for playing
(C) New perspectives have emerged — the New with the spirit of democracy by giving tickets
Agricultural Policy and the Disinvestment to law-breakers in elections.
formula, being the cases in point. (C) People with criminal records are finding easy
Sentence Arrangement 515

to win elections and enter into the power 25. (A) Although the Constitution provides for the
circle. removal of a judge through impeachment, it
(D) Criminalisation of politics has become a is silent on deviant behaviour not amounting
grave concern for our democracy today. to proven misbehaviour.
1) ABCD 2) DCBA 3) ADBC (B) It has been left to Parliament to decide on a
4) CBAD 5) BDAC case-to-case basis.
Grave = serious and important (C) The issue of disciplining the judges is, of
21. (A) Jesus wanted his followers to live like light. course, a tougher one.
(B) It is not meant to change a situation or a (D) But no laws have be en enacte d by the
person. Parliament for investigating misconduct of
(C) Light is that which when given, enables that judges.
which is already present to be recognised. 1) ABCD 2) DCBA 3) CDAB
(D) Light has no content. 4) BCAD 5) CABD
1) ADBC 2) DCBA 3) ABCD Impeach = to accuse a public official or politician
4) CBAD 5) BCDA of committing a serious crime.
Enable = to make soembody abl e to do Deviant = different in moral or social standards
s omething by giving them the from what is considered normal or
necessary authority or means acceptable
22. (A) The techniques and theories are many but 26. (A) A comparison with China is also irrelevant.
the aim is one — to lead us to a non-dualistic (B) The apprehension of a slowdown is perhaps
state of mind where we feel one with our exaggerated.
Creator. (C) It is too early to say what impact the latest

K
(B) Meditation is done to quieten the mind, to judgement will have on economic reforms.
control thoughts and to move on a path of (D) A de mocracy proceeds at its own pace,
higher consciousness. through the process of checks and balances.
(C) It generates tolerance, stillness in mind and 1) ABCD 2) DCBA 3) ACDB
a general feeling of well-being. 4) CDAB 5) CBAD
(D) It brings about an intimate feeling of oneness Apprehension = anxiety about something in the
with everyone and with the entire universe. future; fear that something will
be unpleas ant or that
1) ABDC 2) DABC 3) CADB
something unpl easant wil l
4) BCAD 5) CBAD happen.
Quieten = to become or make somebody/
27. (A) For the poor, the sick, the hungry, the
something less disturbed, noisy etc
homeless and the unemployed, peace is

KUNDAN
Bring about = to mike something happen
23. (A) One group of emotions makes us feel restless release from poverty, is healthcare, is a meal,
and uneasy, the other brings about balance is a roof, is a job.
and harmony. (B) It is now generally understood that peace is
(B) We label the two as positive and negative. much more than the absence of war.
(C) All emotions flow from two basic emotions — (C) What is not as universally acknowledged is
love and fear. how much more, and how the essence of
(D) It also encourages us to accept the happy peace beyond war varies critically from person
ones and reject the others. to person.
1) ABCD 2) DCBA 3) BCAD (D) There is no peace, it is true, without freedom
4) CABD 5) BCDA from fear, but there is no peace without
24. (A) Also, the police needs to be more cooperative freedom from want.
and encourage people to come forward with 1) BCAD 2) ABCD 3) CBDA
information. 4) DBAC 5) ADBC
(B) We must remember that a pre dominant 28. (A) It is human to find fault with others.
section of the minority community has (B) The prejudices thus acquired blunt our power
nothing to do with nefarious activities, but to reason and we become less receptive to
unforunately keeps quiet out of fear or a false new thoughts and ideas.
sense of brotherhood. (C) We get conditioned unconsciously through
(C) Bomb blasts can only be controlled by a subtle environmental influence.
vigilant public which notices and reports (D) We also tend to give our own interpretation
suspicious activities. to doctrines.
(D) The people in the largely minority localities 1) DCBA 2) CBAD 3) BADC
need to be extra-vigilant. 4) ADCB 5) BCAD
Prejudice = dislike or distrust of a person, group,
1) ABCD 2) DACB 3) CBDA
custom etc that is based on fear or
4) ADBC 5) BADC false information rather than on
Nefarious = wicked; morally bad; evil
516 Test of English Language

reas on or experience and that Heed = to pay attention to s omething/


inf luences ones attitude and somebody
behaviour towards them. 31. (A) However, the maxim does not apply to politics.
Blunt = to cause something to have less (B) This is the only way others with political
power or effect ambitions will learn a lesson about what they
Subtle = difficult to detect or describe; fine;
may or may not do after they have achieved
organised in a clever and complex
way
their targets.
Doctrine = a belief or set of beliefs held and (C) One must not hit a man when he is down.
taught by a church, a political party, (D) One has every justification to hit a politician
a group of scientists etc who has been toppled off his ministerial chair
29. (A) This is evident from the fact that there are for his misdeeds and cripple him till he can
parties which favour disinvestment in the never rise again.
states under their control while opposing it 1) ABCD 2) DCBA 3) CADB
at the Central level. 4) ACBD 5) DABC
(B) It is pity, though, that there is an element of Maxim = a well-known saying that expresses
dishonesty behind this rupture. a general truth or rule of conduct
(C) However, the earlie r agreements on the Topple = to cause somebody to lose their
position of power or authority
reforms process seem to have broken down
Misdid = a wicked act; crime
because of political compulsions. Cripple = to damage or harm somebody/
(D) It would obviously have been easier for the something seriously

K
government if a consensus existed among the 32. (A) Nor was there a banner proclaiming ‘mission
political partie s on the matte r of accomplished’ in the background.
disinvestment. (B) Only then did the world know that a US
1) ABCD 2) DCBA 3) CDAB President had paid the first-ever visit to Iraq.
4) BCAD 5) ACBD (C) Inste ad, he arrive d unannounce d in a
Rupture = An ending of friendly relations darkened Air Force One at Baghdad airport in
30. (A) In an ugly throwback to the dark decade of the midst of utmost secrecy and left two- and-
the 1980s, Assam is once more in the grip of a-half hours later.
an intense anti-immigrant frenzy. (D) President George W Bush wasn’t wearing a
(B) With elements from the extremist outfits flight suit this time.
active ly aiding and abe tting the mass 1) CABD 2) BDCA 3) ACDB
violence, the situation on the ground has 4) CDAB 5) DACB

KUNDAN
rapidly spiralled out of control. Proclaim = to announce something
(C) Across large parts of the state, armed mobs Accomplish = to succeed in doing something;
have taken to the streets, targeting the life to compl ete something
and prope rty of the Hindi-spe aking s uccess full y; to achieve
community. something
(D) While the state government can legitimately In the midst of = while something is happening
or being done.
accuse the Centre of not sending in enough
Ut most = greatest; most extreme
forces to put down the violence, it has only
33. (A) Thus democratic elections do not remedy the
itself to blame for not heeding the danger
criminalisation of politics since all parties
signals early enough.
now are replete with criminals.
1) DCBA 2) ACBD 3) CDAB
(B) There was a time when voters used to vote
4) DBCA 5) BDAC
out politicians suspected of crimes, but not
Throwback = a person or thing that shows the
characteris tics of somebody/
any more.
something in the past. (C) People vote on the basis of caste and religion
Frenzy = a s tate of extreme excitement; regardless of the criminal re cord of the
extreme and wil d activity or candidates.
behaviour (D) Since politicians remain out of jail regardless
Outfit = a group of people working together; of how much they embezzle or even murder.
an organisation 1) ADBC 2) DABC 3) BDCA
Abet = to help or encourage somebody to
4) ADCB 5) DCBA
commit an offence or do something
Replete = wel l provided or supplied with
wrong
something; filled with something;
Spiral = to increase rapidly and continuously
very full of food
Legitimate = that can be defended; reasonable;
Embezzle = to use money placed in one’s care
in accordance with the law or rule;
wrongly, especially so as to benefit
legal
oneself.
Put down = to stop or suppress something by
force
Sentence Arrangement 517

34. (A) So it needn’t cause any moral outrage that (C) Just as bad money drives out good money,
cricket superstar Sachin should ask the bad politicians have very nearly cleared the
government to waive the roadworthiness test political arena of good politicians committed
for his new Ferrari. to moral principles and values.
(B) Fearing that the state-of-the-art car is not (D) Through the ir dubious ways, the y are
compatible to Indian road conditions, he has distorting and redefining morality.
sought to avoid the Indian road test. 1) DCBA 2) CDAB 3) BACD
(C) Earlier , a hue and cry was raised over his 4) ABCD 5) CABD
asking for a duty waiver for the same car, Compatible = that can exis t together without
gifted to him by auto giant, Fiat. problems or conflict
(D) Excellence must be rewarded, only then will Canard = false report or rumour
Propagate = to spread an idea, a belief , a
more people be inspired to excel.
knowledge etc more widely
1) ABCD 2) DABC 3) BCAD Vitiate = to spoil the quality or reduce the
4) CDAB 5) CABD force of something
Outrage = a strong feeling of anger and Arena = a place or scene of activity or conflict
shock Dubious = not certain and slightly suspicious
Waive = to choose not to insist on about something; doubtful
something in a particular case, Distort = to give a false account of something
even though one has a legal or 37. (A) A freedom that is limitless has no meaning
official right to do so.
— the re can be no right without a
Roadworthy = fit to be driven on a public road
State-of-the-art = using the mos t modern or
corresponding duty.

K
advanced techniques or (B) Also, a distinction needs to be made between
methods freedom and licence.
Compatible = suited; that can exist together (C) However, we have not evolved a corresponding
without problems or conflict; awareness about the value of restraint and
that can be used together self-control.
Hue and cry = a general alarm or loud public (D) We are all acutely conscious of our right to
protest
freedom.
Waiver = a document that records the
waiving of a legal right, etc.
1) DCAB 2) ABCD 3) CABD
35. (A) The women’s reservation Bill continues to 4) BACD 5) BCDA
be stalled by almost all political parties. 38. (A) What is evidently needed is some criterion,
(B) And even when women have made it to the some test for determining the authenticity
top in politics, little is done to help other of a claim that any given political system is a

KUNDAN
women. democratic one.
(C) Though the overt reasons for doing so may (B) The very word ‘democracy’ has acquired
differ from party to party, the net result universal prestige, something it did not
remains the same: keeping women out of possess a hundred years ago.
power. (C) Nearly every country in the world either
(D) Indian politics has yet again paid just lip- claims to be democratic or to be on the way
service to the cause of women’s emancipation. to democracy.
1) CDAB 2) BCAD 3) CABD (D) But despite this general attachment to the
4) DACB 5) DBCA idea of democracy, there is clearly no general
Stall = to avoid giving a definite answer or agreement as to which political societies
taking action, in order to get more deserve to bear the name.
time; to delay 1) DABC 2) CBAD 3) BACD
Overt = done or shown openly or publicly; 4) CBDA 5) ABCD
not secret or hidden 39. (A) By contrast, our homes are often scenes of
Emancipate = to set somebody free especially from strife and rivalry.
political, legal or social restrictions
(B) So, it would be better for the sake of humanity
Lip-service (or give/pay lip-service
to something) = to say that one approves of or
if pace, harmony and divinity prevails on
supports something while not doing earth.
so in practice (C) It is easier to visualise God in the starry
36. (A) That politics and morality are not compatible heavens than in our own homes.
is a canard propagated deliberately by a few (D) The stars, so remote from our humdrum
who have come to dominate public life. earthly e xiste nce, suggest to our minds
(B) As a result, the entire social system and the infinite stillness, harmony and wisdom.
e nvironme nt is vitiate d—re sulting in 1) ABCD 2) DCBA 3) CDAB
wide spre ad corruption and une thical 4) BCAD 5) ADCB
behaviour.
518 Test of English Language

Strife = angry or violent disagreement; 1) ADCB 2 ) BDCA 3) DCBA


conflict 4) CDBA 5) CABD
Rivalry = competition between people Dialect = the form of a language used in a
wanting the same thing part of a country or by a class of
For the sake of = in order to help somebody/ people with grammar, words and
something or because one likes pronunciation that may be different
somebody/something f rom other forms of the s ame
Humdrum = lacking excitement or variety language
40. (A) Globalisation has led to openness in trade, Incarnation = a person or thing that strongly
investment and financial flows. displays a particular quality
(B) This asymmetry is neither an accident nor a Allegiance = support of or loyalty to government,
coincidence. ruler etc
(C) It also e xte nds to flows of se rvice s, At large = as a whole; in general
technology, information and ideas across 44. (A) We should judge and determine if our acts
national boundaries. are correct, whether they are appropriate
(D) But the cross-border movement of people is before the Lord who has granted us life.
highly restricted. (B) We should live in a manner that material
1) DCAB 2) ACDB 3) CDBA considerations and personal benefits don’t
4) BCAD 5) CABD matter.
41. (A) One major issue is the de bate around (C) Every one of us should find the time for
foreigners, especially whether we can have meditation and communion with the Almighty.

K
someone of foreign origin as leader of our (D) If we find we have acted properly, we should
country. fear no one.
(B) But as it shines across middle class, diasporic 1) DCBA 2) ABCD 3) CADB
culture, claiming that opportunity has found 4) ACBD 5) DACB
Communion = a group of people with the same
a ne w ge ography, one must look at its
religious beliefs
shadows.
45. (A) Therefore, all this breast-beating about the
(C) Packaged as an election issue, it hides more
few knocking advertisements that are being
fundame ntal proble ms of citize nship,
aired, taking potshots at our political leaders,
hospitality and history.
is needless.
(D) Shining India is now a fact of life.
(B) The right to information is the bedrock of a
1) ABCD 2) DCBA 3) BCDA
true democracy.
4) CBAD 5) DBAC

KUNDAN
(C) By the very fact that these politicians are in
Diaspora = the process by which people of a
particular nation become scattered
the public space and are seeking our mandate
and settled in other countries to represent us and rule this country for the
42. (A) At best, it has had one or two dangerous next five years, we have the right to know
bowlers in an attack of four. both their pluses and minuses.
(B) Except for a brief spell in the 1970s, India (D) What is not spelt out but is implicit in this
has never had a match-winning bowling right is that the information can be both
combination. negative as well as positive.
(C) But this comes as no surprise to those who 1) ABCD 2) DCBA 3) CDAB
have followed Indian cricket over the last 50 4) DABC 5) BDAC
years. Bedrock = basic facts or principles
Mandate = the authority to do something, given
(D) India has one of the worst bowling attacks in
to a government or other organization
the world. by the people who support it.
1) ABCD 2) DCBA 3) CDAB Implicit = implied, but not expressed directly
4) BCAD 5) CBAD 46. (A) An individual’s decline begins with desire and
43. (A) Language, it was once said, is a dialect ends in misery.
backed by an army. (B) Animals take life as it comes, but when man
(B) De spite more than 50 ye ars of state fails to do a certain thing or to possess certain
sponsorship, neither Urdu nor Hindi, at least things, he gets angry and slips into mental
in its official incarnation, has gained the depression.
allegiance of the people at large. (C) To keep the mind happy, we have to learn to
(C) Yet, the history of ‘national’ languages in the be content with minimum wants.
region tells a very different story. (D) Human beings are the only species in God’s
(D) On the face of it, this is perhaps more true creation who make an issue out of nothing.
of the Indian subcontinent than any other 1) BACD 2) DCBA 3) ABCD
part of the world. 4) DBAC 5) ACBD
Sentence Arrangement 519

47. (A) There are times in history when political Tout = to offer or propose somebody/
formations need to look beyond ideological something in the hope that people
strait-jackets. will believe or accept them/it
Lopsided = with one side lower, smaller etc than
(B) Such junctures demand imaginative reading
the other; not evenly balanced.
of the present political moment and a break
51. (A) On the contrary, the new states have been
from rehearsed responses.
successful in containing socially divisive
(C) It now needs to decide whether to be part of
tendencies by taking on board the interests
a ruling dispensation in New Delhi or not.
of hitherto marginalised sections of the
(D) Poll 2004 has place d be fore the Le ft a
population.
challenge that it had once refused to take
(B) The process needs to be encouraged.
on.
(C) This has led to a more equitable distribution
1) BDCA 2) DABC 3) CDAB
of political powe r as we ll as e conomic
4) ABDC 5) BCDA
resources.
Straitjacket = a thing that stops or restricts growth
or development
(D) There is no empirical evidence to suggest that
Juncture = at a particular, especially important, creation of new states has led to any increase
stage in a series of events in social tensions.
Rehearse = to give a list of things, especially 1) BACD 2) ACBD 3) CDAB
things that have been mentioned 4) BCDA 5) DACB
many times before On the contrary = (used at the begining of a clause
Dispensation = permission to do something that or sentence to emphasise) that
is not normally allowed what follows is true, and it is

K
Take on = to accept opposite of what was said
48. (A) The be st option the n is to e ngage previously
constructively with the regimes and promote Contain = to prevent s omething f rom
democratic institutions. spreading in a harmful way and
(B) Democracy cannot be foisted on any society becomeing more serious
Take som ethi ng on board = to accept a
from the outside.
responsibility, etc; to
(C) Neither can such states be wished away. understand and appreciate
(D) No amount of external pressure can force something fully.
regimes to turn democratic, as is evident in Hitherto = until now; until a particular time
the case of China and Myanmar. Equitable = fair and just; reasonable
1) CABD 2) ABCD 3) BDCA Empirical = based on obs ervation or
4) DACB 5) CDAB experiment, not on theory

KUNDAN
Regime = a method or system of government 52. (A) Children in most schools are packed like
Foist (foist something on/upon sardines into dingy classrooms and spend
somebody) = to force somebody into accepting most of their time jostling for space instead
something that is not wanted of learning.
49. (A) It has been seen that illiterate people are (B) Education is not just about imparting
basically guided by emotions. literacy; it is about developing a well-rounded
(B) Emotions are the main motivating factor for personality in the child.
most people. (C) This has contributed to stress, lack of interest
(C) Hence it is important to counter this with a in le arning and othe r physical and
broad-based education system that is both psychological disorders.
scientific and spiritual. (D) This is difficult to achieve in today’s crowded
(D) This is especially so when illiteracy is high. classrooms which have made learning a
1) ABCD 2) BDAC 3) BCDA joyless experience.
4) CABD 5) CDAB 1) DABC 2) CDAB 3) BDAC
50. (A) Independent statehood is touted as a remedy 4) ADBC 5) ADCB
for such lopsided development. ike sardine = pressed tightly together in a way
(B) With state governments according unequal that is uncomfortable or unpleasant
priority to different regions within a state, Dingy = dirty and dark; not cheerful or bright
some parts are inevitably neglected. Jost le = to compete with other people in a
(C) The principal argument in favour of smaller forceful manner in order to gain
something; to push roughly against
states is that of regional imbalance.
somebody usually in a crowd
(D) But there is little evidence to suggest that Impart = to give a quality to something
the creation of smaller states has actually 53. (A) If it were so, the one billion-plus and still
resolved the issue. growing population, overcrowded metros and
1) CBAD 2) DABC 3) BDAC schools would have by now led to a crisis,
4) DBCA 5) ADBC triggering regression.
520 Test of English Language

(B) That hasn’t happened. Inoculate = to inject a person or an animal with


(C) We continue to grow — in numbers as well a mild form of a disease as a way of
as in terms of income per head and literacy preventing them/it catching the
disease itself.
levels.
57. (A) In a TV set an echo within the crystal is used
(D) In India, lack of space has never majorly
to set the timing for the electron beam that
deterred achievers.
scans the screen.
1) CABD 2) BCAD 3) ABCD
(B) Toshiro Higuchi and his colleagues at the
4) BADC 5) DABC
Regression = the process of returning to an earlier
Department of Precision Engineering at the
or less advanced form or state University of Tokyo built their motor using a
Deter = to make somebody decide not to do sliver of lithium niobate 60 millimetres long
something and 15 millimetres wide.
54. (A) The re are still two we eks to go for the (C) But when these waves bounce along the
American presidential election. surface of a crystal small objects on top will
(B) Our newspapers and TV channels are full of ‘surf’ along with them.
Kerry-Bush debate. (D) Applying a small voltage to a piezoelectric
(C) But the kind of media attention it is attracting crystal flexes and stretches its atomic lattice,
in India, makes it appear as if India is creating a surface acoustic wave.
America’s 51st state. 1) ABCD 2) DBCA 3) ADCB
(D) Every statement and its implication has been 4) CDAB 5) DACB

K
discussed threadbare. 58. (A) The name malaria itself has its origin from
1) ACBD 2) BACD 3) DACB ‘mal air’ conveying ‘foul air’.
4) CBAD 5) DBCA (B) In those days malaria was considered a
Thr eadbare = not adequate or effective; old and mysterious scourge on mankind causing
worn thin untold havoc over centuries.
55. (A) It is only a means giving a fair chance to (C) No one knew its cause and how it spread.
those who have been discriminated against (D) Rather, it was widely believed, the disease
for centuries. was transmitted through a foul, poisonous
(B) The myth that reservations will bring doom gas known as Marsh Miasma.
to the private sector is being deliberately 1) ADCB 2) BCDA 3) CDBA
propagated. 4) ABDC 5) BACD
(C) Instead of lowering the cut-off percentage for Scourge = a person or thing that causes trouble

KUNDAN
the reserved categories, a large number of or suffering
scholarships should be made available to Untold = too many or too much to be counted,
them. measured etc
(D) Reservation does not mean employing the Foul = an act or a piece of play that is
against the rules of game; dirty and
incompetent.
smelling bad.
1) CDAB 2) BDAC 3) ADCB
59. (A) If there is a large problem that fits ESs, then
4) DCAB 5) CBAD
perhaps this is the approach to be followed.
Di scrim inati on = treating a person or group
differently (usually worse) than
(B) Many companie s choose to build large
others. systems which usually must be run on
Myth = a thing or person that is imaginary mainframe computers.
or not true (C) The benefits of large systems are that the
Doom = death or ruin; any terrible fate savings can be large.
Propagate = to spread an idea, a belief , (D) The downside to building large systems is
knowledge, etc more widely that they require special programmers to
56. (A) That is because all vaccines lose their ability build.
to provide protection against disease over a 1) ADCB 2) BDAC 3) CABD
period of time. 4) BCAD 5) ADBC
(B) Vaccines have saved millions of lives and vast Downside = a negative aspect; a disadvantage
sums of money have gone into preserving the or set of disadvantages
inoculations at low temperatures. 60. (A) Chemical reactions are about to become
(C) The loss of potency becomes quicker when easier to control, as German researchers have
they are exposed to high temperatures. built a laser that ‘learns’ to direct them.
(D) In order to maintain their quality, they must (B) But chemists seldom know the energies of
be stored at the appropriate temperature from bonds well enough to tailor the laser for the
the time they are manufactured right up to task.
the moment of use. (C) Now the team has built the first working
1) DBCA 2) CDAB 3) ACDB system that relies on the feedback to find
4) BACD 5) CBDA the right laser pulse for the job.
Sentence Arrangement 521

(D) By exciting specific bonds, precisely shaped Log = to enter information in an official
laser pulses can split large molecules into written record.
desired products. Optimise = to make something as good or as
favourable as possible
1) ADBC 2) BCDA 3) DBAC
Nasty = painful; severe
4) DABC 5) BDAC
64. (A) And if one has more products one will have
Tailor = to make or adapt something for a
particular purpose, person or type
to forecast which market, which city, which
of person. shop and when they will need a particular
61. (A) A carbon crystal that might eventually be used brand and a particular size.
to make optical circuits (which use light (B) The focus of most CPG companies used to
beams instead of electric currents) has been be first to get the manufacturing right.
de ve lope d by an Ame rican e le ctronic (C) There are lots of issues around the supply
company. chain management side.
(B) They then chemically removed the gemstone, (D) Once the y achie ve this they go for the
leaving the carbon in a form they dub ‘inverse distribution right.
opal’. 1) CABD 2) CDBA 3) CBAD
(C) The resulting crystal can strongly diffract 4) ADBC 5) BDCA
visible light, a quality the researchers have 65. (A) Moreover, the anti-venom contains a mixture
exploited to build an optical switch. of fore ign prote ins and cause s alle rgic
(D) Researchers took a a synthetic version of the reactions in many patients.
gemstone opal and filled the spaces in its (B) Antibodies are then harve ste d from the
animals’ blood.

K
lattice with carbon.
1) BCDA 2) ABCD 3) ADBC (C) Until now, anti-ve nom for neutralising
4) CDBA 5) DACB snakebite toxins has been made by injecting
62. (A) Dobson’s conclusions, they contend, are a horses or sheep with small quantities of
stretch based on highly circumstantial venom to produce an immune reaction.
evidence and at odds with evolutionary (D) However, a single dose of anti-venom costs
biology. around $ 15 and several may be needed to
(B) “We sometimes have to rock the boat,” said treat each patient.
Dr Karl W Butze r, a ge ographe r at the 1) ABCD 2) DCBA 3) BADC
University of Texas at Austin. 4) CDAB 5) CBDA
(C) Pale ontologists who spe cialise in 66. (A) This result is reported in the October 16th
Neanderthal rese arch have raised sharp issue of Science.

KUNDAN
objections. (B) Antarctica is not shrinking according to a
(D) But some anthropologists and othe r report from the European Space Agency (ESA).
geographers said that the data see me d (C) The same investigation provides evidence that
impressive and that the inte rpretations one part of West Antarctica may be rapidly
should be taken seriously. losing its ice to the ocean.
1) ABCD 2) ABDC 3) CADB (D) A team of British, Dutch and Ame rican
4) BDCA 5) CABD scientists, led by Prof Duncan Wingham at
63. (A) Call monitoring features are useful in terms the University College, London, based the
of security, but also enable tracking of call findings on ERS (European Remote Sensing)
volume and logging of all connections so that satellites’ data collected over five years.
administrators can optimise the number of 1) CABD 2) BDAC 3) ACDB
ISDN lines ordered. 4) DBCA 5) CDBA
(B) Call monitoring is also an important element 67. (A) Storage of cadaver organs for transplant
of any ISDN data communications solution. re mains a pe re nnial proble m and is
(C) Given that ISDN costs are ofte n usage- influenced by legal, ethical and other issues.
related, this checking and recording also (B) The function of the organs must be protected
serves to prevent any nasty surprises that before and during procurement from the
user might receive with the monthly phone donor.
bill. (C) Organ transplant for end-stage disease is well
(D) At the same time, usage logs can provide accepted worldwide.
managers with the justification required to (D) The supply of organs from heart-beating dead
add ISDN lines as the need for additional donors is inade quate to mee t the total
bandwidth arises. demand.
1) ABDC 2) BACD 3) DABC 1) CADB 2) CBAD 3) BACD
4) DACB 5) ACDB 4) CDAB 5) ADCB
Cadaver = a person’s dead body
Perennial = constantly occurring
522 Test of English Language

68. (A) The remarkable point has been made in this 1) CADB 2) CABD 3) ADBC
context that language owes its origin to art. 4) DCBA 5) ABDC
(B) Both art and language involve symbols that Hitherto = until now; until a particular time
have some kind of “socially shared meaning”. Dev oi d of som et hi ng = without s omething;
(C) Dr Ian Tattersall of the American Museum of completely lacking in something
Scorching = very hot
Natural History in New York is quoted as
72. (A) Fallow condition of the land favours the loss
saying: “Empathy, intuitive reasoning, and
of valuable top soil by water erosion during
future planning are impossible without
monsoon by run-off and loss of soil moisture
language.”
by evaporation during dry periods.
(D) Art is symbolic representation, and a society
(B) This makes inter-culture operations difficult.
that indulged in art must have had a language.
(C) This favours weeds and harbours pests and
1) BDCA 2) CDAB 3) BADC
diseases.
4) BCAD 5) ADBC
Empathy = the ability to imagine and share
(D) Most of the farmers keep the land fallow up
another person’s feel ings , to the bearing period.
experience etc 1) ABDC 2) CDBA 3) DCBA
Intuitive = of or coming from intuition 4) ACBD 5) CDAB
Indulge = to all ow oneself to enjoy the Fallow = that has been dug but then left
pleasure of something without crops being planted on it,
69. (A) And for good reason: Experts cannot seem to in order to allow essential chemical
agree, either. elements, etc to increase in it.

K
(B) For example, the biomechanics laboratory at Harbour = to contain and allow something to
develop
Ce ntine la Hospital Me dical Ce ntre in
73. (A) The lesions are small, smooth, later enlarge
Inglewood, Calif., completed a study this year
and become slightly depressed.
comparing the effectiveness and safety of
(B) Phytopthora blight produce s diffe re nt
eight popular abdominal movements.
symptoms on affected plants, depending on
(C) Even the research is not definitive.
the age and the plant part affected.
(D) Exercisers are often confused about the best
(C) Initially, purple to dark necrotic lesions girdle
way to strengthen and tone up the midriff.
the basal portion of the stem and later occur
1) ACBD 2) DACB 3) BCDA
on the aerial parts of the seedlings.
4) DBCA 5) DABC
Midriff = the middle part of the human body,
(D) The seedling infection commences with the
onset of heavy rains during July-August.

KUNDAN
between the waist and the chest.
Tone up = to make one’s body stronger, fitter 1) BDCA 2) DCBA 3) ADCB
etc 4) ABCA 5) CDBA
70. (A) There is significant variability in seedlings Lesion = a wound; an injury
raised through seeds. Blight = any plant disease
(B) Small cardamom, the queen of spices, enjoys Necrotic = adjective of ‘Necrosis’.
Necrosis = the death of most or all of the cells
a unique position in the spices market.
in an organ or tissue due to disease,
(C) Clonal multiplication and tissue culture injury, or failure of the blood supply.
method of propagation are advantageous for Girdle = to surround something
true-to-type and uniform crop. Aerial = existing or suspended in the air
(D) It is propagated through seeds and rhizomes. Onset = a beginning, especially of something
1) ACDB 2) CDBA 3) BDCA unpleasant.
4) CADB 5) ADBC 74. (A) It is de sirable that political partie s
Seedling = a young plant newly grown from a themselves develop a code of conduct that
seed will sustain the level playing field.
True to something = being or acting as one would (B) Any code should evolve more as a healthy
expect from something convention having the force of a statute.
71. (A) Bananas cultivated in the Cauvery delta have (C) It is said that conduct cannot always be laid
begun to develop severe salt injury. down by the law.
(B) The wetland tracts hitherto considered as (D) As regards giving statutory status to the
devoid of such problem have now shown signs model code, some have opposed it.
of salt injury by way of marginal scorching 1) ABCD 2) DCBA 3) BDCA
characterised by yellow lining. 4) CADB 5) ACBD
(C) The lower leaves get dried up at a faster rate 75. (A) Social attitudes sometimes lag behind social
and the number as well as the size of the realities in a period of transition.
leaves showed a decline. (B) Perhaps it is time for a fe w determined
(D) The scorching is so severe that the leaf area pushe s to be gin the first ste p of
gets reduced to even 50 per cent.
Sentence Arrangement 523

reconstructing social structures so that they system.


address modern needs. (C) This could be tolerated in the past but can
(C) The roles of the primary care-giver in a family be catastrophic in a nucle ar we apons
ne e ds to be re de fine d to include male environment.
members. (D) The Indian security system with its colonial
(D) If the social superstructure does not reflect structures is quite unsuited to manage the
the current needs then it has to be changed. de mands of de fe nce in strate gic and
1) ADCB 2) ACDB 3) BDCA operational terms.
4) BCDA 5) ABDC 1) ACBD 2) DBAC 3) CBAD
Lag behind = to go too slowly; to fail to go as fast 4) BCAD 5) CDBA
as others Prone = likely to suffer from
76. (A) A close integration in the working of nations Systemic = of or affecting the whole of the body
to achieve political and economic goals is only Catastrophic= a sudden great disaster
to be welcomed. Strategic = that gives an advantage in a war;
(B) It is as if a movement towards securing a forming part of a plan or an aim to
achieve a specific purpose or to gain
world vision is gradually gaining force.
an advantage
(C) The interaction between nations is greater
80. (A) Women had played a very active role in Iran’s
than it ever was.
Islamic revolution from the beginning.
(D) Today, the world is taking on the form of a
(B) It was more an imitation of western culture
global community.
than true freedom for women.
1) DACB 2) ABDC 3) DCBA
(C) However, these reforms were quite superficial

K
4) CDAB 5) ABCD
as women had no freedom to participate in
Take on something = to begin to have a particular
characteris tic, quality or
political activities.
appearance; to ass ume (D) The Shah had imposed modern se cular
something reforms and had abolished the veil and
77. (A) The imagination paints many of these as western dress was encouraged.
unknown terrors. 1) DCBA 2) CBAD 3) BADC
(B) Outside this circle is a vast darkness that 4) ADCB 5) CBDA
holds many mysteries. Abolish = to end the existence of a law, a
(C) The earliest maps were rough sketches of an practice, an institution etc
Veil = a covering of fine net or other usual
unknown world made by the wisest of our
trans parent material worn,
ancestors. especially by women, to protect or

KUNDAN
(D) The geographer of those days was like a hide the face, or as a part of hat, etc.
traveller who stands at night in an unknown 81. (A) Perhaps this will take a long time as it is
place holding a flickering lantern that casts very difficult to get together equals and to
a little circle of light near his feet. decide who is primus inter pares.
1) ABCD 2) DCBA 3) CDAB (B) In this, China and India have to play major
4) CDBA 5) DCBA roles not to mention Japan and the ASEAN.
Flicker = to move backwards and forwards (C) But as a first step, we can have a cooperative
lightly and quickly
community structure embracing certain parts
78. (A) One of the great delusions of modern times
of China adjacent to India, north Myanmar,
is the prevailing belief that we are more
Bangladesh and the Seven sisters of the
civilised than our ancestors.
Northeast.
(B) At no other period in the history of mankind
(D) In the coming years, India should pave the
has the word ‘civilisation’ been so often
way for an Asian Common Market like its
misused and misunderstood.
Europe counterpart.
(C) It is not in the abundance of material goods
1) DBAC 2) BCAD 3) CABD
alone that civilisation lies.
4) ACDB 5) BDCA
(D) Neither does it consist in the advance of To pave the way = to create a situation in which
scientific knowledge, however amazing it may somebody will be able to do
be. something or something can
1) BACD 2) CBAD 3) ACDB happen
4) BADC 5) ABCD 82. (A) But it has always been the other way round
Delusion = a false opinion or belief, especially for Indian participants who either lost their
one that may be a symptom of cool or are subjected to a sudden illness.
madness (B) Besides all these shortcomings, what is
79. (A) It has been prone to systemic failures. glaringly missing is the genuine government
(B) If it has retained substantial strength in the efforts.
tactical field, it is in spite of the slow moving
524 Test of English Language

(C) The Government of India should concentrate have be e n major re asons for its
more on de ve loping the ne ce ssary backwardness.
infrastructure for Indian sportspersons to 1) BCAD 2) ACBD 3) CBAD
acquire skills to compete in international 4) DACB 5) CDAB
events. Skew = to change something from its usual
(D) It is important that any sports participant in position or direction
big events like Olympics should have a poised 86. (A) The number of HIV-positive is increasing at
mind besides being physically fit at least an alarming rate.
during the days of his or her stay in the sports (B) Eve n with crore s be ing spe nt on AIDS
venue. aware ne ss programme s, we see m to be
1) BCAD 2) CDBA 3) ACDB making little headway.
4) CBAD 5) DABC (C) It is obvious that prevention and control
The other way round = in the opposite position or strategies have been ineffective.
direction; the opposite of what is (D) According to reports, there are around 50
expected or supposed million HIV-positive in the world.
Shortcoming= a fault, eg in somebody’s character, 1) DBAC 2) BACD 3) DCBA
a plan or a system; a defect 4) BADC 5) CABD
Glaring = bright in an unpleasant way; angry; Headway = progress, especially in difficult
fierce; that cannot or should not be circumstances
ignored; shocking
87. (A) It is the result of a long-drawn campaign by
Poised = in a state of balance; showing calm

K
self-control; in a state of physical
disabled rights activists.
tension, ready for action. (B) It is only because of a sustained campaign
Glaringly = shockingly and the judiciary’s pre ssure that the
83. (A) That is what health is all about — not illness executive has finally given in.
and hospitals. (C) The move to open up non-technical A and B
(B) On the contrary, it is concerned with the category posts in the civil service to the
well-being and ability to perform tasks in a disabled is commendable.
vibrant society. (D) The judiciary’s stand in this regard has also
(C) The word ‘health’ is often misunderstood and been encouraging.
linke d with hospitals and tre atme nt by 1) DABC 2) CADB 3) BCAD
medical doctors. 4) ABCD 5) CDBA
(D) After all, staying healthy is the most important Commendable = deserving praise

KUNDAN
thing. 88. (A) The Indian government’s response to post-
1) BACD 2) ACDB 3) DBAC election Iraq shows a new flexibility and
4) CBDA 5) CADB vibrancy in our foreign policy.
Vibrant = full of life and energy; exciting (B) The way India deals with Iraq can be used as
84. (A) In India, banking is the largest national a benchmark for future foreign policy.
instrument for socio-economic development. (C) An ability to react quickly is required today.
(B) The sure st way to e nsure that is to (D) As the geopolitical situation changes rapidly,
strengthen banks and professionalise their India cannot afford to have a rigid foreign
managements. policy.
(C) When banks fail, taking with them the money 1) ADCB 2) BCDA 3) CDAB
of depositors, confidence in the banking 4) DABC 5) CABD
sector slips and damages the investment 89. (A) What we should be worried about is the
climate. mismatch between the two.
(D) We need focussed and well-directed lending (B) Internal reforms are crucial to the long-term
to agriculture and industry; banks can and health of the Indian economy.
should play a crucial role in a growing (C) If the pace of internal reforms does not keep
economy. up with that of globalisation, it can ruin the
1) ABCD 2) DCBA 3) CABD kind of vibrancy that the Indian economy has
4) DABC 5) ADBC built for itself.
85. (A) Today it is arguably the most backward and (D) The quicker the speed of reforms at home,
under-developed state. the better chance we will have of integrating
(B) It gave the country leaders like Rajendra with the global economy.
Prasad and Jayaprakash Narayan. 1) DCBA 2) ADBC 3) BDAC
(C) In the early years of independence, Bihar was 4) ABCD 5) CBDA
one of the best-administered states. 90. (A) The fact that India is thinking of asking the
(D) Consistent neglect by the Centre in allocation US for frontline we apons is a positive
of funds and policies skewed against the state development.
Sentence Arrangement 525

(B) But the sum and substance of this (C) Only once after Independence did India grow
relationship has changed considerably over at above 7% in consecutive years — in 1994-
the years. 95, 1995-96 and 1996-97, when industrial
(C) This is a major foreign policy shift and a delicensing spurred an increase in capacity.
we lcome change from the country’s (D) We are expected to round off the current fiscal
overdependence. with 6.9% growth in GDP, despite a mere
(D) The former Soviet Union and its successor 1.1% growth in agriculture.
have be e n India’s principal armaments 1) ADCB 2) BCDA 3) CABD
supplier for over 30 years. 4) DBCA 5) BACD
1) CBAD 2) DCAB 3) BCAD Invariably = always
4) CADB 5) ACDB Spur = to stimulate somebody/something;
Armament = weapons, especially large guns, to encourage or be a reason for
tanks etc. somebody to act or make an effort
91. (A) It was America which made Saddam Hussein, 94. (A) Drugs are big business after all.
once its blue-eyed boy, fight eight-year proxy (B) This is because pharmaceutical companies
war against Iran. do not actively promote generic drugs.
(B) But once its interests were served, the US (C) Several generic drugs are better than brand
saw him as a threat to the region. name drugs.
(C) It is ironical that the US is now training its (D) Yet doctors largely prescribe brand name
gun on Iran. drugs.
(D) It finally hounded him out of office on the 1) BACD 2) CABD 3) DBAC
4) CDBA 5) ADCB

K
pretext of WMD.
1) ABCD 2) CABD 3) CDBA Generic = shared by or including a whole group
or class of things; not specific
4) BACD 5) DACB
Somebody’s blue-eyed boy = a person treated with
95. (A) But as long as adve rtising achieves its
special favour by somebody objective it is effective.
Proxy = a person who is given the authority (B) People classify advertising as good or bad
to act on behalf of another. depending on how it appeals to them.
To hound somebody out (of something/....) = (C) In the ultimate analysis there is only effective
to force somebody to leave and ineffective advertising.
something/a place, especially by (D) The re is no such thing as good or bad
making their life there very difficult.
advertising.
Pretext = an excuse; a reason given for doing
something that is not the real reason
1) ACBD 2) DBAC 3) ABCD

KUNDAN
92. (A) For democracy to succeed, the urge has to 4) CABD 5) BCDA
come from the people. 96. (A) Private property rights alone enable the
(B) But despite democratic elections, peace will freedom of expression.
be elusive in Iraq as it is in J&K. (B) However, in reality, this is a completely
(C) The American leadership urged people to meaningless right.
appre ciate the gre at task the US has (C) Whe re these rights are non-e xistent, or
undertaken to introduce democracy in Iraq. flouted by the authorities, the freedom of
(D) The Iraq elections are being touted as a great expression disappears.
democratic success. (D) Journalists are the first to de fe nd the
1) ACBD 2) CDAB 3) ABCD 'freedom of expression'.
4) DCBA 5) BADC 1) BACD 2) CADB 3) ABCD
Ur ge = strong desire or impulse 4) BDCA 5) DBAC
Elusive = tending to escape or disappear; Flout = to show that one has no respect for
difficult to find or capture; difficult something by openly refusing to
to remember obey it.
To urge = to try hard to persuade somebody 97. (A) The system of nature set up by God already
to do something rests on the basis of peace.
Tout = to offer or propose somebody/ (B) Peace is no external factor to be artificially
something in the hope that people imposed upon man.
will believe or accept them/it. (C) Peace is inherent in nature itself.
93. (A) For arguably the first time ever, India’s (D) If this syste m is not disrupte d, it will
economy looks both buoyant and stable. continue to stay the course set for it by the
(B) In earlier decades, the occasional high-growth Almighty.
year meant very little, as it was invariably 1) DACB 2) ACBD 3) BCAD
preceded by a year of very low or negative 4) DBAC 5) ADCB
growth. Impose = to try forcefully to make somebody
accept an opinion, a belief etc
526 Test of English Language

Inherent = existing as a natural or permanent (C) Besides, we do need proper price signals for
feature or quality of something/ everyday consumption, and fast.
somebody (D) It would summarily distort inve stme nt
Disrupt = to make it difficult for something to
decisions.
proceed eg by causing noise,
problems, interruptions etc
1) BDCA 2) DCAB 3) ADCB
98. (A) Which is why we need to promptly stop the 4) CABD 5) ABCD
Sheer = complete; nothing more than
ring-fencing of petro-product sales, and
Folly = a foolish or unwise act, idea or
thoroughly open up retailing. practice
(B) In a liberalising economy it is sheer folly to
provide sky-high protection for oil refining.

Exercise-2
Directions (Q. 1-5) : Rearrange the following (B) That rests with the judge, and it is ultimately
six sentences (A), (B), (C), (D), (E) and (F) in the for the judge to decide which side is right,
proper sequence to form a meaningful paragraph. and how justice should prevail.
Then answer the questions given below them. (C) When he was asked what he thought of an
(A) While doing so, we may also correct any advocate supporting a cause which he knew
distortions that we may discern. to be bad, Johnson’s answer was that the

K
(B) With all our experie nce and insight, we advocate did not know it to be good or bad till
should be able to visualize them well in the judge determined it for him and for others.
advance. (D) But, he must do so fairly, and without
(C) The celebration of the 50th anniversary of concealing from it anything that it is his duty
the country’s independence is a historic to divulge.
moment. (E) There is a belief that an advocate’s function
(D) Also, it is a time to consolidate on the gains consists, for the most part, of showing white
that we have made. as black and black as white.
(E) But, most of all, it is a time to gear up for the (F) He is, after all, the client’s mouthpiece, and
opportunities and challenges that lie ahead. he must put before the court all aspects of
(F) It is a time to introspect and evaluate what the case which are favourable to his client.
we have achieved in the last five decades. (G) The only answer that one can give to this

KUNDAN
1. Which of the following should be the FOURTH popular misconception is the famous answer
statement after rearrangement? that Johnson gave to Boswell.
1) E 2) D 3) C 4) B 5) A (H) But he is not concerned with the final result.
2. Which of the following should be the SIXTH 6. Which of the following will be the FIR ST
(LAST) statement after rearrangement? sentence in the passage?
1) A 2) B 3) C 4) D 5) E 1) C 2) E 3) F 4) G 5) B
3. Which of the following should be the SECOND 7. Which of the following will be the FIF TH
statement after rearrangement? sentence in the passage?
1) F 2) E 3) D 4) C 5) B 1) F 2) G 3) D 4) C 5) H
4. Which of the following should be the THIRD 8. Which of the following will be the SECOND
statement after rearrangement? sentence in the passage?
1) B 2) C 3) D 4) E 5) F 1) C 2) A 3) F 4) G 5) E
5. Which of the following should be the FIRST 9. Which of the following will be the LAST sentence
statement after rearrangement? in the passage?
1) F 2) E 3) D 4) C 5) B 1) A 2) D 3) F 4) B 5) H
Distort = to give a false account of something 10. Which of the following will be the FOURTH
Discern = to perceive, know or f ind out sentence in the passage?
something 1) F 2) C 3) E 4) D 5) A
Introspect = to examine one’s own thoughts,
11. Which of the following will be the SEVENTH
feelings and motives
sentence in the passage?
Directions (Q. 6-11): Rearrange the following
1) D 2) B 3) H 4) A 5) C
eight sentences (A), (B), (C), (D), (E), (F), (G), and H
Prevail = to exist or happen generally; to be
in the proper sequence to form a meaningful widespread
paragraph; then answer the questions given below Divulge = to make s omething known,
them. especially a secret
(A) Therefore, the duty of the advocate is to do Mouthpiece = a person, newspaper etc that
his best for his client. expresses the opinions of others
Sentence Arrangement 527

Directions (Q. 12-16): Rearrange the following (D) The state exists for the citizens, not the
seven sentences (A), (B), (C), (D), (E), (F) and (G) in citizens for the state.
th e pr oper seq uence to for m a mean ingf ul (E) But it is a partnership of present with past
paragraph then answer the questions given below and future.
them. (F) It is also the trustee for its future.
(A) People thoroughly dedicated to social service (G) This is mainly be cause it is the
but not fulfilling the eligibility requirements representative and effective organ of the
would not be able to contest elections. largest and most inclusive community to
(B) Those who fulfil the stipulated criteria of age which he belongs.
and formal education may not be necessarily 17. Which sentence should be the FOURTH in the
devoted to social service. paragraph?
(C) This syste m has both advantage s and 1) A 2) B 3) C 4) D 5) E
disadvantages. 18. Which sentence should be the SIXTH in the
(D) The re fore , imposing such e ligibility paragraph?
re quire me nts is like ly to be counte r- 1) A 2) B 3) C 4) D 5) E
productive. 19. Which sentence should be the FIRST in the
(E) In certain democratic countries, elections paragraph?
can be contested by anybody. 1) A 2) B 3) C 4) D 5) E
(F) People would be deprived of the probable 20. Which sentence should be the SEVENTH (LAST)
benefit accrued from services of such people. in the paragraph?
(G) There are no eligibility requirements of formal 1) A 2) B 3) F 4) D 5) E

K
education and upper age limit stipulated in 21. Which sentence should be the SECOND in the
their Constitution. paragraph?
12. Which sentence should be the FOURTH in the 1) A 2) B 3) G 4) D 5) E
paragraph? Allegiance = s upport of or l oyal ty to a
1) A 2) B 3) C 4) D 5) E government, ruler, cause etc
13. Which sentence should be the LAST in the Fellowship = a group or society of people
sharing a common interest or
paragraph?
aim
1) A 2) B 3) C 4) D 5) E Contemporary = belonging to the same time; a
14. Which sentence should be the FIRST in the person who lives or lived at the
paragraph? same time as another, usually
1) G 2) F 3) E 4) D 5) C being roughly the same age
15. Which sentence should be the SECOND in the Trustee = a country given responsibilities

KUNDAN
paragraph? f or governing a particular
1) G 2) F 3) E 4) D 5) C territory by the United Nations
Organisation
16. Which sentence should be the THIRD in the
Directions (Q. 22-26): Rearrange the following
paragraph?
six sentences (A), (B), (C), (D), (E) and (F) in the
1) A 2) B 3) C 4) D 5) E
Stipulate = to state something clearly and firmly
proper sequence so as to make a meaningful
as a requirement paragraph, then answer the questions given below
Deprive of = to take something away f rom them.
somebody/something; to prevent (A) We feel these things are glorious because of
somebody/s omething f rom the splendid triumphs.
enjoying or using something (B) Because of these sacrifices we realise the
Accrue = to increase over a period of time; to victories of peace are even more glorious than
allow something to collect over a
victories of war.
period of time; to accumul ate
something
(C) The word victory is associated in our minds
Directions (Q. 17-21): Rearrange the following with war.
seven sentences (A), (B), (C), (D), (E), (F) and (G) in (D) We are impressed by their sacrifices.
the proper sequence so as to make a meaningful (E) It calls up visions of battles, bloodshed and
paragraph; then answer the questions given below conquests by force.
them. (F) But when we think of the philosophy of great
(A) The individual owes allegiance and obedience men, scholars, social reformers, scientists
to the state. and philanthropists we start thinking in a
(B) It is its organ for the present action, the different way.
custodian of its tradition. 22. Which sentence should be the FOURTH in the
(C) For, the nation is not only a fellowship of paragraph?
contemporaries. 1) B 2) C 3) D 4) E 5) F
528 Test of English Language

23. Which sentence should be the THIRD in the Fauna = all the animals of an area or a period
paragraph? of time
1) A 2) B 3) C 4) D 5) E Expedition = an organised journey or voyage with
a particular aim
24. Which sentence should be the FIRST in the
Ornithology = the scientific study of birds
paragraph?
Directions (Q. 32-35): Rearrange the following
1) A 2) B 3) C 4) D 5) E
six sentences (A), (B), (C), (D), (E) and (F) in the
25. Which sentence should be the LAST (SIXTH) in
proper sequence to form a meaningful paragraph;
the paragraph?
then answer the questions given below them.
1) A 2) B 3) C 4) D 5) E
(A) But all three have one focus individual
26. Which sentence should be the SECOND in the
performance improvement.
paragraph?
(B) The importance of each component will vary
1) B 2) C 3) D 4) E 5) F
from organization to organization according
Splendid = magnificent; very impressive
Triumph = the state of feeling great satisfaction
to the complexity of the operations.
or joy as the result of success or (C) They are individual development, career
victory development and organizational development.
Conquest = the action or an instance of (D) Since individual performance improvement is
defeating somebody/something the heart of HRD programme, HRD can be
Philanthropist = a person who is concerned for the described as the area of congruence among
welfare or benefit of others and who the three components.
supports good causes especially by
(E) There are three fundamental component

K
giving money
areas of human resource development.
Directions (Q. 27-31): Rearrange the following
(F) It will also vary according to the criticality of
six sentences (A), (B), (C), (D), (E) and (F) in the
human resources to organizational efficiency
proper sequence to form a meaningful paragraph;
and organization’s commitment to improve
then answer the questions given below them.
human resources.
(A) They collected plants, counted birds and
32. Which of the following will be the SIX TH
photographed the terrain and the fauna and
sentence?
made their recommendations.
1) C 2) F 3) B 4) D 5) A
(B) In spring of 1963, an alarmed King Hussain
33. Which of the following will be the FOURTH
invited a group of British scholars, scientists
sentence?
and naturalists.
1) F 2) C 3) D 4) B 5) A
(C) He also wanted them to cover the deserts to

KUNDAN
34. Which of the following will be the SECOND
the east of the mountains.
sentence?
(D) He wanted them to conduct an extensive
1) D 2) E 3) B 4) F 5) C
survey of the mountains on the eastern side
35. Which of the following will be the FIR ST
of the Dead Sea.
sentence?
(E) The problem of conservation of forests and
1) D 2) E 3) F 4) B 5) C
forest birds and nature in general was thus
Congruent = suitable or fitting for something
set rolling.
Directions (Q. 36-40): Rearrange the following
(F) Accordingly, an expedition of internationally
six sentences (A), (B), (C), (D), (E) and (F) in the
known e xpe rts in conservation, botany,
proper sequence to form a meaningful paragraph;
ornithology, etc went to Jordan.
then answer the questions given below them.
27. Which of the following should be the FIFTH in
(A) The application of economic, environmental
the paragraph?
and consumer pressures have been on an
1) A 2) B 3) C 4) D 5) E
increase in recent years.
28. Which of the following should be the FIRST in
(B) As a result, our agro-food production and
the paragraph?
technology are amongst the most advanced
1) A 2) B 3) C 4) D 5) E
in the world.
29. Which of the following should be the LAST in
(C) They are thus able to provide expertise and
the paragraph?
technology to satisfy the needs of agro-food
1) A 2) B 3) C 4) D 5) E
production.
30. Which of the following should be the SECOND
(D) In turn, the support industrie s have
in the paragraph?
developed to an equally advanced state.
1) F 2) E 3) D 4) C 5) B
(E) They have also equipped themselves with the
31. Which of the following should be the FOURTH
ne ce ssary expe rtise to satisfy the most
in the paragraph?
e xacting re quire me nts of the ove rse as
1) F 2) E 3) D 4) C 5) B
markets.
Terrain = a stretch of land with regard to its
natural features
Sentence Arrangement 529

(F) The se have gre atly influe nce d the Directions (Q. 46-50): Rearrange the following
development of the agriculture and food seven sentences (A), (B), (C), (D), (E), (F) and (G) in
industries in our country. th e pr oper seq uence to for m a mean ingf ul
36. Which of the following should be the SECOND paragraph, then answer the questions given below
sentence after rearrangement? them.
1) F 2) E 3) D 4) C 5) B (A) The history of that system is, however, a
37. Which of the following should be the THIRD warning than a stimulus to reorganise a
sentence after rearrangement? similar scheme.
1) F 2) E 3) D 4) C 5) B (B) However, we can’t resign ourselves merely
38. Which of the following should be the FOURTH because there are not data.
sentence after rearrangement? (C) One of the commone st risks which
1) F 2) E 3) D 4) C 5) B agricultural life is exposed to in this country
39. Which of the following should be the FIFTH is famine or failure of crops.
sentence after rearrangement? (D) There are, however, no reliable data on which
1) F 2) E 3) D 4) C 5) B such a scheme of insurance can be based.
40. Which of the following should be the SIXTH (E) A kind of Famine Insurance System was
(LAST) sentence after rearrangement? attempted by the British Government of India
1) F 2) E 3) D 4) C 5) B in the last century.
Expertise = expert knowledge or skill, especially (F) Still, the need for such a scheme to cover
in a particular field the losses due to famine, cattle plague, crop
Directions (Q. 41-45): Rearrange the following pests, etc can’t be undermined.

K
seven sentences (A), (B), (C), (D), (E), (F) and (G) in (G) It is obviously because of failure of rain and
th e pr oper seq uence to for m a mean ingf ul the consequence is starvation.
paragraph; then answer the question given below 46. Which of the following should be the fourth
them. sentence after rearrangement?
(A) In other words, floods and droughts are built 1) A 2) B 3) C 4) D 5) E
into the countries monsoonal ecology. 47. Which of the following should be the sixth
(B) “It pours cats and dogs” they said. sentence in the paragraph?
(C) In the remaining 8,660 hours there is hardly 1) E 2) D 3) B 4) C 5) A
any precipitation. 48. Which of the following should be there in the
(D) If the rain is not caught and stored, it will be third position in the paragraph?
impossible to live in this country. 1) B 2) C 3) D 4) E 5) F
(E) But most of it pours down in a mere hundred 49. Which of the following should be the first

KUNDAN
hours. sentence after rearrangement?
(F) India gets more rain annually 1,100 mm, on 1) A 2) B 3) D 4) C 5) E
an average than any other part of the world. 50. Which of the following should be the second
(G) When the British came to India, they were sentence after rearrangement?
struck by the amount it rained here. 1) B 2) G 3) D 4) E 5) F
41. Which of the following will be the FIR ST Stimulus = a thing that encourages or excite
sentence? somebody/something to activity,
1) G 2) A 3) C 4) D 5) F greater effort etc
42. Which of the following will be the SECOND Undermine = to make s omething/s omebody
sentence? gradually weaker or less effective
Starvation = suffering or death caused by lack of
1) C 2) D 3) F 4) G 5) B
food
43. Which of the following will be the FOURTH
Directions (Q. 51-55): Rearrange the following
sentence?
six sentences (A), (B), (C), (D), (E) and (F) in the
1) B 2) C 3) E 4) F 5) D
proper sequence to form a meaningful paragraph;
44. Which of the following will be the FIF TH
then answer the questions given below them.
sentence?
(A) Some people believe that at prese nt its
1) A 2) C 3) D 4) F 5) G
importance is decreasing because of rapid
45. Which of the following will be the LA ST
economic and social changes.
sentence?
(B) The extent of its importance may be slightly
1) D 2) E 3) F 4) A 5) C
le ss in citie s as compare d to rural
To rain cats and dogs = to rain very heavily
To be struck by = to be favourably impressed by
communities.
somebody/something; to like (C) Some even go to the extreme and say that it
somebody/s omething very will soon become obsolete because of these
much changes.
(D) The family is an important socialisation
agency both in rural and city communities.
530 Test of English Language

(E) The difference in the degree of importance Dizzy = unable to balance; confused
does not matter much and therefore has no Prophecy = the power of s aying what wil l
significance. happen in the future
(F) There are others who believe that the family Directions (61-65): Rearrange the following six
has survived such storms in the past and it sentences (A), (B), (C), (D), (E) and (F) in the proper
will do so in the future also. sequence to form a meaningful paragraph; then
51. Which of the following should be the FIRST answer the questions given below them.
sentence after rearrangement? (A) We we re inte re ste d by contrast in
1) F 2) E 3) D 4) C 5) B understanding what lessons actual teams and
52. Which of the following should be the SIXTH non-teams had for others to choose to
(LAST) sentence after rearrangement? struggle with change and performance.
1) F 2) E 3) D 4) C 5) B (B) Still, we suspe cte d that most of the se
53. Which of the following should be the FIFTH focussed on persuading readers that “teams
sentence after rearrangement? are important”.
1) F 2) E 3) D 4) C 5) B (C) After all we thought teams are a well-known
54. Which of the following should be the THIRD subject and there must be a thousand books
sentence after rearrangement? on the subject already.
1) F 2) E 3) D 4) C 5) B (D) By going down this path we hoped to discover
55. Which of the following should be the SECOND something to say that was different from most
sentence after rearrangement? books on the subject.

K
1) F 2) E 3) D 4) C 5) B (E) We approached the idea of a book on teams
Obsolete = no longer used; out of date cautiously.
Directions (Q. 56-60): Rearrange the following (F) Alternatively they focussed on providing you
five sentences (A), (B), (C), (D) and (E) in the proper advice on building teams as an objective in
sequence to form a meaning paragraph; then itself.
answer the questions given below them. 61. Which of the following will be the se cond
(A) The reasons for formal education getting sentence?
nullified are that we teachers have limited 1) A 2) B 3) F 4) C 5) D
vision, our judgements about students are 62. Which of the following will be the first sentence?
hasty and we are more knowledge-centred 1) E 2) A 3) B 4) C 5) D
than student-centred. 63. Which of the following will be the third sentence?
(B) Life educates as nothing else does. 1) E 2) C 3) B 4) F 5) D

KUNDAN
(C) Churchill rose to dizzy heights despite his 64. Which of the following will be the fifth sentence?
teachers’ prophecies to the contrary. And 1) C 2) D 3) B 4) F 5) A
there are many more such examples. 65. Which of the following will be the last sentence?
(D) Life’s teachings sometimes supplement the 1) C 2) D 3) E 4) F 5) B
education received in the classroom and at Persuade = to make s omebody believe
something; to convince somebody
other times nullify it.
Directions (Q. 66-70): Rearrange the following
(E) Education re ce ive d in the classroom is
five sentences into a meaningful paragraph and
insignificant as compared to what life teaches
answer the questions given below:
us.
(A) However, with innovation coming into play
56. Which of the following will be the SECOND
unit-linked/market-linked products have also
sentence?
found a place in insurance business after
1) A 2) B 3) C 4) D 5) E
privatisation.
57. Which of the following will be the THIRD
(B) It is also worth mentioning here that world
sentence?
over unit-linked products constitute quite a
1) A 2) B 3) C 4) D 5) E
substantial chunk of the total portfolio of
58. Which of the following will be the FIR ST
insurance companies.
sentence?
(C) There was a time when only traditional
1) A 2) B 3) C 4) D 5) E
insurance products used to dominate the
59. Which of the following will be the FOURTH
arena.
sentence?
(D) The emergence of these products of various
1) A 2) B 3) C 4) D 5) E
insurance companie s combine s the
60. Which of the following will be the LA ST
characteristics of both endowment insurance
sentence?
policies and mutual funds.
1) A 2) B 3) C 4) D 5) E
Nullify = to make something lose its effect; to
(E) The insurance industry in India is evolving
act against something and assuming different proportion since it
Hasty = acting or deciding too quickly, was privatised.
without enough thought.
Sentence Arrangement 531

66. Which of the following will be the Fourth (A) The north is bordered by mountain ranges
sentence in the paragraph? while the remaining sides of Greece are
1) A 2) B 3) C 4) D 5) E enclosed by the Mediterranean sea.
67. Which of the following will be the Second (B) The people there were called the Greeks.
sentence in the paragraph? (C) Greeks, therefore, became good navigators.
1) A 2) B 3) C 4) D 5) E (D) The typical character of Greek civilisation is
68. Which of the following will be the Last sentence due to the geographic conditions prevailing
in the paragraph? there.
1) A 2) B 3) C 4) D 5) E (E) The name Greece comes from one of the
69. Which of the following will be the First sentence islands to the south-east of Europe known
in the paragraph? as Graecia.
1) A 2) B 3) C 4) D 5) E 76. Which of the following will be the THIRD
70. Which of the following will be the Third sentence sentence?
in the paragraph? 1) C 2) D 3) A 4) B 5) E
1) A 2) B 3) C 4) D 5) E 77. Which of the following will be the FOURTH
Innovation= the process of making changes or sentence?
in-troducing new ideas methods etc 1) B 2) E 3) A 4) D 5) C
Chunk = a fairly large amount of something 78. Which of the following will be the SECOND
Arena = a place or scene of activity or conflict
sentence?
Endowment = money, property etc given to provide
an income.
1) A 2) E 3) D 4) C 5) B
Emerge = to develop or become noticeable 79. Which of the following will be the LA ST

K
Directions (Q. 71-75): Rearrange the following sentence?
seven sentences (A), (B), (C), (D), (E), (F) and (G) in 1) C 2) D 3) B 4) E 5) A
th e pr oper seq uence to for m a mean ingf ul 80. Which of the following will be the FIR ST
paragraph; then answer the questions given below sentence?
them. 1) B 2) C 3) A 4) E 5) D
(A) Japanese toys, for instance, are in great Directions (Q. 81-85): Rearrange the following
demand despite the heavy import duty. sentences to form a meaningful paragraph and
(B) The toys that they produce are, almost without answer the questions given below:
exception, of inferior quality. (A) We must explore new methods of boosting
(C) The ir manufacture rs he re ne e d to be agricultural development and grow more food.
reminded of this. (B) The scientists should be e ncourage d to
(D) The two toy-libraries in Mumbai also rely contribute.

KUNDAN
largely on foreign-made toys. (C) Food can also be had by import.
(E) But making them is no child’s play. (D) The most important factor in any planning
(F) Toys are meant for children. for India’s development and economic uplift
(G) Not surprisingly, many parents prefer to buy is that of turning a hungry, discontented
the imported variety even though these are people into a happy well-fed one.
usually much more expensive. (E) Whatever be the way and means, India must
71. Which of the following will be the THIRD feed its hungry millions.
sentence? (F) They should be given due scope for carrying
1) C 2) B 3) D 4) E 5) G on experiments and researches.
72. Which of the following will be the LA ST (G) The problem, therefore, reduces itself to one
sentence? of agricultural development.
1) C 2) D 3) B 4) E 5) G 81. Which of the following is the fourth sentence
73. Which of the following will be the FIR ST in the paragraph?
sentence? 1) A 2) D 3) E 4) G 5) F
1) E 2) G 3) A 4) F 5) B 82. Which of the following is fifth sentence in the
74. Which of the following will be the FOURTH paragraph?
sentence? 1) G 2) A 3) D 4) C 5) E
1) C 2) E 3) B 4) G 5) D 83. Which of the following is the third sentence in
75. Which of the following will be the SIX TH the paragraph?
sentence? 1) F 2) A 3) G 4) D 5) E
1) F 2) E 3) C 4) D 5) A 84. Which of the following is the last sentence in
Child’s play = a thing that is very easy to do the paragraph?
Directions (Q. 76-80): Rearrange the following 1) F 2) C 3) D 4) B 5) G
five sentences (A), (B), (C), (D) and (E) in the proper 85. Which of the following is the second sentence
sequence to form a meaningful paragraph and then in the paragraph?
answer the questions given below them. 1) B 2) E 3) C 4) G 5) F
532 Test of English Language

Explore = to examine something thoroughly in (D) Some rhymes can be traced to popular ballads,
order to test it or find out about it folk songs and games, political satire, ancient
Carry on = to continue doing something proverbs, cries of street vendors, real or
Directions (Q. 86-90): Rearrange the following legendary events.
seven sentences (A), (B), (C), (D), (E), (F) and (G) in (E) About twenty-five years later the book was
th e pr oper seq uence to for m a mean ingf ul reprinted in the United States in Worcester,
paragraph; then answer the questions given below Massachusetts.
them. (F) In fact, until the eighteenth century Mother
(A) It takes its recourse to progressive march Goose did not have a name in print in English
towards perfection. literature.
(B) But, one may conclude, while science is 91. Which of the following will be the last sentence?
inclined towards reason, spiritualism is the 1) A 2) B 3) C 4) D 5) E
essence of religion. 92. Which of the following will be the fourth
(C) In religion deviation from the set course is sentence?
permissible, though some more rationalistic I) F 2) E 3) D 4) C 5) B
religious leaders also allow questioning and 93. Which of the following will be the fifth sentence?
their satisfactory answers. 1) F 2) E 3) D 4) C 5) B
(D) Many people believe that science and religion 94. Which of the following will be the first sentence?
are contrary to each other. 1) A 2) B 3) C 4) D 5) E
(E) The tools of religion, on the other hand, are 95. Which of the following will be the se cond

K
faith, intuition, and the spoken word of the sentence?
enlightened. 1) A 2) B 3) C 4) D 5) E
(F) The me thod of scie nce is obse rvation, To grow in something = to gain a larger amount of
experiment and experience. a particular quality or feeling
(G) There is no doubt that the methods of science Verse = a group of lines forming a unit in a
and religion are different. poem or song
86. Which of the following will be the SECOND Ballad = a simple song or poem especially
sentence? one that tells a story
Satire = the art or practice of mocking people,
1) F 2) E 3) D 4) B 5) G
institutions etc and making them
87. Which of the following will be the FOURTH appear ridiculous in order to show
sentence? how foolish, wicked or incompetent
1) B 2) A 3) D 4) F 5) C they are.

KUNDAN
88. Which of the following will be the LA ST Legendary = very wel l- known; f amous and
sentence? inspiring admiration
1) C 2) A 3) D 4) B 5) E Directions (Q. 96-100): Rearrange the following
89. Which of the following will be the FIR ST six sentences (A), (B), (C), (D), (E) and (F) in a proper
sentence? sequence so as to form a meaningful paragraph.
1) C 2) B 3) D 4) F 5) A Then answer the questions given below them.
90. Which of the following will be the SIX TH (A) The child will be taught that hard work is
sentence? necessary to bring about academic success,
1) C 2) F 3) A 4) G 5) D which is the forerunner to occupational
Recourse = a source of help in a diff icul t success.
situation, or the use of this (B) In the case of the city family educated to
Essence = the most important quality, feature professional standards, the process is likely
or characteristic of something to take a different form.
Enlightened = free from prejudice, false belief etc
(C) The rural family will transmit these values
Directions (Q. 91-95): Rearrange the following
to the child in order to prepare it for its future
six sentences (A), (B), (C), (D), (E) and (F) in the
role as an adult.
proper sequence so as to form a meaningful
(D) In the rural community emphasis will be
paragraph, then answer the questions given below
placed upon values such as group solidarity
them.
and the belief in the natural superiority of
(A) It was further revived by a Boston publishing
the male.
firm, and from that time Mothe r Goose
(E) The child is more likely to be taught the
continued and grew in fame and interest till
values necessary for success in a world
date.
dominated by individual achievement.
(B) The first collection of verses under her name
(F) Thus the child will grow up placing greater
was published in London in book form by John
value upon the family as a unit than upon
Newberry.
himself as an individual : more emphasis
(C) These were known long before they were
upon a segregation of the roles of husband
designated as Mother Goose rhymes.
and wife than upon equality, and so on.
Sentence Arrangement 533

96. Which of-the following will be FO UR TH Kinship = a family relationship; a close feeling
sentence? between people that develops as a
1) F 2) E 3) D 4) C 5) B result of common origins, attitudes
etc
97. Which of the following will be the FIR ST
Dir ection ( Q. 106-110): Rearran ge t he
sentence?
following six sentences (A), (B), (C), (D), (E) and
1) A 2) B 3) C 4) D 5) E
(F) in the proper sequence to form a meaningful
98. Which of the following will be the SECOND
paragraph; then answer the questions given below
sentence?
them.
1) A 2) B 3) C 4) D 5) E
(A) We at Infoquicktech thus make sure that our
99. Which of the following will be the FIF TH
clients receive the best and the quickest
sentence?
services in the field of IT.
1) F 2) E 3) D 4) C 5) B
(B) Information Technology has transformed a lot
100. Which of the following will be the LA ST
in this decade and this metamorphosis is
sentence?
exceedingly quick.
1) A 2) B 3) C 4) D 5) E
To bring something about = to make something
(C) The user can now get the desired information
happen in a fraction of a second.
Forerunner = a sign of what is to follow (D) Our newer packages now perform in a far
Solidarity = unity agreement and support better way than our clients used to get in
resulting from shared interests, the past.
feelings, actions, sympathies etc (E) Our search engines have been astoundingly
Segregation = the action of isolating people or powerful to process and fetch the required

K
things or the state of being isolated
information to the user.
Directions (Q. 101-105): Rearrange the following
(F) These improvements perform a real magic,
five sentences (A), (B), (C), (D) and (E) in the proper
in the true sense of the term, to facilitate
sequence to form a meaningful paragraph; then
quicker access.
answer the questions given below them.
106. Which of the following will be the FIF TH
(A) The socialisation function of the family is a
sentence after rearrangement?
generalised one and is aimed at preparing us
1) A 2) B 3) C 4) D 5) E
for membership of kinship group and the
107. Which of the following will be the SIXTH (LAST)
community.
sentence after rearrangement?
(B) The family is only one of the varieties of
1) A 2) B 3) C 4) D 5) E
agencies of socialisation.
108. Which of the following will be the FIR ST
(C) In other words, through the socialisation

KUNDAN
sentence after rearrangement?
process we learn the basic facts necessary
1) A 2) B 3) C 4) D 5) E
for the performance of a variety of social roles
109. Which of the following will be the SECOND
in the society in which we grow up.
sentence after rearrangement?
(D) Socialisation is the process by which cultural,
1) A 2) B 3) C 4) D 5) E
social and moral values and be lie fs are
110. Which of the following will be the FOURTH
transmitted from one generation to the next.
sentence after rearrangement?
(E) The way in which the process operates will
1) A 2) B 3) C 4) D 5) F
depend largely upon the views taken by the Metamorphosis = a change of form or nature
parents of what their children ought to be Astounding = extremely surprising
like when they are grown up. Fetch = to go and find and bring back
101. Which of the following should be the SECOND somebody/something
sentence after rearrangement? Directions (Q. 111-115): Rearrange the following
1) A 2) B 3) C 4) D 5)E six sentences (A), (B), (C), (D), (E) and (F) in the
102. Which of the following should be the FOURTH proper sequence to form a meaningful paragraph;
sentence after rearrangement? then answer the questions given below them.
1) A 2) B 3) C 4) D 5) E (A) For other wishes, it enlists the help of those
103. Which of the following should be the FIRST who make or own what the child desires.
sentence after rearrangement? (B) It grants some wishes with its own funds
1) A 2) B 3) C 4) D 5) E depending upon the availability of funds.
104. Which of the following should be the THIRD (C) I was deeply moved when I learnt about the
sentence after rearrangement? activity of “Make-a-Wish Foundation”.
1) A 2) B 3) C 4) D 5)E (D) From parents, friends, or hospital attendants,
105. Which of following will be the FIFTH (LAST) the foundation learns about the child’s wish
sentence after rearrangement? for anything from a special toy to a visit to
1) A 2) B 3) C 4) D 5) E Disneyland.
534 Test of English Language

(E) I am sure you also now must have been Invade = to enter a place in large numbers,
moved by the noble act of the foundation. especially so as to cause damage; to
(F) It grants the wishes of children who are crowd into something
C rab = a sea creature that has a hard shell
terminally ill.
and ten l egs two of which are
111. Which of the following should be the FOURTH pincers
sentence after rearrangement? Malignant = that cannot be controlled and is
1) F 2) E 3) D 4) C 5) B likely to prove fatal
112. Which of the following should be the SIXTH Directions (Q. 121-125): Rearrange the following
(LAST) sentence after rearrangement? six sentences (A), (B), (C), (D), (E) and (F) in the
1) F 2) E 3) D 4) C 5) B proper sequence to form a meaningful paragraph;
113. Which of the following should be the SECOND then answer the questions given below them.
sentence after rearrangement? (A) Happiness, if at all found, is accompanied
1) F 2) E 3) D 4) C 5) B with sorrow.
114. Which of the following should be the THIRD (B) This attitude helps us to cope up with our
sentence after rearrangement? unhappiness.
1) F 2) E 3) D 4) C 5) B (C) The world is full of miseries, problems, risks
115. Which of the following should be the FIRST and discomfiture.
sentence after rearrangement? (D) It also helps us to be sympathetic to others
1) F 2) E 3) D 4) C 5) B who are more unhappy.
Directions (Q. 116-120): Rearrange the following (E) Therefore, it seems to be wise to compare

K
six sentences (A), (B), (C), (D), (E) and (F) in the our lot with the lot of those who are less
proper sequence to form a meaningful paragraph; fortunate.
then answer the questions given below them. (F) None can find here perfect happiness.
(A) It is a general term used to describe over 200 121. Which of the following should be the FIRST
individual diseases. sentence after rearrangement?
(B) The abnormal cells grow without any control, 1) B 2) C 3) D 4) E 5) F
invade through normal tissue barriers and 122. Which of the following should be the FIFTH
reproduce indefinitely. sentence after rearrangement?
(C) The word “cancer” comes from Latin, meaning 1) B 2) C 3) D 4) E 5) F
a crab. 123. Which of the following should be the SIXTH
(D) These characteristics include development (LAST) sentence after rearrangement?
within any tissue of a malignant growth. 1) B 2) C 3) D 4) E 5) F

KUNDAN
(E) A tumour was called cance r because of 124. Which of the following should be the SECOND
swollen veins around the area resembling a sentence after rearrangement?
crab’s limbs. 1) B 2) C 3) D 4) E 5) F
(F) These diseases progress differently over a 125. Which of the following should be the FOURTH
pe riod of time and share ce rtain sentence after rearrangement?
characteristics. 1) B 2) C 3) D 4) E 5) F
116. Which of the following should be the SECOND Cope up with = to have the capacity to deal
sentence after rearrangement? with something successfully
1) A 2) B 3) C Discomfiture = noun of the verb ‘discomfit’.
4) D 5) E Discomfit = to confuse or embarras s
somebody
117. Which of the following should be the THIRD
Directions (Q. 126-130): Rearrange the following
sentence after rearrangement?
seven sentences (A), (B), (C), (D), (E), (F) and (G) in
1) A 2) B 3) C
th e pr oper seq uence to for m a mean ingf ul
4) D 5) E
paragraph; then answer the questions given below
118. Which of the following should be the FOURTH
them.
sentence after rearrangement?
(A) Equally, if you show disloyalty to your
1) A 2) B 3) C
company or colleagues, your manners will be
4) D 5) None of these
seen as unacceptable.
119. Which of the following should be the FIFTH
(B) Much of this book is concerned with this
sentence after rearrangement?
golden rule.
1) A 2) B 3) C
(C) The other half is good manners invoke taking
4) D 5) None of these
positive action to make the other person feel
120. Which of the following should be the SIXTH
good about your relationship with him.
(LAST) sentence after rearrangement?
(D) They indicate to the person you are dealing
1) A 2) B 3) C
with whether you can be relied on to act
4) D 5) None of these
correctly and fairly when he does business
with you.
Sentence Arrangement 535

(E) Avoiding these negatives is only half the Directions (Q. 136-140): Rearrange the following
story. sentences into a meaningful paragraph and then
(F) Manners are the patterns of behaviour. answer the questions given below it.
(G) If he thinks that you are boorish, selfish or (A) Development of drought resistance could
undisciplined your relationship is unlikely to benefit large numbers of farmers.
be proper. (B) Hence the human race has no choice but to
126. Which of the following will be the SEVENTH adapt to these impacts.
sentence? (C) India has to be concerned about climatic
1) C 2) D 3) E 4) B 5) A changes.
127. Which of the following will be the FIR ST (D) This impact can run into de cade s and
sentence? centuries.
1) G 2) F 3) A 4) B 5) C (E) Environme nt day is thus an important
128. Which of the following will be the THIRD occasion to assess the past and our future.
sentence? (F) Since there is a possibility of adverse impact
1) G 2) E 3) F 4) B 5) A on agriculture which could deter growth.
129. Which of the following will be the FIF TH 136. Which is the THIRD sentence of the paragraph?
sentence? 1) A 2) D 3) B 4) C 5) E
1) A 2) G 3) E 4) F 5) D 137. Which is the LAST (SIXTH) sentence of the
130. Which of the following will be the SIX TH paragraph?
sentence? 1) C 2) B 3) F 4) D 5) E
1) B 2) D 3) F 4) A 5) C 138. Which is the FIRST sentence of the paragraph?

K
Invoke = to ask, call, beg or pray for something 1) A 2) D 3) C 4) B 5) E
Boorish = of or like a rough or rude person 139. Which is the FIFTH sentence of the passage?
Directions (Q. 131-135): Rearrange the following 1) F 2) D 3) E 4) A 5) C
six sentences (A), (B), (C), (D), (E) and (F) in the 140. Which is the SECO ND se nte nce of the
proper sequence to form a meaningful paragraph; paragraph?
then answer the questions given below them. 1) B 2) D 3) F 4) C 5) E
(A) To propose the idea of becoming perfected is Adapt = to become adjusted to new
not the same as saying that we human beings condition; to make something
can be perfect. suitable for a new use, situation,
(B) It only means that we are capable of learning, etc.
changing and growing throughout the span Deter = to make somebody decide not to do
something
of our earthly life.

KUNDAN
Directions (Q. 141-145): Rearrange the following
(C) Learning is a process that begins at birth and
six sentences (A), (B), (C), (D), (E) and (F) in the
lasts till death.
proper sequence to form a meaningful paragraph;
(D) This development through understanding new
then answer the questions given below them.
things, transforming and de ve loping
(A) Basic human needs also include a sense of
throughout may be conside re d as the
belongingness, a feeling of control over one’s
perfection of soul.
life.
(E) Given that we are here to continually learn on
(B) Motivation and inspiration energise people
the journey of life, it seems that the ultimate
into action.
goal of learning is the perfection of our souls.
(C) Ability to live up to one’s ideals besides all
(F) Nor that we should be perfect in everything.
these is also a fundamental need.
131. Which of the following should be the FIRST
(D) This is done not by pushing them in the right
statement after rearrangement?
direction as control mechanisms.
1) A 2) B 3) C 4) D 5) E
(E) Such feelings touch us deeply and elicit
132. Which of the following should be the SECOND
powerful response.
statement after rearrangement?
(F) But it is done by satisfying basic human needs
1) A 2) B 3) C 4) D 5) E
for achievement.
133. Which of the following should be the THIRD
141. Which of the following should be the FIRST
statement after rearrangement?
statement after rearrangement?
1) A 2) B 3) C 4) D 5) F
1) A 2) B 3) C 4) D 5) E
134. Which of the following should be FOURTH
142. Which of the following should be the SECOND
statement after rearrangement?
statement after rearrangement?
1) A 2) B 3) C 4) D 5) F
1) A 2) B 3) C 4) D 5) E
135. Which of the following should be the LAST
143. Which of the following should be the FOURTH
statement after rearrangement?
statement after rearrangement?
1) A 2) B 3) C 4) D 5) E
Earthly = of this world; not spiritual
1) A 2) B 3) C 4) D 5) E
536 Test of English Language

144. Which of the following should be the FIFTH Directions (Q. 151-155): Rearrange the following
statement after rearrangement? sentences (A), (B), (C), (D), (E) and (F) in the proper
1) A 2) B 3) C 4) D 5) E sequence to form a meaningful paragraph and then
145. Which of the following should be the SIXTH answer the questions given below it.
(LAST) statement after rearrangement? (A) Strict obedience to these rules is called
1) A 2) B 3) C 4) D 5) E discipline.
Elicit = to draw facts, a response, etc from (B) In the same way, a society where rules are
s omebody, s ometimes with not followed cannot survive for long.
difficulty (C) Only then a society can be run in an orderly
To live up to something = to behave as well as or
fashion.
be as good as expected
(D) A society can exist properly only when men
Directions (Q. 146-150): Rearrange the following
living in it agree upon certain rules of conduct.
six sentences (A), (B), (C), (D), (E) and (F) in the
(E) For example, if the people on the road do not
proper sequence to form a meaningful paragraph;
obey traffic rules there will be complete
then answer the questions given below them.
disorder and confusion.
(A) There are a number of items in the atomic
(F) Students must obey their teachers, children
energy programme which are being made
their parents, citizens the laws and so on
indigenously.
and so forth.
(B) Given the overall energy situation in India,
151. Which of the following is the FIFTH sentence?
the use of nuclear power in some measure is
1) A 2) B 3) C 4) E 5) F
inescapable even while thermal and hydro

K
152. Which of the following is the SIXTH (LAST)
power continue to be the dominant elements.
sentence?
(C) However, commercial aspects of exploiting
1) A 2) B 3) C 4) D 5) E
nuclear capabilities, especially for power-
153. Which of the following is the SECOND sentence?
generation programmes, have been recently
1) A 2) B 3) C 4) D 5) E
given high priority.
154. Which of the following is the FIRST sentence?
(D) Atomic energy programmes have been subject
1) A 2) B 3) C 4) D 5) E
to se ve re re strictions for ve ry obvious
155. Which of the following is the THIRD sentence?
reasons as the Department of Atomic Energy
1) A 2) B 3) C 4) E 5) F
is becoming self-reliant in areas in which
And so for th; And so on ( and so fort h) =
only a few countries have such capability. used for indicating things additional
(E) Eve n to me e t the se nucle ar powe r to or s imil ar to thos e al ready

KUNDAN
requirements, India critically requires a mentioned
commercial-level power-generation capability, Directions (Q. 156-160): Rearrange the following
with its commensurate safety and nuclear six sentences (A), (B), (C), (D), (E) and (F) in the
waste management arrangements. proper sequence to form a meaningful paragraph;
(F) Thus, in the Indian context energy security then answer the questions given below them.
is also crucial, perhaps much more than it is (A) To address these issues Indian corporates
for the USA, because India imports a good are increasingly turning eco-friendly.
part of its crude oil requirements, paying for (B) At present, however, there are only a dozen
it with precious foreign exchange. green buildings in the private sector.
146. Which of the following will be the FIF TH (C) However, though an eco-friendly building may
sentence after rearrangement? cost more upfront, it is cost-effective because
1) A 2) B 3) C 4) D 5) E of lower operating costs in the long run.
147. Which of the following will be the THIRD (D) Today there is growing concern about global
sentence after rearrangement? warming, energy and water crises.
1) A 2) B 3) C 4) D 5) E (E) The reason is the construction cost of an eco-
148. Which of the following will be the SECOND friendly building is 15% to 20% more than
sentence after rearrangement? putting up a conventional building.
1) A 2) B 3) C 4) D 5) E (F) Planting trees, using energy-saving lighting
149. Which of the following will be the FIR ST systems and constructing eco-friendly green
sentence after rearrangement? buildings are some of the measures they are
1) A 2) B 3) C 4) D 5) E taking.
150. Which of the following will be the FOURTH 156. Which of the following will be the FIR ST
sentence after rearrangement? sentence after rearrangement?
1) A 2) B 3) C 4) D 5) E 1) A 2) B 3) C 4) D 5) E
Commensurate = in the right proportion; 157. Which of the following will be the SECOND
appropriate sentence after rearrangement?
1) A 2) C 3) D 4) E 5) F
Sentence Arrangement 537

158. Which of the following will be the THIRD (F) However, from the basics of food, clothing
sentence after rearrangement? and shelter we gradually seek luxury cars and
1) B 2) D 3) C 4) E 5) F holidays.
159. Which of the following will be the FIF TH 166. Which of the following is the FIRST sentence
sentence after rearrangement? after rearrangement?
1) C 2) D 3) E 4) F 5) A 1) A 2) B 3) C 4) D 5) E
160. Which of the following will be the SIXTH (LAST) 167. Which of the following is the SIXTH (LAST)
sentence after rearrangement? sentence after rearrangement?
1) B 2) C 3) D 4) E 5) F 1) A 2) B 3) C 4) D 5) E
Upfront = in advance; honest; open; frank 168. Which of the following is the FIFTH sentence
Directions (Q. 161-165): Rearrange the following after rearrangement?
six sentences (A), (B), (C), (D), (E) and (F) in the 1) A 2) B 3) C 4) D 5) (F)
proper sequence to form a meaningful paragraph; 169. Which of the following is the SECOND sentence
then answer the questions given below them. after rearrangement?
(A) The re is ample justification for the 1) A 2) B 3) C 4) D 5) E
phenomenon of the description. 170. Which of the following is the THIRD sentence
(B) In the present era also, efforts are being after rearrangement?
made to rejuvenate Pali. 1) A 2) B 3) C 4) D 5) E
(C) A curious researcher can study them provided Directions (Q. 171-175): Rearrange the following
he has mastery over Pali language. six sentences (A), (B), (C), (D), (E) and (F) in the
(D) The Indian tradition describes Buddha as an proper sequence to form a meaningful paragraph;

K
advocate of analytical method. then answer the questions given below them.
(E) Pali was perhaps the most commonly used (A) Securitization Act provides teeth to banks to
language in those days. deal well with such defaulters.
(F) This justification is found in se ve ral (B) Therefore, the banks should first examine
discourses. thoroughly the genuineness of the clients and
161. Which of the following should be the FIRST prevent them from turning into defaulters.
sentence after rearrangement? (C) Recovery of loans from the borrowers is one
1) A 2) B 3) C 4) D 5) E of the biggest problems of almost all of them.
162. Which of the following should be the SECOND (D) It is because most of them have a tendency
sentence after rearrangement? to misutilize the funds.
1) A 2) B 3) C 4) D 5) E (E) Most banks these days have been facing a
163. Which of the following should be the FOURTH lot of different problems.

KUNDAN
sentence after rearrangement? (F) Most of the borrowers don’t have money to
1) A 2) B 3) C 4) D 5) E repay.
164. Which of the following should be the FIFTH 171. Which of the following should be the FOURTH
sentence after rearrangement? statement after rearrangement?
1) A 2) B 3) C 4) D 5) E 1) A 2) B 3) C 4) D 5) E
165. Which of the following should be the SIXTH 172. Which of the following should be the FIFTH
(LAST) sentence after rearrangement? statement after rearrangement?
1) A 2) B 3) C 4) D 5) E 1) A 2) B 3) C 4) D 5) E
Discourse = spoken or written language 173. Which of the following should be the SIXTH
Rejuvenate = to make somebody/something look (LAST) statement after rearrangement?
or feel younger or more lively
1) A 2) B 3) C 4) D 5) E
Advocate = a person who supports or speaks
in favour of a cause, policy etc
174. Which of the following should be the SECOND
Directions (Q. 166-170): Rearrange the following statement after rearrangement?
sentences (A), (B), (C), (D), (E) and (F) into a 1) A 2) B 3) C 4) D 5) E
meanin gful par agraph and t hen answer t he 175. Which of the following should be the FIRST
questions given below it. statement after rearrangement?
(A) According to conventional wisdom, no amount 1) A 2) B 3) C 4) D 5) E
Defaulter = a person who fails to do what he or
can suffice.
she supposed to do, eg to appear in
(B) In the process of seeking it is money which a law-court; a person who fails to
unfortunately gets a bad name. pay a debt etc
(C) How much money is enough? Directions (Q. 176-180): Rearrange the following
(D) The modern reasoning is that money coming six sentences (A), (B), (C), (D), (E) and (F) in the
in should cover our basic needs. proper sequence to form a meaningful paragraph;
(E) The real culprit, however, is a living thing then answer the questions given below them.
called desire, which although it is difficult
to, can be controlled.
538 Test of English Language

(A) Thus rapid development is still unable to 182. Which of the following should be the SECOND
meet demand. sentence after rearrangement?
(B) Surplus funds from hikes in passenger fares 1) B 2) C 3) D 4) E 5) (F)
and cuts in staff have made this possible. 183. Which of the following should be the THIRD
(C) This de monstrate s that it has be e n sentence after rearrangement?
transformed into a modern high-standard 1) A 2) B 3) C 4) D 5) E
design and high-service reliable system. 184. Which of the following should be the FIFTH
(D) However, China’s railways are not problem- sentence after rearrangement?
free. 1)A 2) B 3) C 4) D 5) (F)
(E) China’s railways have been able to generate 185. Which of the following should be the LAST
the funds needed for the construction of new (SIXTH) sentence after rearrangement?
railway lines internally. 1) A 2) B 3) C 4) D 5) E
(F) For example, at present about 2,80,000 cars Lure = to attract or tempt a person or an
are requested daily to transport goods but animal
only half the requests can be met. Directions (Q. 186-190): Rearrange the following
176. Which of the following is the FIFTH sentence six sentences (A), (B), (C), (D), (E) and (F) in the
after rearrangement? proper sequence to form a meaningful paragraph;
1) A 2) C 3) D 4) E 5) F then answer the questions given below them.
177. Which of the following is the SIXTH (LAST) (A) He then hid nearby to watch and see who
sentence after rearrangement? would remove it.

K
1) A 2) C 3) D 4) E 5) F (B) He saw a purse full of gold lying in the middle
178. Which of the following is the SECOND sentence of the road. It was the king’s reward to the
after rearrangement? person who did something about the problem.
1) A 2) C 3) D 4) E 5) F (C) The king was tire d of his subje cts only
179. Which of the following is the THIRD sentence complaining but doing nothing to solve their
after rearrangement? problems.
1) A 2) C 3) D 4) E 5) F (D) A youth on his way to market saw the stone,
180. Which of the following is the FIRST sentence put down his produce and rolled the stone to
after rearrangement? the side of the road.
1) A 2) C 3) D 4) E 5) F (E) Many people passed by but dodged their duty
Demonstrate = to show something clearly by of moving the stone instead of blaming the
giving proof or evidence king for not keeping the highways clear.

KUNDAN
Directions (Q. 181-185): Rearrange the following (F) One day he placed a heavy stone in the middle
sentences (A), (B), (C), (D), (E) and (F) into a of the road.
meanin gful par agraph and t hen answer t he 186. Which of the following is the SECOND sentence
questions given below it. after rearrangement?
(A) Moreover salaries in public sector enterprises 1) B 2) C 3) D 4) E 5) F
are not as competitive as those offered by 187. Which of the following is the SIXTH (LAST)
private or foreign corporates. sentence after rearrangement?
(B) This trend should be a wake-up call for 1) A 2) B 3) C 4) D 5) E
stakeholders to examine why employees are 188. Which of the following is the FIRST sentence
seeking better opportunities with private after rearrangement?
companies in India and abroad. 1) A 2) B 3) C 4) D 5) F
(C) Public Sector Enterprises (PSEs) have been 189. Which of the following is the THIRD sentence
experiencing severe challenges in attracting, after rearrangement?
motivating and retaining their key staff. 1) A 2) B 3) C 4) D 5) E
(D) Having ide ntifie d the se as the re asons 190. Which of the following is the FIFTH sentence
employees leave PSEs, it is important to after rearrangement?
empower stakeholders to find ways to remedy 1) B 2) C 3) D 4) E 5) F
the situation. Nearby = near in position; not far away; at a
(E) One reason is that young employees lured short distance from somebody/
away by private firms are more willing to something
undertake professional risks. Dodge = to move quickly and suddenly to one
side or out of the way in order to
(F) Employees in specialist roles especially have
avoid somebody/something
become increasingly difficult to retain.
Directions (Q. 191-195): Rearrange the following
181. Which of the following should be the FIRST
six sentences (A), (B), (C), (D), (E) and (F) in the
sentence after rearrangement?
proper sequence to form a meaningful paragraph;
1) A 2) B 3) C 4) D 5) E
then answer the questions given below them.
Sentence Arrangement 539

(A) One of them said, “I never buy imported 200. Which of the following should be the THIRD
goods.” sentence in the paragraph?
(B) Patriotism was the topic of discussion. 1) A 2) B 3) C 4) D 5) E
(C) “Well,” said the third, “I haven’t passed the Tap = to extract or obtain a supply of
foreign language ie English, since I started something from a source
my schooling.” Directions (Q. 201-205): Rearrange the following
(D) Some stude nts we re discussing among six statements (A), (B), (C), (D), (E) and (F) in the
themselves. proper sequence to form a meaningful paragraph;
(E) “I never see foreign films,” said another. then answer the questions given below them.
(F) In the discussion, everyone was trying to (A) Other factors are important in determining
prove how he was more patriotic than the whether a stock market decline causes an
rest of them. economic setback.
191. Which of the following will be the SECOND (B) We tend to associate a stock market crash
statement after rearrangement? with an economic slump.
1) A 2) B 3) C 4) D 5) E (C) The two most important factors are the
192. Which of the following will be the FOURTH impact on the banking sector and policy
statement after rearrangement? response to the crash.
1) A 2) B 3) C 4) D 5) E (D) That is because we have seen such a link
193. Which of the following will be the SIXTH (LAST) several times in the past.
statement after rearrangement ? (E) But there is no automatic link between the
1) A 2) B 3) C 4) D 5) E stock marke t crash and the e conomic

K
194. Which of the following will be the FIR ST downturn, say experts.
statement after rearrangement? (F) For example, the Great Depression followed
1) A 2) B 3) C 4) D 5) E the Great Crash of 1929 in the US.
195. Which of the following will be the FIF TH 201. Which of the following should be the FIRST
statement after rearrangement? statement after rearrangement?
1) A 2) B 3) C 4) D 5) E 1) A 2) B 3) C 4) D 5) E
Directions (Q. 196-200): Rearrange the following 202. Which of the following should be the SECOND
six sentences (A), (B), (C), (D), (E) and (F) in the statement after rearrangement?
proper sequence to form a meaningful paragraph; 1) A 2) B 3) C 4) D 5) E
then answer the questions given below them. 203. Which of the following should be the THIRD
(A) In fact, today, social entrepreneurship is no statement after rearrangement?
different from starting a profit-motivated 1) B 2) C 3) D 4) E 5) F

KUNDAN
company. 204. Which of the following should be the FIFTH
(B) The major challenge they face is employee statement after rearrangement?
selection and retention. 1) A 2) B 3) C 4) D 5) E
(C) For decades social development in India 205. Which of the following should be the SIXTH
meant charity. (LAST) statement after rearrangement?
(D) However, the challenges social organisations 1) A 2) B 3) C 4) D 5) E
experience are tougher. Slump = a period when business is bad,
sales are few etc
(E) This is because while volunteering for social
Downturn = a reduction in economic or business
work is not new, attracting talented people activity
and tapping their potential at lower costs is Directions (Q. 206-210): Rearrange the following
difficult. six sentences (A), (B), (C), (D), (E) and (F) in the
(F) In recent years, however, economic changes proper sequence to form a meaningful paragraph;
have brought busine ss se nse and then answer the questions given below them.
professionalism to the sector. (A) In this early period a good memory was a
196. Which of the following should be the FIFTH prerequisite for success and poets like Homer
sentence in the paraaraph? memorised their work before it was ever
1) A 2) B 3) C 4) D 5) E written down.
197. Which of the following should be the SIXTH (B) If we have to remember everything will it not
(LAST) sentence in the paragraph? increase the feeling of stress?
1) B 2) C 3) D 4) E 5) F (C) Today memory is widely regarded as a useful
198. Which of the following should be the SECOND aid to survival.
sentence in the paragraph? (D) However, it is not what we grasp but what we
1) B 2) C 3) D 4) E 5) F fail to—forgetting a file, key points at an
199. Which of the following should be the FIRST interview— which causes stress.
sentence in the paragraph? (E) Some people, however, are of the view that
1) A 2) B 3) C 4) D 5) E
540 Test of English Language

having an exceptional memory in a world of proper sequence to form a meaningful paragraph;


high pressure working is a disadvantage. then answer the questions given below them.
(F) To our ancestors, though, in the absence of (A) Assuming that all these reasons are true,
the printing press it was much more—it was the fact remains that there is an urgent need
the slate on which history was recorded. to check the accelerated costs and initiate
206. Which of the following will be the THIRD suitable measures.
sentence after rearrangement ? (B) Some people attribute it to the increasing
1) A 2) B 3) C 4) D 5) E greediness among the medicos.
207. Which of the following will be the FIR ST (C) The impact of these measures will be visible
sentence after rearrangement? only after a considerable passage of time.
1) A 2) B 3) C 4) D 5) E (D) Healthcare costs have been sky-rocketing in
208. Which of the following will be the SECOND our country.
sentence after rearrangement? (E) The measures include yoga classes with
1) B 2) C 3) D 4) E 5) F emphasis on physical and mental exercises
209. Which of the following will be the SIXTH (LAST) and also change in food habits.
sentence after rearrangement? (F) Certain others feel that it is because of
1) B 2) C 3) D 4) E 5) F drastic changes in people's lifestyle and
210. Which of the following will be the FIF TH eating habits.
sentence after rearrangement? 216. Which of the following would be the FIRST
1) A 2) B 3) C 4) D 5) E statement after rearrangement?

K
Prerequisite = a thing required as a condition for 1) A 2) B 3) C 4) D 5) E
something to happen or exist 217. Which of the following would be the SECOND
Directions (Q. 211-215): Rearrange the following statement after rearrangement?
six sentences (A), (B), (C ), (D), (E) and (F) in the 1) A 2) B 3) C 4) D 5) E
proper sequence to form a meaningful paragraph; 218. Which of the following would be the FOURTH
then answer the questions given below them. statement after rearrangement?
(A) It was a cycling race launched in 153, by Henri 1) A 2) B 3) C 4) D 5) E
Desgrange, a magazine editor in Paris. 219. Which of the following would be the FIFTH
(B) The Tour de France is a te st of human statement after rearrangement?
endurance. 1) A 2) B 3) C 4) D 5) E
(C) His idea worked and the magazine boomed. 220. Which of the following would be the SIXTH
(D) His aim was to boost the circulation of his (LAST) statement after rearrangement?

KUNDAN
magazine. 1) A 2) B 3) C 4) D 5) E
(E) He wanted to achieve this by covering every Attribute = to regard something as belonging
stage of the three-week-long, 3,500 kilometre- to, caused by or produced by
long cycling race. somebody/something
(F) Till today the race remains more popular than Medico = a medical student or doctor
he could ever have dreamed. Sky-rocket = to rise to a very high level
211. Which of the following should be the FIRST Directions (Q. 221-225): Rearrange the following
sentence after rearrangement? six sentences (A), (B), (C), (D), (E) and (F) in the
1) A 2) B 3) C 4) D 5) E proper sequence to form a meaningful paragraph;
212. Which of the following should be the SECOND then answer the questions given below them.
sentence after rearrangement? (A) If China is the world’s factory, India has
1) A 2) B 3) C 4) D 5) E become the world’s outsourcing centre—
213. Which of the following should be the THIRD keeping in line with this image.
sentence after rearrangement? (B) But India’s future depends crucially on its
1) A 2) B 3) C 4) D 5) E ability to compe te fully in the Cre ative
214. Which of the following should be the FIFTH Economy—not just in tech and software, but
sentence after rearrangement? across design and entrepreneurship; arts,
1) A 2) B 3) C 4) D 5) F culture and e nte rtainme nt; and the
215. Which of the following should be the SIXTH knowledge-based professions of medicine,
(LAST) sentence after rearrangement? finance and law.
1) B 2) C 3) D 4) E 5) F (C) While its creative assets outstrip those of
Endurance = the abiltiy or willingness to suffer other e merging competitors, India must
patiently and without complaining address several challenges to increase its
or to tolerate a difficult situation for international competitiveness as the world
a long time is in the midst of a sweeping transformation.
Directions (Q. 216-220): Rearrange the following (D) This transformation is evident in the fact that
six sentences (A), (B), (C), (D), (E) and (F) in the the world is moving from an industrial
Sentence Arrangement 541

e conomy to a Cre ative Economy that 226. Which of the following should be the LAST
generates wealth by harnessing intellectual (SIXTH) sentence after rearrangement?
labour, intangible goods and human creative 1) A 2) B 3) C 4) D 5) F
capabilities. 227. Which of the following should be the THIRD
(E) Its software industry is the world’s second- sentence after rearrangement?
largest, its tech outsourcing accounts for 1) A 2) B 3) C 4) D 5) F
more than half of the $300 billion global 228. Which of the following should be the FOURTH
industry, according to a technology expert. sentence after rearrangement?
(F) If the meeting of world leaders at Davos is 1) A 2) B 3) C 4) D 5) E
any indication, India is rapidly becoming an 229. Which of the following should be the SECOND
economic ‘rock star’. sentence after rearrangement?
216. Which of the following should be the SIXTH 1) A 2) B 3) C 4) D 5) E
(LAST) sentence after the rearrangement? 230. Which of the following should be the FIRST
1) A 2) B 3) C 4) D 5) E sentence after rearrangement?
217. Which of the following should be the THIRD 1) A 2) B 3) C 4) D 5) E
sentence after the rearrangement? Irrevocable = that cannot be changed or altered;
1) A 2) B 3) C 4) D 5) E final
218. Which of the following should be the FIFTH Directions (Q. 231-235): Rearrange the following
sentence after the rearrangement? six sentences (A), (B), (C), (D), (E) and (F) in the
1) A 2) B 3) C 4) F 5) E proper sequence to form a meaningful paragraph;
219. Which of the following should be the FIRST then answer the questions given below them.

K
sentence after the rearrangement? (A) As a re sult the non-stop te nsions and
1) F 2) B 3) C 4) A 5) E anxieties at work often result in health-
220. Which of the following should be the SECOND related problems.
sentence after the rearrangement? (B) The truth is we cannot change the world of
1) A 2) B 3) C 4) D 5) F work.
Outstrip = to be f as ter, better or more (C) We spend at least half our waking hours at
successful than somebody you are work.
competing against; to become larger, (D) We have the re fore to take charge and
more important, etc than somebody/ transform the way in which we respond to
something our work environment.
In the midst of something = while something is
(E) So how can we control these problems and
happening or being done; while you
are doing something
perform at work?

KUNDAN
Sweeping = having an important effect on a (F) However, we can change the way we feel and
larger part of it deal with various situations.
Harness = to control and use the force or 231. Which of the following should be the LAST
strength of something to produce (SIXTH) sentence after rearrangement?
power or to achieve something 1) B 2) C 3) D 4) E 5) F
Intangible = that does not exist as a physical 232. Which of the following should be the FIFTH
thing but is s till valuable to a
sentence after rearrangement?
company
To account for something = to be the explanation
1) B 2) C 3) D 4) E 5) F
or cause of something 233. Which of the following should be the SECOND
Directions (Q. 226-230): Rearrange the following sentence after rearrangement?
six sentences (A), (B), (C), (D), (E) and (F) in the 1) A 2) B 3) C 4) D 5) E
proper sequence to form a meaningful paragraph; 234. Which of the following should be the FIRST
then answer the questions given below them. sentence after rearrangement?
(A) Se ttle me nt in ‘re al time ’ me ans the 1) A 2) B 3) C 4) D 5) E
transaction is not subjected to any waiting 235. Which of the following should be the FOURTH
period. sentence after rearrangement?
(B) It is a funds transfer mechanism. 1) A 2) B 3) C 4) D 5) E
(C) Moreover, as the money transfer takes place Waking = the state of not being asleep
in the books of RBI it is final and irrevocable. Directions (Q. 236-240): Rearrange the following
(D) The acronym RTGS stands for Real Time six sentences (A), (B), (C), (D), (E) and (F) in the
Gross Settlement. proper sequence to form a meaningful paragraph;
(E) While ‘gross se ttle me nt' me ans the then answer the questions given below them.
transaction is settled without bunching it (A) Nobody likes to practise it, no matter how
with any other transaction. easy and how beneficial it is.
(F) The transfer of funds takes place on a real (B) An ounce of patience is worth, or at times
time and gross basis. even better than, a pound of brains.
542 Test of English Language

(C) Patience, a virtue, is considered to be even Subtle = difficult to detect or describe


better than wisdom. Verse = a group of lines forming a unit in a
(D) This is the worst drawback in us, in our poem or song
Allusion = a brief or indirect reference
national character.
Directions (Q. 246-450): Rearrange the following
(E) Every one of us agrees with this fact in
five sentences (A), (B), (C), (D) and (E) in the proper
principle, without reservation.
sequence to form a meaningful paragraph; then
(F) But, unfortunate ly, whe n it come s to
answer the questions given below them.
practising patience, there is a problem.
(A) For instance , if we me asure the room
236. Which of the following should be the FIRST
temperature continuously and plot its graph
statement after rearrangement?
with time on the X-axis and temperature on
1) A 2) B 3) C 4) D 5) E
the Y-axis, we get a continuous waveform,
237. Which of the following should be the SECOND
which is an analog signal. Analog is always
statement after rearrangement?
continuous.
1) A 2) B 3) C 4) D 5) E
(B) The absence or presence of something can
238. Which of the following should be the THIRD
be used to plot a digital signal.
statement after rearrangement?
(C) An analog signal is a continuously varying
1) A 2) B 3) C 4) D 5) E
signal, similar to a sinusoidal waveform.
239. Which of the following should be the FIFTH
(D) Any signal can be classified into one of the
statement after rearrangement?
two types: analog and digital.
1) A 2) B 3) C 4) D 5) E

K
(E) In contrast, a digital signal takes the form of
240. Which of the following should be the SIXTH
pulses, where we have something or nothing.
(LAST) statement after rearrangement?
246. Which of the following should be the FIRST
1) A 2) B 3) C 4) D 5) E
Ounce of something = a very small quantity of
sentence after rearrangement?
something 1) A 2) B 3) C 4) D 5) E
At times = sometimes 247. Which of the following should be the FIFTH
Virtue = behaviour that shows high moral sentence after rearrangement?
standards; goodness 1) A 2) B 3) C 4) D 5) E
Directions (Q. 241-245): Rearrange the following 248. Which of the following should be the FOURTH
six sentences (A), (B), (C), (D), (E) and (F) in the sentence after rearrangement?
proper sequence to form a meaningful paragraph, 1) A 2) B 3) C 4) D 5) E
then answer the questions given below them. 249. Which of the following should be the THIRD

KUNDAN
(A) In all varieties of humour, especially the sentence after rearrangement?
subtle ones, it is therefore what the reader 1) A 2) B 3) C 4) D 5) E
thinks which gives extra meaning to these 250. Which of the following should be the SECOND
verses. sentence after rearrangement?
(B) But such a verse may also be enjoyed at the 1) A 2) B 3) C 4) D 5) E
surface level. Directions (Q. 251-255): Rearrange the following
(C) Nonse nse ve rse is one of the most sentences (A), (B), (C), (D), (E) and (F) to make a
sophisticated forms of literature. meanin gful par agraph and t hen answer t he
(D) This fulfils the author’s main intention in questions which follow :
such a verse which is to give pleasure. (A) However while reading they would not know
(E) However, the reader who understands the when to pause and what to emphasise.
broad implications of the content and allusion (B) Since then their use has been regularised
finds greater pleasure. and the punctuation rules have been followed
(F) The reason being it requires the reader to by all.
supply a me aning be yond the surface (C) In earlier days, people learnt by reading out
meaning. loud.
241. Which of the following is the FIFTH sentence? (D) But not e ve rybody use d the same
1) D 2) E 3) B 4) C 5) A punctuations for the same thing.
242. Which of the following is the SIXTH (LAST) (E) To addre ss this proble m, various signs
sentence? de picting various punctuations we re
1) F 2) E 3) D 4) A 5) C introduced.
243. Which of the following is the FIRST sentence? (F) Thus firme r guide line s re garding
1) E 2) A 3) F 4) D 5) C punctuations were framed so that everyone
244. Which of the following is the SECOND sentence? used them in similar way.
1) A 2) E 3) F 4) B 5) C 251. Which of the following sentence should be the
245. Which of the following is the THIRD sentence? FIRST after rearrangement?
1) A 2) B 3) F 4) C 5) D 1) A 2) B 3) C 4) D 5) E
Sentence Arrangement 543

252. Which of the following sentence should be the (C) This means that it gives permission for the
SECOND after rearrangement? buyer to use its name and sell its products.
1) A 2) B 3) D 4) E 5) F (D) He pays money to the franchisor, and agrees
253. Which of the following sentence should be the to obey the rules the franchisor makes.
THIRD after rearrangement? (E) A franchising agreement includes two parties.
1) A 2) E 3) D 4) F 5) C (F) The franchisor is the business house/entity
254. Which of the following sentence should be the which grants the franchisee license.
FIFTH after rearrangement? 261. Which of the following should he the FOURTH
1) B 2) C 3) A 4) E 5) F sentence after rearrangement?
255. Which of the following sentence should be the 1) B 2) C 3) D 4) E 5) F
SIXTH (LAST) after rearrangement? 262. Which of the following should be the LAST
1) C 2) E 3) D 4) B 5) F (SIXTH) sentence after rearrangement?
Directions (Q. 256-260): Rearrange the following 1) B 2) C 3) D 4) E 5) F
sentences (A), (B), (C), (D), (E) and (F) to make a 263. Which of the following should be the SECOND
meaningful paragraph and then answer the sentence after rearrangement?
questions which follow: 1) B 2) C 3) D 4) E 5) F
(A) Had it been not for them, Indian banks would 264. Which of the following should be the THIRD
have had their hands tied down. sentence after rearrangement?
(B) Today, almost all the countries are facing the 1) E 2) F 3) A 4) D 5) B
heat of recession. 265. Which of the following should be the FIRST
(C) One of these is the strict RBI and SEBI rules sentence after rearrangement?

K
which re gulate d banking se ctor ve ry 1) A 2) B 3) C 4) D 5) E
efficiently. Directions (Q. 266-270): Rearrange the following
(D) This could have led to massive losses to sentences (A), (B), (C), (D), (E) and (F) to make a
them, which could have percolated to other meanin gful par agraph and t hen answer t he
sectors as well. questions which follow:
(E) However, there are a few things which help (A) The only way in which this problem can be
India in bouncing back from the state of solved is by making artificial blood, which has
recession. remained a distant dream for science.
(F) Like others, India too has not remained (B) Donation of blood is considered to be the
immune to the epidemic. most noble of all the charities.
256. Which of the following sentences should be the (C) If they succeed, it would be noted as one of
THIRD after rearrangement? the most important inventions in the history

KUNDAN
1) A 2) E 3) D 4) F 5) C of mankind.
257. Which of the following sentences should be the (D) This is because this donated magic potion
FIRST after rearrangement? can give life to anothe r pe rson in an
1) A 2) B 3) C 4) D 5) E emergency.
258. Which of the following sentences should be the (E) A group of scientists, however, has dedicated
SECOND after rearrangement? itself towards making this a reality.
1) A 2) B 3) D 4) E 5) F (F) A growing problem, however, is that the
259. Which of the following sentences should be the requirement for safe blood is increasing
SIXTH (LAST) after rearrangement? whereas the number of donors is decreasing.
1) A 2) E 3) D 4) B 5) F 266. Which of the following sentences should be the
260. Which of the following sentences should be the FIRST after rearrangement?
FIFTH after rearrangement? 1) A 2) B 3) C 4) D 5) E
1) B 2) C 3) A 4) E 5) F 267. Which of the following sentences should be the
Percolate = to move gradually through a surface SECOND after rearrangement?
containing tiny holes or spaces 1) A 2) B 3) C 4) D 5) F
Bounce back = to recover well after trouble, illness, 268. Which of the following sentence should be the
hardship etc
THIRD after rearrangement?
Epidemic = the rapid spread of a diseas e
among many people in the same
1) A 2) D 3) C 4) E 5) F
place 269. Which of the following sentences should be the
Directions (Q. 261-265): Rearrange the following FIFTH after rearrangement?
six sentences (A), (B), (C), (D), (E) and (F) in the 1) A 2) B 3) C 4) E 5) F
proper sequence to form a meaningful paragraph; 270. Which of the following sentences should be the
then answer the questions given below them. SIXTH (LAST) after rearrangement?
(A) In turn, the buyer is called the franchisee. 1) A 2) B 3) C 4) D 5) E
(B) These two parties are called the franchisor
and franchisee.
544 Test of English Language

Noble = having or showing very f ine (E) Without cheques, they are likely to keep large
personal qualities, eg honour and amounts of cash in their homes, making
honesty themselves vulnerable to theft.
Potion = a drink of medicine or poison, or a
(F) British banks have thus vote d to phase
liquid used in magic
cheques out in favour of these more modern
Directions (Q. 271-275): Rearrange the following
payment methods.
sentences (A), (B), (C), (D), (E) and (F) to make a
276. Which of the following should be the LAST
meanin gful par agraph and t hen answer t he
(SIXTH) sentence after rearrangement?
questions which follow:
1) B 2) C 3) D 4) E 5) F
(A) This de cision on which force is to be
277. Which of the following should be the THIRD
maximised lies with every individual.
sentence after rearrangement?
(B) This belief comes from the fact that science
1) B 2) C 3) D 4) E 5) F
refle cts the social forces prevailing at a
278. Which of the following should be the FIFTH
particular time.
sentence after rearrangement?
(C) We must maximise the constructive forces
1) A 2) B 3) C 4) D 5) E
of science and the destructive ones should
279. Which of the following should be the FIRST
be minimised. . .
sentence after rearrangement?
(D) Contrary to popular belief, the greatest enemy
1) B 2) C 3) D 4) E 5) F
of mankind is not science but war.
280. Which of the following should be the SECOND
(E) She/he must understand that science can
sentence after rearrangement?
only help in providing ways to reach at either

K
1) A 2) B 3) C 4) D 5) F
war or peace and is actually not responsible
Elderly = rather old; past middle age
for causing these. Vulnerable = that can be hurt, harmed or attacked
(F) During peaceful times science is constructive easily especially because of being
and during war, science is perverted to small or weak
destructive ends. Phase something out = to withdraw or stop using
271. Which of the following sentences should be the something gradually or in stages.
SIXTH (LAST) after rearrangement? Directions (Q. 281-285): Rearrange the following
1) A 2) B 3) C 4) D 5) E sentences (A), (B), (C), (D), (E) and (F) to make a
272. Which of the following sentences should be the meanin gful par agraph and t hen answer t he
THIRD after rearrangement? questions which follow:
1) A 2) E 3) F 4) D 5) C (A) According to it, organised retail stores are

KUNDAN
273. Which of the following sentences should be the not wolve s at the doors of frie ndly
FIFTH after rearrangement? neighbourhood grocery stores as there is room
1) A 2) B 3) C 4) E 5) F for expansion of both.
274. Which of the following sentences should be the (B) Many have been crying foul over the entry of
FIRST after rearrangement? organised retail stores expressing concern
1) A 2) B 3) C 4) D 5) E over their impact on small store owners.
275. Which of the following sentences should be the (C) The final winner in the competition, however,
SECOND after rearrangement? is the common man who gets to choose
1) A 2) B 3) D 4) E 5) F between the most suitable options and in
Pervert = to make somebody, their mind, etc turn fights with the runaway inflation in prices
turn away from what is considered of essential commodities.
right or natural (D) In spite of this potential for expansion, it is
Directions (Q. 276-280): Rearrange the following doubtless that the small store owners face a
six sentences (A), (B), (C), (D), (E) and (F) in the decline in profit in initial years if organised
proper sequence to form a meaningful paragraph; retailers set up stores in the vicinity.
then answer the questions given below them. (E) But a study conducted over a period of two
(A) Howe ve r, if this happe ns it will cause years goes a long way towards allaying these
problems for the elderly who mainly use fears.
cheques. (F) This impact, however, wears off once they
(B) The use of cheques has fallen dramatically learn to take on the competition which in
in the past few years. turn enhances efficiency all around.
(C) Thus, cheques may be phased out gradually, 281. Which of the following sentences should be the
making sure that the needs of all consumers, FIRST after rearrangement?
including the elderly, are met. 1) A 2) B 3) C 4) D 5) E
(D) This is because more and more consumers 282. Which of the following sentences should be the
are transferring money electronically by direct THIRD after rearrangement?
debit or credit cards. 1) A 2) E 3) D 4) F 5) C
Sentence Arrangement 545

283. Which of the following sentences should be the 286. Which of the following sentences should be the
SIXTH (LAST) after rearrangement? FIFTH after rearrangement?
1) A 2) B 3) C 4) E 5) F 1) A 2) B 3) C 4) E 5) F
284. Which of the following sentences should be the 287. Which of the following sentences should be the
SECOND after rearrangement? THIRD after rearrangement?
1) A 2) B 3) C 4) E 5) F 1) A 2) B 3) C 4) D 5) E
285. Which of the following sentences should be the 288. Which of the following sentences should be the
FIFTH after rearrangement? FIRST after rearrangement?
1) D 2) B 3) C 4) E 5) F 1) A 2) B 3) C 4) D 5) E
Keep the wolf from = to have enough money to 289. Which of the following sentences should be the
the door avoid hunger and need SIXTH (LAST) after rearrangement?
Foul = very unpleasant; very bad; 1) A 2) C 3) D 4) E 5) F
terrible
290. Which of the following sentences should be the
Runaway = happening very rapidly or
easily
SECOND after rearrangement?
Vicinity = the area round a place 1) A 2) B 3) D 4) E 5) F
Allay = to make something less; to Stimulate = to make somebody/something more
relieve something active or al ert; to encourage
Wear something off = to disappear or remove somebody/something
something gradually. Turn-around = a complete change in a situation or
Take on = to assume something; to begin a trend, especially from bad to good
to have a particular Exorbitant = much too high or great;
characteris tic, quality or unreasonable

K
appearance Abundance = a quantity that is more than enough;
Enhance = to increase or improve further a very large amount
the good qual ity, val ue or Overshadow = to dominate somebody/something;
status of somebody/ to make somebody something seem
something less important or noticeable
Directions (Q. 286-290): Rearrange the following Directions (Q. 291-295): Rearrange the following
sentences (A), (B), (C), (D), (E) and (F) to make a six sentences (A), (B), (C), (D), (E) and (F) in the
meanin gful par agraph and t hen answer t he proper sequence to form a meaningful paragraph;
questions which follow : then answer the questions given below.
(A) While these disadvantages of biofuels are (A) With all the bid information being available
serious, they are the only alternate energy and tracke d online , corruption has
source of the future and the sooner we find considerably reduced.

KUNDAN
solutions to these problems the faster we (B) Today, most, ie over ninety-five per cent,
will be able to solve the problems we are now households, in the city enjoy broadband
facing with gasoline. connection.
(B) This fuel can also help to stimulate jobs (C) All city contracts are now bid for online.
locally since they are also much safer to (D) Over twenty years ago the city government,
handle than gasoline and can thus have the Central Government and the private sector
potential to turn around a global economy. made a concerted effort to shift the economy
(C) These include dependence on fossil fuels for to include IT.
the machinery required to produce biofuel (E) As our cities do expand and become more
which ends up polluting as much as the complex, such a system will make governance
burning of fossil fue ls on roads and more manageable.
exorbitant cost of biofuels which makes it (F) This level of connectedness has changed not
very difficult for the common man to switch only the city’s economy but also how it is
to this option. governed and how business is conducted.
(D) This turnaround can potentially help to bring 291. Which of the following should be the FIRST
world peace and end the need to depend on sentence after rearrangement?
foreign countries for energy requirements. 1) A 2) B 3) C 4) D 5) E
(E) Biofuels are made from plant sources and 292. Which of the following should be the SECOND
since the se source s are available in sentence after rearrangement?
abundance and can be reproduced on a 1) A 2) B 3) C 4) D 5) F
massive scale, they form an energy source 293. Which of the following should be the THIRD
that is potentially unlimited. sentence after rearrangement?
(F) However everything is not as green with the 1) A 2) C 3) D 4) E 5) F
biofuels as it seems as there are numerous 294. Which of the following should be the FIFTH
disadvantages involve d, which at time s sentence after rearrangement?
overshadow their positive impact. 1) A 2) B 3) C 4) D 5) E
546 Test of English Language

295. Which of the following should be the LAST Deprive somebody/something of something =
(SIXTH) sentence after rearrangement? to take something away f rom
1) B 2) C 3) D 4) E 5) F somebody/something; to prevent
Bid = to offer a price in order to buy somebody/s omething f rom
something especially at an auction; enjoying or using something
a price offered in order to buy Directions (Q. 301-305): Rearrange the following
something at an auction or in sentences (A), (B), (C), (D), (E) & (F) to make a
business meanin gful par agraph and t hen answer t he
Concerted = arranged or done together with questions which follow:
somebody (A) If tomorrow’s children meet these two crucial
Directions (Q. 296-300): Rearrange the following criteria they are likely to have the opportunity
sentences (A), (B), (C). (D), (E) and (F) to make a both to pursue work to fulfil their dreams
meanin gful par agraph and t hen answer t he and make an impact on the world around.
questions which follow: (B) In the 21st century, however, this no longer
(A) In fact, according to mainstream economists, holds true as the e xpe ctations of
it is inevitable and a necessary evil in any organisations have changed.
economy. (C) The concept of job in the 20th century was
(B) It is thus important for every nation to often equated with toil.
maintain this reserve of labour force to (D) In order to meet these changed expectations,
maintain an optimal level of unemployment. employees need commitment and access to
(C) Unemployment is popularly believed to be an continuous learning.

K
index which measures the economic condition (E) Jobs were acquired, learned and performed
of a nation. until retirement and did not have to be
(D) This is because it helps avert inflation by rewarding.
providing a reserve army of labour which (F) They demand more creativity and expertise
keeps wages in check. and want employees to be responsible for both
(E) The problem, however, only emerges when outputs and outcomes.
gove rnme nts indire ctly facilitate 301. Which of the following should be the FIRST
unemployment in order to curb inflation sentence after rearrangement?
through various policies and frameworks, 1) A 2) B 3) C 4) D 5) E
de priving a large population of its 302. Which of the following should be the SECOND
fundamental rights. sentence after rearrangement?

KUNDAN
(F) But contrary to popular belief unemployment 1) A 2) B 3) C 4) D 5) E
is not always disadvantage ous to the 303. Which of the following should be the THIRD
economy of a state. sentence after rearrangement?
296. Which of the following sentences should be the 1) B 2) C 3) D 4) E 5) F
SIXTH (LAST) after rearrangement? 304. Which of the following should be the FIFTH
1) A 2) B 3) C 4) E 5) F sentence after rearrangement?
297. Which of the following sentences should be the 1) A 2) B 3) C 4) D 5) F
FIRST after rearrangement? 305. Which of the following should be the LAST
1) A 2) B 3) C 4) D 5) E (SIXTH) sentence after rearrangement?
298. Which of the following sentences should be the 1) A 2) B 3) C 4) D 5) F
FIFTH after rearrangement? Toil = work that is hard and makes one
1) A 2) B 3) C 4) E 5) F very tired
299. Which of the following sentences should be the Outcome = the effect or result of an action or
SECOND after rearrangement? event
1) A 2) B 3) D 4) E 5) F Output = the amount of something that a
pers on, a machine or an
300. Which of the following sentences should be the
organisation produces
THIRD after rearrangement?
Directions (Q. 306-310): Rearrange the following
1) A 2) E 3) D 4) F 5) C
six sentences (A), (B), (C), (D), (E) and (F) in the
Inevitable = certain to happen
Optimal = best or most favourable
proper sequence to form a meaningful paragraph;
Avert = to turn something away; to prevent then answer the questions given below them.
something; to avoid something (A) It is therefore a contributing factor to the
Emerge = to develop and become noticeable, growth of landfills and waterway pollution,
important or prominent both of which are costly and energy-intensive
Facilitate = to make something especially an to solve.
action or a process, easy or easier (B) Making an effort to use those resources and
Curb = to restrain something; to prevent
avoid polystyrene ones can help to decrease
something from getting out of
control
your environmental impact.
Sentence Arrangement 547

(C) Non-biodegradable essentially means that 311. Which of the following will be the THIRD
any polystyrene that makes its way into a sentence?
landfill will stay there indefinitely, never 1) A 2) B 3) C 4) D 5) E
breaking down and returning to the earth. 312. Which of the following will be the SECOND
(D) Polystyrene, as a product, is very convenient sentence?
to use, but it has some important effects we 1) A 2) B 3) C 4) D 5) E
should consider when making choices as 313. Which of the following will be the FOURTH
consumers, sentence?
(E) While recycling polystyrene material can 1) A 2) B 3) C 4) D 5) E
cushion the environmental blow of its use, 314. Which of the following will be the FIR ST
alternatives are available that are created sentence?
from renewable resources and biodegrade 1) A 2) B 3) C 4) D 5) E
more readily. 315. Which of the following will be the FIFTH (LAST)
(F) For example, while polystyrene has some sentence?
excellent uses and is technically recyclable, 1) A 2) B 3) C 4) D 5) E
it is not a substance that biodegrades. Ageing = the process of growing old
306. Which of the following should be the LAST Oblivious = not aware of or not noticing
(SIXTH) sentence after rearrangement? something
1) A 2) B 3) C 4) D 5) E Directions (Q. 316-320): Rearrange the following
307. Which of the following should be the FOURTH six sentences (A), (B), (C), (D), (E) and (F) in the
sentence after rearrangement? proper sequence to form a meaningful paragraph;

K
1) E 2) F 3) A 4) D 5) B then answer the questions given below them.
308. Which of the following should be the FIRST (A) A legal framework is thus now available for
sentence after rearrangement? promoting energy efficiency in all sectors of
1) A 2) B 3) C 4) D 5) E the economy.
309. Which of the following should be the SECOND (B) The increasing preference for commercial
sentence after rearrangement? energy has led to a sharp increase in the
1) B 2) C 3) D 4) E 5) F demand for electricity and fossil fuels.
310. Which of the following should be the FIFTH (C) There is still a considerable potential for
sentence after rearrangement? repairing such damage and reducing energy
1) A 2) B 3) C 4) D 5) E consumption by adopting energy-efficiency
Landfill = an area of land where waste material measures at various sectors of our country.
is buried under layers of earth (D) This framework is nothing but the Energy

KUNDAN
Cushion = to soften the effect of an impact; to Conservation Act, 2001, the success of which
protect somebody/something from greatly depends on the people who take the
s omething harmf ul , sometimes lead in supporting this programme.
excessively. (E) This use of fossil fue ls has re sulted in
Directions (Q. 311-315): Rearrange the following emission of a huge quantity of carbon dioxide
five sentences (A), (B), (C), (D) and (E) in the proper causing serious environmental damage.
sequence to form a meaningful paragraph; then (F) These adopted measures will not only reduce
answer the questions given below them. the need to create new capacity requiring high
(A) During this ageing process, you had first investment, but also result in substantial
heard statements such as: “You can’t ride environmental benefits.
your bicycle until you are seven” and, “You 316. Which of the following should be the FIRST
can’t drive a car until you’re 18.” sentence after rearrangement?
(B) But, you need to remember that part of you 1) A 2) B 3) C 4) D 5) E
has an ageless mind and is quite oblivious 317. Which of the following should be the FOURTH
to the physical ageing process — you just sentence after rearrangement?
need to encourage it to overcome this excuse 1) E 2) F 3) A 4) D 5) B
of being old. 318. Which of the following should be the SECOND
(C) The age excuse of being too old comes from sentence after rearrangement?
an inclination to identify yourself with the 1) B 2) C 3) D 4) E 5) F
number of trips you have made around the 319. Which of the following should be the LAST
sun. (SIXTH) sentence after rearrangement?
(D) The age of your body can seem to be quite an 1) A 2) B 3) C 4) D 5) E
obstacle on the road to changing long-held 320. Which of the following should be the FIFTH
thinking habits. sentence after rearrangement?
(E) Then at some point you discovered that you 1) A 2) B 3) C 4) D 5) E
went from being not old enough to being too
old.
548 Test of English Language

Directions (Q. 321-325): Rearrange the following (B) “Speak to her,” he said, “She’s into books”.
six sentences (A), (B), (C), (D), (E) and (F) in the (C) The friend who had brought me there noticed
proper sequence to form a meaningful paragraph; my noticing her.
then answer the questions given below them. (D) In late 2003, I was still paying taxes in
(A) The Reserve Bank of India (RBI), which had America, so it horrifie d me that the US
last intervened in the foreign exchange Consulate was hosting a “Gallo drinking
market in June 2009, was seen buying dollars appreciation event”.
to stem the rupee’s runaway appreciation. (E) Behind them, a pianist was playing old film
(B) The bank had for so long resisted a rate tunes, and a slim short woman was dancing
increase, but finally toed the line of other around him.
banks as its cost of funds has gone up. 326. Which of the following would be the FOURTH
(C) The local currency recently rose to a 25- sentence?
month high against the dollar on sustained 1) A 2) B 3) C 4) D 5) E
capital inflows. 327. Which of the following would be the FIRST
(D) Some interesting things have happened in sentence?
the Indian financial system in the past few 1) A 2) B 3) C 4) D 5) E
weeks. 328. Which of the following would be the FIFTH
(E) The RBI also announced a `12,000 crore buy- (LAST) sentence?
back of government bonds from the market 1) A 2) B 3) C 4) D 5) E
to infuse money into a liquidity-starved 329. Which of the following would be the SECOND

K
banking system. sentence?
(F) Finally, the State Bank of India, the nation’s 1) A 2) B 3) C 4) D 5) E
large st lende r, has raise d its minimum 330. Which of the following would be the THIRD
lending rate, or base rate, by 10 basis points. sentence?
(One basis point is one-hundredth of a 1) A 2) B 3) C 4) D 5) E
percentage point) Directions (Q. 331-335): Rearrange the following
321. Which of the following would be the THIRD six sentences (A), (B), (C), (D), (E) and (F) in the
sentence? proper sequence to form a meaningful paragraph;
1) B 2) A 3) D 4) E 5) F then answer the questions given below.
322. Which of the following would be the SIXTH (A) Ironically the same pare nts who are
(LAST) sentence? considered to be ignorant are thought to be
1) B 2) C 3) D 4) E 5) A very enlightened in choosing private schools

KUNDAN
323. Which of the following would be the SECOND over state-run ones.
sentence? (B) This is all the more reason why we should
1) A 2) C 3) D 4) E 5) F include the m during the planning and
324. Which of the following would be the FOURTH implementation of the system.
sentence? (C) This is apparent at every stage from policy
1) B 2) C 3) D 4) A 5) E making to imple me ntation as critical
325. Which of the following would be the FIRST decisions are made without the participation
sentence? of the stakeholders, an attitude that can only
1) B 2) C 3) D 4) E 5) F be de scribe d as e ithe r arrogance or
Intervene = to come or be between indifference.
To stem = to stop something which is (D) In reality, every parent decides which school
fl owing, s preading or is a good one, based on his/her own set of
increasing
values, perceptions and aspirations.
Runaway = happening very rapidly and
easily
(E) The root cause of most of the ills that plague
To toe the line = to obey the orders and express our e ducation syste m is the e normous
the opinions of one’s group or distance that separates the power centres
party; to conform within the system and the schools where the
Infuse = to put a quality into somebody/ action takes place.
something; to fill somebody/ (F) It is often said in defense of such an approach
something with a quality that poor parents are too ignorant to be
Directions (Q. 326-330): Rearrange the following partners in a meaningful dialogue.
five sentences (A), (B), (C), (D) and (E) in the proper 331. Which of the following should be the SIXTH
sequence to form a meaningful paragraph; then (LAST) sentence after rearrangement?
answer the questions given below. 1) A 2) B 3) C 4) D 5) E
(A) “What a waste of my tax money,” I thought, 332. Which of the following should be the THIRD
walking past the pe ople having fre e sentence after rearrangement?
Californian Chardonnay. 1) A 2) B 2) C 3) D 5) F
Sentence Arrangement 549

333. Which of the following should be the SECOND Ignorant = rude through lack of knowledge of
sentence after rearrangement? or respect for good manners
1) F 2) D 3) C 4) E 5) A Enlightened = having or s howing an
understanding of what the current
334. Which of the following should be the FIRST
situation requires , rather than
sentence after rearrangement? following conventional thought
1) A 2) B 3) C 4) D 5) E Apparent = clearly seen or undrestood; obvious
335. Which of the following should be the FOURTH Arrogant = behaving in a proud and superior
sentence after rearrangement? manner; showing too much pride in
1) E 2) D 3) C 4) B 5) A oneself and too l ittl e
Ironically = it seems ironic that considerations for others
Ironic = happening in the opposite way to Indifference = a lack of interest, feeling or reaction
what is expected, and often causing towards somebody/something
amusement because of this Plague = to cause trouble or difficulty to
somebody/something especially
continually or repeatedly

Answers and explantions


Exercise-1

K
1. 5 2. 1 3. 3 4. 2 5. 4 51. 5 52. 3 53. 5 54. 1 55. 2
6. 4 7. 2 8. 5 9. 1 10. 4 56. 4 57. 5 58. 2 59. 4 60. 1
11. 3 12. 4 13. 3 14. 5 15. 2 61. 3 62. 3 63. 2 64. 5 65. 5
16. 3 17. 2 18. 1 19. 5 20. 2 66. 2 67. 1 68. 5 69. 2 70. 3
21. 1 22. 4 23. 4 24. 3 25. 5 71. 5 72. 3 73. 1 74. 2 75. 2
26. 5 27. 1 28. 4 29. 2 30. 2 76. 3 77. 4 78. 5 79. 2 80. 4
31. 3 32. 5 33. 3 34. 2 35. 4 81. 1 82. 5 83. 4 84. 5 85. 3
36. 4 37. 1 38. 4 39. 3 40. 2 86. 4 87. 2 88. 1 89. 3 90. 5
41. 5 42. 2 43. 1 44. 3 45. 5 91. 2 92. 4 93. 1 94. 4 95. 2
46. 4 47. 4 48. 3 49. 2 50. 1 96. 5 97. 3 98. 1

KUNDAN
Exercise-2
(1-5): C come s first be cause it is the only (22-26):
independent sentence. We then place the 22. 5 23. 1 24. 3 25. 2 26. 4
sentences having “it is a time”, the phrase (27-31):
that refers to C. Among three such sentences 27. 1 28. 2 29. 5 30. 3 31. 1
— D, E and F —, F comes first because of its (32-35): The sequence of the sentences: ECADBF
plain structure . Also in D makes it the 32. 2 33. 3 34. 5 35. 2
subsequent sentence. While most of all in E (36-40):
makes it the last among these three. Thus, 36. 1 37. 3 38. 2 39. 4 40. 5
our sentences are CFDE. (41-45):
Now, look at the words “visualise them” in B. 41. 5 42. 4 43. 3 44. 2 45. 1
These words are a clear reference to the words (46-50): The right sequence will be CGEAFDB.
in E: “opportunities ... challenges ... ahead.” 46. 1 47. 2 48. 4 49. 4 50. 2
Thus, E is followed by B. The remaining (51-55):
sentence, i.e. A, comes at the end. Thus, we 51. 3 52. 1 53. 4 54. 2 55. 5
have C F D E B A (56-60): Order of the sentences of the passage is
1. 1 2. 1 3. 1 4. 3 5. 4 BEDAC.
(6-11): The order of the sentences will be EGCAFDHB. 56. 5 57. 4 58. 2 59. 1 60. 3
6. 2 7. 1 8. 4 (61-65):
9. 4 10. 5 11. 3 61. 4 62. 1 63. 3 64. 5 65. 2
(12-16): (66-70): The right order of the sentences will be
12. 1 13. 4 14. 3 15. 1 16. 3 ECADB.
(17-21): 66. 4 67. 3 68. 2 69. 5 70. 1
17. 3 18. 2 19. 1 20. 3 21. 3 (71-75):
71. 1 72. 2 73. 4 74. 3 75. 5
550 Test of English Language

(76-80): (181-185): The correct sequence of sentences will be


76. 2 77. 3 78. 5 79. 1 80. 4 CFBEAD.
(81-85): 181. 3 182. 5 183. 2 184. 1 185. 4
81. 1 82. 4 83. 3 84. 1 85. 2 (186-190): The correct sequence of sentences will be
(86-90): CFAEDB.
86. 5 87. 2 88. 4 89. 3 90. 1 186. 5 187. 2 188. 3 189. 1 190. 3
(91-95): Correct order of the sentence will be DCFBEA. (191-195): The correct sequence of sentences will be
91. 1 92. 5 93. 2 94. 4 95. 3 DBFAEC.
(96-100): Correct order of the se ntence will be 191. 2 192. 1 193. 3 194. 4 195. 5
DCFBEA (196-200): The correct sequence of sentences will be
96. 5 97. 4 98. 3 99. 2 100. 1 CFADBE.
(101-105): The arrangement should be DBACE. 196. 2 197. 4 198. 5 199. 3 200. 1
101. 2 102. 3 103. 4 104. 1 105. 5 (201-205): The correct sequence of sentences will be
(106-110:): Correct order of the sentence will be BDFEAC.
BDEFCA. 201. 2 202. 4 203. 5 204. 1 205. 3
106. 3 107. 1 108. 2 109. 4 110. 5 (206-210): The correct sequence of sentences will be
(111-115): Correct order of the sentence will be CFAEBD.
CFDBAE. 206. 1 207. 3 208. 5 209. 3 210. 2
111. 5 112. 2 113. 1 114. 3 115. 4 (211-215): The correct sequence of sentences will be
(116-120): Correct order of the sentence will be BADECF.

K
CEAFDB. 211. 2 212. 1 213. 4 214. 3 215. 5
116. 5 117. 1 118. 5 119. 4 120. 2 (216-220): The correct sequence of sentences will be
(121-125): Correct order of the sentence will be DBFAEC
CFAEBD. 216. 4 217. 2 218. 1 219. 5 220. 3
121. 2 122. 1 123. 3 124. 5 125. 4 (221-225): The correct sequence of sentences will be
(126-130); Correct order of the sentence will be FAEBCD
FDGAECB. 221. 4 222. 5 223. 3 224. 1 225. 1
126. 4 127. 2 128. 1 129. 3 130. 5 (226-230): The correct sequence of sentences will be
(131-135):The correct sequence of sentences will be DBFAEC.
CEAFBD. 226. 3 227. 5 228. 1 229. 2 230. 4
131. 3 132. 5 133. 1 134. 5 135. 4 (231-235): The correct sequence of sentences will be
(136-140): Correct sequence of the sentences will be CAEBFD.

KUNDAN
CFDBAE. 231. 3 232. 5 233. 1 234. 3 335. 2
136. 2 137. 5 138. 3 139. 4 140. 3 (236-240): The correct sequence of sentences will be
(141-145): Correct sequence of the sentences will be CBEFAD.
BDFAEC. 236. 3 237. 2 238. 5 239. 1 240. 4
141. 2 142. 4 143. 1 144. 5 145. 3 (241-245): The correct sequence of sentences will be
(146-150): The correct sequence of the sentences will CFBDEA.
be BDACEF. 241. 2 242. 4 243. 5 244. 3 245. 2
146. 5 147. 1 148. 4 149. 2 150. 3 (246-250): The correct sequence of sentences will be
(151-155): The correct sequence of the sentences will DCAEB.
be. DEBAFC 246. 4 247. 2 248. 5 249. 1 250. 3
151. 5 152. 3 153. 5 154. 4 155. 2 (251-255): The correct sequence of sentences will be
(156-160): Correct sequence of the sentences will be CAEDFB.
DAFBEC. 251. 3 252. 1 253. 2 254. 5 255. 4
156. 4 157. 1 158. 5 159. 3 160. 2 256-260: The correct sequence of sentences will be
(161-165): Correct sequence of the sentences will be BFECAD.
DAFCEB. 256. 2 257. 2 258. 5 259. 3 260. 3
161. 4 162. 1 163. 3 164. 5 165. 2 (261-265): The correct sequence of sentences will be
(166-170): Correct sequence of the sentences will be EBFCAD.
CADFBE. 261. 2 262. 3 263. 1 264. 2 265. 5
166. 3 167. 5 168. 2 169. 1 170. 4 (266-270): The correct sequence of the sentence will
(171-175): The correct sequence of sentences will be be BDFAEC.
ECFDBA. 266. 2 267. 4 268. 5 269. 4 270. 3
171. 4 172. 2 173. 1 174. 3 175. 5 (271-275): The correct sequence of sentences will be
(176-180): The correct sequence of sentences will be DBFCAE.
EBCDFA. 271. 5 272. 3 273. 1 274. 4 275. 2
176. 5 177. 1 178. 2 179. 3 180. 5
Sentence Arrangement 551

(276-280): The correct sequence of sentences will be (316-320): The correct sequence of sentences will be
BDFCAE. BECFAD.
276. 4 277. 5 278. 1 279. 1 280. 4 316. 2 317. 2 318. 4 319. 4 320. 1
(281-285): The correct sequence of sentences will be (321-325): The correct sequence of sentences will be
BEADFC. DCAEFB.
281. 2 9282. 1 283. 3 284. 4 285. 5 321. 2 322. 1 323. 2 324. 5 325. 3
(286-290): The correct sequence of sentences will be (326-330): “he” in sentence (B) is used for “The friend”
EBDFCA. in sentence (C). Hence (B) must follow (C). Playing
286. 3 287. 4 288. 5 289. 1 290. 2 piano and a dancing woman, as mentioned in sentence
(291-295): The correct sequence of sentences will be (E), is what the author thinks as a waste of his money
DBFCAE. state d in sente nce (A). Sentence (D) is giving the
291. 4 292. 2 293. 5 294. 1 295. 4 introduction of the topic. Hence the correct sequence
(296-300): The correct sequence of sentences will be is DAECB.
CFADBE. 326. 3 327. 4 328. 2 329. 1 330. 5
296. 4 297. 3 298. 2 299. 5 300. 1 (331-335): Every other sentence except (E) has a key
(301-305): The correct sequence of sentences will be connector in it, suggesting that (E) inde ed is the
CEBFDA. opening sentence. ‘This’ in sentence (C) is used for the
301. 3 302. 5 303. 1 304. 4 305. 1 situation de scribe d in (E). He nce, (C) follows (E).
(306-310): The correct sequence of sentences will be Sentence (C) supports the sentence (B). Hence, (B)
DFCAEB. follows (C). (A) follows (F) because (A) is contradicting
306. 2 307. 3 308. 4 309. 5 310. 5 the condition mentioned in (F) using ‘ironically’. Hence,

K
(311-315): The correct sequence of sentences will be the correct sequence is ECDBFA
DAECB. 331. 2 332. 5 333. 3 334. 5 335. 5
311. 5 312. 1 313. 3 314. 4 315. 2

KUNDAN
300 Test of English Language

Chapter 8

Vocabulary Test
Directions: In each of the following questions 5. (A) Fallacy (B) Adage
four words are given of which two are most nearly (C) Dictum (D) Endorse
the same or opposite in meaning. Find the two 1) B-D 2) C-D 3) B-C
words which are most nearly the same or opposite 4) A-D 5) A-B
in meaning and find the number of the correct Fallacy = a false or mistaken belief.
letter combination. Adage = a traditional phrases expressing a
1. (A) Enthralling (B) Respecting general truth.
Dictum = a s hort, of ten well -known,
(C) Projecting (D) Alluring
statement; a saying.
1) A-B 2) B-C 3) C-D Endorse = to give one’s approval or support to
4) A-D 5) B-D a claim, statement, course of action.
Enthrall = to capture the whole of somebody’s 6. (A) Elevate (B) Frugal
attention; to interest or entertain
(C) Exult (D) Lament
somebody greatly
Allure = the quality of being attractive or
1) C-D 2) A-B 3) B-C
charming 4) B-D 5) A-D
2. (A) Swoop (B) Perturb Elevate = to raise somebody/something to a
higher place or rank.
(C) Plump (D) Boil
Frugal = using as little as poss ible of
1) A-D 2) B-C 3) A-C something especially money or food.
4) B-D 5) C-D Exult = to show or feel great joy because of
Swoop = to come down suddenly with a something that has happened.
rushing movement Lam ent = to feel or express great sorrow or
Plump down = to fall or drop something suddenly regret for somebody/something.
and heavily 7. (A) Surreptitious (B) Taciturn
Perturb = to make somebody very worried; to
(C) Exaggerate (D) Covert
disturb somebody
1) A-D 2) A-B 3) A-C
3. (A) Concise (B) Elegant
4) B-D 5) C-D
(C) Indifferent (D) Indecorous
Surreptitious = done or acting secretly or in a way
1) B-C 2) A-C 3) A-B that one does not want others to
4) C-D 5) B-D notice.
Concise = brief; giving a lot of information in Taciturn = saying very little; not communicating
few words a lot.
Elegant = graceful and attractive in Exaggerate = to make something seem larger,
appearance or manner better, worse etc than it really is.
Indifferent = having no interest in somebody/ Covert = concealed or secret; not open.
something; not caring about 8. (A) Handy (B) Sparse
somebody/something
(C) Redundant (D) Exhausted
Indecorous = showing a lack of dignity, good
manners or good taste
1) A-C 2) B-C 3) B-D
4. (A) Acquit (B) Defend 4) C-D 5) A-B
Handy = useful; convenient
(C) Forbid (D) Condemn
Sparse = in small amounts; thin and scattered
1) B-C 2) A-C 3) C-D Redundant = no longer needed
4) B-D 5) A-D Exhausted = very tired
Acquit = to declare somebody to be not guilty 9. (A) Timid (B) Conceited
of crime etc.
(C) Humane (D) Modest
Defend = to act, speak or write in support of
somebody/something; to protect
1) A-C 2) B-D 3) B-C
somebody/something from harm; to 4) A-D 5) C-D
guard somebody/something Timid = easily frightened; shy
For bid = to order s omebody not to do Conceit = excessive pride in oneself or in
something one’s powers; abilities etc.
Condemn = to say that one disapproves strongly Humane = having or showing sympathy,
of somebody/something; to criticize kindness and understanding.
somebody/something
Vocabulary Test 301

Modest = not taking much or boasting about 18. (A) Plunge (B) Grouch
one’s abilities. (C) Grumble (D) Despise
10. (A) Conversion (B) Desistance 1) A-B 2) A-C 3) A-D
(C) Substitution (D) Cessation 4) B-C 5) B-D
1) A-B 2) C-D 3) A-D Plunge = a sudden violent move.
4) B-D 5) A-C Grouchy = bad-tempered or complaining.
Desist = to stop something/doing something Grumble = to complain or protest in a bad-
Cessation = the action or act of stopping; a pause tempered way, usually not loudly; a
11. (A) Desecrate (B) Describe complaint or protest.
(C) Damage (D) Descent Despise = to feel contempt for somebody/
1) B-C 2) C-D 3) A-C something.
4) A-D 5) A-B 19. (A) Prosaic (B) Interesting
Desecrate = to treat a sacred thing or place badly (C) Stupid (D) Marathon
or without respect 1) A-C 2) B-C 3) B-D
Descent = a coming or going down 4) C-D 5) A-B
12. (A) Asinine (B) Profound Prosaic = dull; not romantic
(C) Intelligent (D) Cruel Marathon = a task, event etc that lasts a long
1) B-C 2) B-D 3) A-C time and requires a lot of effort or
patience.
4) A-B 5) A-D
Asinine = very stupid or foolish
20. (A) Debility (B) Strength
Profound = having or showing great knowledge (C) Prosperity (D) Sociability
or understanding of a subject; deep; 1) A-C 2) A-B 3) B-D
intense; very great 4) B-C 5) A-D
13. (A) Vivacious (B) Weird Debility = physical weakness especially as a
(C) Rabid (D) Fanatical result of illness.
1) A-B 2) B-C 3) B-D 21. (A) Audacious (B) Venturous
4) C-D 5) A-C (C) Abstruse (D) Silent
Vivacious = having or showing a lively attractive 1) A-C 2) B-C 3) C-D
personality. 4) A-B 5) B-D
Weird = odd; not usual or conventional. Audacious = showing a willingness to take risks.
Rabid = (of feeling or opinions) violent or Venture = to take the risk of losing or failing in
extreme. something.
Fanatic = a person who is too enthusiastic Abstruse = difficult to understand.
about something. 22. (A) Encomium (B) Extol
14. (A) Feed (B) Avoid (C) Eulogise (D) Euphemism
(C) Hoick (D) Eschew 1) A-B 2) B-C 3) B-D
1) B-D 2) A-B 3) A-C 4) A-D 5) C-D
4) A-D 5) C-D Encomium = a speech or piece of writing that
Hoick = to lift or pool something in a specified prais es s omeone or something
direction es peciall y with quick highly.
sudden movement Extol = praise enthusiastically.
Eschew = to keep away f rom something Eulogise = to praise somebody/something
deliberately; to avoid something. highly in speech or writing.
15. (A) Stupendous (B) Calm Euphemism = an expression that is gentler or less
(C) Livid (D) Pretentious direct than the one normally used
to refer to something unpleasant or
1) B-D 2) A-D 3) A-B
embarrassing.
4) A-C 5) B-C
23. (A) Recluse (B) Pandemic
Stupendous = extremely great or large
Livid = extremely angry
(C) Transparent (D) Opaque
Pretentious = claiming importance, value or style, 1) A-B 2) C-D 3) A-C
especially without good cause. 4) A-D 5) B-D
16. (A) Shambles (B) Flexible Recluse = a person who lives alone and likes
(C) Periodic (D) Organised to avoid other people
Pandemic = (of a disease) occurring over a whole
1) A-D 2) B-C 3) A-C
country or the whole world.
4) C-D 5) A-B
24. (A) Diminutive (B) Intelligent
Sham bles = a scene of complete disorder
(C) Large (D) Prolific
17. (A) Prophetic (B) Strange
1) B-D 2) C-D 3) A-C
(C) Poor (D) Sterling
4) A-B 5) C-B
1) A-B 2) C-D 3) B-D
Diminutive = extremely or unusually small
4) A-D 5) B-C Prolific = producing many work
Sterling = of excellent quality; reliable; genuine
302 Test of English Language

25. (A) Enormous (B) Malign Meagre = small in quantity and poor in quality
(C) Absorb (D) Slander 33. (A) Cutting (B) Establishing
1) A-C 2) B-C 3) C-D (C) Transferring (D) Pruning
4) B-D 5) A-D 1) A-B 2) C-D 3) B-C
Enormous = very large, huge 4) A-C 5) A-D
Malign = harmful Prune = to trim the shape of a tree, bush etc
Slander = a false spoken statement intended by cutting away some of the
to damage somebody’s reputation. branches etc es pecial ly to
26. (A) Concentration (B) Dissociation encourage new shoots to grow.
(C) Distraction (D) Deliberation 34. (A) Fixed (B) Stiff
1) A-D 2) B-C 3) A-C (C) Indelible (D) Soapy
4) C-D 5) D-B 1) A-B 2) A-D 3) A-C
Dissociate = to say that one does not agree with 4) B-C 5) C-D
or support somebody/something; to Stiff = not easily bent, folded, moved,
separate people or things in one’s changed in shape etc
thought or feelings Indelible = that cannot be rubbed out or
Distraction = a thing that prevents somebody from removed
concentrating on what they are 35. (A) Interminable (B) Long
doing or thinking about. (C) Endless (D) Interfering
Deliberation = careful consideration or discussion 1) A-C 2) B-C 3) C-D
27. (A) Exaggeration (B) Reiteration 4) A-B 5) B-D
(C) Imagination (D) Repetition 36. (A) Wobbly (B) Steady
1) A-D 2) B-D 3) C-D (C) Slight (D) Outlandish
4) B-C 5) A-B 1) A-B 2) B-D 3) C-D
Reiterate = to repeat something that has already
4) B-C 5) A-C
been said especially for emphasis.
Wobbly = tending to move in an unsteady way
Exaggeration = the action of making something
from side to side
seem larger, better, worse etc than
Outlandish = looking or sounding strange
it really is.
37. (A) Withstand (B) Climate
28. (A) Implies (B) Leads
(C) Hot (D) Surrender
(C) Confirms (D) Connotes
1) A-B 2) B-C 3) A-D
1) C-B 2) A-D 3) B-A
4) B-D 5) C-D
4) D-C 5) D-B
Withstand = to resist something
Imply = to suggest something indirectly
rather than stating it indirectly
38. (A) Perky (B) Lively
Connote = to suggest something in addition to (C) Honest (D) Kind
the main meaning. 1) A-B 2) B-C 3) C-D
29. (A) Surfaced (B) Nurtured 4) B-D 5) A-C
(C) Created (D) Developed Perky = lively and cheerful
1) B-A 2) B-C 3) C-A 39. (A) Reverie (B) Stirring
4) C-D 5) B-D (C) Serene (D) Fascination
Nurture = to help the devel opment of 1) A-D 2) B-D
something 3) C-D
30. (A) Expanded (B) Proclaimed 4) A-B 5) B-C
(C) Shrunk (D) Facilitated Serene = calm and peaceful
1) A-D 2) B-D 3) C-D Stirring = very exciting
4) B-C 5) A-C Reverie = a state in which one ignores what is
Facilitate = to make something especially an happening around one and has
action or a process easy or easier pleasant thoughts.
Proclaim = to announce something Fascination = a very strong attraction
31. (A) Indelible (B) Erasable 40. (A) Pandemonium (B) Scramble
(C) Insignificant (D) Temporary (C) Wriggle (D) Order
1) A-C 2) C-B 3) A-B 1) A-B 2) B-C 3) C-D
4) B-D 5) C-D 4) A-D 5) A-C
Indelible = that cannot be rubbed out or Pandemonium = wil d and nois y disorder or
removed confusion.
Scramble = an act of walking or climbing with
32. (A) Intangible (B) Restless
difficulty or over rough ground.
(C) Vast (D) Meagre Wriggle = to twist and turn with quick short
1) C-A 2) C-D 3) C-B movements.
4) B-A 5) B-D 41. (A) Stimulate (B) Apprehend
Intangible = difficul t to define; that has no (C) Facilitate (D) Understand
physical existence
Vocabulary Test 303

1) A-B 2) B-C 3) A-C 49. (A) Push (B) Thrive


4) B-D 5) C-D (C) Flourish (D) Arrange
Stimulate = to make somebody/something more 1) A-C 2) A-D 3) C-D
active or alert 4) B-C 5) B-D
Facilitate = to make something especially an Thrive = to become and continue to be
action or a process easy or easier successful, strong, healthy etc.
Apprehend = to understand somebody/
something Flourish = to be succes sf ul , active or
42. (A) Dense (B) Graze widespread; to prosper
(C) Pristine (D) Fresh 50. (A) Refuse (B) Discourage
1) B-C 2) C-D 3) B-A (C) Lurk (D) Hide
4) A-C 5) B-D 1) A-C 2) C-D 3) B-D
Pristine = Fresh and clean 4) B-C 5) A-D
43. (A) Prolixity (B) Brevity Lur k = to be or stay hidden especially when
waiting to attack or appear.
(C) Agreement (D) Proposition
51. (A) Delirious (B) Confluent
1) A-B 2) B-C 3) C-D
(C) Curt (D) Gracious
4) A-C 5) A-D
Prolix = using too many words and therefore
1) A-B 2) B-C 3) C-D
boring. 4) B-D 5) A-D
Brevity = the quality of using few words when Delirious = extremely excited and happy
speaking or writing Confluence = the place where two rivers flow
Proposition = an idea or a plan of action that is together and become one
suggested, especially in business. Curt = (of a person’s manner or behaviour)
44. (A) Suffuse (B) Deplete appearing rude because very few
words are used, or becaus e
(C) Fight (D) Delay
something is done in a very quick
1) B-C 2) C-D 3) A-C way
4) A-D 5) A-B Gracious = (of people or behaviour) kind, polite
Suffuse = to s pread al l over or through and generous , es pecial ly to
somebody/something somebody of a lower social position
Deplete = to reduce something by a large 52. (A) Punishment (B) Divergence
amount so that there is not enough
(C) Confluence (D) Confidence
left.
1) B-C 2) B-D 3) C-D
45. (A) Forensic (B) Delectable
4) A-B 5) A-C
(C) Leaflike (D) Charming
Diverge = to separate and go in different
1) A-C 2) B-D 3) A-D directions
4) B-C 5) A-B 53. (A) Doleful (B) Slothful
Forensic = connected with the scientific tests
(C) Laughable (D) Terrible
used by the police when trying to
solve a crime.
1) A-C 2) B-C 3) B-D
Delectable = extremely pleasant to taste, smell 4) C-D 5) A-D
or look at. Doleful = sad; miserable
46. (A) Benevolent (B) Alarming Slothful = lazy
Terrible = very unpleasant or serious causing
(C) Charitable (D) Stupendous
one to feel very unhappy or upset
1) A-B 2) B-C 3) C-D Laughable = causing people to laugh; ridiculous
4) A-C 5) B-D 54. (A) Dormant (B) Active
Benevolent = kind, helpful and generous
(C) Inconsequential (D) Antiquated
Stupendous = extremely l arge or impres sive,
especially greater or better than you
1) A-B 2) A-C 3) A-D
expect. 4) B-C 5) C-D
47. (A) Convenient (B) Intolerant Inconsequential = unimportant or irrelevant; trivial
Antiquated = old-fashioned and no longer
(C) Enduring (D) Protestant
appropriate
1) A-B 2) A-C 3) B-C
55. (A) Ailing (B) Pessimist
4) B-D 5) C-D
(C) Indispensed (D) Stylish
Enduring = lasting for a long time
1) C-D 2) B-C 3) A-B
48. (A) Eject (B) Spread
4) A-C 5) B-D
(C) Mark (D) Spout
Ailing = ill and not improving
1) B-D 2) A-C 3) B-C Pessimist = a person who expects the worst to
4) A-B 5) A-D happen
Spout = a stream of liquid coming out of 56. (A) Transferable (B) Unimpressive
somewhere with great force.
(C) Estimable (D) Equitable
Eject = to push something out suddenly and
with a lot of force.
1) A-B 2) B-D 3) B-C
4) A-D 5) C-D
304 Test of English Language

Estimable = worthy of great respect 64. (A) Articulate (B) Decipher


Equitable = fair and just; reasonable (C) Senseless (D) Decode
57. (A) Prompt (B) Doubtful 1) C-B 2) A-D 3) B-A
(C) Finery (D) Doctrinaire 4) D-C 5) D-B
1) A-B 2) B-D 3) C-D Decipher = to s ucceed in unders tanding
4) A-D 5) B-C something such as a code, an old
Finery = colourful and elegant clothes or document, somebody’s writing etc
decoration Articulate = able to express one’s ideas clearly
Doctrinaire = strictly applying or insisting on a in words
theory in all circums tances , 65. (A) Moderate (B) Easy
regardless of practical problems or (C) Significant (D) Strenuous
disagreement 1) B-D 2) A-B 3) A-C
58. (A) Census (B) Censure 4) B-C 5) C-D
(C) Reprimand (D) Universe Strenuous = requiring great effort or energy;
1) A-B 2) A-D 3) A-C making great efforts; energetic
4) B-C 5) C-D 66. (A) Focus (B) Trivial
Censure = strong criticism; disapproval (C) Vital (D) Site
Reprimand = a spoken or written statement 1) A-B 2) B-D 3) A-C
off iciall y expres sing s evere
4) C-D 5) B-C
disapproval of somebody or their
Trivial = of little importance; concerned with
actions.
unimportant things.
59. (A) Reason (B) Discernible Vital = essential to the existence, success
(C) Valid (D) Perceptible or operation of something
1) A-D 2) B-C 3) A-C 67. (A) Defer (B) Dispute
4) C-D 5) B-D (C) Prefer (D) Challenge
Discernible = that can be seen, tasted, noticed etc 1) B-C 2) A-C 3) B-A
something but not without effort
4) B-D 5) C-D
Perceptible = that can be felt or noticed with the
Defer = to delay something until a later time;
senses; great enough to be noticed.
to postpone something
60. (A) Critical (B) Space Dispute = to question whether something is
(C) Concourse (D) Courtyard true or valid
1) A-D 2) B-D 3) C-D 68. (A) Consequence (B) Potential
4) B-C 5) A-B (C) Influence (D) Ability
Concourse = an open area forming part of a 1) D-C 2) B-D 3) B-C
building or large group of buildings,
4) A-C 5) D-A
where people can walk about.
69. (A) Rebuke (B) Oppose
61. (A) Mitigation (B) Risking
(C) Praise (D) Distrust
(C) Appreciation (D) Alleviation
1) A-B 2) B-C 3) C-A
1) C-A 2) C-D 3) A-D
4) C-D 5) B-D
4) A-B 5) B-D
Rebuke = to expres s sharp or s evere
Mitigate = to make something less severe,
disapproval to somebody especially
violent or painful
officially.
Alleviate = to make something less severe; to
ease something
70. (A) Waive (B) Speculate
62. (A) Reiteration (B) Honouring (C) Pursue (D) Revise
(C) Reverberation (D) Hollow 1) A-B 2) C-B 3) D-C
1) A-C 2) B-C 3) C-D 4) C-A 5) D-B
Waive = to choose not to insist on something
4) A-B 5) B-D
in a particular case even though one
Reiterate = to repeat something that has already
has a legal or official right to do so.
been said especially for emphasis
Speculate = to guess
Reverberate = to be repeated several times as an
Pursue = to do, have or take part in something
echo.
71. (A) Contrary (B) Compatible
63. (A) Refurbish (B) Furnish
(C) Incomparable (D) Ambiguous
(C) Innovate (D) Renovate
1) A-B 2) B-C 3) C-D
1) C-A 2) C-D 3) C-B
4) A-C 5) B-D
4) B-A 5) A-D
Contrary = opposite in nature, tendency or
Refurbish = to restore and decorate a building
direction; the opposite
etc; to develop and improve
Compatible = that can exis t together without
something
problems or conflict; that can b e
Innovate = to make changes; to introduce new
used together
ideas, methods etc.
Ambiguous = not clearly stated or defined; that
Renovate = to get old buildings back into good
can be interpreted in more than one
condition
way
Vocabulary Test 305

72. (A) Pliable (B) Dependable 78. (A) Opaque (B) Translucent
(C) Flexible (D) Viable (C) Transverse (D) Transvestite
1) A-D 2) B-C 3) B-D 1) A-D 2) B-D 3) C-A
4) C-D 5) A-C 4) B-A 5) None of these
Pliable = easily bent, shaped or twisted; Opaque = not allowing light to pass through;
flexible not transparent
Viable = that can be done; that will work; Translucent = allowing light to pass through but
possible. not transparent
73. (A) Contingent (B) Permissive Transverse = lying or acting across something
(C) Confirmed (D) Endorsed Transvestite= a person who dresses in the clothes
1) B-A 2) C-A 3) C-B of the opposite sex especially for
sexual pleasure
4) B-D 5) D-C
Contingent = dependent on something that may
79. (A) Exorbitant (B) Expeditious
or may not happen (C) Quick (D) Quest
Endorse = to give one’s approval or support to 1) C-D 2) A-B 3) A-D
a claim, statement, course of action 4) C-B 5) None of these
etc Exorbitant = much too high or great; un-
74. (A) Repeat (B) Reverberate reasonable
(C) Retaliate (D) Reciprocate Expeditious = done with speed and efficiency
1) B-D 2) C-D 3) A-C Quest = the act of seeking something; a long
search for something
4) A-B 5) B-C
Reverberate = to be repeated several times as an
80. (A) Explicit (B) Cautious
echo. (C) Introvert (D) Clear
Retaliate = to harm, injure etc s omebody 1) A-B 2) B-D 3) A-C
because they have upset one or 4) A-D 5) C-D
caused one harm or injury Explicit = clear and easy to understand
Reciprocate = to give and receive something in Cautious = careful
return; to make a mutual exchange Introvert = a person who is more interested in
of something her or his own thoughts and
75. (A) Ecstasy (B) Depression feelings than in things outs ide
(C) Intoxication (D) Compression herself or himself, and is often shy
1) A-B 2) B-D 3) B-C and unwill ing to take part in
activities with others
4) C-D 5) None of these
Ecstasy = a feeling or state of great joy or
81. (A) Fearful (B) Beautiful
happiness. (C) Hostile (D) Amicable
Intoxicate = drunk or under the influence of 1) B-D 2) C-D 3) A-B
drugs 4) B-C 5) A-D
Compress = to press or squeeze something into Hostile = very unfriendly or aggressive and
a smaller space ready to fight and argue
76. (A) Tranquillity (B) Loyalty Amicable = done or achieved in a polite or
(C) Calamity (D) Uproar friendly way and without arguing
1) A-C 2) B-D 3) B-C 82. (A) Unite (B) Association
4) C-D 5) A-D (C) Separate (D) Distant
Tranquillity = quietness and peacefulness 1) A-C 2) A-B 3) B-C
Calamity = an event that causes great harm or 4) B-D 5) A-D
damage; a disaster 83. (A) Loud (B) Prominent
Uproar = a lot of noise made by peopl e (C) Salient (D) Legible
shouting, especially because they 1) A-C 2) B-D 3) C-D
are angry.
4) A-D 5) B-C
77. (A) Vilification (B) Nullification Prominent = important and well-known
(C) Denigration (D) Falsifiction Salient = most important and noticeable
1) A-B 2) B-C 3) A-C Legible = clear enough to be read easily
4) B-D 5) A-D 84. (A) Fraud (B) Barbarian
Vilify = to say unpleasant or insulting things (C) Guilty (D) Civilized
about somebody 1) A-C 2) A-B 3) B-D
Nullify = to make something lose its effect; to
4) A-D 5) C-D
act against something
Barbarian = uncivil ized; showing a lack of
Denigrate = to criticize somebody/something
education and good manners.
unfairly; to claim that somebody
something is inferior, worthless etc.
85. (A) Instigate (B) Enquire
Falsify = to alter a document etc falsely; to (C) Construe (D) Interpret
present something falsely 1) A-C 2) A-B 3) C-D
4) B-D 5) A-D
306 Test of English Language

Instigate = to make something begin or happen 93. (A) Purposefully (B) Inaccurately
Enquire = to ask to be told something by (C) Inadvertently (D) Unchangeably
somebody 1) A-C 2) A-B 3) B-C
Construe = to unders tand or interpret the
4) B-D 5) A-D
meaning of words, s entences ,
Inadvertent = not done deliberately or
actions etc in a particular way.
intentionally
86. (A) Superficial (B) Superfluous
94. (A) Germane (B) Generate
(C) Enlightened (D) Surplus
(C) Reliable (D) Irrelevant
1) A-C 2) A-B 3) B-C
1) B-D 2) B-C 3) A-B
4) B-D 5) A-D
4) C-D 5) A-D
Superficial = not thorough, deep or complete
Germane = connected with something; relevant
Superfluous = more than is needed or wanted
Enlightened = free from prejudice, false beliefs etc;
95. (A) Disciple (B) Student
having or s howing an (C) Academy (D) Martyr
understanding of what the current 1) A-C 2) C-D 3) B-C
situation requires , rather than 4) B-D 5) A-B
following conventional thought. Disciple = a follower of a religious, political,
87. (A) Appalling (B) Sinister artistic etc leader or teacher
(C) Perturbed (D) Threatening Martyr = a person who is killed or made to
1) A-B 2) B-D 3) A-C suffer greatly because of her of his
religious or other beliefs
4) A-D 5) D-C
Appalling = shocking; extremely bad
96. (A) Magnetic (B) Cherished
Sinister = suggesting evil, or that something (C) Valued (D) Forlorn
bad may happen 1) C-D 2) A-B 3) B-D
Perturb = to make somebody very worried; to 4) B-C 5) A-D
disturb somebody Cherish = to keep a feeling or an idea in one’s
88. (A) Imprison (B) Torture mind or heart and think of it with
(C) Excruciating (D) Extract pleasure.
1) B-D 2) B-C 3) A-B Forlorn = unhappy and lonely or abandoned
4) C-D 5) A-C 97. (A) Ordinary (B) Vague
Imprison = to put or keep somebody in prison (C) Custom (D) Exceptional
Excruciating= intens el y painful; mentall y 1) A-B 2) B-C 3) C-D
agonising; very embarrassing 4) A-D 5) B-D
Extract = to take or get something out usually Vague = not clearly expressed or perceived
with effort or by force 98. (A) Damaged (B) Hurried
89. (A) Pertinent (B) Impolite (C) Condemned (D) Measured
(C) Irrelevant (D) Insecure 1) A-B 2) C-D 3) B-D
1) A-C 2) B-D 3) C-D 4) B-C 5) A-D
4) A-D 5) B-C Measured = slow and with a regular rhythm
Pertinent = relevant to something 99. (A) Remote (B) Troubled
90. (A) Consent (B) Nascent (C) Secluded (D) Apparent
(C) Emerging (D) Insecure 1) B-C 2) A-D 3) B-D
1) A-C 2) B-D 3) B-C 4) C-D 5) A-C
4) A-D 5) A-B Secluded = not visited or seen by many people;
Consent = agreement; permission away from the company of others
Nascent = beginning to exist; not yet fully Apparent = clearly seen or understood; obvious
developed 100. (A) Preposterous (B) Illuminating
Emerging = developing and becoming (C) Absurd (D) Striking
noticeable, important or prominent 1) A-D 2) C-D 3) B-C
91. (A) Elated (B) Eccentric 4) A-C 5) B-D
(C) Explicit (D) Abnormal Preposterous = contrary to reason or common
1) A-B 2) B-D 3) A-C sense; utterly absurd or ridiculous.
4) A-D 5) D-C Illuminating = helping to make something clear or
Elated = very happy, excited or proud easier to understand; revealing
Eccentric = unusual; not conventional or normal Absurd = not reasonable or sensible; foolish
Explicit = clearly and fully expressed and ridiculous
92. (A) Abundance (B) Incomparable 101. (A) Jeopardise (B) Repudiate
(C) Projection (D) Plethora (C) Confuse (D) Disown
1) A-C 2) A-B 3) C-D 1) A-B 2) B-C 3) C-D
4) B-D 5) A-D 4) A-C 5) B-D
Plethora = a quantity greater than what is Jeopardise = to cause something to be harmed,
needed or can be used; an excess lost or destroyed.
Vocabulary Test 307

Repudiate = to refuse to deal with or be 1) A-B 2) A-C 3) A-D


connected with s omebody any 4) B-C 5) B-D
longer; disown Ominous = suggesting that something bad is
102. (A) Virtuous (B) Conservative going to happen; threatening
(C) Effeminate (D) Virile Pious = having or showing a deep respect
1) A-D 2) C-D 3) B-C for God and religion
4) A-B 5) B-D Livid = extremely angry
Effeminate = having characteristics considered to 110. (A) Jagged (B) Smooth
be typical of a woman (C) Ignoramus (D) Igneous
Virile = having or showing typically male 1) B-C 2) A-D 3) C-D
strength or energy 4) B-D 5) A-B
Virtuous = having or s howing high moral Jag ged = with rough, pointed, often sharp
standards or goodness edges
103. (A) Capricious (B) Firm Ignoramus = an ignorant person
(C) Nostalgic (D) Drastic 111. (A) Infuriate (B) Iniquitous
1) A-B 2) A-D 3) A-C (C) Idiocy (D) Stupidity
4) B-C 5) B-D 1) C-D 2) B-C 3) A-B
Capricious = showing sudden changes in 4) B-D 5) A-C
attitude or behaviour. Infuriate = to make somebody extremely angry
Nostalgia = the feeling of sadness mixed with and annoyed
pleas ure when one thinks of a Iniquitous = very unfair or wicked
happy period, event etc earlier in 112. (A) Penetrate (B) Penitence
one’s life
(C) Jovial (D) Regret
104. (A) Rare (B) Deliberate
1) A-B 2) B-C 3) B-D
(C) Perennial (D) Occasional
4) A-C 5) A-D
1) A-B 2) B-C 3) C-D Penetrate = to make a way into or through
4) A-C 5) B-D something
Perennial = constantly occurring Penitence = sorrow or regret for having done
105. (A) Morbid (B) Chaotic something wrong
(C) Thunder (D) Diseased Jovial = very cheerful and friendly
1) A-B 2) B-C 3) C-D 113. (A) Savage (B) Spiritual
4) A-D 5) A-B (C) Stupor (D) Gentle
Morbid = having or showing an interest in sad 1) B-C 2) B-D 3) C-D
or unpleasant things, especially 4) A-D 5) A-C
disease or death. Savage = wild or fierce; cruel or very violent
106. (A) Frail (B) Enormous Stupor = the state of being nearl y
(C) Pseudo (D) Imitation unconscious or not fully aware of
1) C-D 2) A-B 3) B-C what is happening, caused by
4) A-C 5) B-D shock, drugs, alcohol etc.
Frail = physically weak or delicate; easily 114. (A) Chaos (B) Contradictory
damaged or broken; morally weak (C) Order (D) Vigilant
Pseudo = not genuine; pretended or insincere 1) A-B 2) B-C 3) C-D
107. (A) Hectic (B) Terse 4) A-C 5) A-D
(C) Concise (D) Immense 115. (A) Contrive (B) Design
1) A-B 2) B-C 3) C-D (C) Implement (D) Intervene
4) A-D 5) B-D 1) A-B 2) B-C 3) C-D
Hectic = very busy; full of activity and 4) B-C 5) A-C
excitement Contrive = to design or make a plan, machine
Ter se = using few words and perhaps not etc in a clever or elaborate way or in
friendly or polite order to deceive somebody
Immense = extremely large or great Intervene = to come or be between
108. (A) Insignificant (B) Innocuous 116. (A) Project (B) Decipher
(C) Wholesome (D) Deleterious (C) Entail (D) Involve
1) B-C 2) C-D 3) B-D 1) A-B 2) B-C 3) C-D
4) A-D 5) A-C 4) B-D 5) A-D
Innocuous = not intended to offend Entail = to involve something as a necessary
Wholesome = good for one’s health or inevitable part or consequences
Deleterious = harmful 117. (A) Tangible (B) Stewed
Insignificant = having little or no value usually
(C) Unkempt (D) Tidy
meaning or importance
1) A-C 2) B-D 3) A-B
109. (A) Ominous (B) Pious
4) B-C 5) C-D
(C) Threatening (D) Livid
308 Test of English Language

Tangible = clear and definite; real; that can be 1) A-D 2) B-D 3) C-D
perceived by touch 4) A-C 5) B-C
Stewed = tasting too strong and bitter 126. (A) Eagerly (B) Radically
because it has been left in the pot
(C) Vigorously (D) Severely
too long
Unkempt = not kept tidy
1) A-C 2) B-D 3) B-C
Tidy = arranged neatly and in order 4) A-B 5) C-D
118. (A) Trice (B) Whet 127. (A) Spurious B) Authorised
(C) Syncope (D) Stimulate (C) Attested D) Genuine
1) A-C 2) B-D 3) A-B 1) A-D 2) A-C 3) A-B
4) A-D 5) B-C 4) B-D 5) B-C
Trice (in a trice) = very quickly and suddenly Spurious = not genuine; false or fake
Whet = to excite or stimul ate 128. (A) Sensitive (B) Immune
somebody’s desire, interest etc (C) Vulnerable (D) Covered
Syncope = temporary loss of cons cio- 1) A-B 2) B-C 3) C-D
usness caused by a fall in 4) A-C 5) A-D
blood pressure Immune = not af fected or infl uenced by
119. (A) Confusing (B) Irritating something
(C) Bewildering (D) Dictating Vulnerable = that can be hurt, harmed or attacked
1) A-B 2) A-C 3) A-D easily especially because of being
4) B-C 5) B-D small or weak
Bewildering = confusing 129. (A) Inspiration (B) Intuition
120. (A) Melodious (B) Witty (C) Influence (D) Innovation
(C) Loony (D) Eccentric 1) A-B 2) A-C 3) A-D
1) A-B 2) B-C 3) C-D 4) B-C 5) B-D
4) A-D 5) B-D 130. (A) Laudably (B) Gigantic
Witty = full of clever humour. (C) Immense (D) Munificent
Loony = crazy or odd; mad 1) A-B 2) A-C 3) A-D
121. (A) Robust (B) Pesky 4) B-C 5) B-D
(C) Annoying (D) Abhorring Gigantic = of very great size or extent; huge
1) A-B 2) C-D 3) A-C Immense = extremely large or great
4) B-D 5) B-C Munificent = extremely generous; large in amount
Robust = vigorous, healthy and strong or splendid in quality.
Pesky = causing trouble; annoying 131. (A) Mendacious (B) Truthful
Abhore = to feel hatred and disgus t for (C) Accurate (D) Disgruntled
somebody/something especially for 1) A-B 2) B-C 3) C-D
moral reasons 4) B-D 5) A-D
122. (A) Bridled (B) Hurled Mendacious = not telling the truth; lying
(C) Abusive (D) Unchecked Disgruntle = feeling rather bitter or angry because
1) A-B 2) B-C 3) C-D something has happened to upset
4) A-D 5) B-D one
Bridle = to keep one’s feelings etc under 132. (A) Penetrate (B) Stimulate
control; to restrain something (C) Exasperate (D) Please
Hurl = to throw s omething/s omebody 1) A-B 2) B-C 3) C-D
violently in a particular direction 4) A-C 5) B-D
123. (A) Awesome (B) Nomadic Exasperate = to irritate or annoy somebody greatly
(C) Blushing (D) Shameless 133. (A) Mutilated (B) Disgorged
1) A-B 2) B-C 3) C-D (C) Duplicity (D) Deception
4) A-C 5) B-D 1) A-D 2) A-C 3) A-B
Nomadic = a person who does not stay long in 4) B-C 5) C-D
one place Mutilate = to injure or damage somebody/
Blush = to become red in the face because s omething very severely by
of shame, embarrassment etc. breaking or tearing off a necessary
124. (A) Curb (B) Courage part
(C) Control (D) Identity Disgorge = to pour people or things out in a
1) A-B 2) B-C 3) C-D great mass.
4) A-C 5) B-D Deception = the action of making somebody
Curb = to prevent something from getting believe something is not true.
out of control; to restrain something 134. (A) Flamboyant (B) Extravagant
125. (A) Components (B) Ornamentation (C) Cunning (D) Concomitant
(C) Accessories (D) Attachments 1) A-B 2) B-C 3) C-D
4) A-C 5) A-D
Vocabulary Test 309

Flamboyant = tending to attract attention; very 140. (A) Spiritual (B) Immoral
confident and noticeable (C) Temporal (D) Agonising
Extravagant = willing to use more of something 1) A-B 2) B-C 3) C-D
especially money, than is necessary
4) A-C 5) A-D
or appropriate
Spiritual = of the human spirit or soul; not of
Cunning = clever at deceiving people
physical things
Concomitant = accompanying; happening at the
Agonising = causing great suffering or anxiety
same time as something else
Temporal = of this life, not spiritual
135. (A) Lyrical (B) Lewd Spirit = a person’s mind of feelings as
(C) Liaison (D) Dull distinct from her or his body
1) A-B 2) B-C 3) C-D 141. (A) Fraud (B) Amuse
4) A-C 5) A-D (C) Befuddled (D) Confuse
Lyrical = express ing strong emotion or 1) A-B 2) B-C 3) C-D
enthusiasm especially in poetry, art
4) A-C 5) A-D
music etc.
Amuse = to make somebody laugh or smile
L ew d = treating or referring to sex in a crude
Befuddled = made stupid; confused
or offensive way.
Liaison = a close working rel ations hip
142. (A) Yield (B) Clamour
involving communication between (C) Protest (D) Penalise
groups of people, units of an 1) A-B 2) B-C 3) C-D
organisation etc. 4) A-C 5) A-D
136. (A) Pong (B) Odour Clamour = a loud demand or protest
(C) Mordant (D) Sarcastic 143. (A) Scatty (B) Sceptic
1) B-D 2) B-C 3) C-D (C) Sturdy (D) Sane
4) A-C 5) A-D 1) A-B 2) B-C 3) C-D
Pong = a strong, usually unpleasant, smell 4) A-C 5) A-D
Odour = a distinctive, usually unpleasant, Scatty = slightly mad; crazy
smell. Sceptic = a person who doubts that a claim,
Mordant = sharply critical; very sarcastic statements etc is true
Sarcastic = using or expressing remarks that Sturdy = strong and firm
imply the opposite of what they Sane = having a normal healthy mind; not
appear to mean and are intended to mad
upset or mock somebody. 144. (A) Bizarre (B) Offensive
137. (A) Lucrative (B) Vague (C) Usual (D) Brisk
(C) Distinct (D) Denudation 1) A-B 2) B-C 3) C-D
1) A-B 2) B-C 3) C-D 4) A-C 5) B-D
4) A-C 5) A-D Bizarre = very strange; not at all logical
Lucrative = producing much money; profitable Brisk = quick; active; busy
Vague = not specific or exact 145. (A) Diminishing (B) Infernal
Distinct = easily heard, seen, felt or (C) Pleasing (D) Vehement
understood; definite 1) A-B 2) B-C 3) C-D
Denude = to make something bare; to take the
4) A-C 5) A-D
covering off something
Diminish = to become or make something
138. (A) Effeminacy (B) Effrontery smaller or less; to decrease
(C) Effervescence (D) Impertinence Infernal = used to express annoyance; of or
1) A-B 2) B-C 3) C-D like a hell.
4) B-D 5) A-D Vehement = showing or caused by strong feeling;
Effeminate = having characteristics considered to passionate
be typical of a woman. 146. (A) Soaked (B) Folded
Effrontery = bold or rude behaviour without any (C) Grubby (D) Washed
feeling or shame 1) A-B 2) B-C 3) C-D
Effervescent = lively, excited and enthusiastic.
4) B-D 5) A-D
Impertinent = not showing respect; rude
Grubby = dirty
139. (A) Exquisite (B) Delicate
147. (A) Bracing (B) Invigorating
(C) Tempting (D) Berserk
(C) Dwindling (D) Depicting
1) A-B 2) B-C 3) C-D
1) A-B 2) C-D 3) B-D
4) A-C 5) A-D
4) A-C 5) B-C
Exquisite = extremely beautiful or delicate;
Bracing = cool and fresh; invigorating
finely or skilfully made or done
Depict = to show or represent somebody/
Delicate = very carefully made or formed; fine
something as a picture
Tempting = attractive; inviting
Invigorate = to make somebody feel more lively
Berserk = out of control with anger
and healthy
Dwindle = to become gradually less or smaller
310 Test of English Language

148. (A) Charming (B) Placid 1) A-B 2) B-C 3) C-D


(C) Adventurous (D) Calm 4) A-D 5) B-D
1) A-B 2) B-C 3) C-D Testimony = a thing that is evidence of or
4) A-C 5) B-D demonstrates something
Placid = calm and peaceful Aura = a distinctive atmosphere that seems
149. (A) Assertion (B) Beatitude to surround and be caused by a
person or thing
(C) Repulsive (D) Insistence
Augment = to increase something
1) A-B 2) B-C 3) C-D
156. (A) Discomfit (B) Baffle
4) A-C 5) A-D
(C) Epicure (D) Enumerate
Assertion = the action of claiming or stating
something forcefully
1) A-B 2) A-C 3) A-D
Beatitude = great happiness; blessedness 4) B-C 5) B-D
Repulsive = causing a feeling of strong dislike Discomfit = to confuse or embarrass somebody
or disgust Baffle = to be too difficult or strange for
Insistent = not allowing refusal or opposition; somebody to understand, solve or
tending to demand something explain
forcefully, not accepting a refusal Epicure = a person who takes a special
150. (A) Vocal (B) Benign interest in and gets great pleasure
from food and drink
(C) Unpleasant (D) Drastic
Enumerate = to name things on a list one by one.
1) A-B 2) B-C 3) C-D
157. (A) Unkempt (B) Unremitting
4) A-C 5) A-D
(C) Slackening (D) Distasteful
Benign = kind; gentle; mild; pleasant
1) A-B 2) B-C 3) C-D
151. (A) Abysmal (B) Diligence
4) A-D 5) B-D
(C) Zenith (D) Nadir
Unkempt = not kept tidy
1) A-B 2) B-D 3) C-D Unremitting = never stopping
4) B-C 5) A-C Slacken = to become or make something
Abysmal = extremely bad; extreme; very great slower, less active
Diligence = steady effort; careful hard work Distasteful = a feeling that somebody/something
152. (A) Elude (B) Avoid is unpleasant or unacceptable;
(C) Harness (D) Hatch dislike
1) A-B 2) A-C 3) A-D Tidy = arranged neatly and in order
4) B-C 5) B-D 158. (A) Apathetic (B) Wrath
Elude = to escape somebody/something (C) Whirl (D) Twirl
especially by a clever trick; to avoid 1) A-B 2) A-C 3) A-D
somebody/something 4) B-C 5) C-D
Harness = to control and use a natural force to Apathetic = showing or feeling a lack of interest,
produce electrical power etc. enthusiasm or concern
Hat ch = to come out of an egg. Wrath = great concern
153. (A) Languid (B) Gorgeous Whirl = to move or make somebody/
(C) Knack (D) Ability something move quickly round and
1) A-B 2) A-D 3) B-C round
4) C-D 5) B-D Twirl = to spin something
Languid = moving slowly and involving very 159. (A) Onus (B) Testimony
little physical effort (C) Burden (D) Recitation
Gorgeous = giving pleasure and satisfaction; 1) A-B 2) B-C 3) C-D
wonderful 4) A-C 5) B-D
Knack = an ability; a skill at performing some Onus = a duty or responsibility for doing
special task something
154. (A) Gregarious (B) Quixotic Testimony = a thing that is evidence of or
(C) Sociable (D) Discernible demonstrates something
1) A-B 2) B-C 3) C-D Recitation = an act of saying a series of things
4) A-C 5) B-D aloud.
Gregarious = liking to be with other people 160. (A) Blushing (B) Unconscionable
Quixotic = having fine romantic ideals and good (C) Reasonable (D) Whiff
intentions that are not at all practical 1) A-B 2) B-C 3) C-D
or realistic 4) A-C 5) B-D
Discern = to perceive, know or f ind out Blush = to become red in the f ace
something; to see, taste, notice etc because of shame; embarrass-
something but not without effort. ment
155. (A) Testimony (B) Aura Unconscionable = excessive; greater or longer
(C) Augment (D) Decrease than is reasonable
Vocabulary Test 311

Whiff = a trace or hint; a smell especially 164. (A) Stout (B) Impotent
one that is only smelt for a (C) Fragile (D) Eloquent
short period of time 1) A-C 2) B-D 3) C-D
161. (A) Traduce (B) Harass 4) A-D 5) A-B
(C) Oblige (D) Defame Stout = strong and thick; rather fat; built in
1) A-B 2) B-C 3) C-D a solid way
4) A-C 5) A-D Fragile = easily damaged or broken; delicate
Traduce = to say damaging false things about Eloquent = having or s howing express ive
somebody/something especially to language especially to impress or
make people think badly about persuade an audience
them. 165. (A) Equivocal B) Effeminate
162. (A) Visionary (B) Glorious (C) Enumerate (D) Manly
(C) Uncanny (D) Natural 1) A-B 2) B-C 3) C-D
1) A-B 2) B-C 3) C-D 4) A-C 5) B-D
4) A-C 5) B-D Equivocal = not clear or definite in meaning or
Uncanny = not natural; mysterious and slightly intention; that can be interpreted in
frightening. more than one way; ambiguous
163. (A) Whimsical (B) Traverse Effeminate = having characteristics considered to
(C) Squalid (D) Sordid be typical of a woman
1) C-D 2) B-C 3) A-B 166. (A) Fervour (B) Flimsy
4) B-D 5) A-C (C) Dissipate (D) Waste
Whimsical = unus ual and rather playf ul ; 1) A-B 2) B-C 3) C-D
capricious but charming 4) A-C 5) B-D
Traverse = to travel or extend across an area Fervour = strength or intensity of feeling;
Squalid = very dirty and unpleasant, especially enthusiasm
because of poverty or lack of care. Flimsy = light and thin; weak; not convincing
Sor did = dirty and unpleasant Dissipate = to waste something foolishly

Answers
1. 4; same 29. 5; same 57. 1; opposite
2. 3; same 30. 5; opposite 58. 4; same
3. 5; opposite 31. 3; opposite 59. 5; same
4. 4; opposite 32. 2; opposite 60. 3; same
5. 3; same 33. 5; same 61. 3; same
6. 1; opposite 34. 1; same 62. 1; same
7. 1; same 35. 1; same 63. 5; same
8. 1; opposite 36. 1; opposite 64. 5; same
9. 2; opposite 37. 3; opposite 65. 1; opposite
10. 4; same 38. 1; same 66. 5; opposite
11. 3; same 39. 5; opposite 67. 4; same
12. 3; opposite 40. 4; opposite 68. 2; same
13. 4; same 41. 4; same 69. 3; opposite
14. 1; same 42. 2; same 70. 4; opposite
15. 5; opposite 43. 1; same 71. 1; opposite
16. 1; opposite 44. 5; opposite 72. 5; same
17. 2; opposite 45. 2; same 73. 5; same
18. 4; same 46. 4; same 74. 4; same
19. 5; opposite 47. 3; opposite 75. 1; opposite
20. 2; opposite 48. 5; same 76. 5; opposite
21. 4; same 49. 4; same 77. 3; same
22. 2; same 50. 2; same 78. 4; opposite
23. 2; opposite 51. 3; opposite 79. 4; same
24. 3; opposite 52. 1; opposite 80. 4; same
25. 4; same 53. 5; opposite 81. 2; opposite
26. 3; opposite 54. 1; opposite 82. 1; opposite
27. 2; same 55. 3; same 83. 3; same
28. 2; same 56. 3; opposite 84. 3; opposite
312 Test of English Language

85. 3; same 113. 4; opposite 140. 4; opposite


86. 4; same 114. 4; opposite 141. 3; same
87. 4; same 115. 1; same 142. 2; same
88. 2; same 116. 3; same 143. 5; opposite
89. 1; opposite 117. 5; opposite 144. 4; opposite
90. 3; same 118. 2; same 145. 2; opposite
91. 2; same 119. 2; same 146. 3; opposite
92. 5; same 120. 3; same 147. 1; same
93. 1; opposite 121. 5; same 148. 5; same
94. 5; opposite 122. 4; opposite 149. 5; same
95. 5; same 123. 3; opposite 150. 2; opposite
96. 4; same 124. 4; same 151. 3; opposite
97. 4; opposite 125. 3; same 152. 1; same
98. 3; opposite 126. 2; same 153. 4; same
99. 5; same 127. 1; opposite 154. 4; same
100. 4; same 128. 2; opposite 155. 3; opposite
101. 5; same 129. 2; same 156. 1; same
102. 2; opposite 130. 4; same 157. 2; opposite
103. 2; same 131. 1; opposite 158. 5; same
104. 3; opposite 132. 3; opposite 159. 4; same
105. 4; same 133. 5; same 160. 2; opposite
106. 1; same 134. 1; same 161. 5; same
107. 3; opposite 135. 5; opposite 162. 3; opposite
108. 4; opposite 136. 3; same 163. 1; same
109. 2; same 137. 2; opposite 164. 1; opposite
110. 5; opposite 138. 4; same 165. 5; opposite
111. 1; same 139. 1; same 166. 3; same
112. 3; same
288

Chapter 22

Diction
(Words Often Mistaken For One Another)

Introduction 5. Abortion, Miscarriage


Both these words refer to a premature expulsion of
‘Diction’ is the choice of words. In English there are the foetus from the womb. The difference lies in the
many words which are usually confused. Some words following:
have similar me aning, but cannot be use d ‘Abortion’ can refer to a premature coming out of
interchangeably; that is, a choice must be made the foetus on account of some biochemical or harmonic
according to the grammatical situation. disturbance. It can also refer to a deliberate expulsion
1. Abbreviations, Acronyms of the foetus by a physician in the interest of the
An ‘abbreviation’ is a short way of writing a word or mother’s health and life or for some other reason.
phrase which could also be written out in full, using The use of ‘miscarriage’ on the other hand is confined

K
only letters of the alphabet and possibly full stops. to a natural coming out of the foetus on account of
For example, some bioche mical or harmonic disturbance .
Dr for Doctor ‘Miscarriage’ cannot be used in the case of a deliberate
lb for Pound (s) expulsion of the foetus.
e.g. for for example
6. Accord, Accordance
An abbreviation does not normally have a distinct
pronunciation of its own. These properties distinguish When you do something of your ‘own accord’, you do it
abbreviations from ‘acronyms’ and ‘initialisms’ like NATO voluntarily, without being asked instructed to do it.
and BBC, from ‘clipped forms’ like gym and phone, and Note the preposition ‘of’ here ‘on your own accord’ is
from ‘symbols’ like * and 5. not standard English. Howe ve r, whe n you do
something in obedience to instructions, you do it ‘in
2. Ability, Capacity, Capability accordance with’ those instructions.

KUNDAN
Applied to a person, the first two (Ability, Capacity)
7. Accurate, Precise
me an about the same , but the y don’t be have
grammatically in the same way. For example, These words are not interchangeable. Something
which is ‘accurate’ is correct; the opposite is ‘inaccurate’,
You have an ability to do something,
which means ‘wrong to some extent’. But ‘precise’ means
but a capacity for doing something.
‘correct to a very high degree of detail’, and its opposite
As for the third (capability), this is best used in the ‘imprecise’ means lacking a sufficient degree of detail’.
plural and with no material following: your ‘capabilities’ For example,
are your abilities and your talents as a whole.
Accurate records must be kept.
3. Abolition, Abolishment Can you give more precise definition of the word?
In most contexts, the noun derived from the verb If I try to explain the word ‘loris’ as denoting a ‘nocturnal
‘abolish’ is ‘abolition’, but ‘abolishment’ is sometimes primate’, then I am accurate, since a loris is indeed a
preferred in legal and financial contexts. Therefore, nocturnal primate, but I am rather imprecise, since
we write of the abolition of slavery but possibly of the there are several other kinds of nocturnal primates
‘abolishment’ of mortgage relief. But ‘abolition’ is usually besides lorises, and my account fails to distinguish
possible in any context. lorises from lemures, bushbabies and aye-ayes.
4. Abuse, Misuse, Disabuse 8. Adapt, Adopt
To ‘misuse’ something is to use it wrongly. To ‘abuse’ ‘To adapt’ something is ‘to modify’ it, often especially
it is to misuse it so badly that you damage it. To to make it suitable for a particular purpose. ‘To adopt’
‘disabuse’ somebody of an idea is to show her that something is ‘to take it up’. For example,
idea is wrong. For example, Austin’s Pride and Prejudice has been adapted for
They will be quickly disabused of the notion t hat television.
linguistics is an easy subject. We are adopting a new procedure for budget control.
The derived nouns are ‘adaptation’ for the first ‘adoption’
289

for the second. Some people now use ‘adoption’ in place Almost (nearly, practically) all the boys had left before
of ‘adaptation’, but this is not recommended. the bell rang.
She is almost (nearly, practically) always grumbling
9. Adjacent, Adjoining like this.
Almost (nearly, practically) always grumbling like this.
Things are ‘adjacent’ when they are side by side. But
things are ‘adjoining’ when they share a common Now, look at the sentences given below:
boundary. For example, The speaker said nearly nothing. (Incorrect)
The speaker said almost (practically) nothing. (Correct)
We were sitting in adjacent seats.
Nearly any boy in the class would answer this question
We had adjoining rooms in the hotel.
correctly. (Incorrect)
In many circumstances either word can be used. But Almost (practically) any boy in the class would answer
note that ‘adjacent’ does not mean ‘near’ or ‘close’, this question. (Correct)
and wordings like very adjacent are therefore frowned Near ly no one ever believed t hat he was honest .
on by careful writers. Write ‘very close’ or ‘very near’ (Incorrect)
Practically (almost) no one ever believed that he was
instead. Note also the following difference in grammar:
honest. (Correct)
Their land is adjacent to ours, but their land adjoins ours. I nearly never found him working in t he library.
10. Affect, Effect (Incorrect)
I almost (practically) never found him working in the
These two are often confused. Apart from one or two library. (Correct)
technical uses, the word ‘affect’ is strictly a verb. In
contrast ‘effect’ is primarily a noun. For example, 13. Alphabet(s), Letter(s)

K
This won’t affect our chances. An ‘alphabet’ is a whole set of ‘letters’ used for writing
This will have no effect on our chances. words. For example, the Roman alphabet (the one we
There is, however, a verb spelled ‘effect’ and meaning are using at the moment), which has 26 letters, from
‘bring about’. For example, A to Z, or the Devanagari alphabet which we use for
writing Hindi, Marathi and Nepali.
General Raman managed to effect the withdrawl of
his troops. The plural form ‘alphabets’, is used in the sense
illustrated below:
But this last verb is rare, except perhaps in the
expression ‘effect a change’. Tamil, Telugu, Bengali and Punjabi have all their own
alphabets which differ from one another.
11. All, Whole
‘All’ and ‘whole’ have almost similar meaning. ‘Whole’ 14. Already, Yet, All ready
‘Already’ expresses that something has happened early

KUNDAN
means ‘complete’; ‘entire’ etc. ‘All’ means ‘everything’ or
‘everyone of’. But there is much difference in their uses. or earlier than it might have happened. ‘Yet’ is generally
(a) When the article ‘the’ or a possessive adjective used in negative and interrogative sentences. ‘Already’
(my, your, his, our, etc) is to be used with ‘all’, it (the or and ‘Yet’ are commonly used with present perfect
possessive adjective) is put after the word ‘all’, but tense. ‘All ready’ are two different words that express
before the word ‘whole’. For example, their separate meaning. For example,
All the members were present. Has the postman come yet? (Not, already)
The whole world knows it. These mangoes are not ripe yet? (Not, already)
He lost all his money. The mangoes are not yet ripe. (Not, already)
They got their whole share. He has already finished his task.
Are you all ready? (Here ‘ready’ means ‘prepared’)
Note: It is wrong to write - the all boys. Have you met the Police Inspector yet. (suggests enquiry)
(b) ‘All’ may be followed by of+personal pronoun in Have you already met the Police Inspector? (answer
the objective case, provided such a pronoun has no ‘yes’ is expected)
noun in apposition. If there is a noun in apposition
to pronoun (not after ‘of’) after ‘all’, it (pronoun) will 15. Also, Even
be put in the nominative case. For example, ‘Also’ and ‘even’ are almost similar as regards their
all of us meaning. But there is some difference in their usage.
all of them (i) ‘Also’ is generally used in the middle of a clause
all you students etc or sentence. ‘Also’ modifies the word after which it is
Note: In expressions like ‘all day’, ‘all night’, ‘all put. For example,
morning’, ‘all afternoon’, the article ‘the’ should not be I also beat my servant.
put after ‘all’. (=other persons also beat him)
I beat also my servant.
12. Almost, Nearly, Practically (=I beat him and punished him in other ways also.)
All these words can be used before ‘all’, ‘always’ and I beat my servant also.
‘every’. For example, (=I beat other persons as well.)
290

(ii) ‘Even’ is generally used after the auxiliary verb or 20. Amount, Number
the intransitive forms of be (is/am/are/was/were). It ‘Amount’ refers to a quantity of something (a singular
should not be used before a subject. For example, noun) that cannot be counted — a large amount of
1. He has lost his bag. money. ‘Number’ refers to countable items (a plural
He has even lost his watch. (not, Even he has lost....) noun). With most plurals it is better to use number —
2 . This lady is rude to everybody. a large number of mistakes. For example,
She is even rude to her husband. (not, Even she is
The amount of money paid is none of my concern.
rude....)
A large number of teachers have gone on strike.
(iii) ‘Also’ can be used at the beginning of a sentence,
Note: A number of (many) is plural in sense and
provided it refers to the whole sentence. In such a
takes a plural verb. (A number of people were shot
case it becomes necessary to put a comma (,) after
dead.) The number of (= a mathematical or numerical
the word ‘also’. For example,
figure) is singular. It takes a singular verb. (The
This is a good book, but is not voluminous. Also, it number of boys present is very small.)
costs much. (not, It also costs much.)
21. Ancestors, Forefathers
(iv ) ‘Even+if’ can begin a sentence or clause. For
example, Both these words refer to persons from whom one is
descended. Similarly both these words refer to persons
I shall go out, even if it rains today. more remote than grandparents. The difference lies
Even if I become a rich man, I shall never be proud of
in the following:
wealth or power
‘Ancestor’ has ‘ancestress’ as its feminine form.
16. Alter, Change There is no feminine word for ‘forefathers’. Unlike

K
The difference between these two words lies in the ‘forefathers’, ‘ancestor’ can freely be used in the case
following: of a woman ancestor as well. One can, for example,
‘Alter’ refers to changes in matters of detail. For describe a great-grandmother as ‘one of my ancestors’
example, but not as, ‘one of my forefathers’
Customer says tailor, “Please alter this shirt; it is too 22. Ancient, Old
large for me”.
‘Ancient’ is the opposite of ‘modern’ whereas ‘old’ is
‘Change’ is preferred when the reference is to a the opposite of ‘young’. Moreover ‘ancient’ refers to very
significant change, a loss of identity, a thing being old things. And ‘old’ can at times be used in the case
replaced by another. For example, of a very young child as well. As for example,
He seems t o hav e changed quit e a lot since his The ancient Rome was the cradle of civilization.
marriage. The ancient Mariner narrated his story of adventure.

KUNDAN
Can the Ethiopian change his skin, or the leopard his Death overtakes all alike, the young and the old.
spots? My baby is only two months old.
Women take a long time to change their dress.
He has been changing places quite often. 23. Annex, Annexe
17. Alumnus, Alumna The verb is ‘annex’, a country can annex territory, but
Each of the se words denote s a graduate of an it cannot ‘annexe’ it. The noun meaning ‘extension to a
educational institution, but there is a sex difference. building’ is ‘annexe’ in a British English.
A man is an ‘alumnus’ (plural ‘alumni’), while a woman 24. Anticipate, Expect
is an ‘alumna’ (plural ‘alumnae’). A group of mixed sex To ‘anticipate’ is to guess or imagine in advance what
are ‘alumni’. And there is no such word as ‘alumnis’. will happen, and take the necessary action to face it.
18. Amiable, Amicable For example,
An ‘amiable’ person is good-natured and easy to get I tried to anticipate the kind of questions they were
along with. The word is commonly applied only to likely to ask me.
people, though occasionally we find it extended to To ‘expect’ is to believe or think that something will
occasions, as in an amiable conversation. happen or come. For example,
But ‘amicable’ is not applied to people at all; instead,
We are expecting a letter from our Boss.
it is applie d to human inte ractions and the ir
outcomes, as in an amicable settlement (of a dispute) 25. Anxious, Eager
and an amicable relationship. The meaning here is ‘Anxious’ means ‘nervous’ or ‘worried’ and is usually
‘friendly’ and ‘good natured’. followed by ‘about’. ‘Eager’ means ‘looking forward’. It
19. Amoral, Immoral is usually followed by to. For example,
An ‘amoral’ person is one who does not understand I am terribly anxious about these children.
the difference between right and wrong. An ‘immoral’ Our company is eager to expand into new markets.
person understands the difference but does wrong any
way.
291

26. Anyway, Any way overly bold’, ‘take something upon oneself without
This is one word when it means ‘regardless’, but two permission’, as in we are presuming upon his hospitality.
words when it means ‘in any manner’. Therefore, we 32. Attend, Attend to
write,
Here, to avoid confusion, it is best to regard ‘attend’
Do it anyway. (=Do it regardless.) but, Do it any way and ‘attend to’ as two distinct items in respect of
you like. meaning: ‘attend’ means ‘to be present’ and ‘attend to’
means ‘to provide help of some kind’ or ‘to deal with
27. Apparently, Evidently
somebody or something’ as illustrated below:
The second of these is stronger than the first. For
We would like all of you to attend this meeting.
example,
In a serious road accident, the most important thing is
Apparently she is a good worker. to attend to the injured.
(=I gather that she is a good worker or I am told that Salesman in a shop : Are you being attended to, Sir?
she is a good worker.) I have some urgent business to attend to.
Evidently she is a good worker.
(=It is clear that she is a good worker.)
Note: Occasionally, however, another phrase, ‘attend
on’, is used to convey the meaning of ‘to look after’ or
28. Appendix, Supplement ‘serve’. For example,
An ‘appendix’ is that material which is related to the Our President has a team of doctors to attend on him.
main work, but is not, strictly speaking, an integral But here, the preposition ‘on’ is optional and most
part of the main work. ‘Appendix’ in other words, refers British speakers do not use it.
to that material, which, if included in the body of the

K
main work, would make the main work less compact 33. Avenge, Revenge
and thereby reduce its quality. Both these words indicate a sense of desire for doing
A ‘supplement’, on the other hand, is that additional some harm to the wrong-doer. The difference lies in
material which is thought necessary for completing the following:
the main work. ‘Avenge’ may indicate that the person has the ends
A ‘supplement’ adds new material to a book etc and of justice in mind. It tells us, in other words, that he
thereby makes it up-to-date. wants the wrong-doer to be subjected to his just and
merited punishment. For example,
29. Appraise, Apprise
Mehmet a avenged the destruction of his family by
To ‘appraise’ something is to estimate its value,
killing the evil Aga who had caused it.
whereas ‘to apprise’ is ‘to inform’. For example,
‘Revenge’ often suggests that a person is being guided

KUNDAN
I am having an art dealer in to appraise these paintings.
We have apprised him of the facts.
by malice and unwillingness to forgive. For example,
Chelsea revenged their earlier defeat a 3-1 win in the
The common error is to use appraise where apprise is
second leg.
required. But who needs apprise anyway. Just write
‘inform’ or ‘tell’. The derived noun is ‘vengeance’ in both the cases.

30. Assist, Help 34. Await, Wait


A person who ‘assists’ has a much more subsidiary The words ‘await’ and ‘wait’ are similar in meaning but
role to play than a person who ‘helps’. For example, a the differ in use.
person who is drowning will cry ‘help, help’ and persons ‘Await’ is used as a transitive verb. Hence it should
in the neighbourhood would rush to help him. A never be followed by ‘for+noun/pronoun’. For example,
carpenter who is making a boat will ask his children I am awaiting your reply. (Not, awaiting for your reply.)
to ‘assist’ him in doing that. An Assistant Registrar
‘Wait’ is always used as an intransitive verb. Hence it
assists the registrar in performing his duties. A shop-
should be followed by ‘for+noun/pronoun’. For example,
assistant assists the shopkeeper in performing in his
duty. I am waiting for your reply.

31. Assume, Presume 35. Balance, Change


Both of these mean ‘suppose’ and their meanings often Customer : What is the price of this toy, please?
overlap. But there is a subtle distinction. To ‘assume’ Shopkeeper : Twenty-five rupees, sir.
something is to take it for granted as the basis of Customer : Please pack it for me. Oh, I’m sorry, I
discussion or an argument, without offering any don’t have any small notes. If I give
you a thousand rupees note, can you
evidence for it. This may be done purely in order to
give me the balance?
explore the consequences of the assumption. To
‘presume’ something is to suppose it is so because we In the above dialogue, standard English speaker would
have no good reason to suppose otherwise. use the word ‘change’ in place of the word ‘balance’.
Of course, presume alone has a second sense of ‘be Another somewhat related meaning of the word
292

‘change’ is ‘coins rather than notes’. For example, 39. Bear, Endure, Stand, Tolerate
Can you please give me change for this ten-rupee coin? All these words refer to one’s response to something
distressing, irksome or painful.
This meaning of the word ‘change’ seems to be well
‘Bear’ is the most general of these words. For
known in India.
example,
36. Bank, Shore, Coast, Beach I could not bear to see the goat being slaughtered.
The place where the land meets the water can be either I could not bear the sight of that old and sickly man
the ‘bank’ or the ‘shore’ or the ‘coast’ or the ‘beach’. being crushed to death by a lorry.
The edge of a river is called its ‘bank’ whereas the ‘Stand’ is its colloquial equivalent. For example,
edge of a sea is its ‘shore’. The land bordering the sea
I cannot stand this kind of nonsense.
is ‘coast’. We use it when we think of places on maps,
I cannot stand that ugly woman.
of weather, or naval defence. A ‘beach’ is part of the
‘shore’ that is smooth, without rocks or cliffs: The only ‘Endure’ is normally used in the case of great bodily
word that can be used for lakes is ‘shore’. For example, hardship. For example,
Calcutta is situated on the bank of the river Hoogly. He alone can endure that kind of physical torture.
He has a house on the shore of lake Mansarowar. ‘Tolerate’ is used in the case of human beings and
Our ship is fast heading towards the shore.
their behaviour. For example,
Our ship sailed from port to port along the coast.
He cannot tolerate that rude fellow.
37. Be, Become I cannot tolerate this insult.
‘Be’ and ‘become’ are two different verbs. The one is
40. Begin, Commence, Start
not generally used in place of the other, as they differ

K
in meaning. In many cases ‘begin’ and ‘start’ can replace each other.
‘Be’ means ‘to exist’, ‘to remain’, ‘to live’, ‘to happen’, For example,
‘to take place’. ‘Be’ has three forms in the present tense. After eight o’clock in the morning the water level started
They are (a) is (b) am (c) are. It has two forms in the (began) rising.
past tense (a) was (b) were. In future tense ‘shall/ I began (started) learning French at the age of ten.
will’ is put before ‘be’. For example, The meeting began (started) at 10 o’clock.

God is (=exists) everywhere. Some of the differences between these two words are
Let them be (=remain) where they are. the following: The opposite of ‘start’ is ‘stop’, whereas
He is (=lives) at Patna these days. the opposite of ‘begin’ is ‘end’. For example,
When is the wedding going to be? (=take place) He began t he day with hope but ended it in
‘Become’ means ‘to change from one state or position to disappointment.

KUNDAN
another’. For example, The conversation stopped, and it refused to start again.

He became king. ‘Begin’ and ‘start’ tend to be preferred in ordinary use.


(=Formerly he was not a king, there came a change in ‘Commence’ is more formal than these two verbs and
his power or position.) is often preferred in legal proceedings and officialese
Mango becomes yellow when it ripens. and also on ceremonial occasion. For example,
(=There is a change in colour or state.)
The proceedings will commence in July.
‘Become’ has another meaning also. It means ‘to be
In certain cases the use of ‘commence’ may suggest
suitable to’. For example,
a prolonged and elaborate beginning. For example,
This hat becomes (=suits) his.
Modesty becomes a woman. He commenced being a severe and ardent student.

38. Beautiful, Handsome 41. Beside, Besides


‘Beautiful’ and ‘pretty’ are generally used for women, Both of these words can be prepositions, but their
children and things, but not usually for men. We use meanings are different. The word ‘beside’ means ‘next
‘handsome’ for men. For example, to’, while ‘besides’ means ‘also’, ‘in addition to’. For
example,
Look at that beautiful girl.
How pretty the Chinese doll is! He came in and seated himself beside me.
Lo ! There comes a pretty child. He has two other cars besides this one.
He is a handsome young man.
42. Biennial, Semi-annual, Bi-annual
We can use handsome for women having a fine figure,
The word ‘biennial’ means ‘happening every two years’,
vigour and dignity. For example,
while ‘semi-annual’ means ‘happening twice a year’,
Jane Austen was a handsome lady with chiselled face ‘happening every six months’. The common mistake is
and a fine figure. writing ‘biennial’ where ‘semi-annual’ is intended.
There is also the somewhat unusual word ‘bi-
293

annual’ which means ‘happening twice a year’, but you 47. Canvas, Canvass
are advised to avoid this. The ‘canvas’ is a noun denoting a kind of coarse cloth,
43. Bona fide, Bona fides used, for example, by oil painters. In contrast, ‘canvass’
is usually a verb, and it occurs most often in the
Of these two, ‘bona fide’ is an adjective meaning
expression canvassing for votes. The verb gives rise to
‘genuine’, while ‘bona fides’ is a noun meaning ‘good
the uncommon derived noun ‘canvass’ meaning ‘a
faith’, ‘honest intentions’. So, we write a bona fide offer
search for votes’.
but there are doubts about his bona fides. The noun is
singular: write His bona fides is not in doubt, not......are 48. Catch, Hold
in doubt. However, like most latin expressions, these
Please catch the bag while I am locking the car.
two are hardly necessary: it is preferable to write a
genuine offer and there are doubts about his honesty. This use of ‘catch’ is not acceptable in sandard English.
We catch a ball, for instance, by arresting its movement
44. Both in the air. This, howeve r, is obviousely not the
It is possible to use ‘both’ alone. For example, meaning implied in the above example. The right word
Both are acceptable. here would be ‘hold’ not ‘catch’.
Both of them have been found guilty. The word ‘catch’ suggests an instantaneous action;
Note the ‘both’ always takes plural agreement. It is the word ‘hold’, on the othe r hand, de note s a
also possible to combine ‘both’ with ‘and’. For example, continuous state, and it is latter meaning that the
speaker intends to convey in the above example.
Both the Egyptians and the Israelis will attend the
talks. 49. Cause, Reason

K
But it is wrong to combine ‘both’ with ‘as well as’. For ‘Cause’ is something that produces a result or an effect
example, whe re as ‘reason’ is some thing that justifie s or
explains a result. For example,
Both the Egyptians as well as the Israelis will attend
the talks. (Incorrect) She is the cause of all my unhappiness.
The reason of my silence is my illness.
A wrong use of ‘both’ is illustrated by the following
example: 50. Ceiling, Roof
Both these words are synonymous. (Incorrect) ‘Ceiling’ is the under surface or the overhead interior
lining of a room. ‘Roof’ is the top covering of a building,
Being synonymous is not a property that a single object
bus, car or tent. For example,
can have in isolation, and the correct form is as given
below: The ceiling of my drawing-room is being painted.

KUNDAN
The roof of his house is leaking.
These words are synonymous. (Correct) Even the roof of this bus is crowded with passengers.
Anothe r e rror occurs, if ‘both’ is followe d by
preposition, that preposition must be repeated after 51. Centenary, Centennial
‘and’: write both in Britain and in America, and not both Both are correct for ‘hundredth anniversary’. But British
in Britain and America or in both Britain and America. English prefers ‘centenary’ while American English
prefers ‘Centennial’.
45. Buy, Purchase
‘Buy’ is more general word than ‘purchase’. It is the 52. Chaste, Pure
word of choice in the case of small, casual and day- I often find the chaste Hindi of the AIR news bulletins
to-day transactions. For example, difficult to understand.
I bought two small books for my children yesterday. The word ‘chaste’ has two possible connotations: one,
I bought three spoons last week. sexual and the other, simplicity in style. Surely
‘Purchase’ on the other hand, refers to a transaction neither of these is intended by the speaker of the
of some importance. ‘Buy’ can replace ‘purchase’ in above sentence. What he intends to say instead is
nearly all cases. But in the case of small day-to-day ‘pure Hindi’, not that mixed with say, Urdu and English
transactions, the use of ‘purchase’ makes the sentence words in common use. The right word here is ‘pure’,
sound odd. For example, not ‘chaste’.
Go and purchase three eggs from that shop. (Jarring) 53. Childish, Childlike
Go and buy three eggs from that shop. (much more Both these adjectives are now applicable to children
likely)
and also to adults. ‘Childish’ refers to the unpleasant
46. Callous, Callus features of childhood, for example, triviality, greed,
tendency to quarrel about petty things and lack of
A ‘callus’ is a hard patch of skin. But ‘callous’ is an
manners. For example,
adjective meaning ‘indifferent to suffering’, as in his
callous behaviour. This is my childish effort at using a fork.
Don’t be so childish!
294

‘Childlike’, on the other hand, refers to the admirable (iii)Bring to an end


qualities of childhood, for e xample , simplicity, I have decided to close the accounts.
innocence and lack of inhibitions.
58. Cloth, Clothes
54. Citizen, Inhabitant
A ‘cloth’ is a piece of fabric used for a specialised
The ‘citizens’ of a country are those inhabitants who purpose, such as a dishcloth or a table-cloth. The
have all the civil rights and privileges of a full member plural is ‘cloths’. But ‘clothes’ are garments and this
of that country. The ‘citizens’ of a country can be the word has no singular form other than item of clothing.
native people of that country; they may also be Do not write ‘cloths’ when you mean ‘clothes’.
foreigners who have been naturalized. For example,
59. Cold and Cool, Hot and Warm
Many of the Indians in Britain have become British
Citizens. ‘Cold’ suggests a lower temperature than ‘cool’.
Temperature is uncomfortably low. ‘Cool’ suggests a
The ‘inhabitants’ of a country are all the people who pleasantly low temperature. For example,
live in that country.
I can’t stand this cold wave.
55. Clean, Clear A nice cool breeze is blowing.
‘Clean’ refers to something which is free from dirt. For ‘Hot’, likewise, suggests a higher temperature than
example, ‘warm’. It is often uncomfortable. But ‘warm’ is a
Keep your hands clean. pleasantly high temperature. For example,
It may also refer to something pure and innocent. For You can’t work long hours on a hot day.
example, It was warm, but not hot, day before yesterday.

K
He lives a very clean life. 60. Comic, Comical
‘Clear’, on the other hand, refers to an object which is Something is ‘comic’ if it is intended to be funny. The
easy to see through. For example, word is mainly applied to skits (=a short piece of
A piece of clear glass. humorous writing or a performance that makes of fun of
The clear water of the swimming pool. somebody/something by copying them), songs, plays and
It can also refer to something which can be easily the like. For example,
heard and understood. For example, Tom Lehrer is famous for his comic songs.
His voice was very clear. But something is ‘comical’ if it is funny unintentionally.
Now, look at the sentences given below: For example,

You must keep the utensils clear. (Incorrect) Her portrayal of Ophelia was comical.

KUNDAN
You must keep the utensils clean. (Correct)
61. Compare to, Compare with
56. Client, Customer Careful users of English make a useful distinction
A person who goes to a solicitor or an architect and between these. To compare X to Y is to say that X is
pays for his help or advice is a ‘client’. People who buy similar to Y. But to compare X with Y is to note the
goods from shopke e pe rs and trade sme n are similarity and differences between X and Y. For
‘customers’. For example, example,
A successful solicitor has a large number of clients. A shop assistant must compare your signature with
This shopkeeper has all attention to his customers. the signature on your credit card.
The critics compared his work to that of Martin Amis.
57. Close, Shut
In expressions like the following, either of the two 62. Compatible, Comparable
words is considered suitable. For example, Things are ‘compatible’ if they can work together or be
Close (shut) the door, please, will you? used together successfully. For example,
She closed (shut) her eyes to all his faults. I can’t mail her this attachment because her PC is not
Only ‘close’ can be used in the following contests: compatible with my Mac.
(i) In the case of an institution not functioning But ‘comparable’ means ‘of roughly the same kind or size’.
normally. For example, For example,
The schools are open but the colleges are all closed. House prices in Delhi are now comparable to those in
The shops closed at 6 pm. Mumbai.
This theatre will remain closed until the end of the
The common error is to write ‘compatible’ where
month.
‘comparable’ is intended. Avoid errors like Prices are
(ii) Conclude now compatible with those in Mumbai. And note the
I would now request the chair man to close t he differe nce in pre position: ‘Compatible with’, but
discussion. ‘Comparable to’, not ‘Comparable with’.
295

63. Complement, Compliment A whole is composed of its parts : The NATO forces
A ‘complement’ is something which is necessary or are composed of soldiers from eight countries.
suitable for completeness or harmony. For example, Its part constitute a whole : Soldiers from eight
countries constitute the NATO forces.
a ship’s complement (the officers and crew required
to sail it) 67. Contagious, Infectious
our full complement (all that we need or are entitled
‘Contagious’ is used for a disease spreading by contact.
to)
the complement of a verb (a phrase whose presence is
‘Infectious’ is used for a disease that can be spread by
required by the verb) means of germs carried in the atmosphere or in water.
For example,
The related verb meaning ‘go well with’ is also spelled
‘complement’. For example, Measles is highly contagious.
Cold is infectious.
This necklace will complement your outfit.
68. Continuous, Continual
A ‘compliment’ is an expression of admiration. For
‘Continual’ shows that an action or an event has been
example,
taking place repeatedly. For example,
He paid her a number of flowery compliments.
Our computing system is suffering continual crashes.
The related verb is spelled identically. For example, Her work was disturbed by continual interruptions.
He complimented her on her outfit. ‘Continuous’, on the other hand, shows the unbroken
The common mistake here is to write ‘compliment’ where continuity of an action or an event. For example,
‘complement’ is intended. This error is easy to avoid if

K
England’s canals once provided a continuous waterway
you re call the similarity in spe lling be twe e n from the Themes Estuary to the Irish Sea.
‘complement’ and ‘complete’. The nerve centre in our brain need a continuous supply
of blood.
64. Complex, Complicated
69. Conveyance, Transport
These near-synonyms are not quite interchangeable.
The more formal word is ‘complex’, which in technical Conveyance will be provided to the delegates from the
railway station to the university.
use means ‘having an elaborate structure’, ‘not easily
You can claim conveyance allowance.
treatable’. The more informal ‘complicated’ means ‘hard
to understand or to resolve’. Therefore, a difficult One of the meanings of the word ‘conveyance’ is indeed
proble m in Mathe matics is comp lex , while a vehicle, but in the situations exemplified above,
somebody’s messy personal life is complicated. standard English uses ‘transport’ instead. And I see
no danger of your not being understood if you use the

KUNDAN
65. Comply, Conform ‘transport’ in place of ‘conveyance’ in the above contexts.
These verbs mean about the same, ie ‘to obey a rule,
an order etc’, but they take different prepositions. For 70. Convince, Persuade
example, To ‘convince’ someone means to change his or her
opinion; to ‘persuade’ someone means to move him or
We comply with our instructions, but we conform to
her to action. ‘Convince’ is properly followed by ‘of’ or
the required standards.
‘that’, whereas, ‘persuade’ is followed by ‘to’. For
66. Comprise, Consist, Compose, Constitute example,
The se four ve rbs are very frequently confused, We could not convince him of our innocence.
producing awful things like, The NATO forces are We persuaded him to concede our demands.
comprised of soldiers from eight countries, and Thirty-
two pieces comprise a chess set. 71. Correspond to, Correspond with
A whole comprises its parts : The NATO forces To say X corresponds to Y is to say that X and Y match
comprise soldiers from eight countries. This verb can up in some systematic way, that X and Y occupy
never be passivized or followed by ‘of’ : hence ‘comprises comparable places in two different systems, or simply
of’ and ‘is comprised of’ are always wrong. The bad that X and Y are in agreement. For example, we can
example above should be written as follows : A chess write These findings correspond to the predictions of the
set comprises thirty-two pieces. theory. In contrast, X corresponds with Y means only
Note also that ‘comprise’ does not mean ‘contain’; that X and Y are exchanging lette rs, as in I am
‘include’, unless what follows it is an exhaustive list. corresponding with a scholar in England.
So, example is wrong: Not a single television news 72. Custom, Habit
programma comprises a majority of tabloid content.
‘Customs’ are social and ‘habits’ are personal. A ‘habit’
Now, look at the following sentences carefully, that
is something a person has been doing for a long time.
will illustrate the use of ‘consist’, ‘compose’ and
‘Custom’ is something that belongs to a large number
‘constitute’ :
of people of the same sort: all Christians or all
A whole consists of its parts : The NATO forces consist
Hindus. For example,
of soldiers from eight countries.
296

Don’t let yourself get into bad habits. 79. Dissatisfied, Unsatisfied
Social customs vary from country to country. When you are ‘dissatisfied’ you are disappointed,
frustrated, unhappy. When you are ‘unsatisfied’, you
73. Delusion, Illusion
feel that you need more of something. Only a person
‘Delusion’ is false opinion or belief which is sincerely can be dissatisfied, while an abstract thing like hunger
believed to be true. ‘Illusion’ is false idea or belief or a demand for goods can be unsatisfied.
which appears to be true. For example,
80. Distrust, Mistrust
He is under the delusion that he is the wealthiest
man of this place. There is a subtle distinction here which should be
Macbeth felt that the Birnam wood was moving. It was an maintained. ‘To distrust’ somebody is to suspect that
optical illusion. he is dishonest, while ‘to mistrust’ him is merely to
lack confidence in him. If you consider your deputy to
74. Despite, In spite of be wholly honest but somewhat incompetent, then
The se are equivalent in me aning, but note the you may mistrust him but you don’t distrust him.
difference in usage. For example,
81. Drier, Dryer
The mat ch went ahead despite the bad weather .
There is great variation here, and neither spelling can
(Incorrect)
The match went ahead in spite of the bad weather. be called wrong in any sense. But most authorities
(Correct) prefer ‘drier’ for ‘more dry ’ and ‘dryer’ for any machine
that dries such as a ‘hair dryer’ or a ‘spin dryer’.
75. Die, Dice
82. Earthy, Earthly, Earthen
Traditionally, ‘die’ is singular and ‘dice’ is plural. For
These adjectives are all different. Something which is

K
example, we throw a die but we throw the dice if we are
‘earthy’ is either similar to earth (as in an earthy texture
throwing two or more of them. This usage is still almost
or an earthy smell) or coarse and vulgar (as in earthy
universal in American English. In Britain, however,
language) But ‘earthly’ means ‘pertaining to our human
dice is now commonly used also as the singular. For
existence on earth’ as opposed to ‘heavenly’ or ‘celestial’:
example, throw a dice. This usage is now recognised
our earthly existence, earthly powers. It also occurs as
by most British dictionaries. If you are writing for a
a meaningless modifier in fixed expressions like no
British readership only, you can use dice as a
earthly use. Finally ‘earthen’ means ‘made of earth or of
singular, but, if you have a wider readership in mind,
clay’, as in an earthen floor or earthenware.
you should use ‘die’ as the singular. In any case, avoid
the confused phrases illustrated by throw two die and 83. Eastward, Eastwards
throw two dices. ‘Eastward’ is an adjective but ‘eastwards’ is an adverb.
For example,

KUNDAN
76. Dirty, Filthy
‘Dirty’ is a general word indicating dirt of any kind: The strong eastward wind (or east wind) helped the
dirty hands, dirty clothes. It can also refer to moral boat going from the western to the eastern Bank of the
river.
dirt: dirty jokes, dirty stories. ‘Filthy’ means ‘extremely
The wind was blowing eastwards.
dirty; covered with filth.’ It is a much stronger term
than dirty. 84. East, Eastern
He put the dirty dishes in the sink. ‘East’ would normally indicate clearly marked divisions
Let me take my filthy boots off. to be understood in terms of political boundaries. For
Nobody relishes your filthy jokes. example,
East Germany
77. Discover, Invent
‘Eastern’ would indicate divisions which are not sharply
‘Discover’ means ‘to find out; get knowledge of; bring
defined. For example,
to view something existing but not yet known’. ‘Invent’
means ‘to create or design something not existing Eastern Europe; Eastern India
before’. For example, Similarly, ‘north’, ‘south’ and ‘west’ would indicate
It was Columbus who discovered America. Marconi sharply defined divisions whereas ‘northern’, ‘southern’
invented the wireless. and ‘western’ would indicate divisions which are not
sharply defined. For example,
78. Distinterested, Uninterested
(a) North Pole, North America, North Carolina, South
‘Disinterested’ means ‘impartial’. ‘Uninterested’ means Australia, but Southern England
‘bored’ or ‘lacking interest’. (b) South Africa, South America, South Carolina, South
As a disinterested observer, who do you think is right? Australia, but Southern England.
I am completely uninterested in cricket. (I do not find (c) West Bengal, West Indies, West German, West
cricket at all interesting) Columbia, but Western Europe
297

Note: Western Australia, Northern Hemisphere, 89. Egoist, Egotist


Southern Hemisphere etc are exceptions. ‘Egoist’ means ‘a selfish person, one who puts his
85. Eatable, Edible interest first’. ‘Egotist’ means ‘a self-centred person,
one who is continuously speaking of himself or trying
It is possible to find contexts in which the two words
to attract attention to himself.’
might replace each other, but in most cases they
convey two different shades of meaning. ‘Eatable’ An egoist is all too eager to serve his own end.
means ‘likeable’, ‘enjoyable’, ‘tasty and fresh enough to He is an egotist for he always blows his own trumpet.
be eaten with some pleasure’. For example,
90. Elder, Older
The food the patients are required to eat in that hospital
Normally, ‘elder’ is used in the case of the members
is hardly eatable.
These bananas are over-ripe but yet they are eatable.
of a family. ‘Older’ can be used in the case of persons
having no family relationship. For example,
An ‘edible’ substance is ‘a substance which can be used
1. He married his older daughter to a doctor and the
as food or for making food’. For example,
younger one to an engineer. (Incorrect)
The price of edible oil is likely to come down next He married his elder daughter to a doctor and the
month. younger one to an engineer (Correct)
2 . Jaggu is elder than all other students in his class.
Mostly, ‘edible’ is used in an attributive position and
(Incorrect)
‘eatable’ in a predicative position. Jaggu is older than all other students in his class.
86. Economic, Economical (Correct)
The adjective ‘economic’ means ‘pertaining to economics ‘Elder’ is sometimes used in the case of a person older

K
or to the economy’, as in an economic advisor or current than another person (especially son) of the same
economic conditions. But ‘economical’ means ‘using a name. For example,
minimum of money or resources’, as in an economical Some people say that Sam, the elder, was mor e
meal or an economical manufacturing process. Hence an successful as a politician than sam, the younger.
economic solution to a problem is a solution which
Normally, ‘older’ but not ‘elder’, can be used in the
involves economics, while an economical solution is
predicative position in a sentence. For example,
one which saves money. The common error here is to
write ‘economic’ where ‘economical’ is intended: an Rajesh is elder than I am. (Incorrect)
economic meal. Rajesh is older than I am. (Correct)
Note: The ‘elder’ in the phrase ‘elder statesman’ refers
87. Effective, In effect
to those senior statesman (generally those who have
The adjective ‘effective’ means ‘producing a satisfactory

KUNDAN
retired from office) who exerts a great deal of influence
re sult’: an effective solution is one that works. and whose advice and guidance are valued because of
Accordingly, the adverb effectively means ‘with a their age and experience.
satisfactory outcome’: The salmonella outbreak has been
effectively contained means that salmonella poisoning 91. Elicit, Illicit
is no longer spreading. The phrase ‘in effect’ means ‘in The verb ‘elicit’ means ‘call forth’, as in My complaint
practice’, and it is usually applied to something which elicited only a pompous reply. The adjective ‘illicit’ means
is officially or notionally not done but which gets done ‘illegal’ or ‘contrary to prevailing mores (=the customs
anyway. For example, or conventions considered typical of or essential to a group
The gover nment’s new measur es in effect prevent or community)’, as in illicit trading. Do not write ‘illicit’
refugees from entering the country legally at all. when you mean ‘elicit’.
Here ‘in effect’ indicates that no such outcome was 92. Emigrant, Immigrant
intended or sought, but that this outcome has arisen ‘To emigrate’ is to go away from one country to another
in practice regardless. with the purpose to settling their. ‘To immigrate’ is to
88. Effeminate, Womanly, Womanish come to a country as a settler. For example,
‘Effeminate’ and ‘womanish’ are derogatory terms used The immigrants in England have not been able to
for a man having qualities that are regarded as typical integrate themselves with the native people.
The British emigrants to Canada and Australia are
of women. Both the words mean ‘unmanly’. ‘Womanly’
mostly teachers, doctors and engineers
refers to qualities that are regarded as typical of or
suitable to woman. For example, 93. Emotional, Emotive
A henpecked husband is generally effeminate. These words overlap somewhat in their meanings. But
Art thou a man? Thy form cries out thou art: thy tears ‘emotional’ is usually preferred in the se nse of
are womanish. SHAKESPEARE
‘displaying emotion’, as in ‘an emotional outburst’, and
She showed a womanly concern for t he aggrieved
family.
also in the sense of ‘calling forth emotions’, ‘as in an
emotional moment’. In contrast, ‘emotive’ is a rather dry
298

and abstract word, encountered most commonly in not in size or numbers; it means ‘improve the quality or
the expression emotive language, meaning language status’ of something, as in the following sentences:
which is carefully crafted to induce emotions, such (b) It is time now to consider how to enhance the
as the language of some poems and of some political quality of our education.
speeches. If it’s spontaneously and artlessly affecting,
Therefore, in sentence (a) above, we should use
it’s emotional; if it’s coldly calculated to produce an
‘increased’ or ‘raised’ instead of ‘enhanced’.
emotional response, it’s emotive. When in doubt, prefer
‘emotional’. 99. Envious, Jealous
Both these words refer to a person who wants to have
94. Empathy, Sympathy what someone else has. He is, in fact, unhappy about
The word ‘sympathy’ means ‘pity’, while ‘empathy’ means another person’s possession of something that he
‘more specifically identification with the other person’. himself would like to possess. For example,
You can sympathize with anybody who is in a bad He is jealous of their success.
position, but you can only empathize with her if you He is envious of their success.
can readily see yourself in her shoes. For example,
‘Jealous’, however, refers to a stronger and more
She showed no sympathy when I told her I was in unpleasant feeling. In some cases, it means unhappy
trouble.
and angry because someone who should like you, likes
There is a natural love and empathy between them.
someone else more. When I kiss the baby, it makes
95. Empty, Vacant the older child jealous.
‘Empty’ means ‘containing nothing’. It refers to an 100. Environment, Surroundings
object; an empty box, an empty room. ‘Vacant’ also ‘Environment’ may sometimes refer to the aggregate of

K
me ans ‘e mpty’. It is used for a place or space, physical surroundings and climatic, social, and cultural
especially one that is usually filled or intended to be conditions which affect not only the physical but also
filled. There is a vacant place over there. It is also the intellectual, emotional and spiritual growth of a
used of a job not at present filled. I have applied for person. For example,
the vacant post. It also means lack of interest or
It is so satisfying to work in an academic environment
serious thought. A vacant mind breeds depression. like this.
For example,
‘Surroundings’ generally refers to physical objects like
An empty vessel sounds much.
buildings, trees, flowers which may be pleasing or
Is this seat vacant?
depressing. For example,
96. Endemic, Epidemic, Pandemic He lives in very beautiful surroundings.

KUNDAN
An ‘endemic’ disease is one that persists for a long W ild animal do not liv e long in t he ar tif icial
surroundings of a zoo.
time in a particular area: Malaria is endemic in tropical
Africa. An endemic disease does not necessarily affect Now, look at the sentence given below:
large numbers of people at one time. An ‘epidemic’ The surroundings of the university are very beautiful
disease is one that affects very many people at one but the environment of the university campus is not
time in one place: There is an epidemic of flu in California. at all conducive to academic work.
A ‘pandemic’ disease is an epidemic that affects a vast
area, such as the Black Death in the fourteenth 101. Epigram, Epigraph, Epitaph, Epithet
century or the flu outbreak of 1919. An ‘epigram’ is a pithy statement, especially a witty
one. An ‘epigraph’ is an inscription on a building or a
97. Enervate, Invigorate, Energize
monument, or a quotation at the beginning of a book
The first two words have almost opposite meanings. or a chapter suggesting its theme. An ‘epitaph’ is an
When you are ‘invigorated’, you become filled with inscription on a tombstone , or any re mark
energy and enthusiasm. But, when you are ‘enervated’, commemorating a dead person. An ‘epithet’ is a
you are drained of energy, for example, by a gruelling descriptive word or phrase added to somebody’s name,
ordeal or by a serious illness. Do not write enervate or used in place of his name, as when King Richard I
when you mean invigorate. The word ‘energize’ means of England is called the Lionheart.
the same as ‘invigorate’, but many people dislike it.
Prefer invigorate. 102. Equity, Equality
Quite apart from its several legal and financial senses,
98. Enhance, Increase
equity me ans ‘impartiality’, ‘fairne ss’, ‘lack of
(a) The eduction allowance of the state employees will
favouritism’. It is an error to use this word when you
be enhanced soon.
mean ‘equality’ that means ‘the state of being equal’.
The meaning is clear to all who know English, but the
use of the word ‘enhance’ is not appropriate here.
‘Enhance’ does mean ‘increase’, or rather ‘improve’, but
299

103. Error, Mistake, Slip a clause in our contract, or we can invoke a legal
An ‘error’ is a deviation from an expected standard. precedent, or we can invoke the assistance of a
For example, powerful patron. Invoking is done deliberately, and
only a human being can invoke anything.
This book is full of printing errors.
His essay is full of grammatical errors.
In contrast, to ‘evoke’ something is to call it up in
somebody’s mind. For example, a song, a play or a
This word may also refer to a deviation from a moral story may evoke memories of your childhood, or an
standard. For example, unpleasant act may evoke an angry response. Evoking
May God pardon him for the errors of his misguided is not usually deliberate, and it is not usually done
life. by human beings.
‘Mistake’ can replace ‘error’ in such cases. But only 108. Explicit, Implicit
‘mistake’ and not ‘error’, can be used if the intended
‘Explicit’ means ‘stated outright’, ‘clear and fully
meaning is either of the following:
expressed’. It refers to a statement, rule, etc. ‘Implicit’
(i) something done because of carelessness or
means ‘implied or understood though not directly
forgetfulness:
expressed’. For example,
He took my pen by mistake.
I gave you explicit instructions not to tamper with the
(ii) wrong decision: records.
Your request for information contains an implicit
It was a mistake to appoint him.
threat.
In mathematics ‘error’ can refer to the difference

K
between the observed value and the true value of a 109. Famous, Notorious
quantity. For example, ‘Famous’ me ans ‘widely and favourably known’.
These errors are of a very small magnitude. ‘Notorious’, on the other hand, is widely but only
unfavourably known to the people in general. For
Only ‘error’ and not ‘mistake’ can be used in this
example,
context. A ‘slip’ refers to a minor mistake in speech
or writing, a mistake on account of carelessness or Naushad is a famous music director.
absentmindedness. For example, Phulan Devi is a notorious dacoit.

That slip of the tongue in the middle of his speech 110. Farther, Further
made everybody laugh.
‘Farther’ refers to additional distance (How much farther
This is not a genuine mistake; it is only a slip of the
pen. is it to the Juhu beach?), and ‘further’ refers to
additional time or amount, or other abstract matter (I

KUNDAN
104. Especially, Specially don’t want to discuss this matter any further).
‘Especially’ means ‘particularly’ or ‘more than other You are to use further (i) when the sense is ‘additional’
things’; ‘specially’ means ‘for a specific reason’. For (further evidence, further information), or ‘in addition’
example, (Have you anything further to say?), and (ii) when it is a
verb meaning ‘advance’, ‘promote’ (to further one’s own
I especially treasure the memory of days long gone by.
interests). As adjective or adverb denoting distance,
These boots were made specially for me.
farther is the correct word.
105. Everybody, Everyone, Every One 111. Fast, Soon
The words ‘everybody’ and ‘everyone’ mean the same
Come fast, child. You are getting late for school. ‘Fast’
thing, though everyone is sometimes considered slightly means ‘quick’, ‘quickly’, ‘swiftly’, and in some context,
more formal than everybody. These words can only be even ‘soon’. Therefore, all the following expressions
applied to people. But ‘every one’ is different. This are all right in standard English.
means ‘each one’, ‘every single one of them’, and it a fast car; a fast learner; to run fast;
can be applied to things as well as to people. For leave this place as fast as you can.
example, But — and there lies the problem — ‘fast’ is not used
He left several dozen notebooks, but his widow burned together with ‘come’. In the given example, therefore,
every one. we had better use either ‘run fast’ or ‘come soon’ or
‘come at once’.
106. Every day, Everyday
112. Fatal, Lethal
The adverbial phrase is ‘every day’: write This happens
every day, not This happens everyday. But the adjective Though both pertain to death, these words are not
is ‘everyday’, as in an everyday occurrence. equivalent. We apply ‘fatal’ to something which has
caused someone’s death: a fatal accident is an accident
107. Evoke, Invoke which has killed someone involved in it, and a fatal
To ‘invoke’ something is to call upon it, or to appeal to disease is a disease which has killed the person
it, for help or inspiration. For example, we can invoke contracting it. In contrast, we apply ‘lethal’ to something
300

which is capable of killing someone: a lethal weapon is produced or to say for whom the book is meant or to
a weapon which can kill, regardless of whether it has say something else which would not appear relevant
yet killed anybody, and a lethal disease is a disease inside the main body of the book. Unlike a ‘foreword’,
which is capable of killing those who suffer from it. a ‘preface’ is normally written by author of the book.
There is, however, a complication with diseases. A ‘Preface’ can be used with ‘edition’. Therefore, phrases
disease which invariably kills its victims may be like ‘preface to the first edition’. ‘preface to the second
described as fatal. edition’ and ‘preface to the third edition’ are common
and readily acceptable. ‘Foreword’, on the other hand,
113. Few, A Few
refers normally to the book and not to one of its
These two have quite different senses: ‘few’ means editions.
‘hardly any’, while ‘a few’ means ‘some, but not many’.
The first implies that the total is not far from zero, 118. Foot, Leg
while the second implies that the total is greater than
The ‘foot’ is the lower end of the leg beginning at the
zero. So, Few of my students speak French means ‘Hardly
ankle. It includes the arch, the heel and the toes.
any of my students speak French’, while A few of my
The ‘leg’ is the part of the body from the hip to the
students speak French means ‘I have some students
ankle without foot.
who speak French’.
‘Leg’ can be used in idioms like the following:
114. Fewer, Less (i) pull somebody’s leg
Though colloquial English is often different, standard (ii) not have a leg to stand on = have nothing in support
written English uses ‘fewer’ with things that can be of one’s opinion
counted and ‘less’ with things that cannot be counted: ‘Foot’ is used in idioms like the following:
fewer people but less money. Do not write less students

K
(i) put one’s foot down = protest firmly
or less players. (ii) have one foot in the grave = be near death
However, do not write fewer than six weeks. Here
the expression six weeks denotes only a single period 119. Gambit, Gamut
of time, and not a collection of six individual objects, In chess, a ‘gambit’ is the deliberate sacrifice of material
and so the required wording is less than six weeks. for the sake of gaining a superior position. The term
115. Fictitious, Fictional may be extended to any kind of calculated manoeuvre
Both of these words mean ‘made up’, ‘invented’. But made by someone who hopes to gain an advantage, as
they are not quite interchangeable. The word ‘fictional’ in a conversational gambit, but it should not be used
means ‘pertaining to fiction’, ‘found in fiction’, as in more broadly to label any kind of manoeuvre at all.
Shangri-La is a fictional country and Macbeth was a And do not confuse this word with ‘gamut’. This word
historical King of Scotland, not a fictional one. In contrast, means ‘the whole range’, as in the example She ran

KUNDAN
‘fictitious’ means ‘false’, ‘fraudulent’, ‘non-existent’, the gamut of emotions, meaning that she exhibited
as in Cyril Burt’s supposed collaborator was found to be every possible emotion.
fictitious. 120. Gender, Sex
116. Farther, Further Strictly spe aking, ‘gender’ is a grammatical
‘Further’ means ‘some more or additional’. For example, phenomenon found in certain languages in which
nouns are classified into two or more classes requiring
Don’t make further delay.
different agreement. In contrast, ‘sex’ is a matter of
‘Farther’ means ‘more far or more distant’. For example, biology: you are born into one sex or the other. You
Delhi is farther than Varanasi from Patna. should not use gender when you mean sex: write She
was discriminated against because of her sex, not ...
117. Foreword, Preface because of her gender. Using gender for sex sounds
A ‘foreword’ is a short write-up appearing at the prissy: it suggests that you are too embarrassed to
beginning of a book. It does not form part of the main use the word sex in any sense at all.
body of the book. Ge ne rally, a ‘foreward’ says 121. Gather, Collect, Accumulate, Amass
something about the value of the book or something
You can ‘gather’ things which are irregularly distributed,
about its author and is intended thereby to put the
or not clearly separated from one another: to gather
reader in the right frame of mind. A ‘foreword’ can be
information/flowers/crops. ‘Collect’ is like gather. It
written by the author of the book but very often it is
sugge sts that the things you are gathering are
written by someone other than the author of the book.
separate, or can be dealt with one at a time: Collect
A ‘preface’, too, is a brief write-up appearing at the
the magazines and put the m on the shelf/I am
beginning of a book. It does not form part of the main
collecting data for my research project. It (collect) is
body of the book and in this respect it is similar to a
used especially when you want to keep things together
‘foreword’. Normally, the function of a ‘preface’ is to
to form a collection. He collects coins/stamps. If you
state the circumstances in which the book was
‘accumulate’ things, you collect more and more of them
301

over a period of time (often without having a strong The deaf can’t hear.
intention to do this): I’ve accumulated quite a lot of Please listen to what I am saying.
rare paintings over the years. ‘Amass’, however, is We listened but heard nothing.
rather formal. It is used especially of money, goods or
128. Hide, Skin
power collected gradually, but in very large amounts.
Harshad Mehta has amassed a fortune through share- The raw and undressed skin of large animals (like
market. horses, elephants, etc) used for commercial purpose
is called ‘hide’. The thin outer covering on the body of
122. Glance, Glimpse a man or an animal or a fruit is ‘skin’. For example,
‘Glance’ means ‘to look at something quickly’ whereas This leather factory gets two hundred tonnes of hide
‘glimpse’ means to see by chance, just for a moment’. as its raw material.
For example, He has specialized in skin diseases.
I can tell at a glance what ails this man. Let me peel the skin of this orange.
I caught a glimpse of the Rashtrapati Bhawan as I
drove quickly past. 129. Historic, Historical
A ‘historical’ event is one which really happened, which
123. Give, Take (an exam) is not fictitious or mythical. In contrast, ‘historic’ event
My grandson, who is in class IX now, will give school-
is one which is so memorable that it will live in people’s
final exam next year. memories for a long time. For example,
This sentence will be all right only if ‘my grandson’ Neil Armstrong’s landing on the moon was a historic
event.
(who is just studying in class IX now) becomes an

K
He described that event in historical perspective.
examiner of class X next year; otherwise not.
It is the examiner (or the teacher) who gives an exam; 130. Homogeneous, Homogenous
the student mere takes it. The adjective ‘homogeneous’ (five syllables) means
Isn’t this just the opposite of what happens in many ‘having a uniform composition’: Stir the mixture until it
Indian languages? And that, incidentally, explains the appears homogeneous. But the adjective ‘homogenous’
source of this incorrect use. (four syllables) is only a technical term in comparative
anatomy: homogenous structures are structures in
124. Guest, Visitor
different creatures having a common evolutionary
A ‘guest’ is a person who is staying with you, or a origin, like the forelegs of dogs and the wings of bats.
person whom you have invited to a social occasion, The second should not be used when the first is
such as a party or a meal or a person that you have intended.

KUNDAN
invited to your house or to a particular event that you
are paying for. 131. Hope, Expect
(Note that there are some other meanings of ‘guest’ If you ‘hope’ for something, you want something to
too, but our concern is with the above meaning) happen and usually have some reason to expect that
A person, on the other hand, who just pays a it will happen. On the other hand, if you ‘expect’
courtesy call on you and then goes away (after having something, you think or believe something will happen
a cup of tea, if you like) is, in standard English, or someone will arrive.
generally not regarded as a guest; he/she is a ‘visitor’. The central point of difference between the two
words, then, is the following:
125. Hang, Hanged, Hung
In hoping you ‘want’ something to happen; in
Standard English makes a curious distinction here. expecting, on the other hand, you ‘think’ or ‘believe’
In most circumstances, the verb ‘hang’ is irregular that it will happen. Obviously, the difference between
and has the parts hang, hung, hung; The picture was ‘want’ and ‘think’ or ‘believe’ is large enough to
hung in the living room. However, when ‘hang’ means perceive.
‘put to death with a rope’, it is regular, and has the
parts hang, hanged, hanged: They hanged him; He was 132. House, Home
hanged. It is not recommended to write They hung him A ‘house’ is a building for people to live in. And the
or He was hung. place where you live in is your ‘home’, whatever type of
house it is. Your home is the place to which you belong
126. Hangar, Hanger
and where you feel comfortable. It also means the
You put your coat on hanger, but you keep a plane in place where one is born and habitually lives. For
a hangar. example,
127. Hear, Listen He has shifted to a newly built house.
‘Hear’ means ‘to perceive sound, etc, with the ears.’ Bilaspur is my home but I’m living in Delhi just now.
‘Listen’ means ‘try to hear, pay attention to.’ For
example,
302

133. Hurt, Injured, Wounded worth. If something is ‘impracticable’, it can’t be done


‘Injured’ and ‘hurt’ both mean physical damage caused at all.
by an accident. But ‘wounded’ implies physical damage 141. Imply, Infer, Insinuate
caused in an attack. For example,
‘Imply’ means ‘to give or make a suggestion; to express,
In a train accident twenty passengers were injured. show, or mean indirectly’. But ‘infer’ means ‘conclude,
She was badly hurt in a bus and a truck collision. reach an opinion from facts or reasoning.’ Writers or
The enemy attacked our outpost and wounded five
speakers imply something whereas the listener or the
soldiers.
reader infers it. For example,
134. Hyperbola, Hyperbole Ravi’s letters imply that he is doing brisk business.
A ‘hyperbola’ is a certain mathematical curve. But Am I to infer from your remarks that you think I am a
liar?
‘hyperbole’ is overstatement, exaggeration, as in She
is infinitely more talented than her boss. To ‘insinuate’ something is to suggest it indirectly, by
means of hints or innuendo (an indirect remark about
135. If ... was/were
somebody/something usually suggesting something
When the subordinator ‘if’ introduces a counterfactual bad or rude). Only a human being can insinuate
clause—that is, a clause representing something anything. It is wrong to write the following:
which is not true—then, in formal writing, the verb-
These results insinuate that our current model needs
form ‘were’ is required, not ‘was’. Examples, If I were
revision
dictator, I would ban men from wearing shorts; If Susie
were not so plain-spoken, she might be a cabinet minister Use ‘imply’ instead of ‘insinuate’.
by now. Spoken English commonly uses was here, 142. Incident, Accident

K
but writers should learn this formal and possibly
‘Incident’ refers to event, especially of less importance
unnatural use of were.
than othe rs; happe ning that attracts ge ne ral
136. Illegal, Illicit attention. But ‘accident’ means ‘an unexpecte d,
The word ‘illegal’ means ‘prohibited by law’. The word unpleasant occurrence’. For example,
‘illicit’ can also have this meaning, but more commonly Tell me about the strongest incident of your life.
it me ans ‘contrary to pre vailing social more s Drive slowly lest you should meet with an accident.
(conventions)’, as in an illicit love affair.
143. Index, Indexes, Indices
137. Illegible, Unreadable
When an ‘index’ is an alphabetical listing at the back
An ‘illegible’ text is one in which the letters and the of a book, its plural is ‘indexes’. In all other uses, and
words cannot be made out, and hence one which particularly in mathematical senses, the plural is

KUNDAN
cannot be interpreted. An ‘unreadable’ text is one which ‘indices’.
is so poorly written that a reader cannot be expected
to struggle through it. 144. Infamous, Notorious
‘Infamous’ means ‘disgraceful’, ‘worthy of hatred’. It is
138. Imbue, Inculcate
not directly related to ‘famous’, and something which
You ‘imbue’ a person with ideas. The meaning the word is infamous is not necessarily well known. The Serbian
‘imbue’ ‘to fill somebody or something with strong feelings, atrocities in Kosovo were infamous even before anyone
opinions or values’. But you ‘inculcate’ ideas in a person. outside had heard of them. The word does not mean
‘Inculcate’ means ‘to cause somebody to learn and ‘famous’ or ‘celebrated’, and it certainly does not mean
remember ideas, moral principles etc especially by ‘quaint’ (=attractive because of being unusual or old-
repeating them often’. For example, fashioned).
My t eacher s imbued me with a st r ong sense of Something which is ‘notorious’ is not only bad but
professionalism. well-known. So, a notorious torturer is someone who
My t eacher s inculcated a st r ong sense of is widely known as a torturer.
professionalism in me.
145. Ingenious, Ingenuous, Disingenuous
139. Imminent, Immanent The word ‘ingenious’ means ‘extremely clever’, and it
The common word ‘imminent’ means ‘about to happen’, may be applied either to a person or to a plan or a
as in A storm is imminent. The rare word ‘immanent’ device constructed by such a person. But ‘ingenuous’
means ‘inherent’, ‘permanently present throughout means ‘naive’, ‘easily deceived’ when applied to a
the universe’, and is chiefly confined to religon and person and ‘naive, foolish’ when applied to a plan or
metaphysics. an action. The opposite of the second, ‘disingenuous’,
means ‘calculating’, ‘deceptive’, ‘slightly dishonest’.
140. Impractical, Impracticable
If something is ‘impractical’, it can be done, but doing 146. Inimitable, Inimical
it would require more time, money or effort than it’s The word ‘inimitable’ means ‘unique’, ‘not capable of
303

being imitated’. It has been so vastly overused in show only with a past simple tense. For example,
business that it is now best avoided. The word ‘inimical’ I felt a sudden pain in my chest just now.
means ‘hostile’. The second should not be used when
the first is intended. It is wrong to write ‘I have/had felt....just now’.

147. Intimate, Notify 152. Keep, Put


‘Keep’ denotes the continuity of an activity. It indicates
Please intimate me the court’s judgement in the matter. that a thing is going to be in a certain condition for
There are two problems here: some time . ‘Put’ on the othe r hand, de note s a
(a) the grammar of the verb ‘intimate’. momentary action. Now, look at the sentence given
(b) its meaning below:
As for grammar ‘intimate’, like ‘explain’, ‘appeal’ and (a) Will you please keep this letter on my desk?
‘listen’, is followed by the preposition ‘to’ before the
Indirect object (like ‘me’, ‘you’, ‘him’, ‘Mohan’, ‘Sita’). In the use of the verb ‘keep’ in this sentence, the
Therefore, we have to say ‘intimate to me’. speaker is clearly referring to ‘the act of placing the
Next its meaning. Broadly it means the same thing letter’. But this in not the function of the verb ‘keep’.
as ‘inform’ but with a difference. When we say ‘intimate’, This verb refers to ‘the state of retention at a particular
we mean ‘inform in an indirect way, by hints, for place’ as in the following sentence:
example. Furthermore, the word is rather formal, so (b) Keep your passport in a safe place.
not appropriate in an informal situation. And for just the act of placing something somewhere,
Now, ‘notify’. This too means ‘inform’, but with a as implied in sentence (a), the right word is ‘put’.

K
difference. If you notify me of something, you inform
me about it officially. Further, notice that between 153. Lady, Woman, Female
‘notify’ and ‘me’ there is no preposition. So it patterns My neighbour has appoint ed a new lady f or
like the verb ‘inform’. dishwashing and mopping the floor.
148. Issue, Child Even if times have changed a lot, and in this democratic
age we no longer restrict the use of the word ‘lady’ to
How many issues do you have?
refer only to a woman from upper classes, they have
The word ‘issue’ meaning child is nowadays used only not changed so drastically as to legitimise the use of
in the legal language, especially in such sentences this word for domestic helper or servant – except
as: perhaps in jokes. In the above sentence, therefore,
My uncle died without an issue. So I inherited his the right word is ‘woman’, which is general service
property. word to refer to any ‘adult female human being’.

KUNDAN
In other than legal use, the word is so rare that some Now, the word ‘female’. Yes, I have heard it used as
modern dictionaries do not list this meaning of the a noun in contexts similar to the one exemplified
word at all. above, but there is a small problem. Since this word
is more often used as an adjective (as in ‘female
149. Judicial, Judicious employees’, ‘female singer’, ‘female issues’) than as
The word ‘judicial’ means ‘pertaining to judges or to a noun, if you use it in the given example, you
the courts’, as in a judicial review or a judicial separation. unwittingly emphasise the gender of the person you
In contrast, ‘judicious’ has nothing to do with the law: are referring to. Therefore, where no such emphasis
it means ‘prudent’, ‘carefully considered’, ‘showing is required, I would suggest the use of the word
good judgement’, as in a judicious choice and judicious ‘woman’.
advice.
154. Later, Latter
150. Jurist, Juror ‘Later’ is the opposite of ‘earlier’. For example,
A ‘jurist’ is a person who is knowledgeable about the The meeting was to begin at 10 o’clock; Rajesh came
law, especially one who writes about legal matters. much earlier than 10 o’clock and Ramesh came much
The word is not a fancy equivalent for ‘judge’. A juror later.
(note the spelling) is a member of a jury. ‘Latter’ means ‘the second of the two mentioned’. This
151. Just, Justly, Just Now word is the opposite of ‘former’. For example,
‘Just’ me ans ‘a moment ago’. ‘Justly’ me ans ‘in Of these two Nobel Prize winners in India – Raman
accordance with justice or the law’. For example, and Tagore – the former was a physicist, the latter a
poet.
He was justly punished for his crimes.
He has just gone out.
155. Lazy, Idle
Note that normally, only a present perfect tense is ‘Lazy’ means ‘unwilling to work’, ‘doing little work’.
used with ‘just’. ‘Idle’ means ‘not working’ or ‘not being used’. The two
‘Just now’ also means ‘a moment ago’ but it is used words are not synonymous.
304

Rajo is a very lazy boy. (doesn’t like working) from that of like. If I write Like your teacher, I advise
He is sitting idle. (not working) you to learn algebra, then I am not your teacher, and I
He spent many idle hours (did nothing) during the am merely comparing myself to your teacher. However,
holidays.
if I write As your teacher, I advise you to learn algebra,
That machine is lying idle. (not being used)
then I am indeed your teacher, and I am speaking in
156. Legend, Legendary my capacity as your teacher.
A ‘legend’ is a largely fanciful story, or series of stories, 159. Likeness, Resemblance, Similarity
about the past which may have some basis in fact. ‘Like’ and ‘similar’ may re fe r to a similarity in
King Arthur is a good example of a legendary figure: appearance or in any other feature. ‘Resemblance’, on
there may well have been a historical Arthur who did the other hand refers mostly to similarity in physical
something or other fairly important, but the tales of appearance. For example,
King Arthur and the Knights of the Round Table are
These two girls resemble each other very much. I think
entirely medieval fabrications. they are twins.
The adjective ‘legendary’ may safely be extended to
a real person who has achieved such outstanding ‘Like’ is a general word. It may indicate any degree of
success in some field as to appear almost larger than similarity ranging from virtual identity in all respects
life. Good examples are the cricketer Don Bradman, to a chance similarity in only respect.
the base ball playe r Babe Ruth, and Napole on ‘Similar’ emphasises the likeness between different
Bonaparte. But it is out of order to apply the label things, the implication being that the difference may
legendary to a real person who is merely well known, be ignored or overlooked for a time.
or who is known only to a small group of specialists. 160. Look, Watch

K
The Egyptian Pharaoh Peribsen is a well-documented ‘Look’ is often used as an intransitive verb. It means
historical figure. Nobody apart from Egyptologists has to direct the sight with attention; to give attention; to
ever heard of him, but this fact does not make him seem; to appear. It is generally followed by a preposition
legendary. + noun. For example,
157. Lie, Lay Look at the map of India on the wall.
(a) When ‘lie’ is used as a noun it means ‘a false (Here ‘Look at’ means ‘direct your sight with attention
statement made with the intention of deceiving’. to’)
For example,
He looks sad.
Don’t tell a lie.
(Here ‘looks’ means ‘seems or appears’)
(b) When ‘lie’ is used as an intransitive verb it means
‘to utter falsehood with an intention to deceive’. In this She looks (=appears) like her mother.

KUNDAN
meaning the past and past participle of ‘lie’ is ‘lied’ ‘Watch’ is often used as transitive verb. It means ‘to
only. keep in view, to observe attentively’. For example,
(c) As an intransitive verb ‘lie’ has other meanings We watch things that change, move or develop.
also. It means ‘to be in a horizontal or nearly horizontal Watch that man; just see what he is going to do.
position’, ‘to be situated’, ‘to be or remain passively’.
‘Watch’ is sometimes used as an intransitive verb.
In these meanings ‘lay’ is the past form and ‘lain’
Then it means ‘to keep vigil, to be on the alert’. For
is the past participle form of the verb ‘to lie’.
example,
(d) ‘Lay’ is used as a transitive verb. It takes an
object. It means ‘to cause to lie’, ‘to place or set down’, The police watched all night for the thieves.
‘to spread something on’, ‘to deposit’, ‘Laid’ is its past
161. Luxuriant, Luxurious
and past participle form.
Note carefully that ‘lay’ is the present form of the The word ‘luxuriant’ means ‘lush, rich’, as in luxuriant
verb ‘to lay’ and past form of the verb ‘to lie’. vegetation. But ‘luxurious’ means ‘sumptuous’, as in a
luxurious apartment. Misuse is common in both
158. Like, As directions, but is as yet tolerated by no commentator.
In formal English, ‘like’ cannot be use d as a In any case, luxurious is increasingly replaced as a
conjunction. Though common in speech, the usage proposed modifier by luxury, as in a luxury apartment.
illustrated by We should proceed like we did last time This new form has a small advantage in brevity, but
must be avoided: write We should proceed as we did no other advantage I can think of. Stick to luxurious in
last time. Similarly, write He batted as though he were careful writing.
possessed, not He batted like he was possessed. But
162. Male, Masculine, Female, Feminine
do write He batted like a man possessed, not He batted
as a man possessed. The words ‘male’ and ‘female’ denote the biological sexes:
However, it is not true that ‘as’ cannot be a a male lion, a female engineer. The words ‘masculine’ and
preposition. It can be, but its meaning is different ‘feminine’ are applied to characteristics perceived as
305

typical of each sex, or to individual people seen as My nehpew’s marriage takes place next month.
exhibiting these characteristics: a masculine man is a Only this definition of ‘marriage’ refers to the ‘marriage
man perceived as having manly characteristics, and a ceremony’. And for this definition, we have another
feminine style is a style perceived as appropriate to word, ‘wedding’, which is unambiguous because,
women. You should not use masculine and feminine to unlike the word ‘marriage’, it does not have any other
mean merely male and female: it is wrong to write a meaning. It is for this reason that the anniversary
masculine name for a male name (like Henry) or the feminine you celebrate in honour of the ceremony of your
gender for the female sex. marriage are called ‘wedding annive rsaries’ not
The adjective ‘effeminate’ is a contemptuous label ‘marriage anniversaries’.
applied to a man perceived as exhibiting stereotypical
female characteristics. The nearest counterpart for a 166. Masterful, Masterly
woman perceived as exhibiting the characteristics of The adjective ‘masterful’ means ‘dominating’, ‘exercising
a man is ‘mannish’. authority very effectively’. We may write, for example,
of King Alfred’s masterful leadership. But ‘masterly’
163. Mannequin, Manikin, Mannikin
means ‘very skilful’: we may write of a masterly analysis
A ‘mannequin’ is a plastic human figure displayed in of the problem.
the window of a clothing store. Occasionally the word
is also applied to a human model on the catwalk. A 167. Maternal, Motherly
‘manikin’ is a tiny man, either a boy or a dwarf, and it ‘Maternal’ means ‘related on the mother’s side’ or
is also a model of the human body used in teaching ‘inherited from the mother’s side’. For example,
medicine or art. For the second, a variant spelling He is my maternal uncle.

K
‘mannikin’ is recognised by some dictionaries, but this He is going to sell all his maternal property.
is widely disliked and should be avoided.
‘Motherly’ means ‘mother-like’. For example,
164. Marketing, Shopping I was overwhelmed by her motherly treatment.
We have run out of groceries. Can we go out marketing I cannot ignore has motherly advice.
this evening?
168. May, Might
The intended meaning of the word ‘marketing’ here is
In the present tense, either of these is acceptable,
‘buying’. But this is almost the opposite of what the
though the first perhaps suggests a somewhat greater
word ‘marketing’ actually means, viz, “the activity of
degree of confidence than the second: ‘We may have a
presenting, advertising, and selling a company’s products
peace treaty this month’ and ‘We might have a peace treaty
in the best possible way”. Thus, marketing is even
this month’ are both permissible.
broader than selling and has nothing to do with

KUNDAN
The difficulties begin in the past tense. A past-
buying.
tense verb-form can normally only be followed by might,
Actually the word the speaker needs to use in place
and not by may. So, the required form is ‘Susie said
of ‘marketing’ is ‘shopping’, which means “the activity of
that she might be here’, and not ‘Susie said that she may
going to shops or stores and buying things”.
be here’. Likewise, write We believed that we might have
165. Marriage, Wedding a chance, and not We believed that we may have a chance.
Under the word ‘marriage’, you will find three concepts The use of may in such sentences is decidedly non-
dealt with in any good dictionary. These are given standard, and it will cause many readers to grind their
below: teeth.
(a) A legal relationship between a husband and wife. An even greater problem arises with the pair may
For example, have and might have. In standard English, these two
are very sharply distinguished, as follows. If I write
In any happy marriage, both husband and wife have to
‘We may have won’, this means ‘Maybe we won, and
exercise a good deal of tolerance.
maybe we didn’t: I don’t know what the facts are.’ But, if
Obviously, there is no reference here to the ‘marriage I write We might have won, this carries the very different
ceremony’. Also, whe n you say that somebody’s meaning ‘We didn’t win, but, in slightly different
marriage is on the rocks, you certainly do not mean circumstances, our winning was possible.’
that their marriage ceremony, which took place In other words, might have is counterfactual: it is
sometime in the past, is on the rocks. always followed by something which is not true. But
(b) The state of being married. For example, may have is not counterfactual: it is followed by
Last month a senior citizen couple celebrated fifty five something which is not known to be false. This
years of their marriage. contrast is of central importance in standard English,
Here again there is no reference to the ‘marriage and mastery of it is essential. So, ignore all those
ceremony’. football coaches who routinely intone If it hadn’t been
(c) The ceremony in which two people get married. for that dodgy call, we may have won. Standard English
For example, absolutely requires might have won here, and, if you
find this unnatural, you will simply have to grit your
306

teeth and learn it. You can’t imagine how awful that The seriousness of her crime was mitigated by the
non-standard may have sounds to careful writers. appalling treatment she had enduced.
It is not possible to follow mitigate with against.
169. Maybe, May be
You are probably familiar with the phrase mitigating
These two are often confused. But there is a simple circumstances, which means ‘circumstances that
way to tell them apart: the single word ‘maybe’ can reduce the seriousness of an offence’.
always be replaced by ‘perhaps’ without changing the
meaning, while the two words ‘may be’ cannot. So, 174. Moguls, Mogul, Mughal
write Two decisions may be more important than others The Muslim rulers of India were formerly called
but Two decisions, maybe more, will be more important ‘Moguls’ in English, but today the spelling ‘Mughals’
than others. Note that ‘perhaps’ can replace maybe in is increasingly preferred and is recommended here.
the second example, but not may be in the first. But our informal word for a powerful person, often
especially in the entertainment industry, is always
170. Meet, See
written ‘mogul’, as is the unrelated word ‘mogul’ for a
I think I must meet my boss to settle this issue. mound of hard snow on a ski slope or a snowboard
‘Meet’ is not the right word here; the right word is course.
‘see’. You meet someone somewhere either by sheer 175. Moneys, Monies
chance or by arrangement when both you and the other Though the plural of ‘money’ is hardly ever required in
person come to the same place especially for such ordinary contexts, that plural is the regular ‘moneys’.
meeting. But an exception occurs in legal language, in which
171. Meteor, Meteorite the irregular plural ‘monies’ is usual in the sense of
‘sum(s) of money’.

K
A ‘meteor’ is a bright streak of light in the sky produced
by a small body from space travelling through the 176. Moral, Morale
atmosphere at enormous speed, what is informally As a noun, ‘moral’ means ‘ethical conclusion’ or
called a shooting star. The word is also applied to the ‘lesson’. ‘Morale’ means ‘spirit’ or ‘state of mind’, or
small body producing the light. A ‘meteorite’ is a lump ‘the condition of courage, determination and pride in
of stone or metal on the earth’s surface, the remains the mind of a person, team, army, etc.’ For example,
of a ‘meteor’ which was big enough to survive passage
through the atmosphere without being burned up. The moral of this story is that death is inevitable.
The morale of our soldiers is high.
172. Migrate, Emigrate, Immigrate
177. No More, No Longer
To ‘migrate’ is to move from one place to another. This
‘No more’ is used to express ‘quantity’ or ‘degree’. ‘No
movement may be more or less permanent, as when
longer’ is used to express ‘time’. For example,

KUNDAN
persistent drought forces people to leave their homes
and migrate to a new area, or it may be temporary or There is no more bread in my house.
cyclical, as when nomadic hunter-gatherers migrate The people of t his village no longer suppor t the
from place to place according to the seasons. To congress party. (Never write.....no more support...)
‘emigrate’ is to leave one’s homeland: The potato blight 178. Offer, Opt for
forced many Irish people to emigrate. To ‘immigrate’ is
In this university, if you enrol yourself for MA in
to travel to a new home: During the nineteenth
English, you can offer as many as three Courses in
century, millions of Europeans immigrated to the USA. English language.
Of course, a migrant always travels from one place to
another place, and your choice of word depends on I don’t know of a university where students offer
whether you want to emphasise the leaving or the courses. The fact is that they can only choose or ‘opt
arrival. So, we speak of emigrants from the Ireland of for’ the courses of their liking out of those the
the famine, but immigrants from elsewhere to the USA. university ‘offers’ them. This mistake is similar to
the one discussed under ‘give, take (an exam)’
173. Militate, Mitigate
179. Opht halmol ogist, Oculist, Op tometr ist,
These similar-looking but unrelated verbs are often
Optician
confused. The verb ‘militate’ normally only occurs in
the e xpre ssion militate against, which me ans An ‘ophthalmologist’ is a me dical practitione r
‘disfavour’, ‘work against’. For example, specialising in diseases of the eye. Another word with
the same meaning is ‘oculist’, which is now rarely used.
Her punk hairstyle and pierced face militate against An ‘optometrist’ is a person who is qualified to examine
her chances of getting a teaching job.
the eyes and to prescribe spectacle lenses. An optician
It is not possible for a person to militate against anything,
or for anything to militate against a person. is not qualified to prescribe lenses, but merely sells
spectacle frames. In practice, a high-street shop
The verb ‘mitigate’ means ‘soften’, ‘reduce the severity usually contains both ‘optometrists’ and ‘opticians’. In
of’: Britain, an ‘optometrist is often called an ‘ophthalmic’
optician, while an optician is called a dispensing optician.
307

180. Oral, Verbal body. Thus, we say ‘headache’, ‘heartache’ ‘toothache’,


Something which is ‘oral’ is spoken: for example, an ‘stomach ache’, but not ‘legache’.
oral agreement is one which is made by speaking, with 185. Pair, Couple
nothing written down. In contrast, anything which is
A ‘pair’ means (i) two things of the same kind to be
‘verbal’ is expressed in words, whether spoken or
used together: a pair of shoes/gloves; (ii) single article
written. Hence a verbal agreement may be either
with two parts always joined: a pair of scissors/
spoken or written down. It is an error to write verbal if
trousers/tongs; (iii) two persons closely associated,
what you mean is oral. It may help to remember that
eg, an engaged or married couple: the happy pair (=two
non-verbal communication is communication by
newly married persons).
means of postures, gestures and expressions, without
Any two things of the same kind can be spoken of
the use of words.
as a ‘couple’: I saw a couple of rats in the garden. It
181. Ordinance, Ordnance also means (i) two people who live or spend time
An ‘ordinance’ is a bylaw or a decree. In contrast, together, especially a husband and a wife: look at the
‘ordnance’ is munitions—that is, ammunition, shells newly married couple; (ii) a few; several; small number:
or artillery. Curiously, the official map-making body She’ll be back in a couple of minutes.
of the British or Irish government is the Ordnance 186. Per cent, Percent
Survey.
The preferred British form is ‘per cent’, while the
182. Ostensible, Ostensive, Ostentatious preferred American form is ‘percent’. However, the
The word ‘ostensible’ means ‘apparent’, ‘seeming’, American form is gaining ground in Britain and is now

K
‘professed’: so, for example, an ostensibly independent acceptable in some quarters, though not in all.
assessment is an assessment which is apparently Note also the following. If interest rates rise from
inde pe nde nt, which is publicly pre se nte d as 10% to 11%, then they have not risen by one per cent,
independent. As this example may indicate, today the but by ten per cent. (If you were paying £100 a month
word almost always carries a suggestion of deception: before, you are paying £110 a month now - an increase
the writer of this phrase is delicately implying that, of ten per cent.) But you can write that rates have
even though somebody has called the assessment gone up by one percentage point.
‘independent’, it is not really independent. In other 187. Perquisite, Prerequisite
words, the assessment has been rigged. Therefore,
A ‘perquisite’ is a privilege attached to a certain job,
you should not use ostensible if you do not mean to
such as a company car or free use of a gym. The word
imply any such thing: use ‘apparent’ or ‘seeming’
is informally shortened to perk. But a ‘prerequisite’ is
instead.

KUNDAN
a condition you must satisfy in order to qualify for
The rare word ‘ostensive’ is primarily a technical
something: A good degree in French is a prerequisite for
term in philosophy. You should avoid it unless you
this job.
are sure of its technical sense.
An ‘ostentatious’ action is one which is pretentiously 188. Persecute, Prosecute
flamboyant, which is intended to impress gullible To ‘persecute’ somebody is to deliberately make him
people: Her ostentatious clothes struck me as vulgar. suffer, usually because of his race, religion, social
Do not use ostensive or ostentatious when you mean group or political beliefs: Queen Mary persecuted the
ostensible, and do not use ostensible if you mean only Protestants. To ‘prosecute’ somebody is to bring legal
‘seeming’. charges against him: Shoplifters will be prosecuted.
183. Output, Outcome There is also a much rarer sense of prosecute, ‘continue
with’, now largely confined to war, as in Beyazit
The word ‘output’ means ‘the amount of something
prosecuted his war against the Europeans until
that a person, a machine or an organisation produces’.
Tamerlane’s invasion forced him to desist.
Whereas ‘outcome’ means ‘the result or effect of an
action or event’. For example, 189. Persons, People
We are waiting to hear t he f inal outcome of the The word ‘person’ has two plural forms – ‘people’ and
negotiations. ‘persons’, the commoner one being ‘people’. The form
Manufacturing output has increased by 10 per cent. ‘persons’ is used in very formal or legal language. The
difference is easily seen in the following examples:
184. Pain, Ache
In yesterday’s rail accident two (or three/twenty/many)
When used as nouns, I don’t see much difference people were killed.
between these two words. Both stands for the feeling
(‘Persons’ would be inappropriate here)
of great discomfort in your body. May be ‘ache’ refers
to a steady and fairly strong pain in some part of your Persons who wish to file a petition should do so before
body, but more than that it is by convention that we the end of this month
use ‘ache’ (not, ‘pain’) for pain in certain parts of the (‘People’ would be inappropriate here.)
308

190. Perspicacious, Perspicuous consensus of opinion consensus


A ‘perspicacious’ person is one who is shre wd, each and every each
pe rce ptive or discerning. Only a pe rson can be effective demonstration demonstration
perspicacious. A ‘perspicuous’ thing is a thing which end result result
is clear, lucid and easy to understand, such as a exactly the same the same
perspicuous explanation. A pe rson cannot be final completion completion
perspicuous. The derived nouns are perspicacity and free gift gift
perspicuity, respectively. As so often, you can avoid future plans plans
tangling these words up by simply avoiding them in in the field of linguistics in linguistics
favour of simpler synonyms like perceptive and lucid. minute detail detail
a new innovation an innovation
191. Pick, Choose one and the same the same
‘Pick’ suggests merely selection, sometimes in a rather particular interest interest
perfunctory manner. ‘Choose’ suggests careful thought period of two weeks two weeks
and deliberation, and the weighing of one thing against personal opinion opinion
another. We pick a winner and pick a football team, prior experience experience
but choose a birthday or wedding present, choose a razed to the ground razed
site for a new building, choose a name for a child, and rectangular in shape rectangular
choose one of the several things that are offered. For red in colour red
example, refer back refer
He has been picked for the Indian Cricket Team. repeat again repeat
She chose her words carefully hoping to avoid a quarrel. revert back revert

K
shorter in length shorter
192. Play, Game small in size small
‘Play’ is often used to express a drama or a dramatic summarize briefly summarize
performance. Thus ‘play’ is also a piece of literature surrounded on all sides surrounded
written for the theatre or television. For example, temporary respite respite
Julius Caesar is one of Shakespeare’s early plays. track record record
true facts facts
‘Game’ means ‘a sport of any kind; a contest for usual custom custom
recreation’. Carrom, chess, football, volle y ball, very widespread widespread
badminton etc are games. (They are not plays.) For
example, 194. Practical, Practicable, Pragmatic
Chess is a very slow game. A ‘practical’ person or thing is effective and down-to-

KUNDAN
e arth, not conce rne d with or involving e mpty
193. Pleonasm flamboyance . The labe l is a complime nt. But
A ‘pleonasm’ is the use of a word which merely repeats something which is ‘practicable’ is merely something
a bit of meaning which is already present in another which can be done, regardless of whether doing it
word. Here is an example: The two armies combined would be valuable. This word cannot be applied to a
together. Things which combine are ne ce ssarily person. Except in linguistics and philosophy, in which
together as a result, and so that together is pleonastic. it has special technical senses, ‘pragmatic’ means
Write instead The two armies combined. And here is ‘concerned only with results in the real world, and
another example: There might be potential commercial not with theoretical or moral stances’, and the word
developments. Here might carries the same meaning can be applied to both people and policies.
as potential: write either There might be commercial So, if you have a task to perform, a practicable
developments or T here are potential commercial approach is one which can be done, a practical approach
developments. is a sensible one, and a pragmatic approach is one
Be low is a list of some of the most fre que nt which is meant to be effective.
pleonasms, with corrected forms. 195. Price, Cost
Pleonastic Improved ‘Price’ means the amount of money for which a thing
adequate enough adequate is offered: What price did you pay for the house? Cost
an added bonus a bonus (n) is like price, but is used less for objects, and more
3 a.m. in the morning 3 a.m. (i) for services: the cost of having the car painted (ii)
a total of 200 people 200 people for general things: the cost of living. The amount of
basic essentials essentials money we pay for something is what it costs (v) us.
close proximity proximity
close scrutiny scrutiny The price of gold has risen sky-high.
How much did this motorbike cost you?
completely exhausted exhausted
309

196. Premiss, Premise, Premises supporter (or proponent) of legalized abortion, not He is a
In logic, a statement upon which a following statement protagonist of ....
is based is usually a ‘premiss’ (plural premisses) in 202. Proved, Proven
British English, but the spelling ‘premise’ (plural
The participle of ‘prove’ is usually ‘proved’ in standard
premises) is also acceptable in Britain, and is universal
English. Examples: We have proved it; This hypothesis
in American English. In all varieties of English, the
has never been proved. In such cases, the variant
plural form spelled ‘premises’ is a formal or legal term
‘proven’ is much less acceptable, especially in British
for a property.
English, though it cannot quite be called wrong there,
197. Prescribe, Proscribe and it is in fact rather common in American English,
To ‘prescribe’ something is to lay it down as a rule or a where both forms are accepted as standard. However,
procedure to be followed (or, in the case of medicine, the adjective is almost always proven: her proven
of course, to give instructions to the patient for its competence; a proven remedy. Note also the special case
use). In contrast, to ‘proscribe’ something is to prohibit of not proven in Scottish law.
it or condemn it. So, a prescribed book is a set book, a 203. Quantum Jump, Quantum Leap
book which students following a particular syllabus
In physics, when a particle makes a ‘quantum jump’, it
must read or study, while a proscribed book is a
moves from position A to position B without ever being
banned book.
anywhe re in be twe e n - that is, the jump is
198. Proceed, Precede discontinuous (and also very small by everyday
The verb ‘proceed’ means ‘to move on; to continue in a standards). Journalistic prose commonly uses quantum

K
course of action or set of actions.’ But the verb ‘precede’ jump or quantum leap to mean ‘a sudden and dramatic
means ‘to come before; to happen just before’. For change, especially an increase’, as in a quantum leap
example, in house prices. Not only is this expression now a cliche,
it results from a bad misunderstanding of the physical
We can now proceed to the visiting room.
My work is proceeding according to plan.
sense of the term. You should avoid the expression
My name precedes yours in the alphabet. in formal writing, unless you are using it in its physical
The flash of lightning preceded the sound of thunder. sense.
204. Race, Nation, State, Tribe
199. Program, Programme
All these words stand for large groups into which
In American usage, the spelling is ‘program’ in all
human beings may be divided. The largest of these
circumstance s. In British usage , the spe lling
group is a ‘race’. It means ‘a group of people of the
‘programme’ is preferred for most purposes: a theatre

KUNDAN
same class or physical type’. A ‘nation’ is ‘a group of
programme, a radio programme. But even in Britain a
people who share a common history and usually a
computer program is always so spelled. In all senses,
language, and usually but not always live in the same
and in all varie tie s, the de rive d adje ctive is
area’. A ‘state’ is either a politically independent
programmatic.
country, or one of the states making up a country,
200. Prophecy, Prophesy such as Bengal, Bihar, Uttar Pradesh, etc. A ‘tribe’ is
The noun is prophecy; the verb is prophesy. ‘Prophecy’ ‘a social group smaller than a nation, sharing the
means ‘ a statement that tells what will happen in common customs and usually the same language, and
the future’; whereas ‘prophesy’ means ‘to say what often following an ancient way of life.’ For example,
will happen in the future’. If you prophesy a fall in We all belong to the Aryan race.
share prices, then this outcome constitutes your Do you know anything about the Indian nations of
prophecy. And there is no such word as prophesize. America?
The President spoke on radio to the nation.
201. Protagonist, Antagonist Most former colonies have now become self-governing
The word ‘protagonist’ means ‘first actor’, and it states.
originally denoted the single most important character He is a member of the Zulu tribe.
in a Greek drama. Some people therefore object to
recognising more than one protagonist in a single 205. Rare, Scarce
matter, but all authorities on English usage now agree Things that are uncommon, and perhaps valuable, are
that this extended sense is perfectly acceptable: you ‘rare’; a rare bird/coin/disease. Ordinary useful things
may safely write He is one of the protagonists in this that we have not got enough of, are ‘scarce’. It means
affair, meaning ‘He is one of the principal actors’. ‘not much or many compared with what is wanted’.
But it is a serious error to write protagonist to mean Apples are scarce this season.
‘supporter’. Even though an ‘antagonist’ is an opponent We can use rare, but not scarce, about time: One
or adversary, the word protagonist has nothing much of my rare (=not happening often) visits to New York.
to do with antagonist, and it does not contain the It is a rare bird (hard to come by). Mango is scarce
element pro-, meaning ‘in favour of’. Write He is a in winter, and costs a lot.
310

206. Ravage, Ravish and in the phrase as regards. In al l othe r


To ‘ravage’ a territory is to wreak destruction on it. As circumstances, the required word is ‘regard’. In
for ‘ravish’, this has had several sense, but most particular, write with regard to this issue, not with
commonly it is a quaint or delicate word for rape. The regards to this issue.
common error is writing ‘ravish’ where ‘ravage’ is 212. Regretful, Regrettable
required, as in this example: The fighting has ravished
The word ‘regretful’ means ‘full of remorse’, and only a
Lebanon.
person can be regretful. But ‘regrettable’ me ans
207. Raze, Rase ‘unfortunate’, and it is applied to circumstances or to
The spelling is ‘raze’ in all varieties of English; the behaviour. Write Her behaviour at the party was
older ‘rase’ is now obsolete. Note that to raze a regrettable, and she is now suitably regretful.
building is to demolish it, to level it, so that nothing 213. Release Launch
remains of it except perhaps the foundation. It is
therefore a pleonasm to write The buildings were razed It will be a good idea to have the book released by a VIP
to the ground. Prefer instead The buildings were razed. at a special ceremony.
There is no partial razing. Now, it is a curious fact of standard English that,
while, CDs, videos, and films are released, books are
208. Rebut, Refute
only ‘lauched’. If you ask me for the reason, I won’t be
These words are frequently confused and frequently able to give you one.
used wrongly. To ‘rebut’ a statement is to offer clear
evidence or a reasoned argument against it. To ‘refute’ 214. Release, Relieve
a statement is to prove it wrong. Neither word means Your request for transfer has been granted, but I can’t

K
deny or contradict, and this is where most of the trouble say when they are going to relieve you.
arises.
Whe n we fre e some one from his or he r duty,
Suppose I say to you ‘All swans are white.’ If you
responsibility, or contract, we ‘release’ them. So when
simply reply ‘No, they’re not’, then you have denied
a worker is officially freed to go on transfer, for
my statement (and contradicted me), but you have
example, he is ‘released’ not ‘relieved’.
neither rebutted nor refuted anything. If you produce
It is all right, however, to use the word ‘relieve’ in
an e ncyclopaedia article on black swans or an
the following context:
ornithologist who can report on his experience with
black swans, then you have rebutted my assertion - I know you are tired, but please do carry on for a while.
that is, you have assembled good evidence against it. Seema will come soon to relieve you.
If you show me a black swan, then you have refuted He re, ‘relieve’ me ans ‘to re move or re duce an
my assertion - that is, you have proved it wrong. unpleasant feeling or pain’.

KUNDAN
Do not make the common error of writing refute, or
215. Repel, Repulse
even rebut, when all you mean is deny. It is usually
very difficult to prove that a statement is wrong, and Often confused. To ‘repulse’ is to drive back, usually
refute should accordingly be used with great care. in war: The Chechens repulsed the Russian attack. To
‘repel’ is to excite disgust in: Susie is repelled by the
209. Refer, Allude smell of raw fish.
When you ‘refer’ to something, you do so explicitly,
216. Repellent, Repugnant, Repulsive
citing it by name. However, when you ‘allude’ to
something, you do so inexplicitly, without naming it. All three of these adjectives mean ‘loathe some’,
I can refer to my wife by writing Jan or my wife, but, if ‘disgusting’, ‘offe nsive to the se nse s or to
I write somebody I know, with Jan in mind, I am only sensibilities’. But repulsive is the strongest word of
alluding to her. the three: while you merely keep away from something
which is repellent or repugnant, you recoil from
210. Referee, Umpire something repulsive.
A person who controls and regulates a basketball, or The word repellent also me ans ‘resistant’ (for
billiards or football or hockey or rugby or a boxing example, to water), as in water-repellent fabric. And
match is known as a ‘referee’. But an ‘umpire’ is a person note the spelling, with -ent, not -ant.
who regulates and controls the matches of either
217. Residence, House
badminton or baseball or cricket or tennis or wrestling.
Raman was the referee of the Durand Cup Final. (a) You can come to my residence in Sector 14.
The umpire Dicky Bird gave the batsman the benefit ‘Residence’ is a house, especially large or impressive
of doubt. one. For example,
211. Regard, Regards 10, Race course is the Indian PM’s official residence.

The noun ‘regards’ is only appro priate in the


construction illustrated by Give your wife my regards
311

218. Respective, Respectively 222. Shade, Shadow


The se some what formal words show that two ‘Shade’ is any place sheltered from the sun. And the
sequences should be interpreted as consisting of dark shape made by the shade of something is
paired items. For example, Jan and Larry drank whisky ‘shadow’. For example,
and brandy, respectively means that Jan drank whisky dThe tree gives us pleasant shade in summer.
while Larry drank brandy. The tree cast its shadow on the wall.
It is, of course, necessary to have the same number
of items in each list. Consider the following mess: 223. Shall, Will
Alice, Susan and Brenda are respectively divorced and Over the years, these two words have called forth
separated. This is gibberish, since a list of three is torrents of ink, not all of which has been well spilled.
linked to a list of two, and the reader has no chance But the only rule you need is a simple one: if one
of figuring out what the writer is trying to say. These word or the other feels completely natural, then use
words are very frequently used pointlessly, when no it without hesitation. Just forget what you think you
ambiguity is possible, as in the example Henman and may have read in some usage manual. What that
Kafelnikov won their respective matches. It is manual said was very likely wrong anyway, or else it
inconceivable that the two players could win each was painstakingly describing the sort of English
other’s matches, or win somebody else’s matches, and spoken fifty years ago in another country.
so that respective should not be there: write Hemnan In speech, both words are very often replaced by ’II
and Kafelnikov won their matches. anyway, as in This’ll do the trick. Like all contractions,
In the same vein, it is hardly necessary to write this one should be used sparingly in careful writing,
the following: After the meeting broke up, the delegates

K
and not at all in very formal writing.
returned to their respective homes. Just write ... returned In American English, shall has almost completely
to their homes, or, better still,... returned home. Who disappeared, except in the kind of question illustrated
will suspe ct the de le gate s of re turning to one by Shall I open the window? (‘Would you like me to
another’s homes? open the window?’). Some (not all) Americans also
219. Rob, Steal retain the word in one or two fossilized expressions:
A certain writer, who shall remain nameless... The
Things are ‘stolen’ and people are ‘robbed’. The verb
negative form shan’t is entirely dead in the US.
‘steal’ has, as its object, what is taken by the thief.
British English seems to be going the same way,
For example,
though shall is still vigorous in some circles, and shan’t
Somebody has stolen my scooter. still finds some use.
He stole a few antique coins from the museum. There is a traditional textbook ruling that runs as

KUNDAN
The verb ‘rob’ has, as its object, either the person follows. For simple futurity, you use shall after I or
whose things are taken, or the ‘place’ from where we but will after everything else, while, to express
things are taken. The object of the verb rob is never determination or command, you use will after I or we
the things that are taken. For example, but shall after everything else. By these rules, the
The bank was robbed last week. required forms are We shall finish tonight (simple
He has been robbed of all his belongings. statement) versus We will finish tonight (expressing
determination), but They will finish tonight (simple
220. Sceptic, Skeptic, Septic statement) versus They shall finish tonight (an order).
A person who is not inclined to believe things readily As grammarians never tire of pointing out, these
is a ‘sceptic’ in British English but a ‘skeptic’ in bizarre rules do not accurately describe the real usage
American English; these two spellings represent the of careful speakers at any time or in any place in the
same pronunciation. The de rived adjective s are history of English, and they are little more than a
likewise sceptical and skeptical. But ‘septic’ is a medical fantastic invention. If you are one of that handful of
term meaning ‘infected’, as in a septic toe. The common speakers for whom these rules now seem completely
error here is writing ‘sceptic’ where ‘septic’ is intended. natural, then by all means go ahead and follow them.
But, if you are not, just forget about them, and use
221. Sensual, Sensuous your natural forms.
A person addicted to bodily pleasures derived out of
food, drink and sex is termed ‘sensual’. A person fond 224. Sick, Ill
of beautiful sights and sounds and of objects which The adjective ‘sick’ formerly meant ‘ill’, ‘in bad health’,
are pleasant to smell, taste and touch is a ‘sensuous’ and it still does in American English. In British
person. For example, English, however, sick is now commonly restricted to
the senses of ‘experiencing nausea’ and ‘mentally
He is all for purely sensual pleasures.
Keats’s poetry is remarkably sensuous. disturbed’, and ‘ill’ is preferred as the everyday word.
However, the older sense of sick survives in Britain
in such expressions as sick pay and sick leave, and
also in the locution He’s a sick man.
312

225. Silken, Silky anything—though hardly e ve r a drug—which


A silken cord is a cord made of silk, while a silky cord encourages you to undertake or to pursue an action.
is a cord which looks like silk, but very likely is not. A The plural of stimulus is stimuli.
cat may have a silky coat, but not a silken coat. However, 233. Storm, Cyclone, Typhoon, Hurricane
the word silken is now somewhat old-fashioned, and
The general word for rough and especially windy
today we normally use silk to label something made
weather conditions is ‘storm’. A large, violent storm
of silk: a silk tie, not a silken tie.
with a circular wind is called a ‘cyclone’ in the tropics,
226. Smell, Stink a ‘typhoon’ in the western pacific, and a ‘hurricane’ in
A ‘smell’ can be good or bad but a ‘stink’ is always bad, the western Atlantic ocean.
offensive smell, especially a very strong one. For A fierce storm is raging.
example, This year a cyclone took a heavy toll of life in Bangla
Desh.
The smell of these roses is sweet.
The Western Pacific region, was lashed by typhoon.
I can’t stand the pungent smell of this acid.
The coastal region of the Western Atlantic ocean was
The stink of burning rubber is unbearable.
laid waste by a hurricane.
227. Smile, Grin
234. Struck, Stricken
A ‘smile’ is ‘an e xpression of the face showing
The past participle of strike is normally ‘struck’. This
amusement or happiness’. A ‘grin’ is ‘a very wide smile
has often struck me as curious; He was struck by her
which usually shows the teeth’. For example,
beauty. The form ‘stricken’ is now normally an adjective:
He smiled and then accepted the proposal. a stricken look, a stricken cry. However, stricken can be
She stood there with an embarrassed grin on her face.

K
used with names of diseases: He was stricken with
polio.
228. Sometime, Some time, Sometimes
Of these three, ‘sometimes’ is easy. It means ‘now and 235. Substitute, Replace
again’, and it is always written as one word: My wife Suppose the football player Shearer is forced to leave
and I sometimes play Scrabble. The other two are more the match with an injury, and Owen comes off the
difficult, and usage varies. When some is unstressed, bench to take his place. Clearly a substitution has
the form is ‘some time’: We’ll need some time to consider occurred, but who has been substituted?
this. When the sense is ‘an indefinite length of time’, In standard English, the player who has been
it is usual to write some time: She arrived some time substituted is Owen, not Shearer: Shearer has been
after dinner. But some people write ‘sometime’ here, replaced (by Owen), while Owen has been substituted
and this style cannot be considered wrong. When the (for Shearer).

KUNDAN
sense is ‘at an indefinite time in the future’, sometime
236. Suppress, Depress, Oppress
is usual: We’ll talk about this sometime next week. But
the adjective meaning ‘occasional’ or ‘former’ is always I recently read this account of a cricketer: His recent
sometime: his sometime colleague. loss of form has suppressed his average. (Baseball fans
may read this as His recent slump...) What the writer
229. Sped, Speeded intended, of course, was depressed. These three verbs
The past tense and past partciple of the verb speed all have quite different meanings, but the y are
are ‘sped’ when the verb means ‘hurry’: She sped to sometimes confused.
the scene. But it is ‘speeded’ when the meaning is To ‘suppress’ something is to stop it, to prohibit it,
‘drive too fast’: She got a ticket because she had speeded. to prevent it from becoming public or widespread. You
can suppress a smile, a political movement or a book,
230. Stalactite, Stalagmite
for example. To ‘depress’ something is to make it lower:
In a cave, ‘stalagmites’ stick up while ‘stalactites’ hang you can depress a batting average, the share price of
down. If you have trouble with this, think of ants in a stock, or a person’s spirits. To ‘oppress’ people is to
the pants: the mites go up, and the tights go down. treat them harshly when you have power over them:
231. Stationary, Stationery only people can be oppressed.
The adje ctive ‘stationary’ means ‘not moving’: a 237. Suspicious, Suspect
stationary van is a van which is not moving. The noun If you have a dark feeling that some stranger is up to
‘stationery’ means ‘writing materials’, such as paper no good, then you are ‘suspicious’ (of the stranger,
and ink: a stationery van is a van which delivers and of his behaviour), while his behaviour may be
stationery. Do not confuse the two. described either as suspicious or as suspect. All these
232. Stimulant, Stimulus represent standard use, but you might do well to
A ‘stimulant’ is a drug which (technically) increases remember suspect and to use it when you can, since
physiological activity or (informally) makes you more an expression like a suspicious visitor is potentially
alert. Caffeine is a good example. But a ‘stimulus’ is ambiguous: is it the visitor who suspects something,
or is it someone else who suspects the visitor?
313

238. Systematic, Systemic 242. Timings, Hours (of work), Opening hours,
The word ‘systematic’ means ‘orderly and thorough’, office hours
as in a systematic search. But ‘systemic’ me ans At the entrance to an office or at clinics, etc you may
‘pertaining to a system’, as in systemic poison, a poison find a notice headed as ‘Timings’. What follow this
which affects the entire body. heading are the hours during which that office or clinic
is open. To convey such information ‘timing’ is not
239. Tall, High
the right word. You can convey the desired meaning
‘Tall’ refers to heights from base to top, in proportion by using any of the following words or phrases:
to breadth: a tall tree, a tall person. ‘High’ refers to
distance above—usually though not always, above the hours, hours of work, opening hours, or (in the case of
an office) office hours.
ground or the floor: a high hill, a high window, etc.
We use high (opposite low) for measurement of most ‘Timing’ means ‘the act of choosing when something
things (not people), especially when we are thinking happens’ or ‘a particular point or period of time when
of distance above the ground. We use tall (opposite something happens or is planned’. For example,
short) for people, and also for things that are high and The timing of the decision was a complete surprise.
narrow. Please check your flight timings carefully.
He is rather tall for his age.
How high is that mountain? 243. Travel, Journey, Voyage
‘Travel’ means ‘moving from place to place.’ If a person
240. Tell, Say, Speak moves from place to place over a period of time, we
speak of his or her travels. A ‘journey’ is the time spent

K
Our teacher told that we must do five sums every day.
and the distance covered in going from one particular
The given sentence is incorrect because the grammar place to another. A ‘voyage’ has the same meaning
of English requires an indirect object (like me, him, but is only by sea. For example,
her, John, Mary etc) after the verb ‘tell’ in sentences
He returned home after years of travel abroad.
like the one above. And when there is no indirect object
It was a long journey across the desert.
in the situation, we use ‘say’ instead of ‘tell’. The verb The voyage from Bombay to England takes a lot of time.
‘say’ does not necessitate the use of an indirect object
after it. 244. Troop, Troupe
As for the verb ‘speak’, all that is necessary for A ‘troop’ is a group of soldiers or Scouts; a ‘troupe’ is a
remediation purposes is to remember that language group of actors, dancers or other performers.
we ‘speak’, but a word or a sentence, we ‘say’.
Thus, a sentence like ‘He speaks English fluently’ 245. Vacations, Vacation

KUNDAN
is correct, but a sentence like ‘He spoke only two
sentences, is not. In the second sentence, we must W her e ar e you planning t o go dur ing t he next
vacations?
use the verb ‘said’ instead of ‘spoke’.
The right word there is ‘vacation’ (singular), not
241. That, Which, Who
‘vacations’ (plural). In its singular form itself, the word
A relative clause may often be introduced either with conveys the desired meaning, ie, a period of the year
‘that’ or with ‘which’ (for things) or ‘who’ (for people). during which educational institutions and higher
So, you may write either the topic that I want to consider courts are officially closed.
or the topic which I want to consider (or, in a more Of course, the plural form, ‘vacations’ also exists,
informal style, the topic I want to consider). However, it and its use is illustrated below:
is impossible to use that if the relative clause is non- We have three ‘vacations’ in a year – summer
restrictive - that is, if it does not serve to identify the vacation, autumn vacation and winter vacation.
thing under discussion, but only serves to provide
more information about that thing. So, you must write 246. Vicious, Viscous
the Suez Canal, which was opened in 1869, and you The adjective ‘vicious’ means ‘very nasty, violent’, while
cannot write the Suez Canal, that was opened in 1869. ‘viscous’ means ‘very thick and slow-running’. For
Note in particular that a noun denoting a group of example,
people takes which, not who. You cannot write the Thugs are vicious, but honey is viscous.
battalion who had captured the fortress because a
battalion, though composed of people, is not itself a 247. Volcano, Vulcanism
person: write the battalion which had captured the A mountain which explodes is a ‘volcano’, and the
fortress. derived adjective is ‘volcanic’. However, volcanic activity
It is possible to use that with people, but the is ‘vulcanism’, and a scientist who studies volcanoes
result is often rather clumsy. While the linguists that is a ‘vulcanologist’. Why the difference? The forms with
are working on this problem is not quite wrong, it doesn’t ‘O’ are derived from Italian, while those with ‘U’ are
sound as good as the linguists who are working on this derived from Latin.
problem. Prefer who with people.
314

248. Waste, Wastage example, summer, autumn, winter). So, in the above
The word ‘wastage’ is not a fancy equivalent for ‘waste’, sentence, you should use ‘season’ in place of ‘weather’.
and you should not write things like this: Low-flow 251. Wind, Breeze, Gust, Gale
toilets reduce wastage of water. Waste is failure to use
‘Wind’ means ‘a moving current of air’. A ‘breeze’ is
something which could easily be used. But wastage is
usually ‘a pleasant gentle wind’. A ‘gust’ is ‘a strong,
loss resulting from unavoidable natural causes, such
sudden rush of air’, whereas a ‘gale’ is ‘a very strong
as evaporation.
wind’. For example,
249. Wear, Put on Cold wind is blowing.
The verb ‘wear’ indicates a continuous state (eg, wear The morning breeze is good for health.
clothes = have clothes on your body), the verb ‘put on’ A gust of wind blew his hat away.
denotes the action involved. To further clarify this Our chimney was blown down in a gale.
diffe re nce , I cannot do be tte r than quote this
252. Wordiness
explanation from Cobuild Dictionary:
A very common failing is the writing of three or six
“When you put on clothing, you place it on your body
words where one or two will do. It is impossible to list
in order to wear it”.
all the examples, but here are a few of the most
Thus, ‘Which shoes are you going to wear today? is all frequent cases, with suggested replacements:
right, but not: ‘Wear your shoes now’. Instead we say Avoid Write
‘Put on your shoes new’. as to whether whether
We will have noticed by now that the verbs ‘keep’, at the present time now
‘hold’ and ‘wear’ belong to one category (the category at this moment in time at present or now

K
denoting a continuous state), while the verbs ‘put’, because of the fact that because
‘catch’ and ‘put on’ belong to another (the category by virtue of the fact that because
denoting an action). due to the fact that because
250. Weather, Season has a tendency to tends to
in the absence of without
After a long winter, there is a change of weather in in the event that if
February.
in the near future soon
‘Weather’ is the condition of the atmosphere at any in the not too distant future eventually
particular place and time (for example, rainy, hot, prior to before
sultry, cloudy), and it may even change from hour to subsequent to after
hour. ‘Seasons’, on the other hand, represent the the question as to whether whether
periods into which a year is usually divided (for

KUNDAN
Word Power

Word Power
1. MANIAS
Bibliomania an obsession with rare books
Demonomania a morbid delusion of being under the influence of an evil spirit
Dipsomania a morbid compulsion to drink
Dromomania a morbid compulsion for travel
Egomania a morbid obsession with oneself

K KUNDAN
Graphomania a morbid mania for writing
Hypomania a mild manic excitement
Kleptomania a morbid compulsion to steal
Logomania a mania for talking
Macromania a delusion that one’s body has become very big
Micromania a delusion that one’s body has become very small
Megalomania a morbid delusion of one’s grandeur
Monomania single fixed obsession
Nymphomania excessive sexual desire in a female
Pseudomania a mania for making false and exaggerated statements
Pyromania compulsion to set fire for thrill
Scribblomania a mania for making meaningless marks on paper
Theomania a delusion that one has become a god

2. PHOBIAS
Acrophobia Morbid fear of high places or heights
Agoraphobia Morbid fear of wide open places
Algophobia Morbid fear of pain
Anglophobia Morbid fear of England and English things
Anthropophobia Morbid fear of mankind
Astraphobia Morbid fear of lightning, thunder and storms
Astrophobia Morbid fear of celestial space
Ballistophobia Morbid fear of missiles
Bibliophobia Morbid fear of books
Chirophobia Morbid fear of hands
Claustrophobia Morbid fear of closed, confined places
Demophobia Morbid fear of crowds
Dendrophobia Morbid fear of trees

K KUNDAN
Ergophobia
Francophobia
Gynaephobia
Morbid fear of work
Morbid fear of the French
Morbid fear of women
Hematophobia or hemophobia or hemeratophobia Morbid fear of blood
Hippophobia
Hydrophobia
Monophobia
Morbid fear of horses
Morbid fear of water
Morbid fear of being alone
Mysophobia Morbid fear of contamination or germs
Nyctophobia Morbid fear of darkness
Ochlophobia Morbid fear of crowd
Panophobia Morbid fear of everything in general
Pathophobia Morbid fear of disease
Peccatophobia Morbid fear of sinning or having sinned
Pedophobia Morbid fear of children
Phonophobia Morbid fear of speaking aloud
Photophobia Morbid fear of light
Podophobia Morbid fear of feet
Pyrophobia Morbid fear of fire
Russophobia Morbid fear of Russians
Sitophobia Morbid fear of food
Syphilophobia Morbid fear of syphilis
Thanatophobia Morbid fear of death

K KUNDAN
Toxophobia Morbid fear of being poisoned
Trichophobia Morbid fear of hair
Triskaidekaphobia Morbid fear of the number 13
Xenophobia Morbid fear of foreigners
Zoophobia Morbid fear of animals or some particular animal

3. KILLER-WORDS
(A)
1. apicide the killing of bees
2. avicide the killing of birds
3. canicide the killing of dogs
4. germicide the killing of germs
5. herpeticide the killing of snakes
6. insecticide the killing of insects
7. pesticide the killing of pests
(B)
1. filicide the killing of one’s children
2. fratricide the killing of one’s brother
3. genocide the killing of a race of people
4. homicide the killing of human beings
5. infanticide the killing of one’s infant
6. matricide the killing of one’s mother
7. parricide the killing of one’s parents
8. patricide the killing of one’s father
9. populicide the killing of populations
10. regicide the killing of king
11. sororicide the killing of one’s sister

K KUNDAN
12. suicide the killing of oneself
13. uxoricide the killing of one’s wife

4. THE ANGRY MAN


You have, in English language, very many words to express anger in its myriad forms.
1. acrimony bitterness of temper
2. altercation quarrel
3. animosity active enmity
4. antagonism active opposition
5. antipathy constitutional aversion
6. contentiousness quarrelsomeness
7. diatribe piece of bitter criticism
Word Power

8. indignation feeling of surprised anger


9. irascibility angry behaviour
10. tantrum a fit of ill-temper
11. virulence full of hatred
12. wrath indignation; great anger

5. RULES AND FORMS OF GOVERNMENT


You know cracy is an element meaning rule. You find it in the word democracy which means the rule of the people.
There have been many other types of rules such as these:
1. Aristocracy rule by the nobility

K KUNDAN
2. Autocracy rule by absolute monarch
3. Bureaucracy rule by desk officials
4. Gerontocracy rule by old men
5. Gynecocracy rule by women
6. Kakistocracy rule by the worst citizens
7. Mobocracy rule by mob
8. Neocracy rule by new, inexperienced officials
9. Pantisocracy rule (government) by all
10. Plutocracy rule by wealthy people
11. Theocracy rule by church authorities
And look at these, too:
1. Anarchy a state without government
2. Autarchy government of a country by one person with unlimited power
3. Biarchy government by two people
4. Endarchy government from an inner centre of control
5. Monarchy government by a monarch (king)
6. Oligarchy government in the hands of a few
7. Panarchy government by all; universal rule
8. Pentarchy government by five rulers or powers
9. Polyarchy government by many persons
10. Thearchy government by the gods

6. KNOW YOUR DOCTORS


In the field of medicine -ologist denotes a specialist in a specific type of disease. Let us introduce to you ten of these
specialists with the diseases they specialize in.
1. Cardiologist specializes in heart diseases
2. Dermatologist specializes in skin diseases

K KUNDAN
3. Endocrinologist specializes in diseases of glands of internal secretion
4. Gerontologist specializes in diseases of old age
5. Gynaecologist specializes in female diseases
6. Neurologist specializes in nerve diseases
7. Ophthalmologist specializes in eye diseases
8. Otologist specializes in ear diseases
9. Pathologist specializes in detecting the nature and origin of diseases
10. Rhinologist specializes in nose diseases
And these, too:
1. Orthodontist specializes in treating persons with crooked teeth
2. Osteopath specializes in treating persons with bone displacement
3. Paediatrician specializes in children’s diseases
4. Podiatrist specializes in feet ailments
5. Psychiatrist specializes in treating persons with mental aberrations

7. BRANCHES OF SCIENCE AND LEARNING


Science Scientist Subject
Archaeology Archaeologist prehistoric remains
Astronomy Astronomer heavenly bodies
Anthropology Anthropologist mankind
Entomology Entomologist insects
Etymology Etymologist derivation of words
Botany Botanist plant life

K KUNDAN
Biology
Chromatology
Geology
Graphology
Ecology
Embryology
Ethnology
Biologist
Chromatologist
Geologist
Graphologist
Ecologist
Embryologist
Ethnologist
physical life
colours
rocks
handwriting
environment
beginning of life
races of mankind
Lexicography Lexicographer dictionary compiling
Meteorology Meteorologist weather forecast
Numismatics Numismatist coins and medals
Ornithology Ornithologist lives and habits of birds
Phrenology Phrenologist skull and brain
Philology Philologist languages
Pomology Pomologist fruits
Psychology Psychologist the human mind
Radiology Radiologist radioactivity
Paleontology Paleontologist fossils and other forms of early life
Seismology Seismologist earthquakes
Speleology Speleologist caves
Sociology Sociologist societies and human behaviour in groups
Zoology Zoologist different kinds of animals
Phonology Phonologist speech sound of a language or languages
Philately Philatelist stamp collecting
Cartography Cartographer map drawing
Choreography Choreographer ballet designing
Lithography Lithographer stone engraving

K KUNDAN
Topography Topographer physical features of a place through map or chart
Demography Demographer statistics of human population

8. PROFESSIONS
(1)
1. Architect He draws plans for a building.
2. Astronaut A space-traveller
3. Bookmaker He takes bets, especially on horse races, and writes down in his notebook.
4. Balloonist He ascends in a balloon.
5. Chemist He sells medicines.
6. Chauffeur He drives a motorcar.
7. Collier He works in a coal mine.
Word Power

8. Compositor He sets type in a printing press.


9. Curator An official in charge of a museum or art gallery
10. Custodian Caretaker of a public building
(2)
1. Cobbler Mender or maker of shoes
2. Dietician An expert in food and nutrition
3. Draper He sells cloth, fabric.
4. Druggist He sells medicines.
5. Farrier He shoes horses.
6. Fishmonger He deals in fish.

K KUNDAN
7. Flautist
8. Florist
9. Fruiterer
10. Glazier

1. Grazier
2. Hawker
A flute player
A person who grows or sells flowers
A person who deals in fruit
A person who sets glass in windows, doors, etc
(3)
A person who pastures cattle for the market
He travels from place to place selling miscellaneous articles.
3. Invigilator A person who watches over students taking an examination
4. Ironmonger A person who deals in iron or hardware
5. Jockey A professional horse rider in a race
6. Lapidist He cuts precious stones.
7. Librarian A person in charge of a library
8. Milliner A person who makes and sells ladies’ hats, etc
9. Oculist He is an eye-doctor.
10. Optician A specialist in making eyeglasses
And look at these, too:
1. Optometrist He prescribes corrective lenses.
2. Pawnbroker He lends money and keeps goods as security.
3. Physician He attends to sick people and prescribes medicines.
4. Playwright He writes plays.
5. Plumber He repairs water-pipes and cisterns.
6. Potter He makes pots, cups, etc.
7. Scavenger He is a refuge-collector.
8. Sculptor He carves marble or stone.
9. Surveyor He measures land.
10. Technician He is a technical expert.

K KUNDAN
11. Tinker He goes from place to place, mending pots, pans, etc.
12. Tobacconist He sells tobacco and cigarettes.
13. Usurer He lends money at an exorbitant rate of interest.
14. Violinist He plays the violin.

9. TRAITS
(1)
1. Supercilious a person who is overbearing, proud and haughty
2. Misanthrope a person who hates mankind
3. Libertine a person without a moral
4. Impetuous a person who acts suddenly and without thought
5. Pugnacious a person having an inclination to fight
6. Rapacious a person having a grasping nature; greedy
7. Malicious a person who is full of malice and ill will
8. Insidious a person who does harm secretly and craftily
9. Petulant a person unreasonably impatient or irritable
10. Hypocrite a person who is feigning to be what he is not
(2)
1. Extrovert a person more interested in what goes on around him than in his own thoughts and feelings
2. Introvert a person more interested in his own thoughts and feelings than in things outside himself
3. Egoist a person who always thinks of himself
4. Egocentric a person who is self-centred

K KUNDAN
5. Gregarious a person who likes the company of others
6. Judicious a person who possesses a sound judgment
7. Boisterous a person who is noisy and cheerful
8. Taciturn a person who is habitually silent or reserved
9. Erudite a person who is scholarly and learned
10. Punctilious a person who is precise in the observance of forms or ceremonies
(3)
1. Epicurean a person who loves the refinements of pleasure and believes that pleasure is the chief good
2. Stoic a person who is unmoved by joy or grief
3. Atheist a person who denies the existence of God
4. Catholic a person who is broad in outlook
5. Philistine a person of materialistic taste indifferent to art and literature
6. Fastidious a person difficult to please
7. Connoisseur a person who is a critical judge in matters of taste
8. Antiquarian a person interested in the study of ancient things
9. Ascetic a person who practises self-denial and leads an austere life
10. Gourmet a person who has a good taste for food
(4)
1. Optimist He looks at the bright side of life.
2. Pessimist He looks at the dark side of life.
3. Martinet He is a stickler for discipline.
4. Aesthete He has a highly developed sense of beauty.
5. Pedant He makes a display of his learning.
6. Sycophant He is a servile flatterer of the rich and powerful.
7. Hedonist He leads his life purely for pleasure, especially physical pleasure.
8. Infidel He has no belief in a specific religion.

K KUNDAN
9. Suave He is polished in manner.
10. Docile He is quiet and easily controlled managed or influenced.
11. Conservative He is opposed to great or sudden change.
12. Misogynist He hates women.
13. Philanthropist He is kind and helpful to those who are poor or in trouble.
14. Pornographer He treats of sexual subjects in pictures or writing in a way that is meant to cause sexual
excitement.
15. Eccentric He is a person who is peculiar, not normal in behaviour.
16. Misogamist He hates the institution of marriage.
17. Misologist He hates learning and knowledge.
18. Gullible He is easily tricked and persuaded to believe something.
19. Recluse He lives in complete seclusion.
Word Power

20. Termagant She is a noisy, quarrelsome woman.


(5)
1. Imperious He is commanding, haughty and arrogant.
2. Impertinent He is impudent, saucy and he does not show respect.
3. Henpecked He is ruled by his wife.
4. Garrulous He talks too much about unimportant things.
5. Egalitarian He favours the doctrine of equal rights and opportunities for all citizens.
6. Plagiarist He takes and uses somebody else’s ideas, words, etc as if they were his own.
7. Chauvinist He is a person with unreasoning partiality to his place or group.

K KUNDAN
8. Debonair He is suave and light-hearted.
9. Amiable He is easy and pleasant to talk to.
10. Dilettante He studies something but not seriously and not with real understanding: a dabbler in art,
science or literature.

10. ANIMAL TRAITS


Adjectives Description
aquiline eaglelike
asinine asslike
bovine oxlike or cowlike
canine doglike
elephantine elephantlike
equine horselike
feline catlike
leonine lionlike
ovine sheeplike
porcine piglike
serpentine serpentlike
ursine bearlike
vulpine wolflike

11. MAN’S ATTITUDE TO GOD


1. Atheism disbelief in God
2. Agnosticism skepticism about God
3. Deism belief in the existence of a Divine Being, but without acceptance of revelation or religious dogma
4. Henotheism belief in one of a group of gods, without asserting that he is the only God
5. Monotheism belief in one God
6. Pantheism belief that God is nature

K KUNDAN
7. Polytheism belief in many gods
8. Theism belief in the existence of one God, Creator and Ruler of the universe, but without the denial of
revelation

12. APPROPRIATE VERBS


to foment trouble to grab the land
to explode crackers to comb the whole area
to apprehend danger to waste time
to scotch the rumour (put an end to) to squander money
to vitiate the atmosphere (spoil) to fritter away one’s energy
to sling mud at someone to exact an exorbitant rate of interest
to fling remarks
to tarnish somebody’s image to flex the muscle (bend and move)
to pry into somebody’s affairs to mollify one’s anger
to nab the criminal (arrest) to stoop low
to remand one to jail custody to foil an attempt
to amass wealth to pacify the angry mob
to hoard grains to gnash one’s teeth
to pool one’s resources to apportion blame (to divide and share out)
to court danger
to ascertain the fact to set the bones
to curry somebody’s favour (win) to amputate an arm (cut off)
to lull a baby to sleep to extract the tooth

K KUNDAN
to sing a lullaby to convene a meeting
to refurbish the image (to make to deliver a fiery speech
bright and fresh again) to stage a demonstration
to tilt the balance to take out a procession
to nurse a grudge to break the police cordon
to harbour ill will to pelt stones
to rectify a mistake (put right) to fast unto death
to strike terror to go on strike
to defy authority (disobey) to call off the strike
to struggle against odds to control the unruly mob
to climb stairs to cast vote
to scale the wall to unfurl or hoist the flag
to mop the floor to bare one’s bosom
to dust the books to expunge an objectionable remark
to twirl the moustache to box the ears
to cast aspersion on someone to acknowledge someone’s claim
to lob the ball to concede the demand (admit as true)
to spoon a catch to choose a career
to assign reason to embark on an adventure
to cite precedent to draw up a plan
to concoct a story to execute a plan
to offer pleas to toss the coin
to confirm an appointment to spill every ounce of one’s energy
to ratify a treaty to flash a news
to inflict a wound to leap with joy
to dress the wound to bask in the sunshine
to feel the pulse to commit a crime

K KUNDAN
to administer oxygen to commit to memory
to transfuse blood to stitch clothes
to graft the skin to sew on the buttons
to transplant the heart to play the violin
to draw an inference to invoke the gods
to play the host to address the chair
to lay the table to set fire to something
to roast meat to set something on fire
to fry fish to kindle a fire
to bake bread to propound a theory
to cook rice to promulgate an ordinance
to whet one’s appetite to communicate news
Word Power

to satiate one’s hunger to channelize one’s energy


to quench one’s thirst to wring wet clothes
to deploy troops to distort facts
to demolish an unauthorised to disfigure one’s face
structure to sever connections
to desecrate a place of worship to disrupt a meeting
(to use in an unworthy way) to lynch a rapist
to extort money to launch a rocket
to grease somebody’s palm to torment a person
to flaunt one’s riches (show off) to oppress the poor

K KUNDAN
to eradicate crime, poverty
to walk the distance
to discard a worn garment
to revoke a will
to abjure one’s religion
to suppress a rebellion
to restrain an action
to malign an innocent person
to respond to kindness
to rivet one’s attention (to attract
and hold strongly)
to extort a confession from someone
to generate heat, electricity
to unfold a plan
to fulfil a promise to air one’s grievances
to oppose a measure to peel off an orange
to resist an attack to weave threads together
to forge new friendship to foretell the future events
to renew old contact to propitiate the gods (win the favour of)
to bury the dead to encourage the timid
to spit blood, venom to succour the endangered
to delineate a character to support the weak
to brag about something to repulse an attack

13. COMMONLY USED VERBS


A particular noun needs a particular verb to express what is done to it. As for example
do research, take an exam, give a talk, play a game, make an effort, perform a task, have a rest.
The verbs commonly used with particular nouns are plentiful. You will do well to know them in their right combina-
tions. Here are a few common expressions:

You can have


breakfast a walk a good time
lunch a ride a nice evening
tea a game of cards a bad day

K KUNDAN
dinner a rest a thought
coffee a lie-down an idea
a meal a sleep a talk
a drink a dream a chat
a glass of juice a headache a conversation
a party an illness a row
a shave a fit a quarrel
a bath an accident a fight
a wash an operation a word with somebody
a shower a holiday a baby (give birth)
a swim a day off a nervous breakdown
a try a go a setback
You can make
an accusation a journey
an arrangement love
an attempt a meal (prepare a meal)
a boat a mistake
a cake money
a change a movement
a comment a noise
a decision an offer
a demand peace

K KUNDANan effort
an estimation
an exception
an excuse
a fuss
a gesture
a guess
a plan
progress
a profit
a promise
a recommendation
a request
a statement
an impression (on someone) war

You can take


action a liking to
advantage (of something or someone) a look
a bath medical/legal advice
care my word for it (believe me)
charge (of) notice (of something)
a chance (on something) an objection to
courage offence (at something)
a deep breath (great) pains over something
a degree (obtain) a pill
driving lessons a quick look round
an exception to responsibility (for)
a fancy to risks
fright (at something) a seat
heart something to heart
hold (of something) a taxi
a holiday things easy

K KUNDAN
liberties with your time over something (not hurry)

You can do
your best
business (with someone) (someone) a good turn/harm
a course (of study) your homework
some damage the housework
a dance a job
the washing-up research
your duty the shopping
(someone) a favour (some) work
the gardening your hair
Word Power

good us the honour

You can give


(someone) a chance a performance
a chase to permission
a command an opinion
details an order
evidence (somebody) a ring (telephone him)
a groan a talk/lecture

K KUNDAN
information way
a party

You can play You can perform A person or


something can go
cards a duty bald
fair (fairly) a function blind
a game an operation crazy
the game (observe the rules of the game) a piece of music grey
the man (act like a man) a play mad
a musical instrument tricks rusty (iron)
(some) music sour (milk)
a part stale (bread)
prank (on someone)
a record (cassette, tape, etc) wrong (machine)
a role
a trick (on somebody)
a tune
Different Verbs for Different Actions
A train moves off from the platform. A boat puts off from the shore.
A tree sheds its leaves. A bird casts off its feathers.
You scrub the floor. You polish the car.
You shake your head. You shrug your shoulders.
You waste time. BUT You squander money.
You sharpen the knife. You quicken your steps.
You unravel a secret. You unearth a buried treasure.
You cook rice. You bake bread.

K KUNDAN
You sit for/take an exam You mark/set an exam
(if you’re a student). (if you’re a teacher)
You pare the nails. You clip the moustache.
Trains rumble. Wheels rattle.

14. IDIOMATIC NOUN PHRASES


(a) Collection of people
an army of soldiers a congress of representatives, delegates
an assembly of representatives a band of musicians, followers
a contingent of army personnels, boy scouts a batch of pupils, candidates
a corporation of people a battalion of soldiers
a corps of volunteers, soldiers a bench of judges
a council of ministers, advisers a bevy of girls, ladies
a crew of sailors a board of directors, trustees, examiners
a crowd of people a body of men, soldiers, police, laws, etc
a flurry of attentive callers a gang of robbers, thieves, convicts, prisoners
a brigade of cavalry, infantry, artillery a gathering of people
a caravan of pilgrims, merchants a guild of tradesmen, artisans
a choir of singers a horde of barbarians, savages
a circle of friends, acquaintances a host of people
a class of persons, students a mob of rioters
a clique of schemers a multitude of people

K KUNDAN
a colony of people a muster of troops
a company of actors, merchants a panel of judges, jurymen
a concourse of people a pack of fools, knaves
a conference of preachers, delegates a party of musicians, people
a congregation of pilgrims, worshippers a platoon of soldiers, musketeers
a posse of policemen a procession of people
a school of thinkers, learned men a queue of people
a syndicate of merchants, businessmen a regiment of soldiers
a senate of councillors, university members a team of payers
a throng of people a squad of soldiers drilling
a train of followers a squadron of soldiers, ships
a tribe of aborigines a staff of teachers, officials, servants
a troop of soldiers a stream of people, visitors
a troupe of dancers, artistes a string of coolies
a union of workers, tradesmen
(b) Collection of animals, birds and insects
an army of ants a leash of hares, hounds
a bevy of swans a litter of puppies, kittens, piglings
a brood of chickens (brought forth at one birth)
a colony of termites a muster of peacocks
a column of ants a nest of ants
a covey of partridges (flying together) a pack of asses, hounds, wolves
a school of whales, porpoises a drove of cattle (when driven)
a shoal of herrings, fishes a flight of birds, locusts, insects
a string of camels a flock of sheep, geese, chickens
a stud of ponies, horses a gaggle of geese

K KUNDAN
a swarm of flies, ants, bees, locusts a haul of fishes (taken in a net)
a team of horses, oxen a herd of deer, swine, cattle (when pasturing or driven together)
a train of donkeys a troop of lions, monkeys
a hive of bees
(c) Collection of things
an album of photos, snapshots, a barrage of questions
stamps a basket of fruits
an alliance of states, powers, etc a batch of loaves (baked together)
an anthology of poems a battery of guns, lights, cameras
an archive of public records a beam of rays
an assortment of jobs a block of flats, houses
a bale of cotton, wool a bouquet of flowers
Word Power

a bout of illness a forest of trees


a budget of letters, news a galaxy of stars
a bunch of flowers, grapes, plan- a gallery of pictures
tains, keys a garland of flowers
a cache of arms a glut of injuries
a catalogue of books (ie their names) a group of islands
a grove of trees a cargo of wheat
a heap of stones, ruins, sand a carillon of bells
a hoard of jewels, gold a catch of fishes (taken in a net)
a host of subjects a cavalcade of memories

K KUNDAN
a jumble of things (put together without any order)
a chain of mountains, events
a chest of drawers
a cloud of dust
a clump of trees
a cluster of stars, islands, nuts
a clutch of eggs
a cellar of wine
a league of powers, states, nations
a library of books
a lump of mud, coal, lead
a mass of cloud, ruins
a maze of tanglewood, worldly affairs
a code of laws
a miscellany of composition (on various subjects) a column of smoke
a commission of enquiry a mop of grey hair
a commonwealth of nations a mine of information
a confederation of states, powers a museum of art
a consignment of goods a nosegay of flowers
a constellation of stars an outfit of clothes
a convoy of helicopters, lorries a pack of playing-cards
a course of lectures, events a packet of cigarettes
a crop of weeds, hair a peal of thunder, bells
a curriculum of studies a pencil of rays
a faggot of twigs, sticks a pile of arms, woods, books
a fall of leaves, snow, rain a pool of blood
a family of plants, languages a quiver of arrows
a fleet of cars, ships a range of mountains, hills
a flight of steps, stairs a riot of colours
a flotilla of boats, ships a roll of names
a rosary of beads a stockpile of nuclear arms, gunpowder
a round of shots, duties, applause a row of chairs, trees
a storm of protest a sea of troubles
a stretch of wheat field a series of events, lectures

K KUNDAN
a string of Pearls, beads
a suite of rooms, apartments, furniture
a shock of hair
a shower of arrows, abuses, rains, bullets
a train of wagons
a treasure trove of sunken ships
a tuft of grass, hair, feathers
a set of rules, tools
a sheaf of arrows, cornstalks
a syllabus of studies
a tissue of lies
a species of animals, plants
a stack of arms, hay, wood (piled together)
a volley of questions, abuses, shots
a stock of goods a whirlpool of misery

15. MASCULINE AND FEMININE


1. People
Masculine Feminine Masculine Feminine
actor actress king queen
author authoress landlord landlady
bachelor spinster lord lady
boy girl male female
bridegroom bride man woman
brother sister manservant maidservant
brother-in-law sister-in-law master mistress
conductor conductress monk nun
count countess negro negress
duke duchess nephew niece

K KUNDAN
earl countess poet poetess
emperor empress priest priestess
father mother prince princess
father-in-law mother-in-law proprietor proprietress
fiance fiancee salesman salesgirl
friar nun shepherd shepherdess
gentleman lady sir madam
god goddess son daughter
godfather godmother son-in-law daughter-in-law
grandfather grandmother stepfather stepmother
headmaster headmistress steward stewardess
heir heiress tutor governess
hero heroine waitor waitress
host hostess widower widow
husband wife wizard witch
instructor instructress
2. Animals
bear she-bear bullock heifer
billy-goat nanny-goat cock hen
boar sow cock-sparrow hen-sparrow
buck doe colt filly
bull cow dog bitch
drake duck peacock peahen
fox vixen ram ewe
gander goose stallion mare
he-goat she-goat steer heifer
leopard leopardess tiger tigress

K KUNDAN
lion

Adult
bear
cat
lioness

Young one
cub
kitten
tom-cat

16. YOUNG ONES


1. Animals
Adult
hen
horse
tabby-cat

Young one
chicken, chick
colt, filly, foal
cow calf leopard cub
deer fawn lion whelp
dog puppy ram lamb
fox cub sheep lamb
frog tadpole stag fawn
Word Power

goat kid tiger cub


hare leveret wolf cub
2. Birds, Fish and Insects
ant grub housefly maggot
bee grub mosquito wriggler, larva
butterfly caterpillar moth caterpillar
cockroach nymph owl owlet
duck duckling salmon parr
eagle eaglet swan cygnet

K KUNDAN
goose gosling trout fry

17. WORDS DENOTING MOVEMENT, PLACES AND HOMES


1. Movement
(a) Birds (b) Animals
A bird flies. A bear lumbers.
A cock struts. A deer bounds.
A crow flies. A donkey trots.
A duck waddles. An elephant ambles.
An eagle swoops. A horse gallops, trots.
A lark soars. A hound bounds.
An owl flits. A lamb frisks.
A sparrow flits. A lion prowls.
A turkey struts. A mouse scampers.
A vulture flaps, rises or hobbles. A rabbit leaps.
A wolf lopes.
(c) Persons
A person
hobbles, dawdles, limps, lumbers, meanders,
marches, plods, shuffles, paces, slouches,
staggers, strides, strolls, waddles, walks.
(d) Walking Verbs
We are not tagged to the calendar, tethered to the post, stuck up at just one place. Every single moment, all the time, we
rather step out of our house to see what the world is like. We do move from place to place. In search of job. In pursuit of an ideal.
In quest of peace.
We walk at varying paces and in a variety of ways. As for example, a baby toddles, an idler slouches, a soldier marches,
and a farmer, after the day’s toil, plods his weary way home. And so on.

K KUNDAN
You will find here two sets of verbs: walking verbs and verbs of movement.
Walking Verb
1. To toddle
2. To limp
3. To strut
4. To stride
5. To stroll
Definition
to walk with short tottering steps (as does a baby)
to walk as if lame
to walk in an affected manner/proudly
to walk with long steps
to walk in a quiet, unhurried way
6. To slouch to walk in a lazy, tired way
7. To stagger to walk or move unsteadily (from weakness, a heavy burden, drunkenness, etc)
8. To plod to walk slowly and wearily
9. To dawdle to walk slowly, wasting time
10. To lumber to move in a heavy, clumsy, noisy way
11. To march to walk smartly in steps
12. To meander to follow a winding path, moving slowly
and gently (eg meandering river)
Now look at these verbs of movement.
1. Swoop come down with a rush (like the bird of prey)
2. Skim glide over with an occasional touch
3. Scamper run quickly like a frightened animal
4. Sprint run a short distance with speed
5. Bolt run away quickly, dart off

K KUNDAN
6. Bound move or run in jumping movements
7. Flash move so swiftly as to be visible for a short time
8. Flit fly or move lightly and quickly (Bees flit from flower to flower.)
9. Hasten move with speed
10. Run move with quick steps (faster than walking)
11. Rush go with speed
Look at these, too.
1. Tremor shaking movement of the ground
2. Shiver movement of body due to cold
3. Vibrate regular movement backwards and forwards
4. Oscillate regular movement of a suspended thing from one point to another
5. Flicker movement of the flame
(e) Specific movement words
A baby toddles. A tired traveller plods.
A king strides on. A lame person limps or hobbles.
A drunkard staggers. A conceited fellow struts.
(f) Miscellaneous
The nose blows. The wind blows.
The earth rotates. Water flows.
The watch ticks. The fan whirls.
The wound bleeds. Arrows shoot.
The train runs. The machine works.
2. Places
(a) Where things are made
Beer in a brewery Bread, cakes in a bakery

K KUNDAN
Films in a studio Flour or paper in a mill
Goods of all kinds in a factory Iron implements in a smithy
Leather in a tannery Milk products in a dairy
Money (coins) in a mint Ships in a dockyard
Alcoholic drinks in a distillery
(b) Where games are played
Badminton on a court Boxing in a ring
Billiards on a cloth-covered
table (a billiard table)
Cricket on a pitch Chess on a chessboard
Football on a ground, field Hockey on a ground, field, astroturf
Running on a track, course Skating in a rink
Word Power

Tennis on a court Water polo in a swimming pool


Wrestling in an arena
(c) Where persons, things, animals, etc are kept, grown, exhibited, stored
Aeroplanes are kept in a hangar.
Athletic competitions are held in a stadium.
Athletic exercises are performed in a gymnasium.
Astronomical observations are made in an observatory.
Bees are kept in an apiary.
Birds are kept in a cage, an aviary.

K KUNDAN
Birds and animals are exhibited in a zoo.
Clothes are kept in a wardrobe.
Clothes or linen are washed and ironed in a laundry.
Dead bodies are buried in a cemetery.
Dead bodies are cremated in a crematorium.
Fishes are kept in an aquarium.
Fruit trees are grown in an orchard.
Gas is stored in a cylinder.
Goods are stored in a depot.
Grains are stored in a granary.
Grapes are grown in a vineyard.
Guns are kept in an armoury.
Historical relics or curios are kept in a museum.
Luggage at a railway station is kept in a cloakroom.
Medicines are compounded in a dispensary.
Motorcars are kept in a garage.
Plates, dishes, pots and other cooking utensils are washed in a scullery.
Scientific experiments are conducted in a laboratory.
Sick people are treated in a hospital.
Ships are built or repaired in a dock.
Ships are loaded or unloaded in a quay.
Water is collected and stored in a reservoir.
Wild animals are kept in a menagerie, zoo.
Wine and other provisions are stored (underground) in a cellar.
Young plants grow in a nursery.
3. Homes
(a) Persons (b) Creatures

K KUNDAN
An Arab in a dowar Bees in a hive
A convict in a prison A bird in a nest
An Eskimo in an igloo A cow in a byre, pen
A gypsy in a caravan A dog in a kennel
A hermit in a hermitage An eagle in an eyrie
A king or a queen in a palace A fox in a hole
A knight in a mansion A fowl in a coop
A lumberman in a log cabin A hare in a burrow
A lunatic in an asylum A horse in a stable
A monk in a monastery A lion in a den
A nobleman in a castle A mouse in a hole
A nun in a convent, nunnery An owl in a barn, on a tree
A parson in a parsonage A pig in a sty
A peasant in a cottage A pigeon in a dovecote
A priest in a presbytery A rabbit (tame) in a hutch
A prisoner in a cell, jail A rabbit (wild) in a burrow, warren
A Red Indian in a wigwam, tepee A sheep in a pen, fold
A soldier in barracks, camp A spider in a web
A Swiss in a chalet A squirrel in a drey
A vagrant on the pavement A tiger in a lair
A Zulu in a kraal A wasp in a nest
Wild animals in den, lair

18. WORDS DENOTING VARIOUS SOUNDS

K KUNDAN
(a) Animals
Apes gibber
1. Cries of Animals, Birds and Insects

Dogs bark (at somebody)


yelp or whine (in pain)
snarl or growl
(in anger) bay (at the moon)
howl (in distress)
Asses bray
Bulls (Oxen) bellow
Calves low
Camels grunt Donkeys bray
Cats mew, purr, caterwaul (when Elephants trumpet
they quarrel) Foxes yelp, bark
Cattle low Frogs croak
Cows low, moo Goats bleat
Crocodiles grunt Hogs grunt
Horses neigh, snort, whinny Oxen low, bellow
Hounds bay Pigs grunt, squeak
Hyenas laugh Puppies yelp
Jackals howl Rabbits squeak
Kittens mew Serpents hiss
Lambs bleat Sheep bleat
Lions roar Snakes hiss
Mice squeak Tigers growl, roar
Monkeys chatter, gibber Wolves howl, yelp

K KUNDAN
(b) Birds
Birds twitter, chirp, carol, sing, Owls hoot, screech, scream
warble, chirrup Parrots screech, chatter, talk
Cocks crow Pea-fowls scream
Crows caw Pigeons coo
Cuckoos coo Ravens croak
Doves coo Rooks caw
Ducks quack Sea-gulls scream
Eagles scream Sparrows chirp, twitter
Geese cackle, gobble, hiss (in anger) Squirrels squeak
Hawks scream Swallows twitter
Hens cackle, cluck, chuckle Swans cry
Word Power

Kites scream Thrushes whistle


Larks sing, warble Turkeys gobble
Magpies chatter Vultures scream
Nightingales sing, warble
(c) Insects
Bees hum, buzz, murmur Crickets chirp
Beetles drone Flies buzz
2. Miscellaneous Sounds
(A)

K KUNDAN
the babbling of a brook
the beat of a drum
the blaring of loudspeaker, trumpet
the booming of cannons, guns
the buzz of a telephone, conversation
the chiming of bells, clock
the clinking of coins, sword
the churning of sea water
the clanging of arms
the clangour of a bell, a hammer, a trumpet
the clanking of chains
the clatter of hoofs, plates, spoons, knives
the clicking of knitting needles
the rumbling of heavy vehicles, cloud
the crack of a whip the crackling of fire, dry leaves, wood
the rustling of leaves, clothes, wind, paper the crash of shells
the shuffling of feet the crinkling of paper
the sighing of breeze the gnashing of teeth
the slam of a door the gong of bells
the splashing of water, oars the grating of machines
the swishing of cane, ladies’ skirts the gurgling of water
the tapping at the bolted door the hissing of steam
the thudding of cannons, coconuts the jingling of coins
the thundering of clouds the lapping of water, waves
the tick of a clock the pattering of rain
the tinkling of bracelets, glasses the peal of church bells, thunder, laughter
the tolling of bells the tooting of a horn
the popping of corks the twanging of bowstrings
the prattling of a brook the whirring of an aeroplane, wings, computers
the purring of motor cars the rattling of wheels, cart, carriage
the whistling of wind the whizzing of arrows, motorcars
the rippling of water, rain the zooming of aeroplanes
the roaring of thunder, cannon,
waves, laughter

K KUNDAN
(B)
Arms clang
Babies lisp
Bells ring, chime, peal, jingle, tinkle, toll

Brakes screech, rasp


Bugles blow
Keys jingle
Leaves rustle
Railway engines hoot, whistle, roar
Rain patters
Reeds whisper
Shoes creak
Bullets whizz Silk rustles
Chains clank Steel clinks
Coins jingle, tinkle Streams babble
Doors creak Teeth chatter
Drums beat Thunder roars, rumbles, rolls
Footsteps sound Trumpets blare
Guns boom, explode, roar Water ripples
Hoofs clatter Wind whistles, sighs

19. NATION AND NATIONALITY


(Nationality Words)
(1)
Country Person Nation Adjective

K KUNDAN
Belgium a Belgian the Belgians Belgian
Bulgaria a Bulgarian the Bulgarians Bulgarian
Germany a German the Germans German
Italy an Italian the Italians Italian
Mexico a Mexican the Mexicans Mexican
Morocco a Moroccan the Moroccans Moroccan
Nigeria a Nigerian the Nigerians Nigerian
Norway a Norwegian the Norwegians Norwegian
Russia a Russian the Russians Russian
USA an American the Americans American
Czechoslovakia a Czech the Czechs Czech
Greece a Greek the Greeks Greek
Thailand a Thai the Thais Thai
Burma a Burmese the Burmese Burmese
China a Chinese the Chinese Chinese
Guyana a Guyanese the Guyanese Guyanese
Japan a Japanese the Japanese Japanese
Lebanon a Lebanese the Lebanese Lebanese
Portugal a Portuguese the Portuguese Portuguese
Switzerland a Swiss the Swiss Swiss
(2)
Britain a Briton/Britisher the British British
Denmark a Dane the Danes Danish
England an Englishman/ the English English
Englishwoman
Finland a Finn the Finns Finnish
France a Frenchman/ the French French

K KUNDAN
Holland/the
Netherlands
Ireland

Poland
Scotland
Frenchwoman
a Dutchman/
Dutchwoman
an Irishman/
Irishwoman
a Pole
a Scot
the Dutch

the Irish

the Poles
the Scots
Dutch

Irish

Polish
Scottish
Spain a Spaniard the Spanish Spanish
Sweden a Swede the Swede Swedish
Turkey a Turk the Turks Turkish
Wales a Welshman/Welshwoman the Welsh Welsh
Word Power

20. PLURAL FORMS OF CERTAIN NOUNS


When you speak of one thing, the noun is singular. And when you refer to two or more things, the noun is plural.
Most nouns form their plural by adding -s.
Singular Plural Singular Plural
boy boys chair chairs
hand hands face faces
packet packets
If the singular noun ends in s, ch, sh, x, or z, add -es.
Singular Plural Singular Plural
bus buses box boxes

K KUNDAN
torch
brush

baby
lady
torches
brushes

babies
ladies
buzz

Some singular nouns ending in y make plurals by changing the y into -ies.
Singular Plural Singular
party
story
If the singular noun ends in f or fe, we generally change the f or fe into ves.
buzzes

Plural
parties
stories

Singular Plural Singular Plural


calf calves life lives
leaf leaves self selves
loaf loaves shelf shelves
thief thieves sheaf sheaves
wife wives wolf wolves
But we say roof—roofs; chief—chiefs; safe—safes; belief—beliefs, handkerchief—handkerchiefs, cliff—cliffs.
Some nouns ending in o have plurals in -es.
Singular Plural Singular Plural
echo echoes potato potatoes
hero heroes tomato tomatoes
mosquito mosquitoes
Some plurals are irregular.
Singular Plural Singular Plural
foot feet goose geese
child children tooth teeth
mouse mice
Some nouns do not change in the plural.
deer, grouse, sheep, swine, shad
The following is the list of the plural form of some of the problem words:

K KUNDAN
Singular
addendum
alga
alumnus
analysis
antenna
apex
Plural
addenda
algae
alumni
analyses
antennae
apexes, apices
Singular
forum
fresco
fulcrum
fungus
genesis
genius
Plural
forums, fora
frescoes
fulcrums, fulcra
fungi
geneses
geniuses
appendix appendices (books, genus genera
theses, etc.) half halves
appendixes (in hoof hooves, hoofs
anatomy) hypothesis hypotheses
aquarium aquaria, aquariums impetus impetuses
axis axes index indexes (to books,
bacillus bacilli theses, etc)
bacterium bacteria indices (in Maths)
bamboo bamboos kilo kilos
basis bases knife knives
beau beaux lacuna lacunae, lacunas
bonus bonuses larva larvae
cactus cacti maestro maestros, maestri
corps corps matrix matrices
corrigendum corrigenda maximum maxima

K KUNDAN
crematorium crematoria medium media
crisis crises memorandum memoranda
criterion criteria menu menus
crux cruces minimum minima
curio curios momentum momenta, momentums
datum data
desideratum desiderata mother-in-law mothers-in-law
dictum dicta narcissus narcissi, narcissuses
dynamo dynamos
emporium emporia nebula nebulae
erratum errata nucleus nuclei
facsimile facsimiles oasis oases
focus focuses, foci (in sci- phenomenon phenomena
entific context) plateau plateaux, plateaus
formula formulae, formulas prospectus prospectuses
quiz quizzes syllabus syllabuses, syllabi
quorum quorums symposium symposia
quota quotas tableau tableaux
radio radios terminus termini, terminuses
radius radii thesis theses
sanatorium sanatoria, tornado tornadoes
sanatoriums torpedo torpedoes
serf serfs trauma traumata, traumas
series series ultimatum ultimatums, ultimata
serum sera
sheaf sheaves vacuum vacuums, vacua (in
solo solos scientific context)

K KUNDAN
species species
spectrum spectra vertebra vertebrae
sphinx sphinxes veto vetoes
stadium stadiums, stadia virtuoso virtuosi
stimulus stimuli virus viruses
stratum strata vista vistas
substratum substrata volcano volcanoes
vortex vortices, vortexes

21. THEY GO IN PAIRS


In idiomatic phrases there are certain words that always go in pairs, as for example, kith and kin, root and branch.
The right words are to be placed in the right order. You just can’t alter the order of words. It will be silly of you if you say, kin
and kith, or branch and root.
Word Power

Here is a list of words that generally go in pairs:


bag and baggage friends and foes loaves and fishes
black and white gods and goddesses lock and key
black and blue gold and silver men and women
boys and girls hale and hearty part and parcel
bread and butter heart and soul prince and princess
carrots and radishes hearth and home questions and answers
cats and dogs here and there root and branch
cups and saucers high and low rack and ruin

K KUNDAN
dark and dreary horse and carriage slow and steady
doctors and nurses hot and cold sons and daughters
doors and windows hue and cry spick and span
duke and duchess ins and outs stout and strong
dull and drab joys and sorrows sum and substance
East and West knives and forks tooth and nail
fair and square ladies and gentlemen up and down
far and wide law and order ups and downs
fat and flabby lean and thin weal and woe

22. BITS AND PIECES OF THINGS


In English there are many words to talk about a piece of something. The substance may be solid (a piece of wood),
may not be solid (a drop of wine), may be solid but quite thin (a slice of bread), or solid but small (a grain of salt), or solid but
not so small (a block of stone).
Here is a list of such words. It will let you know what substances these words refer to.
a bar of chocolate/soap a lump of coal/earth/clay
a bit of bread/paper/clay/wood a patch of cloud/grass
a block of stone/wood a piece of bread/paper/wood/clay
a chip/flake/splinter/sliver of stone/wood a puff/wisp of smoke
a rasher of bacon a chunk/hunk of bread/meat
a ray of light a clod of earth
a scrap of cloth/paper a clot of blood
a scrap/shred of paper/cloth a crumb of bread/cake
a segment of orange a dash/squirt/squeeze of lemon juice/oil
a shred of cloth/paper a slab of stone/cheese
a drip of water a slice of bread/cake
a drop of water/wine/oil a splinter/sliver of glass
an ear of corn a square of chocolate

K KUNDAN
a flake of snow a strip of land/lawn/sandy beach
a fragment of matter a tablet of stone
a grain/pinch of salt a tract of land
a grain of sand a wad of cotton

23. DIMINUTIVES
Diminutives are such words that denote small specimen of big things. They are ‘nouns that express smallness, either
actual or imputed, in token of affection or contempt’. The chief suffixes forming diminutives are: et (coronet), let (streamlet) en
(chicken), ock (hillock), ling (sapling).
The list of commonly used diminutives given below will be of immense help to you in the choice of right words.
Word Diminutives Word Diminutives
animal animalcule dear darling
ankle anklet drop droplet
ball ballot eye eyelet
bird birdie flower floweret, floret
book booklet globe globule
brook brooklet hill hillock
car chariot home hamlet
cigar cigarette lance lancet
city citadel lock locket
corn kernel man manikin
crown coronet mouth muzzle
dame damsel nave navel

K KUNDAN
nest nestling seed seedling
nose nozzle shade shadow
part particle star asterisk
pill pillow statue statuette
poet poetaster stream streamlet
puss pussy table tablet
ring ringlet top tip
river rivulet tower turret
room roomette umbrella parasol
rose rosette village hamlet
sack satchel wagon wagonette
scythe sickle weak weakling
Look at these words, too:
reticule a small net hand bag
ridicule small, mocking laughter
molecule a tiny particle of matter
funicle a small cord
fledgling a baby bird; an inexperienced person
gosling a baby goose
princeling a prince of a small country
sapling a very young tree; youth
morsel a very small piece of food
globule a small drop of a liquid or melted solid
infinitesimal so small that it cannot be measured

24. ANNIVERSARY
semicentennial 50th anniversary

K KUNDAN
centennial 100th anniversary
sesquicentennial 150th anniversary
bicentennial 200th anniversary
tricentennial 300th anniversary
tetracentennial 400th anniversary
pentacentennial 500th anniversary
hexacentennial 600th anniversary
Look at these words, too.
They denote the varying periodic occurrence of an event.
biennial occurring once in two years
triennial occurring once in three years
Word Power

quadrennial occurring once in four years


quinquennial occurring once in five years
sexennial occurring once in six years
septennial occurring once in seven years
octennial occurring once in eight years
decennial occurring once in ten years

25. HOW OLD ARE YOU?


I’m 15 years old. I’m an adolescent.
I’m 60 to 69 years old. I’m a sexagenarian.

K KUNDAN
I’m 70 to 79 years old. I’m a septuagenarian.
I’m 80 to 89 years old. I’m an octogenarian.
I’m 90 to 99 years old. I’m a nonagenarian.
I’m 100 years old. I’m a centenarian.

26. COMMONPLACE COMPARISONS


The English language abounds in striking similes in which one thing is likened to another in such a way as to clarify
and enhance an image. As for example, as dead as a doornail, as blind as a bat, and as cool as cucumber. The apt use of such
comparative phrases imparts clarity and vividness to our expressions.
Here is a comprehensive list of comparative phrases of as.........as pattern:
(A)
As active as quicksilver As crisp as new bank notes
As ageless as the sun As crooked as a corkscrew
As agile as a monkey, as a cat As cross as two sticks
As alike as two beans, as two peas As cruel as death, as winter
As ambitious as the devil, as Lady Macbeth As cunning as a fox
As arid as the sands of the Sahara As dark as midnight, as a dungeon, as pitch
As bald as a billiard ball, as a coot, as an egg, as a badger As dead as a doornail, as a herring, as mutton, as a dodo
As bashful as a schoolgirl As deaf as an adder, as a beetle, as a post
As beautiful as a rainbow, as the sunset As dear as life
As big as an elephant, as a whale As deep as the ocean, as a well
As black as a crow, as coal, as ebony, as jet, as pitch, as As desolate as a tomb
midnight, as a raven, as soot, as ink As devoted as a faithful dog
As blind as a bat, as a mole, as a beetle As different as chalk from cheese
As blue as the indigo, as the sky As dirty as ditch-water
As boundless as the ocean As distant as the horizon
As bounteous as nature As dry as a bone, as dust, as a mummy, as a stick

K KUNDAN
As brave as Achilles, as a lion As dumb as a statue
As brief as a dream, as time, as a candle As easy as ABC, as lying
As bright as a button, as day, as light, as a new pin, as silver As eloquent as Cicero
As brittle as glass As elusive as quicksilver
As busy as a bee As empty as space
As cheap as dirt As faithful as a dog
As cheerful as a lark As fast as a hare, as light
As clear as crystal, as daylight, as noonday As fickle as the weather
As cold as charity, as a frog, as stone, as marble, as ice As ferocious as a lion
As cool as a cucumber As fierce as a tiger
As cosy as a bird’s nest As firm as a rock, as steel, as faith
As countless as the desert sands, as the stars As fit as a fiddle
As flat as a billiard table, as a board, as a pancake As pure as a lily
As free as the air, as a bird As red as blood, as crimson, as a rose, as fire, as scarlet
As fresh as a daisy, as a rose, as dew As regular as clockwork
As gay as a lark As sacred as a shrine
As gentle as a dove, as a lamb, as a fawn As sharp as a needle, as a razor
As gloomy as night As simple as ABC
As glorious as the sun As slippery as an eel, as a serpent
As good as gold As slow as a snail
As graceful as a swan As sly as a fox
As greedy as a dog, as a hog, as a wolf As smooth as butter, as oil, as velvet

K KUNDAN
As green as grass As snug as a bug in a rug
As grey as smoke As soft as butter, as fur, as silk, as wax, as wool
As grim as death, as hell As solid as a rock
As hard as flint, as marble, as granite, as nails As solitary as a tomb
As harmless as a babe, as a dove As sound as a bell
As heavy as lead, as sand As spineless as a jelly fish
As helpless as a babe As spotless as snow
As high as heaven, as the stars As steadfast as the sun
As honest as the day As steady as a rock
As hungry as a church mouse, as a wolf As stealthy as a cat
As impatient as a lover As straight as an arrow, as a lance
As industrious as an ant As stubborn as a mule
As inevitable as death As superstitious as sailors
As innocent as a babe, as a dove As sure as death, as fate
As large as life As sweet as honey, as sugar
As light as a feather, as a butterfly, as air As swift as an arrow, as thought, as lightning, as the wind
As lively as a cricket As talkative as a magpie
As mad as a hatter, as a March hare As tall as a maypole, as a steeple
As mean as a miser As tender as a bud, as a lamb, as a chicken
As meek as a dove, as lamb As tough as leather, as nails
As mischievous as a kitten, as a monkey As transparent as glass
As natural as life As ugly as a scarecrow
As new as day As uncertain as the weather
As obstinate as a mule As unchangeable as the past
As old as the hills As vain as a peacock
As pale as death, as a ghost As variable as the weather
As peaceful as sleep As venomous as a snake

K KUNDAN
As piercing as light As wary as a fox
As plain as a pikestaff As watchful as a hawk, as a sentinel
As pretty as a picture As white as snow, as wool
As progressive as time As wily as a fox
As proud as a peacock As wise as Solomon
As punctual as clockwork
27. LOVER WORDS
1. Lover of books Bibliophile
2. Lover of children Paedophile
3. Lover of animals Zoophile /Zoophilist
4. Lover of trees Dendrophile
Word Power

5. Lover of hands Cheirophile


6. Lover of feet Podophile
7. Lover of horses Hippophile
8. Lover of knowledge Bibliologist
9. Lover of words Philologist
10. Lover of nomadic life Nomads
11. Lover of vegetables Vegetarian
12. Lover of meat Carnivore
13. Lover of human flesh Cannibal
14. Lover of grandeur Megalomaniac

K KUNDAN
15. Lover of war
16. Lover of peace
17. Lover of democracy
18. Lover of dictatorship
19. Lover of mobocracy
20. Lover of one’s country
21. Lover of good food
Warmonger
Pacifist, Peace-loving
Democrat
Dictator
Mobocrat
Patriot
Gourmet
22. Lover of good taste in art, painting, etc Connoisseur
23. Lover of travelling round the world Globetrotter
24. Lover of God and religion Theist
25. Lover of alcoholic drink Drunkard
26. Lover of society Sociable
27. Lover of money Mercenary
28. Lover of sports and games Sports fan, Sportsman
29. Lover of mankind Humanitarian, Philanthropist
30. Lover of material things Materialist
31. Lover of self Narcissist

28. HATER WORDS


1. Hater of women Misogynist
2. Hater of mankind Misanthrope
3. Hater of knowledge Misologist
4. Hater of alcoholic drinks Teetotaller
5. Hater of war Pacifist
6. Hater of idol worship Iconoclast
7. Hater of pomp and show Austere
8. Hater of marriage Celebate, Misogamist

K KUNDAN
9. Hater of worldly life Recluse

29. GRAPHY WORDS


Autobiography story of a person’s life written by himself
Biography story of a person’s life written by someone else
Calligraphy elegant handwriting
Chronography a record of past time; history
Demography study of births, deaths, diseases, etc to show the condition of a community
Epistolography the art of letter-writing
Geography science of the earth’s surface, physical features, divisions, climate, products,
population, etc
Homography method of spelling a word which is like another but with a different meaning
Lithography process of printing from parts of a flat stone or sheet of zinc or aluminium that
are prepared to receive a greasy ink
Microcosmography a description of man as a microcosm
Orthography writing with correct penmanship and spelling
Polygraphy literary productiveness; writing on a variety of subjects
Seismography the study of earthquakes
Photography art or process of taking photographs
Topography description of the features, eg rivers, valleys, roads, of a place or district
Telegraphy the technique of sending message by electrical signals
Videography the recording of sound and vision, eg of television programmes by magnetic
tape

K KUNDAN
Xeroxography a scientific method of photostating

K KUNDAN
552 Test of English Language

Chapter 19

Reading Comprehension
Direct ions : Read t he foll owin g passages find it quite useful. Teach yourself to feel that life
carefully and answer the questions given below would still be worth living even if you were not, as of
them. Certain words are given in bold to help you course you are, immeasurably superior to all your
to locate them while answering some of the friends in virtue and in intelligence. Exercises of this
questions. sort prolonged through several years will at last enable
you to admit facts without flinching and will, in so
Passage 1 doing, free you from the empire of fear over a very
The happy man is the man who lives objectively, large field.
Potent = convincing; having great power
who has free affections and wide interests, who
Bestow = to present something as a gift to
secures his happiness through these interests and somebody
affections and through the fact that they in turn make Encase = to surround or cover something
him an object of interest and affection to many others. closely, especially in order to protect
To be the recipient of affection is a potent cause of it.
happiness, but the man who demands affection is not Simulated = artificial, but made to look, feel etc
the man upon whom it is bestowed. The man who like the real thing.
receives affection is, speaking broadly, the man who Merely = only; simply
Nevertheless = in spite of something; however; still
gives it. But it is useless to attempt to give it as a
Persuade = to convince somebody
calculation, in the way in which one might lend money

K KUNDAN
Conviction = a firm opinion or belief ; the
at interest, for a calculated affection is not genuine appearance of being sincere, firmly
and is not felt to be so by the recipient. believed or truly meant
What then can a man do who is unhappy because Dispell = to make something go away
he is encased in self? So long as he continues to Immemorial = of or from a time so long ago that no
think about the causes of his unhappiness, he one can remember it; ancient
continues to be self-centered and therefore does not Prolong = to make something last longer; to
extend something
get outside it. It must be by genuine interest, not by
Flinching = making a s udden automatic
simulated interests adopted merely as a medicine. movement because of pain, fear or
Although this difficulty is real, there is nevertheless shock
much that he can do if he has rightly diagnosed his 1. According to the passage, calculated affection
trouble. If for example, his trouble is due to a sense 1) appears to be false and fabricated
of sin, conscious or unconscious, he can first 2) makes other person to love you
persuade his conscious mind that he has no reason 3) turns into permanent affection over a period
to feel sinful, and then proceed, to plant this rational of time
conviction in his unconscious mind, concerning 4) leads to self-pity
himself meanwhile with some more or less neutral 5) gives a feeling of courage
activity. If he succeeds in dispelling the sense of 2. Who according to the passage is the happy man?
sin, it is possible that genuine objective interests 1) Who is encased in self
will arise spontaneously. If his trouble is self-pity, 2) Who has free affection and wide interests
he can deal with it in the same manner after first 3) Who is free from worldly passions
pe rsuading himse lf that the re is nothing 4) Who has externally centred passions
extraordinarily unfortunate in his circumstances. 5) None of these
If fear is his trouble, let him practise exercises 3. Which of the following statements is NOT TRUE
designed to give courage. Courage has been recognized in the context of the passage?
from time immemorial as an important virtue, and a 1) The happy man has wide interests.
great part of the training of boys and young men has 2) Courage has been recognised as an important
been devoted to producing a type of character capable virtue.
of fearlessness in battle. But moral courage and 3) Unhappy man is encased in self.
intellectual courage have been much less studied. 4) A man who suffers from the sense of sin must
They also, however , have their technique. Admit to tell himself that he has no reason to be
yourself every day at least one painful truth, you will sinful.
Reading Comprehension 553

5) Issue of inte lle ctual courage has be e n 11. Which of the following statements is SIMILAR
extensively studied. in meaning to the word ‘flinching’ as used in
4. Which of the following virtues, according to the the passage?
passage, has been recognised for long as an 1) wincing 2) convincing 3) explaining
important virtue? 4) providing 5) debating
1) Patriotism 2) Sacrifice Wincing = showing pain, distress or
3) Courage 4) Self-consciousness embarras sment by a s light
5) None of these movement of the muscles in the face.
5. Which of the following words is SIMILAR in 12. How can one ge t out of the vicious circle
meaning of the word ‘bestowed’ as used in the mentioned in the passage?
passage? 1) By practising skills of concentration
1) Conferred 2) Accommodated 2) By inculcating the habit of self-absorption
3) Trusted 4) Withdrawn 3) Being true to othe rs and one ’s inte rnal
5) Directed circumstances
6. Which of the following, according to the passage, 4) Admitting to oneself that others could be right
has not been studied much? 5) None of these
To inculcate = to fix ideas, principles, etc firmly in
1) Feeling of guilt and self-pity
somebody’s mind especially by
2) The state of mind of an unhappy man often repeating them
3) How to get absorbed in other interests 13. Which of the following words is OPPOSITE in
4) Moral and intellectual courage meaning of the word ‘dispelling’ as used in the
5) None of these passage?
7. What should a man do who is suffering from the 1) giving 2) accumulating
feeling of self-pity? 3) projecting 4) scattering
1) He should control his passions and emotions. 5) receiving
2) He should persuade himself that everything 14. What according to the passage is the real cause
is alright in his circumstances. of happiness?
3) He should seek affection from others. 1) Material rewards and incentives received
4) He should develop a feeling of fearlessness. 2) Critical analysis of the happy state of mind

K KUNDAN
5) He should consult an expert to diagnose his 3) Affection received from others
trouble. 4) Calculated risk taken
8. What happens to a man who demands affection? 5) None of these
1) His feelings are reciprocated by others. 15. What happens when you think about the cause
2) He tends to take a calculated risk. of your unhappiness?
3) He becomes a victim of a vicious circle. 1) You try to introspect and look critically at
4) He takes affection for granted from others. yourself.
5) None of these 2) You realize that life can be lived in different
Reciprocate = to give and receive something ways.
in return; to make a mutual
3) You try to practice exercise designed to give
exchange of something
Vicious circle = a continuing situation in which
coverage.
one problem or need leads to 4) You remain a self-centered person.
another and the new problem 5) None of these
makes the first problem worse.
9. If a man is suffering from a sense of sin, Passage 2
1) he should invite opinion of others Management is a set of processes that can keep a
2) he should admit his sin at once complicated system of people and technology running
3) he should consciously realize that he has no smoothly. The most important aspects of management
reason to feel sinful include planning, budgeting, organising, staffing,
4) he should develop a fearless character controlling, and problem-solving. Leadership is a set
5) he should develop an internal focus of control of processes that creates organizations in the first
10. Which of the following statements is TRUE in place or adapts them to significantly changing
the context of the passage? circumstances. Leadership defines what the future
1) All passions stem from unhappiness. should look like, aligns people with that vision, and
2) The happy man lives subjectively. inspires them to make it happen despite the obstacles.
3) Any virtue has a dark side also. This distinction is absolutely crucial for our purposes
4) One feels happy if one receives affection. here: Successful transformation is 70 to 90 per cent
5) Any affection is always genuine. leadership and only 10 to 30 per cent management.
To stem from = to have something as its origin Yet for historical reasons, many organizations today
or cause
don’t have much leadership. And almost everyone
554 Test of English Language

thinks about the problems here as one of managing 1) Leaders are reactive whereas managers are
change. proactive.
For most of this century, as we created thousands 2) Organisations are facing problems of not
and thousands of large organizations for the first time getting good managers.
in human history, we didn’t have e nough good 3) Organisations are pursuing the strategy of
managers to keep all those bureaucracies functioning. status quo.
So many companies and unive rsities de veloped 4) In today’s context, organisations need leaders
manage me nt programme s, and hundre ds and much more than managers in transforming
thousands of pe ople were encouraged to le arn them.
management on the job. And they did. But, people 5) None of these
were taught little about leadership. To some degree, Proactive = creating or controlling a situation by
management was emphasized because it’s easier to causing things to happen rather
teach than leadership. But even more so, management than reacting to events.
Stauts quo = the situation or state of affais as it
was the main item on the twentieth-century agenda
is now, or as it was before a recent
be cause that’s what was ne e de d. For e ve ry change
entrepreneur or business builder who was a leader, 2. Why did companies and universities develop
we needed hundreds of managers to run their ever programmes to prepare managers in such a large
growing enterprises. number?
Unfortunately for us today, this emphasis on 1) Companie s and unive rsitie s wante d to
management has often be en institutionalized in generate funds through these programmes.
corporate cultures that discourage employees from 2) A large number of organisations were created
learning how to lead. Ironically, past success is usually and they needed managers in good number.
the key ingredient in producing this outcome. The 3) Organisations did not want spend their scarce
syndrome, as I have observed it on many occasions, resources in training managers.
goes like this: success creates some degree of market 4) Organisations wanted to create communi-
dominance, which in turn produces much growth. After cation network through trained managers.
a while keeping the ever larger organization under 5) None of these
control becomes the primary challenge. So attention 3. Which of the following statements is NOT TRUE

K KUNDAN
turns inward, and managerial compe tencies are in the context of the passage?
nurtured. With a strong emphasis on management 1) Bureaucratic culture can smother those who
but not on leadership, bureaucracy and an inward focus want to respond to changing conditions.
take over. But with continued success, the result 2) Leadership produces change and has the
mostly of market dominance, the problem often goes potential to establish direction.
unaddressed and an unhealthy arrogance begins to 3) Pressure on managers comes mostly from
evolve. All of these characteristics then make any within.
transformation effort much more difficult. 4) Leadership centres on carrying out important
Arrogant managers can over-evaluate their current functions such as planning and problem-
performance and competitive position, listen poorly, solving.
and learn slowly. Inwardly focused employees can have 5) Managers believe that they are the best and
difficulty seeing the very forces that present threats that their idiosyncratic traditions are superior.
and opportunities. Bureaucratic cultures can smother Idiosyncratic = adjective of ‘Idiosyncrasy’.
those who want to respond to shifting conditions. And Idiosyncrasy = a person’s particular way of
the lack of leadership leaves no force inside these thinking, behaving etc that is
organisations to break out of the morass. clearly different from that of
Adapt = to become adjusted to new others.
conditions, etc. 4. Which of the following is not the characteristic
Syndrome = any set of opinions, events, actions of bureaucratic culture?
etc that are characteristic of a 1) Managers listen poorly and learn slowly.
particular condition 2) Managerial competencies are nurtured.
Nurture = to help the devel opment of
3) Employees clearly see the forces that present
something
Smother = to prevent s omething f rom
threats and opportunities.
development or being noticed; to 4) Prevalence of unhealthy arrogance.
suppress 5) Manage rs te nd to stifle initiative and
Mor ass = a situation that is confusing and innovation.
complicated or prevents progress Stifle = to suppress or control something
Respond = to do something as a reaction of 5. Which of the following is SIMILAR in meaning
something to the word SMOTHER as used in the passage?
1. Why, according to the author, is a distinction 1) suppress 2) encourage 3) instigate
between management and leadership crucial? 4) criticise 5) attack
Reading Comprehension 555

6. How has the author defined management? 3) Inspiring people to realise the vision
1) It is the process of adapting organisations to 4) Carrying out the crucial functions of
changing circumstances. management
2) It is the system of aligning people with the 5) None of these
direction it has taken. 13. Which of the following characteristics helps
3) It refers to creating a vision to help direct the organisations in their transformation efforts?
change effort. 1) Emphasis on le ade rship but not on
4) Creating better performance through customer management
orientation. 2) A strong and dogmatic culture
5) None of these 3) Bureaucratic and inward-looking approach
7. Management education was emphasized in the 4) Failing to acknowledge the value of customers
management programmes because and shareholders
1) establishing direction was the main focus of 5) None of these
organisations Dogmatic = insisting that one’s belief’s are right
2) motivating employees was thought to be done and that others should accept them,
by managers without paying attention to evidence
or to other opinions
3) strategies for producing change was the main
14. Why were people taught little about leadership
focus of organisations
in management programmes?
4) organisations wanted to cre ate powe rful
1) Teachers were busy in understanding the
guiding coalition
phenomenon of leadership.
5) management was the main item of agenda in
2) Enough study material was not available to
organisations
facilitate teaching of leadership.
8. What is the historical re ason for many
3) Focus of these programmes was on developing
organisations not having leadership?
managers.
1) A view that leaders are born, they are not made
4) Leadership was considered only a political
2) Le ade rs lack manage rial skills and
phenomenon.
organisations need managers
5) None of these
3) Leaders are weak in carrying out traditional

K KUNDAN
15. Which of the following state me nts is/are
functions of management
definitely true in the context of the passage?
4) Le aders allow too much complace ncy in
(A) Bureaucracy fosters strong and arrogant
organisations
culture.
5) None of these
Complacency = a calm feeling of satisfaction
(B) Leadership competencies are nurtured in
with oneself, one’s work, etc large-size organisations.
9. In the passage, management is equated with (C) Successful transformation in organisations
1) Organisation is 70 to 90 per cent leadership.
2) Leadership 1) Only A and B 2) Only A and C
3) Organisational vision 3) Only B and C 4) Only B
4) Bureaucracy 5) Only C
5) Managerial training Fost er = to help the devel opment of
something; to encourage or promote
10. Why does the attention of large organisations
something
turn inward?
1) Their managers become arrogant. Passage 3
2) They have to keep themselves under control.
3) Their success creates market dominance. In the second week of August 1998, just a few days
4) They want to project their predictability. after the incidents of bombing the US embassies in
5) None of these Nairobi and Dar-es-Salaam, a high-powered, brain-
11. Which of the following is SIMILAR in meaning of storming session was held near Washington D.C., to
the word NURTURED as used in the passage? discuss various aspects of terrorism. The meeting was
1) created 2) developed 3) thwarted attended by ten of America’s leading experts in various
4) surfaced 5) halted fields such as germ and chemical warfare, public
Thwar t = to prevent somebody doing what health, disease control and also by the doctors and
they intend to; to oppose a plan, etc the law-enforcing officers. Being asked to describe the
successfully horror of possible bio-attack, one of the experts
12. What, according to the author, is leadership? narrated the following gloomy scenario.
1) Process which keeps the system of people and A culprit in a crowded business centre or in a busy
technology running smoothly shopping mall of a town empties a test tube containing
2) Planning the future and budgeting resources some fluid, which in turn creates an unseen cloud of
of the organisation germ of a dreaded disease like anthrax capable of
556 Test of English Language

inflicting a horrible death within 5 days on any one Overthrow = to remove somebody/some-
who inhales it. At first 500, or so victims feel that thing from a position of power
they have mild influenza which may recede after a using force
Beyond redemption = too bad to be improved or
day or two. Then the symptoms return again and their
saved
lungs start filling with fluid. They rush to local Ruthless = having or showing no pity or
hospitals for treatment, but the panic-stricken people feeling for others; hard and
may find that the medicare services run quickly out of cruel
drugs due to excessive demand. But no one would be Cacophony = a mixture of loud unpleasant
able to realise that a terrorist attack has occurred. sound
One cannot deny the possibility that the germ involved Perpetrator = a person who commits a crime
would be of contagious variety capable of causing an and does something
considered wrong
epidemic. The meeting concluded that such attacks,
Fanatic = a person who is too enthu-
apart from causing immediate human tragedy, would siastic about something
have dire long-term effects on the political and social Diehard = a person who strongly opposes
fabric of a country by way of ending people’s trust on change and new ideas
the competence of the government. Inflict somebody/something on somebody = to
The experts also said that the bombs used in Kenya make somebody accept
and Tanzania were of the old-fashion variety and something that is unpleasant
involve d quantitie s of high e xplosive s, but new or not welcome
Intent on doing
terrorism will prove to be more deadly and probably
something = having the s pecified f irm
more elusive than hijacking an aeroplane or a gelignite intention; determined to do
of previous decades. According to Bruce Hoffman, an something
American specialist on political violence, old terrorism 1. In the context of the passage, the culprit’s act
generally had a specific manifesto - to overthrow a of emptying a test tube containing some fluid
colonial power or the capitalist system and so on. can be classified as
These terrorists were not shy about planting a bomb 1) a terrorist attack
or hijacking an aircraft and they set some limit to 2) an epidemic of a dreaded disease
their brutality. Killing so many innocent people might 3) a natural calamity

K KUNDAN
turn their natural supporters off. Political terrorists 4) panic created by an imaginary event
want a lot of people watching but not a lot of people 5) None of these
dead. “Old terrorism sought to change the world while Dreaded = greatly feared
the new sort is often practised by those who believe Calamity = an event that causes great harm or
that the world has gone beyond redemption”, he added. damage; a disaster
Hoffman says, “New terrorism has no long-term 2. In what way would the new terrorism be different
agenda but is ruthless in its short-term intentions. from that of the earlier years?
It is often just a cacophonous cry of protest or an (A) More dangerous and less baffling
outburst of religious intolerance or a protest against (B) More hazardous for victims
the West in general and the US in particular. Its (C) Less complicated for terrorists
perpetrators may be religious fanatics or diehard 1) A and C only
opponent of a government and see no reason to show 2) B and C only
restraint. They are simply intent on inflicting the 3) A and B only
maximum amount of pain on the victim.” 4) All the three
Chemical warfare = the use of poisonous gases 5) None of these
and other harmful chemicals as Baffle = to be too difficult or strange for
weapons in war somebody to understand, solve or
Gloomy = making somebody feel sad and explain
depressed Hazar dous = dangerous; risky
Recede = to move backward f rom a 3. What was the immediate provocation for the
previous position or away from meeting held in August 1998?
an observer, or to appear to do 1) the insistence of America’s leading
this
2) the horrors of possible bio-attacks
Contagious = spreading by contact; sprea-
ding easily from one person to
3) a culprit’s heinous act of spreading germs
another 4) people’s lack of trust in the government
Epidemic = the rapid spread of a disease 5) None of these
among many people in the same Provocation = the action of making somebody
place angry by del iberatel y doing
Dire = very serious or urgent something annoying or offensive
Elusive = difficult to find or capture Heinous = very wicked
Gelignite = a powerful explosive 4. What could be the probable consequences of bio-
attacks, as mentioned in the passage?
Reading Comprehension 557

(A) several deaths 3) It can differentiate between the innocent


(B) political turmoil people and the guilty.
(C) social unrest 4) It is free from any political ideology.
1) A only 2) B only 5) It advocates people in changing the socio-
3) C only 4) A and B only political order
5) All the three Directions (Q. 10-12): Choose the word which
Turmoil = a state of great dis turbance, is most OPPOSITE in meaning of the word printed
confusion or uncertainty in bold as used in the passage
Unr est = a state of disturbance in which 10. gloomy
people are angry or dissatisfied and
1) discouraging 2) disgusting
likely to protest or fight
3) bright 4) tragic
5. The author’s purpose of writing the above passage
5) versatile
seems to explain
Versatile = having many uses; turning easily or
1) the methods of containing terrorism readily from on subject, skill or
2) the socio-political turmoil in African countries occupation to another
3) the deadly strategies adopted by modern 11. cacophonous
terrorists 1) loud 2) melodious 3) sonorous
4) reasons for killing innocent people 4) harsh 5) distant
5) the salient features of terrorism of yesteryear Sonorous = having a full deep sound; sounding
Salient = main; most noticeable or important impressive and important
Yesteryear = the recent past 12. intolerance
6. According to the author of the passage, the root 1) forbearance 2) permissiveness
cause of terrorism is 3) adaptability 4) acceptance
(A) religious fanaticism 5) faithfulness
(B) socio-political changes in countries Forbearance= patience; restraint; tolerance
(C) the enormous population growth Directions (Q. 13-15): Choose the word which
1) A only 2) B only is most nearly the SAME in meaning of the word
3) C only 4) A and B only printed in bold as used in the passage.
5) All the three 13. perpetrators

K KUNDAN
Enormous = very larg; huge 1) opponents 2) followers
7. The phrase “such attacks”, as mentioned in 3) sympathisers 4) leaders
the last sentence of the second paragraph, refers 5) manoeuvrers
to 14. elusive
1) the onslaught of an epidemic as a natural 1) harmful 2) fatal 3) destructive
calamity 4) baffling 5) obstructing
2) bio-attack on political pe ople in the 15. inflicting
government 1) elevating 2) imposing 3) alleviating
3) attack aimed at damaging the reputation of 4) reflecting 5) soothing
the government
4) bio-attack manoeuvred by unscrupulous Passage 4
elements
An independent, able and upright judiciary is the
5) None of these
Onslaught = a violent attack hallmark of a free democratic country. Therefore, the
Manoeuvre = to control or infl uence process of judicial appointments is of vital importance.
s ome bo dy/ s o mething At present, on account of the Supreme Court’s last
skillfully advisory opinion, the role of the executive and its
Unscrupulous = without moral principles; not interference in the appointment of judges is minimal,
honest or fair which, in light of our previous experience, is most
8. The sole objective of the old terrorism, according welcome. However, there is a strong demand for a
to Hoffman, was to National Judicial Commission on the ground of wider
1) plant bombs to kill innocent people participation in the appointment process and for
2) remove colonial power or capitalist system greater transparency. The composition, the role and
3) make people realise the incompetence of the the procedures of the proposed National Judicial
government Commission, must be clearly spelt out, lest it be a
4) give a setback to socio-political order case of jumping from the frying-pan into the fire.
5) None of these Recently, there has been a lively debate in England
9. Which of the following statements is true about on the subject. A judicial commission has been
new terrorism? proposed but there are not many takers for that
1) Its immediate objectives are quite tragic. proposal. In the paper issued this month by the Lord
2) It has far-sighted goals to achieve. Chancellor’s Department on judicial appointments,
558 Test of English Language

the Lord Chancellor has said, “I want every vacancy 1) Not having enough judges from backward
on the Bench to be filled by the best person available. communities.
Appointments must and will be made on merit, 2) Inte rfe re nce of the e xe cutive in the
irrespective of ethnic origin, gender, marital status, appointment of judges.
political affiliation, sexual orientation, religion or 3) Professional misconduct of judges.
disability. These are not mere words. They are firm 4) De lay that occurre d in the judicial
principle s. I will not tole rate any form of appointments.
discrimination.” 5) None of these
At present, there are hardly any persons from the In the light of something = in view of something;
ethnic minorities manning the higher judiciary and considering something
so far not a single woman has made it to the House of 4. The role and proce dure of the National
Lords. The most significant part of Lord Chancellor’s Commission must be spelt out clearly
pape r is the re quire me nt that “alle gations of 1) because executive wing will depend on it
professional misconduct made in the course of heavily.
consultations about a candidate for judicial office must 2) because judges will take judicial decisions on
be specific and subject to disclosure to the candidate”. the basis of it.
This should go a long way in ensuring that principles 3) it will be represented by a cross-section of
of natural justice and fair play are not jettisoned in the society.
the appointment process, which is not an uncommon 4) it will bring a qualitative change in the
phenomenon. interpretation of law.
Upright = placed in a vertical position 5) None of these
Hallmark = a feature or quality that is typical of 5. What has been the subject of lively debate in
somebody/something England?
Minimal = very small in size or amount; as small 1) Role of judiciary in free and de mocratic
as possible nations
Lest = in case; in order to prevent
2) Appointment of judicial commission
something from happening; to avoid
the risk of; because of the
3) Seniority as the basis of appointme nt of
possibility of judges

K KUNDAN
From/out of the frying-pan into the fire = from 4) Appointment of judicial posts
a bad situation to one that is worse 5) None of these
Affiliate = to link a group, a company or an 6. What, according to the author, is the typical
organization very closely with characteristic of an independent democratic
another, larger one country?
Jettison = to get rid of something/somebody
1) Objective process of judicial appointments.
that you no longer need or want; to
discard; to abandon
2) Supreme Court’s advisory opinion on legal
1. What, according to the passage should go a long metters.
way in judicial appointments? 3) Responsible, free and fair judiciary.
1) Decision that all sections of the society are 4) Lively and frank debate in the society on the
represented. role of judiciary.
2) Candidate’s qualifications and seniority are 5) None of these
considered. 7. Which, according to the passage, is not an
3) Candidate must know the charge of uncommon phenomenon?
professional misconduct levelled against him. 1) An independent and upright judiciary
4) The re should be strong re ason for 2) Delays taking place in legal pronouncements
discrimination. 3) Justice being denied to poor people
5) None of these 4) Partiality and subje ctivity in judicial
2. According to the passage, there has been a appointments
demand for a National Judicial Commission to 5) None of these
Pronouncement = a formal public statement
1) clear the backing of court cases.
8. Which of the following words is SIMILAR in
2) make judiciary see eye to eye with executive.
meaning as the word jettison as used in the
3) wipe out corruption at the highest places.
passage?
4) make the appointment process of judges more
1) sacrifice 2) accept 3) modify
broad-based and clear.
4) destroy 5) advocate
5) safeguard the interest of natural justice and
9. Which of the following forms part of what the
fair play in judicial pronouncement.
Lord Chancelor has said?
3. Which of the following could be in the author’s
1) Appointments to judicial posts must take into
mind when he says ‘in the light of our previous
consideration the aspirations of the weaker
experience’?
sections of the society.
Reading Comprehension 559

2) Vacancies in the judiciary must not remain health-care system is urban-based, closely geared to
unfilled. drugs, hospitals and expensively trained apathetic
3) Merit should be the sole criterion for judicial doctors. The bulk of the population in poor countries,
appointments. who live in rural areas, are left untouched by all this
4) Selective discrimination may be preached and and must rely on traditional healers. The answer is
also practised. to turn out medical/health personnel sufficiently, but
5) None of these not expensively, trained to handle routine complaints
10. Which of the following according to the author and to get villagers to pay adequate attention to
is the most welcome thing? cleanliness, hygienic sanitation, garbage disposal and
1) The ne gligible role to be playe d by the othe r e le me ntary but crucial matte r s. More
executive in the appointment of judges. complicated ailments can be referred to properly
2) Coordinating role played by the executive in e quippe d ce ntre s in district towns, citie s and
the appointment of judges metropolises. Traditional healers, whom villagers
3) The appointment of judges from the ethnic trust, can be among these intermediate personnel.
minority classes Some third-world countries, including India, have
4) Appointment of judges purely on the basis of launched or are preparing elaborate schemes of this
merit nature. But the experience is not quite happy. There
5) None of these is resistance from the medical establishment which
11. Which of the following groups of words is sees them as little more than licensed quackery but
SIMILAR in meaning as the word lest as used is not prepared either to offer condensed medical
in the passage? courses such as the former licentiate course available
1) in spite of 2) for fear that in this country and unwisely scrapped. There is the
3) for want of 4) in order to question of how much importance to give to indigenous
5) with regard to system of medicine. And there is the difficult matter
12. What does the expression “from the frying-pan of striking the right balance between preventive
into the fire” mean? healthcare and curative medical attention. These are
1) Seeing one dream after the other complex issues and the Milan conference would
2) Making plan after plan perhaps be more fruitful if it were to discuss such

K KUNDAN
3) Crossing one hurdle after the other specific subjects.
4) Jumping from one high place to another Catch-phrase = a popular phrase that is connected
5) None of these with the politician or entertainer who
used it and made it famous.
Passage 5 Swear by something
/somebody = (not used in the progressive tenses)
After the “Liberal” a new catch-phrase is being to be certain that something is good
coined: `A New Health Order’. Talking about setting it or useful
up is the theme of the WHO-sponsored international Entrench = to establish something very firmly
conference on primary health and medical care, so that it is very difficult to change
Radical = thorough and complete
currently being held at Milan in Italy. While much
Accomplish = to succeed in doing or completing
has been said and written on establishing “new order”, something; to achieve
little has actually been done. Will the conference at Muster = to find as much support, courage
Milan too swear by the “new health order”, go home etc as you can
and then forget about it, while the present medical Carry out = to do something that you have said
and healthcare set-up in poor countries further you will do or have been asked to
entrenches itself? This does not have to be the fate do
of the radical resolutions that will undoubtedly be Geared = designed or organized to achieve a
particular purpose, or to be suitable
passed at Milan. Unlike creating a new world economic
for a particular group of people
or information order, establishing a new health set- Apathetic = showing no interest or enthusiasm
up is essentially a matter for individual countries to Rely on = to need or depend on somebody/
accomplish. No conflict of international interests is something
involved. But this advantage is, at least until it begins Healer = a person who cures people of illness
to take concrete shape, only theoretical. The million- and disease using natural powers
dollar question is whether individual third-world rather than medicine
governments are able and willing to muster the will, Turn out = to be present at an event
Sanitation = the equipments and systems that
the re source s, the administrative and othe r
keep places clean, especially by
infrastructure to carry out what it is entirely within removing human waste
their power to attain and implement. Crucial = extremely important because it will
The dimensions of the problem are known and the affect other things; critical; essential
solutions broadly agreed on. The present medical and Ailment = an illness that is not very serious
Elaborate = carefully prepared and organized
560 Test of English Language

Quackery = the methods or behaviour of 3) improving the economic condition of the


somebody who pretends to have masses.
medical knowledge 4) expediting the setting up of a new health
Condense = to put a lot of information into a small
order.
space
Licentiate = a person with an official permission
5) making cheap drugs available.
to work in a particular profession 6. To make the conference really useful, the author
Scrap = to cancel or get rid of something that suggests ....
is no longer practical or useful 1) resolving the international conflicts involved.
Indigenous = native; belonging to a particular 2) that it should address itself to specific issues.
place rather than coming to it from 3) it should give importance to indige nous
somewhere else system of medicine.
Preventive = intended to try to stop something
4) that it should not pass radical resolutions.
that causes problems or difficulties
from happening
5) None of these
Curative = able to cure illness; healing 7. What does the author suggest for the cure of
1. The author is doubtful whether .... the cases involving complications?
1) an individual country can set up a new health 1) Treating such cases at well-equipped hospitals
order. in district places
2) the Milan conference would pass radical 2) Training such victims in preliminary hygiene
resolutions. 3) Training semi-skilled doctors to treat such
3) under-developed countries have the capacity cases
to organize their resources. 4) Issuing licences to semi-skilled doctors to
4) traditional he ale rs could be traine d as treat such cases
intermediate health personnel. 5) None of these
5) the problem has been understood at all. 8. The medical establishment seems to be reluctant
2. The author has reservations about the utility of to trust the ....
the Milan Conference because .... 1) allopathic medical practitioners.
1) it is expected only to discuss but not decide 2) traditional healers.
upon anything. 3) urban-based medical practitioners.

K KUNDAN
2) earlier conferences had failed to reach any 4) expensively trained allopathic doctors.
decisions. 5) None of these
3) the medical profession is opposed to a new Reluctant = hesitating before doing something
because you do not want to do it or
health order.
because you are not sure that it is
4) while “new orders” are talked and written the right thing to do
about, not much is actually done. 9. For a new health order, the author recommends
5) None of these all of the following EXCEPT
3. The contents of the passage indicate that the 1) motivating village rs to pay atte ntion to
author is opposed to .... cleanliness
1) traditional healers. 2) setting up well equipped centres in district
2) licentiate practitioners. towns
3) allopathic system of medicines. 3) discontinuing the present expensive medical
4) hospitals. courses
5) None of these 4) training traditional healers to function as
4. It can be inferred from the contents of the medical health personnel
passage that the author’s approach is ... 5) striking a balance be twe e n pre ve ntive
1) sarcastic 2) constructive healthcare and curative medical attention
3) indifferent 4) fault-finding Directions (Q. 10-12): Choose the word which
5) hostile is most nearly the SAME in meaning as the word
Sarcastic = showing or expressing a way of
printed in bold as used in the passage.
using words that are the opposite
of what you mean in order to be
10. LAUNCHED
unpleasant to somebody or to make 1) participated 2) accomplished
fun of them. 3) elevated 4) planned
Indifferent = having or showing no interest in 5) started
somebody/something Accomplished = very good at a particular thing;
Hostile = very unfriendly or aggressive and having a lot of skills
ready to argue or fight 11. MUSTER
5. The author thinks that the solution to the 1) enlist 2) summon
problem of medical/health care lies in .... 3) manifest 4) extend
1) opening hospitals is rural areas. 5) enrich
2) conducting inexpensive medical courses.
Reading Comprehension 561

Enlist = to persuade somebody to help you considerations. This rich interpersonal relationship
or to join you in doing something between the physician, patient and family has, barring
Summon = to arrange an official meeting a few exceptions, prevailed till the recent past, for
Manifest = to show something clearly, especially
caring was considered as important as curing. Our
a feeling, an attitude or a quality
Enrich = to improve the quality of something,
indigenous systems of medicine like ayurveda and yoga
often by adding something to it have been more concerned with the promotion of the
12. ENTRENCH health of both the body and mind and with maintaining
1) being deteriorating a harmonious relationship not just with fellow-beings
2) surround completely but with nature itself, of which man is an integral
3) establish firmly part. Healthy practices like cleanliness, proper diet,
4) enclose carefully exercise and meditation are part of our culture which
5) finish radically sustains people even in the prevailing conditions of
Directions (Q. 13-15): Choose the word which poverty in rural India and in the unhygienic urban
is most nearly OPPOSITE in meaning of the word slums.
printed in bold as used in the passage. These systems consider disease as an aberration
13. CONDENSED resulting from disturbance of the equilibrium of health,
1) concentrated which must be corrected by gentle restoration of this
2) envigoured balance through proper diet, medicines and the
3) expanded establishment of mental peace. They also teach the
4) lengthened graceful acceptance of old age with its infirmities
5) inexplicable resulting from the normal degenerative process as
Inexplicable = that cannot be understood or explained well as of death which is inevitable.
14. CRUCIAL This is in marked contrast to the western concept
1) trivial of life as a constant struggle against disease, ageing
2) critical and death which must be fought and conquered with
3) significant the knowledge and technology derived from their
4) marvellous science: a science which, with its narrow dissective
5) conspicuous and quantifying approach, has provided us the

K KUNDAN
Trivial = not important or serious; not worth unde rstanding of the microbial cause s of
considering communicable diseases and provided highly effective
Critical = extremely important because a future technology for their prevention, treatment and control.
situation will be affected by it; crucial This can rightly be claimed as the greatest contribution
Conspicuous = easy to see or notice; likely to attract
of western medicine and justifiably termed as ‘high’
attention
technology. And yet the contribution of this science
15. RESISTANCE
in the fie ld of non-communicable dise ase s is
1) opposition
remarkably poor despite the far greater inputs in
2) agreement
research and treatment for the problems of ageing
3) repulsion
like cancer, heart diseases, paralytic strokes and
4) acceptance
arthritis which are the major problems of affluent
5) compliance
Repulsion = a feel ing of very strong dis like to
societies today.
something that you f ind extremel y Endeavour = an attempt to do something new or
unpleasant difficult
Compliance = the practice of obeying rules or requests Connote = to suggest a feeling, an idea, etc as
made by people in authority well as the main meaning
Virtually = almost or very nearly, so that any
Passage 6 slight difference is not important
Interpersonal = connected with relationships
There is no field of human endeavour that has been between people
so misunderstood as health. While health which Barring = except for; unless there is/are
connotes well-being and the absence of illness has Prevail = to exist or be very common at a
particular time or in a particular place
a low profile, it is illness representing the failure of
Indigenous = native; belonging to a particular
health which virtually monopolizes attention because place rather than coming to it from
of the fear of pain, disability and death. Even Sushruta somewhere else
has warned that this provides the medical practitioner Aberration = a fact, an action or a way of behaving
power over the patient which could be misused. that is not usual, and that may be
Till recently, patients had implicit faith in their unacceptable
physician whom they loved and respected, not only Restoration = the act of returning something to its
for his knowledge but also in the total belief that correct place, condition or owner
Infirmity = weakness or illness over a long
practitioners of this noble profession, guided by ethics,
period
always placed the patient’s interest above all other
562 Test of English Language

Degenerative= (of an illness) getting or likely to get 2) Technology and science.


worse as time passes 3) Western physician and western-educated
Inevitable = that you cannot avoid or prevent Indian physician.
Dissect = to study something closely and/or
4) Indian and western concepts of life.
discuss it in great detail
Affluent = having a lot of money and a good
5) Knowledge and technology.
standard of living 7. Why does the author describe the contributions
Proponent = a person who supports an idea or of science as remarkably poor?
course of action 1) It concentrates more on health than on
1. Which of the following has been described as illness.
the most outstanding be ne fits of mode rn 2) It suggests remedies for the poor people.
medicine? 3) It demands more inputs in terms of research
(A) The re al cause and ways of control of and technology.
communicable diseases 4) The cost of treatment is low.
(B) Evolution of the concept of harmony between 5) None of these
man and nature 8. Which of the following can be inferred about the
(C) Special techniques for fighting ageing position of the author in writing the passage?
1) Only B and C 2) Only A and B (A) Arde nt supporte r of weste rn syste m in
3) Only A 4) Only B present context.
5) Only C (B) Supremacy of ancient Indian syste m in
2. In India traditionally the doctors were being today’s world.
guided mainly by which of the following? (C) Critical and objective assessment of the
1) High technology present situation.
2) Good knowledge 1) Only A 2) Only B
3) Professional ethics 3) Only C 4) Neither B nor C
4) Power over patient 5) None of these
5) Western concept of life Ardent = very enthusiastic and s howing
3. What caution have proponents of indigenous strong feelings about something/
systems sounded against medical practitioners? somebody; passionate

K KUNDAN
1) Their undue concern for the health of the 9. The author seems to suggest that
person. 1) we should give importance to improving the
2) The ir e mphasis on re se arch on non- health rather than curing of illness.
communicable diseases. 2) we should move towards becoming an affluent
3) Their emphasis on curing illness rather than society.
preventive health measures. 3) ayurveda is superior to yoga.
4) The ir e mphasis on re storing he alth for 4) good interpersonal relationship between the
affluent members of the society. doctor and the patient in necessary but not
5) None of these sufficient.
4. Why has the field of health not been understood 5) ayurvedic medicines can be improved by
properly? following western approaches and methods of
1) Difficulty in unde rstanding distinction sciences.
Interpersonal = connected with relationships
between health and illness.
between people
2) Confusion between views of indigenous and
Directions (Q. 10-12): Choose the word which
western system.
is most OPPOSITE in meaning of the word printed
3) Highly advanced technology being used by the
in bold as used in the passage.
professionals.
10. INEVITABLE
4) Not given in the passage.
1) Undesirable 2) Unsuitable
5) None of these
3) Detestable 4) Avoidable
5. Why, according to the author, have people in
5) Available
India survived in spite of poverty?
11. CONCERNED
1) Their natural resistance to communicable
1) Diluted 2) Liberated
diseases is very high.
3) Indifferent 4) Divested
2) They have easy access to western technology.
5) Relaxed
3) Their will to conquer diseases
12. DEGENERATIVE
4) The ir harmonious re lationship with the
1) Recuperative 2) Revolving
physician
3) Productive 4) Innovative
5) None of these
5) Integrative
6. Which of the following pairs are mentioned as Recuperate = to get back your health, strength or
‘contrast’ in the passage? energy after being sick, tired injured
1) Western concept of life and science. etc; to recover
Reading Comprehension 563

Directions (Q. 13-15): Choose the word which Implication = a possible effect or result of an action
is most nearly the SAME in meaning as the word or a decision
printed in bold as used in the passage. Flourish = to develop quickly and be successful
or common; thrive
13. CONNOTES
Cartload = a large amount of something
1) Helps 2) Cures 3) Follows In black and white = in writing or in print
4) Confirms 5) Implies Myriad = an extremel y large number of
14. ABERRATION something
1) Observation 2) Alternative 3) Deviation Tint = a s hade or smal l amount of a
4) Outcome 5) Stimulate particular colour
Stimulate = to make somebody interested and Scourge = a person or thing that causes trouble
excited about something or suffering
15. DERIVED 1. How do the British economists and political
1) Constructed 2) Sprung 3) Directed scientists react to budget secrecy? They are
4) Processed 5) Continued 1) in favour of having a mix of secrecy and
openness.
Passage 7 2) indifferent to the budgeting techniques and
taxation policies.
We have inherited the tradition of secrecy about
3) very critical about maintenance of budget
the budget from Britain where also the system has
secrecy.
been strongly attacked by eminent economists and
4) advocates of not disclosing in advance the
political scientists including Peter Jay. Sir Richard
budget contents.
Clarke, who was the originating genius of nearly every
5) None of these
important development in the British budgeting
Indifferent = having or showing no interest in
techniques during the last two decades, has spoken somebody/something
out about the abuse of budget secrecy: “The problems 2. The author thinks that openness in budget is
of long-term tax policy should surely be debated essential as it leads to
openly with the facts on the table. In my opinion, all 1) prevention of tax implications
governments should have just the same duty to 2) people’s reluctance to accept their moral
publish the ir e xpe nditure policy. Inde e d, this

K KUNDAN
duties
obligation to publish taxation policy is really essential 3) exaggerated revelation of the strengths and
for the control of public expenditure in order to get weaknesses of economy
re alistic taxation implications.” Re alising that 4) making our country on par with Finland
democracy flourishes best on the principles of open 5) None of these
government, more and more democracies are having 3. The author seems to be in favour of
an open public debate on budget proposals before 1) maintaining secrecy of budget
introducing the appropriate Bill in the legislature. In 2) judicious blend of secrecy and openness
the United States the budget is conveyed in a message 3) transparency in budget proposals
by the President to the Congress, which comes well 4) replacement of public constitution by secrecy
in advance of the date when the Bill is introduced in 5) None of these
the Congress. In Finland the Parliament and the people 4. The secrecy of the budget is maintained by all of
are already discussing in June the tentative budget the following countries except
proposals which are to be introduced in the Finnish (A) Finland
Parliament in September. Every budget contains a (B) India
cartload of figures in black and white - but the dark (C) United States
figures represent the myriad lights and shades of 1) Only A 2) Only B
India’s life, the contrasting tones of poverty and 3) Only C 4) A and C
wealth, and of bread so dear and flesh and blood so 5) B and C
cheap, the deep tints of adventure and enterprise and 5. Which of the following statements is definitely
man’s ageless struggle for a brighter morning. The TRUE in the context of the passage?
Union budget should not be an annual scourge but a 1) The British Government has been religiously
part of pre se ntation of annual accounts of a maintaining budget secrecy.
partnership between the Government and the people. 2) Budget secrecy is likely to lead to corrupt
That partnership would work much better when the practices.
nonsensical secrecy is replaced by openness and 3) Consulting unjustifiable taxes with public
public consultations, resulting in fair laws and the helps make them accept those taxes.
people’s acceptance of their moral duty to pay. 4) The re should be no control on public
Inherit = to receive money, property, etc from
expenditure in democratic condition.
somebody when they die
Eminent = famous and respected especially in
5) None of these
a particular profession Religiously = very carefully or regularly
564 Test of English Language

6. Sir Richard Clarke seems to deserve the credit Incompatible = the two actions, ideas, etc that
for are i ncompat ibl e are not
1) transformation in the British budge tary acceptable or possible together
because of basic differences
techniques.
12. DUTY
2) maintenance of secrecy of the British budget.
1) obligation 2) imposition 3) tax-liability
3) detection of abuse of transparency in budget.
4) function 5) job
4) bringing down the tax load on British people.
Directions (Q. 13-15): Choose the word which
5) None of these
is most OPPOSITE in meaning to the word printed
7. From the contents of the passage, it can be
in bold as used in the passage.
inferred that the author is
13. FLOURISHES
1) authoritarian in his approach.
1) disappears 2) degenerates 3) vanishes
2) a democratic person.
4) blooms 5) opens
3) unaware of India’s re ce nt e conomic
Degenerate = to become worse, for example by
developments. becoming lower in quality or weaker;
4) a conservative person. deteriorate
5) None of these Bloom = to become heal thy, happy or
8. Which of the following state me nt(s) is/are confident
definitely False in the context of the passage? 14. DEBATED
(A) Transparency helps unscrupulous elements 1) questioned severely
to resort to corrupt practices. 2) opposed strongly
(B) Open approach of Government is a sign of 3) accepted unconditionally
healthy democracy. 4) discussed frankly
(C) People’s acceptance of their moral duties can 5) implemented forcibly
best be achieved through openness and 15. IMPORTANT
public consultations. 1) major 2) uncountable
1) Only A 2) Only B 3) Only C 3) significant 4) unscheduled
4) A and B 5) B and C 5) trivial
Unscrupulous = without moral principl es ; not Trivial = not important or serious; not worth

K KUNDAN
honest or fair; unprincipled considering
To r esor t of som ethi ng = to make use of
something especially something Passage 8
bad, as means of achieving
something often because there is In a disarmingly frank talk at the Indian Merchants
no other possible solution Chamber in Mumbai, the Japanese Ambassador in
9. For making the budget realistic, the Government India dwelt at length on issues that exercise the minds
should of Japanese investors when they consider investment
1) refrain from making public the proposed proposals in India.
provisions before finalisation. Raising the question “What comparative advantages
2) discuss it secretly within themselves. does India offer as an investment market?”, he said
3) e ncourage the public to se nd in the ir though labour in India is inexpensive, wage levels
suggestions. are offset by productivity level to a large extent.
4) consult the public, defend their own plans and Acknowledging that the vastness of the Indian
accept public suggestions. market is a great inducement for investment in
5) None of these manufacturing industry, he wondere d if it was
To refrain from = to stop yourself from doing justifiable to provide that overseas remittance of
s omething especiall y profit in foreign exchange be fully covered by exchange
something that you want to do earnings as had been done. Significantly, on the eve
Directions (Q. 10-12): Choose the word which of the Prime Minister’s visit to Japan, the government
is most nearly the SAME in meaning to the word delinked profits repatriation from exports, meeting this
printed in bold as used in the passage. demand.
10. SCOURGE The Ambassador said foreign investors needed to
1) ritual 2) presentation be assured of the continuity and consistency of the
3) whip 4) compromise liberalisation policy and the fact that new measures
5) remedy had been put into force by means of administrative
Whip = a written ins tructions tell ing notifications without amending government laws acted
members of a political party how to
as a damper.
vote on a particular issue
The Ambassador pleaded for speedy formulation of
11. MYRIAD
the exit policy and pointed to the highly restrictive
1) adequate 2) functional 3) incompatible
control by the government on disinvestment by foreign
4) abundant 5) excellent
partners in joint ventures in India.
Reading Comprehension 565

While it is all too easy to dismiss critical comment Ordeal = a diff icul t or unpleas ant
on conditions in India contemptuously, there can experience
be little doubt that if foreign investment is to be wooed Dogged = showing determination; not
giving up easily
ass iduous ly, we will have t o me e t e xacting
Improvise = to make or do something using
international standards and cater at least partially to whatever is available, usually
what we may consider the idiosyncrasies of our because you do not have what
foreign collaborators. The Japanese too have passed you really need
through a stage in the fifties when their products were Gainsay = to say that something is not
derided as sub-standard and shoddy. That they have true; to disagree with or deny
come out of that ordeal of fire to emerge as an economic something
superpower speaks as much of their doggedness to Benchmark = something that can be
measured and us ed as a
pursue goals against all odds as of their ability to
standard that other things can
improvise and adapt to internationally acceptable be compared with
standards. 1. The author has appreciated the Japanese for their
There is no gainsaying that the past record of 1) quality of products manufactured in the fifties.
Japanese investment is a poor benchmark for future 2) passing through an ordeal.
expectations. 3) perseverance in raising quality of products.
Disarming = making people feel less angry
4) future expectations.
or suspicious than they were
before
5) None of these
To dwell on/upon Perseverance = the quality of continuing to try
something = to think or talk a lot about to achieve a particular aim
something, especiall y despite difficulties
something it would be better 2. According to the Japanese Ambassador, which
to forget; to look at something of the following motivates the foreign investors
for a long time to invest in Indian manufacturing industry?
Offset = to use one cost, payment, or 1) very large scope of Indian market
situation in order to cancel or 2) overse as re mittance of profit in foreign
reduce the effect of another exchange

K KUNDAN
Inducement = s omething that is given to
3) assurance of continuity of the liberalisation
somebody to persuade them to
do something; incentive
policy
Remittance = a sum of money that is sent to 4) high productivity levels
somebody in order to pay for 5) None of these
something 3. The purpose of the author in writing this passage
Repatriate = to send money or profits back seems to be to
to your own country 1) discourage foreign investment in India.
Damper = a piece of metal that can be 2) critically e xamine Indian inve stme nt
moved to allow more or less air
environment.
into a fire so that the fire burns
more or less strongly
3) paint a rosy picture of India’s trade and
Contemptuous = feeling or showing that you commerce.
have no respect for somebody/ 4) criticize government’s liberalization policy.
something; scornful 5) raise the expectations of foreign investors.
Assiduous = working very hard and taking 4. Which of the following suggestions were expected
great care that everything is by the Japanese Ambassador?
done as well as it can be; (A) speedy formulation of the exit policy
diligent
(B) imposing restrictions of disinvestment by
Woo = to try to get the support of
somebody
foreign partners in joint ventures in India
Exacting = needing or demanding a lot of (C) continuity and consiste ncy of the
effort and care about details. liberalisation policy
Cater to somebody 1) All the three
/something = to provide the things that a 2) A and B only 3) B and C only
particular type or person wants, 4) A and C only 5) None of these
especially things that you do 5. According to the Japanese Ambassador, India
not approve of
offers a comparative advantage to fore ign
Idiosyncrasy = a person’s particular way of
behaving, things, etc especially
investors in terms of
when it is unusual; an unusual 1) inexpensive labour
feature 2) abysmally low wage levels
Shoddy = made or done badly and with 3) higher productivity
not enough care; second-rate 4) skilled workforce
5) None of these
566 Test of English Language

Abysmal = extremely bad or of a very low Shoddy = in a poor condition because they
standard have been used a lot
6. For seeking more and more foreign investment, Directions (Q. 13-15): Choose the word which
the author suggests that we should is most OPPOSITE in meaning of the word printed
1) satisfy fully the whims of our fore ign in capital as used in the passage.
collaborators. 13. INDUCEMENT
2) dismiss all critical comme nts on Indian 1) incentive 2) motive
conditions. 3) breach 4) temptation
3) link profit repatriations to exports. 5) impediment
4) raise the quality of product to match Impediment = something that delays or stops the
international standards. progress of something; obstacle
5) None of these 14. JUSTIFIABLE
Whim = a sudden wis h to do or have 1) unreasonable 2) formidable
something, especially when it is 3) irrevocable 4) unscrupulous
something unusual or unnecessary. 5) inevitable
7. From the passage it can be inferred that the Formidable = if people, things or situations are
author is formidable, you feel fear and/or
1) a political commentator. respect for them because they are
2) a secretary of the Japanese Ambassador. impressive or powerful, or because
they seem very difficult
3) a Japanese investor.
Irrevocable = that cannot be changed
4) an Indian investor.
15. CONTEMPTUOUSLY
5) None of these
1) amicably 2) reasonably
8. The author attributes Japan’s emergence as an
3) respectfully 4) methodically
economic superpower to
5) indecisively
(A) their ability to overcome any ordeal. Amicable = done or achieved in a polite or
(B) their tenacity and perserverance despite friendly way and without arguing
unfavourable circumstances.
(C) their ability to improvise and adapt to globally Passage 9
acceptable quality levels.

K KUNDAN
Since July 1991, the government of India has
1) A and B only 2) B and C only
effectively put the liberalisation policy into practice.
3) A and C only 4) All the three
The drastic steps even include some administrative
5) None of these
Tenacious = determined reforms for pruning the government agencies. Last
9. Which of the following statement(s) is/are true year the Japanese business circles represented by
about the critical comments on investment the Ishikawa Mission called attention of their Indian
conditions in India? counterparts to what they considered to be the major
(A) These comments are difficult to be countered. impediments in India. However, thanks to the almost
(B) These comments are received from various revolutionary reforms put into effect by the Indian
international quarters. government, those impediments either have been
(C) These comments are based more on biases removed or now are on their way out. This development
than on facts. gives a new hope for the future of economic co-
1) Only C 2) Only B operation between the two countries. At the same
3) Only A 4) A and B only time, it should be borne in mind that there is a stiff
5) A and C only competition with other countries, notably China and
Directions (Q. 10-12): Choose the word which South-East Asian countries, in this regard. The
is most nearly the SAME in meaning to the word succe ss storie s of ASEAN countries welcoming
printed in capital as used in the passage. Japanese investments with adequate infrastructure
10. ASSIDUOUSLY are already known in India but it may be useful if
1) persistently 2) hastily further studies of Japanese joint ventures in ASEAN
3) feebly 4) deliberately countries be made by Indian business circles. The
5) innocently coastal areas of China have initiated a very active
Feeble = very weak; not effective campaign to welcome foreign economic participation.
11. IDIOSYNCRASIES Beyond our bilateral relationship, India’s more
1) demands 2) needs active participation in global economy is needed. India
3) deviations 4) ideologies certainly deserves a far bigger share of world trade
5) identity considering its vast resources. It is strongly hoped
12. SHODDY that the Indian government’s recently initiated effort
1) extraordinary 2) shabby of enlarging its export market would bear fruit.
3) cheap 4) disadvantageous India has steadfastly maintained its parliamentary
5) unprofitable democracy since independence. Considering its size,
Reading Comprehension 567

its population and its internal complexity, the overall 4. Which of the following is TRUE about the
maintenance of national integrity and political stability author’s view regarding India’s participation in
under parliamentary democracy is remarkable and world trade?
admirable indeed. Here lies the base for the status of 1) India should actively contribute in a big way
India in the world. By effectively implementing its as it had tremendous resources.
economic reform with the support of public opinion, 2) India’s sharing in global economy has already
this democratic polity of India has again demonstrated been very fast and beyond its resources.
its viability and resilience. At the same time, it gives 3) India should refrain from making efforts in
hope and inspiration to the whole world which faces enlarging its export market.
the difficult problem of North-South confrontation. 4) India needs to first strengthen its democracy.
Drastic = extreme in a way that has a sudden, 5) None of these
s erious or viol ent ef fect on Tremendous = very great; huge
something Refrain from = to stop yourself from doing
Prune = to cut out parts of something something, especiall y
Impediment = something that delays or stops the something that you want to do
progress of something 5. On India’s implementing liberalisation policy, the
Stiff = more difficult or severe than usual author seems to be
St eadfast = not changing in your attitudes or
1) unreasonably critical.
aims
Viability = that can be done; that will be
2) sarcastic.
successful; feasible 3) appreciative.
Resilience = the ability of people or things to feel 4) unconvinced about its effectiveness.
better quickl y af ter something 5) None of these
unpleasant, such as shock, injury Sarcastic = showing or expressing a way of
etc. using words that are the opposite
1. The Ishikawa Mission during its visit to India of what you mean in order to be
emphasized on unpleasant to somebody or to make
1) future economic co-operation between Japan fun of them
and India. 6. It can be inferred from the content of the passage
2) ne e d for re moving policy and/or that the author is a/an

K KUNDAN
implementation hurdles. 1) political analyser
3) need for a stiff competition. 2) Japanese bureaucrat
4) striking down revolutionary reforms. 3) economist
5) None of these 4) Japanese politician
2. How did the Indian government react to the 5) Indian Prime Minister
hurdles in the way of bilateral trade between 7. The author seems to appreciate India’s national
India and Japan? integrity and political stability particularly in view
1) The governme nt, in principle , agreed for of which of the following?
removal of these hurdles. (A) the size of the country
2) Bureaucracy succeeded in maintaining a status (B) India’s population
quo. (C) its internal complexity
3) Gove rnme nt thought it was against 1) None of the three
liberalisation policy. 2) All the three
4) The Japanese delegation could not forcefully 3) A and B only
argue their case. 4) B and C only
5) It failed to remove these hurdles. 5) A and C only
Status quo = the situation as it is now, or as it 8. The author feels that India has a better status
was before a recent change in the world market because of its
3. What is the result of Japanese investments in 1) success in political stability and national
ASEAN nations? integration in democratic set-up.
1) It could not gather momentum for want of 2) vast population.
infrastructure. 3) giant size.
2) The e xpe rime nt faile d be cause of stiff 4) effective bilateral relationship with other
competition from other countries. countries.
3) China and South-East Asian countrie s 5) foreign economic participation.
objected to Japanese investments. 9. Which of the following statements is TRUE in
4) The passage doe s not provide comple te the context of the passage?
information. (A) India’s successful experiment of economic
5) None of these reform has become an inspiration to the
For want of something = because of lack of some- world.
thing; because something is not (B) Size, population and internal complexity of
available
568 Test of English Language

our country are the barriers in the way of ever since the dawn of civilization persons in power
attaining national integrity and political have always tried to supervise or control education.
stability. It has been the hand-maid of the ruling class. During
(C) A few government agencies were not in favour the Christian era, the ecclesiastics controlled the
of liberalisation policy at the beginning. institution of education and diffused among the people
1) A only 2) B only the gospel of the Bible and religious teachings. These
3) C only 4) All the three gospels and teachings were no other than a philosophy
5) None of these for the maintenance of the existing society. It taught
Directions (Q. 10-12): Choose the word which the poor man to be meek and to earn his bread with
is most nearly the SAME in meaning as the word the sweat of his brow, while the priests and the
printed in bold as used in the passage. landlords lived in luxury and fought duels for the
10. STEADFASTLY slightest offence. During the Renaissance, education
1) quickly 2) violently 3) adversely passed more from the clutches of the priest into the
4) religiously 5) faithfully hand of the prince. In other words, it became more
Religiously = very carefully or regularly secular. It was also due to the growth of the nation-
11. RESILIENCE state and powerful monarchs who united the country
1) quietening 2) amplifying 3) existence under their rule. Thus, under the control of the
4) adaptability 5) rejuvenation monarch, education began to devise and preach the
Quieten = to become calmer or less noisy; to infallibility of its masters, the monarch or king. It
make somebody/something calmer also invented and supported fantastic theories like
or less noisy
the Divine Right Theory and that the king can do no
Amplify = to increase something in strength,
especially sound
wrong etc. With the advent of the industrial revolution
Rejuvenate = to make somebody/something look education took a different turn and had to please the
or feel younger or more lively new masters. It now no longer remained the privilege
12. PRUNING of the baron class but was thrown open to the new
1) activating 2) trimming 3) punishing rich merchant class of society. Yet education was still
4) encouraging 5) empowering confined to the few elite. The philosophy which was
Directions (Q. 13-15): Choose the word which in vogue during this period was that of ‘laissez-faire’

K KUNDAN
is most OPPOSITE in meaning of the word printed restricting the function of the State to a mere keeping
in bold as used in the passage. of law and order while, on the other hand, in practice
13. STIFF the law of the jungle prevailed in the form of free
1) stubborn 2) indelible 3) tense competition and the survival of the fittest.
4) yielding 5) soapy Impart = to pass information, knowledge, etc
Stubborn = determined not to change your to other people
opinion or attitude Hand-maid = a female servant; something that
Indelible = impossible to forget or remove; supports and helps something else
permanent Ecclesiastic = a priest or minister in the Christian
14. VAST Church
Gospel = one of the four books in the Bible
1) minute 2) meagre 3) minor
about the life and teaching of Jesus
4) innumerable 5) intangible Meek = quiet gentle and always ready to do
Intangible = that exists but that is difficult to what other people want without
describe, understand or measure expressing your own opinion
15. IMPEDIMENTS By the sweat of
1) exaggeration 2) compendium someone brow = by working very hard
3) obstacle 4) aggravation Duel = a competition or struggle between
5) furtherance two people or groups
Compendium =a collection of facts, drawings and Infallible = never wrong; never making
photographs on a particular subject, mistakes; that ever fails; always
especially in a book doing what it is supposed to do
Aggravate = to make an illness or a bad or With the
unpleasant situation wors e; to advent of = the coming of an important event,
worsen person, invention etc
Furtherance = the process of helping something to Devise = to invent something new or a new
develp or to be succes sf ul ; way of doing something
advancement Privilege = a special right or advantage that a
particular person or group of people
Passage 10 has
Baron = a nobleman of the lowest rank
It is an old saying that knowledge is powe r. Vogue = a fashion for something
Education is an instrument which imparts knowledge Laissez-faire = the policy of all owing private
and, therefore, indirectly controls power. Therefore, business to develop without
government control
Reading Comprehension 569

1. Who controlled education during the era after 2) The tiny droplets of sweat on the forehead
the industrial revolution? 3) The wrinkles visible on the face
1) The baron class 4) The sign of innocence
2) The priests 5) None of these
3) The prince 9. Why have persons in power always tried to
4) The monarch supervise or control education?
5) None of these 1) Because they wanted to educate the whole
2. What does the theory of Divine Right of king public.
stipulate? 2) Because they wanted to deprive the common
1) That kings are gods. man of the benefits of education.
2) They have the right to be worshipped like gods 3) Because it involved a huge expenditure on the
by their subjects. state exchequer.
3) That the right of governing is conferred upon 4) Because it is an instrument of knowledge and
kings by god. therefore power.
4) That the rights of kings are divine and therefore 5) None of these
sacred. Exchequer = the government department that
5) None of these controls public money
Stipulate = to state clearly and firmly that 10. What does the philosophy of Laissez-Faire stand
something must be done or how it for?
must be done; to specify 1) Joint control of the means of production by
3. What does the expression ‘hand-maid of the the state and private enterprise
ruling class’ mean? 2) Individual freedom in the economic field
1) Private mistress of the prince 3) State control of the means of production
2) Something fully under the control of the ruling 4) Full de ve lopme nt of the individual’s
class personality
3) Private maid-servants of the prince 5) None of these
4) The symbol of authority of the prince
5) None of these Passage 11

K KUNDAN
4. Who controlle d e ducation during the
The development of nationalism in the third world
Renaissance?
countries, as is well known, followed a very different
1) The common people
trajectory from that in the advanced capitalist
2) The prince
countries. In the latter it was a part of the process of
3) The church and the priests
the emergence of the bourgeois order in opposition
4) The secular leaders of the society
to feudalism, while in the former it was a part of the
5) None of these
anti-colonial struggle. The impact of colonialism,
5. What does the word “infallibility” mean?
though it differed across countries, had on the whole
1) That every man is open to error
been in the direction of transcending localism and
2) That some divine power is responsible for
unifying supra-local economic structures through the
determining the fate of men
introduction of market relations. The struggle against
3) The virtue of not making any mistake
colonialism, consequently, took the form of a national
4) Sensitivity
struggle in each instance in which people belonging
5) None of these
to diffe re nt tribe s or linguistic communitie s
Virtue = an attractive or useful quality
participated. And the colonial power in each instance
6. What did the ruling class in the Christian era
attempted to break this emerging national unity by
think of the poor man?
splitting people.
1) That he is the beloved of god
The modus operandi of this splitting was not just
2) That he deserves all sympathy of the rich
through political manipulation as happene d for
3) That he should be strong
instance in Angola, South Africa and a host of other
4) That he is meant for serving the rich
countries; an important part of this modus operandi
5) None of these
was through the nurturing of a historiograpy that
7. Who controlled the institution of education during
just denied the existence of any overarching national
the Christian era?
consciousness. The national struggle, the national
1) The church and the priests
movement were given a tribal or religious character,
2) The monarchs
they were portrayed as being no more than the
3) The secular leaders of society
movement of the dominant tribe or the dominant
4) The common people
religious group for the achievement of narrow sectional
5) None of these
ends. But the important point in this colonialism,
8. What do you mean by the ‘sweat of his brow’?
while, on the one hand, it objectively created the
1) Very hard work
condition for the coming into being of a national
570 Test of English Language

consciousness at a supra-tribal, supra-local and supra- 4. What was the role of introduction of market
religious level, on the other hand it sought deliberately relations in the process of economic integration?
to subvert this very consciousness by using the same 1) It had diffe re nt impacts in all colonial
forces which it had objectively undermined. exercises.
Trajectory = the curved path of something that 2) It overthrew the capitalistic approach in the
has been fired, hit or thrown into third world countries.
the air 3) It advocated importance of localism and
Emerge = to become known
restricted economic growth.
Bourgeois = belonging to the middle class
Transcend = to be or go beyond the usual limits
4) It broke the shackles of localism and helped
of something unify the economic structures.
Modus operandi = a particular method of working 5) None of these
Nurture = to have a feeling, an idea, a plan, etc Shackles = a particular state, set of conditions
for a long time and encourage it to or circumstances, etc that prevent
develop you from saying or doing what you
Overarching = very important, because it includes want
or influences many things 5. How did nationalism originate in the third world
Subvert = to try to destroy the authority of a countries?
political, religious etc system by 1) as a struggle against feudalism
attacking it secretly or indirectly 2) as vehement opposition to colonialism
Undermine = to make s omething especiall y
3) to advocate capitalistic movement
somebody’s confidence or authority,
gradually weaker or less effective.
4) to strengthen localism
Historiography = the study of writing about history 5) None of these
1. Which of the following was the advantage of Vehement = showing very s trong feel ings ,
especially anger
struggle against colonialism?
6. What was the motive of colonial powers in writing
1) Tribal groups held their separate identity
a distorted history?
throughout the struggle.
1) to emphasise the existence of domination by
2) Communities got divided on the basis of
one tribe over other weaker tribes
religion and language.
2) to make people aware of and to integregate

K KUNDAN
3) Backwardness of tribals was eradicated.
on the basis of their rich cultural heritage
4) Awareness beyond linguistic and religious
3) to make people aware of their glorious religion
identity was generated.
and widely used language
5) None of these
4) to give an impression to general people that
2. How did colonial power react to topple the anti-
there was no national consciousness and to
colonial structure?
prevent them from being united
1) by splitting pe ople on the basis of the ir
5) None of these
financial positions
7. Which of the following statements is definitely
2) by using tempting economic strategies
true in the context of the passage?
3) by creating linguistic, tribal and religious
(A) Colonialism internally helped awakening
divides
nationalism among people of different tribes,
4) by instigating tribals against anti-communal
religions, etc.
forces
(B) Advanced capitalist countries had nurtured
5) None of these
Topple = to become unsteady and fall down;
nationalism as an opposition to feudalism.
to make somebody l os e their (C) The national struggle was not successful
position of power or authority because the colonial powers succeeded in
Instigate = to make something start or happen, dividing the people.
usually something official 1) A and B only 2) B and C only
3. The author has given the example of Angola, 3) A only 4) B only
South Africa, etc in order to 5) C only
1) bring out the similarity of tactics used by the 8. From the content of the passage it appears that
rulers of colonies to divide the natives the author is
2) emphasise how nationalism has be come 1) a vehement critic of anti-feudalism
almost extinct and capitalism has borne roots 2) a staunch follower of capitalistic pattern
3) support the argument that feudalism was 3) an impartial commentator of historical and
oppose d by pe ople in unde rde ve lope d political events
countries also 4) a person holding colonialism in high esteem
4) lay stress on the fact that tribals in those 5) a historian with view coloured in favour of
countries were divided on account of language nationalism
5) None of these Staunch = strong and loyal in your opinions
Extinct = no longer active and attitude
Reading Comprehension 571

9. The colonial powers tried to camouflage national Passage 12


movement and to show it as only
1) a historical fact having ancient roots India is a country of villages. Rural population still
2) skirmish led by a dominating tribe or a dominates the urban population as far as the number
religious group with selfish motive is considered. This is despite the fact that there is
3) dominance of narrow sectional ends over rampant migration of rural families to urban centres.
national goals Generally, the gains of being a unit of the urban
4) survival of the fittest in the struggle against population are less than the disadvantages and risks
colonialism that are in-built in the urban life. Crime, riots, etc
5) None of these are some of the examples of such risks of urban life.
Camouflage = to hide somebody/something by The forces that generate conditions conducive to
making them or it like the things crime and riots are stronger in urban communities
around, or like something else than in rural areas. Urban living is more anonymous
Skirmish = a short fight between small group living. It often releases the individual from community
of soldiers etc, especially one that re straints more common in tradition-orie nte d
is not planned
societies. But more freedom from constraints and
Directions (Q. 10-12): Choose the word which
controls also provides greater freedom to deviate. And
is most nearly the SAME in meaning as the word
living in the more impersonalized, formally controlled
given in bold as used in the passage.
urban society means that regulatory orders of conduct
10. TRAJECTORY
are often directed by distant bureaucrats. The police
1) result 2) tradition
are strangers executing these prescriptions on an
3) path 4) consequence
anonymous set of subjects. Minor offences in small
5) precedence
town or village are often handled without resort to
Precedence = the condition of being more
important than somebody else and official police action. As disputable as such action
therefore coming or being dealt with may seem to be, it results in fewer recorded violations
first of the law compared to those in the big cities. Although
11. TRANSCENDING perhaps causing some decision difficulties for the
1) widening 2) reviving police in small town, formal and objective law

K KUNDAN
3) encompassing 4) surpassing enforcement is not always acceptable to the villagers.
5) piloting Urban area with mass population, greater wealth,
Encom pass = to include a large number or range more commercial establishments and more products
of things of our te chnology also provide more fre que nt
12. EMERGENCE opportunities for theft. Victims are impersonalized,
1) renaissance 2) onslaught prope rty is insure d, consume r goods in more
3) imposition 4) development abundance are vividly displayed and are more
5) rise portable. The crime rate increases despite formal moral
Onslaught = a strong or violent attack education given in schools.
Directions (Q. 13-15): Choose the word which Rampant = unchecked; existing or spreading
is most OPPOSITE in meaning of the word given everywhere in a way that cannot be
in bold as used in the passage. controlled
13. SPLITTING Conducive = making it easy, possible or likely for
1) severing 2) uniting something to happen
Anonymous = without any unusual or interesting
3) adding 4) collecting
features
5) assembling Restraint = a rule, a fact, an idea etc that limits
14. NURTURING on controls what people can do
1) weakening 2) cultivating Abundance = a large quantity that is more than
3) demolishing 4) fostering enough
5) poisoning Vivid = producing very clear pictures in your
15. SUBVERT mind
1) conquer 2) escalate 1. Which of the following would be the best title
3) create 4) emanate for the above passage?
5) strengthen 1) Crime and Punishment
Conquer = to take control of a country or city 2) Hazards of Urban Life
and its people by force 3) Lure of Village Life
Escalate = to become or make something 4) Rural-Urban Rift
greater worse, more serious etc 5) Urban Crimes and their Reasons
Emanate = to produce or show something Lure = to persuade or trick somebody to go
somewhere or to do something by
promising them a reward
572 Test of English Language

2. The passage mainly emphasises the Mould = to s trongl y infl uence the way
1) comparative account of wealth in rural and somebody’s character, opinions, etc
urban areas develop.
2) increasing crime rate in rural areas 8. It can be inferred from the passage that urban
3) need for formal moral education to be given crime can be controlled by
in schools 1) greater emphasis on moral education
4) reasons for growing crime rate in urban 2) enforcement of law by distant bureaucrats
centres as compared to that in rural areas 3) vivid display of expensive consumer goods
5) None of these 4) making e xpe nsive consume r goods le ss
3. The author thinks that risks and disadvantages portable
are 5) None of these
1) more than the gains in urban life 9. The author’s view of ‘Traditional Societies’ is best
2) almost negligible in rural life expressed by which of the following?
3) outweigh the gains of rural life 1) They provide less freedom for the individual
4) surpassed by the gains of urban life in many circumstances.
5) None of these 2) They have lower crime rates because of the
Outweigh = to be greater or more important than moral teachings in schools.
something 3) They provide inadequate freedom for personal
Surpass = to do or be better than somebody/ movements and travel.
something 4) The y do not have ade quate mode rn
4. Which of the following is a characteristic of an technology.
urban setting? 5) They are ruled and controlled by distant
1) Unreported minor crimes bureaucrats.
2) Deviation from freedom 10. According to the passage, the crime in small
3) Less forceful social control towns
4) Minimal opportunities of crime due to better 1) is less frequently re ported or dealt with
law enforcement officially
5) Fewer recorded violations of the law 2) is brought well unde r control by distant
Minimal = very small in size or amount; as small

K KUNDAN
bureaucrats
as possible 3) leads to an impersonalized style of living
5. Which of the following statements is TRUE in 4) is often dealt with objective law enforcement
the context of the passage? 5) always causes difficultie s for the police
1) The display of consumer goods is the main authorities
cause of crime. 11. Which of the following statements is NOT TRUE
2) Lack of personal contacts increases crimes in the context of the passage?
in urban areas. 1) Moral e ducation imparte d in schools is
3) Small communities have more minor crimes ineffective in checking crime rate.
than in urban centres. 2) There is less freedom in the current society
4) Urban crimes cannot be prevented. than in a traditional society.
5) Police in urban areas settle minor disputes 3) Urban are as are thickly populate d and
without official action. commercialized.
6. Which of the following inference(s) can be drawn 4) Anonymous living in urban areas may lead to
from the contents of the passage? a freedom to deviate from rules.
(A) Migration of people from rural areas to urban 5) Urban areas provide more opportunities for
centres is almost negligible. crime than rural areas do.
(B) Strangers can enforce laws in more impartial 12. According to the passage, all of the following
manner than known people can. contribute to higher crime rates in urban areas
(C) Wealth has concentrated more in urban EXCEPT.
centres than in the rural areas. 1) vivid display of consumer goods
1) A and B only 2) B and C only 2) higher standard of living
3) A and C only 4) All the three 3) urban impersonalized living
5) None of these 4) increasing population
7. The behaviour of people is generally moulded 5) inadequate police force
because of social control in Directions (Q. 13-16): Choose the word which
1) formally controlled urban societies is most nearly the SAME in meaning to the word
2) the presence of the police authorities given in bold as used in the passage.
3) an anonymous form of living 13. SUBJECTS
4) non-traditional societies 1) topics 2) people 3) crimes
5) None of these 4) rules 5) provinces
Reading Comprehension 573

14. HANDLED is a kind of joy in serving others with virtuous motives,


1) reported 2) settled 3) dispensed in sacrificing what one has for the good of others. An
4) punished 5) judged act of goodness is of itself an act of happiness.
15. DISPLAYED The secret of perfect happiness lies in renunciation.
1) exhibited 2) kept 3) rested Wealth may give us joy for a while and fame may provide
4) removed 5) sold us with fleeting excitement. But they cannot give us
16. CONDUCIVE permanent happiness. Kings have everything to make
1) prohibitive against 2) helping to reduce them happy and yet they feel unhappy. It is because
3) prone to minimize 4) tending to promote they do not practise renunciation.
5) helping to deteriorate There is a sense of joy in doing one’s work honestly
Prone = likely to suffer from something or to and efficiently. A research-worker feels joy in research
do something bad; liable and a journalist in writing. In doing one’s duty
Directions (Q. 17-20): Choose the word which since re ly, one fee ls peace of mind which is an
is most OPPOSITE in meaning to the word given important essence of happiness.
in bold as used in the passage. It is only by cultivating spirit of renunciation, self-
17. DEVIATE sacrifice, contentment and sincere work that one can
1) obviates 2) break 3) locate really be happy. The strings of misfortune spare none
4) follow 5) concentrate but they will not cow such a person.
Obviate = to remove a problem or the need for Contentment = a feel ing of happiness or
something satisfaction
18. VIVIDLY Virtue = an attractive or useful quality
1) unintentionally 2) unimpressively Inculcate = to cause somebody to learn and
3) unscrupulously 4) unwillingly remember ideas, moral principles,
5) unpopularly etc, especially by repeating them
Unscrupulous = without moral principles; not often
honest or fair; unprincipled Ascetic = related to a simple and strict way of
19. RELEASES living
Renunciation =self-denial; the act of rejecting
1) liberates 2) closes 3) confines
physical pleasures, especially for
4) provides 5) strengthens

K KUNDAN
religious reasons
20. RAMPANT Fleeting = lasting only a short time; brief
1) reluctant 2) enforced 3) uninhibited C ow = to frighten somebody in order to
4) unrestricted 5) controlled make them obey you; intimidate
Reluctant = hesitating before doing something 1. What does a contented man do?
because you do not want to do it or 1) He pursues and assimilates the basic virtues
because you are not sure that it is of life
the right thing to do.
2) He faces boldly the adversities of life
Passage 13 3) He encounters the strings of misfortunes
4) He gives up bad habits effortlessly
We all seek happiness but few, very-few, indeed, 5) None of these
get it. We are unhappy partly because we desire much Pursue = to do something or try to achieve
more than what we can hope to attain. Our countless something over a period of time
desires are hard to be satisfied. And that is what Assimilate = to fully understand an idea or some
makes us so sad in life. information so that you are able to
use it yourself.
The secret of happiness lies in the simplification
2. Which of the following is the correct chain of
of life. Simple living encourages high thinking. It leads
things, as mentioned in the passage, leading to
to contentment. Contentment gives us inner wealth,
happiness?
the wealth of the mind and of the soul. A contented
1) Contentment, high thinking, simple living,
man devotes himself to virtues like truth, beauty, love,
inner wealth
goodness, kindness and charity. By pursuing and
2) Simple living, high thinking, inner wealth,
inculcating these virtue s, a man can fe e l true
contentment
happiness.
3) High thinking, simple living, inner wealth,
I do not mean that for simplification of life, a man
contentment
should become an ascetic. The happiness of a sadhu
4) Inner wealth, simple living, contentment, high
is of a negative kind. I want positive kind of happiness.
thinking
For this I must live in the midst of life and faithfully
5) Simple living, high thinking, contentment,
carry out my responsibilities to my home and my
inner wealth
country. But all this should be done in the spirit of
3. According to the passage , the e sse nce of
selfless service. A man who wants to lead a happy
happiness lies in ______
life, should also make others happy. In making others
1) worldly desires
happy he will taste real and lasting happiness. There
574 Test of English Language

2) matching one ’s abilitie s with the work 3) Leading a simple life of an ascetic
undertaken 4) Carrying out all worldly activities in the spirit
3) avoiding all unfortunate events of selfless service
4) adopting a simple life style 5) None of these
5) following the dictates of nature
Dictate = an order or a rule that you must obey Passage 14
4. We are unhappy partly because ______
Agriculture dominates change in India through its
1) We have no inner strength
causal links with factor and product markets. It
2) We have lost moral and spiritual values
employs 60 per cent of the labour force and contributes
3) We have countless, unfulfilled desires
26 per cent of the gross domestic product. In the poorer
4) Our lives have become extremely complicated
states, its contribution to the domestic product is
5) We carry several stresses and strains
close to 40 per cent. Low productivity in agriculture
5. Which of the following statements is NOT TRUE
has led to the concentration of the poor in this sector.
in the context of the passage?
Due to the sheer size of the agricultural economy
1) One feels peace of mind in doing one’s duty
and the importance of its major products (cereals) in
sincerely
the diets of the poor, gains in agricultural productivity
2) Re nunciation is the re sult of pe rfe ct
have significant pote ntial impact on pove rty.
happiness.
Theoretically, it is possible to reduce poverty as well
3) Multiplicity of desires make us unhappy.
as expand the domestic market for industry by raising
4) Making others happy makes one happy.
labour productivity in agriculture and spreading its
5) An act of goodness is an act of happiness
gains among the low-income groups. Modelling of the
6. Which of the following is OPPOSITE in meaning
linkages between agricultural and industrial growth
to the word ‘fleeting’ as used in the passage?
has shown that a 10 per cent increase in agricultural
1) permanent
output would increase industrial output by 5 per cent
2) passing
and urban workers would benefit by both increased
3) fast
industrial employment and price deflation. However,
4) momentary
there is an asymmetry of adjustments in the demand
5) pleasing
and supply of agricultural goods. An increase in non-

K KUNDAN
Momentary = lasting for a very short time; brief
agricultural production would lead to an immediate
7. A man who wants to live a permanent happy life
incre ase in demand for inte rme diate and final
should ______
agricultural goods, whereas supply-side adjustments
1) make others happy
involving reallocation of resources and net additional
2) master the art of renunciation
investment for capacity expansion take a much longer
3) pursue wealth and fame
period. There is a widely held view that in a large
4) inculcate the virtues of life
country like India, the de mand stimulus for
5) keep a tight control on his desires
industrialisation would come mainly from agriculture
8. Which of the following is SIMILAR in meaning
with less social and economic costs.
as the word ‘cow’ as used in the passage?
Interdependencies in food and labour market are
1) spare
important for the development process. An upward
2) conquer 3) discard
shift in the food supply curve would simultaneously
4) provoke 5) mould
Discard = to get rid of something that you no result in an upward shift in the labour demand curve.
longer want or need The magnitude of the interdependence depends on
Provoke = to cause a particular reaction or have the technique of production causing the shifts in the
a particular effect food supply curve. Similarly, an upward shift in the
9. Which of the following statements is TRUE in labour supply curve shifts up the food demand curve.
the context of the passage? The extent of interdependence between the forces of
1) Human beings seldom seek happiness labour supply and food demand depends on the
2) Doing one’s work sincerely hardly makes one e mployme nt-output elasticity and the income
happy elasticity of demand for food. The recent estimate of
3) Peace of mind is tangent to happiness the employment output elasticity in agriculture is
4) Simple life is like a life of an ascetic around 0.5, income elasticity of food is in the range of
5) A happy man is also not spare d by the 0.55-0.50 and that for cereals is 0.25-0.30. The other
misfortunes important interdependency which plays a crucial role
10. What according to the passage, is a positive kind in inducing indirect employment, is that between food
of happiness? and other sectors through demand linkages. Since food
1) Keeping ones motives and feelings under accounts for a major share in the budget of the poor
control and any reduction in the food price leaves a significant
2) Love for life and a country proportion of income for other items, a lower food price
Reading Comprehension 575

stimulates employment in industrial and service (A) Urban workers get agricultural products at a
sectors. On the other hand, an increase in the food cheaper rate.
price would increase the wage costs of industrial (B) Urban workers get more job offers in the
products and hence the prices of industrial products. agricultural sector.
In the absence of adjustments through exports, it (C) Urban workers get more job offers in the
would result in demand deficiency. Clearly, the most industrial sector.
favourable situation in India is one in which labour 1) None 2) A and B only
demand outpaces its supply and food supply outpaces 3) B and Conly 4) A and C only
its demand. 5) All A, B and C
Wage rates cannot fall below a certain minimum 3. Which of the following has the same meaning
determined by the costs of subsistence living and the as the word ‘sheer’ as used in the passage?
labour supply curve turns elastic at the subsistence 1) simple 2) undiluted
wage rate. Demographic pressure cannot push the 3) mere 4) outright
wage rate below the subsistence level. People would 5) unassisted
be willing to starve rather than work unless the energy 4. Which of the following has the same meaning
expended in physical work is compensated by the as the word ‘deflation’ as used in the passage?
energy provided by food. Foodgrain price usually 1) reduction 2) index
determines the subsistence wage rate in agricultural 3) inflation 4) improvement
as we ll as in the urban informal se ctor since 5) diffusion
foodgrains account for about four-fifths of the calorie 5. Which of the following is meant by “the labour
intake of the poor. supply curve turns elastic at the subsistence
Causal = connected with the relationship wage rate” as used in the passage?
between two things, where one 1) People refuse to work at the minimum wage
causes the other to happen rate.
Stimulus = something that helps somebody/
2) People still work at the minimum wage rate.
something to develop better or more
quickly
3) People are eager to work at the minimum wage
Interdependent = that depends on each other rate.
Induce = to persuade or influence somebody 4) People have no option but to work at the

K KUNDAN
to do something minimum wage rate.
Stimulate = to make s omething devel op or 5) None of these
become more active; to encourage Eager = very interested and excited by
something something that is going to happen
Outpace = to go, rise, improve, etc faster than or about something that you want
somebody/something to do; keen
Starve = to suffer or die because you do not 6. Which of the following statements is not true
have enough food to eat in the context of the passage?
Expend = to use or spend a lot of time, money,
1) Increase in labour productivity in agriculture
energy, etc
Sheer = used to emphasise the size, degree
can reduce poverty.
or amount of something 2) Agricultural sector can provide the impetus
Deflation = a reduction in the amount of money for greater industrialisation at lower cost.
in a country’s economy so that prices 3) Increase in food supply will increase the
fall or remain the same demand for labour forces.
1. Which of the following, according to the passage, 4) Concentration of low-income group people is
signifies influence of agricultural products on relatively higher in the non-agricultural
poverty? sector.
(A) Higher labour productivity in agriculture 5) All are true
reduces poverty. Impetus = something that encourages a
(B) Agricultural products is the main constituent process or activity to develop more
of the food of the poor. quickly
(C) Agriculture output spurs industrial growth 7. Which of the following in addition to
which ultimately helps the poor. employment-output elasticity, according to the
1) A and B only 2) B and C only passage, creates indirect employment?
3) A and C only 4) All A, B and C 1) Interlinkage of demand of food and other
5) None of these sectors
Spur = to make something happen faster 2) Interdependence of forces of labour supply and
or sooner food demand
2. Which of the following, according to the passage, 3) Income elasticity of demand for food
benefits do the urban workers get from increased 4) All of these
agricultural production? 5) None of these
576 Test of English Language

8. Which of the following, according to the passage, 3) The production cost of non-agricultural
can lead to demand deficiency in India? products and services reduces.
1) Widespread import of foodgrains 4) Industrial sector can afford to employ more
2) Oversupply of agricultural products people at lower cost.
3) Increase in prices of industrial products 5) None of these
4) Foresight in gauging the demand-supply of 15. Which of the following has the same meaning
labour as the word ‘causal’ as used in the passage?
5) None of these 1) casual 2) precursor 3) effective
Foresight = the ability to predict what is likely 4) causing 5) experiencing
to happen and to use this to prepare Precursor = a person or thing that comes before
for the future somebody/something similar and
Gauge = to calcul ate something that l eads to or inf luences its
approximately development
9. Which of the following, according to the passage,
will be the result of increase in non-agricultural Passage 15
production?
A few weeks ago I ran into an old friend who is
(A) Increase in demand for non-agricultural
currently one of the mandarins deciding India’s
products
economic and financial policies. He asked, “And so,
(B) Incre ase in de mand for inte rme diate
how is IIT doing?” As one can only indulge in friendly
agricultural products
banter at such gatherings, I responded with, “Not so
(C) Increase in demand for final agricultural
well actually. Your market-friendly policies have forced
products
us to raise the fee, so we have 50% fewer PhD
1) Only A and B 2) Only B and C
applicants this year. Not batting an eyelid, he shot
3) Only A and C 4) Only B
back: “Obviously. Your PhD students don’t have any
5) All A, B and C
market value.” Taken aback, I shifted to a more
10. Which of the following is most opposite in
serious tone and tried to start a discussion on the
meaning of the word ‘interdependence’ as used
need for research in these globalised times. But he
in the passage?
had alre ady walked away. The last word on the
1) correlated 2) dependence

K KUNDAN
imperatives of the ‘market’ had been spoken.
3) independence 4) relativity
Actually, this view of higher education should not
5) interrelated
have surprised me. Worthies who look at everything
11. Which of the following is most opposite in
as consumer products classify higher education as a
meaning of the word ‘elasticity’ as used in the
‘non-merit’ good. Non-merit goods are those where
passage?
only the individual benefits from acquiring them and
1) flexibility 2) brittleness 3) rigidity
not the society as a whole. Multilateral agencies like
4) adamancy 5) peculiarity
The World Bank have too been pushing countries like
12. Which of the following, according to the passage,
India to stop subsidies to higher education.
is the cause for increase in food supply?
When Ron Brown, former US commerce secretary
1) Less demand by the industrial sector
visited India, a public meeting was organized at IIT
2) Bumper food production due to adequate
Delhi. At that meeting I asked him : “I understand
monsoon
that since the 19th century all the way up to the 1970s,
3) Change in technique of food production
most land grant and state universities in the US
4) Not mentioned in the passage
virtually provided free education to state citizens. Was
5) None of these
that good for the economy, or should they have charged
13. The wage rate of which of the following sectors
high fees in the early 20th century?” He replied, “It
is dependent on the foodgrains price?
was great for the economy. It was one of the best
(A) Agriculture sector
things that the US government did at that particular
(B) Informal urban sector
time in American history - building institutions of
(C) Organised urban sector
higher education which were accessible to the masses
1) A only 2) B only
of the people. I think it is one of the reasons why our
3) C only 4) A and B only
economy grew and prospered, one of the ways in which
5) All A, B and C
the US was able to close some of its social gaps. So
14. Why, according to the passage, lower food price
people who lived in rural areas would have the same
stimulates employment in industrial and service
kind of access to higher education as people living in
sectors?
other parts of the country. It was one of the reasons
1) Poorer people cannot afford to buy non-food
for making America strong.”
products.
Our policy-makers se e m unaware that the ir
2) Low price of food items provides the poor with
mentors in the US did not follow policies at home
extra funds to buy other products and service.
which they now prescribe for other countries. Ron
Reading Comprehension 577

Brown’s remarks summarise the importance of policy- Opine = to express a particular opinion
makers in the US place on higher education as a Echo = to be full of a sound; to send back
vehicle for upward mobility, for the poorer sectors of and repeat a sound
Eminent = famous and respected, especially in
their population. Even today, a majority of Americans
a particular profession
study in state -run institutions. Some of the se Pontificate = to give your opinions about
institutions, like Berkeley and the Universities of something in a way that shows that
Michigan, Illinois, Ohio, Wisconsin and Texas, are you think you are right
among the best in world. The annual tuition charged 1. The author of the passage seems to be a/an
from state residents (about $ 5000 a year) is about a 1) official working in economic affairs department
month’s salary paid to a lecturer. Even this fee is 2) financial advisor to Gove rnme nt or a
waived for most students. In addition, students bureaucrat in finance department
receive stipends for books, food and hostel charges. 3) social activist devoted to illiteracy eradication
The basic principle is that no student who gets programme
admission to a university should have to depend on 4) educationist in IIT or some such educational
parental support if it is not available. institution
Ron Brown’s remarks went unnoticed in India. Every 5) industrialist e mploying highly qualifie d
other day some luminary or the other opines that technocrats
universities and technical education institutions 2. What was the net tangible impact of raising fees
should increase their charges and that such education on the higher level of technological research?
should not be subsidized. Most editorials echo these 1) The number of prospective researchers was
sentiments. Eminent industrialists pontificate that reduced to almost a half.
we should run educational institutions like business 2) The market value of PhD students was almost
houses. Visiting experts from the Bank and the IMF, lost.
in their newly emerging concern for the poor, advise 3) Research studies attained a higher market
us to divert funds from higher education to primary value.
education. 4) Research became more and more relevant to
To run into somebody = to meet somebody by market demands.
chance 5) In the current globalised times, the need for

K KUNDAN
Mandarin = a powerful official of high rank,
research was less than ever.
especiall y in the civil s ervice;
bureaucrat
3. According to the author, the US policy-makers
Indulge = to allow yourself to have or do consider education as a
something that you like, especially 1) hindrance in the way to economic growth and
something that is considered bad prosperity
for you 2) means for achieving upward mobility for the
Banter = friendly remarks and jokes poor
Not bat an eyeli d = to show no surprise or 3) wastage of resources and a totally futile
embarrassment when something
exercise
unusual happens
Shoot = to move suddenly or quickly in one
4) matter of concern only for the parents of the
direction; to make somebody/ students
something move in this way 5) None of these
Take aback = to shock or surprise somebody very Hindrance = the act of making it more difficult for
much somebody to do something or for
Globalised = if something, for example a business something to happen
company, globali ses or is Futile = having no purpose because there is
globalised, it operates all round no chance of success
the world 4. Who among the following support the view that
Imperative = a thing that is very important and higher education should be free to everyone
needs immediate attention or action aspiring for it?
Worthy = an important person (A) Editors and Journalists
Prosper = to develop in a successful way; to
(B) Industrialists
be successful, especially in making
money
(C) Visiting Experts from the Bank and the IMF
Mentor = an experienced person who advises 1) A only 2) B only
and hel ps somebody with l ess 3) C only 4) All the three
experience over a period of time 5) None of these
Waive = to choose not to demand something 5. Which of following makes the policy-makers
in a particular case, even though you classify education as “non-merit” commodity?
have a legal or official right to do so; 1) The tendency of people to seek any individual
forgo.
benefits
Luminary = a person who is an expert or a great
influence in a special area or activity
578 Test of English Language

2) The attitude of giving unreasonably more been


weightage to society 1) pressurising India and other countries to stop
3) The tendency of viewing everything as mere substantial higher education
consumer product 2) insisting on discontinuance of subsidies to
4) Undue pressure from International Agencies higher education
like the World Bank, etc 3) analyzing the possibilitie s of increasing
5) None of these subsidies to higher learning
6. What was Ron Brown’s reaction to the author’s 4) emphasising on the needs of lowering fees
question on free education provided by US for higher education
universities to their citizens? Ron Brown 5) forcing countries like India to strengthen only
1) criticized the US govt for its action. industrial development
2) appreciated the author but remained non- Directions (Q. 10-12): Which of the following
committed. is MOST NEARLY THE SAME in meaning as the
3) ignored the fact and gave an ambiguous word printed in bold as used in the passage?
reaction. 10. UNAWARE
4) mentioned that the author’s information was 1) Famous 2) Ignorant 3) Familiar
not correct. 4) Unworthy 5) Negligent
5) None of these 11. WAIVED
Ambiguous = not clearly stated or defined 1) Moved 2) Charged 3) Condoned
7. The basic principle adopted by the renowned 4) Overlooked 5) Paid
State-run Universities in the US is that the Condone = to accept behaviour that is morally
students wrong or to treat it as if it were not
1) must pay the lecturer’s salary from their own serious
resources 12. MASSES
2) should earn while they learn and pay higher 1) Institutions 2) Groups
education fees 3) Students 4) Officers
3) must seek the necessary help from their 5) Parents
parents on whom they depend Directions (Q. 13-15): Which of the following

K KUNDAN
4) need not be required to depend upon their is MOST OPPOSITE in meaning of the word
parents for acquiring higher education printed in bold as used in the passage?
5) None of these 13. GLOBALISED
8. What was the outcome of the US strategy of 1) Universalised 2) Liberalised
imparting fre e unive rsity e ducation to US 3) Earthly 4) Prospering
citizens? 5) Decentralised
1) Education was easily accessible to the vast 14. PROSPERED
majority. 1) Declined 2) Progressed
2) US citize ns found it unaffordable and 3) Improved 4) Decomposed
expensive. 5) Enlightened
3) US economy suffered due to such a lop-sided 15. CONCERN
decision. 1) Worry 2) Anxiety
4) US Govt could not plug the loopholes in their 3) Sympathy 4) Indifference
economic policies. 5) Nullification
5) None of these Indifference = a lack of interest, feeling or
Impart = to pass information, knowledge, etc reaction towards somebody/
to other people; to convey something
Nullification = invalidation; negation
9. Multilateral agencies like The World Bank have
Reading Comprehension 579

Answers and explanations


Answers and explanations
Passage 1
1. 1 2. 2 3. 5 4. 3 ‘nurtured’ and ‘developed’ are synonymous.
5. 1; The meaning of the word ‘bestow’ as mentioned 12. 3 13. 5 14. 3 15. 2
in the passage is ‘to present something as a Passage 3
gift to somebody’. Hence the words ‘bestowed’ 1. 1; Ascertain the hidden meaning of the sentence:
and ‘conferred’ are synonymous. “but no one would be able to realise that a
6. 4 7. 2 8. 3 terrorist attack has occurred”. So, undoubtedly
9. 3 10. 4 the culprit’s act can be classified as a terrorist
11. 1; The meaning of the word ‘flinch’ as mentioned attack.
in the passage is ‘to make a sudden automatic 2. 2; “New terrorism has no long-term agenda but
movement because of pain, fear or shock’. Out its ruthless in its short-term intentions”. This
of the given words, meaning of the word statement from the passage supports (B).
‘wince ’ is ‘to show pain, distre ss or While, in the light of passage, (C) also seems
embarrassment by a slight movement of the suitable.
muscle s in the face ’. He nce the word 3. 5; The immediate provocation for the meeting
‘flinching’ and ‘wincing’ are synonymous. held in August 1998 has not been given among
12. 3 the options. It was the incidents of bombing
13. 2; The meaning of the word ‘dispel’ as mentioned the US embassies in Nairobi and Dar-es-
in the passage is ‘to make something go away’. Salaam.
Hence the word ‘dispelling’ and ‘accumulating’ 4. 5; Bio-attack will result in several deaths which
are antonymous. will lead to political turmoil creating social
14. 3 15. 4 unrest.

K KUNDAN
Passage 2 5. 3
1. 4 2. 2 6. 1; ‘Religious intolerance’, as cited in the last
3. 4; As give n in the passage , planning and paragraph, stands behind terrorism.
problem-solving are the most important 7. 4 8. 4 9. 1
aspects of the management, and not of the 10. 3; The meaning of the word ‘gloomy’ as mentioned
leadership. in the passage is ‘nearly dark’; ‘not well lit’.
4. 3; “Inwardly focusse d e mployee s can have Hence the word ‘gloomy’ and ‘bright’ are
difficulty seeing the very forces that present antonymous.
threat and opportunities.” This sentence of 11. 2; The meaning of the word ‘cacophony’ as
the last paragraph makes option (3) wrong. mentioned in the passage is ‘a mixture of loud
5. 1; Bure aucratic culture is against any unple asant sounds’. He nce the words
transformation; so it suppresses those who ‘cacophonous’ and ‘me lodious’ are
want to bring any change in organisations. antonymous.
Note: The meaning of the word ‘smother’ as 12. 1; The meaning of the word ‘intolerant’ is ‘not
me ntione d in the passage is ‘to pre ve nt willing to accept ideas, opinions behaviour etc.
something from developing or being noticed’; different from one’s own. Out of the given
‘to suppress of stifle something’. Hence the words, ‘forbe arance ’ me ans patie nce ,
words ‘smothe r’ and ‘suppre ss’ are restraint, tolerance etc. Hence the words
synonymous. ‘intole rance ’ and ‘forbe arance ’ are
6. 3; Planning can be defined as ‘creating a vision’, anotnymous.
which is an important aspect of management. 13. 2; The meaning of the word ‘perpetrator’ as
7. 5 mentioned in the passage is ‘a person who
8. 5; For most of this century, as a large number of commits a crime and doe s some thing
organisations were created for the first time conside re d wrong’. He nce the words
in human history, emphasis was given on ‘perpetrators’ and ‘followers’ are nearly similar
management and leadership was overlooked. in meaning.
9. 4; Managers are also bureaucrats. 14. 4; The meaning of the word ‘elusive’ is ‘tending
10. 2 to escape or disappear’; ‘difficult to find or
11. 2; The me aning of the word ‘nurture d’ as capture’. Out of the given choices, ‘baffle’
mentioned in the passage is ‘to help the me ans ‘to be too difficult or strange for
development of something’. Hence the words somebody to understand, solve or explain’.
580 Test of English Language

Hence the words ‘elusive’ and ‘baffling’ are and ‘enlist’ are synonymous.
nearly similar in meaning. 12. 3; The me aning of the word ‘e ntre nch’ as
15. 2; The meaning of the word ‘inflict’ is ‘to make mentioned in the passage is ‘to establish
somebody accept something that is unpleasant somebody/something very firmly with the
or not welcome’. Hence the words ‘inflicting’ result that the change is very difficult and
and ‘imposing’ are synonymous. unlikely’.
Passage 4 13. 4; The me aning of the word ‘conde nse’ as
mentioned in the passage is ‘to put something
1. 3; The author feels this to be the most significant
in fewer words’. Hence the words ‘condensed’
part of Lord Chancellor’s paper.
and ‘lengthened’ are antonymous.
2. 4; This is what is implie d by “gre ate r
14. 1; The meaning of the word ‘crucial’ as mentioned
transparency”.
in the paragraph is ‘very important especially
3. 2; Note the context carefully.
for its effect on something’. Out of the given
4. 5; The passage does not give any specific reason.
words, ‘trivial’ means ‘of little importance’;
5. 2
‘concerned with unimportant things’. Hence
6. 3; See the first sentence of the passage.
the words ‘crucial’ and ‘trivial’ are antonymous.
7. 4; See the last sentence of the passage.
15. 4; The word ‘resistance’ and ‘acceptance’ are
8. 1; The me aning of the word ‘je ttison’ as
anotnymous.
mentioned in the passage is ‘to abandon or
reject something that is not wanted’. Hence Passage 6
the words ‘je ttison’ and ‘sacrifice ’ are 1. 3; From the last paragraph of the given passage.
synonymous. 2. 3; From the fourth line of the second paragraph.
9. 3; “The be st pe rson available ” implie s the 3. 3 4. 1
supremacy of merit. 5. 5; In the last sentence of the second para.
10. 1 6. 4; Second and third para tells about Indian
11. 2; The meaning of the word ‘lest’ as mentioned concept of life and treatment while the last
in the passage is ‘for fear that’; ‘in order that para tells about western concept of life and
... not’. knowledge about medical science.
12. 5; When you jump from the frying-pan into the fire, 7. 5; It is clearly given in the last sentence of the

K KUNDAN
you move from a bad situation to one that is passage: “the contribution of this science in
worse. the field of non-communicable diseases is
Passage 5 remarkably poor....”
8. 2 9. 1
1. 3; Go through the last line of the first paragraph.
10. 4; The me aning of the word ‘ine vitable’ as
2. 4; While much had been said and written on
mentioned in the passage is ‘impossible to
establishing “new order”, little has actually
avoid’; ‘certain to happen’. Hence the words
been done.
‘inevitable’ and ‘avoidable’ are antonymous.
3. 5 4. 2 5. 2
Meaning of the word ‘detest’ is ‘to have a
6. 2; These are complex issues and the Milan
strong fe eling of dislike for some body/
conference would perhaps be more fruitful if
something’, therefore , ‘detestable’ means
it were to discuss such specific subjects.
‘that one detests’.
7. 1; More complicated ailments can be referred to
11. 3; The meaning of the word ‘conce rne d’ as
properly equipped centres in district towns,
me ntione d in the passage is ‘to have a
cities etc.
conne ction with or re sponsibility for
8. 2; The re is re sistance from the me dical
some thing’. Out of the give n words,
establishment which sees them as little more
‘indifferent’ means ‘having no interest in
than licensed quackery. Here reference is made
some body/some thing’; ‘not caring about
to traditional healers.
something’. Hence the words ‘concerned’ and
9. 3
‘indifferent’ are antonymous.
10. 5; The meaning of the word ‘launch’ as mentioned
12. 1; The meaning of the word ‘degenerative’ as
in the passage is ‘to put into action’; ‘to start’.
mentioned in the passage is ‘(of medical
Hence the words ‘launched’ and ‘started’ are
condition) getting or likely to get worse’. Out
synonymous.
of the given words, ‘re cuperative’ means
11. 1; The me aning of the word ‘muste r’ as
‘helping one to recuperate’. The meaning of
mentioned in the passage is ‘to succeed in
the word ‘recuperate’ is ‘to recover after being
creating a particular feeling or attitude in
ill, tired, weak etc’; ‘to regain health, energy
oneself or in other people’. Out of the given
or strength’. Hence the words ‘degenerative’
words, ‘enlist’ means ‘to obtain something as
and ‘recuperative’ are antonymous.
help, support etc.’; ‘to get somebody to provide
13. 5; The me aning of the word ‘connote ’ as
help, support etc’. Hence the words ‘muster’
me ntioned in the passage is ‘to sugge st
Reading Comprehension 581

something in addition to the main meaning’. analysis regarding foreign investment in India.
Out of the given words, ‘imply’ means ‘to 4. 1
suggest something indirectly rather than 5. 5; Comparatively though labour is inexpensive
stating it directly’. Hence the words ‘connotes’ in India, but at the same time productivity is
and ‘implies’ are synonymous. not high. Therefore, it cannot be cited as an
14. 3; The me aning of the word ‘aberration’ as advantage here.
mentioned in the passage is ‘departure form 6. 4; If fore ign inve stme nt is to be wooe d
what is normal, usual or expected, typically assiduously, we will have to meet exacting
one hat is unwelcome’. Hence the words international standards.
‘aberration’ and ‘deviation’ are synonymous. 7. 1; The author is a political commentator because
15. 2 he talks about the government policy and
Passage 7 makes various proposals regarding foreign
investment in India.
1. 3; Eminent British economists and political
8. 4
scientists have strongly attacked the tradition
9. 2; The passage reflects the views of the Japanese
of budget secrecy.
ambassador who also talks about the
2. 5; It leads to the control of public expenditure
problems faced by foreign investors in India.
in order to set realistic taxation implications.
10. 4; The meaning of the word ‘assiduous’ is ‘working
3. 2; He has presented the example of both, the
hard and the showing careful attention to
open budget system and the secret budget
detail’. Meaning of the word ‘persistent’ is
system, practised by various countries and
‘re fusing to give up’. He nce the words
has looked into all their aspects.
‘assiduously’ and ‘pe rsiste ntly’ are
4. 4 5. 5
synonymous.
6. 1; Sir Richard Clarke was the originating genius
11. 4; The meaning of the word ‘idiosyncrasy’ is ‘a
of nearly every important development in the
person’s particular way of thinking, behaving
British budgeting techniques during the last
etc that is clearly different from that of others’.
two decades.
He nce the words ‘idiosyncrasie s’ and
7. 2
‘ideologies’ are synonymous.

K KUNDAN
8. 1; The statement goes against the idea of the
12. 3; The meaning of the word ‘shoddy’ as mentioned
passage.
in the passage is ‘of poor quantity’; ‘done or
9. 4; An open public debate on budget proposals
made badly’. Out of the given words, ‘shabby’
should be he ld be fore introducing the
means ‘in poor condition through much use
appropriate bill.
of being badly cared for’. Hence the words
10. 3; The me aning of the word ‘scourge ’ as
‘shoddy’ and ‘shabby’ are synonymous.
mentioned in the passage is ‘ a whip used
13. 5; The meaning of the word ‘inducement’ as
especially formerly for punishing people’.
mentioned in the passage is ‘a thing that
Hence the words ‘scourge’ and ‘whip’ are
persuades somebody to do something’; ‘an
synonymous.
ince ntive ’. Out of the give n words,
11. 4; The meaning of the word ‘myriad’ as mentioned
‘impediment’ means ‘a person or thing that
in the passage is ‘an extremely large number’.
delays or stops the progress or movement of
Hence the words ‘myriad’ and ‘abundant’ are
something’. Hence the words ‘inducement’
synonymous.
and ‘impediment’ are antonymous.
12. 1
14. 4; The words ‘justifiable’ and ‘unreasonable’ are
13. 2; The meaning of the word flourish’ as mentioned
antonymous. The me aning of the word
in the passage is ‘to grow in a healthy way’.
‘unscrupulous’ is ‘without moral principles’;
Out of the given words ‘degenerate’ means ‘to
‘not honest or fair’.
pass into a worse physical, mental or moral
15. 3; The root word of ‘contemptuously’ is ‘contempt’.
state than one which is considered normal or
Meaning of ‘contempt’ is ‘the feeling that
desirable’. Hence the words ‘flourish’ and
somebody/something is completely worthless
‘degenerate’ are antonymous.
and cannot be respected’. Hence the words
14. 3 15. 5
‘conte mptuously’ and ‘re spe ctfully’ are
Passage 8 antonymous. The me aning of the word
1. 3; The meaning is implied in the last sentence. ‘amicable’ is ‘based on or achieved through
2. 1; The Japanese ambassador acknowledges that polite discussion and without quarreling’.
the vastness of the Indian market is a great Passage 9
induce me nt for inve stme nt in the
1. 2; Japanese business circles represented by the
manufacturing industry.
Ishikawa Mission called attention of their
3. 2; The author describes the Indian investment
Indian counterparts to what they considered
scenario in toto. He presents a comparative
to be the major impediments in India.
582 Test of English Language

2. 1; The Indian gove rnme nt put into e ffe ct through the introduction of market relations.
revolutionary reforms to remove the hurdles. 5. 2; It originated as anti-colonial struggle.
3. 4 6. 4; It was meant to deny the existence of any
4. 1; India deserves a far bigger share of world trade overarching national consciousness.
considering its vast resources. 7. 1; Colonialism was the cause be hind the
5. 3 6. 3 7. 2 8. 1 9. 1 awakening of the spirit of nationalism among
10. 5; The me aning of the word ‘ste adfast’ as the masses of a country.
mentioned in the passage is ‘firm and not 8. 3 9. 2
changing’; ‘constant’. He nce the words 10. 3; The me aning of the word ‘traje ctory’ as
‘steadfastly’ and ‘faithfully’ are synonymous. mentioned in the passage is ‘the curved path
11. 4; The me aning of the word ‘resilie nce ’ as of something that has been fired, hit, thrown
mentioned in the passage is ‘the ability of into the air, eg a missile’. Hence the words
people to recover quickly from shock, injury ‘trajectory’ and ‘path’ are synonymous.
e tc’. He nce the words ‘re silie nce ’ and 11. 4; The meaning of the word ‘transcend’ is ‘to be
‘adaptability’ are synonymous. or go beyond the normal limits of something’.
12. 2; The meaning of the word ‘prune’ is ‘to reduce He nce the words ‘transce nding’ and
the e xte nt of some thing by cutting ‘surpassing’ are synonymous.
unnecessary parts’. Out of the given words, 12. 5 13. 2 14. 3
‘trim’ means ‘to make something neat or 15. 5; The me aning of the word ‘subve rt’ as
smooth by cutting away untidy parts’. Hence mentioned in the passage is ‘to destroy the
the words ‘pruning’ and ‘trimming’ are authority of a political system, religion etc’.
synonymous. Hence the words ‘subvert’ and ‘strengthen’
13. 4; The meaning of the word ‘stiff’ as mentioned are antonymous. Meaning of the given word
in the passage is ‘to an extreme degree’; ‘very ‘emanate’ is ‘to come or flow from something/
much’. Out of the given words, ‘yielding’ means somebody or from a place’.
‘(of a substance) that can bend or move when Passage 12
pressed’; ‘soft rather that stiff’. Hence the
1. 2; The passage presents a comparative sketch
words ‘stiff’ and ‘yielding’ are antonymous.
of rural and urban life focussing on the risks

K KUNDAN
14. 2
associated with the urban life.
15. 5; The meaning of the word ‘impediment’ as
2. 4
mentioned in the passage is ‘a person or thing
3. 1; Generally, the gains of living in urban areas
that delays or stops the progress or movement
are less than the disadvantages and risks that
of some thing’. Out of the give n words,
are in-built in urban life.
‘furtherance’ means ‘the process of helping
4. 3; Urban living often releases the individual from
the progress or development of something’.
community restraints.
He nce the words ‘impe dime nt’ and
5. 2; Life in urban society is more impersonalized.
‘furtherance’ are antonymous. Meaning of the
6. 2
given word ‘compendium’ is ‘a collection of
7. 5; Traditional societies, i.e. villages.
detailed items of information especially in a
8. 4
book’.
9. 1; In ‘traditional socie tie s’ the re is always
Passage 10 community restraint over the individuals.
1. 5 2. 3 3. 2 10. 1; Minor offences in small town or village are
4. 2; During the Renaissance, education passed often handled without resort to official police
more from the clutches of the priest into the action.
hand of the prince. 11. 2
5. 3 6. 4 12. 5; All the others contribute to higher crime rates
7. 1; During the Christian era, the ecclesiastics in urban areas.
controlled the institution of education. 13. 2 14. 2 15. 1 16. 4 17. 4
8. 1 9. 4 18. 2 19. 3
10. 5; Go through the last sentence of the passage. 20. 5; Meaning of the word ‘rampant’ is ‘existing or
Passage 11 spreading everywhere in a way that cannot be
controlled’.
1. 4
2. 3; The colonial powers tried to divide people on Passage 13
linguistic and religious lines. 1. 1; A contented man devotes himself to virtues
3. 1 like truth, beauty, love goodness, kindness
4. 4; The impact of colonialism had on the whole and charity.
been in the direction of transcending localism 2. 5; Go through the second para.
and unifying supra-local economic structures 3. 4
Reading Comprehension 583

4. 3; Our countless desires which are hard to be 8. 3; Increase in the prices of industrial products,
satisfied make us sad in life. in the abse nce of adjustme nts through
5. 2; Pe rfe ct happine ss is the re sult of exports, would result in demand deficiency.
renunciation. 9. 2; An increase in non-agricultural production
6. 4; ‘Fleeting’ means ‘passing quickly, lasting only would le ad to an imme diate incre ase in
a short time’. ‘Momentary’ means ‘lasting for demand for intermediate and final agricultural
a very short time’. goods.
7. 2; It is only by cultivating the spirit of 10. 3 11. 3 12. 4
renunciation, self-sacrifice, contentment and 13. 4; Foodgrain price usually de te rmine s the
sincere work that one can really be happy. subsistence wage rate in agricultural as well
8. 5; Cow when used as verb means make somebody as in the urban informal sector.
do as one wants by frightening him; intimidate 14. 2; Since food accounts for a major share in the
somebody. Mould (Verb) means guide or budget of the poor and any reduction in the
control the de ve lopme nt of some body/ food price leaves a significant proportion of
something; shape or influence. income for other items, a lower food price
9. 2 10. 4 stimulates employment in industrial and
Passage 14 service sectors.
15. 4
1. 2; (b) The worth of agricultural products like
ce re als in the die ts of the poor have Passage 15
significant potential impact on poverty. 1. 4; Clear from the way he is linked with IIT.
(c) Poverty can be reduced by expanding 2. 1; The author clearly said to his old friend, “Your
agriculture which will also result in industrial market-friendly policies hve forced us to raise
growth. the fee, so we have 50% fewer PhD applicants
2. 4; With increase in agricultural output, the this year.”
urban workers would benefit by both increased 3. 2; This is what the author deduces from what
industrial employment and price deflation. “Ron Brown’s remarks summarise”.
3. 3 4. 5; Read the last para.

K KUNDAN
4. 1; Here the word ‘deflation’ has been used with 5. 3; Read the second sentence of the second para.
price, and means lowering or ‘reducing’ price. 6. 2
5. 5; The supply of labour can be affected at the 7. 4; Read the last sentence of the third para.
subsistence wage rate. 8. 5; 1 is more of a restatement. The real outcome
6. 4; Low productivity in agriculture has led to the was the prospering of the US economy.
concentration of the poor in this sector. 9. 2 10. 2 11. 4 12. 2 13. 5
7. 4; All the points are mentioned in the second 14. 1 15. 4
para.
Chapter 20

Blood Relation
Introduction Now consider another example,
Ex. 2: Nandini is the only daughter of Madan’s sister
Problems of this type involve analysis of certain blood Sangita’s br other . How is Nandini related to
relations and then inferring on the basis of the given Madan?
informations. Some examples of such questions are given 1) Daughter 2) Niece
below that will better illustrate our point: 3) Cousin 4) Niece or Daughter
Ex. 1: Pointing to a man in a photograph, a woman said, 5) None of these
‘His brot her ’s f at her is t he only son of my Soln.: Nandini = the only daughter of Madan’s sister
grandfather.’ How is the woman related to the Sangita’s brother
man in the photograph? = the only daughter of Madan/Madan’s
1) Mother 2) Aunt 3) Sister brother
4) Daughter 5) Grandmother = Madan’s daughter/niece
Soln.: For such type of questions, it is best to ‘back- Hence (4) is the correct answer.
track’, ie start with the last information and Ex. 3: M is brother of K. T is sister of M. J is brother of
proceed backward for example, if it is given that T. H is father of J. How is K related to H?
he is the son of the mother of my grandfather’s 1) Niece 2) Son 3) Brother
daught er , then we pr oceed wit h the last 4) Sister 5) Data Inadequate
infor mat ion: ‘gr andf at her ’s daught er ’. Soln.: To solve such types of questions, we need to draw
Grandfather’s daughter means aunt (sister of a family tree diagram. We have already discussed,
father). Now, ‘mother of my grandfather’s daughter’ in det ail in ear lier chapters, ( “Puzzle” and
becomes ‘mother of my aunt’. Mother of my aunt “Coded Relationship”) about “What is family
is my grandmother. Hence, he is the son of the tree?” and “How to draw the family tree for the
mother of my grandfather’s daughter become ‘son given relationship-string?”. Let us draw family
of my grandmother’ which is either father or tree on the basis of the information given in the
uncle. above question, we have,
Now, solve the given example, H(+)
His brother’s father is only son of my grandfather |
 His brother’s father is my father (since only (+)J — T(–) — M(+) — K (?)
son of my grandfather = my father). Now, his Obviously, sex of K is not known. Therefore, K
brother’s father is my father is either son or daughter of H. Hence option (5),
 His brother is my brother ie ‘Data inadequate’ is the correct answer.
 He is my brother.
Hence, woman is the sister of the man in the
photograph. Hence (3) is the correct answer.

Exercise–1
1. Pointing to a man in a photograph, Leesha said, ‘his photograph 2.” How is Mr ‘X’ related to the person in
mother’s only daughter is my mother’. How is Leesha photograph 2?
related to that man? 1) Grandson 2) Granddaughter
1) Nephew 2) Sister 3) Wife 3) Greatgrandson 4) Either 1 or 2
4) Niece 5) Granddaugher 5) None of these
2. Pointing to a photograph of a man, Neha said, “He 4. A man pointing to a photograph says, “The man is
has no, sisters or daughters, but his mother is the my nephew’s maternal grandfather.” How is the
only daughter of my mother.” How is the man in the person in the photograph related to his brother who
photograph related to Neha’s mother? has no other sibling?
1) Brother-in-Law 2) Grandson 1) Brother-in-law 2) Father
3) Nephew 4) Can’t be determined 3) Father-in-law 4) Can’t say
5) None of these 5) None of these
3. Mr ‘X’ shows three photographs to Mr Y. Pointing 5. Pointing to a man in the photograph, another man
towards phot ograph 1, he says, “The person in says: “He is the father of the husband of the only
photograph 1 is my uncle, who says ‘uncle’ to the daughter of my grandfather’s only son.” How is the
per son in photogr aph 2, and t he per son in man in the photograph related to the man (who
phot ogr aph 3 is called ‘uncle’ by t he person in speaks)?
524 Verbal Reasoning (Commonsense Reasoning)

1) Grandfather 2) Uncle 1) Sister-in-law 2) Mother-in-law


3) Brother-in-law 4) Father 3) Mother 4) Cousin
5) None of these 5) Data inadequate
6. Pointing to a boy, Purnima says, “He is the only son 16. Pointing to a photograph, Y says, “He is the only
of the sister of my brother.” How is the boy related to brother of the only daughter of my sister’s maternal
Purnima? grandmother.” Pointing to another photograph, X
1) Can’t be determined 2) Son says, “he is the only brother of the only daughter of
3) Nephew 4) Brother my sister’s maternal grandmother.” If among the two
5) None of these photographs, one was either of Mr X or Mr Y, and
7. Pointing to Priti, father of Rishu says, “She is the the photograph, towards which Y was pointing, was
daughter of the daughter of the wife of the only son not of Mr X, then how is Y related to Mr X?
of the grandfather of my sister.” How is Sushama 1) grandfather 2) paternal uncle
related to Priti if Sushama is the sister of Rishu’s 3) maternal uncle 4) Can’t say
father? 5) None of these
1) Can’t say 2) Mother 3) Aunt 17. Pointing to a woman in a photograph, a man says,
4) Niece 5) None of these “She is the mother-in-law of the sister-in-law of the
8. Pointing to a woman in a photograph, a man says, only sister of my son.” How is the woman related to
“She is the mother-in-law of the husband of the only the man?
granddaughter of my own mother-in-law”. How is 1) sister 2) sister-in-law
the woman related to the man? 3) mother 4) wife
1) daughter 2) wife 3) sister-in-law 5) Can’t say
4) Niece 5) daughter-in-law 18. Pointing to a man, Radhika said, “His sister is my
9. Introducing a man, a woman says, “His wife is the daughter’s brother’s mother.” How is the man related
only daughter of my father.” How is the man related to Radhika?
to the woman? 1) Brother 2) Father 3) Uncle
1) Brother 2) Father-in-law 4) Grandfather 5) Cousin
3) Maternal uncle 4) Paternal uncle 19. Pointing to a photograph, Shanti said, “She is the
5) None of these mother of my brother’s uncle’s son.” How is Shanti
10. Pointing towards a girl, a teacher said, “She is the related to the person in the photograph?
only daughter of the only son of the wife of the father- 1) aunt 2) niece 3) mother
in-law of my wife”. How is the girl related with the 4) daughter 5) None of these
teacher? 20. Pointing to a person in a photograph, a lady said,
1) Daughter 2) Niece “Her sister is the daughter of my father ’s son’s
3) Sister 4) Daughter-in-law mother.” How is the person related to the lady?
5) None of these 1) Aunt 2) Cousin 3) Sister
11. Two persons were quarrelling over their relationship 4) Mother 5) Can’t be determined
with a man in a photo. One was saying that the man 21. Pointing to a man in the photograph Sheela said
was his grandfather, and another was also saying “His father’s only daughter is sister of my father”.
the same. But they were not brothers. What was the How is sheela related to that man?
relationship between them? 1) Wife 2) Sister 3) Niece
1) Brother and sister 2) One of them was lying 4) Daughter 5) Either Niece or Daughter
3) Cousins 4) Data inadequate 22. Pointing to a lady in the photograph, Rajesh said,
5) None of these “she is my grandfather’s only sons’ mother.” How is
12. Pointing to Kedar, Veena said, “His mother’s brother the women related to Rajesh?
is the father of my son Nitin.” How is Kedar related 1) Daughter 2) Sister 3) Mother
to Veena? 4) Grandmother 5) Aunt
1) Niece 2) Aunt 3) Nephew 23. Pointing to a photograph, a woman says, “This man’s
4) Sister-in-law 5) None of these son’s sister is my mother”. How is the woman related
13. Pointing to a photograph, a man said to a woman, to the man in the photograph?
“She is one of the sisters of the son of the only son 1) Daughter 2) Mother
of your grandfather.” If the woman in the photograph 3) Granddaughter 4) Mother-in-law
is the sister of the son of the man’s father-in-law, 5) None of these
then how is the man related to the woman (with 24. A man pointing to a lady says, “Her brother is the
whom the man was talking)? father of my only son’s sister”. How is that lady
1) Husband 2) Brother 3) Brother-in-law related to the man?
4) Either 1) or 3) 5) None of these 1) Daughter 2) Sister
14. Pointing towards a photograph, Raju said, “His sister 3) Granddaughter 4) Niece
is the mother of Vinay, the son of my brother.” How 5) None of these
is Raju is related to Vinay? Raju is the father of Alok. 25. Pointing towards a person in a photograph, Anita
1) Brother 2) Uncle said, “He is the only son of the father of my sister’s
3) Nephew 4) Brother-in-law brother.” How is that person related to Anita?
5) None of these 1) Father 2) Cousin
15. A man, pointing to a photograph, says, “The lady in 3) Maternal Uncle 4) Brother
t he phot ograph is my nephew’s mat ernal 5) None of these
grandmother.” How is the lady in the photograph 26. Pointing to a woman in a photograph a man says:
related to the man’s sister who has no other sister? “She is the only daughter of the only daughter of my
K KUNDAN
Blood Relation 525

brother-in-law’s father.” How is the woman related 1) daughter 2) wife


to the man? 3) daughter-in-law 4) sister-in-law
1) wife 2) mother 3) daughter 5) None of these
4) niece 5) Can’t say 38. Pointing to a woman in a photograph a man says:
27. Pointing to a woman in a photograph a man says: “She is the grandmother of the son of my daughter-
‘She is the mother of my son’s only daughter.’ How is in-law’s mother-in-law.” How is the woman related
the woman related to the man in the photograph? to the man?
1) daughter 2) wife 1) mother 2) mother-in-law
3) mother 4) daughter-in-law 3) sister 4) wife
5) Can’t say 5) sister-in-law
28. Pointing to a man in a photograph a woman says: He 39. Pointing to a woman in the photograph a man says.
is the father of the only grandson of my father-in- “This woman is the wife of the father of my brother-
law. How is the man related to the woman? in-law.” How is the woman related to the man?
1) Husband 2) Brother 3) Son 1) mother 2) mother-in-law
4) Nephew 5) Brother-in-law 3) sister 4) sister-in-law
29. Pointing to a man in a photograph a woman says. 5) daughter
“He is the only son of the only daughter-in-law of my 40. Pointing to a woman a man says, “She is the sister-
only son’s father”. How is the man related to the in-law of the only daughter of my mother-in-law’s
woman? husband”. How is the woman related to the man?
1) Son 2) Father 3) Son-in-law 1) daughter 2) mother
4) Grandson 5) Can’t say 3) sister-in-law 4) daughter-in-law
30. Pointing to a woman in a photograph a man says, 5) sister
“She is the only daughter of my father’s only daughter- 41. Pointing to a man in the photograph a woman says:
in-law.” How is the woman related to the man? “He is the son of my sister’s mother’s husband.” How
1) wife 2) daughter is the man related to the woman?
3) daughter-in-law 4) sister 1) Brother 2) Father
5) sister-in-law 3) Uncle 4) Brother-in-law
31. Pointing to a man in a photograph a woman says: 5) Father-in-law
“He is the father of my only daughter-in-law’s father- 42. Pointing to a man in the photograph a woman says:
in-law”. What is the man to the woman? “He is the father of my daughter-in-law’s brother-in-
1) Father 2) Brother law.” How is the man related to the woman?
3) Husband 4) Brother-in-law 1) husband 2) brother
5) Father-in-law 3) brother-in-law 4) father
32. Pointing to a woman in a photograph a man says: 5) father-in-law
“She is the only daughter of my wife’s mother-in- 43. Pointing to a woman in a photograph, a man says,
law”. How is the woman related to the man? “She is the mother of my daughter-in-law’s husband’s
1) Daughter 2) Wife only sister.” How is the woman related to the man?
3) Sister 4) Sister-in-law 1) wife 2) mother 3) sister
5) Mother-in-law 4) daughter 5) sister-in-law
33. Pointing to a woman in a photograph a man says: 44. Pointing to a woman in a photograph a man says,
“She is the only daughter of my father’s mother-in- “She is the mother of the father of my wife’s only
law”. How is the woman related to the man? son.” How is the woman related to the man in the
1) daughter 2) mother question?
3) daughter-in-law 4) mother-in-law 1) mother 2) wife
5) None of these 3) daughter-in-law 4) mother-in-law
34. Pointing to a photograph a woman says: “He is the 5) Can’t say
brother of the daughter of my mother-in-law’s only 45. Pointing to a man in a photograph, a woman says,
son”. How is the man related to the woman? “He is the only son of the only daughter-in-law of my
1) Son 2) Brother only son’s father.” How is the man related to the
3) Brother-in-law 4) Nephew woman?
5) Can’t be exactly determined 1) Son 2) Father 3) Son-in-law
35. Pointing to a photograph a woman says: “He is the 4) Grandson 5) Can’t say
son of the wife of my husband’s father”. How is the 46. Pointing to a man, a woman says: “He is the brother-
man related to the woman? in-law of t he only son-in-law of my mot her’s
1) son 2) son-in-law husband.” How is the man related to the woman?
3) brother-in-law 4) brother 1) brother 2) husband 3) brother-in-law
5) None of these 4) son 5) son-in-law
36. Pointing to a man in a photograph a woman says: 47. Pointing to a person a man said; “This man is the
“He is the f ather -in-law of the wife of t he only son of my brother-in-law’s father’s only daughter.”
grandson of my own father-in-law”. How is the man How is the man related to the speaker?
related to the woman? 1) Son 2) Father
1) son 2) husband 3) cousin 3) Brother 4) Brother-in-law
4) nephew 5) son-in-law 5) None of these
37. Pointing to a woman in a photograph, a man says 48. Showing a photograph Rajeev told Shweta. “His
“She is the mother-in-law of the wife of the father of mother is the only daughter of your father.” How
my only son”. How is the woman related to the man? was Shweta related to the man in the photograph?
K KUNDAN
526 Verbal Reasoning (Commonsense Reasoning)

1) Aunt 2) Mother 3) Wife 58. Pointing to a person Rohit said ‘He is the younger of
4) Daughter 5) None of these the two brothers of the daughter of my father’s wife’.
49. Pointing to a photograph, a man said, “She is the How is the person related to Rohit?
daughter of my grandfather’s only son.” How is the 1) nephew 2) son 3) uncle
woman related to the man? 4) brother 5) father
1) Mother 2) Daughter] 59. Pointing to a woman, Anil said, ‘She is the daughter
3) Sister 4) Sister-in-law of my grandfather’s only son’s wife.’ How is Anil
5) None of these related to that girl?
50. Pointing to a woman, a man says: “She is the mother 1) Brother 2) cousin 3) Father
of my son-in-law’s brother’s only sister-in-law.” How 4) Uncle 5) Husband
is the woman related to the man? 60. Pointing to a boy in a photograph, Akhil says, “He is
1) mother 2) wife 3) sister the son of my mother’s only son.” How is Akhil related
4) mother-in-law 5) sister-in-law to that boy?
51. Pointing to a lady a man said, her mother’s husband’s 1) Uncle 2) Brother 3) Father
sister in my aunt. How is the lady related to the 4) Cousin 5) None of these
man? 61. Pointing to a boy, Meena says, “He is the son of my
1) Mother 2) Sister 3) Aunt grandfather’s only son.” How is the boy’s mother
4) Daughter 5) Granddaughter related to Meena?
52. Pointing to Rekha, Anuj said, “She is the daughter 1) Mother 2) Aunt
of my grandfather’s son.” How is Rekha related to 3) Sister 4) Data inadequate
Anuj? 5) None of these
1) Aunt 2) Sister 3) Niece 62. Pointing to Kedar, Veena said, ‘His mother’s brother
4) Mother 5) Sister-in-law is the father of my son Nitin.’ How is Kedar related
53. Pointing to a lady in the photograph, Dev Anand to Veena?
said, “This woman is my sister’s father’s son-in-law’s 1) Niece 2) Aunt
wife”. How is Dev Anand related to the woman? 3) Nephew 4) Sister-in-law
1) Mother 2) Sister 5) None of these
3) Can’t be determined 4) Wife 63. Pointing to a boy, Namrata says, “He is the son of my
5) None of these grandfather’s only child.” How is the boy related to
54. Pointing to a photograph Monica said, “He is the Namrata?
brother of the son of my grandfather’s only daughter. 1) Brother 2) Cousin
“How is the man related to Monica? 3) Uncle 4) Data inadequate
1) brother 2) cousin 3) nephew 5) None of these
4) Can’t say 5) None of these 64. Pointing to a girl, Abhishek said, “She is daughter of
55. Pointing to a joker, Madhu says, “He is the son of my the only child of my father.” How is Abhishek’s wife
father’s brother’s only sister-in-law.” How is the joker related to that girl?
related to Madhu? 1) Daughter 2) Mother 3) Aunt
1) Son 2) Brother 3) Cousin 4) Sister 5) None of these
4) Uncle 5) None of these 65. Pointing to a woman, Nirmal said, “She is the daughter
56. Pointing to a photograph Arun said, “She is the mother of my wife’s grandfather’s only child.” How is the
of my brother’s son’s wife’s daughter”. How is Arun woman related to Nirmal?
related to the lady? 1) Wife 2) Sister-in-law
1) cousin 2) aunt 3) Sister 4) Data inadequate
3) daughter-in-law 4) uncle 5) None of these
5) None of these 66. Pointing to a girl, Arun said, “She is the only daughter
57. Pointing to a man, Rohit recalled, ‘He is the son of of my grandfather’s son.” How is the girl related to
the mother of the father of my daughter.’ How is Arun?
Rohit related to the man? 1) Daughter 2) Sister
1) cousin 2) brother 3) nephew 3) Cousin Sister 4) Data inadequate
4) uncle 5) son 5) None of these

Exercise–2
1. L is mother-in-law of K, who is mother-in-law of R. 3. Pankaj is the brother of Rekha. Rekha is the wife of
How is the husband of R related to X, who is the Rahul. Rahul is the brother of Suman. What is the
father of P? It is also given that K is the daughter-in- relation of Suman to Pankaj?
law of X and P has no brothers or sisters. 1) Sister 2) Sister-in-law
1) Grandson 2) Grandfather 3) Son 3) Brother 4) Brother-in-law
4) Can’t say 5) None of these 5) No specific relation
2 . If Raju is the son of the wife of the son of the father 4. X’s mother is the mother-in-law of the father of Z. Z
of Amrendra’s father, how is Raju related to Amrendra, is the brother of Y while X is the father of M. How is
if Raju’s grandfather has no granddaughter? X related to Z?
1) Brother 2) Cousin 1) Paternal uncle 2) Maternal uncle
3) Either 1 or 2 4) Can’t say 3) Cousin 4) Grandfather
5) None of these
K KUNDAN 5) Brother-in-law
Blood Relation 527

5. Binod’s son is the son-in-law of Achyut’s father. Rani 1) father-in-law 2) mother-in-law


is Achyut’s only sister. Binod has only one grandson 3) son-in-law 4) daughter-in-law
Pramod. How is Pramod related to Rani? 5) brother-in-law
1) Father 2) Uncle 12. Kalyani is mother-in-law of Veena who is Sister-in-
3) Son 4) Brother law of Ashok. Dheeraj is father of Sudeep, the only
5) Nephews brother of Ashok. How is Kalyani related to Ashok?
6. If A is a brother of B, C is the sister of A, D is the 1) Mother-in-law 2) Aunt
brother of E, E is the daughter of B, F is the father of 3) Wife 4) Cousin
C, who is the uncle of D? 5) None of these
1) A 2) C 13. D is brother of K. M is sister of K. T is father of R,
3) B 4) None of these who is brother of M. F is mother of K. At least how
5) Can’t be determined many sons does T and F have?
7. If P is brother of Q and R is sister of Q, then how is 1) Two 2) Three
Q related to P? 3) Four 4) Data inadequate
1) Uncle 2) Father 5) None of these
3) Brother 4) Sister 14. M is sister of D. R is brother of D. F is father of M
5) Can’t be determined and T is mother of R. How is D related to T?
8. A said to B that B’s mother was the mother-in-law of 1) Brother 2) Son
A’s mother. How is A’s mother related to B’s mother? 3) Daughter 4) Data inadequate
1) Daughter-in-law 2) Mother-in-law 5) None of these
3) Sister 4) Aunt 15. K is brother of T. M is mother of K. W is brother of M.
5) Sister-in-law How is W related to T?
9. While going to his office P meets a man Q who is 1) Maternal uncle
related to P, because P has R, a son, who is married 2) Paternal uncle
to T. T is the daughter of Q. T has daughter A. How 3) Grandfather
is P related to A? 4) Data inadequate
1) Grandfather 2) Grandmother 5) None of these
3) Uncle 4) Father-in-law 16. M is sister of K. D is brother of K. F is mother of M.
5) None of these How is K related to F?
10. Ramesh says that “Rekha is the sister-in-law of the 1) Son 2) Daughter
only daughter of my mother-in-law’s husband.” How 3) Son or Daughter 4) Data inadequate
is Rekha related to Ramesh? 5) None of these
1) Sister-in-law 2) Mother 17. D is brother of B. M is brother of B. K is father of M.
3) Daughter-in-law 4) Sister T is wife of K. How is B related to T?
5) Daughter 1) Son 2) Daughter
11. A is the brother of B. C is married to D. If C is the 3) Son or Daughter 4) Data inadequate
nephew of B, how is A related to D? 5) None of these

Answers and explanations


Exercise–1

1. 4; Leesha’s mother - man’s sister, ie, Leesha is man’s 7. 1;


niece.
2 . 2; Man’s mother is the only daughter of Neha’s X’s grandfather
mother . Ther ef or e, Neha’s mot her is t he Maternal grandmother  Father of X
grandmother of man. Hence man is the grandson of Priti
of Neha’s mother.
3. 5; Mr X is the grandson of the brother of the person
in photograph 2. Mother of Priti Rishu’s father (say X)
4. 3; The man in the photograph is the father-in-law
of the man’s brother. Priti Rishu
5. 5; Only daughter of one’s grandfather’s only son 
One’s sister. Now father of the husband of one’s Now, if Sushma is the only sister of X then she
sister means father-in-law of one’s sister. Hence is the mother of Priti, otherwise aunt.
‘None of these’ is correct choice. 8. 3; The only granddaughter of the man’s own mother-
6. 1; If Purnima is the only daughter of her father then in-law means the daughter of the man’s brother-in-
the boy is her son. But, here we do not know the law.
number of sisters Purnima has. Now, the daughter’s husband’s mother-in-law
means wife of the man’s brother-in-law. Hence,
K KUNDAN the woman is man’s sister-in-law.
528 Verbal Reasoning (Commonsense Reasoning)

9. 5; The only daughter of the woman’s father is the 24. 2; “Her brother is the father of my only son’s sister
woman her self . Ther ef or e, t he man is the (= daughter)”
husband of the woman.  “Her brother is the father of my daughter (=
10. 1; The wife of the father-in-law of the person’s wife himself)”
 the mother of the person.  “Her brother is himself”
Now, the only son of the person’s mother is the  the lady in picture is her sister
person himself. 25. 4; He is the only son of the father of my sister’s
Hence, the girl is the person’s only daughter. brother (= brother)
11. 4; Here, how many sons and daughters the man in  He is the only son of the father of my brother
the photo has is not clear. Therefore we can’t (= father)
reach a definite conclusion.  He is the only son of my father (= brother)
12. 3; His mother’s brother is the father of my son 26. 3; Daughter of daughter of brother-in-law’s father =
= His mother’s brother is my husband daughter of daughter of father-in-law = daughter
= His mother is my husband’s sister of wife = daughter.
= He is the son of my husband’s sister 27. 4; mother of son’s daughter = son’s wife = daughter-
= He is my husband’s nephew in-law
13. 4; “The son of the only son of woman’s grandfather” 28. 1; Grandson of my father-in-law = my son
 the brother of the woman. Hence the woman  his father = my husband.
in the photograph is the woman’s sister or the 29. 4; my only son’s father = my husband.
woman herself with whom the man is talking. daughter-in-law of husband = daughter-in-law.
Again “the sister of the son of man’s father-in- only son of daughter-in-law = grandson.
law”  either wife or sister-in-law of the man. 30. 2; Father’s daughter-in-law = wife
Still we are uncertain that who is the wife of the daughter of wife = daughter
man but it is certain that the man is either 31. 5; daughter-in-law’s father-in-law = husband
husband or brother-in-law of the woman. husband’s father = father-in-law
14. 2; From the last part it is clear that Raju is male 32. 3; wife’s mother-in-law = mother
and Vinay is his nephew, ie Raju is uncle of daughter of mother = sister
Vinay. 33. 2; father’s mother-in-law = grandmother
15. 5; No specific relation. Because, my nephew implies daughter of grandmother = mother
either my sister’s son or my brother’s son. 34. 1; Mother-in-law’s only son = husband.
16. 3; Each was pointing towards photographs of his/ Daughter of husband = daughter.
her maternal uncle. Among the two photographs, Brother of daughter = son.
one was either of Mr X or Mr Y. It is given that 35. 5; Husband’s father = father-in-law
the photograph, towards which Y was pointing, wife of father-in-law = mother-in-law
is not of Mr X. Obviously, t he photogr aph, son of mother-in-law = husband.
towards which X was pointing is of Mr Y. Hence, 36. 2; “Only grandson of my father-in-law” = “husband”.
Mr Y is maternal uncle of Mr X. “Wife of my son” = “my daughter-in-law”.
17. 4; Sister of son = daughter. “Fat her-in-law of my daught er -in-law” =
Sister-in-law of daughter = daughter-in-law. “husband”.
Mother-in-law of daughter-in-law = wife. 37. 5; She is the mother.
18. 1; Man’s sister = Radhika’s daughter’s brother’s 38. 1; daughter-in-law’s mother-in-law = wife. Son of
mother = Radhika’s son’s mother = Radhika wife = son. Grandmother of son = mother.
 Man = Radhika’s brother 39. 2; Father of my brother-in-law = My father-in-law.
19. 2; The per son (she) = mot her of Shant i’s (my) Wife of my father-in-law = My mother-in-law.
brother’s uncle’s son = mother of Shanti’s uncle’s 40. 5; My mother-in-law’s husband = My father-in-law.
son Only daughter of my father-in-law = Wife.
= Shanti’s uncle’s wife (Since mother of son = Sister-in-law of wife = Sister.
wife) 41. 1; mother’s husband = father,
= Shanti’s aunt sister’s mother’s husband = sister’s father = father,
 Shanti = the person’s niece. son of father = brother.
2 0 . 3; My father’s son’s mother = My mother 42. 1; Daughter-in-law’s brother-in-law = son.
Daughter of my mother = My sister Father of son = husband.
Now, the problem boils down to: Her sister = my 43. 1; Husband’s only sister = sister in law,
sister daughter-in-law’s sister in law = daughter,
So she is either the lady’s sister or the lady mother of daughter = wife
herself. 44. 1; My wife’s only son = my son.
21. 5; The man in the picture can be her father or uncle, Father of my son = myself.
so she is either niece or daughter of the man. Mother of myself = my mother.
2 2 . 4; Rajesh said “she is my grandfather’s only son’s 45. 4; my only son’s father = my husband.
mother” daughter-in-law of husband = daughter-in-law.
 “she is my father’s mother” only son of daughter-in-law = grandson.
 “she is my grandmother” 46. 1; Mother’s husband = father.
23. 3; Woman says, “This man’s sister is my mother” Only son-in-law of father = husband
 “This man’s daughter is my mother” Brother-in-law of husband = brother.
 Man is grandfather (from mother’s side) 47. 1; Father’s only daughter = only sister.
So, woman is granddaughter Brother-in-law’s only sister = wife
K KUNDAN Son of wife = son.
Blood Relation 529

48. 2; Only daughter of your (Shweta’s) father  Shweta Daughter of my mother  my sister.
herself. Thus Shweta is the mother of the man Brother of my sister  myself or my brother
in the photograph. 59. 1; Anil’s gr andf ather ’s only son’s wife = Anil’s
49. 3; Grandfather’s only son  father  sister. mother, Anil’s mother’s daughter is his sister.
50. 2; A’s brother’s only sister-in-law = A’s wife. Therefore Anil is the girl’s brother.
 Son-in-law’s brother’s only sister-in-law 60. 3; Photograph is the son of Akhil’s mother’s only
= son-in-law’s wife = daughter. son.
 mother of daughter = wife. or, Photograph is the son of Akhil.
51. 2; Lady’s mother’s husband = lady’s father, lady’s or, Akhil is the father of the boy.
father’s sister = lady’s aunt. As is given, lady’s 61. 1; One’s grandfather’s only son  one’s father. And
aunt is man’s aunt, therefore, lady is man’s sister. the son of one’s father  One’s brother or oneself.
52. 2; Anuj’s grandfathers only son = Anuj’s father Hence, the mother of the boy is Meena’s mother.
Now, Anuj’s father’s daughter = Anuj’s sister = 62. 3;
Rekha. Veena
53. 2; Sister’s father means father ** *
Father’s son-in-law’s wife means sister ( ) — (  )  (  )
Hence Dev Anand’s sister. | |
54. 4; Her gr andf ather’s only daughter may be her Kedar () Nitin
mother or aunt. Hence, Kedar is Veena’s nephew.
55. 2; Brother’s only sister-in-law = Wife 63. 1; Son of Namrata’s grandfather’s only child is
Father’s wife = Mother Namrata’s brother
Son of mother = Brother 64. 2; Girl is daughter of the only child of Abhishek’s
56. 5; The lady in question is Arun’s brother’s daughter- father or, Girl is daughter of Abhishek
in-law. Thus Arun is father-in-law’s brother of Hence girl is daughter of Abhisek’s wife.
the lady. 65. 1; W oman = daught er o f Nir mal's wif e's
57. 2; Rohit’s daughter’s father  Rohit. grandfather's only child
Mother of Rohit’s daughter’s father  Rohit’s = daughter of Nirmal's wife's father
mother. = Nirmal's wife
Son other than Rohit of Rohit’s mother  Rohit’s 66. 4; Girl = the only daughter of Arun's grandfather's
brother son
58. 4; My father’s wife  my mother = the only daughter of Arun's father or uncle
= Arun's sister or cousin

Exercise–2
1. 1; X (+) L (-) F (+)
| |
P (+)  K (-) (–) C — A(+) — B
| |
H  R (-) D — E (–)
Hence, H is the grandson of X.  A is uncle of D
2 . 3; Father of the father of Amrendra  Grandfather 7. 5; Q can be either brother or sister of P, but since
of Amrendra. Now, t he wif e of t he son of sex of Q is not known, the exact relationship
Amrendra’s grandfather is either aunt or mother can’t be determined.
of Amrendra. Hence, Raju, who is the son of the 8. 1; If B’s mother is mother-in-law of A’s mother, so,
lady, is either brother or cousin. A’s mother is daughter-in-law of B’s mother.
3. 5; Pankaj (+) — Rekha (-)  Rahul (+) — Suman 9. 1;
Hence, Suman is brother of Pankaj’s brother-in-
law. Hence no direct relation can be determined. P Q
4. 2; Mother-in-law of father Son   Daughter
= (Maternal) grandmother. RT
Since X is son of Z’s maternal grandmother, X \ A / Daughter
must be Z’s maternal uncle. Note that X is a male
because he is a father. Thus P is the grandfather of A.
5. 3; It is clear from the diagram below 10. 4; Mother-in-law’s husband  Father-in-law. Father-
in-law’s only daughter  wife. Wife’s sister-in-
law  sister.
11. 1; A — B
(+)
|
C  D
(+) (–)
6. 1; If we represent the information diagrammatically
it becomes

K KUNDAN
530 Verbal Reasoning (Commonsense Reasoning)

12. 5; 15. 1;

16. 3;
13. 1;
T (+) F (-)

D(+) K M(-) R(+) 17. 3;


K T (- )
Thus, D and R certainly are sons. (+)
14. 4;
D(+) – B – M (+)

D is either son or daughter of T.

K KUNDAN
4 Practice Book on Analytical Reasoning

Chapter Two

Assumptions
What is an Assumption? that the girl won’t fail (effect) because she is very
clever (cause). Obviously it is assumed that very
An assumption is something which is assumed, supposed clever girls do not fail.
and taken for granted. When somebody says something Ex. 5: Statement:
he doesnot put everything, every aspect of his idea into Of all the TV sets manufactured in India, X brand
words. There is a lot which he leaves unsaid. That which has the largest sale.
he leaves unsaid, that which he takes for granted, may be Assumption:
defined as an assumption. The sale of all the TV sets manufactured in India
See the example given below that will better illustrate is known.
the concept of assumption. Ex planation:
Ex. 1: Statement: The assumption is valid. Here, it is claimed that
“According t o me, you should get your child of all the TV sets manufactured, X brand has the
examined by specialist doctor.” largest sale. No such claim could be made if the
Assumptions: sale figures of all brands was not known. Hence,

K
Specialist doctors are able to diagnose better than it must have been implicitly assumed in the
ordinary doctors. statement that the sale figures of all brands is
Ex planation: known.
The assumption is valid. One is advising (perhaps
his f r iend) t o get his child examined by a Some Standard Types of Assumptions (Validity
specialist doctor. Obviously, he must be assuming of a Given Assumption)
t hat specialist doctors diagnose bett er t han
or dinar y ones, ot her wise he would not have Following are the standard categories of assumptions:
advised thus.
Ex. 2: Statement:
(a) Existence / Non-existence of the subject
The book is intended to guide the layman to study This category makes a very simple assumption that
yoga in the absence of a teacher. what is being talked about must be existing. Similarly, if
Assumptions: its absence is being talked about, it must not be existing.

KUNDAN
I. A teacher of yoga may not be available to Ex. 6: Statement:
everyone. Love marriages mostly end in divorce.
II. Yoga can be learnt with the help of a book. Valid Assumptions:
Ex planation: I. Love marriages do take place.
Bot h assumpt ions ar e cor r ect . The book is II. There are cases of divorce.
int ended to t each yoga in t he absence of a Note:  Above was an example where we assume
teacher. This means that the absence of teachers existence of what  is being talked about.)
is a possibility: hence I is valid. That the book Ex. 7: Statement:
intends to teach yoga implies that II is also valid. The company will not go into profit unless a
Ex. 3: Statement: foreign-trained manager is brought.
The next meeting of the Governing Body of the Valid Assumption:
institute will be held after one year. At present there are no foreign-trained managers
Assumption: in the company.
The institute will remain in function after one Note:  Abov e  was  an example wher e we
year. assume non-exist ence of somet hing whose
Ex planation: absencc is being discussed.
The assumption is valid. The common practice
is to hold meetings of only those bodies that are (b) Adjectives
functional. So, if it is being announced that the We know that an adjective is something which denotes
next meeting will be held after one year, the a quality of the subject. Naturally then, if an adjective is
announcers must be assuming that the institute attached (unconditionally) to any subject, it must be
will remain functional after one year. assumed that “the subject does have the quality as denoted
Ex. 4: Statement: by the adjective’’. For example:
The girl is too clever to fail in the examination. Ex. 8: Statement:
Assumption: The social nature of man leads to cooperation
Very clever girls do not fail in the examination. and coordination within the society.
Ex planation: Valid Assumption:
The assumption is correct. The statement says Man is social.
Assumptions 5

Ex. 9: Statement: assumptions if any positive aspect of X is mentioned and


The bright-red sky looked beautiful enough to a course of action Y is suggested. The logic will be exactly
bring out the poet in him. on the same lines.
Valid Assumption:
The sky appears bright-red sometimes. (e) Analogy
In some cases it is concluded that, because a cause
(c) Cause-effect
leads to some effect in one type of objects, it will also lead
Some stat ement s ment ion a cause-and-ef fect to the same effect in another type of objects. This is an
relationship. The conjunctions between the clauses are example of reasoning by analogy. In such cases it is
usually ‘therefore’, ‘as’, ‘hence’, ‘thus’ etc. In all such cases assumed that “ The effect of the cause on both the species
it would be a valid assumption to say that “ this cause is similar.’’
leads to this effect”. There may be different versions of Ex. 14: Statement:
the question: sometimes the cause-effect relationship may Properly-fed and starved monkeys were made to
be explicitly stated (see Ex 10), sometimes it may be in run through maze (puzzle). It was seen that starved
the form of “because no cause, hence no effect”(see Ex 11) monkeys could not make their way fast. This
and sometimes in the form of “Although cause, yet no proves that the lower intelligence of people in
effect” (see Ex 12). poor countries is the result of malnutrition.
Ex. 10: Statement: Valid Assumption:
It r ained last night. The grounds must have The effect of malnutrition on the intelligence of
become wet. the monkeys is parallel to those on human beings.
Valid Assumption:
When it rains, grounds become wet. (f) Advertisement/notices/appeals
Ex. 11: Statement: In the cases of advertisements, notice, appeals etc.

K
As you do not have the expertise, you cannot be following assumptions will be considered valid:
selected. ( i ) An advertisement / appeal / notice does have
Valid Assumption: some effect (see Assumption I, Ex. 15, 16, 17, 18).
Expertise is essential for selection. ( i i) In case of an advertisement, that which is being
Ex. 12: Statement: highlighted is looked for and expected by the people
Although the city was under knee-deep water (see Assumption II, Ex. 15).
for four days in this monsoon, there was no ( i ii ) In case of a public-interest notice, it is the duty of
outbreak of cholera. those who issue it, to issue such notices (see
Valid Assumption: Assumption II, Ex. 18).
Water-logging usually leads to cholera. ( iv) In case of a public interest notice, what is being
advised must be beneficial for people and its non-
(d) Course of action practice harmful in some way (see Assumption III,
Sometimes a fact / report / observation / study / data IV, Ex. 18).

KUNDAN
is given followed by a suggested course of action. Let us (v) In case of an appeal, the reason for issuing it exists
call the given fact / data etc X and the suggested course (you can determine the reason using your common
of action Y. Then either some negative aspect of X is sense) (see Assumption II, Ex. 17).
mentioned and a course of action Y is suggested or some ( vi) In case of an of ficial notice, t he eff ect of it s
positive aspect of X is mentioned and a course of action Y implement at ion will be benef icial f or t he
is suggested. In the former case, ie, when some negative organisation (see Assumption II, Ex. 16).
aspect of X is mentioned, the following assumptions will Ex. 15: Statement:
be valid: Banking Services Chronicle—the only magazine that
( i ) X needs improvement. (see Assumption I, Ex. 13) gives exclusive articles on reasoning and quicker
( i i) The negat iv e aspect s of X (if mentioned) ar e mathematics. —an advertisement.
undesirable/harmful. (see Assumption II, Ex. 13) Valid Assumptions:
( i ii ) Y will improve X. (see Assumption III, Ex. 13) I. The advertisement will have some effect on
( iv) The advantages of adopting Y far outweigh the those who read it.
disadvantages (if any) of not adopting it. (see II. People look forward to exclusive articles on
Assumption IV, Ex. 13) reasoning and quicker mathematics.
Ex. 13: Statement: The working atmosphere in our Ex. 16: Statement:
public sector units can only be described as From next month onwards, it has been made
indisciplined and uncoordinated. Therefore, some compulsory for every worker to submit a daily
harsh disciplinary actions need to be taken. report. - a notice issued in a company.
Valid Assumptions: Valid Assumptions:
I. The working atmosphere of our public sector I. The notice will be read by the workers.
units is not ideal / needs improvement. II. The daily submission of reports by workers
II. Indiscipline and lack of coor dination are will prove beneficial for the company.
undesirable in any industry. Ex. 17: Statement:
III. Taking har sh disciplinar y act ion would Donate money for the earthquake victims. — an
improve the working atmosphere of our public appeal
sectors. Valid Assumptions:
IV. The benefits outweigh the disadvantages (if I. The appeal will have some effect on people.
any) of taking harsh disciplinary actions. II. Ther e has been an ear thquake and the
Similarly, you may contemplat e and evaluate t he condition of the earthquake victims is pitiable.
6 Practice Book on Analytical Reasoning

Ex. 18: Statement: revenue. Yes, when they raise prices they do try to raise
Please do not lean out of the running train. —a rices of only those goods in particular whose prices are
notice in the railway compartment lower than what could be tolerated. But even then, the
Valid Assumptions: word “Very low” in the given assumption is definitely
I. The people are likely to pay attention to this questionable.
notice.
II. It is the duty of the Railways to issue such (ii) When an Assumption Cannot be Outrightly
notices. Rejected
III. Leaning out of running trains is dangerous.
IV. Not leaning out of running trains ensures (a) Restatement
safety.
An assumption will be invalid if it is a mere restatement
Cases and Reasons wherein an Assumption — putting it in different words of the given statement.
The following examples illustrate this point:
Becomes Invalid or Incorrect Ex. 22: Statement:
Br oady, t her e may be t wo cat egor ies of inv alid Since certain sections of the society are going to
assumptions: be unhappy whenever reforms are implemented,
( i ) Where the assumption is outrightly incorrect, and there is little that can be done to prevent it except
( i i) W her e t he giv en assumpt ion does not look abandoning it.
outrightly incorrect but a close look shoWs that it Invalid Assumption:
is invalid. Some people would not like the implementation
of reforms.
(i) Wh en an As sump tion can be Outr ight ly Ex. 23: Statement:
Rejected Of all the TV sets manufactured in India, brand
See the examples given below that will illustrate the X has the largest sale.

K
concept: Invalid Assumption:
Ex. 19: Statement: No other brand of TV sets has as high a sale as
“Use aluminium—the versatile metal—for packing”. brand X.
—an advertisement (b) Obversion
Invalid Assumption:
Aluminium is the only metal used for packing. Obversion is a slightly different case of restating the
It  is  obvious by the statement that aluminium is same fact. In it, two of the trio (subject, verb, predicate)
 a versatile metal for packaging. These may be many more are changed into negative which changes the appearance
but not all would  be versatile. Where does one get any of the sentence without changing its meaning. An obverted
hint t hat aluminium is t he only met al possible for form of the statement is an invalid assumption.
packaging? Ex. 24: Statement:
Ex. 20: Statement: Friendship is beneficial.
“Get your child examined by a specialist doctor”.— Invalid Assumptions:

KUNDAN
A tells B. I. Enmity is harmful.
Invalid Assumption: II. Enmity is not beneficial.
B will not heed to A’s advice. III. Frendship is not harmful.
Generally one advises somebody with an assumption Ex. 25: Statement:
that the advice would  be  listened to. How could we “Smoking is injurious to health.” - a notice
conclude that B will not heed to A’s advice? Invalid Assumption:
Take another example to further illustrate this point. Non-smoking promotes health.
Statement:
(c) Conversion
A’s advice to B—“If you want to study Accounts, join
Institute Y.” The given assumption will be invalid if it is only a
Assumptions: converted form of the given statement.
I. Institute Y provides good Accounts education. Ex. 26: Statement:
II. B listens to A’s advice. Some / many historians harm the society by
[Here, both the assumptions are valid. If A advises B distorting facts.
to join a particular institute, A must have assumed Invalid Assumption:
that the particular institute was a good institute. Some of those who distort facts and harm society
While advising B, A must also have thought that B are historians.
would listen to A’s advice.] Ex. 27: Statement:
Ex. 21: Statement: No unexperienced fellow could be employed.
The government has increased the price of bread. Invalid Assumption:
Invalid Assumption: No one who could be employed should lack in
The price of bread was very low. experience.
Generally  prices  are  increased not beause they are
low but because these are compulsions agency that goes (d) Inference
around raising prices of everything that is cheap. In fact, The given assumption is invalid if it is an inference
all Govts try  their best to keep inflation under control derivable from the given statement. Because we know that
(that is, tï keep the prices low.) They raise prices only a statement is based on assumption but an inference is
because they are not left with  any choice and they need based an statement.
Assumptions 7

Ex. 28: Statement: reason is that I, III and IV use definitive words
Ram  went to Gaya on 27th July. Shyam  went such as ‘best’, ‘only’ and ‘definitely’. The statement
 two days after him. in the question mentions a fact that the BoP crisis
Invalid Assumption: has worsened and then makes a suggestion that
Shyam went to Gaya on 29th July. export s should be boosted. Undoubtedly, the
Ex. 29: Statement: author of the advice is assuming that exports
Religion is based primarily and mainly upon fear; should help the country overcome the BoP crisis
it is partly the fear of the unknown and partly or that exports are a good (or, say, ‘reasonably
the wish to feel that one has a saviour who will good’) solution to BoP crisis. But there is no hint
stand by in times of despair and defeat. whatsoever that exports are the only solution or
Invalid Assumption: the best solution or a definitely effective solution.
Man’s fear of the unknown and defeat makes him The above example illustrates how the use of definitive
religious. words may lend a different ‘tone’ to a sentence and how
one should be careful about them.
(e) Long-drawn conclusion
(b) Conjunctions
An assumption will be invalid if it makes too far-fetched
reasoning or long-drawn conclusion, even if it appears to When a statement consists of two clauses and the
be probably correct. clauses are  connected  by a conjunction, the nature of
The following examples illustrate the point:  the  conjunction used, goes a long way in detecting the
Ex. 30: Statement: assumpt ion t hat t he author must hav e made. Some
Religious instruction leads t o a cur iosity for significant conjunctions are : ‘because’, ‘so’, ‘therefore’,
knowledge. So all teaching should be done in ‘despite’, ‘in spite of’, ‘even after’, ‘although’, ‘as’,
religious spirit. ‘as a result of’. When one clause of the sentence mentions

K
Invalid Assumption: an event / fact / suggestion called A (let us say) and the
Curious persons are good persons. other clause of the sentence mentions another event/
Ex. 31: Statement: f act /suggest ion called B, t hen depending upon t he
The Central Excise Collectorate has begun the conjunction the following assumptions can be concluded.
exercise of smooth transition from licence system
t o simplif ied r egistr at ion system f or all the (a) A (because / as a result of ) B  It is assumed that
manufacturers. - a news item B leads to A.
Invalid Assumption: Ex. 33: Statement:
The Central Excise Collectorate  had carefully The lit er acy scenario will improve af t er the
reviewed the licence system. national awareness drive.
Valid Assumption:
Keywords National awareness drive on literacy is a good
In judging the validity of a given assumption, special means of improving the literacy rate.

KUNDAN
case should be taken of some keywords. See the difference (b) A (therefore / hence) B  It is assumed that A leads
that a single word/phrase can make. to B.
(a) Definitive Words Ex. 34: Statement:
There are some words that lend a greater degree of The r ecor d has been broken by an Indian,
emphasis — more weight — on the sentence than some therefore all Indians must be feeling very proud.
others. These words impart a kind of exclusiveness to Valid Assumption:
the sentence and thereby reduce the range or scope of the Breaking of a record by a fellow countryman makes
sentence. Some of these keywords are: ‘only’, ‘best’, other citizens proud.
‘strongest’, ‘all’, ‘definitely’, ‘certainly’etc. All these (c) A (even after / in spite of / despite) B  It is assumed
words have some kind of certainty associated with them that usually A does not occur when B occurs.
and you should be able to understand it. Consider the
following examples: Ex. 35: Statement:
Ex. 32: Statement: There was a murder last night even after the
The BoP crisis has worsened and the government police had arranged for maximum security around
should make every effort to boost exports. the area.
Assumptions: Valid Assumption:
I. Exports are the best solution to avert the BoP Arrangement of maximum security is usually
crisis. sufficient to prevent murders.
II. Exports are a reasonably good solution to the (d) Not A (even after/in spite of/despite) B  It is
BoP crisis. assumed that usually A occurs when B does.
III. Exports are the only solution to overcome
the BoP crisis. Ex. 36: Statement:
IV. The BoP crisis will definitely be averted by There was no outbreak of cholera this year in
boosting exports. spite of the continuous deposition of rain water
V. The BoP crisis will probably be averted by for four days.
boosting exports. Valid Assumption:
Ex planation: Deposition of rain water usually leads to cholera.
In t he abov e example, II and V ar e v alid
assumptions while I, III and IV are not . The
8 Practice Book on Analytical Reasoning

(c) Connotive Phrases (a) It would be highly misleading to say that ...
(b) Nothing could be farther from truth than....
Sometimes it so happens that an author would say what
(c) It is baseless to say that ....
he wants to say but you may miss that he has said any
Although the role of connotative phrases in the type of
such thing because the words that the author uses to say
questions that are asked thesedays is very limited, they
it are slightly indirect, slightly unconventional. We shall
have been mentioned so they do not escape your eyes
call these connotative or connotive phrases. For example,
whenever you come across them.
“It is true that ...” can be written as
(a) It would be correct to say that... Thumb Rule
(b) Even the most sceptic of men would agree that...
(c) It can be claimed with reasonable degree of truth ( i ) The answer-choice “either of them is implicit”
that... is very rarely correct for assumption questions.
Similarly, “It is false” can be written as: ( i i) Remember Ex. 6 to Ex. 18 and f ollow t hese
examples as rules.

Practice Exercise–1
Directions: In each question below is given a his work? Sounds ridiculous! It is the same thing
statement followed by two assumptions numbered I about a film: one must see it in the form the director
and II. An assumption is something supposed or taken wants us to see it.
for granted. You have to consider the statement and Assumptions:
the following assumptions and decide which of the I. Boards usually pressurise directors of a film to
assumptions is implicit in the statement. Give answer cut some shots.
1) if only assumption I is implicit. II. Filmmaker s should hav e t he r ight to show

K
2) if only assumption II is implicit. whatever they want.
3) if either I or II is implicit. 7. Statement: Except emer gency ser v ices like
4) if neither I nor II is implicit. ambulance, fire brigade and the police, no vehicle
5) if both I and II are implicit. should be given priority on the roads.
1. Statement: Teachers should strive to acquire learning Assumptions:
competence, develop commitment to objectives and I. There are some vehicles other than emergency
improve performance. services which are given priority on the roads.
Assumptions: II. People will not mind the disturbance in traffic
I. Nowadays, teachers are money-minded and have caused by emergency services vehicles.
strayed away from their main objective. 8. Statement: Mahatma Gandhi would have been pained
II. The future of a country depends a lot on teachers. to see that leaders in his country have been reduced
2 . Statement: Most of the defence personnel are capable to high living and simple thinking, instead of the
of choosing between right and wrong since they have other way round.

KUNDAN
basic qualifications. Assumptions:
Assumptions: I. Mahatma Gandhi is father of the nation.
I. Basic qualification is necessary to differentiate II. Mahatma Gandhi believed in simple living and
between right and wrong. high thinking.
II. Some def ence per sonnel ar e unable t o 9. Statement: The next time you pick the handset of
differentiate between right and wrong. your telephone, you may hear a pre-recorded voice
3. Statement: Mr ‘X’ has already played the best part of reminding you of your democratic right — the right
his cricketing life and now he is just passing time. to cast your vote.
Assumptions: Assumptions:
I. Mr ‘X’ should now quit cricket. I. The pre-recorded voice will increase the polling
II. Mr ‘X’ does not want to quit cricket. percentage.
4. Statement: Unless the officials do their duty honestly II. People avoid casting their votes.
and wit h the spir it t o serv e the nat ion, the 10. Statement: The Bhagavad Gita speaks of God’s
constitution of any number of safety boards amounts intervention in the affairs of human beings to restore
to nothing but paper work. righteousness.
Assumptions: Assumptions:
I. Safety boards are useless. I. Bhagavad Gita is a holy book.
II. Employees of t he depar t ment can pr event II. Human beings are guided by God.
accidents. 11. Statement: Some are forced to leave their novels just
5. Statement: In India, whenever a disaster occurs it is when they are reaching its tantalising end.
time for the government to constitute a committee. Assumptions:
Assumptions: I. Novels are very interesting and no one wants to
I. Making committees is an unnecessary exercise put it down without finishing it.
and is meant to fool the public. II. Students generally are afraid of their parents
II. No commit t ee’s r ecommendat ions has been while reading novels.
implemented in the past. 12. Statement: There is no good coaching arrangement
6. Statement: A film is no different from a poem, a story for CBSE board exam available anywhere in the
or a painting. Can one think of erasing a painting countr y. Students are forced to depend on sub-
for instance? Or, ask a poet to delete some lines of standard private tuitions of their locality.
Assumptions 9

Assumptions: 21. Statement: The management of the municipal


I. Coaching is necessary for fetching good marks corporation has come up with an ingenious idea for
in the examination. shirking work.
II. Now a days coaching is a good business. Assumptions:
13. Statement: The railways have been fudging accident I. Municipal corporation has lack of man-power.
figures for years in order to paint a rosy picture of II. The employees of municipal cor por at ion ar e
their safety performance. inefficient in their work.
Assumptions: 2 2 . Statement: People think nothing of spitting out from
I. These days railway accidents are very common. a moving bus, throwing empty Uncle chips packet
II. Passengers prefer safest mode of transportation. by the roadside and dropping banana skins from the
14. Statement: We might come from different cultures, car window and even flushing construction debris
but we are part of one world. What I found in meeting down the sewer lines.
my wife and eventually marrying her is that human Assumptions:
values are not necessarily different. I. People lack civic sense.
Assumptions: II. The speaker knows civic manner.
I. The speaker and his wife belong to two different 23. Statement: Infiltration of criminals into politics is a
countries. dangerous development and the voters should oppose
II. Gener ally, people dif f erent iat e bet ween t wo such candidates irrespective of their party labels.
cultures. Assumptions:
15. Statement: Not even a collapsing world looks dark I. Criminals can fight elections.
to a man who is about to make his fortune. II. Ev en big nat ional par t ies gr ant t icket s t o
Assumptions: criminals.
I. Entrepreneurs are ready to face any challenge 24. Statement: The big boys are getting ready — and

K
that comes their way. this battle could be more viciously fought than ever
II. The statement will only boost up the morale of before.
the listener. Assumptions:
16. Statement: We protect other living things such as I. There is a price war going on between two leading
plants and animals because they are not really any companies.
threat to us. II. People enjoy watching such battles.
Assumptions: 25. Statement: With the help of General Motors, Daewoo
I. Carnivorous animals are not a threat to us. in India could be one of the strongest players in the
II. Man is strongest and wisest among all living auto market.
creatures. Assumptions:
17. Statement: The government of India has requested I. Daewoo alone is not sufficient to be strongest in
f or a loan f r om t he Inter nat ional Bank f or the auto market.
Reconstruction and Development (IBRD) towards the II. General Motors is a multinational company.
cost of the construction works for widening the 26. Statement: There were no tickets to this show. But

KUNDAN
exist ing t wo lanes to 4/6 lanes of the Nat ional entry was restricted.
Highways. Assumptions:
Assumptions: I. The organisers know who are to be allowed in
I. IBRD will provide the loan to the government of the show.
India. II. There are ways possible other than that through
II. The number of vehicles is increasing in India. tickets to restrict the entry in the show.
18. Statement: Although our literacy rate is increasing, 27. Statement: Do not buy any gold jewellery till this
our basic beliefs have not undergone any significant Monday.
change. We do not think logically on certain matters. Assumptions:
Assumptions: I. The prices of gold jewellery is to be slashed down
I. Thinking of an individual depends on his literacy. after Monday.
II. Literate persons can think better than illiterate II. Law and order problem will not be there after
persons. Monday.
19. Statement: All astrology is fake. Some eminent people 28. Statement: “Buy the most contemporary design. That
have categorically stated that stars and planets have way you’ll be seen driving the latest. And in case
no influence on human life. you want to sell it, you will always get a good price
Assumptions: ....” — Advertisement of a car.
I. The speaker believes the statement of eminent Assumptions:
people. I. People want to buy those models of cars whose
II. Eminent people never give wrong statements. resale value is high.
20. Statement: The DTC has the lowest fares in the II. The design of a car plays an important role in
country and perhaps, as a consequence, provides purchasing it.
also the worst city bus service anywhere. 29. Statement: The dawn of the new millennium could
Assumptions: affect computers and computer-run activities.
I. The fare of buses depends on the prices of diesel Assumptions:
oils. I. The dawn of the new millennium will be sad for
II. The increased f are will improve the ser vices some people.
provided by DTC buses. II. The Y2K specialist can solve the problem.
10 Practice Book on Analytical Reasoning

30. Statement: If a doctor’s degree is your ambition, then 40. Statement: Paging indust r ies have been going
the choice of a good guidance institution is half the through tough times partly because of the fact that
job done. the industry went through a period of unsustainable
Assumptions: growth where subscribers were attracted more by the
I. Without proper guidance, it is very hard to be a gifts accompanying a pager than by the service which
doctor. it offered.
II. Institute alone can’t help a student in his success. Assumptions:
31. Statement: You need one who is as serious as you I. Gifts accompanying a product always attract large
are — and equally dedicated. number of customers.
Assumptions: II. Unsustainable growth is found harmful in the
I. Nothing can be done alone. long run.
II. The speaker is talking about a student and his 41. Statement: A brand is essentially a seller’s promise
teacher. to consistently deliver a specific set of features,
32. Statement: The skills are aplenty and the ambition benefits and services to the buyers.
and capability are already in our hands; now, we Assumptions:
need to break down the barriers. I. A branded good always gives satisfaction to the
Assumptions: buyers.
I. Skills, ambit ion and capability ar e t he pre- II. An unbranded good does possess the same set of
requisites for any major task. features as that of branded goods.
II. Breaking of barriers is very hard task. 42. Statement: From the worldwide advertising hype one
33. Statement: The gestation period for a business on would imagine that the new year 2000 will be a magic
the net is a tenth or a hundredth of that in the wand changing with a swishing wave, India and
physical world. the world.
Assumptions: Assumptions:
I. Business through net is preferable. I. One’s imagining gr eat ly depends on

K
II. Saving of time is very important in business. advertisement.
34. Statement: He continues to be busy as he was while II. There will be nothing new on the millennium
handling the premier investigating agency. eve.
Assumptions: 43. Statement: If there were posters available of these
I. Nowadays he is busy writing books. men, they would have replaced Shahrukh and Sachin
II. The speaker is talking about the ex-CBI chief. on many host el walls by now. — about t op IT
35. Statement: The Shiv Sainik is like a burning torch, professionals.
who shall burn the evil and show the path of life to Assumptions:
those struggling in darkness. I. The posters of IT professionals are not available
Assumptions: in the market.
I. Shiv Sainik is a ray of hope for the desperate. II. Nowadays top IT professionals are most popular
II. Those struggling in darkness can take the help among students.
of Shiv Sainiks. 44. Statement: The Net can be for India what oil was

KUNDAN
36. Statement: Political circles in the capital were puzzled for West Asia.
over the publication of an advertisement in some Assumptions:
newspapers. I. Most IT entrepreneurs of the world are Indians.
Assumptions: II. India can have a monopoly in IT sector in the
I. Politicians read newspapers. world market.
II. The advertisement in the newspaper was a rare 45. Statement: Have fun this new year eve, but not at
one. others’ cost. — Delhi Police
37. Statement: “You need eyes and ears everywhere.” — Assumptions:
CIA officer. I. Generally, people celebrate new year’s party on
Assumptions: other’s expenses.
I. CIA is the best detective agency of the world. II. People will enjoy the new year eve with a bang.
II. It is very hard to keep eyes and ears everywhere. 46. Statement: Reckless partying can lead to health
38. Statement: The Environmental Pollution Authority problems and you may reach a hospital straight from
hs proposed that the auto-rickshaws which clog the discotheque.
Delhi streets and pollute its air should be converted Assumptions:
to run on clean fuels. I. Heavy drinking is the main reason of health
Assumptions: problems.
I. The “clean fuels” which are accessible in the II. Sometimes dancing may lead to health problems.
market are likely to reduce pollution. 47. Statement: Live a full life. Don’t drink and drive. —
II. In the city pollution auto-rickshaw population is Delhi Traffic Police in its ad.
a significant factor. Assumptions:
39. Statement: Protection of national interest becomes I. Driving after drinking may cause accidents.
paramount in the case of MNCs in many situations. II. Some people generally drive after drinking.
Assumptions: 48. Statement: The finance ministry freshly brought out
I. Parties other than MNCs are always found prudent a huge revenue deficit because customs and excise
in protecting natinal interest. and done badly due to severe industrial slowdown.
II. MNCs are often fond misusing environmental Assumptions:
resources which is a national property. I. Excise and customs has a good stake in revenue
Assumptions 11

collection of the government. dresses t hen t hey will concent r at e mor e on


II. Customs and excise has a strong link-up wiht studies.
industrial performance. 57. Statement: “Reservation policy for SC/ST should be
49. Statement: If you are a middleman, the Internet’s extended for another ten years.” — Labour Minister.
promise of cheaper prices and faster services can Assumptions:
“disintermedi-ate” you. I. Reservation is still necessary for them to uplift
Assumptions: them socially and economically.
I. Middlemen are often found as a major hindrance II. SCs/STs are not so capable so they are given
to efficient working of the economy. reservation.
II. Middlemen have the main role of assisting the 58. Statement: “Tr affic police will be provided with
t ransact ion bet ween t he producer and the alcometers to identify drunken drivers.” — Police
consumer. chief.
50. Statement: The Net has a lot of useless sites floating Assumptions:
around and www.bigwaste.com is one of them, the I. So many accidents take place everyday because
difference being that it admits to being a big waste of of drunken drivers.
time. II. Alcometers is a machine which can identify the
Assumptions: drunken persons very easily.
I. Most of the websites claim to be useful ever when 59. Statement: Residents of XYZ colony agitated for
they are not so. irregular water supply.
II. The Net has no mechanism to distinguish useless Assumptions:
sites from useful ones. I. Agitation is a right way to get the water supply.
51. Statement: “We are in talks with the Japanese Bank II. Residents of the colony have acute problem of
for International Cooperation (JBIC) for the second water supply.

K
t ranche of Rs 1,600 cr or e.” - Delhi Met ro Rail 60. Statement: “Come and join my political campaign.”
Corporation (DMRC) chairman — A political leader to film stars.
Assumptions: Assumptions:
I. DMRC has already received the first tranche of I. People have more faith on the film stars.
the loan from JBIC. II. Film stars are big crowd-pullers.
II. Talks for receiving the first tranche of the JBIC 61. Statement: “There should be no screening of film
loan have been finalised. Kohram until further decision.” — A High Court order.
52. Statement: “In every community where we sell our Assumptions:
brands, we must remember we do not do business in I. People abide by the decision of High Court.
markets; we do business in societies.” — A marketer II. Kohram is an objectionable movie.
Assumptions: 62. Statement: “Nobel laureate Dr. Amartya Sen will be
I. Shops and markets are of no use in selling a presented a Lifetime Card for free travel.” — Chairman
brand. of Indian Airlines.
I. The understanding of social behaviour is a must Assumptions:

KUNDAN
for the marketers. I. This is a good way of advertising the company.
53. Statement: Consumers are often deceived by terms II. Dr Amartya Sen is the pride of India.
like “goods once sold will not be taken back” or the 63. Statement: “This time electronic voting machines
goods are transported at “owner’s risk”. will be used during elections. ”— Chief Election
Assumptions: Commissioner.
I. Bearing in mind these phrases, consumers are Assumptions:
reluctant to seek compensation. I. It will reduce malpractices during counting of
II. Such phrases do not provide a blanket exemption votes.
for the seller or the transporter. II. Electronic voting machine is convenient for the
54. Statement: Some days you settle down peacefully voters to use.
with your morning coffee. Then you open the papers 64. Statement: “The ElectionCommission should curtail
and the adrenaline picks in. election expenditure.” — President of India
Assumptions: Assumptions:
I. Coffee and papers are an integral part of the I. Election Commission can be directed only by the
common man’s life. President of India.
II. On some days the papers carry sensational news. II. It is the Election Commission which can curtail
55. Statement: Mr Speight with the help of some gunmen the election expenditure.
arrested the PM of Fiji. 65. Statement: The world gold production has continued
Assumptions: to grow irrespective of the price hike.
I. The Prime Minister was corrupt and was not keen Assumptions:
in the development of the country. I. Quantity of gold produced doesn’t affect its price
II. Democratic system is very weak in the country. in the market.
56. Statement: “Students are not allowed to wear western II. In case of price hike the production of gold should
dresses on the university campus.” — V.C. of a be reduced.
reputed university. 66. Statement: In t he countr y, which has ov er 70
Assumptions: t elev ision channels, hundreds of newspapers,
I. It looks vulgar and encourages more eve-teasing. thousands of magazines and a sensation through
II. If the girls will come to the college in Indian ‘dotcoms’, the demand for journalists would never
die down.
12 Practice Book on Analytical Reasoning

Assumptions: II. The speaker has witnessed the vices of the society
I. TV channels and dot com companies ar e being controlled by undisciplined leaders.
mushrooming by the day. 75. Statement: Today when there is so much talk about
II. Journalists are the pillar of our democracy. r ev ising the Const it ut ion, we hav e to consider
67. Statement: “Strict action would be taken against the whether it is the Constitution that has failed us or it
Municipal Corporation of Delhi employees if found is we who have failed the Constitution.
guilty of connivance in illegal encroachments.” — Assumptions:
The new Municipal Commissioner. I. Only the revision of Constitution will not serve
Assumptions: the intended purpose unless there is willingness
I. Some employees of MCD indulge in corruption. among the people to abide by it.
II. The warning given by Municipal Commissioner II. All is not right with governance.
will minimise corruption. 76. Statement: People of Indian Origin (PIO) maintain
68. Statement: Thick milk does not mean high-quality their cultural traditions and values even though they
milk — Issued in public interest by a reputed milk have settled comfortably in their adopted countries.
company. Assumptions:
Assumptions: I. Indian culture is flexible enough to be adjusted
I. There is misconception among the people that with any other culture.
thick milk is good. II. Indian culture is superior to other cultures.
II. Some companies are producing adulterated thick 77. Statement: DTC aut hor it ies have planned t o
milk. advertise various products on bus tickets to put them
69. Statement: Citizens and nations reveal their character to good use.
by the way they treat their elderly and the disabled. Assumptions:
Assumptions: I. Passangers will read the advertisement printed
I. Elderly and disabled are the most respectable in on tickets.
society. II. DTC authorities will get advertisements under

K
II. Elderly and disabled are the people who need this proposal.
help of the society. 78. Statement: The par ent s of st udent s ar e
70. Statement: Unemployment has increased in spite of demonstrating at Legislative Council to protest against
the Indian economy growing well. f ee hike r ev ision by t he capit al’s educat ional
Assumptions: institutions.
I. Economic growth is supposed to create more job Assumptions:
opportunities. I. The parents are unable to afford increased fee.
II. There are certain factors other than economic II. In view of large-scale protestation Legislative
growth which influence employment. Council may direct the capit al’s educat ional
71. Statement: It is not unknown in India that inquiry institutions to stop the fee hike.
reports, usually compiled with considerable care, are 79. Statement: “All the State Govts and UTs should go
shelved and forgotten as soon as the initial outrage for compulsory video-filming of the post-mortem
dies down. examinations in the cases of custodial deaths.” —

KUNDAN
Assumptions: National Human Rights Commission
I. The credibility of inquiry reports has considerably Assumptions:
gone down in India. I. Doctors sometimes give false post-mortem report
II. Inquiry reports in India are never implemented in case of custodial deaths.
up to the expectations of the victims of a crisis. II. The kith and kin of the victims do not get justice
72. Statement: India is suffering because citizens have for custodial violence.
given and taken votes based on caste/religion and 80. Statement: Income in the hands of women
not principles. contributes much more to the household food security
Assumptions: and child nutrition than the income controlled by
I. The days of value-based voting are numbered in men.
India. Assumptions:
II. The voting based on principles has the potential I. W omen pay mor e at tent ion to household
to nullify the problems created by the politics of nutrition.
caste and religion. II. Men are more interested in outside affairs.
73. Statement: “Our population is now 100 cr. Let’s have 81. Statement: “The state’s executive machinery should
small family for stronger India.”— a nasal message take all necessary measures to stop hazardous and
sent by the MTNL. poisonous ef fluent s f rom being discharged int o
Assumptions: Yamuna.” — A court order
I. Public messages sent through communication Assumptions:
media are effective tools to achieve the desired I. The water of Yamuna is contaminated.
end. II. State machinery has failed to t ake adequate
II. The runaway population growth seems to be a measures to prevent the problem of water pollution
major cause of worry for the government. in Yamuna.
74. Statement: A society is in danger when those who 82. Statement: If betting and match-fixing play a vital
have never learned to obey have been given the right r ole in cricket , which makes the game
to command. ungentlemanly and ugly, there is absolutely no use
Assumptions: of wasting precious time to watch the game of cricket
I. A good commander should be disciplined. which cheats the innocent.
Assumptions 13

Assumptions: hope that these enhanced outlays will go a long way


I. This contagious disease may spray to other games. in ensuring a safe and sound journey for the millions
II. Cricketers indulge in betting and match-fixing, of passengers.” — Railway Minister in her speech.
and are exploiting innocent public feelings. Assumptions:
83. Statement: Metropolises might provide everything I. Railways has failed to ensure adequate safety
— education, jobs and a good lifestyle — but they measures for its passengers.
scor e miser ably when it comes t o lif e’s basic II. Safety measures are likely to prevent railway
necessities. mishaps in future.
Assumptions: 92. Statement: There was a time when the rivers were
I. The speaker is talking about irregular water and pure. The earth clean, the air clear. Those days have
electricity supply. gone. But there are still ways for you to live healthy.
II. A large number of people in metros are still Assumptions:
deprived of life’s basic necessities. I. The environment affects the health of people.
84. Statement: If you lose after competing hard that’s II. People ar e r esponsible f or making t he
fine. If you lose without trying hard, that’s what environment polluted.
disappoints me. 93. Statement: It may sound harsh but it is true that
Assumptions: people no longer feel safe in the hands of the police.
I. Losing after trying hard is excusable. Assumptions:
II. Losing without effort is regrettable. I. Police are unable to protect the citizens.
85. Statement: In India that is short of nearly every II. There is no certainty of one’s life in police custody.
essential commodity, there is only one product that 94. Statement: Gov er nment s, NGOs and social
we stock in excess: ministers. organisations routinely come up with announcements
Assumptions: and inf or mat ive adv er tising t o cr eat e public

K
I. The more the number of ministers the easier it is awareness about various health threats and lifestyle-
to stock larger commodities. related diseases.
II. The strength of ministry should be reduced. Assumptions:
86. Statement: Courts are no longer cathedrals. They I. These advertisements will help to minimize the
are casinos where the throw of the dice matters. health-related problems.
Assumptions: II. Such advertisements create awareness among the
I. Nowadays, the courts do not provide justice to people.
deserving people. 95. Statement: Ideas have always been more potent than
II. The courts are not fulfilling the objective for which the actual protagonists who act them out.
they were established. Assumptions:
87. Statement: There are lessons to be learnt even in I. Famous protagonists can make any idea famous
defeats. despite their less importance.
Assumptions: II. Bot h ideas and pr otagonist s hav e t he same
I. Some people do not take their defeats seriously. importance in any play.

KUNDAN
II. The people who learn in their defeat may become 96. Statement: The NDMC is wasting money and
successful in future. valuable resources, asking people to file their property
88. Statement: The Union Govt needs to give greater returns by issuing big advertisements.
attention to, and provide larger resources for, primary Assumptions:
education and primary health. I. Nowadays people are not interested in filing their
Assumptions: property taxes.
I. Primary education and primary health are in II. Advertisements on smaller scale can also serve
deteriorating state in our country. the same purpose.
II. Primary education and pr imar y healt h ar e 97. Statement: Winners don’t do different things. They
essential for improving all-round living standards do things differently.
of people of any country. Assumptions:
89. Statement: People are bound to reject changes in I. Doing things differently matters a lot.
the basic structure of t he Constitution and any II. Slow and steady wins the race.
interference in the traditional communal harmony. 98. Statement: “If the Indian men play to potential they
Assumptions: have a good chance to make the grade this time in
I. This is a statement given by a leader of a party in Thomas Cup.” — The coach of Indian badminton
opposition. team.
II. The government in power is making a move to Assumptions:
amend the Constitution. I. Indian men have the potential to win the trophy.
90. Statement: Ov er t he year s, successiv e r ailway II. Indian men have the potential but they do not
ministers have utilised the employment potential of want to perform good.
the railways in order to boost their political careers. 99. Statement: Police-community relationship has been
Assumptions: going downhill over the years and the gap between
I. Ov er t he year s, Railways has wit nessed public expectation and police performance has been
tremendous vacancies for employment. constantly widening.
II. The political career of a political leader depends Assumptions:
upon the direct benefit given by him to the people I. Police is a part of community.
of his constituency. II. The police-community r elationship should be
91. Statement: “Safety will be the primary concern. I healthy.
14 Practice Book on Analytical Reasoning

100.Statement: If a region is crying for development, 108. Statement: “Develop a strategic plan for Internet
establish a new university, an advanced medical adoption. Do not treat the Internet any differently
institute and a centre for agricultural research there than you would treat any other crucial business
— and wait for the results. decision.” — A Manager, IBM India
Assumptions: Assumptions:
I. Only creation of new states would not speed up I. All crucial businesses need a strategic plan to
development. succeed.
II. Dev elopment can be achiev ed by spreading II. Internet is the need of the hour.
education. 109. Statement: Nowadays some people have made it a
101. Statement: For members at IIC, seek out those who business to or ganise rallies and demonstrations
hav e topped t he ser v ices exams, dist inguished agianst big projects citing environmental and social
themselves in graduate studies or have made a mark reasons.
at an early age in the arts or the media. Assumptions:
Assumptions: I. Big projects are necessary.
I. The present selection process for membership II. Environmental and social reasons attract people
has lack of transparancy. towards rallies.
II. Present members of IIC ar e not up t o the 110. Statement: Various state governments will compete
expectations of the IIC. with one another to announce cash prizes they intend
102.Statement: “Disconnect your TV cable connections” to facilitate Olympic medal-winners with.
— A maulana issued a fatwa because of thin attendance Assumptions:
during prayers at the mosque. I. Such announcements by the state governments
Assumptions: will boost the morale of players.
I. TV programmes are more popular than prayers. II. Winning medal in Olympics is a matter of pride
II. Prayer is more important than watching TVs. for the state governments.
103. Statement: The long-term health of the nation and 111. Statement: The textbook variety of the profession

K
its democratic policy should be primary concern of and practice of democracy has not worked in countries
all political parties in contrast to short-term remedies like India where ground realities are different.
which are at best pain-killers. Assumptions:
Assumptions: I. Democracy is not suitable for poor countries.
I. The short-term policy benefits the political parties II. The ground realities of different countries differ
in winning elections. from one another.
II. Only long-term democratic policy is in the best 112. Statement: “The days of ghazal can never be over.”
interests of the country. — A singer
104. Statement: No civilised state today can deliberately Assumptions:
v iolat e t he inter nat ional declar at ions and I. Aspiring singers should concentrate on ghazal
conventions and covenants relating to the minorities. rather than on pop.
Assumptions: II. Ghazal has been written off by many.
I. All civilised st at es enforce t he international 113. Statement: “Railway wagons of Eastern Railway will

KUNDAN
declarations, conventions and covenants equally. carry your message.” - ad by Eastern Railway.
II. Minorities all over the world should be given Assumptions:
protection by the concerned governments. I. Other railways are already in practice and earning
105. Statement: “A statesman or a man who has to deal profits.
with public affairs cannot ignore realities. The non- II. The railway will get responses for the above ad.
recognition of realities leads to artificial policies and 114. Statement: Order can be maintained only when law
programmes.” — JL Nehru becomes enforceable. Law implies the corpus of rules
Assumptions: or injunctions that need to be obeyed by one and all
I. Artificial policies and programmes can not provide without exception.
advantage to people. Assumptions:
II. In general, people at the helm of affairs ignore I. The acceptance of a law by the people is a must
realities and undertake populist measures. for its effectiveness.
106. Statement: In the present globalised scenario it is II. There is a section of people who are law unto
time for a total overhaul of the system. themselves.
Assumptions: 115. Statement: Sanskrit is a ‘dead’ language and its study
I. The prevailing system is not suitable for the in schools is obsolete.
necessities of a globalised economy. Assumptions:
II. Globalisation is the buzzword of the new world I. Sanskrit has no utility in our day-to-day life.
order. II. Schools should teach the students only such
107. Statement: Consumer is the focal point of economic matters as are related to what they want to do in
liber alisation, globalisat ion and ref orms. Any their lives.
manufacturing activity or service to succeed has to 116. Statement: “In India, the wheels of justice hardly
satisfy him. move.” — An American newspaper
Assumptions: Assumptions:
I. Ver y few companies ar e concer ned wit h I. Judicial process in India is dilatory, expensive,
consumer satisfaction. and beyond the reach of common people.
II. Consumer satisfaction is best investment in a II. It is necessary for a civilised society to have a
competitive economy. prompt judiciary.
Assumptions 15

117. Statement: To India and Indians, monsoon can be a 126. Statement: Keeping in mind the visual aspect of
bringer of bounty as well as harbinger of death. theatre, only selected incidents of Buddha’s life have
Assumptions: been taken.
I. The economy and the vast majority of the people Assumptions:
in India are still dependent on rain. I. The romantic aspect of Buddha’s lif e will be
II. In India, almost every year some part or the other shown in the play.
is ravaged by floods. II. Not every aspect of one’s life is worthy of being
118. Statement: So long as there is a caste-based society enacted on the stage.
in our country, there is no harm in having caste- 127. Statement: “If the play on Buddha is received well,
based organisations to look after the welfare of their we may even do a play on the life of Lord Krishna
castes. and Ram.” — A director
Assumptions: Assumptions:
I. Ther e is not hing wrong in hav ing r eligious I. If one play goes well, then others also will.
organisations to spread their ideals. II. The success of this play will be an indication of
II. Political parties cannot look after the welfare of the people’s taste.
different castes like caste-based organisations 128. Statement: In order to check incidents of fire in the
can. Walled City area, godowns of hazardous chemicals
119. Statement: After destroying the Kshatriya kings, and paper of the area will be shifted to Narela and
Parasuram asked the gods a way of penance. Ghazipur respectively.
Assumptions: Assumptions:
I. Parasuram deemed the act of destroying the kings I. Incident s of f ir e cannot occur in Nar ela or
to be a sin. Ghazipur.
II. There is a way out even after having committed II. Paper is a highly combustible material.

K
such ghastly acts. 129. Statement: “W hile weighing sweets, we do not
120. Statement: We grew up in joint families, which meant include the weight of the box which is used to carry
less space for the individual. the sweets.” — A notice in a sweet shop
Assumptions: Assumptions:
I. Big houses were unknown in the past. I. Cost price of a box is not less than the cost of
II. One has to make much adjustment in a joint any sweets in terms of the cost price of each
family. according to weight per gram.
121. Statement: Our urban young are less at ease with II. People prefer to pay only for the weight of the
their own language than with English! sweets they buy.
Assumptions: 130. Statement: “In my opinion if one is desirous of
I. One f eels most comf or table with one’s purchasing a car one should buy Mercedes only
mothertongue. because the least number of cars that are stolen each
II. English language books sell more t han their year is that of Mercedes.” —Mr X says to Mr Y.
vernacular counterparts. Assumptions:

KUNDAN
122. Statement: The average American can afford hobbies I. People who want to purchase a car are financially
that a typical middle-class Indian can only dream of. capable of buying Mercedes car.
Assumptions: II. Mercedes car is one of those cars, percentage
I. Ther e is much dif fer ence in mat er ial share of which is meagre in total cars sold in
circumstances between India and the US. India.
II. Economic prosperity offers a conducive climate 131. Statement: “People of state X were ‘forced to eat rats’
for a variety of interests to flourish. due to starvation and malnutrition.” — A public
123. Statement: Banking has traditionally been a highly- interest writ petition filed in the court by an NGO.
sought-after career because of its stability and the Assumptions:
growth prospects it offers. I. The cour t will issue a not ice to t he stat e
Assumptions: government to relieve the vast sections of the
I. People of modern times do not value stability any people of such acute distress.
longer. II. The court has the power to issue direction to the
II. People love stability but not stagnation. r espect iv e st at e to ensur e pr ot ection and
124. Statement: “Well, if we accept all the demands - preservation of human rights.
we’ll end up with 763 new states!” — A caption in a 132. Statement: “With Corp Junior Account stop worrying
cartoon about money, concentrat e on your st udies. Our
Assumptions: features are: No delays, no transaction cost ... plus
I. Demands have been voiced for the creation of the safety of dealing with a Government of India
763 states. enterprise.” — An advertisement of a Bank X
II. The demand of new st at es is hit ting t he Assumptions:
headlines. I. Most of the people have faith in dealing with a
125. Statement: “I am actually enjoying my experience in Government of India enterprise.
the forest and even my health is quite fine.” — A II. Customers want t he cheapest and t he most
message from a hostage hasslefree service.
Assumptions: 133. Statement: “When forces like the Special Operation
I. The well-wishers of the hostage may panic. Group (SOG) are set up, some excesses are bound to
II. The well-wishers of the hostage may go into a happen and some innocents suffer. But we should
frenzy. be satisfied with the success of this SOG.”—View of
16 Practice Book on Analytical Reasoning

a citizen of an area where SOG has launched its 139. Statement: “Our state does not need State Human
operation Rights Commission, therefore we have abolished State
Assumptions: Human Rights Commission.” — CM of state ‘X’
I. Seen in the larger interest of the people, agonies Assumptions:
of the innocent is a small price to pay. I. There will be no cases of violation of human rights
II. It is quite difficult for SOG to get the desired in state ‘X’.
result without any negative impact. II. State ‘X’ has alternative provision to tackle cases
134. Statement: Instead of burning the leaves, bury them of human rights violation.
in compost pits, by which it gets converted to natural 140. Statement: “We need to instil a sense of pride among
manure, making it beneficial for the soil. — A notice the people for the rich cultural heritage of the country
issued in public int er est by Depar tment of and its various regions.” — Speaker of the Lok Sabha
Environment Assumptions:
Assumptions: I. Culture is what gives meaning to our lives and
I. Whenever leaves are burnt in the open, the air identity to us as a social community.
gets laden with tiny particulate matter which II. To instil a sense of pride among the people for
raises air pollution to alarming levels, which the rich cultural heritage is almost impossible.
causes severe r espirat or y disorder s and eye 141. Statement: Faculty of a reputed institute ‘Y’ has
infections to those exposed to it. advised the students to buy Magical Book Series of
II. Benefits gained from ashes of leaves burnt are BSC Publishing Co if they do not want to miss the
not as much as the benefits gained from the opportunities of being recruited as PO in PNB.
natural manure obtained from leaves by burning Assumptions:
it. I. Students may ignore the advice and continue to
135. Statement: “A tempting cup of garma garam Georgia prepare with the study materials provided only
now awaits you at every street corner. So no matter by coaching Y.
who you are or where you go, a Georgia Vending II. Students of the institute Y have enough money

K
Machine will hand you the same clean, delicious to arrange for the books.
cup of tea in Regular, Adrak, Elaichi and Masala. 142. Statement: “Always use zebra crossings and subways
And if you’re looking for a change try the Regular, to cross the road safely.” — A notice by the City Traffic
Mocha and Cappuccino coffee. One sip will make Police
you realise why every other alternative is a mere Assumptions:
compromise!”—An advertisement I. Road safety is not an opt ion but a must for
Assumptions: pedestrians.
I. Most of the people need delicious cup of tea or II. Safety can be increased by making people aware
coffee with a change in taste. of tips regarding road safety.
II. Every person is addicted to either tea or coffee. 143. Statement: “High-tension wire can be extremely
136. Statement: “If you ask me about the daunting dangerous. Keep a safe distance from them.” — A
challenges t hat I face, I would say t hat my notice by Delhi Transco Limited
government’s first priority is to improve the existing Assumptions:

KUNDAN
law and order situation. Then follows the issue of I. High-t ension wir es ar e laid in complet e
prices of commodities.”—Mr Y, a newly appointed accordance with the law along routes that are
PM of country X legally approved by the concerned authorities.
Assumptions: II. Buildings are sometimes constructed near high-
I. If a citizen of country X can sleep peacefully, he/ tension lines after they have been laid, thereby
she can then think of providing food to his/her reducing the minimum required safe distance to
family, ponder over education and move about high tension wires.
freely in the country. 144. Statement: “Honourable citizens of state, ‘X’ please
II. Prices of the commodities affect the common man rise and rat on your power-thrieving neighbour. You
greatly. could win a motorcycle, a colour TV set or a washing
137. Statement: “Human rights are only for human beings, machine.” — UPPCL
not for terrorists. It is an insult to human rights to Assumptions:
protest on behalf of those found guilty of terrorism.” I. In order to get these gadgets, people of state ‘X’
— Mr X will put , ev en t heir good r elat ion wit h t he
Assumptions: neighbour on stake, and inform the UPPCL, if
I. Terrorists do not deserve sympathy. their neigbbour indulges in power-thieving.
II. Act of terrorism is against humanity. II. Cost on initiative to encourage citizens will be
138. Statement: “The finding of ‘higher resistance’ in less than the increase in the revenue through
malarial parasites was also significant as the analysis such incentives.
was done using t he moder n technique of DNA 145. Statement: “Irrespective of who comes to power,
sequencing. Although the sample size was only 50, what is of prime importance is to heal the badly
there could be no doubt about the results.” — A shaken lives of t he people of stat e ‘X’ wit h t he
scientist assurance that t hings would not go haywir e in
Assumptions: future.” — A citizen of state X
I. Modern techniques are more trustworthy than Assumptions:
traditional or obsolete techniques. I. The tenets of good governance and development
II. Lesser the size of the sample, higher the chances must form the bedrock of the party that forms the
of reliability. next government in the state.
Assumptions 17

II. To concentrate on the prime needs of the citizen Assumptions:


is the only way by which the image of tarnished I. Production under rigorous quality control regime
state ‘X’ can be improved. gives trustworthy results.
146. Statement: “There should not be any delay in filing II. BIS has been entrusted with the responsibility
a complaint. If a policeman is not helpful or harasses to set quality benchmarks.
a woman, the victim should note his name and belt 153. Statement: By 2010, India will be the country with
number and send a complaint to senior officials the greatest number of heart patients.
specifying t he place wher e he was on duty.” — Assumptions:
Women’s cell lawyer I. The number of heart patients all over the world
Assumptions: can be found out.
I. Most of the victims are literate. II. By 2010, India will be the most populated country
II. Senior off icials will take disciplinar y act ion in the world.
against the guilty policemen. 154. Statement: Unlike other known forms of threat, AIDS
147. Statement: “Buy durable pillows of company ‘X’. The spreads it s t entacles slowly but steadily. When
pillows have been made from 100% imported downs billions of dollars are being pumped in to counter
and feather; have the natural property to take the terrorism, such a seriousness is not being observed
contour and shape of your head, neck and shoulder against this sure killer. — Comments of a citizen
while you sleep; and are very popular abroad as well Assumptions:
as in the domestic market.” — An advertisement I. Government’s perception of AIDS requires to
Assumptions: undergo a sea change.
I. Immense popularity of a product is a sign of the II. AIDS poses more threats to human life than
good quality of a product and its usefulness. terrorism.
II. People desire for blissful sleep that keeps them 155. Statement: “When Mr X and Mr Y stepped out for

K
ticking for the rest of the day. toss on the ground it was evident that one of them
148. Statement: “Oil the wheels that India moves on” is was in positive frame of mind while the other was
the cry of bicycle manufacturers in the country. t ense, f earing def eat and eliminat ion fr om t he
Assumptions: tournament.”—An ex-captain of a cricket team
I. The bicycle and components industry is a priority Assumptions:
focus area possessing capability of thrusting I. Panic always sets in when a team is playing after
export. a spate of defeats.
II. Bicycle is the most affordable mode of transport. II. There are some days when a player achieves
149. Statement: “No representative is authorised to collect magical heights, when he is unstoppable and
cash. Do not pay cash to anybody. All subscribers when every ot her player in t he arena looks
ar e r equested to make payment s only t hr ough woefully inadequate.
cheques/DD in f av our of t he company. ”— An 156. Statement: “Mail your grievance and confidential
instruction to subscribers by company X information to the commissioner of police.” — Request
Assumptions: of city police to citizens

KUNDAN
I. If it is not said explicitly, the subscriber may Assumptions:
claim for their payments in cash in case any I. All categories of people, be it the poor or the senior
irregularities are made by the representatives. citizens, might have grievances.
II. Representatives of company X are not trustworthy. II. People have blind faith in the efficiency of flying
150. Statement: “On this auspicious occasion of Republic squad of vigilance branch.
Day let us take the resolution to help the drought- 157. Statement: “Beware! Recycled coloured plastic bags
affected 4.3 crore population and 4.5 crore livestock contain harmful colour pigments, which, on coming
of the state.”—An appeal by the CM of state X to its in contact with food particles, make food unfit for
citizens consumpt ion, result ing in sev er e food poison,
Assumptions: allergies and in ext reme cases ev en death. ”—A
I. Citizens’ par ticipation will ensure pr ide and scientist
respect of all sections of society as well as overall Assumptions:
development of the state. I. Non-biodegradability nature makes plastic bags
II. Miseries of people affected by drought will be an environmental hazard.
lessened through people’s participation. II. Use of plastic bags is harmful and has cascading
151. Statement: Better understanding and cooperation effects on human life.
among the exporters is needed. Effort should be to 158. Statement: “In view of a likely fiscal deficit of around
complement each other, rather than competing among five per cent, there is no denying the need for a
themselves. — View of an agriculturist consensus to trim government expenditure.” — View
Assumptions: of Mr X
I. Exporters stand to lose if they act as competitors. Assumptions:
II. Farmer-industry co-operation can work wonders. I. Trimming government expenditure is an effective
152. Statement: Bottled water companies ‘X’ and ‘Y’ ruled tool to curb fiscal deficit.
out any contamination of their products, saying these II. Tr imming government expenditure is not an
had been produced under rigorous quality control effective tool to curb fiscal deficit.
regime meeting all standards set by the Bureau of 159. Statement: India’s dismal performances in the World
Indian Standards (BIS). Cup notwithstanding, cricket betting is on in full
swing in the country. — A news
18 Practice Book on Analytical Reasoning

Assumptions: 169. Statement: “Policies and programmes and seminars


I. Dismal performances of India discourages betting cannot change the women’s status. There is a greater
business all over the world. need for attitudinal change in the society towards
II. Bookies are making merry. working women.” — View of Mr X
160. Statement: I heard the news as soon as I woke up Assumptions:
in the morning. I felt shattered. I. It is possible to make change in the attitude of
Assumptions: the society towards working women.
I. The incident took place before I woke up. II. Status of the working women is not satisfactory.
II. The news was shocking. 170. Statement: “Government of State X’s step, according
161. Statement: “God bless all the parents whose children to which it has been decided to provide cooked meal
passed away in the Yamuna waters.” — A student. to the students of all primary schools of the states,
Assumptions: is not a proper and judicious step.”—Criticism by a
I. The student believes in God. person
II. Some children have died by drowning. Assumptions:
162. Statement: “I think illiterate and drunk drivers I. Students may hesitate to eat the cooked meal
should not be employed, at least for children.” — provided by the Government.
says X. II. The cooked meal served to the students may be
Assumptions: hazardous for the health of students.
I. Literacy helps in driving. 171. Statement: “Although no war was witnessed during
II. Children have a status equal to the adults. the tenure of prime ministership of Mr X, a large
163. Statement: Owing to urgent repair on a main line number of top bravery medals were conferred upon
from the Wazirabad waterworks, water supply will many cops in the name of curbing terrorism and
be at low pressure on Thursday. eliminating terrorists of different organisations
Assumptions: belonging to our own states.” — View of a citizen
I. Repair impedes water supply. Assumptions:

K
II. Water can be had even on low pressure. I. It is disgraceful of cops to name the act of killing
164. Statement: Another strange thing I noticed last night of terrorists of our own country as an act of
was that the lights of the drawing room were on. bravery.
Assumptions: II. A war is less harmful for a country than menace
I. A thief must have come in. of terrorism.
II. Usually, the drawing room lights are off at night. 172. Statement: “Completely eliminat ing the say of
165. Statement: “Women should be given v ocat ional executive is not acceptable; merit, ability, competence,
training in their work.” — An advice integrity and suitability of the candidate alone are
Assumptions: not enough for appointment of High Court Judges.”
I. Vocational training may increase efficiency of an — A journalist
individual. Assumptions:
II. Women are likely to get benefited from vocational I. A person’s social outlook, concern for public
training. interest and promotion of equality and his/her

KUNDAN
166. Statement: “People who swindle crores of taxpayers’ political outlook are also equally important.
money go scot-free while petty thieves, who probably II. Execut iv e consult ation will ensur e gr eat er
r ob out of desper ation, ar e given thir d-degr ee transparency of the appointment.
punishment and ar e of ten vict ims of cust odial 173. Statement: “Never use obvious PIN like your car
deaths.” — View of Mr X registration no., birthdate, telephone no., etc. Change
Assumptions: your pin every three months.” — An instruction to
I. All persons should be equal before law. customers of a bank who enjoy ATM facilities
II. Punishment should be giv en t o t he culpr it Assumptions:
accor ding t o t he degree of the inf ringement I. ATM cards issued by a bank to customers might
committed by the culprit. be lost by some customers.
167. Statement: “Look under your seat. There could be a II. Some people may try to use ATM cards of others
bomb. Raise alarm. Earn r eward.” — A writt en clandestinely to withdraw money.
instruction to passengers of a bus 174. Statement: “We pr ov ide subsidised t ea t o t he
Assumptions: labourers — pay only Rs 2 instead of Rs 2.50 and get
I. Passengers will read the instruction and may a cuppa of tea.” — An advertisement
abide by it. Assumptions:
II. Passenger s’ par t icipation may ensur e mor e I. Labourers may afford a cup of tea at Rs 2 per
security. cuppa.
168. Statement: The Government X has proposed to create II. Subsidised prices attract customers.
two new service cadres of Rural Development Service 175. Statement: Dowry system can not be eradicated from
and Rural Engineering Service to strengthen the our society unless we change the people’s mindset.
Panchayati Raj model. Assumptions:
Assumptions: I. Existence of dowry system is not desirable in a
I. New cadres may help implement the Government’s society.
schemes for empowerment of rural communities. II. It is possible to bring a radical change in people’s
II. Ther e may be some pr act ical dif ficult ies in mindset.
implementation of various schemes launched by 176. Statement: “No war, yet 1874 Indian Armymen killed
the Government X. or hurt during the 10-month forward deployment
Assumptions 19

along the Indo-Pak border last year.” — Opposition Assumptions:


leader I. Magisterial inquiry may be able to reveal the cause
Assumptions: of death of the man in the police custody.
I. Our Armymen have to soldier on without even II. Transferring the entire staff may pave the way to
t he necessit ies like decent helmet s, pr oper hold free and fair enquiry.
webbing or bullet-proof jackets. 183. Statement: “Bring alive your education dreams. Avail
II. The casualt ies during t he 10 mont hs hav e loan today and pay as your start earning.” — An
surpassed the estimated limit of casualties along advertisement of a bank XYZ
the Indo-Pak border. Assumptions:
177. Statement: Instead of limiting capital punishment I. Most of t he st udent s who need loan, prefer
to the rarest of the rare cases, it should be made flexible repayment options besides no processing
mandator y f or cr imes like mur der , r ape, drug fees or service charges.
trafficking, child molestation and all anti-national II. Students who need loans prefer loans without
activities. If a person does not respect the law, let collateral security.
him at least fear it. 184. Statement: In an attempt to make the conditional
Assumptions: access system “consumer-friendly”, the government
I. When a man is sentenced to death for whatever issued a notification asking cable operators to declare
crime he has committed, he should at least die a pay channel rates by June 15.
painless death. Assumptions:
II. A person convicted of a heinous crime deserves I. Cable operators may not be able to declare pay
no kindness. channel rates by June 15 due to lack of adequate
178. Statement: “It is tall claims that our state X is time.
pr ogr essing on indust r ial f ront . Rising II. Everyone has their own business compulsions

K
unemployment by leaps and bounds is enough to and wish to test their own market.
collaborate it.” — View of a politician of state X 185. Statement: “Despite a draconian crackdown against
Assumptions: software pirates announced by country X last year,
I. If the state were progressing on the industrial the country remains the world’s third largest market
f r ont it should have been ref lect ed in t he for bootleg computer programs.” — A journalist
unemployment situation. Assumptions:
II. Progress on industrial front reduces financial I. Computer pir acy is declining globally but
crunch. increasing in country X.
179. Statement: In a bid to discourage cattle owners from II. A draconian crackdown against software pirates
letting their animals loose on the roads the Municipal may bring the bootleggers to heel.
Corporation of City X has enhanced the fine from Rs 186. Statement: “Problems with your spouse and too little
4000 to Rs 5000. time to hash things out? Online couple therapy may
Assumptions: be for you.” — An advertisement
I. Increase in fine may decrease the stray cattle Assumptions:

KUNDAN
menace. I. People may have faith in online therapy.
II. The subsequent fine is quite a large sum and it II. Couples on the run expect comparable results
should prompt the owner to get his cattle back as from online “chats” mediated by a professional
early as possible. and face-to-face therapy on a psychologist’s couch.
180. Statement: “Learn to solve Quantitative Aptitude in 187. Statement: “If you see any street lights on during
3 seconds besides Dat a Inter pr et at ion (without the day, or notice pilferage of electricity, do inform
written steps) by our experts or take back Rs 10,000/ us.” — A notice by NDPL to common people
- as penalty.” — An advertisement of XYZ Coaching Assumptions:
Institute I. W atching TV t oget her and using one
Assumptions: fan/cooler/AC will help conserve electricity.
I. It may not be possible to read each of the questions II. Power generat ion is limited; joint eff ort s t o
within three seconds by a candidate. conserve power would help maintain reliable
II. It will be a hard nut to crack for all aspirants to supply of electricity.
solve questions of Data Interpretation without any 188. Statement: One of the groups of militants of our
written work. organisation, which had been successful in making
181. Statement: High Court of state ‘X’ has decided to do its way into the Indian territory, kidnapped a local
away with the summer vacations in order to dispose resident of the area and forced him to act as a guide.
of pending cases. Assumptions:
Assumptions: I. Local residents of a particular area possess good
I. Vacation for the courts is a vestige of the Raj knowledge about the path in and around the area.
period. II. Despite the use of force against an individual,
II. The step taken by the judiciary will compel the he or she may guide properly and accurately.
Government to fill up all the vacancies in the 189. Statement: Being unable to accommodate adequate
court. training session for the national team, country X
182. Statement: The Government has transferred the has decided not to defend the LG Cup it won last
entire staff of police station ‘X’ and a magisterial year in the city Z.
inquiry has been ordered into the mysterious death Assumptions:
of a man in the police station. I. Without playing several matches against major
soccer-playing nations, it will be futile to defend
the LG Cup.
20 Practice Book on Analytical Reasoning

II. Underpreparedness of the team may result in Assumptions:


failure for country X. I. Future wars would be fought through air and
190. Statement: “I want Multinational Companies (MNCs) aerospace.
to be out of the water business. I don’t mind them in II. Speedy communication ensures faster decision-
road construction or some other areas.” — Statement making.
of a waterman while educating the people on the 197. Statement: “We are taking care of this historic house,
community’s water right putting our lives at risk.” — A statement made by an
Assumptions: employee of ASI
I. Water is basic to life and the community should Assumptions:
own it. I. The house is more precious than human lives.
II. The MNCs are on the prowl to buy the rivers. II. One takes utmost care of something when one
191. Statement: Courier companies, led by big names are puts one’s life at risk to do so.
planning 25-30% hike in courier and parcel rates, 198. Statement: “From the dawn of civilisation, India’s
as an initiative, following 67% hike in on-board tradition of respect for cultural diversity and spiritual
courier rates by domestic airlines. values has been the bedrock for its intercultural
Assumptions: dialogue and interaction with civilisations, countries
I. If the cour ier rates were not increased, the and nations.” — Ministry of HRD
industry would suffer huge losses which would Assumptions:
retard its growth, leading to closure of operations. I. Other countries have no respect for cultural
II. The hike in courier and the parcel rates may be diversity and spiritual values.
able to reimburse the expenses incurred due to II. The concept of a “Dialogue Among Civilisations”
hike in on-board courier rates by the domestic has assumed gr eat er impor t ance wit h t he
airlines. emergence of the global scenario.
192. Statement: “Before you hire a domestic help, get his/ 199. Statement: The cabinet decided to wind up various
her background verified through us. Your family’s existing authorities on environment and set up a

K
safety depends on this simple step.” — City police National Authority headed by a Supreme Court judge
Assumptions: besides six regional authorities to help Ministry of
I. Police verification of domestic help is an effective Environment and Forests.
step in crime prevention. Assumptions:
II. Ver if icat ion of people’s domest ic help’s I. These new authorities may be able to look after a
antecedents is a simple process. particular eco-sensitive area or a regional issue.
193. Statement: “My life has become a burden for too many II. These new authorities may help re-organise and
people and my step of jumping off a multistoreyed str eamline the of functioning of Minist ry of
housing complex will bring happiness for many Environment and Forests.
people. I do not blame anyone for my act. Please let 200. Statement: “It is very regrettable though issues
me die peacefully.” — A suicide note written by a 19- per taining to t he elderly and the disadvantaged
year-old girl. children do find space in the media, the “treatment”
Assumptions: given to them is often shallow and rarely reflects

KUNDAN
I. Jumping off a multistoreyed housing complex will their actual condition.” — View of Mr ‘X’
cause death. Assumptions:
II. Jumping off a multistoreyed housing complex is I. The media should not gloss over the real problem
the easiest among the various way of ending lives, faced by the old.
such as hanging, burning and consuming poison. II. The r adio and pr int media t ouch upon the
194. Statement: “Despite hike in tariff, power is still problems of the elderly but t he informat ion
cheap in the city X.” — Power Minister supplied has little practical value.
Assumptions: 201. Statement: “Gov ernment employees, including
I. The cost of purchasing electricity or its generation doct or s working in st ate-r un hospitals and
is much more in city X than in the neighbouring dispensaries, have no right — fundamental, legal,
states. moral or equitable — to go on strike.” — Supreme
II. Hike in tariff makes the power costly. Court
195. Statement: “Supporting the Reserve Bank of India’s Assumptions:
clean not e policy, we hav e intr oduced special I. Government employees hold society to ransom
polythene currency note packets. These are secure, by going on strike.
transparent and unstapled. Please do help us to II. Strike as a weapon is mostly misused, which
maintain this.” — a request to customers of Bank results in chaos and total maladministration.
XYZ 202. Statement: “Never t ouch or even go near any
Assumptions: unclaimed object lying ar ound in public areas,
I. Writing anything on currency notes is not in tune however attractive they may seem.” — A notice issued
with RBIs clean note policy. in public interest by city police X.
II. Only people’s participation is enough to get the Assumptions:
desired result expected through RBI’s new policy. I. Attractive and unclaimed objects lying around
196. Statement: “India needs a dedicated military satellite public areas are not supposed to be disastrous
for future defence purposes where communication elements by the common people.
will play a vital role in quick decision-making.” Chief II. Unclaimed object lying around in public areas
of Air Staff may be disastrous.
Assumptions 21

203. Statement: “No matter which career path you choose Assumptions:
in photography, there are skills you need in order to I. The vehicle people want to sell could be misused
succeed: visual skills, technical, camera and lighting by terrorists.
skills and digital imaging skills besides the most II. Each and every individual wants to buy old cars/
important element of success — proper training.” — vehicles.
View of Mr Z 209.Statement: I will not resign until proved guilty. - A
Assumptions: politician in his speech
I. A strong sense of visual style and compositional Assumptions:
skills in addition to training in the technical I. There are demands for his resignation.
int r icacies of phot ography may make a II. Ther e are some charges levelled against t he
photographer more successful. politician.
II. A sound educat ion pr ov ided by a qualit y 210. Statement: Buy ‘X’ dairy milk - fresh, pure and
photography training can help one to gain the hygienic. - An advertisement
skills one needs to succeed. Assumptions:
204. Statement: “The monsoon is her e. So ar e t he I. Other brands of milk are also available.
mosquitoes Beware of total diseases! So do not leave II. ‘X’ dairy’s milk is the most expensive.
in open on the roof of the house, unused/broken 211. Statement: Woman is the embodiment of sacrifices.
articles like bottle, cups, tyres etc.” — A notice issued Assumptions:
in public interest by Directorate of Health Services I. Man should not make sacrifices as this role is
[DHS] entrusted to females.
Assumptions: II. Women usually make sacrifices.
I. Mosquito spreads fatal diseases and steps to 212.Statement: It is none other than humans who destroy
control mosquito breeding can prevent the spread humanity.

K
of the diseases. Assumptions:
II. Unused/broken articles are capable of holding I. Man is man’s enemy.
rainwater. II. Humans are destructive.
205. Statement: The Law Commission of India has called 213. Statement: The government should reject the demand
for sweeping changes in life insurance laws not only for a separate state for tribals in state A.
to promote insurance business in the country but Assumptions:
also to protect policy holders from hassles in claiming I. The government has the power to grant a separate
settlements. state.
Assumptions: II. Tribals are generally very cruel.
I. The interests of policy holders had not been 214. Statement: When the hammer does not work, use a
entirely satisfied particularly in the area of claims sledge hammer.
settlement. Assumptions:
II. Sweeping changes in life insurance laws are I. A sledgehammer is more powerful than a hammer.
possible. II. Different tools are required in different conditions.

KUNDAN
206. Statement: “Are you creative and outgoing? Do you 215. Statement: This move has been taken from the point
enjoy communicating ideas and working with others? of view of exchange rate management.
A career in photography offers the opportunity to Assumptions:
create art and work in a rewarding professional field I. Such a move is not desirable.
of endeavour.” — An advertisement II. No other move is possible.
Assumptions: 216. Statement: “Vote for the Congress to complete the
I. Making commer cial quality phot ogr apher s unfinished agenda st arted by great leaders like
requires technical expertise and creativity. Nehru, Indira Gandhi and Rajiv Gandhi, and to tackle
II. There are some people who want to be successful problems faced by the country such as poverty,
photographer. injustice and exploitation of the poor, women and
207. Statement: “If you are serious about professional children.” — a leader.
photography and committed to taking the next step, Assumptions:
Advanced Studies Classes at photography institutes I. Nehru, Indira Gandhi and Rajiv Gandhi were
will teach you what you need to know to get started.” Congress leaders.
— A tells B II. Poverty is an issue of serious concern to the
Assumptions: voters.
I. Those who are looking for a stepping-stone to 217. Statement: One should see the problems the so-called
gain confidence and knowledge need training Asian Tigers are undergoing, and the huge loans
classes. they are forced to take from the IMF. India should
II. Professional photographers at the institutes offer lear n fr om what has happened in t hese Asian
hands-on instruction, opportunities to enhance countries.
portfolios, and lifelong contacts that can help Assumptions:
students build careers in photography. I. If India does not exercise caution, it will also go
208. Statement: “If you are selling your car/vehicle, retain the way Asian Tigers have.
copies of identification documents with photographs II. Taking loan from IMF is not a good sign.
of buyer, such as driving licence, ration card, election 218. Statement: “I endorse X’s demand to reveal the names
ID card etc. A simple way to fight terror.” — A notice involv ed in t he kickbacks immediately.” — Y, a
issued in public interest by city police X political leader
22 Practice Book on Analytical Reasoning

Assumptions: Independent states (CIS)


I. Both X and Y belong to the same party. Assumptions:
II. Y will be a beneficiary of the revelation of names. I. The hotbed of instability is vulnerable and needs
219. Statement: Ali Sardar Jafri is a representative in some special measures.
the fight against injustice and oppression in society. II. A flexible combination of political, legal and
Assumptions: economic measures to t ar get t he sources of
I. Jafri is not the only one who is fighting against terror ist , extremist and narcotic threats are
injustice and oppression in society. possible.
II. The problems of injustice and oppression will be 226. Statement: “The government has planned to unveil
solved through fighting against these. a new FDI policy by the next month to attract foreign
220. Statement: Political pressure seems to have resulted investors.” — Spokesperson of the govt
in the soft stand taken by RBI in favour of Sahara Assumptions:
and Peerless groups, thus discriminating against I. The new FDI policy will be able to attract those
other NBFCs. foreign investors who are ready to invest in India
Assumptions: but shying away t hemselv es due t o some
I. An influence in the corridors of power leads to unadaptable process of clearness.
relaxations. II. Foreign investors will welcome this new policy.
II. Both Sahara and Peerless are NBFCs. 227. Statement: “Our country has decided to give all types
221.Statement: “The fundament al r equir ement is a of support to country X in its objective to itself polio-
government that seeks a consensus and cooperation. free by 2002.” — Health minister of country Y
It is when governments behave arbitrarily and try to Assumptions:
impose their will without discussion and when they I. Without the support of country Y, country X will
consider dissent as anti-national and approach the not be able to make itself polio-free.
house with closed mind that disruption arises.” — II. Country X is not capable of making itself polio-
Leader of the opposition free by 2002.

K
Assumptions: 228. Statement: “Our country X has decided not to provide
I. If the Government takes a decision keeping the air passage for the aeroplanes of country Y because
view of opposition in mind, there might not be it was later found hatching conspiracy to br ing
any act of disruption. anarchy in our country through destructive work of
II. Views of opposition is a matter of great concern. sev er al milit ant out f it s. ” — Spokesman of t he
222. Statement: “It has never happened in Parliamentary government
history that a member has been selected by the Prime Assumptions:
Minister, given oath by the President, and another I. The pressure mounted by this move is likely to
member does not want to ask him a question.” — PM change the overt and covert policy of country Y
of country X when some members of Lok Sabha refuse towards intentions country X.
to ask any question from the defence minister. II. This move is likely to attract global attention
Assumptions: towards the evil of country Y against country X.
I. If a person does not give due respect to a person 229. Statement: Bhar at Sanchar Nigam Limited has

KUNDAN
who has been appointed constitutionally, it is a announced a sharp reduction in STD rates for both
matter of concern. peak and non-peak hours despite knowing the fact
II. Happening of new things in parliament ar e that this move will compel it to incur a loss of Rs
incorrect and illegal. 3000 crore in the first six months.
223.Statement: “The Indir a Gandhi Nat ional Open Assumptions:
University (IGNOU) has indicated that it would soon I. Other major telecom companies may reduce their
be commissioning 15 more transmitters for ‘Gyan STD rates for both peak and non-peak hours.
Vani’, a radio channel on education, to include Delhi, II. Reduct ion in STD r at es is likely t o pr ovide
Visakhapatnam, Lucknow, Bhopal, Mysore, Raipur, enormous profit in the long run.
Ahemadabad, Rajkot, Guwahati and Varanasi.” — 230. Statement: Keshav’s mother instructed him to return
Director of IGNOU. from the city by train instead of taking the river route
Assumptions: if flood situation became grimmer.
I. People of t hese area need educat ional Assumptions:
information. I. Keshav may not be able to decide on a proper
II. IGNOU’s existing infrastructure is capable of course of action if the situation of flood turns
making these facilities available. worse.
224.Statement: The PM of country X warned country Y II. Both the routes will continue to function even
not to interfere with the Island’s legislative election after the aggravated flood situation.
because the vote represents the people’s will. 231. Statement: People living in border areas of country
Assumptions: X have started selling their crops at low prices after
I. Leaders of country Y have tried to influence the the tense situation with neighbouring country Y.
polls during the past elections. Assumptions:
II. The election in country X is its internal affairs. I. The tense situation in the border area is likely to
225.Statement: “To defuse the hotbed of instability in lead to a war, which may further harm their
country X, it is necessary to use a flexible combination livestock.
of forces—political, legal and economic measures— II. The neighbour ing countr y might succeed in
to target the sources of terrorist, extremist and forefeiting their crops if disputes erupt in the
narcotic threats.” — A member of Commonwealth of border areas.
Assumptions 23

232. Statement: Buy our magical book on quicker maths 239. Statement: “Entry of HTVs is banned.” —A notice
and get techniques to solve those complicated sums on the main entrance of a flyover
within seconds which you were unable to solve in Assumptions:
quicker way. — An advertisement of X pub co of books I. People know the meaning of HTV.
for students II. The notice will have no impact on the people.
Assumptions: 240.Statement: “While solving a sum related to cubes
I. Students know the procedure of solving the sum and cuboids you compelled me to draw a sketch of
through tradit ional or other met hods except cuboid on the black board but what will happen when
quicker methods. you will go through the chapter of Profit and Loss,
II. Only the books of quicker maths of X publishing where you will have to calculate profit gained by a
company contains short-cut techniques. dishonest shopkeeper?” — A teacher X to a student
233. Statement: We have neither sufficient vehicles for Y.
patrolling nor night vision glasses and sophisticated Assumptions:
weapons; how can we be able to curb the militancy? I. The st udent will r equire a sketch of while
— A Commander of Border Security Force calculat ing pr of it ear ned by t he dishonest
Assumptions: shopkeeper.
I. Militancy can’t be curbed. II. The student should try to solve the sum related
II. Physical facilit ies of securit y f or ces help in to cubes and cuboids without drawing a sketch
increasing of efficiency of forces. of cuboid.
234. Statement: Since my childhood I always liked the 241. Statement: “Despite knowing the fact t hat the
poem of poet X and I made my efforts to learn from examination paper of X will consist of hundr ed
the style of the poet to be a good poet. — Mr Y questions on Reasoning we can’t predict the ratio of
Assumptions: verbal to non-verbal reasoning.” — A tells B.

K
I. It is possible to learn some qualities from the Assumptions:
poems of a good poet. I. Examination paper of X will consist of both verbal
II. Poems of poet X were in existence during the and non-verbal reasoning.
childhood of Mr Y. II. Some candidat es who are appear ing f or the
235. Statement: Our book on Non-Verbal Reasoning has examination X need some information regarding
been written systematically for students who are new the ratio of the questions of verbal and non-verbal
to the subject and now they do not require to join reasoning.
any coaching institute to learn the subject. — An 242.Statement: “The inv estor s who hav e made
advertisement investment through Agents may get their coupons
Assumptions: from the Agents on production of original savings
I. Students need a systematically written book on instrument to the Agent.” — A condition to public
Non-Verbal Reasoning. under a small saving schemes for investment
II. Students need the help of a coaching institute. Assumptions:
236. Statement: “Please note that the candidates have to I. Agents will have sufficient coupons to distribute

KUNDAN
bring their own typewriters at the time of the test of to investors.
typing skill.” — A condition in an advertisement II. No coupon will be provided directly from the main
Assumptions: office.
I. Candidates will not be able to bring their own 243. Statement: “How can an alliance be possible with
typewriters. those hypocritical parties who fight for the poor and
II. Candidat es will be able t o br ing t heir own use billionaires as symbols?” — Leader of party Y
typewriters at the time of skill test. Assumptions:
237. Statement: “Instances of brides being illtreated for I. Party ‘Y’ is the only real fighter for the poor and
insufficient dowry are too common to make news the downtrodden.
and it seems that even position and rank are no II. The objectives of the parties should reflect in
guarantee that a woman will receive her in-laws’ their deeds also.
respect.” — A report in a newspaper 244. Statement: The courts should strive to give speedy
Assumptions: and fair justice and keep off from politics as well as
I. It does not behove a woman at a higher post to be corr uption. At t he same t ime it should dev elop
deprived of in-laws’ respect for insufficient dowry. commitment to objectives and improve its performance.
II. Torturing for dowry is not a trivial issue. Assumptions:
238. Statement: “If you are keen to improve your English I. Nowadays, courts are delaying in giving justice
vocabulary, access the online dictionary service Your and have strayed away from their main objectives.
dictionary.com (http://www.yourdictionary.com) and II. The future of humans depends a lot on the
sign up wit h its ‘send me the W ord of the Day judiciary.
everyday’ mailing list service.” —An advice to the 245. Statement: “Most of the students are capable of
readers of a newspaper choosing right course for study as well as for their
Assumptions: career because they have educated parents.” — Mr X
I. People want to acquire a good vocabulary that Assumptions:
enables one to use the right word in the right I. Educated parents are necessary to choose the
context. right course for study or for a prosperous career.
II. Learning of a new word everyday via e-mail is II. Educated par ent s become helpf ul t o t heir
the only possible way. offsprings when the latter need some advice.
24 Practice Book on Analytical Reasoning

246. Statement: “How is it that only the northern part of 252. Statement: “Wit h our adv anced br oadband
the country is involved in the Ram temple movement technology, you can now experience the best action
when the whole nation has Ram devotees? If we on the internet without the worry of huge telephone
want to rise above these petty political machinations bills, and at prices as low as Rs 500 per month. —
there is an urgent need for a new breed of Indians An advertisement of company X.
with a mindset attuned to the present scenario and Assumptions:
instilled with values generated by centuries of true I. Now more people will have access to internet.
Indian civilisation.” — Mr Y II. People need to experience the best action on the
Assumptions: internet.
I. India comprises different types of mindsets. 253. Statement: “English is easy if you follow our book
II. True Indian civilisation among the citizens is English Is Easy.” — An advertisement of a Publishing
desirable to inculcate the citizens with a lesson Company
of harmony. Assumptions:
247. Statement: The leader of country ‘X’ has sent his I. Most people aspire to learn English.
special envoy to the capital of country ‘Y’, seeking II. One can’t learn English without following the
the help of its PM to defuse the western crisis. — A book English Is Easy.
report in a newspaper 254. Statement: “Soft drinks’ penetration in rural areas
Assumptions: is only 10 per cent. If we have to hook the rural folk
I. ‘Y’ is one of the western countries which has we need to give them single-serve soft drink packs
responded positively in building international at low price points.” — CEO of ‘X’ Company
peace. Assumptions:
II. Efforts made by the PM of ‘Y’ is likely to defuse I. Purchasing power of the people living in rural
the crisis in the western country. areas is low and it will make the people unable
248. Statement: “With 17” PC Monitor of company X, get to purchase the beverage of company ‘X’ even at
Laxmark Z13 colour Inkjet Printer absolutely free low price points.

K
with one-year guarantee period.” — an advertisement II. Low margin-high volumes is a way of successful
Assumptions: business.
I. More people will now be attracted towards 17” 255. Statement: “Why do we put pressures on children
PC Monitor of company X due to this scheme. to do exercises when they become adamant not to do
II. Offer of providing free goods to the customers the same? Why not opt for other mediums such as
with some other goods of a company is likely to break-dance etc to fulfil the desired objectives?”
increase the sale of the company. Assumptions:
249. Statement: “Medical and defence communities should I. Break-dances and exercises have some common
always keep vigil as prompt diagnosis and early benefits.
intervention could reduce morbidity and mortality II. Childr en would not hav e any hesit ation
and mitigate the ill-effects of a biological attack.” — participating in breakdances.
Mr ‘X’ makes ‘Y’ aware of his opinion. 256. Statement: “We do not know why government is
Assumptions: saying an agreement has been reached on the issue

KUNDAN
I. Biological attack is not desirable for society. of proposed dilution of government holding in the
II. Medical and defence communities are equipped joint sector automobile company, when neither party
with the instruments of diagnosis. is, however, willing to divulge details of the proposed
250.Statement: “New auto policy allows 100% foreign rights issue.” — Senior representative of Company
funding and has kept in mind the need to address ‘Y’.
emerging problems and make the auto sector WTO- Assumptions:
compatible”. — Heavy Industries Minister I. An agreement is said to be reached only when
Assumptions: both the parties divulge details of the relevant
I. Auto policy needs to be formulated according to issues.
the needs of the citizens. II. Gov er nment makes people f ool by f alse
II. A policy aimed at making any sect or W TO- representation.
compatible and hassle-free is considered to be a 257. Statement: “Almost two months after a girl was found
good policy. dead in a multi-storey apartment, the police are
251. Statement: “‘X’ steel company has approached the exploring the possibility of using a lie-detector to
problem of rebar corrosion holistically by developing verify statements.” — A report in a newspaper
continuously cast, thermo-mechanically-treated (TMT) Assumptions:
low-car bon, high-st r engt h and high-duct ilit y I. The lie detector may not help the police to confirm
weldable rebars with superior corrosion resistance their suspicions.
for concrete structure work.” — MD of ‘X’ Steel II. So f ar t he police hav e not used scient if ic
Company. techniques of interrogation and investigation to
Assumptions: unravel the mystery behind the death of the
I. Corrosion of reinforcing steel in concrete is a deceased.
complex phenomenon. 258. Statement: “Indian software institutes fulfil the
II. A marriage between high-quality steel bar and world’s demand of excellent software.” — A news
high-performance concrete is desirable to ensure heading
longt erm durabilit y Assumptions:
of reinforced concrete in construction work. I. Indian software institutes are quite capable of
producing world-class software.
Assumptions 25

II. In the field of software India has become far better Assumptions:
than any other country of the world. I. A person equipped with true qualities of humanity
259. Statement: Mr ‘x’, the fast bowler of country ‘y’, bid is mor e likely t o encourage harmony among
adieu to Test cricket to focus more on next year’s communities.
World Cup. — Mr Z II. To assuage the feelings of the communities is
Assumptions: not a hard nut to crack.
I. Mr ‘x’ will be fit for next year’s World Cup. 266. Statement: “How painful it is that even fifty years
II. World Cup cricket is more important than Test aft er Independence a sect ion of t he count ry’s
cricket. population is not being considered as its citizens!”
260. Statement: “As far as prediction about your success — A note in a newspaper about citizenship of a section
in the forthcoming PO examination is concerned, it in country X.
becomes necessary to know your level of preparation.” Assumptions:
— ‘A’ tells ‘B’ I. Fifty years is a long time span during which it is
Assumptions: feasible to provide citizenship to every section of
I. Knowledge of pr epar ation of ‘B’ will indicate the population of country X.
whether ‘B’ will succeed in the examination or II. Independence of a country ensures more powers
not. to its citizens.
II. Every competitor needs prediction about the result 267. Statement: “Sanjay Leela Bhansali (a film director)
of his/her preparation for the examination. should be lauded and not criticised for the huge budget
261. Statement: “If Miss ‘J’ can ask for the extradition of of Devdas (a film).” — A comment by Mr X
Mr ‘Z’, supremo of a militant organisation, for the Assumptions:
assassination of former PM, why can’t families of I. A person is not worthy of criticism only because
Br it ish citizens, killed in st at e ‘X’ wit h t he of violation of limit of a single factor.

K
connivance of chief minister ‘Y’, ask f or his II. Unless a person pumps in money, he/she cannot
extradition to UK to stand trial?” — A criticism made be the best in a field like cinema.
by a person. 268. Statement: “Had my father been alive, he would never
Assumptions: have allowed me to join films.”— Miss X, a singer
I. Killings in st at e ‘X’ wer e as deadly as t he Assumptions:
assassination of the former prime minister. I. Sometimes tragedy brings prosperity in the life
II. Mr ‘Y’ is a culprit and so is Mr ‘Z’. of an individual.
262. Statement: “If you have any information about drugs II. Film industry witnesses many ups and downs,
you can contact the officers of Narcotics Control therefore there is no guarantee that prosperity
Bureau.” — An instruction to the people by Narcotics will come in life.
Control Bureau 269. Statement: “Our study materials do not necessarily
Assumptions: satisfy one’s greed but certainly satisfy one’s need to
I. Every person has information (more or less) about become a Pr obat ionar y Of f icer (PO). ” — An
drug trafficking. advertisement of ‘X’ coaching Institute

KUNDAN
II. If it is not said explicitly, people will not convey Assumptions:
the information they have about illegal drug I. Need to become a PO can be satisfied with certain
trafficking. amount of efforts but one’s greed can not be
263. Statement: “Please change the colour pattern of your satisfied with certainty.
number plate of vehicle on or before 1st July, 2002 II. Desires of most of the people (who aspire for PO)
as per the directions by Transport Department.” — A are not conducive to their real requirements.
request by Transport Department to owners of two- 270.Statement: “If you believe that stars and planets
wheelers and four-wheelers. influence the course of your life, then the four-album
Assumptions: series Navagraha is meant for you. It is the collector’s
I. The stipulated time is enough for the owners to series for those who want to keep their fortunes
make requisite changes in number plates. intact.”— An advertisement
II. People will abide by the request of the Transport Assumptions:
Department. I. Some people hanker af t er a pr osper ous lif e
264. Statement: “Since most diseases result from faulty without labouring.
lifestyles or bad eating habits, why should animals II. Astronomy plays a vital role in one’s life and
suffer in the research for cure for these man-made cautions one against unfavourable situations.
ills?” — View of Mr Y 271. Statement: “Only you know what is better strategy
Assumptions: for you. So listen to various views but implement
I. Initiatives aimed at prevention are better than what suits you the best.” —An ultimate success
cure of a disease. formula to youngsters by an IAS topper among women.
II. It is not desirable to harass a species for the faults Assumptions:
of another species. I. Some people need help f r om other s while
265. Statement: “There is a need to take steps that would choosing an appropriate strategy for preparations.
assuage the feelings of all the communities and II. Each individual has the capacity to judge herself
strengthen the forces of tolerance, brotherhood and as well as adopt right courses of action after self-
amity in state ‘X’, where communal violence has done evaluation.
immense damage.” — Mr Z, leader of opposition. 272.Statement: “There should be a joint patrolling by
Indian and Pakistani forces to check infiltration of
militants into Jammu and Kashmir.”—PM of India
26 Practice Book on Analytical Reasoning

Assumptions: Assumptions:
I. Joint patrolling scheme is the only possible and I. Scholarship will attract the children to school.
practical way to check infiltration. II. Those children who are deprived of money but
II. Pakistan Gover nment will tur n down India’s intelligent will attend the school and scholarship
proposal of joint patrolling of LoC as unworkable will make their aspirations feasible.
in the present circumstances. 280. Statement: “Smartness cannot be the only criteria
273. Statement: “Unrestricted, free and fair competition for deciding a person’s personality.”— Mr Y
is the only process to bring out the most competent Assumptions:
and committed of the aspirants to political leadership.” I. Persons with equal smartness are not necessarily
— A note in a newspaper treated equally.
Assumption: II. Personality of a person is not linked only with
I. Polit ical leadership is facing character crisis smartness.
everywhere. 281. Statement: “Everyone desires to buy a mobile phone.”
II. Existing condition of the country is conducive to — a student
unrestricted, free and fair competition so that Assumptions:
the most competent and committed candidate can I. Mobile phones are not a necessity but an item of
be found out. luxury.
274. Statement: “Readers are recommended to make II. Mobile phone makes one more active.
appropriate enquiries before sending money, incurring 282. Statement: “Top rankers of forthcoming examination
any expenses or entering into any commitment in for probationary officers will definitely be from our
r elat ion to any adv er tisement published in our classr oom coaching.” — Co-or dinat or of XYZ
newspaper.” — An instruction to readers through classroom coaching
newspaper ‘X’. Assumptions:
Assumptions: I. Most of the intelligent competitors are from XYZ
I. Most of the readers can be persuaded easily by coaching institutes.

K
false and alluring advertisements. II. Question-paper of the forthcoming examination
II. Appropriate enquiries are likely to reduce the will be entirely based on the concept-building
cases of fraud and other malpractices. papers of XYZ institute.
275. Statement: “Register as a voter and secure your voting 283. Statement: “It has become the need of the hour that
right.”— Notice issued in public interest by Chief we should entrust all the power, regulating our
Electoral Officer f ranchisees, t o one per son t o maint ain smoot h
Assumptions: r unning of our f r anchisees as well as t he
I. Some people still lack right of voting due to lack organisation.”— An employee of XYZ institute.
of proper registration. Assumptions:
II. “Voting right” is essential to make one’s life more I. Entrusting powers to more persons will cause
dynamic, comfortable and prosperous. f lexibility and indiscipline and as a r esult
276. Statement: “We realise that your needs change over regulation of organisational work will be hindered.
time. Our wide range of insur ance solutions is II. Smooth running of franchisees is necessary for

KUNDAN
designed keeping t hese changes in mind. So, the organisation.
whatever stage of life you are at, whatever your needs 284. Statement: “Buy ‘X’ washing machine and win a free
may be, you can count on us to have a solution.” — t r ip t o t he Asian Games in Kor ea. ” — An
An advertisement advertisement of a product in India.
Assumptions: Assumptions:
I. People are attracted towards renovations. I. Most of the people do not want to wash the clothes
II. A wide range of insurance solutions is likely to manually.
garner more customers. II. Free trips to the Asian Games in Korea will give
277. Statement: The Election Commission has deputed a good entertainment to the buyers.
nine-member high level team of official to assess the 285. Statement: “If you are facing distress-like situations,
situation in state ‘X’ and gauge the feasibility of say physical assault , molestat ion, ev e-teasing,
holding early Assembly elections. harassment or any such situation but are reluctant
Assumptions: to report to the Police Station, call 3317004 and get
I. State ‘X’ has just got out of its predicament as a prompt assistance from specially trained Women
result of communal riots. Police Of f icer s. You will be ensur ed of
II. People of state ‘X’ will co-operate with the high confidentiality.” Director-General of police of state
level team of officials in its objectives. ‘X’.
278. Statement: If the government wants to stop child Assumptions:
marriages, why not pass a law incorporating a clause I. In some cases, despit e facing dist r ess-like
that marriages so solemnised will be illegal and situations women do not file a complaint against
treated as child rape? the bad guys due to some social compulsions.
Assumptions: II. Victims of physical assaults etc are more likely
I. Child marriages still exist in our society. to express t heir tr auma t o specially t rained
II. Sever e punishment is mor e likely to rid t he W omen Police Of f icer s t han t o male police
society of evils. officers.
279. Statement: “Provide scholarships to the children in 286. Statement: “The film Ek Chhotisi Love Story is OK to
primary schools to increase the number of students watch once and nothing more. I am convinced that
in school.” — View of Mr X Koirala (the actress of the film) and Nair (the director
Assumptions 27

of the film) are together in the ongoing debate on II. Playing wit h t he minor it y car ds giv es some
vulgarity in the film. The former has decided to get a leaders political berth but the status of their
stay while the latter has decided to go ahead with subjects remains unchanged.
the release only because they knew the movie didn’t 294. Statement: India’s strength lies in its pluralism and
have much in it. —View of a viewer of the movie Ek diver sity. In f act, even western countries which
Chhotisi Love Story tended to be quite monolithic are fast becoming
Assumptions: pluralistic and diverse.
I. Raising much hue and cry on trivial issues is Assumptions:
likely to give boost to a weak film. I. Diversity has become the order of the day.
II. A controversy over a film generates curiosity II. No country can ignore the diversity among their
among viewers. populace.
287. Statement: Religion-based parties are a negation of 295. Statement: When corrupt practices become a way of
our political system. Rather than solve problems, they life, society degenerates and political stability is
will only aggravate them. endangered.
Assumptions: Assumptions:
I. India is a secular country. I. Corruption harms every segment of society.
II. Indian people follow different religions. II. Only corruption-free society can have political
288. Statement: There can be no heroism without courage. stability.
But courage is characteristic of many kinds of people 296. Statement: “Those without a voter identity card will
who are by no means heroes, such as pirates, robbers not be permitted to vote even if their names are in
and blood-thirsty raiders. the eligible voters’ list.” — Election Commissioner
Assumptions: on the eve of election
I. A person who risks or loses life only for the sake Assumptions:

K
of some noble cause is a hero. I. Voter identity cards have been issued to all the
II. Heroism can not be thought of without some sort voters in the country.
of violence. II. Voter identity cards have not been issued to all
289. Statement: For making any democracy successful, the voters in the country.
judiciary must be free from the control of executive 297. Statement: The disinv est ment in banks will
or legislature. eliminat e scams and also reduce NPAs thus
Assumptions: strengthening the nation’s finances.
I. Judiciary is the only wing which cares for the Assumptions:
needs and aspirations of citizens. I. Privatisation of banks will improve the Indian
II. Any kind of interference in the functioning of economy.
judiciary will hamper its spirit. II. Corruption in private organisations is less than
290. Statement: The progress in the IT field has been that in government organisations.
tremendous, despite the country running into rough 298. Statement: If universities in each state adopt the
weather on political as well as economic front. regional language as their medium of instruction, it

KUNDAN
Assumptions: will severely rest rict the int er -stat e mobilit y of
I. Though the political and economic scene in the students and teachers.
countr y has been abysmal it has not cast a Assumptions:
shadow over the growth of the IT sector. I. Inter-state mobility of students and teachers is a
II. Ther e is no linkage between t he polit ical, major problem for the government.
economic and corporate-related activities within II. There are so many languages in India.
the country. 299. Statement: Care should be taken to see that language
291. Statement: Democracy will suffer great damage if learning does not become much of a burden on the
t he all-perv ading nexus bet ween cr iminals and students.
politicians is not effectively questioned by enlightened Assumptions:
people from all walks of life. I. Students are already overburdened.
Assumptions: II. Languages do not play any important role in
I. Criminalisation of politics will give a major blow professional life.
to the rule of law. 300.Statement: Lack of transparency in financial and
II. It is only in the hands of the people to break the investment policies and bureaucratic delays are major
nexus between politicians and criminals. hurdles as India tries to integrate its economy into
292. Statement: Politicians turning into ministers is a the global market.
nat ur al t ransfor mat ion ev en if t he qualit y of Assumptions:
performance rarely matches expectations. I. The investment policies need to be changed for
Assumptions: large-scale globalisation in India.
I. No ministers have adequate quality and skills. II. Bureaucrats in India are corrupt.
II. Ministers should be able to handle their job 301. Statement: Neglecting the manufacturing sector and
efficiently. veering towards the information technology industry
293. Statement: Some ambit ious political leader s is what has led to the slowdown in the economy.
themselves use grievances of minorities as empty Assumptions:
slogans to grab political power. I. The growth in manufacturing sector can boost
Assumptions: the Indian economy.
I. Such leaders have no mass base nor genuine II. Infor mat ion t echnology indust r y needs less
support , except for an occasional emot ional manpower than manufacturing sector.
upsurge.
28 Practice Book on Analytical Reasoning

302.Statement: “Imparting greater economic value to the Assumptions:


time and labour of the poor through technological I. Politics is a convenient means to become high
and knowledge empower ment should receiv e and mighty.
priority.” — An economist’s opinion II. Political leaders use party funds for their personal
Assumptions: benefits.
I. The poor get very little wages for their labour. 310. Statement: Public sect or is a great sour ce of
II. Only upliftment of the poor can enhance the cor rupt ion. And in f act this is t he reason why
economy of a country. government servants make concerted efforts to ensure
303. Statement: “International players are role models and that the transition to privatisation of this sector is
set examples by their actions on the field. Whatever stalled.
we do on field makes impact on younger generation.” Assumptions:
— Zimbabwean captain Heath Streak I. Public sector employees want to stay earning
Assumptions: money through wrong means.
I. International players should be cautious in their II. Private companies can manage the country’s
approach. resourses more efficiently.
II. Most of international players lack good behaviour 311. Statement: With the entire administrative system of
on the field. the country steeped in corruption, any attempt at
304. Statement: Globalisation should not be seen as an diluting the powers of the CVC must be thwarted by
automatic remedy for all economic maladies. It must all, and especially the media.
be tinkered so that its benefits are not denied to the Assumptions:
poor. I. Media works as the most important instrument
Assumptions: of control against the people in power.
I. Until poverty is wiped out, economy can’t be II. CVC reputation is beyond reproach.
boosted up. 312. Statement: The emergence of acute competition
II. Globalisation is the only solution for making because of fresh entry of companies benefits the

K
economy healthy. consumer.
305. Statement: “Nowadays the audiences too have become Assumptions:
very intelligent. You cannot con them with money.” I. The monopoly of a company in a specific sector
— Anupam Kher after his removal from the multicrore is not good for consumers.
television game show Sawal Das Crore Ka II. When competition among companies increases,
Assumptions: the consumer get better quality, new technology,
I. Sawal Das Crore Ka is not a very popular game discounts and innovative sales schemes.
show. 313. Statement: India suf fer s a huge loss of power
II. Intelligent persons cannot be lured by money only. annually. Technically described as transmission and
306. Statement: No reform of income-tax law can be distribution losses, this is a euphemism for large-
completed without a reasonable and rational personal scale theft.
income-tax rate structure in our country. Assumptions:
Assumptions: I. India has an adequate power generation capacity.

KUNDAN
I. Until the present exemption limit for individuals II. Large-scale transmission and distribution losses
is not raised, tax evasion and black money can’t are not possible.
be checked. 314. Statement: The most important quality a model
II. Developed countries have reasonable income-tax should possess is conf idence. One need not be
rate structure. conventionally good-looking to be a model.
307. Statement: India has had a long and honourable Assumptions:
history of writers on social concerns but the problem I. To become a model what counts is not a good-
is that Indians don’t have a sense of history. looking f ace but knowledge, conf idence and
Assumptions: elegance.
I. The individual bias of an author affects the spirit II. Good looks are no criteria for being a model.
of history but this is not the case with social 315. Statement: “The economic fortunes of the USA would
issues. determine the health of the Indian economy.” — FICCI
II. The present issues of social concern can not president
constitute a part of history. Assumptions:
308. Statement: The functioning of Nehru’s democratic I. Indian economy has no independent existence
system remains the best guarantee of Indian pluralism. of its own.
But it has also served to create and perpetuate India’s II. The US economy influences the world economy.
various particularisms. 316. Statement: Integrity and simplicity of life are not
Assumptions: regarded as virtues any more.
I. No leadership other than that of Nehru’s could Assumptions:
ensure harmony in diversity in India. I. Nowadays cunning and fraud is the gateway of
II. In a diverse country like India it is not easy to success.
mitigate particularisms. II. Only those qualities are virtues which help to
309. Statement: Anybody smitten by the urge of seizing earn name and fame.
and wielding power forms a political party, gets it 317. Statement: Legislat ion alone cannot change
registered and starts collecting funds for the party entrenched customs and social attitudes. Women’s
through various means. plight is compounded by their ignorance of rights
Assumptions 29

and reluctance to seek legal remedies even when Assumptions:


they are aware of them. I. High-rise buildings are very much unsafe for
Assumptions: human lives during natural calamities.
I. Only literacy can change entrenched customs and II. The large scale of migration from the rural areas
social attitudes. to the towns and lack of proper civic planning
II. Women are generally subject to ill-treatment in there is behind the massive loss of human lives.
our society. 326. Statement: Good batsmen or good bowlers cannot be
318. Statement: “In 20 years, physics would become judged from their performance in one or two matches.
redundant because scientists will have solved the Assumptions:
mysteries of the universe.” — Stephen Hawking. I. Giving frequent chances to freshers is a must for
Assumptions: judging their capacity as a good batsman or
I. Physics is the science in which we study about bowler.
the Universe. II. The average of runs scored by a batsman and
II. Most of the mysteries of the universe have already wickets taken by a bowler stretched over several
been solved. matches is the real index to judge them as good
319. Statement:The investment in human development or bad performers.
must cont inue because only wit h t he help of 327. Statement: “Invest in the pension plans of LIC and
educational institutions of a high standard can the ensure lifelong security and best returns.” — An
country maintain the flow of talent. advertisement.
Assumptions: Assumptions:
I. A country can’t be branded as developed if it lacks I. People will respond to this ad.
talented persons. II. People want lifelong security.
II. In the present scenario high-standard educational 328. Statement: “They started it and as long as they were

K
institutes are lesser in number than that required. doing it, it was all fine. But the moment we do it, it
320. Statement: Urdu is neither the language of Indian all goes wrong.” — Finance minister of country `X’
Muslims only nor of all of them. It is an Indian on the Opposition’s stand on privatisation.
language which belongs to all of us like all great Assumptions:
languages. I. The Opposition should respond positively to well-
Assumptions: intentioned actions of the government.
I. Urdu originated and developed in India. II. Privatisation is good for the health of country X’s
II. Muslims under st and that Ur du is their own economy.
language. 329. Statement: Good mar ks alone don’t make you
321. Statement: “There is enough for man’s need, and successful in life. It is how many marks you got by
not for his greed.” — Gandhiji understanding the subject that matters.
Assumptions: Assumptions:
I. Needs can always be satisf ied wit h cer tain I. Higher marks can be scored in exams by rote.
amount of effort but greed can not be at any cost II. The utility of education is not confined to exams

KUNDAN
. alone; it has much wider importance in our life.
II. Man is greedy by nature. 330. Statement: Don’t take a job just for money. Take a
322. Statement: There is no value-based education at all. job to serve people and benefit them more and more.
Educat ion has become commer cial with sev er al Assumptions:
malpractices taking place at various levels. I. Most of the people work solely for money.
Assumptions: II. A job/assignment undertaken out of interest and
I. Commercialisation of education has immensely social responsibility not only gives money and
harmed our social interests. status to a person but also a mission in life.
II. Values and education should be synonymous with 331. Statement: For the educated and ambitious people
each other. the only alternative to enforced poverty now seems
323. Statement: Hist ory should not be used in t he t o seek business f or t une and employment
contemporary context to promote hatred between opportunities abroad.
communities. Assumptions:
Assumptions: I. Government polices are insufficient in providing
I. Some politicians have come to power mainly by employment to the educated people.
using history for political ends. II. Educated and ambitious people are not satisfied
II. History should be seen within itself and not wit h the av ailable business f or t unes and
outside its purview. employment opportunities.
324. Statement: “People will be ready to spend big money 332. Statement: “Despite being busy directing films and
on a course, if big money can be made in a related producing serials for TV, I am always on the look-
industry.” — An industrialist out for acting assignments” — An actor director in
Assumptions: an interview.
I. The demand of any professional course is related Assumptions:
to its relevance. I. Direction is a responsibility which one can’t enjoy
II. A less expensive course lands a person in a less- because it is a high-calibre job.
earning net. II. A good actor is paid more heavily than a good
325. Statement: An earthquake is a natural phenomenon director.
but the scale of death is definitely not. 333. Statement: Any democratic process is deeply flawed
if it excludes women from polity.
30 Practice Book on Analytical Reasoning

Assumptions: Assumptions:
I. Like men, women are an integral part of society. I. The soul is like a clothes-hanger.
II. Except in a few developed countries, women have II. The soul is immortal.
not been given proper representation in polity. 342. Statement: Those who say that the Indians have a
334. Statement: Stagnant industrial growth in the past dangerous level of tolerance — to the extent that
two years has crippled job opportunities in urban they would tolerate even non-independence for peace
India. — forget the history of India.
Assumptions: Assumptions:
I. Industrial growth in India is linked with global I. Some persons believe that Indians are tolerant to
industrial ups and downs. a dangerous level.
II. Industries are the mainstay for job-seekers in II. Some persons do not believe that Indians are
urban areas. tolerant to a dangerous level.
335. Statement: Winning votes and capturing power alone 343. Statement: A chariot is made of wheels, horses etc.
is not enough. Winning the hearts of the people also But none of these parts is the chariot itself.
matters. Assumptions:
Assumptions: I. A chariot is an incomplete construction.
I. People expect their representatives to behave II. Parts can never substitute the whole.
impartially, transparently and work for public 344. Statement: When I retaliate, I shall go one step
welf are. further in violence.
II. Winning votes and winning the hearts of the Assumptions:
people are interrelated. I. Violence has been used against me.
336. Statement: Central Recruitment Board has declared II. I do not take an offence lying down.
final results for Bank PO and (CRB) enlisted 218 345. Statement: When we can’t even buy a fan, how can
candidates, twenty more than the announcement in we afford a cooler?
advertisement. Assumptions:

K
Assumptions: I. We can’t afford a cooler.
I. Sur plus candidat es might be selected II. A cooler is costlier than a fan.
provisionally. 346. Statement: Some people think that they can achieve
II. The CRB has the power to alter the number of anything with practice. They forget that there is a
candidates to be recruited. thing called “born genius”.
337. Statement: “I don’t think our bowlers bowled badly. Assumptions:
We just didn’t have the runs on the board to win us I. Practice is not necessary for success.
the game.”— A captain of ‘X’ cricket team. II. A born genius is always successful.
Assumptions: 347. Statement: Some of our staff are not giving their full
I. Bowlers alone cannot bring victory for a country output. We must punish them. - A manager to his
in a match. colleague
II. In spite of bad bowling, batsmen can bring the Assumptions:
match in their team’s favour. I. The manager is in a position to punish the erring

KUNDAN
338. Statement: The anti-defection law is ineffective not staff.
because it is defective but because the enforcing II. The manager may not be in a position to punish
authority, the speaker of the legislature, is obsessed the erring staff.
by party politics.— Chief Election Commissioner of 348. Statement: “Persons who have got the passes need
country X. not stop for security checks. They can go inside
Assumptions: without this formality.” - a notice at a function
I. Enforcing authority needs to be impartial in a Assumptions:
democratic country. I. The function is not very important from the
II. Country X is suffering from horse-trading in security point of view.
politics. II. Those having passes can be trusted.
339. Statement: We have to fashion liberalisation for 349. Statement: “I always listened to my inner voice. And
ourselves. If you have to wear a hat of liberalisation, that is why I was saved from sin on many occasions.”
please wear one that fits your head. — Former PM of — Gandhi in his autobiography
country X. Assumptions:
Assumptions: I. Our inner voice asks us not to sin.
I. The economy of country X is not conducive to II. Gandhi never committed a sin.
meet liberalisation. 350. Statement: How can you be so rude? Your father
II. The policy of liberalisat ion pursued by t he was such a polite person!
incumbent government is adversely affecting the Assumptions:
interests of many sections of our society. I. Politeness is hereditary.
340. Statement: “All chocolates are bad for teeth. So why II. Politeness is not hereditary.
not eat the tastier ones?” — an advertisement 351. Statement: Whenever I come to watch the match,
Assumptions: India loses a wicket. So I will not see the crucial
I. One should eat chocolates. India-Pak match on Friday.
II. Nobody eats chocolates. Assumptions:
341. Statement: Our soul accepts new material bodies I. My coming to watch is one cause of India losing
giving up the old and useless ones, just as a person a wicket.
puts on new garments. II. India will win the match on Friday.
Assumptions 31

352. Statement: There must be some addictive substance I. In other countries, the police do not have to
in Gutkha. Whenever I don’t chew it for more than undergo stress and strain while doing their duty.
two hours, I feel uneasy. II. The police are expected to do their duties without
Assumptions: stress or strain.
I. Feeling uneasy is a sign of addiction. 363. Statement: If children are to manage our world in
II. Gutkha should be taken once every two hours. future, then they need to be equipped to do so.
353. Statement: I can’t make it in the UPSC exam. I am Assumptions:
just not hardworking. I. The world has always educated children.
Assumptions: II. It is possible to educate children.
I. Hard work is necessary to succeed in UPSC exam. 364. Statement: There is no shopping complex for this
II. UPSC exam is tough. colony; people have to go to the main market.
354. Statement: I hope to succeed in the SBI PO exam. Assumptions:
They had more vacancies. I. This colony may be far from main market.
Assumptions: II. The people do not want to go to the main market.
I. The more the vacancies, the greater the chances 365. Statement: Take t his ‘ov en’ home and you can
of success. prepare very tasty dishes which you were unable to
II. SBI PO exam is easier. prepare earlier. - An advertisement of X brand oven.
355. Statement: It is high time the RBI took a look at Assumptions:
infrastructure lending by commercial banks. I. The user knows the procedure recipe of tasty
Assumptions: dishes but does not have the proper oven to cook.
I. Infr astr uctur e lending is the best ser vice II. Only ‘X’ brand oven can cook very tasty dishes.
commercial banks can provide. 366. Statement: “Please note that the company will provide
II. At present the RBI does not pay attention to accommodation to only outside candidates if selected.”

K
infrastructure lending by commercial banks. — A condition in an advertisement.
356. Statement: By wining the Booker Prize, Arundhati Assumptions:
Roy gives Indian writ ing in English global I. The local candidates would be having some or
acceptance. other arrangement for their stay.
Assumptions: II. The company plans to select only local
I. Booker prize is a sign of recognition for English candidates.
wr iting. 367. Statement: Traffic police be given anti-pollution
II. Indian writing in English has come of age. masks while manning traffic signals.
357. Statement: Indians are no longer aghast when they Assumptions:
come across stories of corruption. I. The traffic police will be able to carry out their
Assumptions: work after wearing the mask.
I. Earlier, corruption came to Indians as a shock. II. The masks are safe for wearing and there is no
II. Corruption is not a new phenomenon. other adverse side-effect.
358. Statement: The people promoting aids to hedonistic 368. Statement: Since the First Five-Year Plan, the Indian

KUNDAN
activities are far more dangerous than our external policy-maker s hav e acknowledged t he ser v ices
enemies. rendered by the voluntary agencies.
Assumptions: Assumptions:
I. Our external enemies are not dangerous. I. Voluntary agencies have been in existence in India
II. Hedonistic activities can’t be performed without even before the First Five-Year Plan.
aids. II. Voluntary agencies have contribution in designing
359. Statement: The Indian cr icket team tr ounced of the First Five-Year-Plan.
pakistan 4-1 in Toronto to lift the Sahara Cup. 369. Statement: As a nation we are committed to protect
Assumptions: and promote the interests of all those who are socio-
I. Both India and Pakistan participated in the Sahara economically vulnerable.
Cup. Assumptions:
II. Only India and Pakistan part icipated in t he I. It is possible to protect and promote interests of
Sahara Cup. socio-economically weak people.
360. Statement: This book is for those who are interested II. A nation should have certain commitments for
to know more about ‘Indian History’. its people.
Assumptions: 370. Statement: ‘This book has been written for every
I. People who are interested to know about the one and does not requir e reader s to hav e any
author may read books. experience in handling computers.’ — An author of
II. Every book may attract some readers. a book on computers.
361. Statement: Helping the poor is the real service to Assumptions:
humanity. I. It is possible to learn computers with the help of
Assumptions: a book only.
I. Poor people are in need of help from others. II. It is possible to learn to handle computers only
II. If we do not help poor, we will not be called after reading the book.
human beings. 371. Statement: Health is the foundation of well-being,
362. Statement: The police in India have to cope with virtue, prosperity, wealth, happiness and salvation.
t remendous st ress and st r ain while having t o Assumptions:
maintain security and order. I. Happiness results in health and well-being.
Assumptions: II. People desir e t o be happy, pr osperous and
virtuous.
32 Practice Book on Analytical Reasoning

372. Statement: ‘Authorised Indian Edition — illegal for Assumptions:


sale or distribution outside India’ — A publisher’s I. Employees are likely to be sensitive enough to
note on the cover page of a book. learn by observing the behaviour of their bosses.
Assumptions: II. Normally bosses are considered as sources of
I. Indian editions may be in demand in nearby reward and punishment.
countries. 380. Statement: ‘But, out of A, B, C and D products, you
II. It may be possible to sell or distribute this book buy ‘B’, which alone is based on int er nat ional
outside India. technology. - A shopkeeper tells a customer.
373. Statement: Pollution is a slow poison, and therefore Assumptions:
social scientists and the media must work together I. The cust omer s nor mally accept t he
to create sensitivity among people. recommendation of the shopkeeper.
Assumptions: II. Use of international technology is supposed to
I. Media is well informed and aware about the effects ensure better quality standards.
of pollution. 381. Statement: The organisat ion should pr omot e
II. Media is likely to influence people to raise their employees on the basis of merit alone and not on the
sensitivity towards various problems. basis of length of service or seniority.
374. Statement: In country ‘X’ a public servant cannot Assumptions:
claim immunity from prosecution for any objectionable I. Length of service or seniority does not alone
act committed while performing his official duty. reflect merit of an employee.
Assumptions: II. It is possible to determine and measure merit of
I. A public ser v ant is likely to commit an an employee.
objectionable act while performing his official 382. Statement: Highly brilliant and industrious students
duty. do not always excel in the written examination.
II. Every one is equal before law. Assumptions:
375. Statement: The entry of multinational companies in I. The wr itt en examinat ion is good mainly for

K
India has led to higher eff iciency of the Indian mediocre students.
companies who are competing with them. II. The brilliant and industrious students cannot
Assumptions: always write good answer in the exam.
I. Employees of multinationals may serve as models 383. Statement: ‘Country A would explore all channels
for Indian company’s employees. to diffuse current tensions with country B and bring
II. Competition will reduce many Indian companies peace on its borders.’ — Statement of spokesperson
to ashes. of country A.
376. Statement: ‘Only candidates having B. Tech., B.E., Assumptions:
MBA and MCA with at least one year’s exposure to I. Country A is desirous to diffuse current tension
software will be considered for admission to our and restore peace with country B
course’ - Admission criterion of a reputed software II. It is desirable to use more than one channel when
training institute. complex issues are to be settled amicably.
Assumptions: 384. Statement: Two months ago, it was announced that

KUNDAN
I. The candidates having requisite background are Central government pensioners would get dearness
likely to complete the course successfully. relief with immediate effect but till date, banks have
II. The institute is choosy about admitting candidates not credited the arrears.’ — A statement from a
to its courses. Pensioners’ Forum.
377. Statement: ‘If you would like to have any more Assumptions:
information of XYZ credit card, call us between 8.00 I. Most of the banks normally take care of the
am and 8. 00 pm 365 days of t he year ’ -An pensioners.
advertisement of ‘XYZ credit card company’. II. Two months’ time is sufficient for the government
Assumptions: machinery to move and give effect to pensioners.
I. Competition produces more friendly customer 385. Statement: ‘The bridge was built at the cost of Rs
service. 128 crores and even civil bus service is not utilising
II. The company values and appreciates the need of it. What a pity to see it grossly underutilised!’ — A
the customers. citizen’s view on a new flyover linking east and west
378. Statement: ‘You are expected to be frank and objective sides of a suburb.
while wr it ing your self appr aisal report ’. -An Assumptions:
instruction for writing self-appraisal report I. The building of such bridges does not serve any
Assumptions: public objective.
I. Unless cautioned, people may tend to be a little II. There has to be some accountability and utility of
shy and less objective while writing their self- money spent on public projects.
appraisal report. 386. Statement: ‘Use our product to improve memory of
II. Every self-appraisal report helps the person in your child; it is based on natural herbs and has no
his further development. harmf ul side-eff ects. ’ - Adver t isement of a
379. Statement: The higher echelons of any organisation pharmaceutical company.
are expected to be models of observational learning Assumptions:
and should not be considered as merely sources of I. People generally opt for a medical product which
rewards and punishments. is useful and has no harmful side effects.
II. Impr ov ing memory of child is considered as
important by many parents.
Assumptions 33

387. Statement: The traders of State K would observe a will be convenient. — Appeal of residents of Cheka
statewide bandh as the state has failed to meet their Naka to the city bus company.
demand to resolve sales tax and other issues. Assumptions:
Assumptions: I. The convenience of the city bus company is much
I. The traders of State K have earlier tried other more important than the needs of the consumers.
usual procedures to get their problems solved. II. The city bus company is indif f er ent t o t he
II. State K is not keen to solve the problem of traders. aspirations of the residents of Shramnagar.
388. Statement: India must earn a lot of foreign exchange 397. Statement: Desirable and qualified candidates should
to achieve her target of economic development. submit their application form along with the requisite
Assumptions: qualifications and their biodata. — An advertisement
I. India desires to achieve the target of economic for admission.
development. Assumptions:
II. It is possible f or India to earn more foreign I. Merely having qualification and aptitude for the
exchange. job does not make a person suitable for the job.
389. Statement: “As you want to succeed in life, you must II. Many candidates shall apply because they are
work hard.” `A’ tells `B’. interested in the job.
Assumptions: 398. Statement: It has been felt that at a time when the
I. ‘B’ is capable of doing hard work. airline faces tough competition and is passing through
II. All those who have worked hard have succeeded critical economic conditions, the remaining higher
in life. posts should be opened for outside professionals
390. Statement: The nutritional status of children in India instead of filling them up with insider applicants.
is bet t er compar ed to that in other dev eloping Assumptions:
countries. I. The internal applicants only aspire for promotion

K
Assumptions: without contributing much to the organisation.
I. It is not possible t o est imat e nut rit ional II. It is most likely that problems of the airline would
requirement of children in other countries. be solved by experienced professionals.
II. India can become a developed country. 399. Statement: KLM company has decided t o issue
391. Statement: He teaches behavioral science but see debentures to mop up resources.
how he behaves with others? Assumptions:
Assumptions: I. KLM company has already explored other sources
I. Our behaviour is controlled by others. to collect resources.
II. One is expected to follow what one preaches. II. There are very few competitors in the market for
392. Statement: Economic development and social justice the products of KLM company.
should go hand in hand. 400.Statement: “Tenders are invit ed f r om r eput ed
Assumptions: contractors for pre-qualification.” — The tender notice
I. Only economic development can bring social of a public sector company
justice. Assumptions:

KUNDAN
II. Life without social justice is worth not living. I. The company seeks to do quality business.
393. Statement: “I have not received telephone bills for II. The company expects contractual and competitive
nine months in spite of several complaints.” — A rates for its work.
telephone customer’s letter to the editor of a daily. 401. Statement: The state government ‘X’ is committed to
Assumptions: restrict smoke levels on the roads of the metropolis
I. Every customer has a right to get bills regularly as per the desired parameters.
from the telephone company. Assumptions:
II. The customers complaints point to defect in the I. It is possible to determine the smoke levels.
service which is expected to be corrected. II. A commit ted gov er nment can car ry for war d
394. Statement: Greater public participation results in welfare measures for its people.
good civic governance. — Statement of Municipal 402.Statement: “To reduce the oil pool deficit it has been
Commissioner of city ‘A’. decided to hike the prices of diesel and petrol.”—
Assumptions: The spokesman of the government
I. The municipal office is not competent to effect Assumptions:
good civic administration. I. The amount earned by this increase may be
II. Good civic governance is a matter of collective substantial enough to reduce the deficit.
will and effort of the people and administration. II. There may be widespread protests against the
395. Statement: To investigate the murder of the lone price hike.
resident of a flat, the police interrogated the domestic 403. Statement: The X passenger car manufact uring
servant, the watchman of the multistoried buildings company announced a sharp reduction in the prices
and the liftman. of their luxury cars.
Assumptions: Assumptions:
I. The domestic servant, watchman and the liftman I. There may be an increase in the sale of their
can give a clue about the suspected murder. luxury cars.
II. Generally in such cases the persons known to II. Other such car manufacturers may also reduce
the resident is directly or indirectly involved in their prices.
the murder. 404. Statement: A foreign film producer rendered his
396. Statement: If the city bus which runs between Cheka apology before Indian society for misinterpreting an
Naka and Vande Park is extended to Shramnagar, it Indian epic.
34 Practice Book on Analytical Reasoning

Assumptions: 412. Statement: The government has set up a fact-finding


I. Indians are very sensitive to the misinterpretation mission to look into the possible reasons for the
of their epics. recent violence in the area.
II. It is possible to derive wrong meaning from the Assumptions:
epic. I. The mission may be able to come up with credible
405. Statement: Lalit’s mother instructed him to return information about the incidents.
home by train if it rains heavily. II. The people in the area may cooperate with the
Assumptions: mission and come for war d to giv e det ailed
I. Lalit may not be able to decide himself if it rains information related to the incidents.
heavily. 413. Statement: An advertisement: If you want to follow
II. The trains may ply even if it rains heavily. the footprints of an ideal leader, wear ‘X’ brand of
406. Statement: The Government of India has decided to shoes.
start a track II dialogue with its neighbour to reduce Assumptions:
tension in the area. I. Most people like to become ideal leaders.
Assumptions: II. One can’t become ideal leader unless one wears
I. The neighbouring country may agree to participate ‘X’ brand of shoes.
in the track II dialogue. 414. Statement: Every citizen must be committed to the
II. The people involved in track II dialogue may be social cause; if he is not, his citizenship should be
able to persuade their respective Governments. cancelled.
407. Statement: The host in one of t he popular TV Assumptions:
programmes announced that the channel will contact I. It is possible t o f ind out whet her a
the viewers between 6.00 a.m. and 8.00 p.m. on citizen is committed to the social cause or not.
weekdays and the lucky ones will be given fabulous II. Citizenship of any citizen can be cancelled.
prizes. 415. Statement: An advertisement: Now you can own a
Assumptions: new car in just Rs 1,999 per month.

K
I. The people may remain indoors to receive the Assumptions:
phone call. I. People do not want to buy used cars.
II. More people may start watching the programme. II. Most people can afford to pay Rs 1,999 per month
408. Statement: The ‘X’ group of employees’ association for a new car.
have opposed Voluntary Retirement Scheme to the 416. Statement: Beware of dogs. Our dogs do not bark but
employees of some organisations. they are trained to distinguish between genuine
Assumptions: guests and intruders.
I. Only those employees who are not efficient may Assumptions:
opt for the scheme. I. Barking dogs rarely bite.
II. The response of the employees may be lukewarm II. Our dogs could be dangerous for intruders.
towards the scheme and it may not benefit the 417. Statement: W it hout r efor ming t he entir e
organisation to the desired level. administ r ativ e system, we cannot eradicat e
409. Statement: In view of the statement on the ongoing corruption and prejudice from the society.

KUNDAN
strike of work by the employees, the government has Assumptions:
agr eed to wor k out an ef f ect ive social secur it y I. The existence of corruption and prejudice is good.
programme. II. Ther e is enough f lexibility t o change t he
Assumptions: administrative system.
I. The striking employees may not be satisfied with 418. Statement: The regulatory authority has set up a
the announcement and continue the agitation. review committee to find out the reasons for unstable
II. The str iking employees may withdr aw t heir stock prices.
agitation with immediate effect and start working. Assumptions:
410. Statement: The head of t he or ganisation I. The investors may regain confidence in stock
congratulated the entire staff in his speech for their market by this decision.
sincere effort to bring down the deficit and urged II. The review committee has the expertise to find
them to give their best for attaining a more profitable out the causes for volatility in the stock market.
position in future. 419. Statement: “Get rid of your past for future, get our
Assumptions: new-generation fridge at a discount in exchange of
I. The employees may get motivated and maintain old.” — An advertisement
and if possible enhance their present level of Assumptions:
work. I. The sales of the new fridge may increase in the
II. The employees may now relax and slow down in coming months.
their day-to-day work as there is no immediate II. People prefer to exchange future with past.
threat of huge deficit. 420. Statement: “Learn computer at no cost and make
411. Statement: “Private Property, trespassers will be your life more meaningful.”—An advertisement
prosecuted” — A notice on a plot of land. Assumptions:
Assumptions: I. People prefer to join courses without any fees.
I. The passerby may read the notice and may not II. Knowledge in comput er makes lif e mor e
tresspass. meaningful.
II. The people ar e scar ed of pr osecut ion and, 421. Statement: The government has decided to launch
therefore, never tresspass. food-for-work programme in all the drought-affected
areas.
Assumptions 35

Assumptions: savings schemes and also pay taxes.


I. The government has the machinery to implement II. The total tax collection may increase substantially.
the food for work programme in all the drought 430. Statement: The Government has decided to levy 2
affected areas. per cent surcharge on the tax amount payable for
II. There is enough food in stock to implement the funding drought relief programmes.
programmes successfully. Assumptions:
422. Statement: The head of the organisation has decided I. The Government does not have sufficient money
to reward those employees who will help reducing to fund drought relief programmes.
expenditure substantially by suggesting innovative II. The amount collected by way of surcharge may
techniques. be adequate t o f und t hese dr ought r elief
Assumptions: programmes.
I. The employees may be able to come out with 431. Statement: The ‘X’ Housing Finance Company has
innovative ideas. offered its services to search a suitable home at no
II. The employees may be encouraged to apply their extra cost for those who avail housing loan from it.
mind to earn the reward. Assumptions:
423. Statement: The civ ic authority has adv ised the I. The customers may prefer to take housing loan
residents in the area to use mosquito repellents or from ‘X’ Housing Finance Company as they can
sleep inside nets as a large number of people are sav e a lot of their time and money spent in
suffering from malaria. searching a suitable home.
Assumptions: II. No other Housing Finance Company has offered
I. Local residents have enough money to arrange any such extra services alongwith housing loan.
for the repellents or nets. 432. Statement: World Health Organisation has decided
II. People may ignore and continue to get mosquito to double its assistance to various health programmes

K
bites as they have other pressing needs. in India as per capita expenditure on health in India
424. Statement: The Government should engage the Army is very low compared to many other countries.
for the rapid rehabilitation of people affected by the Assumptions:
cyclone. I. The enhanced assist ance may subst ant ially
Assumptions: increase the per capita expenditure on health in
I. Only the Army can rehabilitate the people affected India and bring it on par with other countries.
by the cyclone quickly. II. The Government funding is less than adequate
II. The Army can take up works other than war also. to provide basic medical facilities in India.
425. Statement: His recent investment in the shares of 433. Statement: The government has decided to hold the
company ‘A’ is only a gamble. employers responsible for deducting tax at source
Assumptions: for all its employees.
I. He may incur loss on his investment. Assumptions:
II. He may gain from his investment. I. The employers may still not arrange to deduct
426. Statement: Government should deploy the Army at tax at source for its employees.

KUNDAN
least this year for the rehabilitation of people affected II. The employees may not allow the employers to
by cyclone because cyclone visits suddenly. deduct tax at source.
Assumptions: 434. Statement: The X-Airlines has decided to increase
I. The Army should be deployed for all such sudden the passenger fare by 15 per cent with immediate
incidents. effect.
II. Some precautionary plan is being made to prevent Assumptions:
destruction caused by cyclone. I. The demand for seats of the X-Airlines may remain
427. Statement: It is not true always that the adoption of unchanged even after the hike of fare.
latest technology ensures increased productivity and II. Other air line companies may also hike t he
capacity. passenger fares.
Assumptions: 435. Statement: “Our bank provides all your banking
I. It is possible to prove that increased productivity requirements at one location.” — An advertisement
and capacity are due t o adopt ion of latest of a bank
technology. Assumptions:
II. The productivity and capacity can be increased I. Cust omer s pr efer to car r y out all banking
by discarding latest technology. transactions at one place.
428. Statement: If you could not collect the required II. People may get attracted by the advertisement and
amount by oral call you must publish an advertisement carry out their transactions with this bank.
in a widely read newspaper. 436. Statement: Bank ‘A’ has announced reduction of half
Assumptions: percentage on the interest rate on retail lending with
I. People rarely respond to oral call. immediate effect.
II. Gener ally people ar e reluct ant t o r ead an Assumptions:
advertisement in a newspaper. I. Other banks may also reduce the retail lending
429. Statement: The Union Government has decided to rates to be in competition.
withdraw existing tax relief on various small savings II. The Bank ‘A’ may be able to attract more customers
schemes in a phased manner to augment its tax for availing retail loans.
collection. 437. Statement: The ‘M’ Cooperative Housing Society has
Assumptions: put up a notice at its gate that salespersons are not
I. People may still continue to keep money in small allowed inside the society.
36 Practice Book on Analytical Reasoning

Assumptions: of a school has informed the Principal that they will


I. All the salespersons will stay away from the ‘M’ not send their children to the school unless the school
Cooperative Housing Society. authority reduces the fees with immediate effect.
II. The security guard posted at the gate may be able Assumptions:
to stop the salespersons entering the society. I. Majority of the parents may agree with the PTA
438. Statement: It is not tr ue that the might iest and may not send their wards to the school.
superpower always wins wars and gets accolades II. The school authority may accede to the demand
from other countries. of the PTA and reduce the fees.
Assumptions: 447. Statement: ‘If you are fir st class graduate, our
I. Winners are sometimes admired and appreciated. organisation is the best place for you to work.” — An
II. Winners are occasionally criticised. advertisement
439. Statement: Nobody can predict as to how long our Assumptions:
country would take to contain the unfortunate and I. No other organisation may require first class
disastrous terrorist activities. graduates as they may not get adequate number
Assumptions: of applications.
I. It is impossible to put an end to terrorist activities. II. First class graduates may get attracted and apply
II. Efforts to control the terrorist activities are on. to this organisation.
440. Statement: Wars must be discouraged vehemently 448. Statement: Provide mid-day meals to the children
even though majority of the victims might have been in primary schools to increase the number of students
a nuisance to peace-loving people. attending schools.
Assumptions: Assumptions:
I. Some people create problems to peace-loving I. Mid-day meals will attract the children to the
people. schools.
II. Wars kill majority of the wicked people. II. Those children who are otherwise deprived of
441. Statement: In the recently imposed war, global public good food will attend the schools.

K
opinion was dishonoured by the economically strong 449. Statement: Salary cannot be the only criteria for
and scientifically advanced superpower. deciding a person’s potential.
Assumptions: Assumptions:
I. Superpowers need not take any heed of global I. Persons with equal potential are not necessarily
public opinion. paid equally.
II. Global public opinion should have been against II. Salary of a person is not linked only with the
the imposition of war. potential.
442. Statement: Wars must be discouraged vehemently 450. Statement: “Everyone desires to buy a personal
even though majority of the victims might have been computer.” — statement of a college student.
a nuisance to peace loving people. Assumptions:
Assumptions: I. Personal computers are not a need but a luxury.
I. Innocent people are also killed in wars. II. Use of personal computer improves quality of skill.
II. Vehement opposition to wars may have some 451. Statement: “The cit y’s t op-ranker f or SSC

KUNDAN
desirable impact. examination this year will be definitely from our
443. Statement: The gov ernment has decided to pay school.” — Principal of a School `X’.
compensation to the tune of Rs 1 lakh to the family Assumptions:
members of those who are killed in railway accidents. I. The teachers of the school have prepared their
Assumptions: students thoroughly.
I. The government has enough funds to meet the II. Most of int elligent student s in t he cit y ar e
expenses due to compensation. studying in School ‘X’.
II. There may be reduction in incidents of railway 452. Statement: “It has become a necessity to computerise
accidents in near future. all the functions of our Institute to maintain the
444. Statement: The X-Air lines has t empor arily present position.” — statement of the Director of XYZ
suspended flights to a few destinations for the next Institute.
four days due to the strike call given by the Pilots’ Assumptions:
Association. I. Unless comput er ised, the Inst it ut e will f all
Assumptions: behind in the race.
I. The airlines may be able to restore all the flights II. The functions of the Institute are too complex to
after four days. be handled manually.
II. The Pilots’ Association may withdraw the strike 453. Statement: “‘Music Loving Club’ has announced a
call within four days. wide v ar iety of pr ogr ammes by the visiting
445. Statement: The civic authority has appealed to the musicians.” — An advertisement
citizens to cooperate in curbing rampant power theft Assumptions:
in the locality. I. A large number of people are interested to get
Assumptions: entertainment through the programme.
I. The local citizens group may respond to the request II. The art ist es may att r act a large number of
and form groups of people to detect such cases of audience.
power theft. 454. Statement: Central Bank, which is the largest bank
II. Those who are engaged in stealing power may in the country, has decided to reduce its workforce
stop doing so for fear of social castigation. by 30 per cent so that its banks may work efficiently.
446. Statement: The Parent Teacher Association (PTA) Assumptions:
Assumptions 37

I. The Bank can perform all its activities after the 462. Statement: The Govt. has made an appeal to all the
reduction in workforce. citizens to honestly pay income tax and file returns
II. The surplus employees may be asked to adopt r ef lect ing t he t r ue income level to help the
early retirement scheme before leaving the bank. Government to carry out developmental activities.
455. Statement: The Principal instructed all the teachers Assumptions:
to be careful in class because some students may I. People may now st ar t paying more taxes in
disturb other students. response to the appeal.
Assumptions: II. The total income tax collection may considerably
I. The teachers may handle the situation properly increase in the near future.
and they may restrict the naughty students. 463. Statement: “We should revise the wage structure in
II. The students will welcome the decision of the order to retain our good employees.” — Comment of a
Pr incipal. member of the Management Committee of a company
456. Statement: The State Government has abolished the Assumptions:
scheme of pr ov iding concessional air t icket t o I. Wage structures of other companies involved in
students. parallel activities are better.
Assumptions: II. Wage structure is one of the most important factors
I. Students will not travel by air in future. for the employees of the company.
II. The students who resort to travel by air can bear 464. Statement: “Do not indulge in unfair practices or
the expenses of air ticket. else you will be sacked from the college.” — Warning
457. Statement: The police has served a notice to the to the students from the principal of a college.
residents with immediate effect to be careful of the Assumptions:
antisocial elements wandering in their areas. I. Some of the students were found using unfair
Assumptions: practices.

K
I. The local residents may pay heed to this notice. II. The principal’s warning may deter the students
II. The antisocial elements may ply to other areas. from using unfair practices.
458. Statement: The General Administration Department 465. Statement: “Invest in our regular scheme and earn
has issued a circular to all the employees informing an interest of at least 10% .” — Advertisement by a
them that henceforth the employees can avail their Financial Institution
lunch break in any of the half-hour slot between Assumptions:
1.00 p.m. and 2.30 p.m. I. 10% interest may att ract a good number of
Assumptions: investors.
I. The employees may welcome the decision and II. No other scheme offers interest as high as 10%.
avail lunch break at different time slots. 466. Statement: “Make use of computers to maintain your
II. There may not be any break in the work of the data and get rid of the hassle.”—An advice to the
organisation as the employees will have their administrative staff by the management
lunch break in different time slots. Assumptions:
459. Statement: The state government has decided to I. Members of the administrative staff are conversant

KUNDAN
appoint four t housand primary school teachers in operating computers.
during the next financial year. II. Maintaining administrative data is an important
Assumptions: activity for any organisation.
I. Ther e ar e enough schools in t he stat e t o 467. Statement: “For easier and faster communication,
accommodate four thousand additional primary use our broadband connections.” — An advertisement
school teachers. Assumptions:
II. The eligible candidates may not be interested to I. Many people ar e looking up for best
apply as the Government may not finally appoint communication facilities.
such a large number of primary school teachers. II. Broadband connections are preferred mode of
460. Statement: The school aut hority has decided to communication.
increase the number of students in each classroom to 468. Statement: “Enrol with us before 30th November to
seventy from the next academic session to bridge the get the advantage of our 20% discount offer.” - An
gap between income and expenditure to a large extent. advertisement by a coaching class
Assumptions: Assumptions:
I. The income generated by way of f ees of t he I. Discount offer is bound to attract good students
additional students will be sufficient enough to as well.
bridge the gap. II. Even those students who cannot afford to pay
II. The school may get all the additional students in the fees of coaching classes may join this class.
each class from the next academic session. 469. Statement: “Join our Yoga institute to keep yourself
461. Statement: The Government has decided against completely fit.” — An advertisement
reduction of prices of petroleum products though Assumptions:
there is a significant drop in the crude oil prices in I. People may prefer exercise to medication.
the international market. II. There is an awareness to a great extent about
Assumptions: Yoga exercises among people.
I. The prices of crude oil in the international market 470. Statement: If you want to get a good job you must
may again increase in the near future. have at least the basic knowledge of computers.
II. The present price difference of petroleum products Assumptions:
will help t he gov er nment to wit hstand any I. All good jobs involve use of computers.
possible price rise in future. II. Computer knowledge has been made an essential
criterion by most of the companies nowadays.
38 Practice Book on Analytical Reasoning

471. Statement: As a measure to avoid occurrence of the II. Ther e may be adequat e number of eligible
epidemics due to monsoon the civic authorities have chartered accountants who may want to join a
organised free vaccination camps all over the city. nationalised bank.
Assumptions: 479. Statement: The municipal authority announced
I. There may be a good r esponse to the camps before the onset of monsoon that the roads within
organized by civic authorities. the city will be free of potholes during monsoon.
II. People are generally aware about the need for Assumptions:
vaccination. I. The roads were repaired so well that potholes
472. Statement: In view of the large number of cases of may not reappear.
suicides committed by the farmers in State X the II. People may not complain even if the potholes
St at e Government has decided to waive off t he reappear.
agricultural loans granted to the farmers. 480. Statement: “Our Europe Holiday Package costs less
Assumptions: t han some of the holiday packages wit hin the
I. This may stop further cases of suicides committed country.” — An advertisement by an Indian travel
by the farmers in State X. company
II. This move of the Government may be welcomed Assumptions:
by the public at large. I. People may prefer to travel to foreign destinations
473. Statement: The government has decided to provide t han to t he places within t he count r y at
monetary relief to the farmers in the drought-hit areas. comparable cost.
Assumptions: II. People generally take their travel decisions after
I. The farmers of the affected areas may accept the getting information from such advertisements.
government relief. 481. Statement: The retail vegetable vendors increased
II. The government machinery may be able to reach the prices of vegetables by about 20 per cent due to
the affected farmers to provide relief. non-availability of vegetables at lower prices at the
474. Statement: All t he st udent s of a school wer e wholesale market.

K
instructed by the Principal to reach school at least Assumptions:
15 minutes before the stipulated time for the coming I. The customers may totally stop buying vegetables
month. at higher prices.
Assumptions: II. The customers may still buy vegetables from the
I. The parents of the students of the school may retail vendors.
protest against the Principal’s instruction. 482. Statement: A large number of students and parents
II. The par ents may r equest t he Principal t o stood in the queue to collect forms for admission to
withdraw the instruction. various under-graduate courses in the college.
475. Statement: Railway authority has started Internet Assumptions:
booking facilit y of long-distance trains and also I. The college authority may be able to admit all
delivering the tickets at the doorstep through courier those who stood in the queue.
service at a little extra cost. II. The college authority may have adequate number
Assumptions: of forms for all those standing in the queue.

KUNDAN
I. Many customers may now book t heir tickets 483. Statement: The General Administration Department
through internet, resulting into less crowd at has issued a circular to all the employees informing
ticket booking offices. them that henceforth the employees can avail their
II. Most of the customers may still buy their railway lunch break at any of the half-hour slots between
tickets at the booking counters. 1.00 pm and 2.30 pm.
476. Statement: “If you have obtained 75 per cent or more Assumptions:
marks in X Std examination, your admission to our I. The employees may welcome the decision and
coaching class f or XII Std is guaranteed. ”— An avail lunch break at different time slots.
advertisement II. There may not be any break in the work of the
Assumptions: organisation as the employees will have their
I. Bright students do not generally opt for attending lunch break at different time slots.
coaching classes. 484. Statement: The Government has decided against
II. The coaching class has adequat e capacity to reduction of prices of petroleum products though
accommodate all such students. there is a significant drop in the crude oil prices in
477. Statement:The municipal corporat ion has given the international market.
permission for holding fun fairs in the local football Assumptions:
ground during the holiday season. I. The prices of crude oil in the international market
Assumptions: may again increase in the near future.
I. The local r esident s may protest against the II. The present price difference of petroleum products
corporation’s decision. will help t he gov er nment to wit hstand any
II. Many people may not participate in the fun fair. possible price rise in future.
478. Statement: A nat ionalised bank issued an 485. Statement: The Govt has made an appeal to all the
advertisement in the national dailies asking the citizens to honestly pay income tax and file returns
eligible candidates to apply for 100 posts of chartered r ef lect ing t he t r ue income level to help the
accountants. Government to carry out developmental activities.
Assumptions: Assumptions:
I. The eligible chartered accountants may respond I. People may now st ar t paying more taxes in
to the advertisement. response to the appeal.
Assumptions 39

II. The total income tax collection may considerably Assumptions:


increase in the near future. I. The management of the organisation may not
486. Statement: The state government has decided to grant leave to most of these employees.
appoint four t housand primary school teachers II. These employees may be able to complete their
during the next financial year. education during the sabbatical leave.
Assumptions: 494. Statement: The college administ r at ion has
I. Ther e ar e enough schools in t he stat e t o instructed all the students to stop using cell phones
accommodate four thousand additional primary within the college premises.
school teachers. Assumptions:
II. The eligible candidates may not be interested to I. The students may stop using cell phones in the
apply as the Government may not finally appoint college premises.
such a large number of primary school teachers. II. The students may continue to use cell phones in
487. Statement: The school authority has decided to the college premises.
increase the number of students in each classroom 495. Statement: The Govt has decided to levy congestion
to seventy from the next academic session to bridge tax on passengers traveling by air to and from the
the gap between income and expenditure to a larger metro cities.
extent. Assumptions:
Assumptions: I. The tax so collected may be adequate to meet part
I. The income generated by way of f ees of t he of the expenses for providing additional resources
additional students will be sufficient enough to to handle huge traffic.
bridge the gap. II. Passengers traveling by air to and from these
II. The school will get all the additional students in cities may be able to pay extra amount by way of
each class from the next academic session. congestion tax.

K
488. Statement: Even though t he number of sugar 496. Statement: The local citizens group submitted a
factories is increasing at a fast rate in India, we still memorandum to the civic authority for allowing them
continue to import it from other countries. to convert the vacant plot in the locality into a garden
Assumptions: at their own cost.
I. Even the increased number of factories may not Assumptions:
be able to meet the demand of sugar in India. I. The local citizen group may be able to gather enough
II. The demand for sugar may increase substantially funds to develop the garden.
in future. II. The civic authority may not accede to the request
489. Statement: The government announced a heavy of the local citizen group.
compensation package for all the victims of the 497. Statement: Most of the private companies have
terrorist attacks. decided against awarding annual increase in the
Assumptions: salaries of their employees for the previous year due
I. Such incidents of terror may not occur in near to the current economic situation.
future. Assumptions:

KUNDAN
II. Compensation may mitigate the anger among the I. Majority of the employees may leave their job to
citizens against the current government. protest against the decision.
490. Statement: Many organizations have switched over II. These companies may announce hike in salaries
to online mode of examinations. next year.
Assumptions: 498. Statement: Mr X st ar ted at 9.00 am f rom his
I. Candidates from all parts of the country may be residence to attend a meeting scheduled to be held
well-versed using computers. at 11.00 am and instructed his assistant to meet
II. Online mode of examinations helps in recruiting him at the venue of the meeting and hand over the
more capable personnel. relevant documents.
491. Statement: Government has decided to relocate all Assumptions:
the factories from the city with immediate effect to I. Mr X may arrive at the meeting place before 11.00
reduce pollution. am.
Assumptions: II. Mr X’s assistant may be able to arrive at the venue
I. Pollution in the city is being caused only because before commencement of the meeting.
of the factories existing there. 499. Statement: The cit y t ranspor t corpor ation has
II. People may be able to manage travelling daily to introduced air-conditioned buses on various routes
the relocated factories. to attract people travelling to their work places by
492. Statement: Gambling through lotteries is banned by car and hence reduce congestion on the roads.
t he Cent r al Gov er nment in all t he st ates wit h Assumptions:
immediate effect. I. Majority of the people may still prefer to travel to
Assumptions: their work places in their own cars.
I. This may save innocent citizens from getting II. Many people may now opt for these buses for
cheated of their hard-earned money. travelling to their work places.
II. The citizens may not gamble in any other way if 500.Statement: The state govt has announced an amnesty
the lotteries are banned. scheme for all the housing societies defaulting on
493. Statement: Many employees of the organisation payment of municipal taxes asking these societies to
applied for special sabbatical leave of two years to pay upfront six per cent of the dues and regularize
pursue higher education. their status without any penalty.
40 Practice Book on Analytical Reasoning

Assumptions: Assumptions:
I. Most of the defaulting housing societies may now I. The traffic department may be able to divert
opt for the amnesty scheme and pay up their dues. movement of vehicles through alternate roads.
II. Other housing societies which have been paying II. The people travelling in the nearby areas may
their taxes regularly may file case against the demonstrate to protest against the authority’s
govt for discriminatory practices. decision.
501. Statement: The railway authority has announced 508. Statement: A major retail store announced thirty per
suspension of movements of train on the main track cent reduction on all food items during the weekend.
within the city limit for carrying out major repair Assumptions:
works on Saturday and Sunday and advised the I. People may still prefer buying food items from
commuters to plan their journey accordingly. other stores.
Assumptions: II. A large number of customers may visit the retail
I. The commuters may protest against the decision store and buy food items.
of the railway authority and may disrupt other 509. Statement: The railway authority has rescheduled
transport services. the departure time of many long-distance trains and
II. The municipal authority may be able to deploy put up the revised timing on its website.
additional buses during Saturday and Sunday to Assumptions:
help the commuters. I. The passengers may note the change in departure
502.Statement: “If you are a first-class graduate with good times from the website.
communication skills and also have work experience II. The passengers may be able to notice the change
of at least two years in sales-related activities, you are and board their respective trains before departure.
welcome in our or ganizat ion. ” —An employment 510. Statement: The school authority has decided to give
advertisement. five grace marks in English to all the students of Std
Assumptions: IX as the performance of these students in English
I. Many with good communication skills may not was below expectation.

K
respond to the advertisement. Assumptions:
II. All the first-class graduates may possess good I. Majority of the students of Std IX may still fail in
communication skills. English even after giving grace marks.
503. Statement: Many people fell ill after consuming meal II. Majority of the students of Std IX may now pass
at a wedding reception and were rushed to the nearby in English after giving grace marks.
govt and private hospitals. 511. Statement: The civic administration has asked the
Assumptions: residents of the dilapidated buildings to move out as
I. The relatives of the affected people may refuse to these buildings will be demolished within the next
take them to the Govt hospitals. thirty days.
II. The nearby hospitals may be able to attend to all Assumptions:
the affected people. I. The civic administration may be able to demolish
504. Statement: The govt has recently announced an these buildings as per schedule.
incentiv e package f or set ting up new business II. The residents of these buildings may vacate and

KUNDAN
v ent ures in t he r ur al ar eas and promised stay elsewhere.
uninterrupted power supply to all the units. 512. Statement: The captain of the school football team
Assumptions: selected only fourteen players to play all the eight
I. The govt may be able to supply adequate power to matches of the interschool football competition.
all such units. Assumptions:
II. People living in the rural areas may welcome the I. There may be adequate number of football players
govt decision. for all the matches.
505. Statement: The municipal aut horit y blocked II. The captain may be able to play in all the matches.
movement of traffic in and around the temple on the 513. Statement: Mohan requested his mother to arrange
main festival day. for food for about thirty persons as he invited all his
Assumptions: friends to celebrate his birthday.
I. Very large number of devotees may visit the temple Assumptions:
on the main festival day. I. Most of Mohan’s friends may come to his house
II. People travelling to the areas near the temple on his birthday.
may postpone their journey by a day unless they II. There may not be more than thirty who may attend
have very urgent work in that area. Mohan’s birthday party.
506. Statement: The government has instructed all the 514. Statement: A very large number of aspiring students
private schools in the city to maintain the current applied for admission to the professional courses run
fees for at least two more years. by the renowned college in town.
Assumptions: Assumptions:
I. The authorities of private schools may not follow I. All the applicants may be able to get admission to
the govt instruction as they are not dependent the college.
on govt funds. II. The admission process adopted by the renowned
II. The parents of the students of private schools of college may be fair to all the applicants.
the city may still be eager to pay higher fees. 515. Statement: The stat e administ rat ion banned
507. Statement: The municipal authority has decided to gathering of more than fifty people at any place during
demolish the old bridge on a bus road for constructing the visit of foreign dignitaries to the city.
a new flyover.
Assumptions 41

Assumptions: 523. Statement:The driver of the huge truck pulled the


I. People may avoid gathering at any place in the emergency brakes to avoid hitting the auto rickshaw
city dur ing t he period of visit of f or eign which suddenly came in front of the truck.
dignitaries. Assumptions:
II. Many people may ignore the prohibitory orders I. The auto rickshaw driver may be able to steer his
and gather to get a glimpse of the dignitaries. vehicle away from the oncoming truck.
516. Statement: The Govt decided to levy a toll tax of Rs II. The truck driver may be able to stop the truck
100 f or ever y v ehicle using t he super highway before it hits the auto rickshaw.
connecting the two big cities of the state. 524. Statement: The doctor warned the patient against
Assumptions: any further consumption of alcohol if he desired to
I. Majority of the vehicles travelling between these get cured from the ailment and live a longer life.
two cities may not use the superhighway. Assumptions:
II. The govt may not be able to recover the cost I. The patient may follow the doctor’s advice and
incurred for constructing the superhighway from stop consuming alcohol.
the toll tax collection. II. The doctor may be able to cure the patient from
517. Statement: The teachers of all the degree colleges t he ailment if t he pat ient st ops consuming
went on an indefinite strike in protest against the alcohol.
Govt’s decision to postpone the pay revision to next 525. Statement: The Chairman of the company urged all
year. the employees to refrain from making long personal
Assumptions: calls dur ing wor king hour s in or der t o boost
I. The Govt may suspend all the striking teachers. productivity.
II. The Govt may r ev ise t he pay of t he college Assumptions:
teachers in the current year. I. Majority of the employees may respond positively

K
518. Statement: The govt has decided t o ear mark a to the Chairman’s appeal.
separate lane in the metropolis for passenger vehicles II. Most of the employees may continue to make long
with more than one occupant. personal calls during working hours.
Assumptions: 526. Statement: The local cultural club decided to organise
I. The move may help decongest the roads of the a musical event to raise money for the construction
metropolis. of the club building.
II. Many people may resort to car pool system to avoid Assumptions:
traffic snarls. I. The local residents may not allow t he club to
519. Statement: Manish invited all his friends to his organise the musical event in the locality.
house for dinner on his birthday and requested his II. The money collected by organising the musical
mother to arrange for the birthday party. event may be substantial enough for the club to
Assumptions: start construction.
I. Most of Manish’s friends may attend his birthday 527. Statement: The traffic police department has put up
party. huge notice boards at all the major junctions of the

KUNDAN
II. Manish’s mother may be able to make all the city, warning drivers to refrain from using cell phones
arrangements including food for all his friends. while dr iving or else t heir licences will be
520. Statement: The civic authority of the metropolis has impounded.
decided to suspend sanctioning of new building Assumptions:
proposals for six months and assess the impact of I. The dr iv er s of t he v ehicles may ignor e t he
the current building projects on the city’s amenities. warning and continue using cell phones while
Assumptions: driving.
I. The builders’ lobby may move the court against II. The traffic police department may be able to nab
the civic body’s decision. most of the offenders and impound their licences.
II. The civic authority may be able to complete the 528. Statement: The largest domestic airlines corporation
impact study in about six months. has announced new summer schedules in which
521. Statement: The railway authority has announced more number of flights in trunk routes are introduced.
that it will carry out major repair work for two days Assumptions:
beginning Saturday on the main line connecting the I. More number of passengers may travel by this
two big cities in the state, bringing the rail service airlines corporation during summer months in
to a halt. trunk routes.
Assumptions: II. Other airlines companies may also increase the
I. People may reschedule their journey in view of number of flights in all the sectors.
the railway authority’s decision. 529. Statement: The chairman of the company decided
II. People may still plan their travel by train between to hold a grand function to celebrate silver jubilee
the two cities even on these two days. during the next weekend and invited a large number
522. Statement: The govt has directed all the degree of guests.
colleges to declare results of all the examinations Assumptions:
within a fortnight after the last date of examination. I. The company officials may be able to make all the
Assumptions: necessar y pr eparat ions f or the silver jubilee
I. The college authorities may not be able to declare celebration.
all the results within the stipulated time. II. Majority of the guests invited by the chairman
II. Many college authorities may not be able to may attend the function.
conduct all the examinations in time.
42 Practice Book on Analytical Reasoning

530. Statement: The largest computer manufacturing city and relocate themselves elsewhere in the
company slashed the prices of most of the desktop state.
models by about 15 per cent with immediate effect. II. Majority of the people living in the shanties along
Assumptions: the beach may try to relocate to higher places
I. The company may incur heavy losses due to during monsoon.
reduction in prices of the desktop. 538. Statement: Please send an official letter rather than
II. The sales of desktop manufactured by the company semiofficial on this subject this time.
may increase substantially in the near future. Assumptions:
531. Statement: The school authority decided to rent out I. The format and emphasis of different types of
the school premises during weekends and holidays letters is different.
for or ganising var ious functions to augment its II. We can send different types of letters on the same
resources to meet the growing needs of the school. subject.
Assumptions: 539. Statement: Please check the availability of two tickets
I. The parents of the school students may protest from Delhi to Lucknow.
against the decision of the school authority. Assumptions:
II. There may not be enough demand for hiring the I. The person checking knows the desired mode of
school premises for organising functions. travel.
532. Statement: The local civic body has urged all the II. The person checking knows the details of the
residents to voluntarily reduce consumption of potable person travelling.
water by about 30 per cent to tide over the water 540. Statement: If you want to increase your writing
crisis. speed, use 0.7 pen.
Assumptions: Assumptions:
I. Many r esidents may r educe consumpt ion of I. There are different types of pen available.
potable water. II. The person being told understands what is 0.7
II. Many activists may welcome the civic body’s move pen.

K
and spread awareness among residents. 541. Statement: In order to build more space, extra FSI
533. Statement: A very large number of people stood in the needs to be bought.
queue for buying tickets for the one-day international Assumptions:
cricket match scheduled to be played in the city on I. The person being told does not know the meaning
the next day. of FSI.
Assumptions: II. More space will reduce the construction cost.
I. No other one-day international cricket match may 542. Statement: Let there be a signboard also indicating
be played in the city for the next six months. the directions and instructions.
II. Majority of those who stood in the queue may be Assumptions:
able to get ticket for the one-day international I. Signboard can be prepared without using any
cricket match. language.
534. Statement: The highway police authority put up large II. Signboard is the only effective tool to indicate
boards at regular intervals indicating the speed limit directions.

KUNDAN
and dangers of over-speeding on the highways. 543. Statement: Banks should always check financial
Assumptions: status before lending money to a client.
I. Most of the motorists may drive their vehicles Assumptions:
within the speed limit on the highways. I. Checking bef ore lending would give a t rue
II. Motorists generally ignore such cautions and over- picture of the client’s financial status.
speed on the highways. II. Clients sometimes may not present the correct
535. Statement: The employees’ association urged its picture of their ability to repay loan amount to
members to stay away from the annual function as the bank.
many of t heir demands wer e not met by t he 544. Statement: The government has decided to run all
management. commercial vehicles on bio-fuels in order to save the
Assumptions: depleting fossil fuel reserves.
I. Majority of the members of the association may Assumptions:
not attend the function. I. It is possible to switch over from fossil fuels to
II. The management may cancel the annual function. bio-fuels for vehicles.
536. Statement: The sarpanch of the village called a II. Sufficient amount of bio-fuel can be produced in
meeting of all the heads of the families to discuss the country to run all commercial vehicles.
the problem of acute shortage of drinking water in 545. Statement: To save the environment enforce total ban
the village. on illegal mining throughout the country.
Assumptions: Assumptions:
I. The sarpanch had earlier called such meetings I. Mining which is done legally does not cause any
to discuss about various problems. harm to the environment.
II. Most of the heads of families may attend the II. Mining is one of the fact or s responsible for
meeting called by the sarpanch. environmental degradation.
537. Statement: The municipal corporation advised all 546. Statement: Give adequate job-related training to the
the people living in the shanties along the beaches employees before assigning them full-fledged work.
to move to higher places during monsoon. Assumptions:
Assumptions: I. Training helps in boosting the performance of
I. Many people living in the shanties may leave the employees.
Assumptions 43

II. Employees have no skill sets before training is 552. Statement: A leading NGO decided to open a library
provided to them. cont aining books and newspapers of all major
547. Statement: Take a ferry or a boat instead of a bus to publishers in a remote village.
reach the Kravi islands faster. Assumptions:
Assumptions: I. All other nearby villages already have similar
I. The islands being in remote location are not easily libraries.
accessible. II. There is adequate number of literate people in
II. Ferries and boats are available to travel to Kravi the village.
islands. 553. Statement: If parking space is not available in office,
548. Statement: A leading university has begun a practice park your vehicles in the mall and walk to the office.
of displaying results only on the Internet rather than Assumptions:
on the main notice boards. I. The mall is at a walkable distance from the office.
Assumptions: II. The office does not allow visitors’ vehicles in its
I. All the students enrolled with the university have premises.
access to Internet at home. 554. Statement: Farmers must immediately switch over
II. Most of the student s r eferred t o the results to organic fertilizers from chemical fertilizers for
displayed on both the internet as well as the better yield.
notice boards earlier. Assumptions:
549. Statement: In order to replenish the nutrients in I. All the farmers use only chemical fertilizers.
the soil, it is important to grow different types of II. Organic fertilizers are readily available to the
crops every alternate season. farmers.
Assumptions: 555. Statement: An advertisement by bank X — ‘Our
I. A crop can never be grown for the second time in interest rates for education loans are lower than those

K
the same field. of any other bank.’
II. If a different crop is grown in the successive Assumptions:
season, no additional nutrients such as fertilizers I. Some other banks also provide education loans.
are required to be added to the soil. II. Interest rates charged on education loans are
550. Statement: If farmers want to improve their yield, different for different banks.
they must use organic fertilizers in place of chemical 556. Statement: For any kind of problem with your mobile
fertilizers. phone, contact our helpdesk immediately.
Assumptions: Assumptions:
I. Chemical fertilizers have certain ill effects on I. Helpdesk has a solution to all kinds of problems
health. r elat ed t o mobile phones or will guide
II. Chemical fertilizers do not produce as much yield accordingly.
as the organic fertilizers. II. Unless the problem is reported immediately, it
551. Statement: Store eatables in the deep freeze in order cannot be solved.
to preserve these for a long time. 557. Statement: Use our medicine to fight the problem of

KUNDAN
Assumptions: obesity.
I. Food material remains eatable even after deep Assumptions:
freezing for a long time. I. Other slimming medicines available in the market
II. It is not possible to store any eatable at room do not reduce weight.
temperature even for a shorter period of time. II. Obesity cannot be controlled without medicines.

Practice Exercise–2
Directions: In each question below is given a 3) Only I and III 4) All I, II and III
statement followed by three assumptions numbered I, 5) None of these
II and III. An assumption is something supposed or 2. Statement: Despit e st r ong opposition, t he
taken for granted. You have to consider the statement controversial Prohibition of Forcible Conversion of
and the assumptions and decide which of the Religion Bill was passed by the Tamil Nadu Assembly
assumptions is implicit in the statement. Then decide with the AIDMK and the BJP outvoting the combined
which of the answers (1), (2), (3), (4) and (5) is the opposition of the DMK, the Congress, the Pattali
correct answer. Makkal Katchi and the Left parties. — A news
1. Statement: “Bar Council of India (BCI) has decided Assumptions:
to go on a hunger strike to protest the implementation I. Conversions create resentment among several
of the Legal Services Authority (Amendment) Act.” — sections and also inflame religious passions,
Chairman of BCI leading to communal clashes.
Assumptions: II. Conversions only lead t o the isolation of the
I. The amendment has several loopholes and is converted.
bound to hurt the litigants’ interests. III. Strong opposition puts hindrance before a bill in
II. All the members of BCI will welcome the decision. taking final shape.
III. The hunger strike held in front of the court will 1) All I, II and III 2) Only I and II
put a pressure on the respective authority. 3) Only I and III 4) Only II and III
1) Only I 2) Only I and II 5) None of these
44 Practice Book on Analytical Reasoning

3. Statement: For the third time in a row this week, and will be compelled to improve the condition
the Indira Gandhi International Airport was put on of the institutes if the names of the institutes are
a full alert today after Air India received an anonymous published in the weekly magazine.
message that two terrorists laden with explosives II. The move will be helpful for admission-seekers
were aboard its Mumbai-Delhi-Hong Kong flight. — from being a victim of degraded standard of the
An authority of Indira Gandhi International Airport institutions.
Assumptions: III. The editor may concede the request and take
I. Anonymous message is not likely to be hoax call. positive steps in this regard.
II. Anonymous message is likely to be a hoax call. 1) Only I and II 2) Only II and III
III. Aler t ness might be helpf ul in reducing t he 3) Only I and III 4) All I, II and III
menace. 5) None of these
1) Only I and III 2) Only II and III 8. Statement: “City ‘X’ should have more women driving
3) Either I or II and III 4) Only III public transport because most do not like to speed
5) None of these unnecessarily, prefer not to talk on mobiles while
4. Statement: “You should be very careful so that the driving, do not listen to blaring music and rarely
society does not adversely comment on the police skip red lights.” — A minister of state X
leadership.” — A statement made by a CBI officer to Assumptions:
the newly recruited young officers I. More women driving buses and autos will usher
Assumptions: more job opportunities for women.
I. The society always indulge in adverse comments II. Women are more disciplined than men on the
II. People’s trust in the police force is not up to the roads.
desired level. III. If women start driving buses and autos it would
III. Society has the efficiency to judge accurately about increase chances of safety for commuters.
the police leadership. 1) Only I and II 2) Only II and III
1) Only I and II 2) Only II and III 3) Only I and III 4) All I, II and III

K
3) Only I and III 4) None 5) None of these
5) None of these 9. Statement: “Performing complex mental tasks whilst
5. Statement: “If we really want to reduce the menace driving is dangerous.” — A psychologist
of smoking, we need to reflect our intention through Assumptions:
our deeds, our creations such as movies, in which I. Thinking too much reduces the ability to spot
incidence of smoking is shown much higher than potential problems and react to them safely.
the actual cigarette consumption among the Indian II. Handsfree phone conversations are as safe—or
population.” — View of Mr X. as risky — as talking to a passenger, depending
Assumptions: upon the demands of conversation.
I. There is a strong link between films and viewers’ III. Insecurity of humans is a man-made problem.
behaviour. 1) Only I and II 2) Only II and III
II. Smoking shown in mov ies r esult s in 3) Only I and III 4) All I, II and III
corresponding increase in smoking among the 5) None of these

KUNDAN
public. 10. Statement: The boar d of direct ors of ‘X’ coaching
III. Menace of smoking can be reduced by proper institute — a premier coaching institute — has decided
planning followed by appropriate efforts. to charge a fixed amount of Rs 10,000/month from each
1) Only I 2) Only II of its franchisees in urban area and Rs 5000/month in
3) Only II and III 4) Only I and III rural area.
5) All I, II and III Assumptions:
6. Statement: “Why ar e only highprof ile people I. Profits gained by franchisees in urban area differ
considered to be adventurous? The villagers in India, from that in rural areas.
who have no buses, walk barefoot for miles on dusty, II. Franchisees will be able to pay the respective
untarred roads. Isn’t that more adventurous than charges according to the respective categories.
rafting or gliding?” — View of a person III. It is feasible to expand branches and control them
Assumptions: by fixing a target.
I. It is a Herculean task to walk barefoot for miles 1) Only I 2) Only II
on dusty untarred roads. 3) Only I and II 4) Only III
II. Rafting and gliding are considered adventurous 5) None of these
by the people. 11. Statement: “Most schools in India continue to be
III. Walking barefoot for miles on dusty untarred roads t eacher-dr iv en, wher e st udent s hav e litt le
is not considered adventurous by the people. participation in the learning process. There is no
1) Only I and II 2) Only II and III response to societal changes that have been induced
3) Only I and III 4) All I, II and III by globalisation, developments in communications,
5) None of these networking and technology.” — A criticism made by
7. Statement: “You should publish the names of the Mr X
ten worst colleges or institutions, which will lead to Assumptions:
an ov erall improvement of Indian institutes.” — I. True education means greater participation of
Request of a reader to the editor of a weekly magazine. students and congruence with societal changes.
Assumptions: II. Education can’t be obtained by assuming teacher
I. The management and administr at or s of the as the only source.
respective institutes will come under pressure
Assumptions 45

III. It is desirable that education be imparted by 16. Statement: Hurdles in restoring security in travelling
considering those factors in mind which are in through train will be removed by the railway ministry
tune with the needs of the student. and travelling by train will once again be perceived
1) Only I 2) Only II as safe.
3) Both I and II 4) Both II and III Assumptions:
5) None of these I. The railway ministry has cr eat ed hurdles in
12. Statement: “We must select educat or s and not travellers’ security.
academics. They should have the ability to teach in II. Today travelling by train is not considered safe.
the canteen or even at bus stop!” — Mr X III. It is possible to make train journey safe.
Assumptions: 1) Only I and II 2) Only II and III
I. Teaching is an ar t . It asks f or a thor ough 3) Only I and III 4) All I, II and III
knowledge of the subject as well as the ability to 5) None of these
put across that knowledge to others. 17. Statement: “Addictive ‘gutka’ and ‘paan masala’ with
II. The skills required in transmitting knowledge or without tobacco has been banned from Aug 1, 2002
are, at times, quite different from a person’s in Maharashtr a, wher e even school and college
receiving skills. student s wer e increasingly becoming t heir
III. It is not necessarily true that a good student will consumers.” — A court notice
also be a good teacher. Assumptions:
1) Only I 2) Only I and III I. Some of the ‘gutka’ manufacturers may not abide
3) Only II and III 4) All by the court’s directions.
5) None of these II. School childr en and college student s ar e
13. Statement: The World Food Summit in Rome was on vulnerable to gutka.
the verge of becoming a fiasco on its opening day III. Consumers as well as manufacturers of ‘gutka’

K
when western countries stayed away, prompting will abide by the court’s directions.
accuslations of indif ference to the f ate of the 1) Only II 2) Either I or III and II
malnourished and starving people because the third 3) Either I or III only 4) Only II and III
world countries send civil servants and ministers 5) None of these
rather than prime ministers and presidents. 18. Statement: “Some lessons from Israel, especially in
Assumptions: the field of curbing terrorism, would be relevant to
I. Prime ministers and presidents are more efficient us.” — Defence Minister of India.
than civil servants and ministers. Assumptions:
II. Prime ministers and presidents enjoy more power I. Israel has a good strategy and a better facility to
than civil servants and ministers. counter terrorism.
III. A meaningful outcome can be obtained only when II. Isr ael has a bit ter experience of count er ing
the real powers attend a summit. terrorism.
1) Only I and II 2) Only II and III III. Isr ael and India hav e inher it ed a lot of
3) Only I and III 4) None commonness in terms of land, people and society.

KUNDAN
5) None of these 1) Only I and II 2) Only I and III
14. Statement: Mandatory verification of identity before 3) Only II and III 4) Only II
acquiring a prepaid cellular connection is taking a 5) None of these
toll on numbers. — CEO of a cellular company 19. Statement: “The return of country X as a full-fledged
Assumptions: member of the Commonwealth is dependent on the
I. Lesser subscribers have joined due to decrease ‘credibility’ of the election which will be in process
in subscription rate. next year.” — Head of Commonwealth Observers’
II. Unnecessar y delay as well as r ed-t apism Group (COG).
discourages consumer’s appetite. Assumptions:
III. Holders of cellulars do not want to reveal their I. ‘Cr edibilit y’ of the election pr ocess can be
identity. measured in tangible terms.
1) Only I and II 2) Only II and III II. Election process in country X always remains a
3) Only I and III 4) None matter of debate for the world community.
5) None of these III. World community has a common desire to restore
15. Statement: “If you did not have a mobile connection democracy and its ethics all over the world.
of MTNL you will feel a jolt when you come to know 1) All I, II and II 2) None
about a unique, free incoming facility from 10 PM to 3) Only I 4) Only II
8 AM, without any condition, for any network.” — X 5) None of these
tells Y 2 0 . Statement: “The Indian Meteorological Department
Assumptions: has proposed to hold a brainstorming session of
I. X knows about the facilities provided by other weather experts next month to try and study the
mobile services. causes for the failure of the just-concluded summer
II. X wants Y to be a subscriber of MTNL mobile monsoon in the country.”—Spokesperson of Indian
services. Meteorological Department
III. People like free incoming facilities without any Assumptions:
condition. I. The behaviour of the just-concluded monsoon was
1) Only I 2) Only I and II intriguing.
3) Only II and III 4) Only I and III II. Thorough scientific investigation is likely to reveal
5) None of these the causes of failure of the monsoon.
46 Practice Book on Analytical Reasoning

III. Brainstorming sessions organised in the past 1) Only I 2) Only I and II


didn’t prove fruitful. 3) Only II and III 4) Only I and III
1) All I, II and III 2) None 5) All I, II and III
3) Only II 4) Only I and II 26. Statement: “A court can convict an accused solely
5) None of these on t he basis of a dying declarat ion but such a
21. Statement: “Why are you looking sad? Did you not declaration should be free from any doubt and the
get bonus this year also”? — Mr X says to Mr Y victim making the statement should be mentally fit.”
Assumptions: — Supreme Court
I. Mr X is the well-wisher of Mr Y. Assumptions:
II. Expression of sadness on one’s face is a sign of I. One who is not ment ally f it always makes
one’s financial crunch. statements untrustworthy in nature.
III. If one gets bonus, one does not remain sad. II. Declaration made by the dying person is likely to
1) All 2) Only I be true.
3) Only I and II 4) Only II and III III. It is possible to distinguish whether a declaration
5) None of these is dubious or not.
22. Statement: Non-Brahmin well-versed with rituals, 1) Only I 2) Only I and II
could be appointed as a pujari as well. — Ruling of 3) Only II and III 4) Only III
Supreme Court. 5) None of these
Assumptions: 27. Statement: “The ‘X’ st ate cabinet endorsed our
I. Brahmins don’t have a monopoly over performing proposal to stagger shopping timing by allowing
puja in a temple. markets to remain open til 10 PM to make it more
II. Mere eligibility for a post is enough to lay claim dynamic.” — A leader of traders
to the candidatureship for the post. Assumptions:
III. People will comply with the verdict given by SC. I. If it is taken well by traders, the state government
1) All I, II and III 2) Only I and II will permanently alter the closing time for all

K
3) Only II and III 4) Only I and III prominent markets in the city.
5) None of these II. There is a need to boost the commercial activities
23. Statement: “Every successful person who claims to in the city.
have come from a village has achieved success after III. Keeping the market open longer is likely to
he or she left the village.” — A leader of party X enhance sales as well as making shopping more
Assumptions: convenient for people with late working hours.
I. The average Indian village is a place with little 1) Only I 2) Only I and II
capital, low technology and limited market access. 3) Only II and III 4) Only I and III
II. Opportunity for growth is more in metro cities. 5) None of these
III. Opportunities for growth are scarce in villages. 28. Statement: The high court of state ‘X’ has directed
1) All I, II and III 2) Only I and III the state government ‘X’ to issue a notification making
3) Only II and III 4) Only I and II speed governors compulsory for all four-wheelers in
5) None of these the city to check high-speed driving.

KUNDAN
24. Statements: “Today I am rejecting your proposal to Assumptions:
play a cricket match against your team because of I. Speed governors will put an end to accidents.
absence of Mr Z but tomorrow I am ready to play II. Speed gov er nors will help in reducing t he
against your team at any cost.” — Mr X says to Mr Y possibilities of road accidents.
Assumptions: III. High speed of vehicles on the r oad causes
I. Mr Z will be available tomorrow. accidents.
II. Match will be played tomorrow irrespective of 1) Only I 2) Only I and II
availability of Mr. Z. 3) Only II and III 4) Only I and III
III. Mr Y will be ready to play a match against Mr X 5) None of these
tomorrow. 29. Statement: “We believe that the adjustment that has
1) All I, II and III 2) Either I or II and III already taken place in the value of the rupee is
3) Only II and III 4) Only I and III adequate. There is no ground for a further weakening
5) None of these of the rupee.” — RBI governor
25. Statement: “An asteroid which burned up in the Assumptions:
earth’s atmosphere in June could have triggered a I. The value of the rupee has depreciated recently.
mistaken nuclear war between India and Pakistan II. Dollar-buying has been on the rise.
had it detonated over South Asia.” — Statement of Mr III. Currencies do not fall after a certain level.
X in a report in The New York Times. 1) None 2) Only III
Assumptions: 3) Only I and III 4) Only II
I. India and Pakistan are equipped with nuclear 5) Only I and II
weapons. 30. Statement: For realising a 7% GDP growth rate, we
II. Neither India nor Pakistan had the sophisticated would need our gross investments to increase to 28-
sensors t hat could det ermine the dif ference 30 per cent. This is not a tall order.
between a natural NEO (near-earth object) impact Assumptions:
and a nuclear detonation. I. 7% GDP growth rate seems to be a far cry.
III. India and Pakistan have no good relationship II. The more you invest, the better the GDP growth.
with each other. III. The GDP growth rate is the indicator of a nation’s
development.
Assumptions 47

1) Only I 2) All the three I. Nothing is impossible if proper efforts are made.
3) Only II 4) Only II and III II. Technology needs ar e diff erent f or dif fer ent
5) Only III sections of society.
31. Statement: “Let us enter into a deal with company Y III. It is possible to develop appropriate technologies
to come out of our financial crisis.” — Company X for various economic sections of the society.
Assumptions: 1) Only I 2) Only III
I. Company Y can bail company X out of its financial 3) Only II 4) Both II and III
crisis. 5) None of these
II. Two companies can enter into a deal. 37. Statement: “We have the distinction of being the
III. Company Y is financially sound. only company in India as well as the second in the
1) Only II and III 2) Only II world to have won an ISO 9002 quality certification
3) Only I and III 4) Only III in our line of business”- Statement of company X’s
5) None of these Chairman.
32. Statement: “Though party A is firm on the ouster of Assumptions:
party B, it is unlikely to reap any benefit from it.” — I. There were not many companies in the line of
a political observer business of Company ‘X’.
Assumptions: II. Gett ing ISO 9002 in the line of business of
I. Party A expects to benefit from the ouster of party Company ‘X’ is not easy.
B. III. The company ‘X’ desires to expand its business.
II. Polit ics is a game of manoeuv ring and 1) Only I 2) Only II
manipulations. 3) Only III 4) Only II and III
III. The political scenario is peaceful. 5) None of these
1) Only I 2) Only II 38. Statement: Move into the upper echelons without

K
3) Only III 4) All the three paying a steep price. Book a luxurious flat with us.
5) None of these — Advt. of a construction company for its prestigious
33. Statement: The ‘licence r aj’ might hav e been project
vanquished by the reforms but ‘inspector raj’ is Assumptions:
thriving. I. It is possible to join the select band of rich people
Assumptions: by hard work.
I. The ‘licence raj’ is the same as the ‘inspector raj’. II. Staying in luxury without paying steep price is
II. The reforms should have put an end to the ‘licence the criterion of upper crust of society.
raj’. III. Booking a luxurious flat is very easy now.
III. The ‘inspector raj’ is a menace. 1) Only II 2) Only III
1) None 3) Only II and III 4) None
2) All the three 5) None of these
3) Only I and III 39. Statement: The employees’ association has appealed
4) Only II and III to the Managers of Company ‘S’ to introduce written

KUNDAN
5) Only III examination for Clerical cadre recruitment to prevent
34. Statement: “A rare opportunity to be a professional selection of incompetent persons.
while you are at home.” — An advertisement for Assumptions:
computer-literate housewives by a computer company I. So far the Company ‘S’ used to select candidates
Assumptions: without conducting a written examination.
I. Some housewiv es simultaneously desir e t o II. A wr it ten examinat ion can help t o identif y
become professional. competent persons.
II. Computer industry is growing at a fast pace. III. At higher level written examination may not be
III. It is possible to be a professional as well as a of much use.
housewife. 1) Only I and II 2) Only II and III
1) Only I and II 2) Only II and III 3) Only III 4) Only I and III
3) Only I and III 4) Only II 5) None of these
5) None of these 40. Statement: The Government of India has set up one-
35. Statement: India’s economic growth has come at a stop facilitation counters manned by trained staff to
terrible price of increased industrial and vehicular attend to the various needs of the foreign tourists at
pollution. all the international airports.
Assumptions: Assumptions:
I. Pollution is a part of industrial society. I. There are adequate trained staff available to man
II. Indian economic gr owt h is based on only these counters in shifts.
industrial growth. II. The services provided by these counters will help
III. A count r y desir es economic gr owt h wit h boost inflow of foreign tourists.
manageable side-effects. III. Majority of the foreign tourists need variety of
1) Only I 2) Only II services when they reach India.
3) Only I and III 4) Only III 1) Only I and II are implicit
5) None of these 2) Only II and III are implicit
36. Statement: Efforts to develop technologies more 3) Only III is implicit
appropriate to the needs of the poorest sections of 4) All are implicit
society need to be further intensified. 5) None of these
Assumptions:
48 Practice Book on Analytical Reasoning

41. Statement: The ‘X’ car manufacturing company has III. The general public may support the cause of the
decided to increase price of the cars in A, B and C cinema hall owners and put pressure on the
segments ranging from 5% to 10% with immediate government to accept their demands.
effect as the steel prices have risen considerably in 1) None is implicit
the recent past. 2) Only I is implicit
Assumptions: 3) Only III is implicit
I. The prices of cars other than in A, B and C 4) Only II is implicit
segments are already very high and need not be 5) All are implicit
increased. 45. Statement: A one-day token strike was called by the
II. The rival car manufacturing companies may also employees in Government organisations to protest
increase the prices of cars in these segments. against privatization of profit-making Public Sector
III. There may be adequate demand in the market of Undertakings.”
the cars in these segments even after the price Assumptions:
hike. I. The Government may favourably consider the
1) Only I is implicit views of the employees.
2) Only II is implicit II. Strike is the most popular tool used by people to
3) Only III is implicit protest.
4) Both I and III are implicit III. The strike may bring pressure on the Government,
5) All are implicit forcing them to reconsider the decision.
42. Statement: The university authority has decided to 1) Only I is implicit
decentralise conduct of terminal examinations and 2) Only II is implicit
give this responsibility to each college for its students 3) Only I and III are implicit
to avoid delay in declaration of results. 4) All I, II and III are implicit
Assumptions: 5) None of these
I. The colleges are equipped t o car r y out t his 46. Statement: “Buy a variety of items from our shop

K
responsibility. and get upto 20% discount.” - An advertisement
II. There may not be uniformity in evaluation standard Assumptions:
across the colleges. I. Many people will visit the shop to get the benefit
III. The students may welcome this new development. of discount.
1) None is implicit II. Customers may go to some other shop if the
2) Only II and III are implicit discount is not offered.
3) Only I and II are implicit III. Of fering discount is the best way t o att ract
4) Only I and III are implicit customers.
5) None of these 1) Only I is implicit
43. Statement: The civic authority has decided that all 2) Only II is implicit
the factories located inside the city limit be shifted 3) Only III is implicit
outside to reduce the level of environmental pollution 4) Only I and II are implicit
in the city. 5) None of these

KUNDAN
Assumptions: 47. Statement: “The municipal corporation of the city is
I. The pollution level in the city in future may reduce granting permission to builders for new construction
after these factories are shifted outside the city of high-rise buildings in the city, despite the appeal
limit. from the environmentalists to avoid overcrowding.”
II. Enough usable land is available outside the city Assumptions:
limit for these factories. I. The cor porat ion will take car e t o f ulfil the
III. Many of these factories may shift to some other minimum requirements providing infrastructure
smaller town to remain profitable. in the area for the newly constructed building.
1) Only I is implicit II. Corporation is expecting good earnings by way of
2) Only I and II are implicit taxes from the high-rise buildings.
3) Only II is implicit III. The corporation has taken the residents of the
4) Only II and III are implicit area into confidence and assured them of no
5) None of these inconvenience to them due to new construction.
44. Statement: All the single-screen theatre halls in the 1) Only I is implicit
city have declared indefinite strike and have warned 2) Only II is implicit
t hat they will not withdr aw str ike unless t he 3) Only III is implicit
gover nment accept s t heir demand of reducing 4) Only I and II are implicit
entertainment tax to 50% of the present level and 5) None of these
also treat these halls at par with multi-screen halls 48. Statement: “Our school provides all facilities like
on all the relevant matters. school bus serv ice, computer tr aining, spor t s
Assumptions: facilities. it also gives opportunity to participate in
I. The employees of all the cinema halls may various extra-curricular activities apart from studies.”
disagree with the management and may appeal - An advertisement by a public school.
to the government to declare the strike illegal. Assumptions:
II. The government may accept all the demands of I. Nowadays extra-curricular activities assume more
the striking cinema halls to avoid any backlash importance than studies.
from the public. II. Many parents would like to send their children
to the school as it provides all the facilities.
Assumptions 49

III. Overall care of the child has become the need of (C) The Govt may not allow the apex body t o
the time as many women are working. implement its decision in all the colleges as it
1) Only I is implicit may lead to chaos.
2) Only II is implicit 1) None is implicit
3) Only I and II are implicit 2) Only (A) is implicit
4) All I, II and III are implicit 3) Only (B) is implicit
5) None of these 4) Only (C) is implicit
49. Statement: “Graduates with first-class are eligible 5) Only (A) and (B) are implicit
to apply for the admission to MBA courses in our 52. Statement: Govt has urged all the citizens to use
Institute”. - An advertisement by a Management electronic media for carrying out their daily activities,
Institute. whenever possible, instead of using paper as the
Assumptions: manufacture of paper requires the cutting down of a
I. Only those who are first-class graduates can cope large number of trees causing severe damage to the
up with the studies for MBA courses. ecosystem.
II. There are plenty of first class graduates who are Which of the following assumptions is/are implicit
likely to apply for admission to MBA. in the above statement?
III. The reputation of the Institute may get affected if (A) Most people may be capable of using electronic
students having less than first class are admitted. media to carry out various routines.
1) Only I is implicit (B) Most people may have access to electronic media
2) Only II is implicit for carrying out their daily routine activities.
3) Only III is implicit (C) People at large may reject the govt’s appeal and
4) Only I and II are implicit continue using paper as before.
5) All I, II and III are implicit 1) Only (A) is implicit

K
Directions (Q. 50-54): In each question below is 2) Only (B) is implicit
given a statement followed by three assumptions (A), 3) Only (A) and (B) are implicit
(B) and (C). An assumption is something supposed or 4) Only (C) is implicit
taken for granted. You have to consider the statement 5) None of these
and the following assumptions and decide which of 53. Statement: The Govt has decided to auction constru-
the assumptions is implicit in the statement. ction of highways to private entities in several blocks
50. Statement: The police authority cordoned off the across the country on build-operate-transfer basis.
entire locality for the entire day and stopped all Which of the following assumptions is/are implicit
vehicular movement for the visit of a top functionary in the above statement?
of the government in view of the threat perception (A) An adequate number of private entities may not
and advised all the residents in the area to limit respond to the Government’s auction notification.
their movement outside their dwellings. (B) Many private entities in the country are capable
Which of the following assumptions is/are implicit of constructing highways within a reasonable
in the above statement? time.

KUNDAN
(A) The police personnel may not be able to control (C) The Govt’s proposal of build-operate-transfer may
the vehicular movement in the locality and may financially benefit the private entities.
seek help from the armed forces. 1) Only (A) and (B) are implicit
(B) People living in the locality may move out of their 2) Only (B) and (C) are implicit
houses for the day to avoid inconvenience. 3) Only (B) is implicit
(C) The Govt functionary may request the police 4) Only (A) and (C) are implicit
authority to lift the ban on the movement of 5) None of these
residents of the locality outside their dwellings. 54. Statement: The airlines have requested all their bona
1) None is implicit fide passengers to check the status of flight operations
2) Only (A) is implicit before leaving their homes as heavy fog is causing
3) Only (B) is implicit immense problems to normal flight operations.
4) Only (C) is implicit Which of the following assumptions is/are implicit
5) Only (B) and (C) are implicit in the above statement?
51. Statement: The apex body controlling universities (A) Majority of the air passengers may check the flight
in the country has decided to revise the syllabus of status before starting their journey to the airport.
all the technical courses to make them focused (B) The Govt may take serious objection to the notice
towards the present needs of the industry, thereby issued by the airline company.
making the technical graduates more employable than (C) Majority of the passengers may cancel their tickets
they are at present. and postpone their jour ney till the situation
Which of the following assumptions is/are implicit becomes normal.
in the above statement? 1) None is implicit
(A) Technical colleges af f iliated to dif f erent 2) Only (A) is implicit
universities may not welcome the apex body’s 3) Only (B) is implicit
decision and may continue with the same syllabus 4) Only (C) is implicit
as at present. 5) Only (A) and (C) are implicit
(B) The industry may welcome the decision of the
apex body and scale up their hiring from these
colleges.
50 Practice Book on Analytical Reasoning

Answers and explanations


Practice Exercise–1
1. 1; I is implicit. That is why the speaker is desirous 18. 1; The comparison of literacy with logical thinking
of showing the teachers the correct path. II is hints about I. But comparison of literate and
not implicit: teachers should be ideal but there illiterate can’t be assumed from the statement.
is no clue why. 19. 5; As speaker is using the statement of eminent
2 . 2; As in the statement it is given that most of the people to give str engt h to his statement, he
...., hence II is implicit. I is not implicit. Basic believes in eminent people and assumes that they
qualification is not necessary but sufficient to are always true.
differentiate between right and wrong. 2 0 . 2; The worst bus service is because of lowest fares.
3. 5; Both ar e implicit. The phr ase ‘passing time’ So, it is clear that the speaker is assuming that
confirms both the assumptions. increase in fare will improve the service. Hence
4. 2; Even a good suggestion by safety boards cannot II is implicit. The speaker is relating fare with
prevent accidents if the employees will not desire services only. Hence I can’t be an assumption.
to implement those suggestions. Hence II is 21. 2; Why did management search ingenious idea for
implicit and I is not implicit. shirking work? Definitely because of II. Hence II
5. 4; Nothing can be assumed about the success and is assumed. Nothing can be assumed about the
failure of the committees. employee strength of the corporation.
6. 5; The context is that of films. And the comparisons 2 2 . 2; I may be a restatement or conclusion, but it is not
are to “erasing” a painting and “deleting” lines. an assumption. Hence I is not implicit. As the
Hence I is implicit. The entire statement is a plea speaker is pointing out about bad manners, it

K
for II. can be assumed that he knows civic manner.
7. 1; I is implicit; hence the distinction is being made. 23. 5; As t he speaker is adv ising to oppose cr iminal
II is not implicit: giving priority to emergency candidates in elections, it can be assumed that
services is a social necessity; one can’t bother criminals can fight elections. Hence I is implicit.
for whether the people will mind or not. The phrase irrespective of their party labels hints
8. 2; Mahatma Gandhi would have been pained to see about II.
“high living and simple thinking” leaders means 24. 4; I is not exhaustive. It may be about elections on
that he believes in the exactly opposite nature, tough fight between two popular candidates; may
ie simple living and high thinking. Hence II is be the war between the US and Russia .... Hence
implicit. I cannot be assumed from the given I is not implicit. About II, nothing can be assumed.
statement. 25. 1; As the speaker is predicting that General Motors
9. 5; Why does the concerned authority need to remind and Daewoo can be strongest player in the auto
people about democratic right? Definitely because maket only together, clearly means that he is

KUNDAN
of II. Hence II is implicit. When people will cast assuming that Daewoo alone is not sufficient to
t heir v ot e natur ally polling per cent age will be No. 1.
increase. Hence I is implicit. 26. 5; Imposing tickets generally serves two purposes
10. 1; The book which speaks about God is holy book. — restricting unlimited entry and raising money.
11. 1; ‘Forced to leave’ clearly indicates that no one The latt er has no use her e. Now, in t he
wants to put it down on their own and everyone statement, it is given that there were no tickets
enjoys reading it. Hence I is implicit. II has no and entry was restricted, clearly indicates that
relation with the statement. both are implicit.
12. 2; Why too many sub-standard private tuition centres 27. 4; I and II may be the reason for stating such a
in all locality? — only because of assumption II. warning. But these are not exhaustive.
Hence II is implicit. I is not implicit. 28. 5; Both are implicit. That is why the advertiser is
13. 5; Why does Railways need to show a good safety stressing on both design and its resale value.
performance? Definitely because of II. 29. 4; II can’t be assumed from the statement because
14. 5; As speaker is comparing cultures and talking nothing is given about Y2K specialist. I is not an
about the world, I is implicit. As speaker used assumption but a conclusion.
the term I found... means II is implicit. 30. 5; The term half the job done helps in assuming both
15. 4; I is an implication, not an assumption. II is not the assumptions. Half means students too need
implicit because we don’t know the context in to do hard labour for the rest half. Hence II is
which the statement is said. implicit. I is implicit since the half provided by
16. 4; I is not implicit: the statement has nothing to do guidance is also necessary.
with what the animals eat. II is not implicit 31. 4; II can’t be assumed. It may be about two business
because (a) merely giving protection does not fit partners, two scientists... As there is no must
one with these attributes; and (b) even if (a) were term in the statement, I is not implicit.
true, we are not protecting all the animals. 32. 1; The assumpt ion I is v alid. The speaker is
17. 5; The government is thinking that IBRD will give assuming that by possessing such qualities, one
the loan, that is why they have applied for it. Hence may overcome bottlenecks in the way to success.
I is implicit. Why do they need to widen the road? W e can’t assume whet her possessing such
Definitely because of II. Hence II is implicit. qualities is hard or not. Hence II is not implicit.
Assumptions 51

33. 4; The speaker is comparing between the two types illness-causing element — is the culprit.
of business. So neither we can assume which 47. 5; I is implicit in the tone of warning. II is implicit
one is preferable nor whether saving of time in because when we st op people f r om doing
business is important or not. It is certain that something, we assume that they generally do so.
time can be saved, but whether it is important or 48. 5; The way the poor performance on excise and
not can’t be assumed from the statement. customs fronts has been shown to be the cause
34. 4; Neither the name of the department nor the post of the “huge revenue deficit” makes I implicit.
held by him can be assumed from the statement. Similarly, II is implicit from the way it affected
Hence II is not implicit. Also, nothing about his customs and excise.
present job status can be assumed. Hence I is not 49. 2; Assumption I is in fact a conclusion. Assumption
implicit. II is implicit from the word “disintermediate”.
35. 1; II may be a conclusion but it is not an assumption. 50. 1; I is implicit from the “difference” being pointed
I is implicit. That is why the speaker is saying out. II is beyond the scope of the statement.
so. 51. 2; Talks for second tranche is expected only after
36. 5; How did they know about the advertisement? talks for the first tranche have been finalised.
Hence it can be assumed that they read newspapers. But the actual amount may still be in the pipeline.
Why did they get puzzled over the publication of 52. 2; The sentence should not be taken too literally.
the advertisement in t he newspaper? All the 53. 5; Both the assumptions are implicit. This is why
advertisements can’t puzzle all persons of the same t he speaker believ es t hat consumer s ar e
profession. Hence it can be assumed that the “deceived”.
advertisement was a rare one. 54. 2; I is not implicit. Maybe, the “you” here refers only
37. 4; Neither can we assume about the CIA nor about t o t he elit e. But II is implicit ; t hat is why
the toughness of its work from the statement. “adrenaline kicks in” on opening the papers.

K
38. 5; Assumption I is implicit because otherwise it 55. 4; Both may be conclusions, but not assumptions.
would not hav e been pr escr ibed. As t he 56. 1; The VC is assuming I; t hat is why he has
Environmental Pollution Authority is concerned r est r ict ed wear ing west er n dr esses on t he
with auto-rickshaws, assumption II seems quite campus. We can’t relate their studies with their
plausible. dress and hence assumption II is not implicit.
39. 4; National interest could or could nto be paramount 57. 4; The assumptions are not implicit because it is
for other industrial concerns. So assumption I is possible that the minister has neither of these
not implicit . There is not even a mention of convictions but is acting under sheer political
env ir onment al r esour ces. We can t reat compulsion.
assumpt ion II as a possibilit y but it is not 58. 5; The move to make use of alcometers must have
necessarily so. been necessitated because of the large number of
40. 2; Gift scheme gave a spurt to paging industry but accidents. Hence I is implicit. II is implicit from
we cannot generalise it. So assumption I is not the fact that it will identify drunken drivers.
implicit because of the word always. II is implicit. 59. 2; The statement is merely a report while I is a

KUNDAN
That is why paging industries are now going value-judgement. Hene it is not implicit. II is
through tough times. implicit from “irregular water supply”.
41. 4; Seller’s promise doesn’t mean that the good will 60. 2; Film stars are public figures and popular so they
give satisfaction to buyers. Hence I is not implicit. can gather more crowd. And assuming this, the
Nothing can be assumed about unbranded goods political leader has invited them to pull the crowd.
from the statement. Hence II is not implicit. Hence II is implicit.
42. 1; The connection between “advertising hype” and 61. 1; When the High Court passes an order, it assumes
“one would imagine” makes I implicit. II is not I. II is not implicit because the court has merely
implicit: what the speaker assumes is that there assumed that the film is controversial. Whether
might be some change but not on the grand scale it is objectionable will be established only when
being talked about. the decision is finally taken.
43. 5; The speaker has used the name of Sachin and 62. 5; Companies like Indian Airlines regard conferring
Shahrukh because of t heir popularity among of such lar gesse on popular f igur es like Dr
young students. Replacing their posters clearly Amartya Sen as a standing publicity of its own.
hints that II is implicit. I is clearly an assumption. Hence both the assumptions are implicit.
44. 5; West Asia has a large stock of oil resources; that 63. 5; W hy t he need to use t he elect ronic v oting
is why t hey hav e monopoly in this sector . machine? Chief Elect ion Commissioner is
Similarly, the speaker is assuming that India has definitely assuming I. Any system won’t be
human resources (in the form of IT entrepreneurs) recommended if it is not convenient to use for
which will lead it to become no. 1 in this field or common people. Hence II is implicit.
contribute a large part to India’s economy. Hence 64. 2; The term only is objectionable in I. Hence I is not
both are implicit. implicit. Why has the President directed the
45. 2; Here “others’ cost” means others’ peace, others’ Election Commission to curtail the expenditure?
safety.... but certainly not others’ expenses. Hence Definitely, he is assuming II.
I is not implicit. II is implicit; that is why Delhi 65. 2; The speaker is assuming II; that is why he has
police has issued the statement. used the term irrespective while comparing the
46. 3; When we say that “reckless partying can lead to gold production with price hike (inflation). I is
health problems”, we assume that either heavy not implicit; the assumption is that price hike
drinking or dancing — we can’t think of any other affects gold production.
52 Practice Book on Analytical Reasoning

66. 4; I may be a conclusion but it is not an assumption. 82. 2; I is not implicit . The st at ement has no
Hence I is not implicit. Nor is he assuming about relationship to other games. II is implicit because
t he role of journalist s in t he societ y bef or e the speaker talks about the consequence only after
delivering his statement. Hence II is not implicit. assuming this.
67. 5; Whenever a warning of this sort is given, the 83. 2; It is hard to reduce the generalised version to a
following assumptions are implicit: I. There is particular field. Other possibilities may also be
something foul in the air; there can be no smoke considered, such as sanitation problem, ... Hence
without fire; and II. The warning will have a I is not implicit. II is implicit because it is this
positive effect in checking the problem. that makes the speaker take potshot at metros.
68. 1; I is implicit; it is to dispel this myth that the 84. 4; Both are sort of restatements.
statement has been issued in public interest. but 85. 2; From the tone of the statement it is clear that the
being inferior in quality is not the same as being speaker is not satisfied with the large (excess)
adulterated; hence II is not implicit. number of ministers in India and wants reduction
69. 5; The need to treat properly the elderly and the in this number. Hence II is an assumption. I is
disabled stems from both the assumptions. not an assumption.
70. 1; The speaker is assuming I; that is why he has 86. 5; Both assumptions I and II have nearly same
st at ed so. In his opinion as t he economy is meaning. Clearly, the speaker is assuming that
growing well, employment rate should increase. courts are not fulfilling the objective (provide
He is not assuming about any other factor that justice to deserving people) for which they were
influences employment. established. The rich can change the judgment
71. 4; From the statement it is clear that the speaker in their favour (throwing dice).
has no doubt about the credibility of inquiry 87. 2; Taking defeat seriously and taking lessons from
reports. Hence I is not implicit. II has too strong the defeat are two different matters. Hence I is
a word in never. not implicit. Why do we need to take lessons
72. 4; I is not implicit because it implies that value- from our defeat? The answer clearly is to be

K
based voting still prevails, though it does not have successful in future. Hence II is implicit.
too long a futur e. But t he statement clearly 88. 2; I is not implicit. It is possible that the situation
suggests that value-based voting is absent. What is improving instead of deteriorating, but t he
about Assumption II, which talks of the potential stat ement is being made because t his
of value-based voting? Well, the only potential improvement is not enough. On the other hand,
the speaker has in mind is that it’s a step in the II is implicit because it is this essentiality that
positive direction. But can it undo what has been makes the speaker talk about what the Govt
done? We don’t know. needs to do.
73. 5; Such messages ar e sent to get contr ol of a 89. 2; The statement may be given by a lawyer or any
problem. II is the implicit problem. I is expected other critic. So we can’t assume that this is given
to help in getting control of the problem. by an opposition leader. But t he speaker is
74. 1; The speaker is assuming that there should be assuming II. That is why he takes about people
certain qualities in the commander to command rejecting the changes.

KUNDAN
the society well. I is a genuine quality. Hence I 90. 5; I is implicit: this is what the speaker has in
is implicit. It is possible t hat the speaker is mind when he talks of having “util is ed t he
predicting on the basis of his knowledge and not employment potential.” The speaker is assuming
experience. Hence II is not implicit. II that is why he is relating the employment
75. 5; The speaker is assuming I; that is why he has potential of railway with the political career of
at tached equal impor t ance t o both — t he the leader.
constitution and adherence of the people to its 91. 5; The railway minister is assuming that previous
provisions. The speaker assumes II; that is why ministers have failed in providing adequate safety
he delves into its causes. to the passengers. That is why she will give more
76. 1; I is implicit ; t hat is why t he PIOs hav e attention to safety, hoping that her step will
mainatined t he Indian cultur e even in alien pr ev ent accident in f utur e. Hence bot h t he
sett ings. But II is not implicit because a assumptions are implicit.
comparaison between the two cultures is nowhere 92. 1; I is implicit here. That is why the speaker is
hinted at. talking about another way of living when the
77. 5; DTC aut horit ies are assuming bot h t he envir onment has been polluted. II can’t be
assumptions; that is why they have planned so. assumed from the statement.
78. 2; Even able parents may be not willing to pay the 93. 3; From the term harsh it is clear that the speaker
increased fee. Hence I can’t be assumed. Why is assuming that the police are not serving the
are they demonstrating in front of Legislative purpose for which they are there. The meaning
Council? Clearly, they are assuming II. Hence II of the two assumptions is different. Hence either
is implicit. I or II.
79. 5; The NHRC is assuming both; that is why this 94. 5; Why are these advertisements given by these
new system has been made compulsory. organisations? Clearly, they are assuming both I
80. 2; I is a restatement. But II is implicit because it is and II.
on this assumption that the comparison has been 95. 4; Both assumptions contradict the statements.
made. 96. 3; Either of the two is implicit. Maybe, the speaker
81. 5; Why did the court intervene and direct the state’s t hinks t hat irr espect iv e of how much you
executive machinery? advertise, you won’t attract tax-filers. The other
Assumptions 53

possibility is: why waste money when less of it Hence I is not implicit. In assumption II, the
can be as effective? term ‘civilized society’ twists his view.
97. 1; I is implicit: it does matter a lot; that is why it 117. 5; Both are implicit.
goes on to make winners. II does not fit here. 118. 2; I is not implicit. Note: So long as there is ....
98. 1; The coach is assuming I, otherwise he won’t means option is situational. He assumes that
stress on his men’s potential. II can’t be assumed: caste-based organisations are the best caretaker
the intention of player is a different matter. of caste-based society. Hence II is implicit.
99. 2; The speaker is assuming II; that is why he is 119. 1; I is implicit in the word penance — an act to
concerned about the failure of the relationship. wash away one’s sins. II is not implicit. From
100. 2; The speaker is assuming II that is why he has the statement, we don’t know whether the gods
referred to things related to education. Assumption showed Parasuram the way out.
I has not been hinted at in the statement. 120.2; Here “less space” does not imply physical space
101. 2; From the given statement it cannot be inferred so much. Instead, it implies less freedom.
whether transparency exists or not. Hence I is 121. 1; Note the exclamation mark(!). Which implies that
not implicit. II is a valid assumption; that is why t he phenomenon is sur pr ising. Hence I is
the speaker has jotted down new guidelines for implicit. II is a conclusion, not an assumption.
selection process. 122.5; Both are implicit in the affordability factor.
102. 5; The maulana is assuming both. He feels that 123. 2; We don’t know whether “people of modern times”
t hough pr ogr ammes ar e mor e popular than are opposed to what has “traditionally” been there.
prayers, they are less important. Hence the appeal. Hence I is not implicit. II is implicit because
“growth prospects” are as important as “stability”.
103. 2; The speaker is assuming II, that is why he has 124. 2; Note that the caption is one in a cartoon. So 763
put emphasis on long-term measures rather than is clearly an exaggeration.

K
short-term ones. Nothing about the election- 125. 3; The message from the hostage is an attempt to
winning interests has been hinted at here. keep his well-wishers normal. If not calmed, they
104. 2; I may have a broad range. Hence it is not implicit. may react in either of the two ways.
II can be assumed from the statement. 126. 2; I is not implicit because the speaker does not
105. 1; Nehru is assuming I; that is why he is against have any particular aspect in mind. II is implicit,
artificial policies. II is not implicit. hence the editing with the visual aspect in mind.
106. 5; I is implicit; that is why the need for overhaul of 127. 2; The statement is not one about staging of plays
the system. II is also implicit in the phrase “the in general. Hence I is not implicit. It is about
present globalised scenario”. whether the depiction of a religious figure on
107. 2; II is definitely an assumption. But nothing about stage appeals to the audience. Hence II is implicit.
the policies of different companies can be found 128. 2; I is not implicit. The assumption is not that fire
out from the statement. cannot occur but that it is less likely in Narela or
108. 2; I is not implicit. We don’t know about success of Ghazipur. II is implicit; that is why, of all the items,
businesses in general. II is implicit; hence the hazardous chemicals and paper are being moved out.

KUNDAN
appeal for Internet adoption. 129. 2; The objective behind the notice is t o attr act
109. 1; The way ‘business’ is used in the given statement customers.
implies assumption I. Whether the rallies will 130. 1; Only I is implicit. That is why X opines to buy
get the support of the people on such issues or Mercedes cars if one is desirous of buying a car.
not, is not in the mind of the speaker. He is only 131. 5; Filing a writ in court is aimed at seeking legal
against such demonstrations. Hence II is not action against the concerned state. Hence, both I
implicit. and II are implicit.
110. 5; The announcement of awards serves both the 132. 5; The features highlighted by the advertisement
purposes. are based on assumptions I and II.
111. 4; Her e the assumpt ion is: The met hod of 133. 5; Why are some excesses bound to happen? The
governance of a country should be according to speaker must be assuming II. Again, to be satisfied
the ground realities of that country. Hence I is with the success of SOG implies that the speaker
not implicit. II is not implicit because we don’t must be assuming I also.
know whether the speaker has other countries 134. 2; Assumption I goes very deep. Hence, it is not
in mind. implicit. But, assumption II is implicit. That is
112. 2; II is implicit by the very need of the singer to why the notice stresses on buying the leaves
defend ghazal. I is not implicit because the singer instead of burning it.
does not compare ghazal to pop. 135. 1; Assumpt ion I is implicit . That is why t he
113. 2; Eastern Railway may be initiator of this practice. advertisement highlights “And if you’re looking
Hence I is not implicit. Assuming I, the Eastern for a change”.
Railways officials have advertised so. 136. 4; I is not implicit because it assumes too many
114. 5; Because he is assuming I that is why he has things: education and free movement are beyond
used the term ‘enforceable’ in his stat ment. the scope of the statement. II is not implicit
Speaker is assuming II; that is why he stressed because the PM only assumes that law and order
that law should be obeyed by all without exception. affects the common man more than prices do.
115. 5; Both are implicit. The term ‘dead’ clears speaker’s 137. 5; Both I and II are implicit. The speaker does not
intention. consider ter r or ists as human. He must be
116. 4; The newspaper’s view on the expensiveness of assuming II. Hence, II is implicit. Why is the
justice can’t be guessed from the given statement. speaker not in favour of using human rights to
54 Practice Book on Analytical Reasoning

pr ot est f or t hose found guilt y? He must be quality control regime. Hence, the companies’
assuming I also. authorities must be assuming I. Assumption II is
138. 1; The t one of t he st at ement implies that t he also implicit in the last part of the statement.
speaker must be assuming I. That is why he 153. 1; I is implicit. Otherwise how can the number of
uses the word ‘significant’ for the analysis done. heart patients in all countries of the world be
The wor d ‘alt hough’ used in t he stat ement compared to one another? II is not implicit because
implies that the speaker makes the assumption the statement does not talk about population.
that lesser the size of sample, lesser the chances 154. 5; Both I and II are implicit. I is implicit. That is
of reliability. why the speaker points towards the lackadaisical
139. 3; It is not clear from the statement what the CM is approach of the government in combating AIDS.
exactly assuming. But the tone of the statement The word ‘sure killer’ implies the speaker must
implies he must be assuming either I or II. be assuming II also.
140. 1; Why the need to instil such a sense of pride 155. 4; I is not implicit because ‘spate of defeats’ can’t be
among the people? The speaker must be assuming correlated with the statement. On a similar basis
I. II is not implicit because one does not desire II also can’t be correlated with the statement.
for a need unless one assumes it to be feasible. 156. 4; The request of the city police implies that the
141. 2; Note that if one advises others to purchase a thing cit y police assume t hat “some people hav e
X, the former assumes that the latter have enough grievances”. Assumption I is not implicit due to
money to purchase it. the words “All categories”. II is also not implicit
142. 5; Both I and II are implicit. Why did the city traffic because the statement does not say anything about
police need to issue such a notice? It must have flying squad of vigilance branch.
assumed II. Again the word ‘always’ used in the 157. 2; Non-biodegradable nature of plastic bag can’t be
notice makes I implicit. cor r elat ed with t he stat ement because t he
143. 4; Both I and II can’t be cor related wit h t he stat ement does not say whet her plast ic is
statement. Hence, neither I nor II is implicit. biodegradable or nonbiodegradable. Hence, I is

K
144. 5; I is an assumption because the statement uses not implicit. But II is obviously implicit. That is
the gadgets as an allurement. II is obvious. Note why the scientist uses the word ‘Beware’ in his
that the initiative is aimed at improving the statement before asserting the negative features
financial condition of the company by reducing of plastic bags.
theft. Hence, the company must be assuming II. 158. 1; Only I is implicit. That is why Mr X advocates
145. 1; I is implicit. The “badly shaken lives” will be for the need for a consensus to trim government
headed through development. “That things would expenditure.
not go haywire” can be assured through good 159. 2; I is not implicit because of its last portion, ie “all
governance. II is not implicit: we don’t know over the world”. II is implicit since betting is on
whet her t he image of t he st at e has been in full swing.
tarnished. 160. 4; Both I and II are implications, not assumptions.
146. 5; How can an illiterate person write the name and I and II are not that on which the statement is
belt number of erring policeman? Hence, the based but something that is implied by the

KUNDAN
speaker must be assuming I. Again, t he statement.
instruction to send a complaint to senior officials Consider this statement: “I heard a shocking news
implies that the speaker must be assuming II also. and felt shattered.” Then t he assumption is:
147. 5; I is implicit ; that is why the adv er t isement “Shocking news shatters a person.”
emphasises the popularity of product abroad as 161. 1; I is implicit: an atheist would not invoke god for
well as in domestic market. The natural property blessings. II is not an assumption but a re-
focussed on in the ad is conducive to facilitating statement: “passed away in the Yamuna waters”
sound sleep. Hence, II is also implicit. itself means “died by drowning in the Yamuna.”
148. 2; I may or may not be an assumption. But II is 162. 1; I is implicit: the proposal that drivers employed
obviously implicit. That is why the manufacturers should not be illiterate has to be based on this
need to oil the wheels that India moves on. assumpt ion. II is not implicit. In f act , the
149. 1; Only I is implicit. Why does the company need assumption is just the contrary. The use of “at
to issue such instruction? It must be assuming I. least for children” assumes that children are or
The motive behind the instruction may be to should be privileged ones.
ensur e conv enience in keeping t he r ecor ds 163. 5; I is implicit because of the cause-ef fect
regarding transaction of the company. Hence, II relationship. II is also implicit because a thing
is not implicit. cannot happen unless it is possible.
150. 2; I may or may not be an assumption. But II is 164. 2; I is not an assumption. It is an inference being
obvious. That is why the CM appeals for people’s drawn out of the “strange thing.” But assumption
participation. II is implicit. It is this departure from the usual
151. 1; W hy does the agr icultur ist f ocus on ‘bet ter that makes “last night” strange.
under standing’, ‘co-oper at ion’ and 165. 5; Why does the person gives an advice to provide
‘complementing’ among the exporters? He must vocational training to women in work? There must
be assuming I. Assumption II can’t be correlated be a positive and constructive role of the training.
with the statement. Hence, II is not implicit. Hence I is implicit. II is also implicit. Otherwise
152. 5; The ground on which the authorities of the bottled the training will be useless.
water companies ruled out any contamination in 166. 5; The view of the speaker is in the form of a critic.
their product is its production under rigorous He feels against the discrimination in punishment
Assumptions 55

given to the swindlers and petty thieves. Hence, at delving into the reasons of the mysterious
he must be assuming I and II also. death and also at punishing the guilty. Order of
167. 5; The purpose to get help from the instruction to magisterial inquiry implies that the government
ensure security can’t be fulf illed unless t he must be assuming I. Why has the entire staff of
instruction is read by the passengers. Hence, both police station X been transferred? The government
I and II are implicit. must be assuming II also.
168. 1; Name of the given service cadres implies that 183. 4; The st at ement does not ment ion anyt hing
these cadres may help in upliftment of the living r egar ding r epayment st ruct ur e or collat eral
condition of rural people. Now, the government security. Hence, neither I nor II is implicit.
proposition implies that the government must be 184. 4; The timeframe given by the government implies
assuming I. that the government must be assuming that the
169. 5; I is implicit because it make no sense to talk of time given is adequate for operators to declare
something without the existence of its possibility. pay channel rates. Hence, assumption I is not
II is implicit; that is why the speaker suggests implicit. Assumption II is not implicit because it
tools to make change in the women’s status. is beyond the scope of the statement.
170. 4; We cannot say what ground the critic assumes 185. 2; Global computer piracy is not mentioned in the
on the basis of which be terms it “not a proper statement. Therefore, assumption I is not implicit.
and judicious step”. But assumption II is implicit. That is why the
171. 4; I is not valid because we don’t know whether the journalist uses the word ‘despite’ in his statement.
contrast expressed in the view is critical or merely 186. 1; Why has such an advertisement been published
a statement of fact. II, if at all an assumption, by the advertiser? Definitely, the advertiser is
can only be false. assuming I. Assumption II is not implicit because
172. 2; I is not implicit because it goes into unnecessary it goes into details.

K
details. II is implicit; that is why the speaker 187. 2; Assumption I may not be implicit. It is beyond
str esses on t he r ole of executiv e in the the scope of the statement due to lack of any
appointment of High Court Judges. specific clue. But assumption II seems quite
173. 5; The instruction to customers is aimed at ensuring plausible. That is why people’s participation has
saf ety and r est r icing unaut hor ised person’s been sought.
accessibility to any individual’s account. Hence, 188. 5; Assumpt ion I is implicit . That is why t he
t he speaker must be assuming that an milit ant s kidnapped t he local r esident .
unauthorised person can access others’ account. Assumption II is also implicit. Otherwise the
174. 5; Both I and II are implicit. Why is the rate of the militants would not have faith in the resident.
cup of tea reduced to Rs 2 for labourers? I must 189. 2; The tone of the statement implies that the speaker
have beeen assumed. Again, the real motto of an assumes training session is necessary for the
ad is to increase the number of customers as well team. Hence, assumption II is implicit. But the
as sales. Hence, II must also be assumed. statement does not indicate what type of training
175. 1; Only I is implicit. II is not implicit because of the session it should be. Hence, assumption I is not

KUNDAN
word ‘radical’. implicit.
176. 2; I goes into details. Hence, I is not implicit. But II 190. 1; Since the given statement has been made by the
is implicit; that is why the opposition leader water man while educat ing people on t he
issues such a statement. communit y’s r ight , t he wat erman must be
177. 2; I is irrelevant. Hence I is not implicit. But II is assuming indulgence of MNCs in water business
implicit; that is why the speaker suggests the as disastrous for the people. Note that the speaker
stringent punishment. is not averse to indulgence of MNCs in road
178. 1; Here II is irrelevant. Hence, II is not implicit. construction or some other areas. This implies
But I must have been assumed by the politician; water business has some special importance.
ot her wise how can the st at us of indust rial Hence assumption I is implicit. Assumption II is
progress be related with unemployment? not implicit because the indulgence of MNCs does
179. 1; I is implicit because this is the motive behind not necessarily imply that they are going to buy
increasing the fine. II is not implicit because of the rivers.
the second part of the statement. 191. 2; Assumption I goes into details, therefore, it is
180. 2; The adver tisement is based on the following not implicit. Again the initiative to be taken is
assumptions: aimed at countering the problem. Hence, II must
* Lear ner s want t o solv e more and mor e be implicit.
questions in less time. 192. 5; The request made by the city police to common
* Learners want to solve DI without written people implies that the city police assumes that
steps. the verification of domestic help is necessary to
* It is possible to solve Quantitative Aptitude in put a tab on increasing rate of crime. Hence, I is
3 seconds. implicit. Assumption II is also implicit. If it is a
* Penalty in case of failure of claim will make a “simple step”, the process involved must be simple.
positive impact on readers of the ad; etc. 193. 1; The tone of the statement implies that the speaker
Hence, I is not implicit. II is obvious. That is must have assumed I. That is why she uses the
why the advertisement goes like this. words “let me die peacefuly.” But the statement
181. 4; We have nothing substantial to correlate the given gives no indication about the comparison to other
assumptions. Neither I nor II is implicit. ways of suicide. Hence, assumption II is not
182. 5; The initiative taken by the government is aimed implicit.
56 Practice Book on Analytical Reasoning

194. 2; Assumption I is not implicit because it rather professional photographers’ contribution. Hence
contradicts the statement. But assumption II is II is not an assumption.
implicit; that is why the word “despite” has been 208.1; Why retain a photograph document? So that it
used in the statement. comes handy in fighting terror. In other words,
195. 4; Assumption I is not implicit because introducing who knows the buyer may be a terrorist or may
polythene packets may have nothing to do with serve as a means of terrorism. Hence I is implicit.
writing on the notes. Assumption II is not implicit II is beside the statement.
because of the word ‘only’. 209.5; If there were no demands for resignation, why
196. 2; I may or may not be an assumpt ion. But would the politician say, “I will not resign”? Hence
assumption II is implicit ; t hat is why I is implicit. Again, he talks of being proved guilty.
communication is termed as having a vital role A proof of guilt comes only after the levelling of
in quick decision-making. charges. Hence II is implicit.
197. 2; The tone of the statement implies that the person 210.1; I is implicit because advertisement of a particular
assumes that the historic house is more precious brand is usually placed in order to compete with
than his life. But assumption I includes all human other available brands. II is not implicit because
lives. Hence, I is not implicit. Again, caring for price and quality are not inextricably related.
the house, putting lives to risk, implies that the 211. 2; I is a bizarre assumption. The statement does not
speaker must be assuming II. assume anything about what men or women
198. 2; Nothing has been ment ioned about other should (not) do. II is implicit because to become
count ries. Hence, I is not implicit . But the embodiment of something, much has to be
assumption II is implicit. That is why the ministry proved first.
focusses on intercultural dialogue and interaction 212.2; I is not an assumption but an inference. II is
with civilisations, countries and nations. implicit because unless one is destructive, one
199. 5; The decision of t he Cabinet t o set up new cannot destroy something.
authorities implies that the cabinet must be 213. 1; I is implicit because the power to reject something

K
assuming I. Hence, I is implicit. II is also implicit; is usually possible only when one has the power
that is why the cabinet displaced the existing to grant it.
authority with a new one. II has no connect ion what soev er wit h the
200. 1; I is implicit; that is why in the speaker’s view statement.
the matter is regrettable. II is not implicit. Note 214. 3; If A needs to be substituted by B, we assume that
that the statement says about media, not the radio B is mor e appropriate t han A. Now, t he
or print media especially. appropriateness of sledgehammer may be due to
201. 4; The decision of the SC implies that the SC must either of the following reasons: (a) a sledgehammer
be assuming the strike by the employee to be is more powerful; (b) though not more powerful,
detrimental for the society and at the same time it works because different tools are required in
it must be dissatisfied with the way of strike. different conditions. (b) may be explained by the
But the word ‘ransom’ can’t be correlated with following example: A chalk cannot be said to be
the statement. Hence, I is not implicit. The same more powerful than a pen. But when it comes to

KUNDAN
is t r ue f or t he wor ds ‘chaos’ and ‘t otal writing on a blackboard, you would use a chalk,
maladministration’. not a pen.
202. 5; Why did the city police need to issue such notice? 215. 4; I is not implicit: since the statement is silent on
It must be assuming that people lack awareness whether the move would have a good or a bad
of the ill-effects of touching unclaimed objects. impact, it does not make any assumption about
Hence, both I and II are implicit. the move’s desirability. II is also not implicit. In
203.5; I is implicit; that is why the speaker mentions fact, the assumption is on the contrary. If we say
the desired skills in addition to proper training. that this move has been taken from a particular
Again, why does an entr ant need a pr oper point of view (that of exchange rate management),
t raining? Obv iously, the speaker must be we assume that other moves are also possible.
assuming II also. 216. 5; I is implicit. Look at how their name is taken in
204.5; Why did the DHS need to issue the notice? How t he same br eat h along with t he mention of
can these initiatives save people from the fatal Congress. When a leader draws attention of the
diseases? Obviously I is the assumption behind voters to a problem, he assumes that the problem
it. Again, how can unused/broken articles lying seriously concerns them.
on the roof can be mosquito-friendly? Obviously, 217. 1; I is implicit : India is being asked to learn. Which
II is the assumption behind it. assumes India may also be threatened by a similar
205.5; Why was the need felt to make the change? There problem. Note carefully that II is not implicit.
was certainly something dissatisfactory. Hence I What is assumed is that being forced to take loan
is the assumption behind it. II is obvious. from the IMF is not a good sign. If one simply
206.2; I is rejected because nothing has been told about takes loan, that is, voluntarily so, that may be
technical expertise. The advertisement must have considered a healthy growth.
been targeted at the people who want t o be 218. 2; I is not an assumption but an inference, and that
successf ul photogr apher . Hence II is an too a wrong one. II is implicit : why else would a
assumption. political leader ask for revelations if it were not
207.1; The person A is certainly assuming that to get for some gain?
start ed one needs some teaching. Hence I is 219. 1; I is implicit because of the word representative.
clearly an assumption. But nothing is said about W hen we t alk of X being a leader or a
Assumptions 57

representative, we assume that X leads a group of not make effort to reach an impossible target. The
persons with similar bent of mind. In other wor ding ‘Since my childhood’ makes II also
words, he is not the only one of that kind. II is implicit.
not implicit. 235. 3; The ad assumes that there are two options: either
220. 5; I is implicit. If X seems to have happened, it is lear n f r om a good book or join a coaching
usually assumed that X happens. Again, II is also institute. However, the former option is assumed
implicit: since the speaker compares Sahara and to be preferable.
Peerless to other NBFCs, he assumes that these 236. 2; Gener ally any condit ion is giv en in an
two are also NBFCs. adv er tisement (especially in the for m of
221. 5; Assuming II only, the leader of the opposition instruction) with an assumption that people have
has made such statement. I is also implicit because the ability to follow it. Hence I is not implicit. II
the root cause of disruption in the house is lack is obvious.
of consensus. 237. 5; I is implicit because the focus of the statement is
222. 1; I is implicit but II is not. Happening of new things on a woman at higher post. II is implicit from the
might be good or bad in nature. wording ‘too common to make news’.
223. 5; Whenever a plan is aimed at for a particular 238. 1; You don’t advise someone unless he needs it.
locality, the planner assumes that the locality Hence I is implicit. But II is not because of the
needs that plan. Hence I is implicit. II is also word ‘only’.
implicit. If an institution makes a plan to provide 239. 1; Whenever a notice is issued it is assumed that
facilities to the people, clearly those facilities are there will be some impact of this notice on the
missing and are required. people. Hence II is not implicit. But I is obvious.
224. 2; I is not implicit. II is implicit. Because if any 240.5; X giv es an example and compar es in such a
country warns another country not to interfere manner that he assumes a situation similar to

K
with the happening in its country, then it must t he exist ing problem; ther ef or e he must be
have been assumed that the happening in its assuming I. II is also implicit because it is the
own country is its internal affair. motive behind the statement of X.
225. 5; The statement is made by assuming II. That is 241. 5; The word ‘ratio’ used by A makes I implicit. Need
why such measures have been aimed at. The talk of prediction of the ratio of questions of verbal to
of the requirement of the combination makes I those of non-verbal reasoning makes II implicit.
implicit. 242.1; Authorising the agents to distribute coupons
226. 2; Assumption I is not implicit because it goes into makes I implicit. But II can’t be correlated with
details which cannot be assumed. II is implicit the statement.
because when you announce a policy, you 243. 2; I is not implicit due to the word ‘only’. II is obvious
generally assume that it will find favour with because this is the assumption on which the
the target group. criticism made by the leader of Y is based.
227. 4; External support makes things easier. But we 244. 1; I is implicit. That is why the speaker is desirous
cannot assume that the success of a programme of showing the courts the right path. II is not

KUNDAN
hinges only on this external support. Hence I is implicit. The judiciary should be ideal but there
not implicit. II is not implicit because the objective is no clue why.
wouldnot be set if the capability did not exist in 245. 2; I is not implicit because of the word ‘necessary’.
the first place. II is obviously implicit.
228. 5; Both I and II are implicit because the purpose of 246. 1; Confinement of Ram temple movement in the
the move is to pressurise country Y as well as to northern part of India indicates that the speaker
attract global attention. must be assuming I. II is not implicit. Mr Y does
229. 2; Assumption I is not implicit because though the assume the desirability of true Indian civilisation.
perceived threat of competition may be one of the But is it for “harmony”? We don’t.
reasons behind this move, we can’t say so with 247. 2; I is not implicit. But II is implicit because seeking
certainty. But II is implicit: shrewd warriors often help from the PM of the country Y implies that
lose the battle to win the war. the leader of country X has some hope from the
230. 5; I is implicit because this is why Keshav’s mother PM of the country Y.
needed to instruct him. II is also implicit because 248. 5; People looks for goods which are cheap as well
a choice between two options can be made only as durable. Hence I is implicit. Since the scheme
when both the options exist. has been initiated with the aim of attracting more
231. 1; War between two countries makes the life of the cust omer s and fur t her str engt hening t he
people miserable. Selling of crops at low prices company, II is also implicit.
is on the basis of assumption I. II is not implicit 249. 5; Advocating prompt diagnosis makes II implicit.
because it is far-fetched. Again, making effort in the direction of mitigating
232. 1; I is implicit from the phrase “unable to solve in the ill-effects of a biological attack indicates that
quicker way”. The wor d ‘only’ makes t he the speaker must be assuming I.
assumption II non-implicit. 250.2; Assumption II is implicit. That is why the minister
233. 2; I is not implicit because t he commander is stresses on “WTO compatible”. I is irrelevant.
considering about the equipment required to curb 251. 5; Both I and II are implicit. Efforts made by the
militancy. He must be assuming II. Consideration company is aimed at the problem related with
over equipment is related with the efficiency of corrosion. Hence the MD must be assuming I. II
forces. is also implicit on the same ground.
234. 5; Efforts of Y obviously makes I implicit. One does 252.5; Purpose of advertisement is aimed at making more
58 Practice Book on Analytical Reasoning

people have access to internet. Hence I is implicit. 267. 4; I is not implicit because we don’t know what X
Advocating f or best action on the int ernet assumes about other factors. II is not implicit
indicates that the speaker must be assuming II. either. We don’t know why X assumes him to be
253. 1; I is implicit ; t hat is why t he company has lauded.
advertised thus. II is not implicit: the ad does 268. 4; I is not implicit because we can’t say with certainty
not claim it to be the only way to learn English. that Miss ‘X’ assumes her life to be full of prosperity
254. 2; Low pr ice points hav e been accept ed as and the prosperity in her life came only after the
instruments for penetration in rural areas. Hence death of her father. II may or may not be the true
II must have been assumed. I is obviously not reason behind the reluctance of her father. Hence,
implicit . Because if a company decides t o II is also not implicit.
penetrate in rural areas by providing the drinks 269. 5; I is implicit ; that is why the adv er t isement
at low price points then the company must be advocates to satisfy one’s need. II is also implicit
assuming that people in rural areas will be able because if study materials of ‘X’ coaching institute
to purchase the cold drinks at reduced price. are capable of satisfying one’s need for PO, it
255. 5; Talking of break-dances as a substitute for exercise must be capable of satisfying the real requirements
makes I implicit . At the same t ime when a of a true aspirant for PO. Hence, the speaker
substitute is talked about it must have been must be assuming II.
assumed that it will be suitable for the existing 270. 2; I may or may not be a true assumption. Hence I is
situation. Hence II is also implicit. not implicit. II is obviously implicit; otherwise
256. 4; I is not implicit. In fact, the statement assumes there would not have been the advertisement.
that it’s no use saying that an agreement has 271. 5; Both I and II are implicit. If someone gives a
been reached as long as the det ails are not ultimate success formula to youngster, then he/
divulged. II is distorting the statement too much. she assumes I. Second sentence of the statement
257. 4; The statement is based on the hindrances and makes II implicit.
ill-eff ect s of delay in proper act ion. Neit her 272. 4; Hence, I is not implicit because of only. II is also

K
assumption carries the above sense. Hence both not implicit because if PM is talking about joint
are not implicit. patrolling then PM must be assuming that other
258. 1; Assumption I holds good because otherwise the party will also support the plan.
said institutions could not have fulfilled the 273. 2; Suggestion made in the note is aimed at
r espectiv e demands of ot her count r ies. adopting a way t hat brings out committ ed
Assumption II is not implicit: just because you aspirants for political leadership. This implies
lend someone a part icular help (excellent in political leadership is facing a crisis of t rue
nature) does not imply your overall superiority. leadership. But can we assume t hat it is so
259. 5; When you prepare yourself for an event, you everywhere? No. Hence, I is not implicit. If one
assume your participation in it. Bidding adieu to suggests something to someone, he assumes
Test cricket and focussing on the forthcoming his/her suggestion is pract ically possible.
World Cup makes asumption II implicit. Hence, II is implicit.
260.1; I is implicit in the words “it becomes necessary”. 274. 5; One does not give instruction to any one unless

KUNDAN
II is not implicit because of the word “every”. he/she assumes the latter needs it. This implies
261. 5; Comparison made by the person between events that the readers need the instruction. This makes
I and II indicates that the speaker must be I implicit. Again, if one suggests someone an
assuming both I and II. initiative, the former assumes the initiative is likely
262.2; I is not implicit because of the word ‘ev ery’. to solve the respective problem of the latter. Hence
Narcotics Control Bureau assumes that some of II is also implicit.
the people must have information; but it does not 275. 1; Why such notice? CEO must be assuming I.
assume that all the people have informat ion 276. 5; The advertisement t alks about renov ation in
regarding drug trafficking. II is obviously implicit. schemes and wide range of solutions. Why such
That is why instruction is being given. ads? It is likely t hat these t wo qualities of
263. 5; The request comprises a fixed time. If one fixes a schemes will attract customers. Hence both I and
deadline for a particular thing, one must be II are implicit.
assuming I. II is also implicit because if a person 277. 2; I is not an assumption. Deputing a high level team
requests for something from another, the former for a particular objective implies that the EC is
assumes that the latter will abide by it. assuming the team will succeed in its objective and
264. 5; I is implicit. Look at how Mr Y is blaming “faulty for this people of the state will co-operate with the
lifestyles” and “bad eating habits” — they are team.
inst ances of brazen v iolat ion of pr ev entiv e 278. 5; The statement is in the form of a suggestion and
measur es. II is implicit , that is why Mr Y it is aimed at stopping child marriages. Note that
expresses his resentment. one does not suggest any initiative to remove a
265. 4; The statement assumes nothing about the kind problem unless he assumes the problem is still
of people who are fit to deliver the goods. Hence in existence. Hence, I is implicit. Now, why a
I is not implicit. Again, the statement says there sever e punishment ? The speaker must be
is a need to assuage the feelings. But it does not assuming. II. Hence, II is also implicit.
assume whet her doing so would be easy or 279. 5; The scheme will attract students, particularly
difficult. Hence II is not implicit. those students who have no money to pursue
266. 1; Only I is implicit because of the words ‘even fifty education. So both assumptions are implicit.
years after’. II is not implicit. 280. 2; Tone of the statement makes II implicit.
Assumptions 59

281. 4; The student doesn’t necessarily assume the basis told so. While making the statement, his prime
of this desire. concern was corruption and not political stability.
282. 4; Neit her I nor II is implicit . Only intelligent Hence II is not implicit.
competitors are not the basis of the statement of 296. 2; If ‘I’ were the Election Commissoner’s assumption,
the co-ordinator. Hence I is not implicit. II may he would not have talked of “those without a voter
or may not be assumption. Hence, II is also not identity card”. So II is a valid assumption.
implicit. 297. 2; I is a r estat ement . Hence does not f ollow.
283. 2; I goes deep into r eason. Hence I is not an Disinvestment in banks means moving towards
assumption. II is obvious; that is why the person privatisation. Clearly, the speaker is assuming
suggests his/her view. II; that is why he talks about elimination of scams.
284. 2; I may or may not be an assumption. Advertisement 298. 2; I is not implicit. On the contrary, the restriction
of a product is done keeping in mind that there of this motility is assumed to be a problem. II is
must be some buyers. Hence, I is not implicit due implicit ; that is why we t alk of “regional
to the word ‘Most’. But since the objective of the language”.
advertisement is to attract customers by giving 299. 4; It is possible that because of the difficulty of
them a chance of free trip to Asian Games, the language learning courses, students quit their
speaker must be assuming II. Otherwise, why studies in the middle. Assuming this reason, he
will people prefer a free trip to a foreign country? may have stated so.
285. 5; Why an assurance of ensuring confidentiality 300.1; There may be other reasons for bureaucratic
along with an easy access to register a complaint? delays, e.g. long enquiry process by different
The speaker must be assuming I. Again, why departments on different angles. Hence II is not
specially t rained Women Police Of ficers? The implicit. I is implicit; that is why he has told so.
speaker must be assuming II also. 301. 4; He may be assuming that manufacturing sector

K
286. 5; The tone of the viewer implies the entire hue and information technology industry should be
and cry over the film is not genuine. Since the r un t oget her wit h equal str ess t o boost the
viewer is convinced that the duo (the director economy.
and the actress) is together, it implies that he is 302.1; It is clear that the economist assumes that at
assuming I and II. present the time and labour poor does not have
287. 1; While making the statement assumption I was much economic value. Hence I is implicit. Now,
in the mind of the speaker; that is why he is why does he advocate for the measure? This is
against the r eligion-based part ies in Indian not very clear. Maybe he also has the economy of
polit ics. II is not an assumption because the country in mind. But not necessarily. He may
secularism may be practised even in a country be more int er est ed in t he poor than in the
where there is only one religion. country.
288. 1; The speaker is assuming I; that is why in spite 303. 1; I is implicit. It is possible that his statement is
of the courageous deeds of robbers, pirates, .... general and his intention is only to explain the
and other anti-social elements, t hey are not impact of players on the youngsters.

KUNDAN
heroes. Courageous work can be undertaken in 304. 4; The speaker is more concerned about poverty and
both violent and non-violent ways. Hence II is wants that the impact of globalisation reach to
not implicit. the poor to eliminate their poverty. He is thinking
289. 2; Judiciary also cares but it is not the only wing little about the booming of the economy.
which cares. Hence I is not implicit. Clearly, the 305. 5; From the tone of Kher ’s r emar ks bot h the
speaker is assuming t hat judiciar y must be assumptions can be considered.
independent so that it can do its best. Hence II 306. 4; Assumption I is not exhaustive. Hence it is not
is implicit. implicit. The speaker has nothing to do with other
290. 4; I is more of a restatement. If there were no linkage countries. He is talking about his country only.
between corporate and political or economic Hence II is not implicit.
conditions of the country, the speaker would not 307. 2; The speaker is assuming that history and social
use despite in his statement. Hence II is not concerns are two different subjects and both have
implicit. different ways of writings. Hence II is implicit.
291. 5; Why does the speaker want that people take But I is not implicit.
action in the matter? Clearly, he is assuming II. 308. 4; It is clear that Nehru was the best leader to
I is also implicit in “Democracy will suffer”. ensure harmony. This implies that there were
292. 2; The speaker is assuming II; that is why he is not other leaders also to ensure harmony though
happy wit h t he qualit y of per for mance of Nehru was the best. Hence I is not implicit. II
ministers. Hence II is implicit. I is an extreme can be a conclusion.
case. It is possible that most of the ministers are 309. 5; From the tone of the statement it is clear that the
inefficient but not all. Hence I is not implicit. speaker is assuming both.
293. 2; II is implicit. The term “ambitious” confirms it. It 310. 5; According to the speaker, it is because of I the
is possible that they have some mass base and public sector employees do not want the transition
they use the grievances to take lead over their of their sector into private sector. Hence I is
rivals. Hence I is not implicit. implicit . II is implicit because corr uption
294. 4; Here both may be conclusions but certainly are decreases efficiency.
not assumptions. The speaker is a supporter of 311. 5; Why is the speaker against diluting the powers
pluralistic society. of CVC? Clearly he is assuming II. The speaker
295. 1; The speaker is assuming I; that is why he has is assuming I; that is why he is pleading for the
60 Practice Book on Analytical Reasoning

media to do its work. opposing only for the sake of opposing. Which
312. 5; If competition benefits the consumer, monopoly means the measure he is advocating, namely
must be harmful to him. Hence I is implicit. II is privatisation, is good. Hence II is implicit. And
implicit because these are the ways in which a once II is assumed, I also becomes implicit.
consumer benefits from competition. 329. 2; Higher marks can also be scored by using unfair
313. 2; If there is a huge loss of power, it is being assumed means. Hence I is not implicit. II is implicit.
that a lot of power is being generated. But one 330. 1; I is implicit; hence the warning. II is not implicit
can’t assume whet her t he generat ion or it s because we can’t assume t hat all social
capacity is adequate. Hence I is not implicit. II is responsibility jobs give money.
assumed from the element of suspicion present 331. 1; The speaker is assuming I. And its solution he
in the statement. has advised in his statement. II is also implicit.
314. 4; I is a restatement. Hence it is not implicit. Good Hence the need to seek fortune abroad.
looks are not the same as being conventionally 332. 4; I and II may or may not be an assumption. It is
good-looking. Confidence brings good looks even possible that he is assuming that an actor is more
to the conventionally not-so-good-looking. Since popular among the audience than directors and
good looks ar e being r uled out only in t he producers and that is why he looks out for acting
conventional sense, II is not implicit. assignments.
315. 4; Influence of one economy on the other does not 333. 1; I is implicit; that is why the speaker talks about
imply t hat t he latt er has no independent the flaw in the “democratic process”. II is not
existence. Hence I is not implicit. Just because implicit in the given theoretical statement.
the US economy influences the Indian economy, 334. 2; I is not implicit because the statement has nothing
one can’t conclude that II is being implied. The to do with “global” phenomenon. II is implicit;
influence can be felt even if India’s bilateral trade that is why job opportunities have been crippled.
with the US is of a large volume. 335. 1; I is a valid assumption. This is the way to win
316. 5; The speaker assumes that the very notion of value the hearts of the people. II is not implicit. The

K
system has changed. statement points to the fact that winners of votes
317. 2; The speaker is assuming II that is why he is often pay little attention to hearts.
talking about the entrenched customs and social 336. 2; How do we know whether the select ion is
attitudes towards women and about their plight provisional? Hence I is not implcit. II is implicit:
getting compounded. I is not an assumption here. that is why it has selected 20 more candidates.
318. 5; I is implicit by the link established by the speaker 337. 1; The blame is being squarely laid on the batsmen.
between “physics” and “myst er ies of t he 338. 5; I is implicit from the speaker’s demerit being
universe”. II is also implicit because 20 years is pointed out. II is also implicit. That is why “the
not too big a period. anti-defection law is ineffective”.
319. 1; I is implicit fr om t he t hr ust on human 339. 2; The former prime minister is against the way
development through educational institutions. II liberalisation policy is being pursued by the govt
is not implicit: we don’t know whether t he and not against the idea of liberalisation itself.
investment is needed to open new institutions 340. 4; The ad does not promot e chocolate eating in

KUNDAN
or merely maintain the old ones. general; it only says that if one eats chocolates,
320.1; Assuming I, it is an Indian language. II is not why not eat the tastier one? So I is not implicit.
the assumption here because it is possible that II is of course contrary to what the ad says.
other communities have this bias. 341. 4; The soul accepts new bodies just as a person
321. 5; You can arrive at the statement only when you accepts new clothes - this being equated to the
assume I. Hence I is implicit. He is assuming II soul being a clot hshanger is v ery crude
and he wants that this habit should be given up. philosophy. I should be rejected. Again, t he
322.5; Clearly, the statement is a lament for the fact immortality of the soul is not proved beyond doubt
that education is turning into just a money- in the given statement. Unless a statement clearly
spinning business. It is getting detached from said that the soul never ceases to accept new
values, which are at the care of a society. Hence, clothes this assumption would be suspect.
both are implicit. 342. 5; The statement talks about some persons who say
323. 4; Who are using history in the contemporary context that Indians are tolerant to a dangerous level. So
can’t be determined. Hence I is not implicit. It I is assumed in as much as such people are
may be used to spread harmony among various assumed to be existing. Further, the author refutes
communities. Hence II is not implicit. the claims of such persons. Hence, the author
324. 1; I is implicit because it is this demand that makes himself is proof that there are some persons who
people “spend big money”. II is beside the point. do not believe that Indians are tolerant to a
325. 4; There may be several other reasons for the large- dangerous level.
scale deaths during earthquakes. eg use of low- 343. 2; The crux of the statement is that although the
quality building materials, electric current. wheels, horses etc. make up the chariot, they
326. 5; Clearly, the speaker does not believe in one-match are not the chariot. In other words, parts can
wonders. Hence, I is implicit. II is also implicit never substitute for the whole. So II is implicit. I
because consistently good performance over a is not hinted at in the statement.
long stretch can be measured in terms of average. 344. 5; The phrase “one step further” implies that some
327. 5; Because of II LIC has advertised so. If I were not violence has already occurred and the author
the assumption, they would never advertise. plans to move “one step further” from this level
328. 5; The finance minister says that the Opposition is of violence. Further the word “retaliate” implies
Assumptions 61

that violence has been against the author to 356. 1; I is implicit because if Booker Prize brings global
which he plans to give a befitting reply. Hence acceptance, it must be a sign of recognition. II is
both I and II are implicit. not implicit. The coming of age of Indian writing
345. 5; I is obvious from the statement. II is obvious by in English may be an inference, but not an
t he way cooler is compar ed t o f an in the assumption.
statement. 357. 5; If a thing is no longer so, we assume that earlier
346. 4; I is not true for everybody. The statement means it was so. Hence I is implicit. Again, if corruption
to say that practice may not be sufficient for came as a shock earlier, we further assume that
success if one is pitted against a born genius. In corruption was there. Hence II is implicit.
other words, it means that practice is necessary 358. 4; If X is more dangerous than Y, we assume that Y
but it may not be sufficient. II is not implied is also dangerous. But I is not implicit because it
beyond doubt. It is supported by the statement states just the contrary. II is also not implicit.
only in a vague fashion. We rather assume that hedonistic activities can
347. 1; If a manager contemplates punishment for the be performed with aids.
er ring st af f, a capability t o punish must be 359. 1; I is implicit because one team cannot defeat the
assumed. ot her unless bot h par t icipate. II cannot be
348. 2. Allowing pass-holders without checking does not assumed. There may have been other teams which
necessarily mean less risks. It could be done were eliminated before India met Pakistan in a
because of time factor or other reasons. But one five-match final.
thing is definite. Those having passes must be 360. 2; I is not implicit because it talks about a subject,
assumed worthy of trust. Without this being true, not the author. II is implicit because when we
allowing t hem wit hout check-ups makes no say that a book is for a particular target segment,
sense. Hence II is implicit. we assume that such a target segment exists.

K
349. 1; If listening to the inner voice can save one from 361. 1; I is implicit: how else can helping the poor be a
sin, assumption I must be valid. Assumption II is service? II is not implicit because not all human
uncertain. The statement merely says that Gandhi beings are in the “service” business.
was saved from sinning on many (not every) 362. 2; I is not implicit because the statement is based
occasions. So chances of a few transgressions only on what happens in India. II is implicit
can’t be ruled out merely on the basis of the given because of the concern shown by the statement.
statement. 363. 2; I is not implicit because we don’t know whether
350. 1; If a polite person’s son being rude is a matter of the statement is based on past experience. II is
shock then it must be assumed that politeness implicit because education is very important in
usually passes from father to son. So I is implicit. equipping the children.
351. 1; First the author says that whenever he watches, 364. 5; Both the assumptions are implicit because of the
India lose a wicket. Then he declares that “so” need felt for a shopping complex in the colony
he will not see t he next crucial match. The itself.
assumption here obviously is that if he doesn’t 365. 1; The term ‘which you were unable to prepare earlier’

KUNDAN
watch the match India will not lose wickets. In clearly means even though t he user knows the
other words, his coming to watch is one cause of procedure of some special preparation, he does not
India losing a wicket. II is uncertain. No hint is have a proper oven to cook. Hence I is implicit. The
made on winning or losing. statement assumes that ‘X’ can cook tasty dishes but
352. 4; Feeling uneasy is not a sign of addiction in itself. does not rule out all other ovens from doing so.
Feeling uneasy after avoiding Gutkha is a sign of 366. 1; Accommodation to only outside candidates clearly
addiction. [Read I carefully. If you have the book indicates that local candidates will be having
Magical Book: Analytical Reasoning, refer to page it t heir own ar r angement f or stay. Hence I is
immediately.] Secondly, a thing that is suspected implicit. It is no where mentioned that outside
to be addictive can never be advised to be taken candidates will not be selected. Hence II is not
every two hours! Both I and II are not implicit. implicit.
353. 5; The author says that he won’t succeed in UPSC 367. 5; Providing anything for a solution automatically
exam. Reason? Because he is not hardworking. leads to the fact that the provider has assumed
Obviously, hard work is essential for success at that neither it will disturb while working nor it
UPSC exam. Further, if hard work is a necessary has any adverse side effect, and that it will rather
prerequisite of any exam, the exam can’t be easy. promote the work.
Both I and II are implicit. 368. 1; I is implicit. Because of I, the policy-makers have
354. 1; The author says that he hopes to succeed in SBI acknowledged the ser v ices r ender ed by t he
PO exam. Reason? They had more vacancies. voluntary agencies. II is not implicit. Designing
Clearly, more vacancies are taken as giving a and service contribution are two different things.
higher chance to succeed. So I is implicit. II is 369. 5; If it is impossible then why would a nation commit
not. The statement makes no comment about the so? Hence I is implicit. The term As a nation used
degree of toughness of the exam. in the statement and then about its commitment
355. 2; If we say that it is high time X did something, for its people confirms II.
we assume that X is not doing that at present. 370. 4; As only is used in both the assumptions, none is
Hence, assumption II is implicit. But I is not implicit.
implicit. Infrast ructure lending is one of the 371. 2; I is not implicit. The statement puts it the other
services but whether it is the best we do not way round. That is, health results in happiness.
know. II is implicit, hence the statement.
62 Practice Book on Analytical Reasoning

372. 5; Definitely because of assumption I and II the II is implicit because it makes no sense to talk of
publisher has warned in the cover page of the somet hing wit hout the exist ience of it s
book. possibility.
373. 5; Sensitivity can be created by the media only when 389. 1; ‘A’ is assuming I, that is why he has suggested
it possesses both these attributes. hard work to B. Hence I is implicit. The word All
374. 1; I is implicit. The provision would not have existed is too strong in II. Hence II is not implicit.
if there were no likelihood of such an act being 390. 4; Assumption I contradicts the statement. Nothing
committed. II is not implicit. Merely debarring can be assumed about the scale of becoming
public servants from immunity does not ensure developed. Hence II is not implicit.
equality for all. 391. 2; I is not implicit. There is no hint regarding who
375. 3; Higher efficiency will be brought about in either cont r ols our behav iour . II is implicit . The
t he case. W hen you have good models, the statement takes exception at the fact that he does
environment improves. On the other hand, even not apply to himself what he teaches.
if this not be case, fear of being eliminated leads 392. 2; I is not implicit. The statement only says that
to adaptation in the Darwinian fashion. the two should go together, not that one results
376. 5; I is implicit . That is why t he demand f or from the other. II is implicit; that is why the
qualification and experience. II is also implicit. emphasis on social justice.
Unless the institute were choosy, it would have 393. 5; The customer is assuming I that is why he has
welcomed one and all, without imposing any lodged several complaints of not receiving his
restrictions. telephone bills. As he has informed about this to
377. 2; I is not implicit: Competition from whom? There the editor of a daily, he expects to correct the
are no such hints in the statement. II is implicit: system. Hence II.
it is this appreciation that has propelled the credit 394. 2; The statement does not point to the competence
card company towards such an excellent customer of anybody. It merely hints at the power of co-
service. operation.

K
378. 1; I is implicit; hence the caution “to be frank and 395. 5; The police is assuming I that is why they have
objective”. II is not implicit. The statement does interrogated them. Hence I is implicit. II is a more
not hint towards the motive of the report. generalised form of I. Hence implicit.
379. 5; I is implicit; that is why the talk of “observational 396. 4; We are in no position to pass any judgment at
learning”. II is also implicit; note the use of the present. We may assume these only if the appeal
word merely. is rejected.
380. 5; I is implicit : it is wit h this belief that t he 397. 4; I is not implicit because this is an ad for admission,
shopkeeper makes the recommendation. II is also not for a job. II is also not implicit for the same
implicit; that is why the shopkeeper emphasises reason. You don’t seek admission.
on “international technology”. 398. 2; The intention of internal applicants can’t be
381. 5; I is implicit. Note that the statement makes a assumed from the given statement. II is implicit;
distinction between merit and seniority. II is t hat is why t hey want t o recr uit out side
implicit: only when you can determine something professionals.

KUNDAN
that you make it the basis for further decision. 399. 1; Whenever a decision is taken by the company it
382. 2; What we are being told about brilliant students goes through every aspect threadbare. Hence I is
does not have anything to do with mediocre implicit. Debentures can be issued even when
students. Hence I is not implicit. But II is implicit. there are a large number of competitors. Hence II
This must be the reason why t he brilliant is not implicit.
students do not always excel. 400. 5; The company’s authority is assuming both that
383. 5; I is implicit. If the desire were not there, why is why they have invited tenders from reputed
the statement? II is also implicit ; hence the contractors. The term reputed confirms I. II is a
emphasis on “explore all channels”. universal assumption as the principal reason of
384. 2; I is not implicit. The grievance is against the inviting tenders.
govt, not against the banks. Banks serve only as 401. 5; The govt has promised to bring down the smoke
a medium for transfer of money. II is implicit level because it is possible to determine the level.
from the phrase “two months ago”. Hence I is implicit. II is implicit because
385. 2; I is not implicit in its present form. The author containing pollution is also a welfare measure.
assumes that it is unproductive unless utilised. 402.1; Any measure is taken with its efficacy in mind.
II is implicit ; hence t he lament on it s Hence I is implicit. But II is not implicit because
underutilisation. there is nothing in the statement that suggests
386. 5; The company is assuming II, that is why it has the fear of a threat.
manuf act ur ed t he medicine. W hy has t he 403. 3; The company may have either of the two situations
company stressed on the constituents and the in mind. Generally, reduction in price is expected
effect in its advertisment? Definitely because the to lead to increase in sales. Hence I is implicit.
company management is assuming I. Alternatively, it is possible that the decision has
387. 1; Failure to meet the demands is possible only been taken with other manufacturers in mind.
when the demands have been raised in the first In that case II may be implicit even without
place. Hence I is implicit. II may be a conclusion assumption I.
but not an assumption. 404. 5; W hy t he apology? It is a submission t o the
388. 5; I is implicit. Tools for an objective are talked about sensitivity of the Indians. Hence I is implicit. II
only when the desire for such an objective exists. is implicit in the word misinterpreting.
Assumptions 63

405. 5; Lalit’s mother is instructing him what course to enthuse employees to evolv e innov ativ e
take if it rains heavily. Clearly, she assumes his techniques. Hence, II is implicit. On the other
incompetence in taking a decision of his own. side, seeking help from employees in order to
Hence I is implicit. Returning by train is possible evolv e an innovative t echniques implies that
only when the trains ply. Hence II is implicit. employees may have the required calibre. Hence,
406. 5; A decision to start any sort of dialogue with one’s I is also implicit.
neighbour assumes that the neighbour will also 423. 1; I is implicit; that is why the civic authorities are
participate in it. Hence I is implicit. II is implicit advising so. II is not implicit. No one gives advice
because track II dialogue can be meaningful only to another unless the former assumes that the
when the respective Govts agree to abide by it. latter will follow the advice.
407. 5; The host of the programme is assuming both. 424. 2; Engaging the Army for the rehabilitation work by
Announcement to distribute fabulous prizes has the government implies that t he gover nment
the clear purpose to enhance the viewership. assumes that the Army possesses the required
408. 4; The employees’ association is generally concerned ability to rehabilitate the affected people rapidly.
with the welfare of employees and not with the Hence, II is implicit but I is not implicit because
benef it of t he or ganisat ion. Hence II is not of the word ‘only’.
implicit. I, also, is not an assumption. 425. 3; Investment in the shares of company ‘A’ has been
409. 2; Assuming II only, the government has agreed to termed as ‘a gamble’. This implies the speaker
work out an effective social security programme. must be assuming that the investment may either
410. 1; To motivate the employees and hence for the incur loss or bear profit.
enhancement of their work, t he head of t he 426. 4; I is not implicit (note the word ‘at least this year’).
organisation congratulated the entire staff in his II can’t be correlated. Hence, II is also not implicit.
speech and appreciated their effort. 427. 1; The wor ds ‘not t r ue always’ implies t hat

K
411. 1; Whenever such notices are displayed it is assumed sometimes the move bears positive results. Hence,
that those who are concerned with the notice the speaker must be assuming I. II is obviously
will read the notice and follow the messages in not implicit.
it. Hence I is implicit. If II were true, it is not 428. 4; The tone of the statement only implies that the
necessary to display such notices. speaker assumes t hat newspaper s ar e mor e
412. 5; Here both the assumptions are valid. If it were effective than oral call. The given assumption I
not so, there would be no benefit of setting up is not implicit because of the word ‘rarely’. II is
such mission. also not implicit because it contradicts the valid
413. 1; I is implicit; that is why the advertisement has assumption. Note that here the valid assumption
been given. Second one is absurd. is “people will r ead an adver t isement in a
414. 5; I is obviously implicit. II is also implicit; that is newspaper”.
why cancellation of citizenship has been talked 429. 2; I is not implicit. It is assumed that people will
about. pay taxes. That is why II is implicit. But the
415. 2; I is vague. But II is implicit because the fixation Govt does not necessarily assume that people will

KUNDAN
of amount as Rs 1999 per month must have been cont inue wit h t hese schemes. Maybe it is
done after assuming II. discouraging people from being mere savers rather
416. 2; The wording “Beware of dogs” makes II implicit. than investors.
417. 4; I is not implicit. The valid assumption is that 430. 5; No government decides to impose extra taxes
the existence of corruption and prejudice is not unless it assumes that the amount it has is
desirable. II is also not implicit. The statement insufficient to serve its pur pose. Hence, I is
is silent on whether the administrative system implicit . II is also implicit ; ot her wise the
can be reformed. government would have increased the levy from
418. 5; Why a review committee to find out the reasons the targeted 2% .
for unstable stock prices? Concerned authority 431. 1; I is implicit from the tone of the offer of the ‘X’
must have assumed I. Hence I is implicit. One Housing Finance company. But II is not implicit.
does not entrust a work to another unless the 432. 2; The real concern of WHO is the low per capita
former assumes that the latter has the efficiency expenditure on health in India. Now, how will
to do that work. Hence, II is also implicit. t he init iat iv e t aken by W HO minimise the
419. 5; Assumption I is implicit . Why has such concern? The assumption is that the enhanced
advertisement been published by the advertisers? assistance may substantially increase the per
Definitely, the advertiser is assuming I. II is also capita expendit ure on healt h in India. But ,
implicit; that is why advertiser advocates doing assumption I is not implicit due to the last part
so. of it, ie ‘and bring it on par with other countries.’
420. 5; The idea behind facilitating learning computer Again, no one provides help to others unless one
at no cost is aimed at attracting learners. Hence assumes t hat t he other needs it . Hence,
I is implicit. II is also an assumption; that is assumption II is implicit.
why t he adv er tisement adv ocat es lear ning 433. 4; When decisions are taken, the assumptions are
computer. positive. It is assumed that various sections would
421. 5; Both I and II are implicit. The government does facilitate the implementation of the decision.
not launch its programme unless it assumes that Hence neither is implicit.
it has the basic infrastructure to implement it. 434. 1; The step taken by the airlines must have been
422. 5; Why a reward for a good suggestion? Obviously, aimed at to earn more revenue. The desired result
authorities are assuming that the reward will can’t be obtained without assuming I. Hence, I is
64 Practice Book on Analytical Reasoning

implicit. II may or may not be an assumption. request made by the authorities implies that the
435. 5; Generally, an advertisement is given to attract authorities must be assuming that their effort will
customers. Hence, assumption II is implicit. The reduce the problem of power theft. Now, how
advertisement mentions some feature. Why will can t hey assume t his? Obv iously, II is an
the mentioned feature attract customers? Hence assumption.
I must be assumed. 446. 5; Why did PTA take such a harsh decision? They
436. 2; Why did bank ‘A’ reduce the interest rate on retail must be assuming that its decision may compel
lending? The step taken by the bank must be the principal to reconsider the school fees. Hence,
aimed at generating more revenue. Hence, II must assumption II is implicit. Again, how can the
be assumed. I may or may not be an assumption. decision usher such result? Obviously, PTA must
Hence, I is not implicit. be assuming that parents of the students will
437. 5; When a notice is given by an organisation to a cooperate. Hence, assumption I is also implicit.
group of persons, the former assumes that the 447. 2; Assumpt ion I can’t be cor related wit h t he
latter will follow it. Hence, I is implicit. statement. Hence, assumption I is not implicit.
438. 2; The tone of the statement implies that sometimes Now, why did the aut hor it y go f or t he
the mightiest superpower does not get accolades advertisement? Obviously, II must be assumed.
from other countries even after winning the war. Hence, only II is implicit.
Hence the speaker must be assuming II. The tone 448. 5; The scheme will attract the children, particularly
of the statement stresses on the failure of the those who have no good food available at home.
mightiest to get accolades from other countries So both assumptions are implicit.
despite winning t he war . Hence, I is not 449. 1; I is implicit. If it wee not so, equal potential would
necessarily implicit. have implied equal salary and vice versa. This
439. 2; I is not implicit. In fact, the statement suggests one-to-one cor respondence could hav e made
it is possible to contain terrorist activities. What salary the only criteria to judge potential.
is disputed is “how long”. But II is implicit. The If II were implicit, if would mean salary is linked

K
action has begun, though its end is not in sight. with factors other than potential. That is, potential
440. 1; I is implicit from the victims being “nuisance to can’t be the only criteria to decide salary. But
peace-loving people.” II is not an assumption. t his is on a t r ack dif fer ent fr om wheat the
Note that the assumption here may be: “Majority statement goes on.
of the people killed in wars are wicked.” 450. 4; Material desires of a person are determined by
441. 4; Dishonour ing of global public opinion by the three factors — the basic necessity, the utility of
superpower implies t hat the speaker must be items purchased and the appetite for luxury items.
assuming that the global public opinion is against Hence I can’t be the assumption. II is also not
the imposition of war. But note that II says: “... implicit.
should have been ... .” Hence, not implicit. I is not 451. 1; As a Principal, he t rust s his t eacher that they
implicit either. It is, in fact, contrary to the speaker’s would have prepared the students thouroughly. So
assumption. that they could appear in the list of toppers. Hence
442. 5; The word ‘vehemently’ used in the statement I is implicit.

KUNDAN
implies that the speaker must be assuming II. Only intelligence cannot be t he basis of t he
Why is the speaker in favour of discouraging wars Principal’s statement. Hence II is not implicit.
vehemently even though majority of the victims 452. 5; I is implicit from the need “to maintain the present
might hav e been a nuisance to peace-loving position”. But why the question of lag? Must be
people? The speaker must be assuming that war because computers are much more efficient than
is disastrous for peace-loving people also. Hence, manual labourers. And this holds true only for
I is implicit also. complex tasks. Hence II is implicit.
443. 1; The decision taken by the government implies 453. 5; Why has the club announced a wide variety of
that the government must be assuming I. Note music programmes by the visiting musicians? The
that no organisation/government/person takes organisers must be assuming that the musicians
decision to compensate the vict im unless it may be able to play a variety of programmes to
assumes that it has adequate fund to satisfy the interest the people. Besides this, they must be
expenses. II can’t be correlated. Hence, II is not assuming also that the programme will attract
implicit. audience. Thus, both I and II are implicit.
444. 1; Why has the suspension of flights been mad for 454. 1; I is implicit; that is why the bank expects to
a limited period of four days? It must have been wor k ef f iciently. II may or may not be an
assumed by the authorities of X-Airlines that the assumption. Hence, II is not implicit.
crisis may be over after this limited period. Hence, 455. 1; I is implicit; that is why the principal instructed
assumption I is implicit. II may or may not be an the teachers to carry the responsibility. II is not
assumption. Hence, II is not implicit. implicit. The mischievous section may not welcome
445. 2; Request made by civic authorities to the citizens the decision.
implies that the authorities must be assuming that 456. 2; I is not implicit. In reality it contradicts the actual
the citizens will respond positively. Now look at assumption. Again, why, has gover nment
assumption I. It may or may not be an assumption abolished the scheme? The government must be
because the way of responding by the citizens may assuming II.
not necessarily be the same as mentioned in the 457. 1; The notice has been aimed at reducing criminal
assumption. activities. How can the criminal activities be
Hence, assumption I is not implicit . Again, reduced without people’s participation? Hence,
Assumptions 65

the police must be assuming I. But we are not * Most of the students want coaching classes
sure about II. Hence, II is not implicit. at a lesser fee.
458. 5; If any circular is issued for an individual or a 469. 2; I is not implicit because of the word 'medication'.
gr oup of people then it is assumed that t he The objectiv e of the adv er t isement can't be
individual or the group of people will abide by it. achieved without II. Hence, II is implicit.
Hence, I is implicit. Again, why did the authority 470. 2; I is not implicit because of the word "All". II is
need to issue such cir cular? It must hav e a implicit: it is this that makes the speaker say
const r uct iv e pur pose r egar ding t he ser vices "you must have at least the basic knowledge of
provided by the organisation. Hence, II is also computers.
implicit. 471. 5; How can the objective of the civic authorities be
459. 1; Why did the government take such an initiative? meaningful without the people's participation?
The government must be assuming that these Hence, I must be assumed. Again, how can the
teachers may be useful in improvement of primary people's participation be possible wit hout II?
education. How can the objective mentioned above Hence, II must be assumed.
be obtained without assuming I. II is absurd. 472. 5; I is an assumption because the government is
Hence, II is not implicit. assuming that the initiative taken by it is an effort
460. 4; I is not implicit because according to the statement to rectify the problem. II is obvious because the
the move is supposed to bridge the gap between initiative taken by the government has been aimed
income and expenditure to a larger extent. II may at benefiting the farmers.
or may not be an assumption because of the word 473. 5; If the government has made a plan to provide relief
‘will’. to the farmers, it must be assuming that the plan
461. 1; The government is adamant not to reduce the can be executed. The objective can't be fulfilled
prices of pet roleum product s despit e the without I and II. Hence, both I and II are implicit.

K
significant drop in the crude oil prices in the 474. 4; The Principal must be assuming that the students
international market. Why? It must have some will abide by the instruction. Note that if a person
const r uct ive objectiv e which compelled t he instructs someone or a group of persons, the
government to do so. Hence, I is implicit. We former assumes that the latter will abide by it.
can’t correlate II with the statement. Hence, II is Hence, both I and II are not implicit.
not implicit. 475. 1; The initiative taken by the railway authority has
462. 5; Why did the government go for the appeal? It been aimed at making the ser vice-customer
must be assuming that the people will abide by it friendly. Hence, I is implicit. II may nor may not
and t he appeal may generat e mor e r evenue. be an assumption. Hence, II is not implicit.
Hence, both I and II are implicit. 476. 2; From the adv ert isement, it is obv ious t he
463. 2; I may or may not be an assumption. Note that if the advertiser must be assuming II. Hence, II is
two companies X and Y are paying equal salaries implicit. I goes rather against the statement.
to their employees and the growth prospects of Hence, I is not implicit.
employees is better in Y (in comparison of X) then 477. 4; The step taken by municipal corporation has been

KUNDAN
many employees working for the company X may aimed at good entertainment for the local residents
prefer to work for the company Y. But II is obvious. during the holidays. The municipal corporation
Hence, only II is implicit. must be assuming that local r esident s may
464. 5; Why did the principal of the college warn the participate in the fun fair with great fanfare. Since
students? The principal must be assuming that I and II go against the above assumption both I and
t he war ning may bring t he sit uat ion under II are not implicit.
control. And how can the situation be under 478. 5; The advertisement would become meaningless if
control without II? Hence, II must be assumed. the two assumptions were not implicit.
Again, what compelled the principal to go for 479. 1; I is implicit in the claim being made in t he
such a warning. It is I. Hence, I is also implicit. announcement. When you make a claim, you
465. 1; How can the advertisement be meaningful without highlight your achievement. But II is irrelevant.
I? Hence, I is implicit. II may or may not be an Such "cooperation" from the people one does not
assumption. Hence, II is not implicit. expect.
466. 5; If ‘A’ advises ‘B’ to do something, then the former 480. 5; Assumption I is implicit as the motive behind
assumes that the latter has the required expertise the comparison. II is implicit in the purpose of
to do it. Hence, I is implicit. Again, why did the advertising.
management need t o adv ise regarding use of 481. 2; Whenever a vendor increases prices, he assumes
computers? It is obvious that the management must t hat his commodit ies will sell ev en at t he
be assuming it to be necessary. Hence, II is also increased price.
implicit. 482. 2; Admission is still a far way off. We are only at
467. 1; I is implicit the for m collect ion stage and hence I is not
468. 4; I highlights. Hence I is not implicit. II can't be implicit. But II is implicit as it is this assumption
correlated with the statement; hence II is not that has prompted people to stand in the queue.
implicit. The given advertisement is based on 483. 5; A decision is taken when it is felt that it would be
the following assumptions: accepted by most of the people concerned. Hence I
* A discount may make the course fee seem less is implicit. II is also implicit as the reason behind
burdensome. the need.
* A discount may at tr act more and mor e 484. 4; It is not necessary that price rise be there on the
students. mind of the govt while taking the decision. Hence
66 Practice Book on Analytical Reasoning

neither I nor II is implicit. In fact, the truth is 502. 4; If the response does not come, as Assumption I
that our petroleum companies are running losses says, the ad won't make sense. Hence I is not
even after the drop in international prices. implicit. II is not implicit; hence the qualification
485. 5; Both are imminent positive outcomes assumed. "with good communication skills".
486. 1; I is implicit because teachers can't be appointed 503. 2; Assumption I is absurd. In such illnesses, the
in a vacuum. II is more of a presumption. patients are rushed to hospitals, irrespective of
487. 5; When a move is made, it is assumed to be effective. how the relatives may behave. II is implicit in
Hence I is implicit. It is also assumed that the r ushing t he af f ect ed people t o t he near by
stipulated target will be met. Hence II is implicit. hospitals.
488. 1; Assumption I is implicit because it is this that 504. 5; W hen a pr omise is made by someone, t hey
makes us import sugar in spite of the increase in assume that they would be able to fulfil it. Hence
the number of sugar fact ories. But II is not I is implicit. Since the business ventures are
implicit because "future" is beyond the scope of planned to be set up in rural areas, II must be
the statement. implicit.
489. 2; Compensation is a way of sympa-thising with 505. 5; I is implicit as the cause of blocking the traffic. II
the victims, not a deterrent to terrorism. Hence is implicit because whenever we plan something,
II is implicit but I is not. we assume that the conditions may be favourable
490. 1; Assumption I is implicit because only then the enough.
switching over makes sense. But II need not be 506. 4; Whenever an instruction is given, it is assumed
an assumption. The switching over may have that it may be followed. Hence I is not implicit.
been prompted by economic factors or those of II is also not implicit. If at all the govt does
convenience. assume something about the parents, it is to the
491. 2; Assumption I is ruled out because of the word contrary: that the parents may not be able to pay
only. But II is im plicit be cause wi t hout higher fees.
considering this factor relocation won't make 507. 1; I is implicit as diversion would be necessary

K
sense. while the construction goes on. II is not implicit
492. 1; Assumption I is implicit as the govt's moves are as negative reactions are not assumed.
generally aimed at protecting the interests of 508. 2; When prices are reduced, the motive is to attract
the masses. But II is not implicit because of "any customers. Hence I is not implicit but II is.
other way" . Ther e might be other means of 509. 5; Both have been assumed by the railway authority
gambling which t he gov t does not consider while putting the revised timing on the website.
significantly detrimental for the people. 510. 2; Grace marks are given with the assumption that
493. 2; When one applies for leave, one assumes that it adding these marks will lead to positive result.
would be granted. Hence I is not implicit. But 511. 5; I is implicit in the deadline given. II is implicit
Assumption II is implicit because only then the in the notice given to the residents to move out.
period of "two years" assumes meaning. 512. 1; I is implicit; otherwise more players would have
494. 1; When you instruct someone to do something, been selected. II is not implicit because even
you assume that he may do it. Hence I is implicit without the captain, ther e will be suf ficient

KUNDAN
and II is not. number of players.
495. 5; Assumpt ion I is implicit as this is the pur pose 513. 1; I is implicit in the preparation being made. But
assumed while levying the tax. II is also implicit II is not implicit as "about thirty" implies "nearly
because when a rule is framed, it is assumed that thirty — maybe a few less, maybe a few more".
people are capable of following it. 514. 2; I is not implicit because of the word " All" . II is
496. 1; Assumption I is implicit in "at their own cost". implicit as applicant s assume a f air select ion
Assumption II is contrary to what the citizens process.
may have assumed. 515. 1; When an order is passed, it is assumed that
497. 4; Were it assumed that the employees might leave, people will comply with it.
such a decision would not be taken. Hence 516. 4; Assumption I is irrelevant while II is contrary
Assumption I is not implicit. Assumption II is to the assumption being made.
not implicit because " next year " is pr esent 517. 2; When workers go on a strike, they assume a
nowhere in the picture. positive response, viz, the authorities may heed
498. 5; Both are implicit in Mr X's instructions to his to their demands.
assistant. 518. 5; Note that t he "separat e lane" decision is an
499. 2; II is implicit a s it ser v e s t he pur p ose of incentiv e. Now an incent iv e is given wit h a
introducing AC buses. Contrary to this, I is not motive in mind (Assumption I) and with the hope
implicit as it defeats the purpose. of people falling for it (Assumption II).
500.1; I is implicit because whenev er a scheme is 519. 5; Assump t ion I is implicit in t he inv it at ion.
announc ed, it is assu med t hat peop le will Assumption II is implicit in the request Manish
welcome it. II is not implicit. Had fear been there makes to his mother.
on the mind of the govt, it would have refrained 520. 2; Assumption I is not implicit because adverse
from introducing such a scheme. impacts are not assumed. II is implicit in the
501. 4; I is not implicit because an announcement is period of suspension being six months.
made wi t h t he assump t ion t hat it will be 521. 1; Assumpt ion I is impl icit because such an
positively received. II is not implicit because the announcement is made with the hope that people
municipal authority is not covered by the ambit will heed to it. For the same reason, Assumption
of the statement. II is not implicit.
Assumptions 67

522. 4; With such negative assumptions, the statement 543. 1; I is implicit in t he norm prescribed in t he
would lose its meaning. sentence. This is why checking is being advised.
523. 2; The driver does not have control on what the Again, what would the banks check? Obviously,
auto driver will do. Hence I is not implicit. But II what the clients reveal. Banks would assume the
is implicit as one assumes the outcome while revelation to be true. Hence II is not implicit.
taking an action. However, II is vague. If cross-checking is what
524. 5; I is implicit in the very giving of the advice. II is the speaker has in mind, II would become implicit.
implicit in the condition attached. 544. 1; If the govt has taken the decision, it must have
525. 1; I is implicit : when you ur ge someone t o do assumed t hat it s implementat ion would be
something, you assume a positive response. For possible. II would not be implicit because of
the same reason, II is not implicit. “produced in the country”. It is possible that the
526. 2; If I were implicit, such a decision would not be govt has import on its mind for the said purpose.
taken. II is implicit in the purpose of fund-raising 545. 2; It is not implied that legal mining does not harm
that has been mentioned in the statement. and illegal mining does all harm. It is only implied
527. 4; When a warning is given, it is assumed that it t hat illegal mining is mor e har mf ul for the
will be heeded to. Hence I is implicit. II is not environment. This happens because norms are
believed to be the motive behind a warning (even flouted with impunity in illegal mining.
though the police often create such an impression 546. 1; I is implicit in the need for training. But II takes
among the people t hat they ar e seen as things to an extreme with the phrase “no skill
extortionists; well! Let that be on a lighter note.). sets”.
528. 1; Without passengers, the increase would make 547. 2; I is not an assumption because there may be
no sense. Hence I is implicit. II may be a probable several reasons for the preference being stated.
reason but not a necessary one for the increase. II is implicit because you ask someone to employ

K
529. 5; I is implicit: you don’t hold a function without a means only when you assume that the means
being r eady for t he preparat ions. II is also is available.
positive: when you invite someone, you assume 548. 4; The assumption is that students have access to
that he will come. the Internet. But it is not necessary that they
530. 2; Price have been slashed with the assumption have this access at home. Hence I is not implicit.
that sales will increase, thus leading to profit. Again, past practice may not have been borne in
531. 4; Both of these assumptions are negative. mind while switching over to Int er net -only
532. 1; A r equest is made t o the people wit h the display. Hence II is not implicit.
assumption that they would comply. Hence I is 549. 4; I is rather contrary to the assumption. The need
implicit. II talks of activists and is hence beyond to grow different types of crops is talked about
the scope of the statement. precisely because there is a likelihood of farmers
533. 2; I is not implicit: Even if other matches are played growing the same crop again and again unless
in the near future, people may like to see this instructed. Again, growing different type of crops
is important but not sufficient. Hence II is not

KUNDAN
match. II is implicit: When you stand in the
queue, you hope to get ticket. implicit.
534. 1; The boards have been put with the assumption 550. 2; I is not implicit because health is not the focus
that they may have a positive impact. of the statement. II is implicit in the very need
535. 1; Only I is implicit. II may not be there in the for substitution.
association’s mind. It may only be assuming that 551. 1; I is implicit because only then does the storage
a reduced at t endance may dr iv e home t he make sense. II is not implicit as the statement
employees’ point. has nothing to do with “a shorter period of time.”
536. 2; There is no indication of an earlier meeting. 552. 2; I is not implicit as the action of the NGO need
Hence I is not implicit. II is implicit in the calling not be based on comparison. II is implicit as
of the meeting. lit er acy is necessar y t o make t he librar y
537. 2; I is not implicit because the corporation has not successful.
advised the people to leave the city. 553. 1; I is implicit because only then does the direction
538. 5; I is implicit from the need of different types of to walk make sense. II is not implicit as the
letters on different occasions. II is implicit from st atement makes no distinction between t he
“official” and “semi-official”. members of the office and the visitors.
539. 1; I is implicit because only t hen t he person 554. 2; I is not implicit because of the word “all”. II is
checking can check the availability. But II is not implicit because only then does the appeal to
implicit. Personal details are generally required switch over make sense.
at the time of booking tickets. 555. 5; The comparison is possible only when I is
540. 5; Obviously, 0.7 is one of the types. Hence I is assumed. Same for II.
implicit. II is also implicit or else the statement 556. 1; I is implicit because of the use of “any” in the
would make no sense. statement. II is not implicit. It is possible that
541. 4; When we mention something to someone, we t he problem can be solv ed ev en later . The
assume t hey know it s meaning. Hence I is instruction to contact “immediately” is only for
implicit. But II is not implicit as the details cannot the sake of convenience of the customer.
be assumed. 557. 4; I would be implicit if the statement said “Use
542. 4; The statement does not give any clue to the use only our medicine ...”. Again, the instruction to
of language. Hence I is not implicit. II is not use is only of an advisory nature, not mandatory.
implicit because of only. Hence II is not implicit.
68 Practice Book on Analytical Reasoning

Practice Exercise–2
1. 5; I is not implicit: will it hurt the litigants’ interests is why the speaker distinguishes an educator
or the lawyers’? II is not implicit because of the from an academic.
word all. III deviates by mentioning the venue. 13. 5; The last portion of the statement makes I implicit.
2 . 5; Only III is implicit. Note that the statement is a Dissatisfaction of western countries is owing to
piece of news. What the journalist assumes about the absence of prime ministers or president of
the conversion and its impact can’t be explicitly member-countries. Hence II is also implicit. On
said. But the words “despite strong opposition” the same basis III is also implicit.
indicate that the journalist must be assuming III. 14. 5; Eit her II or III is implicit. The number have
3. 1; Why was the airport was put on a full alert? The decreased because of “mandatory verification of
authority must be assuming I. Hence, I is implicit identity”. Why? Either it is the fear of consequent
but II is not. Again, to put the airport on a full alert procedural delay or people fight shy of revealing
indicates that the authority must be assuming III their identity. Assumption I is unrelated to the
also. statement.
4. 5; Suggestion or direction to young officers by the 15. 5; I is correct because otherwise X can’t hint at the
officer is aimed at making the new recruits aware fact about the unique facility provided to MTNL
of their responsibilities. Assumptions I can’t be subscribers. II is also correct. Why else should X
correlated with the above statement. II is an tell this to Y? III is also correct. Why will Y feel
assumpt ion: t his is what leads t o adv er se a jolt? Hence, all are correct.
comments. III is not implicit: chances are the 16. 2; If someone takes an initiative to remove the
judgment is inaccurate; hence the extra caution. hurdles, it is not implied that he/she is the one
5. 5; How can the menace of smoking be reduced if who has created it. Hence, I is not implicit. II is
we restrict or reduce the showing of smoking in implicit because it is t he problem for which

K
movies? Obviously, the speaker must be assuming initiatives are being taken. III is also implicit
assumptions I and II. Assumptions III is also because one does not undertake an effort unless
implicit. That is why the speaker stresses on he/she assumes the difficulty can be overcome.
those deeds and creations which are conducive 17. 4; I is not implicit. A direction is given assuming
to reducing the menace of smoking. that the person, for whom the direction is meant,
6. 4; The tone of t he stat ement implies t hat , in t he will follow it. Hence III is implicit. While stating
speaker’s opinion, walking barefoot for miles on the intensity of the problem, school children and
dusty roads should be considered adventurous. college st udents are ment ioned. Hence, t he
Hence, the speaker must be assuming I. To convince speaker must be assuming II.
others about his opinion he makes comparison 18. 5; People take lessons from those who have a better
between walking barefoot rafting gliding. Why? He idea and experience to deal with the problems
must be assuming the assumptions III and II also. which people are f acing. Hence I and II are
7. 3; The reason of the person’s request to the editor implicit . Again, comparison bet ween the two

KUNDAN
to publish the names of the worst colleges, is countries implies that assumption III holds.
obvious. How will the request lead to an overall 19. 2; That credibility can be gauged is all that we can
improvement in the institution? The person must assume. Whether it can be measured is beyond our
be assuming I. Hence, assumption I is implicit. scope. Hence I is not implicit. II is not implicit
II may not be an assumption. But III is implicit because of ‘always’. Besides, both II and III attempt
because if someone requests for something to t o equat e t he Commonwealt h with t he wor ld
others, the former assumes that the latter will community.
concede it. 2 0 . 4; What induced Indian Meteorological Department
8. 2; I is not an assumption because the minister does to propose a brainstorming session on the issue
not have the employment perspective. The positive of just-concluded summer monsoon? It is the
features used in the statement in order to support assumption I which made them do so. II is
more women driving public transport implies that obv ious; ot her wise why is a br ain st or ming
the speaker must be assuming II and III. session needed? We sometimes take an initiative
9. 5; Only I is implicit. II and III can’t be correlated on the basis of positive result obtained through a
with the statement. Hence, II and III are not similar initiative in the past. A negative result of
implicit. a similar br ainstorming session can’t be an
10. 3; I is implicit; that is why there is discrimination assumpt ion f or t he pr oposed brainst or ming
in charges. II is also implicit; one does not take a session. Hence, III is not implicit.
decision unless one assumes it feasible. But 21. 4; Mr X may be taunt ing Mr Y. Hence I is not
nothing can’t be said with certainty about III. implicit. II and III are implicit from the way
11. 5; I and II are implicit because the speaker stresses “looking sad” and “getting bonus” have been linked
on lit tle participation of students. III is also by Mr X.
implicit because the speaker wants an education 2 2 . 1; I is implicit; that is why ruling goes like this.
system which is designed keeping the changes Declaring non-Brahmin, having desired qualities,
of society in mind. also eligible for pujari implies that II must be an
12. 3; I is not implicit because it mentions the words assumption. No court delivers its verdict unless
‘thorough knowledge’. Hence I can’t be correlated it assumes that people will abide by it.
with the statement. II and III are implicit; that 23. 5; I is not implicit because it goes into too much of
Assumptions 69

specifics. II is not implicit because leaving the is trying to highlight company’s achievement and
village does not necessarily mean migrating to by doing that expansion of his business is in his
metro cities. III is obvious. mind.
24. 3; Since Mr X is determined to play against Mr Y’s 38. 4; Nothing is hinted about “hard work” or “upper
team the next day, he must be assuming III. II is crust of society. Hence neither I nor II is implicit.
implicit because of the words “at any cost”. III is also not implicit: In general, it is not very
25. 5; The phrase ‘nuclear war’ used in the statement “easy”; rather, one has to pay “a steep price”.
indicates that the speaker must be assuming I. The 39. 1; To appeal to start the written examination clearly
word, ‘mistaken’ implies that he must be assuming II hints about assumption I. Employees’ association
also. Why does the speaker think of the possibility of assumes II that is why they have appealed so.
war between India and Pakistan? He must be 40. 4; Why did the government take such an initiative?
assuming III also. It mut be assuming III. Hence, III is implicit.
26. 3; I is not implicit because of the word ‘always’. Again, any measure is taken by a government
The impor tance of ‘dying expr ession’ for with its efficacy in mind. Hence, II is implicit.
conviction implies that the Supreme Court must Again, how will t he initiat iv e taken by t he
be assuming II. How can a person judge the gover nment f ulfil the object i ve? Hence, the
veracity of a statement without having the capacity government must be assuming I.
to distinguish whether the statement is doubt- 41. 3; I need not be implicit because we don’t know
free or doubtful. Hence, III is also implicit. about t he company’s policy r egar ding ot her
27. 3; I is not implicit. But II and III are obviously segments. Again, if a manufacturer increases the
implicit; otherwise shopping time would not have price of products, the manufacturer assumes that
been proposed to be enhanced. there may still be adequate demand in the market
28. 3; I assumes too much. But why is there a need of of its products. Hence, III is implicit. Thus reject

K
speed governors for all four-wheelers? The court the options 1), 2), 4) and 5). Note that II may or
must be assuming II and III. may not be an assumption.
29. 3; The governor says that there would not be “further 42. 4; If an or ganisat ion or a per son entr usts a
weakening of rupee. ” Which is based on the responsibility to an organisation or a person, the
assumption that the rupee has weakened in the former assumes that the latter will carry out the
recent past. Hence I is implicit. II is not implicit; r esponsibilit y pr oper ly. And how can an
dollar-buying may be one of the reasons for the organisation or a person carry the entrusted
depreciations of the rupee. But one cannot assume r esponsibilit y wit hout hav ing the requir ed
this. III is implicit: note the use of the word expertise? Obviously, I is implicit. Also, it is
adequate. supposed that a university takes a decision with
30. 3; I is wide off the mark. In fact, it’s just the contrary a constructive purpose which will have positive
of what the statement says. II is implicit: when a impact on the students’ future. Obviously, the
statement is of the form A should be done for B, university authority must be assuming III. Hence,
we assume that A leads to B. III is not implicit. III is implicit. Again, II can’t be correlated with

KUNDAN
31. 5; All the three are implicit. Since company X talks the statement. Hence, II is not implicit.
of entering into a deal, it must be assumed that 43. 2; Why did the civic authority take such an initiative
two companies can enter into a deal. So II is in order to reduce the pollution? It must be assuming
implicit. Again, I is implicit because how can that the move will fulfil the desired objective. Hence,
company Y help company X unless it is capable I is implicit . And, how can t he mov e be made
to do so? Besides, this capability must be based effective? Obviously, the civic authority must be
on being financially sound. So III is implicit. assuming II. Hence, II is implicit. III can’t be
32. 1; A statement of the form though X, Y is unlikely is correlated with the statement. Hence, III is not
based on the assumption that X leads to Y. implicit.
33. 4; I is false; if the two were the same, how could 44. 1; The strike involves only two parties: the cinema
they meet two different fates? II is implicit; look halls and the government. The employees or the
at the way in which ‘inspect or raj’ is being public have nothing to do with the strike. Hence
contrasted with ‘licence raj’, which the reforms I and II are not implicit. II is not implicit because
have put an end to. III is implicit: if you wish to of the word all.
get rid of something, you assume that it is a 45. 3; Why did the employees call a strike? They must be
menace. assuming that strike can be an effective tool. It can
34. 3; Assuming I, the company has advertised f or put pr essure on t he gover nment and t he
housewives. If it were not possible, the company government may be compelled to consider the views
would not go for such an ad. Hence III is implicit. of the employees. Hence, I and III are implicit. II
35. 3; I is implicit in the phrase “industrial ... pollution.” goes beyond the scope of the statement. Hence, II
II is not implicit because of the word only. III is is not implicit.
implicit from the concern shown at the “terrible 46. 4; Let us delve int o the object iv e of t he
price”. adv er tisement . The main objectiv e of the
36. 4; The t erm ‘not hing’ has a br oad range of advertisement must be to attract more customers.
connotations. Hence I is not implicit. Since the How can the objective get fulfilled? It must be
speaker is t alking of sect ional need-based assumed t hat t he f eat ur e ment ioned in the
technology development, he is assuming both II advertisement may attract customers. Hence, I
and III. and II are implicit. III is not implicit because of
37. 4; Since it is hard to get ISO 9002 certification, he the word ‘best’.
70 Practice Book on Analytical Reasoning

47. 5; None is implicit. The step taken by municipal III is beyond the scope of the statement; hence
corporation of the city must have kept the positive III is not implicit.
aspects under consider ation. But I goes into 50. 1; All these assumptions are in directions contrary
details. Hence, I is not implicit. II and III can’t be to what the statement says.
correlated with the statement. Hence, II and III 51. 3; W henev er such a decision is t aken, t he
are not implicit. assumptions are that it would be welcome and
48. 2; What is the real objective of the advertisement? allowed to implement.
The object ive must be t o enr ol mor e school 52. 3; The urging of the govt makes sense only when
children through attracting a large number of (A) and (B) are implicit.
parents. And in order to obtain this objective, 53. 2; The decision to auction assumes response to it.
the advertisement has been adorned with many Hence (A) is not implicit . Unless the priv ate
features with an assumption that the features entities are capable, the decision would make no
may att r act par ents because par ents want sense. Hence (B) is implicit. (C) is implicit as
maximum facilities. Hence, II is implicit. But I is without financial benefit, private entities would
not implicit because I may or may not be an not turn up for the auction.
assumption. III can’t be cor related with t he 54. 2; (A) must be implicit to make the request
statement; hence III is not implicit. meaningful. The govt is out of picture here. Hence
49. 2; I is not implicit because of the word ‘only’. II is (B) is not implicit. (C) is not implicit as the case
obvious; that is why graduates with first class may be only of delay, not of cancellation of flight.
only are being considered eligible for the course.

K
KUNDAN
Cause and Effect 213

Chapter Six

Cause and Effect


Introduction Immediate Cause
Usually, in such questions the candidates are asked to An immediate cause is the one that is close to its result,
determine whether a given event is the cause or the effect in time. In other words, an immediate cause means a
of some ot her ev ent . The idea behind put ting t hese cause that immediately precedes the effect. Sometimes
quest ions is t o ascer t ain t he analytical and logical changes take place in the world in a continuous flux. For
reasoning ability of the aspirants. example, if a becomes angry then B laughs, if B laughs
then C cries, if C cries then D shouts and if D shouts
What is a Cause? then E explodes. Now, if A becomes angry; E will explode.
For our purposes, cause means the logical or scientific Here, each of the events (A’s anger, B’s laughter, C’s
reason of an event. Sometimes grandmothers may say that cries, D’s shouting) are cause f or the ev ent t hat E
exploded. However, the immediate cause behind the event

K
Ram met with an accident because a black cat crossed his
path. So, she is effectively saying that the “cause” of Ram’s was the shouting of D.
accident was the black cat who crossed his path. But our
Principal Cause
rational scientific view of cause does not agree with this.
For our purposes a cause is (a) either a scientifically A principal cause is the most important factor behind
proven (b) or a logically explicable reason that explains the occurrence of the effect. Sometimes an event is caused
an ev ent . For example, t he water -logging and by a variety of factors but one among them can be said to
contamination maybe a scientifically proven cause for an be the most dominant factor without which the event may
outbreak of cholera. On the other hand, if Ram reaches not have occurred at all.
college late because he could not get the bus in time, it is For example, if a student gets very good marks in
a case of a logical reason. Although no scientist ever proved Mathematics the principal cause must be that he is good
that if Ram missed a bus he would be late, it is logically in Mat hematics. If somebody says t hat t his year
consistent and simple explanation for his being late. Mathematics paper was easy, this may only be a secondary

KUNDAN
cause.
Cause and Effect
Rule of Antecedence
It is a fundamental property of nature that events do
not just happen; they happen because there was a cause Naturally the cause will always occur before the effect.
behind them. These causes are the conditions under Hence if two events are given then the effect that is
which these ev ent s (or r esult s or ef f ect s) happen. chronologically antecedent to the other can only be regarded
Something can be said a cause of another event only if it as a possible cause. In other words, we can look for possible
is a necessary, as well as sufficient condition for that effect causes by checking which of the two events occurred
to take place. first. For checking this we can check the tense etc. of the
( i ) A necessary condition for the occurrence of a sentences. We have the following rule regarding this:
specified event is a circumstance in whose absence Past Tense ( antecedent t o) Pr esent Per fect
the event cannot occur. (antecedent to) Present Continuous (antecedent to)
For example, the presence of oxygen is a necessary Future Tense.
condition for any fire. No fire take place without For example,
oxygen being present . But , alt hough it is a Past Tense: I eat mango/I was eating mango etc.
necessary condition, the presence of oxygen is not Present Perfect: I have eaten mango.
a sufficient condition for fire to occur. Perfect Continuous: I am eating mango.
( i i) A sufficient condition for the occurrence of an Future Tense: I will eat mango.
event is a circumstance in whose presence the
event must occur. Point to Remember
In t he above example, fire t akes place only if the In exams, generally you are asked to identify whether
substance is (a) cumbstible (b) the substance reaches a the given statement is the possible or probable cause or
minimum temperature and (c) there is oxygen present. the possible or probable eff ect . But , in some of t he
Thus, here (a), (b) and (c) make a sufficient condition examinations, we are generally asked to find out if a cause
and together they make the cause for the effect of fire. is immediate as well as principal. Roughly it means that
Note that each of the three makes a necessary condition the said cause must not only be the main reason behind
for fire to take place and when they are combined they the event, it should also be sufficiently close to the event,
make a sufficient condition. Thus, there may be several in time.
necessary conditions for the occurrrence of an event and
that they must all be included in the sufficient condition.
214 Practice Book on Analytical Reasoning

Practice Exercise–1
Directions: Given below are pairs of events ‘A’ and Event (B): The talks between the government and
‘B’. You have to read both the events ‘A’ and ‘B’ and agitating transporters have failed.
decide their nature of relationship. You have to assume 14. Event (A): Mr A will fight the election from Amethi
that the information given in ‘A’ and ‘B’ is true and parliamentary constituency.
you will not assume anything beyond the given Event (B) : Mrs S has decided to retain Bellary
information in deciding the answer. Mark answer parliamentary constituency.
1) If ‘A’ is the effect and ‘B’ is its immediate and 15. Event (A): Pakistan banks began a push against
principal cause. defaulters.
2) If ‘A’ is the immediate and principal cause and ‘B’ Event (B): Pakistan’s new military ruler is encouraged
is its effect. by international reaction and understanding of what
3) If ‘A’ is an effect but ‘B’ is not its immediate and he was trying to do.
principal cause. 16. Event (A): A gay 14-year-old youth is expelled from a
4) If ‘B’ is an effect but ‘A’ is not its immediate and prestigious private school.
principal cause. Event (B): The youth has announced he is in love
5) None of these with a schoolmate.
1. Event (A): The CBI has arrested the deputy director- 17. Event (A): The Duchess hinted in a newspaper that
general. she would like to marry Andrew again.
Event (B): He was alleged to have amassed a huge Event (B): Britain’s prince Andrew has ruled out
amount of assets through illegal and corrupt means. remarrying his ex-wife, the Duchess of York.

K
2 . Event (A): India is lodging a strong protest with 18. Event (A): A handful of women have managed to
Pakistan for firing missiles at IAF helicopters. transform the living conditions near their homes.
Event (B): Pakistani troops were now targeting even Event ( B): The project, ‘Bet t er Env ir onment
IAF aircraft flying well within the Indian side of the Campaign” was started by a group of retired defence
international border. officers’ wives in 1995.
3. Event (A): In one of the worst train disasters 400 19. Event (A): The national selectors have decided to
people were killed. retain the same 14 for the third and final test of the
Event (B): People were going from Delhi to Assam. series against New Zealand.
4. Event (A): A village in Delhi has reported over 52 Event (B): India has won the second test against
jaundice cases this season. New Zealand.
Event (B): At a number of places in the village, 20. Event (A): Since Rani’s own child had died at birth,
leakages in pipelines have resulted in accumulation she was desperate for another.
of water around the pipes. Event (B): Rani had quietly kidnapped the child,

KUNDAN
5. Event (A): Doordarshan warned cable operators of less t han two days old, and shipped out of t he
stern action if they failed to show its channels on hospital.
prime band for clearer reception. 21. Event (A): The number of rape cases is highest in
Event (B): Doordarshan announced 24-hour telecast Delhi in the country.
for programmes from August 15. Event (B): Delhi has the highest degree of social
6. Event (A): The Delhi high court issued show cause isolation in the country.
notices to 86 public schools in the capital. 22. Event (A): The new government has ordered a major
Event (B): The educational institutions had violated reshuffle in the top echelons of bureaucracy.
court directives of not hiking fee beyond 40 per cent. Event (B): Only two days before, the new coalition
7. Event (A): Manisha Mishra was preparing food in government took over.
the kitchen. 23. Event ( A): The f inance minist r y has imposed
Event (B): Anand, her husband, after coming from additional taxation.
his office ran into the kitchen. Event (B): The fiscal deficit of the country has been
8. Event (A): Mrs ‘A’ called Mr ‘B’ a traitor. reduced.
Event (B): Mrs ‘A’ made an apology. 24. Event (A): The ministerial talks between India and
9. Event (A): If you are a habitual traffic rule breaker Pakistan failed.
this won’t come as good news to you. Event (B): Led by a strong believer in people-to-people
Event (B): The traffic department has enhanced the contact, the Gandhian Nirmala Deshpande, a busload
number of red lights manned by the police. of Indian women spent a week in Pakistan.
10. Event (A): It seems power problems are to stay. 25. Event (A): The high-profile criminal was arrested
Event (B): Many units of power plants are shut down and put behind bars.
for annual maintenance. Event (B): The cops made more than adequate security
11. Event (A): The good news is that Elizabeth’s coming. arrangements in jail.
Event (B): Censor board has lifted up the objections 26. Event (A): A new TV channel has been launched
from Shekhar Kapoor’s film. with much fanfare.
12. Event (A): The government announced an increase Event (B): The govt has come up with a policy to put
in diesel prices by 40% . a check on the proliferation of TV channels.
Event (B): The Delhi Transport Corporation has 27. Event (A): Actress X decided to play more serious
revised the fares of the buses to double what it was. roles in films.
13. Event (A): Transporters have threatened to go on Event (B): The last film of actress X, in which she
strike in protest against the hike in diesel prices. played a glamous girl, turned out to be a flop.
Cause and Effect 215

28. Event (A): The migratory route of elephants in Area 44. Event (A): The opposition are protesting against the
X has been blocked by construction of dams and other gov er nment ov er t he decision of wit hdrawal of
structures. subsidy on food and fertilisers.
Event (B): Elephants of Area X have turned into Event (B): Govt has decided to withdraw subsidy on
violent creatures — destroying crops and trampling several items to reform the economy.
people to death. 45. Event (A): Indian’s infant mortality rate (IMR) is
29. Event (A): Three-wheelers went on strike in protest rising.
against the hike in petrol prices. Event (B): The economy of India is strengthening.
Event (B): The talks between the government and 46. Event (A): The Bar Council of India (BCI) has ordered
the leaders of the three-wheelers’ union have failed. all evening law colleges in the country to close down
30. Event (A): Party A was unhappy with the results of from the coming academic session.
UP assembly by-elections where it lost all the seats. Event (B): The standard of education in evening law
Event (B): Party A has changed the chief Minister of colleges was found in the state of deterioration.
UP. 47. Event (A): The government has framed rules to
31. Event (A): High Court has ordered to clean city X. regulate the level of noise pollution in urban areas
Event (B): More than 20 people died of cholera in from various sources.
city X. Event (B): Loudspeakers can be used only after
32. Event (A): CBI has called Kapil Dev and Sidhu for obtaining permission from a competent authority.
interrogation. 48. Event (A): No traffic shall be allowed on Jail Road.
Event (B): Kapil Dev and Sidhu have been charged Event (B): Due to the ongoing flyover construction,
in match-fixing. the Traffic Police have made several diversions.
33. Event (A): CBSE has declared the results of class 49. Event (A): Country X has signed CTBT.

K
XII. Event (B): Country X has already tested all her
Event (B): A boy committed suicide after seeing his nuclear weapons.
bad results in class XII. 50. Event (A): Tom Cruise has once again done a great
34. Event (A): There was a heavy rain for two days after job in MI- II.
a month of scorching heat in the capital. Event ( B): MI-I was f ull of act ion, t hr ill and
Event (B): There will be a pleasant weather over the excitement.
next 2-3 days. 51. Event (A): Many Indian companies are patching up
35. Event (A): One MLA has been found to have assets with multinational insurance companies.
more than his income. Event ( B): Gov t of India has allowed MNCs in
Event (B): The Chief Minister has ordered all the insurance sector.
MLA to disclose their assets. 52. Event (A): J&K govt demanded state autonomy.
36. Event (A): AIIMS doctors are on strike from today. Event (B): Parliament has disapproved the autonomy
Event (B): One patient died in AIIMS because no proposal by a considerable margin.
doctor was available for check-up. 53. Event (A): Mr X resigned from the post of coach of

KUNDAN
37. Event ( A): Two IPS of ficer s hav e r esigned in Indian cricket team.
Maharashtra. Event (B): BCCI is looking for a foreign coach for
Event (B): IPS officers are not being given proper Indian cricket team.
facilities in Maharashtra. 54. Event (A): Mr X received the highest-ever advance
38. Event (A): Curfew has been imposed in Kashmir for for a work of non-fiction.
indefinite period. Event (B): The work sold millions of copies.
Event (B): 45 people have been shot dead by the 55. Event (A): The 1,000-rupee note is being printed
mililants in Kashmir. once again.
39. Event (A): Fiji is facing political uncertainty. Event ( B): The 1,000-rupee not e went out of
Event (B): Fiji’s economy has been hit hard. circulation in 1978.
40. Event ( A): Used syr inges, glucose and blood 56. Event (A): The country’s fiscal deficit has widened
transfusion pipes were being reused. over the years.
Event (B): Supreme Court made it mandatory for Event (B): The govt has desided to abolish posts
hospitals to dispose of their medical waste. meant for carrying out obsolete functions.
41. Event (A): Modern Food Industries Ltd. employees 57. Event (A): Two pistols disappeared from a police
have moved the Delhi High Court to quash the sale training institute.
of their company. Event (B): An old woman was shot dead with a pistol
Event (B): The govt. has sold 74 per cent of Modern in the vicinity of the institute.
Food Industries Ltd. shares to Hindustan Lever for 58. Event (A): University ‘X’ and its affiliated colleges
Rs. 106 crore. have hiked the fees.
42. Event (A): In a committee it has been decided that Event (B): The govt has reduced the subsidy on higher
unauthorised constructions will be demolished. education.
Event (B): The Court has questioned the Urban 59. Event (A): Pradeep, father of a Class XII student,
Development Minister for his order to demolish the logged on to the website www.cbse.nic.in at 11 am on
unauthorise constructions. May 31.
43. Event (A): The National Human Rights Commission Event (B): The CBSE result s for class XII were
has ordered Rs. 1 lakh interim compensation to be announced on May 31.
paid to the legal heirs of the deceased in Meerut. 60. Event (A): An inmate died within the jail under
Event (B): A few hawkers in Meerut were driven to mysterious circumstances.
commit suicide due to humiliation by the police.
216 Practice Book on Analytical Reasoning

Event (B): An inquiry by a sub-divisional magistrate 73. Event (A): The Centre advised the Indian Hockey
has been ordered into the incident. Federation (IHF) against participation in the Sultan
61. Event (A): The Met department predicted that there Azlan Shah tournament to be held in Ipoh, Malaysia.
would be a normal south-west monsoon. Event (B): Indian IT professionals in Malaysia faced
Event (B): The south-west monsoon arrived over the harassment and were meted out ill-treatment by the
Andaman Sea as per schedule and is advancing at a Malaysian police.
steady rate. 74. Event (A): Next week India will play in the semifinal
62. Event (A): The US House of Representatives voted match of World Cup cricket in Johannesburg.
in favour of “permanent normal trade relations” with Event (B): Air India will operate a special flight to
China. Johannesburg in the coming week.
Event (B): There was European Union agreement 75. Event (A): A special campaign to check drunken
with China on WTO membership. driving would be launched next week on National
63. Event (A): 32 Indian Army men were taken hostage Highways.
by the rebels in Sierra Leone. Event (B): Ten persons were killed and twenty-three
Event (B): India is dithering in its approach to the injured on the National Highways last year.
Sri Lankan crisis. 76. Event (A): The women’s self-help groups established
64. Event (A): Miss world contestants flew out of Nigeria. in large numbers in the rural areas across State X
Event (B): A riot broke out in the capital of Nigeria have infused dynamism into the village families and
when some people of a particular community became supported the drive for empowerment of women.
rampant against the dissemination of western culture Event (B): Positive changes have been witnessed in
in the capital. State X, especially in the tribal areas in the southern
65. Event (A): An appropriate amending legislation will parts.

K
be brought before the parliament in its next session. 77. Event (A): In a significant move, cabinet of State X
Event (B): A parliamentary committee has asked the has approved a proposal by the State commercial tax
government to expedite the process of bringing in department to issue tax payer identification number
appropriate legislation to safeguard investors’ interest to all businessmen registered under the value-added
and strengthen Investor Education and Protection tax system.
Fund for proper compensation to those whose money Event (B): The Railway Ministry has enhanced the
has been locked up in fraudulent companies. allocation from Rs 3119 crores to Rs 3995 crores in
66. Event (A): Demand of Chinese bikes in India has the current budget for rolling stock procurement to
witnessed a terrific growth in India since last month. meet the next year’s passenger and freight traffic
Event (B): A large number of people visited the trade targets.
fair organised in New Delhi three months ago where 78. Event (A): Reserve Bank of India (RBI) cut the bank
various ranges of bikes of different countries were rate recently.
displayed. Event (B): Today, State Bank of India further reduced

KUNDAN
67. Event (A): All India Confederation of Scheduled the lending rates.
Castes and Tribes Organisation chairman Udit Raj 79. Event (A): Over the years, the breed of independent
today resigned from the Government and floated the film-makers has been growing steadily and while
‘Justice Party’. they have managed to give voice to the voiceless,
Event (B): Next month assembly elections will be their films do not reach the discerning audiences.
held in Gujarat. Event (B): The MLF Co, an NGO, has decided to
68. Event ( A): Tension t r iggered in Manoharpur step in by setting up a national distribution centre
prompted the authorities to deploy additional forces for socially relevant films.
to maintain law and order in the locality. 80. Event (A): Women have achieved parity with men in
Event ( B): Unidentif ied miscr eant s allegedly obtaining four-year college degrees and are more likely
desecrated an idol in a temple in Manoharpur. to work in managerial and professional careers today
69. Event (A): After long years of wait, the capital got its than 20 years ago.
metro last year. Event (B): The new high-tech economy is leaving
Event (B): Pollution levels in the capital have come women behind men.
down. 81. Event (A): India is today gearing to become a leading
70. Event (A): Last week MTNL changed its telephone producer and exporter of a range of minerals.
numbers from seven to eight digits. Event ( B): India is endowed wit h r ich mineral
Event (B): MTNL has again made changes in its resources.
billing cycle by extending payment dates for bills. 82. Event (A): Parliament gave nod to Tobacco Control
71. Event (A): Saving capacity per person in India has Bill last week.
decreased enormously. Event (B): Non-smokers no longer have to be victims
Event (B): There is a slight decrease witnessed in of passive smoking.
the per capita income of India. 83. Event (A): The people of state X have claimed that
72. Event (A): The cold wave sweeping state ‘X’ further vegetables like parwal and kakora are treated with
aggravated today as tribal areas experienced moderate malachite green to make them look fresh.
snowfall while other hilly areas had a glimpse of Event ( B): The of ficials of Food Adult er ation
snow. Depar tment has st ar t ed collecting samples of
Event (B): Thousands of tourists who have come to vegetables from suspicious places in state X.
state ‘X’ to celebrate Christmas are keeping their 84. Event (A): According to an NGO report, there are
fingers crossed and praying for snowfall, hoping for high levels of pesticides in Coca Cola, Pepsi, and
a white Christmas. other aerated drinks.
Cause and Effect 217

Event (B): Members of Parliament have demanded Event (B): Life of women is becoming insecure in
for a ban on the supply of aerated drinks in the rural areas.
Parliament. 96. Event (A): The government has finalised the contours
85. Event (A): The attack by militants on Parliament in of a massive debt swap operation.
Dec 2001 haunts our parliamentarians whenever Event (B): Central government’s current year’s gross
militants attack any target in the country. fiscal deficit touched 5.3% of GDP.
Event (B): New parking labels have been introduced 97. Event (A): Subscribers of Hutch and Airtel cellular
f or t he members of Parliament , aiming at services companies are increasing day by day despite
strengthening the security of the Indian Parliament facing tough competition from other cellular services
and the parliamentarians. companies.
86. Event (A): Growth of GDP forecasts of India have Event ( B): Recently, Hut ch and Air tel hav e
been lowered by NCAER, IMF and RBI recently. announced another joint scheme, according to which
Event (B): In India, drought has ravaged several crops subscribers of these giants will not have to pay for
and the government finances are under stress. incoming calls.
87. Event (A): A rich smell of recovery is spreading across 98. Event (A): In the world’s biggest conference on AIDS
several sectors of the economy, raising hopes of a in Barcelona, officials in their declaration called for
boom in employment and incomes in India. $ 10 billion a year to be donated to the global AIDS
Event (B): India never slid into a recession in the fund and for at least two million people with HIV in
past ten years. developing count r ies t o r eceiv e antir etr ov ir al
88. Event (A): The Centre categor ically rejected the treatment by 2004.
Liberation Tiger of Tamil Eelam’s plea for using either Event (B): According to recent UN report, about 70
Chennai or any other part of the southern states as million people will die of AIDS over the next two

K
v enue f or peace t alks wit h t he Sr i Lankan decades in the 45 worst affected countries.
Government. 99. Event (A): Star (TV channel) has decided to start a
Event (B): The LTTE was banned by the Centre in new news channel of Hindi.
the aftermath of the assassination of the former prime Event (B): Aaj Tak (TV channel) has decided to start
minister, Rajiv Gandhi, at Sriperumbudur in 1991. a news channel of English.
89. Event (A): In an effort to further reduce its manpower, 100.Event ( A): Major player of chocolate industr y,
public sector Steel Authority of India (SAIL) has Cadbury’s, is reportedly planning to reduce the cocoa
announced a new voluntary retirement scheme for even further and add more wafer — presumably to
its employees from January 15. offset an increase in raw material cost.
Event (B): SAIL has a plan to reduce its manpower Event (B): Ivory Coast, the world’s largest producer
to around one lakh in the next 3-4 years. of cocoa, has been in the grip of a bitter civil war.
90. Event (A): Recently the government has established 101. Event (A): The government has decided to reduce
a bamboo innovation studio and allied workshops in its stake in nationalised banks to 33 per cent.
Agartala. Event (B): The bank employees have gone for one-day

KUNDAN
Event (B): Ther e has been wit nessed a r adical strike against the privatisation of public sector banks.
change in the choices of people using bamboo as an 102.Event (A): The government has decided to roll back
alternative to timber and steel. the hike in the prices of cooking gas and kerosene.
91. Event (A): The Congress government in Uttaranchal Event (B): Some ministers had resigned in protest
marked the completion of its first 100 days on the against the hike in prices of cooking gas and other
first day of the ongoing week. petroleum products.
Event (B): The Congress party organised a fete in 103. Event (A): The Supreme Court has ordered to close
Uttaranchal’s capital recently. all industrial units in the residential areas in Delhi.
92. Event ( A) : Gov er nment has emphasised on the Event (B): A mob of about 1,500 people went on the
pr ogr ammes aimed at eliminating pov er t y and rampage and the police had to use teargas to disperse
illiteracy with new zeal and strong willpower. them and restore normalcy.
Event (B): Minister of state Mr X told the plenary 104. Event (A): The br igand Veerappan has r eleased
session of the International Labour Organisation in Rajkumar after 108 days of abduction.
Geneva on a day last week that India is following a Event (B): The state government ‘A’ has activated
proactive policy to eliminate child labour, which is the Special Task Force (STF) to arrest Veerappan.
closely linked to poverty and illiteracy. 105. Event (A): Cricketer ‘A’ has denied to talk with media
93. Event (A): There have been repeated complaints about persons.
malfunctioning electronic meters in the past months. Event (B): BCCI special commissioner K Madhavan
Event (B): Weights and measures department has has quizzed cricketer ‘A’ for nearly six hours for his
asked all meter-manufacturers to get their sample role in betting and match-fixing scam.
meters tested again from Electronic Regional Test 106. Event (A): A tr aveller f ound cockroaches in the
Laboratory. crevices of his seat and also in the toilets of India’s
94. Event (A): Science has made the world a global most prestigious train, the Rajdhani Express.
village. Event (B): He drew the attention of catering manager,
Event (B): Today a man can contact any person in Western Railway, to the insects.
the world within seconds. 107. Event (A): The police has arrested an ISI agent who
95. Event (A): Because of the proper distribution of gas had been spying in India for the past six years.
ovens the life of the women of rural areas has become Event (B): The police received an information about
more comfortable. an ISI agent having set up base at Delhi from Military
Intelligence.
218 Practice Book on Analytical Reasoning

108. Event (A): The Indian Board for Wildlife headed by 122. Event (A): State A faced an earthquake and suffered
the PM has not met since 1998. heavy damage due to fall of skyscrapers.
Event ( B): A London-based env ironment al Event (B): State A has decided to take stern action
investigation agency has brought to light the massive against t he builder s who hav e illegally made
killing of a hundred tigers during this year in India. apartments using sordid technique and low-grade
109. Event (A): The CM offered prayers daily for the safe construction material.
return of the abducted actor Rajkumar during the 123. Event (A): The sale of cars has dropped in the state
107-day period. X since last Monday.
Event (B): The abductor and brigand Veerappan has Event (B): The state ‘X’ has suspended registration
released the actor Rajkumar after the 107-day period. of cars which do not conform to the emission norms
110. Event (A): The BCCI has decided to ban and suspend stipulated recently.
some players who were accused of match-fixing. 124. Event (A): Indian government declared the postal
Event (B): Delhi Police had tapped the conversation strike illegal and invoked the Essential Services
between a bookie and some players. Maintenance Act to deal with the strike.
111. Event (A): Many fashionable commercial centres have Event (B): Employees of postal department started a
mushroomed in Delhi in the last few years. strike for their demands.
Event (B): Connaught Place, in the heart of Delhi, 125. Event (A): Garment Industry has decided to protest
has steadily lost its glory as the capital’s business against excise duty levied on readymade garments.
and entertainment hub. Event (B): Govt decided to levy 16 per cent excise
112. Event (A): 13 Royal Bengal tigers were found dead duty on readymade garments in the new budget.
in Nandan Kanan Zoo near Bhubaneshwar last 126. Event (A): Prices of gold have gone up in the local
month. market.

K
Event (B): A tiger gave birth to a baby in its cage in Event (B): India has won several prizes in design of
Nandan Kanan Zoo near Bhubaneshwar yesterday. gold ornaments.
113. Event (A): The state ‘A’ government has announced 127. Event (A): Today, the Prime Ministers of country ‘P’
the commencement of the second phase of action and ‘Q’ have decided to take steps to improve the
against polluting industries. bilateral relations.
Event (B): The state ‘A’ government took action Event ( B): Next week a Commit t ee of For eign
against 27 polluting industries falling in the F- Ministers and Senior Officers of country ‘P’ and ‘Q’
category of the Master Plan in the first phase. will wor k out fur t her st eps t o improv e t he
114. Event (A): The CNG-run vehicle owners are paying relationship.
a heavy penalty f or their eco-f riendly and cost- 128. Event ( A): Recent ly t he prices of t he per sonal
effective decisions. computers (PCs) have come down
Event (B): The drivers wait in long queues at CNG Event (B): Some school children are showing keen
filling stations because of fuel shortage and low interest in learning computers.

KUNDAN
pressure. 129. Event (A): This year Bank ‘M’ has celebrated its silver
115. Event ( A): The cour t has or der ed t o pay a jubilee.
compensation of Rs 25000 to 10-year-old boy Inder. Event (B): More customers are getting attracted to
Event (B): The court has found Inder’s employer guilty the market branch of Bank M.
of subjecting him to cruelty and harassment. 130. Event (A): Recently the traffic jams on MG Road of
116. Event (A): There were riots in city ‘X’. city ‘Z’ are not only reduced but the traffic has also
Event (B): Curfew was imposed on city ‘X’. become manageable.
117. Event (A): The police control room received a tip-off Event (B): The flyover on MG Road of city ‘Z’ has
about a bomb in the Red Fort. recently been made operational and the number of
Event (B): Laskhkar-e-Taiba militants went on a traffic police personnel has been increased.
shooting spree. 131. Event (A): The interview panel has recommended 5
118. Event (A): ITC, a tobacco company, has decided to candidates for 3 vacancies which are to be filled in
withdraw from all sponsorship of sporting activities. immediately in Company Z.
Event (B): The Cent re has decided to br ing in Event (B): The 5 candidates have been asked to
legislation for banning sponsoring of sports and contact Company Z next week to know their result
cultural events by tobacco companies. and accordingly to collect appointment letters.
119. Event A: Sri Lanka has allocated Rs 63.39 b for 132. Event (A): The sensex crosses 4000-point mark in
defence. the stock exchange, the highest mark in last 12
Event (B): India has allocated Rs 130 b for its defence. weeks.
120.Event (A): The Indian cricket captain warned his Event (B): The government announces its credit
Austr alian counterpart not to speculat e on his policy for agriculture.
involvement in preparing pitches for the three-Test 133. Event (A): At last, yesterday night Shirin could get
series. the medicines prescribed by the doctor for her ailing
Event (B): Australian cricket team is coming to India father.
to play a Test series. Event (B): The doctor has conducted some tests on
121. Event (A): Many girls started taking part in beauty Shirin’s father who has been suffering from high
contests in several towns in India. fever.
Event (B): Indian girls succeeded in capturing all 134. Event (A): The sales of cars has dropped in the state
the titles of beauty contests, i.e. Miss World, Miss ‘X’ since last Monday.
Universe and Miss Asia. Event (B): The state ‘X’ has suspended registration
Cause and Effect 219

of cars which do not confirm to the emission norms 140. Event (A): The public transport company of city M
stipulated recently. has recently changed the routes of some roads to
135. Event (A): The revised entry fee for internationally facilitate travellers.
known Taj Mahal monument has been increased to Event (B): The public transport company of city M
Rs 50 and Rs 500 f or Indians and f oreigner s has recently cancelled some routes because there
respectively. were few commuters.
Event (B): There has been adequate increase in the 141. Event (A): We can get anything with money.
number of foreign tourists visiting India in view of Event (B): Today money is the most important.
the improved infrastructure facilities. 142. Event (A): Due to mechanisation the life of human
136. Event (A): The financial position of the Electricity beings is becoming more comfortable in urban areas.
Division of state XYZ has weakened and it has made Event (B): Life is becoming insecure in urban areas.
demand to the government for more subsidies. 143. Event (A): The government has decided recently to
Event (B): While the Electricity Division of state XYZ prov ide addit ional dear ness allowance t o it s
has revised the pay and perks of its employees, employees.
several subscribers and farmers have refused to pay Event (B): Consumer Price Index is increasing for
long pending dues. the last five months.
137. Event (A): Rickshaw, taxi, tempo and other means 144. Event (A): The children of younger generation do
of gener al communicat ion of cit y X ar e on an better in their studies.
indefinite strike since last week. Event (B): The parents of children now realise the
Event (B): The prices of vegetables and other food importance of education very well.
articles in city X have increased. 145. Event (A): There is considerable increase in the
139. Event (A): The state government ABC has proposed number of people having computers.

K
to start a project “alert citizen” to curb possible crimes Event ( B): Computer education is being made
and nab the criminals. compulsory in schools.
Event ( B): Public cooper ation is imper at iv e in
controlling crime.

Practice Exercise–2
Directions (Q. 1-66): Below in each question are B. The parents of the aspiring students launched a
given two statements (A) and (B). These statements severe agitation last year protesting against the
may be either independent causes or may be effects of high fees charged by the unaided institutions.
independent causes. One of these statements may be 4. A. The Pr ime Minist er has v isited the drought-
the eff ect of the other statement. Read both the af fect ed areas and pr omised gover nment

KUNDAN
statements and decide which of the following answer assistance to help the farmers.
choice correctly depicts the relationship between these B. A large number of farmers in the drought-affected
two statements. Mark answer areas have been suffering due to drought situation
1) if statement (A) is the cause and statement (B) is and are unable to feed their family.
its effect. 5. A. All the lakes supplying water to the city started
2) it statement (B) is the cause and statement (A) is overflowing at the end of the second month of
the effect. the monsoon.
3) if both the statements (A) and (B) are independent B. The normal life in the city has been disrupted
causes. quite a few times in the first two months of the
4) if both the statements (A) and (B) are effects of monsoon due to water-logging in various parts.
independent causes. 6. A. A large number of primary schools in the rural
5) if both the statements (A) and (B) are effects of areas are run by only one teacher.
some common cause. B. There have been huge dropouts from the primary
1. A. Major part of the sugarcane crop was affected by schools in rural areas.
pests, resulting into huge loss incurred by the 7. A. The police resorted to lathi charge to disperse
farmers in the state. the unruly mob from the civic headquarters.
B. The farmers in the state who were cultivating B. The civic administration has recently hiked the
sugarcane earlier have now switched over to property tax of the residential buildings by about
grapes’ cultivation this year. 30 per cent.
2. A. There has been a high increase in the incidents 8. A. The government has decided to allow private
of atrocities against women in the city during airline companies in India to operate to overseas
the past few months. destinations.
B. The police authority has been unable to nab the B. The national air carrier has increased its flights
culpr it s who ar e commit t ing crime against to overseas destinations.
women. 9. A. The prices of f oodgrains and other essential
3. A. The government has recently fixed the fees for commodit ies in t he open market have risen
professional courses of fered by t he unaided sharply during the past three months.
institutions which are much lower than the fees B. The political party in opposition has given a call
charged last year. f or gener al str ike t o pr otest against t he
government’s economic policy.
220 Practice Book on Analytical Reasoning

10. A. A large number of people have fallen sick after 2 2 . A. Most of the steel-manufacturing companies in the
consuming sweets from a particular shop in the country made considerable profit during the last
locality. financial year.
B. Major part of the locality is flooded and has B. Many Asian countries have been importing huge
become inaccessible to outsiders. quantities of steel from India.
11. A. The traffic police removed the signal post at the 23. A. Many seats in the private engineering colleges
intersection of two roads in a quiet locality. in the state have remained vacant this year.
B. Ther e hav e been many accident s at t he B. The government engineering colleges in the state
intersection involving vehicles moving at high could not accommodate all the students who
speed. sought admission this year.
12. A. The local steel company has taken over the task 24. A. The banks have decided to give advances to the
of development and maintenance of the civic roads priority sector at the rate of interest at par with
in the town. the corporate sector.
B. The local civic body requested the corporate bodies B. The percentage of bad loans given by the banks
to help them maintain the civic facilities. to the priority sector is very low compared to
13. A. The Govt has suspended several police officers corporate sector.
in the city. 25. A. The government of India has allowed the private
B. Five persons carrying huge quantity of illicit liquor air line companies t o oper at e on specified
were apprehended by the police. international routes.
14. A. Majority of the students in the college expressed B. There has been a considerable increase in the
their opinion against t he college authority’s flow of foreign tourists to India.
decision to break away from the university and 26. A. Govt -owned oil companies have r educed t he

K
become autonomous. prices of petroleum products wit h immediate
B. The univ er sit y aut hor ity has expr essed it s effect.
inabilit y to provide gr ants to it s constituent B. Govt has made an appeal to the public to reduce
colleges. consumption of petroleum products.
15. A. Huge tidal waves wrecked the vast coastline early 27. A. The police authority has recently caught a group
in the morning, killing thousands of people. of housebreakers.
B. Lar ge number of people gather ed along t he B. The citizens' groups in the locality have started
coastline to enjoy that spectacular view of sunrise. night vigil in the area.
16. A. Majority of the employees of the organisation 28. A. The Govt has decided to allow private universities
signed t he statement pr ot esting against t he to run their own courses after obtaining necessary
management’s personnel policy. approvals.
B. The management of the organisation has decided B. There has been a continuous decrease in the
that all those employees who are above 58 years number of students enrolled for various courses

KUNDAN
of age may opt for voluntary retirement scheme. offered by the Govt-run universities.
17. A. The public sector telecom service provider reduced 29. A. The wholesale grain market is flooded with fresh
the monthly rental substantially with immediate stock.
effect. B. The Govt has recently increased the procurement
B. All the private sector telecom service providers price of grains.
have reduced their charges last week. 30. A. The school authority has asked the X Std students
18. A. The largest oil company in the private sector to attend special classes to be conducted on
decided to raise its supply of crude oil to the Sundays.
government-controlled refineries with immediate B. The parents of the X Std students have withdrawn
effect. t heir war ds f rom at t ending priv ate t uit ions
B. The government has recently notified all the conducted on Sundays.
refineries under its control to reduce the refining 31. A. The v illager s hav e decided t o boycot t t he
capacity in future months. forthcoming assembly elections.
19. A. The government has decided to make all the B. The state government has recently revised the
information related to primary education available electoral roll.
to the general public. 32. A. The school authority has decided to increase
B. In the past, the general public did not have access tuition fees by 30 per cent from the next academic
t o all t hese inf or mat ion r elat ed t o pr imar y year.
education available with the government. B. The Govt has urged the local public to enrol all
2 0 . A. The local traders’ association urged all its members their children to schools in the area.
to close down their shops for a day to protest 33. A. A recent tiger census in the tiger reserve in the
against the government’s new tax policy. state has reported significant reduction over the
B. Many shopkeepers decided to close down their last census.
shops for the day and gave a day off to their B. The Govt has initiated an enquiry to ascertain
employees. the facts relating to tiger population in the state.
21. A. The state govt has now decided to increase the 34. A. Many shops in the locality r emained closed
stamp duty on house purchases with immediate throughout the day.
effect. B. Many offices in the locality closed during the
B. The r eal est ate prices have decr eased day.
considerably during the last few months. 35. A. Majority of the residents of the housing society
Cause and Effect 221

participated in the dinner hosted by one of the performance of students from college ‘X’ was better
members of the society. than that of students from college ‘Y’.
B. Most of the people living in the housing society B. Majority of the students depend upon coaching
invite other members for the functions at their classes for university examinations.
house. 50. A. The Gov er nment of state ‘X’ decided t o ban
36. A. The train services in the suburban areas of the working of women in night shifts and also in
city were disrupted for four hours. late evening hours.
B. The overhead electrical wire snapped between B. The per cent age of wor king women has a
two stations in suburban area of the city. significant rise in the last one decade.
37. A. State Government has ordered immediate ban on 51. A. Fr equent r obberies in jeweller y shops wer e
airing of certain movie channels on television. recorded in distant suburbs of the city.
B. A few social activists have come together and B. Shop owners in the city and suburbs demanded
demanded ban on telecasting ‘Adult’ movies on improvement in security situation from the police
television. authorities.
38. A. Employment scenar io in t he count r y has 52. A. Party ‘X’ won clear majority in the recently held
remarkably improved recently. state assembly elections.
B. The number of pr ospect ive job-seekers going B. Of late, there was unrest in public and also among
abroad has increased recently. the members of the ruling party of the state.
39. A. Government has tightened security checks at all 53. A. Staff members of the university decided to go on
important places and also at various public places. strike in protest during the examinations.
B. Incidences of terrorist attacks are increasing day B. The univ er sit y administr at ion made all t he
by day. arrangements for smooth conduct of examination

K
40. A. The High Court has fixed a time limit for repairing with the help of outsiders.
all the roads in the city. 54. A. In t he univer sit y examinat ion, ov erall
B. Road Development Authorities in the city are performance of students from college ‘X’ was better
carrying out road repair work on urgent basis. than that of students from college ‘Y’.
41. A. There is an outbreak of several epidemics in the B. Majority of the students depend upon coaching
country. classes for university examinations.
B. It was a worst flood situation ever experienced 55. A. The Gov er nment of state ‘X’ decided t o ban
in the past in most parts of the country. working of women in night shifts and also in
42. A. The prices of pet roleum product s dr opped late evening hours.
marginally last week. B. The per cent age of wor king women has a
B. The State Govt reduced the tax on petroleum significant rise in the last one decade.
products last week. 56. A. Fr equent r obberies in jeweller y shops wer e
43. A. Majority of the citizens in the locality belong to recorded in distant suburbs of the city.
higher income group. B. Shop owners in the city and suburbs demanded

KUNDAN
B. The sales in t he local super market ar e improvement in security situation from the police
compar at iv ely much higher t han in other authorities.
localities. 57. A. The State Education Board has decided to do away
44. A. A major fire destroyed part of the oil refinery with pr eparing mer it lists for SSC and HSC
owned by a private company. examinations.
B. Govt has decided t o incr ease t he supply t o B. A large number of students scored very high marks
retailers from the public sector refineries. in the recently held SSC examination.
45. A. The Govt medical college has decided to increase 58. A. The State Government decided to grant permission
the number of seats in undergraduate course from for opening more junior colleges in the state.
the next academic session. B. Percentage of qualif ied st udent s in SSC
B. The Govt has decided to withdraw its grant from examination was higher this year compared to
all the Gov t medical colleges f r om t he next the past few years.
academic session. 59. A. Increase in rainfall and rising flood situations
46. A. Every year a large number of people spend their are regular phenomena for past few years.
vacation in various tourist destinations within B. People avoid going out in heavy rains.
the country. 60. A. The health department has advised people to drink
B. Every year large number of people spend their boiled and filtered water and maintain hygiene
vacation in various tourist destinations outside during the monsoon.
the country. B. The health department has instructed the civic
47. A. Party ‘X’ won clear majority in the recently held hospitals to equip themselves with adequate stock
state assembly elections. of medicines during monsoon.
B. Of late, there was unrest in public and also among 61. A. The Government has made it compulsory to wear
the members of the ruling party of the state. a helmet for the riders of two-wheelers.
48. A. Staff members of the university decided to go on B. The number of cases of road accident involving
strike in protest during the examinations. two-wheelers has been increasing every year.
B. The univ er sit y administr at ion made all t he 62. A. Parents in the locality decided to stop sending
arrangements for smooth conduct of examination their children to school by private vehicles.
with the help of outsiders. B. A major accident of a private van carrying school
49. A. In t he univer sit y examinat ion, ov erall children led to deaths of a few and injuries for
many children.
222 Practice Book on Analytical Reasoning

63. A. The city observed the lowest temperature of the 78. A. The share prices are touching an all-time low.
last decade accompanied by heavy fog during the B. Most of the organizations have been grounding
week. or terminating employees and undergoing cost-
B. Most of the flights from the city were indefinitely cutting exercises wherever possible.
delayed causing panic among the passengers. 79. A. A substantial increase in unhealthy competition
64. A. In the past few years the job market has improved has been observed among the students.
for the professionally qualified youth. B. A rise of 23% is reported every year in the cases
B. Many youth are not able to get jobs up to their of suicide after declaration of grade 10th and 12th
expectations. examination results.
65. A. During peak hours roads are overcrowded with 80. A. The glaciers at the poles of the earth are melting
vehicles causing traffic jams in most parts of the at a fast rate.
city. B. In recent times there has been a substantial
B. Many companies are planning to launch low- increase in the incidents of earthquakes and
priced vehicles. volcanic eruptions.
66. A. During evening hours roads are overcrowded with 81. A. Though mobile phones find a good number of
vehicles causing very slow movement of vehicles. users in rural India, computers and Internet still
B. Pollution lev el in t he air has subst ant ially remain a distant dream.
increased in the recent past. B. In the recent past there has been a large-scale
67. A. This year the cut-off percentages for admission migration from the rural parts of India to the urban
to junior colleges have increased over the last sectors.
year. 82. A. There has been a continuous increase in average
B. This year performance of students in Xth final temperature during winter in many parts of the

K
exam was considerably higher than the previous country over the past few years.
year. B. There has been significant changes in the wind
68. A. The conditions of most of the national highways pattern across the country over the last few years.
are very bad. 83. A. The conditions of all the major roads in the city
B. Govt has now sanctioned a huge amount of money have deteriorated causing hardship to motorists.
to maintain the national highways. B. The municipal authority has sanctioned significant
69. A. Many students of the local school have failed in amount to repair all the major roads in the city.
English Language paper in t he annual 84. A. The BPO sector has laid off a large number of
examination. employees in the recent months.
B. Many students of the local school have failed in B. Very few projects are now being outsourced to
Mathematics paper in the annual examination. BPO sector.
70. A. Rain and thunder showers bashed the city during 85. A. Ther e has been shar p decline in sales of
the past three days. passenger cars during the last few months.

KUNDAN
B. Many people stayed indoor during the past three B. Many finance companies have announced
days. attractive schemes of car loans with moderate
71. A. There has been a considerable increase in the interest rate.
sale of fat-free food articles. 86. A. All the airlines companies in India have increased
B. Now people have become more conscious about the air fares in all routes with immediate effect.
their health condition and food habits. B. There has been substantial reduction in aviation
72. A. There have been heavy rains in the catchment fuel prices in India during the past few weeks.
area of the lakes supplying drinking water to the 87. A. Computer education has been made compulsory
city. for all the classes by many schools.
B. The municipal aut horit y has suspended the B. The current job market prefers computer-literate
proposed cut in water supply to the city. workforce.
73. A. Many pilgrims used Govt transport to travel to 88. A. The standard of education in evening colleges of
the holy shrine. the State has been deteriorating.
B. The cost of travel by private transport is very high. B. The standard of school education has been fast
74. A. The prices of vegetables have increased substant- deteriorating in the State.
ially during the past few weeks. 89. A. All domestic airlines increased the fares in all
B. Consumer price index at the end of the previous sectors with immediate effect.
week increased by 2 per cent. B. Railways increased the fare of all its classes with
75. A. Many anti-social elements have been caught by immediate effect.
the police from the locality. 90. A. The prices of f r uit s and vegetables f ell
B. Many people in the locality have been detained substantially over the last few days.
by the police for questioning. B. The quality of fruits and vegetables improved
76. A. Many employees of the company proceeded on a considerably over the last few days.
day’s leave on Friday. 91. A. Recent floods in the area changed the nutritional
B. Bot h Thursday and Sat ur day wer e declared contents of the soil.
holiday by the company. B. Farmers in the area switched over to cultivating
77. A. Many schools have banned the sale of fast food rice instead of wheat.
in their premises. 92. A. The residents report ed of increased criminal
B. Obesity in youngsters has been linked to their activities in the area to the local police station.
poor eating habits. B. Many criminals were arrested by searching the
residence of the suspected individuals.
Cause and Effect 223

93. A. Govt has awarded a high-stake reward scheme B. Customers have been complaining about poor
for such persons as may provide any information services in the bank’s branches.
about the suspect. 108. A. The production of pulses has dropped for the
B. Four members of a family were brutally murdered third consecutive year.
by unidentified gunmen. B. India has decided to import pulses this year.
94. A. There have been sporadic events of stone-pelting 109. A. Budgetary allocation for building a better railway
throughout the day in the affected areas of the network, eg constructing new railway lines, has
city. increased.
B. Many wounded people were brought to the nearby B. There has been a substantial drop in the number
hospitals from the affected areas of the city. of passengers opting for air travel.
95. A. Many people left from the city for their native 110. A. Indian citizens are willing to incur the cost of
places during the summer months. using environment-friendly technology.
B. Many tourists gathered in the city during summer B. Many countries are taking steps to cut their carbon
months. emissions.
96. A. All the schools declared holiday on the next day 111. A. The government has amended tax laws to boost
of the major festival. exports.
B. All the colleges declared holiday on the next day B. The export sect or has been passing through
of the major festival. difficult times due to heavy tax burdens.
97. A. Many elderly people are continuously harassed 112. A. The Gov t has mar ginally incr eased t he
by the youngsters in the locality. procurement price of wheat for the current crop.
B. Many children living in the locality play till late B. The current wheat crop is expected to be twenty
in the evening. per cent more than the previous wheat crop.

K
98. A. The state govt has decided to change the syllabus 113. A. The braking system of the tourist bus carrying
of mathematics for Std IX from the next academic 40 passengers failed while negotiating a stiff
year. climb on a hilly road.
B. Many students from the state could not secure B. The tourist bus fell into the gorge killing at least
admission to the colleges of their choice. ten passengers and seriously injuring all the
99. A. Majority of the employees of the manufacturing remaining.
company received a hefty bonus at the end of the 114. A. The stat e gov t has decided to boost English
current financial year. language education in all the schools from the
B. The manuf act ur ing company has made next academic year.
considerable profit in the current financial year. B. The lev el of English language of t he school
100. A. The municipal authority decided to carry out repair students of the State is comparatively lower than
work of the pipeline under the main arterial road that of the neighbouring states.
of the city. 115. A. The municipal authority demolished the tea stall
B. Vehicular movement has been diverted through located on the footpath on the busy road.

KUNDAN
alternate roads for a period of fifteen days. B. A large number of people have been taking their
101. A. There is a significant drop in the number of evening tea at the tea stall located on the footpath
people travelling by air during the last quarter. on the main road, blocking pedestrian movement.
B. There is a significant drop in the number of 116. A. Majority of the students left the local school as
people travelling by long-distance trains during the school building was in a dilapidated condition.
the last quarter. B. The school authority decided to close down the
102. A. Govt has decided to distribute part of the foodgrain school immediately and shift the r emaining
stock through Public Distribution System to people students to a make-shift school.
below poverty line. 117. A. State Govt has ordered immediate ban on airing
B. There has been bumper kharif crop for the last of certain movie channels on television.
two seasons. B. A few social activists have come together and
103. A. Most of the students enrolled themselves for the demanded ban on telecasting ‘Adult’ movies on
educational tour scheduled for next month. television.
B. The school authority cancelled the educational 118. A. Employment scenar io in t he count r y has
tour scheduled for next month. remarkably improved recently.
104. A. The prices of fruits have dropped substantially B. The number of pr ospect ive job-seekers going
during the last few days. abroad has increased recently.
B. The pr ices of foodgr ains have incr eased 119. A. Government has tightened security checks at all
substantially during the last few days. important places and also at various public places.
105. A. The road traffic between the two towns in the B. Incidences of terrorist attacks are increasing day
state has been disrupted since last week. by day.
B. The rail traffic between the two towns in the 120.A. The high court has fixed a time limit for repairing
state has been disrupted since last week. all the roads in the city.
106. A. Heavy showers are expected in the city area during B. Road development authorities in the city are
the next forty-eight hours. carrying out road repair work on an urgent basis.
B. The inter-club cricket tournament scheduled for 121. A. There is an outbreak of several epidemics in the
the week was called off. country.
107. A. The bank has provided a link on its website to B. There was a worst flood situation ever experienced
obtain feedback from customers. in the past in most parts of the country.
224 Practice Book on Analytical Reasoning

122. A. The government of India has allowed the private 127. A. Ther e is an unpr ecedent ed increase in t he
air line companies t o oper at e on specified number of young unemployed in comparison to
international routes. the previous year.
B. There has been a considerable increase in the B. A lar ge number of candidat es submitt ed
flow of foreign tourists to India. applications against an advertisement for the post
123. A. Many seats in the private engineering colleges of manager issued by a bank.
in the state have remained vacant this year. 128. A. The pr ices of vegetables have incr eased
B. The government engineering colleges in the state considerably during this summer.
could not accommodate all the students who B. There is tremendous increase in the temperature
sought admission this year. during this summer, thereby damaging crops
124. A. The banks have decided to give advances to the greatly.
priority sector at the rate of interest at par with 129. A. Heav y downpour with high-velocit y wind is
the corporate sector. probable in the coastal areas in next twenty four
B. The percentage of bad loans given by the banks hours.
to the priority sector is very low as compared to B. A soap manufacturing company increased its
the corporate sector. production by more than 100 in the last month.
125. A. The state government has now decided to increase 130. A. There has been considerable reduction in the
t he st amp dut y on house purchases wit h number of people affected by water-borne diseases
immediate effect. in City A during this rainy season.
B. The r eal est ate prices have decr eased B. The government opened four new civil hospitals
considerably during the last few months. in City A at the beginning of the year.
126. A. Most of the steel manufacturing companies in 131. A. There is increase in water level of all the water

K
the country have made considerable profit during tanks supplying drinking water to the city during
the last fiscal year. the last fortnight.
B. Many Asian countries have been importing huge B. Most of the trains were cancelled last week due
quantities of steel from India. to waterlogging on the tracks.

Practice Exercise–3
1. Cause: All the major rivers in the state have been 1) All t he airline companies had t hr eatened t o
flowing way over the danger level for the past few suspend their services during peak hours.
weeks. Which of the following is/are possible effect(s) 2) The Govt has increased its tax for peak time flights.
of the above cause? 3) The aircrafts are routinely put on hold over the
(A) Many villages situated near the riverbanks are airports while landing during peak time, causing

KUNDAN
submerged, forcing the residents to flee. extra fuel consumption.
(B) Govt has decided to provide alternate shelter to 4) The airline companies can now charge unlimited
all the affected villagers residing near the river additional charge for peak time flights.
banks. 5) None of these
(C) The entire state has been put on high flood alert. 4. Cause: The cement manufacturing companies have
1) Only (A) 2) Only (A) and (B) increased the price of cement by about fifteen per
3) Only (B) and (C) 4) All (A), (B) and (C) cent with immediate effect.
5) None of these Which of the following is/are possible effect(s) of
2. Effect: This year majority of the final year students the above cause?
of the management institute have opted for finance (A) Gov t will dir ect the cement manuf act uring
speciali- sation. companies to reduce the price increase to five
Which of the following can be a probable cause of per cent.
the above effect? (B) The prices of residential flats and commercial
1) Last year most of the st udent s wit h HR companies will see an upward trend.
specialisat ion got better job of fers than ot her (C) The construction companies may stop all ongoing
specialisations. construction projects with immediate effect.
2) The management institute of fer s only finance 1) Only (B)
specialization to its final year students. 2) Only (A)
3) Last year the students with finance specialisation 3) Only (C)
bagged most of the lucrative offers vis-a-vis students 4) Only (B) and (C)
with other specialisations. 5) None of these
4) The management institute has recently started its 5. Effect: There has been unprecedented increase in
finance specialisation in addition to Marketing and t he number of inst it utions t r aining f or spoken
HR being offered earlier. phonetic English in all the major cities of India during
5) None of these the last few years.
3. Effect: Govt has allowed all the airlines to charge Which of the following can be a probable cause of
additional amount as peak time congestion charges the above effect?
for the flights landing between 6.00 a.m. and 10.00 1) Many parents want their children to speak fluent
a.m. English.
Which of the following is a probable cause of the 2) Various activities are being outsourced to India by
above effect? many European and North American countries.
Cause and Effect 225

3) English is no longer being taught in the schools 10. Eff ect: Majority of the employees of t he ailing
and colleges in India organisation opted for voluntary retirement scheme
4) India has highest number of English-speaking and left the organisation with all their retirement
educated youth compared to any other country. benefits within a fortnight of launching the scheme.
5) None of these Which of the following can be a probable cause of
6. Effect: At least 20 school children were seriously the above effect ?
injured while going for a school picnic during the 1) The company has been making huge losses for the
weekend. past five years and is unable to pay salary to its
Which of the following can be a probable cause of employees in time.
the above effect ? 2) The management of t he company made huge
1) The teacher accompanying the school children fell personal gains through unlawful activities.
ill during the journey. 3) One of t he compet it ors of t he company went
2) The bus in which the children were travelling bankrupt last year.
met with an accident while taking turn on the 4) The company owns large tracts of land in the state
main highway. which will fetch huge sum to its owners.
3) The driver of the bus in which the children were 5) None of these
travelling did not report after the break at the 11. Cause: The Govt has recently increased its taxes on
halting place on their journey. petrol and diesel by about 10 per cent.
4) The school authority banned all school picnics for Which of the following can be a possible effect of
the next six months with immediate effect. the above cause?
5) None of these 1) The petroleum companies will reduce the prices
7. Cause: Gov t has r ecent ly decided t o hike t he of petrol and diesel by about 10 per cent.

K
procurement price of paddy for the rabi crops. 2) The petroleum companies will increase the prices
Which of the following will be a possible effect of of petrol and diesel by about 10 per cent.
the above cause? 3) The petroleum companies will increase the prices
1) The farmers may be encouraged to cultivate paddy of petrol and diesel by about 5 per cent.
for the rabi season. 4) The petrol pumps will stop selling petrol and diesel
2) The farmers may switch over to other cash crops till the taxes are rolled back by the Govt.
in their paddy fields. 5) None of these
3) There was a drop in production of paddy during 12. Effect: The temple at the religious site wears a
Kharif season. deserted look with the number of devotees trickling
4) Govt may not increase the procurement price of down.
paddy during the next Kharif season. Which of the following can be a possible cause of the
5) Govt will buy paddy from the open market during above effect?
the next few months. 1) A structural engineer had visited the temple a
8. Cause: A severe cyclonic storm swept away most part month back and had declared the structure unsafe.

KUNDAN
of the state during the last two days. 2) The temple is facing a drastic depletion of its funds
Which of the following cannot be a possible effect which had accumulated over the years due to
of the above cause ? offerings made by devotees.
1) Heavy rainfall was reported in most part of the 3) The local corporation decided to donate a huge
state during the last two days. amount of money to the temple for its renovation.
2) Many people were rendered homeless as their 4) The village housing the religious site has qualified
houses were flown away. priests to perform religious ceremonies.
3) The communication system of t he stat e was 5) A famous actor recently visited the temple and paid
severely affected and continues to be out of gear. his respects to the deity.
4) Govt has ordered that all the offices and schools 13. Effect: As a step to regulate private hospitals, the
should be kept open. state health department is framing rules to ensure
5) All are possible effects. all such hospitals are registered with it.
9. Eff ect: The prices of pet roleum product s hav e Which of the following can be a possible cause of
increased by about twenty per cent in the past two the above statement?
months. 1) The department realized that private hospitals
Which of the following can be a probable cause of the charge much less for treatment as compared to
above effect ? government hospitals.
1) The prices of foodgrains and vegetables have shot 2) Government-run hospitals do not maintain the
up by more than thirty per cent. same standards as private hospitals.
2) The tr uck owners’ association has decided to 3) The department realized that several hospitals were
increase their rent by about 20 per cent with rejecting cases stating lack of infrastructure.
immediate effect. 4) Apart from the number of doctors, nurses and beds,
3) The prices of crude oil in the international market the kind of procedure a hospital can carry out based
have increased considerably during the past few on its infrastructure will also be registered and
weeks. detailed.
4) People have decided to demonstrate against the 5) Pr iv at e hospit als not regist er ing wit h t he
Govt’s apathy towards rise in prices of essential department shall be forced to do so and will have
commodities. to pay hefty penalties.
5) None of these
226 Practice Book on Analytical Reasoning

Answers and explanations


Practice Exercise–1
1. 1; The CBI has arrested him because of Event (B). political level. But it can’t be considered a direct
2 . 1; Because of Event (B) India has lodged his protest. consequence of the failure of the ministerial talks.
Hence A is the effect of the cause B. The environment for such talks has been built
3. 3; Death of 400 people is an effect. But the cause is over the years.
not given. It may be accident, bomb blast, or food- 25. 2; The security arrangements were made to check
poisoning, murder in the process of robbery or the high-profile criminal from escaping.
anything 26. 4; The launching of the new TV channel can only
4. 3; In Event (B) it is not clearly mentioned whether be one of the causes of formulating the new policy.
this leaked water again enters the pipeline or 27. 1; The flop film must have forced the actress to take
not. Because drinking dirty or impure water may a fresh look at the roles she played.
be the cause of jaundice. But only accumulation 28. 5; Here A may be the cause of B or vice versa.
of water is not related with jaundice. 29. 5; The events could take place either ways.
5. 1; The 24-hour telecast is the immediate cause of 30. 4; Ev ent A may be one of the reasons but not
the warning. Because telecast in the wee hours necessarily the principal one.
will be available t hr ough sat ellite, not 31. 1; Clearly, the High Court order has come as a
terrestrially. consequence of the numerous deaths.

K
6. 1; As educational institutions had violated the court 32. 1; The name of the players are there; so CBI has
directions so Delhi high court issued show cause called them for interrogation.
notices to these schools. 33. 2; A boy committed suicide after the result was
7. 4; Running into the kitchen is an effect. Several published; so ‘A’ is the cause.
causes are possible. But A is not the immediate 34. 2; Rain after scorching heat is the cause of pleasant
cause. weather. Hence A is the cause and B is its effect.
8. 2; Making an apology is an effect of event (A). 35. 4; As the assets of one MLA was found more than
9. 4; Increasing the number of red lights is an effect. his income, so the CM has ordered all the MLAs
It may be because of heavy traffic, because of t o show t heir asset s. But bot h may be
modernisation and computerisation.. . But A is independent incidents. It may be possible that it
not the cause. was in the election menefesto.
10. 1; Power problems are definitely the effect. And here 36. 2; Event A is the cause and B is its effect. Because
event (B) looks a reasonable cause behind it. doctors were on strike, there was no one to attend

KUNDAN
11. 5; It can’t be derived from these two sentences that the patient.
Elizabeth is Shekhar Kapoor’s film. Both (A) and 37. 3; A is definitely the effect of some cause but B may
(B) are effects of some causes. Because of the not be its immediate and principal cause. Why
phrase The good news is, (A) is an effect because did only two of them resign?
from the phrase we can conclude that there were 38. 1; The imposition of the curfew is clearly as direct
some hindrances in its coming. Several causes consequence of the macabre incident.
are possible. 39. 4; Event B is an effect but A may not be its principal
12. 2; The revision in fare is definitely because of (A). cause.
13. 3; Event (A) is an effect but (B) is not the principal 40. 2; Event A is the principal cause. Due to this cause
cause. Hike in diesel prices is the cause behind it. Supreme Court has directed the hospitals to
14. 5; No r elat ion between Amet hi and Bellar y dispose the medical waste.
parliamentary constituencies is given. 41. 1; The govt. has sold 74% of Modern Food Ind. Ltd.
15. 5; Again, the two sentences are independent. shares to Hindustan Lever. Because of this the
16. 1; Because of his announcement he has been employees have moved the court. Event B is the
expelled. cause and A the effect.
17. 2; Andrew talked so after reading the newspaper. 42. 4; Event B is an effect but it is not necessary that
18. 1; Because of the project they have managed to event A is the cause behind it. It is not known
transform the living conditions near their homes. who were in that committee.
19. 1; Because of the term second and third, we can 43. 1; The hawkers was driven to commit suicide which
establish the relation between the two sentences. is a violation of human rights and that is why
2 0 . 2; Because of the desperation for another child, Rani NHRC has ordered for compensation So, event B
kidnapped the child. is the cause and event A is its effect.
21. 3; ‘B’ may be cause, but several other causes may be 44. 1; B is the cause against which the opposition are
possible. Hence ‘B’ is not t he pr incipal and protesting. Hence A is its effect.
immediate cause. 45. 5; Both A and B are effect of some unknown reason.
2 2 . 1; It is osur common experience that such reshuffles 46. 1; To enhance the standard of education in evening
often take place when a new govt come in. law colleges the BCI has ordered to close down
23. 2; Additional taxation leads to greater revenues these colleges from the coming academic session.
which in turn lead to reduction of fiscal deficit. Hence (B) is the cause and (A) is its effect.
24. 4; Interaction between the two countries at the 47. 2; The qualified use of loudspeakers is a consequence
popular level is a fallout of failure of talks at the of the rules framed by the govt.
Cause and Effect 227

48. 1; The ban on traffic on Jail Road is a result of the given to IHF is due to the harassment Indian
diversions made. professionals were meted out by the Malaysian
49. 3; Having conducted N-tests could be the reason for police.
signing CTBT. But the major cause would be 74. 2; Special f light t o Johannesbur g is aimed at
something else: international opinion. carrying cricket lovers especially. Hence event
50. 5; Note that the two are different films. (A) is the immediate and principal cause and Event
51. 1; The govt discision has determined the conciliatory (B) is its effect.
approach of the Indian companies. 75. 3; Event (B) is the cause behind the effect shown
52. 2; Only when the demand was made could t he in the event (A) but not immediate and principal.
proposal have gone to the govt. 76. 4; Event (A) is the cause behind the effect shown
53. 5; W e can’t det er mine t he cause-eff ect chain. in event (B) but it is not the immediate and
“Looking for a foreign coach” could be independent principal cause.
of the resignation. 77. 5; The two statements have no correlation with each
54. 4; The large sales may be a consequence of media other.
hype which surrounded the work because of the 78. 2; Event (A) is the immediate and principal cause
high advance. But there is no direct connection. and event (B) is its effect. Note that the bank rate
Moreover, after a certain extent, works sell purely is a reference rate also. If the bank rate is raised
on the basis of their intrinsic merit. both the deposit and lending rates are raised.
55. 3; It is true that if the notes had not gone out of Similarly, when the bank rate is reduced both
circulation, there might have been no need of the deposit and lending rates are reduced.
their re-print. But much time has elapsed and a 79. 2; Event (A) is the immediate and principal cause
direct causal relationship can’t be established. and event (B) is its effect. Failure of independent

K
56. 2; B is an exercise in cutting expenditure, which filmmakers has compelled the NGO to step in by
has become imperative given the widening fiscal setting up a national distribution centre for their
deficit. films.
57. 5; Even a remote connection can’t be established 80. 5; The two are independent events contradicting
with any bit of certainty. each other.
58. 1; Since the subsidy has been reduced, t he 81. 3; Event (A) is the effect but event (B) is not its
institutions will have to compensate for it by immediate and principal cause because to get the
recovering a large portion from the students. abov e out come pr oper use of r ich mineral
59. 1; W hen t he result s wer e declar ed, it was the resources is equally important.
father’s curiosity that led him to the website. 82. 2; It is the parliament’s nod to Tobacco Control Bill,
60. 2; The inquiry is a logical consequence in case of a which gave the non-smokers a new horizon.
mysterious death. 83. 2; Event (B) is the ef f ect and event (A) is the
61. 4; The arrival o the monsoon is certainly an effect immediate and principal cause behind it. The
but its prediction can’t be the cause. move t aken by the Food and Adulter ation

KUNDAN
62. 1; Once China becomes a member of the WTO, the Department is the result of the claim made by
US won’t be able to discriminate against it. So the people.
the EU agreement must have led the US to prepare 84. 2; Once again, ev ent (A) is t he immediat e and
in advance. principal cause of event (B). The demand sought
63. 5; The two crises are totally unrelated. While in by the parliamentarians is due to the report
Sierra Leone the Indian soldiers are working revealed by the NGO.
under the aegis of the UN, a military intervention 85. 4; The new security measures have been taken not
in Sri Lanka would involve coming in direct touch. only due to the attack on Parliament in Dec 2001
64. 1; Event (A) is the effect and the immediate and but also to ensure that the Parliament be more
principal cause behind it is the rampant situation saf e for Par liament ar ians, aiming at the
in Nigeria explained in Event (B). importance of this section of people.
65. 1; The effect described in event (A) is the outcome 86. 1; The estimated loss, owing to the drought, is the
of the request of the parliamentary committee. immediate and principal cause behind the change
66. 3; Event (A) is the effect of Event (B) but Event (B) is in the forecast of the different bodies.
not the immediate and principal cause behind it. 87. 5; Note that Event (B) is a non-event. An event is
67. 5; There is no cause-effect relationship. when something happens, not when something
68. 1; Event (A) is the effect and Event (B) is the factor does not happen.
responsible for it. Hence, your answer is either 88. 2
1) or 3). Now, check if Event (B) is the immediate 89. 3; Here (A) is the effect. Rejection came from the
and principal cause behind it. Since your answer government because the government considers
is yes, your answer is 1). LTTE a terrorist organisation. (B) explains the
69. 4; Event (B) is the effect and Event (A) is one of the nature of the organisat ion. But it is not the
factors responsible for it. Hence, Event (A) is not principal and immediate cause.
the immediate and principal cause. 90. 1; (A) is the effect because VRS has been aimed at
70. 2; Event (B) is the effect and Event (A) is the main the plan to reduce man power and it is t he
factor responsible for the effect. principal cause behind it.
71. 3; Event (A) is the effect and Event (B) is one of the 91. 1; By an intuitive look it is clear that ‘A’ is the effect
factors responsible for the effect. and ‘B’ is the cause. Hence, we reject options 2),
72. 5; There is no cause-effect relationship. 4) and 5). Now check: Is event (B) immediate and
73. 1; Here event (A) is the effect because the advice principal cause? You get ‘yes’ as your answer.
228 Practice Book on Analytical Reasoning

Hence the correct answer is 1. may be its cause but it is not the immediate and
92. 2; Here ‘A’ is the immediate principal cause and ‘B’ principal cause.
is its effect. 113. 5; Both events are independent of each other.
93. 5; The two events have no causative link with each 114. 1; The second phase is possible only after the first
other. phase.
94. 2; Event (B) is in a form of course of action. And 115. 1; Because of the shortage of CNG filling stations
cause of its requirement is event (A). Hence event the owners of CNG-run vehicles are suffering.
(A) is its immediate and principal cause. 116. 1; Because of (B) the court has ordered so.
95. 5; It is very difficult to determine which is the cause 117. 2; When such riots take place, curfew is a direct
and which the effect. Science has made contact consequence.
easy. And easy contact has made the world a 118. 5; Both are independent events.
global village. 119. 1; Because of (B), ITC has decided so.
96. 5; The two events can’t be connected. 120.5; Both are independent events.
97. 1; Massive debt swap operation has been initiated 121. 3; There whould be no such speculation on the part
due to the menace of the government’s current of the Australian caption if his team were not
year’s gross fiscal deficit. visiting India in the first place. So the latter is a
98. 3; Subscribers of Hutch and Airtel have increased cuase. But the immediate cause of the warning
and one of the factors of this increase may be the is the speculation.
recent joint scheme. 122. 5; A can be the cause of B and vice versa.
99. 5; The two events have no apparent causative link 123. 2; Because of the earthquake the weak buildings
with each other. have collapsed and so the government has decided
100.5; There is no cause-effect relationship. to take action against those corrupt builders.

K
101. 1; Critical condition of Ivory coast has compelled 124. 1; If less cars are allowed, the sale is bound to drop.
Cadbury to take such an initiative. 125. 1; Event A is clearly a reaction to event B.
102.2; Reduction of its stake in nationalised banks by 126. 1; The protest (Event A) is in response to the levy
the government is a steps towards privatisation imposed (event B).
of bank. And this is a reason of strike in bank by 127. 5 128. 2 129. 4 130. 5 131. 2
the bank employees. 132. 2 132. 3 134. 3 135. 1
103. 1; Because of event (B), government has decided to 136. 3; There may be several causes for the increment
roll back the hike in the prices of cooking gas in entry fee in the Taj Mahal. Event (B) may be
and kerosene. one of the causes. It is also possible that the
104. 4; The SC order is not the immediate cause for the hike in maintenance cost is the other cause of
mob frenzy. It is the implementation of the order. this increment.
105. 5; Both events (A) and (B) are independent of each 137. 1
other. It is possible that because of the court’s 138. 2; The prices of vegetables has increased because

KUNDAN
pressure or central government’s pressure, the of the strike of rickshaw, taxi and other general
STF has been activated against him. communication. Hence B is the effect and A is
106. 3; This may be the reason for his denial to talk the immediate cause.
with the media persons but not necessarily the 139. 1; Because of (B), government has started the project
principal and immediate cause. “alert citizen” to curb crimes. Hence `A’ is the
107. 2; As he found cockroaches in the crev ices, he effect and (B) is the cause.
immediately drew the attention of the catering 140. 5
manager. 141. 2
108. 1; The receiving of the information is the immediate 142. 4; Insecurity is a long-run impact of this habit of
cause of the arrest. too much comfort. Comfort leads to a distaning
109. 5; Here both events are effects of different causes. from follow-beings. And this loneliness lat er
110. 4; The chief minister’s prayers signify that he was results in insecurity.
t aking int er est in Rajkumar’s release. But 143. 1; Dearness allowance of the employee depends on
Verappan’s concession can not be considered a the consumer price index.
direct effect of the prayers per se. 144. 1
111. 3; There may have been several steps before BCCI 145. 3; One of the reasons for increase in no. of home
has decided to ban some players. computers in their being made compulsory in
112. 4; (B) may be because of the traffic congestion around schools. Other reasons are a general interest in
CP, bad parking arrangements, etc .... (A) also computers for various purposes.

Practice Exercise–2
1. 1; Here ‘A’ is the cause and ‘B’ is its effect. Farmers 3. 2; The tense of both the statements implies that
do not want to incur any loss further. Hence the the event mentioned in ‘B’ happened before ‘A’.
r eason which led far mer s t o swit ch over t o Through an intuitive look we come to know that
another crop is obvious. ‘A’ is an initiative taken by the government with
2 . 2; A crime has a direct relationship with the condition a view to the people’s concern described in ‘B’.
of law and order. Incidents of crime increase if Hence, ‘B’ is the cause and ‘A’ is its effect.
the condition of law and order is not up to mark. 4. 2; See t he words ‘has’ and ‘hav e been’ in t he
Hence, ‘A’ is the effect of the cause ‘B’. statements ‘A’ and ‘B’ respectively. From these
Cause and Effect 229

words we get that ‘B’ happened before ‘A’. Now, 2 0 . 1; ‘B’ is the result of the request made by the local
why did the PM give such an assurance? The traders’ association to its member, as mentioned
answer lies in t he ev ent ment ioned in t he in ‘A’. Hence, ‘A’ is the cause and ‘B’ is its effect.
statement ‘B’. Hence, ‘A’ is the effect and ‘B’ is its 21. 4; It is difficult to link stamp duty and real estate
cause. prices with certainty.
5. 5; Both the events are the effects of a common cause, 2 2 . 2; It there are large exports, considerable profit is
ie heavy rain in the city. quite likely.
6. 1; Drop out from the primary schools from rural areas 23. 4; Both (A) and (B) are contradictory statements (to
may be an effect of poor facility of education. Note each ot her). The st udent s who did not get
that lack of interest in study because of poor admission in t he gov er nment engineering
infrastructure increases the number of dropouts colleges must have gone to the private engineering
from schools. Hence, A is the cause and B is its colleges. But this has not happened. Hence, both
effect. the statements (A) and (B) are the effects of
7. 2; The event mentioned in B caused resentment independent causes.
among the people, which made the people unruly, 24. 4; This is a case of contradictions. Statement (B)
as a result of which the police resorted to lathi says that the priority sector is the better sector
charge to br ing the situation under control. for the banks to invest in. Despite this, the banks
Hence, B is the cause and A is its effect. have decided to increase the rate of interest on
8. 5; Both A and B are the effects of a common cause: the advances to be given to the priority sector.
to facilitate travelling for overseas destinations Hence, (B) is obviously not behind the decision
through air route. of increasing the rate of interest. Rather, it should
9. 5; Note that the real concern of the opposition is have gotten some favour. Hence, both (A) and (B)

K
the government’s economic policy. Also, a faulty are effects of independent causes.
economic policy results in sharp increase in 25. 2; Why is there a need for allowing the private
prices of commodities. Hence, both A and B are air line companies t o oper at e on specified
the effects of a common cause - the economic international routes? Obviously, an increase in
policy of the government. the flow of foreign tourists to India is behind the
10. 4; Both A and B are effects of independent causes. decision. Hence, (B) is the cause and (A) is its
The ef fect mentioned in A is t he r esult of effect.
‘adulteration’ whereas the effect mentioned in B [Note: Here, we have to look for such a statement
is the result of ‘natural disaster’. which may be the effect of the other statement.]
11. 4; Both A and B are effects of different causes. I is 26. 5; Both are the efforts made by the government to
an init iat iv e t aken to ensur e driving free of keep the price of petroleum products low. Hence,
hindrances whereas the cause which led the both ‘A’ and ‘B’ are effects of a common cause.
event B is the lack of signal posts. 27. 1; The event mentioned in t he st at ement ‘A’
12. 2; Statement A mention the initiative taken by the compelled the cit izens to init iat e securit y

KUNDAN
local steel company because of the request made measures as mentioned in the statement ‘B’.
by the local civic body. Hence, B is the cause and Hence, ‘A’ is the cause and ‘B’ is its effect.
A is the effect. 28. 2 ; Here the decision taken by the government (as
13. 4; Both A and B are eff ects. A is the r esult of mentioned in statement ‘A’) is aimed at equipping
disciplinary act ion taken by t he government private universities so that these universities can
against the carelessness of the police. Whereas facilitate more students to pursue their desired
B is the result of alacrity and vigilance of the course.
police. Now, from statement ‘B’ we come to know that
14. 2; A is the result of resentment caused by B. Hence, the government is compelled to do so because of
B is the cause and A is its effect. the continuous decrease in the number of students
15. 4 enrolled for various courses. Hence, ‘B’ is the
16. 2; From the given two statements (A) and (B) it is cause and ‘A’ is its effect.
obv ious t hat the employees have t aken t he 29. 1; Here the effort (mentioned in ‘B’) has been taken
initiative because of the resentment caused by by the government to control the prices of the
the policy announced by the management. Hence, grain so that the interest of the farmers can be
(B) is the cause and (A) its effect. protected. Note that fresh stock of gains in the
17. 2; Here, it is clear that the event mentioned in (B) market can reduce the prices of grain sharply.
happened earlier than the event mentioned in Hence, ‘A’ is the cause and ‘B’ is its effect.
(A). See the key wor ds ‘last week’ and ‘fr om 30. 1; Here the decision taken by the parents is the
immediate effect’. From the two statements it r esult of the decision t aken by t he school
seems that the initiatives taken by the public authority. Hence, 'A' is the cause and 'B' is its
sector telecom service provider may have been effect.
taken in order to compete with the private sector 31. 4; Here ‘A’ is the effect of the resentment among the
telecom service provider. Hence, (B) is the cause villagers. Whereas ‘B’ is the effect of the efforts
and (A) its effect. aimed at electoral reforms. Hence, both ‘A’ and ‘B’
18. 4; Here, both the statements (A) and (B) are effects are effects of independent causes.
of independent causes. 32. 4; 'A' may be the effect of the increase in demand 'B'
19. 2; The condition mentioned in ‘B’ compelled the is the effect of the aim to increase literacy rate.
government to take initiative as mentioned in ‘A’. Hence, both 'A' and 'B' are effects of independent
Hence, ‘B’ is the cause and ‘A’ is its effect. causes.
230 Practice Book on Analytical Reasoning

33. 1; The initiative mentioned in 'B' has been aimed at 57. 2; High marks have become so commonplace that
because of the concern raised in 'A'. Hence 'A' is merit lists have lost their meaning.
the cause and 'B' is its effect. 58. 2; The large number of SSC qualif ier s has
34. 5; Both the effects 'A' and 'B' may have a common necessitated the opening of more junior colleges.
cause like bandh called by a political party, etc. 59. 4; A is the effect of some climate change. But B is
35. 2; Here 'B' is the cause of the event mentioned in something that has been happening since time
'A'. immemorial.
36. 2; Here the impediment mentioned in 'B' is caused 60. 5; Both the statements have been necessitated by a
by 'A'. Hence, 'B' is the cause and 'A' is its effect. common cause: the fear of outbreak of diseases
37. 2; It is the effort of the social activists which forced during monsoon.
the government to take the initiative. Hence, 'B' 61. 2; The increase in accidents again and again has
is the cause and 'A' is its effect. led the govt to strike a cautionary note.
38. 5; Both 'A' and 'B' are the effects of the new economic 62. 2; Clearly, the accident in B led to the decision in
scenario of the world. Hence, both 'A' and 'B' are A.
effects of a common cause. 63. 1; The fog in A led to the flight delays in B.
39. 2; The preventive measure taken by the government 64. 3; The two are contradictory and therefore can't be
is to restrict unpleasant incidences caused by connected.
terrorists. Hence, 'B' is the cause and 'A' is its 65. 5; Both are effects of a common cause – people's
effect. desire to drive cars.
40. 1; It seems that activeness of Road Development 66. 5; The overcrowding in A has led to pollution in B.
Authorities is the result of the order issued by 67. 2; Since Class X results were higher, the cutoff
the High Court. Hence, 'A' is the cause and 'B' is percentages for junior colleges (Class XI) are

K
its effect. bound to increase.
41. 2; The flood has led to the epidemics. 68. 1; The bad condition of the highways has prompted
42. 2; Reduction in taxes aff ects t he pr ices of the the govt to take the step.
product. Hence, 'B' is the cause and 'A' is its effect. 69. 5; Both seem to be the effects of poor teaching in
43. 1; Paying capacity of a customer plays an important the local school.
role in shopping. Hence, 'A' is the cause and 'B' 70. 1; Clearly, rain and thunder showers forced the
is its effect. people to stay indoors.
44. 1; Destruction of an oil refinery can create problem 71. 2; Health-consciousness has led to the sale of fat-
in ensuring smooth availability of oils. Hence, 'A' free food articles.
is the cause and 'B' is its effect. 72. 1; Since there have been heavy rains, there will
45. 2; Withdrawal of grant will increase the financial now be ample water and this will obviate the
bur den of t he gov er nment medical colleges, need to cut the water supply.
whereas the increase in the number of seats will 73. 2; Obviously, the high cost of private travel has

KUNDAN
increase the revenue of the government medical given a boost to Govt transport.
colleges. Hence, 'B' is the cause and 'A' is its 74. 1; The rise in vegetable prices will be one of the
effect. factors in the increase in consumer price index.
46. 4 75. 5; Both are the effects of some crime having been
47. 5; It can be safely assumed that Party X was earlier committed or a suspicion thereof.
in the opposition and has benefited from anti- 76. 2; Clearly, employees have been prompted to go on
incumbency. But A and B are consequences of a leave on Friday because they would like to take
common cause — bad governance by the ruling advantage of (B).
party. 77. 2; In order to check the menace of poor eating habits,
48. 1; Since the staff members have gone on strike, the schools have banned the sale of fast food.
help of outsiders has been sought. 78. 5; Bot h ar e ef f ect s of a common cause — the
49. 4; (A) seems to have happened as X is a better recession.
college. (B) seems to be the result of the falling 79. 1; The unhealthy competition leads to the negative
standard of teaching in colleges. mindset that is responsible for suicide.
50. 4; (A) might have happened because harassment of 80. 4; (A) is the result of global warming. (B) is the result
women is on the rise. While (B) seems to be the of tectonic shifts.
result of a change in gender role perception. 81. 3; Both are independent causes.
51. 1; The r obberies hav e led t o a demand of 82. 2; Changes in wind pattern have caused increase
improvement in security situation. in temperature.
52. 1; Since the prices of cement have increased, there 83. 1; The motorists' hardship has led to the municipal
are fewer customers now. An impact on profit sanction.
has made businessmen less likely to venture in 84. 2; Lack of projects has led to the lay-off.
this industry. Hence the govt has decided to 85. 1; The attractive schemes are attempts to boost sales.
provide tax break. 86. 4
53. 5; Both the events are the consequences of atrocities 87. 2; Job market is an impor tant consideration in
committed by corrupt police officials. determining the curriculum of schools.
54. 4; There may have been different reasons for the 88. 5; There seems to be some common cause that is
opting of these subjects. leading t o deter ior at ion in bot h kinds of
55. 4; There seems to be no connection between the education.
two events. 89. 5; Hike in fuel prices seems to be the common cause.
56. 2; Clearly, heavy discounts led to the rush. 90. 4; The two effects are unrelated.
Cause and Effect 231

91. 1; The farmers' decision is apparently an effect of 112. 1; The govt initiat iv e has led to great er wheat
change in the soil. cultivation.
92. 1; The report to the police led to the criminal's 113. 1; Brake failure led to the accident.
arrests. 114. 2; Since the level is lower, the govt has decided to
93. 2; The murders have led to the announcement of boost English language education.
the award. 115. 2; Since pedestrian movement was getting blocked,
94. 1; The st one-pelt ing led t o the wounded being the authority demolished the tea stall.
brought to the hospitals. 116. 1; The leaving of students led to the urgent action
95. 4; A is the effect of summer vacation while B is the by the authority.
effect of trying to get relief in summer. 117. 2; The pressure of the social activists has led to the
96. 5; Both are the effects of the fact that festivals cause banning.
fatigue. 118. 4; The two are contradictory and must be the effects
97. 4; (A) is the effect of the presence of some wicked of independent causes.
youngsters living in the locality while (B) is the 119. 2; The large number of terrorist attacks has led to
effect of a rather peaceful atmosphere. tightened security checks.
98. 3; There seems to be no connection between the 120.1; The court order has led to the urgent work.
syllabus of Std IX and admission to colleges. 121. 2; The flood has led to epidemics.
99. 2; The "considerable profit" has led to the "hefty 122. 2 123. 4
bonus". 124. 2 125. 2
100. 1; The repair work has led to the diversion. 126. 2
101. 5; Both are the effects of a recession in the economy. 127. 1; The effect mentioned in B has direct relationship
102. 2; The bumper crop has led to the largesse shown with the number of unemployed persons. Hence,

K
by the govt. A is the cause which led to B.
103. 3; (A) happened so that students could see more of 128. 2; Soar ing of pr ices of vegetable has direct
the world. (B) happened so that the school may relationship with the availability of vegetables
attend to other important tasks. and also with the demand of vegetable. The event
104. 4; (A) has happened because of increased supply of mentioned in B has a negative impact on the
fruits. (B) has happened because of decreased availability of vegetables. Hence, B is the cause
supply of foodgrains. which led to B.
105. 5; Both the statements seem to have a common 129. 5; Both B and B are effects of independent causes.
cause—agitation on a large scale. Manuf act uring of soaps or increase in it s
106. 1; The fear of rain has led to the tournament being production has no relationship (as mentioned)
called off. with the coastal environment. Hence, option 5).
107. 2; The surge in complaints has led the banks to 130. 5; Is B the cause which led to A? Answer is ‘No’.
receive them in an electronic mode. Again, is B the cause which led to B? Answer is
108. 1; The shortage in production has led to the import ‘No’. Thus, bot h A and B

KUNDAN
decision. are eff ects of independent causes. Hence,
109. 5; Both are the effects of a common cause. The optoption 5).
Railways has gone in a reviving mode. 131. 3; Increase in water level of all water tanks and the
110. 5; Bot h ar e the ef f ect s of gear ing up on t he problem of water-logging on the tracks are the
environmental front. result of increase of water availability or surplus
111. 2; The bur den is sought to be r educed by the of water (due to rain). Hence, both the events are
amended tax laws. effects of the same cause.

Practice Exercise–3
1. 4 2. 3 3. 3 9. 3; Crude prices have a direct bearing on the prices
4. 1 5. 2 of petroleum products.
6. 2; W e of t en hear of accident s leading t o such 10. 1; The effect is a win-win solution to this cause.
injuries. 11. 3; Tax is only one component of the price. So, the
7. 1; The very purpose of hiking the procurement price increase in price will be there but the increase
of a crop is encouraging the farmers to cultivate percentage will be lower.
it. 12. 1; The engineer’s caution may have led to people
8. 4; In such a cataclysmic scenario, the govt is likely keeping away from the temple.
to order the closure of offices and schools. 13. 3; Such rejections will cause difficulty to the people.
Inequality
Introduction Ex. 1: Inequalities: A > B, C > D
Here, there are four terms A, B, C and D but they
Problems based on inequalities and coded inequalities do not have a common term. Hence these two
involve essentially combination of two elementary problems inequalities cannot be combined.
(as very name of the chapter suggests) (i) Inequalities and Ex. 2: Inequalities: A  B, M  N
(ii) coding Here, also there are four terms A, B, M and N but
In such problems coding part is not a big challenge the common term is missing. Hence they cannot
because coding scheme is told entirely in the question be combined.
itself. Therefore, to decode the inequalities in a given ( 2) Two inequalities can be combined if and only if
problem would not mean any more headache than a couple the common term is greater than (or ‘greater
of extra seconds. than or equal to’) one and less than (or ‘less

K
Essentially it is a problem of inequalities and it is than or equal to’) the other. (And the combined
this aspect that should be mastered. Hence we first learn inequality will have the common term in the middle
the basics of inequalities. with the greater and the smaller terms on the two
extremes.) Look at the examples given below:
What is an Inequality?
Ex. 3: Inequalities: A > B, B > C.
We know that the result of multiplication between 3 Here, common term B is greater than one term C
and 2 and the number 6 are equal. Since they are equal and less than the other, A. So a combination is
it is an equality. In the same way, 3 × 3  6. Here the possible.
product of 3 and 3 is not equal to the number 6. And Combined inequality:
since they are not equal, it is an inequality. A > B > C or C < B < A
Ex. 4: Inequalities: A  B, C < B.
Signs of Inequalities Here common term B is less than (or equal to)
There are, usually, four types of inequalities as given one term, A, and greater than the other term, C:

KUNDAN
below: Hence, combination is possible.
( i ) Greater than: To denote mathematically, we use Combined inequality:
‘>’ to denote greater than. For example, 3 × 3 > 6. A  B > C or C < B  A
( i i) Less than: To denote mathematically, we use ‘<’ Ex. 5: Inequalities: B < A, B  C.
to denot e less t han. For example, Here, common term B is less than one term, A;
3 × 1 < 6. and greater than (or equal to) the other term, C.
( i ii ) Greater than or equal to: Sometimes we are faced Hence, combination is possible.
with two numbers where we don’t know the exact Combined inequality:
state of inequality between them. For example, we A > B  C or C  B < A
may have two numbers x and y and all that we Ex. 6: Inequalities: A > B, C > B.
know is that ‘x is not less than y’. In this case x Here, common term is less than both the other
can be either equal to or greater than y. Such t erms, A and C. So a combination is not
situations can be represented as ‘’ sign. Thus possible.
we have ‘’ meaning ‘greater than or equal to’. Ex. 7: Inequalities: A  B, B < C.
So, x  y means x is either greater than or equal to y. Here, common term B is less than (or equal to)
( iv) Less than or equal to: Similarly, as above, we both terms. No combination possible.
may have two numbers a and b and all that we Ex. 8: Inequalities: B  A, C  B.
know is that ‘a is not greater than b’. In this case Here, common term B is greater than (or equal to)
a can be either equal to or less than b. Such situations one term, A, and less than (or equal to) another
can be represented as ‘’ sign. Thus we have ‘’ term, C. Hence, combination is possible.
meaning ‘less than or equal to’. Combined inequality:
So, a  b means a is either less than or equal to b. A  B  C or C  B  A.
Ex. 9: Inequalities: A  B, B  C. Here, common term B
Combining Inequalities is less t han (or equal to) bot h A and C
combination is not possible.
W e have t wo golden rules f or combining t wo
Ex. 10: Inequalities: B  A, B  C.
inequalities as given below:
Here, common term B is less than (or equal to)
(1) Two inequalities can be combined if and only if
one term A; and greater than (or equal to other
they have a common term. Look at the examples
term, C. Combination is possible.
given below:
Combined inequality:
A  B  C or C  B  A.
Deriving a Conclusion from a Combined Step I:
Inequality Neatly and quickly decode the symbols.
We have another golden rule, we call it as third golden The question itself tells you which code stands for
rule, for deriving a conclusion from a combined inequality which arithmetical operation. for example, we have been
as given below: given that P  Q means P > Q. Therefore replace ‘’ by ‘>’
Combine the two inequalities and draw a conclusion by wherever you see them.
letting the middle term disappear. The conclusion—inequality You should take one code at a time and replace it by its
will have an ‘’, sign (or a ‘’ sign) if and only if both the signs original mathematical symbol in all the given questions
in the combined inequality were ‘’ (or ‘’; as the case may be). before going to the next code. And you should do it quickly.
Hence, the conclusion will normally have a ‘>’ (or a ‘<’ Step II:
sign strictly, unless the ‘’, sign or (‘’) appears twice in
the combined inequality. Look at the examples given below Take one conclusion at a time and decide which
that will illustrate the concept: statements are relevant for evaluating the conclusion.
Ex. 11: Derive a conclusion from the following combined Now, this needs some thinking. What do we mean by
inequalities: relevant statements? By a relevant statement we mean
(i) x > y > z the statement that is not useless for deriving a conclusion.
(ii) x < y < z If there is a conclusion, say, x > y then a statement like b
Soln.: (i) x > z > c is useless because this statement has neither x nor y.
((ii) x < z. Therefore any analysis of this statement can’t tell us
Ex. 12: However, when we have ‘’ signs in the combined anything about the conclusion: x > y. For any conclusion,
inequalit ies, t hen the pr ocess is a bit mor e the relevant statements are those that can be combined to
thought-provoking. For example, consider the prove or disprove that conclusion. How do we find the

K
following combined inequality: relevant statements?
x  y > z. It is simple ....
Here, x is either greater than y or equal to y. To decide which stat ement s ar e r elev ant f or a
Therefore the minimum value for x is equal to y. conclusion; take the two terms of a given conclusion and
But y is always greater than z. Therefore, x is see if each of them separately appears with a single
always greater t han z (because even when x common term in the given statements in the question.
attains its least value it is equal to y and y is These statements will be our “relevant statements”.
always greater than z. Therefore x will always be To understand the above look at the example given
great er t han z). Hence, the inequalit y as below:
conclusion is: Ex. 16: Suppose after performing Step I, we have the
x > z. following given statements and conclusions.
Ex. 13: Now consider the combined inequality Given Statements:
x > y  z. M > N, L = M, O > N, L  K

KUNDAN
Here, x is always greater than y and y is either Conclusions: I. M < K II. L > N
greater than z or equal to it. When y is greater Now, take conclusion I (M < K). For this we find
than z; x will obviously be greater than z. Even that the relevent statements are
when y is equal to z; x will be greater than z M = L, L > K
because x is always greater than y. Hence, in all Now, take conclusion II (L>N). For this relevant
cases, our conclusion is: statements are:
x > z. M > N, L = M
Ex. 14: Now consider the combined inequality
Step III:
xyz
Here x is either greater than y or equal to y. When Use the three golden rules to combine the relevant
x is greater than y; we have: x  z which gives the statements and derive a conclusion from it. Those three
conclusion x > z ... (a) [see Ex. 13] golden rules are:
When x is equal to y; we have: Rule 1: There must be a common term.
x = y  z which gives the conclusion Rule 2: The common term must be less than (or equal
xz ... (b) to) one term and greater than (or equal to) another.
Combining conclusions (a) and (b), we have Rule 3: The conclusion-inequality is obtained by letting
xz the common-term disappear and it has a ‘’ or a ‘’ sign if
Ex. 15: Derive a conclusion from the following combined and only if the both the inequalities in second step had a
inequalites: ‘’ or a ‘’ sign. In all other cases, there will be a ‘>’ or a ‘<‘
(i) L = M  K sign in the conclusion.
(ii)) L = M > N For illustration consider the previous example. We
Soln.: (i) L  K have found that
(ii) L > N ( i ) for conclusion I (M < K) the relevant statements
are:
Strategy to Solve Problems on Inequality and M = L, L  K
Coded Inequality We combine them and get M = L < K
 M  K. This is Step III.
There are various steps needed to solve the problem
Now, M  K does not imply that M < K because M 
as given below:
K allows for M to be even equal to K, which is not
t rue in case of M < K Hence t his conclusion
(Conclusion I) doesnot follow.
( i i) for Conclusion II (L > N), the relevant statements Note: Check 4 merely tells you that one number can only
are: have three positions vis-a-vis another number. It
M > N, L = M can be either less than or equal to or greater than
After combining, we get L = M > N  L > N the other.
Hence, Conclusion II follows. This is true universally of any two numbers. That is,
After performing these three steps, if a conclusion is [A  B or A > B] is a universally correct statement, because
established and verified, well and good. If not, then perform A can be either (less than or equal to) or (greater than) B.
the following four checks: Thus, for any two numbers A and B the following are
Check 1: Check if the conclusion directly follows from only always correct:
single given statement. (i) (A  B) or (A > B)
Somet imes a st at ement may be in t he f or m of (ii) (A < B) or (A  B)
A  B and one conclusion may be in t he f or m of (iii) (A > B) or (A  B)
B  A. Obviously both these are completely identical but (iv) (A  B) or (A < B)
sometimes we are prone to ignore such minor tricks of We can call these four pairs of statements as our
the examiner. complementary pairs. Obviously since one out of the
Ex. 17: For example consider the following: two statements will always be true in such cases, we
(Let  mean >,  mean ,  mean =,  mean <,  choose “either follows” as our answer. But remember, we
mean ) choose this as our answer only if neither of the two
Let, given statement: E  F, C  D, F  G, D  F statements have been otherwise proved in any previous
Conclusion: I. G  F. step. This is because if we have already proved that A < B
Here, conclusion I is G  F or G  F and it is is definitely true it is ridiculous to still settle for the rather
identical to F  G or F  G. Hence, it directly uncertain- looking answer that “ either A < B is true or A

K
follows from one single statement.  B is true”. To understand this point better, consider Ex
Check 2: The conclusion you reach after the Third Step 18 and Ex 19 below.
may be identical to the given conclusion although it may not Ex. 18: Statements: A  B, B = D, D  C, C  B
look so in the first glance. Check. Conclusions: I. A  C
For Example if you arrive at a conclusion- inequality: A II. A < C
 B then a given conclusion B  A is obviously true. Soln.: Here, both conclusions are between A and C. We
Check 3: If after the third step you get a conclusion that see that A and C appear with a common term B
has a ‘’ (or a ‘’) sign and the two given conclusions have a ‘>’ in
(or a ‘<‘) sign and a ‘=’ sign between the same terms; the choice A  B and C  B.
either I or II follows is correct. (Very Imp.) So these are our relevant statements. They can
For example, suppose you reach A  B after performing be combined because they have a common term
the Third Step. Now suppose the given conclusions are: and because the common term B is less than ( or
I. A > B and II. A = B. Then, the choice “either I or II equal to) one term A and greater than (or equal to)

KUNDAN
follows” is correct Similarly if you conclude that M  N another term B.
and the given conclusions are I. M < N and II. M = N then Combining we get: A  C.
again the same answer follows. So, conclusion I follows.
Note: Check 3 merely tells you that if you have concluded Ex. 19: Statements: A  B, B = D, D  C, B  C
that [A  B] you can as well write it as [A > B or A Conclusions: I. A  C
= B]. Again, if you have concluded that [A  B] you II. A < C
can as well write it as [either A < B or A = B]. Soln.: By the same analysis as in Ex 18; our relevant
Check 4: If the two given conclusions have a statements are: A  B, B  C.
(i) ‘’ and ‘>’ signs, or Now, these two can not be combined because the
(ii) ‘<’ and ‘’ signs, or common term B is less than (or equal to) both A
(iii) ‘>’ and ‘’ signs, or and C. Hence, no conclusion is possible.
(iv) ‘’ and ‘<‘ signs But t he t wo conclusions I and II f orm a
between the same terms; and if neither of the conclusion complementary pair. And hence either of them
has been accepted in any of the steps above; the choice “either must follow.
of the two follows” is correct. (Important) [Note that we had the same complementary pair as our
For example, suppose in a given question, the given answer-choices in both Ex 18 and Ex 19. Yet we did not
conclusions are: choose “either follows” in Ex 18 because there conclusion
I. A  B II. A < B I was definitely established. But in Ex 19 no conclusion
And suppose that neither of them have been proved to was definitely established and therefore we choose “either
be true by virtue of any of the preceding steps. follows” as our answer.]
Then since they have the same pair (A and B) and the
signs are ‘’ and ‘<‘; the choice either follows is correct.
Illustrative Example
We will now demonstrate how to utilise the above Now we will take each of the questions separately and
discussed method to quickly solve problems of this type. perform step II and III for each of the conclusions.
Directions (Q. 1-5): In the following questions, the 1. (i) Conclusion I: Relevant statements are:
symbols , ,  ,  , and  are used with following L = M, L  K.
meaning: Combining, we get: M  K. This does not match
‘P  Q’ means ‘P is greater than Q’. with the given conclusion; M < K.
‘P  Q’ means ‘P is either greater than or equal to Q’. (ii) Conclusion II: Relevant statements are:
‘P  Q’ means ‘P is equal to Q’. M > N, L = M.
‘P  Q’ means ‘P is smaller than Q’. Combining, we get L > N. Hence, only conclusion
‘P  Q’ means ‘P is either smaller than or equal to Q’. II follows. Correct answer: 2
Now in each of the following questions, assuming 2 . (i) Conclusion I: Relevant statements are:
the given statements to be true, find which of the two E = F, F  G.
conclusions I and II given below them is / are Combining, we get E  G. This does not match
definitely true. Give answer with the conclusion given: E > G.
1) if only conclusion I is true; (ii) Conclusion II: Relevant statements are:
2) if only conclusion II is true; C < D, D = E.
3) if either I or II is true; Combining we get, C < E. This does not match
4) if neither I nor II is true; and with C = E. Hence both conclusions are rejected.
5) if both I and II are true. Check 1, 2, 3, 4 are also futile. Correct answer: 4
1. Statements: M  N, L  M, O  N, L  K 3. (i) Conclusion I: Relevant statements are:

K
Conclusions: I. M  K O = N, M = O, L  M.
II. L  N Combining the first two, we get M = N. Combining
2. Statements: E  F, C  D, F  G, D  E this with L  M, we get: L  N. This means that
Conclusions: I. E  G conclusion I does not follow.
II. C  E (ii) Conclusion II: We have already seen that L  N
3. Statements: L  M, O  N, L  H, M  O follows. This is different from L > N. So conclusion
Conclusions: I. L  N II does not follow. But, by virtue of Check 3,
II. L  N choice 3 is correct.
4. Statements: Z  Y, K  L, Y  X, Z  K 4. (i) Conclusion I: Conclusion I is Y > L. Now, from
Conclusions: I. Y  L the given statements, Y and L do not appear
II. Y  L separately with a single common term. Y appears
5. Statements: R  I, S  C, S  I, C  O with Z, Z with K and K with L. Hence we will
Conclusions: I. C  I hav e to t ake these t hree as our r elev ant

KUNDAN
II. S  R statements. They are:
Soln.: In First Step, we quickly decode the symbol. Thus Z  Y, Z > K, K = L
we have: Combining Z  Y and Z > K, we get: Y > K. Now,
1. Statements: M > N, L = M, O < N, L  K combining it wit h K = L; we get Y > L. So
Conclusions: I. M < K conclusion I follows.
II. L > N (ii) Conclusion II: Conclusion II is Y = L. Which is
2. Statements: E = F, C < D, F  G, D = E not true as Y > L has been proved.
Conclusions: I. E > G Correct answer: 1
II. C = E 5. (i) Conclusion I: Conclusion I is C < I. C and I
3. Statements: L  M, O = N, L < H, M = O appear separately with S in S = C and S  I. So
Conclusions: I. L = N these two are our relevant statements. Combining,
II. L > N we get: C  I. This means conclusion I is not
4. Statements: Z  Y, K = L, Y < X, Z > K true.
Conclusions: I. Y > L (ii) Conclusion II: Conclusion II is S > R. Now, S
II. Y = L and R appear separately with a common term I;
5. Statements: R < I, S = C, S  I, C > O in R < I and S  I. So these two are our relevant
Conclusions: I. C < I statements and combining them we get: R < S.
II. S > R By Check 2, it is the same as S > R. Hence
conclusion II follows.
Correct answer: 2
Coded Inequality
Exercise
Directions (Q. 1-5): In the following questions, cer- 1) if only conclusion I is true;
tain symbols are used with the following meaning: 2) if only conclusion II is true;
P  Q means P is greater than Q. 3) if either I or II is true;
P © Q means P is either greater than or equal to Q. 4) if neither I nor II is true; and
P = Q means P is equal to Q. 5) if both I and II are true.

K
P @ Q means P is smaller than Q. 6. Statements: P  A, I  D, K  A, I  P
P @ Q means P is either smaller than or equal to Q.
Conclusions: I. I  K II. D  A
Now in each of the following questions assuming the
given statements to be true, find which of the two conclu- 7. Statements: U  S, N  A, H  U, A  S
sions I and II given below them is/are definitely true? Give Conclusions: I. U  A II. A  U
answer
8. Statements: E  D, K  A, E  R, A  D
1) if only conclusion I is true;
2) if only conclusion II is true; Conclusions: I. K  A II. K  R
3) if either I or II is true; 9. Statements: Y  L, A  R, M  Y, M  R
4) if neither I nor II is true; and
Conclusions: I. R  L II. A  M
5) if both I and II are true.

KUNDAN
1. Statements: I © V, R @ D, E @ V, R = I 10. Statements: E  L, A  N, E  V, A  V
Conclusions: I. DV II. E = I Conclusions: I. N  V II. A  E
2. Statements: T = A, E @ L, T @ E, R  A Directions (Q. 11-15): In the following questions, the
Conclusions: I. T  A II. T @ R symbol $, #, £,  and  are used with the following mean-
3. Statements: C @ A, O @ C, E = P, E  A ing:
Conclusions: I. P  A II. O @ A P $ Q means P is greater than Q.
4. Statements: M © B, R @ B, M @ U, R = E P # Q means P is either greater than or equal to Q.
Conclusions: I. M  R II. M = R P £ Q means P is equal to Q.
5. Statements: S © M, K  A, S @ U, A @ M P  Q means P is smaller than Q.
Conclusions: I. A = S II. K  M P  Q means P is either smaller than or equal to Q.
Directions (Q. 6-10): In the following questions, the Now in each of the following questions assuming the
symbols  ,  ,  ,  and  are used with the following given statements to be true, find which of the two conclu-
meaning. sions I and II given below them is/are definitely true? Give
P  Q means P is greater than Q. answer
1) if only conclusion I is true;
P  Q means P is either greater than or equal to Q.
2) if only conclusion II is true;
P  Q means P is equal to Q. 3) if either I or II is true;
P  Q means P is smaller than Q. 4) if neither I nor II is true; and
P Q means P is either smaller than or equal to Q. 5) if both I and II are true.
11. Statements: D # H, I  R, R  H
Now in each of the following questions assuming the
Conclusions: I. D $ I II. R  D
given statements to be true, find which of the two conclu-
12. Statements: K  I, S $ H, K # H
sions I and II given below them is/are definitely true? Give
Conclusions: I. S £ K II. H  I
answer
13. Statements: A £ M, J $ T, A  T Conclusions: I. O + W II. B – F
Conclusions: I. T $ M II. A J 23. Statements: Z × M, B  S, N ÷ Z, N  S
14. Statements: R $ A, H  U, A £ U Conclusions: I. S ÷ M II. B + N
Conclusions: I. A $ H II. H £ A 24. Statements: F  E, L × B, F ÷ S, B + E
15. Statements: M  U, K # A, M $ K Conclusions: I. L + B II. L – S
Conclusions: I. U # A II. A  M 25. Statements: V × T, O – B, I ÷ V, B  T
Directions (Q. 16-20): In the following questions, Conclusions: I. V + B II. B – V
the symbols @, @  ,  and  are used with the following Directions (Q. 26-30): In the following questions,
meaning: the symbol £, $, @,  and > are used with the following
P @ Q means P is greater than Q. meanings:
P @ Q means P is either greater than or equal to Q. P £ Q means P is greater than Q.
P  Q means P is equal to Q. P $ Q means P is either greater than or equal to Q.
P Q means P is smaller than Q. P @ Q means P is equal to Q.
P  Q means P is either smaller than or equal to Q. P  Q means P is smaller than Q.
Now in each of the following questions assuming the P > Q means P is either smaller than or equal to Q.
given statements to be true, find which of the two conclu- Now in each of the following questions assuming the
sions I and II given below them is/are definitely true. Give given statements to be true, find which of the two conclu-
answer sions I and II given below them is/are definitely true. Give
1) if only conclusion I is true; answer

K
2) if only conclusion II is true; 1) if only conclusion I is true;
3) if either I or II is true; 2) if only conclusion II is true;
4) if neither I nor II is true; and 3) if either I or II is true;
5) if both I and II are true. 4) if neither I nor II is true; and
16. Statements: X @ Y, Z  R, Y  Z 5) if both I and II are true.
Conclusions: I. X @ R II. Y  R 26. Statements: I > J, K @ J > L, H $ J
17. Statements: B @ C, C  D, D  E Conclusions: I. J £ L II. I > H
Conclusions: I. B @ E II. D  E 27. Statements: G $ F, D > E, E @ F
18. Statements: M  L, N  O, L  N Conclusions: I. E  G II. E @ G
Conclusions: I. O @ M II. L  O 28. Statements: L > K, M  N, K @ M
19. Statements: H @ G, E  F, F  G Conclusions: I. N £ L II. K > N

KUNDAN
Conclusions: I. F  H II. F  H 29. Statements: A £ B, B > C, C > D
20. Statements: J  K, L  K  M, I @ K Conclusions: I. A £ D II. C @ D
Conclusions: I. K  M II. J  I 30. Statements: W £ X, Y > Q, X @ Y
Directions (Q. 21-25): In the following questions, Conclusions: I. W £ Q II. X @ Q
the symbols ÷, ×, +, – and  are used with the following Directions (Q. 31-35): In the following questions,
meanings: the symbol  ,  ,  ,  and  are used with the following
P ÷ Q means P is greater than Q.
meanings:
P × Q means P is either greater than or equal to Q.
P  Q means P is greater than Q.
P + Q means P is equal to Q.
P – Q means P is smaller than Q. P  Q means P is either greater than or equal to Q.
P  Q means P is either smaller than or equal to Q. P  Q means P is equal to Q.
Now in each of the following questions assuming the P  Q means P is smaller than Q.
given statements to be true, find which of the two conclu- P  Q means P is either smaller than or equal to Q.
sions I and II given below them is/are definitely true? Give Now in each of the following questions, assuming
answer the given statements to be true, find which of the two con-
1) if only conclusion I is true; clusions I and II given below them is/are definitely true. Give
2) if only conclusion II is true; answer
3) if either I or II is true; 1) if only conclusion I is true;
4) if neither I nor II is true; and 2) if only conclusion II is true;
5) if both I and II are true. 3) if either I or II is true;
21. Statements: Q ÷ B, J – E, L  B, J × Q 4) if neither I nor II is true; and
Conclusions: I. J + L II. E + B 5) if both I and II are true.
22. Statements: F ÷ M, B  O, F – W, B + W
31. Statements: R  S, O  P, P  S conclusions follow. Give answer
1) if only conclusion I follows
Conclusions: I. R  O II. P  R
2) if only conclusion II follows
32. Statements: F  Z, H  A, F  H 3) if either conclusion I or II follows
Conclusions: I. Z  A II. A  F 4) if neither conclusion I nor II follows
5) if both conclusions I and II follow.
33. Statements: O  R, U  V, O  V 41. Statements: S a  T, U   T, S a V
Conclusions: I. U  O II. V  R Conclusions: I. T b V II. U   V

34. Statements: W X, Z  Y, W  Y 42. Statements: L b  M, L a  N, N b O
Conclusions: I. Y  X II. W  Z Conclusions: I. N b  O II. N a M
43. Statements: R b  T, N a  T, S b T
35. Statements: I  J, L  M, J  M
Conclusions: I. R b  U II. R   U
Conclusions: I. J  L II. L  J 44. Statements: Y b Z, S   Z, T b Z
Directions (Q. 36-40): In the following questions, Conclusions: I. Y b  T II. Y b S
the symbols  ,   ,  ,   , and  are used as fol- 45. Statements: M b  L, M a N, O a  N
lows: Conclusions: I. L b  O II. N   M
A  B means A is greater than B. Directions (Q. 46-50): In a particular method of cod-
A   B means A is greater than or equal to B. ing the symbols a, a  , b, b  and   are used with the
following meaning:

K
A  B means B is greater than A.
A a B means A is greater than or equal to B
A   B means B is greater than or equal to A. A a  B means A is equal to B
A  B means A is equal to B. A b  B means B is greater than or equal to A
Now, in the following questions, assuming the three A b B means B is greater than A
given statements to be true, decide upon the validity of the A   B means A is greater than B
given conclusions. Give answer On the basis of the above scheme and assuming each
1) if only conclusion I follows of the given statements to be true, decide which of the given
2) if only conclusion II follows conclusions follow. Give answer
3) if either conclusion I or II follows 1) if only conclusion I follows
4) if neither conclusion I nor II follows 2) if only conclusion II follows

KUNDAN
5) if both conclusions follow 3) if either conclusion I or II follows
36. Statements: P   Q, Q   R, R  T 4) if neither conclusion I nor II follows
5) if both conclusions I and II follow.
Conclusions: I. P   R II. P  R 46. Statements: P   Q, P a R, R b  S
37. Statements: N  M, C  N, M  D Conclusions: I. P a  R II. Q   S
Conclusions: I. C  D II. N  D 47. Statements: T a  N, D b  E, E a N
38. Statements: Q  L, Q   R, T  P Conclusions: I. D b  N II. E b T
48. Statements: K b  L, L   M, L a  N
Conclusions: I. L   P II. R  L Conclusions: I. K a N II. M   N
39. Statements: X   Y, Y  Z, A  Z 49. Statements: P   T, S b T, R b  S
Conclusions: I. X  Z II. Y   A Conclusions: I. S   R II. S a  R
50. Statements: X b Y, Y   T, T b  X
40. Statements: S  T, S  M, M  P Conclusions: I. Y   X II. T a  X
Conclusions: I. S  P II. T  M Directions (Q. 51-55): In the following questions,
Directions (Q. 41-45): In a particular method of cod- the symbols ©, $, #, @ and  are used with the following
ing the symbols a, a  , b, b  and   are used with the meaning:
following meaning: P © Q means P is greater than Q.
A a B means A is greater than or equal to B P $ Q means P is either greater than or equal to Q.
A a  B means A is equal to B P # Q means P is equal to Q.
A b  B means B is greater than or equal to A P @ Q means P is smaller than Q.
A b B means B is greater than A P  Q means P is either smaller than or equal to Q.
A   B means A is greater than B. Now in each of the following questions, assuming
On the basis of the above scheme and assuming each the given statements to be true, find which of the two con-
of the given statements to be true, decide which of the given clusions I and II given below them is/are definitely true. Give
answer answer
1) if only conclusion I is true; 1) if only conclusion I is true.
2) if only conclusion II is true; 2) if only conclusion II is true.
3) if either I or II is true; 3) if either conclusion I or II is true.
4) if neither I nor II is true; and 4) if neither conclusion I nor II is true.
5) if both I and II are true. 5) if both conclusions I and II are true.
51. Statements: A $ K, M # N, K  N, Q © A 61. Statements : A £ N, B = M, A * 5, B # 5
Conclusions: I. A # M II. K @ Q Conclusions: I. N # B II. M @ N
52. Statements: K # S, L  X, D $ K, S  X 62. Statements: T * C, U @ Y, T = Y, U £ T
Conclusions: I. S  L II. D $ S Conclusions: I. C # U II. U @ T
53. Statements: F # T, H  M, T © R, F @ M 63. Statements: D # E, F £ G, D @ H, F # E
Conclusions: I. R  H II. M  T Conclusions : I. H = G II. G # E
54. Statements: G $ I, D © E, E @ I, I  D 64. Statements: I @ K, R = U, O @ I, R * O
Conclusions: I. G $ E II. G # D Conclusions: I. O @ U II. O # U
55. Statements: V © W, L @ W, V $ P 65. Statements: A @ B, N * M, M £ A, B # N
Conclusions: I. V © L II. P # L Conclusions : I. M # N II. M = N
Directions (Q. 56-60): In the following questions, Directions (Q. 66-70): In the questions given below,
the symbols #, *, @, $ and  are used with the following certain symbols are used with the following meanings:
meaning: P * Q means P is neither equal to nor smaller than Q.

K
P # Q means P is greater than Q. P  Q means P is not smaller than Q.
P * Q means P is either greater than or equal to Q. P $ Q means P is neither greater nor smaller than Q.
P @ Q means P is equal to Q. P £ Q means P is neither greater than nor equal to Q.
P $ Q means P is smaller than Q. P @ Q means P is not greater than Q.
P  Q means P is either smaller than or equal to Q. Now in each of the following questions, assuming
Now in each of the following questions, assuming the given statements to be true, find which of the two con-
the given statements to be true, find which of the two con- clusions I and II given below them is/are definitely true. Give
clusions I and II given below them is/are definitely true. Give answer
answer 1) if only conclusion I is true.
1) if only conclusion I is true; 2) if only conclusion II is true.
2) if only conclusion II is true;

KUNDAN
3) if either conclusion I or II is true.
3) if either I or II is true; 4) if neither conclusion I nor II is true.
4) if neither I nor II is true; and 5) if both conclusions I and II are true.
5) if both I and II are true. 66. Statements: M £ N, O  N, P @ M
56. Statements: P @ S, A * P, S * Z Conclusions: I. N * P II. P £ N
Conclusions: I. S # A II. Z # P 67. Statements: A * B, B @ D, A $ E
57. Statements: T @ R, V  R, R * N Conclusions: I. E £ D II. B * E
Conclusions: I. T $ V II. V @ N
68. Statements: X  Y, Y * Z, M  Y
58. Statements: I $ L, K @ L, S # I
Conclusions: I. K  S II. I $ K Conclusions: I. Z £ M II. X * M
59. Statements: V @ X, N  J, X # J 69. Statements: M * U, V  U, R $ M
Conclusions: I. N * X II. J * V Conclusions: I. V £ M II. V $ M
60. Statements: W # O, D $ E, O # E 70. Statements: H @ T, R $ N, T £ N
Conclusions: I. W $ E II. D $ O Conclusions: I. H £ N II. H $ R
Directions (Q. 61-65): In the questions given below, Directions (Q. 71-75): Study the following informa-
certain symbols are used with the following meanings: tions carefully and answer the questions given below:
A = B means A is greater than B. A ÷ B means A is greater than B.
A * B means A is either greater than or equal to B. A × B means A is either greater than or equal to B.
A # B means A is equal to B. A = B means A is either smaller than or equal to B.
A £ B means A is not greater than B. A + B means A is equal to B.
A @ B means A is neither greater than nor equal to B. A - B means A is smaller than B.
Now in each of the following questions, assuming Now assuming the statements to be true, find which
the given statements to be true, find which of the two con- of the two conclusions I and II given below them is/are defi-
clusions I and II given below them is/are definitely true. Give nitely true. Give answer
1) if only conclusion I is true. clusions I and II given below them is/are definitely true. Give
2) if only conclusion II is true. answer
3) if either conclusion I or conclusion II is true. 1) if only conclusion I is true;
4) if neither conclusion I nor conclusion II is true. 2) if only conclusion II is true;
5) if both conclusion I and conclusion II are true. 3) if either I or II is true;
71. Statements: B ÷ D, H - I, M = N, Q × R 4) if neither I nor II is true; and
Conclusions: I. H + M II. I + N 5) if both I and II are true.
72. Statements: M × N, E - F, K ÷ L, C = L 81. Statements: C + E, F $ U, U * Q, Q @ C
Conclusions: I. M + F II. C - K Conclusions: I. Q $ E II. Q @ E
73. Statements: Y + B, X - Y, A ÷ B 82. Statements: U * V, V $ X, W @ M, X + W
Conclusions: I. A ÷ X II. B × X Conclusions: I. W @ U II. M # X
74. Statements: C - F, P - T, C × T, F - R 83. Statements: E + Q, F $ V, G * Q, F @ E
Conclusions: I. C ÷ P II. F ÷ T Conclusions: I. E $ V II. E + G
75. Statements: Q + S, S ÷ Z, Z - W, S = T 84. Statements: F * G, H $ I, I # F, H @ L
Conclusions: I. T × W II. W ÷ T Conclusions: I. I @ L II. I * G
Directions (Q. 76-80): In the following questions, 85. Statements: Q $ R, O # N, N @ S, S * Q
the symbols ©, =, @,  , and @ used with the following Conclusions: I. Q + O II. R $ N
meanings: Directions (Q. 86-90): In the following questions the
P © Q means P is not smaller than Q. symbols  , ©, #, @ and @ used with the following mean-

K
P = Q means P is neither greater than nor smaller than Q. ings:
P @ Q means P is not greater than Q. A  B means A is neither smaller than nor equal to B.
P  Q means P is neither smaller than nor equal to Q. A © B means A is not smaller than B.
P @ Q means P is neither greater than nor equal to Q. A # B means A is neither smaller than nor greater than B.
Now in each of the following questions, assuming A @ B means A is neither greater than nor equal to B.
the given statements to be true, find which of the two con- A @ B means A is not greater than B.
clusions I and II given below them is/are definitely true. Give Now in each of the following questions, assuming
answer the three statements to be true, find which of the two conclu-
1) if only conclusion I is true; sions I and II given below them is/are definitely true. Give
2) if only conclusion II is true; answer

KUNDAN
3) if either I or II is true; 1) if only conclusion I is true.
4) if neither I nor II is true; and 2) if only conclusion II is true.
5) if both I and II are true. 3) if either conclusion I or conclusion II is true.
76. Statements: A = B, B © D, B @ H 4) if neither conclusion I nor conclusion II is true.
Conclusions: I. H = D II. A = D 5) if both conclusions I and II are true.
77. Statements: X  Y, Y = Z, Z © T 86. Statements: E # T, S @ L, L @ E
Conclusions: I. X  T II. Y = T Conclusions: I. S # T II. S @ T
78. Statements: E © F, G = F © H, I @ F 87. Statements: P © Z, R @ K, Z @ R
Conclusions: I. F = H II. E © I Conclusions: I. P  R II. K  Z
79. Statements: P @ Q, R © S, Q = R 88. Statements: M © S, Q @ D, M # D
Conclusions: I. P @ R II. Q = S Conclusions: I. Q  M II. D  S
80. Statements: A @ B, B © C, C © D 89. Statements: B # G, R @ B, Y  R
Conclusions: I. A @ D II. C = D Conclusions: I. Y  G II. Y @ G
Directions (Q. 81-85): In the following questions,
90. Statements: D © V, X  D, V  E
the symbols *, #, $, @ and + are used with the following
meanings: Conclusions: I. X  V II. E @ D
P * Q means P is not smaller than Q. Directions (Q. 91-95): In the following questions,
P # Q means P is neither greater than nor smaller than Q. the symbols  , ©, @, @ and  are used with the following
P $ Q means P is not greater than Q. meanings:
P @ Q means P is neither smaller than nor equal to Q. P  Q means P is not smaller than Q.
P + Q means P is neither greater than nor equal to Q. P © Q means P is neither greater than nor smaller than Q.
Now in each of the following questions, assuming P @ Q means P is not greater than Q.
the given statements to be true, find which of the two con- P @ Q means P is neither smaller than nor equal to Q.
P  Q means P is neither greater than nor equal to Q. P © Q means P is neither greater than nor smaller than Q.
Now in each of the following questions, assuming P @ Q means P is not greater than Q.
the given statements to be true, find which of the two con- P @ Q means P is neither smaller than nor equal to Q.
clusions I and II given below them is/are definitely true. Give P  Q means P is neither greater than nor equal to Q.
answer Now in each of the following questions, assuming
1) if only conclusion I is true; the given statements to be true, find which of the two con-
2) if only conclusion II is true; clusions I and II given below them is/are definitely true. Give
3) if either I or II is true; answer
4) if neither I nor II is true; and 1) if only conclusion I is true;
5) if both I and II are true. 2) if only conclusion II is true;
91. Statements: Q © T, B  Q, T  A 3) if either I or II is true;
Conclusions: I. T @ B II. A @ Q 4) if neither I nor II is true; and
92. Statements: U @ S, W  S, S @ O 5) if both I and II are true.
Conclusions: I. U  W II. W  O 101. Statements: A@L, L  K, K@Z
93. Statements: H © K, J @ K, R @ H Conclusions: I. Z © L II. A©K
102. Statements: O@A, U©O, A©E
Conclusions: I. J @ H II. R  J
Conclusions: I. U@A II. E@U
94. Statements: W © Z, M @ L, Z  L 103. Statements: P@T, G  P, F©G
Conclusions: I. M  Z II. L @ W Conclusions: I. F©P II. F@T

K
95. Statements: V  P, C © D, P @ D 104. Statements: W@E, P@E, V@W
Conclusions: I. V @ D II. C  P Conclusions: I. E©V II. E@V
Directions (Q. 96-100): In the following questions 105. Statements: M  L, O©L, J  L
the symbols #, *, @, $ and = are used with the following Conclusions: I. M@J II. O  J
meanings: 106. Statements: H@I, K@L, I©L
A # B means A is neither smaller than nor equal to B. Conclusions: I. I@K II. K©I
A * B means A is not smaller than B. Directions (Q. 107-111): In the following questions,
A @ B means A is neither smaller than nor greater than B.
the symbols  ,  ,  ,  and  are used with the following
A $ B means A is neither greater than nor equal to B.
A = B means A is not greater than B. meanings:

KUNDAN
Now in each of the following questions, assuming P  Q means P is not smaller than Q.
the three statements to be true, find which of the two conclu- P  Q means P is neither greater than nor smaller than Q.
sions I and II given below them is/are definitely true. Give P  Q means P is not greater than Q.
answer P  Q means P is neither smaller than nor equal to Q.
1) if only conclusion I is true.
P  Q means P is neither greater than nor equal to Q.
2) if only conclusion II is true.
3) if either conclusion I or conclusion II is true. Now in each of the following questions, assuming
4) if neither conclusion I nor conclusion II is true. the given statements to be true, find which of the two con-
5) if both conclusions I and II are true. clusions I and II given below them is/are definitely true. Give
96. Statements: K # L, K @ E, L $ F answer
Conclusions: I. L # E II. F * L 1) if only conclusion I is true;
97. Statements: P@ H, P @ J, P = K 2) if only conclusion II is true;
Conclusions: I. J = K II. J @ H 3) if either I or II is true;
98. Statements: 3 * 4, 5 $ 6, 5 = 4 4) if neither I nor II is true; and
Conclusions: I. 3 * 5 II. 6 $ 5 5) if both I and II are true.
99. Statements: Q @ R, S # R, S * T 107. Statements: X  F, T  C, F  C
Conclusions: I. R $ S II. Q # S Conclusions: I. X  T II. C  X
100. Statements: U @ W, W $ Z, W = Y 108. Statements: 2  8, 6  8, 5  6
Conclusions: I. Z $ Y II. Z * Y
Conclusions: I. 5  8 II. 5  2
Directions (Q. 101-106): In the following questions,
the symbols  , ©, @, @and  are used with the following 109. Statements: S  N, O  R, S  R
meanings: Conclusions: I. N  O II. O  S
P  Q means P is not smaller than Q.
110. Statements: J  Y, J  O, Y  K 118. Statements: K=L, ML=N, J@L
Conclusions: I. LN II. K=J
Conclusions: I. O  K II. O  K
119. Statements: I@H, F=G, GH
111. Statements: D  B, R  L, L  D Conclusions: I. G  I II. GI
Conclusions: I. R  D II. L  B 120. Statements: N=M, O  P, MO
Directions (Q. 112-116): In the following questions, Conclusions: I. P@N II. M=P
the symbols *, $, ,  and  are used with the following 121. Statements: D@E, E=F, F=G
meanings: Conclusions: I. D@G II. FG
P * Q means P is greater than Q. Directions (Q. 122-126): In the following questions,
P $ Q means P is either greater than or equal to Q. the symbols £, $, @,  and > are used with the following
P  Q means P is equal to Q. meanings:
P  Q means P is smaller than Q. P £ Q means P is greater than Q.
P  Q means P is either smaller than or equal to Q. P $ Q means P is either greater than or equal to Q.
Now in each of the following questions, assuming P @ Q means P is equal to Q.
the given statements to be true, find which of the two con- P  Q means P is smaller than Q.
clusions I and II given below them is/are definitely true. Give P > Q means P is either smaller than or equal to Q.
answer Now in each of the following questions, assuming
1) if only conclusion I is true; the given statements to be true, find which of the two con-
clusions I and II given below them is/are definitely true. Give

K
2) if only conclusion II is true;
3) if either I or II is true; answer
4) if neither I nor II is true; and 1) if only conclusion I is true;
5) if both I and II are true. 2) if only conclusion II is true;
112. Statements: I$R, Q  C, D  R, QI 3) if either I or II is true;
Conclusions: I. C*R II. D I 4) if neither I nor II is true; and
5) if both I and II are true.
113. Statements: PL, DK, P  D, Q*L
122. Statements: J > R, P @ R > L, H $ R
Conclusions: I. P*K II. P  Q Conclusions: I. R £ L II. J > H
114. Statements: XY, N  X, DO, D*Y 123. Statements: G $ F, D > S, S @ F
Conclusions: I. O*Y II. N  Y Conclusions: I. S  G II. S @ G

KUNDAN
115. Statements: L$J, QJ, L  M, QD 124. Statements: Y > H, P  N, H @ P
Conclusions: I. L*Q II. LQ Conclusions: I. N £ Y II. H > N
116. Statements: K$L, J*O, K  T, O  L 125. Statements: M £ N, N > Q, Q > D
Conclusions: I. OK II. J*L Conclusions: I. M £ D II. Q @ D
Directions (Q. 117-121): In the following questions, 126. Statements: V £ X, U > Z, X @ U
the symbols @, @,,  and = are used with the following Conclusions: I. V £ Z II. X @ Z
meanings: Directions (Q. 127-131): In the following questions,
P @ Q means P is greater than Q. the symbols @, @,  ,  and  are used with the following
P @ Q means P is either greater than or equal to Q. meanings:
P  Q means P is equal to Q. P  Q means P is greater than Q.
P  Q means P is smaller than Q. P @ Q means P is either greater than or equal to Q.
P  Q means P is equal to Q.
P = Q means P is either smaller than or equal to Q.
P @ Q means P is neither greater than nor equal to Q.
Now in each of the following questions, assuming
P  Q means P is either smaller than or equal to Q.
the given statements to be true, find which of the two con-
Now in each of the following questions assuming the
clusions I and II given below them is/are definitely true. Give
given statements to be true, find which of the two conclu-
answer
sions I and II given below them is/are definitely true. Give
1) if only conclusion I is true;
answer
2) if only conclusion II is true;
1) if only conclusion I is true;
3) if either I or II is true;
2) if only conclusion II is true;
4) if neither I nor II is true; and
3) if either I or II is true;
5) if both I and II are true.
4) if neither I nor II is true; and
117. Statements: A@M, D=H, MD
5) if both I and II are true.
Conclusions: I. A@H II. MH
127. Statements: D  F, E  B, F  E 4) if neither conclusion I nor II is true.
Conclusions: I. D  B II. F  B 5) if both conclusions I and II are true.
128. Statements: K  B, B  C, C  M 137. Statements: E £ F, O  F, P @ E
Conclusions: I. K  M II. C  M Conclusions: I. F * P II. P £ F
129. Statements: R  S, N @ O, S  N 138. Statements: C * P, P @ D, C $ G
Conclusions: I. O  R II. S  O Conclusions: I. G £ D II. P * G
130. Statements: H @ P, R  F, F  P 139. Statements: S @ T, Q $ N, T £ N
Conclusions: I. F @ H II. F  H Conclusions: I. S £ N II. S $ Q
131. Statements: T V, U  V  M, I  V 140. Statements: L * K, V  K, R $ L
Conclusions: I. V  M II. T @ I Conclusions: I. V £ L II. V $ L
Directions (Q. 132-136): In the following questions,
141. Statements: H  J, J * Z, M  J
the symbols @, #,  , © and  are used with the following
Conclusions: I. H * M II. Z £ M
meanings:
Directions (Q. 142-146): In the following questions,
P @ Q means P is not smaller than Q.
the symbols =, >, +, < and × are used with the following
P # Q means P is neither greater than nor smaller than Q.
meanings:
P  Q means P is not greater than Q.
P = Q means P is not smaller than Q.
P © Q means P is neither smaller than nor equal to Q.
P > Q means P is neither greater than nor smaller than Q.
P  Q means P is neither greater than nor equal to Q. P + Q means P is not greater than Q.
Now in each of the following questions, assuming P < Q means P is neither smaller than nor equal to Q.

K
the given statements to be true, find which of the two con- P × Q means P is neither greater than nor equal to Q.
clusions I and II given below them is/are definitely true. Give Now in each of the following questions, assuming
answer the given statements to be true, find which of the two con-
1) if only conclusion I is true; clusions I and II given below them is/are definitely true. Give
2) if only conclusion II is true; answer
3) if either I or II is true; 1) if only conclusion I is true;
4) if neither I nor II is true; and 2) if only conclusion II is true;
5) if both I and II are true. 3) if either I or II is true;
132. Statements: R©K, P@S, P#K 4) if neither I nor II is true; and
Conclusions: I. S#K II. S  R 5) if both I and II are true.

KUNDAN
133. Statements: A#E, I©O, U  A, I@U 142. Statement: A = B, P + R, A × P
Conclusions: I. I©A II. E#O Conclusions: I. B < P II. R = B
134. Statements: Q#R, T  L, R  M, L  Q 143. Statement: N < O, E > F, M × O
Conclusions: I. M©T II. T  R Conclusions: I. M < N II. E > O
135. Statements: Z  N, D  G, B  Z, D#N 144. Statement: K > L, G = I, Y + Q
Conclusions: I. G©Z II. N  B Conclusions: I. G × L II. G = L
145. Statements: R = U, B < R, K + U
136. Statements: H@Y, E@C, K  Y, K©C
Conclusions: I. U × B II. B < K
Conclusions: I. H  E II. E  H 146. Statements: M > H, 2 × H, 2 + N
Directions (Q. 137-141): In the questions given be- Conclusions: I. M × 2 II. M < N
low, certain symbols are used with the following meanings: Directions (Q. 147-151): In the following questions,
P * Q means P is neither equal to nor smaller than Q. the symbols +,  , =, * and @ are used with the following
P  Q means P is not smaller than Q. meanings:
P $ Q means P is neither greater nor smaller than Q. P + Q means P is not smaller than Q.
P £ Q means P is neither greater than nor equal to Q. P  Q means P is neither greater than nor smaller than Q.
P @ Q means P is not greater than Q. P = Q means P is not greater than Q.
Now in each of the following questions, assuming P * Q means P is neither smaller than nor equal to Q.
the given statements to be true, find which of the two con- P @ Q means P is neither greater than nor equal to Q.
clusions I and II given below them is/are definitely true. Give Now in each of the following questions, assuming
answer the given statements to be true, find which of the two con-
1) if only conclusion I is true. clusions I and II given below them is/are definitely true. Give
2) if only conclusion II is true. answer
3) if either conclusion I or II is true. 1) if only conclusion I is true;
2) if only conclusion II is true; 3) if either conclusion I or II is true.
3) if either I or II is true; 4) if neither conclusion I nor II is true.
4) if neither I nor II is true; and 5) if both conclusions I and II are true.
5) if both I and II are true. 157. Statements: N $ P, QP, M  N
147. Statements: B + F, P @ R, M  F, M = P Conclusions: I. M@P II. N$Q
Conclusions: I. B + M II. M @ R 158. Statements: N  U, OE, E@N
148. Statements: E = H, 6 * P, R @ E, 6  R Conclusions: I. E@U II. U£O
Conclusions: I. E * P II. H * P 159. Statements: M£P, Z$H, M  Z
149. Statements: M @ N, R  Q, A * Q, M * R Conclusions: I. P@Z II. M@H
Conclusions: I. A * R II. N = Q 160. Statements: U  V, V@X, W$U
150. Statements: B @ D, E = T, T + P, P * B Conclusions: I. W$V II. W@X
Conclusions: I. P = D II. P * D 161. Statements: D£E, E@F, F  G
151. Statements: P = Q, N  M, M * R, R + P Conclusions: I. D$G II. E@G
Conclusions: I. P * N II. Q = M Directions (Q 162-166): In the questions given below,
Directions (Q. 152-156): In the following questions, certain symbols are used with the following meanings:
the symbols, *, # $, @ and + are used with the following P  Q means P is neither equal to nor smaller than Q.
meanings: P  Q means P is not smaller than Q.
P * Q means P is not smaller than Q. P $ Q means P is neither greater nor smaller than Q.
P # Q means P is neither greater than nor smaller than Q. P £ Q means P is neither greater than nor equal to Q.

K
P $ Q means P is not greater than Q. P @ Q means P is not greater than Q.
P @ Q means P is neither smaller than nor equal to Q. Now in each of the following questions, assuming the
P + Q means P is neither greater than nor equal to Q. given statements to be true, find which of the two conclu-
Now in each of the following questions, assuming the sions I and II given below them is/are definitely true. Give
given statements to be true, find which of the two conclu- answer
sions I and II given below them is/are definitely true. Give 1) if only conclusion I is true.
answer 2) if only conclusion II is true.
1) if only conclusion I is true; 3) if either conclusion I or II is true.
2) if only conclusion II is true; 4) if neither conclusion I nor II is true.
3) if either I or II is true; 5) if both conclusions I and II are true.
4) if neither I nor II is true; and 162. Statements: TU, Q@R, R £ U

KUNDAN
5) if both I and II are true. Conclusions: I. TQ II. R@T
152. Statements: P @ Q, Q * R, R # S, T + S 163. Statements: H£B, JC, HJ
Conclusions: I. T + P II. S $ Q Conclusions: I. BC II. C@H
153. Statements: S $ A, A + K, R * K, L * R 164. Statements: Q£T, WX, QX
Conclusions: I. S $ L II. L * A Conclusions: I. W$Q II. X£T
154. Statements: G @ H, F + H, K * P, M $ P 165. Statements: Y$Z, BA, Y£A
Conclusions: I. K @ M II. K # M Conclusions: I. AZ II. Y£B
155. Statements: A + P, Z + A, N$Z, L$N 166. Statements: KL, N@O, L$O
Conclusions: I. P@L II. P#L Conclusions: I. LN II. N$L
156. Statements: A@P, Z@A, N*Z, L*N Directions (Q. 167-171): In the following questions,
Conclusions: I. P#L II. P+L the symbols  and # are used with the following meanings:
Directions (Q. 157-161): In the following questions the  PQ means P is not smaller than Q.
PQ means P is neither greater than nor smaller than Q.
symbols @, $, £,  and  are used with the following mean-
P#Q means Q is neither greater than nor smaller than Q.
ings: PQ means P is not greater than Q.
P @ Q means P is either equal to or smaller than Q. #PQ means P is neither smaller than nor equal to Q.
P $ Q means P is neither greater than nor smaller than Q. PQ# means P is neither greater than nor equal to Q.
P £ Q means P is neither greater than nor equal to Q. Now in each of the following questions, assuming the
P  Q means P is either greater than or equal to Q. given statements to be true, find which of the two conclu-
P  Q means P is not equal to Q. sions I and II given below them is/are definitely true. Give
Now in each of the following questions, assuming the answer
given statements to be true, find which of the two conclu- 1) if only conclusion I is true.
sions I and II given below them is/are definitely true. Give 2) if only conclusion II is true.
answer 3) if either conclusion I or II is true.
1) if only conclusion I is true. 4) if neither conclusion I nor II is true.
2) if only conclusion II is true. 5) if both conclusions I and II are true.
167. Statements: #XF, TC, FC 179. Statements: T © O, O © P, P – A, A × Z
Conclusions: I. #XT II. CX Conclusions: I. P © T II. O © A
168. Statements: #28, 68, 56# 180. Statements: D × E, S ÷ E, S – X, M @ S
Conclusions: I. 58 II. 5#2 Conclusions: I. D @ M II. D + M
169. Statements: SN# OR, SR 181. Statements: A + P, A @ T, T – N, N × S
Conlusions: I. #NO II. OS Conclusions: I. P × S II. P + S
170. Statements: JY#, JO, Y#K Directions (Q. 182-186): In the questions given below,
Conlusions: I. OK II. OK# certain symbols are used with the following meanings:
171. Statements: DB, RL#, #LD P @ Q means P is neither equal to nor smaller than Q.
Conclusions: I. #RD II. #LB P × Q means P is not smaller than Q.
Directions (Q. 172-176): In the questions given below, P – Q means P is neither greater nor smaller than Q.
certain symbols are used with the following meanings: P ÷ Q means P is neither greater than nor equal Q.
A @ B means A is greater than B. P + Q means P is not greater than Q.
A * B means A is either greater than or equal to B. P © Q P is not equal to Q.
A # B means A is equal to B. Now in each of the following questions, assuming the
A $ B means A is either smaller than or equal to B. given statements to be true, find which of the two conclu-
A + B means A is smaller than B. sions I and II given below them is/are definitely true. Give
Now in each of the following questions, assuming the Answer
given statements to be true, find which of the two conclu- 1) if only conclusion I is true.
sions I and II given below them is/are definitely true. Give 2) if only conclusion II is true.
answer 3) if either conclusion I or II is true.

K
1) if only conclusion I is true. 4) if neither conclusion I nor II is true.
2) if only conclusion II is true. 5) if both conclusions I and II are true.
3) if either conclusion I or II is true. 182. Statements: G@H, T+H, T-K, K©Z
4) if neither conclusion I nor II is true. Conclusions: I. G×Z II. G+Z
5) if both conclusions I and II are true. 183. Statements: N©U, U+M, N×B, M–R
172. Statements: B*I, Y+A, N$I, Y@N Conclusions: I. R×U II. M×B
Conclusions: I. B @ A II. Y + I 184. Statements: P÷M, R×M, S–R, S@Y
173. Statements: M*A, P$O, A*N, N$P Conclusions: I. R@Y II. P÷Y
Conclusions: I. M # N II. O # N 185. Statements: L©M, M@N, R÷N, R×F
174. Statements: Q$P, T#Q, T*N, N@J Conclusions: I. M@F II. F÷N
Conclusions: I. P @ N II. P # N 186. Statements: G@H, I÷H, P×Q, Q@R

KUNDAN
175. Statements: L#K, S#K, L*B, R$B Conclusions: I. G÷I II. P©R
Conclusions: I. R + K II. R # S Directions (Q. 187-192): In the following questions,
176. Statements: H@T, T@D, G$F, G*P the symbols !, @, #, $ and * are used with the following
Conclusions: I. P $ F II. P # F meanings:
Directions (Q. 177-181): In the questions given below, P!Q means P is neither smaller than nor equal to Q.
certain symbols are used with the following meanings: P@Q means P is not smaller than Q.
P @ Q means P is neither equal to nor smaller than Q. P#Q means P is neither greater than nor smaller than Q.
P × Q means P is not smaller than Q. P$Q means P is neither greater than nor equal to Q.
P – Q means P is neither greater nor smaller than Q. P*Q means P is not greater than Q.
P ÷ Q means P is neither greater than nor equal to Q. Now in each of the following questions, assuming the
P + Q means P is not greater than Q. given statements to be true, find which of the two conclu-
P © Q P is not equal to Q. sions I and II given below them is/are definitely true. Give
Now in each of the following questions, assuming answer
the given statements to be true, find which of the two con- 1) if only conclusion I is true.
clusions I and II given below them is/are definitely true. Give 2) if only conclusion II is true.
answer 3) if either conclusion I or II is true.
1) if only conclusion I is true. 4) if neither conclusion I nor II is true.
2) if only conclusion II is true. 5) if both I and II are true.
3) if either conclusion I or II is true. 187. Statements: N!O, O#Q, R$Q, O*S
4) if neither conclusion I nor II is true. Conclusions: I. N!R II. Q#S
5) if both conclusions I and II are true. 188. Statements: T@U, U!Q, Q$S, S$U
177. Statements: J – O, T + O, T © Y, E × J Conclusions: I. T!S II. T!Q
Conclusions: I. E © T II. E @ T 189. Statements: C$D, D#E, E!F, F!G
178. Statements: T ÷ R, P @ A, R + A, L – T Conclusions: I. D!G II. C!G
Conclusions: I. L ÷ P II. A @ L 190. Statements: K@L, L!I, I$M, I!N
Conclusions: I. L$M II. K!N
191. Statements: V*X, X$Y, Y#Z, Y!A 202. Statements: T @ I, R # T, N £ I, S @ I
Conclusions: I. Y!V II. X!A Conclusions: I. N $ S II. N © S
192. Statements: L!M, M!N, L@O, P*O Directions (Q. 203-207): In the following questions,
Conclusions: I. N$L II. P*L the symbols !, @, #, $ and  are used with the following
Directions (Q. 193-197): In the questions given below, meanings:
certain symbols are used with the following meanings: P!Q means P is neither smaller than nor equal to Q.
P @ Q means P is neither equal to nor smaller than Q. P@Q means P is not smaller than Q.
P × Q means P is not smaller than Q. P#Q means P is neither greater than nor smaller than Q.
P$Q means P is neither greater than nor equal to Q.
P - Q means P is neither greater nor smaller than Q.
PQ means P is not greater than Q.
P ÷ Q means P is neither greater than nor equal to Q. Now in each of the following questions, assuming the
P + Q means P is not greater than Q. given statements to be true, find which of the two conclu-
Now in each of the following questions, assuming the sions I and II given below them is/are definitely true. Give
given statements to be true, find which of the two conclu- answer
sions I and II given below them is/are definitely true. Give 1) if only conclusion I is true.
answer 2) if only conclusion II is true.
1) if only conclusion I is true. 3) if either conclusion I or II is true.
2) if only conclusion II is true. 4) if neither conclusion I nor II is true.
3) if either conclusion I or II true. 5) if both I and II are true.
4) if neither conclusion I nor II is true. 203. Statements: G@H, TS, H#O, O!S
Conclusions: I. SG II. H@T

K
5) if both conclusions I and II are true.
204. Statements: M@E, A#P, LP, A$M
193. Statements: B @ C, A @ B, C × D Conclusions: I. E!A II. EP
Conclusions: I. A × D II. B @ D 205. Statements: Z!E, RZ, E@B, A$R
194. Statements: M - N, N + P, P × R Conclusions: I. E!R II. B$A
Conclusions: I. P × M II. R + P 206. Statements: KT, S@G, G#H#T
195. Statements: X @ R, R ÷ S, X - T Conclusions: I. S!K II. K#S
Conclusions: I. T × S II. T ÷ S 207. Statements: P@S, AD, A!I, I#S
196. Statements: R @ M, M - Y, Y × Z Conclusions: I. P@I II. S$D
Conclusions: I. Z ÷ R II. Z @ R Directions (Q. 208-212): In the following questions,
197. Statements: T - Y, P × X, P + Y the symbols !, @, #, $ and  are used with the following
Conclusions: I. P - X II. P - Y meanings:

KUNDAN
Directions (Q. 198-202): In the following questions, P!Q means P is neither smaller than nor equal to Q.
P@Q means P is not smaller than Q.
the symbols #, @, ©, $, £ are used with the following mean-
P#Q means P is neither greater than nor smaller than Q.
ings:
P # Q means P is greater than Q. P$Q means P is neither greater than nor equal to Q.
P @ Q means P is either greater than or equal to Q. PQ means P is not greater than Q.
Now in each of the following questions, assuming the
P © Q means P is equal to Q.
P $ Q means P is smaller than Q. given statements to be true, find which of the two conclu-
P £ Q means P is either smaller than or equal to Q. sions I and II given below them is/are definitely true. Give
Now in each of the following questions, assuming the answer
given statements to be true, find which of the two conclu- 1) if only conclusion I is true.
2) if only conclusion II is true.
sions I and II given below them is/are definitely true. Give
answer 3) if either conclusion I or II is true.
1) if only conclusion I is true; 4) if neither conclusion I nor II is true.
2) if only conclusion II is true; 5) if both I and II are true.
3) if either I or II is true; 208. Statements: P ! Q, M K, T K
Conclusions: I. T ! M II. T # M
4) if neither I nor II is true; and
209. Statements: S $ M, M ! L, L @ Z
5) if both I and II are true.
Conclusions: I. S # Z II. S ! Z
198. Statements: K © Y, N © I, I @ K, N £ P 210. Statements: D ! F, F # S, S  M
Conclusions: I. P # I II. Y $ I Conclusions: I. F ! M II. F @ M
199. Statements: V # I, C $ I, C © Y, K @ C 211. Statements: J # V, V $ N, R J
Conclusions: I. Y £ K II. I # K Conclusions: I. N ! R II. J @ N
200. Statements: K @ N, S # T, I $ N, T £ I
212. Statements: L @ U, C $ L, C ! B
Conclusions: I. T £ K II. T $ K Conclusions: I. C @ U II. CU
201. Statements: L © E, E # P, P © R, R @ D Directions (Q. 213-217): In the following questions the
Conclusions: I. D $ L II. D © L symbols +, ×, ?, @ and $ are used with the following mean-
ing: P # Q means P is not equal to Q.
A + B means A is neither smaller nor greater than B. Now in each of the following questions, assuming the
A × B means A is neither equal to nor smaller than B. given statements to be true, find which of the two conclu-
A ? B means A is neither greater nor equal to B. sions I and II given below them is/are definitely true. Give
A @ B means A is either greater or equal to B. answer
A $ B means A is not equal to B. 1) if only conclusion I is true.
Now, in each of the following questions, assuming the 2) if only conclusion II is true.
given statements to be true, find which of the two conclu- 3) if either conclusion I or II is true.
sions I and II given below them is/are definitely true. Give 4) if neither conclusion I nor II is true.
answer 5) if both conclusions I and II are true.
1) if only conclusion I is true. 223. Statements: S+P, LQ, P#R, R$Q
2) if only conclusion II is true. Conclusions: I. LS II. QP
3) if either conclusion I or II is true. 224. Statements: MT, GT, GH, TK
4) if neither conclusion I nor II is true. Conclusions: I. HK II. HM
5) if both conclusions I and II are true. 225. Statements: TF, TP, G#T, T#P
213. Statements: R@P, B?P, E+B, F×B Conclusions: I. G#P II. G$P
Conclusions: I. P?F II. R×F 226. Statements: K$R, R#T, T#Z, Z$S
214. Statements: G$H, L+K, H+E, M$K Conclusions: I. K + T II. KT
Conclusions: I. G@E II. L@M 227. Statements: Q$M, M+N, N$P, LP
215. Statements: B@K, T?B, T@K, K×S Conclusions: I. Q+L II. P*M
Conclusions: I. T@S II. T×S Directions (Q. 228-232): In the questions given below

K
216. Statements: F×K, P+K, P@H, T?P certain symbols are used with the following meaning:
Conclusions: I. T?F II. H?F In the following questions, the symbols  ,  ,  ,  and
217. Statements: D$L, L+P, P$T, T@Z  are used with the following meanings:
Conclusions: I. D$T II. T+D A  B means A is neither greater than nor equal to B.
Directions (Q. 218-222): In the questions given below, A  B means A is neither greater than nor smaller than B.
certain symbols are used with the following meanings: A  B means A is not greater than B.
P + Q means P is neither smaller nor equal to Q.
A  B means A is not smaller than B.
P  D means P is not smaller than Q.
P $ Q means P is neither greater nor smaller than Q. A  B means A is neither smaller than nor equal to B.
P  Q means P is neither greater nor equal to Q. Now in each of the following questions, assuming the
P  Q means P is not greater than Q. given statements to be true, find which of the two conclu-
sions I and II given below them is/are definitely true. Give

KUNDAN
Now in each of the following questions, assuming the
given statements to be true, find which of the two conclu- answer
sions I and II given below them is/are definitely true. Give 1) if only conclusion I is true.
answer 2) if only conclusion II is true.
1) if only conclusion I is true. 3) if either conclusion I or II is true.
2) if only conclusion II is true. 4) if neither conclusion I nor II is true.
3) if either conclusion I or II is true. 5) if both conclusions I and II are true.
4) if neither conclusion I nor II is true. 228. Statements: M  N, L  M, L  O
5) if both conclusions I and II are true. Conclusions: I. O  N II. O  N
218. Statements: F + A, I  R, I  A, I + L
Conclusions: I. L  R` II. F + L 229. Statements: A  C, P  Q, Q  R
219. Statements: P  M, K P, S  P, SG Conclusions: I. A  R II. P  R
Conclusions: I. M + G II. G  K
220. Statements: NT, G$L$T, S  L 230. Statements: W  X  V, X  Y, Y  Z
Conclusions: I. N + L II. N $ L Conclusions: I. Z  V II. W  V
221. Statements: M$P, SM, SZ, TP
Conclusions: I. PZ II. PZ 231. Statements: M  N, N  O, O  P
222. Statements: H  K, T  H, K $ F, S $ T Conclusions: I. M  P II. O  M
Conclusions: I. K  T II. F  S
Directions (Q. 223-227): In the questions given below, 232. Statements: R  A, S  B, A  B
certain symbols are used with the following meanings:
Conclusions: I. S  R II. B  R
P + Q means P is neither smaller nor equal to Q.
P  D means P is not smaller than Q. Directions (Q. 233-237): In the following questions
P $ Q means P is neither greater nor smaller than Q. the symbols @, @, =,  and  are used with the following
P  Q means P is neither greater nor equal to Q.
meanings:
P  Q means P is not greater than Q.
A @ B means A is greater than B, Directions (Q. 243-247) : In the following questions;
A @ B means A is either greater than or equal to B, the symbols m, m  , n , n  and  are used as follows:
A = B means A is equal to B, A m B means A is greater than B
A  B means A is smaller than B, and A m  B means A is greater than or equal to B
A  B means A is either smaller than or equal to B. A  B means A is equal to B
A n B means A is less than B
Now in each of the following questions, assuming
A n  B means A is less than or equal to B.
the three statements to be true, state which of the two con-
Now, assume the three given statements to be defi-
clusions I and II given below them is definitely true. Give
nitely true in each of the given questions. Then decide which
answer
of the given conclusions are definitely true. Give answer
1) if only conclusion I is true;
1) if only conclusion I follows
2) if only conclusion II is true;
2) if only conclusion II follows
3) if either I or II is true;
3) if either conclusion I or II follows
4) if neither I nor II is true; and
4) if neither I nor II follows
5) if both I and II are true.
5) if both I and II follow
233. Statements: P = Q, R @ P, Q  S 243. Statements: B m C, A m B , C m  D
Conclusions: I. S @ P II. S = P Conclusions I. A m  D II. B m D
234. Statements: X @ Y, Y = R, Y  S 244. Statements: M  N, N n  P, P m  R
Conclusions: I. X @ R II. R  Y Conclusions I. P m  M II. R n  P

K
235. Statements: P  Q, Q  R, R = S. 245. Statements: X m R, R n S, X  T
Conclusions: I. P  S II. S @ Q Conclusions I. T m  S II. T n S
236. Statements: A @ B, B  C, C @ D 246. Statements: R m M, M  Y, Y m  Z
Conclusions I. Z n R II. Z m A
Conclusions: I. A = D. II. A  D.
247. Statements: X  Y, P m  X, P n  Y
237. Statements: P  Q, R @ S, Q = R
Conclusions I. P  X II. P  Y
Conclusions: I. P  R II. P @ R
Directions (Q. 248-252): In the following question
Directions (Q. 238-242): In the following question
the symbols #, *, @, $ and = are used with the following
the symbols #, *, @, $ and = are used with the following
meanings:
meanings:
A # B means A is greater than B.
A # B means A is greater than B.

KUNDAN
A * B means A is greater than or equal to B.
A * B means A is greater than or equal to B.
A @ B means A is equal to B.
A @ B means A is equal to B.
A $ B means A is lesser than B.
A $ B means A is lesser than B.
A = B means A is lesser than or equal to B.
A = B means A is lesser than or equal to B.
Now in each of the following questions, assuming
Now in each of the following questions, assuming
the three statements to be true, find which of the two conclu-
the three statements to be true, find which of the two conclu-
sions I and II given below them is/are definitely true. Give
sions I and II given below them is/are definitely true. Give
answer
answer
1) if only conclusion I is true.
1) if only conclusion I is true.
2) if only conclusion II is true.
2) if only conclusion II is true.
3) if either conclusion I or conclusion II is true.
3) if either conclusion I or conclusion II is true.
4) if neither conclusion I nor conclusion II is true.
4) if neither conclusion I nor conclusion II is true.
5) if both conclusions I and II are true.
5) if both conclusions I and II are true.
248. Statements: S $ M, M # L, L * Z
238. Statements: E # F, F @ G, H * G
Conclusions: I. S @ Z II. S = L
Conclusions: I. G $ F II. G $ E
249. Statements: J @ L, V $ N, R = J
239. Statements: A @ B, A * C, C # D
Conclusions: I. R $ N II. J * N
Conclusions: I. B = C II. A @ D
250. Statements: L * U, C $ L, C # B
240. Statements: O * P, O # Q, O = R
Conclusions: I. U @ C II. L # B
Conclusions: I. P $ R II. R @ P
251. Statements: D # F, F @ S, S = M
241. Statements: N = O, P $ O, P # R
Conclusions: I. D # M II. F * M
Conclusions: I. N $ O II. O = R
252. Statements: P # T, M = K, T @ K
242. Statements: L $ M, N * M, M * O
Conclusions: I. T # M II. T @ M
Conclusions: I. L $ N II. N * O
Directions (Q. 253-257): In the following question 262. Statements: QR, PQ, BR, GP
the symbols #, *, @, $ and = are used with the following Conclusions: I. GB II. PR
meanings: Directions (Q. 263-267): In the following questions,
A # B means A is greater than B. the symbols , , ,  and  are used with the following
A * B means A is greater than or equal to B. meanings.
A @ B means A is equal to B. PQ means P is not smaller than Q.
A $ B means A is lesser than B., PQ means P is neither greater than nor smaller than Q.
PQ means P is not greater than Q.
A = B means A is lesser than or equal to B.
PQ means P is neither smaller than nor equal to Q.
Now in each of the following questions, assuming PQ means P is neither greater than nor equal to Q.
the three statements to be true, find which of the two conclu- Now in each of the following questions, assuming
sions I and II given below them is/are definitely true. Give the given statements to be true, find which of the two con-
answer clusions I and II given below them is/are definitely true. Give
1) if only conclusion I is true. answer
2) if only conclusion II is true. 1) if only conclusion I is true;
3) if either conclusion I or conclusion II is true. 2) if only conclusion II is true;
4) if neither conclusion I nor conclusion II is true. 3) if either I or II is true;
5) if both conclusions I and II are true. 4) if neither I nor II is true; and
253. Statements: S = T, T * U, T $ V 5) if both I and II are true.
263. Statements: LM, SM, MN, QL
Conclusions: I. T $ S II. U = V
Conclusions: I. NQ II. QN

K
254. Statements: K # L, K * M, M $ N 264. Statements: SG, RJ, KR, KS
Conclusions: I. L @ M II. M @ K Conclusions: I. RG II. JS
255. Statements: F = G, F * H, F = K 265. Statements: MG, TK, SG, KS
Conclusions: I. G = K II. K $ H Conclusions: I. MT II. TS
256. Statements: T @ S, R $ Q, Q @ T 266. Statements: NS, GS, QTG, LN
Conclusions: I. Q * S II. S # R Conclusions: I. GL II. GL
257. Statements: B # C, C $ D, E $ C 267. Statements: PS, TS, KN, KM
Conclusions: I. D # B II. D = C Conclusions: I. PT II. NM
Directions (Q. 258-262): In the following questions, Directions (Q. 268-272): In the question given below
the symbols , , ,  and  areusedwiththe following certain symbols are used with the following meanings:
meanings: A  B mean A is neither smaller than nor equal to B.

KUNDAN
A  B mean A is not smaller than B.
P  Q means P is not smaller than Q.
A  B mean A is neither greater than nor equal to B.
P  Q means P is neither greater than nor smaller than Q. A  B means A is not greater than B.
P  Q means P is not greater than Q. A  B means A is equal to B.
P  Q means P is neither smaller than nor equal to Q. A  B means A is not equal to B.
P  Q means P is neither greater than nor equal to Q. Now in each of the following questions, assuming
Now in each of the following questions, assuming the given statements to be true, find which of the two con-
the given statements to be true, find which of the two con- clusions I and II given below them is/are definitely true. Give
clusions I and II given below them is/are definitely true. Give answer
answer 1) if only conclusion I is true.
1) if only conclusion I is true; 2) if only conclusion II is true.
2) if only conclusion II is true; 3) if both I and II are true.
4) if neither I nor II is true.
3) if either I or II is true;
5) if either I or II is true.
4) if neither I nor II is true; and 268. Statements: LM, PN, SP, MN
5) if both I and II are true. Conclusions: I. PM II. MS
258. Statements: PL, NQ, MN, ML 269. Statements: GP, QM, ZM, QP
Conclusions: I. PN II. NP Conclusions: I. GP II. GQ
259. Statements: RL, ML, LC, MP 270. Statements: RL, YT, YS, RS
Conclusions: I. LP II. RP Conclusions: I. SL II. YL
260. Statements: RL, GP, LQ, PQ 271. Statements: PG, RS, TR, LG
Conclusions: I. GL II. RG Conclusions: I. PT II. PT
261. Statements: TS, MH, TQ, GH 272. Statements: ED, FG, GH, FE
Conclusions: I. SG II. SG Conclusions: I. DG II. HE
Directions (Q. 273-277): In the following questions,
the symbols @, #, $, £ and © are used with the following meaning:
meanings: P  Q means P is greater than Q.
P # Q means P is not smaller than Q. P © Q means P is either greater than or equal to Q.
P $ Q means P is neither greater than nor smaller than Q. P = Q means P is equal to Q.
P © Q means P is not greater than Q. P @ Q means P is smaller than Q.
P @ Q means P is neither smaller than nor equal to Q. P @ Q means P is either smaller than or equal to Q.
P £ Q means P is neither greater than nor equal to Q. Now in each of the following questions assuming the
Now in each of the following questions, assuming given statements to be true, find which of the two conclu-
the given statements to be true, find which of the two con- sions I and II given below them is/are definitely true. Give
clusions I and II given below them is/are definitely true. Give answer
answer 1) if only conclusion I is true;
1) if only conclusion I is true; 2) if only conclusion II is true;
2) if only conclusion II is true; 3) if either I or II is true;
3) if either I or II is true; 4) if neither I nor II is true; and
4) if neither I nor II is true; and 5) if both I and II are true.
5) if both I and II are true. 284. Statements: I © V, R @ D, E @ V, R = I
273. Statements: G@U, L©U, T$L, T#M Conclusions: I. D  V II. E = I
Conclusions: I. G @ M II. M © U 285. Statements: T = A, E @ L, T @ E, R  A
274. Statements: L#G, S©Q, P@Q, K£G Conclusions: I. T  L II. T @ R
Conclusions: I. P @ K II. P £ K 286. Statements: C @ A, O @ C, E = P, E  A
275. Statements: F©N, M#Z, Y$Z, Y$N

K
Conclusions: I. P  A, II. O @ A
Conclusions: I. F £ M II. M $ F 287. Statements: M © B, R @ B, M @ U, R = E
276. Statements: T£Z, P@Z, P@M, Q£M Conclusions: I. M  R II. M = R
Conclusions: I. P # R II. P © R 288. Statements: S © M, K  A, S @ U, K @ M
277. Statements: G$M, P$L, M#P, S©P Conclusions: I. A = S II. K  M
Conclusions: I. S # G II. P©G Directions (Q. 289-294): In the following questions,
Directions (Q. 278-283): In the following questions, the symbols +, -, ×, ÷, @ and © are used with the following
the symbols  , ©, @, @ and  are used with the following meanings:
meanings: P + Q means Q is not smaller than P.
P  Q means P is not smaller than Q. P - Q means Q is neither greater than nor smaller than P.
P © Q means P is neither greater than nor smaller than Q. P × Q means Q is not greater than P.
P @ Q means P is not greater than Q. P ÷ Q means Q is neither smaller than nor equal to P.

KUNDAN
P @ Q means P is neither smaller than nor equal to Q. P @ Q means Q is neither greater than nor equal to P.
P  Q means P is neither greater than nor equal to Q. P © Q means Q is not equal to P.
Now in each of the following questions, assuming the Now in each of the following questions, assuming the
given statements to be true, find which of the two conclu- given statements to be true, find which of the two conclu-
sions I and II given below them is/are definitely true. Give sions I and II given below them is/are definitely true. Give
answer answer
1) if only conclusion I is true; 1) if only conclusion I is true;
2) if only conclusion II is true; 2) if only conclusion II is true;
3) if either I or II is true; 3) if either I or II is true;
4) if neither I nor II is true; and 4) if neither I nor II is true; and
5) if both I and II are true. 5) if both I and II are true.
278. Statement: T @ S, N  G, L  S, G @ L 289. Statements: A × B, R + G, L©G, B@L
Conclusions: I. T  L II. T © L Conclusions: I. A@R II. A ÷ R
279. Statement: Z  H, Z  M, G  H, M @ N 290. Statements: M÷T, T©L, B×L, R@B
Conclusions: I. G @ M II. N  H Conclusions: I. L@T II. L÷T
280. Statement: K  W, S © W, L @ S, D  K 291. Statements: R©P, P©T, S+T, S@K
Conclusions: I. S  K II. W @ L Conclusions: I. R-T II. R©T
281. Statement: L @ K, N @ K, S  P, Q  P 292. Statements: M©N, N-K, K©S, G÷N
Conclusions: I. L @ N II. S  Q Conclusions: I. M©K II. M©S
282. Statement: C © M, T © M, U  M, P  M 293. Statements: T+U, W×U, G©S, P©S
Conclusions: I. T @ U II. P @ U Conclusions: I. W×T II. G-P
283. Statement: C  H, L  C, H @ X, M @ L 294. Statements: N@T, T-M, M-Z, K+Z
Conclusions: I. L @ H II. L  H Conclusions: I. N×K II. K+T
Directions (Q. 284-288): In the following questions, Directions (Q. 295-299): In the following questions
the symbols  , ©, =, @ and @ are used with the following the symbols @, ©, ®, # and  are used with the following
meanings: meanings:
P © Q means P is neither smaller than nor equal to Q. P # Q means P is not equal to Q.
P Q means P is neither greater than nor equal to Q. P © Q means P is either greater than or equal to Q.
P ® Q means P is not smaller than Q. P ® Q means P is equal to Q.
P # Q means P is not greater than Q. P @ Q means P is smaller than Q.
P @ Q means P is neither greater than nor less than Q. P  Q means P is either smaller than or equal to Q.
Now in each of the following questions, assuming the Now in each of the following questions assuming the
given statements to be true, find which of the two conclu- given statements to be true, find which of the two conclu-
sions I and II given below them is/are definitely true. Give sions I and II given below them is/are definitely true? Give
answer answer
1) if only conclusion I is true; 1) if only conclusion I is true;
2) if only conclusion II is true; 2) if only conclusion II is true;
3) if either I or II is true; 3) if either I or II is true;
4) if neither I nor II is true; and 4) if neither I nor II is true; and
5) if both I and II are true. 5) if both I and II are true.
295. Statement: GQ, R®Q, T#R, S@T 305. Statements: J©K, M#N, LN, K®M
Conclusions: I. R©G II. S#R Conclusions: I. K@L II. L@K
296. Statement: K@M, V#T, S@T, M®S 306. Statements: L@G, B@K, L®S, B#L
Conclusions: I. KS II. KV Conclusions: I. G©S II. S@K
297. Statement: G@K, Z#Y, YX, K®X 307. Statements: WJ, W@S, J@M, E#W
Conclusions: I. Z#G II. K©Z Conclusions: I. M©S II. M@S

K
298. Statement: P®R, R®Q, RS, R©L 308. Statements: P®N, N#M, M©G, KG
Conclusions: I. S#P II. S©P Conclusions: I. P©K II. P@K
A C M G 309. Statements: D©G, G®S, KS, P®K
299. Statement:  ,  , G.B@H.A B@D Conclusions: I. PD II. G©K
B D N H Directions (Q. 310-315): In the following questions
C G the symbols andare used with the following
Conclusions: I. A  G II. 
D H meanings:
Directions (Q. 300-304): In these questions, certain P  Q means Q is not smaller than P.
symbols have been used to indicate relationships between P  Q means Q is neither greater than nor smaller than P.
elements as follows: P  Q means Q is not greater than P.
‘A * B’ means ‘A is either equal to or greater than B’ P  Q means Q is neither smaller than nor equal to P.
‘A $ B’ means ‘A is equal to B’

KUNDAN
P  Q means Q is neither greater than nor equal to P.
‘A £ B’ means ‘A is either equal to or smaller than B’
‘A & B’ means ‘A is smaller than B’ and Now in each of the following questions, assuming
‘A @ B’ means ‘A is greater than B’. the given statements to be true, find which of the two con-
In each question, four statements showing relation- clusions I and II given below them is/are definitely true. Give
ships have been given, which are followed by two conclu- answer
sions I & II. Assuming that the given statements are true, 1) if only conclusion I is true;
find out which conclusion(s) is/are definitely true. Mark 2) if only conclusion II is true;
answer 3) if either I or II is true;
1) if only conclusion I is true. 4) if neither I nor II is true; and
2) if only conclusion II is true. 5) if both I and II are true.
3) if either conclusion I or II is true. 310. Statements: S  A, P  A, P  L, G  L
4) if neither I nor II is true and; Conclusions: I. S  L II. L  S
5) if both conclusions I and II are true. 311. Statements: M  N, N  P, P  Q, R  Q
300. Statements: H & G, K$S, S*J K$H Conclusions: I. N  Q II. N  Q
Conclusions: I. J & G II. J&K 312. Statements: H  A, H  G, G  S, A  T
301. Statements: L$M, N£M, N@J, R$J Conclusions: I. T  G II. T  G
Conclusions: I. L*R II. N£L 312 Statements: R  Z, R  H, R  G, G  X
302. Statements: G$P, P*Q, Q*S, P@T Conclusions: I. Z  G II. H  Z
Conclusions: I. G@T II. S£G 314. Statements: P  L, L  S S  K, L  M
303. Statements: N@L, M$N, R&L, L*D Conclusions: I. K  M II. M  S
Conclusions: I. M$D II. M*D
304. Statements: T$S, U&T, S*W, V$S 315. Statements: A  B, Y  X, C  B, Y  Z
Conclusions: I. W&T II. T$W Conclusions: I. X  Z II. C  A
Directions (Q. 305-309): In the following questions, Directions (Q. 316-320): In the following questions,
the symbols #, ©, ®, @ and  are used with the following the symbols #, ©, ®, @ and  are used with the following
meanings: P  Q means P is either smaller than or equal to Q.
P # Q means P is not equal to Q. Now in each of the following questions assuming the
P © Q means P is either greater than or equal to Q. given statements to be true, find which of the two conclu-
P ® Q means P is equal to Q. sions I and II given below them is/are definitely true.
P @ Q means P is smaller than Q. 1) if only conclusion I is true;
P  Q means P is either smaller than or equal to Q. 2) if only conclusion II is true;
Now in each of the following questions, assuming the 3) if either I or II is true;
given statements to be true, find which of the two conclu- 4) if neither I nor II is true; and
sions I and II given below them is/are definitely true. Give 5) if both I and II are true.
answer 326. Statements: Q  B, J  E, L  B, J  Q
1) if only conclusion I is true; Conclusions: I. J  L II. E  B
2) if only conclusion II is true; 327. Statements: V  T, O  B, I  V, B  T
3) if either I or II is true; Conclusions: I. V  B II. B  V
4) if neither I nor II is true; and 328. Statements: F  E, L  B, F  S, B  E
5) if both I and II are true. Conclusions: I. L  B II. L  S
316. Statements: P @ K, P © M, G ® T, M @ T 329. Statements: Z  M, B  S, N  Z, N  S
Conclusions: I. M @ K II. M @ G Conclusions: I. S  M II. B  N
317. Statements: R © N, A ® B, S  B, Z @ B 330. Statements: F  M, B  O, F  W, B  W
Conclusions: I. N @ B II. A # R Conclusions: I. O  W II. B  F
318. Statements: G  L, G ® T, T # P, P © K Directions (Q. 331-335): In the following questions the
Conclusions: I. L © K II. L @ K symbols @, @, =, © and © are used with the following mean-

K
319. Statements: T # K, K # L, L © G, S  G ings:
Conclusions: I. T @ L II. L @ T P @ Q means Q is neither greater than nor equal to P.
320. Statements: A @ T, Z ® A, Z © K, P  K P @ Q means Q is not greater than P.
Conclusions: I. P @ Z II. P @ A P = Q means Q is equal to P.
Directions (Q. 321-325): In the following questions, P © Q means Q is neither less than nor equal to P.
the symbols $, ©, ×, @ and # are used with the following P © Q means Q is not less than P.
meanings: Now in each of the following questions, assuming the
P $ Q means P is not equal to Q. given statements to be true, find which of the two conclu-
P © Q means P is neither greater than nor smaller than Q. sions I and II given below them is/are definitely true. Give
P @ Q means P is not greater than Q. answer
P × Q means P is neither smaller than nor equal to Q. 1) if only conclusion I is true.

KUNDAN
P # Q means P is neither greater than nor equal to Q. 2) if only conclusion II is true.
Now in each of the following questions, assuming the 3) if either conclusion I or II is true.
given statements to be true, find which of the two conclu- 4) if neither conclusion I nor II is true.
sions I and II given below them is/are definitely true. Give 5) if both conclusions I and II are true.
answer 331. Statements: C@W, L©D, D©C
1) if only conclusion I is true; Conclusions: I. W@D II. C@L
2) if only conclusion II is true; 332. Statements: M@V, U = M, V © T
3) if either I or II is true; Conclusions: I. U@V II. V = T
4) if neither I nor II is true; and 333. Statements: U = M, P @ U, M @ B
5) if both I and II are true. Conclusions: I. P = B II. P@B
321. Statements: L$T, S#T, W©S, W×K 334. Statements: L@ N, J©P, P@L
Conclusions: I. L×S II. W#L Conclusions: I. J = L II. P = N
322. Statements: D$E, F$E, F©G, H©G 335. Statements: H@G, D@E, H = E
Conclusions: I. H#E II. E#H Conclusions: I. D@H II. G©D
323. Statements: T@K, G@H, T©H, L©K Directions (Q. 336-340): In the following questions,
Conclusions: I. L×G II. L©G the symbols @, +, ,  and $ are used with following mean-
324. Statements: J×G, G©M, M$N, N©S ings:
Conclusions: I. S#J II. S$G A @ B means B is neither greater nor equal to A.
325. Statements: P×Q, M#Q, T$Q, T©D A + B means B is not greater than A.
Conclusions: I. M#P II. D@P A $ B means B is equal to A.
Directions (Q. 326-330): In the following questions, A  B means B is neither smaller nor equal to A.
the symbols  ,  ,  ,  and  are used with the following. A  B means B is not less than A.
P  Q means P is greater than Q. Now in each of the following questions, assuming
P  Q means P is either greater than or equal to Q. the given statements to be true, find which of the two con-
P  Q means P is equal to Q. clusions I and II given below them is/are definitely true. Give
P  Q means P is smaller than Q. answer
1) if only conclusion I is true; 3) if either I or II is true;
2) if only conclusion II is true; 4) if neither I nor II is true; and
3) if either I or II is true; 5) if both I and II are true.
4) if neither I nor II is true; and 346. Statements: B@K, E=H, KE
5) if both I and II are true. Conclusions: I. B@H II. KH
336. Statements: X + Z, T  Z, T@M, NM
347. Statements: R=S, MS=N, J@S
Conclusions: I. X + T II. N  T
337. Statements: X @ Z, Z $ T, T + M, N  M Conclusions: I. SN II. R=J
Conclusions: I. X  N II. X $ N 348. Statements: J@H, F=X, XH
338. Statements: T@M, N  M, X$N, ZX Conclusions: I. X  J II. XJ
Conclusions: I. T@Z II. M + X 349. Statements: G=Q, O  P, QO
339. Statements: Z + X, T  X, M  N, P@N Conclusions: I. P @ G II. Q=P
Conclusions: I. T@P II. T  P 350. Statements: L@U, U=F, F=C
340. Statements: A@B, B + C, C  D, D  E Conclusions: I. L@C II. FC
Conclusions: I. A @ E II. A  E
Directions (Q. 351-355): In the following questions,
Directions (Q. 341-345): In the following questions,
the symbols @, , , $ and # are used with the following the symbols  ,  ,  ,  and  are used with the following
meanings: meanings:
P Q means P is either equal to or smaller than Q.
P $ Q means P is neither greater than nor smaller than Q. P  Q means P is not smaller than Q.
P # Q means P is neither greater than nor equal to Q. P  Q means P is neither greater than nor smaller than Q.

K
P @ Q means P is greater than Q. P  Q means P is not greater than Q.
P Q means P is not less than Q. P  Q means P is neither smaller than nor equal to Q.
Now in each of the following questions, assuming the
given statements to be true, find which of the two conclu- P  Q means P is neither greater than nor equal to Q.
sion I and II given below them is/are definitely true. Give Now in each of the following questions, assuming
answer the given statements to be true, find which of the two con-
1) if only conclusion I is true; clusions I and II given below them is/are definitely true. Give
2) if only conclusion II is true; answer
3) if either I or II is true; 1) if only conclusion I is true;
4) if neither I nor II is true; and 2) if only conclusion II is true;
5) if both I and II are true. 3) if either I or II is true;

KUNDAN
341. Statements: Z # N, F N, FK 4) if neither I nor II is true; and
Conclusions: I. K$N II. K@Z
5) if both I and II are true.
342. Statements: D$T, TM, M#K
Conclusions: I. M$D II. D@M 351. Statements: M  N, H  Q, Q  M
343. Statements: WA, BA, B@M Conclusions: I. H  M II. Q  N
Conclusions: I. B#W II. W$B
344. Statements: JM, M$N, N#T 352. Statements: C  B, L  S, S  C
Conclusions: I. T@J II. T$J Conclusions: I. B  S II. C  L
345. Statements: VF, F@R, RG 353. Statements: I  H, E  F, I  F
Conclusions: I. G#V II. G@V
Conclusions: I. E  I II. H  E
Directions (Q. 346-350): In the following questions,
the symbols @, @, ,  and = are used with the following 354. Statements: V  O, R  V, O  B
meanings: Conclusions: I. R  B II. R  B
P @ Q means P is neither smaller than nor equal to Q. 355. Statements: L  U, T  L, U  W
P @ Q means P is either greater than or equal to Q. Conclusions: I. T  W II. U  W
P  Q means P is equal to Q.
Directions (Q. 356-360): In the following questions,
P  Q means P is smaller than Q.
P = Q means P is not greater than Q. the symbols  ,  ,  ,  , and  are used with the following
Now in each of the following questions, assuming meanings:
the given statements to be true, find which of the two con- P  Q means P is not smaller than Q.
clusions I and II given below them is/are definitely true. Give P  Q means P is neither greater than nor smaller than Q.
answer
P  Q means P is not greater than Q.
1) if only conclusion I is true;
2) if only conclusion II is true; P  Q means P is neither smaller than nor equal to Q.
P  Q means P is neither greater than nor equal to Q. P = Q means P is not greater than Q.
Now in each of the following questions, assuming P  Q means P is neither equal to nor smaller than Q.
the given statements to be true, find which of the two con- P $ Q means P is not smaller than Q.
clusions I and II given below them is/are definitely true. Give P  Q means P is neither greater nor smaller than Q.
answer P £ Q means P is neither greater than nor equal to Q.
1) if only conclusion I is true; Now in each of the following questions, assuming
2) if only conclusion II is true; the given statements to be true, find which of the two con-
3) if either I or II is true; clusions I and II given below them is/are definitely true. Give
4) if neither I nor II is true; and answer
5) if both I and II are true. 1) if only conclusion I is true.
356. Statements: K  L, M  P, J  K, P  L 2) if only conclusion II is true.
Conclusions: I. K  P II. P  K 3) if either conclusion I or II is true.
4) if neither conclusion I nor II is true.
357. Statements: F  M, A  L, F  G, A  U 5) if both conclusions I and II are true.
Conclusions: I. L  U II. A  F 366. Statement: C $ D, F  E, G = E
358. Statements: P  B, J  H, S  B, J  P Conclusions: I. F £ G II. F  G
Conclusions: I. J  S II. H  B 367. Statement: Y  Z, X $ G, Y £ L, G  L
359. Statements: M  Z, B  R, Z  C, M  R Conclusions: I. Y £ X II. L  Z

K
Conclusions: I. R  Z II. B  M 368. Statement: A $ D, B = C, A  R, B  A
Conclusions: I. R  D II. R  D
360. Statements: X  Y, U  V, X  S, V  Y
369. Statement: Z = Y, U  V, Y £ K, R £ V
Conclusions: I. U  V II. U  S
Conclusions: I. K  Z II. U £ R
Directions (Q. 361-365): In the following questions,
370. Statement: W = Q, R  X, Q $ X
the symbols ©, =, @,  and @ are used with the following
Conclusions: I. W £ X II. W  X
meanings:
P © Q means P is not smaller than Q. Directions (Q. 371-375): In the following questions,
P = Q means P is neither greater than nor smaller than Q. the symbols  ,  ,  ,  and  are used with the following
P @ Q means P is not greater than Q. meanings:

KUNDAN
P  Q means P is neither smaller than nor equal to Q. P  Q means P is not greater than Q.
P @ Q means P is neither greater than nor equal to Q. P  Q means P is neither greater than nor smaller than Q.
Now in each of the following questions, assuming
P  Q means P is not smaller than Q.
the given statements to be true, find which of the two con-
clusions I and II given below them is/are definitely true. Give P  Q means P is neither smaller than nor equal to Q.
answer P  Q means P is neither greater than nor equal to Q.
1) if only conclusion I is true; Now in each of the following questions, assuming
2) if only conclusion II is true; the given statements to be true, find which of the two con-
3) if either I or II is true; clusions I and II given below them is/are definitely true. Give
4) if neither I nor II is true; and answer
5) if both I and II are true. 1) if only conclusion I is true;
361. Statements: X@Y, Z©V, Y©Z 2) if only conclusion II is true;
Conclusions: I. X@V II. Y=V 3) if either I or II is true;
362. Statements: Q@P, S©R, P=S 4) if neither I nor II is true; and
Conclusions: I. Q@S II. P=R 5) if both I and II are true.
363. Statements: F©E, G=E©H, I@E 371. Statements: M  J, K  B, M  A, J  K
Conclusions: I. E=H II. F©I
Conclusions: I. M  K II. A  J
364. Statements: A  B, J@L, B=J
372. Statements: E  A, N  A, C  E, N  D
Conclusions: I. A  L II. B=L
Conclusions: I. C  N II. A  D
365. Statements: M=N, N@Q, N©R
Conclusions: I. Q=R II. M=R 373. Statements: K  I, J  V, G  I, V  G
Directions (Q. 366-370): In the questions given be- Conclusions: I. G  K II. K  G
low, certain symbols are used with the following meanings: 374. Statements: Y  Z, R  T, S  Y, R  Z
Conclusions: I. S  Z II. Y  T Conclusions: I. G&I II. J$H
382. Statements: X!Y, Z&K, Z?Y
375. Statements: L  K, C  H, C  A, H  L
Conclusions: I. Y&K, II. X!Z
Conclusions: I. C  L II. A  K 383. Statements: N?L, M£L, P$N
Directions (Q. 376-380): In the following questions, Conclusions: I. M&P II. N?M
the symbols  ,  ,  ,  and  are used with the follow- 384. Statements: Q?S, T!S, R£T
ing meanings: Conclusions: I. R!Q II. Q$R
P  Q means P is not smaller than Q. 385. Statements: C£D, E$F, E&C
Conclusions: I. C&F II. D$F
P  Q means P is neither greater than nor smaller than Q. Directions (Q. 386-390): In the questions given be-
P  Q means P is not greater than Q. low, certain symbols are used with the following meanings:
P  Q means P is neither smaller than nor equal to Q. P * Q means P is neither equal to nor smaller than Q.
P  Q means P is not equal to Q. P  Q means P is not smaller than Q.
Now in each of the following questions, assuming P $ Q means P is neither greater nor smaller than Q.
the given statements to be true, find which of the two con- P £ Q means P is neither greater than nor equal to Q.
clusions I and II given below them is/are definitely true. Give P @ Q means P is not greater than Q.
answer Now in each of the following questions, assuming
1) if only conclusion I is true; the given statements to be true, find which of the two con-
2) if only conclusion II is true; clusions I and II given below them is/are definitely true. Give

K
3) if either I or II is true; answer
4) if neither I nor II is true; and 1) if only conclusion I is true.
5) if both I and II are true. 2) if only conclusion II is true.
376. Statements: X  Y, X  Z, Z  S 3) if either conclusion I or II is true.
Conclusions: I. X  S II. S  Y 4) if neither conclusion I nor II is true.
5) if both conclusions I and II are true.
377. Statements: A  B, C  B, C  D 386. Statements: R*S, S$O M@O
Conclusions: I. D  A II. D  B Conclusions: I. R*M II. S*M
378. Statements: T  U, W  V, V  U 387. Statements: R  G, G@K, R$L
Conclusions: I. W  T II. V  T Conclusions: I. L @ K II. K£L

KUNDAN
379. Statements: L  N, K  L, M  N 388. Statements: Q  X, Y@X, Z@Y
Conclusions: I. Q  Z II. Y  Q
Conclusions: I. L  M II. L  M
389. Statements: A  T, S£T, N@S
380. Statements: P  Q, R  S, P  R Conclusions: I. A  S II. A  N
Conclusions: I. Q  S II. S  P 390. Statements: A$T, T@M, Q  M
Directions (Q. 381-385): In the following questions,
Conclusions: I. Q*T II. Q$T
the symbols ?, !, £, $ and & are used with the following
Directions (Q. 391-395): In the following questions,
meanings:
the symbols #, $,  , * and @ are used with the following
P ? Q means P is not smaller than Q.
P ! Q means P is neither greater than nor smaller than Q. meanings.
P £ Q means P is not greater than Q. A # B means A is not greater than B.
P $ Q means P is neither smaller than nor equal to Q. A $ B means A is neither smaller than nor equal to B.
P & Q means P is neither greater than nor equal to Q. A  B means A is neither greater than nor smaller than B
Now in each of the following questions, assuming A * B means A is neither greater than nor equal to B.
the given statements to be true, find which of the two con- A @ B means A is not smaller than B.
clusions I and II given below them is/are definitely true. Give Now in each of the following questions, assuming
answer the given statements to be true, find which of the two con-
1) if only conclusion I is true; clusions I and II given below them is/are definitely true. Give
2) if only conclusion II is true; answer
3) if either I or II is true; 1) if only conclusion I is true;
4) if neither I nor II is true; and 2) if only conclusion II is true
5) if both I and II are true. 3) if either I or II is true;
381. Statements: G£H, H$I, I?J 4) if neither I nor II is true; and
5) if both I and II are true. 1) if only conclusion I follows
391. Statements: L*S, P@R, S#R 2) if only conclusion II follows
Conclusions: I. P$L II. S#P 3) if either conclusion I or II follows
392. Statements: I. G$R, H#R, G@M 4) if neither conclusion I nor II follows
Conclusions: I. M$R II. R*M 5) if both conclusions follow
393. Statements: Y#T, J  T, O  T, E@J 401. Statements: R  M, M  P, R   L
Conclusions: I. Y*E II. E  Y Conclusions: I. M  L II. P  L
394. Statements: H@P, H*D, T*P, X  T 402. Statements: T   P , P  S, P  M
Conclusion: I. H@T II. D$X
Conclusions: I. S  M II. T  S
395. Statements: F#O, C  I, L*I, L@O
Conclusions: I. F*I II. F#I 403. Statements: M  T, T   Z, S  M
Directions (Q. 396-400): In the following questions, Conclusions: I. Z  M II. Z  M
the symbols ©, @, ,  and @ are used with the following 404. Statements: Z  B, N   S, B  N
meanings:
Conclusions: I. B  Z II. S   B
P © Q means P is not smaller than Q.
P @ Q means P is neither greater than nor smaller than Q. 405. Statements: L  C, C  Z, Z   F
P  Q means P is not greater than Q. Conclusions: I. C  F II. C  F
P  Q means P is neither smaller than nor equal to Q. Directions (Q 406-410): In a particular method of

K
P @ Q means P is neither greater than nor equal to Q.
Now in each of the following questions, assuming coding the symbols a, a  , b, b  and   are used with the
the given statements to be true, find which of the two con- following meaning:
clusions I and II given below them is/are definitely true. Give A a B means A is greater than B
answer A a  B means A is greater than or equal to B
1) if only conclusion I is true; A b B means B is greater than A
2) if only conclusion II is true; A b  B means B is greater than or equal to A
3) if either I or II is true;
4) if neither I nor II is true; and A   B means A is equal to B.
5) if both I and II are true. On the basis of the above scheme and assuming each
396. Statements: U © V, U@W, M  N, W  N of the given statements to be true decide which of the given

KUNDAN
Conclusions: I. V@W II. W@V conclusions follow. Give answer
397. Statements: S ©T, X@Y, S Y 1) if only conclusion I follows
Conclusions: I. T  X II. Y  T 2) if only conclusion II follows
398. Statements: Z  R, S © D, R@A, A S
3) if either conclusion I or II follows
Conclusions: I. Z © D II. R © S
399. Statements: U © V, N K, L@V, U N 4) if neither I nor II follows
Conclusions: I. V © N II. K@L 5) if both I and II follow
400. Statements: A @ B, C  D, D © A, C  E 406. Statements: M a  O, L b O, O a P
Conclusions: I. E  D II. C  B Conclusions I. L a O II. O a L
Direction (Q 401-405): In the following questions, 407. Statements: N b L, L a S, S a Q
the symbols  ,   ,  ,   and  are used as fol- Conclusions I. Q b L II. Q a N
lows: 408. Statements: A a B , B b C, C a D
A  B means A is greater than B. Conclusions I. A   D II. A a  D
A   B means A is greater than or equal to B. 409. Statements: M b N, O a P, O b U
A  B means B is greater than A. Conclusions I. P   U II. P a U
A   B means B is greater than or equal to A. 410. Statements: Q a S , U b S , U   R
A  B means A is equal to B. Conclusions I. Q a U II. Q b U
Now, in the following questions, assuming the three
given statements to be true, decide upon the validity of the
given conclusions. Give answer
Answers and Explanations
1. 1; I  V ... (i), R < D .... (ii), E < V .... (iii), R = I ... (iv) Hence II is not true.
From (i), (ii) and (iv) D > R = I  V 22. 4 23. 1 24. 4 25. 3
 D > V Hence I is true. 26. 2; I  J ... (i), K = J  L ... (ii), H  J ... (iii)
From (i) and (iii), I  V > E  II is not true. I is false from (ii).
2. 2 3. 5 From (i) and (iii), I  J  H  I  H. Hence II is true.
4. 3; M  B ... (i), R  B ... (ii), M < U ... (iii), R = E ... (iv) 27. 3 28. 1 29. 4 30. 4
From (i) and (ii), M  B  R  M  R Hence either 31. 1; R  S ... (i), O  P ... (ii), P < S ... (iii)
I or II is true. Combining all these, we get, R  S > P  O  R > O.
5. 4 Hence I is true.
6. 4; P > A ... (i), I < D ... (ii), K  A ... (iii), I  P ... (iv) From (i) and (iii), R  S > P  R > P or P < R. Hence II
From (i), (iii) and (iv), I  P > A  K  I > K, Hence is not true.
I is not true. 32. 4; F < Z ... (i), H  A ... (ii), F > H ... (iii)
From (i), (ii) and (iv), D > I  P > A  D > A, Hence II From (ii) and (iii), F > H  A ... (iv)  F > A. Hence II
is not true. is not true.
7. U  S .... (i), N < A ... (ii), H > U ... (iii), A  S ... (iv) From (i) and (iv), Z > F > H  A  Z > A. Hence I is
From (i) and (iv), U  S  A  U  A  Either U not true.

K
= A or U > A or A < U. Hence either I or II is true. 33. 2; O < R ... (i), U > V ... (ii), O  V ... (iii)
8. 4 9. 1 10. 4 From (ii) and (iii), U > V  O. Hence no relation be-
11. 1; D  H .... (i) I  R... (ii) R < H ... (iii) tween U and O can be established. Hence I is not true.
From (i), (ii) & (ii), D  H > R  I  D > I. Hence I is From (i) and (iii), V  O < R  V < R. Hence II is true.
true. 34. 2; W = X ... (i), Z > Y ... (ii), W < Y ... (iii)
From (i) and (iii) D  H > R  R < D. Hence II may be From (i) and (iii), Y > W = X  Y > X. Hence I is not
true but not necessarily so. true.
12. 2; K < I .... (i), S > H ... (ii), K  H ... (iii) From (ii) and (iii), Z > Y > W  Z > W or W < Z. Hence
From (ii) and (iii), S > H  K  I is not true. II is true.
From (i) and (iii), I > K  H  I > H or H < I. Hence II 35. 3; I > J ... (i), L  M ... (ii), J = M ... (iii)

KUNDAN
is true. From (ii) and (iii), L  M = J  L  J. Hence either I
13. 5; A = M ... (i), J > T ... (ii), A < T ... (iii) or II is true.
From (i) and (iii), M = A < T  M < T or T > M. 36. 3; P  Q .... (i); Q  R ... (ii); R = T ... (iii)
From (ii) and (iii), J > T > A  J > A or A < J. Now, we cannot establish a relationship between P
14. 3; R > A .... (i), H  U ... (ii), A = U ... (iii) and R from the given equations. But note the conclu-
From (ii) and (iii), A = U  H  A > H or A = H. sions : I. P  R ; II. P < R. At least one of the two must
be true for any two numbers P and R.
15. 4; M  U ... (i), K  A ... (ii), M > K ... (iii)
37. 5; N = M ... (i); C > N ... (ii); M = D ... (iii)
From (i), (ii) and (iii), U  M > K  A  U > A. Hence From (i), (ii) and (iii), C > D ; thus I is true
I may be true but not necessarily so. From (i) and (iii), N = D; thus II is true.
From (ii) and (iii), M > K  A  M > A or A < M. 38. 2; Q < L .... (i); Q  R ... (ii); T < P ... (iii)
Hence II may be true but not necessarily so. No relationship can be established between L and P
16. 4; X > Y ... (i), Z  R ... (ii), Y = Z ... (iii) from the above equations. So I is not true.
Combining all we get, X > Y = Z  R  X > R may be From (i) and (ii), we get L > Q  R  R < L. Thus II is
true but not necessarily so. true.
From (ii) and (iii) Y = Z  R  Y = R may be true but 39. 4; X  Y .... (i) ; Y = Z ... (ii) ; A > Z ... (iii)
not necessarily so. From (i) and (ii), X  Z. Thus I is false.
17. 4 18. 1 19. 3 20. 2 From (ii) and (iii), Y < A. Thus II is false.
21. 4; Q > B ... (i), J < E ... (ii), L  B ... (iii), J  Q ... (iv) 40. 1; S < T ... (i) ; S > M ... (ii) ; M = P ... (iii)
From (i), (iii), and (iv), J  Q > B  L  J > L. From (ii) and (iii), S > P. Thus I is true.
Hence I is not true. From (i) and (ii), T > M. Thus II is false.
From (i), (ii) and (iv), E > J  Q > B  E > B 41. 2; S = T .... (i); U > T .... (ii) ; S  V .... (iii)
From (i) and (iii), T  V .... (iv). So I is false. From (iii) and (iv), we get K > L  C  K>C or C<K.
From (ii) and (iv), U > V. So II is true. Hence II is true.
42. 4; L  M .... (i) ; L = N .... (ii); N < O .... (iii) I cannot be established because there is no link betwen
From (iii) I, can’t be established. (i) and (ii).
From (i) and (ii), N  M. So II is false. 73. 1; Y = B ... (i), X < Y ... (ii), A > B ... (iii)
43. 3; U is not mentioned anywhere in the statements. So we Combining all, we get X<Y = B<A  X<A or A>X.
cannot establish a relationship with U. However, R is Hence I is true. It also implies that B > X. Hence II may
either lesser than or equal to or greater than U. So be true but not necessarily so.
either I or II is correct. 74. 5; C < F ... (i), P < T ... (ii), C  T ... (iii), F < R ... (iv)
44. 2; Y < Z .... (i); S > Z .... (ii); T < Z .... (iii) From (ii) and (iii), we get C  T > P  C > P. Hence I
I does not follow because no relationship can be es- is true.
tablished between Y and T. From (i) and (iii), F > C  T  F > T. Hence II is true.
From (i) and (ii), Y < S. Hence, II is true. 75. 3; Q = S ... (i), S>Z ... (ii), Z < W ... (iii), S  T ... (iv)
45. 4; M  L .... (i); M  N .... (ii); O = N .... (iii) From (ii) and (iv), we get Z < S  T  Z < T ... (A)
From (i), (ii) and (iii), L  M  N = O. So neither I nor Now, from (A) and (iii), we get T > Z < W  no con-
II follows. clusion i.e. either T > W, or T = W or T < W. Hence
46. 4; P > Q .... (i); P  R .... (ii); R  S .... (iii) either conclusion I or conclusion II is true.
P = R is one of the possibilities from (iii), but we can’t 76. 4; A = B ... (i), B  D ... (ii), B < H ... (iii)

K
be certain
From (ii) and (iii), we get H > B  D  H > D. Hence
47. 4; T = N .... (i); D  E .... (ii); E  N ... (iii)
I is not true.
From (ii) and (iii), D = N is one of the possibilities. So
From (i) and (ii), we get A = B  D  A  D. Hence
D  N may follow but not necessarily so. From (i) and
II may be true but not necessarily so.
(iii), E  T. Thus II does not follow.. 77. 1; X > Y ... (i), Y = Z ...(ii), Z  T ... (iii)
48. 4; K  L .... (i); L > M .... (ii); L = N .... (iii) Combining all, we get X > Y = Z  T  X > T. Hence
From (i)and (iii), I does not follow. I is true. II may be true but not necessarily so.
From (ii) and (iii), II does not follow.
78. 2; E  F ... (i), G = F  H ... (ii), I  F ... (iii)
49. 3; P > T .... (i); S < T .... (ii); R  S .... (iii)
From (i) and (iii), we get E  F  I  E  I. Hence II
From (iii), either S > R or S = R.

KUNDAN
is true. I is not true from (ii).
50. 1; X < Y .... (i); Y > T .... (ii); T  X .... (iii)
79. 1; P < Q ... (i), R  S ... (ii), Q = R ... (iii)
I follows from (i). II does not follow from (iii).
From (i) and (iii), we get P < Q = R  P < R. Hence I is
51. 2; A  K... (i), M = N ... (ii), K  N ... (iii), Q > A ... (iv)
true. From (ii) and (iii), we get Q = R  S 
From (i), (iii) and (ii), we get, A  K  N = M  A =
M may be true but not necessarily so. Q  S. Hence I may be true but not necessarily so.
From (i) and (iv), we get, Q > A  K  Q > K or K < 80. 4; A < B ... (i), B  C ... (ii), C  D ... (iii)
Q. Hence II is true. Combining all, we get A < B  C  D  No relation-
52. 2 53. 4 54. 4 55. 1 56. 4 57. 4 58. 2 ship between A and D can be established. Hence I is
59. 4 60. 2 not true.
61. 2; A  N ...(i); B > M ...(ii); A  5 ...(iii); B = 5 ...(iv) II is not necessarily true from (iii).
Combining these, we get N  A  5 = B > M 81. 3; C < E ... (i), F  U ... (ii), U  Q ... (iii), Q > C ... (iv)
Hence, N  B and M < N. Hence I does not necessar- From (i) and (iv), we get Q > C < E  Either Q > E or Q
ily follow and II follows. = E or Q < E. Hence either I or II is true.
62. 4 63. 4 64. 4 65. 4 82. 4;U  V... (i), V  X... (ii), W > M ... (iii) X < W ... (iv)
66. 5; M < N ...(i); O  N ...(ii); P  M ...(iii) Combining (iv), (ii) and (i), we get, W > X  V  U 
Combining these, we get P  M < N  O. no relationship can be established between W and U.
Hence N > P and P < N. Hence I is not true.
67. 4 68. 1 69. 4 70. 1 Aligning (iv) and (iii), we get X < W > M  no rela-
71. 4; B > D... (i), H < I ... (ii), M  N ... (iii), Q  R ...(iv) tionship can be established between M and X.
As there is no common letter between (ii) and (iii), hence 83. 2; E < Q ... (i), F  V ... (ii), G  Q ... (iii), F > E .... (iv)
neither I nor II can be established. Combining (iv) and (ii), we get,
72. 2; M  N... (i), E < F ... (ii), K > L ... (iii), C  L... (iv) E < F  V  E < V. Hence I may be true but not
necessarily so. 93. 1 94. 2 95. 2
From (i) and (iii), we get, E < Q  G 96. 4; K > L ... (i), K = E ... (ii), L < F ... (iii)
 E < G. Hence II is true. From (i) and (ii), we get, E = K > L  E > L
84. 5; F  G .... (i), H  I ... (ii), I = F ... (iii), H > L .... (iv) or, L < E. Hence I is not true.
From converted (ii) and (iv), we get I  H > L II is not true because of (iii).
97. 5; P = H.... (i), P = J ... (ii), P  K... (iii)
 I > L. Hence I is true.
From (iii) and (i), we get, I = F  G  I  G. Hence II From (ii) & (iii), we get, J = P  K  J  K. Hence I
is true. is true. From (i) and (ii) it can be proved that II is true.
98. 1 99. 1 100. 3
85. 1; Q  R ... (i), O = N .... (ii), N > S .... (iii), S  Q ... (iv)
101. 4; A > L ... (i), L < K ... (ii), K > Z ... (iii)
From (ii), (iii) and (iv), we get O = N > S  Q From (ii) and (iii), we get, L < K > Z. Hence no relation-
 O > Q or Q < O. Hence I is true. ship between L and Z can be determined. Hence I is
From converted (i), converted (iv) and converted (iii), not true.
we get, R  Q  S < N  no definite relationship From (i) and (ii), we get, A > L < K. Again, nothing can
between R and N can be established. Hence II is not be concluded about the relationship of A and K. Hence
true. II is not true.
86. 3; E = T...(i), S  L....(ii), L  E...(iii) 102. 4; O  A ... (i), U = O ... (ii), A = E ... (iii)
Combining all, we get, T=E  L  S  T  S or S  T From (ii) and (i), we get, U = O  A  I is wrong.
Hence either I or II is true. Since I is wrong, hence II also can’t be established.

K
87. 2; P  Z ....(i), R  K .... (ii), Z <R.... (iii) 103. 2; P > T ... (i), G  P ... (ii), F = G ... (iii)
From (i) and (iii), we get, P  Z < R  I can’t be From (iii) and (ii), we get, F = G  P  F  P... (iv).
established. Hence I may be true but not necessarily so.
From (ii) and (iii), we get, Z<R  K  Z<K or K>Z. Now, from (iv) and (i), we get, F  P > T  F > T..
Hence II is true. Hence II is true.
88. 4; M  S ....(i), Q  D...(ii), M = D ... (iii) 104. 3; W  E ... (i), P > E ... (ii), V  W ... (iii)
From (ii) and (iii), we get, Q  D = M  Q  M. Hence Combining all, we get, P > E  W  V or, E  V
I is not true.  E is either greater than or equal to V..
From (i) and (iii), we get, S  M = D  S  D. Hence 105. 1; M  L ... (i), O = L ... (ii), J < L ... (iii)

KUNDAN
II may be true but not necessarily. From (i) and (iii), we get M  L > J  M > J. Hence I
89. 4; B = G ...(i), R<B .... (ii), Y>R .... (iii) is true.
Combining all, we get, Y > R < B = G  no conclusion. From (ii) and (iii), we get, O = L > J  O > J. Hence II
Since Y= G is not given in conclusion, hence neither I is not true.
nor II is true. 106. 3; H > I .... (i), K  L ... (ii), I = L .... (iii)
90. 5; D  V ... (ii), X>D ... (ii), V>E .... (iii)
From (ii) and (iii), we get, K  L = I  K  I  either
From (i) and (ii), we get, X > D  V  X>V. Hence I is I > K or K = I.
true. 107. 4; X > F .... (i), T  C ... (ii), F  C ... (iii)
From (i) and (iii), we get, D  V>E  D>E or E<D. Combining all, we get,
Hence II is true. X > F  C  T  X > F = C = T  X > T may be true
(91-95): Symbols can be easily transformed as:
but not necessarily so because it is only one possibil-
(    , ©  =, @   , @  > and   <) ity. Hence I can’t be established. Nor can II be estab-
91. 4; Q = T ... (i), B  Q ... (ii), T  A ... (iii) lished.
From (i) and (ii), we get B  Q = T  B  T 108. 4; 2 > 8 ... (i), 6  8 ... (ii), 5 < 6 ... (iii)
or, T  B. Hence I is not true. From (ii) and (iii), we get, 5 < 6  8  I can’t be
From (i) and (iii), we get Q = T  A  Q  A proved. II also can’t be established.
or, A  Q. Hence II is not true. 109. 2; S < N ... (i), O  R ... (ii), S = R ... (iii)
92. 4; U > S ... (i), W < S ... (ii), S > O ... (iii) From (ii) and (iii), we get, S = R  O  S  O
From (i) and (ii), we get U > S > W  U > W.. or O  S. Hence II is true.
Hence I is not true. From II and (i), we get, O  S < N  No relationship
From (ii) and (iii), we get W < S > O. Hence no relation- between N and O can be established. Hence I is not
ship between W and O can be determined. true.
110. 2; J < Y ... (i), J  O ... (ii), Y = K ... (iii) or P < G. Hence II is false. Now, from (a) and (ii),
Combining all, we get K = Y > J  O  K > O or we get G > P  D  no conclusion.
O < K. Hence II is true. 139. 1; S  T ...(i), Q = N ...(ii), T < N ...(iii)
111. 2; D = B ... (i), R < L ... (ii), L > D ... (iii) From (i) and (iii), we get S  T < N  S < N. Hence I
From (ii) and (iii), no relationship between R and D can is true. From I and (ii), we get S < N = Q  S < Q.
be established. Hence I can’t be proved. Hence II is not true.
From (i) and (iii), L > D = B  L > B. Hence II is true. 140. 4; L > K ...(i), V  K ...(ii), R = L ...(iii)
112. 1; I  R ... (i), Q < C ... (ii), D < R ... (iii), Q = I ... (iv) From (i) and (ii), we get V  K < L  no relationship
From (ii), (iv) and (i), we get C > Q = I  R between V and L can be established. Hence I or II may
 C > R. Hence I is true. be true but not necessarily so.
From (iii) and (i), we get D < R  I  D < I. Hence II is 141. 2; H  J ...(i), J > Z ...(ii), M  J ...(iii)
not true. From (ii) and (iii), we get Z < J  M  Z < M. Hence
113. 2 114. 5 115. 3 116. 4 II is true.
117. 4; A > M ... (i), D  H ... (ii), M = D ... (iii) From (i) and (iii), we get H  J  M  no conclusion.
Combining all, we get, A > M = D  H  No relation- Hence I may be true but not necessarily so.
ship between A and H can be determined. Hence I is (142-146):
not true. We also get, M  H  II may be true but not Here, = means  , > means =, + means  , < means >,
necessarily so. and × means <.

K
118. 2 119. 3 120. 1 121. 4 122. 2 123. 3 124. 1 142. 4; A  B ...(i), P  R ...(ii), A < P ...(iii)
125. 4 126. 4 127. 4 128. 4 129. 1 130. 3 131. 2 From (i) and (iii), we get P > A  B  P > B or B < P
132. 2; R > K ...(i), P  S ...(ii), P = K ...(iii) ...(A). Hence I is false.
From (ii) and (iii), we get K = P  S  K  S ...(a). From (A) and (ii), we get B < P  R  B < R or R > B.
Hence I may be true but not necessarily so. Now, from Hence II may be true but not necessarily so.
(i) and (a), we get R > K  S  R > S or S < R. Hence 143. 4; N > O ...(i), E = F ...(ii), M < O ...(iii)
II is true. From (i) and (iii), we get N > O > M  N > M or M < N.
133. 4; A = E ...(i) I > O ...(ii), U < A ...(iii), I  U ...(iv) Hence I is not true. No relationship between E and O
From (iii) and (iv), we get I  U < A ...(a)  no con- can be established from the given statements.
144. 3; K = L ...(i), G  I ...(ii), Y  Q ...(iii)

KUNDAN
clusion. Hence I is not necessarily true. No relation-
ship between E and O can be established. No relationship between G and L can be established
134. 5; Q = R ...(i), T < L ...(ii), R < M ...(iii), L  Q ...(iv) but both the conclusions together are exhaustive.
From (ii), (iv) and (i), we get T < L  Q = R Hence either I or II follows.
 T < R. Hence II is true. 145. 5; R  U ...(i), B > R ...(ii), K  U ...(iii)
Now, From II and (iii), we get T < R < M  T < M From (i) and (ii), we get B > R  U  B > U or U < B.
or M > T. Hence I is true. Hence I is true From I and (iii), we get K  U < B 
135. 1; Z < N ...(i), D  G ...(ii), B < Z ...(iii), D = N ...(iv) K < B or B > K. Hence II is true.
From (i), (ii) and (iv), we get Z < N = D  G  Z < G 146. 4; M = H ...(i), 2 < H ...(ii), 2  N ...(iii)
or G > Z. Hence I is true. From (i) and (iii), From (i) and (ii), we get M = H > 2  M > 2 ...(A).
we get B < Z < N  B < N or N > B. Hence I is not true.
Hence II is not true. From (A) and (iii), we get M > 2  N  no conclu-
136. 3; H  Y ...(i), E  C ...(ii), K < Y ...(iii), K > C ...(iv) sion. Hence II is not true.
Combining all, we get H  Y > K > C  E  no 147. 5
relationship between H and E can be established. But 148. 5; E  H ...(i), 6 > P ...(ii), R < E ...(iii), 6 = R ...(iv).
I and II together are exhaustive. Hence either I or II is From (ii), (iii) and (iv), we get E > R = 6 > P  E > P..
true. Hence I is true.
137. 5; E < F ...(i); O  F ...(ii); P  E ...(iii) From I and (i), we get P < E  H  P < H or H > P..
Combining these, we get P  E < F  O. Hence II is true.
Hence F > P and P < F. 149. 1; M < N ...(i), R = Q ...(ii), A > Q ...(iii), M > R ...(iv)
138. 4; C > P ...(i), P  D ...(ii), C = G ...(iii) From (iii) and (ii), we get A > Q = R  A > R. Hence I
From (i) and (iii), we get G = C > P  G > P ...(a) is true.
From (i), (iv) and (ii), we get Q = R < M < N  Q < N or From (ii) and (iii), we can’t ascertain the relationship
N > Q. Hence II is not true. between W and Q. Hence, I does not follow. From (i)
150. 3; B < D ...(i), E  T ...(ii), T  P ...(iii), P > B ...(iv) and (iii), we get
From (i) and (ii), we get P > B < D  no definite T > Q  X  T > X.
conclusions; means P > D or P = D or P < D. Hence Hence, Conclusion II (X < T) is true.
either I or II follows. 165. 5; Y = Z ... (i); B > A ... (ii); Y < A ... (iii)
151. 4 Combining all these, we get
B > A > Y = Z  A > Z and B > Y..
152. 5; P > Q .... (i); Q  R .... (ii); R = S .... (iii); T < S .... (iv)
Hence Conclusion I (A > Z) and Conclusion II (Y < B)
Combining all the equations, we get
are true.
P > Q  R = S > T  T < P (conclusion I) 166. 3; K > L ... (i); N  O ... (ii); L = O...(iii)
and S  Q (conclusion II) Combining (ii) and (iii), we get
153. 4; S  A .... (i); A < K .... (ii); R  K .... (iii); L  R .... (iv) L = O  N  L  N, ie L > N or L = N
Combining all the equations, we get Hence, either Conclusion I (L > N) or
S  A < K  R  L  S < L and L > A. Hence, Conclusion II (N = L) is true.
conclusions I and II do not follow. 167. 4; X > F ... (i); T  C ... (ii); F  C ... (iii)
154. 3; G > H .... (i); F < H .... (ii); K  P ...... (iii); M  P ...... (iv) Combining (ii) and (iii), we get
Combining (iii) and (iv), we get T  C  F ... (iv)
KPMKM From (i) and (iv), no relation can be established be-

K
tween T and X nor between C and X.
 K > M (conclusion I) or K = M (conclusion II)
168. 4; 2 > 8 ... (i); 6  8 ... (ii); 5 < 6 ... (iii)
155. 1; A < P ... (i); Z < A .... (ii); N  Z .... (iii); L  N .... (iv)
From (ii) and (iii), no relation can be established be-
Combining all the equations, we get tween 5 and 8. Similarly, the given equations are not
L  N  Z < A < P  P > L (conclusion I) sufficient to establish relation between 2 and 5.
156. 2 169. 2; S < N ... (i); O  R ... (ii); S = R ... (iii)
157. 4; N = P ... (i); Q  P ... (ii); M  N ... (iii) Combining (i) and (iii), we get N > S = R ... (iv)
Combining all, we get From (ii) and (iv), no relation can be established be-
M  N = P  Q  M  P and N  Q tween N and O. From (ii) and (iii), we get O  R = S
Hence neither conclusion I nor II is true.  O  S.
158. 4; N  U ... (i); O  E ... (ii); E  N ... (iii) Hence, conclusion II (O  S) is true.

KUNDAN
No statement gives any clue to establish relation be- 170. 2; J < Y ... (i); J  O ... (ii); Y = K ... (iii)
tween E and U, or U and O. Combining (i), (ii) and (iii), we get
Hence, neither conclusion I nor II is true. K = Y > J  O  K > O. Hence, only conclusion II
159. 4; M < P ... (i); Z = H ... (ii); M  Z ... (iii) follows.
Combining all, we get 171. 2; D = B ... (i); R < L ... (ii); L > D ... (iii)
P > M  Z = H  P > Z and M  H. From (ii) and (iii), no relation can be established be-
Hence neither conclusion I (P  Z) nor conclusion II tween R and D. Hence, I does not follow. But conclu-
(M  H) is true. sion II follows from (i) and (iii).
160. 4; U  V ... (i); V  X ... (ii); W = U ... (iii) 172. 4; B  I ... (i); Y < A ... (ii); N  I ... (iii); Y > N .... (iv)
Combining (i) and (iii), we get From (i) and (iii), we get B  I  N ...... (v) From (ii) and
W = U  V  W  V. Hence conclusion I is not neces- (iv), we get A > Y > N ..... (vi). From (v) and (vi), we do
sarily true. Also, the given statements are not suffi- not get any specific conclusion between B and A or Y
cient to establish a relation between W and X. and I. Hence, neither I nor II follows.
161. 4 173. 4; There is no sign of # in the given statements. Hence,
162. 1; T  U ... (i); Q  R ... (ii); R < U... (iii) neither I nor II follows.
Combining all these, we get 174. 3; Q  P ... (i); T = Q ... (ii); T  N ... (iii); N > J .... (iv)
T  U > R  Q  T > Q and T > R. Hence, Conclusion Combining all statements, we get
I (T > Q) follows. But conclusion II (R  T) is false. P  Q = T  N > J  P  N. Hence, either conclusion
163. 4; H < B ... (i); J  C ... (ii); H > J ... (iii) I (P > N) or conclusion II (P = N) is true.
Combining all these, we get 175. 3; L = K ... (i); S = K ... (ii); L  B ... (iii); R  B ... (iv)
B > H > J  C  B > C and H > C. Hence, Conclusion Combining (iii) and (iv), we get L  B  R  R  L, ie
I (B  C) and Conclusion II (C  H) do not follow.. R < L or R = L.
164. 2; Q < T ... (i); W > X ... (ii); Q  X ... (iii) Again from (i) and (ii), since L = K = S, hence either
conclusion I (R < K) or conclusion II (R = S) is true. From (v), we get N > R (conclusion I). From (ii) and (iv),
176. 1 we get
177. 4; J = O .... (i); T  O .... (ii); T  Y .... (iii); E  J .... (iv) S  O=Q  S  Q
By combining (i), (ii), (iii) and (iv), we get E  J = O Hence, conclusion II is not necessarily true.
 T  Y  E  T. Hence, both the conclusions do 188. 5; T  U .... (i); U > Q .... (ii); Q < S ..... (iii); S < U ..... (iv)
not necessarily follow. From (i) and (iv), we get
178. 5; T < R .... (i); P > A .... (ii); R  A .... (iii); L = T .... (iv) T  U > S  T > S (conclusion I)
Combining all, we get Again, from (i) and (ii), we get
P > A  R > T = L  P > L and A > L. Hence, both the
T  U > Q  T > Q (conclusion II)
conclusions are true.
189. 1; C < D .... (i); D = E .... (ii); E > F ..... (iii); F > G ..... (iv)
179. 2; T  O .... (i); O  P ... (ii); P = A ... (iii); A  Z .... (iv)
From (ii), (iii) and (iv), we get
Hence, conclusion I does not follow from equations (i)
and (ii). But conclusion II follows from equations (ii) D = E > F > G  D > G (conclusion I)
and (iii). But conclusion II is not necessarily true because C
180. 3; D  E ... (i); S < E .... (ii); S = X .... (iii); M > S .... (iv) and G can’t be related.
By combining (i), (ii) and (iii), we get 190. 2; K  L ..... (i); L > I ..... (ii); I < M ...... (iii); I > N ...... (iv)
D  E > S = X ..... (v) From (i), (ii) and (iv), we get
Comparing (iv) and (v) we do not get any specific rela- K  L > I > N  K > N (conclusion II)
tion between D and M. But the given two conclusions But no relation can be obtained between L and M.

K
are complementary to each other. Hence, either of the Hence, conclusion I is not necessarily true.
two must follow. 191. 1; V  X ..... (i); X < Y ...... (ii); Y = Z ...... (iii); Y > A ...... (iv)
181. 4; A  P ... (i); A > T .... (ii); T = N .... (iii); N  S .... (iv) From (i) and (ii), we get
Combining all the equations, we get Y > X  V  Y > V (conclusion I)
P  A > T = N  S  P > S. Hence, both the conclu- But no relation can be obtained between X and A.
sions do not follow. Hence, conclusion II is not necessarily true.
182. 3; G > H ...(i), T  H ...(ii), T = K ...(iii), K  Z ...(iv) 192. 5; L > M ..... (i); M > N .... (ii); L  O ..... (iii); P  O .... (iv)
Combining all, we get G  H  T  K  Z From (i) and (ii), we get
Hence, we do not get any specific relation between G L > M > N  L > N  N < L (conclusion I)
and Z. But the given conclusions form a complemen- Now, from (ii) and (iii), we get L  O  P  L  P 

KUNDAN
tary pair. Hence, either I or II follows.
P  L (conclusion II).
183. 1; N  U ...(i), U  M ...(ii), N  B ...(iii), M = R ...(iv) 193. 2; B > C ... (i); A > B ... (ii); C  D ... (iii)
Combining (i), (ii), (iii) and (iv), we get Combining all these we get
RMU NB  RU A > B > C  D  A > D and B > D
Hence, conclusion I R  U  is true. But we can’t get Hence, conclusion I does not follow but conclusion II
any specific relation between M and B. Hence, conclu- follows.
sion II M  B  is not necessarily true. 194. 5; M = N ... (i); N  P ... (ii); P  R ... (iii)
184. 1; P < M ...(i); R  M ...(ii); S = R ...(iii); S > Y ...(iv) Combining (i) and (ii), we get P  N  M ... (iv)
From (iii) and (iv), R > Y. Hence I follows. But no rela- Hence from (iv), we get P  M (conclusion I)
tion can be obtain between P and Y. Hence II does not Now, P  R  R  P (conclusion II).
follow. 195. 3; X > R ... (i); R < S ... (ii); X = T ... (iii)
185. 5; L  M ...(i); M > N ...(ii); R < N ...(iii); R  F ...(iv) From (i) and (iii), we get X = T > R ... (iv)
Combining all, we get From (iv), T > R From (ii), S > R
L  M  N  R  F  M > F and N > F Thus from (ii) and (iv) we can’t ascertain the relation-
Hence, both the conclusions are true. ship between T and S. But the given two conclusions
186. 2; G > H ...(i); I < H ...(iii); P  Q ...(iii); Q > R ...(iv) make a complementary pair. Hence, either conclusion I
From conclusions (i) and (ii), we get T  S or conclusion II (T < S) is true.
G > H > I. Hence, conclusion I (G < I) is not true. 196. 1; R > M... (i); M = Y... (ii); Y  Z ... (iii)
Again, from conclusions (iii) and (iv), we get P  Q  R Combining all these, we get
 P > R. Hence, conclusion II P  R is true. R  M  Y  Z  Z  R (conclusion I)
187. 1; N > O .... (i); O = Q .... (ii); R < Q .... (iii); O  S..... (iv) 197. 4; T = Y ... (i); P  X ...(ii); P  Y ... (iii)
Combining (i), (ii) and (iii), we get N > O = Q > R ...... (v) Now, P = X (conclusion I) does not follow from (ii).
Similarly, P = Y (conclusion II) does not follow from between M and T. Hence, neither conclusion I nor con-
(iii). clusion II is necessarily true.
198. 4; K = Y ... (i); N = I ... (ii); I  K ... (iii); N  P .... (iv) 209. 4; S < M ... (i); M > L ... (ii); L  Z .... (iii)
Combining all, we get Combining (ii) and (iii), we get M > L  Z ... (iv)
P  N = I  K = Y  P  I and Y  I. Now from (i) and (iv), no specific relation can be ob-
Hence, neither I nor II is necessarily true. tained between S and Z. Hence, neither conclusion I
199. 1; V > I ... (i); C < I ... (ii); C = Y ... (iii); K  C ... (iv) nor conclusion II is necessarily true.
Combining (i), (ii) and (iii), we get V > I > C = Y ... (v) 210. 4; D > F ... (i); F = S ... (ii); S  M ... (iii)
From (iv) and (v), we do not get any specific relation
From (ii) and (iii), we get F  M. Therefore, conclusion
between I and K. Hence, II is not necessarily true.
I and conclusion II are not true.
From (iii) and (iv), we get
K  C=Y  K  Y  Y  K 211. 1; J = V ... (i); V < N ... (ii); R  J ... (iii)
Hence, conclusion I is true. Combining all, we get
200. 2; K  N ... (i); S > T ... (ii); I < N ... (iii); T  I ... (iv) N > V = J  R  N > R (conclusion I) and N > J.
Combining (i), (iii) and (iv), we get Hence, conclusion I is true but conclusion II is not
K  N>I  T  K>T  T<K true.
Hence, conclusion II is true but conclusion I is not 212. 3; L  U ... (i); C < L ... (ii); C > B ... (iii)
true. From (i) and (ii) no specific relation can be obtained
201. 1; L = E ... (i); E > P ... (ii); P = R ... (iii); R  D ... (iv) between C and U. But conclusion I and conclusion II

K
Combining all, we get make a complementary pair. Hence, either conclusion I
L = E > P = R  D  L > D  D < L. or conclusion II is true.
Hence, conclusion I is true but conclusion II is not 213. 4; R  P ... (i); B < P ... (ii); E = B ... (iii); F > B ... (iv)
true. From (i) and (ii), we get R  P > B and from (iii) and (iv)
202. 3; T  I ... (i); R > T ... (ii); N  I ... (iii); S  I ... (iv) we get F > E = B.
Combining (iii) and (iv), we get S  I  N Hence, no specific relation can be obtained between P
 S  N  N  S and F, and R and F. Therefore neither conclusion I (P <
Hence, N < S or N = S. F) nor conclusion II (R > F) is necessarily true.
203. 4; G  H ... (i); T  S ... (ii); H = O ... (iii); O > S ... (iv) 214. 4; G  H ... (i); L = K ... (ii); H = E .... (iii); M  K ... (iv)
Combining all equations, we get From (i) and (iii), we get
G  H = O > S  T  S < G and H > T G  H = E  G  E, ie G > E or G < E. From (ii) and (iv),

KUNDAN
Hence, neither conclusion I (S  G) nor conclusion II we get
(H  T) is true. M  K = L  M  L, ie M > L or M < L
204. 3; M  E ... (i); A = P ... (ii); L  P ... (iii); A < M ... (iv) Hence neither conclusion I (G  E) nor conclusion II
Combining (ii), (iii) and (iv), we get M > A = P  L ... (v) (L  M) is true.
We can’t obtain any specific relation between E and A 215. 2; B  K ... (i); T < B ... (ii); T  K ... (iii); K > S .... (iv)
or between E and P. But these two conclusions make a From (iii) and (iv), we get T  K > S  T > S
complementary pair. Hence, conclusion I (T  S) is not true. But conclusion
205. 4; Z > E ... (i); R  Z ... (ii); E  B ... (iii); A < R ... (iv) II (T > S) is true.
Combining (i) and (iii), we get Z > E  B ... (v) 216. 5; F > K ... (i); P = K ... (ii); P  H ... (iii); T < P ... (iv)
Combining (ii) and (iv), we get Z  R > A .... (vi) From (i), (ii) and (iv), we get
From (v) and (vi), no specific relation can be obtained F > K = P > T  F > T  T < F (conclusion I)
between E and R, or B and A. Hence, neither conclu- From (i), (ii) and (iii), we get F > K = P  H  F > H
sion I nor conclusion II is true.  H < F (conclusion II)
206. 3; K  T ... (i); S  G ... (ii); G = H = T ... (iii) 217. 3; D  L ... (i); L = P ... (ii); P  T ... (iii); T  Z ... (iv)
Combining all, we get S  G = H = T  K  S  K From (i), (ii) and (iii), we get D  L = P  T
ie S > K or S = K Hence, no specific relation can be obtained between D
207. 5; P  S ... (i); A  D ... (ii); A > I ... (iii); I = S ... (iv) and T. But one of the following three relations must be
From (i) and (iv), we get there between D and T:
P  S = I  P  I (conclusion I) a. D > T, ie D  T b. D < T, ie D  T c. D = T
From (ii), (iii) and (iv), we get Hence, either conclusion I (D  T) or conclusion II (T
D  A > I = S  S < D (conclusion II) = D) is true.
208. 4; P > Q ... (i); M  K ... (ii); T  K ... (iii) 218. 5; F > A ... (i); I < R ... (ii); I  A ... (iii); I > L ... (iv)
From (ii) and (iii), no specific relation can be obtained From (ii) and (iv), we get
R > I > L  R > L  L < R (Conclusion I) 226. 3; K = R ... (i); R  T ... (ii); T  Z ... (iii); Z = S ... (iv)
From (i), (iii) and (iv), we get Combining (i) and (ii) only, we get K = R  T  K  T..
F > A  I > L  F > L (Conclusion II) Therefore, one of the given conclusions K > T (con-
Hence, both the conclusions I and II are true. clusion I) and K < T (conclusion II) must be true.
219. 5; P < M ... (i); K  P ... (ii); S  P ... (iii); S  G ... (iv) 227. 5; Q = M ... (i); M > N ... (ii); N = P ... (iii); L  P ... (iv)
From (i), (iii) and (iv), we get Combining all, we get
M > P  S  G  M > G (Conclusion I) Q = M > N = P  L  Q > L (Conclusion I) and
From (ii), (iii) and (iv), we get M > P  P < M (Conclusion II)
K  P  S  G  K  G  G  K (Conclusion II) 228. 3; M  N ....(i); L = M ....(ii); L > O ....(iii)
Hence, both the conclusions I and II are true. Combining (i) and (ii),
220. 3; N  T ... (i) G = L = T ... (ii); S < L ... (iii) L = M  N. The above equations give no specific
From (i) and (ii), we get relation between O and N. Hence, neither conclusion I
N  T = L = G  N  L  N > L (Conclusion I) or nor II follows. But the options when combined give a
N = L (Conclusion II) complementary pair.
Hence, either conclusion I or conclusion II is true. 229. 4; A < C ....(i); P = Q ....(ii); Q  R ....(iii)
221. 1; M = P ... (i); S  M ... (ii); S  Z ... (iii); T  P ... (iv) There is no information regarding the relation between
From (i), (ii) and (iii), we get A and R. Hence I cannot be proved. Combining (ii) and
P = M  S  Z  P  Z (Conclusion I) (iii), we get R  Q  P
But, conclusion II P  Z is not true.  P  R. Hence conclusion II is also not true.

K
222. 3; H  K ... (i); T  H ... (ii); K = F ... (iii); S = T ... (iv) 230. 2; W = X > V ....(i); X > Y ....(ii); Y < Z ....(iii)
From (i) and (iii), we get H  K = F ... (v) From (i), we get W > V, hence conclusion II is true.
From (ii) and (iv), we get H  T = S ... (vi) From (i), (ii) and (iii) no specific relation can be ob-
From (v) and (vi), we do not get any specific relation tained between Z and V. Hence, conclusion I is not
between K and T. necessarily true.
Now, look carefully at conclusion II. F  S can be 231. 5; M  N ....(i); N < O ....(ii); O < P ....(iii)
written as K  T because F = K and S = T.. Combining all we get
Hence, either conclusion I K  T  or conclusion II M  N < O < P  M < P (conclusion I)
and O > M (conclusion II)
K  T  is true. 232. 1; R > A ....(i); S < B ....(ii); A > B ....(iii)

KUNDAN
223. 4; S > P ... (i); L  Q ... (ii); P  R ... (iii); R = Q ... (iv) Combining all, we get, R > A > B > S  S < R. Hence
Combining all, we get S > P  R = Q  L conclusion I is true. But conclusion II (B  R) is not
From the above equation we can’t get any specific true.
relation between L and S. Hence, conclusion I (L < S) is 233. 3; P = Q ... (i); R > P ... (ii); Q  S ... (iii)
not true necessarily. Again the above equation gives
From (i) and (iii), S  P  either S > P or S = P
us the conclusion P  Q. This does not imply neces-
sarily that Q < P. Hence, conclusion II (Q < P) is not 234. 1; X  Y ... (i); Y = R ... (ii); Y < S ... (iii)
necessarily true. From (i) and (ii), X  R. Hence I is true.
224. 5; M  T ... (i); G  T ... (ii); G  H ... (iii); T < K ... (iv) From (ii), II is false.
From the equations (ii), (iii) and (iv), we get 235. 5; P < Q ... (i); Q < R ... (ii); R = S .... (iii)
K > T  G  H  K > H  H < K (Conclusion I) Combining these, we get P < Q < R = S
Again, from the equations (i), (ii) and (iii), we get 236. 4; A  B ... (i); B  C ... (ii); C > D ... (iii)
M  T  G  H  M    H  M (Conclusion II) No relationship can be established between A and D
225. 3; Note that the given conclusions are about the relation from the given statements.
between G and P. The last two equations. G # T and T 237. 1; P < Q ... (ii); R > S ... (ii); Q = R ... (iii)
# P will help decide the answer. From (i) and (iii), P < R. Hence I is true while II is false.
G  T ... (i); T  P ... (ii) 238. 2; E > F ... (i), F = G ... (ii); H  G ... (iii)
From (i) and (ii) G  T  P I is false because of (ii).
The above equation gives no specific relation between From (i) and (ii), G < E. Hence, II is true.
G and P. 239. 4; A = B ... (i), A  C ... (ii); C > D ... (iii)
But the given conclusions are From (i) and (ii), B  C. Which means B is certainly not
I. G  P II. G = P lesser than C. So I does not follow.
Hence, no doubt, one of the above equations must be Again, combining (i), (ii) and (iii), we get
true.
A=B  C> D no other letter. Nor do we have any other relationship
Thus A is certainly not equal to D. Hence, II does not in terms of V.
follow. 250. 2; L  U ... (i); C < L ... (ii); C > B ... (iii)
240. 3; O  P ... (i); O > Q ... (ii); O  R ... (iii) I cannot be established even from (i) and (ii). II is es-
Combining (i) and (iii), we get P  O  R, which im- tablished by (ii) and (iii).
plies P  R. Now, P  R means P is either lesser than R 251. 4; D > F ... (i); F = S ... (ii); S  M ... (iii)
or P is equal to R. In other words, either conclusion I or From (i) and (ii), D > S ... (iv)
II. No relationship can be established between D and M
241. 4; N  O ... (i); P < O ... (ii); P > R ... (iii) even from (iii) and (iv). Hence I does not follow. From
Note: N  O means N is lesser than or equal to O, that is, (ii) and (iii), F  M. Hence II does not follow..
N < O or N = O. It could be either of the two. So 252. 3; P > T ... (i); M  K ... (ii); T = K ... (iii)
neither of them is definitely true. Thus I does not fol- Combining (ii) and (iii), we get T = K  M or T  M
low. That is, either T > M or T = M, In other words, either
Again, from (ii) and (iii), O > R. Hence II does not I or II follows.
follow. 253. 4; S  T ... (i); T  U ... (ii); T < V ... (iii)
242. 5; L < M ... (i); N  M ... (ii); M  O ... (iii) From (i), T  S. Hence I is not true.
From (i) and (ii), L < N. Hence I follows. From (ii) and (iii), U  T < V or U < V. But II is
From (ii) and (iii), N  O. Hence II follows. U  V. Hence II is also not true.

K
243. 1; The statements can be decoded as (1) B > C, 254. 4; K > L ... (i) ; K  M ... (ii); M < N ... (iii)
(2) A > B, (3) C  D. From (1) and (3) we have B  D. From (i) and (ii), no relationship can be established
But conclusion II says B > D which is not the same as between L and M. Hence I is not true.
B  D. Hence II is not definitely true. Further, from (2), From (ii), M  K. Hence II is also not true.
(1), (3) we have A > B > C  D  A  D. Hence I is 255. 4; F  G ... (i); F  H ... (ii); F  K ... (iii)
definitely true. No relationship can be established between G and K.
244. 5; The three statements can be decoded as (1) M = N, (2) Hence I cannot be established
N  P, (3) P  R. Now, (1) and (2) give: M  P which From (ii) and (iii), K  F  H or K  H. Hence I is
is conclusion I while (3) gives conclusion II. false.
245. 3; The three statements give: 256. 2; T = S ... (i); R < Q ... (ii); Q = T ... (iii)

KUNDAN
(1) X > R, (2) R < S, (3) X = T. From (i) and (iii), Q = S. Hence I is not true.
From (1) and (3) we have: T > R. From (3) we have S > Combining (i), (ii) and (iii), S = T = Q > R or S > R. Hence
R. Hence, T and S are both greater than R and so no II is true.
relationship can be established between T and S. But 257. 4; B > C ... (i) ; C < D ... (ii); E < C ... (iii)
any two numbers are either equal or one is less than or No relationship can be established between B and D.
greater than the other. Since I and II cover all these Hence I can’t be established. From (ii), D > C. Hence II
three possibilities, either of them must be true. is false.
246. 1; The three statements give (1) R > M, (2) M = Y, 258. 3; P  L ... (i); N > Q ... (ii); M  N .... (iii); M  L ... (iv)
(3) Y  Z. This can be combined as R > M = Y  Z, Combining all, we get,
i.e. R > Z. This is I. II can’t be evaluated as variable A P  L  M  N  Q Hence, P > N or P = N
is not present in the statements.
259. 1 260. 2 261. 4 262. 4
247. 5; The three statements give (1) X = Y, (2) P  X,
263. 1; L = M ... (i); S  M ... (ii); M > N ... (iii);
(3) P  Y. From (1) and (2), we have P  Y while from
Q  L ... (iv)
(3), P  Y. Now, P  Y and P  Y both can be true if
Combining (i), (iii) and (iv), we get
and only if P = Y. Hence both conclusions are true.
248. 4; S < M ... (i); M > L ... (ii); L  Z ... (iii) Q  L  M  N. Hence conclusion I is true. Con-
I does not follow because no relationship can be es- clusion II is not true.
tablished between S and Z. II also does not follow 264. 5; S < G ... (i); R  J ... (ii)
because no relationship can be established between S K > R ... (iii); K  S .... (iv)
and L either. Combining all these relations, we get
249. 4; J = L ... (i); V < N ... (ii); R  J ... iii) G  S  K  R  J.
The relationship of N is established only with V and Since G > R, therefore R < G is true.
Similarly J < S. Hence II is also true. (conclusion I) is true. Again combining (i), (ii) and (iii)
265. 4 266. 3 267. 2 268. 1 269. 4 270. 5 we get G = P  Q  S. Hence G  S (conclusion II) is
271. 5 272. 3 true.
273. 5; G > U... (i); L  U ... (ii); T = L ... (iii); T  M ... (iv) 303. 4
Combining all these, we get G > U  L = T  M. 304. 3; Since S = T and S  W therefore, T  W. Hence
Hence G > M and M  U. either conclusion I or conclusion II must be true.
274. 4 275. 3 276. 3 277. 2 278. 3 279. 5 280. 4 305. 4; Here J  K ....(i); M  N ....(ii);
281. 2 282. 1 283. 4 284. 1 285. 2 286. 5 287. 3 L  N ....(iii); K = M ..... (iv)
288. 4 Combining (ii), (iii) and (iv) we get K = M  N  L.
(289-294): Here the direction proves a little troublesome. Hence no relationship between K and L can be deter-
But think logically. You infer P + Q means P is either mined.
less than or equal to Q. P × Q means P is either greater 306. 4; Here L < G ....(i); B < K ....(ii); L = S ....(iii); B  L ...(iv).
than or equal to Q. P @ Q means P is greater than Q. P Combining (i) and (iii), we get S < G. Hence I is not true.
÷ Q means P is less than Q. Now, go through all the No relationship between S and K can be determined.
questions with this form of direction. 307. 3; There is no such statement regarding the relationship
289. 4 290. 3 291. 3 292. 1 293. 1 294. 2 between M and S. But conclusions I and II are comple-
295. 5; When we combine the first two equations, ie G < Q mentary to each other.
and R  Q we get R  Q > G. Hence R > G (conclu- 308. 3; No relationship between P and K can be determined.

K
sion I) is true. Combining the third and the fourth But conclusions I and II make a complementary pair.
equations, we get Hence either I or II is true.
R  T = S. Hence S  R is also true. 309. 5; Combining all the given information, we get
296. 4; Combining all the informations we get D  G  S  K  P  D  P and G  K . Hence
K = M  S = T  V. The valid relation between K and conclusions I and II are true.
S is K  S and between K and V is K  V. Hence both 310. 3; Here S = A ..... (i); P  A ..... (ii);
I and II are not true. P  L ..... (iii); L > G ..... (iv)
297. 2; Combining all the informations we get Combining all these we get
G = K  X > Y  Z. From this, we get G > Z and S = A  P  L > G. Hence, S  L ie S > L or S = L.
K > Z. Hence conclusion I is not true. 311. 4; Here, M  N ... (i); N  P ... (ii);

KUNDAN
298. 3 P > Q ..... (iii); R < Q ..... (iv)
A C G M From the above equations no specific relation can be
299. 2;  ... (i) ;  .... (ii); determined between N and Q.
B D H N
312. 1; Here, H > A ..... (i); H < G .... (ii);
G.B = H.A ... (iii); B = D ... (iv);
G = S ..... (iii); A  T ..... (iv)
A G Combining all these we get S = G > H > A  T..
From (iii) 
B H Hence, G > T (Conclusion I) is true. But conclusion II is
not true.
A G C
Hence   . 313. 3; Here, R  Z ..... (i); R = H .... (ii);
B H D
R = G .... (iii); G > X ..... (iv)
A G Combining all these we get
Hence conclusion II is true. If  then value of
B H Z  R = H = G > X. Hence, we get Z  G, ie Z > G or Z
A will depend on value of B. Hence I is not true. = G. Therefore Z > G (conclusion I) and Z = H = G
300. 1; Combining all the given statements, we get G > H = K (conclusion II) make an exhaustive case. Hence, either
= S  J. Hence only conclusion I is true. of them must be true.
314. 2; Here, P < L .... (i); L  S .... (ii);
 K  J , therefore conclusion II is not true.
301. 2; Combining all the given statements, we get S < K ..... (iii); L = M .... (iv)
L = M  N > J = R. From this we get L > R. Hence Combining all these we get K > S  L = M > P..
conclusion I is not true. Conclusion II is obvious from Hence K > M, which makes conclusion I false. But M
the equation.  S (conclusion II) is true from the above equation.
302. 5; G = P .... (i); P  Q .... (ii); Q  S ..... (iii); P > T .... (iv) 315. 5; Here, A > B .... (i); Y  X .... (ii);
Combining (i) and (iv), we get G = P > T. Hence, G > T C < B .... (iii); Y  Z ..... (iv)
Combining equations (ii) and (iv) we get X  Y  Z. than or equal to’) any other term in the given state-
Hence conclusion I is true. Again, combining equations ments, then it can’t be less than (or, “less than or equal
(i) and (iii) we get A > B > C. Hence, C < A is also true. to”) any term in a conclusion. (Such a conclusion is
316. 5; P < K .... (i); P  M .... (ii); G = T .... (iii); M < T .... (iv) definitely false.) Thus II is not true.
Combining (i) and (ii), we get 329. 1; Z  M .... (i), B  S .... (ii), N > Z .... (iii), N  S .... (iv)
K > P  M. Hence, we get M < K ( conclusion I). Again, Combining (i), (ii) and (iii), we get
by combining (iii) and (iv) we get G = T > M. Hence, we S  N > Z  M  S > M. Hence, I is true. Look the
get M < G (conclusion II). equations carefully. Is there any sign of  ? Your an-
317. 4; By an intelligent observation you come to know ele- swer is ‘No’. Hence II is not true.
ments in the first equation have no direct or indirect 330. 4; F > M .... (i), B  O .... (ii), F < W .... (iii), B = W .... (iv)
relations with the elements in the other three equations. Combining (i), (ii), (iii) and (iv), we get
318. 3; G  L .... (i); G = T .... (ii); T  P .... (iii); P  K .... (iv) O  B = W > F > M  O  W. Hence I is not true. II is
also not true because B > F.
Combining all these, we get L  G = T  P  K
331. 2; C > W ... (i); L < D ... (ii); D  C ... (iii).
319. 4; T  K ..... (i); K  L ..... (ii)
From (i) and (ii) we do not get any relation between T Combining (ii) and (iii), we get
and L. The given two conclusions are not exhaustive C  D > L. Hence, C > L (conclusion II) is true. Wee
either. Hence neither I nor II are true. can’t get relation between W and D on the basis of the
320. 4; A < T .... (i); Z = A .... (ii); Z  K .... (iii); P  K .... (iv) given statement. Hence, I is not true.
332. 1; M > V .... (i); U = M .... (ii); V  T .... (iii)

K
Combining all these, we get T > A = Z  K  P
Combining (i) and (ii), we get
Hence we get Z  P and A  P..
M = U > V. Hence U > V (conclusion I) is true. We have
321. 4; L  T .... (i); S < T ..... (ii); W = S .... (iii); W > K .... (iv)
information that V is either less than or equal to T. Hence,
Combining all these, we get L  T > S = W > K
concentrating only on latter (conclusion II) is not true.
Hence there is no specific relationship between L and S
333. 3 334. 4 335. 5
or W.
336-337: Here directions for these questions are in the sim-
322. 3; D  E .... (i); F  E ... (ii); F = G .... (iii); H = G ... (iv)
plest form as following:
Combining all these, we get D  E  F = G = H.
A @ B means A is greater than B.
Hence, we get E  G.
A + B means A is either greater or equal to B.
Therefore E is either greater or less than G.
A $ B means A is equal to B.
323. 3; T  K ... (i); G  H .... (ii); T = H .... (iii); L = K .... (iv)

KUNDAN
A  B means A is less than B.
Combining all these, we get L = K  T = H  G
A  B means A is either less than or equal to B.
Hence L  G ie L > G or L = G
324. 2; J > G .... (i); G = M .... (ii); M  N ... (iii); N = S .... (iv) 336. 5; X  Z ... (i); T  Z .... (ii); T > M ... (iii); N < M ... (iv).
Combining all these, we get J > G = M  N = S Combining (i) and (ii), we get X  T (conclusion I).
We can’t find any specific relation between J and S. Again, combining (iii) and (iv), we get N < T (conclu-
Hence S  J (conclusion I) is not true. Again, since sion II).
M  N and G = M and N = S, therefore G  S (conclu- 337. 4; X > Z .... (i); Z = T .... (ii); T  M .... (iii); N < M .... (iv).
sion II) is true. Combining (i), (ii), (iii) and (iv), we get X > Z = T  M >
325. 1 N. This implies X > N. Hence, both the conclusions are
326. 4; Q > B .... (i); J < E .... (ii); L  B .... (iii); J  Q .... (iv) not true.
By an intelligent observation we find there is no sign of Note: For a quick method you need to use Tip 8, one of the
timesaving tips given in the book Magical Book Se-
 in any equation given in the statements. Hence, we
ries: Analytical Reasoning by MK Pandey.
can conclude within a second that both I and II are not
The first part of tip 8 says, “If a term is not less than (or
true.
“less than or equal to”) any other term in the given
327. 3; V  T .... (i), O < B ... (ii), I > V ... (iii), B  T .... (iv)
statements, then it can’t be less than (or “less than or
From (i) and (iv), we get
equal to”) any term in conclusion. (Such a conclusion
V  T  B  V > B or V = B. Hence, either I or II is true.
is definitely false).”
328. 4; Again, by an intuitive look we come to know L is not
338. 5; T > M ... (i); N  M .... (ii); N = X ... (iii); Z < X .... (iv).
equal to any element in the given equations in the state-
Combining I, II, III and IV, we get
ments. Hence, I is not true. Again, as you have in our
T > M  N = X > Z. Hence, T > Z (conclusion I) and
Magical Book Series on Analytical Reasoning by MK
M  X (conclusion II) are true.
Pandey, Tip 8 says “If a term was not less than (or, ‘less
339. 3; Z  X .... (i); T < X .... (ii); M  N .... (iii); P > N ... (iv)
Combining (i) and (ii), we get Z  X > T, and by combin- ship between H and M can be established. Hence I
ing (iii) and (iv), we get P > N  M. Here, we can’t get does not follow.
any clues by which the relation between P and T can be From (i) and (iii), we get Q  M  N  Q  N. Hence
found out. But the given conclusions are exhaustive in II may be true but not necessarily so.
nature. Hence, either I or II must be true. 352. 2; C > B ...(i), L < S ...(ii), S  C ...(iii)
340. 4; A > B ... (i); B  C ... (ii); C  D ... (iii); D < E ... (iv). From (i) and (iii), we get B < C  S  no relationship
Combining (i) and (ii), we get A > B  C. Similarly, by between B and S can be established. Hence I is not
combining (iii) and (iv), we get
true. From (ii) and (iii), we get L < S  C  L < C
E > D  C. Hence, despite the combination we have no
or C > L. Hence II is true.
information of the relation between A and E. We reject
both conclusions because A might be equal to E. 353. 5; I  H ....(i), E > F ....(ii), I = F ...(iii)
341. 2; Z < N ... (i); F  N ...(ii); F  K ... (iii) From (ii) and (iii), we get E > F = I  E > I. Hence I is
Combining (i), (ii) and (iii) we get true. From I and (i), we get E > I  H  E > H
K  F  N > Z. Hence, we get K  N. On its basis or, H < E. Hence II is true.
conclusion I is not necessarily true. But II is obvious 354. 3; V = O ...(i), R  V ...(ii), O  B ...(iii)
because the combination gives us K > Z. Combining all, we get R  V = O  B  R  B.
342. 3; D = T ...(i); T  M ...(ii); M < K ... (iii) From (i) and (ii), we get Hence either I or II is true.
D = T  M  D  M. Hence, D > M or D = M (Conclusion 355. 4; L > U ...(i), T = L ...(ii), U  W ...(iii)
II or I). Combining all, we get T = L > U  W  no relation-

K
343. 3; W  A ... (i); B  A ... (ii); B > M ... (iii) From (i) and (ii),
ship between T and W can be established. Hence I is
we get W  A  B  W  B. Hence B < W or W = B
not true. From (iii), II may be true but not necessarily
(Conclusion I or II).
so.
344. 1; J  M ... (i); M = N ... (ii) N < T ... (iii) Combining (i), (ii)
356. 3; K  L ...(i), M < P ...(ii), J > K ...(iii), P  L ....(iv)
and (iii), we get T > M = N  J  T > J. Hence I is true
but II is false. From (i) and (iv), we get K  L  P  K  P
345. 4; V  F ... (i); F > R ... (ii); R  G ... (iii) Combining (ii) and  P  K  either K = P or P < K
(iii), we get F > R  G. But comparing F > R  G with 357. 4; F > M ...(i), A  L ...(ii), F < G ...(iii), A = U ...(iv)
equation (i) we get no relation between V and G. How- From (ii) and (iv), we get U = A  L  U  L or
ever, one of the following three cases must be true: L  U. Hence I may be true but not necessarily so.

KUNDAN
1. G < V; 2. G > V; 3. G = V. But the given conclusions do The relation between A and F can’t be established.
not consist of case 3. Hence neither I nor II is true. 358. 2; P > B ...(i), J < H ...(ii), S  B ...(iii), J  P ...(iv)
346. 4; B > K ... (i), E  H ... (ii), K = E ... (iii) From (iv), (i) and (iii), we get J  P > B  S  J > S.
Combining all, we get, B > K = E  H  No relation- Hence I is not true.
ship between B and H can be determined. Hence I is From (i), (ii) and (iv), we get H > J  P > B  H > B.
not true. We also get, K  H  II may be true but not Hence II is true.
necessarily so. 359. 1; M > Z ...(i), B  R ...(ii), Z  C ...(iii), M  R ...(iv)
347. 2; R  S ...(i), M = S  N ...(ii), J  S ...(iii) From (i) and (iv), we get Z < M  R  Z < R or R > Z.
From (ii), I may be true but not necessarily so. Hence I is true.
From (i) and (iii), we get R  S  J  R  J. Hence II From (ii) and (iv), we get B  R  M  No definite
is true. relationship between B and M can be established.
348. 3
360. 4; X  Y ...(i), U  V ...(ii), X > S ...(iii), V = Y ...(iv)
349. 1; G  Q ...(i), O < P ...(ii), Q = O ...(iii)
I is not necessarily true from (ii).
Combining all, we get G  Q = O < P  G < P Combining all, we get U  V = Y  X > S  U > S.
or, P > G. Hence I is true. We also get Q < P  II may Hence II is not true.
be true but not necessarily so. 361. 4; X < Y ...(i), Z  V ...(ii), Y  Z ...(iii)
350. 4; L > U ...(i), U  F ...(ii), F  C ...(iii) Combining all, we get X < Y  Z  V  No relation-
Combining all, we get L > U  F  C  no relation- ship between X and V can be established. Hence I is
ship between L and C can be established. Hence I is not true.
not true. II may be true but not necessarily so. From (ii) and (iii), we get Y  Z  V  Y  V. Hence
351. 4; M  N ...(i), H  Q ...(ii), Q  M ...(iii) II may be true but not necessarily so.
From (ii) and (iii), we get H  Q  M  no relation- 362. 1; Q < P ...(i), S  R ...(ii), P = S ...(iii)
From (i) and (iii), we get Q < P = S  Q < S. Hence I is lished. Hence II not true.
true. 377. 2; A  B ... (i), C > B ... (ii), C  D ... (iii)
From (ii) and (iii), we get P = S  R  P  R  II From (ii) and (iii), we get D  C > B  D > B. Hence II
may be true but not necessarily so. is true.
363. 2; F  E ...(i), G = E  H ...(ii), I  E ...(iii) From II and (i), we get D > B  A  no definite
From (ii), I may be true but not necessarily so. relationship between D and A can be established.
From (i) and (iii), we get F  E  I  F  I. Hence II Hence I is not true.
is true. 378. 5; T  U ... (i), W  V ... (ii), V > U ... (iii)
364. 4; A > B ...(i), J < L ...(ii), B = J ...(iii) From (i) and (iii), we get T  U < V  T < V
Combining all, we get A > B = J < L  no relationship or V > T. Hence II is true.
between A and L can be established. Hence I is not From (ii) and II, we get W  V > T  W > T  W 
true. T. Hence I is true.
From (ii) and (iii), we get B = J < L  B < L. Hence II is 379. 3; L = N ... (i), K > L ... (ii), M  N ... (iii)
false. From (i) and (iii), we get L = N  M  L  M. Hence
365. 4; M = N ...(i), N < Q ...(ii), N  R ...(iii) either I or II is true.
From (ii) and (iii), we get Q > N  R  Q > R  I is 380. 4; P  Q ... (i), R > S ... (ii), P > R ... (iii)
false. Combining all, we get Q  P > R > S  no definite
From (i) and (iii), we get M = N  R  M  R  II relationship between Q and S can be established. Hence

K
may be true but not necessarily so. I is not true. From (ii) and (iii), we get
366. 2; C  D .... (i); F > E ..... (ii); G  E ..... (iii) P > R > S  P > S. Hence II is not true.
From (ii) and (iii), F > E  G  F > G 381. 4; G  H ... (i), H > I ... (ii), I  J .... (iii)
Hence II follows while I does not. From (i) and (ii), we get G  H > I. Hence no relation-
367. 1; Y = Z ... (i); X  G ... (ii); Y < L ... (iii); G > L .... (iv) ship between G and I can be established. Hence I is
Combining these, we get not true. From (ii) and (iii), we get
Z = Y < L < G  X Hence Y < X and L > Z H > I  J  H > J or J < H. Hence II is not true.
368. 3; A  D ... (i); B  C ... (ii); A = R ... (iii); B > A .... (iv) 382. 1; X = Y .... (i), Z < K ... (ii), Z  Y ... (iii)
From (i) and (iii), R  D  either R = D or R > D. From (ii) and (iii), we get K > Z  Y  K > Y
369. 1; Z  Y .... (i); U > V .... (ii); Y < K .... (iii); R < V ... (iv) or Y < K. Hence I is true.

KUNDAN
From (i) and (iii), K > Y  Z  K > Z. Hence I follows.
From (ii) and (iv), U > V > R  U > R. Hence II does
not follow.
370. 4; W  Q .... (i); R > X .... (ii); Q  X ..... (iii)
From (i) and (iii), no conclusion can be drawn between
W and X. Hence I and II don’t follow.
371. 2 372. 1 373. 3
374. 4; Y = Z ... (i), R > T ... (ii), S  Y ... (iii), R < Z ... (iv)
From (i) & (iii), S  Y = Z  S = Z may be true but not
From (i) and (iii), we get X = Y  Z  X  Z. Hence
II may be true but not necessarily so.
383. 5; N  L .... (i), M  L .... (ii), P > N .... (iii)
From (i) and (ii), we get N  L  M  N  M. Hence
II is true.
From II and (iii), we get P > N  M  P > M
or M < P. Hence I is true.
384. 3; Q  S .... (i), T = S .... (ii), R  T ... (iii)
Combining all, we get R  T = S  Q  R  Q 
necessarily so. either R = Q or Q > R. Hence either I or II is true.
From (i), (ii) and (iv), Y = Z > R > T  Y > T. Hence II 385. 2; C  D ... (i), E > F .... (ii), E < C ... (iii)
is not true. Combining all, we get F < E < C  D  F < D
375. 2; L > K ... (i), C  H ... (ii), C < A ... (iii), H = L ... (iv) From or D > F. Hence II is true. I is not true.
(ii) and (iv), C  H = L  C  L. Hence I may be true 386. 1; R > S ... (i), S = O ... (ii), M  O ... (iii)
but not necessarily so. From (ii) and (iii), we get S = O  M  S  M ... (A).
Combining all the equations, we get Hence II is not necessarily true.
A > C  H = L > K  A > K. Hence II is true. From (i) and (A), we get R > S  M  R > M. Hence
376. 4; X > Y ... (i), X  Z ... (ii), Z  S ... (iii) I is true.
From (ii) and (iii), we get X  Z  S  No relation 387. 3; R  G ... (i), G  K ... (ii), R = L ... (iii)
Hence I is not true. No relationship between L and K can be established.
No definite relationship between S and Y can be estab- But I and II together are exhaustive. Hence either I or II
follows. 400. 1; A < B , ... (i), C > D .... (ii), D  A .... (iii), C  E .... (iv)
388. 1; Q  X ... (i), Y  X ... (ii), Z  Y ... (iii) From (ii) and (iv), E > D. Hence I follows. From (i), (ii)
From (i) and (ii), we get Q  X  Y  Q  Y and (iii), we get C > D  A < B . Hence II can’t be
 Y  Q ... (A). Hence II is not true. established.
From (A) and (iii), we get Z  Y  Q  Q  Z. Hence 401. 4; The three statements are decoded as:
I is true. (i) R < M, (ii) M > P, (iii) R  L.
389. 4; A  T ... (i), S < T ... (ii), N  S ... (iii) From (i) and (iii), we have M > R  L.
From (i) and (ii), we get A  T > S  A > S.... (A). Which means M  L. Hence, I does not necessarily
Hence I may be true but not necessarily so. follow. Again, M  L and M > P..
From (A) and (iii), we get A > S  N  A > N. Hence So P = L may or may not be true. So II does not follow
II may be true but not necessarily so. either.
390. 3; A = T ... (i), T  M ... (ii), Q  M ... (iii) 402. 1; The three statements are decoded as:
From (ii) and (iii), we get T  M  Q  T  Q (i) T  P, (ii) P < S, (iii) P = M.
Hence either I or II is true. From (ii) and (iii), we have S > M. So I follows. From (i)
(391-395): In these questions and (ii) we can’t be sure whether T is greater than or
less than S. So II may or may not follow.
$  > (greater than), @   (greater than or equal
403. 3; The statements are decoded as
to),   = (equal to), *  < (smaller than), and # (i) M = T (ii) T  Z, (iii) S > M

K
  (smaller than or equal to). From (i) and (ii), we have Z  M.
391. 5; L < S .... (i), P  R .... (ii), S  R ... (iii) This means that either Z > M or Z = M.
From (i), (ii) and (iii), we get Hence either I or II follows.
P  R  S > L.  P > L. Hence I is true. 404. 4; The statements are decoded as
From (ii) and (iii), II is true. (i) Z < B, (ii) N  S, (iii) B < N.
392. 4; G > R .... (i), H  R .... (ii), G  M .... (iii) From (i) and (iii) we see that: N > B > Z.
From (i) and (iii), no definite relationship between M Obviously, I is wrong. Also, N > B and N  S.
and R can be established. Hence I and II are not true. This is not sufficient to find a relation between B and S.
393. 3; Y  T ... (i), J = T .... (ii), O = T .... (iii), E  J ... (iv) 405. 4; The statements are : (i) L < C, (ii) C > Z, (iii) Z  F..
From (i), (ii), (iii) and (iv), we get From (ii) and (iii), we can’t find any relation between C

KUNDAN
E  O=T=J  Y  E  Y and F.
Hence, either conclusion I or II is true. 406. 2; Since conclusion II is nothing but the second state-
394. 2; H  P... (i), H < D ... (ii), T < P ... (iii), X = T ... (iv) ment, don’t waste your time decoding the given state-
From (i) and (iii), we get ments. II is of course true.
H  P > T  H > T. Hence conclusion I is not true. 407. 1; The statements can be decoded as
From (i), (ii), (iii) and (iv) (i) N < L, (ii) L > S, (iii) S > Q. From (ii) and (iii) it is
D > H  P > T = X  D > X. obvious that Q < S < L  Q < L. Hence, I is true. Since
Hence conclusion II is true. both N and Q are smaller than the same quantity L, it is
395. 1 not possible to compare them. Hence II cannot be de-
396. 1; U  V .... (i), U < W .... (ii), M  N .... (iii), W > N .... (iv) clared true or false.
From (i) and (ii), V < W. Hence I follows while II does 408. 4; The three statements can be decoded as (1) A > B, (2)
not. B < C, (3) C > D. Here, we can’t establish any link
397. 4; S  T ... (i), X < Y ... (ii), S  Y ... (iii) between A and D. Therefore, neither I nor II can be
I can’t be established. declared true or false.
From (i) and (iii), Y  T. Thus II may follow but not 409. 4; The three statements can be decoded as: (1) M < N, (2)
necessarily so. O > P, (3) O < U. From (2) and (3),
398. 4; Z > R ... (i), S  D... (ii), R < A .... (iii), A  S .... (iv) we have, U > O > P  U > P..
I can’t be established. Thus both I and II are wrong.
From (iii) and (iv), R < S. Thus II is false. 410. 1; The three statements can be decoded as (1) Q > S, (2)
399. 4; U  V .... (i), N  K ... (ii), L < V ... (iii), U  N ... (iv) U < S, (3) U = R.
Combining these, we get K  N  U  V > L. From (1) and (2), we have:
So I does not follow. Nor does II follow. U < S < Q  U < Q. Hence, I is correct.
222

  

 
  
ttt
t
ttt
ttttttttttttt ttttttttt ttttt ttt
tttttt ttt
ttt
tttttttttttttt tttttttt tt ttttttttt tttttttttttttttttttttt
tttttttttttttttttttttttttttttttttttttttttttttttttttttt tttttt ttttttt ttttttttttttttttttttttttt ttttt tttttt
tttttt ttt ttttt ttttt t t tttttttt ttt tt ttttttt tttttt ttttttttttttt tttttttttt ttttttt tttttttttttt
tttttttttttt
tt ttttt tttt ttttt tt ttttt ttt ttttt ttttt ttttttttttt tt tttttttttttttttttttttttttttttttttttttttttttttttttttttttttt
tt ttttt tttttt ttttt ttttttttttt tttt ttt tttttt tttttttttt ttt tttt ttttttttttt tttttttttttttttttt tttttttttttttttttttt
ttt tttt tt tttt t ttt tttttttt ttt ttttt tttt ttttt ttttttt t ttttttttttttttt tttttttttttt ttttttttttttt
tttt tt tttttt ttttttt ttt tt tttttt tttttt tttt ttt ttt
t ttttttt ttttt tttt  !  !! ! ! 
    tttttt tttt tttt ttt
   tttt tttttt tttt ttt tt tttt tttttt t t tttt ttttt t tt t tttttt
t tt tttt ttt t
   tttt tttttt ttt tttt ttttttttt ttt ttt tttt ttt tttttt t tt tttt t tttttt t t tttttttt
   tttttt tttt ttt tttt ttttt tttt tttttttt tttttt ttt tttt ttttttt tttttttttt t ttt
    tttttt tttt tttt ttt   !  ttt ttttt ttttt t t tttttttt
t tt tttttttt ttt t
   tttt tttttt tttt ttt tttt ttttt ttttt ttt ttttttt t t t tttt ttttt tttttt ttt tttt t
   tttt tttttt ttt tttt t ttt ttttttt
tttt ttt tt        !tttt t
t ttttttttt
    ttttt t tt ttttt ttt ttttt tttttt ttt ttt ttt tttttttttt tt ttt

ttt
t t tt ttttttttttt tttt tttttt ttt ttttt ttttt tttt ttttt t tt ttttt ttt ttttt tttttt ttt ttt ttt ttttttttt
   ttt tttt ttt ttt t tttttt t t ttt tttt
ttt tttt ttttt ttttttttt
t tt ttttt ttttttttt ttt tttt ttttt 2  ttt ttttt ttttt t t tttttttt
t tttt ttttt tt
ttt tt ttt t ttt tttt t tt ttt tttt ttttt t tt ttttt t tt tttt tttttttt tttt ttttttttt ttt tttt tt t ttt tttttt tttt ttttt
ttttt ttttttt tttt tttt tttttttttt t tt ttttt tttt tt t tttttttttttttttttt tttttttttttt ttttt t tttttttt tttttttt
ttttt tttt tttt tt tttt tttt ttttt ttttttt     tt tttttttt ttt tt tttttttt tttttt ttt tttt t t tttttttt * 
       !  !
ttttttt ttttt tttt ttttt ttttt ttttt ttt ttt ttttt tttttt ttt         
ttttt t tt ttttt tttt ttttttt tttt ttttt t tt tttt   !  2t ttt ttttt tt tt t tttt
ttttt ttttttt ttt
t t ttttttt ttt ttttt ttttt tttt tttttt ttt ttttttttttttttt ttt tttttttttttttttttt ttttttttttttt tttt
ttt tttt ttttt ttttttt tttt tt tttt tt tttt ttttttt     ttttt
t
t tttttttt ttttt tttt tt ttttt ttt ttttttttttt tt
 !  !  !! !
ttt tttt   t tttt    tttttttttttt t t ttt
tt ttttt ttttt ttt ttttt ttt tttt tttttt tttttt
t tttt tttttt ttt ttt ttttttt tttt tttt tt tttttttt ttt ttttttttttttt ttt
ttt tttt tt tttt ttttttt tttt tttt tttttttttt ttt tttttt tttt tt ttttttt ttttt tttttt tt tttt tttt t ttttt tttt ttt ttt ttttttttt
tttttttttt t
t ttt ttttttt tttt ttttt t tt ttttt tt ttttttt ttt ttttt tttt tttt ttttttt tttt ttttt ttttt ttttt ttt tt ttttt tt
ttttt tttttttt tttt ttttt t tt ttt
t tt tttt tttttt tttttt tttt
   t tttttttt tttt tt tt ttttt tttt ttt tttttttttt t
tttttttttt ttt tttt ttt tttt ttt ttt tttttt ttt tttttt ttttttt tt   ! ! 
tttt
t ttt tttttttt ttttttttt
t tt tttt ttttt tttt ttttt t t ttttt t t tttt tttt ttttttt tttt ttt tt tt t t tttttt ttt ttt
ttttt tt ttttttt tttt ttttt tttttt tttt ttttt t tt tttt tttttttttttt t t tttttttttttt ttt ttttt
t tttt ttttt tttt
ttt
t tt tttt tttttt ttttttt     ttt tttttttttttt ttttttttt ttt ttt ttttttttt ttt tttttt t t ttt
t t tttt tt ttttt tttt tt
t tttt tt ttt tt ttttt tt tttttt ttt tttttttttt t t ttttttt ttttt tttt tttttttt ttttt tt
tttttttttt ttt ttttt tttt tttttttttttt ttttttttt ttt ttt tt tttttttt ttt tt ttttttt
t
t ttttt ttt tttt t tttttt ttttt tttt  ttttt
 2   tttttt tttt tttt ttt !     !  t t ttttttt tttt ttttt ttt ttt
   ttt tttttt tttt tttt ttttt ttttttttttttt tt tttttttttttttttt tttttttttttt tttttttt
   ttt tttt tttttt tttt tt tttttttttttttttttttttttt t tttttttttttttttttttttttttttttttt
   ttt tttt tttt tttttt ttttttt
  t ttt tttt ttttt ttttt tttt tttt tt ttttt ttttt ttt tttt  2t ttttt ttt ttttt tttt
ttt
t t t tttt ttt
t tttt ttttt tttt tttttt tttttt tttttttt tt tttttt
  
ttt ttttt t ttttt
t tttt tt ttttt ttttt tttttttttt ttt tttt   !  ! 
tttttt ttt ttttt tttt  t t tttt ttt ttt t t tttttttttttt tttt ttttt tttttt ttt

tttt tttttttt ttttttttt


ttttttttttttttt ttttttt tttttttt ttttt tttt ttttttt tt tt t t ttt ttttt ttt ttttttt ttt tt ttt
ttttt tttttttttttttt tttttttttttttttttttttt tttttttttt tttttttttttt tttttt tttttttttttttt ttttttttttttttt ttttttt
tttttttt t ttttttt t tttt tt ttttt ttttt ttttttttt tttt tttttt ttt tt t tt
  222

ttttt ttttt ttttttttt ttt ttttt tttttt ttttttttt t t tttt ttttt tt tttttttttttttttttttttttt ttttttttttt ttttttttttttttttttt t
ttt tt tttttttt t t ttt ttttt tttt ttt tttt  t tttt ttt ttt tt tttttt ttt
t
tttttttttttttt tttttttttttttttttttttttttttttt tttt ttttttttttttttttttttttttttttttttttt ttttttttttttttttt
tttt tttttttttttt tttt ttt ttt ttttt tt tttttt tt t t ttt ttt tt tttttt ttt tttttt t t tttttt ttttt t
ttt tttt
 ! t t tttt ttttt ttt 2 t tttt tttt tttt tt ttttt ttt tt ttttt
tt tttt t tttttt ttt tttttttttt ttttt tttt ttttt tt tttttttt ttttttttttttttttttttttttttttttttt ttttttttttttt
ttttttt tt ttttttttttttttttt
ttt ttt
t ttttttttttttt tttttt tttttt tttt tttt tttttt ttt tttttt tttt tttttt ttt tttt tt tt
tt tttttt ttt tt ttt tttt tt tttt t ttt tttttttt ttt ttttt tt tt tttttttttttttttt ttttttttttttttt ttttttttttttttttttttttt
ttttt tttttt ttttttttt tttttt ttt ttt ttttt tttt tttt ttttt t ttt ttt ttt ttt tttt t t tt ttttt ttttt ttt ttttt  2 
 !     t tt ttttttttttt ttttt ttt tt tttttttttttt tttt tttttt tttt
tttt tttttt ttt tttttt tttt tttttt ttt tttt tt ttt tt tttttt ttt
   ! !    tttt tttttt ttttttttt tttt ttttttttt tttttttttt tttt ttttt
tttttttttttttttttttttttttttttttttttttt tttttt tttttttttt
ttttttttttt ttttttttt tttttt tttttttt tttttttttttttt
ttt tttt tttt ttttt tttt tttttt tttt tttt tt ttt tttt ttt ttt tttt
ttttt ttt tttt tttttt ttt ttttttt 
t 
t 
t t tt t tttt tttt
ttt tttt tttttt tttt tttt tttt ttt tttt tt tttttttt
t ttt tt t
ttt tttt tt ttttt ttt tttt tttttt t t tttttt ttt tttttt
ttt
t tt tt
tttt ttttt ttt ttttt tttt tttttt ttt tttttt tttt tttttt ttt ttt
tttttt ttttttttttttttt tttttt tt ttt ttttttt ttttttttt ttttt
tt ttt tt tttttt tttt tttt tttttt tttt tttt tt ttt tttt ttt ttt
ttt
tt tttt
t tttttt ttttttt t t tt tttt tt ttttt
tttt tttt ttt ttt tttt tttt tttt
t tttttt tttt tttttt ttt ttt
           2   tt ttt ttttt ttt tttt tttttt tttt tttt tttt ttt tttt tt ttttt
2     ttttttttttttttttttttttttttttttttttt tttttt tttttttttttt
tttt t tttt tttt tttttt tttt tttt ttt ttt tttt tttt ttt ttttt tt
ttttt tttttt ttttttttttttttttt ttttttt ttttttttttttttt tttt tttttt tttt tttt t ttt tttt t
tttt ttt tttttt tttttttttttt ttt tttt ttttt tt ttttt tt ttt tttt ttttttt t
t tt tttt ttttttttt ttt tttttt tt
t ttttt
t ttt  t ttttt t tt ttttttttttt tttt ttt ttttt ttttt tttt 
tttttttttttt ttt tttt ttttttttttttt tt tttt t ttt ttt tt t t ttttt
tttt tt tttt tt t tttttttttt tttttt ttt tttttttttt t ttt ttt tt t t tttt t
ttt ttt tttttt   t ttt tttt
t tttt tt tttt t tt tttttttttt t ttt ttt tt t t tttt tt
ttt ttt tttttt 2   ttt ttttt tttt tttt tt tttt ttttt t ttt ttt tt t t tttt tttt tttt ttt tt
ttttttttttt tttttt tt
t ttt ttt tttttt 2  t ttt tttt tttt
t tttttttttt tt tttttt ttttt tt ttttt ttttttt tttt
ttt t ttttt
t tttttttt tttt tttt ttt
tttt ttttttttttttttt tttttttttttttttttt
tttttttttttttt tt
2    ttt ttt ttttt tttt ttt   t ttt t ttttt
t tt ttt ttttt 2  2t tt t
   ttt ttt ttttt tttt ttttttttt
tttt ttttttttttttttt ttttttttttttttttttttt
ttt
   ttt ttt ttttt tttt tt tt tttttttttttttt t t ttttttt ttt ttt tttttt ttt tttttt ttt t
   ttt ttt ttttt tttt ttttt tttt tttt t tttt    t ttttt tttt 
  
tttt ttt tt tttt t t     ! t tttt ttt tt ttttt tt
t t t tttt 't t ttt t tttttt tt ttttt ttt
tttttt tttt tttttt ttt tttt tt ttttt tttt ttttt t tt ttttt ttt
t t t ttt t t t ttt t tttt t ttt t 't t ttt t ttt ttt t
tttttt ttttt tttttttttt ttt    t ttt tttt tttt
tttttt ttt
tttttttttt ttt tttttt ttt tttttt tttt tttt tttt
t tttttt ttttttttttttttt tttttttttttttt
2    ttt ttt ttttt tttt
't t ttt !t ttt ttt tt tttttt ttt
   ttt ttt ttttt tttt
   ttt ttt ttttt tttt
   ttt ttt ttttt tttt   !     !      
    ttt ttt ttttt tttt  !   
   ttt ttttt tt tttt   !  
tttt ttt tt tttt t t 2   ! t tttt ttt tt ttttt tt ttttt tttt ttttttttttt
t tt ttt tttt tttt
tttttt tttt tttttt ttt tttt tt ttttt tttt ttttt t tt tttt ttt tt ttttt ttt
ttttt tttttt tttt ttt    t tttt
t ttt tttttt ttt  ttt t'ttttt2 't2 ttt 2 
tttt tt ttt tttt ttt ttttttttt ttttt t tt ttttt ttttt tttttt ttt ttt tt ttttt ttt
ttt   ! t ttttt t tt ttttttttttt tttt ttttt t tttttt't 2tttt2t't  ttt  tttt
ttt tttttt tttttttttttt ttt tt ttt ttt tt tttttt ttt t ttttt2t't2tttt 't ttt<t
2     ttt ttt ttttt tttt ttt tt ttttt ttt
   ttt ttt ttttt tttt t tttttt't 2ttttt't < ttt2 'ttttt
   ttt ttt ttttt tttt t ttttt2t'ttttt 't2 ttt= 'ttttt
  
tttt ttt ttttt tt  t2tttt2t't tttt<t't =tttt> 't
    ttt ttttt ttttt tt tttt tttttt ttt ttt tt ttttttt tt
   ttt ttttt ttt tttt ttt tt tttttt ttt
   ttttt ttttt ttt tt tttt 'tt tt t ttttt tt t t'tt tt t tttt
   tt t
tttt ttt tt tttt t t 2   ! t tttt ttt tt ttttt tt ttt tt tttttt ttt
tttttt tttt tttttt ttt tttt tt ttttt tttt ttttt t tt ttttttt ttttt'tt tt t ttttt tt t t'tt tt t ttt
ttttt tttttt tttt ttt    t ttt tttttt tttt tttt t t tt t
tttttttttttttttttttt ttttttttttttttt ttttttttttttt tttttttt  ttt2t'tt tt t ttttt tt t t'tt tt t ttt
 t t tttt ttt tttttt tttt tttt ttt ttt tttt tttt ttttt tttt tttt t tt t
ttttt ttttt tttttt tttt ttt   ! t ttt
t ttttt
22

ttt tt tttttt ttt ttt ttttttt tttttttttttttt t t   


tttt ttt
ttttt'tt tt t ttttt tt t t'tt tt t ttt tttttttttttt t tttttt ttttt tttt 
t tt t   t tttt tttttttttttt tt tttt
t tttt ttttt t
tttt2t'tt tt t ttttt tt t t'tt tt t ttt tt ttt ttttt ttttt
t ttttttttttt ttt tttt tttt t
t tt t ttttt ttttt ttttt ttt tttt tttt ttt tttt tttttttttttt
 2tttt2t'tt tt t ttttt tt t t'tt tt t ttt t t tttt tttttt tttt tttt tttttt t t ttttttttttt
t tt t   tt tttt ttttt t tttttt ttt ttttt ttttt
t t tt tttttttt tttt ttttt tt tttt tttt    tttttttttttttttt tttttt ttttttttt
ttt ttttt ttttttttttttttttt tttttt tttttttt tttttt    ttttt tttttt t tttttt tttt ttt tt tttt
ttttttttttttttttttt ttttttttttttttttttt ttttttt ttt    ttttt t tttttt tttttt tttt ttt tt tttt
ttt tt ttttt tttt tt ttt ttt     t ttttttttttttttttttttt ttttttttt
  t ttttt'tt t t tttt!t tt t t't tttt't tttt ttt ttt ttttt ttttt tt
t t tttt!t tt t t't 2tttt<ttt   tttt tttt tttt ttt
t 2tttt>t'tttt t t tttt ttt t t t't tttttt     t tt ttt t tttt tttt tttt tttttt tt t
ttt ttttt ttttttt
ttttt tttttt tttttttt ttttttttt tttt ttttt tttt ttt tttt tttt tttt ttttttttttt
tttttttttttttttt ttttttttttttttttttt ttttt
tt tttttt t ttt tt tt tt tttttt tttt tttt ttt
tt ttttt tttt tt ttt ttt  t  t t  t t
  t tttt 'tt tt t tttt!t tt t ! t ttt t tttt t t ttttt
ttt't2tttt< 2  t t ttttt tt t t tt ttttt tttttt ttt 
ttt tt ttttt tttttttt
ttt'tt tt t tttt!t tt t ttttt tttt ttt
t tttt ttttt ttt tttt ttttt
ttt'ttttt=  t tttt ttttt tttt tttt ttttt tt tttttttt
t 2tttt>t'tttt!t t tttt ttt!t t  t tttt ttttt tttt tttt ttttt ttttttttt t
ttttt'tttttttt   t ttttt tttt tttt tttt ttttt ttttttttt t
 ttt ttttt ttttttt
ttttt tttttt tttttttt ttttttttt ! t tt tttt tttt tttt ttttt ttttttttt tttt
tttttttttttttttt tttttttttttttttttttt ttttt
tt tttttt t tttt t t ttttt
tt ttttt ttt
t tt ttt ttt 2 t t ttttt ttt t t tt ttttt tttttt ttt ttt ttt ttttttttttt ttt tt
 < t  tttt'tt tt t tttt!t tt t tttttttttt tttttt tttt ttt tttt ttttt ttt t t ttttt ttttt
ttt't2tttt2  t ttt ttt tttttttttt tttttt ttt ttttttttttt tt
tt'tt tt t tttt!t tt t  t ttt ttt tttttt tttttttttt ttt ttt tttttttttt
t t t 't <t ttt t   t ttt ttt tttttttttt tttttt ttt ttttttttttt tt
t 2tttt>t'ttttt t tttt tttt t ! t ttt tttttttttt ttt tttttt ttt ttttttttttt tt
t t t t 't t ttt t t tt  t tttt t t ttttt
ttt ttttt tttttttttt tttttt ttttt ttt ttttt tt tttt ttttt  tttt tttt ttttt
t tttt ttttt ttt ttttt t t t tttt ttttt
ttttttt  t tttt tttt ttt ttt tttt tttttt ttttt
 t tttt ttttt tttttt tttttt tttt ttt tt
 t ttttt tttt tttttt tttttt tttt ttt tt
  ! 
 t ttttt tttttt tttt tttttt tttt ttt tt
!   !      ! t tttttt ttttt tttt tttttt tttt ttt tt
2  2        ! t tttt t t ttttt
          t ttttttttt tttttttttttttttttttttttttttttttttttt tttttt
  !     !   ttt ttt
t tttt ttt ttttt tt t t ttttt ttttt
!  2 2    2 2    t tttt ttttt ttt ttt tttttttt ttt tttttt
!       t ttttt tttt ttt ttt ttttttt ttt ttttttt
 t tttttttt ttttt ttt ttt ttttttt ttt ttt
@ !   !  ! t ttttttt ttt tttttttt ttt ttt tttt tttt
t tttt t t ttttt
ttttttttttttttttttttttttttttttttttttttttttttttt tttt  
ttttttttt tttt tttt ttttt   2 tttt tttt tttt ttt ttt    tt tttt ttttt t ttt t t t ttt t tttt t ttt t 't t ttt 
 22t ttt tttt tttttttttttt tttt ttt ttttt ttt ttt !  2
tt tt ttt ttt tt tttttt ttttt ttt tttt
 22tttttttt ttttttttt tttttttttttt ttttttttttttt tttt   ! tt ttttt ttt ttt t ttttt t tttt
tttttttt tttt ttttt tttt ttttt ttt ttt tt tttttt ttttt tttt t
ttt t 
t tttt tt ttt ttttt tttt tt tttt ttt ttt
ttttttttttttttttttttttttttttt ttt tttttttttt'ttt tt tt tttttttt tt ttttt ttttttttttttttt tt tt tttt
t ttttttt t t'ttttttttt ttttttttttttttttttttttt ttttt tttt ttt ttt ttt tt tttttt tttt tttttttt ttttt

tttttttttt tttttttttttttttttttt tttttttttttttttttt ttttt ttt ttt t


ttttt  t
tt
tt!tttttt t tttttt tttttttttt<tttt

   tttt tt ttttt t t ttttt tttttt ttt 



ttt 2t ttt 2
t t ttt t tttt t ttt 
t ttttt tttttt ttt t
ttt t tt tt tttt tt ttttt tttttttt tttt ttttt tt ttttt
t ttttttt ttttt t t tttt ttttt ttt ttt tttttttt tt
tttt tttt tttt tttttttt t t tttt ttttt tt t ttt t tttttt tttt tttttt ttttt tttttt tttttt
ttttt ttt ttt ttttt tttttt ttt ttttttttttt ttt ttttt  tt tt tt
   tttt tttttttttt tttt t ttt tttt 
t ttt ttt tt !  t ttt  t 't !t tt t t  t ttt t tt t t 
ttttt tttt tttttttt 'ttttt2
ttt ttttttttttttt ttttttttt ttt ttt ttttt tttttt ttt ! 2 <tttt
tttttttttttt t tt tttttt t t ttttttttt tttt ttt ! 2 2t ttt 2t 't t tt t t  t tt t tt t t 
t tttt tt tttt tttttt tttt tttt ttttttt ttt ttt 't ttt 
tttt ttttttt t t tttttttt ttttt tttttt !  tttt
tt tttt tt ttttt ttttttt tttt tttt ttttttt ttt ttt !   ttt
tttt ttttt ttttttt ttt tttttttt ttttt tttttt
  22

t tt ttttt t ttt !t ttt  t ttttt tttt tttttttt ttttt ttt ttt tttttttttt tttttt ttt ttttttttttt ttt 
ttt tttt ttttt ttt ttt ttt tt   
t
t tt tttt ttttt ttt tttt tttt ttt ttttt tttt tttttttt tttt !  t !t tt
ttttttt ttt 2 ttt ttttt ttt
tt t
t tt ttttt ttt ttt t ttt tttt
ttttttt't tttttttt ttttttttttttttttttttt
tt tttt tt ttttt ttttt tttt  tttt tt tttt tt ttttt  't t t t !t t t t t
t t
   tttttttttttttt
t ttttttt tttttt ttttt tttttttttt tttttttttttt't 
ttttt't
t ttt '
tttt't!
t tttt't

ttt
t 
t
t!t t ttttt
t tt't
t tttt't
t ttttt' ttttttt 't
t tttttt 't t t
t tttt tt tttt tt tttt

t t tttttt 't !
t ttt 't
t ttttt 't
t tttttt 't t t
t t 't t t t t t t !t ttttttttttt tttt tttt
t t
tt ttt t tt tt tttt tt ttttt t t ttttt 2 ttttt tttt tttt tttt tttttt tttttt tttt ttt ttt 
     !    2
      !  !  t t tttt
     !    ttt t tttt
t !t 't ttt ttt ttttt tttttttt ttttttt tt
   !     ttt ttttttttttt tttt tt tttt
t t't!t ttt tt
    !    
tt tt tt tttttt
       ! ttt 't !
t ttt 't
t ttttt 't 
t tttttttt 't 
t ttttttt 't

      !  ttt 't 
t tttt 't t t
t tttt tt tttt tt ttttt t 't 
      !  t t !t t t t ttt
t tttt tttt tt ttt ttt ttt ttttttt
  t ttt ttttt tttttttt ttttt t ttttt tt ttt ttttt ttt ttttttt ttt tttt    
ttttt tttt tt ttttttttt tttttt tttt ttt tt tt     ttt tt tttt ttttt t t ttttttttt ttttt ttt tttt
ttttttt tttt tttttttttttt ttttttt tt t
t ttt ttt t tt tttt ttttt tttt ttttttt t t tttt ttttt tttt ttt
tttttttttt tttttttt 'tt2 ttt2t
tt t
tttttttt tttttttt t ttttttttttttt ttttttt ttttt ttttttt
ttt tt tttt tttt tt ttt ttttt ttt ttt tttt tttt tt tt tttt tttt ttt tttt ttttt ttttt tt ttttt t t tt
ttt tt tttt tttt ttt tttttttt
t tttt tt ttt ttt t ttt t tttttt t t tt tt tttt
ttttttttttttttttttttt tttttttttttttttttt t ttt ttttttt ttttttttt t t ttttt tttt
t   
tttt tttt ttt     !      
   t tt tt tttttttt tttt t ttt tttt t ttt ttttt           !  
tttt
t tt tt tt tttttttt tttt tt tttt tt ttttt t  t ttt tttt tttt tttt ttttttttttttt tt
tttt tttt ttt ttt tttt tttttttt ttttttttttt tttt
  t
t tttttt ttttt tt tttttt t t tttttttt t  t tt tt ttttt't t ttt ttttttt 
t 
t >
t =
ttttttttt ttt ttttt ttt
t tttt tttt ttttt ttt ttttttt ttttttttttttttttttttttttttttt
t  tt
ttttttt t t tt ttttt tttt tttt tttt ttt tttt ttt t 't  ttt t ttt ttttttt ttttt t
t tt ttttttt
t
ttttttt t t tttttttt ttttt 
t 
t !t tttt  ttt t tttt tttt tttt tttttt tt t
t ttt ttt tttttt tttttt
tttt t ttttttt t t ttttt ttttt t t ttttttttt tt ttttt tttt tt t
t tttttt tttttttt ttt tttttt ttt t 
tttt tt tttt ttttt t t tttttttt
t tttt tttt tttt t !t t t t t
t tt ttttt tt 't 
t ttt t 't 
t tttt 't 

tttt ttttttt t t tttt ttttt tttt tttt ttttttt t tttt 't !


t tttt 't
t tttttt 't
t tt tt 't t t
t ttt
tttttttt ttttt tt ttttt tttt tttt tttt ttt ttt ttttttttttttttttttttttt tttttttttttttt ttttttt
ttttt ttttt tt tttttttttt t
t  t ttt tt tt ttttt t t t t t !t t tt tttt ttt tttt tt tttt t
t tttttt t t tt ttttt ttttt ttt ttttttttttt ttt tttt t tttt ttttt
 !2 2  tt
t tt tttt tttt ttttt tttt ttt ttttt tttt tt tt tt tttttt tttt tttt ttt t ttt ttttt t t t t
ttttttttttttttttttttttt tttttttttttt tttttttt ttt ttttt tttttttttttttttt tt tttttttt
t t tttt ttttt tttt
t tt tttt ttttt tttt t t ttt tt
tttttt tttttt tttttttttttttttt tttttttttt!tt ! 
tttt ttttt ttt ttttttttt ttt tttt ttttttt tttt
tt t
ttttttt ttttttt ttttttt tttt!t tt'tttttt
ttt
t tt tttt ttttt tttt ttttt ttttt tttt ttt ttt
ttttt tt tttt ttttttt tttt ttttt ttttttt ttt tttt 
ttttttt ttt ttttt ttttt ttt tttt tt tttt tt tttt tttttttttttttt t tttttttt
ttttt ttttttttttttttttttttttt
t
t tt tttt tttt tttt tttt tt ttt tt t t ttttt ttttttttt ttt tttt tt t ttt  t tttt tttttt ttt ttt tt
ttttttt ttt tttt ttttt t t tt tt tt tttt ttt ttt ttttttttt tttttttttttt ttttt t tttt
tttt ttttttttttt ttttttt
tttt ttttttt tttttt
t tt ttttttttt t tttt tttt ttttt ttttttttttttttttt ttttt tttttttttttttttttt tttttt
ttt tttt ttttt tttttt ttt ttttt ttttt ttt tttttttttt tttttttttt ttttt ttt ttttt ttt ttt
! 2 ttt t ttttt
t t t tt tttttt ttttttttt ttt t ttttt
< ttt 
t ttttt t 't

ttt tt ttttt ttttttttt tttt


tttt ttt
t t
t tttt t ttt tttt tt tttt tt ttttt tt
 ! ! 
ttt t t!ttt
t ttttttttt
ttt't
ttt't
ttttt
't 
t ttttttttt 't
t ttttt 't
t tttt 't !
t tttt 't  ttttttt tttt tttt t ttttt ttt tttttttttttt ttttttt
t
t tttttt ttt tttt tt tttt tt ttttt ttt 't t t t !t ttttttt tttt ttt ttttttttttt tttt tttt t ttt tt tt tttt tt tttt
t t ttttttttttt tttt tttt
t tt ttt ttttt ttt ttttt tttt ttttttt t tttt tttt tttt tttttttt
tttttt 't tt tttt tttt tttt ttttt ttttttttt tttt      
   
! 2 t t tt ttttttttttt ttt ttt t t ttttt ttt ttt ttt ttt t t ttttttt
2t ttt 2
t tttt
t t 't
ttt ttt ttttttttttt ttt tt ttttt tttt ttttttttt ttttttttt ttttt ttttt tttttttt ttt
tttttttttt ttttt
t t
t ttttttt ttttt t t tttt tttt ttttttttt ttt tttttt tt tttttttt ttt tt tttttttt tttttt t
2tttt2t't  ttt  tttttttttt tt tt ttt tt ttttttttt ttt tttttttttttt t t tttt

t
t t 't 
t 
t 
t !
t
t
t t tt tt ttttttt t t 't 
tt ttttt tt tttt tt tt tt ttttt tt tttt t ttt tttt tt ttttttt

ttt 't 
t ttttttttttt 't 
t ttt 't !
t ttt 't
t ttttttttt tt tttt ttttttttt tttt tt ttttt tttt tt tttt tt tttt tttttttt
't
ttttttt'ttt
t tttttttttt tttt tt tttt
ttt tttt tt tt tt ttttt tttt tt tttt tt tttt ttttttttt tttt 
t'ttt tt tt!t ttttttttttttttt tttt
t ttttt ttt
t t t tttt tttt tttttt ttttt tt
t ttt
t tttt tttt ttt tt
22<

ttttttt tt ttttt tttt tttttt ttt t ttt ttttt ttttt ttt t tt  2   tttt tttttttt ttttt ttttttt
ttttttt tt ttttt tttt tt tttt tttt ttt tttt tt ttttttt    tttttttt ttttt tttttttt tttt
tttt ttt
tt t ttttttttt
t ttt
t tt ttttttttttttttttttttt tttt    tttttttt tttttttt ttttt tttt
tt tttttttt tt ttt tt ttttt  tt
t ttt
t ttt t ttt ttttttttt tttt    tttttttt ttttt tttttttt tttt
tt tttttttt tt ttt tt ttttttttt 
t 
t t ttt ttt tt ttttttt  =   ttttt tttttttt ttttt ttttttt
tt ttt tt ttttt 
t 
t t ttt tt tt tttttttt tt ttt tt t
   tttttttt ttttt tttttttt tttt
tt ttt tttt tt tt ttttt ttttt ttttttttt ttt ttt tt ttttttt    tttttttt tttttttt ttttt tttt
tt ttt ttttt tt tttttttttttttttttttttttttttttttt ttttttt    tttttttt ttttt tttttttt tttt
tt ttt t
t
t ttttt ttt tttt t ttttttt ttttt tttt  !   !  
    tttt tttttttt ttttt ttttttt ttt tt tt tt tttttt tttt
t tttt tt ttttt ttttt tttt
   tttttttt ttttt tttttttt tttt ttttttttt tttt ttt ttttt ttt tttttttt tttt tttt ttt tttttt t tt
   tttttttt ttttt ttttt ttttttt tttt ttttt ttt ttt t
t ttt tt ttttt tttt tt tttt ttt tttt ttt ttt
t tttt ttt tt ttttt t tt ttt t
t tt tt tttttttt ttt ttt t tttt ttttt tt
ttt ttttt ttt ttt t tttttt t t tttttttttt ttt tt tttttttttt t ttttttt ttttt ttt tt ttttt tttt t tt ttt ttt
tt tttttttt tt ttt t ttt tttttttt tttt ttttttt ttttttt ttt ttt tttt  t
t  t ttt tt
 2   ttttt tttttttt ttttt ttttttt ttttt ttttttt ttttttttt tttt tttt ttttt tttt tttt  t  
   ttttt ttttt tttttttt ttttttt ttt tttt tttttt tttt ttt tttttt ttt t t tttt tt t tttt tttt tttttt
   tttt tttttttt ttttt ttttttt :tttttt tttt tttt ttt t
t t tttttt tttt tt t tttt tttt ttt t
ttttttttttt ttttttt ttt tttttttttt ttt tttt tttt tttt ttttt tttt tttttttttt ttttt tt tttt
t ttt tttt tt ttt
ttttttttt ttttt ttttt tttt tttt t tttttt ttttt ttt t tttt tttttt tt ttttt t ttttt ttttt
ttttt ttttttt t  tt tt ttt ttttttttt ttttt tttt  >   ttttt ttttttt ttttt ttttttt
t tttttt ttttt ttt t    tttttttt ttttt tttttttt tttt
 2    tttt tttttttt    tttttttt ttttt ttttt ttttttt
   tttt tttt tttt tttttttt ttt ttttttttt ttttt ttttt tttt ttt ttt tttt
   tttt ttttttttt ttt tttt ttttt ttt tt ttttt ttttt tttt ttttttt ttt ttt
 ttttttttttt ttttttt ttttttttttttttttt tt ttttttt tttttt tttt ttttt ttttt ttt tttt tt tttt ttt tt ttt
ttttt tttt tttttttttt t tt tttttttt tttt tttttt t ttt ttt
     tttt tttttttttt ttttt
    ttttt ttttttttttt     ttttt tttttttt ttttt ttttttt
    tttttttttt tttttt    tttttttt ttttt tttttttt tttt
 ttttttttttt ttttttt tttttttttttttttt tt ttttttt    ttttt tttttttt tttttttt tttt
ttttt  ttttttttttt ttt ttt tt tttttttt ttttt
t tttttttt ttt

tttttttttt
t t
t ttttttttt ttttt ttttt tttt tttt
! !  !   ! tttttttttttttttt tttttttt tttttttttttt ttttttt ttt
!    ! tt tt tttt
t tttttttt tttttttttttttt tttttttttttt t
t  !  ! t t tt tttt tttttttttt ttt tt ttttttt tttt tttttttttt ttttt tttt ttt tttt ttt tttt tt t
ttttt
t tt ttt tttttt tttt ttttt tttt t t tttt tt tttttttt ttt ttt     ttttt tttttttt ttttt ttttttt
ttttt ttt tt tttt ttttt ttt ttttt tt tttt tttt
t ttt ttttt tt
t t    tttttttt tttttttt ttttt tttt
ttttttttttttt tttt ttttt ttt ttttttttttttttttt ttttttt t    tttttttt ttttt tttttttt tttt
ttt ttt ttt ttt ttttt tt
t ttt tt ttttttt ttttt
t tttt tttt
tttttttt ttt tttttttt ttt tttt ttttt ttt t
t tttt ttt
tt ttt
tttttttttt ttt tttt !ttttt ttttttttt ttttttttt tttt ttttttttt tttttttttttttttt tttttttt ttttttt tttttt
tttt
t tt ttt
t tttttt tttt tttttttttt tttt ttttttttttttttttt tttttt
t ttttttt
    tttt tttttttt ttttt ttttttt ttttt ttt tttt tt tttt ttt tt tttttt
t tttttttt ttt tt
   tttttttt ttttt tttttttt tttt tt tttt ttt tt t
t ttttt tttt tttttttt  
   tttttttt ttttt ttttt ttttttt ttt tt
t tttt ttt tt t tt tttt t tttt ttttt ttttt ttt ttt ttt tttttt tttttttt

  ! ttttttttt tttt ttttt ttttt t tttt tttt ttttttttttt tttt tttttttttt tttt ttttt tttttt ttt
tttttttttt ttt ttt
t tt tttttttt ttttt t ttt ttt t tttttt tt tttttttttttt ttttttttttttttttt
tttttttttttttttttttttttt
tttttttt tttt ttttt tttt t t tttt tt tttttttt ttt tttt ttttt ttt t t ttttttt ttttt tttt 
tttt ttttt ttt ttttt tt tttt tt ttt tttt tttt tt tttttttt
tttt t t tttt tt tttttttt ttt tttt tt tttt ttt tt tttt tttt ttttt    !!
ttt ttt ttt ttt ttttt tt
t ttt tt ttttttt ttttt
t tttt tttt
  t tttt tttttttttttt tt tttt
t tttt ttttt tt
tt ttt
t tttttt tttttt tttt 
t tttt tttttt ttt ttttttttt tttt tttttt tttttt t tttt
t ttttttttttt ttttt tttt tttt tt ttttt tttt
tttt
t tt tttt tttt
t tttttt ttttt ttt ttttt ttt tt tt ttttt ttt tttt tttt ttt tttt ttttttttttttt t t ttt
 <   tttt tttttttt ttttt ttttttt tttttt tttt tttt tttttt t t ttttttttttt
   tttttttt ttttt tttttttt tttt   ttttt ttt tttt t tttttttt t t tttttt ttttt
   ttttt tttttttt tttttttt tttt    t tttttttt ttt tttt ttttt t t tttttt ttttt
t tttt ttt tt    t tttttttt tttttt tttt ttttt t t ttt ttttt
 t  !     tttttttt
t tttt ttttt ttttt t    t tttttttt tttttt ttttt tttt t t ttt ttttt
tttttttttt ttt ttt
t tt tttttttt ttttt t ttt ttt t tttttt tt     t tttttttt tttttt ttttt t t tttt ttt ttttt
tttttttt tttt ttttt tttt ttt ttttt ttt t
t ttttt ttttttttt tttt    t tttttttt tttttt ttttt t t tttttt ttt ttt
ttttt tttt ttt ttttt ttt t
t ttttt ttttttttt ttttt tt tttt ttt    t tttttttt tttttt ttttt t t tttttt tttt tt
ttt tt tttt ttt tt tttt tt tt tttt ttt ttt ttt tttttt tttt
tt tt tttt ttttt tttt tttttttt tttttttt
t tttt tt tttt
ttttt tttt ttttttttttt tttt tttt tt tt tttt tttt tttt ttttt tttttt t tttt ttttt ttttt tttt tttt tttt tt ttttt tttt tttt
tttttttttttt ttt tttt tttt tttt t tt t tttttt
  222

ttt tttt tttt t tttttt tttt ttttt tttt ttttt ttttttttt ttttt ttt 2 ttt tt tt ttttt t t tttttttt
t tt tttt tttt tttt tttt
tttt t tttt ttt ttt tttt ttttttttt ttttt tttt 
 tttt tttt ttt tttt tttt tttt ttt tttt tttt tttt ttttt
t ttttttttttttt ttttttttttttttt tttttttttttt tttt
 t ttt tttt t tttt tttt tttt tttttt ttttt tttt tttttttttttttttttttt tttttttttttttttttttttt tttt
 t tttt tttt ttt ttttt tttttt t tttt tttt tttt ttttttt

 t tttt tttt ttt ttttt t tttt tttttt tttt tttt

ttttttttttttttt tttttttt tt tttttttt tttt ttttttt
 t tttt tttt ttt tttt t tttt tttttt ttttt tttt
! t tttt tttt t tttt tttt ttt tttttt ttttt tttt
ttt ttttt t
t ttt tttt ttttt ttt ttttt t t t ' t t tttttt tt
tttttttt ttt tttttttt
tt t
t tttttttttttt tt ttttt tttt
t tttt t t ttttt
ttttt ttttt ttt ttttttt ttttt t ttt ttttt tttt tttttttttt
2  t ttt ttttt ttttt ttttt ttt tt ttttttt
ttt ttt t tttt !t t
t tt tttt tttt tttt tttt tttt 
ttt tttt ttttt tttttt tt tt ttttt tttt tttt tttt ttt
t t tt tttt tttt tttt tttttttt ttt tttt
tttttttttttt tt
   ttt tttt tttt tttt tttttttt tt tttt
t tt ttt ttt ttttt tttttttt ttttt tttt
   ttt tttttttt tttt tttt tttt tt ttt
 t tttt t  ttttt 
   ttt tttttttt ttttt tttt tttt tt ttt
 t tttt t ! tttttttt
    ttt tttttttt ttttt tttt tttt tt ttt
t tttt t t ttttt
   ttt tttttttt ttttt tttt tt tttt ttt
2  t tt tttt tttt tttt tttttttt ttt tttt

tt tt tttt tttt tt ttt tt ttt  
tt tt t t tttt tttt tttt tttt ttt tttt tttttttttttt tttt
 ttt tt tt ttttttttt tt tttt tttt tttt tttt
ttt tttt ttttt ttttt
 t tttt t  ttttt 
ttttttttttttttttttt tttt
tttttttttttttttt tttt ttt
 t tttt t ! tttttttt tttttt t ttttttttttttttt ttttttttttt ttttttttttt
t tttt t t ttttt tttttttttttt

  t
t t tt tt ttt tt tt tt tttt ttttt
tttt tttt ttt tttt t ttt ttttt ttt tttt ttttt ttttt !    t tt tttt ttttt tttt
t tttt t tttttt
 t tttt tt tt ttt tt tt tt tt t ttttt tttt ttt tttttt ttttt ttt ttttt ttt ttt tt tt tt tttt ttt

 t tttt ttt tt tt tt t tt tt tt ttttt tt


t tt t
t tt ttttt tt t
 t tttt ttt tt tt tt t tt tt tttttt t tttt ttttttttttt ttt ttt t tttt !t t
t ttttt t tttt
! t tttt ttt tt tt tt tt tt tt t ttttt ttt ttt tttt tttt tt ttttt ttttt ttt tt ttt tttt ttt tttt tt
t tttt t t ttttt tttt t t tttt tttttttt ttttttttttt     2
 tttt tttt ttt tttt tt tttt ttttt ttt tttt tttt tttt ttttt  !    t tt tttt ttttt tttt
t ttt tttttt tttt
 t
tt tttt ttttt ttttt ttttttt tt tttttt ttttt t tt ttttt ttttttt ttt ttttttttt ttttt tttt ttttt t t
t tt
 t tt tttt ttttt ttttt ttttttt ttt tttttt ttttt t t tttt tttt t tttt ttttttttttt ttt t
t t t
t t
 t tt ttttt tttt ttttt ttttttt ttt tttttt ttttt ttttt ttttttt tttt tt t ttt ttttt tt tttt ttt tt tttt tttt
 t tt ttttt tttt ttt ttttttt ttttt tttttt ttttt tttttt tttttttttttttttttttttttt
t tttttttttt ttt tttt
! t tt ttttt tttt ttt tttttt ttttt ttttttt ttttt tt ttt tttttttttt
t ttt ttt ttt ttt tt     2
t tttt t t ttttt
  ttttttt ttt tttt ttttttt
t tt ttttttttttt ttttt ttt   !!
tttt t t t tttttttttttt ttt tttt ttt  2 !  !  !    
t tt tttttttt ttttt    
t tt ttttttt tttt t tttt tttttttttttt tt tttt
t tttt ttttt ttt ttttt
t ttttt ttttt ttttt t t tttt
t ttttttttttt ttttt tttt tttt tt ttttt ttttt tttt
tt tttttttt tttttt tt ttttt ttttt tttt tttttt tttttttt ttt tt tt ttttt ttt tttt tttt ttt ttt ttttttttttttt t t tttttt ttt
ttt ttttttttt ttttt tt tttttttt tttttt t ttttttt tttt tttt tttttt t t ttttttttttttt
tt ttttt tttttttt tttt ttttt ttttt tttt tt tt ttttt ttttt   tt t t ttt ttt ttt #ttt tttttt t t ttt tt
tt ttttt ttttt ttttttt tttt ttttt ttt ttttt ttttttt    ttt tt t t ttt ttt #ttt tttttt t ttt tt
tttttttttt ttt t ttttt
t ttt ttttt t
t t tttttttt tttt    ttt tt ttt t t ttt ttt #ttt tttttt t tt
ttt ttttt ttt tt ttt ttttt tttttttt tttttttttt tttt tttt     ttt tt ttt tttttt t t ttt ttt #ttt t tt
ttt tt tttt tttt
t ttttt t ttttt ttt tt tttt ttt tt ttt     ttt tt ttt tttttt t t ttt ttt ttt #ttt
ttttt ttttttt ttt ttt tttt ttt     ttt tt ttt tttttt t t ttt ttt t ttt #tt
t
t tttt ttt tttt ttt tttt tttttttt tttt tttt t ttt tttt ttttt ttttt ttt ttttt ttttt 
 ttt ttttt tt tttt ttttt ttttttt ttt ttt tttt ttt ttt ttttt tt
tt tttt tttt tttttt tttt ttt ttt tttt ttttt ttttt
t tttt ttt
tttt ttt ttttttt ttt ttt ttt t
t ttt ttt ttttt ttt t tttt tt ttt tttt ttttt ttttttttt tttt tttttttttttt ttttt ttt tttt tttt
tttt ttt tttttt ttt tt ttt ttt tttt
t ttttt ttttt ttt tttt tt ttttt
tttt ttt !ttt ttt tt     2  tt tt t t tttt tttt tttt tttt ttt ttttt ttt ttt tttt tttt
2 tt tttt tttt tttt tttt tttt  ttttt
ttt ttttt ttttt ttttt ttt tt ttttttt  t ttt t tt tttt t t tt
   tt ttttt ttttt ttttt ttt ttt ttttttt  t t tt tttt ttt ttt t  t tttt ttt t tt t t tt
   tt ttt ttttt ttttt ttt ttttt ttttttt  t ttt t t ttt t tt ttt ! t t tt tttt ttt t t tt
   tt ttt ttt ttttt ttttt ttttt ttttttt t tttt t t ttttt
    tt ttt ttt ttttt tttttttt ttttt tttt 2  t tttt tttt ttt ttttt ttt tttt
*  t ttt ttttt ttt
t ttttt ttt ttttttttt ttttt !ttt tttt ttt ttt tt tt t t tttt tttt tttt tttttt ttttt ttt t tttt ttt tttt
tttttttt t tttttt t !ttt ttt tt tttt ttt
t ttt ttttt t ttt tttt tttt
tt tttt tttttttt  tt tt ttt ttttttttt ttttt tttt t tt t tt  tt  t tt  t ttt
t ttt ttt t ! t t t 
22=

2  t
ttt tttt
ttt ttttttt ttt ttt tttttttt tttt
t tttt ttt tt
t ttttttt ttt tt
t tttt tt
tt tt ttttt tttt ttt tttt ttttt ttttt ttt ttttt ttttt tt
tttttttt!tt
tttttttt tt
ttttttt tttt
t ttttttt t
 tt  t tt  t ttt ttt ttt tt t tttttt tt ttt
! t t t  !   
 tttt tt ttt ttttt ttttt t ttt tttttt ttt ttt ttt tttt tt t t tt ttttttttt ttttt
t t tt tttt !  t tttt tt
tttt tttt tttt ttt ttttt
tt tt t t tttt tttt tttt tttt tt tttttttt ttt tttt ttttt t t ttttttttttttttttttttttt tttttt! tttt
 t ttt ttt tttt ttt ttt ttt tttt tttttt tttt ttttttttttttttttt t ttttttttttttttttt
t
tt t
 t ttt ttt ttt tttttt ttt tttt ttt tttt tttt tttt! tttttttttttttttttt
t tttttttttttttttttt
 t tttttt ttt ttt ttt ttt tttt ttt tttt tttt ttttttt tttttttttttttttttttttttttt ttttttttttttt
! t tttttt ttt tttttttttt t
ttttt tttt ttttt tttttttttttttttt tt 
t tttt t t ttttt  t tttt ttttttttt tttt tttt tttt
 t ttt ttt ttttttt t tttttttt ttttt tttt tttt t tt
t tt tttttttt ttttt
ttt tttttttttttt tttttttt
t tttt tttttttt t t tt
t tt tt tttt tttttttt ttttt ttt ttttt ttt ttttttttttt ttt
t tttt tttttttt

 tt ttttttt t ttttt
t tt ttttt tttt ttttt tttt ttttt
tt t
t tttt tt ttt tt ttt !
tttttttttttttt tttttttttttttt t ttttttttt ttttt ttttttt *  t ttttttt ttttt ttttt ttttt  t ttt tttttt tt
tttt ttttt tttt tt tt ttttt tttt tt ttttt tt tttt ttttt ttt   ttttt  2t ttt tttttt ttt  
tttt ttttttttt tttt ttt ttttt tttttttt tttt tttt ttttt
ttt
t tttt ttt ttttt ttt tttt tt t ttt tt tttt ttttttt ttt   ! !   !
tttttttttt tt ttt tt ttt tttt tttttt ttt t ttttt
t tt 
ttttt t
t tt t tttttt ttt t ttttt ttt tt tttt tttt tttttt
tttttttttt tt ttt tt ttt ttttttt ttt tttt ttttt
t ttt tttt
ttt
t ttttt tttttt tttt ttttt ttttt ttt ttt ttttt ttt ttt
tt
t t t tttttt tttt tttttt ttt tt tttt ttttttt ttttttttt tt tttt ttt ttt tttttt ttt tttttttttttt tttt tt tttt tt ttt ttt tt
tt ttt ttttttttttttttttttttttttttttttttttt
ttttttttttttt ttttt ttt tttt ttttt t tt tttttttttttt t tttttt
ttt tt tttt ttt tt ttt ttt ttt ttt ttt ttt tttttt ttt tt tttt    ttttttttttttttttttt tttt
tttttttttttttttt tttt ttt
 ttt tt tt tttttttt tt tttt tttt tttt tttt ttttt  tt tttttt t ttttttttttttttt ttttttttttt ttttttttttt
ttttt ttttt tttt  ttttttttttttt
 2 ttttttttttttttttttttttt
ttttt ttttttttt tttt

tt tttttttt tttttttt tttttt t ttttttttttttttt ttttt tttttttttttttttttttttt tttt tttttttt
ttttttttt tt tt tttt ttttt t ttttttttt ttt ttttt ttt  2 t ttttttttttttt ttttttttttttttt tttttttttttt tttt
:tttt tttt ttttt tt tttt tttt tttttttttt ttttt tt ttttttttttttttttttttt tttttttttttttttttttttt tttt
tt tttttttttttttttt ttt ttttt ttttt ttt ttttttt ttttttt
tttttt tt ttt t
t tt tttt tttt ttt tt tt tttt ttttt   ttttttttttttttttt ttt tttttttt tttttttt tttttt t tt
tttt
tttttttttttttttttttttttt tttttt ttttt tttttttttt ttt tttt
tttt:ttttttttttttttt tttttttttttttttttt ttttttt

tttt tttttt ttt ttt t tt tttt t t ttt tt tt tttt tttt tt
tt tttttttt tttttt tt ttt tt ttt tttt t t tt ttttttt
tttt ttttt tt
t tt tt ttttt t tt ttttt tttttttt tttt

tttttt tt ttt t
t tt tttt tttt ttt tt tt tttt tttt t
tttttt tt ttttttttt t tttttt tttt tttt tttttttttt
tttt
tt tt ttttt tttt tttttttttt tt ttt tt ttt ttt t t ttt t

  ttt ttttt ttttt tttt tt tttt tttt tttttt ttttttt t


ttttttt tttttttttttt tttttttt ttttttttttttt tttttttt
tttttttttttttttttttttttttttt ttttttttt ttttttttt
t

t tttt    
ttt tttt ttttttttttttttttttttttttttt ttttttttt

2   t ttt tttt tttt ttt ttttt ttt ttttt t


t ttt tttt tt
ttttttttttt tttttt tttttttttt ttttttt ttttttttttttt
tttttttttttttttt
t!  t tttttttt
ttt
t ttttttttt tt
tttttttttttttttttttttttttttttttttttttttttttttttttttt
tttt tttttt t t tttt ttt tttt tttt ttt ttt tt tttt ttt
tttttttttttttttttttttt ttttttttttt tttttttt tttttt
   
ttttttt ttttttttt tt
2 ttt tt tt tttttttt tt tttt tttt tttt tttt tttt
*  ttttttttttttttt! tttt ttttttttttttt ttttt ttttttttttt
ttt tttttttttt tttt 
t ttttttttttttttttttttttttttt ttttttttttttttttt tttt

tt tttt tttt tttttt ttttttt t t tttttt ttttttt tt ttttt tttt tt tttt ttttt t t tttt
tttttttttttt ttt tttttt ttttttttt

tt ttt
ttttttttttttttttttttttttttt tttttttttt
tttt
ttttttt tttttt tttttttttt ttttttt ttttttttttttt
ttttttttttttttttttttttttttttttttttttttttttttt
ttttttt tttttttttttttttttttttt ttttttttttt ttttttt
tttttttttttttt ttttttttt tt

  22>

  
  !   !  !  !    tttt ttttttttttttttt=tt
 ! !           tttttt tt=ttttttttttttt
tt t ttttt ttt ttttttt ttttt tttt tt tttttttttttt tt tttt    t=tttttttt tttt tttttttt
tt tt tttttttttt tt t ttttt ttt tttttttttttt ttttt ttt t t tt t   ttttt =ttttt tttttt ttttt
t t tttt ttttt ttt tttt ttttt ttt ttt tt tttt ttt tt ttttt tt tttt ttt tt
ttt tttt ttttttt tttttttt ttt ttt tttttttttttttttttt
tttt t t tttttttt tttt ttttttttt ttttt tttt 
tt ttttt tttttttttt t ttttt ttt tttttttttttttt t t ttttt tt < tt tt t t tttt tttt tttt tttttt ttttt ttttt ttt tttt ttt
ttttt ttttt ttt tttt tttt tttt ttt ttt ttttttttttttt t ttttt tttt tttt tttt tttt ttt tttt tttttt tttt tttt tttt ttt
ttttttttttt t t ttttt ttt ttttt ttt tttt t t tt ttt ttttt tttt ttt
tttt ! t ttt tt t ttt tttt ttttt ttttt ttt ttttt  t ttt  t   t t
tttt  t ttt tt t ttt ttt ttttt ttttt tttt ttttt ! t tt t tttt t t ttttt
tttt  t ttt tt t ttt ttt ttttt ttttt tttt ttttt 2 t t tttt tttt t t t ttt tttttt ttt ttt tt tttt tttt ttt tttt tt

tttt !  t ttt tt t tttttt ttt ttttt ttttt tttt tt tt tt t t tttt tttt tttt ttttt ttt tttt tttt tt
tttt ! !t ttt tt t ttttt tttttt ttt tttt ttt tttt  t ttt tttt ttt tttt tttt ttt t
tttt t ttt tt t ttttt ttt ttt tttt tttttt tttt  t tttt ttt tttt ttt ttt tt ttt
tttt ttt ttt ttttt ttttttt ttt tt t tt tttt tttttt ttt ttt  t tttt ttt tt ttt tttt ttt ttt
ttttt tttt
t tt t tttt t t ttttt tttt ttt ttttttt ttt ttt tttt ! t tttt ttttt tttt
tttt t tttt t t ttttt
 t t tttt ttttttt ttt tt t t tttt tttt ttt  tttt ttttt ttt ttt = t t tttt tttt tttt t t ttt tttttt ttt ttt tttt ttt tttt tttt tt
tttttt ttt
t tt tt t t tttt tttt tttt ttttt ttt tttt tttt ttt 
t tt tt t t tttt tttt tttt ttttt ttt tttt ttttt
tttttt ttttt t t tttt tttt t t tttt tttttt ttt tt t t tttt  t tttt ttttt tttt
ttt  t tttt ttt tttt ttt t ttt tttt tt
 t ttttt ttt t tttt   t ttttt ttt t tttt   t tttt ttt tttt ttt ttt ttt t ttt
 t tttttt ttt t tttt  ! t tttt ttttt ttt  ! t ttt ttt ttt t tttt ttt t tttt tt
t tttt t t ttttt t tttt t t ttttt
2  t ttttt ttttt ttt tttttt ttt tttttttttttttttttttt tttt > tt tt t t tttt tttt tttt tttt tttt tt t tttttt ttt
tttt ttt ttt
t tt tt t t tttt tttt tttt ttt tt tttt tt ttttt tttt t ttt ttttttttttt ttttttt tttttt tttttt tt
ttttt ttttt ttt tttt ttttt tttt tttt tttt tt tttt ttttt ttttt
 t t ttt  t tttt  t tt ttttt ttt tttttttttt ttt ttttttt ttttt
 t ttttt ! t t tt  tttttttttttt tttt ttttttt ttttttttt ttt tt ttttttt
t tttt t t ttttt ttttt
2 t ttttt ttttt t ttttttttt tttttttt tttt tttttttttttttt  ttt ttttttt ttttttt ttttt ttttttt tttt tttt ttttt
tttt
t tt tt t t tttt tttt tttt ttttt ttt tttt tttt tt ttttt tttt
ttttttt ttt t ! tttt ttttt tttt
 t tttt ttttt tttttt tttt ttt tttt tttt t t ttttt
 t ttttt ttt tttt tttttt ttttt ttt  tttt tttt ttt tttt ttttt tt ttt tttt tttt tttt ttttt
 t tttt tttt ttttt ttt ttttt ttttt   tttt tttt ttt ttt t t tttt tttt
! t ttttt ttttt tttttt tttt ttt ttt  t ttt tttt tttt ttt t t tttt tttt
t tttt t t ttttt  t t t tttt ttttt ttt ttt tttt ttt
 t ttttt tttttttt tt ttttt t tttttt ttt ttt ttttttt ttttttt  t tttt ttt t t tttt tttt ttttt tt
tttt
t tt tt t t tttt tttt tttt tttt ttt tttt tttt tt ! t tttt ttttt ttt t t tttt tttt tt
tttttt ttt t t tttt t t ttttt
 t ttt ttt ttt ttttt tttt tttt   ! 2    ! 
 t ttttt ttt ttttt ttt tttt tt !   ! !      
 t ttt ttt ttttt ttt tttt tttt 
! t ttttt ttt ttt tttt ttttt tt t ttttttttttttttttttttttttttttttttttttttttttttttttttt
t tttt t t ttttt ttt tt tttttt tttttttt tttttttt tttt tttt ttt ttt ttt tttt t ttt
 t t tttt tttt t t t ttt ttt tt t t tt tttt ttt ttttt ttt tttt tt ttttt tttt ttttt ttt tt tttt tttttt tttt tt t ttt tttt t tttt tt
tt t tttt
t tt tt t t tttt tttt tttt ttttt tt ttttttt tttttttt ttt tttttt ttttt tt tt tttttt ttt tttttttt ttt tttt
ttt tttt ttttt tttt t t ttttt ttt ttt tt tttt tttt ttt ttt ttttttttttttt t t ttttt ttttt tttttt
 t tttt ttttt ttt tt t ttttt tt ttt tt tt ttt t
 t ttttt tt t tttt ttt ttt tttt !  t ttttttt ttttt tttt ttttt tttttt tttt tttt tt
 t tt t ttt ttttt ttttt tttt tt !  t ttt ttttttt ttttt tttt tttt tttt ttttt ttttt
! t tt t tttt ttt ttttt ttttt tt !  t tttttt ttt ttttttt ttttt tttt ttttt tttt ttt
t tttt t t ttttt tttt ttt ttt 
  ! <  !  !   t t ttttt tttt ttt tttt tttttt ttt tt ttt tttttt tt tttt ttt
!   ! !       tttt ttttt tttt ttt tt
t tttt tttt ttt tttt ttttt tttt tt
 tttt tt ttt ttt t
ttt tt tttt t tttttttt
t tt tt ttttt ttt tttttt tt ttttt ttttt  t tttt ttttt ttt ttt tt tttt tttttt tttt ttt
ttt tttt tttt tttt ttttt tttt ttttttt tttt ttt tt tttt ttt ttt  t ttttt tttt ttt tttt tt tttt tttttt ttt ttt
ttttt ttt ttt tttt tttt ttttt ttttt tttt tttt ttt ttt ttttttttttttt t  t tttt ttttt ttt ttt tt tttt tttttt tttt ttt
tttt ttttttt t t ttttt tt tttt ! t ttttt tttt ttt ttt tt tttt tttttt tttt ttt
 t tttt ttt tttt tt =ttt tt tt t tttt t t ttttt
  t tttttttttttttttt=tttt
2=

2 t t ttttt ttt =ttt ttttttt tttt tt ttttt tttt ttt tttt ttttt    !      2  
tttt ttt tttt t ttt ttt t
t =ttt tt tttt ttttttt ttttt ttt tttt tttt t ttt ttttt ttttt ttttt ttttt tttttt
ttttt ttt tttt ttt tttt ttttt tttt ttt tt tt t t ttt tttt ttt ttt ttttt tttt ttttt ttt tt ttttttttt ttt t tt
tttt tttt ttt ttt = t t
ttt ttt tttt ttttt ttt tttt ttttt tt
t ttt tttt tttt ttt
 t tt  t t  tt ttt ttttttt ttt t
t
ttttt ttt tttt ttttt ttt ttt tttt tt

! t ttt t t tttt ttt tttt tttt tttt ttt tt tt t t tttt tttt ttt
2 tt:ttt ttttttt tttt ttttttt ttt tttt tttttt ttt tt ttt ttt ttt
ttt ttttttt tt ttttt tttt ttt tttt ttttt ttttt tttt tt  t ttttt ttt t  t t tttt ttt t  t ttttt ttt t
tttt tttt tttt ttttt tttt ttttt tttt ttttt tttt tt ! t tttttt ttt t t tttt t t ttttt
tttttttttttttttttttttttttttttttttttttt ttttttt t >
tttttttt tttttttttttttttttttttttt
tttttttttttt tttt tt
 t ttttt ttt tttt ttttt ttt tttt tttt ttt ttt tt ttttttttt ttt t ttttt tttt ttttttt tttttt

 t ttttt tttt ttt ttttt ttt tttt tttt ttt tttt ttt ttttttt tttt tttt tttt
ttttttt ttt ttt tttt ttt
 t ttttt ttttt tttt ttt ttt tttt tttt ttt ttt tttt tttt
t tt tttt ttttt tttt tttt tttt
! t tttt ttttt tttt  tttt  t ttt  t ttt
t tttt t t ttttt ! t!ttt tttt
 tttttt tttttttttttttttttttttttttttttttt ttttttt t 2   tt t ttttttt tttt ttttttt ttt t tt
t tttt ttt tttttt

tt ttt ttttttt tt ttttt tttt ttttttt tttt tttt t tttt t ttttttt tttt tttt tttt ttt !t ttt ttt tt tt tt ttt
tt t
tttt tttttt tttt
t ttt
t ttt ttttt tttt ttttt ttt ttt tttt ttttt tttttt ttt tttt tt
t tttt ttt tttt tttt t
tttttttttttttttt ttttttt tttttttt ttttttttttttttt
t tttt tttt ttttt ttttttt tttt ttttt ttttttt ttt tt 
ttttt ttt ttttt ttt ttttt ttt tttt t ttt ttt tt tttt tttt tt  t ttt ttt ttttt tttt ttt tttt tt tt ttttt
tttt tt t ttttt tttt ttttttt ttt ttt  t ttt ttt tttt ttttt ttt tttt tt tt ttttt
 t tttt t t tttt tttttt tttt ttttttt t tttt ttt  t ttt ttt tttt ttttt tttt ttt tt tt ttttt
 t tttt tttt t t tttttt tttt ttttttt t tttt ttt ! t ttt ttt tttt ttttt ttt tttt tttttt tt t
 t tttt t t tttt tttttt tttt t tttt tttt tttttt t tttt t t ttttt
! t tttt t t tttt tttt tttttt ttttttt tttt t ttt 2 ttt tttt tttt tttttttttt ttt tttt tt tttt ttt tt ttt t
t tttt t t ttttt ttttt ttttt tttt ttt
ttt tttt tt tt ttttt ttt ttt ttt ttt

 t t ttttt tttt ttt tttt tt tttt ttt tt ttt ttt tt tttt t tttt tttt ttt tttt tttt tttt ttt tttt tt ttt ttt tt tt
ttttt tttt tttttt tttt tttt tt ttt
t tttt tttt ttt ttt ttttt ttt
ttttt tttt ttt tttt tttttt ttt t  t tttt ttt ttt ttttt tttt ttt tt tt tt
 tttttt tt tttt ttt tttt ttttt tt tttt tt tttt ttt  t tttt ttt ttt ttttt tttt tt tt ttt tt
 tttttt tt tttt tttt ttt ttttt tt tttt tt tttt ttt  t tttt ttt ttt ttttt tttt tt tt ttt tt
 tttttt tt tttt ttt tttt ttttt tt tttt tt tttt ttt ! t tttt ttt ttttt ttt tttt tt tt ttt tt
! tttttt ttt tt tttt tttt ttttt tt tttt tt tttt ttt t tttt t t ttttt
tttt t t ttttt 22  ttt tttt tttt ttt ttt tt ttttttttt ttt t ttt ttt
ttt
< t t tttt ttttt tttt ttttttt ttt tttt ttttt tt tt t ttt tt tt ttt t ttttttt ttt ttttt ttttt ttt ttt ttt
t tttt tttt
tttt ttt ttt ttt tttt ttttt ttttt tttt ttt
t tttt tttt tt ttt tttt tttt tttt ttt tttt ttt tt ttttttttt ttt t tt
tttt ttttt tttt ttt tttt ttttt ttt t  t ttt ttttt ttt ttttt ttt tttt ttt t ttttttt ttt
 t ttt ttt tttt tttt tttt ttttt ttt tttt tttt  t ttt ttttt ttttt ttt ttt tttt ttttttt ttt t ttt
 t tttt tttt ttt ttt ttttt tttt ttt tttt tttt  t ttttt ttt ttttt ttt ttt tttt ttttttt ttt t ttt
 t tttt tttt ttt ttt tttt ttttt ttt tttt tttt ! t ttttt ttttt ttt ttt tttt ttt ttt t ttttttt ttt
! t ttt ttt tttt tttt tttt ttttt tttt ttt tttt t tttt t t ttttt
t tttt t t ttttt 22 tt ttttttttttttttttttttttttttttttttttttt ttt
2 t tttttttttt tttt ttttttttt tttttt tttttttttttttttt tt ttt tt ttttt ttt
ttt tttt t t ttttt tttt ttt ttt tt t

ttt t ttttt tt
t tttt ttttttt tttt ttttt tttt ttttttt ttttt tttt ttttt tttt ttttt tttt tttt tttt tttt ttt tttttt
ttt t ttttttttttttttttt
 tt t tttttttttt ttt tttt tttttt  t tttt ttt ttttt tt tt t t ttt tttt tt
 tt t tttttttttt ttttttt tttttt  t tttt ttt tt tt ttttt t t ttt tttt tt
 tt t tttttttttt ttt tttt tttttt  t ttt tttt ttttt tt tt t t ttt tttt tt
! t tttt ttttt tttt ! t ttt tttt tt tt ttttt t t ttt tttt tt
t tttt t t ttttt t tttt t t ttttt
  ! =2    !  2 t t tttt tttt tttt ttt t ttt ttt tt ttt
tttt tttt tttt tttttt tt
!   ! !       tttt tt tt ttttt

t tttt tttt ttt tttt tttt tttt ttt ttt


 ttttt ttt t
ttt ttttt tttttt t tttt tttttt ttt tttt t ttt  t tt tt ttt tttt tttttt tttttt tttt tttt ttt
ttttttt ttt tttt ttttttt t t tttt tttt ttt tttt tttttttt tt  t tt tt ttt tttt tttttt tttttt tttt tttt ttt
tttt ttttt tttt tttttttttt ttt tt ttttt tttt tttttttt tttt  t tt tt tttt ttt tttttt tttttt tttt tttt ttt
tttttt t tttt ttttt tttt tttt
t ttttttt tttt tttttttt tttttt  ! t tt tt ttt tttt tttttt tttttt tttt tttt ttt
ttt ttt t ttt ttt ttttttttttttt t t tttt ttttt ttttttttt t tttt t t ttttt
ttt t tttt ttt ttttt tttt    ! 22>  !  ! 
   !       !   ! !      
ttttttt ttttt ttttt ttttt tttt t ttt tttt tttt 
   !      2  #t ttttttt ttttttt tttttttttt ttt t ttttttttt t
tttt ttttt ttttttt ttttt t ttt tttt ttttt tttt tt ttttt tttttt ttt t ttttttttt ttt ttttttt ttt tttt ttt
   !         tttttttt ttt ttttt t ttttttt tttt ttttt tttt ttttttt tt
t ttt ttttttt tttt ttttt tttt ttttt ttttt tttt tttt tttttttt ttt tttttt tttt ttt tttt tttt t t tttttt tttt tt
ttttttt ttttt ttttt t ttttttt ttt tt ttttttt ttttt ttt ttt t
  2=

tttt t ttt tttttt ttttt t t tttt ttttt ttt tt t t tttt tttttttt tt ttt t t ttttt
t ttt
t ttt ttt ttttttttt ttt ttt
t t tttt ttttt ttt tttt ttttt ttt tt ttt t t t tt ttt t ttt tttt ttt tt ttt t ttttt ttt ttttt tttt tttt ttt ttt
ttt tttt ttt ttttt tttt ttttt ttt ttt tt ttttt ttt tttt  ttttt tttt tttt
t ttt t   t tttttt tttttt tt t t ttttt ttttt ttt ttttt tt
!   >      ttt t t ttttt ttt t tttttt ttttt ttttt tttttt t
    tttttt ttttt tttt ttt tt tt ttttt tt  ttt ttttt ttt tttttt tt ttttt t tttttt t t tttt
!        ! ttttt tttttt tt t tttttt t t ttttt ttttt ttt tt
    ttttt tt tt tttttt ttt ttt tttt tttt tttt t t ttttt
!      2 * 22 tttt tttt ttt tttt tttttt ttt tt tttt ttt tt tt ttttt ttt
   t tt tt ttt ttttt ttt ttttt tttttt ttt ttt tttt tttttttttt ttttttttt ttttt ttt tt ttt ttt ttt
tttt ttt tt tttttt tt ttttttttt ttttt tttt ttt tttt tt
2 t t tttt tttt tttt ttt tttt ttt tt ttttttttt ttt t tttt tt  tttt ttt ttttt tt ttttttttt ttt tttt ttt tttttttt

tttt tttt ttttt ttt ttttt tttt tttt

t ttttt tttt tttt tt  tttt ttt ttttttttt ttt tttt ttttt ttt tt tttttttt
tttt tttt tttt ttt tttt ttt tt ttttttttt ttt t   tt ttttttttt ttttt ttt ttt ttttttttt ttt tttt ttt
 t ttttt ttttt ttt tttt tttt tttt tttt ! tt ttttttttt ttt ttt tttt tttt ttt ttttttttt tttt
 t ttttt ttttt tttt ttt tttt tttt tttt tttt t t ttttt
 t ttttt ttttt tttt tttt ttt tttt tttt 22 ttt ttttt tttt ttt tttt ttt ttt tt ttt tt tttt ttt tttt
! t ttttt ttttt tttt ttt tttt tttt tttt tttttttt ttttttttttttttttttttttttttttttttttttt ttt
t tttt t t ttttt ttt tttt tttttt tttttttt ttt tttt tt ttttt ttt ttttt ttt
2< ttt tttt tttt t t tt tt t t tttt tttt tttt ttt tttt ttt  ttttttt tttttttt tttttttttttt ttttttt ttt ttt tttt
ttt tttttttt ttt tttt tttt tttt ttt tttt ttt tt ttt tttt
 t t  t tt  t ttt  ttttttt ttttt tttttttt ttttt tttttttttttt ttt tt
! t t t tttt t t ttttt tttttt
22 t ttttttttt tttt tttttttttt ttttttttt
ttttttttttttttttttt  ttttttt tttttttt ttttttt tttttttttttt ttt ttt tttt
tttttt ttt

t ttttt tttt tttt ttt tttt tttt tttt ttt ttt tttt
ttt tt  ! ttttt ttt ttt ttttttt ttttt ttttttt ttttttttttt
 t ttt tttt ttt tttttt tttttt tttt ttt ttttttt
 t ttt tttt ttt tttttt tttttt tttt ttt tttt t t ttttt
 t tttt ttt ttt tttttt tttttt tttt ttt   ! 22=     
! t tttt ttt ttt tttttt tttttt tttt ttt     !    !
t tttt t t ttttt  ! !  
2= t t tttttttt tttt ttttttt tt ttttttt tt ttt
ttt tttttt tt tt   tttt tttt ttttttt t tttttttt :ttttttt ttttt tttt ttttt
ttttt tttttt ttt
t ttttt tttt tttt tttt ttt tt tt t t ttt   
tttt tttt ttt tttt ttt
tttt tttttt ttttt ttt tt tt tttt tt
 !   :ttttttt ttt t ttttt tttt tttt tttttt t ttt tt
 t t  t ttt  t  t ttttttt
! t t t tttt t t ttttt !   ttt t tttt ttttt tttttt t ttt t ttt ttt :tttttt
2> t t tttt tttt tttt ttt ttt tt tt ttt
ttt tt t tttt tttt ttt t ttttttt
:ttt tt

t ttttt tttt tttt ttt tttt tttt tttt ttt ttt t !    tttttttttttttttttt:ttttttttttttt ttttttt
tt tttttt
 t :ttt tttt tt t tttt ttt tttt tt !    tttt :ttttttt tttttt ttttt tttt t tttttttt ttt
 t :ttt tt t tttt tttt ttt tttt tt ttttttt tttt ttt tt
 t :ttt tttt tt t tttt ttt tttt tt ttt ttttt ttt tt ttttttt ttt t ttt tttt tt tt ttt tt tt
! t :ttt tttt tttt tt t ttt tttt tt t t tttt tttttt ttttttttt ttttt ttt tt ttttt ttttttt t t tttt tttt
t tttt t t ttttt t tttt tttt tttt ttt tttt tttttt ttt tt ttt ttt
  ! 222      2 tt tt ttttt ttttt ttt
t tt t ttt ttt tttttt tttt tttt tt
     !    ! ! tttt ttt tt ttt t ttt tttt tt ttttt tttt ttt ttt tt
!   tt ttttt ttt tttt tttttt tttttttttt ttttttt tt
t ttt

   tttttttttttttt ttttttttttttt t tttttttt ttt tttt tttttttt ttttttttt ttttttt tt :tttttttttt ttt t
   ttt t ttttt tttt ttt tttt ttttt tttt tttt
!    tttt ttttttttt ttttttt t t ttt t ttttt ttttt  tttttt ttt tttttttttt
!    tttttt tttt ttttt ttttttttt ttt t ttttttt t  tttt ttt tttttt ttt ttt tttttttttt tt ttttt ttttttt t
!    tttttt t t tttt ttttttt ttttt ttt t tttttttt tt
ttt ttt ttt ttttt tttt tt ttt ttt t  ttt tttt tt ttttt tttttt ttt ttt tttttttttt ttt tttttt
ttt ttttt ttt tt ttttttt tttt ttt t ttt t tt ttt t tt t
ttttt ttt tttt ttttt ttttt ttt tt ttttt ttttttt t t tttt tttt ! ttt ttttttt tt tt ttttt tttttttttt ttt ttt tttttt ttt ttt
t tttt tttt tttttt ttt ttt tttt ttt ttt tttt t t ttttt
2 t t tttt tttttt ttt tt tttt ttt tttttt ttttttt tttt tttttttt tt 2 t tttttttttt ttttttttttttt ttttttttt ttttttt tttttttt
tttt tt tt
t ttttt tttt tttt ttt tttt ttttt tttt ttt ttt t t tttttt ttttt tttttt ttttt ttttttt tttt tt
t tttt ttt
ttt tt ttt t tt ttt tttt ttttt tttt ttt tttt ttt tt ttt t ttt ttt tttt
 t tttt tt ttt ttt ttttttt tttt tttttttt ttttt ttt
 t tt ttt tttttttt tttt tttt ttt tttttt tttttt  tttt ttttt tttttt tttt ttt ttttttt ttttt tttttt t
 t tttttttt ttttttt tttt tttt tt ttt ttt ttttt  t t tttt ttttttt tttttt tttt ttt ttttt ttttt ttttt
! t tt ttt ttttttt tttttttt tttttt tttt ttt ttt  tttttt tttt ttttttt ttt tttt ttttt t t ttttt ttttt
t tttt t t ttttt ! tttt t t ttt tttt ttttt ttttttt ttttt tttttt ttttt
2 :ttt ttt ttt ttttttt tttt ttt tt ttt !t ttt ttt tt ttt tttt t t ttttt
tttt tt ttttt tttt tttttt tt t tttttt ttt ttttt tttt
2=2

2< t t tttt ttttt tttt ttt tt tttt ttt tttt ttt tt ttt t tt  ttttttt ttttttttt tttttttttt ttt ttt ttttttttt ttttt
ttt ttt ttt ttttt tttt ttttt ttttt ttttttt tttttttt
t ttt tt
ttt ttttttttt tttt ttt ttttttt tt ttt t tttttt tttt  ttt ttttttt tttttttttt ttttttttt tttttt ttttttttt tt
ttt tt
 t ttt tttt ttttt ttttttttt ttt ttttt ttttt tttttt ! ttt ttttttt tttttttttt tttttt ttttttttt ttttttttt tt
 t ttttt ttt tttt ttttttttt ttttttt ttttt ttt tttt tt
 t ttttttt ttttt ttttt ttt ttt tttt ttttt tttttttt tttt t t ttttt
! t ttttt ttt ttttt ttt tttt ttttttttt ttttt tttttt 2= ttttttt
tttttttttt tttt ttttttt tttttttt tt ttttttttt
t tttt t t ttttt ttt ttttt ttt ttt ttttttt tttt ttttttttt ttttt ttt
22 ttt ttttt tttt ttt tttt ttt tt tttttttttttt tt tttt ttt tttttt t t tttt tttt ttt tttt ttttttt tttttt t t tttt tttt
ttttt ttttt ttt tttttt ttt ttt tttttttttt ttttttttt tt  tttt ttt ttttt ttttttt ttt ttt ttttttt tttttttt
ttttttt ttttttttt tttt ttt tttt tttttt ttt tttt tttt  ttt ttt ttttttt ttttttttt ttttttt ttttt ttt ttt
tttt ttt ttttt ttt  tttt ttt ttttt ttttttt ttttttttt ttttttt ttt tt
 ttttttt ttttttttt ttttttttt ttt ttt tttttttttt ttttt ! ttttttttt ttttttt ttt ttt ttttttt ttttt ttt ttt
tt tttt t t ttttt

  2
  ! 2  !  !  !  < ttt t ttt ttttt tttt tt tttt ttttt t t ttt tttttt ttt
       tttt ttt tttt tttttt tttt
t ttttttt tttttttttttt tt ttttt tttt ttttt ttt ttttt   t  t t t  t t 
t t ttttttt
t ttttttttttt ttttt tttt tttt tt ttttt ttttt tttt  t  t t t t  t 
ttt tt tt ttttt ttt tttt tttt ttt ttt ttttttttttttt t t tttttt ttt  t  t t t  t t 
tttttt t t ttttttttttttt  t  t t  t t t 
  !t ! t t  t t !t t   ! t tttttt ttt tttttttttt
   !t !t ! t t  t t t   t tttt t t ttttt
   !t  t !t ! t t t t   2 ttt t ttt ttttt tttt tttttt ttttt t t ttt tttttt tttt ttt
   !t  t t !t ! t t t   ttt tttt tt tttt tttt
    !t  t t !t  t ! t t   t t !t t  t t !
   !t  t t !t  t t ! t   t t !t  t  t t !
tttt ttt tttt ttttt tttttttttttt tttt ttttt t ttt tttt tttt  t t !t t  t  t !
ttttt ttt ttttt ttttt  t t !t  t t  t !
 tttt tttt tttt tttttt ttttt ttt ttt tttttt ttttt tt ttt ! t tttttt ttt tttttttttt
ttttt ttt ttttt tttt  t tttt t t ttttt
 t  t !t t  t !t t t     ! =  !  ! 
 t  t !t !t  t t  t t  !   ! !       
 t  t !t !t t t t  t   t tttt tttt ttttttt tttttttttttt tt ttttt tttt tttt
 t  t !t !t t t  t t   ttt tttttt ttttt t t ttttt tttt tttttttt ttttttttttt tttt
! t tttttt ttt tttttttttt tttt tttt tt ttttt ttttt ttttt ttt tt tt ttttt ttt tttt tttt tt
t tttt t t ttttt ttt ttttttttttttt t t tttttt tttt ttttttttttttt
2 ttttttttttt ttttttt ttttttttttt t ttttt tttt ttttttttt    :tt tt    tttttt tttt
 t  t t t  t ! t      :tt tt    tttttt tttt
 t ! t t  t t  t      tt :tt    tttttt tttt
 t ! t t t  t t      tt  :tt   tttttt tttt
 t ! t t t t  t      tt  tttttt :tt   tttt
! t ! t t  t  t t     tt  tttttt  :tt  tttt
t tttt t t ttttt ttt tttt t ttt tttt ttttt ttttt t t tttt ttttttttttttt
2
t t ttttt tttttt tttt ttt tt tttttt ttt tttt tttt tttt tt tttt tttt tttttt tttt ttt ttt tttt ttttt ttttt
t tttt ttt
ttttttt tttt tttt tttt tttt ttttt ttt tt tt t t tttt tttt tttt ttttttttt tttt tttttttttttt tttt
 t  t t  t ! t  t t  ttt tttt ttttt ttttt
 t  t  t tttt  tttt =  t tttttt=tt  t t ttttttt ttttt  t t ttttt
! t t t tttt t t ttttt
t t ttttt tttttt tttt ttt tt tttttt ttt tttttttt ttt
 tttt ttttttt ttt tttt ttttt ttttt t t tttt tttt tttt ttttt ttttttttttt
 t  t  t !t t  t   t tt  t t  t ttt
 t !t t  t  t  t  ! tttt t tttt t t ttttt
 t !t t  t  t  t  >  t ttttt ttttt tttttt t  t t tttt !
 t !t t  t  t  t  tt tt t t tttt tttt tttt tttt ttt ttttt 
! t !t t  t  t  t   t t tttt ! t ttttt t ttttt  t ttttt
t tttt t t ttttt  t t tttt ! t ttttt ttttt tttttt  t 
 t t tttt ttttt ttttt t t ttt tttttt ttt
 t  t t ! t   t t tttt ttttt ttttt tttttt  t t !
t !

! t t ttttt ttttt tttttt  t t tttt !


ttttt tt tt t t tttt tttt tttt tttttt tttt tt t ttttt tttt ttt ttt tt tt tttt

 t t ! t ! t  t t
   t tt tttt ttt tttttt  t t !t ttttt !
 t tttt  t ttttt  t t tt tt tt t t tttt tttt tttt tttttt tttt ttt tttt ttttt ttt
! t t ttt t tttt t t ttttt ttt
  2=2

 t t  t tt  t ttt   t t !t t !t t t 


! t t t tttt t t ttttt  t t t !t !t t t 
 tttt ttt t t ttt tttttt ttt t t tttt !t t ! t ttt ttt tttt  t t !t !t t t t 
tt tt t t tttt tttt tttt ttt tt tttttttt tttt ttttt  t t t !t !t t t 
 t!tt ! t ttttttttttt tttt  ! t tttt ttt tttttttttt
 t!tt ! t ttttttttttt tttt t tttt t t ttttt
 tt!t ttttt! t ttt ttttttt 2  tttt tttt ttt tttt tt tttt ttttt ttt tttt tttt tttt ttttt
! t tttttt ttt tttttttttt  t  t  !t t  t !t  t ! 
t tttt t t ttttt  t ! t  t  !t t  t !t  
2 ttttttttt ttttttttttttt!t ttt t tt tttttttttttt  t ! t  t  t t !t  !t  
tt tt t t tttt  tttt tttt tttttt tttt ttt tttt tttt  t ! t  t t  t !t  t  !
 t ttt  t t  t tt ! t ! t  t t !t  t  t  !
! t  t tttt t t ttttt t tttt t t ttttt
2 tttt tttt t t ttt tttttt ttt t  t tt t t t ttttt tttt  2 tt tt t t tttt tttt tttt ttt ttt t ttttt ttttt ttt ttt
tt tttt tttt ttttt

t t ttttt ttttt tttttt tttt ttt tt tttttt ttt ttttttt  t ! t t t ! t  t !t 
tttt ttttttttttt  t t  t !t t ! t ! t 
 t tttt  t  t  tttt  t t  t !t t ! t  t !
! t tt t tttt t t ttttt  t t  t t !t ! t  t !
 tttt tt t t ttt tttttt ttt t ttttt ! t tttt !t t tt ! t t  t !t t  t ! t !
tt t tt tt t t tttt tttt tttt tttt ttt ttttt tt t tttt t t ttttt
 t t ttttt ! t tttt tt t t ttt !   ! 222<  !  ! 
 t t ttttt ! t tttt t tt t ttt ! !        
 t t ttttt ! t tttt t tt t !t tt t tttt tttttttttttt tt ttttt tttt ttttt ttt ttttt
! t t ttttt ! t tttt t !t ttt tt ttttt t t tttt
t ttttttttttt ttttt tttt tttt tt ttttt ttttt tttt
t ttttt tttt ttt ttt tt tt tttt ttt tt tt ttttt ttt tttt tttt ttt ttt ttttttttttttt t t tttt ttttt
  ! 2  !  !  tttt tttt tttttt t t ttttttttttttt
!          t tttttt tttt ttt ttttt ttt tttttt ttt
t ttttttt tttttttttttt tt ttttt tttt ttttt ttt ttttt   t ttttt tttttt tttt ttt ttt tttttt ttt
t t ttttttt
t ttttttttttt ttttt tttt tttt tt ttttt ttttt tttt   t ttttt tttttt tttttt tttt ttt ttt ttt
ttt tt tt ttttt ttt tttt tttt ttt ttt ttttttttttttt t t tttttt ttt   t ttttt tttttt tttttt tttt ttt tttt tt
tttttt t t ttttttttttttt tttt tttt ttt tttt ttttt ttttt ttt ttttt ttttt
 t  t t ! t  t t !t  tt tttt tttt tttttt tttt ttt ttt tttt ttttt ttttt
t tttt ttt
   t tt ! t  t t!t  ttt tttt ttttt ttttttttt tttt tttttttttttt ttttt ttt tttt tttt
   t t ! tt t t !t ttttt
   t t ! tt!t t  t 22   t tttt ttt ttt ttt ttt tttt tt
tttt ttt tttt ttttt tttttttttttt tttt tttt tttt ttt tttt tttt tt tt t t tttt tttt tttt tttt ttt tttt tttttt ttttt ttt tttt
ttttt ttt ttttt ttttt ttttt
 t t!t ! t t  t tt tttt tttttt tttt tttttt  t ttt tttt ttt ttt ttt tttt tt
ttt ttttt
t tt tt t t tttt tttt tttt tttttt tttt ttt !  t ttt tttt tttt ttt ttt ttt tt
! t  t  t t t   t ttt ttt tttt ttt tttt ttt tt
 t ttttt  t t tt ! t ttt tttt tttt ttt ttt ttt tt
 t t tt ! t tttt ttt tttttttttt t tttt t t ttttt
t tttt t t ttttt 22 t t tttt tt tttt ttttt t t ttt tttttt ttt  ttttt tttttt
<
t t ttttt tttttt tttt ttt tt tttttt ttt tttt tttt tttt tttt t tttttt ttt tt tttttt ttttt tt tt t t ttt
ttttttt tttt tttt tttt tttt ttttt tttt tttt tttt ttt tttt tt ttt tttt
 t t  t  t !t !t t   ttttt ttttttt tttt t ttt tt tttttt tttttt tttt
 t!  t  t  tttt t ttttttt ttttt tttttt ttt tt tttttt tttt
! t t tttt t t ttttt  ttt tt ttttt tttttt tttttt ttttt ttttttt tttt
2 tttt tttt ttt tttt tttttt ttttt ttt tttt tttt tttt ttttt ! ttttt ttttttt tttt t tttttt ttt tt tttttt tttt
 t t t !t  t ! t  t  ttt tttttt tttt tt ttt tttt
 t ! t t !t  t t  t  2 t t tttt tttttt ttt ttttt ttt tttt ttt ttttt ttt tttt

 t ! t t  t  t t !t  tt tt t t tttt tttt tttt tttt ttt tttt tt tttt
 t ! t  t t !t t  t   t ttttt ttttt ttttt tttt ttt ttt tt
! t ! t t  t !t t  t   t ttt ttt ttttt ttt ttttt tttt tttt
t tttt t t ttttt  t ttt ttt ttt ttttt ttttt tttt tttt
= t t tttt tttt ttt tttt ttttt ttttt ttt ttt ttttt
t tttt ttt ! t ttttttttt ttttt ttttt tttttt tt
ttt tttt tttttt tttt t tttttt ttt tttttttttt
  t ! t  t  t t  t t   2  t ttttt ttttt tttt ttttt tttttt t ttt tttt
 t ! t t  t  t  t  t tt tt t t tttt tttt tttt tttttt ttttt ttt tttt ttttt tttt
 t ! t t  t  t t  t  ttt ttttt ttttt
 t ! t t  t  t  t t   t ttt  t t  t 
! t ! t t  t  t  t  t ! t t t tt
t tttt t t ttttt 2< t tttttttttttt ttttttttttttttttttttttttt tt ttttttttt
> tttt tttt ttt tttt ttttt ttt ttt ttt tttttt tttt tttt tt tttt tt
t tttt tttt ttt tttt ttt tttt
tttt ttttt ttttt ttt tttt ttttt
2=

 t tttttt tttttttttt ttt tttttt tt ttt   t ttt ttt ttt ttttt ttttt tttt tttttt ttt
 t tttttttttt tt ttttt tttt tttttt ttt  t ttt ttt tttt ttt ttttt ttttt tttt ttttt
 t tttttttttt tt tt ttt ttt ttttttt ttt  t ttt ttt tttt ttt ttttt ttttt tttt ttttt
! t tttttttt tttttt tt ttttt tttttt ttt  t ttt ttt ttt tttt ttttt ttttt tttt ttttt
t tttttt ttt tttttttttt ! t tttt ttt tttttttttt
  ! 222       t tttt t t ttttt
       !  22 t t tttt tttt ttt tttt tttttt ttttt t t ttt ttttt
t tttt ttt
     ! !      ! ttt tttt ttttt tttt
      !    t ttt tttt ttt ttttt ttt tttttt ttttt
 t t ttt t ttt ttt  t ttt ttt tttt ttttt ttt tttttt ttttt
 t ttt t t ttt ttt  t ttt ttttt ttt tttt ttt tttttt ttttt
 t ttt t t ttt ttt  t ttt tttt ttttt ttt tttttt ttt ttttt
 t ttt t ttt t ttt ! t tttt ttt tttttttttt
tttt tttt ttt tttt ttttt ttttt ttt ttttt ttttt t tttt t t ttttt
*  !        2 tt tt t t tttt tttt tttt ttt tttt tttttt ttttt ttt ttt
    !  tttt tttt ttttt
22  t ttt t tttt ttt  t t ttt  t tt tt t t tt tttttt tt tt ttt ttt
tt tt tttttt tttt ttttt tttttt tttt ttt tttt ttttt tttt  t tt t t ttt tttt tt tt tttttt tt t
ttt tttt ttttt ttttt  t tt t t ttt tt tt tttttt tt tt ttt
 t ttt  t t  t   t tt ttt t t tt tt tttttt tt tt ttt
! t t t tttt t t ttttt ! t tt ttt t t tttt tt tt tttttt tt t
2=  t tt !t tttt ttt t tttt  t tttt t t ttttt
tttt ttt ttttt tt ttt tttt ttttt ttttt 2
t t ttttt tttttt tttt ttt tt tttttt ttt tttt tttt tttt
 t ttt t tt tttt  t !t ttt ttttttt tttt tttt tttt tttt ttttt
 t ttt t tt  t !t tttt ttt  t ttttt ttt tttt ttt tttt t t tt
 t ttt tt !t tttt t tttt   t tt  t tttt  tttt
! t tttt  t t tttt !t tt tt ! t tttt t tttt t t ttttt
t tttt t t ttttt 2< t t ttttt tt t t ttt tttttt ttt t t tttttt ttt ttt ttttt ttt

2> t t ttttttttt t tttt ttttttttt ttttt t ttttt t tttt tttt ttttt ttttt ttt t t ttt tttt tttttt ttt tttt
t   
t ttttt tt tt t t tttt tttt tttt ttt  t t ttt  t tttt
tt tttttttt ttt tttt ttttt  t ttttt ! t tttt ttt tttttttttt
 t ttttt ttttt t  t ttttt  t   t tttt t t ttttt
 t ttttt  t ttttt ttttt  t   t    22  !  !  ! 
 t   t ttttt ttttt ttttt  t  t   ! !       
! t tttt ttttt tttt tt tt ttt tttttt ttttt t t ttttt ttt ttttt ttt tt ttt
t tttt t t ttttt tttttttttttt tt tttt
t ttt ttttttttttt ttttt tttt tttt t
2  t ttttt tttt  t  !t tttt  t tttt ttttt ttttt ttttt ttt tt tt tttt
ttt tttt ttttt ttttt
t tt tt ttttt tttt ttt tttt tttt ttt  t ttt ttttt ttttt tttt ttttt tttt tttt tttt
ttttttttttt   t ttttt ttt ttttt ttttt ttttttttt tttt ttt
t tttt  t tttt  t ttttt  !t tttt   tttttt ttttt ttt tttttttttt tttttttt ttt
 t t  t   t ttt   tttttt tttttttttt ttt ttttttttt ttttttt
! t tt t tttt t t ttttt    t ttttt ttttt tttttttttt ttt tttt tttt ttt
2 t t ttttt ttt t t tt ttttt tttttt ttt tttt  t  t tttt !   t tttttttttt tttttttttt tttt ttttttt ttt
tttt !t ttttt ttttt tt tt t t tttt tttt tttt ttt ttttt t tttt ttttt ttt tttt ttttt tttttt
t t tttt ttttt ttttt 22 t t tttt ttt ttttt t t ttt tttttt tt
 t tttt  t tttt !t tttt !t ttttt   ttttt ttttt tttt tttt =tttt tttt ttt ttt
 t tttt  t tttt !t  t !t tttt tttt tt tt t t tttt tttt tttt tttt ttt tttt ttttt tttt
 t tttt  t tttt !t tttt !t  t tttt  t ttt  t tt  t tttt
! t tttt ttttt tttt ! t ttt t tttt t t ttttt
t tttt t t ttttt 2= t t tttt ttt ttttt t t ttt tttttt tt
  ! 222<  !  !  tttt ttttt ttt tttt ttttt ttttt tttt ttt

!   ! !       tt tt ttt ttttttttt tttt ttttt


  t ttt tttt tttt ttttt ttttt tttt tttt ttt
t tttt tttttttttttt tt tttt
t tttt ttttt tt ttttt tttt  t ttt tttt tttt ttttt ttttt tttt tttt ttt
ttttt
t ttttttttttt ttt tttt tttt tt ttttt ttttt ttttt tt  t ttt tttt ttttt tttt ttttt tttt tttt ttt
tttt tttt ttt tttt ttttttttttttt t t tttt tttttt tttt tttt tttttt t ! t tttt ttt tttttttttt
ttttttttttt t tttt t t ttttt
  t tttt tttt ttttt ttttt ttt ttt ttttt 2>  t ttttt ttttt tttt ttt tttt ttttt tt t tttt
  t ttt tttt tttt ttttt ttttt ttt ttttt tt tt t t tttt tttt tttt ttt tttt ttt ttttt t t tttt tttt
  t ttt ttt tttt tttt ttttt ttttt ttttt ttttt
  t ttt ttt tttt tttt ttttt ttttt ttttt  t ttttt ttttt ttttt ttttt ttt tttt tt t ttt
   t ttt ttt tttt tttt ttttt ttttt ttttt  t ttttt ttttt ttttt ttttt tttt ttt tttt tt
tttt ttt tttt ttttt tttttttttttt tttt ttttt tt ttt tttt tttt  t ttttt ttttt ttttt ttttt tttt ttt tttt tt
ttttt t t ttttt ttttt ! t tttt ttt tttttttttt
22 t t tttt tttt ttt tttt tt tttt ttttt t t ttt ttttt
t tttt ttt t tttt t t ttttt
ttt tttt tttttt tttt
  2=

   ttttt ttttt tttt ttt tttt tttt ttttt tt ttt tttttt ttttt t t ttttt tttt tttttttt ttttttttttt tttt
tt tt t t tttt tttt tttt tttt ttt tttt !ttt tttt tttt tttt tt ttttt ttttt ttttt ttt tt tt ttttt ttt tttt tttt tt
 t tttt ttttt ttttt tttt ttt tttt ttttt tt ttt ttttttttttttt t t tttttt tttt ttttttttttttt
 t tttt ttttt ttttt ttttt tttt ttt tttt tt  t tttt tt tttt t tttt t t tttt !
 t tttt ttttt ttttt ttttt ttt tttt tttt tt   t tttt tttt tt tttt t tttt t t !
! t tttt ttttt ttttt ttttt ttt tttt ttt ttt   t tttt t tttt tt tttt t tttt t !
t tttt t t ttttt   t tttt t tt tttt tttt t tttt t !
  t tttt tttt tttt ttttt tttt ttttt ttttt ttt    t tttt t tt tttt !t tttt t tttt 
tt tt t t tttt tttt tttt tttt ttt tttt ttttt tttt   t tttt t tt tttt !t tttt tttt t 
 t tt  t ttt  t ttt   t tttt t tt tttt !t tttt t tttt 
! t tttt t tttt t t ttttt tttt tt ttt tttt ttttt ttttt t t tttt tttttttttttttt t t ttt
  ! 2<    !  ttttt ttttt
!   ! !       tt tttt tttt tttttt tttt ttt ttt tttt ttttt ttttt
t ttt tt
 tttt tttt tttt tttttttt
t ttttttttttt tttttttttttt tt tttt
t tttt ttttt tt 2  t tt ttttt ttttt ttt  t tttttttt  t t !
tttttt ttt tt tttt t t ttttt tttt ttttttt
t ttttttttttt tttt tt tt t t tttt tttt tttt tttt ttt ttttt tt
tttt tttt tt ttttt ttttt ttttt tttt tttttttttt tttt tttt ttttttt  t tttttttt  t ttttt tt ttttt ttt  t t !
ttttt tttt tttttttttttttt ttt tttttttt tttt tttt ttttt tttt  t tttttttt  t tt ttttt  t ttttt ttt t !
tttt ttttt ttt ttt ttttttttttttt t t tttttt tttt tttttt t  t tttttttt  t tt ttttt ttttt ttt  t t !
tttttttttttttt ttttt tttt ttttt tttt ! t tttttt ttt tttttttttt
 t ttttt tttt t ttttt  t ttttt t  t tttt t t ttttt
  t tttt ttttt t ttttt  t ttttt t  =  t tt t ttt t t t tt tt tt t 
  t tttt t ttttt ttttt  t ttttt t  tt tt t t tttt tttt tttt ttt ttttt t
  t tttt t ttttt ttttt  t ttttt t   t ttt t tt t t tt tt t t tt
   t tttt t ttttt  t ttttt ttttt t   t ttt t tt t t tt tt t tt t 
  t tttt t ttttt  t ttttt ttttt t   t ttt t tt t t tt tt t tt t 
  t tttt t ttttt  t ttttt t ttttt  ! t tttttt ttt tttttttttt
tttt tttt tt ttt tttt ttttt tttttt t tttt t t ttttt
tttttt ttttttttt tttt tttttt ttttttttttttttt tttt ttttttt >  t ttt ttt t t !t tttt tttt 
ttt ttt ttt tt tt t t tttt tttt tttt tttttttt tt tt t t tttt tttt tttt tttttt tttt ttt tttt tttt
2 t t ttttt ttt t t ttt tttttt ttt
ttt t t tttt ttt  t !  t t  t tt  t 
tt t

t tt tt t t tttt tttt tttt tttt ttt tttt ttttt tttt ! t ttt t tttt t t ttttt
 t   t t  t t  tttt tttt t t ttt tttttt tt
! t tt t tttt t t ttttt tttttt t ttttt t  t ttttttttt ttttt 
2 t t tttt tttt ttttt t t ttt tttttt ttt
tttt  t tttttt  t t ttt tt tt t t tttt tttt tttt tttttt tttt ttt tttt ttttt ttt
ttt  t !

t tt tt t t tttt tttt tttt tttt tt tttttttt tt ttt


tttt ttttt ttttt t t tttt ttttt  t t  t tt  t ttt
 t tttt  t  t tttttt ttt !t  t t ttt ! t  t tttt t t ttttt
 t tttt  t t tttt tttttt !t  t ttt   tttt ttt t t ttt tttttt tt
 t tttt  t  t t tttt !t tttttt  t tt ttttt t t ! t ttttttt ttttt t tttt
! t tttttt ttt tttttttttt tt tt t t tttt tttt tttt ttt tt tttttttt tttt ttttt
t tttt t t ttttt  t ! t t t ttttt ttttttt ttttt t tttt
 t t tttt tttt ttttt t t ttt tttttt ttt
ttt t ttt tttttt t   t t t t ! t ttttttt ttttt ttttt tttt
ttttt 

t tt tt t t tttt tttt tttt tttt ttt tttt ttt  t ttttttt t t t ! t ttttt ttttt tttt
tttt ! t tttttt ttt tttttttttt
 t ttt t ttttt t tttttt t t tt t tttt t t ttttt
 t ttt t ttttt t t ttt tttttt    ! 2<    ! 
 t ttt t ttttt t tttttt t ttt  !   ! !      
! t ttt t ttttt t ttt tttttt t  
t tttt t t ttttt t tttt tttt ttttttt tttttttttttt tt ttttt tttt tttt
  t =tttt ttt ! t t ttttt t ttt tt  ttt tttttt ttttt t t ttttt tttt tttttttt ttttttttttt tttt
tt tt t t tttt tttt tttt tttt ttt tttt tttttt tttt tttt tttt tt ttttt ttttt ttttt tttt tttttttttt tttt tttt ttttttt
 t ttt ! t ttttt t ttt tt t =tttt  ttttt tttt tttttttttttt ttt tttttttt tttt tttt ttttt tttt
 t ttt ! t ttttt t =tttt t ttt tt  tttt tttt ttt ttt ttttttttttttt t t tttttt tttt tttttt t
 t ttt ! t ttttt =tttt t t ttt tt  tttttttttttttt ttttt tttt ttttt tttt
! t ttt ! t ttttt t ttt tt =tttt t   t ttttt ttt t t tttt t ttttttt 
t tttt t t ttttt   t ttt ttttt t t tttt t ttttttt 
<  t  t tttt !t t tttt tttt ttttt !   t ttt tttt ttttt t t t ttttttt 
tt tt t t tttt tttt tttt tttttt ttttt ttt
tttt !t ttt    ttt tttt t ttttt t tttttttt 
 t tttt !t t tttt
    t ttt tttt t t ttttt t ttttttt 
 t tttt  t ttt   t ttt tttt t t ttttttt ttttt t 
 t ttt ! tttt ttt tttttttttt   t ttt tttt t t ttttttt ttttt t 
t tttt t t ttttt ttt tttt tt ttt tttt ttttt ttttt t t tttt ttttt
  ! 2  !  !  tt tttt tttt tttttt tttt ttt ttt tttt ttttt ttttt
t tttt ttt
!   ! !        tttt ttt ttt ttt tt tt t t tttt ttttttttt ttttt tttt 
t tttt tttt ttttttt tttttttttttt tt ttttt tttt tttt
2=<

2 tttt tt t t ttt tttttt tt  t t ttttt t ttttttt tttttt t ttttttttt !
ttttt ttttt t t t t ttttt tt ! t t ttttt t tttttt ttttttt t ttttttttt !
tt tt t t tttt tttt tttt tttt tt tttttttt ttt   t t t tttt t t ttttt
tttt tttttt < t t tttt ttttt ttttt t t ttt tttttt tt
 t ttttt ttttt t t t t ttttt tt !t tttt t tttt t ttt:t tt t ttttt
 t ttttt ttttt tt t t t t t tttt tt tt t t tttt tttt tttt ttttt ttt tttt ttttt
 t ttttt ttttt ttttt tt t t t t   t ttt:t tt t tttt tttt t !t tttttt 
! t tttttt ttt tttttttttt  t ttt:t tt t tttt tttt !t tttttt t 
t tttt t t ttttt  t ttt:t tt tttt t tttt !t t tttttt 
2   tttttt ttttt !t t  t ttttt t tttt ! t tttttt ttt tttttttttt
tt tt t t tttt tttt tttt tttttt tttt ttt tttt tttt t tttt t t ttttt
 t   t t  t t < tt tt t t tttt tttt tttt tttt ttt tttt tttt tttt t t ttt
! t tt t tttt t t ttttt tttt tttt ttttt
   ttt tt !t t tttt tt t  t tttttt ttt ttt  t !t t tttttt ttttt ttttttt t t tttt
tt tt t t tttt tttt tttt ttt tttt ttttt tttt t t tttt tttttt  t t ttttttt t !t t tttttt ttttt tttt
 t tttt tt tttttt t !t ttt tt t  t ttt ttt  t t ttttttt t ttttt !t t tttttt tttt
 t tttt tt tttttt t !t ttt tt ttt tttt t   t t ttttttt t ttttt !t tttttt t tttt
 t tttt tt tttttt t !t ttt tt ttt tttt  t  ! t t !t t tttttt ttttt ttttttt t tttt
! t tttttt ttt tttttttttt t tttt t t ttttt
t tttt t t ttttt   ! <2<<  !  ! 
 tttt ttt t t ttt tttttt tt !   ! !       
ttttt tttttt t t !t t ttttttt tttt t tttt tttt ttttttt tttttttttttt tt ttttt tttt tttt

t t ttttt ttttt tttttt tttt ttt tt tttttt ttt ttttttt ttt tttttt ttttt t t ttttt tttt tttttttt ttttttttttt tttt
tttt ttttttttttttt tttt tttt tt ttttt ttttt ttttt ttt tt tt ttttt ttt tttt tttt tt
 t !  t  t ttt ttttttttttttt t t tttttt tttt ttttttttttttt
! t t tttt t t ttttt   t tttt ttttt tttt t !t tt t 
< tttt tttt t t ttt tttttt tt   t tttt t tttt ttttt t !t tt t 
tttttt ttttt t ttttt  t t ttttt     tttt  t t tttt ttttt t !t tt
tt tt t t tttt tttt tttt tttttt tttt ttt tttt ttttt ttt   t tttt  t tt t t tttt ttttt t !
ttt    t tttt  t tt t t t tttt ttttt !
 t   t t  t ttt   t tttt  t tt t t ttttt t tttt !
! t tt t tttt t t ttttt    tttt  t tt t t ttttt !t t ttt
  ! 2<  !  !     tttt  t tt t t ttttt !t tttt 
!         tttt tttt ttt ttt tttt ttttt tttt
t tttt tttt ttttttt tttttttttttt tt ttttt tttt tttt tt tttt tttt tttttt tttt ttt ttt tttt ttttt ttttt
t tttt ttt
ttt tttttt ttttt t t ttttt tttt ttttttt
t ttttttttttt tttt ttt tt tt t t tttt tttt tttt ttttttttt tttt tttttttttttt tttt
tttt tttt tt ttttt ttttt ttttt ttt tt tt ttttt ttt tttt tttt tt <2 tttt ttt t t ttt tttttt tt
ttt ttttttttttttt t t tttttt tttt tttttt t t ttttttttttttt tttt t t t tttttt tttttt t tttt
 t tttt tt tttt  t t tt tt t ttttt ! tt tt t t tttt tttt tttt ttt tt tttttttt tttt ttttt
  t !t tttt tt tttt  t t tt tt t tttt  t t t t tttttt tttttt t ttttt ttt
  t !t tt tt tttt tt tttt  t t t tttt  t tttt t t t tttttt tttttt t tttt
  t !t tt tt  t tttt tt tttt t t tttt  t t t t tttttt tttt tttttt t tttt
   t !t tt tt  t tt tttt tttt t t tttt ! t tttttt ttt tttttttttt
  t !t tt tt  t tt tttt t tttt t tttt t tttt t t ttttt
  t !t tt tt  t tt tttt t ttttt tttt  <2 tttt tttt t t ttt tttttt tt
  t !t tt tt  t tt tttt t ttttt t ttt ttttt t ttttt ttttt  t t t tttt
tttt tttt ttt ttt tttt ttttt tttt
t t ttttt ttttt tttttt tttt tt tttttt ttt tttttttt ttt
tttttttttttttttt tttt tttttttttttttttttttt
t tttttttttt tt ttt t
ttttttttt tttttttttttttttttttttttttttttt ttttttttttttttttt  t  t  t tttt  tttt
2  t t t tttttt tttt t !t tttt  t ttt ! t tt t tttt t t ttttt
tt tt t t tttt tttt tttt tttt ttt ttttt t <  t ttt tt t t t ttt t t tt
 t  t tttttt !t t t tttt t tttt ttt tt tt t t tttt tttt tttt tttttt tttt ttt tttt tttt
 t  t tttttt !t tttt t t tttt t ttt  t ttt  t t  t 
 t  t tttttt !t tttt t t tttt t ttt ! t t t tttt t t ttttt
! t  t tttttt !t tttt t tttt t tttt <   ttttt ttt tttt  t t  t tttttt 
t tttttt ttt tttttttttt tt tt t t tttt tttt tttt tttt ttt ttttt t
= t t tttt ttt t t ttt tttttt ttt
t ttttt !t ttttttt t ttttt  t tttttt  t ttt  t ttttt tttt t 
t ttttt 

t tt tt t t tttt tttt tttt tttttt tttt ttt ttt  t tttttt  t ttt  t ttttt tttt  t 
ttttt tttt  t tttttt  t ttt  t ttttt  t tttt 
 t tttt ttt  ttttt   t tttt t ! t tttttt ttt tttttttttt
! t tttt t t tttt t t ttttt t tttt t t ttttt
> t t tttt tt t t ttt tttttt tt <<   ttttt ttttt  t t t t tttttt ttt
t ttttt ttttttt tttttt t t ttttttttt ! tt tt t t tttt tttt tttt tttttt tttt ttt tttt ttttt ttt
tt tt t t tttt tttt tttt tttt ttt tttt ttt ttt
 t t ttttt t ttttttt t tttttt ttttttttt !  t t  t tt  t 
 t t ttttt t t ttttttt tttttt ttttttttt ! ! t ttt t tttt t t ttttt
  2=2

  ! <22  !  !   t t ttttt t ttttttttt  t tttttt t !t ttt


!   ! !        t t ttttt t ttttttttt  t tttt tttttt t !
 ! t t ttttt t ttttttttt tttttt t !t  t ttt
t tttt tttttttttttt tt ttttt tttt ttttt ttt ttttt t tttt t t ttttt
tttttt t tttttttttttttttttttttttttttttttt tttt tttttt tttttt 22 tttt tt t t ttt tttttt ttt ttttttttt t tttttt !t t 
ttttt tttt tttt ttt ttt ttttttttttttt t t ttt tttttt ttttt t t tttt tttt ttt
t t ttttt ttttt tttttt tttt tt tttttt tt
tttt ttttttt tttttt t t ttttttttttttt tttttttt tttt ttttttttttttt
 t ttttt tttt ttttt ttttttt tttttttt ttttttt  t  t   t
  t ttttt ttttt tttt ttttttt tttttttt ttttttt ! t! t tttt t t ttttt
  t ttttt tttt ttttt ttttttt tttttttt ttttttt 2  t tt tt tttt t t  t ttttttttttt ttttt 
  t ttttt tttt tttttttt ttttt ttttttt ttttttt tt tt t t tttt tttt tttt tttt ttt tttt ttttt tttt
   t ttttt tttt tttttttt tttttttt ttttt tttttt  t tttt t  ttttt   t tttt t
tt
t tttt tt ttt tttt ttttt ttttt ttt ttttt tttttt t t ttt ! t tttt tt t tttt t t ttttt
tttt tttttttttttt ttttt ttt tt tt t t tttt tttt tttt tttttttt 2  t tttt t ttttttt t t tt tt tttt t  t 
tttt tttt tttt ttttt tttt tt tt t t tttt tttt tttt tttt ttt tttt ttttt tttt tttt tttt
<2  t ttt ttt ttttt tt tt ttttttt tt  t tttt ttt  t tttt t  ttttt 
tttt tttt ttt tttt tttttt ttttt ttt ttttt ttttt ! t tttt t t tttt t t ttttt
 t ttttt ttt tt tt ttttttt ttt tt 2< tttt tttt t t ttt tttttt tt
 t ttttt tt tt ttt ttttttt ttt tt ttt t t ttttt t ttt tt ttttttttt t t !
 t ttttt ttt ttttttt tt tt ttt tt tt tt t t tttt tttt tttt ttt tt tttttttt tttt tt t t tttt
! t ttttt ttt tt tt ttt ttttttt tt ttttt
t tttt t t ttttt  t ttt t ttt tt ttttt t t ttttttttt t t !
<= t t ttttt tttt ttt
tttttt ttttt tt tttt ttttttt tttttt ttttt

 t ttt t ttttt t ttt tt t ttttttttt t t !


tt tt t t tttt tttt tttt ttttt ttt tttt ttttt  t ttt t ttt tt t ttttt t ttttttttt t t !
 t tttttt ttttttt tttttt tttt ttttt tt ttttt ! t tttttt ttt tttttttttt
 t tttttt ttttt tt ttttttt tttttt tttt ttttt t tttt t t ttttt
 t tttttt ttttttt ttttt tt tttt tttttt ttttt   ! 22=  !  ! 
! t ttttttt tttttt ttttt tt tttt tttttt ttttt !   ! !       
t tttttt ttt tttttttttt t tttt tttt ttttttt tttttttttttt tt ttttt tttt tttt
<> t t ttttt tt t t tttttt ttt tttttt ttt ttttt ttttttt :tttttt ttt tttttt ttttt t t ttttt tttt tttttttt ttttttttttt tttt
tttttt
t tt tt t t tttt tttt tttt ttttt ttt ttttt ttt t tttt tttt tt ttttt ttttt ttttt ttt tt tt ttttt ttt tttt tttt tt
ttttt ttttt ttt ttttttttttttt t t tttttt tttt ttttttttttttt
 t tttttt :ttttttt ttttttt ttt ttttt tttttt    tt t !t t ttt :tttt t ttt 
 t tttttt ttt ttttt :ttttttt ttttttt tttttt     t tt t !t t ttt :tttt t tt
 t tttttt ttt :ttttttt ttttttt ttttt tttttt     t ttt tt t !t t :tttt t tt
! t tttttt ttt ttttt ttttttt :ttttttt tttttt     t ttt !t tt t t :tttt t tt
t tttttt ttt tttttttttt     t ttt !t ttt tt t t :tttt 
2 t t ttttt tttt t t ttt tttttt ttt  tttttt t tttttttttt tt t    t ttt !t ttt t tt t t :ttt
tttt ttttttt ttttttt
t tttt ttttt ttttt ttt  tttttt ttt     t ttt !t ttt t :tttt tt t 
tt tt ttttttt tttttttt t ttttttttt     t ttt !t ttt t :tttt t tt
 tt  t tt  t t tttt ttt ttt tttt ttttt ttttt ttt ttttt ttttt
! t  t tttt t t ttttt tt tttt tttt tttttt tttt ttt ttt tttt ttttt ttttt
t tttt ttt
2  t tttt tttttt tttt ttt tttt t tttt ttt tt tt t t tttt tttt tttt ttttttttt tttt tttttttttttt tttt
tt tt t t tttt tttt tttt tttt ttt tttt ttttt ttttt t t ttt ttt tttt ttttt ttttt
ttttt ttttt 22  t tttt tttt t ttt tttt t  t !t tt t
 t ttttt  t ttt  t tt tt tt t t tttt tttt tttt tttttt tttt ttt tttt ttttt ttt
! t ttt t tttt t t ttttt ttt
  ! 222<  !  !   t t  t t  t 
!   ! !        ! t tt t tttt t t ttttt
t tttt tttt ttttttt tttttttttttt tt ttttt tttt tttt 2=  t ttttt tttt t !t t ttttttt ttt tt 
ttt tttttt ttttt t t ttttt tttt tttttttt ttttttttttt tttt
t t ttttt tttttt tttt ttt tt tttttt ttt tttttttt ttt
tttt tttt tt ttttt ttttt ttttt ttt tt tt ttttt ttt tttt tttt tt ttttttttttttt
ttt ttttttttttttt t t ttt tttttt tttt tttt ttttttttttttt  t tttt  tttt  t tt
  ttttttttt  t t tttttttt t ttttt tttttt  ! t t t tttt t t ttttt
   tttttt ttttttttt  t t tttttttt t ttttt  2> tttt tt t t ttt tttttt tt
   tttttt t ttttttttt  t tttttttt t ttttt  !t tttttt t tttt t t tttttt tttt
   tttttt t tttttttt ttttttttt  t t ttttt  tt tt t t tttt tttt tttt ttt tt tttttttt tttt ttttt
    tttttt t tttttttt  t ttttttttt t ttttt   t tttttt tttt !t t t t tttttt tttt
   tttttt t tttttttt  t ttttt t ttttttttt   t !t tttt tttttt t t tttttt t tttt
tttt tttt t ttt tttt ttttt ttttt  ttt ttttt ttttt  t tttt !t tttttt t t t ttttt ttttt
tt tttt tttt tttttt tttt ttt ttt tttt ttttt ttttt
t tttt ttt ! t tttttt ttt tttttttttt
ttt tt tt t t tttt tttt tttt ttttttttt tttt tttttttttttt tttt t tttt t t ttttt
ttt tttt ttttt ttttt = tttt tttt t t ttt tttttt tt
22  t ttttttttt tttttt t !t  t t ttttt tttt  t ttt tt !t tttttt tttttt ttttt t ! 
tt tt t t tttt tttt tttt tttt ttt ttttt t tt tt t t tttt tttt tttt tttt ttt tttt t
 t t ttttt t ttttttttt  t tttt t tttttt !  t t ttt tt !t tttttt ! t tttttt ttttt 
2==

 t t ttt tt !t tttttt ! t ttttt tttttt      tttt  t tttt t !t tttt tttt
 t t ttt tt !t tttttt ! t ttttt t ttttt    tttt  t tttt t tttt !t tttt
! t tttttt ttt tttttttttt ttttttttttt ttt tttt t ttt tttt ttttt tttttttttttt ttt
t tttt t t ttttt tttt t ttt tttt ttttt tttt
=  t ttttt ttttt tttt t t  t !t ttttt ttttttttttttt ttttttt tttttttttttt tttt tttttttttttt
tt tt t t tttt tttt tttt tttt ttt tttt t tttttt ttt tttttttttttt tttt
 t ttttt  tttttt !t ttttttt tttt =2 tt ttttttt tttttttttttttttttttttt tttttttttt ttt tt t
 t ttttt  tttttt !t ttttttttttt  tttttt ttttt ttt
t ttt t tt tt tttt !t ttttt 

 t ttttt  tttttt !t tttttt ttttt  t ttttt  t ttttt
! t ttttt  tttttt ttttt t!t ttttt  t t tt ! t t tt
t tttt t t ttttt t tttt t t ttttt
  ! =2=<  !  !  == t t tttt tt tttt ttttt t t ttt tttttt ttt
t ttt ttttt tttt
!   ! !        !t  t ttttt 

t tttt tttt ttt tttt t ttt tttt


tttt ttt tttttt tttt
t tt tttttt tt ttttt ttttttttttt ttt  t t ttt  t ttttt t ttttt tttt t !
tttttt tttt tttt ttttttt tttttt ttt t tttttttt tttt   t t ttt  t ttttt !t ttttt tttt t 
   !t ttttttt ttt t tttttt t tttt t   t t ttt  t ttttt t tttt t !t tttt
    !t ttttttt ttt t tttttt t tttt t  ! t ttttt tttt ttt ttt tt tt tttt
    tttttt !t ttttttt ttt t t tttt t  t tttt t t ttttt
    tttttt t t tttt !t ttttttt ttt t  => t t tttt tttttt ttttt t t ttt tttttt ttt
t tttttt t tttt !t 
    tttttt t t tttt t !t ttttttt ttt ttttttt tttttt
t tttt tttt ttt tt tttttttt tttt ttttt
   tttttt t t tttt t ttt !t ttttttt  t ttttttt t tttt ttttttt t !t t ttttt
ttt tttttttttttt ttt tttt t ttt tttt ttttt ttttttttttt  t ttttttt t ttttttt tttt !t tttttt t 
tttt tttt t ttt tttt ttttt tttt  t ttttttt t t ttttttt tttt !t tttttt 
ttt tt tt t t tttt tttt tttt ttttttttt tttt ttttttttttttt ! t tttttt ttt tttttttttt
tttttttt tttt ttttttttttttttttttttttt ttttttttttttttttttt t tttt t t ttttt
tttttttttttt > t t tttt tt tttt ttttt t t ttt tttttt ttt
 t tttt !t ttttt
=2 t ttttt tttttttttttt ttttttttttttt ttttt tttttttttttt ttttt t t t ttt

t tt t ttttt ttttt tttttt tttt tt


 t !t tttt t ttttttt
t tttt ttt tttt ttttt tttt tt tttttt ttt tttttttt tttt ttttttttttt
 tttt !t ttttt ttt t ttttttttt  t t ttttttt  t tt  t t  tttt
 tttt !t ttttt ttt t  t ttttttttt t ttttttt ! t tttt t tttt t t ttttt
 tttt !t ttttttttt ttttt ttt t  t tttttttt  > tttt tttt ttt tttt tttttt ttttt t t tttt tttttt ttt
tttttt
! tttt !t ttttttttt ttttt ttt  t t tttttttt  ttttt tttt tt t t t t !

tttttt ttt tttttttttt  t t ttttttt ttttt tttt tt t t t !
=2 tttt tttt ttt tttt tttttt ttttt ttt tttt tttt tttt ttttt  t t ttttt ttttttt tttt tt t t t !
  tttttt tttttt  t t ttt ttt t ttttt  t t ttttt !t ttttttt tttt tt t t
 t ttttt t tttttt ttt ttt tttttt  t t ! t t ttttttt !t ttttt t tttt tt t
 t ttttt t ttt ttt t tttttt tttttt  t  t tttt t t ttttt
 t ttttt t ttt ttt tttttt t tttttt t    ! >2><    
! t ttttt t ttt ttt tttttt tttttt  t t       !   !   
t tttt t t ttttt      
= t ttttttttt ttttttttttttt tttt!tttttttttttttttt!    ttttt ttttt t !t ttttt t ttttttt 
tttttttttt  
t tt t ttttt ttttt tttttt tttt ttt tt ttttt     t ttttt ttttt !t ttttt t ttttttt 
ttt tttttttt tttt ttttttttttt     t ttttt !t ttttt ttttt t ttttttt 
 t tttt  tttt  t tt     t ttttt !t ttttt t ttttt t tttttt
! t  t t tttt t t ttttt     t ttttt !t ttttt t ttttttt ttttt 
= tt tt ttttt tttt ttt tttt ttttt ttttt ttt tttt tttttt      t ttttt !t ttttt t ttttttt t tttt
ttttttt !t tttttttttt tttttttttt t tttttt   tttt t ttt tttt ttttt ttttt ttt ttttt ttttt
 t t  t   t t ttt tttt tttt tttt ttttttttt tttt ttttt tttt t ttt ttttttttttt
! t tt t tttt t t ttttt tttttttttttttttttt
=< tttt tttt ttt tttt tttttt ttttt t t ttt tttttt tttttt tttt >2 tttt ttt ttt ttt tttttt tt
ttttt ttt ttttttt ttttt !t t tt tttt tt ttttt  t  
t ttttttt !t t ttttt tttttttt  t tttttt

 t ttttttt  t tt ttttt t ttttt tt tttt !t  tttt tttt ttt tttt t ttt tttt
 t ttttttt  t tt ttttt t tt tttt !t ttttt   t t ttttttt  t ttttt !t t tttttttt tttttt
 t ttttttt  t tt ttttt t ttttt !t tt tttt   t t ttttttt  t ttttt t !t tttttttt tttttt
! t tttttt ttt tttttttttt  t t ttttttt  t t ttttt !t tttttttt tttttt
t tttt t t ttttt ! t t ttttttt  t !t ttttt t tttttttt tttttt
  ! =2>  !  !  t tttt t t ttttt
!   ! !        >2 t tttt ttttt t t ttt tttttt ttt
t ttttt t t ttttt
t tttt tttt ttttttt tttttttttttt tt ttttt tttt tttt tttt ! t :ttt
t tt tt ttttt tttt ttt tttt ttttt tttt
ttt tttttt ttttt t t ttttt tttt tttttttt ttttttttttt tttt  t t  t tt  t ttt
tttt tttt tt ttttt ttttt ttttt ttt tt tt ttttt ttt tttt tttt tt ! t t t tttt t t ttttt
ttt ttttttttttttt t t tttttt tttt ttttttttttttt > tttt tttt ttt tttt tttt ttt tttt tttt tttt ttttt
   tttt tttt ! t tttt  t ttttt    tttttt tttttttt t ttttt t  t ! t ttttt
    tttt tttt !t tttt  t ttttt   t t tttttt ! t ttttt tttttttt  t t ttttt
    tttt tttt !t tttt  t ttttt   t t tttttt t ttttt ! t tttttttt  t ttttt
    tttt  t tttt !t tttt ttttt   t t tttttt t tttttttt ttttt  t ! t ttttt
  2=>

! t t tttttt t ttttt tttttttt  t ! t ttttt    ttt ttt ! t ttttt  t ttttttt  t ttttt 
t tttt t t ttttt     ttttt ttt ttt ! t  t ttttttt  t ttttt 
> tttt ttt t ttt ttttttttt
tttttttt t ttttttttttttttt !     ttttt  t ttt ttt ! t  t ttttttt  t tttt
t :ttttt
t tttt tttt ttt tttt tttttt tt tttttttt      ttttt  t ttttt ttt ttt ! t  t ttttttt 
 t ttttttt t t :tttttt ! t tttttttt t tttttt     ttttt  t ttttt  t ttt ttt ! t  t tttttt
 t ttttttt tttttttt t t :tttttt ! t t tttttt    ttttt  t ttttt  t ttttttt ttt ttt ! t 
 t ttttttt t tttttttt :tttttt t ttttttt ! t      ttttt  t ttttt  t ttttttt  t ttt ttt !
! t tttttt ttt tttttttttt tttt tt ttt tttt ttttt ttttt ttt tttt ttttt ttttt
t tttt t t ttttt t t ttt ttt tttt tttt tttt tttttttt
t tttt tttt ttt
>< tttt tttt ttt tttt tttttt ttttt t t ttt tttttt ttttt tttt ttttt tttttt ttt tttttttttttt ttttt ttt tttttttttttttt t t ttt
ttttt ttt
t ! t tttt t ttttt t tttttttttt  t  t ttt
 tttttt tttt
 t t ttttt  t ! t tttt t t tttttttttt  t ttt 2t t tttt tttttt ttttt t t tttt tttttt ttt
ttt !t ttttttttt  t tttt
 t t ttttt  t ! t tttt t  t t tttttttttt ttt  t tt tttt !
t tt tt t t tttt tttt tttt tttt ttt ttt
 t t ttttt  t ! t t tttttttttt tttt t  t ttt t ttt tttt
! t tttttt ttt tttttttttt  t ttt !t ttttttttt  t tt tttt !t ttttt 
t tttt t t ttttt  t ttt !t ttttttttt  t tt tttt  t ttttt !
  ! >2      t ttt !t ttttttttt  t ttttt  t tt tttt !
        ! t ttttt tttt ttttt tttt ttttt
!        t tttt t t ttttt
   t tttttt tttt tttttt t t t t tt 2 tttt tttt ttt tttt tttttt ttttt t t tttt ttttt
    t t tttttt tttt tttttt t t t tt
tttttttttt  t tttttt  t tttttt t ttttttttt !

    t tttttt t tttttt tttt t t t tt  tttttttttt  t ttttttttt  t tttttt t tttttt !
    t tttttt t t tttttt tttt t t tt  tttttttttt  t ttttttttt  t tttttt !t tttttt 
    t tttttt t t ttt t tttttt tttt   tttttttttt  t ttttttttt  t tttttt t tttttt !
   t tttttt t t ttt t tttt tttttt  ! tttttttttt  t tttttt  t ttttttttt t tttttt !
    t tttttt t t ttt t tttt t ttttt tttt t t ttttt
ttttttttttttttttttttttttttttttttttttttttttttttttttttt  t tttttttttttttt
tttt ttttttt! ttt tttt ttttttttt

ttttt tttttt tt t ttttt tttttt tttt ttt tt tttttt ttt tttttttt ttt
tt tttt tttt tttttt tttt ttt ttt tttt ttttt ttttt
t tttt ttt ttttttttttttt
ttt tt tt t t tttt tttt tttt ttttttttt tttt tttttttttttt tttt  t tttt  tttt  t tt
ttt tttt ttttt ttttt ! t t t tttt t t ttttt
>2 tttttttt ttttttttttttt t tttt t!t tttttttt ttttt 
t t ttttt t ttttttt tttt ttttt ttttttt tttt ttt ttttt tt
 t
t ttttttttttttttttt ttttttttt ttttttttt ttttttt tttttttttt tttt ttttt
tttt ttttttttttt
tttt ! t ttttttt t tttttt  t tttttttt 

 t tttt  tttt  t tt  t t tttttt ! t ttttttt


! t t t tttt t t ttttt  t t ttttt
>= tttt tt ttt tttttt ttt  t tttttt t tttttt ttttt ttttttt  t t tttttt t ttttttt
t ! t tttt tttt ttt tttt tttttt tt ttttttt ! t !t tttttt ! t ttttttt
 t tttttt t tttttt t ttttt tttttttt t ! t tttt t t ttttt
 t tttttt tttttt t t ttttt tttttttt t ! < tttt tttt ttt tttt tttttt t t tttt tttttt ttttt t t ttt
 t t tttttt tttttt t ttttt tttttttt t ! tttttttttttt tt
! t tttttt ttt tttttttttt
ttttt  t ttttttt  t tttttttttt t tttttt 

t tttt t t ttttt  t ttttt  t ttttttt  t tttttttttt t tttttt 
>> tt tt t t tttt tttt tttt tttt ttt tttt tttttt ttttt tt  t ttttt  t ttttttt  t tttttt t tttttttttt 
tttttt t :t tttt tttt  t !t ttt ttt ttttt t !   t ttttt  t ttttttt  t tttttt  t tttttttttt 
 t ! t :t tttt !t ttt ttt tttt  t ttttt  ! t tttttt ttt tttttttttt
 t ! t :t tttt !t tttt ttt ttt  t ttttt  t tttt t t ttttt
 t ! t :t tttt !t ttt ttt  t tttt ttttt    !  2  !  ! 
! t ! t :t tttt !t tttt  t ttt ttt ttttt  !   ! !       
t tttt t t ttttt t tttt tttt ttttttt tttttttttttt tt ttttt tttt tttt
 tt tt ttttt tttt ttt tttt ttttt ttttt ttt ttt tttttt tttt ttt tttttt ttttt t t ttttt tttt tttttttt ttttttttttt tttt
tt tttttttttttt ttttttt ttttttttttttttttttttt tttt tttt tt ttttt ttttt ttttt ttt tt tt ttttt ttt tttt tttt tt
 t t  t   t ttt ttt ttttttttttttt t t tttttt tttt ttttttttttttt
! t tt t tttt t t ttttt    ttttt t t tttt  t tt t ttttt 
 tttt tttt ttt tttt t ttt ttttt t t ttt tttttt ttttt tttt     t ttttt t tttt  t tt t ttttt 
ttttt ttt  t :ttttttt tttttt  t !t tttttt t ttttttt     t ttttt ttttt t tttt  t tt t 
 t  t tttttt !t tttttttt t :ttttttt tttttt      t ttttt t ttttt tttt  t tt t 
 t  t tttttt !t tttttttt :ttttttt  t tttttt      t ttttt t tt t ttttt tttt  t 
 t  t tttttt !t tttttttt  t :ttttttt tttttt     t ttttt t tt t  t ttttt tttt 
! t ttttt ttt ttt tt tt tttt     t ttttt t tt t  t tttt ttttt 
t tttt t t ttttt     t ttttt t tt t  t tttt  t tttt
  ! 2<      tttt tttt ttt ttt tttt ttttt ttttt t t tttt ttttt ttttt
      !      tt tttt tttt tttttt tttt ttt ttt tttt ttttt ttttt
t tttt ttt
        !    ttt tt tt t t tttt tttt tttt ttttttttt tttt tttttttttttt tttt
!    2  !      !  ttt tttt ttttt ttttt
    
2>

2 tttt ttt t t ttt tttttt ttt t t ttt tt t  t ! t ttttt ttt    tt tttttttttttttt tt ttttt t t ttttttttttt
ttt tt t

t t ttttt ttttt tttttt tttt ttt tt tttttt ttt ttttttt tt tt t t tttt tttt tttt tttt ttt ttttt 
tttt ttttttttttttt  t  t t ttttt  t tt tt tttttt  t t tttt
 t tttt  tttt  t tt  t  t t ttttt  t tt tt  t tttttt t tttt
! t t t tttt t t ttttt  t  t t ttttt  t tt tt  t t tttttt tttt
= tttt tttt t t ttt tttttt ttt t t ttttt !t t tt ttt ttttt ! t ttttt tttt ttt ttt tt tt tttt
tttt !
t tt tt t t tttt tttt tttt tttt ttt ttttt t t tttt t t ttttt
 t t ttttt !t tttt !t tttttt t tt tt <   tt tttttttttttttt ttttt ttttt tttttttt!t
 t t ttttt !t tttt !t t tt ttt ttttt tt tt t t tttt tttt tttt ttt tt tttttttt tttt ttttt
 t t ttttt !t tttt t tt ttt tttttt !  t tttt tttt  t t ttt ttttt !t 
! t ttttt tttt ttt ttt tt tt tttt  t tttt tttt  t t ttt ttttt !t 
t tttt t t ttttt  t  t  t tttttttt ttt ttttt !t 
> tttttt t  t t t tttt tttt ttt !t tt ! t tttttt ttt tttttttttt

t t ttttt tttttt tttt ttt tt tttttt ttt tttttttt ttt t tttt t t ttttt
ttttttttttttt 2    ttttt tttt t !t tttt ttttt  t 
 tttt  t ttt  t tt tt tt t t tttt tttt tttt tttttt tttt ttt tttt ttttt ttt
! t t t tttt t t ttttt ttt
 tttt tt t t ttt tttttt ttt t t ttt t ttttt  t  t ttttt  t t  t   t tt
ttttt tt tt t t tttt tttt tttt ttt tt tttttttt tttt ttttt ! t ttt t tttt t t ttttt
 t ttttt ttt  t t  t t tttttt tttt =   t t t ttt tttttt ttt t  t ttt !!t t  t tttttt tt
 t ttttt  t ttt t t  t tttttt tttt tttt
 t ttttt ttt t t  t tttttt tttt
t t ttttt ttttt tttttt tttt ttt tt tttttt ttt ttttttt
! t tttttt ttt tttttttttt tttt ttttttttttttt
t tttt t t ttttt  t tttt  tttt  t tt
 tttttt t ttttt ttttt !t t tttttt ttttt ! t  ! t t t tttt t t ttttt
tt tt t t tttt tttt tttt tttttt tttt ttt tttt ttttt ttt >   t tt t t tttttt t ttttt t t :ttt
ttt
t t ttttt tttttt tttt ttt tt tttttt ttt tttttttt ttt
 t t  t   t tt ttttttttttttt
! t ttt t tttt t t ttttt  tttt  t tt  t t
  !  2>  !  !  ! t ttt t tttt t t ttttt
!   ! !          ! 22     
t tttt tttt ttttttt tttttttttttt tt ttttt tttt tttt         
ttt tttttt ttttt t t ttttt tttt tttttttt ttttttttttt tttt !       ! !  
tttt tttt tt ttttt ttttt ttttt ttt tt tt ttttt ttt tttt tttt tt    !       ! 
ttt ttttttttttttt t t tttttt tttt ttttttttttttt  t ttttt ttttttttt  t tttt t !t tttttt 
   t ttttt t tttttt tt t t ! t ttttt   t tttttt ttttt ttttttttt  t tttt t !t 
    t t ttttt t tttttt tt t ! t ttttt   t tttttt t ttttt ttttttttt  t tttt t !
    t tttttt t ttttt t tt t ! t ttttt   t tttttt t ttttt  t ttttttttt tttt t !
    t tttttt t t ttttt tt t ! t ttttt    t tttttt t ttttt  t ttttttttt !t tttt 
    t tttttt t ttttt t tt t ! t ttttt tttt tt ttt tttt ttttt ttttt t t tttt ttttt ttttt
   t tttttt t ttttt ! t t tt t ttttt tt tttt tttt ttttt tttt ttt ttt tttt ttttt ttttt
t tttt ttt
    t tttttt t ttttt ! t tt t t ttttt tttt tttttttttttt ttttt ttt tttt ttttt tttttt ttt t tt ttttt tt
ttt tttt tt ttt tttt ttttt ttttt t t tttt ttttttttttttt tttt tttt tttt tttttttt
tt tttt tttt tttttt tttt ttt ttt tttt ttttt ttttt
t tttt ttt 2t t t ttttt t t t tt ttttt tttttt ttt  tt ttt t tttttt 
ttt tt tt t t tttt tttt tttt ttttttttt tttt tttttttttttt tttt tttttt t ttttt t tttt ttttt ttt tttt ttttt
ttt tttt ttttt ttttt  t tttttt tttttt t t tt ttt ttttt 
2   t t t ttttttttt ttt t t ttttt ttttt tttt ttttt t   tttttt t tttttt t t tt ttt ttttt 
  tt ttt tttttt t tttttt t t ttttt 

t t ttttt ttttt tttttt tttt ttt tt tttttt ttt ttttttt ! tttt ttt tttttttttt
tttt ttttttttttttt tttt t t ttttt
 t tttt  tttt  t tt 2tt tttttttttt ttttttttttttttttt#ttt ttttttttttttttt
! t t t tttt t t ttttt t tttt ttttt ttt tttt ttttt ttttt t t ttttt ttttt
2  t t t t t ttt ttt tt tt t !t :tttttt  t#ttt tttttttttttttttt

t t tt ttt tttttt tttt ttt tt ttt ttt ttttt ttt ttt  t#ttt tttttttttttttttt
ttttttttttttt  t#ttt tttttttttttttttt
 t tttt  tttt  t tt ! t#ttt tttttttttttttttt
! t t t tttt t t ttttt t tttt t t ttttt
   t tt tt tt ttt !t t tttttt ttttttt !t 22t ttt tt t ttttt tttttt ttt tttt tttt tttt tttttt ttt tttt
tt tt t t tttt tttt tttt tttt ttt ttttt ttt tttttttt
 t t tt tt !t ttttttt tt ttt tttttt !t  t ttttttt ttttttttttt t t t tt ttt tttttt  
 t t tt tt !t ttttttt !t tt ttt tttttt  t t  t   t t
 t t tt tt !t tt ttt tttttt ttttttt !t ! t tt t tttt t t ttttt
! t t tt tt tt ttt !t tttttt ttttttt !t 22 tttt ttttt ttt tttttttttttttttt ttttt ttt tttt tttt ttt
t tttt t t ttttt ttttt
 t tttt t=ttt t ttt t t t ttt 
  2>

 t tttt t t=ttt t ttt t t t tt 2  t tttt tttt t !t t tttt tt t
 t tttt t t=ttt t ttt t t ttt 
t t ttttt tttttt tttt ttt tt tttttt ttt tttttttt ttt
 t tttt t t=ttt t ttt t ttt t  ttttttttttttt
! t tttt t t=ttt t ttt t t ttt   t ttt  t tttt  t tt
t tttt t t ttttt ! t t t tttt t t ttttt
2 ttt tt ttttttttt t t ttttttt ttttttttt
ttttt tttttt    ! 2222     
tttttt t ttttt 
t tttt tttt ttt   t ttt tttt tttt      !
ttttt      ! 
 t ttttt t tttttt t ttttt t ttttt         ! ! 
 t ttttt t tttttt t ttttt ttttt t     !   
 t ttttt t tttttt t ttttt t ttttt    t ttt tt t t t t ttt tttttt
! t ttttt t tttttt ttttt t t ttttt      t ttt tt t t t t ttt ttttt
t tttt ttt tttttttttt   t t tttttt ttt tt t t t t tt
  22  !  !     t t tttttt t ttt tt t t t tt
!   ! !           t t tttttt t ttt ttt tt t t 
t tttt tttt ttttttt tttttttttttt tt ttttt ttt   t t tttttt t ttt t ttt tt t 
ttttt ttt tttttt ttttt t t ttttt tttt tttttttt tttttttttt    t t tttttt t ttt t ttt t tt
ttt tt tttt tttt tt ttttt ttttt ttttt ttt tt tt ttttt tt   t ttt tttt ttttt ttttt t t tttt ttttttttttttt
tttt tttt ttt ttt ttttttttttttt t t tttttt tttt ttttttttttttt tt tttt tttt tttttt tttt ttt ttt tttt ttttt ttttt
t tttt ttt
  t ttttttt t t tttt ttttt  t t tt ttt tt tt t t tttt tttt tttt ttttttttt tttt tttttttttttt tttt
    ttttt ttttttt t t tttt  t t tt ttt tttt ttttt ttttt
    ttttt t ttttttt t tttt  t t tt 22  t ttt t t  t tt t ttt t ! t tttt
    ttttt t tt t ttttttt t tttt  t  tt tt t t tttt tttt tttt tttttt tttt ttt tttt ttttt ttt
    ttttt t tt t t ttttttt t tttt  ttt
  t ttttt t tt t t ttttttt  t t ttt  t tt  t ttt  t t
   t ttttt t tt t t ttttttt  t tttt  ! t  t tttt t t ttttt
ttttttttttttttttttttttttttt tttttttttttttttttttt tttt 22  t ttttt ttt t  t !t  t !!t ttttt ttt
ttttt ttttt tt tt t t tttt tttt tttt tttttt tttt ttt tttt tttt
tt tttt tttt tttttt tttt ttt ttt tttt ttttt ttttt
t tttt ttt  t  t t  t ttt
ttt tt tt t t tttt tttt tttt ttttttttt tttt tttttttttttt tttt ! t tt t tttt t t ttttt
ttt tttt ttttt ttttt 2 tttt ttt t t ttt tttttt ttt t  t tttt t t tttttt tttt :ttt
2   t t tttt ttttt  t t !t ttttt ttttt !
tt tt t t tttt tttt tttt tttttt tttt ttt tttt ttttt ttt
t t ttttt ttttt tttttt tttt ttt tt tttttt ttt ttttttt
ttt tttt ttttttttttttt
 t   t t  t t  tttt  t tt  t t
! t tt t tttt t t ttttt ! t tttt t tttt t t ttttt
2< tttt ttt t t ttt tttttt ttt t tttt t !t t ttttt t t ! 2 tttt tt t t ttt tttttt ttt t t tt t !t tt t  t t tt t
ttttt ttt tt tt t t tttt tttt tttt ttt tt tttttttt ttt tttt tt tt t t tttt tttt tttt tttttt tttt ttt tttt tttt
ttttt  t tt  t t  t ttt
 t t ttttt t tttt t !t t !t ttttt tt ! t t t tttt t t ttttt
 t t t ttttt tttt t !t t !t ttttt tt 2< tttt tttt t t ttt tttttt ttt !t ttt  t tttt t ttttt t 
 t t ttttt t tttt t t !t !t ttttt tt tttttt tt tt t t tttt tttt tttt ttt tt tttttttt tttt ttttt
! t tttttt ttt tttttttttt  t !t ttt tttt t  t ttttt t t ttttt
t tttt t t ttttt  t t tttt t !t ttt ttttt t t ttttt
22 tttt tttt t t ttt tttttt ttt t tttt  t tttttt t t t tttt  t ttt ttttt tttt t  t t t tttttt !
ttttt
t t ttttt ttttt tttttt tttt ttt tt tttttt tt ! t tttttt ttt tttttttttt
tttttttt tttt ttttttttttttt t tttt t t ttttt
 t tttt  t tt  tttt 22  t tttt ttt ttttt t t !t ttttt t tt tt t t ttt
! t  t t tttt t t ttttt tttt tttt tttt ttt ttttt t
2=    ! t tt t tttttt !t t ttt ttttt   t t tttt !t ttttt t ttttt t tt
tt tt t t tttt tttt tttt tttt ttt tttt t t t tttt tttt  t t tttt !t ttttt t ttttt ttt 
ttttt  t t tttt !t ttttt ttttt ttt t 
 t ttttttt ttt ! t ttttt t tt t ! ! t ttttt tttt ttt ttt tt tt tttt
 t ttttttt ttt ! t ttttttt t !t  t tttt t t ttttt
 t ttttttt ttt ! t tt t!t t tttt   !  2=2  !  ! 
! t ttttt tttt ttt ttt tt tt tttt !   ! !       
t tttt t t ttttt t tttt tttt ttttttt tttttttttttt tt ttttt tttt tttt
2>   t ttt t ttttt t t ttttt ttttt t ! ttt tttttt ttttt t t ttttt tttt tttttttt ttttttttttt tttt

t t ttttt tttttt tttt ttt tt tttttt ttt tttttttt ttt tttt tttt tt ttttt ttttt ttttt ttt tt tt ttttt ttt tttt tttt tt
ttttttttttttt ttt ttttttttttttt t t tttttt tttt ttttttttttttt
 t tt  t t  t ttt   t ttttt t tttttt t t tttt ttttt 
! t tttt t tttt t t ttttt    t t ttttt tttttt t t tttt ttttt 
2 tttt tt t t ttt tttttt ttt t ttttt !t ttttt t t !t tttt     t tttttt ttttt t t tttt ttttt 
tttttt tt tt t t tttt tttt tttt tttttt tttt ttt tttt tttt     t tttttt t ttttt t tttt ttttt 
 t t  t ttt  t tt     t tttttt t ttttt  t t tttt tttt
! t t t tttt t t ttttt    t tttttt t ttttt  t tttt t tttt
2>2

tttt   t ttt tttt ttttt ttttt t t tttt ttttttttttttt  t t  t   t t


tt tttt tttt tttttt tttt ttt ttt tttt ttttt ttttt
t tttt ttt ! t tt t tttt t t ttttt
ttt tt tt t t tttt tttt tttt ttttttttt tttt tttttttttttt tttt < tttt tttt t t ttt tttttt ttt t ttt  t !t t t t tt
ttt tttt ttttt ttttt ttttt
2=  t  t ttttt t ttttt tttttt t t :tt tt tt t t tttt tttt tttt ttt tt tttttttt tttt ttttt

t t ttttt tttttt tttt ttt tt tttttt ttt tttttttt ttt  t  t !t t ttt t t t tt t ttttt
ttttttttttt  t ttt t  t !t t t t tt t ttttt
 tttt  t tt  t t  t ttt  t !t t t t tt t tttttt 
! t ttt t tttt t t ttttt ! t tttttt ttt tttttttttt
2> tttttttt ttttttttttttt ttttt t t!tttttttt ttttt t tttt t t ttttt
tt tt t t tttt tttt tttt tttt ttt ttttt tt  2 tttt ttt t t ttt tttttt ttt  t tt tt  t !t tttt tttttt 
 t  t tttt !t ttttt t tt t ttttt  ttt
 t  t tttt !t ttttt t t tt t tttt
t t ttttt ttttt tttttt tttt ttt tt tttttt ttt ttttttt
 t  t tttt !t ttttt t tt t t tttt tttt ttttttttttttt
! t ttttt tttt ttt ttt tt tt tttt  tttt  t tt  t t
t tttt t t ttttt ! t ttt t tttt t t ttttt
 tttt tttt t t ttt tttttt ttt t t t ttttt t :ttttt !t !t ttt =  t !t  t tttt ttttt tttt t !t tttt
tttt
t t ttttt tttttt tttt ttt tt tttttt ttt tttttttt ttt

t t ttttt ttttt tttttt tttt ttt tt tttttt ttt ttttttt ttttttttttttt
tttt ttttttttttttt  tttt  t tt  t t
 t tttt  tttt  t tt ! t ttt t tttt t t ttttt
! t t t tttt t t ttttt   !  >  !  ! 
  t ttt t ttttt t t tttttt tttttt ! !   ! !       
tt tt t t tttt tttt tttt tttt ttt ttttt tt t t tttt tttt t tttt tttt ttttttt tttttttttttt tt ttttt tttt tttt
ttttt ttt tttttt ttttt t t ttttt tttt tttttttt ttttttttttt tttt
 t !t ttt t ttttt t tttttt t ttttt tttt tttt tt ttttt ttttt ttttt ttt tt tt ttttt ttt tttt tttt tt
 t !t ttt t t ttttt t tttttt ttttt ttt ttttttttttttt t t tttttt ttttttttttttt
 t !t ttt t ttttt t t tttttt ttttt    tttt t t tttttt tttt tttttt t !
! t ttttt tttt ttt ttt tt tt tttt    ttttttt tttttt tttttt tttt t!
t tttt t t ttttt    ttttttt!t tttttttttttt tttt
2  t tttt t !t t ttttt tttttt  t tt     tttttt!ttttttttttttt tttt
tt tt t t tttt tttt tttt tttttt tttt ttt tttt ttttt ttt     tttttt!t tttttttttttt tttt
ttt   t tttttt !t tttttt t tttt tttt t
 t t  t tt  t ttt    ttttttt !ttttttt t tttt ttttt 
! t  t tttt t t ttttt t tttt tttt tt ttt tttt ttttt ttttt t t tttt ttttttttttttt
2  t ttttttt t t tttt t t t ! t =ttt tt tttt tttt tttttt tttt ttt ttt tttt ttttt ttttt
t tttt ttt

t t ttttt tttttt tttt ttt tt tttttt ttt tttttttt ttt ttt tt tt t t tttt tttt tttt ttttttttt tttt ttttttttttt
ttttttttttttt ttttt ttt tttt ttttt ttttt
 tttt  t tt  t t > tttt tttt t t ttt tttttt ttt ttttt t tttttt  t t ttttt
! t ttt t tttt t t ttttt tttttt 
  !  =  !  ! 
t t ttttt ttttt tttttt tttt ttt tt tttttt ttt ttttttt
!   ! !        tttt ttttttttttttt
t tttt tttt ttttttt tttttttttttt tt ttttt tttt tttt  t tttt  tttt  t  t
ttt tttttt ttttt t t ttttt tttt tttttttt ttttttttttt tttt ! t tt t tttt t t ttttt
tttt tttt tt ttttt ttttt ttttt ttt tt tt ttttt ttt tttt tttt tt   t tttt tt t ! t !t ttt tttt !t
ttt ttttttttttttt t t tttttt tttt ttttttttttttt tt tt t t tttt tttt tttt tttttt tttt ttt tttt ttttt ttt
   ttttt  t ttt ttttt t ! t t ttttt ttt
    t ttttt  t ttt ttttt t ! t ttttt  t t  t tt  t 
    t tttttt ttttt  t ttt ttttt t ! ! t ttt t tttt t t ttttt
    t tttttt t ttttt  t ttt ttttt !  tttttttt ttttttttttttttt tt!tttttt tttt tt:ttt!
    t tttttt t ttttt ! t  t ttt tttt tt tt t t tttt tttt tttt ttt tt tttttttt tttt ttttt
   t tttttt t ttttt ! t ttttt  t tt  t ttttt  t t tt tt tt tt :ttt !t !
tttt tttt t ttt tttt ttttt ttttt t t tttt ttttttttttttt  t ttttt  t tt tt t !t tt tt :ttt !
tt tttt tttt tttttt tttt ttt ttt tttt ttttt ttttt
t tttt ttt  t ttttt  t tt tt !t t tt tt :ttt !
ttt tt tt t t tttt tttt tttt ttttttttt tttt tttttttttttt tttt ! t tttttt ttt tttttttttt
ttt tttt ttttt ttttt t tttt t t ttttt
  t tt t ttt ttttt ttttt t  t  t ! 2 tttttt ttttt tttt ! t t t ttt tt tt t 
tt tt t t tttt tttt tttt tttt ttt ttttt t tt tt t t tttt tttt tttt tttt ttt ttttt tt 
 t !t tt t t ttt ttttt ttttt  t   t ttttt t tttt ! t t ttt tt tt
 t !t tt t t ttttt  t ttt  t tttt  t ttttt t tttt ! t t t tt
 t !t tt t t ttttt  t ttt ttttt   t ttttt t tttt ! t tt t t ttt tt 
! t ttttt tttt ttt ttt tt tt tttt ! t ttttt tttt ttt ttt tt tt tttt
t tttt t t ttttt t tttt t t ttttt
   t tt ttt tttt tt t t !t  t ttttt ! 2 tttttttt tttttttttttttt ttt ttt t ttttt! tttttt!
tt tt t t tttt tttt tttt tttttt tttt ttt tttt ttttt ttt tt tt t t tttt tttt tttt tttttt tttt ttt tttt tttt
ttt
  2>2

 t   t t  t t  t ttttt  t ttttt ! t ttttt ttttt t 


! t tt t tttt t t ttttt  t ttttt  t ttttt ! t ttttt t ttttt 
  t ttt ttt tttt ttttt !t t t t tttt ! t ttttt tttt ttt ttt tt tt tttt

t t ttttt tttttt tttt ttt tt tttttt ttt tttttttt ttt t tttt t t ttttt
ttttttttttttt <  t ttttt tt t t  t  t t t tt tt tt
 t tttt  tttt  t t tt tt t t tttt tttt tttt tttttt tttt ttt tttt ttttt ttt
! t tt t tttt t t ttttt ttt
  !  >  !  !   t t  t t  t 
!   ! !        ! t tt t tttt t t ttttt
t tttt tttt ttttttt tttttttttttt tt ttttt tttt tttt 2 tttt tt t t ttt tttttt ttttt t t t t  t  t tttt tt
ttt tttttt ttttt t t ttttt tttt tttttttt ttttttttttt tttt
t t ttttt ttttt tttttt tttt ttt tt tttttt ttt ttttttt
tttt tttt tt ttttt ttttt ttttt ttt tt tt ttttt ttt tttt tttt tt tttt ttttttttttt
ttt ttttttttttttt t t tttttt tttt ttttttttttttt  tttt  t tt  t tttt
   tttt !t tttttt  t  t t tttt tt ! t  t t tttt t t ttttt
    tttt tttt !t tttttt  t  t t tt = tttttttt tttttttttttttt ttttt t tttt! tttttt tt
    tttt !t tttt tttttt  t  t t tt tt tt t t tttt tttt tttt ttt tt tttttttt tttt ttttt
   tttt !t tttt t tttttt  t  t ttt   t t  t tttt ! t ttttt ttt ttt 
    tttt !t tttt t ttt tttttt  t   t t t ttt  t tttt ! t ttttt tt
   tttt !t tttt t ttt  t tttttt   t t t  t ttt tttt ! t ttttt tt
tttt tttt t ttt tttt ttttt ttttt t t tttt tttttttttttttt t t ttt ! t tttttt ttt tttttttttt
ttttt ttttt t tttt t t ttttt
tt tttt tttt tttttt tttt ttt ttt tttt ttttt ttttt
t tttt ttt >  t ttt t tttt ttt tt !t  t tt t !
ttt tt tt t t tttt tttt tttt ttttttttt tttt tttttttttttt tttt
t t ttttt tttttt tttt ttt tt tttttt ttt tttttttt ttt
ttttttt ttttt ttttt ttttttttttttt
  ttttttttt tttttttttttttttt tttttttttttttttttt!  tttt  t tt  t t
tt tt t t tttt tttt tttt tttt ttt ttttt tt ! t ttt t tttt t t ttttt
 t ttttt  t ttttt ! t ttttt t t tttt

  2
  ! >  !  !  !   t tttttt tttt tttt tttttt ttt
       ! t tttttt tttt tttt tt tt ttt
tttt tt tttt tttttttttttt tt ttttt ttt ttttt ttt ttttt t tttt t t ttttt
ttttt t t tttt
t ttt ttttttttt ttttt tttt tttt tt ttttt ttttt tttt 
t t ttttt tttttt tttt ttt tt tttttt ttt tttttttt ttt
tt t tttt tttt ttt ttt ttttttttttttt t t tttttt ttt ttttttttttt
ttttttttttttt  t tt  t ttt  t t
  ttttt ttt t tttttt t ttt =tt t ttttt tttt ttt ttt tt ! t t t tttt t t ttttt
   ttt ttt t t ttttt ttt t tttttt t ttt =tt t ttttt ttt  tt tt t t tttt tttt tttt ttt tttt tttttt tttt tttt ttt
tt tttttt t t ttttt t
   ttt ttt tttttt ttttt ttt t t ttt =tt t ttttt tttt tt  t tt  t tt t  t ttttt
   ttt ttt tttttt ttt ttttt ttt t t ttt =tt t ttttt ttt ! t tttt t tttt t t ttttt
    ttt ttt tttttt ttt ttttt tttt ttt t t ttt =tt t tttt   ! <>     ! 
   ttt ttt tttttt ttt ttttt tttt ttttt ttt t t ttt =tt       
   ttt ttt tttttt ttt ttttt tttt ttttt t ttt ttt t =tt  t tt ttttt ttttt ttttt ttttt ttttt ttttt tttt
tttttttttttttttttttttttt tttttttttttttttttttttttttttt  t tt ttttt ttttt ttttt ttttt ttttt ttttt tttt
ttttttttt tttttttttttt ttt tttttttt  t tt ttttt ttttt ttttt ttttt ttttt ttttt tttt
tt tttt tttt tttttt tttt ttt ttt tttt ttttt ttttt
t tttt ttt  t tt ttttt ttttt ttttt ttttt ttttt ttttt tttt
ttt tt tt t t tttt tttt tttt ttttttttt tttt tttttttttttt ttttt  t tt ttttt ttttt ttttt ttttt ttttt ttttt tttt
ttt tttt ttttt ttttt < tt tt t t tttt tttt tttt ttt ttttt 
  !    t  t  t 
  tttttttttttttttttttt tt ttttttttttt ttttttttttttttt ! t t
 tt tt t t tttt tttt tttt ttt tt tttt ttt tttt tttttt t t ttt 2 tt tt t t tttt tttt tttt ttt ttttt ttt
tttt ttttt ttt ttttttt tttt tttt tttttt tttt t t ttttt t  t   t  t
 t tt tt  t tttt  t ttttt ! t t
! t tttttt t tttt t t ttttt = tt tt t t tttt tttt tttt ttt ttttt t
2 tt tt t t tttt tttt tttt tttt ttt ttttt tt ttt tttt tttt  t  t  t
ttttt ! t t 
 tttttt ttttt ttttt tt ttt ttttt ttttttt tt tt ttt ttt ttttt > tt tt t t tttt tttt tttt ttt ttttt tt
 tttttt ttttt ttttt tt ttt ttttttt ttttt tt tt ttt ttt ttttt  t  t  t
 tttttt ttttt ttttt tt ttt ttttt ttttttt tt tt ttt tttttt tt ! t  t
! tttttt ttttt ttttt tt ttt ttttt ttttttt ttt tt tt tttttt tt   ! 2  !  ! 
tttt t t ttttt !        
2 tttt tttt ttt tttt ttttttttt t t tt ttt ttt ttttt ttt t tttt tttt ttttttt tttttttttttt tt ttttt tttt tttt
 t t ttt tttt tttt tt tt ttt ttt tttttt ttttt t t tttt tttt tttttttt ttttttttttt tttt
 t tttttt tttt tttt tttttt ttt tttt tttt tt ttttt ttttt ttttt ttt tttt tttt ttt ttt tttttttttttt
2>

t t tttttt tttt tttttttttttttt  ttt tttttttt ttt ttttt < tt tt t t tttt tttt tttt tttt ttt tttt ttt
ttttttttt tttt t tttttttt ttttttt  t  t  t
  ttttt t tt t tt tt t ttttt t ttttt ! t tt ! t t 
   tt ttttt t t tt tt t ttttt t ! t ttttt tt 2 tt tt t t tttt tttt tttt tttt ttt tttt t
   tt tt tt t t t ttttt t ! t ttttt ttttt tt  t  t  t
   tt tt tt ttttt t t t t ! t ttttt ttttt tt ! t t 
    tt tt tt ttttt t t t t ! t ttttt ttttt tt   ! =22  !  ! 
   tt tt tt ttttt t ! t t t t t ttttt ttttt tt !        
   tt tt tt ttttt t ! t t t t ttttt ttttt tt t tttt tttt ttttttt tttttttttttt tt ttttt tttt tttt
ttttttttttttttttttttttt tttttttttttttttttt ttttttttt ttt tttttt ttttt t t ttttt tttt tttttttt ttttttttttt tttt
tttttt ttt tttt ttttttttt tttttttttttt ttt tttttttt tttt tttt tt ttttt ttttt ttttt ttt tttt tttt ttt ttt tttttttttttt
  !   *      t t tttttt tttt tttttttttttttt tttttt tttttt tttttttt ttt
  !    ttt ttttt ttt tt =tttt ttt tttttt tttttttt tttt t ttttttt
  ttt ttt!t ttttttttt t tt tttttttt tttttt ttttttt
t t tttt t ttttt   ttttttttttttttttttttttttttt ttt t ttttttt
 tt tt t t tttt tttt tttt ttttt ttt tttt ttt ttt tttt tttt    tttttttttttttttttttttttt ttt ttttttt t 
tttttt     ttttttttttttttttttttttt ttt ttttttt tt 
 tttttt t tttt ttt ttt !t t ttttt  t tt tt t tttt    tttttttttttttttttttttt ttt ttttttt ttt 
t t t ttt ttttt     ttttttttttttttttttttt ttt ttttttt tttt 
 tttttt t tttt ttttt ttt ttt !t t  t tt tt t tttt    tttttttttttttttttttt ttt tttttttt tttt 
t t t ttt ttttt tttt t ttttttt ttttt tttt t t tttt ttttttttttttt t tttt tttt
 tttttt t tttt ttttt ttt ttt !t t  t t t t  tttttt ttt tttt ttttttttt tttttttttttt ttt tttttttt
ttttt ttt tt tt ttttt   ! =>  !   
! ttttttttt ttt!ttttttt t tttttt ttt ttt  !  
t ttt tt tt ttttt   tt tttt  t tttt t !t ttt t !t tttttt tttt
tttt t t ttttt = tt ttt ttttt tttt tttt ttttttttttttttttt ttttttttttt
 tt tt t t tttttt tttttttttttt ttttt ttt t ttt  ttt ttttt
tttttt tttt t ttt tttt tttt ttt tttt ttttt  t tttttttttt ttttt tttt t!t ttttt!t t
 tt   t   tt ttt  t tttttttttt ttttt tttt!t t tttttt t!
! t  t tttt t t ttttt  ttt tttttttt ttt tttttt t!t tttttt t!
2 tt tt t t tttt tttt tttt ttttt ttt tttt ttttt ttttt ttt ttt ! t tttttttttt ttttt tttt t!t tttttt t!
ttttt ttt tttttttt ttt ttttt t!t ttttttt t!
 tttttt t tttt ttttt ttt ttt t  t !t t t t  >
t t ttttt tttttt ttttt ttt ttttttt ttt tttttttt ttt
ttttt ttt tt tt ttttt tttttttttttt ttt tttt ttttt ttttt
 tttttt t tttt ttttt ttt ttt t !t  t t t t   t t  t   t t
ttttt ttt tt tt ttttt ! t tt t tttt t t ttttt
 tttttt t tttt ttttt ttt ttt t  t !t t t t    ! 222  !   
ttt ttttt tt tt ttttt  !  
! tttttt t tttt ttttt ttt ttt t  t !t t t t    t t ttttt ttt t tt t ttt !t ttttt tttttt t 
ttttt ttt tt tt ttttt tttttt 
tttt t t ttttt 2  tt t tttt tttt ttttttt ttt tt tt ttttt t t ttt
2
t t ttttt ttttt ttttt ttt ttttttt ttt tttttttt ttt tttttttttttt tttttt ttttt ttttt t ttt tt t ttt !t 
tttttttttttt ttt tttt ttttt ttttt tttttt t t t  
 t tt  t ttt  t   t tttt ttt  ttttt   t tttt t
! t t t tttt t t ttttt ! t tttt t t tttt t t ttttt
 tt t tttt tttt ttttttt ttt tt tt ttttt t t ttt 2 tt tt t t tttt tttttttttttt tttt t ttttt ttt ttt ttt
ttttttttttttt t ttt ttttttttt  tttt tttt tttttt ttt t ttt tttt t t t ttt
ttttttt tttttttttttt ttt t!tt tt tttttt ttttt
ttt tt tt ttttt  t ttttt  t !  t 
 t tttt ttt  ttttt   t tttt t ! t ttttt t tttt t t ttttt
! t tttt t t tttt t t ttttt 22  tt tt t t tttt tttt tttt ttttt ttt tttt ttttt ttttt ttt ttt
  ! 2    !   ttttt
    ! !  !     tttttt ttttt ttttt ttt tt t ttt tttt ttt t t !t 
 !        t t  
   !          tttttt ttttt ttttt ttt tt t tttttt ttt  t t t 
  !       !  !t t 
 t ttttttt ttt  t !!t t t tttttt ttt t  ttt tttttt tt t ttt ttttt ttttt tttttt  t t t 
 t ttttttt ttt t !!t t t  t tttttt ttt t !t t 
 t ttttttt  t !!t t ttt t tttttt ttt t ! tttttt ttttt ttttt ttt tt t tttttt ttt t t !t 
! t ttttttt ttt t !!t  t t tttttt ttt t t t  
t ttttttt ttt t  t !!t t tttttt ttt t ttt tttttt tt t ttt ttttt ttttt tttttt t t !t 
 tt tt t t tttt tttt tttt tttt ttt tttt tt t t  
 t  t  t
! t t 
  2>

   
  
 t ttt tt ttttt ttttttttt ttt tttt t *  t tttttttttttttttttttttttttt tttttttttt
    2  t t tttt tttt t ttt tttt tttt ttttttttt t t tttttttt tttt
ttttt tttttt ttt tt <   t tttt t tttt t tttt t tttt ttttt tttt ttt
2   2 t ttt ttttttt t tt ttttt ttttt ttttt tttttttttttttttttttttttttt!tttt ttttt ttttt
ttt tt tttttttttttttttttt tttttttttttt
2   t ttttt tttt ttttt ttttt ttttt ttttt   tttttttttttttttt tttttttt tttttttt!
ttt tt 2    ttttt ttttttttttttttttttttttttt
   t ttt ttttttt ttttt tttt ttt ttt ttttttttttttttt ttttt ttttttttttttt!tttt
t tttttt tttttttttt ttt tttttt ttttt tttt tttttt tttt   tttttt!ttttttttt tttttttttttt
tttt tttttt ttttt tttt tt tttt tttttttttttttttttttttttttttttttttttttttt t

tttt ttt
t tttt ttt tttt ttt tttttttt
t tt =    t ttt tttt ttt tttt tttt ttt ttt
  <tttttttttttttt tt ttttttttttttt tttttttt!ttttt tttttttttttttt
<  2tttttttt tttttttt  tttttttttttttttttt!tttttt tttt tttt
ttt ttt tt t ttttt tttt :ttt ttttt ttt tttttt ttt tt tttttttttttttttttttt t ttttt ttttttt ttt
tttttttt ttttttttttttttt tt ttttttt tttttttt t ttt >  ttt ttttt ttt ttttt tttt
ttt tttt ttttt ttt tttt ttttt ttt tt ttt
t tttt tt 
 
ttttt tt 
t ttttt tt !
t ttt tt
t tttttt tt t ttt
t ttt 2 t
tttt tttt ttttt tttt ttt tt
tttttt tt tttt ttt ttttt tttt t ttttt tttttttt ttt tttt ttttt tttt
   t t  t!t t tttt tttt ttttt tttt tt tttt ttttt ttt tt ttttt tttt
2   t t t !t t ttt tt ttttt tt tttt tttt tttttt ttttt tttttt t
2   t t !t t  t tttt tt ttttttttt tttt tttt tttttttttt ttttt t t ttt
  t t !t t t  ttttttttttt tttt tttt tttttttttt t tttttttt ttttt
  !t t tt  t  tttt tttt ttttt
t tttttt tttt t ttt tt tttt tt tt

<  !t t t t t  ttttttt tttt ttttttt ttttt ttttt


2  ! t t t t t  ttt ttttttt t
t ttttttttttttttt tttttttttttttt
tt ttt tt t ttt ttt tttt ttttttttt ttttttt tt tttttttt ttttttt
ttttttt
ttttt
t ttttt!
ttttttt
ttttttttt tttt ttttt ttttt
t tttt t tttt
t ttt
  2 t tttt ttttt ttt ttt t tttttt ttt !  tttttttttttttttttt!tttt ttt ttttttttt
tttttt!t ttttttttt tttttt ttttt !  tttttttttttttttttttt ttttttt!ttttt
2  t ttttttt tttt!t !  ttt ttttttttttttt!tttttttttttttttt
tttttttttttttt tt tttt !   ttt ttttt tttttttttttttttttt!tttttt
2   < t tttt tttt ttttt tttt ttt ttt !  tttttttt ttttt ttt!ttttttttttttttttt
tttttt!t  ttt t tt tt ttt !  ttttttttttt tttttttttttt ttt!ttttt
tt tttt tttt ttttt ttt
t tttt ttttt tttt tttt ttt   !  
ttt ttttt ttttt ttt t t !t t t t tt tt ttttttt tt tttttt tt tttt tttt tttt ttttt tttt ttt tt
tttt ttt ttt tt tttt ttttt  ttt tttttttttttttttttttt!ttttttttttttttttt
2 !  t ttt tttt ttttt tttt tttttt tttt !  
ttttt tttt ttttttt!tttttttttttttttt ttttttttttttt tttttttttttttttttt ttttt!ttttt
2 ttt!ttttttttttttttt tttttt ttttt tttt ttttt ttt t ttt tt tttt tttttt tttt ttt
t t ttttt ttttt tttttt tttt ttt ttt 2 ! !  
  tttt ttttt !ttt ttt tt ttt tttt ttttttt tttttt t t ttt ttttt ttt =ttt ttttttt tttt tt ttttt ttt
ttttt ttt t tttttttttt tttttt tttttttttttt!ttttttttttttttttttttttttttttttttttt
   tttttttttttttttttttttttttt ttttttt !  
tttttt!ttttt ttttt ttttttttttttttt =ttt tt tttt ttttttt ttttt tttt ttttt tt
  tttttttttttttttttt!ttttt tttttt ttttttt tttttttttttttt!tttttttttttttttttt
tttttt ttttttttttttttttttttttttt 2  !   
< 
tttt tttt ttttt tttt ttt tt ttt ttttt ttttt ttttt ttttt t ttt ttttt ttt
ttt tt tt ttttt tttt tttttt tttt tt ttttt ttttt ttt tttt tttttttttttttttttttttttttt!ttttt
tttt tttt ttttt tttt tt ttttt tttttt tttt tttt ttt !  
tttttt ttttt ttt tt tttt ttt tt ttttt ttttt t ttt tttttttttttttttttttttt ttttttttt!tttttt
tttt ttttt
t tt tt ttt ttt t tt ttt tt ttttt tttt ttttttttttttt ttttttttttttttt tttttt ttt
ttt tttttt ttt ttttt ttttttt tttttt
t tttt ttt ttttttttt    
tttt ttttt tttttttttt tttt t ttt t tttt tt ttt <t 2t
     !  =2 t
tttt tttt ttttt tttt ttt tt
   !     ttt tt tt ttttt tttt t tt
t tttttt tttt ttttt tttt
    !    tt tttt tttt tttt
t tt tttt tttt tttttt ttttt ttttt
    !    tttttt tttt tttttttt ttt tttt tttttttt tttttttt
      !   t tt
t ttt tttt tttt ttttttt ttttt ttt tt ttt
      !  ttttttttt tt t tttt tttt tttt tt tttt tttt tt tttt
       ! tttt tttt
t ttt ttttttt ttt tttt ttttt t t ttttttt t

2><

t t tt ttttttt tttttt t ttttt ttt tttttt tttt


t t= !      
ttttttt
t ttttt
t ttttt
t ttttt!
t tttt
t t ttt
ttttttt ttttt
tttttBtttttt
ttttttttttttt tttttttttt tttt t !      
!    2    !   t tttttttt t tt
!   2      !    !       
!     2    !     tttttttttt
!   2  2    !     !      
!  2       !    ttttttt!ttt
!           !   !      
!     <      !   !ttttttt ttt
!   <  2      !   2 ! ttttttt tttt tttttttttt tttttttttttttttttt
=  !   tttt tttt ttt tttt tt tt 
ttt ttt tttt ttttt ttt tttt ttttt t tt  
tttttttttttttt!ttttttt ttttt ttttt tttttt t ttttttt t tttt t tttttttt t ttt t tttt t tt tt
 ttt tttttttttttttttttttttt tttttttttttttttt
ttttt ttt tttt ttttt ttt ttt tttt tt !  
tt ttttttt ttttt!tttttttttttttttt ttt ttttttttttttttt ttttttt ttttttt tt tttttttt
>t! 2t  2t  tt ttt t ttttttt tttttttt tttttt tttt t t ttt t t ttt
22t 22t  2  2 22 
22> t ttt tt tt ttttt tttt ttttt tttt tt ttttt tttt ttttt 22 
tttt tttttt tt ttttt tttt tt ttt tttt tt ttt tt tttt 22= 
ttt
t tttttt tttt ttttt ttttt ttt t tt tttttt t ttt
ttttttt tttt ttttttt tt ttttt t ttt tttt t tt tttt ttttt ttt tttt tttttt tttt ttttt tt ttttttt tt t
tt ttttttt tttttttttttttt tttttt ttttttttt tttt ttt tttttt tt t
ttttt ttttt tttttttt tttttttttt t ttt
ttt tttttt tttt ttttt tttt tttttt tttt ttt tt tt tttt ttttt ttt t ttttttttt tttt tttt t ttt ttt t ttt
ttttttt t
t ttttt tttt tttttt tttttt ttttt ttt tttttttttttttttttttttttttttt tttt
tttttttt t tttt
ttttt ttttt tttt ttttttttt tttttt tttt ttt tttt tttttttttt tttttt t tt
t tttttttt tt
t tt tttt
!   >    ttt!t t t ttttttt tttt tttttttttttt tttt ttttttt ttttt ttttt
!       t tt!ttt ttt ttttttt ttt ttt
t tttttt t tt tttttt tttt tttttttt
!     2  ttt!t tt tttttttttttt ttt tttt tttt tttt tt
!   2     t t t!ttt   ttttttt!tt tt tttt
!  2   2   ttt!tt t tttt!   tttt ttttttttt!
!   2      tt t!ttt tttt!   tttt tttttttt tt!
!         ttt!t t t tttt!   tttttttt tt tt!tt
 tt tttt ttttt ttt ttttt ttt ttttt t t !  t tttt tt t t ttt !    tt tttt tttt!tttt
tttttt tttt tttt t tttt ttttt tttttt t tttttt tt ttt!   tttt!tt tttttttt
!      <   t tt!ttt !ttttt !   tttt!tttttttttt
!  ! <   2   ttt!t tt tt tt tt ttt tttt ttttt ttt tt ttt tt ttttttt tttt
ttt
!  ! 2   =   t t t!ttt ttttttttttt tttt ttttttt ttt
tttt ttttttttttttt
2  2 * ttttt tttt tt!tttttttttttttt ttttttttttttttttttttttttttt tttttttt ttttttttt
tttt tttt ttttt ttt ttttt tttt tttt ttt t tt
2  tttttttttttttttt ttt!ttttttttttt 2  t ttt ttt tttt ttttttt ttt tttt tt tttt ttt tt ttt
t t t t t t t t t ttttt ttttt tttt ttt tttt tttt tttt t ttttt ttt tttt ttttttt ttt tttt tttttt ttt t
2< ! 22t 
tt t
t tttt tttt tttt tttt ttt ttt tttt t
2=t  2>t   t ttt ttt ttt tt ttttt ttt tttt ttttt
222 
tttt ttt ttt tt tttt t t ttt ttttt ttt ttt ttt tt tttt t ttttttttttttttttt ttttttttttttttt
t ttttttt ttt tttt
t ttt tttt tttt t tttttttt t t t t tttttttttt t t ttttttt t
ttttt ttttttt tttttttttttttttt!
tttt tttttttt ttttt tttt tttttt tttt tt tt ttt tt 2 
tt tt tttt ttt ttttt ttt tttttt ttt tt tt tt tttt ttt
tttttttttttttttt ttttttttttttt
tttt tttttt tttt tttt tt tt tt ttt tt tt t tttt tt t tttt t ttt
tttt ttttt ttt tt tt ttt ttt tt tt ttt tt ttttt t ttttttt tt t tt
ttt
t tttt ttttttt tttt tt tttt ttt tt tttt
t tt t tttt ttttt! tt
tttt ttt tt tt tt tttt tt tt t ttt tt tt tt tttt tt 2 !
ttt
t tt ttt t ttttt t ttt ttt tttt ttttttt ttt ttt
tttt tttt t tttttt tt
tt t
tttttttttt tttt tttttttttt tttt t
tttt ttt tttt ttttttt tttt tttttt ttt tt tt tt ttt !   tttt t t t tttttt t ttttt ttttt
tttt tt tt tt ttt tt tt tt ttttttttttttt tttttttttttttt
t t t tttttt tt tttttt!t t t ttttttttttttttttttttttttttttttttttt
tt tt tt tt tttt ttttt tt t! ttttttttttttttttttttttttttttt tttttt!
! ttt t t tt tt tt tt ttttt!t t t t!  t ttttt t t ttt ttttttttt tttttt
tt tt ttt tt ttttt ttt tttt tt t t t t t t ! ttttttttttttt!tt ttttttttttt
ttttt tttt ttttt t ttttttt tttt
t tttt ttt ttttt t t t t tt ttttt tttttt t ttttt
tttt tttttttttttttt
   !  2<  t ttt tttt t ttt tttt tttt tttttttt ttt ttt
  !   t ttt tttt tttt tttttt ttt tttt ttttt tttttt
tt t

!       tttt ttttt tttt ttt tttt tttttt ttt tt tttt ttt tt
tttttttttt tttt t
  2>2

!  t t ttt t ttt t ttttt t tttt t ttttt t tttt t  t ttttttt ttttttttt ttttttttt tt
ttttttttttttttttttttttt!tttt ttttttttttt tttttttttttttttttttttttttttttttttttttttttttttttt!
ttttttttttttttt ttttttttttttttttt tttttttttttttttttttttttttttttttttttttttttt
tttttttttttttttttttttttttttttttt ttttttttttttttttt ttttttttt tttttttttttttttttt
t !   t t ttt t tttt t ttttt ttttttttt ttt ttttt tttt 2=  !  t ttttttt ttt ttttt ttt tt
tttttt tttttttttttttttt ttttttttttt ttttttt
22  ttt ttttttt ttt tttt tttttt t tt t ttttt t ttt tttttttttttttttttttttttttttttttttttttt
tttt ttt tt tt t tt t
tt t
t tttt tttttt tttt tt tttttttttttttttttttttttttttttttttttt!
ttt tttt t
tt t
t tttt tttttt ttt tttt ttttttt tttttt tttt ttt tt
!   t tttttt ttt ttt tttttttttt tttttttt tttt t
ttttttttttttttttttttt tttttttttttt ttttttttttt  ttt ttttttttttt tttttttttttttttt tttttttttt
tttttttttttttttttt ttttttttttttttttttttttt tttttttttttttttttttttt tttttttttt tttttttttt!
ttttttttttttttttttttt!ttttttttttttttttttt ttttttttttttttttttttttttttttttttt tt
tttttttttttttttttttttttttttttttttttt

  2
2 t tttt tttt ttttt ttttt ttt ttt ttttt ttttt tttttt tttt t t >   t ttttt ttttt tttttt t t  t t t tttt t !
tttttttttttt ttttttt t t tttttttt ttt ttt ttt tttttt   t t ttttt ttttt tttttt t  t t t tttt !
tttttt tttt tttt tttttt ttt tt ttttttt ttttt   t t ttttttttt ttttt tttttt  t t t t!
tttt tt ttt tttttttt tttttt ttt ttttt t
t tt tttt tttt   t t tttt ! t ttttt ttttt tttttt t 
tttt tttttttt ttt tt ttttt tttt ttttt tttt ttttttttt    t t tttt ! t ttttt t ttttt tttttt 
tttttttt ttt tt t ttttt tttt ttt tttt tt tt ttttt ttt   t t t tttt ! t ttttt t ttttt  t ttttt
ttttttttt ttt tt ttttt tttt tt tttt tttt tttt tttt   t ttt ttttttttttttt t t!tttttt!
ttt tt ttttt tttt ttttt t tt tttttt ttttttt   t t tt tttt ttt tttttt  t !t ttttt !
tttttttttttt ttttttt tttt t tt tttttttttt ttttttt ttt   t t tt tttt !t ttt tttttt  t ttttt !
ttttt   t t tt tttt !t ttt  t tttttt ttttt !
    t  t !t t  t !t t t      t t tt tttt !t ttt  t tttttt !t tttt
  t  t !t !t t  t t t  
tt tt ttttt tttt tttt ttt tttt ttttt tttt ttt
   t  t !t !t t t  t t    ! tt tttt ttt tttt tt t ttt
2  !  t  t t t  t ! t  2 !   t !t ttt t  t t ttt ttttt tttt
  t ! t  t t t  t     t !t ttt t ttttt  t t t ttt tttt
  t ! t t  t t t     t !t ttt t ttttt  t ttttt t tt
  t ! t t  t  t t  2    t  t tt t t t ttttt tttt t tt
2   t  t t  t ! t  t t     t  t t tt t t tttt t ttttt t tt
  t ! t  t t  t  t t    t  t tt t t tttt t t ttttt tt
  t ! t  t  t t  t  t    t  t tt t t tttt t ttttt t tt
  t ! t t  t  t t t 
tt tt t tt t 't t ttttt tttttt tttt ttt tt ttttt
   t ! t  t  t  t  t t       t t ttttt ! t tttt !t t ttt tt
  t ! t  t  t  t  t t    t t ttttt ! t tttt t !t ttt tt
  tttt ttttt ttt ttt ttt tt tttt ttt   t t ttttt ! t tttt t tt t !t tt
    t  t  t t ! t t t ! tt tt tttt ttttt ttt tttttttt tttttttt
t tttttt tttt tt
  t  t t  t ! t  t ! ttt ttttt tt
    t ! t    t ! 2 t
tttt tttt ttttt tttt ttt ttt tttttttt tttt ttttttt
     t ! t ! t  t t ttttttttt ttt ttt ttttttt ttttt
<            ttt tttttttt t t tttt ttttt tttttttt ttttt tttttt
          ttttttt tt tttttttt ttttttttttttttt ttttttttttttttt
           ttttttt ttt tttttttt tttttttt
t tttt tt tttt ttttttt
2 ! ttttttt tttttt tttt ttt tttttttttt ttt ttttt ttttttt tttt tttt ttttttt tt tt ttt ttt
=  ttt tttt ttttt tttt
t tttt tttttttt ttttttt ttttt ttt ttt tttttttt tt
t tttt ttt ttt tttttt tttt tttttttt ttt
tt tttttt tttttttt
t tttttttt tttt ttttt t t tttt tttt ttttttttt ttt ttt ttttttt ttttt
ttttt ttttttt tttt tt tttttt
t tttt tttt ttttt tttt     t!ttt! tt tt tt tttt 
ttttt ttt tttt ttttttttt ttt tttttt t tttt tt ttt   t!tt! tt tt tt tttt 
tt tttt ttttttttt tttt tttt tt t
t tttttttt tttt tttt    t!tt! tt tt tt tttt 
t t tttt ttttt ttttt tttt tttt tttt tttttttt ttt   tt!tt! tt tt tttt tt 
ttttt tttt ttttttt tt t ttt ttttt ttt ttttt t ttt ttt < 
ttttttttttttttt ttt ttt ttttttt ttttttttttttt ttt 2 !  ttt tttt !tt tt t! tt t t
tttt ttttt ttt tttt ttttttttt ttt ttttt   tt! ttttt!tt tttt tt
= !  ttttttt=tt ttttttttttttttt ttttttttt   t! tttt!tt tttt tt
  tttttttt=tt ttttttttttttttt ttttttt   t! tttt tt!tttt tt
  t t tttttt tttttt=tt  t t t t ttttttt tttt =    t! ttt ttt ttt ttt tttttt 
   t! ttttt ttt ttt tt ttt 
  t t tttttt  t tttttt=tt  t t t tttttt   t! ttttt tt tt ttt ttt 
tttt    t! ttttt tt tt tt ttt
   tttttttt ttttttt=tt ttttttttttttt > ! ttttttt ttttt tttt ttt tttttttttt
2>=

2    t  t t  !t t t t  t t !t t  t t !  tttt ttttt t t tttttttttttt ttt tttttttttt tt ttt


  t! tt !tttt tt!tt tt  tttttt tttttt tttttt ttt tttt tttttt tttt ttttttttttt
  t! tt tttt tt!tt !tt  ttt ttt ttttt ttt ttttt tt tttttttttttt t tt ttttttttt
2  ttt ttttttt tt tt ttt ttt tttt tttt ttt ttttttt ttttt ttttttttt ttttttt tttt ttt ttttt
tttt ttt tttt ttt tt 222< t ttt ttttt tttt ttttttttt tttt ttt tttt ttt tttt ttttt
222< t ttt ttttt tttt ttttttttt t ttttttt ttt tttt tttttt t t tttt ttt t t ttttttt
t tttt tttt tttt tttttt ttt t
t ttttttttt tttttttt
t ttttttt ttttttt ttt tttt ttt ttttt ttttt ttttttttt ttttt t t tttt ttt t t tttttttt tt
tttt tt ttttt ttttttt t t tttttttt ttt tttt ttttt t ttt t tttt
t tttt tttt ttttt ttttt ttttt ttt ttt
t tt ttttt tttttttt ttttt ttttttt t ttttttt
t tt tt tt tttttttt ttttt ttttttttt ttttt ttttt tttt ttt
ttt ttttttttt ttt ttttttttttt ttttt tttttttt
t tttt ttttttt ttt tt ttttt tttt
22    t tttt ttt ttt ttt ttt tttt tt 22    t ttt ttt ttt ttttt ttttt tttt tttttt ttt
  t ttt tttt ttt ttt ttt tttt tt   t ttt ttt tttt ttt ttttt ttttt tttt ttttt
  t ttt tttt tttt ttt ttt ttt tt    t ttt ttt tttt ttt ttttt ttttt tttt ttttt
  t ttt tttt ttttttt ttt ttt tt 22 ! ttttttt ttttt tttt ttt tttttttttt
22    t ttttt ttttttt tttt t tttttt ttt tt ttttt 2   t tt tt t t tt tttttt tt tt ttt ttt
tttt   t tt tt tt t t tttttt tt tt ttt ttt
  t ttttt ttttttt tttt t ttt tt tttttt ttttt   t tt ttt tt tt t t tttttt tt tt ttt
tttt   t tt ttt t t tt tt tttttt tt tt ttt
   t tttttttttttt tttt tttt tttttttttttt 2
ttttt 2<    t t t tttttt ttt ttt ttttt ttt
t
t ttttt tt ttttt ttt tttt ttttt tttt   t t t ttt tttttt ttt ttttt ttt
2   t ttttt ttt tttt ttt ttttt ttt tttt   t t t ttt tttt tttttt ttt tttt
  t ttttt ttttt ttt tttt ttt ttttt tt 22 t
tttt tttt ttttt tttt ttt tt
  t tttttttttt ttttt ttt ttttttt tt ttttt tttt ttttttttt t ttttttt ttt tttttt ttt t
  tttttt tttttttttt tttt ttt ttttt tttttttt ttttt tttt tttttttt ttt t t tttttttt ttt
   t ttttt ttttt ttttt tttt ttt ttt tt ttttttttt ttttt ttt t tt ttttt tttt t tttt ttt t
2 !  t ttttt ttttt tttt ttttt tttttt t ttt tttt tttttttt ttttt tttt t tttt tttttt t t ttttttt tt ttttttt

  t ttttt ttttt ttttt tttt tttttt t ttt tttt tt t tttt tttt tt tt ttttt ttttt ttt tttttt
  t ttttt ttttt ttttt tttt tttttt t ttt tttt tt tttttttt ttt ttttttttt ttttt ttt tt tt ttttt tttt
  t ttttt ttttt tttttt ttttt tttt t ttt tttt tttt tttt ttt ttttttttt tttt tttt ttttt ttt tt ttt
   t ttttt ttttt ttttttt t ttt ttttt tttt tttt ttttttttt ttttt ttttt ttt ttt tttt ttttt ttt tttttt
  t ttttt ttttt tttttt t ttt tttt ttttt tttt tttt tttt ttttt tttt tttttttt
  t ttttt ttttt tttttt t ttt tttt ttttt tttt 22 
2<  tt tttt tttt tt t ttt 2= ! ttttttt ttttt tttt ttt tttttttttt
222  ttttttttttttttttttttt tttttt tt ttttttttttt tttt 2> t  t
tttt tttttttt ttttttttttttt tttttttttttt tttt ttt 2< t tttt tttt ttttt ttttt ttt ttt ttttt ttttt ttttt ttt
tttttt ttttttt t t tttttttt ttt ttt ttt tttt tt ttttt ttttttttt ttt ttttttttt ttt tttttt tttt tttt ttt
ttttttttt tttt ttt ttt tttt tttttt ttttttt tttttt ttt tttttttttttttt tttttttttttttttttttttttttttttttttt
ttttt tttttttt ttt tttt t t t tt ttttt tttt tttt ttttt ttttttt ttttt tttt ttt
t tt tt ttttt ttttt tt
tttttttt tttttttt
t tttttttttttttttttttttttttttttt tt ttttttt
t ttttt ttt tttt ttttt ttt tt tttt tttt tttt
tt tttttttt tttttt ttt tttttttt ttttttttt ttttt ttt ttt tt tttt ttt tt ttttt ttttt ttt tttt tt tt ttttt ttt
ttttt ttttttt tttt tttt tttttttttt ttttt ttt t tttttttt ttt ttttt ttt tttt tt tttt ttt tt tttt ttt
ttt t ttttt ttttt ttt tt tttt ttt tt ttttt ttttt ttttt t tt ttttt
22 !  t ttt t tttt ttt  t t ttt t ttt tttttttttttt ttttttt tttt ttttt ttttt ttt tttttttt
  tttt tttttttt t tttt 2t  2t!  
  tttt ttttttttt tttt t <t
  tttt ttttttttt tttt 2 tttttttttttttttttttttt tttttt tt ttttttttttt tttt
    tttt tttt ttttttttt tttt tttttttt tttt ttttttttt tttttttttttt ttttt ttt
  t ttt t ttt t tttttttt  ttttttttt ttttttttt ttttt ttttttt tttttttt ttt
  tttt  tttt ttttt tttt ttttttt tt ttt ttt ttttttt tttt tt tttt ttt
2=   t tt !t tttt ttt t tttt  tttttttttttt t t tttt ttttttttt ttttt ttttttttt ttt t
  t ttt !t tttt tt t tttt  ttttt ttttt ttttt ttttt ttttt ttt tttttt ttttt tttt
  t ttt t tttt tt !t tttt  2   t tt ttttt ttttt ttt  t tttttttt  t t !
  t tttt tt tttt !ttttt    t tttttttt tt ttttt ttttt ttt  t  t t t !
   t ttt t tt  t !t tttt ttt   t tttttttt  t tt ttttt ttttt ttt  t t t !
2> ! ttttttt ttttt tttt ttt tttttttttt =   t tt t ttt t t t t t tt tt tt t 
2   t ttttt tttt  t  !t tttt  t tttt   t ttt t tt t t t t t tt tt tt t 
  t tttt tttt  t  !t ttttt  t tttt   tttt t t ttt t t t ttt tt tt t
  t tttt  t tttt  !t ttttt  t tttt   t ttt t tt t t t tt tt tt t 
  t tttt  t tttt  t ttttt  !t tttt    t ttt t t t tt t t tt tt t tt t 
2   t tttt  t  t tttt !t tttt !t tttt >   t tttttt t tt t!t t tttttttt 
  t tttt  t tttt  t !t tttt !t tttt   tttttttttttttt!tttttttttttt 
   t tttt  t tttt !t !t tttt  t tttt   ttttt ttttttttt!ttttttttt
  t tttt  t tttt !t tttt !t  t tttt   tttt tttttttttttttt!tttt
  t tttt  t tttt !t tttt !t ttttt      ttttt ttttttt!ttttttttttttt
*  t ttt tttt ttttttt ttttt ttt tttt ttttttttttt
  tttt ttttttt!ttttttttttttt
  2>>

    t tttttt t ttttt t  t ttttttttt ttttt    t ttttttt ttttttttttttttttttttttt!
   t tttttt t ttttt t t  t ttttttttt tttt < ! tt tt tttttt tt ttttt t ttttttttttt
t tt tttt tttttt
  t tttttt t ttttt t ttttt t t  t tttttttt tttttttt tttttt tttt tttttttt
  t tttttt t ttttt t ttttt  t t t tttttttt <   t !t t tttttt ttttt ttttttt t t tttt
  t tttttt t ttttt t ttttt  t ttttttttt    t t !t t tttttt ttttt ttttttt t tttt
tttt ttt ttt tttt ttttt ttttt
tt t
t ttttt tt ttt ttt   t tttttttt!tttttttttttttt tttt
tt tttttttt ttttt tttttttttttt tttt    t ttttttttttt!tttttttttttttt tttt
 ! ttttttt tttttt tttt ttt tttttttttt <2<<  tttttttttttttttttttttttttttttttttttttttttttttttt
2< ttttttttttttttttttttttttttttt ttttttttttttttt ttt ttt tttttttttttttttttt ttttt ttttttttttttttttttttt
ttt ttt tttt t tttttt ttt ttttt tttt tttttt tttt ttt ttt tt ttttttttttt t t ttttt tttt tttttttt tttt
tttttt ttt tttttttt tttt t ttt tttt tttt tt ttt tt tttt ttttttttt t ttttttt ttt ttttttt tttttt t
ttttt tttt ttt t ttttttt tttt ttt t ttttttt tt ttttttttt ttt tttttttttttt ttttttt tttttttt ttt
ttttttt ttttttttt ttt tt ttttttt ttttt
ttttt t tttt ttt tt ttt ttttttttt ttt tttttt tttt ttt ttttt t
t tttt t tttttt tttt ttttt ttt tt tttt ttt
tttttttt t tttt ttt tt ttt ttt ttttttt ttttt tt tt tttt tttt tttt tttttt ttttttttt tttt ttttttt ttt
ttttttttt ttttttttttttttttttttttttttttttttttt ttt tt ttttt ttt
ttttttttt ttt tt tttt ttt tt ttttt ttt tttttttt
ttttttttttt
t tt tttttttttttt ttttttt t
2 ! tt tttttttttt ttttt t tttttttttt
tttttttttttttt tttt tttt tt tt tttt tttt tttt tttttt ttttttttt ttt
tttt ttt ttttttttt tttt tttttttt ttttttt tttt ttt tt ttttt ttt
2 t  t tttttt ttttt !t t  t ttttt t tttt tttt tttttttttt tttttttttt t t ttttt tttt ttttttt
  t tttttt ttttt ttttt !t t  t t tttt ttttttt ttttttttt ttttt tttttttt tt tt tttttttt ttt tt
  t tttttt ttttt t ttttt !t  t t tttt tttt tttt tt tt tttt t tttttt ttt ttttt tttt tttt ttt
  t tttttt ttttt t !t ttttt  t t tttt ttt ttt tttt t tttttt ttt ttttttt
   ttttttttttttt!ttttttttttt t <2 ! tt tt ttt ttt tt tttt t t ttttttttttt
t ttttt ttt
  t tttttt ttttt t !t ttttt ttttt t  tttttttt tttttt ttt tttttt tttt ttt ttttttttttt ttt
tt tt tttt tttt ttttttttt t t tttttt tttt ttttttttt tt tttttttt
tttt 
t ttt ttt tttt ttttt tttt <2    t tttttt ttttt ttttt  tt t tttt
   tttt tt!t ttttt ttt tttttttttt ttt    t ttttt t ttttt t ttttt  t t tttt
  ttttt ttttt tt!t tt tttttttttt ttt   ttttttttttttttttttttttt ttt
  ttttt ttttttttttt tt!t tt tttt ttt   tttttttttttttttttt tttttt
  t tttt tt tttttt t ttt tt !t t 
tt t
t ttttt tt ttt tttt ttttt ttttt tttt ttt
t ttttt
ttt ttt ttttttttttttttttt ttttttttt tttttttttttttt ttt t
    t ttttt tttttt t t !t t ttttttt tttt < t !  t ttt tt t t t t ttt t t tt
  t ttttt tttttt t t t !t ttttttt tttt   ttt tttt tttt ttt t tttt
   t ttttt tttttt t !t t t ttttttt tttt   t tt t t ttt t t ttt t tt
  t ttttt tttttt t !t ttttttt t t tttt   ttt t ttttttt ttt ttt 
  t ttttt tttttt t !t ttttttt ttttt t     t tt t t ttt t ttt t ttt 
  t ttttt tttttt t !t ttttttt ttttt t    t tt t t ttt t ttt ttt t t 
<    t tttttt ttttt t ttttt  t t ttttt    t tt t t ttt t ttt t ttt 
   t tttttt ttttt t  t ttttt  t t tttt
tt t
ttttttttttttttttttttttttt ttttttttttttttttt
  t tttttt ttttt t  t ttttt ttttt  t  <   t ttttt t ttt tttt  t t t t  t t tttttt t 
  t tttttt ttttt t  t ttttt ttttt t    t tttttt ttttt ttt tttt  tt t t  t t t 
tt tttttttttttttttttttttttttttt ttttttttttttttttt   t tttttt  t ttttt ttt tttt  t t t t 
t t ttttt tttt tttt ttttttttt ttt ttttt t
t ttttt t ttt ttt   t tttttt  t t ttt ttttt tttt  t t t 
tt tttttt ttttt tttttttttttt ttttt ttt ttttt tttt ttt    t tttttt  t ttt  t ttttt tttt  t t 
2< t
tttt ttt ttt tttt t t tttttttttttt ttt tttt t ttt ttt <<   t ttttt ttttt  t t t t tttttt ttt
tttttttttttttttttttttttttttttt ttttttttt tttttt   t tttt ttttt ttttt  t t t t ttttt
tttt tttttttt tttt ttttttttt ttt ttt ttttttt ttttt   t ttttt tttttttttt t ttttttt
ttttttt t tttt tttt ttt ttt ttt tttt ttttt ttttt   t ttttt ttttttttttt t tttttt
ttttt ttttt t tttt tttt tttttttttt tttt ttt     ttttt ttttttttttt t tttttt
tttttttt ttttt ttttttttt tttt tttttttt tttttttt   t ttttt ttttttttttt ttttttt
t ttttttt ttttt ttttttttt ttt ttt tttt ttt tt tt tttt
tt t
t ttttt t ttt tttt ttttt ttttt tttt tttt ttt
ttttt tttttttttttt ttttt ttttt tttt ttt t ttt ttttt t
2   t t t tttttt tttt t !t tttt  t ttt <22 ttttttttttttttttttttttttt tttttttttt tttt ttttttttt
  t  t t t tttttt tttt t !t tttt ttt tttt ttttt t t tttt tttttt ttt ttt ttttttt tt ttttttt
  t  t tttttt t t tttt t !t tttt ttt ttttt
ttttttttttttttttttttttttt ttttttttttttt ttt

  t  t tttttt !t t t tttt t tttt ttt ttttt tttt ttttt ttt
t ttttt tt tttt ttttt tttt ttt
   t  t tttttt !t tttt t t tttt t ttt ttt tt
=    ttttttt!tttttttt t ttttttt ttttt <2   t ttt ttt ttttt tt tt ttttttt tt
  ttttttt!t ttttttttttttt tt ttttt   t ttttt ttt ttt tt tt ttttttt tt
   ttttttt!tt tttttttttttttt tt tttt   t ttttt ttt tt tt ttt ttttttt tt
tt tt tttt tttt ttttttttt t t tttt tttttt tttt tttt   t ttttt ttt tt tt ttttttt ttt tt
ttttttttt ttt tttt t
t ttttt ttt tttt ttttt ttttt t t ttt <=  ttttttt ttttt tttt ttt tttttttttt
ttttt ttttt <>  ttttttt ttttt tttt ttt tttttttttt
> !   t t tttttt ttttttt tttttt tt ttttttttt ! 2  ttt ttt ttttt ttttt tttt ttttt tttttttttttt tttt
  t t ttttt t ttttttt tttttt t ttttttttt ! tttttttt ttttt ttttt tttt tt ttttt tttttt ttt tt ttttt tt


ttt tttt ttttt tttttt ttttt t


t tt tttttt tttt ttt   t !t ttt tttt tttt tttt t t  t tt t
ttttt tt ttttt tttt  t tttt tttt tt ttttt ttttt ttt   t !t ttt tttt t tttt tttt t  t tt t
  !t tttt tt tttt tt tt ttttttt ttttttt    t !t ttt tttt t tt tt tttt tttt t 
t ttttttttt   t !t ttt tttt t tt tt t tttt tttt 
  !   t tttttt t tttttttttt tttt tt t   t !t ttt tttt t tt tt t tttt  t ttt
ttttttt ttttttt
ttt
t ttttt tt ttt tttt ttttt ttttt ttt
t ttt
  !  2 t tttttt t tttttttttt tt tt ttt tttttttttttt ttttt tttt ttt ttttt 
ttttttt ttttttt 2= !  t ttttt tttt t !t t ttttttt ttt tt 
tttt tt't   t t ttttt tttt t !t t ttttttt ttt t
tt't   t t ttt tt ttttt tttt t !t t tttttt
2 t   t tttt tttttt tttt ttt tttt t tttt   t t ttt tt t ttttt tttt t !t tttttt
  t tttt tttt tttttt ttt tttt t tttt    t t ttt tt t ttttt !t tttt t tttttt
  t tttt ttt tttt tttttt tttt t tttt   t t ttt tt t ttttt !t ttttttt tttt 
  t tttt ttt t ttttt tttt tttttt ttt   t t ttt tt t ttttt !t ttttttt t ttt
   t tttt ttt t ttttt tttttt tttt ttt
tt t
t ttttt tt ttt tttt ttttt tttt
  t tttt ttt t ttttt tttttt tttt ttt 2> !
ttt
t ttt ttt tt tttt t t tttttttttttt tttt tttt ttt
222<  tttt tttt ttttt ttttt ttt ttt ttttt ttttt t tt ttttttttt tttttttt tttttt tttt ttt ttttttttt tttt tttttttt
ttttttt tt ttttttt ttttttt tttt tt tttt ttttttt ttt =    t t ttt tt !t tttttt tttttt ttttt t !
tttttttttttt ttt ttttttt ttttttt ttt ttt tt tttttt    t t ttt tt !t tttttt tttttt ttttt t !
tttttt ttttt tttt tttt ttttttt tttt tt tttt ttttt t   t t ttt tt !t tttttt ! t tttttt ttttt 
ttttttt tt tttttttt ttt tttt tt tt tttttttt ttttt   t t ttt tt !t tttttt ! t ttttt tttttt 
ttt tttt tt tttttttt ttttt ttttt ttt tttt tttttt =   t ttttt ttttt tttt t t  t !t ttttt
tt tttttttttt tttt tttttt
t ttttt tttt ttt
t tt t   t tttttt ttttttttt t  t!t ttttt
tttttttt tttttt ttt tttt tt tttttttt ttttt ttt tttt tttt   t t tttttttttttttt t t !tttttt
ttttttttt t ttt tttt ttttt ttt tt tttt tttttttt   t ttttt  tttttttttttt !tttttt
ttttt ttttt tttttttt
t tttt tttttt ttttttt ttttt tt    t ttttt  t ttttt !tttttt t ttttt
tttttt ttttttttttttttttttttttttttttt tttttttttttttt =2=<  tttttttt ttttt ttttttttttttttttttttttttttttttttt
ttttt ttttt ttt ttt tttt ttttttttt tttttt ttt tttttttttttttttttt tt tttttttttt tttttt tttttttt
tt tttttt ttttttt ttt tttttttt tttttttttt tttttttttttttttttttttttttttt ttttttt
22   t ttttttttt tttttt t !t  t t ttttt tttt  ttttttttt ttttttttttttttttttttttttttttttttttt
  t t ttttt ttttttttt tttttt t !t  t tttt  tt tttttttttttttttttttt ttttttttttttttttt ttttt
  t t ttttt t ttttttttt tttttt t !t  t ttt ttt
ttttttt
t ttt
ttttttt
t
  t t ttttt t ttttttttttttttt t !t  t ttt tttt tttttt ttt ttttt t
t tttttt ttt ttt tttttt ttt
   t t ttttt t tttttt  t ttttttttt t !t ttt ttttt tt ttt ttttt ttt ttttt tttt tttttttt ttttt tttttttt
22     t ttttttttt t tttttt !t t t tttt tt ttt tttt ttttt ttt ttt ttttt tttt t tt ttttttttt ttt
  t ttttttttt t tttttt !t ttt t t ttt ttttttttt tttt ttttt tttt ttt ttttt tt tttt tttt tttt
  t ttttttttt t tttttt !t ttt t t ttt tt ttt ttttt t ttttt tttt tttttttt ttttt tttttttt
  t ttttttttt t tttttt !t ttt t tttt  =2  tt tt ttt ttttt tttt t t tttttttttttt tt tttt tttt tt
2   t tt tt tttt t t  t ttttttttttt ttttt  tttt
  t tt tt t tttt t  t ttttttttttt ttttt  =2 !  t tttttt tttttt  t t ttt ttt t ttttt
  t tt tt t ttttt tttt t  t ttttttttttt    t tttttt tttttt  t t ttt ttt t ttttt
  t tt tt t tttttt tttt t  t tttttttttt   t ttttt tttttt tttttt  t t ttt ttt t
   t tt tt t ttttt t tttt  t t tttttttttt   t ttttt t ttt ttt tttttt tttttt  t t
  t tt tt t ttttt t tttt  t ttttttttttt  =    t tttt!tttttttttttttttt! ttttttttttt
2   t tttt t ttttttt t t tt tt tttt t  t    t tttt!tttttttttttttttt! ttttttttttt
  t tttt t tttt t ttttttt t t tt tt  t     t tttt!tttttttttttttttttttttttttt! t
  t tttt t  t tttt t ttttttt t t tt tt    t tttt!ttttttttttttttttt! tttttttttt
  t tttt t  t tt tt tttt t ttttttt t t 
ttt
t ttttt t ttt tttt ttttt ttttt
tt t
t tttttt tttt
   t tttt t  t tt tt t tttt t ttttttt t  tttttt tttt ttttttt t tttt ttttt ttt ttt tttttttt ttt
  t tttt t  t tt tt t ttttttt tttt t t  ttttttttttt
  t tttt t  t tt tt t ttttttt t tttt t  =    t ttttttt !t tttttttttt tttttttttt 
tttt tt ttt tttt ttttt ttttt
tt tt ttttt t ttt ttt tttttt 
tttttttttttt tttt    t ttttttt !t tttttttttt tttttttttt 
2< ! ttttttt ttttt tttt ttt tttttttttt tttttt 
22= 
tttt ttt ttt tt tttt t t ttttttttttt    tttttttttt t ttttttt !t tttttttttt 
ttt tttt t ttt tttt ttttt tttt ttttttttt tt tttttt 
ttttttttt ttt tttttt ttttttt tttt tttttttt ttt    ttttttttttttttttttttt!t ttttttttt
ttttttttt ttt ttt ttttttt tttttt ttttttt t ttt tttttt 
tttt ttt ttt ttt tttt ttttt tttttt ttttt ttttt t ttt     tt tttttttt t ttttttt t tttttt !
tttt tttttttttt tttt tttt tttttttt ttttt tttttttt
tttttttttt 
tttt tttttttt tttttttt t ttttttt ttttt tttttttt    tttttttttt t ttttttt t tttttt  t !
ttt ttt tttt ttt tt tt ttttt ttttt ttttttttt
22 
tttt tt tttt ttt tttt tttt tttt tttttttttttt tttt
t tt    tt tttttttt t ttttttt t tttttt 
tt ttttttttt tttt tttttttttt !
 t tttt tttt t ttt tttt t  t !t tt t
ttt
t tttt tttttttt tttttttttttt ttt ttttttttt tt
  t !t tttt tttt t ttt tttt t  t tt t ttttt tt
tt t
t tttt tt ttt tttt ttttt tttt
  

=<    ttttttttttttt !tttt tttttt ttttt tt    t t ttttttt  t ttttt !t t t ttttttt
  tttttttt tttttt !tttt tttttt ttttt tttttt
  tttttttt ttt ttttttttttt !ttt tttt   t t ttttttt t  t ttttt t t t !t t t ttttttt
tt ttttttttttttttttttttttttttt
t ttttttttttt tttttt tttttt
tttttttt >2    t t ttttt t t tttttt tttt ! t :ttt
=2>  ttt ttt tt tttt t t ttttttt ttttttttttt   tt tttttttttttttt tttt! tt:ttt
ttttttttttttttttttttttttttttttttttttttttttttttt    tt tttttt ttttttttttt! t:ttt
ttttttttttttttttt ttttttttttttttttttttttttttttt   t t ttttt t tttt ! t tttttt t :ttt
ttt
ttttttttttttttttttttttttttttttttt tttt ttt   t t ttttt t tttt ! t :tttt t ttttt
tt
tt t
t ttttt tt ttt tttt ttttt tttt
tttttt
tttttttt
tt ttttttttttt
tt tttttttttttt > !  t tttttt tttttttt t ttttt t  t ! t ttttt
t tttt ttt tt   t t tttttt tttttttt t ttttt  t ! t ttttt
ttt
t tttt ttttt tttt ttttttttt ttt tttt tttt ttt   t t tttttt t tttttttt ttttt  t t ! t ttttt
tt   tttttttttttttt tttttttt tt! tttttt
t 
t #t 
t t 
t t t tttt ttt tt > ! tt tt ttt ttt tt tttt t t ttttttttttt
t ttttt ttt
ttttttttttttttttttt
tttttttttttttttttttttttt tttt tttttttt tttttt tttt ttt tttttttt
tttt ttt t tttttttttt t tttt tttt tttt tttttt ><    t t ! t tttt t ttttt t tttttttttt  t  t ttt
ttttt tttt tttt ttttt tt ttt ttttt tt
t tt tttt tttt   t t ttttt ! t tttt t t tttttttttt  t  t ttt
ttttttttt tttt tttt tttttttttt ttttttttt tttt tttttt   ttttttt t! t tttt tt tttttttttt tttt
ttttt tttt ttt ttt tttt ttttttttt tttt tttt ttt >2  ttttttttttttttttttttttttttttt tttttttttttt ttttt
ttttttttt tttt ttttttttt t t ttt tttttttt ttt tttt tttttttt
t tttttttt tttt ttttt t t tttt ttttt ttttt tttt
tttttttt tttttttt
t tttttttt tttttttt ttttt tttttttt ttt tttt tt tt ttttt tttt tttt ttttt ttttt ttttt ttt ttt
=2    t tttt ttt tttttt!t ttttt ttttttttt ttt tttttt t tttttt tttt tt tttt tttttttt
   t tttt t tttt tt tt!t ttttt  tttt tttt tt tt tttt tttt tttttttttt tttttt ttt
  t t t tttt tttt!t tt ttttttt  ttttt ttttt tttt ttttt ttt tttttttttttt ttttt tttt ttt
  t tttt t tttt !tttttt tt tt  tttttttt tttt ttttttttt ttt ttt ttttttt tttttt ttt
    ttt t tttt !t ttttt t tt t ttt t ttttt ttt tt ttttt ttttttttt ttt tttttt tt

tt t
t ttttt ttt ttt tttt ttttt tttt tttttttttt tttttttttt ttt tttt tt ttttt ttt ttttttt
==    t t ttt ttttt tttt t !t  t ttttt  tttttttt
t tttt tt ttttt ttttt ttt ttt tttt tt tt ttt
  t t ttt  t ttttt tttt t !t ttttt  >2      t tttt t !t t ttttt tt tt tttt 
   t t ttt  t ttttt ttttt tttt t !t     t t tttt t t !t ttttt tt tt tttt 
  t t ttt  t ttttt !t ttttt tttt t     t t tttt t t tt tt !t ttttt tttt 
=> ! ttt ttt tt tttt t t tttttttttttt ttttttt tttttt ttt   t t tttt t t tt tt !t tttt ttttt 
ttt ttttttttt tttt tttttttt     t t tttt t t tt tt !t tttt  t tttt
>    t  t tttt !t tttttt ttttt t t t ttt
tt t
t tttt ttttt tttttt tttt tt ttttt
  t  t tttt t !t tttttt ttttt t t ttt >= ! tt tttttt ttttttttttt tttt tttttttttttt ttt ttt
   t t ttttt tttt!tttttttttttt t ttttttt tttt tttt
  t t  t t tttt t tttt t !t tttttt ttttt >>    :t tttt tttt  t !t ttt ttt ttttt t t !
  t  t tttt t tttt t ttttt t !t ttttt   t ! t :t tttt tttt  t t !t ttt ttt ttttt 
tttt tt ttt tttt ttttt ttttt ttt
t tttt ttttt ttttt   t ! t :t tttt !t tttt  t ttt ttt ttttt 
tttt tt tttttt ttt tttt tttt ttttttttttt   t ! t :t tttt !t ttt ttt tttt  t ttttt 
>   t ttttttt ttttt tttt tt t t t t !     t t ttttt t t t ttttttt tttttttt t tttt
  t t ttttttt ttttt tttt tt t t t !   t t ttttt t t t ttttttt tttttttt t tttt
  t t ttttt ttttttt tttt tt t t t !    t t ttttt t ttttt t ttttttt tttttttt 
  ttttttt!tttttttt tttttt tt   t t ttttt t ttttt t t ttttttt ttttttt
>2><  tttttttttttttttttttttttttttttttttttttttttttttttt   t t ttttt t ttttt t tttttttt t tttttt
ttt tttttttttttttttttt ttttt ttttttttttttttttttttt
tt tt ttttt tt tttt ttt tttt ttttt tttt
ttttt tttttttttttt t ttttttttttttttttt tttttttt     t t :ttttttttttttt  tt !ttttttt tttttttt
tttttttttt tttttttttttttttttttttttt tttttttttt tt     ttttttt:ttttttt ttt !ttttttttttttttt
ttttttttttt
t ttttttt tttttttt tttt ttttttttt tt   t ttttttt !t:ttttttt ttttttttttttttt
ttt tt ttttttt ttttt    t ttttttt !ttttttttt:ttttttt ttttttt
tttt tttttt ttt ttttt t
t t t tttttt tttt ttttt ttt t   t  t tttttt !t tttttttt  t :ttttttt tttttt 
tttt tttt tt tt tttt tttt tttt tttttt ttttttttt ttt 2<  ttt ttttt tttt ttttttttt ttt ttttttttt ttt ttttt
ttttttt tttt ttt tt ttttt ttt tttt tttt tttttttt ttt tt ttttttt tttttt t ttttt t
tttttttt
t ttt ttttt tt
t ttt tt tttt tttt ttttt tt tttt tttt ttttt tt ttttttt ttt ttttt ttttt tt
tttttttt ttttttttt ttt tttt tt tttt ttt t
t ttttt ttt tttttttttttt tttt tt tttt ttt ttttt tt tt tt
!t t tttttt tttt tttttt ttt tt tttt ttt t ttttt tttttttt
t tttt tttttttttt tttttt ttt tt ttttt ttt
ttttttttt tttt ttttttt tttt ttt tt ttttt ttt 2   t ttt !t ttttttttt  t ttttt  t tt tttt !
ttt
t tttttttttt tttttttttt t t tttttttt tttt tttt    t ttt !t ttttttttt  t  t ttttt tt tttt !
ttttttt ttttttttt ttttt tttttttt tt tt tttttttt ttt tt   t ttt !t ttttttttt  t tt tttt  t ttttt !
tttt tttt tt tt tttt t tttttt ttt tttttttt tttt ttt 2   t tttttttttt  t tttttt  t tttttt t tttttttt
tttt ttt ttt tttt t tttttt ttt tttt !
>2    tt ttttttt!ttttttttttttttt ttttttt   t tttttttttt  t ttttttttt tttttt  t tttttt 
  t t t ttttttt  t t !t t t t ttttt ttttttt !
tttttt   t tttttttttt  t ttttttttt  t tttttt ttttt
t !
2

  ttttttttttt tttttttttt ttttttt !tttttt   t t ttttt t ttttt t tttt ttttt 
   t t ttttt t ttttt t tttt t tttt
 !  t tttt  t tttttt ! t tt tttt t tttttttt 
tt tt t tt t 't t ttttt tttttt tttt ttt tt ttttt
  t tt tttt tttt  t tttttt ! t t tttttttt  2 !  t t t t ttt ttt tt tt t !t :tttttt
  t tt tttt t tttt  t tttttt ! t tttttttt    t !t t t t ttt ttt tt tt t :tttttt
  t tt tttt t tttttt tttt  t ! t tttttttt    t !t tt tt t t t t ttt ttt :tttttt
   t tt tttt t tttttt  t tttt  t ! t ttttttt   t !t tt tt t t t t ttt ttt :tttttt
  t tt tttt t tttttt  t tttttttt tttt  t !    t !t tt tt t t :ttttttt t t ttt tt
  t tt tttt t tttttt  t tttttttt  t tttt !   t !t tt tt t t :ttttttt t t ttt tt
    tttt ! t ttttttt t tttttt  t tttttttt    t !t tt tt t t :ttttttt t ttt t tt
  t tttttttt tttt ! t ttttttt t tttttt  t     t tt tt tt ttt !t t tttttt ttttttt !t
  t tttttttt  t tttt ! t ttttttt t tttttt    t t tt tt tt ttt !t tttttt ttttttt !t
   tttttttt  t tttttt tttt ! t ttttttt t    t t tt tt !t tt ttt tttttt ttttttt !t
   t tttttttt  t tttttt  t tttt ! t ttttttt    t t tt tt !t ttttttt tt ttt tttttt !t t
  t tttttttt  t tttttt  t ttttttt tttt ! t   !     t t ttttt  t  t t tttttt ttttt tt t
  t tttttttt  t tttttt  t ttttttt ! t tttt    t  t t ttttt  t tt tt  t t tttttt tttt
ttt ttttt tttt tttttt tttt ttttt tttt ttttt tt tt ttt    t  t t ttttt  t tt tt  t tttttt t tttt
tttt ttttt t tt ! t tttt tt tt
tt tt tttttt ttttt ttttt tt tt tttt ttttt ttt tttttttt tttttttt
t tttttt tttt tt
ttt !t t t ttt t t t ! t tttt 't t ttt t ttt ttt t ttt tttt
< ! tt tttt ttt tttt tt t ttt < ! tt tttt tttt tttt tt t ttt
2  ttt tttt ttttt tttt tttttttt ttttttt ttttt ttt ttt 2   t ttttt tttt t !t tttt ttttt  t 
tt tttttt ttttttttt ttttt tttt tttttttttt ttttt ttt tt   t t ttttt tttt !t tttt ttttt  t 
ttttt tttt ttt tttt tt tt ttttt tttt tttt ttttt tttt   t t tttt ttttt !t tttt ttttt  t 
ttttttttttttttttttttt tttttttttttt tttttt tttttt ttt   t t tttt !t ttttt tttt ttttt  t 
ttttttttt tttt tttt tttttttttt ttttt ttt ttt ttttt ttt    t t tttt !t tttt ttttt ttttt  t 
tttt ttttt ttt tttttttttttt ttttt tttt tttttttt ttt   t t tttt !t tttt  t ttttt ttttt 
ttttttttt ttt ttt ttttttt tttttt tttt tttt ttttt tt   t t tttt !t tttt  t ttttt t tttt
ttttttttt ttt tttttt ttt tttttttttt ttttttttt
tt tt tttt t ttt tttt ttttt tttt ttt
2    t t ttt tt t  t ! t ttttt tttt ttt =    t  t ttt !!t t t  t tttttt tt t tttt
  t t ttt tt ! t t t  t ttttt tttt ttt    t  t ttt !!t tttttt t  t tt t tttt
   t t ttt tt ! t ttttt t  t tttt ttt   t  t ttt !!t tttttt  t t tt t tttt
  t t ttt tt ! t ttttt  t t tttt ttt   t  t ttt !!t tttttt  t ttttt t tt
  t t ttt tt ! t ttttt  t tttt t ttt
tt tt t tt t 't t ttttt tttttt tttt ttt tt ttttt

tt tt t tt t 't !t ttttt tttttt tttt tt ttttt >   t tt t t tttttt t ttttt t t :ttt
=    t t ttttt !t t tt ttt tttttt tttt !   t t tt t t tttttt t ttttt t :ttt
   t t ttttt !t tttt t tt ttt tttttt !   t t ttttt tt t t tttttt t t :ttt
  t t ttttt !t tttt !t t tt ttt ttttt   t t ttttt t tt t tttttt t t :ttt
  t t ttttt !t tttt !t tttttt t tt tt     t ttttt t tt t t tttttt t :ttt
>   t  t t t tttt tttt ttt !t tt 22 t tttttttt ttttt tttttttt ttt ttttt tttttt tt ttttttttt
  t t t  t t t t tttt tttt ttt !t tt ttttttt tt ttt ttttt tt tttt tt tttt ttttttt ttt
  t t tttt  t t tttt ttt !t tt tttttttttttt ttt ttt ttttttt ttt ttt tt ttttttt ttttt

  t t tttt  t ttt t tttt !t tt ttttt tttt tttt ttttttt tttt tt tttt ttttt t t tttttt
   t t tttt  t ttt !t t tttt tt tt tttttttt ttt tttt tt tt tttttttt tttttt ttt ttt
  t t tttt  t ttt !t tttt t tt tt tttttttt ttttt ttttt ttt tttt tttttt
 ! tt tttt tttt tt t ttt tt tttttttttt tttt tttttt
t ttttt tttt ttt
t tt t
   t ttttt ttttt !t t tttttt ttttt ! t  tttttttt tttttt ttt tttt tt tttttttt ttttt ttt tttt tttt
  t !t ttttt ttttt t tttttt ttttt ! t  ttttttttt t ttt tttt ttttt ttt tt tttt tttttttt
  t !t ttttt ttttt t tttttt ttttt ! t  ttttt ttttt tttttttt
t tttt tttttt ttt ttttt ttt ttt
  t !t ttttt ! t ttttt t tttttt ttttt  tt tttt ttttttttt tttt tttt ttttt ttt tt tttt tttttttt
   t !t ttttt ! t ttttt t tttttt t tttt ttttt ttttt ttt ttt tttt ttttttttt tttttt ttt

tt tt tttt tttt tttt ttt tttt ttttt tttt ttt tt tttttt ttttttt ttt tttt tt
2<  ttttttttttttttttttttttttttt tttttttttttttt ttttt 2! ttttttt ttttt tttt ttt tttttttttt
tttttttt
t tttttttt tttt ttttt t t tttt ttttt ttttt ttt 2  tttt ttttt ttt ttt ttttttt tttt ttt
ttt ttttt ttt tttt tttt ttttt ttttt ttttt ttt ttt 22  tttttttttttttttttttttttt ttt tttttt
ttttttttt ttt tttttt ttt ttt ttt tt tttt tttt tt t
  t tttttttttttttttttttttttttttttt ttt
tttttttt tttttt tttt ttttt t t tttt ttttt ttttt ttt   t tttttttttttttttttttttttttttttt ttt
tttttttt
t ttt ttttt tttt tttt tt tttt tttttt tttttt   t ttttttttt tttttttttttttttttttttttt
ttt ttt ttt tttttt tttt tt tt ttt ttttt tt tttt tt ttt    t ttttttttt tttttttttttttttttttttttt
tttt ttttt ttt tttt ttttttttt ttt tttttt ttttt ttt ttt   t ttttttttt tttttttttttttttttttttttt
tttttt ttttttttt ttt ttttt ttttt ttt tttttttttttt tttt 22   t tttt t=ttt t ttt t t t ttt 
tttt tttt tttttttt tttt ttttttttt ttt tt ttttttt ttttt   t tttt t t=ttt t ttt t t t tt
tttt tttt ttttt ttt ttttttt ttttttttt ttt ttttt   t tttt t t=ttt t t ttt t t tt
2    t t ttttt ttttt tttt ttttt t t    t tttt t t=ttt t ttt t ttt t 
  t t ttttt t ttttt tttt ttttt t t     t tttt t t=ttt t ttt t t ttt 
   t t ttttt t ttttt ttttt tttt t t 
tt t
t ttttt tttt ttt tttt tttttttttttt tttt
  t t ttttt t ttttt t ttttt tttt 
  2

2 t    t ttttt t ttttt t tttttt t ttttt    t t ttt t ttttt ! t tt t ttt t 
  t ttttt t tttttt ttttt t t ttttt    t t ttt t ttttt ! t tt t  t ttt
   t ttttt t tttttt t ttttt t ttttt  tttt ttttt tt tttt ttt tttt ttttt tttt ttt
  t ttttt t tttttt t ttttt ttttt t  22   t ttttt ttt t  t !t  t !!t ttttt ttt
22 t ttt tttt ttttt ttttt tttt tttt ttttt ttttt ttttt tt   t  t ttttt ttt t  t !t !!t ttttt ttt
tttt ttttttt ttttttttt ttt tttttt ttttt ttt tttt tttt   t  t ttttt !!t ttt t  t !t ttttt ttt
ttt tt tttt tttt ttttt t t tttt ttttt ttt ttt ttttt ttt   t  t ttttt !!t tttt ttt t  t !t tttt
ttt tttt tt tt tttt
t tttt tttttttt ttttttt t ttttt    t  t ttttt !!t tttt  t ttt t !t tttt
tttt tt tt ttt tt tttt tttt ttttt t t tttt ttttt ttt tt   t  t ttttt !!t tttt  t ttttt ttt t !
ttttt tttt tttt ttttt ttt tttttttttttt ttttt tttt tttt   t  t ttttt !!t tttt  t ttttt !t ttt
tttttttttttttttttttttttttttttttttttt tttttttttttt tttt ttttt tt tttt ttt tttt tttt
tttt tttttttt ttt tt ttt ttttttt ttttt 2    t ttttttttttttttttt:tttt!
2   t t tttt ttttt  t t !t ttttt ttttt   t  t tttt !t t t tttttt tttt :ttt
  t ttttt t tttt ttttt  t t !t ttttt      t tttt !t tttttt t t tttt :ttt
  t ttttt t ttttt tttt  t t !t ttttt   t  t tttt !t tttttt t t tttt :ttt
  t ttttt t ttttt  t tttt t !t ttttt   t  t tttt !t tttttt t :tttt t ttt
   t ttttt t ttttt  t tttttt tttt t ! tttt t tt t 't !t ttttt tttttt tttt ttt tt ttttt
  t ttttt t ttttt  t tttttt t tttt ! 2     t ttt t!ttt t tttt ttttt
tt tt tttt t ttt tttt ttttt tttt
t ttttt tt tttt ttt ttt   t t tt t !t tt t t  t tt tt ttt
ttttt tttt ttt   t t tt t !t tt t t tttt  t tt t
2< ! ttttttttttttttttt ttttttttttttttttttttttttttt tttt ttttt tt tttt t ttt tttt
t tt ttt ttttttttttttt 2< ! tt tttt ttt tttt tt t ttt
22    t tttt  ttttttt tt tt t t ttttt tttt 22   t tttt ttt ttttt t t !t ttttt 
   t tttt  t tttttt t t t t ttttt tttt   t t tttt ttt ttttt t !t ttttt 
   tttt  t tttttt t ttttt t t tttt   t t tttt !t ttt ttttt t ttttt 
  t tttt  t tttttt t ttttt t t tttt    t tttt !t ttttt ttt tttttt 
  t tttt  t tttttt t ttttt t ttttt    t t tttt !t ttttt t ttt ttttt 

tt tt t tt t 't !t ttttt tttttt tttt ttt tt ttttt 2=2 t ttt ttttt t
t tttt ttttttttt ttttttt ttttt ttt ttt
2=   t ! t tt t tttttt !t t ttt ttttt  t tttttt tt tt tttt tttt ttttt t t ttttt ttt ttt ttttt ttt
  t tttttt ! t tt t!t t ttt ttttt  ttt tttt tt tt ttttt tttt tttt ttttt ttttt ttttt ttt ttt
  t tttttt t ! t tt t !t t ttt tttt ttttttt ttttttttt ttt tttttt ttttt ttt tttt tt ttt
  t tttttt t ttt ! t tt t !t t tttt ttttttttt tttt tttt tt tt tttt tttt ttttt t t tttt tttt
   t tttttt t ttt ! t ttttt tt t !t  ttt ttt ttttt tttt ttt
t tttt tttttttt tttt ttt
  t tttttt t ttt ! t ttttt t tt t ! tttttttttttttttttttttttttttttttttttttttttttttttttt
2>   t ttt t ttttt t t ttttt ttttt t ! tttt ttt tt ttttttt tttttt tttt tttt tttt ttttt tt
  t ttttt ttt t t t ttttt ttttt ! ttttttt ttttttttt ttt ttttt
  t ttttt t ttt t t t ttttt ttttt t ! 2=   t  t ttttt t ttttt tttttt t t :tt
  t ttttt t ttt t t ttttt t ttttt !   t t  t ttttt t ttttt tttttt t :tt
   t ttttt t ttt t t ttttt ! t t tttt   t t :ttt  tttttt t ttttt tttttt 
  t ttttt t ttt t t ttttt ! t ttttt    t t :tttt  tttttt ttttttttttt 
2t     t ttttt !t ttttt t t !t ttttt ttttt     t :ttt t tttttt  t ttttt ttttt 
  t ttttt !t ttttt t tttttt t !t tttt   t t :ttt t tttttt t  t ttttt tttt
  t ttttt !t ttttt t tttttt !t t tttt   t t :ttt t tttttt t ttttt  t tttt
  t ttttt !t ttttt t tttttt !t ttttt  2>      t tttt t t !t ttttt tt t tttt
2 !  t tttt tttt t !t t tttt tt t   t  t tttt !t t t ttttt tt t tttt
  t tt t tttt tttt t !t t tttt    t  t tttt !t ttttt t t tt t tttt
  t tt t t tttt tttt t !t tttt   t  t tttt !t ttttt t t tt t tttt
  t tt t t tttt tttt t !t tttt    t  t tttt !t ttttt t tt t t tttt
   t tt t t tttt t tttt !t tttt     t t t ttttt t :ttttt !t !t tttt tttt
  t tt t t tttt t tttt tttt !t    t t t ttttt t :ttttt !t !t tttt tttt
  t tt t t tttt t tttt t tttt !   t t t ttttt t :ttttt !t ttttt !t ttt
2222 tttttttttt
ttttttttttttttttttttttttttttttttt ttttt t tt t 't t ttttt tttttt tttt ttt tt ttttt
tt tt tttt tttt ttttt t t tttt ttttt ttt ttt ttttt tttt    t ttt ttttttt ttttttttttttt!
ttt tttt tt tt ttttt tttt tttt ttttt ttttt ttttt tt   t !t ttt t ttttt t t tttttt ttttt
ttttttttt ttt tttttt ttttt ttt tttt tt tttt tttttttt   t !t ttt t t ttttt t tttttt ttttt
tttt tttt tt tt tttt tttt ttttt t t tttt ttttt ttt tt   t!t ttt tttttt ttttttttttttt
ttttt ttttt tttt tttt tttttttt tttt tttt tttt    t!t ttt ttttttt tttttttttttt
tttt ttttttttt tttttttttttt ttttt tttt tttt tttttttt ttt 2    ttttt!ttttttttttttt t tt
ttt tt t t tttttt tttttt tttt t ttt ttt t ttttt tt   t t tttt !t t ttttt tttttt  t tt
ttttttttt ttt ttttt   t t ttttt tttt !t t tttttt  t tt
22    ttt t t  t tt t ttt t ! t tttt   t t ttttt  t tttt !t t tttttt tt
  t t ttt t t  t tt t ttt ! t tttt    t t ttttt  t tttt !t ttt t ttttt
  t t ttt ttt t t  t tt t ! t tttt
tt tt ttttt tttt tttt ttt tttt ttttt tttt ttt
  t t ttt t ttt t  t tt t ! t tttt 2    ttttttt t t tttt t t t ! t =ttt
   t t ttt t ttttt ttt t  t tt t !   t t ttttttt t t tttt t t ! t =ttt
  t t ttt t ttttt ! t ttt t  t tt   t t =tttt ttttttt t t tttt t t !


  t t =tttt ! t ttttttt t t tttt t    t tttt tt t ! t !t ttt tttt !t
   t t =tttt ! t t t ttttttt t t ttt   t tttt tttt tt t ! t !t ttt !t
  t t =tttt ! t t t t ttttttt t ttt   t tttt t tttt tt t ! t !t ttt !
= t ttt tttt ttttt tttt
t tttt tttttttt ttttttt ttttt tt   t tttt t tt t tttt ! t !t ttt !
tttt ttttt ttttttttt tttt tttt tttttttttt ttttt ttt tt    t tttt t tt t ! t tttt !t ttt !
ttttt ttttttt tttt tt tttttt
t tttt tttt ttttt tttt   t tttt t tt t ! t ttt tttt !t !
ttttt ttt tttt ttttttttt ttt tttttt ttttt ttt ttt t ttt   t tttt t tt t ! t ttt !t tttt !
tttt tt tttt tttttttt
t tttttttt tttt ttttt t t tttt tttt
tt tt tttt t tttt ttt tttt ttttt tttt ttt
ttttt tttt tttt ttttt ttt tttttttttttt ttttt tttt ttt  ! tt tttt ttt tttt tt t ttt
ttttttttttttttttttttttttttttt tttttttttttttttt 2 !  t ttttt tttt ! t t t ttt tt tt t 
tttt tttt ttttt ttttt tt tt t tttt t tt t ttt ttt tt   t ttttt t tttt ! t t t ttt tt tt
ttttttttt   t ttttt t tttt ! t tt t t t ttt t
 !  t tt t ttt ttttt ttttt t  t  t !   t ttttt t tttt ! t tt t t ttt tt 
  t !t tt t ttt ttttt ttttt t  t  tt tt tttt ttttt ttt tttttttt tttttttt
t tttttt tttt tt
  t !t tt t t ttt ttttt ttttt  t  ttt !ttt tttt
  t !t tt t t ttttt ttt ttttt  t  2    t ttt t tt t tttt t ! t ttttt !
   t !t tt t t ttttt  t ttt ttttt    t ttt t ttttt tt t tttt t! t !
  t !t tt t t ttttt  t ttt  t tttt    t ttt t ttttt ! t tt t tttt t !
tt tttttttttttttttt tttttttttttttttttttttt
tttttt   t ttt t ttttt ! t tttt tt t t !
tttt ttt ttt ttttt t   t ttt t ttttt ! t tttt t tt t !
   t tt ttt tttt tt t t !t  t ttttt !  !  t ttt ttt tttt ttttt !t t t t tttt
  t  t tt ttt tttt tt t t !t ttttt !   t ttttt ttt ttt tttt !t t t t tttt
  t  t tttt tt ttt tt t t !t ttttt !   t ttttt t ttt ttt tttt !t t t tttt
  t  t tttt t tt ttt tt t !t ttttt !   t ttttt t ttt ttt t tttt !t t tttt
   t  t tttt t ttttt tt ttt tt t !t !    t ttttt t ttt ttt t ttttt tttt !t 
  t  t tttt t ttttt !t tt ttt tt t !   t ttttt t ttt ttt t ttttt t tttt !
  t  t tttt t ttttt !t tt t tt ttt ! > tttttttt ttttttttt
ttttt ttttttttt ttttt ttttttttttt
  t  t tttt t ttttt !t tt t !t tt tt ttttttttt tttttttttttt tttttttttttt tttttttttttttt


tt tt tttt tt tttt ttt tttt ttttt tttt ttt ttttttttttttttttttttttttttt tttttttt tttttttttt
< ! tt tttt ttt tttt tt t ttt tt tttttt
ttttttttttttttttttttttt tttttttttttt ttt
2    t ttt tt t!t tttttttttt tttt ttttttttt tttttt t
ttttttttttttttttttttttttttt
  t ttt tt t t!t ttttttttttttt ttttt ttttttttttttttttttttttttttttttttttttt tttt
   t ttt tt ttttttt t!t ttttttt tttttttttttttttttttttttttttt tttttttttttttttt
  t ttt tt ttttttt!t t ttttttt tt ttt tttttt t ttt t t tttttttttt tt tttt ttt ttt tt
  t ttt tt ttttttt!t tttt tttt ttttttttt

tt tt t tt t 't !t ttttt tttttt tttt ttt tt ttttt     ttttttt tttttttttttttttttt!
=   t!t t ttttttttt tttt t!ttttt    t ttttt  t ttttt ! t t t ttttt tttt
  t t !t  t tttt ttttt tttt !t tttt   t ttttt  t ttttt ! t ttttt t t tttt
  t t tttt!t t ttttttttt!ttttt   t ttttt  t ttttt ! t ttttt t t tttt
  t t tttt!t!t t ttttttttttttt   tttttt tttttt! tttttttttttt
   t t tttt!tttttt!t t tttttttt <   t ttttt tt t t  t  t t t tt tt tt
  t t tttt!tttttt t!t tttttttt   t ttttt t tt t t  t  t t tt tt tt
  t t tttt !t ttttt  t ttttt !t ttt   t ttttt t tt t  t t  t t tt tt tt
> t ttt tttt ttttt tttt
t tttt tttt ttttt ttttt ttttt tt   tttttt ttt t tt ttt ttt tt
tttt ttttttt ttttttttt ttt tttttt ttttt ttt tttt tttt    tttttt ttt t tt ttttt ttt
ttt tt tttt tttt ttttt t t tttt ttttt ttt ttt ttttt ttt
tt tt ttttt tttt tttt ttt tttt ttttt tttt ttt
ttt tttt tt tt tttt
t tttt tttttttt ttttttt t ttttt 2     tttttt t t t tt tt tttttttt
tttt tt tt ttt tt tttt tttt ttttt t t tttt ttttt ttt tt   t ttttt t t t t tt t  t  t ttt
ttttt tttt tttt ttttt ttt tttttttttttt ttttt tttt tttt   t ttttt t t t t tt t  t  t ttt
tttt ttttttttt ttt tttt ttttttt ttttttttt ttt ttttt   t ttttt t t t t tt t  t tttt 
tttt tttt tttttttt ttt tt ttt ttttttt ttttt
tt tt t tt !t 't t ttttt tttttt tttt ttt tt ttttt
>    t ttttt t tttttt  t t tttttt tttttt  = ! tt tttt ttt tttt tt t ttt
   t ttttt t tttttt  t  t t tttttt ttttt >    ttt t tttt ttt tt !t  t tt t !
  t ttttt t tttttt  t tttttt  t t ttttt   t tttt ttt t ttt tt !t  t tt t !
  t ttttt t tttttt  t tttttt t  t ttttt   t tttt t ttt ttt tt !t  t tt t !
  tttttt ttttttt tttttttttttttt   t tttt t tt t ttt ttt tt !t  t !

tt tttttt't!tttttttttttt ttttttttt ttttt    t tttt t tt t !t ttt ttt tt  t !
  t tttt t tt t !t ttt tt ttt  t t !
  t tttt t tt t !t ttt tt !t ttt 
  

  2

>    tttt t t ttttt


  ttttt ttttttt tttttt tt tt ttt tttttt tt ttt ttttt tt  ! t ttttt ttt t ttt tttt tttt tttttt ttt ttttt ttt
tttt 2  tttttt  t ttt tttt ttttt tttt
   tttttt ttttt ttttttt tttttt tt tt ttt tt ttt ttttt tt 2  t t ttttt
tttt  ! ttt ttt ttttt t
   tttttt ttttt ttttt ttttttt tttttt tt tt ttt tt tt 2 
ttttt tt     t ttttttt  t !!t t ttt t tttttt ttt t
   tttttt ttttt ttttt tt ttt ttttttt tttttt tt tt tt     t ttttttt ttt  t !!t t t tttttt ttt t
ttttt tt     t ttttttt ttt t  t !!t t tttttt ttt t
    tttttt ttttt ttttt tt ttt ttttt ttttttt tttttt tt tt tt      t ttttttt ttt t !!t  t t tttttt ttt t
tt     t ttttttt ttt t !!t t  t tttttt ttt t
   tttttt ttttt ttttt tt ttt ttttt ttttttt tt tt tttttt tt   t ttt tttt ttttt tt
tt <  t ttt tttt ttttt ttt
   tttttt ttttt ttttt tt ttt ttttt ttttttt tt tt ttt ttttt 2  t ttt tttt ttttt t
tt =22  tttt ttt ttt ttttttttt t t tttt ttttt tttttt ttt
   tttttt ttttt ttttt tt ttt ttttt ttttttt tt tt ttt tt ttttttt tttttt t t tttttttttttt ttt ttt ttttttt ttttt tt
ttttt tt tt ttttt tttt tttt tttt tttt ttttttt ttt
t! 2t  ttttttttttt ttt ttttt tttt ttttttttttt tttt ttt
2  tttttt tttttttttt tttttttttttttttt ttttttttttttt tt tt ttt ttttttttt ttt tttttt tttt ttt ttttttt ttttt
ttt ttttt tttt tttttttt tttt ttttttttttt ttt ttt tttttt
t t tttttt tttt tttt ttttt
<>     => 
      tt tttt  t tttt t !t ttt t !t tttttt tttt
      tttttt tt tttt  t tttt t ttt t !t ttttt !
        tttttt tttt tt tttt  t t ttt !t ttttt t !
       tttttt tttt ttt tt tttt  t !t ttttt t t !
<t 2 =t >t     tttttt tttt ttt ttttt tt tttt !t  t t t !
2 t tttt ttt ttt ttttttttt t t tttt ttttt tttttt ttt = !t tttttt ! t ttt tttt tttt ttt
ttttttt tttttt t t ttttttttttttt
t ttt ttt ttttttt tttt > t tt tt tttttt tttt ttttttt ttt tttttttt ttt
tttt tttttttt tttt ttttttttttt ttt tttt ttttttt tt ttttttttttt
ttt ttttttt tttttt tttt ttttt tttt ttttttttt tt 222 
ttttttttt ttt tttttt tttt tttt tt tt tttt tttttt tt   t t ttttt ttt t tt t ttt !t ttttt tttttt t 
ttttt tttttttttt tttt t tttt tttt ttttttttttt ttt tttttt 
tttt tt tt ttt ttttttttt ttt tttttt tttt tttt tttt    tttttt t ttttttttt ttt t ttt!ttttttt ttttt
tttttttttt tttt tttttttttt tttt ttttttttttt ttt t 
tttt tttttt ttt tttt ttttttt ttttttttt ttt ttttt    tttttt ttttt t t ttt tt t ttt !t ttttt t ttttt
  t t 
  ttt ttt!t ttttttttt t tt tttttttt tttttt    tttttt ttttt ttttt t ttt tt t ttt !t t tttttt 
t t tttt t ttttt t t 
   tttttt ttt ttt !t ttt t ttttt  t t ttttt t      tttttt ttttt ttttt ttt tt t ttt !t t tttttt t 
t tttt t tt tt ttttt t t 
   ttttttt ttttttt ttt!ttttttt ttttttt t    tttttt ttttt ttttt ttt tt t tttttt ttt t t !t 
t ttt tt tt ttttt t t 
   tttttt t tttt ttttt ttt ttt !t t  t t t t     tttttt ttttt ttttt ttt tt t tttttt ttt t t !t 
ttttt ttt tt tt ttttt tt tt
    tttttt t tttt ttttt ttt ttt t !t t  t t t  2  t ttt ttt tttt ttt
ttttt ttt tt tt ttttt 2 t !t ttt ttt tttt t ttt ttttttttt tttt tttt tttttt tttt tt
   tttttt t tttt ttttt ttt ttt t  t !t t t t  tttt 
ttttt ttt tt tt ttttt 22  t tttttt  t ttt tttt ttttt tttt
   tttttt t tttt ttttt ttt ttt t  t !t t t t 
ttttt ttt tt tt ttttt
276 / Magical Book on Puzzle

Chapter 8

Questions Asked In Exams

Puzzle 1
Directions: Study the following information carefully and answer
the questions given below:
Three ladies and four men are a group of friends, ie R, M, T, S, L, W and
Z. Each one has a different profession, ie Lawyer, Travel Agent, Air-hostess,
Doctor, Professor, Consultant and Jeweller and each one owns a different car,
ie Alto, Corolla, Santro, Lancer, Ikon, Scorpio and Esteem, not necessarily in
that order. None of the ladies is a Consultant or a Lawyer. T is an Air-hostess
and she owns an Ikon car. R owns a Scorpio. M is not a Doctor. L is a Jeweller
and he owns Corolla. W is a Lawyer and does not own Alto. Z is a Consultant
and owns Santro. The Doctor owns Esteem car whereas the Professor owns
Scorpio. The Travel Agent owns an Alto. None of the ladies own a Scorpio.
Questions:
1. What car does S own?
1) Alto 2) Santro 3) Lancer
4) Esteem 5) None of these
2. Who owns the car Lancer?
1) Z 2) M 3) W

K KUNDAN
4) Data Inadequate

4) Data inadequate
4. Who is the Doctor?
1) R
4) Data inadequate
5) None of these
3. What is the profession of R?
1) Professor 2) Travel Agent
5) None of these

2) S
5) None of these
3) Doctor

3) L

5. Who are the three ladies in the group?


1) T, R, L 2) T, M, S 3) W, T, M
4) Data inadequate 5) None of these
[PNB Management Trainee Exam–2005]

Puzzle 2
Directions: Study the following information carefully and answer
the questions given below:
Seven people N, K, T, B, M, W and R have their weekly offs on different
days of the week, ie Sunday, Monday, Tuesday, Wednesday, Thursday, Friday
and Saturday, not necessarily in that order. Each of them has a liking for
different cuisine, ie Indian, Italian, Mexican, Chinese, Spanish, Continental
and Thai, not necessarily in that order. K likes Thai food and gets his weekly
off on Thursday. B likes Italian food and does not have off on Sunday. M has
weekly off on Saturday and R has his weekly off on Tuesday. W likes continental
food whereas the one who has weekly off on Monday likes Mexican cuisine. T
does not like Spanish cuisine and has weekly off on Wednesday. The one who
likes Indian food does not have a weekly off on Tuesday or Wednesday.
Questions:
1. Who has a weekly off on Friday?
Questions Asked In Exams / 277

1) T 2) R 3) W
4) Data inadequate 5) None of these
2. What cuisine does R like?
1) Continental 2) Indian 3) Italian
4) Spanish 5) None of these
3. On which day does N have weekly off?
1) Tuesday 2) Friday 3) Monday
4) Sunday 5) None of these
4. Who likes Chinese cuisine?
1) T 2) B 3) R
4) N 5) None of these
5. On which day does W have weekly off?
1) Monday 2) Sunday 3) Wednesday
4) Data inadequate 5) None of these
[PNB Management Trainee Exam–2005]

Puzzle 3
Directions: Study the following information carefully and answer
the questions given below:
(i) P, Q, R, S and T finished a work, working from Monday to Saturday,
one of the days being a holiday, each working overtime only on one of
the days.
(ii) R or T did not work overtime on the first day.
(iii) Q worked overtime the next day after the holiday.
(iv) The overtime work done on the previous day of the holiday was by R.

K KUNDAN
(v) There was a two days’ gap between the days on which P and Q worked

Questions:
over time.
(vi) P worked overtime the next day of the overtime day of S.

1. Which of the following is the correct statement ?


1) P worked overtime last among them.
2) P worked overtime earlier than S.
3) The holiday was on Friday.
4) S worked overtime earlier than Q.
5) None of these
2. On which day did R work overtime?
1) Monday 2) Tuesday 3) Thursday
4) Friday 5) None of these
3. How many days’ gap was there between the days on which P and T worked
overtime?
1) Three 2) Two 3) One
4) Cannot be determined 5) None of these
4. When did T work overtime?
1) On the day previous to that on which S worked overtime
2) On the next day of the day on which Q worked overtime
3) Two days after the day on which S worked overtime
4) Cannot be determined
5) None of these
[Andhra Bank PO Exam–2005]

Puzzle 4
Directions: Study the following information carefully and answer
the questions given below:
278 / Magical Book on Puzzle

Seven friends P, F, R, T, Q, N and D are studying different specialisations


IT, Civil, HRM, Marketing, Finance, Journalism and Pharmacy, not necessarily
in the same order. Each one of them has a liking for a different colour—red,
blue, green, yellow, pink, orange and grey—not necessarily in the same order.
Three of them are girls. P likes yellow colour but does not study IT or HR. The
one who studies Civil likes grey colour and is a girl. Q, who is the sister of N,
studies Marketing and likes pink colour. D’s specialisation is in Pharmacy
and likes red colour. N, the wife of R, studies HR and likes green. F likes grey
and R likes orange. The one who likes blue studies Finance.
Questions:
1. Who is studying Civil Engineering?
1) P 2) T 3) F
4) Cannot be determined 5) None of these
2 Which of the following is the group of girls?
1) F, D, N 2) F, Q, N 3) Q, N, P
4) Cannot be determined 5) None of these
3. Which subject is studied by R?
1) Civil 2) Finance
3) Journalism 4) Cannot be determined
5) None of these
4. Who is studying Journalism?
1) P 2) Q
3) R 4) Cannot be determined
5) None of these
5. Which of the following combinations of person – colour — subject is correct?

K KUNDAN
1) Blue–T–Marketing 2) Pink–N–HR
3) Orance–R–Civil 4) Blue–T–Finance
5) None of these
[UBI PO Exam–2005]

Puzzle 5
Directions: Study the following information carefully and answer
the questions given below:
P, Q, R, S, T, V, W and Z are eight friends studying in three different
engineering colleges - A, B and C in three disciplines - Mechanical, Electrical
and Electronics with not less than two and not more than three in any college.
Not more than three of them study in any of the three disciplines. W studies
Electrical in college B with only T, who studies Mechanical. P and Z do not
study in college C and study in the same discipline but not Electrical. R studies
Mechanical in college C with V, who studies Electrical. S studies Mechanical
and does not study in the same college where R studies. Q does not study
Electronics.
Questions:
1. Which of the following combinations of college-student-specialisation is
correct?
1) C-R-Electronics 2) A-Z-Electrical 3) B-W-Electronics
4) B-W-Electrical 5) B-Z-Electronics
2. In which of the following colleges do two students study in Electrical
discipline?
1) A only 2) B only 3) C only
4) Cannot be determined 5) None of these
3. In which discipline does Q study?
1) Electrical 2) Mechanical 3) Electrical or Mechanical
Questions Asked In Exams / 279

4) Data inadequate 5) None of these


4. In which of the colleges at least one student studies in Mechanical
discipline?
1) A only 2) B only 3) C only
4) Both A and B 5) All A, B and C
5. S studies in which college?
1) A 2) B 3) A or B
4) Data inadequate 5) None of these
[Corporation Bank PO Exam–2006]

Puzzle 6
Directions: Study the following information carefully and answer
the questions given below:
A, B, C, D, E, F, G and H are eight employees working in three departments
- Marketing, Finance and Production - in an organisation with at least two of
them in any department. Each of them has a different choice of TV channels
from Star, Zee, ESPN, DD, Sony, NDTV, Aaj Tak and BBC not necessarily in
the same order.
D likes ESPN and he works in Production department with only G. B and
F do not work in the same department. H likes DD and does not work in
Finance department. A does not work in the same department with either F or
C, who does not work in Marketing department. E works with C in the same
department and likes Star. B likes Aaj Tak and none of his colleagues in the
department likes either Sony or NDTV. G likes Zee. F does not like Sony.
Questions:

K KUNDAN
1. Which channel does A like?
1) DD
4) Data inadequate
2) NDTV
5) None of these
2. In which department does F work?
1) Marketing
4) Marketing or Finance
2) Production

3. Which channel does F like?


3) BBC

3) Finance
5) None of these

1) Zee 2) NDTV 3) BBC


4) Data inadequate 5) None of these
4. Which of the following groups work in Marketing department?
1) BAC 2) BGE 3) HBE
4) BAE 5) None of these
5. In which department does B work?
1) Marketing 2) Production 3) Finance
4) Data inadequate 5) None of these
[Central Bank of India PO Exam–2006]

Puzzle 7
Directions: Study the following information carefully and answer
the questions given below:
A private bank deputed eight of its managers P, T, D, E, J, Q, M and R to
different cities Bangalore, Delhi, Chennai, Hyderabad, Jaipur, Cochin, Pune
and Ahmedabad for marketing of three different products X, Y and Z. The
order of managers, cities and products is not necessarily the same. Each product
is marketed in not less than two cities and not more than three cities.
T goes to Delhi for marketing of product Y. J goes to Hyderabad for marketing
of product Z. E does not go to Bangalore or Jaipur and markets the same
product as M, who goes to Ahmedabad. R goes to Chennai for marketing of
280 / Magical Book on Puzzle

product Z. Same product is marketed in Bangalore and Delhi. E and Q are


marketing the same product. P goes to Pune for marketing of product Z.
Questions:
1. Which of the following groups of managers go for marketing product ‘X’?
1) E, Q, R 2) J, Q, M 3) J, M, R
4) Q, M, R 5) None of these
2. E goes to which of the following cities?
1) Jaipur 2) Cochin
3) Ahmedabad 4) Cannot be determined
5) None of these
3. Which of the following combination of product, manager and city is correct?
1) X - Q - Jaipur 2) X - Q - Cochin 3) X - E - Bangalore
4) Y - D - Cochin 5) None of these
4. Which of the following persons go for marketing of product Y?
1) Q, D, M 2) Q, T 3) T, D, Q
4) T, D 5) None of these
5. Who goes to Bangalore?
1) Q 2) M 3) D
4) D or Q 5) None of these
[Andhra Bank PO Exam–2006]

Puzzle 8
Directions: Study the following information carefully and answer
the questions given below:
(i) A school held competitions for Chess, Table Tennis, Carrom, Kho-

K KUNDAN
Kho and Volleyball during the annual sports week from Monday to
Saturday, each game on one day, one day being a rest day.
(ii) Carrom competition was held not on the first or on the last day but
was held earlier than Table Tennis competition.
(iii) Kho-Kho competition was held on the immediate next day of the Table
Tennis competition day.
(iv) Chess competition was held on the immediate previous day of the rest
day.
(v) Kho-Kho competition day and Volleyball competition day had a two-
day gap between them.
(vi) Volleyball competition was held on the immediate following day of the
rest day.
Questions:
1. Which of the following was a rest day?
1) Wednesday 2) Tuesday 3) Friday
4) Thursday 5) None of these
2. Kho-Kho and Carrom competition days had a gap of how many days between
them?
1) Nil 2) Two 3) Three
4) Four 5) None of these
3. On which day was the Chess competition held?
1) Thursday 2) Friday 3) Monday
4) Wednesday 5) None of these
4. Which of the following is a wrong statement?
Questions Asked In Exams / 281

1) Carrom competition was held on the immediate previous day of Table


Tennis competition.
2) Kho-Kho competition was held on three days after the day on which
Volleyball competition was held.
3) There was a gap of three days between the days on which Chess and
Table Tennis competitions were held.
4) There was a two days’ gap between the rest day and the day on which
Carrom competition was held.
5) None of these
5. Which of the following is the correct statement?
1) Kho-Kho competition was held after Table Tennis competition.
2) Chess competition was held on Thursday.
3) No competition was held on Wednesday.
4) Table Tennis competition was held earlier than Chess competition.
5) None of these
[IDBI PO Exam–2005]

Puzzle 9
Directions: Study the following information carefully and answer
the questions given below:
Eight members A, B, C, D, E, F, G and H belonging to three families X, Y,
Z go for weekend outing in three different cars I, II, III. Four out of the eight
members are females. Members of any one family travel in different cars. Each
car has at least one male and one female member. Each family has at least two
members.

K KUNDAN
A belongs to family Y and he travels in car III. D is wife of E and they
travel in cars I and II respectively. H is son of B, who is wife of G, and they
belong to family Z. C is daughter of F, who is wife of A. C travels in car II. G
does not travel with F.
Questions:
1. Which of the following groups of persons travels in car I?
1) D, F, G 2) D, E, G 3) D, G, H
4) D, F, H 5) None of these
2. Which car has only two members travelling in it?
1) I 2) II 3) III
4) II or III 5) Cannot be determined
3. Which of the following members of families Y and Z travel in different
cars?
1) F, G 2) C, G 3) F, H
4) C, F 5) None of these
4 Which of the following groups of persons is a group of all females?
1) B, D, G 2) A, B, C 3) B, E, F
4) D, E, F 5) None of these
5. Which of the following members of families X and Y travel in the same
car?
1) C, F 2) D, F 3) C, D
4) F, E 5) None of these
[Bank of Baroda PO Exam–2007]

Puzzle 10
Directions: Study the following information carefully and answer
the questions given below:
282 / Magical Book on Puzzle

Seven officers L, M, N, P, Q, R & S work in three different shifts I, II & III


with at least two persons working in each shift. Each one of them has a
different weekly off from Monday to Sunday not necessarily in the same order.
M works in second shift only with R, whose weekly off is on Friday. Q’s
weekly off is on the next day of L’s weekly off and both of them work in
different shifts. P works in third shift and his weekly off is on Saturday. S has
a weekly off on Monday and he works in first shift. The one who has a weekly
off on Sunday works in first shift. L and P do not work in the same shift. L’s
weekly off is on Tuesday.
Questions:
1. Whose weekly off is on Sunday?
1) L 2) M 3) N
4) Q 5) None of these
2. Which of the following group of officers works in shift I?
1) L, N, S 2) L, S 3) N, S
4) L, P, Q 5) None of these
3. On which day is Q’s weekly off ?
1) Tuesday 2) Wednesday 3) Sunday
4) Cannot be determined 5) None of these
4. Which of the following combinations of shift, person and weekly off is
definitely correct?
1) II, M, Sunday 2) III, N, Sunday 3) II, P, Sunday
4) I, L, Tuesday 5) None of these
5. Whose weekly off falls on Thursday?
1) L 2) N 3) Q

K KUNDAN
4) Cannot be determined 5) None of these
[Bank of Maharashtra PO Exam–2007]

Puzzle 11
Directions: Study the following information carefully and answer
the questions given below:
Eight family members Dhruv, Garima, Avinash, Varsha, Aakash, Deepti,
Charu and Moksh are sitting around a square table in such a way that two
persons sit on each of the four sides of the table facing the centre. Members
sitting on opposite sides are exactly opposite each other.
Aakash and Garima are exactly opposite each other. Deepti is immediately
right to Garima. Dhruv and Moksh are sitting on the same side. Moksh is
exactly opposite Avinash, who is to the immediate left of Varsha. Dhruv is
towards right of Deepti.
Questions:
1. Which of the following statements is definitely true?
1) Charu is opposite Varsha.
2) Deepti is to the left of Garima.
3) Avinash is towards the right of Aakash.
4) Moksh is sitting opposite Dhruv.
5) None of these
2. Who is sitting opposite Dhruv?
1) Charu 2) Deepti 3) Varsha
4) Moksh 5) None of these
3. Who is sitting opposite Deepti?
1) Moksh 2) Charu 3) Varsha
4) Charu or Varsha 5) None of these
4. Who is next to Varsha in anti-clockwise direction?
Questions Asked In Exams / 283

1) Garima 2) Avinash 3) Dhruv


4) Deepti 5) None of these
5. Which of the following pairs of persons has both the persons sitting on
the same side with first person sitting to the left of second person?
1) Aakash — Charu 2) Moksh — Charu 3) Dhruv — Aakash
4) Avinash — Charu 5) None of these
[Andhra Bank PO Exam–2007]

Puzzle 12
Directions: Study the following information carefully and answer
the questions given below:
Seven professionals A, B, C, D, E, F and G are practising their professions
in different cities Chennai, Bangalore, Hyderabad, Mumbai, Ahmedabad, Jaipur
and Bhubaneshwar, not necessarily in the same order. Each has a different
profession –Doctor, Engineer, Pharmacist, Lawyer, Counsellor, Professor and
Artist, not necessarily in the same order.
A is a Pharmacist and practises in Bhubaneshwar. D practises in Bangalore
but is not a Doctor or an Artist. The one who practises in Hyderabad is a
Professor. G is a Counsellor and does not practise in Mumbai or Chennai. E is
a Lawyer and practises in Ahmedabad. F practises in Chennai but is not an
artist. C practises in Mumbai.
Questions:
1. What is D’s profession?
1) Doctor 2) Professor 3) Engineer
4) Cannot be determined 5) None of these

K KUNDAN
2. Who is the Professor?
1) B
4) E

3) Doctor - Bangalore
5) None of these
2) C
5) None of these
3) D

3. Which of the following combinations of profession and place is correct?


1) Pharmacist - Jaipur 2) Engineer - Chennai
4) Artist - Mumbai

4. Which of the following persons works in Jaipur?


1) B 2) G 3) C
4) B or G 5) None of these
5. Who is the Doctor?
1) D 2) B 3) C
4) B or C 5) None of these
[NABARD PO Exam–2008]

Puzzle 13
Directions: Study the following information carefully and answer
the questions given below:
Seven candidates Harish, Samir, Nilesh, Shailaja, Nikita, Laxman and
Sujata are to be interviewed for selection as Trainee Officers by different panels
I to VII for different companies A, B, C, D, E, F and G not necessarily in the
same order.
Nilesh is interviewed by panel IV for Company A. Samir is interviewed by
panel III but not for company C or D. Harish is interviewed for company B but
not by panel I or II. Nikita is interviewed by panel VI for company E. Panel VII
conducts the interview for company F. Shailaja is interviewed by panel I but
not for company C. Panel II does not interview Laxman.
Questions:
284 / Magical Book on Puzzle

1. Shailaja is interviewed for which company?


1) A 2) G 3) F
4) D 5) None of these
2. Panel II conducts interview for which company?
1) C 2) F 3) G
4) B 5) None of these
3. Who is interviewed for company G?
1) Nikita 2) Samir 3) Shailaja
4) Laxman 5) None of these
4. Who is interviewed for company F?
1) Shailaja 2) Sujata 3) Laxman
4) Cannot be determined 5) None of these
5. Which candidate is interviewed by panel V?
1) Harish 2) Laxman 3) Sujata
4) Shailaja 5) None of these
[Bank of Maharashtra PO Exam–2008]

Puzzle 14
Directions: Study the following information carefully and answer
the questions given below:
P, Q, R, S, T, V and W are seven friends working in a call centre. Each of
them has different day offs in a week from Monday to Sunday not necessarily
in the same order. They work in three different shifts I, II and III with at least
two of them in each shift.
R works in shift II and his day off is not Sunday. P’s day off is Tuesday

K KUNDAN
and he does not work in the same shift with either Q or W. None of those who
work in shift I has day off either on Wednesday or on Friday. V works with
only T in shift III. S’s day off is Sunday. V’s day off is immediate next day of
that of R’s day off. T’s day off is not on Wednesday. W’s day off is not on the
previous day of P’s day off. S works in shift I. Q does not work in the same
shift with R and his day off is not on Thursday.
Questions:
1. Which of the following is W’s day off?
1) Tuesday 2) Monday 3) Saturday
4) Data inadequate 5) None of these
2. Which of the following is R’s day off ?
1) Friday 2) Thursday 3) Tuesday
4) Wednesday 5) None of these
3. Which of the following groups of friends work in shift II?
1) RP 2) RV 3) QWS
4) Data inadequate 5) None of these
4. Which of the following is Q’s day off ?
1) Friday 2) Wednesday 3) Thursday
4) Monday 5) None of these
5. Which of the following groups of friends work in shift I?
1) RV 2) RP 3) QWS
4) Data inadequate 5) None of these
[Vijaya Bank PO Exam–2008]

Puzzle 15
Directions: Study the following information carefully and answer
the questions given below:
M, N, P, R, T, W, F and H are sitting around a circle facing the centre. P
Questions Asked In Exams / 285

is third to the left of M and second to the right of T. N is second to the right of
P. R is second to the right of W, who is second to the right of M. F is not an
immediate neighbour of P.
Questions:
1. Who is to the immediate right of P?
1) H 2) F 3) R
4) Data inadequate 5) None of these
2. Who is to the immediate right of H?
1) R 2) F 3) M
4) Data inadequate 5) None of these
3. Who is to the immediate left of R?
1) P 2) H 3) W
4) T 5) Data inadequate
4. Who is third to the right of H?
1) T 2) W 3) R
4) F 5) Data inadequate
5. Who is second to the right of F?
1) M 2) R 3) T
4) Data inadequate 5) None of these
6. In which of the following is the first person sitting in between the second
and the third person?
1) NHM 2) PHN 3) TRP
4) TWF 5) None of these
[Union Bank of India PO Exam–2008]

K KUNDAN
Puzzle 16
Directions: Study the following information carefully and answer
the questions given below:
A, B, C, D, E, F, G and H are sitting around a circle facing the centre. F
is third to the right of C and second to the left of H. D is not an immediate
neighbour of C or H. E is on the immediate right of A, who is second to the
right of G.
Questions:
1. Who sits between G and D?
1) H 2) D 3) F
4) E 5) None of these
2. Which of the following is the correct position of B with respect to H?
I. Second to the right
II. Fourth to the right
III. Fourth to the left
IV. Second to the left
1) Only I 2) Only II 3) Only III
4) Both II & III 5) None of these
3. Who is second to the left of C ?
1) A 2) B 3) E
4) D 5) None of these
4. Which of the following pairs of persons has first person sitting to the
right of the second person?
1) CB 2) AE 3) FG
4) HA 5) DB
5. Who is on the immediate right of C?
1) E 2) B 3) D
4) B or D 5) None of these
286 / Magical Book on Puzzle

[State Bank of India PO (Prelims) Exam-2008]

Puzzle 17
Directions: Study the following information carefully and answer
the questions given below:
Seven members L, H, K, T, F, J and R represent different countries in
Olympics, viz, USA, China, Korea, France, Russia, Australia and Japan; each
one competes for a different sport, viz. Volleyball, Archery, Rifle Shooting,
Tennis, Boxing, Athletics and Football. The order of persons, countries and
games is not necessarily the same.
K represents China for Archery. T represents USA but not for Volleyball or
Rifle Shooting. The one who represents Japan competes for Boxing. F competes
for Volleyball but not for Korea. L represents Australia for Athletics. The one
who represents Russia competes for Tennis. J does not represent Korea or
Japan. R competes for Rifle Shooting.
Questions:
1. Which of the following combinations is correct?
1) J - Tennis - France 2) R-Tennis - Russia
3) R - Tennis - France 4) J - Tennis - Russia
5) None of these
2. Who represents Japan?
1) F 2) R 3) J
4) H 5) None of these
3. F represents which country?
1) France 2) Russia 3) Japan

K KUNDAN
4) Korea 5) None of these
4. The one who competes for Rifle Shooting, represents which country?
1) France 2) Korea 3) Japan
4) USA 5) None of these
5. For which game does T compete?
1) Boxing 2) Football
3) Tennis 4) Cannot be determined
5) None of these
[State Bank of India PO (Prelims) Exam-2008]

Puzzle 18
Directions: Study the following information carefully and answer
the questions given below:
Ashwini, Priya, Sudha, Rani, Meeta, Geeta and Mukta are sitting around a
circle facing the centre. Ashwini is third to the left of Mukta and to the
immediate right of Rani. Priya is second to the left of Geeta, who is not an
immediate neighbour of Meeta.
Questions:
1. Who is to the immediate right of Priya?
1) Meeta 2) Sudha 3) Mukta
4) Cannot be determined 5) None of these
2. Who is second to the left of Rani?
1) Ashwini 2) Meeta 3) Priya
4) Sudha 5) None of these
3. Which of the following pairs of persons has the first person sitting to the
immediate left of the second person?
Questions Asked In Exams / 287

1) Rani-Meeta 2) Ashwini-Geeta 3) Sudha-Priya


4) Geeta-Sudha 5) None of these
4. Which of the following groups has the first person sitting between the
other two?
1) Meeta-Ashwini-Geeta 2) Sudha-Rani-Geeta
3) Mukta-Priya-Rani 4) Mukta-Priya-Sudha 5) None of these
5. Which of the following is the correct position of Rani with respect to
Mukta?
I. Third to the right
II. Third to the left
III. Fourth to the left
IV. Fourth to the right
1) I only 2) II only 3) Both I & II
4) Both II & IV 5) Both I & III
[Bank of Baroda PO Exam-2008]

Puzzle 19
Directions: Study the following information carefully and answer
the questions given below:
A, B, C, D, E, F and G are members of a sports club and have liking for
different games, viz Carrom, Table Tennis, Badminton, Bridge, Hockey, Football
and Lawn Tennis but not necessarily in the same order. Each one of them has
a liking for different musical instruments, viz Sitar, Guitar, Harmonium, Flute,
Tabla, Banjo and Santoor, not necessarily in the same order.
B likes Carrom and Banjo. E likes to play Bridge but not Harmonium or

K KUNDAN
Tabla. The one who plays Hockey plays Sitar. F plays Guitar but not Table
Tennis or Lawn Tennis. A plays Badminton and Flute. The one who plays
Lawn Tennis does not play Tabla. C plays Harmonium and G plays Hockey.
Questions:
1. Who plays Santoor?
1) D
4) D or E
2) A
5) None of these
3) E

2. D plays which game?


1) Table Tennis 2) Lawn Tennis
3) Football 4) Cannot be determined
5) None of these
3. Which of the following combinations of game-person-musical instrument
is definitely correct?
1) Badminton - B - Flute
2) Table Tennis - E - Santoor
3) Lawn Tennis - D - Tabla
4) Table Tennis - C - Tabla
5) None of these
4. Who plays Football?
1) C 2) D 3) G
4) F 5) None of these
5. Who plays Table Tennis?
1) C 2) F
3) D 4) Cannot be determined
5) None of these
[Bank of Baroda PO Exam-2008]

Puzzle 20
288 / Magical Book on Puzzle

Directions: Study the following information carefully and answer


the questions given below:
A, B, C, D, E, F, G and H are sitting around a circle, facing the centre. F
sits to the immediate right of D and third to the left of A. G sits third to the left
of D who does not sit next to E. B sits next to G but not next to D. C does not
sit next to either D or A.
Questions:
1. Who sits to the immediate left of A?
1) E 2) F 3) G
4) H 5) None of these
2. What is the position of H with respect to C?
1) Second to the left 2) First to the right 3) Third to the right
4) Second to the right 5) None of these
3. Which of the following pairs sits between G and D?
1) AC 2) DF 3) HB
4) FA 5) None of these
4. Starting from A’s position, if all the eight are arranged in alphabetical
order in clockwise direction, the seating position of which of the following
(excluding A) would not change?
1) B 2) C 3) D
4) H 5) None of these
5. Four of the following are alike based upon their seating arrangement around
the circle. Which is the one that does not belong to that group?
1) FH 2) GE 3) CD
4) BG 5) EF

K KUNDAN
[Bank of Baroda (Agriculture Officer) Exam-2008]

Puzzle 21
Directions: Study the following information carefully and answer
the questions given below:
P, Q, R, S, T, W and Z are seven students studying in three different
institutes - A, B and C. There are three girls among the seven students who
study in each of the three institutes. Two of the seven students study BCA,
two study medicine and one each studies Aviation Technology, Journalism
and MBA. R studies in the same college as P, who studies MBA in college B.
No girl studies Journalism or MBA. T studies BCA in college A and his brother
W studies Aviation Technology in college C. S studies Journalism in the same
college as Q. Neither R nor Z studies BCA. The girl who studies BCA does not
study in college C.
Questions:
1. Which of the following pairs of students study medicine?
1) QZ 2) WZ 3) PZ
4) SZ 5) None of these
2. In which college does Q study?
1) A 2) B 3) C
4) Data inadequate 5) None of these
3. In which of the colleges do three of them study?
1) A 2) B 3) A and B
4) C 5) None of these
4. What is the field of study of Z?
1) Aviation Technology 2) BCA
3) MBA 4) Medicine
Questions Asked In Exams / 289

5) None of these
5. Which of the following three represents girls?
1) SQR 2) QRZ 3) SQZ
4) Data inadequate 5) None of these
[Bank of Baroda (Agriculture Officer) Exam-2008]

Puzzle 22
Directions: Study the following information carefully and answer
the questions given below:
M, D, P, K, R, T and W are sitting around a circle facing the centre. D is
second to the right of P, who is third to the right of K. T is third to the right of
W, who is not an immediate neighbour of D. M is third to the left of R.
Questions:
1. Who is second to the right of T?
1) D 2) K 3) M
4) Data inadequate 5) None of these
2. In which of the following pairs is the second person sitting to the immediate
right of the first person?
1) DT 2) TP 3) PR
4) KW 5) None of these
3. Who is on the immediate left of R?
1) W 2) P 3) K
4) T 5) None of these
4. Who is on the immediate left of M?
1) K 2) W 3) D

K KUNDAN
4) T

4) Data inadequate

Puzzle 23
5) None of these
5. Who is third to the left of D?
1) W 2) P
5) None of these
3) K

[Oriental Bank of Commerce (PO) Exam-2008]

Directions: Study the following information carefully and answer


the questions given below:
P, Q, R, S, T, U, V and W are sitting around a circular table, facing the
centre. P sits third to the right of W and third to the left of Q. S sits second to
the right of T. V sits second to the left of R. T is not the neighbour of Q while
U is neither a neighbour of T nor of W.
Questions:
1. Who sits second to the left of V?
1) R 2) P 3) U
4) T 5) None of these
2. Who sits between U and P?
1) S 2) R 3) V
4) Q 5) None of these
3. Starting from P’s position, if all the eight are arranged in alphabetical
order in clockwise direction, the seating position of how many members
(excluding P) would not change?
1) None 2) One 3) Two
4) Three 5) Four
4. Which of the following pairs has only one person sitting between them, if
the counting is done in the clockwise direction?
1) T, V 2) V, Q 3) W, P
290 / Magical Book on Puzzle

4) R, P 5) None of these
5. Four of the following are alike in a certain way based on their positions in
the seating arrangement and so form a group. Which is the one that does
not belong to that group?
1) W,T 2) P, U 3) S, Q
4) R, P 5) P, Q
[Punjab National Bank (Management Trainee) Exam-2009]

Puzzle 24
Directions: Study the following information carefully and answer
the questions given below:
A group of friends having seven members A, B, C, D, E, F and G contains
four men and three ladies. Each one of them has a different profession—
stockbroker, lawyer, doctor, professor, engineer, businessman and banker—
and each one has passed out of a different college—P, S, V, W, X, Y and Z, not
necessarily in the same order. None of the ladies is a businessman or a
stockbroker. C is a doctor and she has passed out from ‘College X’. A is a
‘College Y’ passout. B is not a professor. E is a banker and is a ‘College S’
passout. F is a stockbroker and has not studied in ‘College P’.
G is a businessman and has studied in ‘College V’. The professor is a
‘College Z’ passout. The lawyer has studied in ‘College P’. None of the ladies
has studied in ‘College Y’ or ‘College S’.
Questions:
1. What is the profession of D?
1) Doctor 2) Stockbroker 3) Engineer

K KUNDAN
4) Professor 5) None of these
2. Who among the following is a lawyer?
1) A 2) B 3) E
4) G 5) None of these
3. Which of the following groups represents ladies in the group of friends?
1) A, B, C 2) E, F, G 3) B, C, D
4) B, E, G 5) None of these
4. From which of the following colleges has the stockbroker passed out?
1) W 2) Y 3) S
4) X 5) None of these
5. Which of the following combinations is correct?
1) B-Doctor-Female 2) C-W-Male 3) A-Businessman-Y
4) D - Professor - Male 5) None of these
[Punjab National Bank (Management Trainee) Exam-2009]

Puzzle 25
Directions: Study the following information carefully and answer
the questions given below:
Eight friends A, B, C, D, E, F, G and H are sitting around a circle facing
the centre. E is third to the left of G, who is on the immediate right of B, who
is third to the left of A. H is second to the right of F, who is not an immediate
neighbour of E. D is not an immediate neighbour of B.
Questions:
1. Who is second to the right of B ?
1) F 2) A 3) H
4) D 5) None of these
2. Which of the following pairs has the first person on the immediate left of
the second person?
Questions Asked In Exams / 291

1) GB 2) AF 3) CE
4) HD 5) None of these
3. Which of the following is the correct position of B with respect to D?
1) Second to the right 2) Second to the left 3) Third to the right
4) Third to the left 5) None of these
4. Who sits between A and D?
1) F 2) E 3) G
4) B 5) H
5. What is E’s position with respect to C?
1) On the immediate right 2) On the immediate left
3) Second to the right 4) Cannot be determined
5) None of these
[Canara Bank PO Exam-2009]

Puzzle 26
Directions: Study the following information carefully and answer
the questions given below:
Seven members H, I, J, K, L, M and N are working in different cities
Ahmedabad, Bangalore, Chennai, Hyderabad, Kolkata, Delhi and Mumbai, not
necessarily in the same order. Each one has a different mother tongue–Tamil,
Kannada, Telugu, Hindi, Marathi, Punjabi and Bangla, not necessarily in the
same order.
J works in Bangalore and his mother tongue is not Tamil or Marathi. K’s
mother tongue is Punjabi and he works in Ahmedabad. L and M do not work in
Chennai and none of them has Marathi mother tongue. I works in Hyderabad

K KUNDAN
and his mother tongue is Telugu. The one who works in Delhi has Bangla
mother tongue. N works in Mumbai and his mother tongue is Hindi. L does not
work in Kolkata.
Questions:
1. What is J’s mother tongue?
1) Telugu
4) Kannada
2) Hindi
5) None of these
3) Bangla

2. Who works in Chennai?


1) H 2) L 3) M
4) L or M 5) None of these
3. Which of the following combinations is correct?
1) Marathi - I - Hyderabad 2) Tamil - M - Kolkata
3) Marathi - I - Chennai 4) Punjabi - K - Delhi
5) None of these
4. Who works in Delhi?
1) H 2) M 3) L
4) K 5) None of these
5. What is M’s mother tongue?
1) Bangla 2) Marathi 3) Telugu
4) Cannot be determined 5) None of these
[Canara Bank PO Exam-2009]

Puzzle 27
Directions: Study the following information carefully and answer
the questions given below:
P, Q, R, S, T, V and W are sitting around a circle facing at the centre. V is
second to the left of P and second to the right of W. T is third to the right of Q
and is not an immediate neighbour of V. S is third to the right of R.
292 / Magical Book on Puzzle

Questions:
1. Who is second to the right of Q?
1) R 2) W 3) T
4) S 5) None of these
2. Who is to the immediate left of S?
1) V 2) T 3) Q
4) W 5) None of these
3. Who is to the immediate right of R?
1) W 2) T 3) P
4) Data inadequate 5) None of these
4. In which of the following groups is the first person sitting between the
second and the third persons?
1) RPQ 2) TWS 3) QPR
4) QVS 5) None of these
5. Who is third to the left of V ?
1) T 2) S 3) W
4) R 5) None of these
[Indian Overseas Bank PO Exam-2009]

Puzzle 28
Directions: Study the following information carefully and answer
the questions given below:
A, B, C, D, E, F, G and H are eight friends sitting around a circular table
facing the centre. A sits second to the left of D, who is third to the left of E. C
sits third to the right of G, who is not an immediate neighbour of E. H sits

K KUNDAN
third to the right of B, who sits second to the right of G.
Questions:
1. Who sits between D and C?
1) Only B 2) Only C and A 3) Only G
4) Only E 5) Only G and E
2. Who sits second to the right of E?
1) B 2) F 3) G
4) C 5) None of these
3. What is the position of A with respect to H?
1) Third to the left 2) Third to the right 3) Second to the left
4) Second to the right 5) Fourth to the left
4. Four of the following five are alike based upon their seating arrangements
and so form a group. Which is the one that does not belong to that group?
1) CH 2) FG 3) DA
4) BE 5) GB
5. Which of the following pairs has the second person sitting to the immediate
left of the first person?
1) DB 2) EH 3) FA
4) GD 5) None of these
[NABARD Bank Officer’s Exam-2009]

Puzzle 29
Directions: Study the following information carefully and answer
the questions given below:
A, B, C, D, E, F and G are seven friends studying seven different branches
of engineering, namely Mechanical, Chemical, Electrical, Electronics, Civil,
Computer and Aeronautical Engineering, not necessarily in this order. Each
of them studies in three different colleges, X, Y and Z. Not less than two study
Questions Asked In Exams / 293

in any college. D studies Electrical engineering in College X. The one who


studies Chemical Engineering does not study in college Z. F studies Aeronautical
Engineering in college Y with only B. A does not study in college X and does
not study Civil Engineering. E studies Computer Engineering and does not
study in college X. G studies Electronics Engineering but not in college X.
None in college X studies Mechanical or Civil Engineering.
Questions:
1. Which of the following groups represents the persons studying in college
Z?
1) D, B 2) C, E, G 3) A, G
4) G, E, A 5) None of these
2. In which of the following colleges does C study?
1) X 2) Y 3) Z
4) Either X or Z 5) Cannot be determined
3. Which of the following combinations is correct?
1) A–Civil–Z 2) B–Chemical–Y 3) C–Chemical–Z
4) G –Electronics –Y 5) None of these
4. B studies which of the following branches of engineering?
1) Chemical 2) Mechanical
3) Civil 4) Cannot be determined
5) None of these
5. Who studies Chemical engineering?
1) B 2) C 3) E
4) A 5) None of these
[NABARD Bank Officer’s Exam-2009]

K KUNDAN
Puzzle 30
Directions: Study the following information carefully and answer
the questions given below:
A, B, C, D, E, F, G and H are sitting around a circle facing the centre. B
is second to the right of H and third to the left of A. D is not an immediate
neighbour of either B or H and is second to the right of F. C is fourth to the
right of G.
Questions:
1. Who is to the immediate right of B ?
1) C 2) E 3) H
4) Data inadequate 5) None of these
2. Who is third to the right of E?
1) C 2) F 3) D
4) Data inadequate 5) None of these
3. In which of the following pairs is the second person sitting to the immediate
left of the first person?
1) BC 2) HE 3) FA
4) GD 5) None of these
4. Who is fourth to the right of H?
1) B 2) A 3) F
4) Data inadequate 5) None of these
5. If E and F interchange their places, who will be second to the right of B?
1) F 2) C 3) D
4) Data inadequate 5) None of these
[United Bank of India PO Exam-2009]
294 / Magical Book on Puzzle

Puzzle 31
Directions: Study the following information carefully and answer
the questions given below:
P, Q, R, S, T, V and W are seven students of a college. Each of them has
a favourite subject from Physics, Chemistry, English, Biology, History, Geography
and Philosophy, not necessarily in the same order. Each of them also has a
favourite sport from Football, Cricket, Hockey, Volleyball, Badminton, Table
Tennis and Basketball, not necessarily in the same order.
R likes Philosophy and his favourite sport is Hockey. The one who likes
Football likes English. T’s favourite sport is not Badminton or Table Tennis. V
does not like either History or Biology. The one whose favourite sport is Basketball
does not like Physics. W likes Chemistry and his favourite sport is Volleyball.
S likes Geography. Q’s favourite sport is Badminton. V does not like English
and his favourite sport is not Basketball. P’s favourite sport is Cricket. The one
whose favourite sport is Badminton does not like Biology.
Questions:
1. Which subject does P like?
1) History 2) Biology 3) Chemistry
4) Data inadequate 5) None of these
2. Who likes History?
1) P 2) R 3) Q
4) V 5) Data inadequate
3. What is Q’s favourite sport?
1) Cricket 2) Table Tennis 3) Football
4) Badminton 5) None of these

K KUNDAN
4. Whose favourite sport is Basketball?
1) S
4) Data inadequate
5. Which subject does T like?
1) Biology
4) Data inadequate
2) W
5) None of these

2) Physics
5) None of these
3) Q

3) Chemistry

[United Bank of India PO Exam-2009]

Puzzle 32
Directions: Study the following information carefully and answer
the questions given below:
P, Q, R, S, T and M are six students of a school, one each studies in Class
I-VI. Each of them has a favourite colour from red, black, blue, yellow, pink
and green, not necessarily in the same order.
Q likes black and does not study in Class IV or V. The one who studies in
Class IV does not like green. P studies in Class II. M likes blue and does not
study in Class IV. The one who likes yellow studies in Class VI. S likes pink
and studies in Class I. R does not study in Class VI.
Questions:
1. In which class does R study?
1) V 2) III 3) IV
4) Data inadequate 5) None of these
2. Which colour does R like?
1) Black 2) Yellow 3) Green
4) Blue 5) None of these
3. Which colour does P like?
1) Green 2) Yellow 3) Red
4) Data inadequate 5) None of these
Questions Asked In Exams / 295

4. Which of the following combinations is correct ?


1) P - II - Yellow 2) Q - III - Green 3) S - I - Black
4) T - V - Yellow 5) None of these
5. In which class does M study?
1) IV 2) III 3) II
4) V 5) None of these
[Andhra Bank PO Exam-2009]

Puzzle 33
Directions: Study the following information carefully and answer
the questions given below:
A, B, C, D, E, F, G and H are sitting around a circle facing the centre. H
is fourth to the left of B and second to the right of F. A is third to the left of C,
who is not an immediate neighbour of F. G is second to the left of A. D is
second to the right of E.
Questions:
1. Who is on the immediate right of F?
1) H 2) A 3) G
4) Data inadequate 5) None of these
2. Who is third to the left of A?
1) C 2) F 3) B
4) Data inadequate 5) None of these
3. In which of the following pairs is the first person sitting on the immediate
left of the second person?
1) EH 2) CE 3) AF

K KUNDAN
4) DB

4) Data inadequate

4) Data inadequate
5) None of these
4. Which of the following pairs represents the immediate neighbours of E?
1) DH 2) HC
5) None of these
5. Who is on the immediate right of H?
1) E 2) C
5) None of these
3) CA

3) H

6. Who is on the immediate right of B?


1) D 2) E 3) F
4) Data inadequate 5) None of these
[Andhra Bank Marketing Associate Exam-2009]

Puzzle 34
Directions: Study the following information carefully and answer
the questions given below:
A, B, C, D, E, F, G and H are sitting around a circle facing the centre. B
is third to the right of F and third to the left of H. C is fourth to the left of A,
who is not an immediate neighbour of F or B. E is not an immediate neighbour
of B. G is second to the right of D.
Questions:
1. Who is to the immediate left of B?
1) D 2) G 3) D or G
4) Data inadequate 5) None of these
2. Who is to the immediate right of H?
1) A 2) E 3) F
4) Data inadequate 5) None of these
3. Which of the following pairs represents the immediate neighbours of F?
1) CH 2) ED 3) HD
296 / Magical Book on Puzzle

4) CE 5) None of these
4. In which of the following pairs is the first person sitting to the immediate
right of the second person?
1) BG 2) GA 3) AH
4) HE 5) CF
5. Who is third to the left of E?
1) A 2) C 3) G
4) Data inadequate 5) None of these
[Bank of Maharashtra (Agriculture Officer) Exam–2009]

Puzzle 35
Directions: Study the following information carefully and answer
the questions given below:
P, Q, R, S, T, V and W are seven students of a school. Each of them
studies in a different standard—from Standard IV to Standard X—not necessarily
in the same order. Each of them has a favourite subject from English, Science,
History, Geography, Mathematics, Hindi and Sanskrit, not necessarily in the
same order.
Q studies in VII Standar d and does not like either Mathemat ics or
Geography. R likes English and does not study either in V or in IX. T studies
in VIII Standard and likes Hindi. The one who likes Science studies in X
Standard. S studies in IV Standard. W likes Sanskrit. P does not study in X
Standard. The one who likes Geography studies in V Standard.
Questions:
1. In which standard does W study?

K KUNDAN
1) VII 2) IX 3) X
4) Data inadequate 5) None of these
2. Which subject does P like?
1) Geography 2) Mathematics 3) English
4) History 5) None of these
3. Which subject does S like?
1) History 2) Geography 3) Mathematics
4) Data inadequate 5) None of these
4. In which standard does P study?
1) IV 2) VII 3) IX
4) X 5) None of these
5. Which of the following combinations of student-standard-subject is correct?
1) T - VIII - Mathematics 2) W - VII - Sanskrit
3) Q - VII - Geography 4) V - X - Science
5) None of these
[RBI Grade ‘B’ Officer’s Exam-2009]

Puzzle 36
Directions: Study the following information carefully and answer
the questions given below:
(i) A, B, C, D, E, F, G and H are eight students, each having a different
height,
(ii) D is shorter than A but taller than G.
(iii) E is taller than H but shorter than C.
(iv) B is shorter than D but taller than F.
(v) C is shorter than G.
(vi) G is not as tall as F.
Questions:
Questions Asked In Exams / 297

1. Which of the following is definitely false?


1) G is shorter than F. 2) C is shorter than F. 3) F is taller than C.
4) B is taller than E. 5) All are true
2. If another student J, who is taller than E but shorter than G, is added to
the group, which of the following will be definitely true?
1) C and J are of the same height. 2) J is shorter than D.
3) J is shorter than H. 4) J is taller than A.
5) None of these
3. Which of the following will definitely be the third from top when the
eight students are arranged in descending order of height?
1) B 2) F 3) G
4) B or G 5) Cannot be determined
4. How many of them are definitely shorter than F?
1) Three 2) Four 3) Five
4) Data inadequate 5) None of these
5. Which of the following is redundant to answer all the above questions?
1) (ii) only 2) (ii) and (iii) only 3) (iii)
and (iv) only 4) (ii) and (v) only 5) All are
necessary to answer the above questions.
[RBI Grade ‘B’ Officers’ Exam-2009]

Puzzle 37
Directions: Study the following information carefully and answer
the questions given below:
A, M, D, P, R, T, B and H are sitting around a circle, facing the centre. M

K KUNDAN
is third to the left of A, who is second to the left of T. D is second to the right
of H, who is second to the right of T. R is second to the right of B, who is not
an immediate neighbour of T.
Questions:
1. Which of the following combinations represents the first and the second
to the left of B respectively?
1) MD 2) DH 3) AM
4) AR 5) DM
2. Who is third to the right of T?
1) D 2) B 3) H
4) M 5) None of these
3. Who is to the immediate left of H?
1) P 2) M 3) T
4) R 5) Data inadequate
4. Who is second to the left of B?
1) D 2) H 3) M
4) Data inadequate 5) None of these
5. In which of the following combinations the third person is second to the
left of the second person?
1) BAR 2) DBM 3) TPH
4) PMH 5) None of these
[Corporation Bank PO Exam-2009]

Puzzle 38
Directions: Study the following information carefully and answer
the questions given below:
A group of seven friends, A, B, C, D, E, F and G work as Economist,
298 / Magical Book on Puzzle

Agriculture Officer, IT Officer, Terminal Operator, Clerk, Forex Officer and


Research Analyst, for Banks L, M, N, P, Q, R and S, but not necessarily in the
same order. C works for Bank N and is neither a Research Analyst nor a Clerk.
E is an IT Officer and works for Bank R. A works as a Forex Officer and does
not work for Bank L or Q. The one who is an Agriculture Officer works for
Bank M. The one who works for Bank L works as a Terminal Operator. F
works for Bank Q. G works for Bank P as a Research Analyst. D is not an
Agriculture Officer.
Questions:
1. Who amongst the following works as an Agriculture Officer?
1) C 2) B 3) F
4) D 5) None of these
2. For which bank does D work?
1) Q 2) L 3) N
4) S 5) None of these
3. What is the profession of C?
1) Terminal Operator 2) Agriculture Officer
3) Economist 4) Cannot be determined
5) None of these
4. Who amongst the following works as a Clerk?
1) C 2) B 3) F
4) D 5) None of these
5. Which of the following combinations of person, profession and bank is
correct?
1) A - Forex Officer - M 2) D - Clerk - L

K KUNDAN
3) F -Agriculture Officer-Q 4) B - Agriculture Officer-S
5) None of these
6. What is the profession of the person who works for Bank S?
1) Clerk 2) Agriculture Officer
3) Terminal Operator 4) Forex Officer
5) None of these
7. For which bank does B work?
1) M 2) S 3) L
4) Either M or S 5) None of these
[Oriental Bank of Commerce PO Exam-2009]

Puzzle 39
Directions: Study the following information carefully and answer
the questions given below:
A, B, C, D, E, F, G, H and K are sitting around a circle facing the centre B
is fourth to the left of G, who is second to the right of C. F is fourth to the right
of C and is second to the left of K. A is fourth to the right of K. D is not an
immediate neighbour of either K or B. H is third to the right of E.
Questions:
1. In which of the following combinations is the third person sitting between
the first and the second persons?
1) EKB 2) CHB 3) AGC
4) FGD 5) None of these
2. Who is fourth to the left of E?
1) A 2) C 3) G
4) Data inadequate 5) None of these
3. Who is second to the right of K?
1) C 2) H 3) F
4) E 5) Data inadequate
Questions Asked In Exams / 299

4. Who is third to the right of H?


1) A 2) D 3) G
4) F 5) None of these
5. Who is fourth to the right of D?
1) K 2) H 3) E
4) B 5) None of these
[Indian Bank PO Exam-2010]

Puzzle 40
Directions: Study the following information carefully and answer
the questions given below:
A, B, C, D, E, F, G and H are sitting around a circular table, facing the
centre. A sits third to the left of C and second to the right of E. B sits second
to the right of D, who is not an immediate neighbour of E. H sits second to the
left of F. G is not an immediate neighbour of D.
Questions:
1. Which of the following pairs has only one person sitting between them, if
the counting is done in clockwise direction?
1) F, G 2) H, G 3) H, C
4) H, B 5) None of these
2. Who sits third to the right of E?
1) D 2) G 3) F
4) B 5) None of these
3. What is the position of G with respect to A’s position?
1) Immediately to the right 2) Second to the left

K KUNDAN
3) Third to the right
5) Fourth to the right
4. Who sits between E and A?
1) F
4) B
2) D
4) Third to the left

5) None of these
3) G

5. Starting from A’s position, if all the eight are arranged in alphabetical
order in clockwise direction, the seating position of how many members
(excluding A) would-remain unchanged?
1) None 2) One 3) Two
4) Three 5) Four
[IDBI PO Exam–2009]

Puzzle 41
Directions: Study the following information carefully and answer
the questions given below:
A, B, C, D, E, F, G, H and K are sitting around a circle facing the centre.
F is fourth to the right of A, who is third to the right of B. K is fourth to the left
of B and third to the right of D. C is third to the right of H. E is second to the
left of G.
Questions:
1. Who is fourth to the left of G?
1) C 2) A 3) D
4) K 5) Data inadequate
2. What is E’s position with respect to B?
1) Second to the left 2) Third to the right 3) Fourth to the right
4) Third to the left 5) Fifth to the right
3. Who is third to the right of K?
1) F 2) E 3) G
300 / Magical Book on Puzzle

4) Data inadequate 5) None of these


4. Who is on the immediate right of F?
1) B 2) G 3) E
4) Data inadequate 5) None of these
5. In which of the following combinations is the third person sitting between
the first and the second person?
1) GFB 2) BGH 3) ADC
4) KEC 5) EGF
[SBI PO Exam–2010]

Puzzle 42
Directions: Study the following information carefully and answer
the questions given below:
A, B, C, D, E, F, G and H are eight employees of an organization working
in three departments, viz Personnel, Administration and Marketing with not
more than three of them in any department. Each of them has a different
choice of sports from football, cricket, volleyball, badminton, lawn tennis,
basketball, hockey and table tennis, not necessarily in the same order.
D works in Administration and does not like either football or cricket. F
works in Personnel with only A, who likes table tennis. E and H do not work
in the same department as D. C likes hockey and does not work in Marketing.
G does not work in Administration and does not like either cricket or badminton.
One of those who work in Administration likes football. The one who likes
volleyball works in Personnel. None of those who work in Administration
likes either badminton or lawn tennis. H does not like cricket.

K KUNDAN
Questions:
1. Which of the following groups of employees work in Administrat ion
department?
1) EGH 2) AF 3) BCD
4) BGD 5) Data inadequate
2. In which department does E work?
1) Personnel 2) Marketing 3) Administration
4) Data inadequate 5) None of these
3. Which of the following combinations of employee-department-favourite sport
is correct?
1) E - Administration - Cricket
2) F - Personnel - Lawn Tennis
3) H - Marketing - Lawn Tennis
4) B - Administration - Table Tennis
5) None of these
4. What is E’s favourite sport?
1) Cricket 2) Badminton 3) Basketball
4) Lawn Tennis 5) None of these
5. What is G’s favourite sport?
1) Cricket 2) Badminton 3) Basketball
4) Lawn Tennis 5) None of these
[SBI PO Exam–2010]

Puzzle 43
Directions: Study the following information carefully and answer
the questions given below:
A, B, C, D, E, F, G and H are sitting around a circle facing the centre. B is
second to the right of D, who is third to the right of F. C is second to the left
Questions Asked In Exams / 301

of A, who is second to the left of F. G is third to the right of E.


Questions:
1. In which of the following combinations is the first person sitting between
the second and the third persons?
1) GCD 2) FGH 3) EFH
4) ABE 5) None of these
2. Who is third to the right of H?
1) G 2) D 3) C
4) Data inadequate 5) None of these
3. Who is on the immediate right of A?
1) B 2) E 3) F
4) Data inadequate 5) None of these
4. What is H’s position with respect to B?
1) Fifth to the right 2) Third to the left 3) Fifth to the left
4) Third to the right 5) Fourth to the left
5. Who is on the immediate left of G?
1) H 2) F 3) D
4) Data inadequate 5) None of these
[Allahabad Bank PO Exam-2010]

Puzzle 44
Directions: Study the following information carefully and answer
the questions given below:
P, Q, R, S, T, V, W and Z are sitting round a circle facing the centre. T is
second to the right of R, who is third to the right of P. S is second to the left

K KUNDAN
of P and fourth to the right of Q. Z is third to the right of V, who is not an
immediate neighbour of P.
Questions:
1. In which of the following combinations is the first person sitting between
the second and the third persons?
1) VTS
4) PWQ
2) TZS
5) VRT
3) QRV

2. Who is second to the right of T?


1) S 2) Z 3) P
4) R 5) None of these
3. What is P’s position with respect to S?
1) Fourth to the left 2) Fourth to the right
3) Fifth to the left 4) Sixth to the left
5) Third to the right
4. Who is on the immediate left of Z?
1) T 2) P 3) S
4) V 5) None of these
5. Who is second to the right of W?
1) R 2) Q 3) Z
4) S 5) None of these
[Coporation Bank PO Exam-2010]

Puzzle 45
Directions: Study the following information carefully and answer
the questions given below:
P, Q, R, S, T, U, V and W are sitting around a circle facing the centre. T
is second to the left of P and third to the right of V. S is second to the right of
W, who is on the immediate right of T. Q is third to the right of U.
302 / Magical Book on Puzzle

Questions:
1. In which of the following pairs is the third person sitting in between the
first and the second persons?
1) USP 2) VRU 3) TQW
4) WPS 5) None of these
2. Who is on the immediate left of T?
1) Q 2) W 3) R
4) Data inadequate 5) None of these
3. Who is second to the right of P?
1) S 2) V 3) U
4) Q 5) Date inadequate
4. What is R’s position with respect to W?
1) Third to the left 2) Fourth to the left
3) Sixth to the right 4) Fifth to the left
5) None of these
5. Who is fourth to the left of R?
1) U 2) P 3) S
4) W 5) None of these
[Punjab and Sind Bank PO Exam-2010]

Puzzle 46
Directions: Study the following information carefully and answer
the questions given below:
Five plays A, B, C, D and E were organised in a week from Monday to
Saturday with one play each day and no play was organised on one of these

K KUNDAN
days. Play D was organised before Thursday but after Monday. Play E was
organised on Saturday. Play C was not organised on the first day. Play B was
organised on the next day on which play C was organised. Play A was organised
on Tuesday.
Questions:
1. On which day was play B organised?
1) Thursday 2) Friday 3) Wednesday
4) Data inadequate 5) None of these
2. On which day was no play organised?
1) Monday 2) Wednesday 3) Thursday
4) Data inadequate 5) None of these
3. Which play was organised on Wednesday?
1) A 2) C 3) D
4) Data inadequate 5) None of these
[Syndicate Bank PO Exam-2010]

Puzzle 47
Directions: Study the following information carefully and answer
the questions given below:
A, B, C, D, E, F, G and H are sitting around a circle facing the centre. F is
second to the right of A and third to the left of C. B is second to the left of C
and fourth to the right of H. D is second to the right of G
Questions:
1. Who is to the immediate left of D?
1) H 2) C 3) G
4) Data inadequate 5) None of these
2. Who is second to the right of E?
1) B 2) G 3) H
Questions Asked In Exams / 303

4) Data inadequate 5) None of these


3. Who is third to the left of B?
1) E 2) H 3) F
4) Data inadequate 5) None of these
4. What is the position of G with respect to A?
1) Third to the left 2) Third to the right 3) Fifth to the left
4) Fourth to the right 5) Fifth to the right
5. In which of the following combinations is the third person sitting in
between the first and the second persons?
1) BGC 2) EFB 3) DAH
4) AEF 5) GCD
[Syndicate Bank PO Exam-2010]

Puzzle 48
Directions: Study the following information carefully and answer
the questions given below:
(a) Six plays are to be organised from Monday to Sunday—one play each
day with one day when there is no play. ‘No play’ day is not Monday or
Sunday.
(b) The plays are held in sets of 3 plays each in such a way that 3 plays are
held without any break, ie 3 plays are held in such a way that there is
no ‘No play’ day between them but immediately before this set or imme-
diately after this set it is ‘No play’ day.
(c) Play Z was held on 26th and play X was held on 31 st of the same
month.

K KUNDAN
(d) Play B was not held immediately after play A (but was held after A, not
necessarily immediately) and play M was held immediately before Q.
(e) All the six plays were held in the same month.
Questions:
1. Which play was organised on Monday?
1) Z
3) Q
2) M
4) Cannot be determined
5) None of these
2. Which day was play Z organised?
1) Tuesday 2) Monday
3) Wednesday 4) Cannot be determined
5) None of these
3. Which date was a ‘No play’ day?
1) 26th 2) 28th
3) 29th 4) Cannot be determined
5) None of these
4. Which of the following is true?
1) Play B is held immediately before play M.
2) Play Z is held after play B.
3) There was a gap after 2 plays and then 4 plays were organised.
4) First play was organised on the 25th.
5) Play B was held on Friday.
5. Which day was play Q organised?
1) Friday 2) Wednesday
3) Saturday 4) Cannot be determined
5) None of these
[Central Bank of India PO Exam-2010]
304 / Magical Book on Puzzle

Puzzle 49
Directions: Study the following information carefully and answer
the questions given below:
(i) In a family of 6 persons, there are two couples.
(ii) The Lawyer is the head of the family and has only two sons — Mukesh
and Rakesh—both Teachers.
(iii) Mrs. Reena and her mother-in-law both are Lawyers.
(iv) Mukesh’s wife is a Doctor and they have a son, Ajay.
Questions:
1. Which of the following is definitely a couple?
1) Lawyer-Teacher 2) Doctor-Lawyer
3) Teacher-Teacher 4) Cannot be determined
5) None of these
2. What is the profession of Rakesh’s wife ?
1) Teacher 2) Doctor
3) Lawyer 4) Cannot be determined
5) None of these
3. How many male members are there in the family?
1) Two 2) Three
3) Four 4) Cannot be determined
5) None of these
4. What is/was Ajay’s Grandfather’s occupation?
1) Teacher 2) Lawyer
3) Doctor 4) Cannot be determined
5) None of these

K KUNDAN
5. What is the profession of Ajay?
1) Teacher
3) Doctor
5) None of these

Puzzle 50
2) Lawyer
4) Cannot be determined

[Central Bank of India PO Exam-2010]

Directions: Study the following information carefully and answer


the questions given below:
(i) There are 8 friends A, B, C, D, E, F, G, H seated in a circle facing the
centre.
(ii) AC, DG, HE and FB are seated adjacent to each other. A is also seated
adjacent to H.
(iii) B is 2nd to the right of H.
(iv) E is 3rd to the right of C.
Questions:
1. Who is 2nd to the left of A?
1) D 2) G
3) F 4) Cannot be determined
5) None of these
2. Who is 3rd to the left of C?
1) G 2) D
3) B 4) Cannot be determined
5) None of these
3. What is C’s position with reference to E?
1) 5th to the right 2) 4th to the left 3) 4th to the right
4) 3rd to the right 5) Cannot be determined
4. Who is 2nd to the right of A?
Questions Asked In Exams / 305

1) B 2) E
3) F 4) Cannot be determined
5) None of these
5. Who among the following pairs may not be seated adjacent to each other?
1) AH 2) DC
3) EB 4) Cannot be determined
5) None of these
[Central Bank of India PO Exam-2010]

Puzzle 51
Directions: Study the following information carefully and answer
the questions given below:
A, B, C, D, E, F, G and H are eight boys studying in VIII, IX and X standards
of a school with not more than three in any standard. Each of them has a
favourite sport from Football, Cricket, Volleyball, Basketball, Lawn Tennis,
Table Tennis, Badminton and Hockey not necessarily in the same order.
D likes Badminton and does not study either in VIII or X. H’s favourite
sport is Hockey and he studies in standard X with only B among them. A likes
Volleyball and does not study in the same standard in which E studies. F
studies in VIII std. and his favourite sports is Football. G does not study in the
same standard in which C studies. None of the students studying in IX std.
likes Cricket or Basketball. B likes Table Tennis. G does not like Lawn Tennis.
Questions:
1. What is C’s favourite sport?
1) Basketball 2) Lawn Tennis 3) Volleyball

K KUNDAN
4) Data inadequate

4) Data inadequate
3. In which std. does G study?
1) IX
4) Data inadequate
5) None of these
2. What is E’s favourite sport?
1) Table Tennis 2) Volleyball
5) None of these

2) VIII
5) None of these
3) Lawn Tennis

3) Either IX or VII

4. Which of the following combinations of students- standard-favourite sport


is not correct?
1) G - VIII - Volleyball 2) F - VIII - Football 3) D - IX - Badminton
4) H-X-Hockey 5) All are correct
5. Which of the following combinations of sports represents the favourite
sports of the students studying in VIII standard?
1) Football, Lawn Tennis, Cricket 2) Football, Cricket, Basketball
3) Football, Volleyball, Basketball 4) Data inadequate
5) None of these
[Punjab National Bank Specialist Officers Exam–2007]

Puzzle 52
Directions: Study the following information carefully and answer
the questions given below:
M, V, K, D, T, J and R are seven friends studying in different classes—
Illrd, IVth, Vth, Vlth, Vllth, VIIIth and IXth standards. Each of them has
different favourite colours—yellow, blue, red, white, black, green and violet. J
likes red and studies in class Vth. R likes violet and studies in Class Illrd. M
studies in Class VIIIth and does not like green and yellow. K likes white and
does not study in Vllth and in IVth. D studies in Vlth and likes black. T does
not study in IVth. V does not like green.
306 / Magical Book on Puzzle

Questions:
1. In which standard does ‘V study?
1) IVth 2) IXth 3) VIIIth
4) Data inadequate 5) None of these
2. What is M’s favourite colour?
1) Red 2) Yellow 3) Green
4) Blue 5) None of these
3. In which standard does K study?
1) IIIrd 2) Vth 3) IVth
4) VIIth 5) None of these
4. What is V’s favourite colour?
1) Green 2) Red 3) Yellow
4) Data inadequate 5) None of these
5. In which standard does T study?
1) IVth 2) VIIth 3) VIIIth
4) IXth 5) None of these
[Dena Bank (SO) Exam–2007]

Puzzle 53
Directions: Study the following information carefully and answer
the questions given below:
Seven Professors A, B, C, D, E, F and G are engaged in evaluation of
answer papers in three different subjects English, Mathematics and History.
At least two persons evaluate the papers in each subject. Each of the evaluators
stay in different buildings P, Q, R, S, T, V and W not necessarily in the same

K KUNDAN
order.
A evaluates English papers only with E and stays in building R. D stays in
building W and does not evaluate Maths papers. The one who stays in building
V evaluates History papers. B and C do not evaluate the papers in the same
subject. Those who evaluate English papers do not stay in building Q. F stays
in building P but does not evaluate History papers. G evaluates same papers as
F. C stays in building T.
Questions:
1. Who stays in building V?
1) E 2) F 3) G
4) B 5) None of these
2. Which of the following combinations of subject, person and buildings is
definitely correct?
1) Maths F-Q 2) Maths G-Q 3) History D-T
4) History E-S 5) None of these
3. Which of the following groups of persons evaluate the Mathematics paper?
1) CF 2) EFG 3) CFG
4) FG 5) None of these
4. Papers in which subject are evaluated by D?
1) History 2) Maths
3) English 4) English or Mathematics
5) History or Mathematics
5. E stays in which building?
1) P 2) Q
3) T 4) Cannot be determined
5) None of these
[Andhra Bank (SO) Exam–2007]
Questions Asked In Exams / 307

Puzzle 54
Directions: Study the following information carefully and answer
the questions given below:
Seven executives A, B, C, D, E, F and G from a company have to visit
seven different places Ahmedabad, Kolkata, Delhi, Chennai, Hyder abad,
Bangalore and Jaipur to market their newly launched product. The order of
persons and cities may not be necessarily the same. Each one flies by a different
airline Spicejet, Kingfisher, Sahara, Jet, Air Deccan, Indian Airlines, Air India,
not necessary in the same order.
C goes to Kolkata but not by Sahara or Jet Airlines. D flies by Air India to
Bangalore. The one who goes to Jaipur does not travel by Air Deccan or Sahara.
E travels by Air Deccan. A does not go to Ahmedabad. F travels to Hyderabad by
Spice jet. B goes to Chennai by Kingfisher. E does not go to Ahmedabad. G does
not go to Jaipur.
Questions:
1. Who travels by Sahara Airlines?
1) A 2) C
3) G 4) Cannot be determined
5) None of these
2. Who goes to Jaipur?
1) A 2) E 3) G
4) D 5) None of these
3. The one who travels by Air Deccan, visits which place?
1) Ahmedabad 2) Delhi 3) Chennai

K KUNDAN
4) Bangalore 5) None of these
[Andhra Bank (SO) Exam–2007]

Puzzle 55
Directions: Study the following information carefully and answer
the questions given below:
Moon TV had decided to celebrate sports programmes week by telecasting
programmes on Badminton, Cricket, Football, Hockey, Tennis and Volleyball
(not necessarily in the same order) in a week starting from Wednesday.
(i) Sixth day of the programme was holiday.
(ii) Tennis was telecast immediately after Hockey and Football was telecast
on fifth day of the programme.
(iii) Badminton was telecast on third day after Saturday and Hockey
programme was telecast on fourth day before the holiday.
(iv) Volleyball was not telecast on first day.
Questions:
1. On which of the following days Volleyball programme was telecast?
1) Second day 2) Seventh day 3) Fourth day
4) Data inadequate 5) None of these
2. Which of the following days would be holiday?
1) Sunday 2) Monday 3) Saturday
4) Tuesday 5) None of these
3. Which of the following pairs of programmes were respectively telecast
before and after the holiday?
1) Badminton and Football 2) Badminton and Tennis
3) Football and Volleyball 4) Football and Badminton
5) None of these
4. Which of the following programmes was telecast on Wednesday?
1) Cricket 2) Tennis 3) Hockey
308 / Magical Book on Puzzle

4) Data inadequate 5) None of these


5. Which of the following days and programme combinations are definitely
false?
1) Wednesday-Cricket 2) Tuesday-Badminton
3) Thursday-Hockey 4) Friday-Tennis
5) Sunday-Volleyball
[North Malabar Gramin Bank Officer’s Exam–2008]

Puzzle 56
Directions: Study the following information carefully and answer
the questions given below:
(i) There is a group of five persons M, N, O, P and Q of a family. They are
businessman, farmer, lawyer, doctor and teacher.
(ii) N is an unmarried teacher, who is M's daughter.
(iii) Q is a lawyer, who is O's brother.
(iv) O in this family is the only husband of a married couple.
(v) M is a farmer, who is father of two sons and an unmarried daughter.
(vi) M's daughter-in-law is a doctor.
1. Who is doctor in this family?
1) N 2) P 3) O
4) Q 5) M
2. Who is businessman in this family?
1) M 2) N 3) O
4) P 5) Q
3. Which of the following is a group of women in this family?

K KUNDAN
1) M, N and Q 2) N and O 3) P and Q
4) N and P 5) None of these
4. Which of the following is a married couple?
1) M and P 2) M and N 3) N and P
4) O and P 5) P and Q
5. Which of the following is a group of males?
1) M, N and O 2) M, Q and P 3) M, O and Q
4) M, P and Q 5) N and P
[Union Bank of India PO Exam–2008]

Puzzle 57
Directions: Study the following information carefully and answer
the questions given below:
(i) A marketing-executive plans to visit each of six companies J, K, L, M,
N and O exactly once during the course of one day.
(ii) He must visit J before K and N.
(iii) He must visit K before M.
(iv) The third company he visits must be L.
Questions:
1. If the marketing-executive visits O first, which company must he visit
second?
1) J 2) K 3) L
4) M 5) N
2. Which of the following could be the order in which the marketing-executive
visits the six companies?
1) J, O, L, K, N, M 2) M, K, L, N, O, J 3) J, N, K, M, L, O
4) L, O, J, N, M, K 5) L, N, J, K, M, O
[Union Bank of India PO Exam–2008]
Questions Asked In Exams / 309

Puzzle 58
Directions: Study the following information carefully and answer
the questions given below:
a, b, c, d, e, f and g are sitting around a circle facing at the centre. d is
third to the left of a who is second to the left of f. e is not a neighbour of either
f or d. c is third to left of b.
Questions:
1. What is e’s position with respect to f?
1) Third to the right 2) Fourth to the left 3) Second to the left
4) Data inadequate 5) None of these
2. Who is sitting between g and b?
1) f only 2) d only 3) Both f and d
4) Data inadequate 5) None of these
3. Who is to the immediate left of d?
1) c 2) g 3) f
4) Data inadequate 5) None of these
4. Which of the following information represents the first person sitting to
the immediate right of the second person?
1) ab 2) fg 3) ce
4) ae 5) None of these
5. Who is second to the right of d?
1) a 2) f 3) e
4) Data inadequate 5) None of these
[Indian Bank Computer Officer’s Exam–2008]

K KUNDAN
Puzzle 59
Directions: Study the following information carefully and answer
the questions given below:
Seven persons R, J, M, Q, L, T and K conduct workshop on Developing
Managerial skills in seven different companies A, B, C, D, E, F and G on a
different day of the week from Monday to Sunday. The order of persons,
companies and days of the week are not necessarily the same.
J organises workshop in Company D on Wednesday. Q does not conduct
workshop for companies A or C and conducts on the next day of L who conducts
the workshop for Company F. T conducts workshop for Company E on Friday.
K conducts workshop on Monday but not for Company C or G. M conducts
workshop for Company A but not on Tuesday.
Questions:
1. Who conducts workshop on Saturday?
1) M 2) Q 3) L
4) Q or L 5) None of these
2. On which day does Q conduct the workshop?
1) Sunday 2) Saturday
3) Tuesday 4) Cannot be determined
5) None of these
3. M conducts workshop on which day?
1) Saturday 2) Sunday 3) Tuesday
4) Thursday 5) None of these
4. Which of the following combinations of person-company and day is correct?
1) K-B-Wednesday 2) R-B-Monday 3) K-C-Monday
4) K-G-Sunday 5) None of these
5. Who conducts workshop for Company C and on which day?
1) R, Thursday 2) R, Tuesday 3) Q, Saturday
310 / Magical Book on Puzzle

4) Q, Sunday 5) None of these


[Punjab National Bank Agriculture Officer’s Exam–2008]

Puzzle 60
Directions: Study the following information carefully and answer
the questions given below:
P, Q, R, S, T, V and W are seven members of a club. Each of them has a
favourite sport from—Chess, Table Tennis, Lawn Tennis, Volleyball, Badminton,
Basketball and Carrom, not necessarily in the same order. Each of them also
has a specific choice of colour from—Blue, Red, Green, Yellow, Grey, Black
and White, not necessarily in the same order.
R likes Green and his favourite sport is Badminton. V’s choice of colour is
neither Red nor Black. T’s favourite sport is neither Table Tennis nor Basketball.
The one who likes Blue does not like Carrom. The one who likes Volleyball
does not like Yellow and Grey. Q’s favourite sport is Lawn Tennis and he likes
Black. S likes White. W likes Basketball. P likes Volleyball. T likes Blue. The
one who likes Basketball does not like Grey.
Questions:
1. What is V’s choice of colour?
1) Black 2) Grey 3) Yellow
4) Data inadequate 5) None of these
2. What is T’s favourite sport?
1) Basketball 2) Volleyball 3) Chess
4) Data inadequate 5) None of these
3. Whose favourite sport is Carrom?

K KUNDAN
1) S 2) R 3) W
4) Data inadequate 5) None of these
4. Whose favourite sport is basketball?
1) S 2) T 3) W
4) R 5) Data inadequate
5. What is W’s choice of colour?
1) Green 2) White 3) Black
4) Data inadequate 5) None of these
6. Which of the following combinations of sport and colour is correct?
1) Table Tennis, Yellow 2) Volleyball, Red
3) Volleyball, Grey 4) Chess, Black
5) Carrom, Green
[IDBI Officers’ Exam–2008]

Puzzle 61
Directions: Study the following information carefully and answer
the questions given below:
J, K, H, R, F, L, N and Q are sitting around a circular table facing the
centre. H is third to the left of L and is to the immediate right of K. R is third
to the left of N but is not a neighbour of H or L. J is second to the right of Q.
Questions:
1. Who is second to the left of N ?
1) Q 2) K 3) J
4) F or J 5) None of these
2. Which of the following groups of persons has the first person sitting
between the next two?
1) LKN 2) QFL 3) JHR
4) JHF 5) None of these
Questions Asked In Exams / 311

3. Who is to the immediate left of R?


1) Q 2) K 3) F
4) N 5) None of these
4. Which of the following is correct position of J with respect to K?
1) Third to the left 2) Third to the right 3) Second to the left
4) Second to the right 5) Fourth to the right
5. Four of the following five are alike in a certain way on the basis of their
seating positions and so form a group. Which is the one that does not
belong to the group?
1) RQ 2) LK 3) HJ
4) JR 5) FN
[Allahabad Bank PO Exam–2008]

Puzzle 62
Directions: Study the following information carefully and answer
the questions given below:
Eight executives B, G, H, K, D, F, T and V are working in three departments
Engineering, Systems and Marketing of the organization and are posted at
different places viz. Chennai, Kozikode, Kolkata, Ranchi, Patna, Bhopal, Nagpur
and Hyderabad not necessarily in the same order. At least two and not more
than three executives work in any of the three departments.
G works in Engineering department at Chennai. H is posted in Ranchi
but not in Systems department. No one from Marketing department is posted in
Hyderabad. The only other person in same department as that of G is posted in
Kolkata. D is posted in Hyderabad and F in Kozikode. V is not posted in Kolkata

K KUNDAN
and works in the same department as that of D. B and T both work in Marketing
department. The one who works in Marketing is not posted in Bhopal. T is not
posted in Nagpur.
Questions:
1. T is posted at which place?
1) Nagpur
3) Bhopal
2) Patna
4) Cannot be determined
5) None of these
2. Who is posted in Kolkata?
1) K 2) T
3) V 4) K or T
5) None of these
3. Which department has only two Executives?
1) Systems 2) Marketing
3) Engineering 4) Marketing or Systems
5) Cannot be determined
4. Which of the following group of persons work in Mar-keting department?
1) KBT 2) BTF
3) BHD 4) BHT
5) None of these
5. Which of the following combinations of department, person and place is
correct?
1) Marketing-B-Bhopal 2) Engineering-G-Kolkata
3) Systems-V-Chennai 4) Systems-T-Patna
5) None of these
[Allahabad Bank PO Exam–2008]
312 / Magical Book on Puzzle

Puzzle 63
Directions: Study the following information carefully and answer
the questions given below:
A, B, C, D, E, F, G and H are sitting around a circle, facing the centre. A
sits fourth to the right of H while second to the left of F. C is not the neighbour
of F and B. D sits third to the right of C. H never sits next to G.
Questions:
1. Who amongst the following sits between B and D?
1) G 2) F 3) H
4) A 5) C
2. Which of the following pairs sit between H and G?
1) BH 2) EF 3) CE
4) DB 5) None of these
3. Four of the following are alike in a certain way based on their positions in
the seating arrangement and so form a group. Which is the one that does
not belong to that group?
1) AE 2) HF 3) BD
4) GE 5) CH
4. Who is to immediate right of A?
1) C 2) D 3) G
4) Data inadequate 5) None of these
5. Who sits second to the right of B?
1) A 2) C 3) D
4) E 5) None of these
6. Which is the position of B with respect to C?

K KUNDAN
I. Second to the right
II. Sixth to the left
III. Third to the left
IV. Fifth to the right
1) Only II
4) Data inadequate
2) Only II and III
5) Both III and IV
3) Only I and IV

[Andhra Bank PO Exam–2008]

Puzzle 64
Directions: Study the following information carefully and answer
the questions given below:
There are five men, Anuj, Kunal, Sourav, Rahul and Harish. The one who
is tallest is not the youngest. Kunal is older than only Harish. Sourav is older
than Rahul but shorter than him. Only one person is taller than Rahul. Anuj
is shortest while younger than only Sourav and Rahul. Only two men are
shorter than Sourav.
Questions:
1. Which of the following men is third tallest of the five?
1) Sourav 2) Rahul 3) Harish
4) Kunal 5) Kunal or Rahul
2. Who among the following men is tallest?
1) Sourav 2) Rahul 3) Kunal
4) Harish 5) Sourav or Harish
3. If the five men are made to stand in a line according to their height, first
in ascending order, then in descending order, then whose position will
remain the same in both the arrangements?
1) Harish 2) Rahul 3) Kunal
4) Sourav 5) Rahul and Anuj
Questions Asked In Exams / 313

4. Who among the following is older than Kunal but younger than Rahul?
1) Harish 2) Sourav 3) Anuj
4) Data inadequate 5) Sourav or Anuj
5. Who among the following is taller than Anuj only?
1) Sourav 2) Harish 3) Rahul
4) Kunal 5) Kunal or Harish
[Andhra Bank PO Exam–2008]

Puzzle 65
Directions: Study the following information carefully and answer
the questions given below:
A, B, C, D, E, F, G and H are sitting around a circle, facing the centre. E
and G always sit next to each other. D sits third to the right of C. F sits second
to the left of H. C never sits next to A while D never sits next to G. H is not the
neighbour of D and C.
Questions:
1. Who sits between A and D?
1) B 2) F 3) C
4) E 5) None of these
2. Who sits second to the left of B?
1) F 2) G 3) A
4) E 5) None of these
3. Who sits to immediate right of F?
1) D 2) C 3) B
4) A 5) None of these

K KUNDAN
4. Which of the following pairs sits between B and F ?
1) HB
4) GC

not belong to that group?


1) CH
2) FD
5) AH
3) BG

5. Four of the following are alike in a certain way based on their positions in
the seating arrangement and so form a group. Which is the one that does

2) BA 3) FE
4) DG 5) AC
[Dena Bank Agriculture Officers Exam–2008]

Puzzle 66
Directions: Study the following information carefully and answer
the questions given below:
Seven members A, B, C, D, E, F and G represent seven different states
Madhya Pr adesh, Uttar Pradesh, Bihar, Kerala, Tamil Nadu, Orissa and
Maharashtra in seven different games Hockey, Chess, Cricket, Badminton,
Table-Tennis, Golf and Billiards. The order of persons, states and games is
not necessarily the same.
D represents Kerala in Chess. E represent s Golf team but not from
Maharashtra or Uttar Pradesh, A represents Madhya Pradesh for Badminton.
C represents Orissa but not for Cricket or Table-Tennis. The one who represents
Bihar, represents “Table-Tennis. The one who represents Hockey represents
Uttar Pradesh. F represents Maharashtra for Cricket. G does not represent
Bihar.
Questions:
1. Who represents Uttar Pradesh?
1) G 2) F
3) B 4) Cannot be determined
314 / Magical Book on Puzzle

5) None of these
2. Who represents Bihar?
1) G 2) E 3) B
4) B or E 5) None of these
3. Who represents Billiards team?
1) G 2) F 3) C
4) B 5) None of these
4. E represents which state?
1) Bihar 2) Kerala 3) Uttar Pradesh
4) Tamil Nadu 5) None of these
5. Which of the following combinations of game and state is correct?
1) Orissa-Chess 2) Orissa-Billiards 3) Tamil Nadu-Cricket
4) Maharashtra-Chess 5) None of these
[Dena Bank Agriculture Officers Exam–2008]

Puzzle 67
Directions: Study the following information carefully and answer
the questions given below:
B, D, F, H, K, W, M and T are sitting around a circle facing at the centre.
F is third to the left of D who is second to the left of H. B is fourth to the right
of H. K is third to the right of M who is not an immediate neighbour of F. T is
not an immediate neighbour of B or D.
Questions:
1. Who is to the immediate right of T?
1) K 2) F 3) H

K KUNDAN
4) Data inadequate 5) None of these
2. Who is second to the left of B?
1) T 2) K 3) D
4) Data inadequate 5) None of these
3. Who is sitting between D and B?
1) T 2) F 3) M
4) W 5) Data inadequate
4. Who is third to the left of H?
1) W 2) F 3) B
4) Data inadequate 5) None of these
5. What is T’s position with respect toW?
1) Third to the right 2) Third to the left 3) Second to the left
4) Second to the right 5) None of these
[Andhra Bank IT Officers’ Exam–2008]

Puzzle 68
Directions: Study the following information carefully and answer
the questions given below:
A, B, C, D, E, F and G are seven boys. Each of them studies a different
subject as Economics, Bio-Chemistry, Biology, Physics, English, German and
Psychology. Each of them likes a different game as Football, Cricket, Badminton,
Table-Tennis, Carrom, Volleyball and Hockey. The order of boys, subject and
games is not necessarily the same.
B studies Biology and likes cricket . D likes football but not studies
Biochemistry or English and G likes Table-Tennis but not studies Physics. F
studies Economics but not likes Carrom. A studies Psychology likes Hockey.
One who studies Physics likes Badminton. C plays Carrom but not studies
Questions Asked In Exams / 315

Bio-chemistry.
Questions:
1. F likes which game?
1) Volleyball 2) Badminton
3) Table-Tennis 4) Cannot be determined
5) None of these
2. Who studies English?
1) E 2) D 3) C
4) G 5) None of these
3. Which one of the following statements is certainly true?
1) One who likes football studies English
2) One who likes Volleyball studies Bio-chemistry
3) One who likes Carrom sudies Psychology
4) One who likes Hockey studies English
5) One who likes Carrom studies English
4. D studies which subject?
1) Economics 2) Physics 3) Biology
4) German 5) None of these
5. Who likes Badminton?
1) D 2) E
3) G 4) Cannot be determined
5) None of these
[Andhra Bank IT Officers’ Exam–2008]

Puzzle 69

K KUNDAN
Directions: Study the following information carefully and answer
the questions given below:
Ashwini, Priya, Sudha, Rani, Meeta, Geeta and Mukta are sitting around a
circle facing the centre. Ashwini is third to the left of Mukta and to the
immediate right of Rani. Priya is second to the left of Geeta, who is not an
immediate neighbour of Meeta.
Questions:
1. Who is to the immediate right of Priya?
1) Meeta 2) Sudha
3) Mukta 4) Cannot be determined
5) None of these
2. Who is second to the left of Rani?
1) Ashwini 2) Meeta 3) Priya
4) Sudha 5) None of these
3. Which of the following pairs of persons has the first person sitting to the
immediate left of the second person?
1) Rani-Meeta 2) Ashwini-Geeta 3) Sudha-Priya
4) Geeta-Sudha 5) None of these
4. Which of the following groups has the first person sitting between the
other two?
1) Meeta-Ashwini-Geeta 2) Sudha-Rani-Geeta

3) Mukta-Priya-Rani 4) Mukta-Priya-Sudha
5) None of these
5. Which of the following is the correct position of Rani with respect to
Mukta?
I. Third to the right
II. Third to the left
316 / Magical Book on Puzzle

III. Fourth to the left


IV. Fourth to the right
1) I only 2) II only 3) Both I and II
4) Both II and IV 5) Both I and III
[Bank of Baroda (SO) Exam-2008]

Puzzle 70
Directions: Study the following information carefully and answer
the questions given below:
A, B, C, D, E, F and G are members of a sports club and have liking for
different games, viz Carrom, Table Tennis, Badminton, Bridge, Hockey, Football
and Lawn Tennis but not necessarily in the same order. Each one of them has
a liking for different musical instruments, viz Sitar, Guitar, Harmonium, Flute,
Tabla, Banjo and Santoor, not necessarily in the same order.
B likes Carrom and Banjo. E likes to play Bridge but not Harmonium or
Tabla. The one who plays Hockey plays Sitar. F plays Guitar but not Table
Tennis or Lawn Tennis. A plays Badminton and Flute. The one who plays
Lawn Tennis does not play Tabla. C plays Harmonium and G plays Hockey.
1. Who plays Santoor?
1) D 2) A 3) E
4) D or E 5) None of these
2. D plays which game?
1) Table Tennis 2) Lawn Tennis
3) Football 4) Cannot be determined
5) None of these

K KUNDAN
3. Which of the following combinations of game-person-musical instrument
is definitely correct?
1) Badminton - B - Flute
2) Table Tennis - E - Santoor
3) Lawn Tennis - D - Tabla
4) Table Tennis - C - Tabla
5) None of these
4. Who plays Football?
1) C 2) D 3) G
4) F 5) None of these
5. Who plays Table Tennis?
1) C 2) F
3) D 4) Cannot be determined
5) None of these
[Bank of Baroda (SO) Exam-2008]

Puzzle 71
Directions: Study the following information carefully and answer
the questions given below:
P, Q, R, S, T, V and W are sitting around a circle facing at the centre. R is
third to the left of V who is second to the left of T. W is second to the right of
P who is not an immediate neighbour of V. S is not an immediate neighbour of
W.
Questions:
1. Who is second to the right of R?
1) T 2) W 3) Q
4) Data inadequate 5) None of these
2. Who is third to the left of P?
Questions Asked In Exams / 317

1) S 2) W 3) Q
4) Data inadequate 5) None of these
3. Who is third to the right of Q?
1) T 2) P 3) S
4) Data inadequate 5) None of these
4. In which of the following pairs is the first person sitting to the immediate
right of the second person?
1) PR 2) PT 3) RW
4) QV 5) None of these
5. Which of the following pairs represents the immediate neighbours of R?
1) PT 2) PQ 3) WT
4) PW 5) Data inadequate
[Punjab National Bank Agriculture Officer’s Exam–2009]

Puzzle 72
Directions: Study the following information carefully and answer
the questions given below:
Eight friends P, Q, R, S, T, V, X and Z are sitting around a circular table
facing centre. P sits third to the right of X and second to the left of R. S and T
do not sit next to each other. S sits second to the right of Q, who is not a
neighbour of P. T sits second to the left of V.
Questions:
1. Four of the following five are similar in a certain way based on their
position in the seating arrangement. Which of the following does not
belong to that group?

K KUNDAN
1) VX
4) XZ

B. Fourth to the left


C. Second to the right
D. Fourth to the right
2) SZ
5) RQ
3) TV

2. What is the position of V with respect to Z in the above arrangement?


A. Third to the right

1) Only A 2) Both A and B


3) Both B and D 4) Only D
5) None of these
3. Who sits to the immediate left of R?
1) V 2) Z 3) T
4) P 5) None of these
4. Which of the following sits between X and V?
1) Q 2) X 3) Z
4) R 5) None of these
5. In which of the following pairs, second person is sitting to the left of the
first person?
1) XS 2) PT 3) RV
4) QV 5) None of these
[Allahabad Bank Agriculture Officer’s Exam–2009]

Puzzle 73
Directions: Study the following information carefully and answer
the questions given below:
Seven friends A, B, C, D, E, F and G work in four different departments ie,
Marketing. Finance HR and IT. Not more than two work in the same department.
Each of them works in different organisations which are P, Q, R, S, T, W and
318 / Magical Book on Puzzle

Z not necessarily in this order. A who does not work for organisation P works
in Marketing Department like C. E works for organisation W and does not
work in Finance Department. B works for organisation T and works in the
same Department as D. Only F works in HR Department. None in Marketing or
IT department works for organisation R. C works for organisation S. The one
who works for organisation Z works for the Finance Department.
Questions:
1. Which of the following combinations is correct?
1) C-S-Finance 2) E-T-IT 3) A - R-Marketing
4) F-Q-HR 5) None of these
2. Which of the following works in IT Department?
1) G 2) D 3) B
4) Data inadequate 5) None of these
3. Who works for organisation R?
1) A 2) F 3) G
4) Z 5) None of these
4. Which of the following pairs works in Finance Department?
1) A, B 2) E, G 3) F, D
4) D, B 5) None of these
5. For which of the following organisations does D work ?
1) P 2) Q 3) Z
4) R 5) None of these
[Allahabad Bank Agriculture Officer’s Exam–2009]

Puzzle 74

K KUNDAN
Directions: Study the following information carefully and answer
the questions given below:
A, B, C, D, E, F, G and H are sitting around a circle facing at the centre.
G is fourth to the right of A who is second to the right of D. E is second to the
right of C who is not an immediate neighbour of G or D. B is second to the
right of F.
Questions:
1. In which of the following pairs is the first person sitting to the immediate
right of the second person?
1) EG 2) GF 3) HE
4) BD 5) None of these
2. Who is third to the left of H?
1) A 2) D 3) B
4) Data inadequate 5) None of these
3. Who is second to the right of H?
1) E 2) G 3) F
4) Data inadequate 5) None of these
4. What is F’s position with respect to C?
A. Fourth to the left
B. Fifth to the left
C. Fourth to the right
D. Third to the right
1) Only A 2) Only B
3) Only A and D 4) Only B and C
5) Only A and C
5. Who is to the immediate right of G?
1) F 2) G 3) D
4) Data inadequate 5) None of these
Questions Asked In Exams / 319

[United Commercial Bank PO Exam–2009]

Puzzle 75
Directions: Study the following information carefully and answer
the questions given below:
B, M, K, H, T, R, D, W and A are sitting around a circle facing at the
centre. R is third to the right of B. H is second to the right of A who is second
to the right of R. K is third to the right of T, who is not an immediate neighbour
of H. D is second to the left of T. M is fourth to the right of W.
Questions:
1. Who is to the immediate left of W?
1) R 2) T 3) B
4) Data inadequate 5) None of these
2. Who is third to the left of M?
1) B 2) W 3) K
4) T 5) None of these
3. Who is third to the left of H ?
1) A 2) T 3) K
4) R 5) Data inadequate
4. Who is to the immediate left of D?
1) H 2) M 3) B
4) Data inadequate 5) None of these
5. In which of the following combinations is the third person sitting in
between the first and the second persons ?
1) WTR 2) BDT 3) MHD

K KUNDAN
4) KAM

Puzzle 76
5) WKR
[Indian Bank Rural Marketing Officers Exam–2009]

Directions: Study the following information carefully and answer


the questions given below:
P, Q, R, S, T, V, W, Y and Z are sitting around a circle facing the centre.
V is second to the left of P who is third to the left of Y. S is fourth to the left of
P. T is third to the right of Q who is to the immediate right of V. W is fourth to
the right of Z.
1. In which of the following combinations is the first person sitting in between
the second and the third persons ?
1) RSZ 2) YTW 3) WPQ
4) VZQ 5) PQV
2. Who is second to the right of Z ?
1) Q 2) P 3) W
4) Data inadequate 5) None of these
3. In which of the following pairs is the first person sitting to the immediate
right of the second person ?
1) VQ 2) ZV 3) SZ
4) RS 5) YT
4. Who is third to the left of S ?
1) Y 2) W 3) T
4) Data inadequate 5) None of these
5. Who is third to the right of W ?
1) S 2) R 3) V
4) Z 5) Data inadequate
[Bank of India Banking Officers’ Exam–2010]
320 / Magical Book on Puzzle

Puzzle 77
Directions: Study the following information carefully and answer
the questions given below:
A, B, C, D, E, F, G and H are eight persons working in three different
departments viz. Operations, Sales and Finance of an organisation with at
least two and not more than three in any department. They are in three different
scales viz. I, II and III with at least two in any one scale.
Both the employees in Operations department are in Scale II. D works in
Sales department and belongs to Scale I. A works in Finance department and
does not belong to Scale I. Two employees in Sales department belong to one
scale. F works with only H in one of the departments. C works with E in one
of the departments. B does not work with either C or A in the same department.
G does not belong to Scale III. E does not belong to Scale I.
Questions:
1. Which of the following combinations is correct?
1) Operation — F — I 2) Operation — H — III
3) Sales — B — II 4) Finance — E — III
5) All are correct
2. Which of the following groups of employees work in Sales department ?
1) DBE 2) DBC 3) DBG
4) Data inadequate 5) None of these
3. H belongs to which scale?
1) I 2) II 3) III
4) Data inadequate 5) None of these
4. G belongs to which scale ?

K KUNDAN
1) II
4) I or II
5. C belongs to which scale ?
1) I
4) I or II
2) III
5) None of these

2) II
5) None of these
3) I

3) III

[Bank of India Banking Officers’ Exam–2010]

Puzzle 78
Directions: Study the following information carefully and answer
the questions given below:
A, B, C, D, E, F, G and H are sitting around a circle facing at the centre.
C is third to the left of A and second to the right of E. B is second to the right
of C. D is second to the right of F who is second to the right of A. G is not an
immediate neighbour of C.
Questions:
1. Who is to the immediate right of C?
1) H 2) G 3) D
4) Data inadequate 5) None of these
2. Who is to the immediate right of H?
1) D 2) G 3) A
4) Data inadequate 5) None of these
3. Who is the immediate left of D?
1) C 2) H 3) F
4) E 5) Data inadequate
4. Who is third to the right of H ?
1) E 2) F 3) D
4) G 5) Data inadequate
5. Who is second to the right of G ?
Questions Asked In Exams / 321

1) A 2) D 3) E
4) B 5) Data inadequate
6. In which of the following is the first person sitting in between the second
and the third person ?
1) BHA 2) CHB 3) EDC
4) EFG 5) None of these
[Baroda Rajasthan Gramin Bank Officers Exam–2010]

Puzzle 79
Directions: Study the following information carefully and answer
the questions given below:
A, B, C, D, E, F, G, H and I are sitting around a circle facing the centre. D
is fourth to the right of A and second to the left of G. C is second to right of G
and third to the left of I. F is fourth to the right of H. B is not an immediate
neighbour of G or A.
Questions:
1. Who is third to the left of A ?
1) E 2) G 3) D
4) F 5) None of these
2. Who is fourth to the right of B ?
1) H 2) C 3) A
4) G 5) None of these
3. Who is to the immediate left of D?
1) B 2) F 3) I
4) H 5) None of these

K KUNDAN
4. Who is second to the right of F ?
1) G
4) Data inadequate
2) C
5) None of these
3) E

5. In which of the following combinations is the first person sitting in between


the second and the third persons ?
1) FBD
4) HIB
2) CEG
5) GEF
3) AHI

[Sarva UP Gramin Bank Officers Exam–2010]

Puzzle 80
Directions: Study the following information carefully and answer
the questions given below:
A, B, D, E, F, G and H are sitting around a circle facing the centre. D is
fourth to the right of H and second to the left of B. F is fourth to the right of B.
G is fourth to the right of E who is not an immediate neighbour of B or D. A is
not an immediate neighbour of D.
Questions:
1. What is B’s position with respect toG?
1) Third to the right 2) Third to the left 3) Fifth to the right
4) Fourth to the left 5) Fourth to the right
2. In which of the following combinations is the third person sitting in
between the first and the second persons?
l) ABC 2) GCD 3) AHE
4) CBA 5) EFG
3. Who is third to the right of A?
1) H 2) E 3) F
4) Data inadequate 5) None of these
4. Who is to the immediate left of D?
322 / Magical Book on Puzzle

1) G 2) C 3) F
4) Data inadequate 5) None of these
5. Who is fourth to the left of G?
1) E 2) F 3) A
4) H 5) Data inadequate
[Bank of Baroda PO Exam–2010]

Puzzle 81
Directions: Study the following information carefully and answer
the questions given below:
Four friends Amrita, Deepa, Smita and Rhea complete their PhD in different
number of years. The one who took maximum time took eight years to complete
her PhD while the one who took the least time took only three years to complete
it. Rhea took more time only than Amrita and completed her PhD in five years.
Smita did not take longer time than Deepa to complete her PhD.
Questions:
1. How many years did Amrita take to complete her PhD?
1) 8 2) 3 3) 4
4) Either 6 or 7 5) Cannot be determined
2. How many years did Smita take to complete her PhD ?
1) 8 2) 3 3) 4
4) 7 5) Cannot be determined
3. Who amongst the following took the maximum number of years to complete
PhD ?
1) Amrita 2) Rhea 3) Smita

K KUNDAN
4) Deepa 5) Cannot be determined
4. How many meaningful English words can be formed with the letters TEID
using each letter only once in each word?
1) None 2) One 3) Two
4) Three 5) More than three
5. If it is possible to make only one meaningful word with the second, fifth,
eighth and the eleventh letters (when counted from left hand side) of the
word DEVELOPMENT, which would be the second letter of the word from
the left ? If more than one such word can be formed, give X as the answer.
If no such word can be formed, give Z as your answer.
1) E 2) L 3) T
4) X 5) Z
[Punjab National Bank (SO) Exam–2010]

Puzzle 82
Directions: Study the following information carefully and answer
the questions given below:
Eight friends P, Q, R, S, T, V, W and Z, out of whom one is a pilot,
professor, businessman, doctor, lawyer, banker, cricketer or an architect (but
not necessarily in the same order), are sitting around a circular table, facing
the centre.
l S who is a banker sits third to right of Z.
l The professor and the architect are immediate neighbours of each other.
Neither the professor nor the architect is an immediate neighbour of
either Z or S.
l Cricketer and the pilot are immediate neighbour of each other. Neither
Z nor W is a pilot.
l The one who is a professor sits second to the right of T who is a
Questions Asked In Exams / 323

lawyer. V who is a cricketer is not an immediate neighbour of the


banker.
l Only R sits between the professor and the doctor. P sits third to the
right of the pilot.
Questions:
1. Which of the following pairs represents the immediate neighbours of the
doctor ?
1) Professor-businessman 2) Pilot - professor
3) Cricketer - businessman 4) Lawyer - architect
5) None of these
2. What is the position of the businessman with respect to the pilot ?
1) Third to the left 2) Second to the left 3) Immediately to the right
4) Fourth to the right 5) Second to the right
3. Who sits third to the right of the professor ?
1) The banker 2) Q 3) The cricketer
4) The lawyer 5) None of these
4. Which of the following is true regarding R ?
1) He is a doctor
2) He is an immediate neighbour of the pilot
3) R sits between Q and T
4) He is an immediate neighbour of the professor
5) None is true
5. How many people sit between the ‘banker’ and ‘W’ when counted in anti-
clockwise direction from the banker?
1) None 2) One 3) Two

K KUNDAN
4) Three 5) Four
6. Who amongst the following is a pilot ?
1) P 2) Q 3) R
4) Z 5) None of these
7. What is the profession of Z ?
1) Businessman 2) Architect 3) Professor
4) Doctor 5) None of these
8. Who amongst the following sits exactly between T and Q ?
1) Cricketer 2) Architect 3) Professor
4) Doctor 5) Banker
[Punjab National Bank (SO) Exam–2010]

Puzzle 83
Directions: Study the following information carefully and answer
the questions given below:
A, B, C, D, E and F are seated in a circle facing the centre. A and C are
seated adjacent to each other and E and B are also seated adjacent to each
other. B is to the immediate left of F. There are two persons between D and E.
A is not seated adjacent to E.
Questions:
1. How many persons are seated between F and E if we go anticlockwise
from F to E ?
1) 1 2) 2
3) 3 4) Cannot be determined
5) None of these
2. Who is to the immediate left of E?
1) C 2) B
3) F 4) Cannot be determined
324 / Magical Book on Puzzle

5) None of these
Directions (Q. 3-5): Four of the following five are alike in a certain
way based on their seating positions in the above arrangement and so
form a group. Which is the one that does not belong to the group ?
3. 1) BA 2) BD 3) CF
4) DE 5) FC
4. 1) BEC 2) CAD 3) FDA
4) DFB 5) ADF
5. 1) AC 2) EB 3) CE
4) AF 5) DA
[Bank of India PO Exam–2010]

Puzzle 84
Directions: Study the following information carefully and answer
the questions given below:
Q, R, S, T, U and V are seated to a straight line facing North. S is second
to the right of T and T is second to the right of Q. R is to the left of Q and is
second to the left of V.
Questions:
1. How many persons are seated between T and V?
1) 1 2) 2 3) 3
4) 4 5) None of these
2. Four of the following five are alike in a certain way based on their seating
position in the above arrangement and so form a group. Which is the one
that does not belong to the group ?

K KUNDAN
1) QV 2) VT 3) US
4) UT 5) RQ
3. What is Q's position with respect to S?
1) Third to left 2) Immediate Next 3) Second to left
4) Fifth to left 5) None of these
4. Which of the following represents persons seated at the two extremes ?
1) RQ 2) US 3) SQ
4) SR 5) None of these
5. If S : T and T : Q, then U : ?
1) T 2) V 3) Q
4) S 5) R
[Bank of India PO Exam–2010]

Puzzle 85
Directions: Study the following information carefully and answer
the questions given below:
V, U and T are seated in a circle facing the centre. A, B and C are also
seated in the same circle but two of them are not facing the centre (facing
opposite direction of the centre). V is second to the left of C. U is second to the
right of A. B is third to the left of T. C is second to the right of T. A is seated
next to V.
Questions:
1. Which of the following are not facing the centre ?
1) BA 2) CA
3) BC 4) Cannot be determined
5) None of these
2. Which of the following is T’s position with respect to B?
1) Third to the right
Questions Asked In Exams / 325

2) Second to the right


3) Third to the left
4) Third either to the right or to the left
5) None of these
3. Which of the following is V's position with respect to C?
1) Second to the right 2) Third to the left
3) Fourth to the right 4) Fourth to the left
5) Cannot be determined
4. Which of the following is true regarding the seating arrangement?
1) A, B and C are seated adjacent
2) V, U and T are seated adjacent
3) There are two persons whose seating arrangement cannot be ascertained
4) Those not facing the centre are seated adjacent
5) There are only two persons seated between V and T
5. Which of the following is A's position with respect to U?
1) Second to the left 2) Second to the right
3) Third to the right 4) Cannot be determined
5) None of these
[Bank of India PO Exam–2010]

Puzzle 86
Directions: Study the following information carefully and answer
the questions given below:
In a Group of 5, each person has an exclusive and different preference
(has/likes) for a pen, a watch and a car. Pen preferences are Parker, Lamy,

K KUNDAN
Pointer, Lexi and Cello. Car preferences are Wagon R, Swift, Santro, Mica and
City. Watch preferences are Timex, Titan, Fastrack, Samay and Citizen.
Suman has Mica and Parker but does not prefer among watches -Titan or
Fastrack. The one who has Swift, likes Fastrack. Mrudula has preference for
City, Cello and Citizen. Amit has preference for Lamy and Timex, Veena prefers
Wagon R and Lexi. Harsh's preference for a watch is not Titan.
Questions:
1. Which watch is Suman's preference?
1) Titan 2) Fastrack
3) Samay 4) Cannot be determined
5) None of these
2. Which pen is Harsh's preference?
1) Lamy 2) Pointer
3) Lexi 4) Cannot be determined
5) None of these
3. Which watch is Harsh's preference?
1) Samay 2) Fastrack
3) Timex 4) Cannot be determined
5) None of these
4. Who's preference is Swift?
1) Harsh 2) Amit
3) Veena 4) Cannot be determined
5) None of these
5. Which watch is Veena’s preference?
1) Sarnay 2) Fastrack
3) Titan 4) Cannot be determined
5) None of these
326 / Magical Book on Puzzle

[Bank of India PO Exam–2010]

Puzzle 87
Directions: Study the following information carefully and answer
the questions given below:
In a seven-storey building, having floors numbered one to seven, P, Q, R,
S, T and V each live on a different floor. (The ground floor is numbered floor
no. 1, the floor above it floor no. 2 and so on.) One of the floors in the building
is vacant. P lives on the fifth floor. No floor below fifth floor is vacant; also no
odd numbered floor is vacant. Only S lives between T and V. T does not live on
an odd numbered floor. T does not live on a floor immediately above or
immediately below R's floor. Q does not live on the bottommost floor.
Questions:
1. Who lives on the topmost floor ?
1) Q 2) S 3) R
4) V 5) No one as it is vacant
2. On which of the following floors does R live ?
1) Seventh 2) Sixth 3) Third
4) Second 5) First
3. Which of the following floors is vacant?
1) Seventh 2) Sixth 3) Third
4) Second 5) Fourth
[United Bank of India PO Exam–2010]

Puzzle 88

K KUNDAN
Directions: Study the following information carefully and answer
the questions given below:
In a sports event, different games are scheduled to be held on seven days,
starting on Monday and ending on Sunday. Two games are scheduled to be
held on Saturday as well as on Sunday and one game on each of the remaining
five days. The games to be held in these seven days are: Basketball, football,
boxing, sprinting, swimming, shooting, weightlifting, wrestling and cycling,
but not necessarily in the same order.
Shooting is scheduled to be held on Thursday. Boxing and cycling are
scheduled to be held on the same day. Wrestling is scheduled to be held three
days before basketball, i.e. two sports are scheduled between wrestling and
basketball. Wrestling is not scheduled to be held on Wednesday. Weightlifting
is scheduled immediately after the day boxing is scheduled. Football is scheduled
immediately after the day wrestling is scheduled. Swimming is not scheduled
on Monday.
Questions:
1. Which of the following games is scheduled to be held on Friday ?
1) Basketball 2) Wrestling 3) Swimming
4) Weightlifting 5) Sprinting
2. Which of the following games is scheduled to be held on Sunday?
1) Football 2) Wrestling 3) Basketball
4) Cycling 5) Swimming
3. On which of the following days is sprinting scheduled ?
1) Saturday 2) Friday 3) Wednesday
4) Monday 5) Cannot be determined
4. On which of the following days is weightlifting scheduled?
1) Monday 2) Tuesday 3) Wednesday
4) Friday 5) Sunday
Questions Asked In Exams / 327

5. Shooting is related to Basketball in the same way as Wrestling is related to


1) Sprinting 2) Football 3) Shooting
4) Weightlifting 5) Boxing
[United Bank of India PO Exam–2010]

Puzzle 89
Directions: Study the following information carefully and answer
the questions given below:
A, B, C, D, E, F, G and H are sitting around a square table in such a way
that four of them sit at four corners of the square while four sit in the middle
of each of the four sides. The ones who sit at the four corners face the centre
while those who sit in the middle of the sides face outside.
Two females sit in the middle of the sides and two at the corners. A sits
second to the left of G. G sits in the middle of one of the sides. C sits fourth to
the right of his wife and his wife is not an immediate neighbour of A or G. B
sits third to right of her husband. B does not sit at any of the corners. Only D
sits between B and H. H is the husband of A. E is a male.
Questions:
1. W hich of the following is t r ue with r espect t o t he given seat ing
arrangement?
1) No two males are immediate neighbours of each other
2) G and H do not face each other in the seating arrangement
3) E and D are immediate neighbours of each other
4) F is a male and sits diagonally opposite to E
5) A sits in the centre of one of the sides of the square table

K KUNDAN
2. Who amongst the following is B's husband?
1) C
4) F
2) G
5) Cannot be determined
3) E

3. How many people sit between B and C when counted in anti-clockwise


direction from B ?
1) None
4) Three
2) One
5) Four
3) Two

4. Who amongst the following is the wife of C?


1) D 2) F 3) B
4) G 5) Cannot be determined
5. What is the position of E with respect to C ?
1) Immediately to the left 2) Second to the left
3) Third to the right 4) Immediately to the right
5) Second to the right
[United Bank of India PO Exam–2010]

Puzzle 90
Directions: Study the following information carefully and answer
the questions given below:
P, T, Q, U, S and R are standing in a straight line facing North. S is
standing third from the left end of the line. R is fourth to the right of T.
Neither P nor R is at the end of the line. Q is second to the left of P.
Questions:
1. Who amongst the following is standing fourth from the left end of the
line?
1) Q 2) R 3) P
4) U 5) None of these
2. If TQ : SP then, ? : RU
328 / Magical Book on Puzzle

1) QS 2) TS 3) SP
4) QT 5) None of these
3. Who amongst the following are at the extreme ends of the line?
1) TU 2) TR 3) RU
4) TP 5) US
Directions (Q. 4-5): Four of the following five are alike in a certain
way based on their positions in the above arrangement and so form a
group. Which of the following does not belong to the group ?
4. 1) SRP 2) QPS 3) PUR
4) TSQ 5) TSU
5. 1) UR 2) PS 3) QT
4) SR 5) RP
[IDBI Assistant Manager Exam–2010]

Puzzle 91
Directions: Study the following information carefully and answer
the questions given below:
Six friends Hetal, Jayshree, Rohini, Meena, Nidhi and Swati, stay in
different areas of a city i.e., Dadar, Matunga, Mulund, Vikroli, Thane and
Kanjurmarg not necessarily in the same order. All six have different hobbies
which are singing, dancing, drawing, travelling, cooking and reading not
necessarily in the same order.
Hetal stays in Vikroli and her hobby is neither dancing nor reading. The
one whose hobby is travelling stays in Mulund. Jayshree's hobby is drawing
and she does not stay in either Dadar or Matunga. Meena stays in Kanjurmarg

K KUNDAN
and her hobby is cooking. Rohini likes reading but she and Nidhi do not stay
in Matunga.
Questions:
1. What is Swati's hobby ?
1) Singing 2) Dancing 3) Reading
4) Travelling 5) None of these
2. Whose hobby is singing ?
1) Nidhi 2) Rohini 3) Swati
4) Hetal 5) None of these
3. Whose hobby is travelling ?
1) Hetal 2) Swati 3) Nidhi
4) Rohini 5) None of these
4. Where does Jayshree stay ?
1) Thane 2) Vikroli 3) Kanjurmarg
4) Mulund 5) None of these
5. Where does Rohini stay ?
1) Matunga 2) Mulund 3) Thane
4) Dadar 5) None of these
[IDBI Assistant Manager Exam–2010]

Puzzle 92
Directions: Study the following information carefully and answer
the questions given below:
Seven flights namely Jet Airways. British Airways, Delta, Quantas,
Emirates, Lufthansa and Air India are scheduled to fly to London. There is
only one flight to London on each of the seven days of the week, starting from
Monday and ending on Sunday.
Delta flies on Wednesday. Air India flies the day next to British Airways.
Questions Asked In Exams / 329

British Airways does not fly on Monday or Friday. Two airlines fly between the
days British Airways, and Emirates fly. Emirates does not fly on Sunday.
Quantas flies a day before Lufthansa.
Questions:
1. On which of the following days does Jet Airways fly?
1) Friday 2) Sunday 3) Tuesday
4) Thursday 5) None of these
2. How many flights fly between Lufthansa and Delta ?
1) None 2) One 3) Two
4) Three 5) Five
3. Which of the following flights flies on Friday ?
1) Air India 2) Quantas 3) Emirates
4) Lufthansa 5) Jet Airways
4. If Delta postpones its flight to Sunday owing to some technical reasons
and all the flights scheduled for Thursday to Sunday are now made to take
oil a day ahead of the schedule, which of the following flights would now
fly on Friday?
1) Lufthansa 2) Jet Airways 3) British Airways
4) Air India 5) Quantas
5. If Emirates is related to British Airways and Delta is related to Quantas in
a certain way based upon the given flight schedule, then Jet Airways will
be related to which of the following based upon the same relationship'?
1) Lufthansa 2) Quantas 3) Delta
4) Air India 5) None of these
[Bank of Maharashtra PO Exam–2010]

K KUNDAN
Puzzle 93
Directions: Study the following information carefully and answer
the questions given below:
A, B, C, D, E, F, G and H are sitting around a circular table facing the
centre. No two males or two females are immediate neighbours of each other.
A is wife of H. A sits third to the left of E. F sits second to the right of D.
D is not an immediate neighbour of A or E. H and C are immediate neighbours
of each other. F is not an immediate neighbour of his wife B.
Questions:
1. Which of the following is true about G ?
1) G is a male
2) G sits exactly between F and H
3) G sits third to the left or E
4) G sits second to the right of B
5) None is true
2. Who sits third to the left of B ?
1) F 2) H 3) D
4) A 5) None of these
3. How many people sit between B and F when counted in anti-clockwise
direction from B ?
1) One 2) Two 3) Three
4) Four 5) More than four
4. Four of the following live are alike in a certain way and so form a group.
Which is the one that does not belong to that group ?
1) H 2) F 3) E
4) G 5) D
5. Which of the following groups consists of only female members of the
330 / Magical Book on Puzzle

group?
1) A, B, H 2) G, F, C 3) C, H, G
4) D, H, C 5) None of these
6. If all the persons are made to sit in alphabetical order in clockwise
direction, starting from A, the positions of how many (excluding A) will
remain unchanged as compared to their original seating positions?
1) None 2) One 3) Two
4) Three 5) Four
[Bank of Maharashtra PO Exam–2010]

Puzzle 94
Directions: Study the following information carefully and answer
the questions given below:
A, B, C, D, E, F, G and H are sitting around a circular table facing the
centre. Each one of them has a different profession viz. doctor, engineer,
architect, teacher, clerk, shopkeeper, businessman and banker.
A sits third to right of teacher. D sits second to left of G. G is not an
immediate neighbour of the teacher. Only one person sits between B who is
the shopkeeper and teacher. The one who is an architect sits third to right of
the shopkeeper. H sits between architect and engineer. E is not an immediate
neighbour of H. Engineer sits third to the right of clerk. Only one person sits
between businessman and F. E is neither a businessman nor a doctor.
Questions:
1. W hich of the following is t r ue with r espect t o t he given seat ing
arrangement?

K KUNDAN
1) E is an immediate neighbour of the engineer
2) E is an architect
3) The clerk is an immediate neighbour of the banker
4) The teacher sits between H and the engineer
5) Shopkeeper sits second to the right of the teacher
2. What is the profession of H ?
1) Businessman 2) Architect
3) Banker 4) Teacher
5) Shopkeeper
3. What is the position of doctor with respect to the banker?
1) Immediately to the left 2) Third to the left
3) Second to the right 4) Fourth to the left
5) Second to the left
4. Who sit/s exactly between the architect and the businessman?
1) C and H 2) Clerk
3) Banker and shopkeeper 4) Doctor
5) C and teacher
5. Who amongst the following is a clerk?
1) C 2) D 3) E
4) F 5) G
[UCO Bank PO Exam–2011]

Puzzle 95
Directions: Study the following information carefully and answer
the questions given below:
Twelve people are sitting in two parallel rows containing six people each,
in such a way that there is an equal distance between adjacent persons. In
Questions Asked In Exams / 331

row-1 P, Q, R, S, T and V are seated and all of them are facing South. In row-
2 A, B, C, D, E and F are seated and all of them are facing North. Therefore, in
the given seating arrangement each member seated in a row faces another
member of the other row.
P sits third to the left of T. Neither P nor T sits at an extreme end of the
line. A sits second to the right of E. Neither A nor E faces T or P. A does not
sit at an extreme end. R does not face A and R does not sit at an extreme end
of the line. Only one person sits between F and C. Neither F nor C faces T. C
does not sit at the extreme end. Only one person sits between V and Q. F is
not an immediate neighbour of B and A does not face V.
Questions:
1. How many persons are seated between B and D?
1) One 2) Two 3) Three
4) Four 5) None
2. V is related to B in the same way as Q is related to C. To which of the
following is P related to, following the same pattern?
1) F 2) A 3) D
4) E 5) None of these
3. Which of the following is true regarding V?
1) V faces A
2) T is not an immediate neighbour of V.
3) C faces the one who is second to left of V
4) V sits at one of the extreme ends of the line
5) R sits third to the right of V
4. Who amongst the following sit at extreme ends of the rows?

K KUNDAN
1) D, F 2) V, S 3) Q, S
4) B, D 5) None of these
5. Who amongst the following faces B?
1) V 2) S 3) Q
4) R 5) T
[UCO Bank PO Exam–2011]

Puzzle 96
Directions: Study the following information carefully and answer
the questions given below:
Six chemicals L, M, N, O, P and Q are kept in bottles of different colours
viz, green, red, blue, white, pink and violet, not necessarily in the same order.
These bottles are arranged from left to right, again not necessarily in the same
order.
Chemical M is kept in white bottle. Chemical L is not kept in green bottle
and is kept to the immediate left of the violet bottle. Chemical O is kept in the
blue bottle and is kept exactly between the bottles containing chemicals L and
M. The red bottle is at the extreme left end. The bottles containing chemical Q
is not kept at either of the ends. The green bottle is kept at the extreme right
end. Chemical P is not kept near the white bottle.
Questions:
1. Four of the following five are alike in a certain way based on their positions
in the above arrangement and so form a group. Which is the one that does
not belong to the group?
l) LM 2) LP 3) QO
4) LQ 5) NO
2. Which bottle contains chemical L?
1) Pink 2) Blue
332 / Magical Book on Puzzle

3) Red 4) Cannot be determined


5) None of these
3. Which of the following combinations of chemical and bottle is correct ?
1) P - Red 2) N - Green
3) P - Green 4) Q - Pink
5) None of these
4. Which bottle contains chemical?
1) Pink 2) Green
3) Violet 4) Cannot be determined
5) None of these
5. If all the six chemicals are arranged alphabetically from left to right,
positions of how many will remain unchanged?
1) None 2) One 3) Two
4) Three 5) Four
6. Which bottle contains chemical N?
1) Green 2) Red
3) Pink 4) Cannot be determined
5) None of these
7. Which chemical is kept in the bottle at the extreme right end ?
1) P 2) N
3) L 4) Cannot be determined
5) None of these
[UCO Bank PO Exam–2011]

Puzzle 97

K KUNDAN
Directions: Study the following information carefully and answer
the questions given below:
Six plays A, B, C, D, E and F are to be staged starting from Monday and
ending on Sunday with one of the days being an off day, not necessarily in the
1 1
same order, Each of the plays has different time duration: hour, 1 hour, 1
2 2
1
hours, 2 hours, 2 hours and 3 hours, again not necessarily in the same
2
order.
1
Sunday is not an off day and a Play of hour duration is staged on that
2
day. Play A is staged immediately before Play E. There are two plays staged
1
between Play F which is for 3 hours and Play C which is for 1 hours. The
2
off day is after the staging of Play E and there are two days between the off day
and Play A. Play D which is for 2 hours is not staged on Monday. The play
1
staged immediately before the off day is of 3 hours. Play A is for less than 2
2
hours.
1. What is the time duration of Play B?
1
1) 2 hours 2) 2 hours 3) 1 hour
2
4) Vi hour 5) None of these
2. Which day is the off day?
1) Tuesday 2) Monday 3) Friday
4) Saturday 5) Cannot be determined
Questions Asked In Exams / 333

3. Which of the following combinations of Play - Day - Time Duration is


correct ?
1) E - Wednesday - 2 hours 2) A - Tuesday - 1 hour
1
3) C - Thursday - 1 hours 4) F - Tuesday - 3 hours
2
5) None is correct
4. On which day is Play D staged?
1) Wednesday 2) Saturday 3) Tuesday
4) Friday 5) Cannot be determined
5. How many plays are staged before the off day?
1) Two 2) One 3) Five
4) Three 5) None of these
Directions (Q. 6-7): Keeping all the other information the same, if D
is staged on Monday, then ________
6. A play of what time duration would be staged on Thursday?
1
1) 2 hours 2) 2 hours 3) 1 hour
2
4) 3 hours 5) Cannot be determined
7. Which day would be the off day?
1) Tuesday 2) Monday 3) Friday
4) Saturday 5) Cannot be determined
[UCO Bank PO Exam–2011]

Puzzle 98

K KUNDAN
Directions: Study the following information carefully and answer
the questions given below:
Eight people S, R, N, L, M, T, O and P are sitting in a circle facing the
centre. All eight belong to different professions — reporter, doctor, cricketer,
teacher , accountant, shopkeeper , paint er and supervisor. They are not
necessarily seated in the mentioned order.
M is sitting third to the left of O. The doctor is to the immediate right of M
and M is not a reporter. R is sitting fourth to the right of P. Neither R nor P is
an immediate neighbour of M. T is a teacher and is sitting third to the right of
the doctor. The shopkeeper is sitting second to the left of the teacher. The
painter is sitting second to the left of M. S the cricketer is sitting exactly
between T and P. The accountant is sitting second to the right of the cricketer.
N is sitting third to the left of T.
Questions:
1. Who amongst the following is a reporter ?
1) O 2) L 3) N
4) R 5) None of these
2. What is S's position with respect to R?
1) Third to the right 2) Second to the right 3) Third to the left
4) Second to the left 5) Fourth to the right
3. How many people are sitting between P and N when counted in an anti-
clockwise direction from N ?
1) One 2) Two 3) Three
4) Four 5) None
4. Four of the following five pairs are alike in a certain way based on their
positions in the above arrangement and so form a group. Which of the
following does not belong to the group ?
1) Teacher - Painter 2) Supervisor - Shopkeeper
334 / Magical Book on Puzzle

3) Cricketer - Reporter 4) Doctor - Accountant


5) Shopkeeper - Doctor
5. Which one of the following statements is false according to the above
mentioned arrangement?
1) N is to the immediate right of the supervisor
2) The cricketer is third to the right of the shopkeeper
3) The doctor is sitting exactly between the supervisor and the accountant
4) L is neither a teacher nor a supervisor
5) There are only three people between S and N
[RBI Grade ‘B’ Officers’ Exam–2011]

Puzzle 99
Directions: Study the following information carefully and answer
the questions given below:
Six friends — Deepak, Varun, Anit, Nilesh, Rajesh and Siddharth are
studying six different specialisations of engineering which are — metallurgy,
t elecommunication, sof tware, mechanical, elect r ical and har dware not
necessarily in the same order. Each one likes a different sport - hockey, cricket,
swimming, football, badminton and tennis again not in the same order.
Nilesh is not studying hardware Rajesh is studying software and likes
hockey. Anit likes swimming and is not studying hardware. The one who
likes football is studying electrical. Siddharth is studying mechanical and
does not like t ennis. The one who likes badmint on is st udying
telecommunication. Deepak and Varun do not like badminton. Deepak does
not like tennis.

K KUNDAN
Questions:
1. Which specialisation is Varun studying?
1) Metallurgy 2) Mechanical 3) Hardware
4) Electrical 5) None of these
2. Which sport does Deepak like?
1) Football 2) Cricket
3) Hockey 4) Cannot be determined
5) None of these
3. Which of the following person-specialisation combination is correct
according to the given information?
1) Nilesh-Hardware 2) Varun-Electrical
3) Anit-Metallurgy 4) Siddharth - Software
5) None is correct
Directions (Q. 4-5): If all six friends are asked to sit in a straight line,
facing north, in an alphabetical order (according to their names), from left
to right, then ________
4. Who will be to the immediate left of the one studying electrical?
1) The one who likes badminton
2) The one who is studying telecommunication
3) The one who is studying hardware
4) The one who likes hockey
5) None of these
5. Which of the following combinations will represent the favourite sport of
the immediate neighbours of Rajesh?
1) Badminton - Football 2) Cricket-Tennis
3) Cricket-Football 4) Tennis - Football
5) Cricket - Badminton
Questions Asked In Exams / 335

[RBI Grade ‘B’ Officers’ Exam–2011]

Puzzle 100
Directions: Study the following information carefully and answer
the questions given below:
Six friends A, B, C, D, E and F working in the same office take different
time to reach office. All of them take time in the multiples of ten in such a
manner that the one who reaches office the earliest, reaches in 10 minutes
and the one who takes maximum time reaches office in 60 minutes. D takes
more time than E but less time than A. A reaches in 30 minutes. B takes less
time than only F.
Questions:
1. How much time does C take to reach office?
1) 60 minutes 2) 50 minutes 3) 40 minutes
4) 20 minutes 5) Cannot be determined
2. Who amongst the following takes maximum time to reach office?
1) B 2) C 3) D
4) F 5) Cannot be determined
3. How many people take more time to reach office than D?
1) Four 2) Three 3) Two
4) One 5) None
[RBI Grade ‘B’ Officers’ Exam–2011]

Puzzle 101
Directions: Study the following information carefully and answer

K KUNDAN
the questions given below:
K, L, M, P, Q, R, S and T are sitting around a square table in such a way
that four of them sit at four corners of the square while four sit in the middle
of each of the four sides. The ones who sit at the four corners face outside
while those who sit in the middle of the sides face the centre of the table.
P sits third to the right of S. S faces the centre. Q sits third to the left of
M. M does not sit in the middle of the sides. Only one person sits between Q
and R. R is not an immediate neighbour of M. T faces the centre. K is not an
immediate neighbour of R.
Questions:
1. What is position of M with respect to L ?
1) Third to the right
2) M and L sit diagonally opposite to each other
3) Second to the right
4) Second to the left
5) Fifth to the right
2. Who sits exactly between Q and R?
1) T 2) P 3) K
4) M 5) S and K
3. Which of the following pairs represents the persons seated in the middle
of the sides who face each other?
1) S, Q 2) K, L 3) M, P
4) R, T 5) T, Q
4. Who amongst the following sit between R and K when counted in anti-
clockwise direction from K?
1) No one sits between R and K as R and K are immediate neighbours of
each other
2) S, P and L
336 / Magical Book on Puzzle

3) P and Q
4) L and R
5) M, S and T
5. If K is made to face the opposite direction, who would sit to his immediate
right?
1) R 2) Q 3) P
4) T 5) S
6. Four of the following five are alike in a certain way and so form a group.
Which is the one that does not belong to that group?
1) L 2) M 3) K
4) P 5) R
[RBI Grade ‘B’ Officers’ Exam–2011]

Puzzle 102
Directions: Study the following information carefully and answer
the questions given below:
Twelve friends A, B, C, D, E, F, G, H, I, J, K and L were born in different
months of the same year. A was born in the month of April and G was born in
the month of August. J was born in the month immediately preceding the
month in which K was born and immediately succeeding the month in which
C was born. J was not born in the month of October nor in February. There is
a gap of two months between the birthdays of L and B. There were 30 days in
the month in which L was born. D was born in the month immediately after
the month in which I was born. There were 31 days in the month in which D
was born. There is a gap of one month between the birthdays of B and F. E and

K KUNDAN
H were born in that months which had 31 days each.
Questions:
1. In which of the following months B was born?
1) December 2) June 3) March
4) November 5) September
2. Four of the following five are alike in a certain way and hence form a
group. Which one of the following does not belong to that group?
1) L 2) A 3) J
4) K 5) B
3. Who among the following was born in the month of February?
1) J 2) L 3) E or H
4) C 5) I
4. How many friends did celebrate their birthdays after F ?
1) None 2) Three 3) Four
4) Five 5) Six
5. If I is related A and B is related to J on the basis of months in which they
were born, then with which of the following L is related on the same
basis?
1) G 2) A 3) K
4) E 5) H
[RBI Grade ‘B’ Officers’ Exam–2011]

Puzzle 103
Directions: Study the following information carefully and answer
the questions given below:
Twelve persons are sitting in two parallel lines in such a way that there
are six persons in each row at equidistance. A, B, C, D, E and F are sitting in
row-1 and they face towards east. P, Q, R, S, T and V are sitting in row-2 and
Questions Asked In Exams / 337

they face towards west. One person of one row faces the other person of the
other row.
P, who is sitting at one of the ends of the row, is second to the right of T.
A does not face P or T. A is third to the left of F. There are two persons
between Q and V. There is only one person between C and D. C and D do not
face P. B is neighbour of C. S, who does not face D, is not the neighbour of Q.
Questions:
1. Who among the following faces F?
1) Q 2) T 3) S
4) R 5) V
2. How many persons are sitting between E and C?
1) One 2) Two 3) Three
4) Four 5) None
3. E is related to R in the same way as B is related to S. On this pattern F is
related to
1) V 2) P 3) T
4) Q 5) None of these
4. Which of the following statements is true about B?
1) Q is the neighbour of that person who faces B
2) C is not the immediate neighbour of B
3) E is second to the right of B
4) T faces B
5) B is second from the left end of the row
5. Who are sitting at the ends of the row?
1) P, V 2) F, A 3) D, E

K KUNDAN
4) P, S 5) None of these
[RBI Grade ‘B’ Officers’ Exam–2011]

Puzzle 104
Directions: Study the following information carefully and answer
the questions given below:
Seven friends A, B, C, D, E, F and G studied in colleges X, Y and Z and
are currently in different professions, namely Medicines, Fashion Designing,
Engineering, Business, Acting, Teaching and Architecture (not necessarily in
the same order). At least two and not more than three friends had studied in
the same college.
C is an architect and studied in college Y. E is not a businessman. Only G
amongst the seven friends studied in college X along with E. F is an engineer
and did not study in college Y. B is an actor and did not study in the same
college as F. A did not study in college Z. Those who studied in college X are
neither Fashion Designers nor teachers. None of those who studied in college
Y is a teacher.
Questions:
1. Who amongst the following have studied in college Z?
1) B, A 2) C, F 3) B, D, F
4) A, D 5) D, F
2. Which of the following groups represents the students of college Y?
1) C, E, G 2) A, C, D 3) A, B, C
4) D, B, C 5) None of these
3. What is the profession of F?
1) Engineering 2) Business 3) Medicines
4) Acting 5) None of these
4. Who amongst the following is in the profession of Medicine?
1) E 2) G 3) A
338 / Magical Book on Puzzle

4) D 5) None of these
5. What is the profession of A?
1) Teaching 2) Medicine 3) Business
4) Fashion Designing 5) None of these
6. Which of the following combinations of person, college and profession is
definitely correct?
1) E-X-Fashion Designing 2) F-X-Engineering
3) A-Y-Businessman 4) D-Z-Teaching
5) None of these
7. Who amongst the following is a teacher?
1) A 2) D 3) E
4) G 5) None of these
[Syndicate Bank PO Exam–2010]

Puzzle 105
Directions: Study the following information carefully and answer
the questions given below:
Eight friends Q, R, S, T, V, W, Y and Z are sitting around a circular table,
facing the centre. There are three males and five females in the group of
friends. No two males are immediate neighbours of each other.
l V sits second to the right of his wife.
l S sits third to the right of V.
l W sits second to the right of her husband Z. Z is not an immediate
neighbour of V’s wife.
l T is a male and Y is not an immediate neighbour of V.

K KUNDAN
l R sits second to the right of Q.
Questions:
1. What is the position of T with respect to Z?
1) Second to the left
2) Immediately to the right
3) Third to the left
4) Second to the right
5) Third to the right
2. Which of the following statements regarding S is definitely correct?
1) S is one of the male members of the group.
2) Both the immediate neighbours of S are females.
3) S sits third to the left of T.
4) W is an immediate neighbour of S.
5) S sits second to the right of Q.
3. Who amongst the following is V’s wife?
1) Q 2) Y 3) R
4) T 5) None of these
4. Who amongst the following has a male sitting to the immediate left and
the right?
1) Y 2) R 3) Q
4) S 5) None of these
5. Which of the following is not true regarding T?
1) T is an immediate neighbour of Z’s wife.
2) No male is an immediate neighbour of T.
3) Q sits second to right of T.
4) The one who sits third to the left of T is a male.
5) All are true
6. Which of the following pairs represents the immediate neighbours of T?
Questions Asked In Exams / 339

1) RQ 2) WZ 3) YV
4) WY 5) None of these
7. How many people sit between V and S when counted in anti-clockwise
direction?
1) None 2) One 3) Two
4) Three 5) Four
8. Who amongst the following sits exactly between V and Y?
1) Q 2) W 3) R
4) T 5) Z
[Syndicate Bank PO Exam–2010]

Puzzle 106
Directions: Study the following information carefully and answer
the questions given below:
In an international meet, representatives A, B, C, D, E, F G and H from
eight different countries, viz Thailand, France, Holland, Austria, US, Spain,
India and Germany (not necessarily in the same order), sit around a circular
table facing the centre. A, who represents Germany, sits third to the left of E.
The one who is from India sits on the immediate right of A. D, who is from
Holland, sits second to the right of B. B is not an immediate neighbour of E. C,
who is from Spain, sits exactly in the middle of people representing US and
India. G, the representative from France, sits second to the left of H, who is
from Thailand.
Questions:
1. How many people sit between A and the representative from Austria when

K KUNDAN
counted in clockwise directions?
1) None
4) Three
2) One
5) Four
3) Two

2. Which of the following is TRUE regarding representative F?


1) F is the representative of Austria.
2) F sits second to the left of the representative from Thailand.
3) The representative from Germany is not an immediate neighbour of F.
4) E sits third to the right of F.
5) None of these
3. In which of the following pairs is the representative from the first country
sitting on the immediate left of the representative from the second country?
1) India-Thailand 2) Austria-US 3) Thailand-Holland
4) Spain-Germany 5) US-Spain
4. What is the position of B with respect to the position of the representative
from the US?
1) Third to the left 2) Fourth to the right 3) Fourth to the left
4) Second to the right 5) Second to the left
5. Which of the following pairs represents the immediate neighbours of the
representative from Holland?
1) CG 2) BE 3) AH
4) HB 5) GE
6. The representative from which of the following countries is seated second
to the left of the Indian representative?
1) Thailand 2) US 3) France
4) Spain 5) None of these
7. The representative from which of the following countries sits exactly in
the middle of the representatives from Thailand and France?
1) Holland 2) Austria 3) Germany
4) US 5) None of these
[NABARD Bank PO Exam–2010]
340 / Magical Book on Puzzle

Answers and explanations


Puzzle 1

Person Profession Car Gender


T Air-hostess Ikon Female
R Professor Scorpio Male
L Jeweller Corolla Male
W Lawyer Lancer Male
Z Consultant Santro Male
S Doctor Esteem Female
M Travel Agent Alto Female
1. 4 2. 3 3. 1 4. 2 5. 2

Puzzle 2

Person Food Day


K Thai Thursday
B Italian Friday
M Indian Saturday

K KUNDAN
1. 5; B
R
W
N
T
2. 4 3. 3
Spanish
Continental
Mexican
Chinese
4. 1
Tuesday
Sunday
Monday
Wednesday
5. 2

Puzzle 3
From the clues (i), (iii), (iv) (v) and (vi) we get three possible arrangements
for Monday to Saturday. These arrangements are as follows:
1. R, Holiday, Q, T, S, P
2. T, S, P, R, Holiday, Q
3. S, P, R, Holiday, Q, T
Reject the possibilities 1 and 2 because of clue (ii). Hence, the arrangement
3 is the correct one. Thus, we get

S.No. Day Person


1. MON S
2. TUE P
3. WED R
4. THU Holiday
5. FRI Q
6. SAT T
1. 4 2. 5; Wednesday 3. 1 4. 2
Questions Asked In Exams / 341

Puzzle 4
Arrange the whole information in the following form.

S. No. Person Specialisation Colour


1 P(+) Journalism Yellow
2 Q(-) Marketing Pink
3 D(+) Pharmacy Red
4 N(-) HR Green
5 R (+) IT Orange
6 F(-) Civil Grey
7 T(+) Finance Blue

1. 3 2. 2 3. 5; IT 4. 1 5. 4

Puzzle 5
College Student
A P(Electronics), Z (Electronics), S(Mechanical)
B W (Electrical), T (Mechanical)
C R (Mechanical), V (Electrical), Q (Electrical)
1. 4 2. 3 3. 1 4. 5 5. 1

K KUNDAN
Puzzle 6
Marketing: H(DD), A(BBC), (Aaj Tak)
Finance: C(Sony), F(NDTV), E(Star)
Production: D(ESPN), G(Zee)
1. 3 2. 3 3. 2 4. 5; H, A and B 5. 1

Puzzle 7
Product Manager and city
1. X Q(Jaipur), E(Cochin), M(Ahmedabad)
2. Y T(Delhi), D (Bangalore)
3. Z J(Hyderabad), R (Chennai), P(Pune)
1. 5; Q, E and M
2. 2 3. 1 4. 4 5. 3

Puzzle 8

Day Game
Mon Chess
Tue Rest
Wed Volleyball
Thu Carrom
Fri Table Tennis
Sat Kho-kho

1. 2 2. 5; One day 3. 3 4. 4 5. 1
342 / Magical Book on Puzzle

Puzzle 9
I . D() (X), F() (Y), H () (Z)

II. E () (X), C () (Y), G () ( Z)

III. ——— A() (Y), B() ( Z)


1. 4 2. 3 3. 1 4. 5 5. 2

Puzzle 10

W orker Shift W eekly Off


L I Tue
M II Thu
N I Sun
P III Sat
Q III Wed
R II Fri
S I Mon

1. 3 2. 1 3. 2 4. 4 5. 5; M

Puzzle 11

K KUNDAN Av
Aa Ch

Mo

Va Dh

Ga De
1. 3 2. 3 3. 2 4. 1 5. 5

Puzzle 12

Professional City Profession


A Bhubaneshwar Pharmacist
B Hyderabad Professor
C Mumbai Artist
D Bangalore Engineer
E Ahmedabad Lawyer
F Chennai Doctor
G Jaipur Counsellor

1. 3 2. 1 3. 4 4. 2 5. 5; F
Questions Asked In Exams / 343

Puzzle 13

Candidate Panel Company


Nilesh IV A
Samir III G
Harish V B
Nikita VI E
Shailaja I D
Laxman VII F
Sujata II C

1. 4 2. 1 3. 2 4. 3 5. 1

Puzzle 14

Friend Shift Day off


P II Tuesday
Q I Monday
R II Wednesday

K KUNDAN S
T
V
W
I
III
III
I
Sunday
Friday
Thursday
Saturday

1. 3 2. 4 3. 1 4. 4 5. 3

Puzzle 15

F
W M

T N

R H
P

1. 1
2 . 5; N
3. 4 4. 4 5. 3 6. 1
344 / Magical Book on Puzzle

Puzzle 16

G
H F

A D

E B
C
1. 3 2. 4 3. 1 4. 5 5. 2

Puzzle 17

M ember Country Sport


K China Archery
T USA Football
F France Volleyball
L Australia Athletics
J Russia Tennis
R Korea Rifle Shooting

K KUNDAN
1. 4

Puzzle 18
H
2. 4 3. 1
Japan
4. 2

Rani
5. 2
Boxing

Ashwini
Sudha

Meeta
Geeta

Priya Mukta

1. 3
2 . 5; Geeta
3. 4 4. 2 5. 5
Questions Asked In Exams / 345

Puzzle 19

M ember Sports Instument


A Badminton Flute
B Carrom Banjo
C Lawn Tennis Harmonium
D Table Tennis Tabla
E Bridge Santoor
F Football Guitar
G Hockey Sitar
1. 3 2. 1 3. 5 4. 4 5. 3

Puzzle 20
A
G E

B C

H F
D
1. 1

K KUNDAN
2 . 5; Third to the left
3. 3 4. 2 5. 4

Puzzle 21

Student College Subject


P( ) B MBA
Q() A BCA
R() B Medicine
S( ) A Journalism
T( ) A BCA
W( ) C Aviation
Z() C Medicine

1. 5; RZ 2. 1 3. 1 4. 4 5. 2

Puzzle 22

W M

D
R

T
P

1. 3 2. 4 3. 1 4. 3 5. 5; R
346 / Magical Book on Puzzle

Puzzle 23

1. 3 2. 1
3. 2;

Thus only R’s position does not change.


4. 4
5. 5; In all others the first person is second to the left of the second person.

Puzzle 24

K KUNDAN M ember
A (+)
B (–)
C (–)
D (–)
Profession
Engineer
Lawyer
Doctor
Professor
College
Y
P
X
Z
E (+) Banker S
F (+) Stockbroker W
G (+) Businessman V

1. 4 2. 2 3. 3 4. 1 5. 5

Puzzle 25

F
A G

H B

D C
E

1. 1 2. 4 3. 3 4. 5 5. 2
Questions Asked In Exams / 347

Puzzle 26

M ember City M other tongue


H Chennai Marathi
I Hyderabad Telugu
J Bangalore Kannada
K Ahmedabad Punjabi
L Delhi Bangla
M Kolkata Tamil
N Mumbai Hindi
1. 4 2. 1 3. 2 4. 3 5. 5; Tamil

Puzzle 27

T R
P
W
Q
S V
1. 1 2. 4 3. 2 4. 51 5. 1

Puzzle 28

K KUNDAN H
E
C
B

F G
A
1. 1 2. 2 3. 4
4. 3; In all others, the second person is second to the right of the first.
5. 5

Puzzle 29

Friend College Branch


A Z Mechanical
B Y Civil
C X Chemical
D X Electrical
E Z Computer
F Y Aeronautical
G Z Electronics
1. 4 2. 1 3. 5 4. 3 5. 2
348 / Magical Book on Puzzle

Puzzle 30

F
A C or G

D B

G or C E
H
1. 4; C or G 2. 2 3. 5 4. 3 5. 5; E

Puzzle 31

Student Subject Sport


P Biology Cricket
Q History Badminton
R Philosophy Hockey
S Geography Basketball
T English Football
V Physics Table Tennis

K KUNDAN
1. 2

Puzzle 32
W

2. 3 3. 4
Chemistry

4. 1
Volleyball

5. 5; English

Student Class Colour


P II Green
Q III Black
R IV Red
S I Pink
T VI Yellow
M V Blue

1. 3 2. 5; Red 3. 1 4. 5 5. 4

Puzzle 33

1. 2 2. 3 3. 4 4. 2 5. 1 6. 5; G
Questions Asked In Exams / 349

Puzzle 34
G
A B
H D

E C
F
1. 1 2. 2 3. 4 4. 5 5. 3

Puzzle 35

Student Standard Subject


P V Geography
Q VII History
R VI English
S IV Maths
T VIII Hindi
V X Science
W IX Sanskrit

1. 2 2. 1 3. 3 4. 5; V 5. 4
Puzzle 36

K KUNDAN
A > D > G ... (ii);
C > E > H ... (iii)
D > B > F ... (iv);
G > C ... (v); F > G ... (vii)
Combining these, we get
A > D > B > F > G > C > E > H
1. 5
2. 2; A > D > B > F > G > C, J > E > H
3. 1
4. 2; G, C, E and H.
5. 5

Puzzle 37

1. 5 2. 4 3. 1 4. 3 5. 2
350 / Magical Book on Puzzle

Puzzle 38

Friend Profession Bank


A Forex Officer S
B Agriculture Officer M
C Economist N
D Terminal Operator L
E IT Officer R
F Clerk Q
G Research Analyst P

1. 2 2. 2 3. 3 4. 3 5. 5
6. 4 7. 1

Puzzle 39

K KUNDAN
1. 4

Puzzle 40
2. 1 3. 2 4. 3 5. 4

1. 2 2. 3 3. 4
4. 5; H
5. 1;

All positions change.


Questions Asked In Exams / 351

Puzzle 41

G
B
F
H
E

D K

A C
1. 1 2. 4 3. 3 4. 2 5. 5

Puzzle 42

Employee Department Sport


A Pers TT
B Admin Football
C Admin Hockey
D Admin Basketball
E Mktg Cricket
F Pers Volleyball

K KUNDAN
G Mktg LT
H Mktg Badminton

1. 3 2. 2 3. 5 4. 1 5. 4

Puzzle 43

1. 4 2. 3 3. 2 4. 5 5. 1

Puzzle 44

1. 5 2. 2 3. 4 4. 3 5. 1
352 / Magical Book on Puzzle

Puzzle 45

1. 5 2. 1 3. 3 4. 1 5. 2

Puzzle 46

Day Play
Monday No play
Tuesday A
Wednesday D
Thursday C
Friday B
Saturday E

K KUNDAN
1. 2 2. 1 3. 3

Puzzle 47

1. 2 2. 1 3. 5; A 4. 4 5. 3

Puzzle 48

Date Day Play


25 Monday A
26 Tuesday Z
27 Wednesday B
28 Thursday No play
29 Friday M
30 Saturday Q
31 Sunday X
1. 5; A
2. 1 3. 2 4. 4 5. 3
Questions Asked In Exams / 353

Puzzle 49

1. 1 2. 3
3. 2; Rakesh, Mukesh and Ajay
4. 4; The family doesn’t have Ajay’s grandfather.
5. 4

Puzzle 50

1. 4; D or G
2 . 5; F

K KUNDAN
3. 1 4. 2 5. 2

Puzzle 51

Students Class Play


A VIII or X Volleyball
B X Table Tennis
C IX Lawn Tennis
D IX Badminton
E VIII or X Cricket or Basketball
F VIII Football
G VIII Basketball or Cricket
H X Hockey
1. 2 2. 4 3. 2 4. 1 5. 4

Puzzle 52
Friend Class Colour
M VIII Blue
V IV Yellow
K IX White
D VI Black
T VII Green
J V Red
R III Purple
1. 1 2. 4 3. 5 4. 3 5. 2
354 / Magical Book on Puzzle

Puzzle 53

A English R
B History V
C Maths T
D Histroy W
E English S
F Maths P
G Maths Q

1. 4 2. 2 3. 3 4. 1 5. 5

Puzzle 54

A Jaipur Jet
B Chennai Kingfisher
C Kolkata Indian Airlines
D Bangalore Air India
E Delhi Air Deccan

K KUNDAN
F Hyderabad Spicejet
G Ahmedabad Sahara

1. 3 2. 1 3. 2

Puzzle 55
On the basis of given information and conclusions as well as sub-
conclusions drawn from them we can construct the following chart:

S. No. Day Sports


1st Day Wednesday Cricket
2nd Day Thursday Hockey
3rd Day Friday Tennis
4th Day Saturday Volleyball
5th Day Sunday Football
6th Day Monday Holiday
7th Day Tuesday Badminton

1. 3; Volleyball was telecast on fourth day, i.e., Saturday.


2 . 2; Monday was holiday.
3. 4; Football was telecast before holiday and Badminton was telecast after
holiday.
4. 1; Cricket was telecast on Wednesday.
5. 5; On Sunday football was telecast.
Questions Asked In Exams / 355

Puzzle 56

M ember Sex Profession Relationship


M M Farmer Father of O, Q and N
N F Teacher Daughter of M
O M Businessman Husband of P
P F Doctor Wife of O
Q M Lawyer Brother of O

1. 2; Clearly, ‘P’ is doctor in this family.


2. 3; Clearly, ‘O’ is businessman in this family.
3. 4; ‘N’ and ‘P’ are women.
4. 4; ‘O’ and ‘P’ are couple.
5. 3; Clearly, ‘M’ ‘O’ and ‘Q’ are male members of this family.
Puzzle 57
1. 1; Of the six companies if ‘O’ is the, first, L is third and order, ‘J’, ‘K’, ‘M’
and ‘J’, ‘N’ is to be followed. Clearly, ‘J’ must be visited second.
2 . 1; According to information, L must be in third place and the order J, K,
M must not be violated. This is followed only in the arrangement J, O,
L, K, N, M.

Puzzle 58

K KUNDAN
1. 5 2. 1 3. 2 4. 4 5. 3

Puzzle 59

Persons Companies Days


R C Tuesday
J D Wednesday
M A Thursday
Q G Sunday
L F Saturday
T E Friday
K B Monday

1. 3 2. 1 3. 4 4. 5 5. 2
356 / Magical Book on Puzzle

Puzzle 60

M embers Colours Sports


R Green Badminton
V Grey Table Tennis or Carrom
T Blue Chess
Q Black Lawn Tennis
S White Carrom or Table Tennis
W Yellow Basketball
P Red Volleyball
1. 2 2. 3 3. 4 4. 3 5. 5 6. 2

Puzzle 61

1. 3 2. 3 3. 1 4. 1

K KUNDAN
5. 5; Second is left of the first.

Puzzle 62

B
G
Marketing
Engineering
Nagpur
Chennai
H Marketing Ranchi
K Engineering Kolkata
D Systems Hyderabad
F Systems Kozikode
T Marketing Patna
V Systems Bhopal

1. 2 2. 1 3. 3 4. 4 5. 5

Puzzle 63

H
E B

C F

G
D
A
1. 2 2. 3 3. 1 4. 2 5. 4 6. 5
Questions Asked In Exams / 357

Puzzle 64
According to age,
Sourav > Rahul > Anuj > Kunal > Harish
According to height,
Kunal > Rahul > Sourav > Harish > Anuj
1. 1 2. 3 3. 4 4. 3 5. 2

Puzzle 65

1. 2 2. 3 3. 4 4. 5 5. 5

Puzzle 66

M embers States Games

K KUNDAN A
B
C
D
Madhya Pradesh
Bihar
Orissa
Kerala
Badminton
Table-Tennis
Billiards
Chess
E Tamil Nadu Golf
F Maharashtra Cricket
G Uttar Pradesh Hockey

1. 1 2. 3 3. 3 4. 4 5. 2

Puzzle 67

1. 1 2. 2 3. 4 4. 1 5. 5
358 / Magical Book on Puzzle

Puzzle 68

Boys Games Subjects


A Hockey Psychology
B Cricket Biology
C Carrom English
D Football German
E Badminton Physics
F Volleyball Economics
G Table Tennis Bio-Chemistry

1. 1 2. 3 3. 5 4. 4 5. 2

Puzzle 69

Rani

Ashwini
Sudha

K KUNDAN Meeta

Priya Mukta
Geeta

1. 3
2 . 5; Geeta
3. 4 4. 2 5. 5

Puzzle 70

M ember Sports Instument

A Badminton Flute
B Carrom Banjo
C Lawn Tennis Harmonium
D Table Tennis Tabla
E Bridge Santoor
F Football Guitar
G Hockey Sitar

1. 3 2. 1 3. 5 4. 4 5. 3
Questions Asked In Exams / 359

Puzzle 71
R

W P

Q T

S
V
1. 3
2 . 5; V is third to the left of P.
3.1 4.2 5.4

Puzzle 72

1. 2 2. 3 3. 3 4. 1 5. 4

K KUNDAN
Puzzle 73

Friends
A
B
Departments
Marketing
Finance
Organisations
Q
T
C Marketing S
D Finance Z
E IT W
F HR R
G IT P
1. 5 2. 1 3. 2 4. 4 5. 3

Puzzle 74

1. 4 2. 3 3. 2 4. 5 5. 1
360 / Magical Book on Puzzle

Puzzle 75

H
D M
B A
K
T R
W
1. 2 2. 5 3. 3 4. 1 5. 5

Puzzle 76

V
Q Z

P S

W
T R
Y
1. 4; V is sitting between Z and Q.
2. 1; Q is second to the right of Z.

K KUNDAN
3. 5; Y is sitting to the immediate right of T.
4. 3; T is third to the left of S.
5. 2; R is third to the right of W.

Puzzle 77

Person Department Scale


A Finance III
B Sales III
C Finance III
D Sales I
E Finance III
F Operations II
G Sales I
H Operations I

1. 4; The combination Finance - E-III is correct.


2. 3; B, D and G work in Sales department.
3. 2; H belongs to scale II.
4. 3; G belongs to scale I.
5. 3; C belongs to scale III.
Questions Asked In Exams / 361

Puzzle 78

1. 1 2. 5 3. 4
4. 4 5. 3 6. 1
Puzzle 79

K KUNDAN
1. 2
4. 3

Puzzle 80
2. 5
5. 5
3. 1

1. 1; B is third to the right of G.


2. 2; D is sitting between G and C.
3. 3; F is third to the right of A.
4. 1; G is to the immediate left of D.
5. 3; A is fourth to the left of G.
362 / Magical Book on Puzzle

Puzzle 81
Amrita > Rhea > Smita > Deepa

Friends Number of Years


Amrita 3
Rhea 5
Smita 6 or 7
Deepa 8
1. 2 2. 4 3. 4
4. 4; Meaningful words are TIDE, TIED and DIET.
5. 1; Second, fifth, eighth and the eleventh letters from left are E, L, M and
T respectively.
Meaningful word is MELT of which second letter from the left is E.

Puzzle 82
T
P S

R Q

W V
Z

K KUNDAN Friends
S
V
T
P
Professions
Banker
Cricketer
Lawyer
Professor
Q Pilot
R Architect
W Doctor
Z Businessman
1. 5 2. 2 3. 5 4. 4 5. 4 6. 2
7. 1 8. 5

Puzzle 83
On the basis of given information following seating arrangement can be
made:
B

F E

D C

A
Questions Asked In Exams / 363

1. 3; There are thr ee persons D, A and C bet ween F and E, if we go


anticlockwise from F to E.
2 . 1; C is sitting to the immediate left of E.
3. 2; In all others there are two persons between the first and the second
person. While in BD, only one person is sitting.
4. 3; Except in FDA, in all others the three persons are seated in clockwise.
5. 4; Except AF, all other pairs indicate the persons seated adjacent to
each other.

Puzzle 84
Following seating arrangement can be possible:

R Q V T U S

1. 5; There is no person between T and V.


2 . 4; Except in UT, in all others the first person is sitting to the immediate
left of the second person.
3. 5; Q is fourth to the left of S.
4. 4; R and S are seated at the two extremes.
5. 2; S is second to the right of T.
T is second to the right of Q.
U is second to the right of V.

Puzzle 85

K KUNDAN
On the basis of given information following seating arrangement can be
possible:

U A

C V

1. 3; B and C are not facing the centre. Since the information ‘U is second
to the right of A’ given in the question indicates that A is facing the
centre.
2 . 4; T is third to the left or right of B.
3. 3; V is second to the left of C or fourth to the right of C.
4. 4; Option 4 is correct.
5. 1; A is second to the left of U.
364 / Magical Book on Puzzle

Puzzle 86
On the basis of information given in the question, following table can be
constructed:

Person Pen Car W atch


Suman Parker Mica Samay
Mrudula Cello City Citizen
Amir Lamy Santro Timex
Veena Lexi Wagon R Titan
Harsh Pointer Swift Fastrack

1. 3; Suman likes Samay watch.


2. 2; Harsh likes Pointer pen.
3. 2; Harsh likes Fastrack watch.
4. 1; Harsh likes Swift car.
5. 3; Veena likes Titan watch.

Puzzle 87
On the basis of given information, following table can be constructed:

Floor Person
7 Q

K KUNDAN 6
5
4
3
2
Vacant
P
T
S
V
1 R

1. 1; Q lives on the topmost floor.


2 . 5; R lives on the first floor.
3. 2; Sixth floor is vacant.

Puzzle 88
Days Games
Monday Sprinting
Tuesday Wrestling
Wednesday Football
Thursday Shooting
Friday Basketball
Saturday Boxing, Cycling
Sunday Weightlifting, Swimming
Questions Asked In Exams / 365

1. 1; Basketball
2. 5; Weighlifting and swimming
3. 4; Monday
4. 5; Sunday
5. 2; Basketball is scheduled immediately after Shooting. Similarly, Football
is scheduled immediately after Wrestling.

Puzzle 89

B (Female) D (Female)
(Female) F

(Male) G H (Male)

(Male) C E (Male)
A (Female)

H is husband of A. C is husband of D. E is husband of B.


1. 5; Option (5) is true.
2 . 3; E is husband of B.
3. 3; Two persons F and G sit between B and C.
4. 1; D is wife of C.
5. 5; Second to the right.

K KUNDAN
Puzzle 90
N

W E

S
l l l l l l
T Q S P R U
1. 3 2. 3 3. 1
4. 5; 2 1
S  R  P
2 1
Q  P  S
2 1
P  U  R
2 1
T  S  Q
2 3
T  S  U
1 1 1 2 1
5. 4; U  R; P  S; Q  T; S  R ; R  P
366 / Magical Book on Puzzle

Puzzle 91

Friends Places of Satying Hobbies


Hetal Vikroli Singing
Jayshree Thane Drawing
Rohini Dadar Reading
Meena Kanjurmarg Cooking
Nidhi Mulund Travelling
Swati Matunga Dancing

1. 2 2. 4 3. 3 4. 1 5. 4

Puzzle 92
On the basis of given information following table can be constructed:

Days Flights
Monday Emirates
Tuesday Jet Airways
Wednesday Delta
Thursday British Airways

K KUNDAN
Friday Air India
Saturday Quantas
Sunday Lufthansa

1. 3; Jet Airways flies on Tuesday.


2. 4; Three flights fly between Lufthansa and Delta.
3. 1; Air India flies on Friday.
4. 5; Quantas will fly on Friday.
5. 4; There are two flights between Emirates and British Airways.

Puzzle 93
Sitting arrangement would be as follows:

(Female)
A (Male)
(Male) H F

(Female) C G (Female)

(Male) E D (Male)
B
(Female)
A is wife of H.
B is wife of F.
Questions Asked In Exams / 367

1. 4; G is a female who sits second to right of B.


2 . 2; H sits third to the left of B.
3. 2; Two persons D and G sit between B and F when counted in anti-
clockwise direction from B.
4. 4; G is a female while all others are males.
5. 5; Female members are A, G, B and C.
6. 3;
(A)
A
(H) (B)
H F

(G) C G (C)

E D (D)
(F)
B
(E)
Puzzle 94
Sitting arrangement is as given below:

A
Businessman

K KUNDAN E
Banker
G
Clerk
D
Doctor

F
Architect

B H
Shopkeeper Teacher
C
Engineer
1. 3; Statement (3) is true.
2. 4; H is a Teacher.
3. 2; D, Doctor is third to the left of Banker E.
4. 4; Doctor D sits between F, Architect and A, Businessman.
5. 5; G is a clerk.

Puzzle 95

Row-1
R L
I E
G F
H
T T
V T Q R P S
E B A C D F
L R
E I
F G
H
T T
Row-2
368 / Magical Book on Puzzle

1. 2; To persons - A and C - are seated between B and D.


2. 1; V and B are opposite diagonally. Therefore, P is related to F.
3. 4; V sits at one of the extreme ends of the line.
4. 2; V and S are at extreme ends of the Row-1.
5. 5; T faces B.

Puzzle 96

L N M O L Q P R
E I
G

Violet
Pink
Red

White

Blue

Green
F H
T T

1. 4; Except in LQ, in all others there is a gap of one bottle.


2. 1; Chemical L is in Pink bottle.
3. 3; P- Green is correct.
4. 3; Chemical Q is in Violet bottle.
N M O L Q P
5. 2;
L M N O P Q
6. 2; Chemical N is in Red bottle.
7. 1; Chemical Pis kept in the bottle at the extreme right.

Puzzle 97
Days Play Duration

K KUNDAN
1
Monday C 1 Hours
2
Tuesday A 1 Hour
1
Wednesday E 2 Hours
2
Thursday F 3 Hours
Friday Off day
Saturday D 2 Hours
1
Sunday B Hour
2

1
1. 4; Duration of Play B is Hour.
2
2. 3; Friday is off day.
3. 2; A—Tuesday — 1 Hour is correct.
4. 2; D is staged on Saturday.
5. 5; Four plays were staged before the off day.
1
6. 2; If D is staged on Monday, Play E of 2 Hours duration will be staged
2
on Thursday.
7. 4; Now, Saturday would be the off day.
Questions Asked In Exams / 369

Puzzle 98
Sitting arrangement:

Supervisor
M
Doctor N L Accountant

Shopkeeper R P Painter

Reporter O S Cricketer
T
Teacher
1. 1; O is a reporter.
2. 1; S is third to the right of R.
3. 4; Four persons - R, O, T and S.
4. 5; Shopkeeper and Doctor are immediate neighbours.
5. 3; The doctor is sitting exactly between the Supervisor and Shopkeeper.

Puzzle 99

K KUNDAN
Friend Specialisation Sport
Deepak Electrical Football
Varun Hardware Tennis
Anit Metallurgy Swimming
Nilesh Telecommunication Badminton
Rajesh Software Hockey
Siddharth Mechanical Cricket

1. 3; Varun is studying Hardware.


2 . 1; Deepak likes Football.
3. 3; The combination Anit - Metallurgy is correct.
(4-5):
Anit Deepak Nilesh Rajesh Siddharth Varun

4. 5; Deepak is studying Electrical. Anit is to the immediate left of Deepak.


Anit is studying Metallurgy and likes Swimming.
5. 5; Immediate neighbours of Rajesh are Nilesh and Siddharth. Nilesh
likes Badminton and Siddharth likes Cricket.

Puzzle 100
E < D < A
10 20 30
B < F
50 60
370 / Magical Book on Puzzle

E  10 Minutes D  20 Minutes
A  30 Minutes C  40 Minutes
B  50 Minutes F  60 Minutes
1. 3; C takes 40 minutes.
2 . 4; F takes maximum time.
3. 1; Four persons.

Puzzle 101

S M
K

Q T

L
P R

1. 4; M is second to the left of L.


2. 2; P sits exactly between Q and R.
3. 5; Q faces T and both are sitting in the middle of the sides.
4. 3; P and Q.
5. 2; Q would sit to the immediate right of K.
6. 5; Except R, all others are seated at the corners.

K KUNDAN
Puzzle 102

M onth
January
Friend
E/H
February I
March D
April A
May K
June J
July C
August G
September L
October E/H
November B
December F

1. 4; B was born in November.


2. 4; Except K, all others are born in the months having 30 days.
3. 3; E or H was worn in January.
4. 1; F was born in December.
5. 2; The month of birth of friends has 30 or 28 days.
Questions Asked In Exams / 371

Puzzle 103

LEFT RIGHT
E P

A Q
N

Row-2
Row-1

B
W E
C S

S
F V

D R
LEFT RIGHT
1. 5; F is facing V.
2 . 2; Two persons—A and B.
3. 4; E and R at the left ends of the rows. B and S are third from the left
end.
F is second from the right end. In Row-2, Q is second from the right

K KUNDANend.
4. 4; T faces B.
5. 3; E and D are sitting at the ends of the row-1.

Puzzle 104

Friend College Subject

A Y Fashion

B Y Acting

C Y Architecture

D Z Teaching

E X Medicine

F Z Engineering

G X Business

1. 5 2. 3 3. 1
4. 1 5. 4 6. 4
7. 2
372 / Magical Book on Puzzle

Puzzle 105

1. 5 2. 4 3. 2 4. 2 5. 5 6. 4
7. 3 8. 1

Puzzle 106

K KUNDAN
1.
2.
3; D and H
5 3. 3 4. 4 5. 3
6. 5; Holland
7. 2

********
PROBLEMS ON SEATING ARRANGEMENT

SEATING ARRANGEMENTS
In this type of questions, some clues regarding
seating or placing (linear/ circular)of some
persons or items is given. The candidate is
required to form the paper sequence using these
clues and answer the questions accordingly.
I) Read the following information carefully and answer the
questions given below:

Six persons A,B,C,D,E and F are sitting in two rows, three


in each. E is not at the end of any row D is second to the
left of F. C the neighbour of E, is sitting diagonally
opposite to D. B is the neighbour of F.

1. Which of the following are sitting diagonally opposite


to each other?

(a) F and C (b) D and A (c) A and C (d) A and F


(e) A and B

2. Who is facing B?

(a) A (b) C (c) D (d) E (e) F

3. Which of the following are in same row?

(a)A and E (b) E and D (c) C and B (d) A and B


4. Which of the following are in one of the two rows?

(a) FBC (b) CEB (c) DBF (d) AEF (e) ABF

5. After interchanging seat with E, who will be the


neighbours of D in the new position?

(a) C and A (b) F and B (c) only B (d) only A (e) only C

Solution:

The given information can be analysed as follows:


E is not at end so; E must be in the middle of one of the
rows.

D is second to the left of F so, order of rows must be D_F.

C is neighbour of E and is sitting diagonally opposite to D


means C is under F in the other row i.e. D _ F _ E C.

B is neighbour of F; s the arrangement must be D B F A E C.

1. Other than D and C, A and F are sitting diagonally


opposite to each other, as seen in the arrangement. So
the answer is(d).

2. Clearly, E is opposite to B in the other row. So, E is


facing B and the answer is (d).

3. Clearly, from amongst the given alternatives, A and E


are in the same row .So the answer is (a).

4. Clearly, from amongst the given alternatives, D,B and F


are in the same row. So, the answer is (c).

5. Clearly, neighbours of E are A and C. So, on


interchanging the seat with E, the new neighbours of D
will be A and C. So the answer is (a).

***********************************************************
*

II) Eight books are kept one over the other counting from
the top the second, fifth and sixth books are on plays.
Two books on plays are between two books on composition.
One book of plays is between two books on poetry. While
the book at the top of the book of literature is a book
of composition.

Which book is fourth from the top?

(a) plays (b) poetry (c) composition (d)literature

Solution:
We analyse the given information as follows :

Let C denote 'composition' , P denote 'plays' , Po denote


'poetry' and L denote 'literature'.
1 2 3 4 5 6 7 8

_ P _ _ P P _ _

_ _ _ C P P C _

Po P Po _ _ _ _ _

_ _ _ _ _ _ C L

So the arrangement becomes:

1 2 3 4 5 6 7 8

Po P Po C P P C L

Clearly the fourth book from the top is on composition. So


the answer is (c).

III) Read the following information and answer the


Questions that follows.

(1)Six friends A, B, C, D, E and F are sitting in a closed


circle facing the center.
(2)E is to the left of D.
(3)C is between A and B.
(4)F is between E and A.

1. Who is to the left of B?


(a) A (b) C (c) D (d) E (e) none of these

2. Who is to the right of C?


(a) A (b) B (c) D (d) E (e)F
3. Which of the above given statements is superfluous?
(a) 1 (b) 2 (c) 3 (d) 4 (e) none of these

Solution:
Clearly, in the circle this arrangement is as shown
D
B E
C F
A

1. Clearly, D is the left of B. So the answer is (C)

2. Clearly, A is to the right of C. so the answer is (a).

3. Since all the statements are necessary to determine the


arrangement none of them is superfluous. So the answer
is (e).

EXERCISE

1. Four girls are sitting on a bench to be photographed.


Shikha is to the left of Reena. Manju is to the right
of Reena. Rita is between Reena and Manju.

Who would be second from the left in the photograph?


(a) Reena (b) Shikha (c) Manju (d) Rita

Answer is (d)
Shikha is to the left of Reena and Manju is to her right.
Rita is between Reena and Manju. So the order is Shikha
,Reena , Rita and Manju. In the photograph Rita will be
second from left.

**********************************************************
2. There are five different houses, A to E in a row. A is
to the right of B and E is to the left of C and right
of A. B is to the right of D.

Which of the houses is in the middle?


(a) A (b) B (c) D (d) E

Answer is (a)

B is to the right of D. A is to the right of B. E is to the


right of A and left of C. So the order is in the middle.
D, B, A, E, C. Clearly A is in the middle.

**********************************************************
3. In a march past, seven persons are standing in a row.
Q is standing left to R but right to P. O is standing
right to N and left to P.Similarly, S is standing
right to R and left to T.

Find out who is standing in middle?


(a) P (b) Q (c) R (d) O

Answer is (b)

Q is left to R and to the right of P i.e. P, Q, R.


O is to the right of N and left of P i.e. N, O, P.
S is to the right of R and left of T i.e. R, S, T.
Clearly, Q is in the middle.

**********************************************************

4. Five children are sitting in a row. S is sitting next to


P but not T. K is sitting next to R who is sitting on
extreme left and T is not sitting next to K. Who are
sitting adjacent to S?
(a) K and P (b) R and P (c) only C d) P
and T (e)Insufficient

Answer is (d)

S is sitting next to P. So, the order S, P or P, S is


followed. K is sitting next to R. So, the order R, K is
followed because R is on the extreme left. T is not next to
P or K. So, the arrangement will be R, K, P, S, T.
Clearly, P and T are sitting adjacent to S.

**********************************************************

5. Five girls are sitting in a row , Rashi is not adjacent


to Sulekha or Abha. Anuradha is not adjacent to Sulekha.
Rashi is adjacent to Monika. Monika is at the middle in
the row. Then, Anuradha is adjacent to whom out of the
following ?

(a) Rashi (b) Sulekha (c) Abha (d) Monika

e) Cannot be determined
Answer is (a)

Clearly, the order is

Anuradha, Rashi, Monika, Sulekha Abha.

Anuradha is adjacent to Rashi

**********************************************************

6. Read the following information carefully and answer the


questions given below it:

(A)There are five friends


(B)They are standing in a row facing south.
(C)Jayesh is to the immediate right to Alok.
(D)Pramod is between Bhagat and Subodh.
(E)Subodh is between Jayesh and Pramod.

1. Who is at the extreme left end?


(a) Alok (e) None of these (b) Bhagat (c) Subodh
(d) Inadequate Data

2. Who is in the middle?


(a) Bhagat (b) Jayesh (c) Pramod (d) Subodh (e)Alok

Solution:

The boys are standing facing south. So, consider left and
right accordingly.

Jayesh is to the right of Alok i.e. Jayesh, Alok.

Pramod is between Bhagat and Subodh i.e. Bhagat, Pramod,


Subodh

Subodh is between Jayesh and Pramod.


So, the sequence is Bhagat, Pramod,Subodh,Jayesh, Alok,

Answer:

1. (a), Alok is at extreme left end.


2. (d), Subodh is in the middle.

***********************************************************
*

7. Study the given information carefully and answer the


questions that follows.

(1)A, B, C, D, E, F and G are sitting on a wall and all of


them are facing east.
(2)C is on the immediate right to D.
(3)B is at an extreme end and has E as his neighbour.
(4)G is between E and F.
(5)D is sitting third from the south end.

1. Who is sitting to the right of E?


(a) A (b) C (c) D (d) F (e) None of these

2. Which of the following pairs of people are sitting at


The 2extreme ends?
(a) AB (b) AE (c) CB (d) FB (e) Cannot be determined

3. Name the person who should change places with C such


that he gets the third place from the north end?
(a) E (b) F (c) C (d) G
4. Immediatly between which of the following pairs of
People are sitting?
(a) AC (b) AF (c) CE (d) CF (E) None of these

Solution:
C is to the right of D. D is third from south.
So, B will be at the extreme end from north because it
should have E as its neighbour.

G is between E and F. SO, the sequence is


B->
E->
G->
F->
East
D->
C->
A->

Answer:

1. (e), G is sitting to the right of E.

2. (a), A and B are sitting at the extreme ends.

3. (C), G should change place with C to make it third from


north.

4. (d), D is sitting between C and F.

**********************************************************

8. In the Olympic games, the flags of six nations were


flown on the masts in the following way:

The flag of America was to the left of Indian tricolor and


to the right of the flag of France. The flag of Australia
was on the right of the Indian flag but was to the left of
the flag of Japan which was to the left of the flag of
China.

Find the two flags which are in the center.

(a) India and Australia (c) Japan and Australia


(b) America and India (d) America and Australia

Solution:

The correct sequence is


France, America, India, Australia, Japan, China.
The two flags in the center are India and Australia.

**********************************************************
9. Mr. A, Miss.B, Mr.C and Miss.D are sitting around
a table and discussing their trades.

(1)Mr A sits opposite to cook.


(2)Miss B sits right to the barber.
(3)The washer man is on the left of the tailor.
(4)Miss D sits opposite to Mr C.

What are the trades of A and B?

(a) Tailor and Barber (c) Barber and Cook


(b) Tailor and Cook (d) Washer man and Cook.

Solution: (b)

C and D sit opposite to each other .So if A sits opposite


to cook, B shall be cook.

Now B is to the right of barber. So, one of the rest say C


will be barber, then D. On the opposite side shall be
washer man or tailor. But washer man is on the left of
tailor and A is to the left of D. So, A is washer man and D
is tailor. Thus, A and B are Washer man and Cook.

**********************************************************

10. On the information given below, answer the questions.

(A) P, Q, R, S and T are sitting in a circle facing the


center.
(B) R is immediate left of T.
(C) P is between S and T.

1. Who is to the immediate left of R?


(a) P (b) Q (c) S (d) T (e) cannot be determine

2. Which of the Following statements can be dispensed with?


(a) None (b) B only (c) C only (d) B or C only (Ans) None

Solution:

Solution for 1st and 2nd questions is in the circle the


arrangement is as shown:
P
S T
Q
R
Answer:

1. (b), Q is to the immediate left of R.


2. (a) None. All the three statements are essential to
find out the answer for the first question so none can
be dispensed with.

**********************************************************

11. Six friends A, B, C, D, E and F are sitting in a closed


Circle facing the center.

A is facing D. C is between A and B. F is between E and A.


Who is to the immediate left of B?

(a) A (b) C (c) D (d) E

Answer is (b)
Clearly, in a circle the arrangement is as shown:

A
C F

B E

D
So, C is to the immediate left of B hence the answer is (b)

According to my opinion there can be another arrangement


based on the given details:
A
F C

E B

Hence D is the immediate left of B… so the answer could be


(c ) as well….

**********************************************************

12. A, B, C, D, E and F are seated in a circle facing the


center. C is between F and B.

A is second to the left of D and second to the right of E.

1. Who is facing A?
(a) B (b) D (c) F (d) either F or B (e) None

2. Who among the following is facing D?


(a) A (b) C (c) E (d) cannot be determine (e) None

Solution: Solution for 1st and 2nd questions is


The circular arrangement is as shown

C
A E

F B
D
Answer:

1. (a), Clearly B is facing A.


2. (b), C is facing D.

According to me there can be another arrangement as


follows:

C
A E

B F

If so the answer for the first question will be (


d) either F or B

**********************************************************

13. Eight friends A, B, C, D, E, F, G and H are sitting in


circle facing the center.

B is sitting between G and D. H is third to the left of B


and second to the right of A. C is sitting between A and G
and B and E are not sitting opposite to each other.

1. Who is third to the left of D?


(a) A (b) E (c) F (d) cannot be determine (e) None

2. Which of the following statements is not correct?

(a) C is third to the right of D.


(b) A is sitting between C and F.
(c) D and A are sitting opposite to each other.
(d) E is sitting between F and D.
(e) E and C are sitting opposite to each other

Solution: Solution for the 1st and 2nd question is

B is between G and D i.e. the order is G B D.

H is third to the left of B and second to the right of A.


So, forming a circle we have:

H A

G D

C is between A and G. But E is not opposite B. So, C is


between A and H.
C
H A
F E
G D
B

Answer:
1. (c), F is third to the left of D.

2. (d), Clearly, E is not sitting between F and D.


**********************************************************

14. A group of eight members sit in a circle. D is between


A and F and is opposite to G.E is to the right of A but
on the left of C, whose right hand neighbour is G. B
enjoys having H to his left and F to his right.

Find the member who is diagonally opposite to A?

(a) B (b) F (c) G(d) H

Answer: (d)

In a circle the arrangement is as shown

G
H C

B E

F A

H is diagonally opposite to A.

____________________________END______________________________________
Exercise–1
Three-statement Data Sufficiency
Directions: Each of the questions below consists
of a question and two or three statements given below the father-in-law of the mother of ‘L’.
it. You have to decide whether the data provided in 1) No ne
the statements are sufficient to answer the question. 2) All the information even together are not sufficient.
1. How is P related to Q? 3) Any one of them
A. P is the mother of R, who is the sister of S. 4) Only C
B. S is the brother of Q, who is the son of T. 5) None of these
C. P is the wife of T. 7. P, Q, R, S, T, U and V are seven policemen standing
1) Only A and B together are sufficient in a column (not necessarily in the same order) facing
2) Only B and C together are sufficient east. Who is exactly in the middle?
3) A and either B or C are sufficient A. The number of persons between S and U is equal
4) B and either A or C are sufficient to that between V and S.
5) All even together are not sufficient B. P, who is 5th from one end, is 3 positions behind
2 . What is the direction of K with respect to L? R.
A. K is east of M, which is south of L. C. Q and P are adjacent to each other, so are T and
B. N is north of L and north-west of K. S.
C. K is south-east of N, which is north of M. 1) Either A or B or C 2) Only B
1) Only either A or B alone is sufficient 3) Only A 4) All together are sufficient
2) Only A alone is sufficient 5) None of these
3) Only B and either A or C are sufficient 8. How many sons and daughters does Amod have?
4) Either A alone or B and C together are sufficient A. Amod’s wife says she has number of sons twice
5) Only B alone is sufficient the number of daughter.
3. How is B related to Y? B. Tom, who is one of the sons of Amod, says that
A. X and Y are brothers. B is the brother of A and A he has one-and-a-half times as many brothers as
is the mother of X. sisters.
B. A is the brother of B and father of X, who is the C. Tinu, who is one of the daughters of Amod, says
brother of Y. that she has 4 times as many brothers as sisters.
C. B is the brother of A and father of X. X is the 1) Any one of them 2) Any two of them
brother of Y. 3) Either B or C 4) Either A or C
1) Any one of the three is sufficient 5) None of these
2) Either (A) or (B) is sufficient 9. Who is the uncle of L?
3) All the three are necessary A. P, brother of M, is father of L; M is father of S.

K KUNDAN
4) Either (B) or (C) is sufficient B. R is father of L’s cousin.
5) None of these 1) A alone is sufficient
4. What is the meaning of the code ‘kullu’ in a code 2) B alone is sufficient
lanugage? 3) Either A alone or B alone is sufficient
A. In that code language rem tez kullu pullu tullu 4) Both A and B together are not sufficient
means ‘Sher Singh is my son’ and ‘gullu sullu rullu 5) Both A and B together are necessary
pullu’ means ‘is he at home’. 10. How is Raju related to Manohar?
B. In that code language nel pullu kullu dela means I. Pramila is the mother-in-law of Rashmi, the wife
‘my daughter is Nirmala’ and setha gama lala means of Raju.
‘sit with me’. II. Pramila’s brother is Raju’s maternal uncle.
C. In that code language nel dela pullu nillu means III. Pramila’s husband is the only son of Manohar.
‘her daughter is Nirmala’ and ‘kettu bala’ means 1) Only I and II
‘go home’. 2) Only I and III
1) B and either A or C 2) Either C or A 3) Only I and either II or III
3) Either A or B or C 4) Any two of them 4) Any two of the three
5) None of these 5) Question cannot be answered ev en wit h the
5. In which direction from city ‘X’ is city ‘Y’ situated? information in all three statements
A. City ‘X’ is to the east of city ‘W’ but both the cities 11. What is the code for ‘rope’ in a code language?
‘Y’ and ‘X’ are in the straight line. I. ‘use the rope’ is written as 'nik ta re’ in the code
B. No cities are to the north of ‘X’. language.
C. No cities except ‘X’ are on the same straight line II. ‘rope is straight’ is written as ‘pe da ta’.
on which ‘W’ is situated. III. ‘always use rope’ is written as 'ma re ta’.
1) Either A or B or C 2) B and C 1) Only I and II 2) Only I and III
3) Only B 4) All are true 3) Only II and III 4) Only II and either I or III
5) None of these 5) Question cannot be answered ev en wit h the
6. How is ‘L’ related to ‘W’? information in all three statements
A. ‘P’ is wife of one of the sons of ‘X’ and mother of 12. Who amongst Subodh, Neeraj, Tara, Meena and Anil
the grandson of the father of ‘Y’. is the first to take the lecture?
B. ‘Y’ is the father of ‘W’ but ‘W’ is not the daughter I. Subodh takes lecture before Meena and Neeraj
of the wife of ‘Y’. but not before Anil.
C. ‘L’ is the brother of ‘T’, who is granddaughter of II. Tara is not the first to take the lecture.
III. Meena is not the last to take the lecture. 1) Only I and II
1) Only I 2) Only I and III
2) Only I and II 3) Only II and III
3) Only I and either II or III 4) Only I and either II or III
4) All I, II and III are necessary 5) None of these
5) Question cannot be answered ev en wit h the 19. How many children are there in the class?
information in all three statements I. Saurabh is fif th f rom the top if arranged in
13. P is in which direction with respect to Q? descending order of marks.
I. M is to the North of R, who is to the West of Q. II. Sulabha, who is ten ranks below Saurabh, is 25th
II. P is to the East of M. from the bottom.
III. P is to the North-East of R. III. Jatin is four ranks above Sulabha.
1) Only I and II 1) Only I and III 2) Only II and III
2) Only I and III 3) Only I and II 4) Any two of three
3) Any two of the three 5) None of these
4) All I, II and III are necessary 20. How many daughters does W have?
5) Question cannot be answered ev en wit h the I. B and D are the sisters of M.
information in all three statements. II. M’s father T is the husband of W.
14. What is Sunita’s rank from top in a class of 45 III. Out of the three children which T has, only one
students? is a boy.
I. Sunita is five ranks below Samir, who is 15th 1) Only I and III are sufficient to answer the question.
from the bottom. 2) All I, II and III are required to answer the question.
II. Radha is 30th from the top and Neeta is 4th from 3) Only II and III are suf f icient t o answer the
the bottom. question.
III. Sunita is exactly in the middle of Radha and 4) Question cannot be answered even with all I, II
Neeta. and III.
1) Only I 5) Only I and II are sufficient to answer the question.
2) Only II and III 21. Who among A, B, C, D, E and F, each having a
3) Either only I or only II and III different height, is the tallest?
4) Only I and either II or III I. B is taller than A but shorter than E.
5) None of these II. Only two of them are shorter than C.
15. Among P, T, J, F and L who scored the highest? III. D is taller than only F.
I. P scored less than J and F. 1) Only I and II are sufficient to answer the question.
II. T scored more than F but less than L. 2) Only I and III are sufficient to answer the question.
III. J has not scored the highest. 3) Only II and III are suf f icient t o answer the
1) Only I and II question.

K KUNDAN
2) Only I and III 4) All I, II and III are required to answer the question.
3) Only II and either I or III 5) All I, II and III even together are not sufficient to
4) Only I and either II or III answer the question.
5) All I, II and III 22. Towards which direction is Village J from Village
16. Pillar ‘P’ is in which direction with respect to pillar W?
’Q’? I. Village R is to the west of Village W and to the
I. Pillar ‘M’ is to the East of pillar T, which is to north of Village T.
the South of pillar ‘Q’. II. Village Z is to the east of Village J and to the
II. Pillar 'P' is to the North of pillar ‘M’. south of Village T.
III. Pillar ‘R' is to the North of pillar ‘P’ and to the III. Village M is to the north-east of Village J and to
East of pillar ‘Q'. the north of Village Z.
1) Only I and II 1) Only III is sufficient to answer the question.
2) Only I and III 2) Only II and III sufficient to answer the question.
3) Only II and either I or III 3) All I, II and III are required to answer the question.
4) Only III 4) Question cannot be answered even with all I, II
5) All I, II and III and III.
17. What is the code for ‘adequate’ in a code language? 5) None of these
I. In the code language, ‘rainfall is adequate’ is 23. On which day of the week did Suresh visit Chennai?
written as ‘vo al ji’. (Assume that the week starts from Monday.)
II. In the code language, 'food is adequate’ is written I. Suresh took a leave on Wednesday.
as 'vo ji fa’. II. Suresh visited Chennai the day after his mother’s
III. In the code language, ‘food is delicious’ is written visit to his house.
as ‘fa vo re’. III. Suresh’s mother visited Suresh’s house on neither
1) Only I and III 2) Only II and III Monday nor Thursday.
3) Only I and II 4) All I, II and III 1) Only II and III are suf f icient t o answer the
5) None of these question.
18. How is Sanjeev related to Radha? 2) Only I and II are sufficient to answer the question.
I. Sanjeev is the son of Radha’s grandfather’s only 3) Only I and III are sufficient to answer the question.
daughter. 4) All I, II and III are required to answer the question.
II. Sanjeev has no siblings. 5) Question cannot be answered even with all I, II
III. Radha has only one brother. and III.
24. How is ‘go’ written in a code language?
I. ‘now or never again’ is written as ‘torn ka na sa’ in that code language.
II. ‘you come again now’ is written as ‘ja ka ta sa’ in that code language.
III. ‘again go now or never’ is written as ‘na ho ka sa torn’ in that code language.
1) Only I and III are sufficient to answer the question.
2) Only II and III are sufficient to answer the question.
3) Only I and II are sufficient to answer the question.
4) All I, II and III are required to answer the question.
5) None of these

Answers and explanations


Three-statement Data Sufficiency
1. 4; From A and B: (-) P  T (+) From (B): It is clear that ‘R’ is uncle of ‘L’.
10. 2; Statement I and statement II both lack information
(-) R — S (+) — Q (+) about Manohar. Hence, reject the options 1), 3) and
Thus, P is mother of Q. 4).
From B and C: (-) P  T (+) Now, check for the possibility of the option 2).
Using, statement I and statement III, we get
S(+) — Q(+)
Answers and explanations
Manohar
Again, P is mother of Q. |
2 . 2; From A: L  Pr amila (  )  * (  ) (Manohar ' s son )
|
M  K Rashmi (  )  Raju ( )
K is south-east of L. Hence A alone is sufficient.
From B: K is east of L but we don’t know whether Hence, Raju is the grandson of Manohar.
north or south. And C is no help either. 11. 4; From I: we get
3. 5; In (B) we have no information about the sex of use the rope  nik ta re ... (i)
‘B’. Hence correct answer is either (A) or (C). From II: we get
4. 1; We can access the answer with the help of B rope is straight  pe da ta ... (ii)
and either A or C. From III: we get
5. 4; From A: The possible situations are as follows: always use rope  ma re ta ... (iii)
1. W — X — Y From I and II: We get that the only word which
2. W — Y — X is common in (i) and (ii) is ‘rope’ and only code
3. Y which is common in (i) and (ii) is ‘ta’. Hence, the

K KUNDAN
| code for ‘rope’ is ‘ta’.
W — X From II and III:
4. W —X We get that the code for ‘rope’ is ‘ta’.
| But from I and III we can’t get the specific code of
Y ‘rope’ among ‘ta’ and ‘re’.
5. Y — W — X Hence, only II and either I or III are sufficient.
From A and B: We reject situation 3. 12. 2; From I it is obvious that either Anil or Tara took
Now, from A, B and C: we reject situations 1, 2 the first lecture. And with the help of II it becomes
and 5. Hence we have the only possible situation clear that Anil took the first lecture.
as indicated in 4. Hence, we need all the three 13. 4; From I and II: we get
informations.
M   P1   P2   P3 —
6. 2; Statement C has no relation either with statement |
A or with B. In the statement C, ‘W’ is not being R _______ Q
talked about. Similarly, in statemnts A and B, ‘L’ P may be towards North or North-west or North-
is not being talked about. Hence all the three East of Q. Since we do not get any specific answer
statements even together are not sufficient. both I and II together are not sufficient.
7. 4; VQPSTRU or UQPSTRV From I and III: We get
8. 2; From A: B = 2S; P3
3 M
From B: B-1 = S;
2 P2
From C: (S-1)4 = B
Where, B = no. of brothers; P1
S = no. of sisters R Q
We have three different equations. Hence any Once, again we do not get any specific answer
two of them will be sufficient. because P may be North-West or North or North-
9. 2; From (A): East of Q.
(+)P —— M(+) From I, II and III: we get
| |
L S
Hence ‘M’ is uncle of ‘L’.
of Radha.
M P 19. 3; From I and II: We get
Sulabha's rank from top = 5 + 10 = 15th
Sulabha's rank from bottom = 25th
Total number of students in the class
= 15 + 25 - 1 = 39
2 0 . 3; From the statements II and III:
R Q

P is to the North of Q.
14. 3; From I: We get Sunita’s rank is (15-5 =) 10th
from the bottom, ie (45 - 10 + 1 =) 36th from the
top. Though, sex of M is not known, it is given in
From II and III: We get statement III, that T has three children and only
Radha's rank from top = 30th one of them is boy. Therefore, we may conclude
Neeta’s rank from top = 45 - 4 + 1 = 42nd that W has two daughters.
21. 4; From statement I:
30  42 E > B > A
Now, Sunita’s rank =
2 From statement II:
= 36th (from top) — > — > — > C > — > —
15. 5; From I and II: We get From statement III:
L > T > F > P Now combining all the above statements, we have
J > P E > B > A > C > D > F
From I, II and III: We get 2 2 . 5; From statement II:
L > T > F > P
L > J > P
Hence, 'L' scored the highest.
16. 4; From III: We get
Q R

From statement III:

It is obvious from the positions of 'P' and 'Q' that

K KUNDAN
the pillar 'P' is to the North-East of the pillar 'Q'.
17. 2; From II and III: We get Now combining statements I and II:
food is adequate  vo ji fa ... (i) R W
food is delicious  fa vo re ... (ii) l l
From (i) and (ii), we get 'vo fa' is the code of 'food l
is', hence the code for 'adequate' is 'ji'. T
18. 2; From I and II: We get l l
J Z
Radha's grandfather
J is in south-west direction from W.
23. 5; Even by combining all the statements, we cannot
find the day of the week on which Sur esh’s
mother visited Suresh’s house.
(+ ) 24. 1; From statements I and III:
(-)
now or never again  torn ka na sa ...(i)
Radha again go now or never  na ho ka sa torn ...(ii)
Sanjeev From (i) and (ii), code for ‘go’ is ‘ho’.
(+)

Hence, Sanjeev is the cousin of Radha.


From I and III: Sanjeev is either brother or cousin
598 Verbal Reasoning (Commonsense Reasoning)

Chapter 24

Syllogism
Introduction (b) Particular Proposition: Part icular proposition
either only partly include or only partly exclude the subject
Syllogism is originally a word given by the Greeks. while making a statement. Examples are,
Which means ‘inference’ or ‘deduction’. Some documents are secret.
Some cloths are not cotton.
Definitions of Some Important Terms
Particular propositions are also further classified as
The terms defined below are used in the well defined ( i ) Particular Positive Proposition: A proposition
method for solving the problems on syllogism. of the form “Some S are P”, for example, “Some
documents are secret”, is called a particular positive
Proposition proposition and it is denoted by the letter “I”.

K
A proposition is a sentence that makes a statement ( i i) Particular Negative Proposition: A proposition
and gives a relation between two terms. It consists of of the form “Some S are not P” for example, “Some
three parts (a) the subject; (b) the predicate; (c) the relation cloths are not cotton”, is called a particular negative
between the subject and the predicate. proposition. And is usually denoted by the letter
Some examples of propositions are being given below: “O”.
( i ) All coasts are beaches.
( i i) No students are honest.
Important Note
( i ii ) Some documents are secret The definition of the A, E, I, O propositions are very,
( iv) Some cloths are not cotton. very important and the student must have the ability to
immediately recognise these types. With this need in mind
Subject and Predicate we are listing these four types of propositions in the
A subject is that part of the proposition about which following table:
The Four Types of Propositions

KUNDAN
something is being said. A predicate, on the other hand,
is that term of the proposition which is stated about or
related to the subject. Type of
Universal Particular
Thus, for example, in the four propositions mentioned Proposition
above, ‘coasts’, ‘students’, ‘documents’ and ‘cloths’ are A Format: I Format:
subjects while ‘beaches’, ‘honest’, ‘secret’ and ‘cotton’ are All S are P Some S are P.
predicates. Positive Example: Example:
All coasts are Some documents
Categorical Propositions beaches. are secret.
E Format: O Format:
A categorical proposition makes a direct assertion. It
No S are P Some S are not P.
has no conditions attached with it. For example, “All S
Negative Example: Example:
are P”, “No S are P”, “Some S are P” etc are categorical
No student are Some cloths are
pr oposit ions, but “If S, then P” is not a cat egor ical
honest.
proposition.

Types of Categorical Propositions Mediate and Immediate Inference


(a) Universal Proposition: Univ er sal pr opositions Mediate Inference: Syllogism is actually a problem of
eit her f ully include the subject or f ully exclude it . mediate inference. In mediate inference conclusion is
Examples are, drawn from two given statements. For example, if two
All coasts are beaches. statements are given: “All tips are balls” and “All balls are
No Students are honest. pencils”, then a conclusion could be drawn that “All tips
Universal propositions are further classified as are pencils”. This is a case of syllogism or mediate inference
( i ) Universal Positive Proposition: A proposition of because conclusion is drawn from two propositions. We
the form “All S are P”, for example, “All coasts are will learn how to draw conclusion (mediate inference)
beaches”, is called a universal positive proposition. from the two given propositions later on in this chapter.
And it is usually denoted by a letter “A”. Immediate Inf erence: In immediat e inf er ence
( i i) Universal Negative Proposition: A proposition conclusion is drawn from only one given proposition. For
of the form “No S are P”, for example, “No students example, let a given statement be “All coasts are beaches”.
ar e honest ”, is called a univ er sal negativ e Then, based on this statement, a conclusion could be
proposition. And it is usually denoted by a letter drawn that “Some beaches are coasts”. This is a case of
“E”. immediate inference.
Syllogism 599

Important Cases of Immediate Inference by analytical method than venn-diagram method. Though,
I will be discussing about Venn-Diagram also in the latter
In order to be able to solve syllogism problems completely section of this book.
and speedily we need to have a thorough idea of immediate The analytical method for solving syllogism completely
inference. There are many aspects or methods of immediate consists of the following easy steps:
infer ence. These include conv er sion, obver sion,
contraposition etc. We shall not study the less important I. Draw mediate inferences
of these methods. We shall see only two important cases II. Draw immediat e inferences (implicat ion or
of immediate inference. conversion)
III. Check for complementary pairs
I. Implications (of a given proposition):
Let us see examples given below: I. Draw Mediate Inferences
(a) Suppose we are given a proposition “All coasts are
beaches”. then this proposition naturally implies There are two extremely simple steps to draw mediate
that the conclusion “Some coasts are beaches” must inferences:
be true. It is very easy to comprehend because if
“all” are beaches, then “some” (“some” is only a

Вам также может понравиться